Download as pdf or txt
Download as pdf or txt
You are on page 1of 1026

m

CLASS XII

(Code 241)

ow
Based on the syllabus of Applied Mathematics (Code 241) prescribed
by CBSEfor Class XII under 10+2 Pattern of Senior School Certificate Examination

e
Fl
re
F
ur
f or
ks
Yo
oo

Dr. R.D. SHARMA


B.Sc. (Hons) (GoldMedalist), M.Sc. (GoldMedalist), Ph.D.
B

Professor and Head of Department of Sciences and Humanities


re

Delhi Skill and Entrepreneurship University (DSEU)


Aryabhatt DSEUAshok Vihar Campus, Delhi
u
ad
Yo
d
Re
in
F

DHANPAT RAI PUBLICATIONS (P) LTD.


22, ANSAR! ROAD. DARYAGANJ, NEW DELHI-110002
E-Mail: ish.dhanpat@gmail.com
Price: t 455.00

To prevent a pirated book being sold to you, always ask the bookseller

ow
to put his stamp on the first page of this book.

e
re
First Edition : 2021
This Reprint : 2022

Flr
F
ou
Published by Isli Kiifuii-, for Dhanpat Rai Publications (P) Ltd.

sr
EVERY GENUINE COPY OFTHIS BOOK HAS A HSHF
fo
k
oo
The title of the book has been Laminated by a High Security Holographic Film (HSHF) to prevent Piracy.
You can see logo of Dhanpat Rai Publications at different angles with special security features.
Y

IiuportiwtNote: Buying & Selling pirated books which do not carry HSHF is illegal and a criminal offence
reB

under the Indian Penal Code. This also deprives the author and the publisher of their due earnings for
providing you the knowledge through this book. Pirated books are also sold as old editions.
uY

© Author
ad
do

This book shall not, by way of trade or otherwise be lent, resold, hired out, or otherwise circulated without the
publisher's prior written consent in any form of binding or coverotherthan that in which it is published. No
part of this book may be reproduced or copied in any form or by any means [graphic, electronic or medianical,
in

including photocopying, recording, taping, or information retrieval system] or reproduced on any disc, tape,
Re

perforated media or other information storage device, etc., withoutthe writtenpermissionof the author and
publisher. Breach of this condition is liable for legal action.
F

DISCLAIMER:

Every effort has been made to avoid errors or omissions in this publication. In spite of this, some errors might
have crept in. Any mistake, error or discrepancy noted may be brought to our notice which shall be taken care
of in the next edition. It is notified that neither the publisher nor the authors or seller will be responsible for any
damage or loss of action to any one, of any kind, in any manner, therefrom.
For binding mistakes, misprints or for missing pages, etc., the publisher’s liability is limited to replacement
within one month of purchase by similar edition. All expenses in this connection are to be borne by the
purchaser.
All disputes are subject to Delhi jurisdiction only.
Dear Teachers & Students
CBSE has introduced Applied Mathematics (Code 241) as an academic elective at Senior

ow
Secondary Level from Academic Session 2020-21 and onwards. This first edition of Applied
Mathematics for Class XII has been written strictly as per the latest syllabus prescribed by
CBSE.

e
The entire text has been divided into 26 chapters. In each chapter, all concepts have been

re
discussed in detail in a lucid manner and have been explained with suitable illustrations
and illustrative examples.

Flr
F
Some unique features of the book are:ou
— Detailed theory with Illustrations

sr
— Algorithmic approach
— Large number of graded Illustrative Examples
— Large number of unsolved Exercises fo
k
oo
— Large number of Multiple Choice Questions (MCQs), Very Short Answer Questions
Y

(VSAQs) and Fill in the Blanks Questions (FBQs).


reB

I avail this opportunity to convey my sincere thanks to Sh. Ish Kapur of Dhanpat Rai
uY

Publications (P) Ltd for his painstaking and sincere efforts in bringing out this book in such
an excellent form.
ad
do

Although every effort has been made to keep this book error free, inspite of tliis some printing
errors might have crept m. I would be extremely thankful if the same are brought to my notice
in

through email to rdsmath.feedback@gmail.com or by post.


Re
F

With my Best Wishes


Dr. R.D. SH/.RMA
APPLIED MATHEMATICS - XII CONTENTS

Preface to the First Edition m


1. MODULAR ARITHMETIC 1.1-1.39

2. ALLIGATION 2.1-2.14

ow
3. BOATS & STREAMS AND PIPES & CISTERNS 3.1-3.30

4. PARTNERSHIP 4.1-4.47

5. RACES AND GAMES 5.1-5.8

e
re
6. NUMERICAL INEQUALITIES AND LINEAR INEQUATIONS 6.1-6.30

Frl
F
7. ALGEBRA OF MATRICES 7.1-7.65

8. DETERMINANTS 8.1-8.21
ou
or
9. INVERSE AND APPLICATIONS OF MATRICES kfs 9.1-9.70

10. INCREASING AND DECREASING FUNCTIONS 10.1-10.29

11. MAXIMA AND MINIMA 11.1-11.53


oo

12. APPLICATIONS OF DERIVATIVES IN COMMERCE AND ECONOMICS 12.1-12.42


Y
B

13. INDEFINITE INTEGRALS 13.1-13.83


re

14. DEFINITE INTEGRALS 14.1-14.17


oYu

15. DIFFERENTIAL EQUATIONS 15.1-15.61


ad

16. APPLICATIONS OF INTEGRATION 16.1-16.27


d

17. DISCRETE RANDOM VARIABLE AND ITS PROBABILITY DISTRIBUTION 17.1-17.48


in
Re

18. BINOMIAL DISTRIBUTION 18.1-18.30


F

19. POISSON DISTRIBUTION 19.1-19.23

20. NORMAL DISTRIBUTION 20.1-20.28

21. INFERENTIAL STATISTICS 21.1-21.23

22. INDEX NUMBERS 22.1-22.32

23. TIME SERIES 23.1-23.17

24. FINANCIAL MATHEMATICS-1 24.1-24.34

25. FINANCIAL MATHEMATICS-II 25.1-25.39

26. LINEAR PROGRAMMING 26.1-26.74

LOGARITHMIC AND OTHER TABLES T.1-T.34


APPLIED MATHEMATICS-XII CO" b

Preface to the First Edition m

UNIT-1: NUME' ■ jANTIFlCATiC:' .Nl ‘ “'EPICAL.. ICATlONb

1. MODULAR ARITHMETIC 1.1-1.39

1.1 Introduction 1.1


1.1
1.2 Divisibility
1.3 Modular Arithmetic 1.2
1.5
1.4 Properties of Congruence
1.5 Linear Congruence 1.25
1.32
1.6 Elementary Arithmetic Functions
2. ALLIGAITON 2.1-2.14

w
2.1 Introduction 2.1
2.1
2.2 Alligation

F lo
3. BOATS & STREAMS AND PIPES & CISTERNS 3.1-3.30
3.1 Introduction 3.1
3.2 Some useful results 3.1

ee
3.3 Some useful terms and formulae 3.11

Fr
4. PARTNERSHIP 4.1-4.47
4.1 Some Definition 4.1

4.2
4.3
Essential elements of Partnership
Partnership Deed
for 4.1
4.3
ur
4.2
4.4 Some important points about Partnership
4.2
4.5 Investment of Capital for equal periods
s
ook

4.6 Investment of Capital for unequal periods 4.12


Yo

4.7 Partner's Salaries, Interest on Capital, Drawings and Advances or Loans 4.24
4.38
4.8 Profit sharing on the admission of a new partner
eB

4.43
4.9 Profit sharing on the retirement of an existing partner
5. RACES AND GAMES 5.1-5.8
our

5.1 Introduction 5.1


ad

5.2 Some useful Terms and Statements 5.1


5.3 Games 5.5
Y

6. NUMERICAL INEQUALITIES AND LINEAR INEQUATIONS 6.1-6.30

6.1 Numerical Inequalities 6.1


Re
nd

6.4
6.2 Inequations
6.3 Solution of an Inequation 6.4
Fi

6.4 Solving Linear inequations in one variable 6.5

6.5 Solution of System of Linear inequations in one variable 6.13

6.6 Some applications of Linear inequaitons in one variable 6.18

6.7 Graphical solution of Linear inequations in two variables 6.21

6.8 Solution of Simultaneous Linear inequaitons in two variables 6.24

UNIT-II: ALGEBRA
7. ALGEBRA OF MATRICES 7.1-7.65
7.1 Matrix 7.1
7.2
7.2 Types of Matrices
7.3
7.3 Equality of Matrices
7.4 Addition of Matrices 7.8
7.10
7.5 Multiplication of a Matrix by a scalar (Scalar Multiplication)
7.6 Subtraction of Matrices 7.10
7.18
7.7 Multiplication of Matrices
(V)
(Vi) Contents

7.8 Transpose of a Matrix 7.47

7.9 Symmetric and Skew-Symmetric Matrices 7.50

8. DETERMINANTS 8.1-8.21
8.1 Determinants 8.1

8.2 Singular Matrix 8.5


8.3 Minors and Cofactors 8.6

8.4 Applications of Determinants in solving a System of linear equations 8.10

9. INVERSE AND APPLICATIONS OF MATRICES 9.1-9.70

9.1 Adjoint of a square matrix 9.1


9.2 Inverse of a Matrix 9.3

ow
9.3 Some useful results on invertible Matrices 9.5

9.4 Simultaneous Linear equations 9.23

9.5 Solution of simultaneous Linear equations by Elimination method 9.24

9.6 Matrix method for the solution of a Non-Homogeneous system of equations 9.31

9.7 Leontief Input-Output Model 9.47

e
UNIT-111: CALCULUS

re
10.

rFl
INCREASING AND DECREASING FUNCTIONS 10.1-10.29

F
10.1 Introduction 10.1

10.2 Solution of rational algebraic Inequations 10.1


10.3 Some Definitions 10.5

r
10.4 Necessary and Sufficient conditions for monotonicity 10.11
ou
11. MAXIMA AND MINIMA fo 11.1-11.53
ks
11.1 Introduction 11.1
11.2 Maximum and Minimum values of a function in its domain 11.1
oo

11.3 Local Maxima and Local Minima 11.6


11.4 First derivatives test for Local Maxima and Minima 11.9
Y
B

11.5 Higher order derivative test 11.15


11.6 Maximum and Minimum values in a closed interval 11.22
re

11.7 Applied problems on Maxima and Minima 11.27

12. APPLICATIONS OF DERIVATIVES IN COMMERCE AND ECONOMICS 12.1-12.42


ou
Y
ad

12.1 Introduction 12.1


12.2 Functions related to Business and Economics 12.1

12.3 Breakeven analysis 12.6


d

12.4 Average and Marginal Costs 12.11


in
Re

12.5 Minimization of Average Cost 12.16

12.6 Total Revenue, Average Revenue and Marginal Revenue 12.23


F

12.7 Maximization of Total Revenue 12.28


12.8 Maximization of Total Profit 12.34

13. INDEFINITE INTEGRALS 13.1-13.83


13.1 Premitive or Antiderivative 13.1

13.2 Indefinite Integrals 13.1

13.3 Fundamental Integration Formulas 13.2

13.4 Some standard results on Integration 13.3

13.5 Geometrical Interpretation of Indefinte Integral 13.9

13.6 Comparison between Differentiation and Integration 13.10

13.7 Methods of Integration 13.11

13.8 Integration by Substitution 13.11

13.9 Some special Integrals 13.30

13.10 Integration by Parts 13.51

13.11 Some important Integrals 13.60

13.12 Integration of rational algebraic functions by using partial fractions 13.66


Contents (vli)

14. DEFINITE INTEGRALS 14.1-14.17


14.1
14.1 Fundamental Theorem oflntegral Calculus
14.1
14.2 Evaluation of Definite Integrals
14.9
14.3 Evaluation of Definite Integrals by substitution
14.11
14.4 Area as a Definite Integral
14.13
14.5 Area using vertical strips
15. DIFFERENTIAL EQUATIONS 15.1-15.61

15.1 Some Definitions 15.1


15.6
15.2 Formation of differential equations
15.20
15.3 Solution of a differential equation
15.24
15.4 Methods of solving a first order first degree differential equations ....
15.45
15.5 Applications of differential equations

ow
16. APPLICATIONS OF INTEGRATION 16.1-16.27
16.1 Introduction 16.1
16.2
16.2 Determining Cost function and Average Cost function when
MarginalCost function is given

e
16.6
16.3 Determiningthe Total Revenuefunctionand the Demand Function ..

re
from a given Marginal Revenue function

rFl
16.4 Determining Maximum profit when Marginal Revenue and Marginal 16.11

F
Cost functions are given
16.17
16.5 Consumer's and Producer's Surplus

r
UNIT-IV: PROBABILITY
ou
17. DISCRETE RANDOM VARIABLE AND ITS PROBABILITY fo
ks
DISTRIBUTION 17.1-17.48

17.1 Introduction 17.1


oo

17.2 Discrete Random Variable 17.1


17.2
Y

17.3 Probability Distribution


eB

17.18
17.4 Mathematical Expectation
17.5 Variance of a Discrete Random Variable 17.20

18.1-18.30
r

18. BINOMIAL DISTRIBUTION


ou

18.1 Introduction 18.1


ad
Y

18.2 Bernoulli Trials 18.1


18.3 Binomial Distribution 18.1

18.4 Mean and Variance of Binomial Distribution 18.19


d

19. POISSON DISTRIBUTION 19.1-19.23


Re
in

19.1 Introduction 19.1

19.2 Poisson Distribution 19.1


F

19.3 Role of the Poisson Distribution 19.2


19.4 Mean of Poisson Distribution 19.14

19.5 Variance of Poisson Distribution 19.14


19.15
19.6 Recurrence formula for the probabilities of Poisson Distribution
20. NORMAL DISTRIBUTION 20.1-20.28

20.1 Introduction 20.1


20.1
20.2 Continuous random variable and its probability density function
20.3 Normal Distribution 20.5
20.6
20.4 Properties of the Normal Distribution and Normal Probability curve
20.5 Standard form of the Normal Distribution 20.7
20.7
20.6 Area under the standard normal probability curve
UNIT-V: INFERENTIAL STATISTICS

21. INFERENTIAL STATISTICS 21.1-21.23


21.1
21.1 Population and Sample.
(viii) Cc ● ● ‘

21.2 Parameters and Statistics 21.2


21.3 Statistical Inferences 21.2
21.4 Students's t-Distribution 21.4

21.5 Applications of t-Dislribution 21.6

;IT-Vi: .:jE; ■ 8ER3 AND TIME BASED DATA

22. INDEX NUMBERS 22.1-22.32


22.1 Index Numbers 22.1
22.2 Characteristics of Index Numbers 22.2
22.3 Uses of Index Numbers 22.2

22.4 Types of Index Numbers 22.3


22.5 Problems in the construction of Index numbers

ow
22.4

22.6 Methods of constucting unweighted Index numbers 22.5

22.7 Methods of constucting weighted Index numbers 22.20


22.8 Limitations of Index numbers 22.29

23. TIME SERIES 23.1-23.17

e
23.1 Time Series 23.1

re
23.2 Method of Moving Averages 23.2

rFl
F
UNIT-VII: FINANCIAL MATHEMATICS

24. FINANCIAL MATHEMATICS-I 24.1-24.34

r
24.1 Recapitulation 24.1
ou
24.2 Sinking Fund
24.3 Perpetuity
fo 24.1
24.7
ks
24.4 Bonds 24.11
24.5 Valuation of Bonds 24.12
oo

24.6 Effective Rate of Interest 24.18


Y

24.7 Nominal Rate of Returns 24.23


eB

24.8 Compound Annual Growth Rate (CAGR) 24.25


24.9 EMI Calculation 24.26

24.10 Depriciation 24.30


r
ou

25. FINANCIAL MATHEMATICS-II 25.1-25.39


ad
Y

25.1 Introduction 24.1


25.2 Joint Stock Company 24.1
25.3 Shares 24.1
d

25.4 Some important terms 24.1


Re
in

25.5 Types of Shares 24.2


25.6 Debentures 24.14
F

25.7 Stocks 24.18

UNIT-VIII: LINEAR PROGRAMMING

26. LINEAR PROGRAMMING 26.1-26.74


26.1 Introduction 26.1

26.2 Linear Programming Problems 26.1


26.3 Some Definitions 26.3

26.4 Mathematical Formulation of Linear Programming Problems 26.3


26.5 Some Definitions and results 26.21

26.6 Graphical Methods of Solving Linear Programming Problems 26.23

26.7 Corner-point Method 26.23


26.8 Iso-Profit or Iso-Cost Method 26.29

26.9 Different types of Linear Programming Problems 26.37


26.10 Some exceptional cases 26.69

Logarithmic and Other Tables T.1-T.34


oorrH ]

MODULAR ARITHMETIC

- MTRODUCTION

Modular arithmetic is a system of arithmetic for integers, where numbers "wrap around" after
reaching a given fixed positive integer, called the modulus or modulo. In order to imderstand
various concepts in the chapter on modular arithmetic, we require to know the concept of
divisibility of integers and its properties. So let us begin with the same.

w
1.2 DIVISIBILITY

DIVISIBILITY ON ^

F lo
Let us first, define the divisibility on the set N of all natural numbers.
A natural number 'a' is said to divide another natural number 'b', if three exists a
natural number c such that b = ca and we write a \ b.

divide 14, because there is no natural number such that 14 = 3 x c.


If a natural number 'a' does not divide a natural number
ree
We observe that 3] 12,because there exists natural number 4such that 12 - 4x 3.But, 3 does not
for F
'h', we write a ^ b.
Let us now define divisibility on the set Z of integers.
QIVISIBiLiTY ON Z A non-zero integer 'a' is said to divide an integer 'b', if there exists an integer c such
Your
oks

that b -ca and we write a \ b.


eBo

We find that 5 divides 20, because there exists integer 4 such that 20 = 4x5.Also,51 -20, because
-20 = (-4) X 5. But, 5 / 22.
NOTE When an integer 'a' divides another integer 'b', we can also sai/ that a is a factor ofbor a is divisor
ad
our

borb is a midtiple of a.
Following are some properties of divisibility on Z.
(i) ± 11 for every a eZ.
Re

(ii) a I 0 also, a\a,a\ (-a) and {-a) \ a for every 0) e Z.


dY

(iii) a\b ^ a\ (-b); (-fl) |(-&) and (-n) ] b for al\a^0,b e Z.


Fin

(iv) a 1 b and b\a ^ a =■ ± b for all a, b e Z.


(v) a I b and a\c => a\bx + cy, x, y eZ.
(vi) aIb and «| c => a\b + c;a\{b-c) and a \ bx, x eZ.
(vii) aIb and a\c a\bc.
(viii) a1b and c \ d ac\bd.
(ix)aIb and b\c => a\c
(x) ac\bc a\b, c ^ 0.
DIVISION ALGORITHM Let a, b 0) be any two integers. Then there exist unique integers q and r such
that q =bq + r, where 0 <r <|&|.
GREATEST COMMON DIVISOR A positive integer d is called the greatest common divisor (gcd) of a and
b,if
(i) d\a and d \ b (ii) c | a and c\b => c | d.
If a, b are positive integers, their gcd is denoted by {a, b).
1.2 APPLIED MATHEMATICS-XII

The g.c.d is also known as the highest common factor (HCF). We find that (27, 45) =9 and
(18,-9) =9.
Also, {a, b) - {a, -b) ={-a, b) = (-<7, -b).
PRIME NUMBER An integer ±\) is called a prime number, if its only divisors are ± 1 and ± p.
For example, ± 2, ± 3, ± 5, ± 7, ± 11 etc are prime numbers.
RELATIVELY PRIME NUMBERS Two positive integers a and b are said to be relatively prime to each other
or co-prime iff their g.c.d. is 1 i.e. (a,b) =1.
Following are some properties of prime numbers :
(i) If p is a prime number and a is any integer, then either {p, r?) =1 or, (p, a) =p.
(ii) If pis a prime ninnber and a, b are two integers, then

ow
p\ab ^ p\a or, p\b.
(hi) Let a, b be two positive integers. If there exist integers x and y such that ax -\-by = l, then {a, b) =l
i.e. a and b are relatively prime.
(iv) Let a, b, c be three integers such that (a, b) =1 and {a, c) =1, then {a, be) =1.

e
(v) The smallest divisor, greater than 1, of an integer > 1, is a prime number.

re
1.3 MODULAR ARITHMETIC
rFl
F
We all are familiar with the usual arithmetic since our childhood. We have learnt that arithmetic
is study of numbers and various ways to generate new numbers by combining them through

r
ou
fo
addition, subtraction, multiplication and division. In our day-to-day life we also use another
kind of mathematics, even if we do not always think of it as such. Almost everyone of us uses
ks
watch or a clock to see time which works on 12-hour time keeping system. If someone asks us
that if it is 3 o’clock now, what will the time be after 27 hours? Do we say that it will be 30
oo

hours or 30 o' clock? Of course we do not. Because clock time "wraps around" every 12 hours.
Y
eB

By using the standard convention of time keeping, we would say that, it will be 6 o' clock. How
can we add 27 to 3 and end up with 6? The reason is that in this system 27 o'clock is same as
3 o'clock;28 o'clock is same as 4 o'clock;29 o' clock is same as 5 o’ clock and 30 o'clock is same
ur

as 6 o'clock. This is how we end up with 6 o'clock. Thus, we find that in 12-hour time
ad

keeping system, 30 o' clock is same as 6 o' clock. This can also be written as: '30 =6 (modulo 12)'
Yo

and we read it as '30 is congruent to 6 modulo 12' Likewise we find that 15 s3 (modulo 12), 17 =5
(modulo 12) etc. Such systems, that "wrap around" after hitting some limit, are called modular
d

arithmetic systems, and play an Important role both in theoretical and applied mathematics.
Re
in

In Mathematics, modular aritlimetic is a system of arithmetic for integers, where numbers


F

"wrap around" upon reaching a given fixed positive integer to leave a remainder. The given
fixed positive integer is known as the modulus or modulo.
As discussed in section 1.1, in the 12-hourclock, clock time "wraps around" every 12 hours and
so the hour number starts over after it reaches 12. So, it is arithmetic modulo 12.

In a modular arithmetic system there are finite number of integers, because numbers "wrap
around" after hitting some limit called modulo. For example, in arithmetic modulo 12 there
areonly 12 integers 0, 1, 2, 3, 10, 11, because 12 s 0 (modulo 12), 13 = 1 (modulo 12),
14 =2 (modulo 12) and so on. Likewise, in arithmetic modulo 7, there are only 7 integers 0,1,2,3,
4,5, 6, because 7=0 (modulo 7), 8 = 1 (modulo 7) and so on.
We find that in a modular arithmetic system there is a relationship between pairs of integers
with respect to a specified positive integer m called the modulus or modulo. For example, in
arithmetic modulo 12, there is a relationship between 18 and 6, because 18 = 6 (modulo 12).
Similarly, there is relationship between 23 and 11; 30 and 6 etc. Likewise, in arithmetic modulo 7,
we find that 30 =2 (modulo 7), -12 = 9 (modulo 7) etc. Such relationships are called congruent
modulo as defined below.
MODULAR ARITHMETIC 1.3

CONGRUENCES Letn, b be two integers and m be a positive integer other than 1. Then, 'a' is said to be
congruent to 'b' modulo in if m divides {a-b) or {a -b) is divisible b\/ in.
If ‘a’ is congruent to 'b' modulom, we write a=b (modulo m) or,a=b (mod m)
Thus,
a=b (modulo m)
m| (a-b)
There exists an integer q such that a-b = mq or,a = b + mq
NOTE 1 If 'a' is not congruent to 'b' modulo m, then ive write a mb (modulo m).
For example
109
(i) 123=21 (modulo 6), because = =17 i.e. (123-21) is divisible by 6.
6 6
57 -(-6) 63
(ii) 57 = -6 (modulo 7), because = 9 i.e. (57 - (- 6)) is divisible by 7.

w
7
23—(—4) 27
(hi) 23^-4(modulo5),because ' ’ —is not an integer i.e. 23-(-4) is not divisible

F lo
5

by 5.

ee
ILLUSTRATIVE EXAMPLES

Fr
EXAMPLE 1 Which of the following statements are true?
(i) 27 = 3 (modulo 4) (h) 11 =56 (modulo 9)
for
(hi) -4 =-114 (mod 11) (iv) 15 =3 (mod 9)
ur
SOLUTION (i) We find that
s

27-3 24
ook

^ = 6, which is an integer
Yo

4
eB

27 - 3 is divisible by 4
27 =3 (modulo 4)
So, the given statement is true,
our
ad

(ii) We find that


11-56 -45
— = - 5, which is integer
9
Y
Re

=> 11 -56 is divisible by 9


nd

11 =56 (modulo 9)
Fi

So, the given statement is true,


(hi) We observe that
-4-(-114) -4 + 114 no
= 10, which is an integer
11 11 11

-4-(-114) is divisible by 11
-4=- 114 (mod 11)
So, the given statement is true,
(iv) We observe that
15-3 12 4
— = —, which is not an integer
9 9 3

15 - 3 is not divisible by 9
15 ^ 3 (mod 9)
So, the given statement is not true.
1.4 APPLIED MATHEMATICS-XII

Find the set of values ofx satisfying: x + l=2^ (mod 7).


SOLUTION We have.
x + l=2^ (mod7)
x + 1 =256 (mod 7)
(.V +1) - 256 is divisible by 7
X - 255 is a multiple of 7
=>
.V - 255 = 7 q, where q is an integer
X = 255 +7q, where qis an integer
Putting <7 =...-2, -1, 0,1,2, we obtain
241,248, 255,262, 269,
Hence, the required set is { ..., 241,248,255,262, 269, } or, {x:x = 255+7q, q eZ^
■ : //250 slO (mod m),find thevalnes ofm.

w
SOLUTION We have.
150 =10 (mod in)

=>
150-10 is divisible by

F lo
140 is divisible by in or, in is a divisor of 140

ee
^ m = 2, 4,5,7,10,14, 20,28, 35,70,140 [●.● 777 > 0 and 777 ^ 1]

Fr
::XAMPI.L ● If 28 = x (mod 6), find the set of values ofx.
SOLUTION We have, for
28 = X (mod 6)
r
^ 28-.V is divisible by 6
You
s

^ 28-j: is a multiple of 6
ook

^ 28-.V = 6{j, where eZ


eB

.r = 28 - 6q, where q eZ
Putting g =..., -2, -1, 0,1, 2,..., we obtain
our

x = ..., 40, 34, 28, 22,16,


ad

Hence, the required set is


.., 40, 34, 28, 22,16,...[ or {.v: x = 28 - 6q, where q e Z|.
dY

EXAMPLE 5 Ifx is an odd integer, prove that x^ =1 (mod 8).


Re

SOLUTION Since x is an odd integer. Therefore, .v = 2?j +1 for some integer n


Fin

Now, .Y = 2n + ) ,n eZ
=> Y^ =(277 + 1)^
=> Y^ = 477^+ 477 + 1
=> Y^ = 477 (77 + 1) +1
Y^ -1 = 477 (77 + 1) ●●●(i)
We know tliat the product of two consecutive integers is always even
77 (77 1) = 27i for some X. e Z. ...(ii)
From (i) and (ii), we obtain
y^-1=8X.
=> Y^-1 is divisible by 8
=> Y^ = 1 (mod 8)
MODULAR ARITHMETIC 1.5

EXERCISE 1.1

1- Which of the following statements are true?


(i) 3=-2 (mod 5) (ii) no ^5 (mod 15) (iii) 25 = 3 (mod 11)
(iv) -110 =-5 (mod 15) (v) 21 s 181 (mod 40) (vi) 39 =12 (mod 7)
Find a value of ni between 70 and 80 satisfying: 14 s 172 (modulo m).
3. Find the set of values of m satisfying:
(i) 23 = 29 (mod m) (ii) 58 = 4 (mod m)
4. Find five values of m satisfying : 21 =181 {modm).
If 100 = .r (mod 7), find the least positive value of .t.
ANSWERS

I. (i) True (ii) True (iii) True (iv) True (v) True (vi) False
2. 79 3. (i) {2,3,6}(ii) 12,3,6,9,18,27,54) 4. 2,4,8,10,40 Z. X = 2

w
1.4 PROPERTIES OF CONGRUENCES

F lo
Following are some useful properties of congruences which are stated and proved as theorems.
THEOREM 1 Let n, b be ani/ two integers and m be a positive integers. Then,
n=b (mod m) o a = b + mq for some integer q.

e
Fre
ntOOl First, let =1? (mod m). Then,
a=b (modm)
for
{a-b) is divisible by m or, m is a factor of {a-b).
r
There exists an integer qsuch that a-b = mq or, a=b + mq.
You
oks

Thus, a =b (mod /n) => a = b+mq for some integer q ... 1


eBo

Conversely, let
a = b + mq for some integer q.
a-b = mq for some integer q.
ad
our

{a-b)
a = b (mod m)
a = b + mq for some integer q ^ a = b (mod m) ● ●●(ii)
dY
Re

From (i) and (ii), we obtain


Fin

a=b (mod m) a =b+mqfor some integer q.


Q.E.D
REMARK The above theorem states that: a=b (mod) a = b +mqsome integer q.
Let us now divide b + mq by m using long division as given below:
m mq + b ( ^
mq
b

We find that b is the remainder when b + mq i.e. 'a' is divided by m.


Thus, finding an integer 'b' (smaller than m) congruent to an integer 'a' modulo m means finding
the remainder when 'a' is divided by m.
For example, 33=1 (mod 4) means 1 is the remainder when 33 is divided by 4. Likewise
2^ = 3 (mod 5) means when 2^ is divided by 5, the remainder is 3.
1.6 APPLIED MATHEMATICS-XII

ILLUSTRATION Find the value of X in the set {0,1,2, 6| such that 5311A = a: (mod 7).
SOLUTION Clearly, x is the remainder when 53114 is divided by 7. When 53114 is divided by 7
the remainder is 5. Hence, a: =5.
THEOREM 2 Let a,b be any tzuo integers and m be positive integer. Then,
a=b (mod m) o a and b have the same remainder when divided by m.
PROOF Let r-[ and /2 be the remainders when a and b respectively are divided by m. Then, there
exist integers and qj such that
a = mqi + rj , 0 <r^ <m -(i)
b = m^2 + ^2' 0 < r2 < HI
First, let fl = cf (mod hi). Then,
a = b (mod hi)
HI I {a-b)

w
in\imq^+r-^)-{mq2+r2) [Using (i)]
ni\m{q-^-q2) + {r^-r2)

F lo
But, m\miq-i-q2).
Thus, we have

ee
m\m{qi-q2) + (i*| -12) andm| hi(^-j -1^2)

Fr
Hil HI (i/i -£/2) + (i-| -rj) - HI iqi -1/2) [●.● a I b and a\c ^ a\b± c]
Hi!(ri-i2) for
(^1 -12) = 0 [-.■ 0 <rj <Hi and 0 <12 <m ^ 0 <rj -r2 <hi]
ur
^1=^2
s
ook

Thus, a = b (mod m) a and b have the same remainder when divided by hi. -(ii)
Yo

Conversely, letnandii have the same remainder when divided bymi.e. }\ =r2.Then, from (i), we
eB

obtain

a = mqi + and b = 1111/2 ^1


our

=>
a-b = (im/i + r{) -(i?it/2 +12)
ad

a-b=m{qj-q2)
HI I (a-b) [v m\m{q^-q2)\
Y

a = b (mod hi)
Re
nd

Thus, a and b have are the same remainder when divided by hi ^ h s b (mod hi) ... (iii)
Fi

From (ii) and (iii), we obtain


a=b (mod hi) <» a and b have the same remainder when divided by m.
Q.Fi.D
RLMARK From the above two theorems, we find that the following statements are equivalent:
(i) asb{modm)
(ii) a = b+ rnq for some integer q
(iii) a and b have the same remainder when divided
by m
Consequently, any one of these two statements can be taken as the definition of congruence.
THEOREt.^
Let a be any integer and m be a positive integer. Then, a = 0 (mod m) <=> m \ a.
.';<OOI First, let fl = 0 (mod hi). Then,
a = 0 (mod hi)
HI I (fl-0)
MODULAR ARITHMETIC 1.7

m a.

Conversely, let m \ a. Then,


m\a
m| (rt-0)
^ rt = 0 (mod m)
Hence, s 0 (mod w) <=> m | a
Q.E.D
THEOREM 4 Let a, b, c be any three integers and m be a positive integer. Then,
(i) i7sf7(modw)
i.e. the relation "congruence modulo m" is reflexive,

w
(ii) a=b{modm)=>b=a{modm)
i.e. the relation "congruence modulo m" is symmetric

Flo
(Hi) a=b (mod m) andb = c (mod m) => a = c (mod m)
i.e. the relation "congruence modulo m" is transitive,

e
(iv) asbimodm) ^-n = -b{modm)

re
PROOF (i) For any integer a, we have

F
ur
r
a-a = 771^0 fo
m \ {a-a)
ks
a = a (mod »0
Yo
oo

(ii) Let rt sb (mod m). Then,


eB

a=b (mod w)=> m| (a-b) => j -{b-a) => m| {b-a) => b (mod m)
(iii) Let a, b, c, be integers such that a=b (mod w) and b = c (mod m).Then,
ur

asb (mod »/) and b = c (mod ?«)


ad

tn\ (a-b) and w| {b-c)


Yo

7n\ (a-b) + {b-c)


7n \ a-c
d
Re
in

a = c (mod t/i)
F

(iv) fls/7(modm)
m I a -b
m \ -{a-b)
m \ {-a +1)
m\{{-a)-{-b))
-a = -b (mod m)
THEOREM b Let a, b, c be any three integers and 7n be a positive integer. Theii,
(i) a=b (modm) z=> a = b +t7iq{modm) for every integer q

i.e. any /nultiple or multiples ofm can be added to or removedfroin either side of a congruence modulo m.
(ii) a = b + c (mod m) ^ a-c sb (mod »i)
i.e. a term on either side of a congruence can be transferred to the other side by changing its sign.
1.8 APPLIED MATHEMATICS-XII

LKQOJ (i) a = b (mod m)


m \ (a-b)
rn \ a-b -mq [●.● m I mq :.m \ o-b and m \ mq m\a-b -mq]
m \ a-{b + mq)
a=b + mq (mod m)
(ii) (7 = + c (mod m)
^ m\a-{b + c)
m\{{a-c) -b))
(n-c)=b (mod m)
ILIUSTRATION
79 = 31 (mod 8). Using Theorem 5 (i), zoefind that 79 s 31 - 8x 3 (mod 8)
i.e. 79 =7 (mod 8).
We find that 53 =102 +41 (mod 15). By using Theorem 5 OV), 53-41 =102 (mod 15)

w
THEOREM 6 Let a, b, c, d be integers and m be a positive integer. Then,
(i) a =b + c(mod m)and c = d (mod m) => a =b + d{modm)

F lo
i.e. any term in a congruence (mod m) can be replaced by another term congruent to it (mod m).
(ii) a=bc (mod m) and c=d (mod m) => a = bd (mod m)

ee
i.e. in a congruence [mod m) any factor of a term can be replaced by another factor congruent to

Fr
it (mod m).
PROOF (i) We have, a = b + c (mod m) and c = d (mod m)
m\ a-{b + c) and ??i| (c-d)
for
ur
m \ a-(b + c) +{c-d)
s
m\a-b-d
ook
Yo

m\a-{b + d)
a=b + d (mod m)
eB

(ii) We have.
a = bc (mod m) and c=d (mod m)
r
ou
ad

m I (fl -be) and m | (c - d)


m\ (a-bc) + b (c-d) [●.' m I a and m\^=> m \ pa + q^]
Y

m \ a-bd
Re
nd

a=bd (mod m)
Q.E.D
Fi

ILLUSTRATION We observe that 53 =102 + 41 (mod 15) and 41 = -4 (mod 15). By using Theorem 6
(i), zve obtain 53 = 102 - 4 (mod 15)
We also observe that 247 = 43 x 4 (mod 15) and 43 = -2 (mod 15). By using Theorem 6 (ii), zve find that
247 = (-2) X 4 (mod 15).
THEOREM 7 Let a, b, c, d be integers and m be a positive integer such that a=b(modm) and
c = d (mod m). Then,
(i) a + c=b + d (mod m) (ii) (a-c)=(b-d) (mod m) (Hi) ac = bd (mod m)
(i) We have,
a=b (mod m) and c=d (mod m)
w| (a-b) andm| (c-d)
m| (fl -b) +(c-d)
m\(a + c) -(b + d)
a + c=b + d (mod m)
MODULAR ARITHMETIC 1.9

(ii) We have.
a=b (mod m) and c = d (mod m)
m| {a~b) and (c-d)
m \ {a-b)~{c-d)
m\{a-c)-{b-d)
{a~c)^ib-d)
(iii) We have,
a=b (mod m) and c = d (mod m)
m\ (a-b) andm| (c-d)
m \ c (a-b)+b (c-d)

w
m I ac -bd
ac H bd (mod m)

F lo
Q.E.D.
THEOREM 8 Let a^,bj,bj-,i = l,2, ...,n be integers and m be a positive integer. Such that
Oj =bj (mod m),i =1,2, ...n Then,

ee
(i) a-i + a2 + ... +a,fcj + &2 + ● ● ●+ b„ (modm)

Fr
(ii) 0^02 ... o,j = bib2 ... b,i(modm)
for
(iii) Cl a-i + C2 02 + ...+ c,j a,, =Ci bi+ C2b2 + ... + c,j b„ (mod m), where Ci, C2, ■■■■ are integers.
ur
PROOF Left as an exercise because it is direct generalization of the theorem 7.
REMARK Replacing Hi, 02,..., by a and b], &2/ ●●●, i’n bi/b in (ii) of the above theorem, lue obtain
s
ook
Yo

a=b (mod m) ^ =b’' (mod m)


eB

ILLUSTRATION (i) We find that


4 sll (mod 7), 26 =19 (mod 7) and 79 = 9 (mod 7)
(i) Using Theorem 8 (i), we obtain
r
ou
ad

4+ 26+ 79 =11+19+9 (mod 7)


(ii) Using Theorem 8 (ii), we obtain
Y

4x26x79 =11x19x9(mod 7)
Re
nd

(iii) Using Theorem 8 (iii), we obtain


Fi

2x4+(-l)x26+ 3x79 =2x11+(-l)x 19+3x9 (mod7)


THEOREM 9 Let a, b be tzuo integers and let m be a positive integer. Then,
(i) a=b (mod ??z) => (a + .r) = (b + x) (mod m) for any integer .x.
(ii) a=b (mod in) => (n-x) ={b - .v) (modzn) for any integers,
(iii) a=b (mod in) ^ ax = bx (mod in) for any non-zero integer x.
(iv) a = b (mod m) ^
a
Mmod m\ , if X is a divisor of a, b and m.
X x\ X J

a b f mod fjj \
(v) a = b (mod m) ^ — , if X is a divisor of a and b and (x, m) = d.
X X d

PROOF (i) a = b {modm)


=> m I (a -b) => m \ {(a + x)-(b + x)) => (a + x)={b + x) (mod m)
(ii) a=b (mod m) ^ m \ (a-b) => »z| {(fl-.x) -(b -x)| ^ (a-x) ={b -x)
1.10 APPLIED MATHEMATICS-XII

(iii) n=b (modm) => m\ {a-b) =>m\{a-b) x => m\ {ax-bx) => ax =bx(mod m)
(iv) a sb (mod m)
m\ (a-b)
{a -b) = mq for some integer q
a b

X X \xj

111 a b^
X X XJ
a bf m m
mod — a: is a divisor of a, b, and in — and are integers
a: a: V xj a: a: X

(v) a = b (mod m)

w
m\ (a-b)
a b

F lo
in X

\x X

?jid'\x
I ' rt bx)dX

ee
d={x,m) and ^ are integers

Fr
m a b m X
=1
d'[ X X) for [I'd
ur
a b f mod—
= —
,m
X d)
s
ok

Q.E.D.
Yo
o

COROLLARY let a, b, c be integers and m be a positive integer such that {c, m) =1 i.e. m and c are
eB

relatively prime. Then,


ac = be (mod m) => a=b (mod m)
r

I’KOPP ac = be (mod ;h)


ad
ou

be ( mod
ac

c c \
—\
d)
where d = (c, m)
Y

a=b (mod m) [v d={c,m) =1]


Re
nd

THEOREM 10 let a, b beany two integers and m be a positive integer. lfn>l is a divisor of m, then
Fi

a=b (mod m)=^ a =b (modn)


I’KOOt
a=b (modm) ^ m \ (a-b)
11 is a divisor of ui ^ n | m ...(ii)
From (i), and (ii), we find that
n I in and m \ {a-b) => «j (a-b) =^> a = fj (mod «)

'■HEf^=!Fn 1
Let a, b be integers and m be a positive integer. Ifa=b (mod???)/ ^ben {a, m) ={b, m)
Let frj ={a,m) and ^2 ={b, w). It is given that a =1? (mod???). Therefore,
A = +????/ for some integer q
Now,
rfj =(a, m)
dj [ A and fr-j I m
d-^ I a-mq and d^ 111
MODULAR ARITHMETIC 1.11

=>
rfj I b and ifj | m [v a = b + mq :.a-mq=h]
di is a common divisor ofb and mboth
dl ^ ^2 ... (i) c/2 is thie greatest common divisor ofb and m]
and, d2 ={b, m)
d2 I b and ^2 m

d2\b +mq and t/2 I


=> ^2 I Cl and ^2
(^2 is a divisor of a and m both
^2 — d-^ ... (i) [●.● di is the gcd of a and m]
From (i) and (ii), we obtain
di = ^2 he. {a, m) = {b, m)
Q.E.D.

low
THEOREM 12 Leta,b,c be integers and m^, m2 be positive integers. Then,
ash (mod Wj) and a = c (mod m2) => b =c(mod d), where d =(m], ;»2)-
PROOF d = (wj, m2) i.e. d is the gcd ofm-^ and m2.
=> d\m-Y and if I nh.

ee
Now, a=b (mod mj^) and a = c (mod m2)
F
Fr
=> nil I '”21 ...(ii)
From (i) and (ii), we obtain
for
ur
d I nil I => d \ {a -b)
d\ (a-c)~ {a-b) => d\{b ~c)=> b = c (mod d)
d I m2 and m2 \ {a-c) d\a-c)
s
Q.E.D.
k
Yo
oo

THEOREM 13 If a =b (modulo nij); i = l,2,...,n. then, a=b (mod M), ivhere M is the LCM of
mi, ni2^..., m^i i-c. M = [wp W2, ●●●/ "^il-
eB

PROOF a=b(modmi);/ = 1, 2,..., «


=> nij I (a-b), / =1, 2, 3
r

or. nil I "^2 I (^“^)/ I i^~^)


ou
ad

{a -b) is a common multiple of m^, m2, ■■■, ni,


But, M is the LCM of Wj, m2, ■■■, m^,
Y

M|(fl-f?)
Re

^ rt=b(modM)
nd

Q.E.D.
Fi

REMARK We find that 136 = 10 (modulo 6), 136 = 10 (mod 9) and 136 = 10 (mod 21). Therefore, by
using the above theorem
136 s 10 (mod [6, 9, 21])
?.e. 136 s 10 (mod 126). [●.● LCM (6,9,21) = 126]
THEOREM 14 Let a^ =1 (modjRj), a^ =1 (modm2). Then =1 (mod???)/ inhere z is the and y i.e.
z = l-T, y\and m is the LCM ofnii and m2 i.e. m = [???i^, ??i2|.
L’BQOr Since z is the LCM of .r and y. Therefore, there exist integers p and g such thatz = xp
and z = yq.
Now, si (mod??i|) and s 1 (modm2)
=> (fl^)P s iP (mod nil) (mod ???2)
=> a^ =1 (mod nil) = 1 (mod m2)
=> si (mod ???i) and si (mod ???2)
1.12 APPLIED MATHEMATICS-XII

0^ = 1 (mod m), where = LCM of m-^, m2 [By Theorem 13]


RHMARK We find that 3^ = 1 (mod 8) and 3® = 1 (mod 10).
LCM of 6 and 8 is 24 and LCM of 8 and 10 is 40. Therefore, by using the above theorem, we obtain :
3^"^=l(mod 40)
ILLUSTRATIVE EXAMPLES

Type I ON PROVIDING RESULTS OF THE FORMa^' c(mod »h) WHERE ,1, b, c AND 11, ARE POSITIVE
INTEGERS

EXAMPLE 1 Prove that:


2222 500 700
(i) 2 = 4 (mod 13) (ii) 3 2 2 (mod 7) (iii) 5 2 6 (mod 23)
2222
SOLUTION (i) In order to prove 2 2 4 (mod 13), we first find an integer k such that

w
2^ 2 ± 1 (mod 13).
1
We know that 2=2 (mod 13)

F lo
^ 2^ 22x2=4(modl3)
=> 2^ 2 2x 4 = 8 (mod 13)

ee
=> 2^ 22x8=1623(modl3)

Fr
^ 2^ 2 3x2 = 6 (mod 13)
^ 2^ 2 2x6 =12 2-1 (mod 13)
for
ur
2222
Thus, we find that 2^ 2 -1 (mod 13). Let us now express 2 in terms of 2^as given below.
s

22222 ^2^) 2^
ook
Yo

Now, 2^ 2-1 (mod 13)


eB

(26)370 2 (_i) 370 [●.● a=b (mod m) <7” 2 Jj” (mod ?7/)]
(26)370 =1 (mod 13) [V (-1)370=1]
our

(2^)370x2^21x2^ (mod 13)


ad

[●.● a =b (mod m) => ax = bx (mod ??i)]


22222 24 (mod 13)
Y

(ii) In order to prove 3^00 = 2 (mod 7), let us first find an integer k such that 3^ 2 ±1 (mod 7).
Re

We know that 3^23 (mod 7)


nd

=> 3^23x3 = 922(mod7)


Fi

33=3x2 (mod 7)
=> 33 = 6 2-1 (mod 7)
Thus, we find that 33=-! (mod 7). Let us now express 3^00in terms of 33,
166
3300 = (3 3) ■ 3^
Now,
33=-! (mod 7)
(33)166 =(_i)166 (^qJ 7) [v a (mod m) ^ ^b” (mod rn)]
(33)166.32 =(_;^)166^ 32 (mod 7) [●.● a=b (mod m) => ax 2bx (mod m)]
3500 2 9 (mod 7)
But, 922 (mod 7). Thus, we obtain
MODULAR ARITHMETIC 1.13

= 9 (mod 7) and 9 = 2 (mod 7)


3^°°= 2 (mod 7) [●.* a=b (modm),b=c(modm)=> a = c(modm)]

= 6 (mod 23), let us first find an integer k such that 5^ is congruent to


700
(iii) In order to prove 5
700
either ± 1 or an integer close to ± 1 mod 23. We find thatS = 25 s 2 (mod 23). So, we express 5
interms of 5 as given below.
700 2x350
Clearly, 5 = (5")
Now,
5^=2 (mod 23)
(52)350 ^(2) 350 (mod 23) [v a =b (mod m) => =b^ (mod m)]
=> 5700= 2^5° (mod 23) ...(i)

w
Let us now find an integer k such that 2* = ± 1 (mod 23).

F lo
We find that

2^ s 9 (mod 23)
2^x22=2^x9(mod23)

ree
2^ = 36 (mod 23)
But, 36 s 13 (mod 23)
F
for
2^ s 13 (mod 23)
r
=> 2’'x2s23x2(mod23)
You
oks

2® s 26 (mod 23)
eBo

But, 26 s 3 (mod 23)


2® s 3 (mod 23)
2®x2s3x2(mod23)
our
ad

2® X 2^ s 3 X 2^ (mod 23)
2®x2®s3x2®(mod23)
dY
Re

2^^ s 24 (mod 23)


Fin

2^1 si (mod 23) [v 24 si (mod23)1


350
Let us now express 2 in terms of 2^^ as given below.
2350 2^
Now, 2^^ si
(211)31 =(1)31 (mod 23) [v flsb(modm) ^ fl^sb”(modm)]
(211)31x29^2^ (mod 23)
2®®°s2^ (mod 23)
But, 2^ s 6 (mod 23) [See above]
● 2®®°s6(mod23) ...(ii)

From (i) and (ii), we obtain 5^®® s 6 (mod 23)


1.14 APPLIED MATHEMATICS-XII

n ON FINDING THE REMAINDER WHEN ,r' IS DIVIDED BY AN INTEGER >

EXAMPLE 2 Find the remainder when:


100
(i) 2 is divided by 11. (ii) 2 is divided by 7.
301
(iii) 2 is divided by 5 (iv) 2^^^ IS divided by 5
SOLUTION (i) We know that ii a = b (mod m) and 0 <b <m, then b is the remainder when a is
divided by Therefore, to find the remainder when 2^^^ is divided by 11, its is sufficients find
an integer & such that 2^^*^ st (mod 11), where 0 < 11.
Now,
1
2‘ =2 (mod 11)
2^=2x2=4(modll)
=> 2^ s2x4=8(modll)

w
2“^ s 2x8 =5 (mod 11) [v 2^ sl6(modll)andl6s5(modll).-. 2^ =5(modll)]
2^ =2x5 =10 (mod 11)
2^ = -1 (mod 11)
(2^)20 ={-l)20(modll)
F lo [vl0=-l(modll)]

ree
2^°°=l(modll)

F
100 for
Hence, 1 is the remainder when 2 is divided by 11.
(ii) We find that
1
r
2‘ =2 (mod 7)
You

2^=2x2 = 4(mod7)
oks

2^ = 2x4=8(mod7)
eBo

2^ =1 (mod 7) [v 8 si (mod7)]
30]
3^100
(2") = (l)^“°(mod 7) [v2 ^(23)100^2]
our
ad

=> 2300 = l(mod7)


2x2^“s2xl(mod7)
dY
Re

2^°^ s 2 (mod 7)
Fin

301
Hence, 2 is the remainder when 2 is divided by 7.
(iii) We find that
2^ s 2 (mod 5)
2^ s2x2 = 4(mod5)
2^ = -1 (mod 5)
301
2x150
(2") = (-l)^^°(mod 5) [●●● 2 ^(22)150 ^2]
2300 = 1 (mod 5)
2300 X 2 six 2(mod5)
2301 s 2 (mod 5)
301
Hence, 2 is the remainder when 2 is divided by 5.
(iv) We find that
2^ = 4s-l (mod 5)
MODULAR ARITHMETIC 1.15

{2^)50 =(_i)su (jnod5)


2IOO = 1 (mods)
100
Hence, 1 is the remainder when 2 is divided by 5.
EXAMPLE 3 Find the least non-negative remainder when:
(i) 3^^ is divided by 7 (ii) is divided by 13
SOLUTION (i) We find that:
3^ = 3 (mod 7)
3^ =3x 3 = 2(mod7)
3^ =3x2 = 6=-l(mod7)
==. (3^)”^^ =(-l)^^ (mod7) = (3^)^^x3^]
50
[●●● 3

(3^)^^ =1 (mod7)

w
(3^)^^x 3^ =lx 3^ (mod7)

F lo
3^^ =9 (mod 7)
:=> = 2 (mod 7) [●.● 9 = 2 (mod 7)]
50
Hence, 2 is the remainder when 3 is divided by 7.

e
Fre
(ii) We find that:
3^= 3 (mod 13)
for
32= 3x3=9(modl3)
r
3^s 3x9 = 27 (mod 13)
You
oks

3^=1 (mod 13) [v 27=1 (mod 13)]


eBo

3v833
(3") = (l)^^^=l(modl3) [●●● 3
2500
= (3^)®^^x3^]
(33)833 X 3 =1X 3 (mod 13)
ad
our

32500 = 3 (mod 13)


2500
Hence, 3 is the remainder when 3 is divided by 3.
EXAMPLE 4 Find the least non-negative remainder when:
Re
dY

(i) 5^^ is divided by 7 (ii) 5^^ is divided by 7


(ii) 5^^ is divided by 7 (iv) 5^^ is divided by 9
Fin

SOLUTION (i) We find that


5^ =5 (mod 7)
=> 5^ =5x5 = 25(mod7)
=> 5^ =4 (mod 7) [●.● 25 = 4 (mod 7)]
=> (5^)^ =4^ =16 (mod 7)
=> 5^ = 2 (mod 7) [v 16 =2 (mod 7)]
=> (5^)^ =2^ =4 (mod 7)
=> 5® =4 (mod 7)
Thus, we have
5^=4 (mod 7) and 5® = 4 (mod 7)
1.16 APPLIED MATHEMATICS-XIl

5^x5® s4x4(mod7) [●-● a = b (mod m), c = d (mod m) => ac=bd (mod m) ]


=16 (mod 7)
=2 (mod 7) I-.- 16 = 2 (mod 7)]

Hence, 2 is the remainder when 5 is divided by 7.


(ii) From (i), we find tliat
5^ =5(mod7),5^ s4(mod 7) and 5® =4(mod7)
5^x5^x5® =5x4x4 (mod 7)
=> 5^^ =5xl6(mod7)
5^^=5x2=10(mod7) [●.● 16 = 2 (mod 7)]

5^^ = 3 (mod 7) [v 10 s 3 (mod 7)]

w
Hence, 3 is the remainder when 5^^ is divided by 7

F lo
(iii) From (ii), we find that
5® = 4 (mod 7)
(5^)2 =4^=16 (mod 7)

ee
Fr
5^^ =2 (mod 7) fv 16 =2 (mod 7)]
Thus, we have, for
5"^ s 2 (mod 7) and 5^^ = 2 (mod 7)
ur
5^x5^^s2x2(mod7)
s

=> 5^^ =4 (mod 7)


ook
Yo

Hence, 4 is the remainder when 5^^ is divided by 7.


eB

Al.lTER (i) We find that


5^s5(mod7)
our
ad

5^s5x5=25(mod7)
5^ s4 (mod 7)
Y

5^s5x4 = 20 (mod 7)
Re

=>
nd

5^s-l (mod 7)
Fi

But, 5^*’ =(5^)^x5


5^ s-1 (mod7)
(5^)^s(-1)3=-1 (mod 7)
(53)3x5 =-5 (mod 7)
s 2 (mod 7) [●.● -5 s 2 (mod 7)1

Hence, 2 is the remainder when 5^^ is divided by 7.


(ii) From (i), we obtain
5^ s-1 (mod7)
(5^)^s(-l)3=-i (mod7)
(5^)^x5^ =-1x5^ (mod7)
MODULAR ARITHMETIC 1.17

5^^h-25 (mod?)
5^^ s 3 (mod 7) [v -25 = 3 (mod 7)1
Hence, 3 is the remainder when 5^^ is divided by 7.
(iii) From (i), we obtain
5^ =-l (mod?)

5^=-l(mod7)
(5^)^ =(-1)^=1 (mod?)
(5^)^x5^ =1x5^ =25 (mod ?)
5^^ =4 (mod?)

low
[v 25 =4 (mod 7)]
Hence, 4 is the remainder when 5^ is divided by ?.
(iv) We find that
s'* =5 (mod 9)

ee
5^ =5x5=25 (mod 9)

F
Fr
5^=7 (mod 9) [v 25 =7 (mod 9)]
53=5x7 = 35 (mod 9)
for
ur
5^ =-l (mod 9) [v 35=-l (mod 9)]
But, 5*°** =(5^3)33^5
ks

53 =-l (mod 9)
Yo
oo

(53)33 =(-1)33 =_i 9)


eB

(53)33x5^(-l)x5(mod 9)
5^°° =-5 (mod 9)
r
ou
ad

5IOO = 4 (mod 9) [●.● -5 =4 (mod 9)]


Y

100
Hence, 4 is the remainder when 5 is divided by 9.
EXAMPLE 5 Find the non-negative remainder when:
nd
Re

30
(i) 7 is divided by 5. (ii) 9*** is divided by 11.
Fi

SOLUTION (i) We find that


7* =7 =2(mod5)
7^=7x2=14 (mods)
7^ =-l (mods) [●.● 14=-l(mod5)]
730 ^ (72)5
7^=-l (mod 5)
(7^)13 ^(^1)15 (mods)
730=-! (mods)
7 30 =4 (mod 5) [●.● -1=4 (mod 5)]
30
Hence, 4 is the remainder when 7 is divided by 5.
30
XI.ITHK 7 = 23^ (mods) [●.● 7=2 (mod 5)]
1.18 APPLIED MATHEMATICS-XII

Now, 2^ =16=1 (mod 5)


(2Y^(lf ^l{mod5)
=> (2^/X 2^ =1x2^ (mod 5)
2^“ =4 (mod 5) ...(h)
Thus we find that 7^^ =2^^ (mod 5) and 2^*^ =4 (mod 5).
7 ^^^4 (mod 5)
Hence 4 is the remainder when 7 is divided by 5.
(ii) We find that
9^ =9 =9 (mod 11)
9^=81 (mod 11)

w
9^ =4 (mod 11) [●.● 81s 4 (mod 11)]

9^ = 9x 4 (mod 11)

F lo
=>

9^ = 36 s 3 (mod 11)
9^^ s 3x9 (mod 11)

ee
9^ s5 (mod 11)

Fr
[v 3x9=27s5(modll)]
9^ s5x9=45 (mod 11) for
9^ si (mod 11) [●.● 45 si (mod 11)]
r
(9^)^s(l)2=i(modll) [v 9^°=(9V]
You
s
ook

=> 9^°sl(modll)
Hence, 1 is the remainder when 9^^ is divided by 11.
eB

EXAMPLE 6 Find the non-negative remainder when:


151
(i) 3 - 2^^ is divided by 5
X
(ii) 2 3^^ is divided by 7
our
ad

SOLUTION (i) We find that


2^ s 2 (mod 5)
dY

2^ s 4 (mod 5)
Re

2^ s 8 (mod 5)
Fin

2^ s 3 (mod 5) [v 8s3(mod5)]

2^x2s3x2 = 6(mod 5)
2^ si (mod 5) [v 6 si (mods)]
But, 250 ^(24)12 ^2^
2^ si (mod5)
(2^)12 ^(1)12
(2'^)^^ si (mod5)
(2^)^^X 2^ six 2^ (mods)
2^° s 4 (mod 5) ...(i)
MODULAR ARITHMETIC 1.19

Now,
3^ = 3 (mods)
=> 3^ = 3x 3 = 9 (mods)
=> 3^ = 4 (mod S) [v 9 =4(mods)]
=> 3^ = 3x4 = 12 (mods)
=> 3^ =2 (mods) [v 12 = 2 (mods)]
=> 3^ =3x2 = 6 (mods)

ow
=> 3^=1 (mods) [v 6=1 (mods)]
=> (3Y°=(l)^°(modS)
=> 32^°=l(modS) ... (ii)

e
From (i) and (ii), we obtain

re
2^® = 4 (mod S) and 3^®° = 1 (mod S)

Frl
F
^ 2^® X 3^®° = 4x1 (mods)
^ 2^0x3200 = 4 (mod S)
ou
r
Hence, 4 is the remainder when 3^^ x 2^® is divided by S
(ii) We find that
so
kf
2^ =2 (mod 7)
oo
2^=4 (mod 7)
Y

2^ =2x4 = 8(mod7)
B

2^=1 (mod 7) [v 8=1 (mod 7)]


re

But, 2^"’'=(2^)^°x2 1
oY
u

2^=1 (mod 7)
ad

(23)50 (mod 7)
d

(2^°=l(mod7)
in

(23)50x21 =2 (mod 7)
Re
F

=> 2^^^ =2 (mod 7) ...(i)

Now, 3^ = 3 (mod 7)
32=9 = 2(mod7)
3^ =3x2 = 6 (mod 7)
=> 3^=-l (mod 7) [-.* 6 = -1 (mod 7)]
But, 3200=(3Y^x32
3^=-l (mod 7)
(33)66 =(_i)66(mod7)
=> (3Y^=l(mod7)
(3Y^x32=lx32^9(mod7)
=> 3200 =2 (mod 7) [v 9 = 2(mod7)] ...(ii)
1.20 APPLIED MATHEMATICS-XII

From (i) and (ii), we obtain


s 2 (mod 7) and = 2 (mod 7)
2151 x3^‘’“=2x2=4(mod7)
Hence, 4 is the remainder when 2^^^ x 3^^ is divided by 7
EXAME’LE 7 Prove that the remainder when 2^*^ is divided by 341 is 1.
SOLUTION We find that: 341 = 11 x 31 and 340 = 68 x 5

Now, 2^ =5 (mod 11) => 2^ = 2x5 =10 (mod 11) => 2^ =-l (mod 11)
(2^)^® =(-l)^® =1 (modll)
2340 si (mod 11) ...(i)

Similarly, 2^ = 32 s 1 (mod 31)

w
(2^)^^ s(l)68 3^j
2^ si (mod 31) ...(ii)
From (i) and (ii), we obtain
2340 = 1 (mod 11) and 2 340 = 1 (mod 31)

F lo
ee
2340 s 1 (mod LCM of 11 and 31) [Using Theorem 14]

Fr
2340 si (mod 341).
Hence, 1 is the remainder when 2
340
is divided by 341.
for
ur
KLMAKK Every positive integer can be expressed as the sum of powers of 2. This fact can also be used to
s

find the remamder ivhen numbers having large exponents are divided by a given positive integer as
ook
Yo

illustrated in the folloiuing example.


287
EXAMPLE 8 What is the remainder when 3 is divided by 23?
eB

SOLUTION Let us first write the exponents 287 as the sum of the power of 2.
287 =2®+ 2^+2^+ 2^+2^1 =256 + 16 + 8 + 4+2 + 1
our
ad

3287 ^ 3256 ^ 3I6 X 3® X 3^ X 3^ X 3^


Let us now find numbers congruent to 3^^^, 3^^, 3®, 3^, 3^ and 3 under modulo 23.
Y

Now,
Re

3^ s 9 (mod 23) ●●●(i)


nd

(3^)^s9^ (mod 23)


Fi

=> 3^ s 81 (mod 23) ...(ii)

3"^ s-11 (mod 23) [v 81s-11 (mod 23)]


(3^)2 =(-11)^ (mod 23)
=> 3® sl21 (mod 23)
3® s 6 (mod 23) [●.● 121 s 6 (mod 23)] ... (iii)
(3®)^s6^ = 36(mod 23)
3^^ s -10 (mod 23) [v 36 s-10 (mod 23)] ... (iv)
(316)2 =(-10)2 =100 23)
3^^ s8(mod 23) [v 100 s 8 (mod 23)]
(332)2 ^g2^ 64 23)
MODULAR ARITHMETIC 1.21

3^ = -5 (mod 23) [v 64 s-5 (mod 23)]


(3^)2 =(_5)2 ^25(mod 23)
3^^® s 2 (mod 23)
(3128)2 ^2^ 23)
=j> 3^^^ s 4 (mod 23) ●●●(v)
3287 ^ 3256+ 16+8+4+2+ 1 ^ 3256 ^ 3I6 ^ 38 ^ 34 ^ 32 ^ 3I
3287 s 4 X (-10) x6x-11x9x3 (mod 23) [Using (i) — (v)]
3287 s-40 X-66x27 (mod 23)
3287 = 6x 3x 4 (mod 23) [■-' -40 s 6 (mod 23),-66 s 3 (mod 23) and 27 =4 (mod 23]
3287 S 72 (mod 23)

w
3287 s 3 (mod 23) [●.● 72 s 3 (mod 23)]
287
Hence, 3 is the remainder when 3

F lo
is divided by 23.
121
EXAMPLE 9 Find the remainder when 5 is divided by 17.
SOLUTION First, we express 121 as the sum of the powers of 2.

ee
Clearly, 121 = 64 + 32 +16 + 8 +1 = 2^ + 2^ + 2*^ + 2 ^ + 2°

Fr
5l2U564^532...16..c8,..l
x5^'^x5"x5

Let us now find numbers congment to5^,5^^,


for
and 5^ under modulo 27.
Now,
r
5^ = 25 s - 2 (mod 27)
You
s
ook

(5^)^s(-2)^=4(mod 27)
eB

5^ s 4 (mod 27)
(5^)^ s 4^ =16 (mod 27)
our
ad

5® s-11 (mod 27) [v 16 s-11 (mod 27)] ...(i)


(5^)^ s(-ll)2 ^121 (mod 27)
5^^ ^13 (mod 27)
dY

[v 121s 13 (mod 27)] ... (ii)


Re

(5^^)^ ^13^ =169 (mod 27)


Fin

5^^ s7 (mod 27) [v 169 s7 (mod 27)] ... (iii)


=> (5^^)^ s7^ =49 (mod 27)
^ -5 (mod 27) [v 49 s-5 (mod 27)] ...(iv)
5 121 .564+ 32+ 16+ 8+ 1
5^21 = 5^x532x5'^x5«x5
5I21 = _5x7xl3x-llx5(mod 27) [Using (i) — (iv)]
5121 ^_35j^i3^_55 27)
5^21 = -8xl3x-l (mod 27) [●●● - 35 s -8 (mod 27), -55 s -1 (mod 27)]
5^^^ s 104 (mod 27)
5^^^ s23 (mod 27) [●●● 104 s 23 (mod 27)]
121
Hence, 23 is the remainder when 5 is divided by 27.
1.22 APPLIED MATHEMATICS-XI!

EXAMPLE 10 Showtiwtl^^ + 1 is divisible by 641.


SOLUTION In order to show that 2 ^ +1 is divisible by 641, it is sufficient to show that 2 +1

leaves remainder to 0 when divided by 641 i.e. 2 +1=0 (mod 641) or 2 = -1 (mod 641)
Now,
4
2^ s 16 (mod 641)

(2^)^ =16^ =256 (mod 641)


2^ = 256 (mod 641)
(2^)^ =(256)^ (mod 641)
2^^ =154 (mod 641) [v (256)2 =154 (mod 641)]

w
(2^^)^s (154)2 (mod 641)

F lo
=> 2^2=640 (mod 641) [v (154)2^640 (mod 641)]
2^2 =_i (mod 641) [●.● 640 = -1 (mod 641)]

ee
=>

Fr
=> 22^+1=0 (mod 641)
Hence 2 +1 is divisible by 641.
for
ur
EX.A.MPLE 11
Find the least non-negative remainder when:
(i) 89 X 111 X135 is divided by 11 (ii) 64 X 65 X 66 is divided by 67
s
ook
Yo

(ii) 175 X 365 x 69 is divided by 17


SOLUTION (i) The least non-negative remainder when 89x111x135 is divided by 11 is the
eB

least non-negative integer r (0 <r < 11) such that 89x 111 x 135 = r (mod 11). We find that
89 si (mod 11), 111 si (mod 11) and 135 s 3 (mod 11)
r
ad
ou

89x111x135 si xlx 3 (modll)


89 X 111 X135 s 3 (mod 11)
Y

Hence, required remainder is 3.


Re

(ii) We find that


nd

64 s -3 (mod 67), 65 = -2 (mod 67) and 66 = -1 (mod 67)


Fi

64 X 65 X 66 s -3 X -2 X -1 (mod 67)
64 X 65 X 66 s -6 (mod 67)
64 X 65 X 66 s 61 (mod 67) - 6 s 61 (mod 67)]
Hence, required remainder is 61.
(iii) We find that
175 = 5 (mod 17), 365 s 1 (mod 17) and 69 s 1 (mod 17)
175 X 365 X 69 s5 X1 X1 (mod 17)
175x 365x69s5(modl7)
Hence, required remainder is 5.
Tlifie HI ON FINDING THE DIGIT AT UNIT’S PLACE OF A GIVEN INTEGER.
We know that the units digit of a positive integer 'a' is the remainder when it is divided by 10.
Therefore, if ^7 s r (mod 10), 0 < r < 10, then r is the digit at units place of integer a.
MODULAR ARITHMETIC 1.23

EXAMPLE 12 Find the units digit in each of the following:


37 132 291 500 183
(i) 13 (ii) 11 (iii) 7 (iv) 6 (v) 183!+ 3
SOLUTION (i) We find that
13 = 3 (mod 10)
13^ =3^ =9 (mod 10)
13^ =~1 (mod 10) [●.● 9 s -1 (mod 10)]
But, 13^^ =(13^)^^xl3
13^ =-l (mod 10)
(132)18 ^(_;^)18 (i^o^io)
(13^)^® =1 (mod 10)

w
(13^)^® X13 = 1X13 = 13 (modlO)
13®^ =3 (mod 10) I-.- 13 = 3 (mod 10)]

Hence, units digit in 13’^ is 3.


(ii) We find that
'^7

F lo
e
Fre
11 =1 (mod 10)
132 132
11 = (1) (mod 10)

11^®^ si (mod 10)


for
Hence, the digit at units place of 11^®^ is 1.
r
You

(iii) We find that


s
ook

7^=49s-l (mod 10)


eB

But, 7291 ^(72)145 x7

7^ s-1 (mod 10)


our
ad

(72)145 ^^_1)145 (jj^odlO)


(72)145^7 ^_7 (jnodlO)
dY

7291 = 3 (mod 10)


Re

[v-7 s 3 (mod 10)]


291
Hence, the units digit Ln 7 is 3.
Fin

(iv) We find that


6^ = 36 s 6 (mod 10)
6®s6x6=36 (mod 10)
6® s 6 (mod 10)
In fact 6" s 6 (mod 10) for all n e N.
s 6 (modlO)

Hence, the units digits in 6®®® is 6.


(v) We have, 51 = 120, 61 = 720,71 = 5040 etc.
In fact units digit in/ilforn >5 is 0. Therefore, units digit in 183! is 0. Consequently, units digit in
183 100

183! + 3 is same as the units digit in 3 .


Now, 3^ = 9 s-1 (mod 10) and, 3^®® =(3^)^^ x 3
1.24 APPLIED MATHEMATICS-XII

3^ s -1 (mod 10)
(mod 10)
(32)91 (mod 10)
(sV^x 3s-3 (mod 10)
(SV^x 3=7(modl0)
3^®^ =7 (mod 10)
Units digit in 3^®^ is 7.
Hence, the units digit in 183!+ 3^®^ is 7.
EXERCISE 1.2

w
1 ● Prove that:

(i) 2^^'^=l(modl7) (ii) 3


200
= 1 (mod 10) (iii) 3^°° s 9 (mod 13)
(iv) 10^^^ =5 (mod 7)
2. Find the least non-negative remainder when;
F lo (iii) 2^^ is divided by 41

e
(i) 2^^ is divided by 7 (ii) 2^^^ is divided by 7

Fre
(iv) 2^^*^ is divided by 5 for
3. Find the remainder when

(i) 79 X 101 X125 is divided by 11 (ii) 42 X 44 X 46 is divided by 45


r
(iii) 127 X 65 X 59 is divided by 7 (iv) 34 x 78xl23x217x 365 is divided by 11.
You
oks

4. Prove that:

(i) 76x204 = 6 (mod 7) (ii) 358 X 499 H 4 (mod 8) (iii) 71 x 73 x 75 s 2 (mod 23)
eBo

5. Find the digit in the unit's place of


12 100 13 50 123 51
(0 2 (ii) 2 (iii) 3 (iv) 3 (v) 3 (Vi) 9
100 249 300 41
ad
our

(vii) 9 (viii) 7 (ix)7 (X) 13


6. Find the digit in the units place of
(i) 111^^^ + 111! (ii) 3^°° +35! (iii) 9^^x 3^°°
dY

7. Find the value of a: in the set {0,1,2, 3, 4,5} such that 73583 = .y (mod 6).
Re

ANSWERS
Fin

2. (i) 4 (ii) 2 (iii) 32 (iv) 1 3. (i) 5 (ii) 3 (iii) 6 (iv) 5


5. (i) 6 (ii) 6 (hi) 3 (iv) 9 (v) 7 (vi) 9 (vii) 1 (viii) 7 (ix) 1 (x) 3
6. (i) 1 (ii) 2 (iu) 1 7. 5

HINTS TO THE SELECTED PROBLEMS

1. (i) 2^ =-l (modl7)=^> (2V° =(“!)'''’ (mod 17) => 2^^° ^1 (mod 17)
(ii) 3^ =-l (mod 10) => (3^)^°” =(-l)^^° (modlO)^ 3^*’'’ =1 (modlO)
(iii) 3^=1 (mod 13)
=> (SY^ =(l)^^modl3)
^ 3
198
= 1 (mod 13) => 3^^®x 3^ =lx 3^ (modl3)=> 3^™ =9 (mod 13)
(iv) We find that
10^ = 3 (mod 7)
1.25
MODULAR ARITHMETIC

10^ = 30 =2 (mod 7)
10^=20=-! (mod 7)
But, 10^^^ =(10^)^^^x 3^.
10^ =-l (mod 7)
/-,n3xl71
(10") = (mod 7)

(103)171^ 3^ s-lx 3^ (mod7)


=> 10^^^ s-9 (mod 7)
10^^^ =5 (mod 7) [v -9s5(mod7)]
7. The value of x is the remainder when 73583 is divided by 6.
1.5 LINEAR CONGRUENCE

w
Recall that nx-bis the general form of a linear equation in one variable in algebra. Similarly, if a,
b and tn are positive integers, then nx = b (mod m) is a linear congruence in modular arithmetic.

congruence
6x =5 (mod7)
where .r is an unknown integer.
F lo
For example, 2:c = 3 (mod 5) and ^x = 2 (mod 8) are linear congruencies. Consider the linear

for F
Any value of x which satisfies congruence (i) is called its solution.
ree
If we put a: = 2 in (i), we find that 6x2=5 (mod 7) is true. Hence .v = 2 is a solution of (i). Let us
now put - 5 in (i), we find that 6x-5=-30=5 (mod 7). Therefore, .v = - 5 is also a solution. It can
easily be verified that -12,9,16 etc are solutions of (i). We find that -12,-5,2,9,16 are integers
Your

to 2 is a solution of (i). All these


oks

congruent to 2 modulo 7. In fact every integer congruent


congruent solutions are by convention considered as one and the same solution and the solution
eBo

is written as .r = 2 (mod 7). We could as well have written the solution as .x = -5 (mod 7) or,
x = 9 (mod 7) and so on. But, we choose to write it as .t = 2 (mod 7) because 2 is tiae least
-negative remainder when 6.v —5 is divided by 7 for various values of .v. Also, this is the usual
ad
our

non

and standard practice.


Under the modulo 7, numbers 0,1, 2, 3, 4, 5, 6 are incongruent to each other (mod 7) and any
other integer is congruent to one of these integers. So, let us consider the set
Re

S = {0,1, 2, 3, 4,5, 6} of these integers. Let us now find whether any of other number from S
Y
Find

other than 2 satisfies (i). We find that


6x0 = 0 (mod 7), 6 X 1 = 6 (mod 7), 6 x 2 = 5 (mod 7), 6 x 3 = 4 (mod 7)
6 X 4 s 3 (mod 7), 6 x 5 = 2 (mod 7), 6 x 6 = 1 (mod 7)
Clearly, only ,v = 2saUsfies equation (i). So, 6x = 2 (mod7) has unique solution x=2 (mod7) in
set S.

In the above discussion the linear congruence 6.r=5(mod7) has unique solution in
S = {0,1, 2, 3, 4,5, 6} given by .v - 2 (mod 7). Let us now consider the equation
6x = 3 (mod 9) ...(ii)

We find that 9=0 (mod 9), 10 = 1 (mod 9), 11=2 (mod 9) and so on. So, under modulo 9, there
are 0, 1, 2, 3, 4, 5, 6, 7, 8 distinct integers. Let us now find out which of these integers satisfy
equation (ii).
We find that
6x0 = 0 (mod 9), 6x 1 = 6(mod 9), 6x2 = 3 (mod 9), 6x 3 = 0 (mod 9)
6x4 = 6(mod 9), 6x5 = 3 (mod 9), 6x6 = 0 (mod 9), 6x7 =6 (mod 9), 6x8=3(mod 9)
1.26
APPLIED MATHEMATICS-XII

Clearly, x = 2, a: =5 and x = 8 satisfy equation (ii) and we write the solution as


X = 2,5,8 (mod 9). Clearly, 2,5 and 8 are incongruent to each other (modulo 9).
It follows from the above discussion that the linear congruence ax = b (modm) may have a
unique solution or it may have more tlian one solution in the set S = {0,1, 2,..(?« -1)j. Solutions
are considered as distinct or different iff they arc incongruent to each other (mod m). It is also
observed that the solution(s) (unique or distinct) lie in the set |0,1,2,(w -1)}.
Following theorems help us to determine whetlier ax=b (mod m) has a unique solution or it has
distinct incongruent solutions. We only state these theorems without giving their proofs.
THEOREM 1 The linear congruence ax=b (mod m) has a unicfue solution, if {a, m) = 1 i.e. the greatest
common divisor of a and m is 1 or a and m are relatively prime to each other.

ow
THEOREM 2 Let ax=b{modm) be a linear congruence such that {a,m) = d and d\b. Then,
ax
= b{modm) has exactly d in congruent solution (modm). If a-q is the least solution of
a b f m\
d d
mod —
d J
, then the solutions of ax ^b (modm) are

e
a: satq, -Tq +—, xq + 2 XQ + {d~l) ^d (mod m)

re
d d

rFl
In order to solve linear congruence ax=b (mod m) by method of trial, we may use the following

F
algorithm.
ALGORITHM

r
ou
STEPl
Find the gcd of a and m. Letd= gcd (a, m) i.e. d = (a, m) fo
ks
STf-PII
Ifd =1, then ax =b (modulo m) has a unique solution belonging to the set {0,1, 2,..., (w-1)).
Try in succession x = 0,1, 2, ...,{ni-l) till we find the value of x which satisfies
oo

ax =b (mod m).
Y
eB

If d:^ 11 check whether d divides b or not. if d does not divide b, then ax =b (mod m) has no
solution

bf mA
r

Ifd divides b, then obtain the linear congruence—x s mod — frotn ax=b (mod m). Find
ou

d d{ d )
ad
Y

m
a solution of—x^ —
^ d d mod —
-■ \d
by using the method of trial. Let a:q be the least solution of
u /
d

a b ( m\
- x = ^ mod ^ , then its incongruent solutions are
Re
in

d d \ d)
m
F

X-Xq, Xg + —, A-g +
d
,..., atq +{d-1) — (mod m).
d d

ILLUSTRATIVE EXAMPLES

Ti/pi’ I ON SOLVING LINEAR CONGRUENCE .rv (mod m) WHEN(<r, I,

EX.AMPir I Solve:7x = 3{mod5)


SOLUTION We find that (7, 5) = 1. So, there is exactly one solution which belongs to the set
{0,1,2, 3, 4}. We find that
7 X 0 s 0 (mod 5), 7 X1 s 2 (mod 5), 7 x 2 = 4 (mod 5), 7 x 3 = 1 (mod 5), 7 x 4 = 3 (mod 5).
Therefore, a: = 4 satisfies 7x = 3 (mod 5).
Hence, x = A (mod 5) is the unique general solution of the given linear congruence.
Find the general solution ofSx = 2 (mod 9).
SOLUTION Here, (8, 9) = 1. So, the given equation has only one solution belonging to the set
{0,1,2, 3,..., 7, 8). We find that
1.27
MODULAR ARITHMETIC

8 X 0 s 0 (mod 9), 8 X1 s 8 (mod 9), 8 x 2 = 7 (mod 9), 8 x 3 = 6 (mod 9)


8x4=5 (mod 9), 8x 6 = 3 (mod 9), 8x7 = 2(mod 9), 8x 8 =1 (mod 9)
Clearly, 8x7=2 (mod 9) implies that x = 7 is a satisfies 8x = 2 (mod 9).
Hence, x =7 (mod 9) is the general solution of the given equation.
EXAMri.LS Solve:55x = 66{mod77)
SOLUTION Here, (55,77)=11 and it divides 66. Therefore, 55x = 66(mod77) has 11
incongruent solutions belonging to the set {0,1,2, ..., 77}.
Now,
55.r = 66 (mod 77)
=> 5.V = 6 (mod 7) [Dividing throughout by 11]

ow
We find that (5,7) = 1. So, this linear congruence has unique solution belonging to
!0,1,2, 3,4,5,6). We find that
5 X 0 =0 (mod 7), 5 X 1 =5 (mod 7), 5 x 2 =3 (mod 7), 5 x 3 =1 (mod 7)
5x4=6 (mod 7), 5 X 5 = 4 (mod 7), 5 x 6 = 2 (mod 7)

e
Clearly, 5x4 = 6 (mod 7) implies that atq = 4 satisfies 5.r = 6 (mod 7).

re
rFl
Here, m = 77 and d = 11. So, the incongruent solutions of the given equation are

F
.r =.to/ ^0 +-T'
d
^0 + -T'
a
●●●' ^0 + Tcl (modm)

r
ou
i.e. .t = 4, 4 + 7, 4 +14,..., 4 + 70 (mod 77) sfo
or. X =4,11,18, ...,74(mod77).
k
EXAMPLE! Find the solutions of 91x =42 (mod78).
oo

SOLUTION We find that (91,78) = 13 which does not divide 42.


Y

Hence, 91x = 42 (mod 78) has no solution.


eB

EXAMPLE 5 Solve: (.X -1) (x + 2) = (x - 2) (x + 7) (mod n), if n is odd.


SOLUTION We have,
r
ou

(x-1) (x + 2) =(x-2) (x + 7) (mod n)


Y
ad

x^ + X-2 = x^ +5x-14(mod n)
X ~5x = -14 + 2 (mod n)
d

-4x = -12 (mod n)


Re
in

4x=12(modw)
[●.● 4 is relatively prime to n]
F

X = 3 (mod n)
EXERCISE 1.3

1 ● Which of the following linear congruences has no solution?


(i) 3x = 2 (mod 6) (ii) 4x =1 (mod 3) (iii) 2x = 1 (mod 3)
(iv) 5x = 3 (mod 4) (v) 4x = 2 (mod 12) (vi) 7 X = 1 (mod 8)
(vii) 12x =5 (mod 9)
2. Find the number of incongruent solutions of each of the following linear congruences:
(i) 3x = 6 (mod 9) (ii) 16x = 8 (mod 4) (iii) 12x = 9 (mod 15)
3. Solve the following linear congruences:
(i) 7x=2(modl3) (ii) 2x = 3 (mod 5) (iii) 3x = 1 (mod 5)
(iv) 2x = 4 (mod 6) (v) 8x=12(modl6) (vi) 3x = 4 (mod 5)
(vii) 3x=2(mod7) (viii) 16x = 8 (mod 17) (ix) 33x=42(modl7)
1.28
APPLIED MATHEMATICS'Xll

(x) 37 s 2x (mod 11) (xi) 7a: s 13 (mod 11) (xii) 7 a: s 3 (mod 5)


(xiii) 5.V H -3 (mod 7)
ANSWERS
1. (v), (vii) 2. (i) 3 (ii) 4 (iii) 3
3. (i) a: s 4 (mod 13) (ii) a: = 4 (mod 5) (iii) As2(mod5) (iv) as2(mod 6)
(v) No solution (vi) a: = 3 (mod 5) (vii) A = 3 (mod 7) (viii) a = 9 (mod 17)
(ix) a: = 9 (mod 17) (x) a: s 2 (mod 11) (xi) As5(mod 11) (xii) As4(mod5)
(xiii) A- s5 (mod 7)

ow
1.5.1 MODULAR MULTIPLICATIVE INVERSE

DEFINITION The 7miltiplicative inverse of an integer '<?' under modulo m is an integer


A e|0,1, 2,(j«-l)i ijfflx A =1 (mod m).
Note that the multiplicative inverse of "a modulo m" exists iff a and m are relatively prime i.e.

e
{a. m) = 1.

re
Let a and b be tvv'o positive integers and d be their gcd i.e. d = {a, b). Using Euclidean algorithm
there exist integers a and y such that ax + by= d.

F
Frl
Now, let a and m be two relatively prime integers i.e. their gcd is 1. Then there exist integers x
and y such that ax + my= 1.
ou
Taking modulo m on both sides, we obtain

osr
ax + my = 1 (mod m)
=> rtA + 0 = 1 (mod m) kf [●.● wy = 0 (mod m)j
=> ax s 1 (mod m)
oo
X is multiplicative inverse of 'a' (mod m).
Thus, A i.e. coefficient of 'a' in ax + my = 1 is the multiplicative inverse of 'a' (mod m).
Y
B

It follows from the above discussion that to find the multiplicative inverse of "a modulo m , we
find the gcd of a and m by using Euclidean algorithm and then by using reverse substitution
re

express gcd{a, m) i.e. 1 in the form ax + my=-\. The coefficient of 'a' in ax + my=\ is the
uY

multiplicative inverse of 'a' (mod ??/).


ILLUSTRATION 1 Find the midtiplicative inverse of 27 (mod 392).
ad
do

SOLUTION Let us first find the gcd of 27 and 392 by using the Euclidean algorithm.
392 = 27x14 + 14
[Dividing 392 by 27]
in

27=14x1+13
Re

14=13x1+1
F

Using reverse substitution, we obtain


1 =14-13x1

=> 1 =14-(27-14xl)xl
^ 1=14x2-27x1

1 =(392-27xl4)x2-27xl
=> 1 =392x2-27x29

=> 1 = 27 X (-29)+ 392x2


Thus, -29 is multiplicative inverse of 27 under modulo 392. But, -29 = 363 (mod 392) and the
inverse must be in the set {0,1, 2,..., 391}. Hence, 363 is required multiplicative inverse.
ILLUSTRATION 2 Find the inidtipHcative inverse of 47 under modulo 249.
SOLUTION Let us find the gcd (249,47) by using Euclidean algorithm as given below.
249=47x5+14
MODULAR ARITHMETIC 1.29

47 =14x 3 + 5
14=5x2 + 4

5=4xl+l

Using reverse substitution, we obtain


1=5-4x1

l=5-(14-5x2)xl
1 =-14x 1+5x3

l=-14xl + (47-14x3)x3
1 =47x 3-14x10

1 =47x 3-(249-47x5)xl0
1 =-10x249 + 47x53

The coefficient of 47 is 53. Hence, the multiplicative inverse of 47 under modulo 249 is 53.

w
1.5.2 SOLVING LINEAR CONGRUENCE . r (mod m) BY USING INVERSE OF 'n' UNDER
MODULO

we may use the following algorithm.


ALGORITHM

STRIM

STtl’Il
F lo
In order to solve a linear congruence ax s b (mod m) by using the inverse of 'a' under modulo m,

Obtain the linear congruence ax=b (mod m).


for F
ree
Ifa and m are relatively prime i.e. {a, m) = 1. Use Euclidean algorithm tofind thegcd of a and m.
STniMlI
Apply reverse substitution technique to express gcd of a and m i.e. 1 in terms of a and m i.e.
r
obtain a relation of the form l=ax + myby using back substitution.
You
oks

S l EP IV The coefficient of 'a' in ax + my=l is the required inverse of 'a'. Let it be c.


eBo

su r .11 Multiply both sides ofaxsb (mod m) by c to obtain


cax = cb (mod m) => x = cb (mod m) [v crt=l(modm)]
ad
our

This gives the solution of ax s b (mod m).

ILLUSTRATIVE EXAMPLES
Re
dY

I upe I ON SOLVING LINEAR CONGRUENCES ,/v (mod III) BY FINDING THE INVERSE OF V
Fin

BY INSPECTION

FX.AMIM.f : Solve :7x = 3 (mod 5)


SOLUTION We find that 7 and 5 are relatively prime i.e. (7,5) = 1. The inverse of 7 belongs to the
set |0,1,2,3,4). Out of these five numbers, we have to find a number ‘a' such that7a s 1 (mod 5).
Clearly, 7x3 = 1 (mod 5). So, the inverse of 7 under modulo 5 is 3.
Now,

7xs3 (mod 5)
=> 3 (7 x) = 3 X 3 (mod 5) [Multiplying throughout by the inverse of 7 i.e. 3]
=> (3x7)j: = 9(mod 5)
X = 4 (mod 5) [●.● 3x7 = 1 (mod5) and 9 = 4 (mod5)]
Hence, x = 4 (mod 5) is the required solution.
I \ ●
' Solve : 2x = 37 {mod 11)
1.30
APPLIED MATHEMATICS-XII

SOLUTION We find that (2,11) = 1. The inverse of 2 is one of the numbers 1,2,3, 10 under
modulo 11. Thus, we have to find a number 'a' out of these 10 integers such that 2a = l (mod 11).
By inspection, we find that 2x6=1 (mod 11). So, the inverse of 2 modulo 11 is 6.
Now,

2.v = 37 (mod 11)


6 (2x) = 6x 37 (mod 11) [Multiplying through out by the inverse of 2 i.e. 6]
6X = 222 (mod 11)
=> j: = 2(modll) [●.● 6x2 = 1 (mod 11), 222 = 2 (mod 11)]
Hence, x = 2 (mod 11) is the required solution.
Tx/pe U ON SOLVING ax =b (mod m) WHEN THE INVERSEOF a (mod ?;;) IS FOUND BY
THE EUCLIDEAN ALGORITHM

w
EXAMPLES So/yc7.r = 3 (mod 19).
SOLUTION We observe that (7,19) = 1. The inverse of 7 (mod 19) belongs to the set (0,1,2,...,

find that
19=7x2+5

F lo
18}. So, the inspection method is not very' convenient. Let us use the Euclidean algorithm. We

e
Fre
7 =5x1+2
5=2x2+l

Using back substitution, we obtain


for
1=5-2x2
r
1=5-2(7 -5x1)
You

1 =-2x7 + 3x5
s
ook

1 =-2x7 + 3(19-7x2)
1 =-8x7 + 3x19
eB

The coefficient of 7 i.e. -8=11 (mod 19) is the multiplicative inverse of 7 (modulo 19).
Now,
our
ad

7.r = 13 (mod 19)


ll(7.Y)=llx3(modl9) [Multiplying throughout by 118]
=> (llx7).Y = 33(modl9)
dY

=> -Y =14 (mod 19) [●.● 11x7 =1 (mod 19) and 33 = 14 (mod 19)]
Re

Hence, .y =14 (mod 19) is the required solution.


Fin

EXAMPLE 4 Solve the linear congruence 5x = 12 (mod 23).


SOLUTION Firstwenotethat(5, 23) =l,henceithasuniquesolution(mod23).UsingEuclidean
algorithm, we obtain
23=5x4 + 3
5 = 3 X1 + 2
3=2xl+l

Using back substitution, we obtain


1 =3-2x1

l = 3-(5-3xl)xl
1 =5x(-l)+3x2
1 =5x(-l)+(23-5x4)x2
1 =5x(-9) + 23x2
The coefficient of 5 i.e. -9 = 14 (mod 23) is the multiplicative inverse of 5 (mod 23).
Now, 5y = 12 (mod 23)
MODULAR ARITHMETIC 1.31

14(5x)= 14x12 (mod 23) [Multiplying throughout by 14]


(14 X 5) -V = 168 (mod 23)
x=7 (mod 23) [●-● 14x5=1 (mod 23) and 168 s 7 (mod 23)]
Hence,x-^7 (mod 23) is the required solution.
EXAMPLE 5 Solve: 12x = 44 (mod 59)
a hf m
SOLUTION We know that n =b (mod m) => mod — ,where d={x,m).
X X d

Vlx = 44 (mod 59)


3x =11 (mod 59) [V (4,59)=!]
We find that (3, 59) = 1, hence it has unique solution (mod 59). Using division algorithm, we
obtain

w
59=19x 3 + 2
3=2xl+l

F lo
Using back substitution, we obtain
1=3-2x1

ee
1=3-(59-19x3)x1

Fr
1 =59x(-1) + 20x 3
The coefficient of 3 i.e. 20 is the inverse of 3 (mod 59).
Now,
for
ur
3x sll (mod 59)
[Multiplying throughout by inverse of 3 i.e. 20]
s
=> 20x 3x = 20xll (mod59)
ook
Yo

=> (20x3) a: = 220 (mod 59)


=> .vs 43 (mod 59)
eB

Hence v s 43 (mod 59) is the solution of the given linear congruence.


EXAMPLE 6 Solve: 12x = 9 (mod 15)
r
ou
ad

SOLUTION We find that d =(12,15) = 3 which divides 9 and the quotient is 3. So, there are 3
incongruent solutions of the given equation.
Y

Now,
Re
nd

12v s 9 (mod 15)


m
Fi

=> 4v s 3 (mods) Dividing throughout by 3 and using a=b (mod m)=> — = — ^ mod —
V

=> 4 (4v) =4x3 (mod 5)


[Multiplying throughout by the multiplicative inverse of 4 which is 4 itself]
^ (4 X 4) V = 12 (mod 5)
=> V 2 2 (mods) [●.● 4x4=1 (mod 5) and 12 = 2 (mod 5)]
Thus, we obtain Vq = 2 (mod 5).
Here m = 15 and d = 3. So, the incongruent solutions are
m 2ni ,, ..in
v=Vo,Vo+-,Vq+-^, ..., Vo + (d-l)-
15 15
i.e. V = 2, 2 + -, 2 + 2x —
3 3

Hence, v = 2, 7,12 (mod 15) are incongruent solutions of the given linear congruence.
1.32 APPLIED MATHEMATICS-XII

EXERCISE 1.4

Solve each of the following linear congruences:


22:f = 39{modl5) 2. 25 a: =15 (mod 29) 3. 31a-=75 (mod 13)
i. 17j:=12(mod44) 5. 47;c = ll(mod249) 6. 37 a: =7 (mod 127)
7. 126x^67 (mod 209) 8. 49a: h 47 (mod 81) 9. 59a: = 31 (mod 67)
10. 210 a: s 40 (mod 212) 11. 33 a: =7 (mod 143) 12. 124:c = 132(mod900)
13. 120.v=52(modll9)
ANSWERS

1. AT =12 (mod 15) 2. a: = 18 (mod 29) 3. a: = 2 (mod 13) 4. a: = 24 (mod 44)


5. A = 85 (mod 249) 6. a = 86 (mod 127) 7. a = 42 (mod 209) 8. a: =77 (mod 81)

w
9. A = 38 (mod 67) 10. a = 86,192 (mod 212) 11. No solution

12. A =168, 393, 618, 843 (mod 900) 13. A =52 (mod 119)
1.6 ELEMENTARY ARITHMETIC FUNCTIONS

F lo
ee
DEFINITION An arithmetic or arithmetical function f{n) is a function whose domain is the set of positive

Fr
integers and whose range is a subset ofR (the set of real numbers) or C (the set of complex numbers).
In other words, a function / :N->K or/:N->C is called an arithmetic or arithmetical or
number theoretic function. for
ur
For example, a function /: N i? defined by
f{n) = Number of divisors of n ...(i)
s
ok

is an arithmetic functions.
Yo

We find that there is only one divisor of 1 which is itself.


o
eB

/(1)=1
There are 2 divisors of 2 viz. 1 and 2.
/(2)=2
r
ad
ou

Integer 3 has two divisors 1 and 3 itself.


/(3)=2
Y

There are 3 divisors of 4 viz. 1,2, and 4.


/(4) = 3
Re
nd

and soon.
Fi

COMPLETELY ADDITIVE ARITHMETIC FUNCTION An arithmetic function f:N->R is called a


completely additive function, if
f(mn) =/ (w) +/(«) for all m, neN
ADDITIVE ARITHMETIC FUNCTION An arithmetic function f:N^R is called an additivefunction, if
f(mn) = f (m) + f{n)for all co-prime natural number m and n.
COMPLETELY MULTIPLICATIVE ARITHMETIC FUNCTION An arithmeticfunction f :'N -> R is called
a completely multiplicative function, if
f(mn) = f(7]i) f(ri) for all m, n sN
MULTIPLICATIVE ARITHMETIC FUNCTION/4« arithmetic function f:N^R is called a
multiplicative function, if
f (mn) =f(m) f{ti)for all co-prime natural numbers m and n.
Let us now discuss some arithmetic functions and their properties:
1.33
MODULAR ARITHMETIC

1.6.1 THE EULER TOTIENT FUNCTION (j. (n)

The function ^:N -> R defined by


1, if n = 1
(i) 4»(n) =
Nwmfcer of positive integers less than n and relatively prime to it, if n > 1.
is called the Eular Toient function
i.e. (j) (?i) is defined as the number of positive integers relatively prime to n zvhich do not exceed n.
We find that

(j)(l) = Number of positive integers less than 1 and relatively prime to it = 1


(t>(2) = Number of positive integers less than 2 and relatively prime to it = 1
4>(3) = Number of positive integers less than 3 and relatively prime to it = 2

w
(|)(4) = Number of positive integers less than 4 and relatively prime to it = 2
Positive integers less than 5 and relatively prime to it are 1,2, 3 and 4.

F lo
(|)(5)=4.
Positive integers less than 6 and relatively prime to it are 1 and 5

ee
(1)(6) = 2
We find that (1, 7) = 1, (2, 7) = 1, (3, 7) = 1, (4, 7) = 1, (5, 7) = 1 and (6, 7) = 1.

Fr
M7)=6
In fact, if p is prime, then 4i (p) = p -1. for
ur
PROPERTIES OF EULER TOTIENT FUNCTION (j) (n)
(i) 4» («) is multiplicative i.e. <{> {m n) = ^ (m) (j) (») for any two relatively prime integers m and n.
s

Infact,if«;i,«2^ are pairwise prime integers, then (j)(nj 112 ■■■Vf.) = (t)(ni)
ook
Yo

(ii) If p is prime integer, then (j) (p) = p -1


eB

(iii) If p is prime integer, then


(7-1 1 'l
Hp‘’)=p“-p =p 1 — , a eN
r

p
ad
ou

-p" =p(p‘’-p'* I)=p4,(p)


fl+1 rt+i
(t) p =P
Y

(iv) If n is an odd integer, then (j> (2n) = (j) (71).


Re
nd

(v) If n > 2, then (j) (77) is an even natural number,


(vi) If 77 is a composite integer, then <i)(n) <n--Jn.
Fi

(vii) Let n = pj^ P2^'^ ■ ■ ■ Pk‘^^ be the canonical decomposition into primes. Then,
1 1
(|) (/z) =77 1 - 1-
Pi P2 Pk

Also, <^{n)={p-f't -Pi"^-l) iPi"^ -Pi"^~'^) iPk"’^

ILLUSTRATIVE EXAMPLES

Fmd the following values of Euler's Totient function (|)(7i):


(i)4>(ll) (li) c|) (13) (iii) (|) (17) (iv) (j)(19).
SOLUTION We know that ^ (p) = p -1, if p is prime.
Here, 11,13,17 and 19 are prime integers.
4.(11)=10, (j)(13)=12, (|)(17)=16 and(|)(19)=18.
1.34
APPLIED MATHEMATICS-XII

EXAMPLE 2
Find the values of Euler's Totienf function (|)(«) at:
(i) 8 (ii) 9 (iii) 16 (iv) 25 (v) 27
fl-1
SOLUTION We know that =p'^-p , where p is prime. Therefore,
(i) m=m^)=2^-2^=4: (ii) <|)(9)=(1)(32) = 32-3^=6
(iii) 4.(16) =([>(24) =2^-2^=8 (iv) (l)(25) = (j.(52)=52-5 = 20
(v) 4>(27) = 4>(3^) = 3^-32=18
EXAMPLE: 3 Find the values of(i) (^(77) and (ii)^{l896) (iii) (|)(9009)
SOLUTION Using the multiplicative property of <f> we obtain
(i) 4>(77)-(i>(7xll)={|,(7)(|)(n)=6xl0=60
(ii) 4) (1896) = 4> (3 X 8 X 79) = 4) (3) 4> (8) 4. (79) 3,8,79 are pairwise prime]

w
= 4)(3)4>(2^)4)(79)
= (3-l)x(2^-2^)x(79-l)

F lo
= 2 x 4 x 78 = 624
[v 4>(2^) =2^-2^ =4 (see property (iii))]

ee
(iii) 4>(9009) = 4> (7 X 9x 11X13) = 4>(7) 4> (9) 4> (11) 4* (13) 7,9,11,13 are pairwise prime]
= 4>(7)4>(32)4>(11)4>(13)

Fr
= (7-l)(3^-3^) (11-1) (13-1) [Using property (ii) and (iii)]
= 6x6x10x12 = 4320 for
ur
EXAMPLE 4 Find the values of Euler’s Totient function 4>(n) at:
(i) 420 (ii) 360 (iii) 6615 (iv) 55125
s
ok
Yo

SOLUTION Using the Fundamental Theorem of arithmetic which states that every positive
integer (^t 1) can be expressed as the product prime integers in a unique way, we obtain
o
eB

420 = 2^x 3x5x7, 360 = 2^ x 3^ x5,6615 = 3^ x5^ x 7 and 55125 = 3^ x5^ x 7^


We know that
r
ou
ad

Therefore,
(i) 4'(420) = 4.(2^x3^x5^x7^)=(22-2b(3-3°)(5-5'^)(7-7'^) = 2x2x4x6=96
Y

(ii) 4>(360)=4)(2^x 3^x5^)=(2^-22)(3^-3^5^-5°)=4x6x4 = 96


Re
nd

(iii) 4)(6615) = 4.(3^ x5“ x7^) =(3^-3^) (5^-5^7^-7”) =18x20x6 = 2160


Fi

(iv) 4>(55125) = 4)(3^x5^x7^)=(3^-3^)(5^-5^)(7^-7^)=6xl00x42=252 00


EXAMPLE 5 If n is a positive odd integer, prove that ^(2n) = 4>(/i) and hence find the value of (bin) at
n = 230.

SOLUTION Since n is a positive odd integer. Therefore, 2 and n are relatively prime i.e. (2,«) = 1-
Using multiplicative property of 4>/ we obtain
4>(2»o =4>(2) 4>(») =4>(«) [V 4»(2)=1]
Now, 4>(230) = 4)(2x 115) =4.(115) =4.(5x 23) = 4.(5) 4>(23) =(5-1) (23-1) =88.
1.6.2 DIVISOR FUNCTION AND SUM OF POSITIVE DIVISORS FUNCTION

DIVISOR FUNCTION Thefunction x :N R associating eachn &N to the number of positive dwisors of
n including the divisors 1 and n, is called the divisor function.
Thus, X (n) = Number of divisors of n (including 1 and n)
1.35
MODULAR ARITHMETIC

Clearly, t(1)=1, t(2)=2, t(3) = 2, t(4) = 3, t(5) = 2, t(6)=4, t(7)=2, t(8) = 4, x (9) = 3
and X (10) = 4.
If p is prime, than x (p) = 2.
SUM OF POSITIVE DIVISORS FUNCTION The function c\N associating each nsNto the sum of
the positive divisors ofn, including 1 and n itself
Thxis, o(7?) =The sum of positive divisors of n (including 1 and n).
We find that
a(l)=l
a(2) = (The sum of the positive divisors of 2.) = 1 + 2 = 3
a( 3) = The sum of the positive divisors of3=l + 3= 4
o
(4) = The siun of the positive divisors of4 = l + 2 + 4= 7
If p is prime, then its positive divisors are 1 and p itself.

w
o(p)=p + l.
ILLUSTRATION 1 Find the values of:

F lo
(i) x(12), and a (12) (ii) x(15), and a (15) (iii) X (19), and a (19)
SOLUTION (i) The divisors of 12 are : 1,2, 3,4, 6, and 12.
X (12) = 6 and, a (12) =1 + 2+ 3 + 4 + 6 + 12 =28

ee
(ii) The divisors of 15 are: 1, 3, 5 and 15 itself.

Fr
X (15) = 4 and a (15) =1+ 3 + 5 + 15 = 24
(iii) We find that 19 is a prime integer. Therefore, it has only two divisors viz. 1 and 19.
for
x(19)=2 and a(19) =1+19 =20
ur
NOTE Ifp is prime, then a (p) = p +1.
s

PROPERTIES OF r AND o: Following are some useful properties of the divisor function x and the
ook
Yo

sum of the divisors function a.


eB

PROPERTY 1 Let n -p/1 p2^‘^ ■ ■ ■Pk"’^ be the canonical decomposition of n. Then,


x(n)=(fli + l) (a2+l)---(rtfc+l).
our
ad

Fl
and, a («) =
Pl-1 P2-1 Pic-1
dY

ILLUSTRATION 2 Fmdfi)x(72), and o(72) 07) x (2700), and o(2700)


Re

SOLUTION (i) We find that/j =72 = 2^ X 3^.


Fin

( 2^ -1 3^-1
= 15x13=195
x(72)=(3 + l)(2 + l)=12 anda(72)= —
2-1 3-1

(ii) We find that n = 2700 = 2^ x 3^x5^.


x(2700)=x(22x3^x52)=(2 + l)(3 + l)(2 + l) = 36
f23-lV3‘^-lV53-l = 7x40x 31 =8680
and. a (2700) =
2-1 3-1 5-1

PROPERTY 3 X (n) is odd ijfn is a perfect square.

PROPERTY 4 X («) is even iff n is not a perfect square.

PROPERTY 5 Let m, n be relatively prime positive integers i.e. {m, n) = 1. Then,


(i) X {m n) = X (m) x (n) {Multiplicativity o/x)
(ii) G (m n) = a {m) ct {n) {Multiplicativity of a)
1.36
APPLIED MATHEMATICS-Xll

PROPER DIVISORS The divisors of an integer n, excluding the divisor n are called proper divisors ofn.
For example, proper divisors of 18 are 1,3,6 and 9 whereas divisors are 1,2,3,6,9 and 18.
SUM OF PROPER DIVISORS FUNCTION A function oq: N R associating each natural number n to
the sum of all proper divisors ofn is called the sum of proper divisors function.
If n = 12, then divisors of 12 are 1,2,3,4,6,12 and its proper divisors are 1,2,3,4, 6.
ao(12)=l+2+3 + 4 + 6=16
The divisors of 28 are 1,2,4, 7,14,28 and its proper divisors are 1,2,4, 7,14.
Oo(28)=l + 2 + 4+7+14=28.
Note tha t a (12) = 1 + 2 + 3 + 6 +12 = 22 and o (28) = 1 + 2 + 4 + 7 +14 + 28 = 56
For ;i = 28, we find that a (n) = 2n and oq (;?) = n. Therefore, oq («) = a (n)
Such integers are called perfect numbers as defined below.
PERFECT NUMBER An integer n is said to be a perfect number, if the sum of its proper divisors is equal
to n.

F low
Thus,«is a perfect number, if og {n) =n or, a («) = In.
Consider n = 496. We find that n = 496 = 2^ x 31.
a(496) = CT(2^X 31) = a(2^) a(31) [Using multiplicity of a]
25 -1 „a+l -1
=> ct(496)= — x(31+l) If p is prime then a (p) = p +1, and a (p^) =
2-1
p-1
re
for F
=> a(496) = 31X 32 = 2x496.
So, 496 is a perfect number.
DEFICIENT NUMBER An integer n is called a deficient number, if ag(«) < n.
Your

We find that
s
eBook

a (15) = a (3 X 5) = a (3) a (5) [Using multiplicity of a, because (5,3) = 1]


a(15)=(3 + l)(5 + l) = 24 [v o(p) =p + l, if p is prime]
0g(15)=24-15=9>15.
ad
our

So, 15 is a deficient number.


(1+1
-1 1
Also,0(32)=a(2^)=-2-1 = 63 va(p»)=£ if p is prime
p-1 '
Re

ag (32) = a (32) - 32 = 63 - 32 = 31 < 32


Y
Find

So, 32 is a deficient number.

N '’TJ: ] Ifp is a prime number, then ctq {p■^) =a(p)-p =(p + l)-p =1.
' ruATl'-'. . Ifp is a prune integer and k is a positive integer, prove that p’^ is a deficient number.
k+l
-1
SOLUTION We know that o(p^) = -
p-1

fc+1

0o(/)=-
-1
-P
1
<
p^ cf
p-1 p-1 p-1 p-1 p-1
Hence, p is a deficient number.
2*, 3^, 5^ etc are deficient number for k >1.
■ ii> ●
i If Pi and p2 are prime numbers, then prove thatpiP2 is a deficient number.
MODULAR ARITHMETIC 1.37

SOLUTION We know that Oq ('0 = ^ (»)


^0iP\Pl)=^(PlP2)-PlP2
= (y(p-i)a(p2)-p\P2 [Using multiplicity of a]
= {p■^+l) iP2+V-P\P2 [●.● a (p) = p +1, if p is prime]
= Pl+P2+^
= PiP2-KPi-1) (P2-1)"2} P2
Hence, pi P2 is a deficient number.
ABUNDANT NUMBER An integer n is called an abundant number, ifa^in) > n.
Consider n -12, we find divisors of 12 are 1, 2,3,4, 6,12.
c7o(12) = ct(12)-12 =(1 + 2+3 + 4 + 6+12)-12 =16 >12.
So, it is an abrmdant number.
Also, tJo(30)=o(30)-30=a(2x 3x5) - 30 = a (2) a(3) a (5) - 30

w
= (2 + 1) (3 + 1) (5 + 1)-30 =42 >30
30 is an abundant number.

F lo
and.
ao(945) = a(945)-945

ee
= a(3^x5x7)-945

Fr
= cj(3^)a(5) a(7)-945 [.● 3^, 5,7 are relatively prime]
_ 3‘^-l
for
X (5 + 1) (7 +1) -945 = 40x 6x 8-945 = 975 >945
ur
“ 3-1
Hence, 945 is an abundantnumber.
s
ook
Yo

NOTE 945 is the first odd abundant number.


eB

1.6.3 MOBIUS FUNCTION

MOBIUS FUNCTION A function p\N ->R defined as


our

(i) p(l)=l
ad

(//) p (/i) = (-1)*^/(/"« = Pi P2 ■■■Pk is a product of k distinct primes.


(Hi) p («) = 0/or all other values ofn
Y

From the above definition, we find that


Re
nd

(i) n (2) =(-!)’=_1


Fi

(ii) t,(3)=(-l)U-l
(iii) 4 = 2^ is not a product of primes. Therefore, p (4) = 0.
(iv) n(5)=(-l)U-i
(V) n (6) =(-1)2=1 [v 6 = 2x3 .●. 6 is product of two distinct primes]
(Vi) ^(7)=(-l)U-l
(vii) m(8)=h(22)=0
(viii) n (105) = n (7 X 5 X 3) = (-1) 2 = -1.
(ix) n (3x5x72) =0
(x) n (2x7x13x19) =(-!)“* =1
1.38 APPLIED MATHEMATICS-Xil

PROPERTIES OF MOBIUS FUNCTION

PROPERTY 1 Let m, n be relatively prime integers i.e. {ni, n) = 1. Then

PROPERTY 2 The sum of the mobius functions over all positive divisors of n (including 1 and n itself) is
zero except n = l.
n

MERTENS FUNCTIONS A function M : N ^ R defined by M («) = ^ |i(it) is called the Mertens


k=l

function.

EXERCISE 1.5

1.
Find the values of the Euler Totient function (|)(«) at:
(i) 91 (ii) 262 (iii) 1200
2.
If p is a prime number, prove that

w
(i) T (p) = 2 and a (p) = p +1

F lo
where t and a are divisor and sum of divisors functions respectively.
3.
If n is an odd positive integer, prove that (l)(2>i) =6(«)-
4.
Find the values of Mobius function p (n) for n -

e
(0 15 (ii) 20 (iii) 24 (iv) 29 (V) 30

Fre
*>. Find the number and sum of all divisors of 252.
6.
Find the sum of all divisors of 960 excluding 1 and itself.
for
7. Find the number of divisors of the form 4n + 2, n > 0 of the integer 240.
8. Find the number of divisors and the sum of all divisors of the following numbers:
r
You

(i) 960 (u) 1920 (iii) 2304 (iv) 100


oks
eBo

ANSWERS

1. (i)78 (ii)130 (iii) 32 4. (i) 1 (ii) 0 (iii) 0 (iv) -1


5. 18,728 6. 2087 7. 4
our
ad

8. (i) 28,3048 (ii) 32,3049 (iii) 27,6643 (iv) 9,217

HINTS TO THE SELECTED PROBLEMS


7. 4n + 2 = 2 (2n +1) and 240 = 2 x 120. So, a divisor of the form 4n + 2 of 240 is an odd divisor
dY
Re

of 120. Clearly, 1,3,5,15 are odd divisors of 120.


Fin

MUL TIPLE CHOICE QUES TIONS (MCQs)


Each of the following questions has four choices (a), (b), (c)and (d) out of which only one choice is correct.
Mark the correct choice.
1.
The least positive integer x satisfying 28 = x (mod 6) is
(a) 2 (b) 4 (c) 3 (d) 1
2.
If X is the least non-negative integer satisfying 218 s x (mod 7), then x^ +1 is equal to
(a) 1 (b) 2 (c) 5 (d) 50
100 :
3.
The unit's digit of 7 is

(a) 1 Cb) 7 (c) 2 (d) 4


4.
The smallest non-negative integer congruent to 2796 (mod 7) is
(a) 1 (b) 2 (c) 5 (d) 3
242
The least non-negative remainder when 3 is divided by 13 is
MODULAR ARITHMETIC 1.39

(a) 3 (b) 6 (c) 9 (d) 1


100
6. The remainder when 2 is divided by 5 is
(a) 1 Cb) 2 (c) 3 (d) 4
7. The remainder when 5^^ is divided by 7 is
(a) 1 (b) 2 (c) 4 (d) 5
24 :
8. The unit digits in 11 IS

(a) 7 (b) 8 (c) 9 (d) 1


600 :
9. The units digit in 6 IS

(a) 2 (b) 4 (c) 6 (d) 8


300 .●
10. The units digit in 3 IS

(a) 1 (b) 2 (c) 3 (d) 6


11. The value of a: in the set {0,1, 2,..., 10} satisfying 3-x =5 (mod 11) is
(a) 10 (b) 9 (c) 8 (d) 2

w
12. The value of .t in the set {0,1, 2,..., 6} such that53814 = x is
(a) 2 (b) 4 (c) 5 (d) 3

1. (b) 2. (b) 3. (a) 4. (d)


F lo
5. (c) 6. (a) 7. (d)
ANSWERS

8. (d)

e
Fre
9. (c) 10. (a) n. (b) 12. (c)
for
r
You
oks
eBo
our
ad dY
Re
Fin
CHAPTER 2

ALLIGATION

2.1 INTRODUCTION

When two or more substances are physically combined, they form a mixture. In other words, a
mixture is a physical combination of two or more substances. The substances are known as the

w
ingredients of the mixture. For example, when salt or sugar or both are combined with water a
mixture is formed. In this chapter, we will discuss problems on finding the amount of each

F lo
ingredient needed to make a mixture of given quantity. The method of finding the same is called
Alligation as defined below.

ee
2.2 ALLIGATION

Fr
Alligation is on old and practical method of solving arithmetic problems related to mixtures of
two or more ingredients. There are two types of alligation,
(i) Alligation Medial
for
(ii) Alligation Alternate
ur
Alligation medial is the method to find the quantity of a mixture when we are given the
quantities of its ingredients. It can be found by using the formula for weighted mean.
s
ook
Yo

Alligation Alternate is the method to find the quantity of each ingredient needed to make a
mixture of given quantity.
eB

In this chapter, we will mainly discuss problems on Alligation Alternate.


Let a mixture M be made of tw'o ingredients and M2. Let the quantities of ingredients M| and
r

M2 be q-[ and 72 ^nd the cost price per unit beCj and C2 respectively. Further, let the cost price
ad
ou

per unit of the mixture M be C,„ .Then,


Cost price of ingredient M| + Cost price of ingredient M2 = Cost price of mixture M
Y

=> Cii^T +C2i?2 {‘7i + ‘?2) [●.● Total quantity of mixture M = qi + q2] ●●● (i)
Re
nd

=> -C,„ =C,„ i72~C2 (?2


Fi

=> (C;i -C,„) =(C„j -C2) q2


‘/l ~^2 ...(ii)
^2 Ci-C III

From (i), we obtain


Cm
_Ci<^l+C2 qi
'll +“?2

This shows that C,„ is the weighted mean of cost prices C| and C2 of ingredients M^and M2
respectively. Therefore,C,„IiesbetweenCi andC2-If we assume thatCj <C2,thenC| <C,„ <C2-
From (ii), we obtain
Quantity of ingredient M^ _ Mean cost price C,„ - Cost price of ingredient M2
Quantity of ingredient M2 Cost price of ingredient Mj - Mean cost price C III
2.2 APPLIED MATHEMATICS'XII

or.
Quantity of ingredient _ Mean cost price - Cost price of cheaper ingredient
Quantity of ingredient M2 Cost price dearer ingredient - Mean cost price.
The above result can be depicted in the following form of alligation grid:
C. P. per unit of ingredient Mj C. P. per unit of ingredient M2
(Cl) (Cj) tN

C c
o 0)
T3 ■o
CJ
b.
tio &

ow
c c

o Mean Cost price o

c
tz
3
6 o
- t

e
re
C,„-C2

Frl ''2 C,-C

F
1

Norr 1 In the above diagram C-^ (cost price of dearer ingredient) is written on the left hand side and that
of cheaper ingredient on the right hand side. The arrow is directed towards the larger cost price.
ou
or
2.2.1 REPEATED DILUTIONS kfs
Suppose a container contains .v litres of some liquid A. From this, y litres are taken out and
replaced by an equal quantity of liquid B. Then, after first replacement
oo

Quantity of liquid A in x litres of mixture of A and B = (.v -y) = .v 1 - l]


/
Y

xj
B

y
Quantity of liquid A in 1 litre of mixture of A and B = 1 - —
re

.V
oYu

I
=>
Quantity of liquid A in y litres of mixture of A and B = y 1 - .V
ad

So, quantity of liquid A left after taking out y litres of mixture of A and B
d

y^
in

y
Re

= .T 1-^ -y 1-
xj Xj
F

y
= {x-y) 1-^ =x 1-^
.V XJ

n2
y
Thus, the quantity of liquid A left after 2 operations = x 1
XJ

Now, y litres of liquid B are added and y litres of mixture of A and 6 are taken out.

The quantity of liquid A in a: litres of mixture at the end of 2 operations = x 1


yf
XJ

\2.
y
Quantity of liquid A in 1 litre of mixture at the end of 2 operations = 1 -
X

^2
y
Quantity of liquid A in y litres of mixture at the end of 2 operations = y 1 -
xJ
ALLIGATION 2.3

n2
y
So, quantity of liquid A left after 3rd operation = a: 1 X) xj

= fl-^ (.V-l/)=.Y 1-^


y
X) V X)

Continuing in this manner, we find that


V
y
The quantity of liquid A left after n operations = a: V 1 - -XJ

Thus, if a container contains x litres of liquid A from which y litres are taken out and replaced by
y litres of another liquid B and the process is repeated n times. Then, after n operations,
f y V'
litres.
The quantity of liquid A in x litres of mixture = x 1 - —
XJ

Quantity of liquid A left (i-i


Quantity of liquid A originally present I

w
XJ

and.

= x-x

Quantity of liquid A left _


1

,T l-i'
F lo
Quantity of liquid B left =x - Quantity of liquid A left
y
XJ

xJ
V'

V'
=x -11

\h'
X

for F
ree
Quantity of liquid B left X 1- i_y
X
Your
ks

y
eBoo

1-
Quantity of liquid A left _ XJ

Quantity of liquid B left i-fi-y V’


xJ
ad
our

2.2.2 MIXTURE OF TWO GIVEN MIXTURES WITH SAME INGREDIENTS

Let there be two mixtures and with same ingredients A and B. Let the ingredients A and
Bin the mixtures and M2 be in the ratio a-y-.bi andfl2 :^2 respectively. Suppose imits ofM]
Re
Y

and q2 units of M2 are mixed to form a resultant mixture M with ingredients A and 6 in the ratio
Find

'/A : ‘iB-
Now,

Quantity of ingredient A in qi units of M| =


a^+bi
h
Quantity of ingredient B in q-[ units of Mj =
^1

Quantity of ingredient A in ^2 units of M2 = ?2


^2+^2

bi
Quantity of ingredient B in q2 units of M2 = ‘72
^2 7^2
2.4 APPLIED MATHEMATICS-XII

= Total quantity of ingredient A in M = ?1 +


^2 ^2

^2
%= Total quantity of ingredient B in M = —— 9l + ^2
<?2 + ^2

fl2
^71 + ^2
^2 ^2
Hence,
%
91 +
^2 92
l«l+^l ^2+^2,
and.

9a
Quantity of ingredient A in the resultant mixture M = = X
(9a + 9fi)
9a +9b

w
_ 9a

F lo
^ (9i + 92) 9a + 9s - 9i + 92]
9a +9b

9b 9b
Quantity of ingredient B in the resultant mixture M = qg = (9a + 9b) - X (gi + ^2)

e
9a +9b 9a + 9b

Fre
Now, for
Quantity of ingredient A in one unit of Mj =
(7i+l7i
r
You

f?2
oks

Quantity of ingredient ,4 in one unit of M2 =


<?2 "I" b')
eBo

9a
Quantity of ingredient A in one unit of the resultant mixture =
9a +9b
our
ad

9b
Quantity of ingredient B in one unit of the resultant mixture =
9a + 9b
dY
Re

ILLUSTRATIVE EXAMPLES
Fin

Tifpe 1 ON MIXTURE OF TWO INGREDIENTS


EXAMPLE 1 In lohat ratio should a shopkeeper mix two types of rice, one costing ? 20 per kg and another
costing ? JO per kg to get a rice variety costing ? 14 per kg.
SOLUTION Supposes kg of rice costing ? 20 per kg is mixed withy kg of rice costing ? 10 per kg
to get a rice variety costing ? 14 per kg. Tlren,
Cost of X kg of rice costing ? 20 per kg = 20 x
Cost of y kg of rice costing ? 10 per kg = 10 y
Cost of (x + y) kg of mixture costing ? 14 per kg = 14 (x + y)

20x + lOy = 14 (x + v) => 6x = 4i/ => - = —


' y 3

Hence, the required ratio is 2: 3.


ALLIGATION 2.5

ALITER The alligation grid is as given below.

.V 1/ = 20 - 14 = 6

Required ratio = .v: y = 4: 6 or, 2: 3.

ow
EXAMPLE 2 The cost of Type 1 rice is ^15 per kg and Type 2 rice is ? 20 per kg. If both Type 1 and Type
2 are mixed in the ratio 2 :3, find the price per kg of the mixed imriety.
SOLUTION Let the price per kg of mixed variety of rice be ? .v. Then, by using the formula for
weighted mean, we obtain

e
2x15+3x20 30 + 60
= .Y=> .y=18.

re
= .Y=>
2+3 5

Hence, the price of mixed variety is ? 18 per kg.


rFl
F
ALITER Let the price per kg of mixed variety of rice be ? x. Clearly, it is the mean price. The
alligation grid is as given below.

r
ou
Type 1 2 fo
ks
oo
Y
r eB
ou

20 - .Y -v-15
ad
Y

Using the alligation grid the ratio of Type 1 and Type 2 rice is(20-.Y) :(.Y-15).But, it is given that
this ratio is 2 : 3.
20-.V

I => 3(20-.y)=2(.y-15)=> 60-3.Y = 2.Y-30=>5.Y=90=> .y=18


d
Re

.y-15
in

Hence, the price per kg of mixed variety of rice is ^ 18.


F

EXAMPLES A Merchant lent out X W,000 in two parts, one at 8% and other at 10% interest. The yearly
average comes out to be 9.2%. Find the amount lent in two parts.
SOLUTION The alligation grid is as given below;

Using alligation law, we obtain


Amount lent at 10% 1.2 3
Amount lent at 8% 0.8 2
2.6 APPLIED MATHEMATICS-XII

Thus, ? 10,000 are divided in two parts in the ratio 3 : 2.


Amount lent at 10% = ? — X 10,000 = ? 6,000
3 + 2

Amount lent at 8% = X 10,000 =? 4,000


3 + 2

EXAMPLE 4 Pure milk costs ? 72 per litre. A milkman adds water to 25 litres of pure milk and sells the
mixture at ? 60 per litre. Hoio mamj litres of water does he add?
SOLUTION Suppose the milkman adds x litres of water to 25 litres of pure milk and sells the
mixture at ? 60 per litre. Then,
Cost of 25 litres of milk = X (25 x 72)
Cost of (25 + x) litres of mixture of = ? 60 (25 + x)
25 X 72 = (25 + x) X 60 ^ 30 = 25 + x ^ x = 5

w
Hence, the milkman adds 5 litres of water.
ALITER Suppose the milkman adds x litres of water to 25 litres of pure milk and sells the

F lo
mixture at X 60 per litre.
The alligation grid is as given below.

ee
Pure milk Wate

Fr
for
ur
s
ook
Yo

By the alligation law


eB

Quantity of pure milk _ 60 _ 5


Quantity of water 12 1
r
ad
ou

Hence, the milkman adds 5 litres of water.


EXAMPLE 5 In what ratio must a grocer mix two varieties of tea worth X 60 a kg and X 65 a kg so that by
Y

selling the mixture at X 68.20 a kg may gain 10%?


SOLUTION We have,
Re
nd

S.P. of mixture = X 68.20 and Gain = 10%


Fi

C.P.=
SP
Gain
C.P.= X
68.20
10
^^[^682^ = X 62
11
1 + 1 +
100 100

The alligation grid is as given below:


Cost of 1 kg tea Cost of! kg tea
of first kind of second kind
X60 X 65

Mean
cost
X62

(65-62) = 3 (62-60) = 2

By using the rule of alligation, we obtain


ALLIGATION 2.7

Tea of first kind : Tea of second kind =3:2


Hence, the grocer must mix in the ratio 3 :2
ALITER We have,
S.P. of mixture = ? 68.20, Gain = 10%
S.P. ? 68.20 ? 682
C.P.= - => C.P.= = ? 62
Gain 10 11
1 + 1 +
100 100

Suppose the grocer mixes x kg of tea worth ? 60 (7 kg and y kg of tea worth ? 65 r? kg.
Then, by using weighted average formula, the mean price of the mixture is
60a: + 65y
X +y

ow
But, the price of mixture is given ? 62.
60a: + 65y = 62
A: + y
X 3
60.V + 65i/ = 62-y + 62i/ => 2x = 3i/ => — = —

e
■ y 2

re
Hence, the grocer mixes two varieties in the ratio 3 : 2.

rFl
F
EXAMPI.P. 6 Five litres of water is added to a certain quantity of pure milk costing ? 60 per litre. If by
selling the mixture at the same price as before, a profit of 20% is made, what is the amount of pure milk in
the mixture?

r
ou
SOLUTION Let the quantity of pure milk be a: litres. It is given that the mixture is sold at a profit
fo
of 207o and the selling price of the mixture is X 60 per litre.
ks
Profit^
S.P. =C.P. 1 +
100
oo

20 A
Y

60 =C.P. 1 + C.P.= 60x-=50


B

100 6

So, the mixture is sold at X 50 per litre.


re

To find the ratio of milk and water in the mixture, let use the alligation rule. The alligation grid is
ou

as given below.
Y
ad

Pure milk Water

X 60 per liter X 0 per liter


d
in
Re
F

X 50 {C. P, of mixture)

(50-0) =50 (60-50) = 10

By using alligation rule, we obtain


Quantity of pure milk 50 5 _ x = -5 => .Y = 25
Quantity of water io“T ^ 5 1
Hence, the amount of pure milk in the mixture is 25 litres.
EXAMPLE 7 In what ratio must water be mixed ivith milk to gain 20% by selling the mixture at cost
price?
SOLUTION Let the C.P. of milk be ^ y per litre. Then, S.P. of the mixture is also X x per litre. It is
given that: Gain = 20%
Gain A
S.P. = C.P. 1 +
100
2.8 APPLIED MATHEMATICS^XII

20
X=C.P.\1 + => .Y = ^c.p.^c.p-j^
100 5 6
5.V
So, the C.P. of mixture is ? —.
6

The alligation grid is as given below.


(C. P. of milk) (C. P. of water)
0

ow
5x
— {C. P. of mixture)
6 V

e
re
lx - 5 .y' X

6 6

Flr
Using alligation rule, we obtain

F
Quantity of milk _ 5.y/6 _ 5
Quantity of water x/6 1
ou
sr
Hence, milk and water are in the ratio 5 :1.

kfo
EXAMPLE 8 Two liquids are mixed in the proportion 3 :2 and the mixture is sold at ^ 110 per litre at a
10% profit. Iffirst liquid costs ^ 20 more per litre than the second, what does it cost per litre?
oo
SOLUTION We have.
S.P. of mixture = ? 110, Profit = 10%
Y

Profit \
reB

S.P. = CP. 1 +
100

10 ^ 11
uY

110=C.P. 1 + =>110 = C.P.=> C.P. = ? 100


100 10

Thus, C.P. of the mixture is ? 100.


ad
do

Let the C.P. of second liquid be ? y. Then, C.P. of first liquid = ? (y + 20).
Thus, we have
in

C.P. of first liquid = ? (y + 20), C.P. of mixture = ? 100.


Re

The alligation grid is as given below:


F

C.P. of first liquid C.P. of second liquid


?(.r + 20)

100

100-Y (y + 20)= .v-80

By using the alligation rule, we obtain


Quantity of first liquid _ 100 -y
Quantity of second liquid y-80

But, it is given that two liquids are mixed in the proportion 3 : 2.


2.9
ALLIGATION

- => 200 - 2.V = 3-y - 240 => 5.y = 440 => .v = 88


.v-80 2

Hence, cost price of second liquid is ? 88.


Ti/pc II ON MIXTURE OF GIVEN MIXTURES
EXAMPLE 9 Two equal glasses filled zuitli mixtures ofalcohol and water in the proportions 2:1 and 1:1
respectively were emptied into a third glass. What is the proportion of alcohol and water in the third glass?
SOLUTION Two ingredients A and B are in the ratio a^: and ^72 : i’2 two mixtures M-j and M2
respectively. If i/^units of M^ and (72units of M2 are mixed to form a resultant mixture M, then the
ingredients A and B in the resultant mixture are in the ratio
^2
7l +
‘IA _ I ^1+^1 ^2 + ^2,

w
h bo
q-l + ^2
_rti+&l ^2 ^2

F lo
Hence, f?-] =2,b\ = l.a2=l,b2 =1 and q-^ =^q2 (glasses are equal)
f 2 f 1 2 1
9l + ^1 +
Alcohol _ ^ 2 + 1 1+1 _ 3 2

ee
[●●● ^2=^ll
Water 1 'l 1 1 1 5

Fr
+
^1 + 3 2
2 + 1 1+1

Hence, in the third glass alcohol and water are in the ratio 7: 5. for
EXAMPLE 10 The milk and water in two vessels P and Q are in the ratio 4: Sand 2 :3. In what ratio, the
ur
mixtures in both the vessels be mixed to obtain a new mixture in vessel R containing half milk and half
water?
s
ook

i?l _ 4 a2 _ 2
Yo

SOLUTION We have
&2 ”3
eB

Let qp units of mixture in vessel P be mixed with qg units of mixture in vessel Q. Then, the
quantities of milk and water iri the mixture are in the ratio given by
r
ad
ou

1 1
/
^2 b\ b2
Y

qp + 1 + a +

^2 + a-y
—L
Re
nd

‘Tw ^1 b2 '
qp + 1 1
Ui+f’i ^’2 ,
Fi

1 + 1 +
^2
h) h
4 2

3. 3

/■/P + g-jQ
It is given that vessel R contains half milk and half water. Therefore,
‘7m
4 2

‘7m „‘7p + c;'?Q 3 3 4 2


‘Tw ^3 ^=1 ^ 7 flp + 5 =7‘7p -‘7Q
7^P + 5‘7q
1 1

7‘7p=5‘?Q
2.10 APPLIED MATHEMATICS-XII

3z.-Z.

Hence, the mixtures in vessels P and Q are mixed in the ratio 7; 5.


\l I IFK Let the C.P. of milk be ? a: per litre.
4 2
Milk in 1 litre mixture of vessel P = — litre. Milk in 1 litre mixture of vessel O = — litre
7 ^5
4 2
C.P. of 1 litre mixture in vessel P = ^ -x, C.P. of 1 litre mixture in vessel Q = ? -a:
7 5
1 X
C.P. of 1 litre mixture in vessel R = C.P. of - litre milk = ? -
2 2

The alligation grid is as given below.

ow
C.P. of 1 litre milk in P

e
re
rFl
F
r
U
ou
By using alligation rule, we find that fo
ks
Quantity of mixture in vessel P _ a:/10 _ 7
Quantity of mixture in vessel Q .r/14 5
oo

EX.A.MPLEll
hi two alloys, the ratio of copper to zinc are 5:2 and 3:4. Hozu many kg of the first alloy
Y
eB

and that of the second alloy should be melted together to obtain 28 kg of a new alloy with equal amount of
copper and zinc?
SOLUTION Wehave, =5:2, ^2 ^: 4and q^~ = q^, where q^ denote the quantity
r
ou

of copper and zinc in the new alloy. Let and q2 be the quantities of two alloys to be melted
ad
Y

together to obtain 28 kg of a new alloy. Then,


+ £/2 = 28
d

^1 02
Re
in

^1 + ^2
(]c fli+fcl t?2 T ^2
Now,
F

(Iz bi
^1 + ^2
IJ ^2 1^2
f5 ^3
^2
V/7 / 7
1 =
2^ (4
7r^\? <72

3
‘71 = ~i/2 ^ 3(/| =q2 ...(ii)
7

From (i) and (ii), we obtain


i^l + 3 = 28 => =7
1^2=3^1-21.
Hence, 7 kg of first alloy and 21 kg of second alloy are melted together to obtain 28 kg of new
alloy.
2.11
ALLIGATION

Txjpe III ON REPEATED DILUTION


EXAMPLE 12 A container contains 100 litres of milk. From this container, 10 litres of milk zvas taken out
and replaced hy equal amount of zvater. This process zvas further repeated tzvice. Hozv much milk is left in
the container?

SOLUTION If a container contains -v litres of liquid A from which y litres are taken and replaced
by V litres of another liquid B and the process is repeated n times. Then,
f vY
After n operations, the quantity of liquid A in x litres of mixture = x 1 — .

Here, =100, y =10 and n = 3.


f V 10

ow
= 100 1- litres
Quantity of milk left after 3 operations = .r 1 —xj 100

' 9
= 100x litres = 72.9 litres.
10

e
EXAMPLE 13 A bottle isfidl ofdettol. One third of its is taken out and then an equal amount of zvater is

re
poured into the bottle to fill it. This operation is repeated four times. Find the final ratio of dettol and water
in the bottle.

rFl
F
SOLUTION Let the original quantity of dettol be x litres and let y litres be taken out which is
replaced by an equal quantity of water. It is given that y = x/ 3.

r
ou
(
After 4 operations the quantity of dettol left = .v 1 —.V fo
ks
n4
V
After 4 operations the quantity of water in the bottle =x-x 1 —XJ
oo
Y
B

Thus, after 4 operations, we obtain


^4
re

1/
.V 1- .V 1-
Quantity of dettol in the bottle _ Xj 3.Y .Y
ou

Vi/ = -
n4
Y

Quantity of water in the bottle


ad

.Y
y -Y - .Y 1 -
-.Y 1-
Xj 3.Y

if
d

1-
in

16/81 16
Re

3J
if 16 65
1- —
F

1- 1- 81
3)
Hence, after four operations, dettol and water in the bottle are in two ratio 16 : 65.
EXAMPLE 14 An 8 litre cylinder contains a mixture ofo.xygen and nitrogen, the volume of oxygen being
16% of total volume. Afezv litres of mixture is released and an equal amount of nitrogen is added. The
process is repeated twice. As a result, the oxygen content reduces to 9% of total volume. Flow many litres
of mixture is released each time?
SOLUTION We have,
16
Original volume of oxygen = 16% of 8 litre = 100 X 8 =1.28 litre

— x8=0.72 litre
Volume of oxygen left after two operations = 9% of 8 litre = 100

Suppose y litres of mixture is released each time. Then,


2.12 APPLIED MATHEMATICS-Xll

n2
Volume of oxygen left after two operations y
Original volume of oxygen 8

0.72 ^2 ^2
=> 1-^ = -=>y = 2.
1.28 8 16 8 4 4 ^
Hence, 2 litres of mixture is released each time.

EXERCISE 2.1

1.
In what ratio must a grocer mix two varieties pulses costing ? 45 and ? 60 per kg
respectively so as to get a mixture worth ? 49.50 per kg?
2. In what ratio must 25% alcohol be mixed with 50% alcohol to get a mixture of 40% alcohol

w
strength?
3. How many kg of salt at ? 10.50 per kg must a man mix with 25 kg of salt at ? 6 per kg so that

Flo
he may, on selling the mixture at ? 10 per kg, gain 25% on the outlay?
4. Five litres of water is added to a certain quantity of pure milk costing ? 60 per litre. If by

ee
selling the mixture at the same price as before, a profit of 20% is made, what is the amount

Fr
of pure milk in the mixture?
5. Alcohol costs ? 35 per litre and Kerosene oil costs ? 25 per litre. In what proportion should
these be mixed so that the resulting mixture may be ? 27.50 per litre? for
ur
6. A cup of milk contains 3 parts of pure milk and 1 part of water. How much mixture must be
withdrawn and water substituted in order that resulting mixture may be half milk and half
ks

water?
Yo
oo

7. A mixture contains milk and water in the ratio 3 : 2. If 4 litres of water is added to the
eB

mixture, milk and water in the mixture become equal. Find the quantity of milk in the
mixture, in litres.
8. A mixture contains milk and water in the ratio 8: .v. When 33 litres of mixture and 3 litres of
r
ou
ad

water are mixed, the ratio of milk and water becomes 2 :1 find the value of ,v.
A vessel contains 56 litres of mixture of milk and water in the ratio 5 : 2. How much water
Y

9.
should be mixed with it so that milk to water ratio becomes 4 : 5?
nd
Re

10. How many litres of water should be added to a 30 litre mixture of milk and water
containing milk and water in the ratio 7:3 such that the resultant mixture has 40% of water
Fi

in it?

11. The ratio of milk and water in the mixture of water and milk is 4 : 3. If 6 litres of water is
added to this mixture, the ratio of milk and water becomes 8:7. What is the quantity of milk
in the original mixture?
12.
35 kg of types S-\ sandal powder, which costs? 614 per kg, was mixed with a certain amount
of type $2 sandal powder, which costs ? 695 per kg. Then the mixture was sold at the rate of
? 767 per kg and 18% profit was earned. What was the amount (in kg) of type $2 sandal
powder in the mixture?

ANSWERS

rd
1 ^
1. 7:3 2. 2:3 3. 20 kg 4. 25 litres 5. 1 : 3 6. — 7. 20 litres
U
8. 3 9. 34 litres 10. 5 litres 11. 48 litres 12. 28 kg
ALLIGATION 2.13

HINTS TO SELECTED PROBLEMS

3 1
6. Let there be 1 litre of mixKire having — litre milk and — litre water. Suppose x litres of

mixture is taken out and same quantity of water is added. Then.


3 3.r 1 X
Quantity of milk left = — Quantity of water left = 4 4
+ .t

It is given that — - — = — - — + a: ^ 3 -- 3a: = 1


® 4 4 4 4
+ 3.y ^ .r = i3
3 a- 2.V
7. Let the quantity of mixture be a litres. Then, quantity of milk = —,
5
Quantity of water = —.
5

ow
2a
After adding 4 litres of water. Quantity of water = ^
+ 4

It is given that -^ = ^ + 4 => a = 20

e
8. In (33 + 3) = 36 litres of mixture the ratio of milk and water is 2 :1

re
Fl
Quantity of milk in 36 litres = 36 x — = 24 litres
2

F
Quantity of water in 36 litres = 36 x i =12 litres
ur
r
Now,
24 8
= - => A = 3.
fo
ks
12-3 A
Yo
9. Suppose A litres of water is mixed. Then,
oo

^ x56 , 40
eB

-=>60=16 + a=>a = 34
16 + a 6
-x56 + a ^
7
ur

10. Total quantity of mixture = 30 litres


ad
Yo

Milk and water in the mixture are in the ratio 7 : 3.


7
Quantity of milk in the mixture = X 30 litres = 21 litres
d

7 + 3
Re
in

3
Quantity of water in the mixture = — X 30 litres = 9 litres
(7+3
F

Suppose A litres of water is added to the mixture so that the resultant mixture has
40% of water. Then.
40
x(30 + a) =9 + a^ a=5
100

11 ● Let the quantity of milk and water in the mixture be 4a and 3a litres respectively. If 6 litres
of water is added to this mixture, then
4a 8
= -=> a=12
3a + 6 7

S.P. of mixture = ? 767 per kg. Gain = 18%


S.P. 767
C.P. of mixture = xl00 = ?- X 100 = ? 650
100 + Gain 100 + 18

The alligation grid is as given below:


2.14 APPLIED MATHEMATICS-XII

lypei-i lype
?614 695

650

695- 650 - 614 = 36

Quantity of type Sj _ 45 5
Quantity of type $2 36 4
It is given that the quantity of type Sj is 35 kg.
5x = 35=>x=7

w
Hence, quantity of type S2 is 4 x 7 = 28 kg
MULTIPLE CHOICE QUESTIONS (MCQs)

F lo
Each of the follozoing questions Izasfour choices (a), (b), (c) and (d) out of which only one choice is correct.
Mark the correct choice.

ree
1. 20 litres of a mixture contains milk and water in the ratio 3:1. The amount of milk, in litres,
to be added to the mixture so as to have milk and water in the ratio 4:1 is
(a) 7 (b) 4 (c) 5
F(d) 6
for
2.
A milkman mixed some water with milk to gain 25% by selling the mixture at the cost price.
The ratio of water and milk respectively, is
r
(a) 5 : 4 (b) 4:5 (c) 1:5 (d) 1:4
You
oks

3.
In what ratio must rice at ? 29.30 per kg be mixed with rice at ? 30.80 per kg so that the
mixture be worth ^ 30 per kg?
eBo

(a) 7:8 (b) 8 : 7 (c) 3:8 (d) 8:3


2
4.
In what ratio must water be mixed with milk to gain on selling the mixture at cost
our
ad

price?
(a) 1 :6 (b) 6:1 (c) 2 : 3 (d) 4:3
5. How much water must be added to 60 litres of milk at 1 — litres for ? 20 so as to have mixture
dY
Re

2
2
worth ? 10— a litre?
Fin

3
(a) 10 litres (b) 15 litres (c) 5 litres (d) 20 litres
6. Two vessels A and B contain milk and water mixed in
the ratio 5 :2 and 7: 6 respectively.
3
The ratio in which these two mixtures be mixed to get a new mixture containing 69 —%
13
milk, is
(a) 2:7 (b) 3:5 (c) 5 : 2 (d) 5 :7

ANSWERS

●1- (c) 2. (d) 3. (b) 4- (a) 5. (b) 6. (a)


CHAPTER 3
BOATS & STREAMS
AND PIPES & CISTERNS

3.1 INTRODUCTION

In this chapter, we will discuss some numerical problems related to the motion of boats being
rowed in a river along the direction of the current (stream) or opposite to it.

w
3.2 SOME USEFUL RESULTS

F lo
Let us discuss some useful terms and result related to the motion of a boat in a river. The
direction of the flow of water in a river is called the direction of the current (stream). Following
are the terms related to it.

ee
DOWNSTREAM The direction along the flow of water in a river i.e. the direction along the stream is called

Fr
the downstream and the speed with which boat runs in this direction is called the speed downstream.
UPSTREAM The direction opposite to flow ofiuater in a river i.e. the direction opposite to the direction of
for
current (stream) is called the upstream and the speed zvith which a boat runs in this direction is called the
ur
speed upstream.
oks

Intuitively, we can say that: Speed upstream < Speed downstream.


Yo
o
eB

T V km/hrU.—"

m Speed of Current {strcam)^-£^-


our
ad
Y

—-T". Downstream:
Re

Boat
Upstream^
nd
Fi

Fig. 3.1

Let the speed of a boat in still water be u km/hr and the speed of the current (stream) in a river
be V km/hr. When a boat is rowed in the direction of the current, then the flowing water in the
river assists the motion of the boat and its speed increases, the increased speed is known as the
speed downstream and is given by
Speed Downstream of the boat = {u + v) km/hr
When boat is rowed against the current i.e. the boat is rowed upstream, then the flowing water
of the river resists the motion of the boat due to which its actual speed is less than the speed of
the boat in still water, the reduced speed is known as the speed upstream and is given by
Speed Upstream of the boat = (w - iJ) km/hr
RESUl i It the speed downstream and the speed upstream of a boat rozving in a river are v^ and
respectively, then
3.2 APPLIED MATHEMATICS-XII

1
Speed of the boat in still water = — (^^ +

Speed of the current (stream) =

PROOF Let the speed of the boat in still water be u km/hr and the speed of the current be
u km/hr Then,
=u + v and =u-v

+ v„ = 2u and -p„ = 2v

n=^{V(i+Vi,) and v =

ow
i.e. Speed of the boat in still water =
^ i'^d Speed of the current (stream) = -V;,).
RESULT2 Let the speed of a boat in Still water be u km/hr and the speed of the water current in a river be

e
V km/hr. If a man roius distance D down stream and same distance upstream, then

re
{» + y) (i/-y)
Average speed for total journey =

Frl u

F
_ Speed Downstream x Speed Upstream
Speed in still water

r
ou
PROOF Let the speed downstream and the speed upstream be and respectively. Then,
o
kfs
=11+v and v = u-v

Let the time taken in downstream and upstream journey be t^ and f,, respectively. Then,
oo

D D
^d = and f1/ ~
Y

u +v u-v
B

Distance covered
Average speed of the journey =
re

Time taken
D+D 2D (u + v) (u-v)
oYu

D D
U +
ad

u
+ ■
u+v u-v

_ speed downstream x Speed upstream


d

Speed in still water


in
Re

ILLUSTRATIVE EXAMPLES
F

EXAMPLE 1 Aarushi can roiv upstream a boat at lOkm/hr and downstreamat 18 km/hr. Find the rate at
luhich she rows the boat in still water. Also, ifnd the speed of the current.
SOLUTION Let u km/hr be be the rate at which Aarushi can row the boat in still water and let
the speed of the current be v km/hr. Then,
Aaruslii's speed upstream = {u-v) km/hr
Aarushi's speed downstream = (u + v) km/hr
It is given that
u-v = 10 and and r/ + d = 18

(u + v) +{u-v) =18 + 10 and (w+t?) -(//-y) =18-10


2u = 28 and 2y = 8
u =14 and y = 4

Hence, Aarushi can row the boat in still water at 14 km/hr and the speed of the current is
4 km/hr
3.3
BOATS & STREAMS AND PIPES & CISTERNS

ALITER We have, = 18 and Vj, = 10


1
1/ = -- (18 + 10) -14 andu =- -i;„) =(18-10) =4
2 2 2
Hence, Aarushi can row in still water at 14 km/hr and the speed of current is 4 km/hr.
EXAMI’LE 2 A man rows downstream 30 km and upstream 20 km. If he takes 5 hours to cover each
distance, then ifnd the speed at which he rows in still water and the speed of the current.
SOLUTION Suppose the man can row in still water at u km/hr and the speed of the current is
V km/hr. Tlien,
Man's speed upstream = (u-v) km/hr ...(i)
Man's speed downstream = (u + v) km/hr

ow
It is given that the man rows downstream 30 km in 5 hours and 20 km upstream in 5 hours.
20
Man's speed upstream = — km/hr = 4 km/hr
...(ii)
■ 30
Man's speed downstream = — km/hr = 6 km/hr

e
From (i) and (ii), we obtain

re
u-v = 4 and u + v = 6

Fl
F
=> {u-v)+(u + v) =4 + 6 and (i<+Z')-0'-^) =6-4
=> 2ii=10 and 2v = 2
ur
r
^ u = 5 and v = \
fo
Hence, Man's speed in still water = 5 km/hr and speed of the current = 1 km/hr
ks
30 20
ALITER Wehave,i>(y=—=6km/hrand =4km/hr
Yo
5 5
oo

~^u) = i (6 -4) =1 km/hr


1 1 1
=-(6 + 4) =5 km/hr and.
2
V =
eB

EXAMPLES A man can row a boat in still water at the rate of 4 km/hr. He finds that it takes him tzuiceas
long to row upstream of a river as to row downstream of the same river. Find the speed of the current of the
ur

river.
ad

SOLUTION We have,
Yo

11 = Speed of boat in still water = 4 km/hr


Let the speed of the current of the river be v km/hr and suppose he takes t hours to row
d

downstream. Then, time taken by him to row upstream is 2t hours.


Re
in

Downstream distance rowed =={u + z>)t ={4+v)t km


Upstream distance rowed ={u-v)x2t =2 (4-v)t km.
F

But, Downstream distance rowed = Upstream distance rowed


4
{4+v)t = 2{4-v)t ^ 4 + v =
= 8-2u=> 3z» = 4=^u= —
3
4
Hence, speed of current of river = — km/hr

EXAMPLE 4 The speed of a motor boat is that of the current of water in a river as 36:5. The boat goes
along with the current in 5 hours 10 minutes. How much will it take to come back?
SOLUTION Let the speed of the motor boat and that of the water current in the river be
u = 36at km/hr and v = 5x km/hr respectively. Then,
Speed downstream = u + v = { 36x + 5a:) km/hr - 41 x km/hr
Speed upstream = u-u =(36.r-5.r) km/hr = 31 x km/hr
Suppose the boat goes d km along the current. Then,
3.4 APPLIED MATHEMATICS-XII

d
Time taken = hour.
4lx

=>
41 :c 60
[It is given that time taken = 5 hours 10 minutes]
d 31 31
= — =s> fr = x41.t
41 .V 6 6

Suppose the motor boat takes t hours to come back with the speed of 31x km/hr. Then,
d Distance covered
Time -
31.T
Speed
{ 1 31 41 5
t = X—x41a: hour = — hour = 6- hours = 6 hours 50 minutes
31x 6 6 6

low
I XAMPL1-. 5 A rjitvican row a boat at 5 km/hr in still zvater. If the speed of zvater current in a river is
1 kin/hr and it takes him 1 hour to row to a place and come back, hozvfar off is the place?
SOLUTION We have,

ee
u = Speed of the boat in still water = 5 km/hr

F
Fr
V = Speed of the water current = I km/hr
Speed downstream = {u + v) km/hr = (5 + 1) km/lir = 6 km/hr
Speed upstream = (u - v) km/hr = (5 - 1) km/hr = 4 km/hr
for
ur
Let the place be at a distance of .r km. Then,
s
Time taken to travel downstream + Time taken to travel upstream = 1 hour.
ook
Yo

-V .T , 5x . 12 - ,,
eB

6 4 12 5

Hence, the place is 2.4 km.


r

I●XA^1PLE6 A, B and Care three towns on the bankof a river zohichflozus unifonnli/such that towzi B is
ou
ad

equidistant from toivns A and C. A man rozos a boat from toiun A to town B and back in 10 hours and he
can roiv dozunstreamfrom AtoC in 4 hours. Find the ratio of the speed of the boat in still water and that of
Y

the zvater current in the river.


Re
nd

SOLUTION Let the speed of the boat in still water be u km /hr and the speed of the water current
in the river be v km/hr. Then,
Fi

Speed downstream = u + v, Speed upstream = ii -v


A xkm — B X km C

rzlonor
jDo\vn.stream-.

j Upstream “

Fig. 3.2
BOATS & STREAMS AND PIPES & CISTERNS 3.5

Letrt =H + i’andi? =u-v.

It is given that the man rows a boat from A to B and back in 10 hours.
X
— =10=>- + -=10
u+v u-v a b

It is also given that the man can row downstream from A to C in 4 hours.
2x ^ 2.V , X ^ - ...(ii)
= 4=> — = 4=>-=2=>x = 2f7
ll + V a a

From (i) and (ii), we obtain


2 + -=10=> - = 8=> t = ...(iii)
b b

From (ii) and (iii), we obtain

w
u 5
2a = 8b=> a = 4b=> u + v = A{u-v)=> 3u=5v=>
V 3

Hence, the required ratio is 5 :3.

Flo
EXAMPLE 7 A man catt row a boat in still water at a speed of 4.5 km/ln. In a river flowing at the rate of
7.5 km/hr he starts his journey from a point downstream and comes back to the same pwint. Find his

e
re
average speed for the total journey.

F
SOLUTION We have,
u
= Speed of boat in still water = 4.5 km/hr, v = Speed of the water current = 1.5 km/hr
ur
Let V be the average speed of the total journey. Then
V =
(u+v) {ll-v) => V =
(45+ 15)(45-15) _6x3 = 4 km/hr
f or
ks
u 45 45
Yo

EXAMPLE 8 A motor boat can travel at 10 km/hr in still water. It travelled 91 bn downstream in river
oo

and then returned back taking altogether 20 hours. Find the rate offlow of water in the river.
B

SOLUTION Let the rate of flow of water in the river be v km/hr.


It is given that the motor boat can travel in still water at ii = 10 km/hr.
re

Speed downstream = u + v = (10 + v) km/hr.


u

Speed upstream = u-v = (10 - v) km/hr.


ad
Yo

91 Distance
Time taken to travel 91 km downstream = Using: Time =
10+u Speed
d

91
Re

Time taken to travel 91 km upstream =


in

10-V
F

It is given that the total time taken is 20 hours.


91 91 10-f+ 10 + U
= 20 =>91 ^ =20
10+u 10-v |(10 + i')(10-u)
91x20
= 20 => 91 =100 => v^ =100-91=>tJ^ =9=> t? = 3.
100

Hence, the water is flowing in the river at 3 km/hr.


EXAMPLE 9 A boat takes 1 hour 30 minutes less to travel 36 km downstream than to travel the same
distance upstream. If the speed of the boat in still water is 10 km/hr, find the speed of the stream.
SOLUTION Let the speed of the stream be v km/hr. It is given that the speed of the boat in still
water is m = 10 km/hr. Then,

Speed downstream = (10 + v) km/hr


Speed upstream = (10 - v) km/hr
3.6 APPLIED MATHEMATICS-XII

36
Time taken to travel 36 km downstream = hr
lO+i;
36
Time taken to travel 36 km upstream = hr
10-i;

It is given that the boat takes 1 hour 30 minutes less to travel 36 km downstream than to travel
the same distance upstream.
36 36 3
\Q-v 10 + y 2
1 1 3
36
AO-v 10 + iJ ; 2
lO+i’-lO + r; 3
36
(10-y)(10 + z?)J 2

w
=?●
72v _ 3
100 2

F lo
=*● 100-i;^=48i;

+ 48f/-100 = 0 => (y + 5.0) {v-2) = 0 ^ v-2 = 0^ v = 2 [vi; + 50 0 asi? >0]

ee
Fr
Hence, the speed of the stream is 2 km/hr.
nXAMPLE 10 A man roios a boat 30 km upstream and 44 km downstream in 10 hours. It he rozvs 40 km
for
upstream and 55 km downstream in 13 hours, then ifnd the rate at which he rows the boat in still water.
ur
SOLUTION Suppose the man rows the boat in still water at u km/hr and the rate of water
current is v km/hr. Then,
s
ook

Speed downstream = {u + v) km/hr


Yo

Speed upstream = {u - v) km/hr


eB

It is given that he rows the boat 30 km upstream and 44 km downstream in 10 hours.


30 44 30 44
= 10 => — + — =10, where x =u-v and i/=u + v
our
ad

u-v u+v
X y

He can also row 40 km upstream and 55 km downstream in 13 hours.


40 55
Y

+
...(h)
Re

u-v u+v X
y
nd

Multiplying (i) by 4 and (ii) by 3 and subtracting we get


Fi

176 165
= 40-39=> — =1 => y = n =>j/ + y=ll ... (iii)
y y y

Putting y = 11 in (i), we obtain


30 30
+ 4 =10^ — = 6 => .r =5 => u-v =5 ●●●(iv)
,Y Y

Adding (iii) and (iv), we obtain


2h =11 +5 => H = 8

Hence, in still water the man rows the boat at 8 km/hr.


At his usual rowing rate, Ravish can travel 12 miles downstream in a certain river in 6
hours less than it takes him to travel the same distance
upstream. But, if he could double is usual rowing
rate for his 24 mile round trip, the doivnstream 12 miles would then take only one hour less than the
upstream 12 miles. What is the speed of the water current?
BOATS & STREAMS AND PIPES & CISTERNS 3.7

SOLUTION Let Ravish's rowing rate in still water be ii km/hr and the speed of water current in
the river be v km/hr. Then,
Speed upstream = {u - v) miles/hour. Speed downstream = {ii + v) miles/hour
12
Time taken to travel 12 miles downstream = hours
H + U

12
Time taken to trave 12 miles upstream = hours
ii-v

It is given that

ow
12 12
— 6~
u-v u +v

12 12
= 6
u-v u+v

24v . 2

e
^ ~Av => u^ =v^ + 4v
2—2 = ^ "
-V

re
u -V

If Ravish doubles his rowing speed, then it is given that

Frl
F
12 12
=1
2u-v 2u + v
ou
24-0
^ = 24v => 4ii^ =v^ + 24v

r
^=l=>4u^ -V ...(ii)

so
4it -V

From (i) and (ii), we get


kf
4{v^+4v)=v^+ 24v [On eliminating u \
oo

2 8
3i.2 ^Sv => u = -
Y
B

3
8
Hence, the speed of the water current in the river is — miles/hour.
re
oY
u

EXAMPLE 12 Boat P trnvels downstream from point A to point B in 3 hours less than the time taken by
boat Q to travel upstream from point B to point C. The distance between points A and B is 20 km which is
ad

half of the distance betioeen B and C. The speed of boat Q in still water is 10 km/hr and the speed of boat P
d

in still -water is equal to the speed of boat Q upstream. Find the speed of boat P in still water.
in

SOLUTION Let the speed of boats P and Q in still water be Up and uq respectively. Let the speed
Re

of the water current be v km/hr.


F

It is given that =10 km/hr and i/p = (10-u) km/hr


L 2U km A 2U km li

Fig. 3.3
20
Time taken by boat P to travel downstream from A to B =
Up +v
40
Time taken by boat Q to travel upstream from point B to point C = -u

It is given that
40 20
-3 =
“Q -V
Up +v
40 20
-3 = 1’.‘ lip = 10-y and = 10]
10-V 10-v + v
3.8 APPLIED MATHEMATICS-XH

40
-3 = 2
10-D
40
=5 =>10-i? = 8 =>i? = 2
10-z;

»p =10-1’ ^ wp =(10-2) =8
Hence, the speed of boat P in still water is 8 km/hr.
EXAMl’LE 13 A boatman rows to a place 45 km distant and back in 20 hours. He finds that he can row
12 km loitfi the streain in the same time as 4 bn against the stream. Find the speed of the boatman in still
water and the sped of stream.
SOLUTION Let the speed of boatman instill water be u km/hr and that of stream be v km/hr.

ow
Then,
Speed downstream = (» + v) km/hr. Speed upstream = (u - v) km/hr
It is given that
45 45
= 20 -(i)

e
II+v u-v

re
12 4
and.
U + V U -V

rFl ...(ii)

F
|%^20and^
1

r
ou
9ii = 2i?' - Iv^ and 3u-2>v=u + v
liP' - 2v^ =9ii and 2i/ = 4u
fo
ks
=>

2u^ -2v^ = 9u and u = 2v


oo

2{2vf-2v^=9{2v)
Y

8v‘^-2v^=18v => =18u=> y = 3.


eB

u-2v ^ u = 6.
r

Hence, the speed of boat man in still water is 6 km/hr and the speed of current is 3 km/hr.
ou

ALlTER Suppose the boatman travels 12 km downstream in x hours. Then,


Y
ad

12
Speed downstream = km/hr - (i)
X
d

4
Speed upstream = - km/hr ...(ii)
in
Re

.Y

It is given that
F

+
Y _4 4
12 4 i:
12 4 45 3 "9 3
X X

4
Substituting y = — in (i), we obtain
3
12
Speed downstream = = 9 km/hr
4/3
4
Speed upstream = = 3 km/hr

Speed of the boatman instill water = ^ (9 + 3) = 6 km/hr.


Speed of the stream = ^(9 - 3) = 3 km/hr.
BOATS & STREAMS AND PIPES & CISTERNS 3.9

EXERCISE 3.1

1. The speed of a boat when rowing downstream is 32 km/hr, whereas when rowing
upstream it is 24 km/hr. Find the speed of the boat in still water and speed of the stream.
2. A man can row a boat 2 km against the stream in 20 minutes and returns back at the same
point in 15 minutes. Find the rate of rowing the boat and speed of the current of the river.
3. A man takes 3 hours 45 minutes to row a boat 15 km downstream of a river and 2 hours 30
minutes to cover a distance of 5 km upstream. Find the rate of rowing the boat and the
speed of river current.
4. A man can row a boat at 6 km/hr in still water. If the current of water in the river is at 2
km/hr, it takes him 3 hours to row to a place and come back. How far is the place?
A man can row a boat at 7.5 km/hr in still water. If in a river flowing at 1.5 km/hr, it takes

w
5.

him 50 minutes to row to a place and back, how far off is the place?
6. A man can row a boat in still water at the rate of 6 km/hr. If the water in a river flows at the
rate of 2 km/hr, it takes 3 hours more in upstream than to go downstream for the same

o
distance. How far is the place?

e
7. A man rows a boat 10 km upstream in a river and returns back to the starting point in 55

re
rFl
minutes. If the water in the river is flowing at the speed of 2 km/hr, find the rate at which

F
the man can row the boat in still water.
8. A boat goes 8 km upstream and then returns. Total time taken is 4 hours 16 minutes. If the

r
speed of the current is 1 km/hr, find the actual speed of the boat.
ou
9. fo
A man can row 6 km/hr in still water. It takes him twice as long to row up as to row down
ks
the river. Find the rate of the stream.

10. A man can row 9— km/hr in still water and finds that it takes him thrice as much time to
oo

row up than as to row down the same distance in the river. Find the speed of the current.
Y
eB

11. The speed of a boat in still water is 5 times that of the current, it takes 1.1 hours to row to a
point Q from point P downstream. The distance between points P and Q is 13.2 km. How
much distance will it cover ^ ^ hours upstream?
ur
ad
Yo

12. A boat can travel 36 km upstream in 5 hours. If the speed of the stream is 2.4 km/hr, how
much time will the boat take to cover a distance of 78 km downstream?
d

13. A man can row 40 km upstream and 55 km downstream in 13 hours. Also, he can row 30 km
Re
in

upstream and 44 km downstream in 10 hours. Find the speed of the man in still water and
speed of the current.
F

14. A man rows his boat a place 48 km distant in 14 hours. He finds that he can row 4 km witli
the stream in the same time as 3km agaiiist the stream. Find the rate of stream.
15. A boatman takes 2 hours for travelling downstream from a point A to a point B coming back
to point A. If AB = 6 km and the water flows in the river at 4 km/hr, find the speed of the
boatman in still water.
3 1 1 ●
16. A man can row — of a km against the stream ^ minutes and returns in 7- minutes. Find
the speed of the man in still water.
17. A man rows to a place 48 km distant and back in 14 hours. He finds that he can row 4 km
with the stream in the same time as 3 km against the stream. Find the rate of stream.
18. A man takes 2.2 times as long to row a distance upstream as to row the same distance
downstream. If he can row 55 km downstream in 2 hours 30 minutes, what is the speed of
the boat in still water?
3.10 APPLIED MATHEMATICS-XII

19. A boat covers 24 km upstream and 36 km downstream in 6 hours while it covers 36 km


upstream and 24 km downstream in 6- hours. Find the speed of boat in still water and the

speed of the steam.

ANSWERS

I. 28 km/hr, 4 km/hr 2. 7 km/hr, 1 km/hr 3. 3 km/hr, 1 km/hr


4. 8 km 5. 3 km 6. 24 km
7. 22 km/hr 8. 4 km/hr 9. 2 km/hr
10. — km/hr 11. 41.6 km 12. 6 hour 30 minutes
3

13. 8 km/hr, 3 km/hr 14. 1 km/hr 15. 8 km/hr


16. 5 km/hr 17. 1 km/hr 18. 16 km/hr

w
19. 10 km/hr, 2 km/hr

HINTS TO THE SELECTED PROBLEMS

16. Speed upstream =


3/4 _ 3/4
11
1“^
1

F lo
-- km/minute
15

ee
4 4

Fr
3/4 3 2 1
Speed downstream = km/minute
7I 4 15 10 for
2
ur
1/1 1 1 1
Speed in still water = km/minute = km/minute = —x 60 km/hr.
2115^10 12 12
s
ook
Yo

17. Suppose the man rows 4 km downstream in x hours. Then,


4 3
eB

Speed downstream = —
.Y
km/hr. Speed upstream = —
Y
km/hr.

48 J8 Y =
= 14=? - + - 14 7^ 14 1
our

4/y 3/y 4 3 12 48 2
ad

55
18. Speed downstream = = 22 km/hr.
5/2
Y
Re

22
nd

Speed upstream = -10 km/hr.


2.2
Fi

1
Hence, speed in still water = — (22 + 10) = 16 km/hr

19. Lot the speed upstream be y km/hr and speed downstream be y km/hr. Then
24 36
— + — =6
^ ... , 36 24 13
...(i) and — + —= — ...(ii)
Y
1/ Y 1/ 2
Adding (i) and (ii), we obtain
^+ i + 1^ ^ ...(iii)
Y y 2 Y y 24
Subtracting (i) from (ii), we obtain
n_n_i 1 ...(iv)
Y y 2 Y y 24
Adding (iii) and (iv), we obtain
3.11
BOATS & STREAMS AND PIPES & CISTERNS

2 6 Q
.V 24

Subtracting (iv) from (iii), we obtain


2 4

y 24

Speedofboatinstillwater = ^(12 + 8) =10 km/hr. Speed of stream = -(12-8) = 2 km/hr.


3.3 SOME USEFUL TERMS AND FORMULAE ON PIPES AND CISTERNS
The problems often asked on pipes and cisterns are similar to the problems on time and work
studied in class XL In the problems on Pipes and Cisterns, we usually come across two types of

ow
pipes described below.
INLET PIPE A pipe connected uuth a tank or a resenyoir or a cistern, that fills it, is called an inlet pipe.
OUTLET PIPE A pipe connected with a tank or a reservoir or a cistern, that empties it, is called an outlet
pipe.

e
th

re
n
If a pipe can fill a tank in n hours, then in 1 hour, it will fill \n- J part of the tank.
(i)

rFl
F
th
fn
For example, if a pipe takes 4 hours to fill a tank completely, then in 1 hour it will fill -

r
ou
of the tank.
sfo til

part of the tank,


(ii) If a pipe can empty a tank in n hours, then in 1 hour, it will empty -
k
oo

For example, if a pipe takes 6 hours to empty a tank completely, then in


i 1 hour it will
th
Y

n
B

empty — part of the tank.


re

3.3.1 SOME USEFUL RESULTS

Two pipes A and B separately can fill a tank in n^ and uq hours respectively. If A and B
ou

RESULT-1
Y
ad

together can fill the tank in n^g hours, then


J- = -L+ -
d

'Ub "6
in
Re

PROOF Let the capacity (volume) of the tank be V units.


Pipe A can fill the tank of capacity V units in hours.
F

V
In one hour pipe A will fill — units
'U

Pipe B can fill the tank of capacity V units in Jig hours.


V
In one hour pipe B will fill units

y y
units.
Thus, in one hour pipe A and pipe B together will fill +

I ”b
y y
+— units.
In hours pipe A and B together will fill
”B

But, it is given that pipes >1 and B together fill the tank completely in n^g hours.
y y ( 1 1 . 1
= 1 => — + — =
1 1
"AB ^
= y => n^B -—-I
”B) ”/\ «B 'U ”6 'Mb
3.12
APPLIED MATHEMATICS-XII

REMARK 1
// a tank of capaciti/ (volume) V units is filled by a pipe in n hour, then part of the tank filled
in 1 hour— units.
n

ih
fv
REMARK 2 If part of a tank of capacity V units is filled by a pipe in 1 hour, then the pipe takes n
V n J

hours to fill the tank completely.


RESULT-2
Two pipes A and B separately can fill a tank inn^ and n^ hours respectively. If the tank can
be filled completely when pipe A fills for 'a' hours and pipe Bfor 'b' hours, then
a b
~ + — = 1

ow
SOLUTION Let the capacity (volume) of the tank be V units.
Pipe A can fill the tank of capacity V units in n^ hours.
V
In 1 hour pipe A will fill units

e
aV

re
So, in '(?' hours, pipe A will fill units.

rFl
F
Pipe B can fill the tank of capacity V units in Ug hours.
In 1 hour pipe B will fill — units

r
«6
ou
So, in 'b' hours, pipe B will fill
bV
units.
fo
ks
If pipe A alone fills the tank for ‘a‘ hours and pipe B alone fills the tank for 'b' hours, then the tank
oo

is completely filled.
Y

aV bV b
B

a
+ = y => — + —=1
"A "a "e
re

REMARK1
Two pipes A and B separately can fdl a tank in n^ andng hours respectively. If pipes A and B
ou

th
Y
ad

are open for a and b hours respectively to fill — part of the tank, then
n )

a ^ b _m
d

Ua ng n
in
Re

RESULT-3 Pipes A, B and C can fill a tank separately in n^, tig and n^ hours respectively. If all the three
F

pipes taken together ifll the tank in ^^ours, then


1 1 1 1
* + +

^ABC tiA ng nc
PROOF Proceed as in Result-1.
REMARK 2
Pipes A and B can fill a tank in and iig hours respectively and pipe C can empty the full
tank in iiq hours. If all the pipes are opened together the tank is filled in n hours, then
1= +
n nj^ ng n^

REMARK 3 Three pipes A, B and C separately can fill a tank in n^, ng an iiq hours respectively. If the
tank can be filled completely when pipes A, B and C are open for a, b and c hours respectively, then
a

iiA ng n^
3.13
BOATS & STREAMS AND PIPES & CISTERNS

RI-MARK4 Two pipes A and B can fill a tank separately in and ng hours respectively. Pipe C can
empty completely filled tank in hours. If pipes A, B and C are opened for a, h and c hours respectively to
make the tank empty, then
a

^
RESULT-4 An inlet pipe A can fill a tank in n^ hours and an outlet pipe B can empty the full tank in
Mg hours (f?g > n^). If on opening both the pipes together, the tank can be filled in n^g hours, then
1 1 1

w
PROOF Let the capacity volume of the tank be V units. Tlien,
units
Part of the tank filled by inlet pipe Ami hour =

o
Part of the tank emptied by outlet pipe B in 1 hour = — units

e
re
V V
But,no >«4.Therefore,— <—.

Frl
F
V P
Part of the tank filled by both the pipes in 1 hour =
"B
ou
or
V P
So, part of the tank filled by both the pipe together in n^g hours = n^g
kfs
"A "B

But, two pipes together fill the tank of capacity P in n^g hours.
oo

u f 1 ^ 1-1 ^ 1
Y

Ua ”bJ l”A «bJ »AB »A %


eB

RESULT-5 An inlet pipe A can fill a lank inn/^hours and an outlet pipe Bean empty the full tank Bin Hg
ur

hours (ng < ). If on opening both the pipes together, the tank can be emptied in ?j^g hours, then
oY

_L= —- —
ad

'Mb "b "a

Two inlet pipes A and B when opened together can ifll a tank in n hours. If inlet pipe A alone
d

RESULT-6

takes 'a' hours more than n to fill the tank completely and inlet pipe B alone takes b hours more than n to
in
Re

fill the tank completely. Then, n = ^fab.


F

PROOF Let the capacity (volume) of the tank be P units.


Inlet pipe A alone takes {n a) hours to fill the tank and inlet pipe B alone takes (r + b) hours to fill
the tank. Therefore,
P
units
Part of the tank filled by inlet pipe A in 1 hour = n + a

P
Part of the tank filled by inlet pipe B in 1 hour = units

P P 'l units
Part of the tank filled by pipes A and B together in 1 hour = + n+b
( . P ^ units
So, Part of the tank filled by pipes A and B together in n hours = n \n + a n +b

But, in n hours pipes A and B together fill the tank completely.


P P
n = P
n+a n+b
3.14
APPLIED MATHEMATICS-XII

n{2n + a+b) V
(« + a) {n + b)

n(2n + a + b) = (n + a) (n + b)
2ir +n{a + b)-n^+n{(i + b) + ab
rP" = ab
u = yfnb

ow
ILLUSTRATIVE EXAMPLES

EXAMPLE 1 Two pipes A and B can fill a tank in 20 and 30 minutes respectively. If both the pipes are

used together, how lojig will it take to fill the tank?

e
SOLUTION We have, = 20 minutes, % = 30 minutes

re
1 ^ 1 ^ 1

Flr
"ab "a "b

F
1 1 1 1 5
= — + — =>
20 30 60
"AB =12
"ab "ab

Hence, pipes A and B together can fill the tank in 12 minutes.


ou"AB 12

sr
A LITER 1 Let tlie capacity of the tank be V units.
Pipe A fills the tank of capacity V units in 20 minutes.
y
Part of the tank filled by pipe /4 is 1 minute = — units.
fo
k
oo
20

Pipe B fills the tank of capacity V units in 30 minutes.


Y
reB

Part of the tank filled by pipe B in 1 minute = — units.


30

Thus, part of the tank filled by pipes A and B together in 1 minute = V_


V
uY

units = — units.
20 30 12

Hence, pipes A and B together can fill the tank in 12 minutes.


ad
do

A LITER 2 Let the capacity of tank be LCM (20, 30) = 60 units.


Then, Efficiency of pipe A = — = 3 units/minute
in

20
60
Re

Efficiency of pipe B =— =2 units/minute


30
F

Combined efficiency of pipes A and B = (3 + 2) = 5 units/minute


Hence, time taken by two pipes to completely fill the tank = — = 12 minutes.
5
EXAMPLE 2
One pipe can fill a tank three times as fast as another pipe. If together the tzuo pipes can fill
the tank in 36 minutes, how much time the slower pipe alone take to fill the tank?
SOLUTION Let the slower pipe alone fill the tank in n minutes. Then the faster pipe alone will
n
fill the tank in
- minutes. It is given that two pipes together can fill the tank in 36 minutes.
1 1 1 1 1 1
Putting ^ and n^j^ = 36 in "AB
— = - + —
36 n n/3 "a "b

36 n n
1 4
= — => /7 = 144
36 n
3.15
BOATS & STREAMS AND PIPES & CISTERNS

Hence, the slower pipe alone will fill the tank in 144 minutes.
EXAMPLE 3 A pump can fill a tank xuilh water in 2 hours. Because of a teak, it took 2— hours to fill the
tank. In how many hours the leak can drain all the water of the tank?
SOLUTION Suppose the leak can drain all the water of the tank in n hours. Here, we have
1 7
n^ = 2, ng =n and =2 -3 hours = -3 hours
1 _ 1 1 [Using Result 4]
'Mb 'M "fJ
1 _1 1
7/3 "2 n

w
3 1 1
7 1 n
1 7-6 1 1
1 1_3 = — => »=14

lo
n 2 7 n 14 n 14

Hence, the leak can drain all the water of the tank in 14 hours.

e
re
EXAMPLE 4 Two pipes A and B can fill a tank in 24 minutes and 32 minutes respectively. If both the
rF
pipes are opened sunultaneously, after how much time B should be closed so that the tank is full in 18

F
minutes?

SOLUTION Let B be closed after n hours. Then, pipe A runs for 18 hours and B runs for n hours to

r
fo
u
fill the tank.
18 n
[Using Result 2]
ks
+ =1
24 32
Yo
3 n n
1-1^ n
oo

+ = 1 => = = -=>n = 8
4 32 32 4 32 4
B

Hence, pipe B must be closed after 8 hours.


EXAMPLE 5 Two pipes A and Bfill a tank in 12 minutes and 15 minutes respectively. If both the pipes
e

are opened simultayieously and pipe A is closed after 3 minutes, then how iijuch more time will it take tofill
ur

the tank by pipe B?


ad
Yo

SOLUTION Suppose pipe B takes n more minutes to fill the tank. Clearly, pipe A takes 3 minutes
and pipe B takes {n+ 3) minutes to fill the tank.
3 n+3
d

= 1 [Using Result 2]
Re
in

12 15
1 n+3
=1
F

4 15
n+3 1
= 1-
15 4
n+3 3
415
45
« + 3 =—
4

n = «-3
4
45-12 33
n - n = — => H = 8— minutes => n = 8 minutes 15 seconds
4 4 4

KX. :; l.E I-A large tank can be filled by two pipes A and B in 60 minutes and 40 minutes respectively.
How many minutes will it take to fill the tank from empty state ifB is usedfor half the time and A and Bfill
together for the other halp
3.16 APPLIED MATHEMATICS-XII

SOLUTION Suppose tank is filled from empty state in n hours.


If is given that B fills for half the time and A and B together for other half. This means that A fills
the tank for — hours and B for n hours.
2
n/2 n
H —1
60 40
[Using Result 2]
n 4«
+ — = 1 => = !=>« = 30
120 40 120

ow
Hence, the tank is filled in 30 minutes.
EXAMPLE 7 A tap can fill a tank in 10 minutes an another can empty it in 6 minutes. If the tank is
already tzvo-fifths fidl and both the taps are opened together, hoiv long will it take before the tank is either
filled completely or emptied completely, as the case may be?

e
SOLUTION We find that the outlet pipe is faster than the inlet pipe. So, the tank will be emptied.

re
Let the tank will be emptied in n minutes. Then,
n_n _ 2
[Using Remark 1 of Result 2 on page 5.2]

Flr
6 10 ”5

F
5n - 3n _ 2

n 2
ou
sr
= — ^ n=b
15 5

Hence, the tank will be emptied in 6 minutes.


fo
k
EXAMPLES A cistern has two inlet pipes A and B. Pipe A alone can fill the cistern in 10 minutes earlier
oo
than pipe B alone. If pipes A and B together can fill the cistern in 12 minutes, hozu much time will pipe A
alone can take to fill the cistern?
Y
reB

SOLUTION Suppose pipe A alone can fill the cistern in n minutes. Then, pipe B alone can fill the
cistern in (n +10) minutes. Thus, we have
=11, ng =?2 +10 and »^g =12
uY

-L=J- +J-
»6
ad
do

J_=l -J:
12 n n + 10
in

1 _ 2n + 10
Re

12 H^ + 10;i
F

n^ + lOn = 24n + 120


n^ -14«-120=0
- 20fi + 6« -120 = 0

{n - 20) (?i + 6) = 0 => w - 20 = 0 ^ n = 20 [●-● n + 6 0 as > 0]


Hence, pipe A alone will take 20 minutes to fill the cistern completely.
EXAMPLE 9 Three pipes A, B and C fill a tank in 10 hours, 12 hours and 15 hours respectively. If all
pipes are opened together, how much time do they take to fill the tank completely?
SOLUTION Let three pipes when opened together fill the tank in n hours.
We have, n^ =10, ng =12 and nr =15-
1 1 1 1
1 +
n
't
3.17
BOATS & STREAMS AND PIPES & CISTERNS

1 1 1 1 1 6+5+4 1 1
= — + — + — => - = - => ?j = 4
n 10 12 15 n 60 n 4

Hence, three pipes together can fill the tank in 4 hours.


EXAMI’Lino A tank can be filled in 5 hours by three pipes A, B and C. The pipe C is twice asfast as Band
pipe B is twice as fast as pipe A. How much time each pipe alone takes to fill the tank?
X

SOLUTION Suppose pipe A takes x hours to fill the tank completely. Then, pipes B and C take -

hours and ^ hours respectively. It is given that the tank is filled completely by three pipes in 5
hours. Thus, we have

ow
Y
and II =5

1 1 1 1
— s h *

n n^ ng tiQ
1 1

e
+

re
5
1 1 2 4
X
1 7
x/2

Fl
.t/4

F
-= — + — + — => - =- => ,y = 35
5 -t -V Y 5 Y
ur 1 3
Hence, pipes A, B and C take 35 hours, 17 - hours and 8- hours respectively.

r
I XANin i; 11 Pipes A and B can fill a tank in 20 hours and 30 hours respectively and pipe C can empty
fo
ks
the full tank in 40 hours. If all the pipes are opened together, how much time will be needed to make the tank
Yo

full?
oo

SOLUTION Let the tank be filled completely in n hours. It is given that


eB

=20, Jig =30 and n^ =40


1 _ J__^ J ]_
ur

n
”A "b 'kz
ad
Yo

n ~ 20 30 40
d

1 6 + 4-3 ^ 7
Re

120 " 120


in

120
F

Ji = hours = 17 — hours
7 7

1
Hence, the tank is filled in 17 — hours.
7

) ' 'MI’I I A pipe A canfdl a tank in 3 hours. There are hvo outlet pipes B and C from the tank which
IJ

can empty it in land 10 hours respectively. If all the three pipes are opened simultaneously, how long will
it take to fill the tank?
SOLUTION Let the tank be filled in n hours. It is given that ii^ = 3, Jig = 7 and = 10.
1 _ J 1_
n n^ Jig nc
1 1 70-30-21 1 19 210 1
=> ^ JI = = 11 —
JI 3 7 10 JI 210 JI 210 19 19

Hence, the tank is filled in 11 — hours.


19
3.18
APPLIED MATHEMATICS-XII

EXAMPLE 13 Tivo pipes A and B can separately fill a cistern in 60 minutes and 75 minutes respectively.
There is a third pipe in the bottotn of the cistern to empty it. If all the three pipes are simultaneously
opened, then the cistern is full in 50 minutes. In hozo much time, the third pipe alone can empty the
cistern ?

SOLUTION Let the third pipe C alone can empty fully filled cistern in uq minutes.
We have, = 60, n^ =75 and n=50
1_
n n^
1 1 1 1

50 ~ 60 ^ nc
_1_ _ 1 1 1 _5 + 4-6 _ 1
lie 60 75 50 ~ 300 “lOO
=>
uc =100

w
Hence, the third pipe alone can empty the cistern in 100 minutes.

F lo
3
EXAMPLE 14 Three pipes A, B and C when opened together can fill a tank in —- hours. Pipes B and C
together take 2 hours to fill the tank -while pipes A and C together take 3 hours to fill the tank. How long
will the pipes A and B together take to fill the tank completely?

ree
SOLUTION Let the pipes A, B and C separately take n^, n^ and hours respectively to fill

F
the tank completely. It is given that for
1 1 ^ 1 _ 1 1
1 ^J__2
't 3/2 iiA Up nc 3
r
_L+± -i ...(ii)
You
oks

)iQ nc 2
J- +
eBo

and
...(iii)
't 3
Adding (ii) and (iii), we obtain
our
ad

1 1 1 1 1 1
— + — + — +— =- + —
[iiA np nc 2 3
2
[Using (i)]
dY

3 6
Re

1 5_2_^
Fin

6 3~6

Putting — = — in (i), we obtain


nc 6
1 1 1 2 1 1 1
-f h — = —

tiA liptiA lip 3 66 3 iiA Up 2


Hence, pipes A and B together take 2 hours to fill the tank.
EXAMPLE 15
Two pipes A and B can fill a tank in 12 minutes and 15 minutes respectively while a third
pipe C can empty thefidl tank in 20 minutes. All the three pipes are opened in the beginning but pipe C is
closed 6 minutes before the tank is filled. In xvhat time will the tank be full?
SOLUTION Let the tank be full in n minutes. Pipes A and B fill the tank for n minutes and pipe
C is kept open for {n - 6) minutes.
n n (n-6) = 1
2^ [Using Remark 3 on page 5.3]
BOATS & STREAMS AND PIPES & CISTERNS 3.19

5n + 4n-3(?i-6) = 1 => +18 = 60 => 6» = 42 n =7


60

Hence, the tank will be filled in 7 minutes.


rxAMPLE i6 A cistern has three pipes A, B and C. Pipes A a}id B are inlet pipes whereas C is an outlet
pipe. Pipes A and B can fill the cistern separately in 3 hours and 4 hours respectively while pipe C can
empty the completelyfilled cistern in 1 hour. If the pipes A, B and C are opened in order at 5,6 and 1 AM
respectively, at what time will the cistern be empty?
SOLUTION Let the cistern be emptied n hours after 5 AM. Clearly, pipes A and B fill the tank for
n hours and {n-1) hours respectively, while pipe C empties the tank for (>? - 2) hours.

ow
{n^2)
= 0 [Using Remark 4 on page 5.3]
3 4 1

4/1+ 3(f7-l)-12(;i-2) = 0 => -5/1 + 21 = 0 21


/7 = — hours ^ /i = 4 hours 12 minutes
12 5

e
Hence, the cistern will be emptied 4 hours 12 minutes after 5 AM i.e. at 12 minutes past 9 AM.

re
rFl
liXAMPl.i; 17 Two pipes A and B can fill a cistern in 12 minutes and 15 minutes respectively zuhile a

F
third pipe C can empty the full cistern in 6 minutes. A and B are kept open for 5 minutes in the begimnng
and then C is also opened. In what time is the cistern emptied?

r
SOLUTION Let the cistern be emptied after n minutes of opening of pipes A and B. Clearly,
ou
fo
pipes A and Bare kept open fornminutes each while pipeC for(/i-5) minutes to make the cistern
ks
completely empty.
(/7-5)
oo

n n
= 0 [Using Remark 4 on page 5.3]
6
Y
eB

5/r + 4//-10(/J-5) = 0 => ●// + 50 =0 => n =50


60
r

Hence, the cistern is emptied after 50 minutes of opening of A and B or after 45 minutes of
ou
ad
Y

opening of pipe C.
lxampm: ih
A tank is fitted with two ijilet pipes A and B and an outlet pipe C. A is twice as efficient as
B which in turn is twice as efficient as C. The empty tank getsfilled in 16 hours luhenall the three pipes are
d

opened. How many hours will be taken toifll the empty tank ifB is plugged and other two pipes are open.
Re
in

SOLUTION Suppose A alone takes n hours to fill the tank. Then, B alone takes 2n hours to fill the
F

tank and C alone takes 4/i hours to empty the tank. The empty tank gets filled in 16 hours when
all the three pipes are opened.
J_1
16 n 2/1 4/2
1 4 + 2-1 1 5
=>l=-=>/i = 20
16 4n 16 4/2 4 n

Let the tank be filled in x hours when pipe B is plugged and pipes A and C are kept open. Then,
i=i-—
X n An

i-A ^ ^ => 80 ^^2


.v = —=26-
X 4n X 4x20 3 3
2
Hence, A and C will fill the tank in 26— hours i.e. 26 hours 40 minutes.
3
3.20 APPLIED MATHEMATICS-XII

EXAMPLE 19 A vessel has three pipes connected to it, two to supply liquid and one to draw liquid. The
fist alone can fill the vessel in 4- hours, the second
in 3 hours and the third can empty it in 1 ^ hours. If all
the pipes are opened simultaneously when the vessel is halffull, how soon zoill it be emptied?
SOLUTION Let the vessel be filled in n hours. In n hours the vessel is half filled. This means that
if each pipe is kept open for n hours the tank is half-filled.
n n ^ _1

ow
~9”^3 T~2
2n+3n-6?i 1 n 1
^ = -4i
= — => n = —
9 2 9 2 2 2
1

e
Hence, the vessel will be emptied in 4^ hours.

re
EXAMPLE 20 A swimming pool is filled by three pipes A, B and C with uniform flow. The first two pipes

Frl
F
A and B operating simultaneously fill the pool in the same time during which the pool is filled by the pipe C
alone. The second pipe B fills the pool 5 hours faster than the ifrst pipe A and 4 hours slower than the third
pipe C. Find the time required by the first pipe to fill the swimming pool.
ou
sor
SOLUTION Let the pipes A, B and C fill the swimming pool separately in , Mg and n^ hours
respectively. It is given that
^AB ^
kf
1 ^ 1
oo
=>

^AB 't
Y

1 1 1
B

=>
, Mg = Myi - 5 and n^ = m^ - 9
”B 't
re

1 1 1
oY

Ua n^ -5 M^ -9
u

-5 1
ad

m^(m^-5) m^-9
d

(2m^ -5) (m^ -9) =m^(m^ -5)


in
Re

2 -23n^ +45 =(»a)^


F

(m^)^-18m^ +45 = 0
(m^-15) (m^-3)=0
=>
n/^ -15 = 0 [●.● Mg > 0 and 11(2 >0 n^ >5 and n^^ >9]
Ha =15
Hence, the first pipe A alone takes 15 hours to fill the tank.
EXAMPLE 22 A tank has two outlet pipes A and B, zuhich together take 6 hours to empty a fidl tank when
they are opened simultaneously. The tank was initially half-full and both the outlets were opened after an
hour, an inlet pipe C was also opened. If the inlet alone can fill the empty tank in 4 hours, how much time
zoill it now take to fill the tank completely?
SOLUTION Let the inlet pipe C take n hours to fill the tank after output pipes A and B were
opened. Further, let outlet pipes A and B take separately and Mg hours respectively to fill the
empty tank. Since the tank was half-full when outlet pipes A and B were opened and after an
hour an inlet pipe C was opened. In order to fill the remaining half tank, one inlet pipe remains
open for (m-1) hours and two outlet pipes remain open for m hours.
3.21
BOATS & STREAMS AND PIPES & CISTERNS

(n-1) -n
1 1 1

4 2

n-1 11 1 1 1
76 (given)
4 6 2 “A %

3{n-l)-2n 1 „
' ' =—=> h-3 = 6=^*?i = 9.
12 2

Hence, the tank will be filled in 9 hours.


EXAMPLE 2.3 Tzoo pipes A and B can fill a tank in 6 hours and 4 hours respectively. If they are opened on
alternate hours and if pipe A is opened first, in how many hours, the tank shall be full?
th
(1 1 5
SOLUTION Pipes A and 6 can fill - + - — part of the tank if each is open for 1 hour
V6 4j U2
alternately i.e. after two hours.

w
( 5 \ 5
Part of the tank filled in 4 hours = 2 — =-
[12 6

F lo
/ 1 Nth
Now, — part of the tank remains unfilled and it is A's turn. A fills the tank completely in 6
UJ ^

ee
th

part in 1 hour. Hence, the tank is filled in (4 +1) = 5 hours.

Fr
hours. Therefore, it fills —
UJ ^
Three taps A, B and C can ifll a lank in 12 hours, 15 hours and 20/io«rs respectively. If A
EXAMPLE 24
for
is
open all the time and B and C are open for one hour each alternately, in how many hours will the tank be
ur
full?
SOLUTION It is given that tap A always remains open and taps B and C are opened alternately
s
ook
Yo

for one hour each. So, after two hours tap A has filled the tank for 2 hours, B for one hour and C
for one hour.
eB

f 2 1 J_ 17
Part of the tank filled after 2 hours = — +
12 15 20 60
our
ad

17 1
We find that — < — < —.
4 60 3
3 < 3x
- „ —17 <1, and,,1 <4x
, —
17 <—.
4
dY

4 60 60 3
Re

So, after 3 rounds of 2 hours each the lank will be partially unfilled and in 4th rounds it
Fin

overflows.
17
Part of the tank filled in 6 (= 2 x 3) hours = 3 x
60~20
17 3
Remaining (unfilled) part of the tank = 1 - 20 20

It is now the turn of A and B


1_
Part of the tank filled by A and B in 1 hour = 12 "^15 20‘
So, the unfilled part of the tank is filled by A and B in 1 hour.
So, the unfilled part of the tank is filled by A and B in 1 hour.
Hence, the tank is filled in (6 + 1) hours i.e. 7 hours.
EXAMPLE 25 Tzoo pipes A and B can fill a tank separately in 10 hours and 15 hours respectively. Third
pipe C can empty the full tank in 20 hours. Pipes A, B and C are opened alternately for one hour each. IfA
is opened first, then how many hours will they take to fill the empty tank?
3.22 APPLIED MATHEMATICS-XII

SOLUTION In first three hours pipes A, B and C are opened alternately for one hour each.
Part of the tank filled by pipes A, B and C in first three hours =
We find that
1 7 1
- < < —
9 60
8 7 7 9
< 8 X — <1 and 1 < 9 x — < —
9 60 60 8

So, after 8 rounds of 3 hours each the tank will be partially empty and in 9 rounds it overflows.
Part of the tank filled after 8 rounds i.e. after 8 x 3 = 24 hours = 8 x — = —
60 15
14^ 1
Unfilled part of the tank = 1 - —
15; 15
Now, its A's turn to fill the tank

w
The tank is filled by pipe A in 10 hour
1 f 1 ^ 2
—^
15‘"

F lo
part of the tank will be filled by pipe/I in vl5
Hence, the tank is completely filled in 24 hours 40 minutes.
—xlO J = —hours
3
= 40 minutes

HXAMPLE26 Pipes A and B Can fill a tank separately in 20 minutes and 30 minutes respectively. PipeC

e
Fre
can empty the full tank in 15 minutes. If pipes A, B and C are opened successively for one minute each, hozo
soon will the tank be filled? for
SOLUTION In first three hours pipes A, B and C are opened successively for one minute each.
.'. Part of the tank filled by three pipes in first 3 minu tes = 'J_ J__2_ 1
r
= —. We find that in
20 30 15 60
You
oks

1 11
55 rounds of 3 minutes each, the part of the tank filled is 55 x —
eBo

60 12

Unfilled part of the tank = 1 11 = JL


12 "l2
1
our
ad

After 55 rounds of 3 minutes each i.e. after 55 x 3 =165 minutes only — th part of the tank

remains unfilled.

Now, it is A's turn to fill the tank.


dY
Re

Part filled by A in 1 minute = —


Fin

20

Unfilled part left = —12 201- = —


30

Now, it is B's turn to fill the tank. In one minute B can fill —th part of the tank whiclt is same as
30 ^
the unfilled part. So, the tank is completely filled.
Hence, the tank is filled in 165 + 1 + 1 = 167 minutes i.e. 2 hours 47 minutes.
lA AMi'i 1 ● Tzuo pipes can fill a tank in 20 and 24 minutes respectively and a zvaste pipe can empty 3
litres per minute. Alt the three pipes working together can fill the tank in 15 minutes. Find the capacity of
the tank.

SOLUTION Let the waste pipe can empty the full tank in n minutes.
It is given that the tank is filled in 15 minutes when all three pipes are opened.
J___l_ 1 1 1 1 1 -=i-=>?j=40.
15 "20 "^24 n n 20 ^ 24 15 40
3.23
BOATS & STREAMS AND PIPES & CISTERNS

Hence, the waste pipe can empty the full tank in 40 minutes.
The rate at which the waste pipe empties the tank is 3 litres per minute. Therefore, the waste
pipe empties 40 x 3 litres = 120 litres in 40 minutes. Hence, the capacity of the tank is 120 litres.
hXAMi’i I ● A cistern can be filled by two pipes filling separately in 12 and 16 minutes respectively.
Both the pipes are opened together for a certain time but being clogged, only -- of the full quantity of water

flows through the former and only ^ through the later pipe. The obstructions, hoioever, being suddenly
removed, thecisterji isfilled in 3 minutesfrom that moment. How long was it before the fullflow began?
SOLUTION Suppose the full flow began after n minutes. Then, part of the tank filled by both the
pipes with obstruction in n minutes.

w
7 1 5 1 n
= n —X — + -X —
8 12 6 16,/ 8
/ 3 3 7

Flo
Part of the tank filled by the both the pipes with full flow in 3 minutes = — + -7 =— ... (ii)
12 16 7 Id

ee
But,

Fr
Part of the tank filled by both the pipes with obstruction in n minutes
+ Part of the tank filled by both the pipes with full flow in 3 minutes = Full tank.
n +—
7 =1^ ^ n n 9
9=4! for
ur
- = — => n =
8 16 8 16 8 16 2 2

Hence, the full flow began after 4 — minutes.


s
° 2
ok
Yo

EXERCISE 3.2
Bo

1. Two pipes 2l and B can fill a tank in 24 hours and 30 hours respectively. If the pipes are
re

opened together in the empty tank, how much time will be taken by them to fill it?
2. Two pipes A and B can fill a tank in 18 hours and 6 hours respectively. If both the pipes are
ou
ad

opened simultaneously, how much time will be taken to fill the tank?
Pipe A can fill a tank in 30 minutes and pipe B can fill it in 45 minutes. If both the pipes are
Y

3.

opened in the empty tank, how much time will they take to fill it?
nd

Two pipes A and B together can fill a cistern in 4 hours. Had they been opened separately,
Re

4.

then B would have taken 6 hours more then A to fill the cistern. How much time will be
Fi

taken by pipe A alone to fill the cistern?


5. Two pipes A and B can fill a tank in 15 minutes and 20 minutes respectively. Both the pipes
are
opened together but after 4 minutes, pipe A is turned off. What is the total time required
to fill the tank?

6. Two pipes A and B can separately fill a cistern in 37^ minutes and 45 minutes respectively.
Both the pipes are opened. After how much time pipe B should be turned off so that the
cistern is filled in just half an hour?
7. A cistern has two pipes. One can fill it with water in 8 hours and other can empty in 5 hours.
In how many hours will the cistern be emptied if botlT he pipes are opened together when —
of the cistern is already full of water?
A tap can fill a tank in 48 minutes whereas another tap can empty it in 2 hours. If both the
taps are opened at 9 : 40 AM, at what time the tank will be filled?
3.24 APPLIED MATHEMATICS-XII

9. Two pipes can fill a tank in 12 hours and 16 hours respectively. A third pipe can empty the
tank in 30 hours. If aU the three pipes are opened and function simultaneously, how much
time will the tank take to be full?

10. A tap can completely fill a water tank in 8 hours. The water tank has a hole in it through
which the water leaks out. The leakage will cause the full water tank to get empty in 12
hours. How much time will it take for the tap to fill the tank completely with the hole?
11. Three pipes A, B and C are connected to a tank. Out of the three A and B are inlet pipes and
C is the outlet pipe. If opened separately, A fills the tank in 10 hours, B fills the tank in 12
hours and C empties the tank in 30 hours. If all three are opened simultaneously, how much
time does it take to fill the tank?

12. Three pipes A, B and C are connected to a tank. Out of three, A is the inlet pipe and B and C

w
are outlet pipes. If opened separately, A fills the tank in 10 hours, B empties the tank in 12
hours and C empties the tank in 30 hours. If all three pipes are opened simultaneously, how

Flo
much time does it take to fill the empty tank?
13. Three pipes A, B and C are connected to a cistern. Out of three A and B are the inlet pipes

ee
and C is the outlet pipe. If opened separately, A fills the tank in 10 hours and B fills the tank

Fr
in 30 hours. If all the three pipes are opened together, if takes 30 minutes extra than if only A
and B are opened. How much time does it take to empty the tank if only pipe C is opened?
for
14. Time required by two pipes A and B working separately to fill a tank is 36 minutes and 45
ur
minutes respectively. Another pipe C can empty the tank in 30 minutes. Initially, A and B
are opened and after 7 minutes, C is also opened. In how much more time the tank would be
ks

completely filled?
Yo
oo

15. Two pipes A and B can fill a tank in 20 hours and 30 hours respectively. If both the pipes are
opened simultaneously, find after how much time should pipe B be closed so that the tank
eB

is full in 18 hours?

16. Two pipes A and B can fill a tank in 15 hours and 20 hours respectively while a third pipe C
r
ou

can empty the full tank in 25 hours. All the three pipes are opened in the beginning. After 10
ad

hours, pipe C is closed. In how many hours will the tank be full?
Y

17. Two pipes A and B can fill a tank in 15 and 12 hours respectively and a third pipe C can
empty it in 4 hours. If the pipes A, B and C are opened at 8 AM, 9 AM and 11 AM
nd
Re

respectively, at what time will the tank be fully empty?


Fi

18. Two pipes A and B can fill a tank separately in 20 hours and 30 hours respectively. Both the
pipes are opened to fill the tank but when the tank is one-third full, a leak develops in the
tank through which one third water supplied by both the pipes goes out. How long will it
take to fill the tank?

19. A leak in the bottom of a tank can empty the full tank in 8 hours. An inlet pipe fills water at
the rate of 6 litres a minute. When the tank is full, the inlet is opened but due to the leak, the
tank is empty in 12 hours. How many litres does the tank hold?
20. Two pipes A and B can fill a cistern in 14 hours and 16 hours respectively. Pipes A and B are
opened simultaneously and it is found that due to leakage in the bottom, 32 minutes extra
are taken for the cistern to be filled up. If the cistern is full, in what time would the leak
empty it?
21. Two inlet pipes A and B can fill a tank in 60 minutes and 30 minutes respectively. An outlet
pipe C drains 24 litres of water in a minute. If all three pipes are opened when the tank is
full, the tank gets emptied in 60 minutes. How much water can the tank hold?
3.25
BOATS & STREAMS AND PIPES & CISTERNS

22. Two pipes A and B can fill a cistern in 32 minutes and 48 minutes respectively. Both the
pipes being opened, find when pipe B be closed so that the cistern gets filled in 24 minutes?
_ ^ANSWERS
1
2. 4—hours 3. 18 minutes 4. 6 hours
1. 13 hours, 20 minutes
2

14 minutes, 40 seconds 6. 9 minutes 7. 10 hours 8. 11 AM

8 12. 60 hours
9. 8 - hours 10. 24 hours n. 6 hours 40 minutes
9

13. 120 hours 14. 39 seconds 15. 3 hours 16. 12 hours 17. 2; 40 PM

ow
18. 16 hours u*. 8640 litres 20. 112 hours 21. 360 litres

22. 12 minutes
HINTS TO SELECTED PROBLEMS

5. Let the tank be filled in u minutes. Pipe A is open for 4 minutes and pipe B for n minutes to

e
re
fill the tank.
n 11 44

rFl
F
15 20 20 15 3

6. Let the pipe B be turned off after n minutes. So, pipe A fills for half an hour and pipe B runs

r
for n minutes to fill the cistern completely.
ou
30

45*^75/2
= -I1 => —// + —
4 =!=>
. —n = -=>
1 n =Q
9 fo
ks
45 5 45 5

7. Let the cistern be emptied in « hours. Then,


oo

)j n 3 3n
= - =>?3=10
Y

5 8 4 40 4
eB

8. Let the tank be filled in n minutes. Then,


n u 531 - 2n
= 1=> = 1=> 3n = 240=>n = 80
r

48 120 240
ou

Hence, the tank is filled after 80 minutes i.e. at 11 AM


ad
Y

10. Let be tank be filled in ii hours. Then


d

ij 8 12 n 24
Re
in

■1 We have, Ha =10, 3Id = 12 and iir = 30. Let the tank be filled in n hours. Then,
1^^ j___l_ 1 1 1 1 1 6+5-2 3 20
F

= — => n = —
n 10 12 30 n 60 20 3
n ug ?ic
20
Hence, the tank is filled in — hours i.e. 6 hours 40 minutes.
3
12. Letthetankbefilled/emptiedcoiTipletelyin3ihours.Wehave,f3^ =10,Mg =12andfi^ -30.
M 10 12 30 M 60 60
n Ua »b
Hence, the tank is emptied in 60 hours
13. Let the pipe C takes jiq hours to empty the tank completely.
1
We have, = 10, >ig = 30, ?i^bC = +2
1 1 1 1 1 4 15
Now, — —I- — => = — + = => n An = —

'Me 'M ”e 'Mb 10 30 30 2


3.26
APPLIED MATHEMATICS-XIl

1 15 1
= 8
"ABC - "AB 2 ^ ^T 2
Now,
1 1 1 1
1 — 1 J 1 1
8 ~ 10 30 15 8 120
=> tiQ = 120
"ABC "A "B 't lie fie

14. Let the tank be filled after u minutes of opening of C. Clearly, A and B are opened for (?? + 7)
minutes and C for n minutes.
11 +7 n + 7 n 5{n + 7) + 4{n + 7)-6n
— =1 => = 1=> 3« + 63=180=> 3n = 117=>»i = 39
36 45 30 180

15. Let the tank B be closed after n hours. Clearly, pipe A is opened for 18 hours and pipe 6 for n
hours to fill the tank.
18 11 II 11 1

w
1 =1 ^ - — ^ n = 3
20 30 30 10 30 10

16. Let the tank be full after ii hours of the closeof pipe C. Then, pipes A and 6 both are kept

Flo
open for (n +10) hours and pipe C for 10 hours to fill the tank.
(/7 + 10) (77 + 10) 10
— =1 ^ 7 (77 +10) -24 = 60 ^ 7 (77 + 10) = 84 ^ 77 + 10 =12 ^ n~2

ee
15 20 25

Fr
18. Let the tank be filled in n hours. Then,
1 n , 1 77 -=>77=16
120 30 3 12 3
for
ur
1 1 ^ 1
19. Part of the tank filled in 1 hour = -
8 12 24
s
1 1
k
Yo

Part of the tank filled in 1 minute =


oo

24x60 1440
th
eB

1 A
Thus part of volume of the tank = 6 litres
1440

Volume of the tank = (1440 x 6) litres = 8640 litres


r
ou
ad

20. Let the full cistern can be emptied by leak in ii hours. Further, let the time taken by two
pipes to fill the cistern be 77| hours. Then,
Y

J-=J_ 1 _ 15 112

77i“14 16^77i"112 =^"1 =


15
Re
nd

112 32
Time taken by two pipes to fill the tank when leakage is open = = 8 hours.
Fi

15 60

8 14 16 n

MULTIPLE CHOICE QUESTIONS (MCQs)


Each of the following questions has four choices (a), (b), (c) and (d), out ofzohicli only one is correct. Mark
the correct choice in each question.
1. A man can row upstream at 10 km/hr and downstream at 18 km/hr. Man's rate in still
water in km/hr is
(a) 14 (b) 4 (c) 12 (d) 10
2. A boat covers 8 km in one hour along the stream and 2 km in one hour against the stream.
The speed of the stream in km/hr is
(a) 3 (b) 5 (c) 2 (d) 4
3.27
BOATS & STREAMS AND PIPES & CISTERNS

3. A man rows downstream 32 km and 14 km upstream. If he takes 6 hours to cover each


distance, then the speed of the stream in km/hr is
(a)i (b) 1 (d) 2

4. A man rows at a speed of 8 km/hr in still water to a certain distance upstream and back to
the starting point in a river which flows at 4 km/hr. The average speed of the journey in
km/hr, is
(a) 12 (b) 6 (c) 4 (d) 8
5. A boat goes downstream at u km/hr and upstream at v km/hr. The speed of the stream, in
km/hr, is
1
(a) -{u-v) (b) u~v (c) + (d) u + v

ow
6. A boat goes downstream at u km/hr and upstream at u km/hr. The speed of the boat in still
water, in km/hr, is
1 1
(a) (b) u-v (c) - {u + v) (d) u + v

e
Fl
A man rows 40 km upstream in 8 hours and a distance of 36 km downstream in 6 hours. The

re
7.

speed of the stream is

F
1 1
(a) — km/hr (b) 5— km/hr (c) 1 km/hr (d) 11 km/hr
ur
r
8. A boatman rows 1 km in 5 minutes, along the stream and 6 km in 1 hour against the stream.
fo
The speed of the stream is
ks
(a) 6 km/hr (b) 10 km/hr (c) 12 km/hr (d) 3 km/hr
Yo

9. The speed of a boat in still water is 10 km/hr. It is can travel 26 km downstream and 14 km
oo

upstream in the same time, the speed of the stream, in km/hr, is


B

(a) 2 (b) 3 (c) 4


(.f
re

10. A boat goes 12 km upstream in 48 minutes. If the speed of the stream is 2 km/hr, the speed
u

of boat in still water is


ad
Yo

(a) 13 km/hr (b) 17 km/hr (c) 6.5 km/hr (d) 8.5 km/hr
11. A man can row a boat in still water at 15 km/hr and speed of water current is 5 km/hr. The
distance covered by the boat downstream in 24 minutes is
d
Re

(b) 8 km (c) 6 km (d) 16 km


in

(a) 4 km
12. A man rows fr km upstream and back again in f hours. If he can row in still water at u km/hr
F

and the rate of stream is v km/lir, then f =


2 2
2nd 2nd
(d)^
U -V

(a) -2
U -V
2 (b)
d (c) U 2 +V
2 d

13. If a man goes 18 km downstream in 4 hours and returns against the stream in 12 hours, then
the speed of the stream in km/hr is
7
(a) 1
(d)f (d) 3

14. If a boat goes 7 km upstream in 42 minutes and the speed of the stream is 3 km/hr, then the
speed of the boat in still water is
(a) 4.2 km/hr (b) 9 km/hr (c) 13 km/hr (d) 21 km/hr
15. Two pipes A and B can fill a tank in 20 minutes and 30 minutes respectively. If both the
pipes are opened together, the time taken by them to half-full the cistern is
(a) 12 minutes (b) 6 minutes (c) 9 minutes (d) 18 minutes
3.28 APPLIED MATHEMATICS-XII

3
l('. An outlet pipe can empty a cistern in 3 hours. The time taken by it to empty — rd of the
cistern is

(a) 2 hours (b) 3 hours (c) 4 hours (d) 6 hours


th
3
' . A pipe fills — part of a tank in 1 hour. The rest of the tank can be filled in
7 3
(a) —hours
3
(b) - hours
4
(c) — hours
3 (d) 44 hours
A pipe can fill a tank in hours and another pipe can empty it in «2 (^2 ^ ”l) hours. If both
the pipes are opened together, the tank will be filled in
«1«2 «1«2

w
(a) (jii-^2) hours (b) hours (c) hours (d) hours
«2 -”1 ”1 -»2

A cistern has two taps which fill it in 12 minutes and 15 minutes respectively. There is also a

Flo
waste pipe in the cistern. When all the three pipes are opened, the empty cistern is full in 20
minutes. How long will the waste pipe take to empty the full cistern?

e
(a) 10 minutes

re
(b) 8 minutes (c) 20 minutes (d) 5 minutes
A pipe can fill a tank in 3 hours. Because of a leak in the tank the pipe takes 3 hours 30

F
minutes to fill the tank. How much time will the leak take to empty the full tank?
ur
(a) 7 hours (b) 6Vi hours (c) Vi hours (d) 21 hours

r
fo
There is a leak in the bottom of a cistern. Before the leak, it could be filled in 4Vi hours. It

now takes Vi hour longer. If the cistern is full, how long will the leakage take to empty the
ks
full cistern?
Yo
oo

(a) 30 hours (b) 5 hours (c) 45 hours (d) 15 hours


A pipa A can fill a tank in 25 minutes and pipe B can empty the full tank in 50 minutes. The
B

time taken by two pipes to fill the tank is


re

(a) 20 minutes (b) 25 minutes (c) 30 minutes (d) 10 minutes


th
r3
u

1
ad

— of a cistern is filled by a pipe in 12 minutes. How much time does it take to fill - of the
Yo

cistern?

(a) 16 minutes (b) 15 minutes (c) 8 minutes (d) 10 minutes


d
Re
in

A pipe can empty — of a cistern in 20 minutes. What part of cistern will be emptied in 9
6
F

minutes

(a) I (C) I8 (d)l8


A pipe can fill an empty cistern in 5 hours. A leak develops in the cistern due to which full
cistern is emptied in 30 hours. With the leak, the cistern can be filled in
(a) 8 hours (b) 6 hours (c) 10 hours (d) 12 hours
Two pipes A and B can fill a tank in 30 hours and 20 hours respectively. Pipe B is kept open
for half the time and both pipes are kept open for the remaining time. The number of hours
taken to fill the tank is
(a) 25 hours (b) 40 hours (c) 35 hours (d) 15 hours
Two pipes A and B can fill a cistern in 10 minutes and 15 minutes respectively. Both the
pipes are opened together, but after 3 minutes pipe B in turned off. How much time will the
cistern take to be full?
3.29
BOATS & STREAMS AND PIPES & CISTERNS

(a) 12 minutes (b) 8 minutes (c) 6 minutes (d) 11 minutes


28. Two pipes A and B can fill a cistern in 10 minutes and 15 minutes respectively. Pipe C can
empty the full cistern in 5 minutes. Pipes A and B are kept open for 4 minutes and then
outlet pipe C is also opened. The cistern is emptied by the output pipe C in
(a) 24 hours (b) 20 hours (c) 18 hours (d) 30 hours
29. Pipe A can fill a tank 6 times faster than a pipe B. If B can fill a tank in 21 minutes, then time
taken by botla the pipes to fill the cistern is
(a) 3 minutes (b) 9 minutes (c) 4Vi minutes (d) 7 minutes
30. Pipes A and B together can fill a cistern in 6 minutes. If A takes 5 minutes less than B to fill
the cistern, then the time in which B alone can fill the cistern is
(a) 15 minutes (b) 10 minutes (c) 30 minutes (d) 25 minutes

ow
31. A cistern normally takes 10 hours to be filled by a tap but because of one open outlet pipe, it
takes 5 hours more

(a) 20 hours (b) 24 hours (c) 30 hours (d) 12 hours


32. Two pipes can fill a tank separately in 12 and 20 hours respectively. The pipes are opened
simultaneously and it is found that due to leakage in the bottom, 30 minutes extra are taken

e
Fl
re
for the cistern to be filled up. If the cistern is full, in what time would the leak empty it?
(c) 115 hours (d) 112 hours

F
(a) 120 hours (b) 100 hours
Two pipes A and B can fill a cistern in 36 and 48 minutes respectively. Both pipes
are
33.
ur
opened together, after how many minutes should B be turned off, so that the cistern be

r
filled in 24 minutes? fo (d) 12 minutes
(a) 6 minutes (b) 16 minutes (c) 10 minutes
ks
Two pipes A and 6 can fill a tank in 20 and 16 hours respectively. Pipe B alone is kept open
Yo
34.
oo

for — of time and both pipes are kept open for the remaining time. In how many hours, the
4
B

tank will be full?

(d) 12^2 hours


re

1
(a) 18— hours (b) 20 hours (c) 10 hours
3
u

A tank has a leak which would empty it in 10 hours. A tap is turned on which delivers 4 litre
ad

35.
Yo

a minute into the tank and now it emptied in 12 hours. The capacity of the tank is
(a) 648 litres (b) 1440 litres (c) 1200 litres (d) 1800 litres
d

36. Three pipes A, B and C can fill a tank in 10,20 and 30 hours respectively. If A is open for all
Re
in

the time and B and C are open for one hour each alternately, then the tank will be full in
(d) 7 ^ hours
1
F

(a) 6 hours (b) 6-hours (c) 7 hours


2

37. A cistern is filled in 20 minutes by three pipes A, B and C. The pipe C is twice as fast as B and
pipe B is thrice as fast as pipe A. How much time will pipe A alone take to fill the tank?
(a) 200 minutes (b) 205 minutes (c) 352 minutes (d) 180 minutes
38. Three pipes A, B and C can fill separately a cistern in 12,16 and 20 minutes. Pipe A is opened
first and pipe B and C are opened after every 2 minutes interval. The time after which the
cistern will be full, is (after the opening of pipe A)
(a) 36 minutes (b) 32 — minutes (c) 34 —minutes (d) 36 —
47
minutes
47 47

39. Two pipes A and 6 can fill a cistern in 30 and 20 minutes respectively. They started to fill a
cistern together but pipe A is closed few minutes later and pipe C fills the remaining part of
the cistern in 5 minutes. After how many minutes was pipe A closed?
(a) 9 minutes (b) 10 minutes (c) 12 minutes (d) 14 minutes
3.30 APPLIED MATHEMATICS-XII

ANSWERS
1. (b) 2. (a) 3. (c) 4. (c) 5. (a) 6. (d) 7. (c) 8. (b)
9. (c) 10. (c) n. (b) 12. (d) 13. (b) 14. (c) 15. (b) 16. (a)
17. (c) 18. (c) 19. (a) 20. (d) 21. (c) 22. (b) 23. (c) 24. (c)
25. (b) 26. (d) 27. (b) 28. (b) 29. (a) 30. (a) 31. (c) 32. (a)
3.3. (b) 34. (c) 35. (b) 36. (c) 37. (a) 38. (d) 39. (a)

w
F lo
e
Fre
for
r
You
s
ook
eB
our
ad dY
Re
Fin
CHAPTER 4
PARTNERSHIP

4.1 SOME DEFINITIONS

w
PARTNERSHIP Partnership is the relationship behueen tivo or more persons who have agreed for setting
up a business, running it and sharing its profits and losses.
Section 4 of Indian Partnership Act 1932 defines a partnersliip as the relation between persons
whose have agreed to share the profits of a business carried on by all or any one of them acting

o
e
for all. Thus, we can say that in partnership, two or more persons carry on a business and share

re
the profits of the business at an agreed proportion.

Frl
The persons xvho have entered into a parhiership with one another are called individual

F
PARTNERS

partners.
When a partner devotes his time for the business in addition to his investment, he is called a
ou
r
working or an active partner and when a partner merely invests money in the business, he is

so
known as a silent or a sleeping partner. According to the Partnership Act, the maximum number
kf
of partners in a firm is ten in the case of a banking business and twenty in the case of any other
business.
oo

FIRM The name of the business under which all partners conduct their business activities is called the
Y

finn.he firm.
B

Collectively partirers are the owners of the firm.


re

CAPITAL The total investment in the business is known as the capital of the business.
oY
u

4.2 ESSENTIAL ELEMENTS OF PARTNERSHIP


ad

There arc three essential elements in the definition of partnership as explained below.
d

AGREEMENT Partnership comes into existence on account of an agreement among the partners zuhich
max/ be verbal or zvritten. So, there jnust be an agreement among the partners of the business.
in
Re

PROFIT-SHARING The aim of the partnership is to share profits or losses. So, there must be an agreement
F

amo7ig the partners to share profits or losses of the business.


PRINCIPAL-AGENT RELATIONSHIP The business activities tnust be carried on bij all or any of them
ac
ting for all. An agreement between the partners is called Partnership Agreement or Partnership Deed.
This agreement explains the rights and duties of individual partners.
In the absence of any one of the above mentioned features, a partnership firm cannot be treated
as a case of partnership firm.
4.3 PARTNERSHIP DEED

As discussed in the previous section that an agreement between the partners of a firm is called
the partnership deed or partnership agreement. This agreement explains the duties and rights of
individual partners. The partnership deed provides the following in formations:
(i) It states the amount of capital contributed or to be contributed by each partner,
(ii) It states the proportion in wliich the profits of the business are to be shared and losses
borne by the individual partners,
(iii) It states the commission to partners either on net profits or on orders secured by them.
4.2 APPLIED MATHEMATICS-XII

(iv) It states the rate of interest, if any, on partner's capitals and drawings,
(v) It states the salary, if any, of each partner,
(vi) It states about the maximum amount which each partner is entitled to draw from the firm
periodically or throughout the financial year in anticipation of his share of profit,
(vii) It states the rate of interest on loan taken by any partner.
For a partnership deed to be effective, it must be in writing and duly registered.
In the absence of the partnership deed, section 13 of Indian Partnership Act will operate which
are listed below:

(i) No interest shall be allowed on the partner's capitals,


(ii) No interest shall be charged on the partner's drawings,
(iii) Interest at 6% per annum shall be allowed on partner's loan to the firm,
(iv) No salary, remuneration or commission shall be paid to any partner for taking part in
carrying out the business of the firm,
(v) All partners must share profits equally and contribute equally to the losses suffered by

w
the firm, irrespective of the amounts of their capitals.
4.4 SOME IMPORTANT POINTS ABOUT PARTNERSHIP

The following are some observations about partnership :


F lo
(i) A partner alone cannot take all the profits of the business but he can take the

e
responsibility of meeting all losses of the business in case he desires to do so.

Fre
(ii) A person can get a share of profit of a business without being a partner of the business.
for
For example, an employee of a business may sometimes be allowed to take a share of
profits of the business in lieu of his remuneration, although he is not a partner of the
business,
r
(iii) Generally, partnership deed provides the ratio in which the partners will share profits. If
You
oks

such a provision is not provided in the partnership deed, partiiers may share profits in
eBo

equal ratio. They may provide in the deed to share profits in capital ratio. If the
partnership deed provides to share the profits in capital ratio and partners put in fresh
capital or withdraw capital during the year, then the fresh capital introduced or capital
ad

withdrawn should be taken into consideration to compute the effective capital ratio. In
our

such a case, profit is divided in the ratio of effective capital investments of the partners.
4.5 INVESTMENT OF CAPITAL FOR EQUAL PERIODS
Re
dY

If partners of a firm invest capital in the business for equal periods, then the profits or losses of
the business at the end of the year are divided among the partners in the ratio of their
Fin

investments.

The following algorithm explains the procedure.


algorithl;

M .rp I
Obtain the investments of the partners.
SI i:r II
Compute the ratio of the capitals of the partners. This ratio is called the profit-sharing ratio.
Let there be n partners in the business and their profit-sharing ratio be Aj : ^2 ^ ^3 ^ — ● ^n-
s; u.
Obtain the total profit. Let the total profit be ofX P.
S'l i !
Compute the profits of the partners by using the following procedure:

Profit offirst partner = ? X P


+ (72 4fJ
4.3
PARTNERSHIP

X P and so on.
Profit of second partner = ?
fl2 + ^2

Foliowing examples illustrate the above algorithm.

ILLUSTRATIVE EXAMPLES

EXAMri.r.i X, y, Z invested ? 126,000, ^ 84,000 and ? 210,000 respectively in a partnership business.


Find their share in profit oft 242,000 after a year.
SOLUTION Ratio of investments of X, Y and Z = 126,000 : 84,000 : 210,000 =3:2:5

ow
Sum of the ratios = 3 + 2 + 5 = 10.
Total profit = ? 242,000
Share of X = ? I—X 242,000 =? 72,600
10

e
re
Share of Y = ? — X 242,000 = t 48,400
UOI J

Frl
F
Share of Z = Ax
10
242,000l = ? 121,000.
Three partners A, B and C start a business. Twice the investment of A is equal to thrice the

r
ou
EXAMPLE 2

pitai ofB and capital ofB isfour times the capital of C. Find the share ofeach out ofa profit of 1297,000.
o
ca
kfs
SOLUTION Let C's capital be t x. Then, B's capital = t 4a‘
Now, 2 (/I's capital) = 3 (B’s capital)
oo

2 (A's capital) = 3 x 4.v


Y

=> A's capital = ? 6x.


B

Now, Ratio of investments of A, B and C = 6.r: 4.v: .t = 6:4; 1


re

Profit-sharing ratio = 6:4:1


Total profit =? 297,000.
oYu
ad

Sum of the ratios = 6 + 4 + 1=11

A's share = t -ix 297,000 = ^162,000


d

11
in

-^X 297,000
Re

B's share = t = 1108,000


11
F

( 1
C's share =? —x 297,000 =? 27,000
11

EXAMPl E 3 A, B, C and D enter into a partnership. A, B, C subscribe ^


— of the capital
3 4 5 ^
respectively and D, the rest. How should they divide a profit oft 2,460?
SOLUTION Let the total capital be of t x. Then,

Capital of A = ? -, Capital of B = ^, Capital oiC = t~,


Capital of D = t x
A _A ,1^
3 4 5 60
XXX 13a
= 20:15:12:13
Ratio of Capitals of A, B, C and D = ^:
3 4 5 60

Profit-sharing ratio =20:15:12:13


4.4
APPLIED MATHEMATICS-XK

Total profit =?2,460.


Sum of the ratios = 20 +15 + 12 + 13 = 60.
f 20
A's share = ? — x 2,460 = ^ 820
60

B's share = ?(—x


60
2,460 -?615
fl5
Cs share =? —x 2,460 =?492
Uo
f 13
D's share = ? — x 2,460 = ?533.
60

UX.^MPLE 4 Three partners invested ? 24,000 in business. At the end of the year one got ? 337.50, the
second X 1,125.00 and the third ? 637.50 as proift. How much amount did each invest ?

w
SOLUTION It is given that profits are in the same ratio as investments.

F lo
Ratios of profits = 33750:1125 : 63750
Sum of the ratios = 33750 + 1125 + 63750 = 2,100
Total investment = ? 14,000

ee
33750

Fr
Investment of first partner X 14,000 =?2,250
V 2100

/1125 for
Investment of second partner = ? X 14,000 =?7,500
2100
ur
Investment of third partner ^ f63750 X 14,000 =?4250.
s

2100
ook
Yo

l.\ ●-.Ml’LE 3
A, B andC are partners in a business with a total capital of^ 240,000. They agreed to
share the profit in the ratio of their capitals. At the end of the year, A receives 112000 as his share of the
eB

profit, B receives ? 23,000 and C receives ? 20,000. Find the capitals of A, B and C.
SOLUTION We have.
r
ad
ou

Total capital = ? 140,000


Ratio of profits = 12,000 :13,000 :10,000 = 12:13:10
Y

Sum of the ratios =12 + 13 + 10 = 35


f 12
Re
nd

Capital of = ? —x 140,000 =^48,000


35
Fi

(13
Capital of /I = ? X 140,000 =? 52,000
35

(10
Capital of C = ? X 140,000 =?40,000
35

EXAMPLE 6
Two partners A and B invested ? 1,250 and ? 850 respectively in a business. They decided
to distribute equally the 60% of the profit, and the remaining 40% as the interest on their capitals. If A
received ? 30 more than B,find the total profit.
SOLUTION Let the total profit be ? x. It is given that 40% of the total profit is distributed as the
( 40 2
interest on the capitals of the two partners. This means that ? X X = ?— X are distributed
100 5

between A and B in the ratio of their capitals.


Now, ratio of capitals =1250:850=25:17
Profit-sharing ratio = 25 : 17
PARTNERSHIP 4.5

Sum of the ratios = 25 + 17 = 42.


25 2
Interest received hy A = t — x — x
^ 42 5 ; 21

17 2
Interest received by B = ? —x — x
^ 42 5 . 105

It is given that A received X 30 more than B.


5x _17x 8x 30 X 105
+ 30=> = 30 => X = = 393.75
21 “ 105 105

Hence, total profit = ? 393.75.


EXA\tPLi: 7 A, B and C are partners. A reccwes 2/3 of the profits, B and C dividing the remainder

ow
equal!}/. A's income is increased b}/ X 200 when the rate of profit rises from 5 to 7 percent. Find the capital
ofB.
SOLUTION Since profits are divided in the ratio of the investments. Therefore, if we assume
that the total capital is X x. Tlien,

e
2

re
A's capital - ^
{ 2
rFl
F
X
Remaining capital -X x —-X =X-.
^ 3 3

r
Since the remaining profit is divided equally among B and C.
ou
B's capital =C's capital = ? -.
X
fo
ks
6

It is given that A's income is increased by X 200 when the rate of profit rises from 5% to 7%. Tliis
oo

means that due to increase of 2% in the rate of profit, the income is increased by X 200.
Y

2% of A's capital = 200


B

^ X — = 200
re

100 3
200 X 300
= 15,000.
ou

X =
Y

4
ad

fx^ 15,000
.'. Capital of B = ? — = X = X 2,500.
V 6 ;
d

3
in
Re

EXAMPLE 8 A, B and Care partners in a business. A receives — of the total profit while B and C share
the remainder equal!}/. A’s profit is increased by X 1,500, when the rate of profit is increased from 10% to
F

12% in a year. Hnd B's share in the total profit.


SOLUTION Let the total profit be X x. Then,
3x
A's profit = X
X

B's profit - ^ ^
X

C's profit “ ^ g
It is given that the A's profit is increased by X. 1500 when the rate of profit increases from 10% to
12%. Therefore,
2% ofXx = ^1,500
2x
=> = 1500
100
4.6 APPLIED MATHEMATICS-XII

X =75,000

Share in the total profit = ? — — = ? 15,000.


^ [ 5 J
EX.4MPLE 9
A, B and C agree to receive profit from their business each in the proportion to their
investments. A and B put back into the business their shares of the profit each year but C does not.
Initially, A invests 15,000, B ^10,000 and C K 30,000. If the profit for the first year is ^ 9000 and for
the second year is ? 30,000, calculate the arnount of money each has in the business during the third year.
SOLUTION We have.
Ratio of investments of A, B and C =5,000:10,000: 30,000 = 1:2:6
Total profit for the first year = ? 9,000.
Slim of the ratios = l + 2 + 6 = 9

ow
(I
A's share = ? — x 9,000 = ? 1,000.
^9

B's share = ? -X 9,000 = ? 2,000


9

e
Fl
re
C's share = ? -X 9,000 =? 6,000.
9

F
It is given that A and B put back their profits in the business each year. Tlierefore, for the second
ur
year, we have

r
Capital of A = ? (5,000 +1,000) = ? 6,000 fo
Capital ofB = t (10,000 + 2000) = ?12,000
ks
Capital of C = ?" 30,000.
Yo
oo

Ratio of capitals of A, B and C = 6,000:12,000 : 30,000 = 1:2:5


Profit-sharing ration = 1:2:5
B

Total profit for the second year = ?10,000.


re

Sum of the ratios = l + 2 + 5 = 8


fl ^
u

A's share = r -x 10,000 =? 1,250.


ad

8
Yo

B's share = ? -X 10,000 =? 2,500


8
d
Re

C's share = ? f-8 x 10,000


in

= ? 6,250.
F

Now, A and B put back their shares of profit in the business. So, their capitals in the tliird year
are as follows:

Capital of A = ? (6,000 +1,250) = ? 7,250.


Capital of B = t (12,000 + 2,500) = ? 14,500.
Capital of C remains same i.e. ? 30,000.
EXAMPLE 10
Rashid, Vijay and fohn entered into a partnership investing ^250,000, ^325,000 and
t 400,000 respectively and agreed to share the profits in the ratio of the capitals invested by them. The
business yielded a total annual proift of ^ 1,260,000. If an amount of ^ 7,500 per month is paid out of the
profit towards rent of the business premises, find the share of each in the remaining profit.
SOLUTION We have. Monthly rent = ? 7,500.
Yearly rent = ? (7500 x 12) = ? 90,000
Hence, Net profit = Total profit-Yearly rent = ? (1.260,000-90,0 00) = ? 1,170,000
Profit-sharing ratio = 250,000 : 325,000 : 400,000 =10:13:16
PARTNERSHIP 4.7

Sum of the ratios = 10 +13 +16 = 39


10
Rashid's profit =? —x
39
1,170,000 =^ 30,000
13
Vijay's profit = ? —x
39
1,170,000 =? 390,000
16
John's profit = ? —x 1,170,000 =^ 480,000
39

EXAMI-!.! M
Four persons enter into partnership, the second has twice as large capital as the first, and
the third as much as half the sum of the capital of the first two and the fourth has a sum equal to the capital
of the others. Distribute the total profits of ^ 1,800,000 amongst the partners in proportion to the
capital
SOLUTION Let the capital invested by the first partner be ? .y. Then

w
Capital invested by the second partner =^2x
3x
Capital invested by the third partner = ^

and.

F lo
ee
3.y^
Capital invested by the fourth partner = ? V .r + 2y + —2

Fr
2

3y 9y
Profit sharing ratio = y:2y:-^:-^ = 2:4:3:9 for
ur
Sum of the ratios = 2 + 4+ 3 + 9=18

^X 1,800,000
s

Profit received by first partner = ? = ? 200,000


ook

18
Yo

Profit received by second partner = ? — X 1,800,000 = ? 400,000


eB

18

f 3
Profit received by third partner = ? - X 1,800,000 =? 300,000
r

U8
ad
ou

and.
f 9
Y

Profit received by fourth partner = ? —x


18
1,800,000 =? 900,000
Re
nd

RXAMFLE 12 A, B and C entered into a partnership luith a capital of ? 700,000, ? 800,000 and
Fi

? 2,400,000 respectively. The business yields a profit of^ 700,000 in a year. If an amount o/? 20,000 per
month is paid towards the rent of the business premises out of the profit, find the share of A in the profit, if
they slwre in proift in the ratio of their capitals.
SOLUTION We have.
Ratio of capitals of A, B and C =700,000 : 800,000 ; 1,400,000 =7 : 8 :14
Since the ratio of profits ot A, B and C is same as the ratio of their capitals. Therefore,
Ratio of profits of A, B and C = 7 : 8:14
Sum of the ratios =7 + 8 +14 = 29

Gross profit = ? 700,000.


Rent paid out of the gross profit = ? 10,000 per month
= ? (12x10,000) =^120,000
Net profit = ^ (700,000 -120,000) = ? 580,000
4.8 APPLIED MATHEMATICS'XII

7
Share of 71 in the net profit = ? —x 580,000 =? 140,000.
29

EXAMPLE a3 A,B,Cand D entered into a partnership. A, B, C respectively subscribed—, -


3 4 5
part of the capital and rest was subscribed by D. D is a working partner and gets 4% of the total profit for
that. The rest is divided amongst A, B, C and D in the ratio of their capitals. Find the shares of A, B, C and
D in a total annual profit of^ 1,000,000.
SOLUTION Let the total capital be of ? x. Then,
X X X
A's share in investment = —; B's share in investment = —: C's share in investment = —
3' 4' 5
fx X vA 13x
D's share in investment = x - — + — + —
3 4 5 60

w
XXX 13x
Ratio of capitals of A, B, C and D = — : — : = 20:15:12 :13
3 4 5 60

Flo
Profit sharing ratio = 20:15:12:13
Sum of the Ratios = 20 + 15 + 12 + 13 = 60

ee
Total annual profit = ^ 1,000,000.
( 4

Fr
D's share as working partner = (4% of total profit) = ? X 1,000,000 =? 40,000.
100

Remaining profit = ? 1,000,000 - ? 40,000 = ? 960,000. for


ur
1^20
A's share = ? — X 960,000 = ? 320,000
60
k s
?(— X 960,000
Yo

B's share = = ? 240,000


oo

60

12
eB

C's share = ? X 960,000 =? 192,000


60
13
r

D's share = ? —x 960,000 =? 208,000


ou
ad

60

D's total share in the profit = ? (40,000 + 208,000) = ? 248,000


Y

EXAMPLR 14 Roma and Mona invested ? 25,000 and ? 28,000 in a hardware business two years ago.
During first year, their net loss was ^ 2,640. During the 2nd year, their net gain was ? 7,590. What is the
nd
Re

present worth of each partner, if the profit (loss) is distributed in the capital ratio at the beginning of each
Fi

year?
SOLUTION Note that the present worth of each partner is equal to the original investment plus
profit minus the loss. In the first year, we have
Capital ratio of Roma and Mona =15,000:18,000 = 5:6
Loss sharing ratio = 5:6
Sum of the ratios =5 + 6 =11.

Total loss = ? 2,640

Loss born by Roma = ? ^ X 2,640


U1
= ?1,200

f 6
Loss born by Mona = ? — X 2,640 = ? 1,440
11

In the beginning of the second year, we have


PARTNERSHIP 4.9

Roma's capital = ? (15,000 -1,200) = ? 13,800


Mona's capital = ? (18,000 -1,440) = ? 16,560
Ratio of capitals = 13,800:16,560 =5:6
Profit sharing ratio =5:6
Sum of the ratios =5 + 6 =11

Total profit = ? 7,590


Roma's share of profit = ? — X 7,590 =? 3,450
11

f 6

ow
Mona's share of profit = ^ —x 7,590 =? 4,140.
11

Thus, present worth of Roma's capital = ? (13,800 + 3,450) = ? 17,250


Present worth of Mona's capital = ? (16,560 + 4,140) = ? 20,700.

e
EXAMPLE 13 Three partners A, BandC invested respectively ? 10,700, ? 12,300 and ? 17,000for two

re
rFl
years. During the ifrst year their net profit zoas ? 4,000. During the second year they earned profit of

F
? 2,000. Find the total profit of each partner.
SOLUTION It is given that

r
A's investment = ?10,700; B's investment = ? 12,300; C's investment = ? 17,000
ou
Ratio of investments = 107 :123:170 fo
ks
Profit sharing ratios = 107 :123:170
oo

Sum of the ratios = 107 +123 + 170 = 400

Total profit of first year = ? 4,000.


Y
B

For first year, we have


re

107
>l's Profit =? X 4,000 =? 1,070
Uoo
ou
Y
ad

(123
B's Profit = ? X 4,000 =?1,230
400
d

170
C's Profit = ? X 4,000 =? 1,700
in
Re

400

Thus, in the beginning of the second year, we have


F

A's total investment = ? (10,700 + 1,070) = ? 11,770


B's total investment = ^ (12,300 + 1,230) = 13,530
C's total investment = t (17000 + 1,700) = ? 18,700
Ratio of capitals = 11,770 :13,530 :18,770 = 107 :123 :170
Profit sharing ratio = 107 :123 :170
Sum of the ratios = 400

Total profit in second year = 2,000.


Thus for second year, we have
107
A's share of profit = ? X 2,000 =?535
400

123
B's share of profit = ? X 2,000 = ? 615
400
4.10 APPLIED MATHEMATICS-XII

170
C's share of profit = ? X 2,000 = ? 850.
400

Hence, A’s total profit = ? (1,070 + 535) = ? 1,605; B's total profit = ? (1,230 + 615) = ?1,845
C's total profit = ? (1,700 + 850) = ? 2,550.

EXERCISE 4.1

1. A, B and C enter into a partnership with ? 26,000, ? 34,000 and ? 10,000 respectively. Find
their shares in a profit of ? 3,500.
X, Y and Z enter into a partnership with ? 11,700, ? 3,900 and ? 9,100 respectively. How
should they divide a profit of ? 3,990 ?

w
3. A, B and C subscribe X 47,000 for a business. A subscribes X 7,000 more than B and B X 5,000
more than C. Find the share of each out of a profit of X 9,400.

Flo
4. A,B,C and D enter into a partnership. A, B,C subscribe —, -, - respectively of the capital
3 5 6

e
and D, the rest. How should they divide a profit of X 7,050?

re
5. Ramesh invests X 30,000 for one year in a business. How much his partner Mahesh should

F
invest in order that the profits after one year may be in the ratio 2:3?
6. There were three partners A, B, C in a business, and each had a share of the profits in
ur
r
proportion to his capital. A's capital was X 240, B's X 640; and A's share was X15 in every
X 100. Find B's share of profits and C's capital.
fo
ks
7. Three persons invested ? 9,000 in a joint business. The second person invested ? 1,000
Yo

more than the first and the third X 1000 more than the second. After two years, they
oo

gained X 5,400. How much will each get ?


B

8. A, B and C start business with X 6,000, X 7,500 and X 9,000 respectively. They earn X 7,020 as
re

annual profit. If X 210 per month out of the profit is paid as rent, find the share of each.
9. A, B and C entered into a partnership investing? 5,000,? 6,500 and? 8,000 respectively. The
u
ad

business yielded a total annual profit of ? 25,200. If an amount of ? 150 per month is paid out
Yo

of the profit towards the rent of the business premises, find the share of each in the
remaining profit.
d
Re

10. Three persons A, B and C subscribe ? 4,700 for a business. A subscribe ? 700 more than
in

6 and B ? 500 more than C. How much will each receive out of a profit of ? 423 ?
F

11. A, B and C enter into a partnership and decide to contribute the total capital of ?12,000
in the following manner. A will contribute 25% of the total capital, B's contribution will be
80% of C's contribution and 50% of the contribution of A and C together. What amount will

each contribute towards the capital ? If the total profit after one year is ? 2,400, find the share
of each in it.
12. Ved, Vinod and Arun started a business jointly by investing ? 420,000, ? 380,000 and
? 400,000 respectively. At the end of the years they had a net profit of ? 240,000. Find the
share in profit of each.
13. Three partners A, B, C invest in a business in such away that 6 (A's capital) = 8 (B's
capital) = 10 (C's capital). Find the shares out of a profit of ? 94,000.
14. A, 6, C subscribe ? 470,000 for a business. A subscribes ? 70,000 more than B and B
invests ? 50,000 more than C. Find the share of each out of a total profit of ? 282,000.
PARTNERSHIP 4.11

15. A, B,C enter into a partnership. A contributes one-third of the whole capital while B
contributes as much as A andC together contribute. If the profit at the end of the year is
184,000, what would each receive ?
16. The ratio of investments of two partners is 11:12 and the ratio of their profits is 2:3. If A
invested the money for 8 months, find for how much time B invested his money.
17. A, B and C entered into a partnership with a capital of ?300,000,?500,000and?800,000
respectively. The business yields a profit of ? 500,000 in a year. If an amount of ? 10,000 per
month is paid towards the rent of the business premises out of the profit, find the share of A
in the profit, if they share in the profit in the ratio of their capitals,
A, B and C entered into a partnership investing? 10,000, ? 13,000 and ? 16,000 respectively.

w
The business yielded a total annual profit of ? 50,400. If an amount of ? 300 is paid per
month out of the profits towards the rent of the business premises, find the share of each in
the remaining profit.

o
1
1 1
19. A, B, C and D entered into a partnership. A, B, C subscribed 3' 4 and - part of the capital

e
re
rFl
respectively and D the rest. How should they shared the profit of ? 954000, if they share it in
the ratio of their investments?

F
Jack and Johan entered into a partnership, investing ? 1,80,000 and ? 1,25,000 for two
years. In the first year, their net profit was ? 61,000. During the second year, their net loss

r
ou
was ? 3,050. What is the present worth of each partner? fo
Shikha and Sharda invested ? 80,000 and ? 90,000 respectively in a printing press. They
ks
●●

decided to share equally the 60% of the profit and the remaining in the ratio of their
oo

capital investments. If Shikha gets ? 8,400 less than Sharda, find total profit earned by them.
3
Y

Aman, Vishesh and Monika form a company and invest some money. Vishesh gets — of the
eB

profit and Aman and Monika share the rest of the profit equally. If Monika gets ? 440 less
ur

than Vishesh, find the profits shared by Aman, Vishesh and Monika.
ad
Yo

? 1,300,? 1,700, ? 500 ?1,890,? 630, ? 1,470


d

? 2,000, ? 3,000, ? 4,400 ? 2,350,? 1410,? 1,175,? 2,115


Re
in

? 45,000 40%, ? 720


F

? 1,200, ? 1,800, ? 2,400 ? 1,200, ? 1,500, ? 1,800


? 6,000, ? 7,800, ? 9,600 ? 198, ? 135, ? 90
n. (i) ? 300, ? 4000, ? 5,000 (ii) ? 600, ? 800, ? 1,000
12. ? 84,000, ? 76,000, ? 80,000 ? 40,000, ? 30,000, ? 24,000
14. ? 132,000, ? 90,000, ? 60,000 ● v"'. ? 28,000, ? 42,000,? 14,000
16. 11 montlis ? 71,250
18. ? 12,000,? 15,600,? 19,200
19. ? 318,000; ? 238,500; ? 190,800; ? 206,700
20. Jack: ? 216,000; Johan: ? 148,750
21. Total profit = 357,000 Shikha's share = 174,300, Sharda's share = ? 182,700
22. Aman ? 220, Monika ? 220, Vishesh ? 660
4.12 APPLIED MATHEMATICS-Xll

HINTS TO SELECTED PROBLEMS


13. We have.
6 (^'s capita!) = 8 (B's capital) = 10 (C's capital)
^ A's capital _ B's capital _ C’s capital [Dividing throughout by the LCM of 6, 8 and 10]
20 15 12

Profit-sharing ratio = 20 :15 :12


14. Let the total capital be ? .v. Then, /I's capital = ^

Now, (B's capital) = /4's capital + C's capital


=> 2 (B's capital) = A's capital + B's capital + C's capital

=> 2 (B's capital) = ? => B's capital = ^ ^

w
.r X
C's capital = ? -j x - 1
2
= ?
6,

F lo
X X
Hence, profit-sharing ratio = —: -r ^=2:3:1
3 2 6
11 X 8 2
16. Ratio of investments = Ratio of profits ^

ee
12 X .T 3

Fr
21. Let the total profit be ? .V. for
60
60% of ? -V i.e. ? X X
— X are divided equally between Shikha and Sharda.
100
r
You
3.r
Remaining profit = ? x ^ This is divided in the ratio of capitals.
s
ook

V 5

We have.
eB

Sliikha's capital = t 80,000, Sharda's capital = ? 90,000.


Ratio of capitals = 80,000 :90,000 =8:9
our
ad

=> Profit sharing ratio = 8:9 and, Sumof the ratios =17
Shikha's share = ? — X
8
2.v'l ^ 16.r ; Sharda's share = ?
(—
9 2.r _ , 18a-
X —
85 '
dY

17 5 17 5 85
Re

Total share of Shikha = ? i x — + +


16a
Fin

2 5 85 10 85

Total share of Sharda = ? f - x — +


2 5 85 10 85

It is given that Shiklia gets X 8400 less than Sharda.


3a- 16a 3a 18a
-8,400 => A = 357,000.
lo”^ 85 10
+
85

4.6 INVESTMENT OF CAPITAL FOR UNEQUAL PERIODS

When the capitals of different partners are invested in the business for different lengths of time,
then to compute profits of partners their capitals invested in the business are first adjusted to a
common period, say one year or one month, by multiplying each partner's capital by the period
for which it is invested. The profit-sharing ratio is then determined from the adjusted effective
capitals.
The following algorithm exhibits the procedure to divide the profit.
PARTNERSHIP 4.13

ALGORITHM

STEP I Obtain the investment of each partner and corresponding time for which the money remains
invested in the business.

Let there be n partners Xi,X2,...,X„ who invested ? A2,A„ respectively for


Ti,72,T3,.. T„ months respectively.
STEIMl Compute the adjusted effective capitals for each partner. The adjusted effective capitals of
Xj, X2, ...,X„for one month areC-[ = Aj xT|,C2 --A2 ^'^2' respectively.
STEP 111 Find the profit - sharing ratio from the effective capitals i.e. compute

ow
^1-^2-^3 ;C„. Also, find the sum of the ratios =C-j +C2 ●●●● + C„.
STEP IV Obtain the total profit say. ? P.
STEP V Compute the shares of partners in the profit by using :
\
C;I

e
Share ofXj = ? X P , i = 1,2,...., n.

re
+ C2 + ● ● ● + C n

Frl
Following examples illustrate the above algorithm.

F
ILLUSTRATIVE EXAMPLES
ou
or
EXAMPLE! A,B and C entered into a partnership investing ^12,000 for 4 months, ? 14,000 for
kfs
8 months and ? 10,000 for 10 months, respectively. Find the shares of each in a profit oft 5,850.
SOLUTION Effective capitals of A, B and C for one month are as follows:
oo

Effective capital of A for one month = ?(12,000 x 4) = ^ 48,000


Effective capital of B for one month = t (14,000 x 8) = ^ 112,000
Y
B

Effective capital of C for one month = t (10,000 x 10) = ? 100,000


Profit sharing ratio = 48,000 :112,000 :100,000 =12:28:25
re

Sum of the ratios = 12 + 28 + 25 = 65


oYu

Total profit = 15,850


ad

f 12 r28
/l'sshare = ? —x 5.850 =? 1080; Fsshare = ? —x 5.850 =?2.520
d

65 (.65
25
in
Re

C’s share = t X 5,850 =? 2,250.


65
F

EXAMPLE 2 Ramesh starts a business luith a capital of t400,000. After 3 months, he is joined by
Manish who brings in 1600,000 and at the end of another three months, Rahim joins with 11,000,000. If
the year's profit is 1540,000, what is the proift of each partner, if they share profits in the ratio of effective
capitals invested by them?
SOLUTION It is given that Ramesh invests ? 400,000 for 12 months, Manish invests ^ 600,000
for (12 - 3) months = 9 months and
Rahim invest 11,000,000 for (12 - 6) months = 6 months.
Effective capital of Ramesh for one month = t (400,000 x 12) = t 4,800,000
Effective capital of Manish for one month = t (600,000 x 9) = ^ 5,400,000
Effective capital of Rahim for one month = t (1,000,000 x 6) = ^ 6,000,000
So,
Profit-sharing ratio = 4,800,000 :54,000,000 : 6,000,000 =48:54:60-8:9:10
Now, 8 + 9 + 10 = 27 and profit = t 540,000
4.14 APPLIED MATHEMATICS-XII

Hence, Ramesh's share of profit = ? — x 540,000 =^160,000


27

Manish's share of profit = ? — X 540,000 = ? 180,000


10
and. Ramesh's share of profit = ? — x 540,000 = ? 200,000.
27

IIX.AMPLE 3 Ammi, Amish and Anant entered into a partnership investing ? 15,200 each for 8,10 and
12 months, respectively. Find the share of each in a profit off 37,575 after one year.
SOLUTION LetC|,C2 and C3 be the effective capitals of Aman, Amish and Anant respectively
for one month. Then,

ow
Ci = f (15,200 X 8) = ? 121,600; C2 = ? (15,200 xl0)=f 152,000
and. C3 = ? (15,200 xU) = f 182,400.
Cl: C2 : C3 = 121,600 :152,000 :182,400 =4:5:6
Thus, profit-sharing ratio 4:5:6

e
Sum of the ratios = 4 + 5 + 6 = 15

Fl
re
and. Total profit P = ? 37,575

F
f 4
Aman's share in profit = ? —x 37,575 =?10,020.
Il5
ur
f 5
Amish's share in profit = f — x 37,575 = f 12,525 or
sf
V15
/ A
k
Anant's share in profit = f -^x 37,575 = f 15,030.
Yo

U5
oo

X.AMPLE 4 A and B started a business. A invested f 200 more than B. If A invested his money for 5
B

months and Bfor 4 months and A gets f 90 more than Bout of a profit off 510, how much did each invest ?
re

SOLUTION Let the investment of B be f x. Then, Investment of A = ^ (x + 200)


Let Cj and C2 be the effective capitals of A and B respectively for one month. Then,
u
ad
Yo

Ci=f [(.Y + 200) x5] = f (5y + 1,000) and C2 = ^ (.t x 4) = ? 4.y.


Profit-sharing ratio =Ci :C2-=5y + 1,000; 4y
d

Now, 5y + 1,000-I-4y = 9y + 1,000 and, total profit = ? 510.


Re
in

5y +1000 4y
A's share = f x510 , 6's share = ? x510
F

9.Y +1,000 9y +1,000

It is given that A gets f 90 more than B.


5y +1,000 4y
x510- X 510 = 90
9y-^ 1,000 9y +1,000

510 (5y +1,000)-4yx 510 = 90 (9y +1,000)


17 (y +1,000) = 3 (9y-k 1,000) => IOy =14,000 => y =1,400.
Thus, B invested f 1,400 and A invested f (1,400 + 200) = f 1,600.
EXAMPLE 5
A and B enter into a partnership zvith capitals as 5:6; at the end of 8 months A withdraws;
if they receive profits in the ratio 5:9, find how long B's capital was used ?
SOLUTION Suppose B's capital was used for y months. Let the capitals of A and B be ? 5y and
6.V respectively. Let and C2 be the effective capitals of A and B respectively for one month.
Then,
PARTNERSHIP 4.15

Cl = ? (5.V X 8) = ? 40.V, C2 = ? (6.V X y) = ? 6.vy


Since A and B receive profits in the ratio 5 : 9. Therefore,
Cl 5 40a: 5
—L = — => = - => 1/ = 12.
C2 9 6.ry 9
Hence B's capital was used for 12 months.
HXAMJ’LE 6 Rai’i ami Ravish are partners m a business. Ravi contributes 1/4 of the capital for
15 months and Ravish receives 2/3 of the total profits. For how long was Ravish's money in the business ?
SOLUTION Let the total capital be ^ a:. Then,
X f X
Ravi's investment = ? — , Ravish's investment = ? a: - — = 5? —
3 a:.
4 I 4 4

Suppose Ravish's money was in the business for y months. Let C-j and C2 be the effective
capitals of Ravi and Ravish for one month. Then,

w
.Y ."l -15.Y . ^ -f3.v ^3.vy
Ci=? — X 15 =K — and, C9 = < — X y 4
4

Ratio of their profits = —: —=1:2


2

F lo
It is given that Ravish receives 2/3 of the total profit. So, Ravi receives 1/3 of the total profit.
1

e
3 3

Fre
But, ratio of capitals = ratio of profits
1^ ^=l=>y=10.
for
Ci:C2=1:2^
4 "" 3A:y 2
r
Thus, Ravish's money was in the business for 10 months.
You

A, B and C entered into a partnership. A subscribes one-half the capital for one-third of the
s

EXAMPLE 7
ook

time, B one-third of the capital for one-fourth of the time and C the rest of the capital for the zvhole time.
Hozv should they divide a profit of ? 2,000 ?
eB

SOLUTION Let the total capital be ? .y and the total time be y months. Then,
Y Y ( X X● \ .Y
= ?-.
A's capital = ^ Capital = ? ^ , C's capital = ^ “2
our
ad

“ 3 6

Total number of months for wliich the capital of:


y
dY

A remained in business = — B remained in business =


Re

3' 4

C remained in business =y.


Fin

LetC],C2 andC3 be the effective capitals of/I, BandC respectively for one month. Then,
f X
.Y
Ci=? — X ●==- C2 = ? - X - = ?^.
I2 3 6 ' 3 4 12 ' \ O 6

Cl -Ct :Co=-^: — ; — = 2:1:2 => Profit-sharing ration = 2:1:2


^ ^ ^ 6 12 6
Now, 2 + 1+ 2=5 and total profit =? 1,000.
A's share = ? -X 1,000 = ? 400, B's share = ? ix 1,000 = ^ 200.
5 Is
2 ^
C's share = ? - X 1,000 = ^ 400.
5 )
1 1 1
:-. A
EXAMPLE 8 A , B, C are partners in a business. Their shares are in the proportion ^ ^
withdraws half his capital at the end of 15 months and after 15 inonths more, a profit 0/? 4,340 is divided.
Find the share of each.
4.16 APPLIED MATHEMATICS-Xll

SOLUTION Let the total capital be ? .v. Then,


Capital ot A Capital of 6 = ^ Capital of C = ?

Let Cj, C2 and C3 be the effective capitals of A, B and C respectively for one month. Then,
a:
— X 15 H
1 X X .-'i
X — 15 = ?
_ 15x , ^Co = ? ~
a: _ 15-t:
X 30 = ?
2 3 2 ^ 4 2

and. C3=? -X 30 = ? 6a:.


^5
ISy ICJr

Cj :C2 : C3 =-^:: 6a: =5 :5 : 4 => Profit-sharing ratio =5:5:4


Now, 5+5 + 4 =14 and total profit = ? 4,340.

w
f 5
ShareofA=? —x 4,340 =?1,550;
14
Share of B = ^ -^x
14
4,340 = ? 1,550

Flo
4
Share of C = ? — x 4,340 = ? 1,240.
U4

e
re
i XAMI’LC 9 A, B and C enter into a partnership and invest ? 1,300; ? 3,300 and ? 3,700. After few
months an additional capital 0/? 500 /s needed which is invested B. A/fcr 32 months they closed the

rF
business with a profit of f 2,527. A's share of profit was X 627. When did B invest the additional capital ?
ur
SOLUTION Suppose B invested the additional capital of ? 500 after x months of start of the
fo
business.

LetCi,C2 andC3 be effective capitals of A, B and C respectively for one month. Then,
ks
Yo

Cj =? (1,100 X 12) =r 13,200


oo

C2 = t (1,300 X 12 + (12 - a:) X 500) = t (21,600 -500a:) and, (1,700 x 12) = ? 20,400.
B

Cl + C2 + C3 = ^ (55,200-500a:)
re

Total profit = ? 2,527


C1
A's share of profit = ?
u

X 2,527
ad

Cl +C2 +C3
Yo

13,200 132
X 2,527 =? X 2,527
55,200-500a: 552-5a:
nd
Re

But, A's share was ? 627.


Fi

132 132 X 2,527


X 2,527 =627 => 552-5.t = => 552-5.V = 532 => a:=4
552-5a: 627

Thus, B invested ? 500 after 4 months.


nXAMPLC 10 A, B and C form a partnership and contribute ? 500,000, ? 400,000 and ? 300,000
respectively toioards capital. They agree to divide the annual profit in proportion to the capital and to the
time it is in use. After 5 months, A withdrazvs ? 50,000 and B adds ? 50,000. At the end of the year, the
profit is ? 122,400. Hozv should this be shared ?
SOLUTION It is given after 5 months, A withdraws ? 50,000 and B adds ? 50,000 to the capital.
Therefore,
Effective capital invested by A = ? (500,000 for 5 months) + (? 450,000 for 7 moths)
= t (500,000 X 5 + 450,000 x 7) for one moth.
= ? 5,650,000 for one month
Effective capital invested by 6 = (? 400,000 for 5 months) + (? 450,000 for 7 months)
PARTNERSHIP 4.17

= ? (400,000 X 5 + 450,000 x 7) for one month


= ? 5,150,000 for one month
Effective capital invested by C = ? (300,000 for 12 months)
= ? (300,000 X 12) for one month = ? 3,600,000 for one month
Profit-sharing ratio =5,650,000:5,150,000: 3,600,000 =113:103:72
Now, 113 + 103 + 72 = 288

/I's share of profit = X 122,400 =?48,025


288

103
B's share of profit = ? X 122,400 =?43,775
288

ow
72
C's share of profit = ? X 122,400 =? 30,600
288

EWMPLE n Shnrma, Bhutani and Mathew had agreed to divide the profits of their partnership in
proportion to their capitals, which were ? 2,500,000, ? 2,200,000 and ? 2,800,000 respectively at the

e
beginningoftheyear. After 2 months, Sharrna added ? 200,000 to his capital and Bhutani withdrew

re
rFl
? 100,000from his capital after 3 months and Mathew added ? 200,000 to his capital after 8 months. If the

F
profit for the year was ? 938,825 what amount should each partner receive?
SOLUTION We have.

r
Effective capital of Sharma = t 2,500,000 for 2 months + t 2,700,000 for 10 months)
ou
fo
= ? 5.00,0000 + ? 27,000,000 = ? 32,000,000
ks
Effective capital of Bhutan! = (? 2,200,000 for 3 months + t 2,100,000 for 9 months)
= ? (2,200,000 X 3 + 2,100.000 x 9) = ^ 25,500,000
oo

Effective capital of Mathew = ? (1.800,000 for 8 months) + ? 2.000,000 for 4 months)


Y
eB

= ? (14,400,000 + 8,000,000) = r 22,400,000


Profit-sharing ratio = 32,000,000 : 25,500,000 : 22,400,000 = 320 : 255 : 224
Now, 320 + 255 + 224 = 799
r
ou

We have, total profit = ? 938,825.


ad
Y

( 320
Sharma's share of profit = ? X 938,825 = ? 376,000
799
d

255
Bhutani's share of profit = ? X 938,825 =? 299,625
Re
in

799
F

and. Mathew's share of profit = X 938,825 =^ 263,200


799

EXAMl’l.E i; A and B engage in a trade, their capitals being in the ratio 3:2. At the end of 3 months, A
takes out a sum equivalent to one-third of his capital and at the end of another 3 months, B puts in a sum
equivalent to zvhatA lookout. If B's profit at the end of the year is ? 11000 more than A’s, find the amount
of profit of each, profits and losses being divided on the basis of average investmerit.
SOLUTION Let the capitals of A and B be ? 3a: and I lx respectively.
Now, Effective capital of A = ? 3a: for 3 months + ? Iv for 9 months
= (? 3.V X 3 + 2x X 9) for one month
= ? 27X for one month
27 a:
A's average investment = 12

Effective capital of B = ? 2x for 6 months + ? 3x for 6 months


= ? (2a: X 6 + 3a- X 6) = ? 30a:
4.18 APPLIED MATHEMATICS-XII

30.t
B's average investment = ?
12

It is given that the profit is divided in the ratio of average investments.


27x 30.r
Profits of A and B are in the ratio = 9:10
12 12

Let the profits of A and B be of ? 9a and 10a respectively. Since B's profit is ? 11,000 more than
A's.

10rt-9fl=ll,000=>rt=ll,000
Hence,
/Ts profit = ? (9 X 11,000) = ? 99,000, B's proOl = ? (10 x 11,000) = ? 110,000
LXAMPrr 13
A, B and C enler into a partnership loith A investing ? 8,000for the whole year. In the
beginning B invests ? 5,000 and after 4 months he increased his capital to ? 6,000. In the beginning C
invests ? 3,000 and after 7 months he increases his capital to ? 5,000. In an annual profit of^ 5,250, find

w
the share of each.

F lo
SOLUTION We have.
Effective capital of A = ? 8,000 for 12 months = ? (8,000 x 12) = ^ 96,000
Effective capital of 6 = ? 5,000 for 4 months + ? 6,000 for 8 months

ee
= ? (4 X 5,000 + 8 X 6,000) = ? 68000.

Fr
Effective capital of C = ? 3,000 for 7 months + ^ 5,000 for 5 months
= f (3,000 X 7 + 5,000 X 5) = ? 46,000.
for
ur
Profit sharing ratio = 96,000 : 68,000:46,000 = 48 : 34 : 23
Sum of the ratios = 48 + 34 + 23 = 105
s
ook
Yo

Annual profit = ? 5,250


48 34
eB

.'. A's share of profit = ? X 5,250 =? 2,400, B's share of profit = ? X 5,250 =? 1,700
105 105
23
C's share of profit = ? X 5,250 =? 1,150
our
ad

105

5
IXAM1M.E14
Karina and Karishma started a business in partnership. Karina invested — of the capital
Y

11

and rest by Karishma. After 8 months Karina withdraws. If they shared the profits in the ratio5:9, find
Re
nd

the time period for which Karishma's capital was used in the business.
Fi

SOLUTION Let the total capital be ? x. Then,


5x 5x
Karina's investment = ? Karishma's investment = ? x
11 ' 11 11.

Let Karishma's capital be used for a period of n months. Then, Karina's capital is used for a
period of 8 months.
{ Sx'] 40.r
Karina's effective capital for one month = ? 8 x — = ? —u
11 11

6n.v
Karishma's effective capital for one month =
11
40.Y 6^2Y
Profit sharing ratio = 11 11
= 20: 3n

But, the profit sharing ratio is given to be 5 : 9


PARTNERSHIP 4.19

7fl S
20:3m=5:9 => — = - => 15n = 180 => n=12.
3n 9

Hence, Karishma's capital was used for 12 months.


EXAMPLII15 Awau and Yamin contributed capitals in the ratio 1 :3 in a partnership. After sometime,
Yamin withdrew his money. Thq/ distributed the yearly proift in the ratio of adjusted capitals, and Yamin
got 2/3 of the profit. Forhozo longivas Yamin's money in the business?
SOLUTION Suppose Yamin withdrew his capital after n months.
It is given that Aman and Yamin contributed capitals in the ratio 1 : 3.
So, let Aman's and Yamin's capitals be ? x and ? 3.r respectively.
Aman's effective capital at the end of the year = ? 12.r
Yamin's effective capital = ? (3.v x n) = ? 3nx

w
Profit sharing ratio = 12.r : 3nx = 4 :
It is given that Yamin got 2/3 of the profit. Therefore,

F lo
1 2
Ratio of profits = —: — = 1:2 ...(ii)
^ 3 3

ee
From (i) and (ii), we have

Fr
4 1
4:j!=1:2=> - =
n 2

EXAMPLE 16 In a partnership, A and B invested money in the ratio 4; 5. After 6 months, A withdrew
for
ur
half of his capital and after next 3 months B added the sum equivalent to that A has withdrawn. If B gets a
profit ivhich is double the share of A, then find the time after which the profit zvas distributed.
s
SOLUTION We have.
ook
Yo

Ratio of initial investments of A and B = 4:5


Let initial investments of A and B be of ? 4.r and ^ 5x respectively.
eB

Suppose profit was distributed after n months.


A invested ^ 4a: for 6 months and 2x for {n - 6) months.
r
ou
ad

A's effective investment for one month = ? [6 x 4.r + (;i - 6) x 2.\] - ? {2n + 12).r
B invested ? 5a: for 9 months and ^ 7x for (n - 9) months.
Y

B's effective investment for one month = ? [9 x 5a: + {n - 9) x 7x] = ? (7» -18) a:
Re

Ratio of investments = (2h + 12) a:: (7n -18) a: = (2>J + 12): (7;j -18)
nd

=> Ratio of profits ={2h +12) :(7n -18)


Fi

But, it is given that B gets profit which is double the share of A.


2m+ 12 1
{2f? + 12):(7M-18)=l:2 => - => 4m + 24=7m-18 => 3m = 42 => n=14.
7m-18 2

Hence, the profit was distributed after 14 months.


EXAMPLE 17 In a partnership, Naveen and Neena invested ^ 36,000 and ? 48,000 respectively. After 3
months, Naveen added ? 12,000 and after next 3 months Neena withdreiv ? 12,000. The profit was
distributed among Naveen and Neena in the ratio 15:14. Find the time period for ivhich the profit ivas
counted. The proifts and losses being distributed in the ratio of adjusted capital investments.
SOLUTION Let the profit was counted for n months.
We have.
Naveen's effective investment for one month = ? [36,000 x 3 + 48,000 x (m - 3)}
= ? (48,000 M - 36,000)
4.20 APPLIED MATHEMATICS-Xli

Neena's effective investment for one month = ? {48,000 x 6 + 36,000 x {n - 6))


= ? (36,000?!+72,000)
Ratio of profits =(48,000 ft - 36,000); (36,000 n + 72,000)
= (48?i - 36): (36?! + 72) = (4?i - 3): (3?! + 6)
But, the profit was distributed among Naveen and Neena in the ratio 15:14.
4”“3
3/1 + 6
^15
14
^ 56?i-42=45?i + 90 => ll?i=132 => ?!=12.
Hence, the profit was counted for 12 months.
t-XAMi’LL 18 A, BanciC started a business. A invested of the total capital, B invested of the total

w
capital andC invested the rest of the capital. After— of the total time period, A zuithdrew his total capital
3

and after ^4 of the total time period, B zvithdreiu his capital. C kept his capital for whole of the time period.

F lo
Find the ratio in which the total proift is divided among each partner.

ee
SOLUTION Let the total capital be ? x
, ● X X ( X

Fr
.X
/I s investment = ?-r/■ B's investment = ?—; C's investment = ? x
2' 3' 2 3 6

Let the total time period of the investment be n months. Then,


for
ur
x) = ,?
/I's effective investment for one month = ? — X —
n nx

^3 2J 6
ks
n nx
Yo

B's effective investment for one month = ? — X — = ?


oo

4 3 12
B

x nx
Cs effective investment for one month = ? nx — = ?
6) 6
re

n e- . . >tx nx nx
Ratio of mvestments = — — =2:1:2
ou

6 12 6
ad

Hence, ratio of profit = 2:1:2.


Y

i \AMPLE 19 A,B and C are partners in a business with capital of ? 900,000; ^ 800,000and^. 25Q,000
respectively. It is agreed that A, B, C will share profit and loss in the ratio 3:2:1 The deed also provides
nd
Re

that if any partner has more loss than his capital and cannot bring in any amount, the balance ivill
be shared by the other partners in the ratio of their capitals. At the end of the year there loas a loss of
Fi

? 10,02,000 and partnership was dissolved. What will each partner receive?
SOLUTION We have.
Profit and loss sharing ratio =3:2:1; Sum of the ratios = 3 + 2 + l= 6
Total loss = ? 10,02,000
f3
Tl's share of loss = ? -X 1,002,000 = ? 501,000
6

B's share of loss = ? -X 1,002,000 = ? 334,000.


6

and. C's share of loss = f -X 1,002,000 = ? 167,000.


6

Clearly, C's loss is more than his capital.


Excess of loss in C's share over his capital = ? (167,000 -150,000) = ? 17,000.
PARTNERSHIP 4.21

This loss is shared by A and B in the ratio of their capital investments i.e.
900,000 : 800,000 = 9:8.
Sum of the ratios = 9 + 8=17

A's share of excess loss = ? — X 17,000 =? 9,000


17
8
and. B's share of excess loss = ? —x 17,000 =? 8,000
17

Thus, yl's total loss = ? (501,000 + 9,000) = ? 510,000.


B's total loss = ? (334, 000 + 8,000) = ? 342,000.
It is given that partnership was disolved at the end of the year. The receipts of A, B and C when
the partnership is disolved are as follows:

w
^'s receipts = t (900,000 - 510,000) = ^ 390,000.
B's receipts = ? (800,000 - 342,000) = ? 458,000; C doesn't receive any amount.
P, Q and R are partners in a business with a total capital of^ 33,000. The profit at the end

Flo
EXAMPLE 20

of the year is ? 25,000, to be divided in proportion to their capitals. If P receives f 4,500 andQ receives
? 5,500 as their share of profits. Find R's capital.

ee
SOLUTION We have.

Fr
Total profit = ? 15,000, P's share in profit = ? 4,500, Q's share in profit = ? 5,500
R's share in profit = ? (15,000 - 4,500 -5,500) = ? 5,000
for
ur
Profit sharing ratio = 4,500 :5,500 :5,000 = 9:11:10
Sum of the ratios = 9 +11 + 10 = 30
s
no
k

R's capital = ? —x 33,000 =? 11,000.


Yo

30
oo

EXERCISE 4.2
eB

1. A, B, C enter into a partnership. A invests ? 1,200 for 4 months, B ? 1,400 for 8 months,
and C ? 1,000 for 10 months. They gain ? 585 altogether. Find the share of each.
r
ou
ad

2. Rameshand Roma entered into a partnership investing ^15,200 each for 8, 10 and 12
months, respectively. Find the share of each in a profit of ? 33,045 after a year.
Y

3. A, B and C enter into a partnership. A invests ? 6,000 for 6 months, 6 invests ? 5,000 for
9 months and C invests ? 4,500 for 12 months. Find the share of each in a profit of ? 13,500
Re
nd

after a year.
Fi

4. How should a profit of ? 4,500 be divided between two partners one of whom has
contributed ? 12,000 for 5 months and the other ? 7,500 for 4 months ?
5. There are three partners A, B and C in a business. A puts in ? 20,000 for 5 months, B ? 12,500
for 6 months and C ? 25,000 for 7 months; and the profits are ? 18,956. How should they
divide the profits ?
6. A, B, C and D entered into a partnership investing ? 12,500 each for 5, 8,9 and 10 months
respectively. A is working partner and gets 12% of the total profit for the same. Find the
share of each in a profit of ? 40,000 after 11 months.
7. A and B entered into a partnership investing ? 7,500 and ? 8,000 respectively. After one
month C also joins the business with a capital of ? 10,200. Find the share of each in a
half-yearly profit of ? 24,048.
8. A began a business with ? 6,500. After 4 months, B joined him with a certain capital. At the
end of the year, the profits were divided in the ratio 5 : 2. How much did B invest ?
9. A began a business with ? 2,800 and was joined afterwards by 6 with 13,200. When did B
join, if the profits at the end of the year where divided in the ratio 3:2?
4.22 APPLIED MATHEMATICS-Xll

10. A and S enter into a partnership for a year. A contributes ? 1,500 and B ? 2,000. After
4 months, they admit C, who contributes ^2,250. If 6 withdraws his contribution after
9 months, how would they share a profit of ? 900 at the end of the year?
11.
A and Centered into partnership. A puts into stock at first ? 2000 and at the end of
8 months ? 1,000 more. B puts in at first ?750 and at the end of 4 months ? 3,000 more,
but took out ? 1,300 at the end of 3 months more. At the years's end they had gained ?1,635.
What should each receive ?
12.
A and B enter into a partnership with investments of ? 6,000 and ? 9,000 respectively. At the
end of 3 months, C also joins with a capital of ? 7,500 while B withdraws ? 2000 after 6
months of the start of the business. Determine the share of each in a profit of ? 31,431.40
after one year.
Ill
13.
A, B and C are partners in a business and their shares are in the proportion A
3 4 5

w
withdraws half his capital at the end of 4 months and after 8 months more a profit of ? 1,694
is divided. What is A's share ?

F lo
14.
A, B and C entered into a partnership. A invested 1/4 of the total capital for 1/4 of the total
time. B invested 1/5 of the total capital for 1/2 of the total time. The remaining capital was
invested by C for the whole time. If at the end of the year, the total profit is ? 1,140, find of

ee
the share of each.

Fr
15. A puts in ? 600 more in a business than B, but B has invested his capital for 5 months while
A has invested his capital for 4 months. If the share of A is ? 48 more than that of B out of the
for
total profit of ? 528, find the capital invested by each ?
ur
16. A and Centered into a partnership investing ? 9,000 and ? 10,500 respectively. After
4 months, C also joined the business with a capital of ? 12,500 while B withdraws ? 2,000.
s
ook
Yo

Determine the share of each in a profit of ? 47,700 after a year.


17.
A starts a business with a capital of ? 1,700 and admits two partners B and C after 3
eB

and 6 months respectively. After a year, the profits are divided in the ratio 2:3:5. What
amount did B and C contribute ?
our

18.
A and B start a business by investing ? 5,000 and ? 4,500 respectively. At the end of
ad

4 months, A withdraws one third of his capital and C enters with a capital of ? 7,000. At the
end of the one year, the profits are ? 5,080, find the share of each in profit.
Y

19.
A, B and C enter into a partnership. A invests ? 6,000 for the whole year. In the beginning, B
Re

invests ? 3,000 and after 6 months, he increases his capital and makes its ? 4,000. In the
nd

beginning C invests ? 2,000 and after 4 months he increases his capital, so as to make it
Fi

? 5,000. In the annual profit of ? 4,320, find the share of each.


20.
A begins a business with X 3,000 capital. Four months later, B joins with X 2,500, three
months still later, C joins with ? 2,800. Out of the profit at the end of the year, A gets 16% for
managing the business and the balance is divided according to their capitals. If gets X 888,
find the profit that each of S and C get.
21. A, B,C enter into a partnership by investing in the proportion of —: —: —. But B withdraws
2 3 4

one half of his capital after 4 months. At the end of the year, a profit of X 175,000 is divided
among them. Find the share of each.
1 1 1
In a partnership, A invests — of the total capital for - of the whole time; 6 invests — of the
6 6 3

capital for — of the time and C invests the rest of the capital for the whole time. Find the

share of each out of a total profit of X 161,000.


PARTNERSHIP 4.23

23. Two merchants A and B enter into partnership. A puts in ? 23,250 and at the end of
the 4 months adds ? 3,750 to his capital while 6 withdraws ? 3,000 at the end of 7 months. At
the end of the year A and B receive equal shares of profits. Find how much was initially
invested by B.
24. A and B are partners in a trade, their capitals being in the ratio 4:3. At the end of 3 months,
A takes out a sum equal to one fourth of his capital, and at the end of another 3 months, B
puts in a sum equal to what A took out. If B's profit at the end of a year is ? 1,000 more than
A's, find the amount of profit of each.
25. A and B are partners in a trade, their capitals being in the ratio 3:1. At the end of 4 months,
A takes out a sum equal to one third of his capital, and at the end of another 4 months, B
puts a sum equal to what A took out. If A's profit at the end of the year is ? 9,000 more than
B's, find the amount of profit of each.
26. A,B,C enter into a partnership with A investing ? 5,000 for the whole year. In the

w
beginning, B invests ? 2,000 and after 4 months he increases his capital to ? 4,000. In the
beginningCinvests?3,000and after5months he increases his capital to ?5,000. In the

Flo
annual profit of ? 4,500, find the share of each.
27. A began business with ? 8,500. After 2 months B joined him with a certain capital. At the end

ee
of the year, the profits were divided in the ratio 5 :2. Find how much money was invested

Fr
by B.
28. Three persons A, B and C are partners in a business. They invested their capital in the ratio
of 20 ; 15 : 12. A withdraws half of his capital at the end of 6 months and two-third of
for
ur
remaining after next 3 months. B withdraws one fourth of his capital after 9 months. After a
year they divided a profit of ? 18,910 in the ratio of their adjusted capitals. Find each
s
person's share.
k
Yo

29. A and B are engaged in a trade. A invested 3/5 of the total capital of the firm and B invested
oo

the rest of the capital. After 3 months, A took one-third of his investments and at the
eB

same time B puts a sum equivalent to what A took out. If A's profit at the end of the year
is ? 1,860 less than B's profit, find the amount of profit of each, profits and losses being
divided on the basis of the adjusted capital investment.
r
ou
ad

30. Harish, Javed and Mohan are partners in a plumbing business. On April 1, Harish invested
? 50,000, Javed invested ? 40,000 and Mohan invested ? 30,000. On July 1, Harish withdrew
Y

? 10,000 and Mohan added ? 10,000 in their investments. After expenses are paid, the net
profit for the year ending 31st inarch is ? 28,800. Find each partner's share, if the profits are
Re
nd

to be divided on the basis of adjusted capital investment ratio.


Fi

ANSWERS

1. ? 108, ? 252, ? 225 2. ? 8,812,? 11,015,? 13,218


3. ? 3,600, ? 4,500, ? 5,400 4. ? 3,000,? 1,500

5. ? 5,416, ? 4,062, ? 9,478 6. ? 10,300, 8,800, 9,900,11,000


7. ? 7,515,? 8,016,? 8,517 8. ? 3,900

9. After 5 months 10. ? 300 each

11. ? 840, ? 795 12. ? 9,609.60, ? 12,812.80, ? 9,009


13. ?560 14. ? 100, ? 160, ? 880
15. ? 1,800, ? 1,200 16. ? 16,200, ? 16,500, ? 15,000
17. ? 3,400, ? 8,500 18. ? 1,600, ? 1,800, ? 1,680
19. ? 1,920, ? 1,120, ? 1,280 20. ? 360,? 252.
4.24 APPLIED MATHEMATICS-XII

21. ? 90,000, ? 40,000, ? 45,000 22. ? 7,000, X 28,000, X 126,000


23. ? 27,000 24. X 13,000, X 14,000
25. ? 21,000, ? 12,000 26. ? 1,800, ? 1,200, ? 1,500 27. ? 40,800
28. share = ? 6,400, B's share = T 6,750 C's share = ? 5760.

29. A'sprofit = ^ 8,370 B’s profit =? 10,239


30. Harish's profit = ? 102,000 Javed's profit = ? 9,600; Mohan's profit = 9,000.
^ HINTS TO SELECTED PROBLEMS
4.
We have. Cl = 12,000x5, C2 = 7,500x4
; C2 = 2 : 1 => Cl + C2 = 3.
(2 ^ (1 ■’1
So, profits of two partners are X — x 4,500 and X —x 4,500 respectively

w
\ 3 / V 3 /
7. Cl = A's effective capital for one month = ? (6 x 7,500) = X 45,000,

F lo
C2 = B's equivalent capital for one month = ? (6 x 8,000) = X 48,000,
C3 =C's equivalent capital for one month = ? (5 x 10,200) = X 51,000.

ee
Cl :C2:C3:15:16:17

Fr
8. Suppose B invested X x. Then,
6,500 X 12 5
Ratio of investments for unit time = Ratio of profits ^ for = —^ X = 3,900
8x 2
ur
12.
Let Cl, C2 and C3 be the effective investments of A, B and C respectively for one month.
Then,
s
ook

Cl = ? {12 X 6,000) = X 72,000, C2 = (6 x 9,000 + 7,000 x 6) = ? 96,000 and,


Yo

C3 = ? (9 X 7,500) = ^ 67,500. Then, Ci: C2 : C3 = 144 :192:135


eB

Let the total capital be X x and the total time be one year. Then,
X X f
A invests ? — for 2 months, B invests ? — for 4 months and C invests X x -
X
--Ifor
6 3 I 6 3)
r
ad
ou

12 months.
.r v'l 14
Y

Profit-sharing ratio - X 2 : — X 4: X - -- xl2 =~:-:6=l:4:18


3 6 3 3 3
Re
nd

4.7 PARTNER’S SALARIES, INTEREST ON 7- ; ■:s


Fi

OR L

If in the partnership deed of a firm its partners decide to take a regular salary from the firm then
it is paid from the total profit before the same is divided among the partners. If the partnership
deed provides interest on capitals, then the interest is also given out of the total profit of the firm
before it is distributed among all the partners. The remaining profit is then distributed among
the partners in the agreed proportions.
If a partner takes a loan from the firm, then the rate of interest to be charged may be given in the
partnership deed. If the interest rates are not mentioned in the deed, then as per section 13 of
Partnership Act the interest at the rate of 6% per annum is charged on the loans/advances.
The partnership deed may provide for interest to be charged on the amounts withdrawn by a
partner (drawings) during the year. If the interest rates are not mentioned in the partnership
deed then no interest is charged on the drawings by a partner.
PARTNERSHIP 4.25

ILLUSTRATIVE EXAMPLES

EXAMPLE 1 In a firm, Mukesh, Bimal and Ahmad contribute as capita! ? 2,500,000, 13,500,000
and ? 4,000,000 respectively. They agree to divide the profits in proportion totheir capitals and to pay
? 200,000 andt 300,000 as salary to Bmial and Ahmad respectively for their sewices to the firm as
Manager and General Manager respectively. What should each receive out of the net profit of^ 2,000,000?
SOLUTION We have. Ratio of the capitals = 2,500,000: 3,500,000 ; 4,000,000
= 25:35:40=5:7:8

Profit-sharing ratios =5:7 : 8


Bimal's salary = ? 200,000, Ahmad's salary = ? 300,000
Total salary = ? 500,000
Profit = ? 2,000,000
Balance profit = ? (2,000,000-500,000) = ^1,500,000
This profit is to be divided in proportion to their capitals.

w
We have.
Profit-sharing ratio =5:7:8; Sum of the ratios = 5 + 7 + 8-20
Mukesh's profit = t
f 7
20

F lo
— X 1,500,000 = ? 375,000

e
Fre
Bimal's profit = ? —
20
x 1,500,000 = ^ 525,000

X 1,500,000 =? 600,000
for
Ahmad's profit = ? 20

Total amount received by Mukesh = ? 375,000


r
You

Total amount received by Bimal = ? (525,000 + 200,000) = ? 725,000


oks

Total amount received by Ahmad = ? (600,000 + 300,000) = ? 900,000.


eBo

EXAMPLE 2 A, B and C enter into partnership with capitals oft 7,500,16,000 and 14,500 respectively.
Thepartnership agreement providesfor 5% interest on capitals, an annual sale oft 1,500 to A and expense
allowance oft 300 to C before distributing profits of the firm. Find the total sum received by each partner,
ad
our

if the total profit amounts to 16,000 and partners agree to share the profits in their capital ratio.
SOLUTION Since the partnership agreement provides 5% interest on capitals invested by the
partners. Therefore,
dY
Re

A's interest = t — X 7,500 = 1375, B's interest = X 6,000 = 1300


100 100
Fin

C's interest = X 4,500 = t 225


100

Total interest paid to the partners = t (375 + 300 + 225) = t 900 ...(i)
Now,
A's salary = 11,500, C's expense allowance = t 300
Total =? 1,800 ...(ii)
Total amoimt to be charged from the profit before distribution
= r (900+ 1,800) [Adding (i) and (ii)]
= t 2,700 ...(iii)
Total profit = t 6,000 ...(iv)
Balance profit = t (6,000 - 2,700) = ? 3,300 [Subtracting (iii) from (iv)]
This balance of the profit will now be divided among the partners in ratio of their capitals.
4.26 APPLIED MATHEMATICS'XII

We have.
Ratio of the capitals =7,500 : 6,000 : 4,500 =5:4:3
Sum of the ratios = 5 + 4+ 3=12
z' 5 f 4
A's profit = ? —
12
X 3,300 = ? 1,375; B's profit = ? 12
X 3,300 = ?1,100

C's profit = ^ — X 3,300 = ?825


12

Total amount received by A = X (375 +1,500 + 1,375) =? 3,250


Total amount received by B = ? (300 +1,100) =? 1,400
Total amount received by C = ? (225 + 300 + 825) = X 1,350
EXAMPLE 3
BImrgava and Bhandari enter into a partnership. Bhargava puts in X 5,000,000 and

w
Bhandari, X 3,000,000. Bhargava is to get 10% and Bhandari 5% of the total profits for any year as
Manager and Accountant respectively, and in addition, each is to get interest at 5% per annum for the
capital put in. The remaining profits are to be divided equally between them. If at the end of the year,

Flo
Bhargava gets X 160,000 more than what Bhandari gets, how much does each get ?

e
SOLUTION Let the total profit be X x. Then,

re
10 X
Bhargava's salary as Manager = X XX =X

F
uoo 10

I 5
ur
Bhandari's salary as Accountant = ? —

Total salary = ^ f — +—
100
X X
f 20
or
ks
10 20 20
Yo
oo

Since each partner gets interest at 5% per annum for the capital invested. Therefore,
B

Bhargava's interest = X X 5,000,000 = X 250,000


100
re

Bhandari's interest = X — X 3,000,000 = ? 150,000


1100
u
ad
Yo

Total interest = X (250,000 +150,000) = X 400,000.


3x
Balance profit = ^ ■^ .r - 20
+ 400,000
20
-400,000
d
Re
in

This balance profit is divided equally between Bhargava and Bhandari.


1 H7.Y
F

Total amount received by Bhargava = X — + 250,000 +- - 400,000 ...(i)


10 2 20

1 fl7x
Total amount received by Bhandari = X — +150,000 + -400,000 ...(ii)
20 2 I 20
Since Bhargava gets X 160,000 more than the Bhandari. Therefore,
1 ri7x 1 fl7x
— + 250,000 + - -400,000 - + 150,000 + - -400,000 = 160,000
10 2 20 20 2V 20

=> — + 100,000=160,000
20

X =1,200,000

Putting X = 1,200,000 in (i) and (ii), we get


Total amount received by Bhargava = X (120,000 + 250,000 + 510,000 - 200,000) = ? 680,000.
Total amount received by Bhandari = X (60,000 + 150,000 + 510,000 - 200,000) = X 520,000
4.27
PARTNERSHIP

EXAMPLE 4 Three partners A, B and C contribute sums of ? 75,000, fl50,000 and ? 300,000
respectively towards a venture and agree to share the profits of the venture in such a way that the rate of
return which each receives is in the proportion to the amount of his contribution. If the proifts for a
year amount to ? 94,500, how much will each receive ?
SOLUTION Since tlie rate of return i.e. rate of interest each receives is in the proportion to the
amount of his contribution. Therefore, rates of interest of A, B and C are in the ratio
75,000:150,000: 300,000=1:2:4
Let r % be the rate of return as A's capital. Then, the rates of return on B‘s and C's capitals are
respectively 2r % and 4r %.
A's share of profits = ? — X 7,500 =?750r
100

{ 2r ...(ii)
B's share of profits = ? X 150,000 = ? 3,000r
100

w
4r
and. C's share of profits = ? X 300,000 =?12,000r ...(iii)
100

Since the profits of a year amount to ? 94,500.

F lo
750r + 3,000r + 12,000r = 94,500 => 15,750r = 94,500 => r =
94,500
= 6

ee
15,750

Fr
Putting r = 6 in (i), (ii) and (iii), we get
A's share of profits = ? 4,500; B's share of profits = ? 18,000; C's share of profits = ? 72,000
for
A, B, C and D entered into a partnership investing ? 14,500 each for 5, 7,9 and 4 months
ur
EXAMPLES

respectively. C is a ivorking partner and gets 10% of the total profitfor the same. Find the share ofeach in a
s

profit o/? 40,000 after 10 months.


ook
Yo

SOLUTION Let C|, C2, C3 and C4 denote the effective capitals of A, B,C and D respectively for
eB

one month. Then,


Cl = ? (5 X 14,500) = ? 72,500; C2 = ? (7 x 14,500) = ? 101,500;
C3 = ? (9 X 14,500) = ? 130,500 and, C4 = t (4 x 14,500) = ? 58,000
our
ad

Cl :C2:C3 :C4 =72,500 :101,500 :130,500 :58,000 =145:203:261 ;116


So, profit-sharing ratio = 145 : 203 : 261:116
Y

We have, total profit = ? 40,000


Re

Share of C as working partner = 10% of ? 40,000= ^ 4,000.


nd

Remaining profit = ^ (40,000 - 4,000) = ? 36,000.


Fi

Now, 145 + 203 + 261 +116=725


145 ^203
Share of A = ^ X 36,000 = ? 7200; Share of B = ? X 36,000 =? 10,080
725 725

261 116
Share ofC = ? X 36,000 = ? 12,960; Share of D = ? X 36,000 = ? 5,760
725 725

Total share of C = ? (4,000 + 12,960) = ^ 16,960.


Hence, the sahres of A, B, C and D are ? 7,200, ? 10,080, ? 16,960 and X 5,760 respectively.
EXAMPLE 6 Rajendra with a capital ofX 2,000,000 in a business, admits Vijay into partnership. Vijay
brings X 300,000 as capital. They agree to charge interest at 5% on the capital. Vijay gets a bonus of 4% on
the turnover before profits are divided between Rajendra and Vijay in the proportion of 5:3. The turnover
during the year amounts to X 6,000,000. If the gross profit isX 747,000, find zvhat Rajendra and Vijay
received ?
4.28 APPLIED MATHEMATICS-XII

SOLUTION Rajendra's interest on capital at the rate of 5% per annum

X 1,000,000 =? 50,000
100

Vijay's interest on .capital at the rate of 5% per annum = ? — X 300,000 =?15,000


100

Vijay's bonus on the turnover of ? 6,000,000 at the rate of 4% = ? — X 6,000,000 = ? 240,000


100

Gross profit =? 747,000


Balance profit = t (747,000 -50,000-15,000 - 240,000) = ? 442,000

ow
It is given that Rajendra and Vijay share the balance profit in the ratio 5: 3.
f5
Rajendra's share in the balance profit =? - X 442,000 =? 276,250
^8
f 3

e
Vijay's share in the balance profit = ? — x 442,000 = ? 165,750

re
Frl
Hence, Total amount received by Rajendra = ? (276,250 + 50,000) = f 326,250

F
Total amount received by Vijay = ? (165,750 + 15,000 + 240,000) = ? 420,750.
EXAMPLE 7 Mnnoj and Manish entered into a partnership investing ? 12,000 and ^ 7,000 respectwely.
ou
or
Manish is a zvorking partner and gets ^ 250 per month as luorkingalloiuance. Find their shares in a profit
off 8,700 after one year.
kfs
SOLUTION
We have. Total profit = f 8,700.
oo

Manish's salary as a working partner = f (250 x 12) = ? 3,000.


Remaining profit = f (8,700 - 3,000) = f 5,700.
Y
B

Ratio of capitals of Manoj and Manish =12,000:7,000 =12:7.


Profit-sharing ratio = 12:7
re

Now, 12 + 7=19.
oYu

fl2
ad

Manoj's share =f — x 5,700 = f 3600; Manish's share = f — X 5,700 = ? 2,100.


19 19
d

Manish's total share = f (3,000 + 2,100) = f 5,100.


in

EXAMPLE 8 A, Band Care partners in a business. Their capitals are respectively f 5,000, f 6,000 and
Re

f 4,000. A gets 30% of the total profit for managing the business. The remauzing profit is divided among
F

the three in the ratio of their capitals. In the end of the year, the profit of A is f 200 more than the sum of the
profits of B and C. Find the total profit and share of each in the business.
SOLUTION Let the total profit be f x.
(30x
Amount received by A for managing the business = 30% of f x = f = rl X.
100 10
3x
Remaining profit =f x = f~.
10) 10
Now,
Ratio of capitals of A, B andC =5,000:6,000:4,000 =5:6:4.
Profit-sharing ratio =5 : 4 : 6
Now, (5+ 6 + 4) =15
Profit of A = ? 7^
15 ^ 10 30
PARTNERSHIP 4.29

/ ^ lx
Profit of B = ? —X —
15 10 25
( 4 7Y^ 14.T
Profit ofC = ? —X— =? ...(ii)
15 10 j 75

Total profit of A = ?
(3:r 1^ ...(iii)
30 J 15
+
10

It is given that:
Profit of A = Profit of B + Profit of C + 200

ow
+ 200 => 40.v = 21.r +14a:+ 15,000 => 5:r= 15000 => a- = 3,000.
15 ~ 25 75
Putting -a = 3000 in (i), (ii) and (iii), we get
Profitof A =?f—X 3,00ol = ?l,600; Profit of B = ? —
7
x 3,000 = ? 840
U5 25

e
14

re
Profit ofC=? —X 3,000 =?560.

Frl
75

F
EXAMPLE 9 Three partners A, B and C subscribe ? 3,000, ? 4,000 and ? 5,000 respectively as capital for
a biismess. A receives 15% of the profits as manager and B, 10% as assistant manager after which the
ou
remaining profits are divided in pnoportion to capital subscribed by each. Find the shares of B and C when

or
A receives ? 540.
kfs
SOLUTION Let the total profit be ? a. Then,
15a
^ 3a
oo

A's profit as manager =15% of a = ? 100 20'


Y

10a
= r^
B

B's profit as assistant manager = 10% of ? a= ? 100 10.


re

3a a
Total working profit of A and B = ? ^"^To 4
V
oYu
ad

a
Remaining profit =? a-—
V 4/ 4
d

Now, ? — are divided among A, B and C in the ratio of their investments.


in
Re

Ratio of investments of A, B and C = 3,000 14,000 :5,000 = 3:4:5.


F

Profit-sharing ratio =3:4: 5


Sum of the ratios = 3 + 4+ 5=12
f 3 3a
A's share from the remaining profit = ? ^ ^ 16

f 4 3a a
B's share from the remaining profit =? =?-4

f 5 Sa'^
C's share from the remaining profit = ^ — x — 16

3a 3a 27a
So, Total amount received by A = ? = ?
16 20 80
. \

Total amount received by^ B = ? 110


-^ + -4 ^7.
20
4.30 APPLIED MATHEMATICS-XII

But, A receives ? 540.


27 a: 540 X 80
=540 => x = = 1600.
80 27

Hence, total profit = X1600.


7a:
Total amount received by B=X X 1,600 =?560.
20 20
f S'T f Sv
Total amount received by C = ^ — =? —xl,600 =?500.
16 16

EXAMPLE 10
A, B and C have respectively X 20,000, X 18,000 and X 12,000 invested jointly in a
business. A and B receive respectively 12% and 8% of the annual profits for services, the remainder being
divided among A, B and C in proportion to their capitals. At the end of the year A receives altogether X 648
more than B. Find zuhat each receives.

SOLUTION Let the total profit at the end of a year be X x.


12a:
Profit received by A for his services = 12% of ? a: = ?

w
100 25
f Sx

Remaining profit =X x -

Sum of the ratios = 10 + 9 + 6 = 25


3a:
25 25

Profit-sharing ratio = 20,000 :18,000:12,000 =10:9:6


F lo
Profit received by B for his services = 8% of ? a: = ?

10
4a:

4.r'\
for F
100

ree
25

A's share in the remaining profit = X — X —

25 5 25
Your

( 9 4a:
. \

B's share in the remainiiAg profit = X — x — ^ , 36.Y


ks

V 25 5 125
eBoo

f 6 4x 24.r
C's share in the remaining profit =X — x — =X
V 25 5 , 125
ad

3.V 8a: ll.r


our

Total amount received by A = X + = ?


25 25 25
2.a: 36a:
Total amount received bv B = ? — + ^ 46.Y
25 125 125
Re
Y

It is given that A receives X 648 more than B.


Find

ll.r 46a: 9.t 648 X 125


+ 648 => = 648 => a: = = 72 X 125 = 9,000.
25 125 125 9
11
Total amount received by A = X — x 9,000 = X 3,960
25
46
Total amount received by B = ? X 9,000 =? 3,312
125
24
Total amount received by C = ? X 9,000 =? 1,728.
U25

EXAMPLE 11 Three men, Kaitl, Kapoor and Rao went into a business. Kaul contributing X 60,000,
/1 \
Kapur X 36,000 and Rao X 30,000 of the capital on the understanding that after allowing — th of the
profits to Rao as a Manager the remainder should be divided amongst them all in proportion to the amount
of the capital contributed by each. At the end of the year, Rao received X 8,000. What ivere the total profits
of the business and how much did Kaul and Kapoor receive ?
4.31
PARTNERSHIP

SOLUTION Let the total profit be ^ x Then,


X
Rao's salary as a Manager = ? - ...(i)
8

f ^ ●..(ii)
Remaining profit = ^ ^ ^ ~q8 8

This profit is divided among the three partners in the ratio of their capitals.
Profit-sharing ratio = 60,000 : 36,000; 30,000 =10:6:5
Sum of the ratios = 10 + 6 + 5 = 21
( 5 7.y^
Rao's profit = ? —

ow
X —

21 24

5.V
Total amount received by Rao = ? - +
^ 8 24 3

It is given that Rao received ? 8000 at the end of the year.

e
re
- = 8,000 => Y = 24,000
3

rFl
F
Thus, the total profits of the business were of ? 24,000.
Putting Y = 24,000 in (i) and (ii), we get

or
ou
Rao's salary as a manager = ? 3,000; Remaining profit = ? 21,000.
This amount is divided among the partners in the ratio of their capitals i.e. 10:6:5.
ksf
f -10
Kaul's share = ? —x 21,000 =? 10,000; Kapoor's share = ? — X 21,000 = ? 6,000
21
oo

21

( 5
Y

and. Rao's share = ? X 21,000 =? 5,000


B

21

Roy Sinhn ami Naidu were partners contributing ? 350,000, ? 280,000 and
re

EXAMPLE 12

40,000 respectively. Roy received 20% and Sinha received 2 3% of yearly profitsfor their services as the
oYu

Manager and the Deputy Manager respectively. They divided the ronaining profits among themselves in
ad

proportion to their capital contribution. At the end of the year 1998, Roy received altogether ? 30,000 more
than Sinhn. What was the total amount of trading profits after charging salaries of partners for the year
d

1998 and how jnuch profit did each receive ?


in
Re

SOLUTION Let the total trading profit be X x. Then,


13y
Roy's salary as Manager = ^ 20y
F

100 ^ ; Sinha's salary as Deputy Manager = ? 100

Total salary = ^ — + 13y^_, 33y


^5 100 100

33y'i
Remaining amount of the profit = ? y - 100 100

This amount is to be distributed among the three partners for their shares of profits in the ratio
of their capitals.
Profit-sharing ratio = 350,000 :180,000 :140,000 = 35 :18:14
Sum of the ratios = 35 + 18 + 14 = 67
35 67 Y
Roy's share of profit = ? 67
X

100 20

, , , , 18 67y 9y
Sinha s share of profit = T — x ——
^ \67 100 50
4.32 APPLIED MATHEMATICS-XII

f 14 67x
Naidu's share of profit = ? — X

V 67 100 50

fX V x'
Now, Total amount received by Roy = ? — + ~ ...(ii)
20 20

fl3x 9x^
Total amount received by Sinha =? —-h ^ Six ...(iii)
1100 50 100

It is given that Roy received ? 30,000 more than Sinha at the end of the year.
11.V 31-y 24x
= 30,000 => = 30,000 => y = 125,000
20 100 100

Hence, the required total amount of trading profits was ? 125,000.

ow
Putting X = 125,000 in (i), (ii) and (iii), we get
f 7
Total amount received by Naidu =? —x 125,000 = ? 17,500
50
f 11
Total amount received by Roy =? —x
20
125,000 =? 68,750

e
Fl
re
31
Total amount received by Sinha = ? X 125,000 =? 38,750.
uoo

F
EXAMPLE 13
A, B and C are partners in a business with a total capital o/? 33,000. The profit at the end
ur
of the year is ? 15,000, to be divided in proportion to the partner's capital. If A receives ? 4,500 as his share
of profit and B ? 5,500, ifnd C's capital.
SOLUTION
Note that if ? Pis the total profit and ? Pj, P2/ P3 ,
f or
P„ are the profits received by.v
ks
a ^ P.
such that El + ^ +... + Pn = 1
Yo
partners, then their profit-sharing ratios are
n

P' P ' ' P


oo

P P P

Here, we have : Total profit = ? 15,000; A's profit = ? 4,500; B's profit = ? 5,500
B

4,500 3 5,500 11
/I's profit-sharing ratio = —; B's profit-sharing ratio =
re

15,000 10' 15,000 ~ 30


C's profit-sharing ratio =1- T + -
11 1
u

110 30
ad

3
Yo

Hence, C's capital =


-

- X 33,000 = ? 11,000.
3
d
Re

EXAMPLE 14
X, Y and Z are partners in a business zuith capital ? 900,000, ? 800,000 and ^150,000
in

respectively. It is agreed that X, Y, Z will share profits and losses in the ratio 3:2:1. The deed provides
F

that if any partner has more loss than his capital and cannot bring in any amounts, the balance will be
shared by other partners in the ratio of their capitals. At the end of a year, there was a loss oft 100,200 and
the partnership business was dissolved. What ivill each partner receive?
SOLUTION
We have. Loss-sharing ratio = 3:2:1.
Sum of the ratios = 34-2 + 1=6.

X's share of loss = ? -X 100,200 = ? 501,000


^ 6
f2
Y's share of loss = ? -
-X 100,200 = ^334,000
6

and. Z’s share of loss =? -X 100,200 = ?167,000


6

Clearly, Z's loss is more than his capital.


Excess of loss in Z's share over his capital = ? 167,000 - ? 150,000 = ? 17,000
PARTNERSHIP 4.33

It is mentioned in the partnership deed that if any partner has more loss than his capital, then the
excess loss will be shared by the other partners in the ratio of their capitals. Therefore, the loss
of ? 17,000 will be shared by X and Y in the ratio of their capitals i.e. in the ratio 900,000 :
800,000 = 9:8.
( 9
X's share of excess loss = ? — x 17,000 = ^ 9,000
17

( 8
and. y’s share of excess loss = ? -2_x 17,000 = X 8,000.
17

Thus,

ow
X's total share of loss = X 501,000 + X 9,000 = X 510,000
y's total share of loss = X 334,000 + X 8,000 = X 342,000
Hence, after partnership business is dissolved,
X's receipt = X 900,000 - X 510,000 = X 390,000

e
re
y's receipt = X 800,000 - X 342,000 = X 458,000; Z's receipt = X 150,000 - X 150,000 =X 0
EXAMPLE 15

Frl
Mr Rno and Mr Ghosh engaged in a business sharing profits and losses in equal ratio, their

F
ca
pifals being X 4,000,000 and X 3,000,000 respectively. They decided to (i) transfer 5% of trading profits
to reserve fund and (ii) give 10% of profit, after transfer to reserve fund, to Mr Rao and 8% of such profits
ou
or
to Mr Ghosh for their service rendered. If Mr Rao received a total amount ofX 581,400for the year, then
how total amount will be received by Mr Ghosh ?
kfs
SOLUTION Let the total profit be X x. It is given that 5% of the trading profits is transferred to
the reserve fund.
oo

I 5x = X^.
Y

Amount transferred to the reserve fund = X


B

100 20

-V ^ 19.Y
re

Balance profit = X x - 20 20
oYu

Mr Rao's share of profit for his service rendered = X


19.Y 1^ ^ 190y
^ 2,000
ad

20 100

19y 8
^ 152.T
d

Mr Ghosh's share of profit for his service rendered = X 20 "" 100 2,000
in
Re

Balance profit =X x
,Y 19y 152y
^ 1558y
2,00^
F

20 2,000 2,000 >

This profit is divided equally between Mr Rao and Mr Ghosh.


1 1558 779
X =X
Mr Ghosh's share from the balance profit = X - x Y

V2 2,000 2,000
779
and. Mr Ghosh's share from the balance profit =X Y.
2,000

, ^ .
Mr Rao s total share = X
,fl90Y
——
779y 969
Thus, + Y.

2,000 2,000 2,000

It is given that the total amount received by Mr Rao is X 581,400.


969
Y =581,400 => Y = 1.200,000
2,000

Now,
4.34 APPLIED MATHEMATICS-XII

152 779
Total amount received by Mr Ghosh = ? .Y + Y
2,000 2,000

^ 931 Y=?
931
X 1,200,000 =? 558,600.
2,000 2,000
EXAMI’LE 16
Agganoal starts a business with a capital ofX 675,000 and after 6 months Bhatia becomes
a partner investing ? 975,000. After 4 months more, Sitndaram joins and brings a capital o/? 1,425,000.
What should each receive out of profit of^ 1,537,500 made of the time Aggarwal has been a year in the
business, reckoning that each receiveslO percent interest per annum on his capital prior to the division
of profits ? Find out the total amount transferred to each partner’s capital account if they share the profits
in the ratio of the time they were in this business.

ow
SOLUTION
It is given that Aggarwal remains in the business for 12 months, Bhatia for 6
months and Sundaram for 2 months. It is also given that the partners share profits in the ratio of
the time they were in the business. Therefore,
Profit-sharing ratio = 12:6:2 = 6:3:1

e
re
Since each partner receives 10% interest per annum on his capital. Therefore,
10

rFl
F
Interest to Aggarwal = ? X 675,000 =? 67,500
100

/ 6 10 'i

r
Interest to Bhatia = ? 97,500 x — = f 48,750
ou
X

12 100
fo
ks
and.
f 2 10
Interest to Sundaram = ? 1,425,000 x—x = r 23,750
oo

12 100
Y

Total interest = ^ (67,500 + 48,750 + 23,750) = ? 140,000


B

Balance profit = ^ (1,537,500 -140,000) = ? 1,397,500


re

The profit-sharing ratio is 6 : 3:1 and 6+3 + 1 =10


ou
Y

AggarwaTs share of profit = ? — X 1,397,500 = ? 838,500


ad

10

Bhatia's share of profit = ? — X 1,397,500 = ? 419,250


d

10
in
Re

( 1
Sundaram's share of profit =? —x 1,397,500 = ? 139,750
F

10

Hence, Amount transferred to AggarwaTs account = ? (838,500 + 67,500) t = 906,000


Amount transferred to Bhatia's account = ? (419,250 + 48,750) = ? 468,000
Amount transferred to Sundaram's account = ? (139,750 + 23,750) = ? 163,500
EXAMPLE 17 Three persons X,Y and Z started a part}iership firm. X and Y contributed services
and ? 50,000 and ? 200,000 in cash respectively. Z contributed 500,000 cash. Each partner is to receive
5% interest o)i the money invested. Also, X and Y are paid ? 500 per month, each for first six months as
salary. After the end of the first year, the profit of the firm amounts to ? 222,500. Find the present worth of
each partner, if the profits are distributed on the basis of their capital investments.
SOLUTION We have.
Investment ofX = ?50,000; Investment of Y = ? 100,000 ; Investment of Z =? 500,000.
( 5
Interest on X's investment @ of 5% per annum = ? X 50,000 =? 2,500
100
4.35
PARTNERSHIP

Interest on Y's investment @ of 5% per annum = ^ ^ X 100,000


100
= ? 5,000

Interest on Z's investment @ of 5% per annum = K — X 500,000 = ? 25,000.


100

Total interest paid = ? (2,500 + 5,000 + 25,000) = ? 32,500


Amount paid to X as salary = ? (5,000 x 6) = ^ 30,000
Amount paid to Y as salary = (5,000 x 6) = ? 30,000.
Total amount paid as salary = t 60,000.
Total amount to be deducted from the profits before distribution
= Interest paid + Salary paid = ? (32,500 + 60,000) = ? 92,500.

ow
Total profit = f 222,500.
Net profit to be distributed ^ ? (222,500 - 92,500) = ? 130,000.
Now,
Ratio of the capital investments =50,000:100,000:500,000 -1:2:10

e
re
Sum of the ratios = 1 + 2 + 10 = 13.

X's share of profit = ^ —


rFl
x 130,000 y = ? 10,000

F
13

Y's share of profit = ? — xl30,00ol = ? 20,000

r
ou
13
10 fo
ks
Z's share of profit = ? —
13
x 130,000 = ? 100,000.

Present worth of X's capital = Initial investment + Interest received


oo

+
Salary received + Profit received
Y
B

= ? (50,000 + 2,500 + 30,000 +10,000) = ? 92,500.


Similarly, we have
re

Present worth of Y's capital = ^ (100,000 + 5,000 + 30,000 + 20,000) = tl55,000


ou
Y

and.
ad

Present worth of Z's capital = ? (500,000 + 25,000 +100,000) - ? 625,000.


EXAMPLE 18 P and Q started a business contributing 100,000 and ^ 120,000 respectively. During the
d

first year Q took a ban of ? 50,000 from the total funds of the firm. The partnership deed provided
an
in
Re

interest of 6% on the capital hwestment and allowed to charge an interest of 10% on the loan. Find the
earnings of P and Q if the year’s profit of^ 30,200 is distributed in the ratio of capital investments after
F

paying the interest on the capital.


SOLUTION We have.
P's investment = ? 100,000; Q's investment = ? 120,000.
( 6
Interest paid to P = ? X 100,000 =^ 6,000.
100

Interest paid to Q = ? — X 120,000 = ? 7,200.


uoo

Total interest paid = ? (6,000 + 7,200) = 113,200.


Total profit = ? 30,200
Profit after paying interest = ? (30,200 -13,200) = ^ 17,000.
10
Interest on Q's loan = ^
100
X 50,000 1 = ^5,000.
4.36 APPLIED MATHEMATICS-Xll

Net profit to be distributed = f (17,000 + 5,000) = f 22,000.


Now,

The capital investment ratio = 100,000 : 120,000 = 5:6


Sum of the ratios = 5 + 6=11.

P's share of profit = ? -X 22,000 = no,ooo


11

f 5
Q's share of profit = ? —x 22,000 =?12,000.
Kll

P's total earnings = Interest + Profit = ? (6,000 +10,000) = ? 16,000


Q's total earnings = Interest received + Profit received - Interest paid
= ? (72,000 + 12,000 -5,000) = ? 79,000.

w
- -- EXERCISE 4.3

1. Ramesh and Ravish entered into a partnership investing ? 35,000 and ? 42,000 respectively.

F lo
Ramesh is a working partner and gets ? 200 per month for the same. Find the share of each
in an annual profit of ? 13,400.

ee
2. A is a working partner and B is a sleeping partner in a business. A puts in ? 5,000 and B puts

Fr
in ? 6,000. A receives 12.5% of profits for managing business, and the rest is divided in
proportion to their capitals. What does each get out of a profit of ? 880 ?
for
3. A and 6 enter into a partnership with ? 6,250 and ? 7,500 respectively. After 4 months, A
ur
withdraws ? 1,250 while B puts in ? 2,500 more. If A is working partner getting 10% of the
annual profit for the same, find the share of each in an annual profit of ? 17,500.
s
ook

4. A, B and C entered into a partnership investing ? 12,000, ? 16,000 and ? 20,000 respectively.
Yo

A is working partner gets 10% of the annual profit for the same. After 5 months, B
eB

invested ? 2,000 more while C withdrew ? 2,000 after 8 months of the start of the business.
Find the share of each in annual profit of ? 97,000.
5. A, 6 and C enter into a partnership in a business with capitals of ? 50,000, ? 60,000
r
ou
ad

and 140,000 respectively. A gets 30% of the profit for managing the business and balance is
divided in proportion to their respective capitals. At the end of the year, A gets ? 8,000 more
Y

than what B and C get together. Find the total profit and the share of each.
6. A, B, C enter into a partnership, investing ? 45,000, ? 60,000 and ? 75,000 respectively.
Re
nd

After 4 months of the start of the business, B invests ? 5,000 more. After 8 months of the
Fi

start of business, C withdraws ^ 5,000. A is a working partner and gets 20% of the annual
profit for the same. Find the share of each in the annual profit of ? 136,250.
7. A is a working partner and B, a sleeping parhier in a business. A puts in ? 48,000 and B,
? 80,000. A receives 10% of the profit for managing and the rest being divided in proportion
of their capitals. If A's share of total annual profit is ? 31,500, find the total profit and B's
share out of it.

8. Three partners A, B, C in a business subscribe ? 30,000, ? 40,000 and ? 50,000 respectively. A


receives 15% of the profit as manager and B, 10% as Deputy Manager. The remaining profit
is divided in the ratio of capitals of A, B, C. At the end of the year, A receives ? 54,000 in all.
Find the total profit and the shares of B and C.
9. A, B, and C started a partnership firm and invested ? 100,000, ? 80,000 and ? 120,000
respectively. C took a loan of ? 70,000 and paid 9.5% interest to the firm. The firm earned a
profit of f 135,400 in addition to the interest from loan. Find each partner's total earnings, if
the total profit is distributed in the ratio of the capital investments.
PARTNERSHIP 4.37

10. Bhargava and Bhandari enter into partnership. Bhargava invested ? 5,000,000 and
Bhandari ? 3,000,000. Bhargava is to get 10% and Bhandari 5% of the total profit for a year as
Manager and Accountant respectively and in addition, each is to get interest at the rate of
5% per annum on the capital they put in. The remaining profits are to be divided equally
between them. If at the end of the year, Bhargava gets ? 160,000 more than what Bhandari
gets, how much does each get?
n. Aman, Anish and Arman contributed ? 7,000, ? 8,000 and ? 9,000 respectively in a
partnership business. They receive 5% interest on their investments. The total profit of
? 3600 was distributed to them after deducting the interest. Find the present worth of each
partner's capital, if the profit is distributed in the ratio 3:4:5.

ow
12. Karina and Kanika organised a partnership with an investment of ? 50,000 by Karina
and X 75,000 by Kanika. They agreed to pay 6% interest an each partner's investment, to pay
Karina a salary of X 9,000 per month for six months and to divide the remaining profits or
losses equally. Find each partner's present worth after a profit of X 150,000 to the firm.
Rajendra with a capital of X 1,000,000 in a business admits Vijay into partnership. Vijay

e
13.

Fl
re
brings X 3,000,000 as capital. They agree to charge interest at 5% per annum in the capital.
Vijay gets bonus of 4% on the turnover before profits are divided between them in the

F
proportion 5:3. The turnover during the year amounts to X 6,000,000. If the gross profit is
ur
X 747,000, find what Rajendra and Vijay received?
14.
or
In a firm Mukesh, Bimal and Ahmed contributed X 2,500,000 X 3,500,000 and X 4,000,000
sf
respectively. They agreed to divide the profit in proportion to their capital and to pay
k
X 20,000 and X 30,000 per month for six months as salary to Bimal and Ahmed for their
Yo
oo

services to the firm as manager and general manager respectively. What should each
receive out of net profit of X 2,000,000?
B

15. Radhika and Gauri invested X 8,000 and X 10,000 respectively, in a partnership business.
e

Each partner is paid 5% of the profit as the interest of investments. Find the present worth
ur

of each partner, if the profit of ? 5,000 is distributed in the ratio of their capital investments,
ad

after paying the interest on investment.


Yo

ANSWERS
d
Re

1. ? 7,400, ? 6,000 2. ?460, 420


in

3. ? 7,600, ? 9,900 4. X 31,300, X 30,900, X 34,800


F

5. X 120,000, A's share = X 64,000, B's share = X 33,600, C's share = X 22,400
6. X 54,250, X 38,000, X 44,000 7. X 72,000, X 40,500
8. X 160,000, B X 56,000, C X 50,000
9. A's profit = X 47,350, B's profit ^ X 37,880, C's profit = X 56,820
10. Bhargava : X 880,000, Bhandari: X 720,000
11. Aman: X 7,950, Anish: X 9,200, Arman: X 10,450
12. Karina's worth X 151,250; Kanika's worth X 123,750
13. Rajendra: X 241,875; Vijay: X 505,125
14. Mukesh ? 425,000, Bimal X 715,000, Ahmed X 860,000
15. Radhika: X 10,250, Gauri: X 12,750
4.38 APPLIED MATHEMATICS-XII

4.8 PROFIT SHARING ON THE ADMISSION OF A NEW PARTNER

The existing partners of a firm may decide to admit a new partner when it needs additional
capital or managerial help or both. A new partner can be admitted only with the consent of all
the existing partnership agreement comes to an end and a new agreement comes into effect. The
new partner is required to contribute a proportionate amount of capital depending upon the
share of profit to be given to him. Accordingly, the profit sharing ratio of the existing partners is
adjusted. Besides his capital, the new partner is asked to bring premium for goodwill of the
business. This premium is shared by the old partner in the ratio in which they contribute to the
new partner's share. This ratio is called sacrificing ratio. If the share of profit sacrificed by each
partner is not given, then it is assumed that partners will contribute to the new partner's share of
profit in their old profit-sharing ratio.
ILLUSTRATIVE EXAMPLES

w
EXAMPLE 1
A and B are in partnership sharing profits and losses in the ratio of 5:3. C is admitted as a

F lo
th
n (1 (1 Y^'
partner for - share which he takes — from A and — from B. Find the shares of A, B, C in a
v5 10 10

ee
profit of ? 369600.

Fr
SOLUTION Old profit-sharing ratio of A and B is 5 : 3.
5 3
A's old share of profit = —, B's old share of profit = — for
ur
1 \th ^th \th
1 1
SinceC is admitted as a partner for - share which he takes — from/land — fromB.
s
10 10
ook
Yo

A's new share of profit = - _i_ - B's new share of profit = — _J_^U
3
8 10 ~ 40 ' 8 10 “ 40
eB

21 11 1
Thus, new profit-sharing ratio = — : :- = 21:11:8
40 40 5
r

Sum of the ratios = 21+11 + 8= 40


ad
ou

r2i
A's share out of the profit of ? 369,600 = ? — x 369,600 = ? 194,040.
40
Y

fll
Re

B's share out of the profit of ? 369,600 = ? X 369,600 =? 101,640


nd

40
Fi

( 8
C's share out of the profit of \ 369,600 = ? 40
X 369,600 = ? 73,920.

EXAMPLE 2
A and B are partners sharing profits and losses in the ratio 5 : 3. C brings ? 96,000 as
premium and is admitted as a partner for one-fifth share. Show how this amount will be shared by A and 6,
assummg that the share of C is contributed by A and B in the profit-sharing ratio. Find also the new
profit-sharing ratio.
SOLUTION Since A and B share profits and losses in the ratio 5:3. Therefore, the premium will
be divided in the ratio 5 : 3. Hence,
(5
A's share of premium = ? —x 96,000 = ? 60,000
f 3
B's share of premium =? —x 96,000 =? 36,000.
V 8
1
A's contribution towards the share ofC = — x -
8 5
PARTNERSHIP 4.39

3 1
B's contribution towards the share of C = — X -
8 5
5 5 1 1 3 _3
A's new share of profit = —; 6's new share of profit = —
8 5 2 ^ 8 8^5 10
3 1
Hence, new profit-sharing ratio = — : :-=5:3:2
3 10 5

f I
EXAMPLE 3 A and B are old partners sharing profits in the ratio of 3 : l.C comes in ivith \o— of the
profit. Calculate sacrificing ratio and new ratio.
SOLUTION Since C's share is given without mentioning as to what C acquires from A and 6
separately, hence, it is assumed that C takes it from A and B in their old ratio. Therefore, sacrifice
made by A and B is in the ratio of 3 :1.

w
Sum of the ratios = 3 + 1 ●-4

We have, C's share = —.

F lo
1 1
A's sacrifice = — x — = — , B's sacrifice =-x- =
4 32 8 4 32

ee
3 1
Hence, sacrificing ratio is — : — = 3:1

Fr
^ 32 32
Computation of new ratio: for
3 3 21 7
; Remaining, share of 6 = - - —
ur
Remaining share ^ 32 ^ 4 32 32
21 7 1
s
Hence, new ratio oi A:B:C =■—: :- = 21:7:4
ook

32 32 8
Yo

EXAMPLE 4 A,B and C are partners sharing profits in the ratio 3:2:1. They admit D for — th share. If
eB

C retains his original share, find the sacrificing ratio of A and B. Also, find the new proift-sharing ratio.
ih
r
ad

Let the total share be 1. Since C retains his original share and D is admitted for -
ou

SOLUTION

share. Therefore, it is assumed that D takes it from A and B in their old ratio i.e. 3 : 2.
Y

Now, 3 + 2=5
Re
nd

A's sacrifice =
3 ,1)
-of — = —X —= — ;
3 1 1 .,.
Bs sacrifice = — ot—
f2 .1] 2
=—x — =—
1 1
6 J 5 6 10 Is 6j 5 6 15
Fi

V5

Hence, sacrificing ratio is — : — = 3 : 2.


^ 10 15
The profit-sharing ratio of A, B and C is 3 : 2 :1 and 3 + 2 + 1 =6
3 1 . 21
A's old share = — 6 s old share — = —.
6 2' 6 3
1 4
Thus, A's new-share = 1-J- = A =— ; B's new-share = — -
2 10 10 5 ' 3 15 15
2 4 11
= 12:8:5:5
Hence, new profit-sharing ratio of A, B, C and D is 5 ' ^ ^
EXAMPLE 5
X and Y are partners sharing profits and losses in the ratio of 4 : 3. They admit Z into
partnership. The new profit-sharing ratio of the partners is 5:3: 2. Calculate the sacrificing ratio of X
and Y.
4.40 APPLIED MATHEMATICS-XII

SOLUTION We have,

Old share of X = - ; New share of X = —


7 ' 10

X's sacrifice = -- 1=A


7 10 ~ 70
Old share of Y = ; New share of Y = —
7 ' 10

Y's sacrifice = —
7 10 “ 70
5 9
Hence, sacrificing ratio =—: — =5:9.
^ 70 70

ow
1
EXAMPLE 6 A and B are partners sharmg proifts and losses
in the ratio of 3 :2.C is admittedfor ^ th
share. A and B decide to share equalli/ in future. Find the sacrificing ratio and new-profit sharing ratio.

e
SOLUTION Let the total share be 1. Since C is admitted for — th share. Therefore,

re
4

1 ^3
rFl
F
Remaining
o
share =1 —
4 4

The remaining share is divided equally between A and B. Therefore,

r
1 3
ou
3 1 3 3
A's new share = — x
2 4
, B's new share = — x — = —
2
fo 4 8
ks
3 3 1
Hence, new profit-sharing ratio = —:
8 8
:- = 3:3:2
4
oo

Now,
Y
eB

3 3 9
i4's sacrifice = Old share - New share = =—
5 8 40
2 3 1
r

B’s sacrifice = Old share - New share =


ou

5 40
ad
Y

9 1
Hence, sacrificing ratio of A and 6 is —: — =9:1
40 40
d

Sumant and Giinicharan are partners sharing profits and losses equally. They admit
Re
in

Rakesh as a new partner, the nezv profit-sharing ratio being 6:5:4 between Sumant, Gurucharan and
Rakesh respectively. Rakesh pays 80,000 as premium for goodwill. Hozv zvill it be shared betiueen Sumant
F

and Gurucharan ?

SOLUTION Initially, Sumant and Gurucharan were partners sharing profits and losses equally.
This means that each of Sumant and Gurucharan was entitled
to ^ of the profits.
After the admission of Rakesh as new partner, the profit - sharing ratio has become 6:5:4.
Sum of the ratios = 6 + 5 + 4=15

Therefore, Sumant and Gurucharan are entitled to receive — and — of the profits.
15 15

Hence,
1
— =— and, Gurucharan sacrificed i - —
Sumant sacrificied

of Rakesh.
2 15
■ 10 '-J 2 15 ^ of the profits in favour
PARTNERSHIP 4.41

Now, the premium paid by Rakesh will be divided bewteen Sumant and Gurucharan in the
ratio of the profits which they have sacrificed, i.e. in the ratio —: - = 3:5.
10 6

Sum of the ratios =3 + 5=8.

Sumant's share of premium = ? V —8 x 80,000 J = ? 30,000


(5 )
and. Gurucharan's share of premium = ? - x 80,000 = ? 50,000.

EXAMPLE 8 A mid B started a business with a capital oft 675,000 and t 975,000 respectively. They
agreed to share profit in the ratio of their capital investments. C joins the partnership with the condition
that A, B and C will share profit equally. C pays t 220,000 as premium of goodwill. How will this
premium be shared between A and B.

w
SOLUTION We have,
A's capital = ? 675,000 ; 6's capital = ? 975,000.

F lo
Ratio of capitals = 675,000 ; 975,000 =9:13
Ratio of profits = 9:13 and. Sum of the ratios = 22
Let the total profit be of t x.

ee
9.V 13.V
; 8's share of profit = —

Fr
A's share of profit = 22 22

After the joining of C, we have for


Profit sharing ratio =1:1:1.
ur
C's share of profit = ?
s
ook
Yo

Thus, Share sacrificed by A in favour of C = ? — - '


9 Y X
^5.
^ 22 3) 66
eB

13.V
Share sacrificed by B in favour of C = ?
22 3j 66
our
ad

Now, the premium paid by C will be divided between A and B in the ratio of profits which they
have sacrificed i.e. in the ratio
5x , 17 X
Y

= 5:17
66 ■ 66
Re
nd

f 5
A's share of premium = ? —x
22
220,000 =? 50,000
Fi

17
B's share of premium = t — x 220,000 = ?170,000
V 22 /

EXAMPLES X, y and 2 are partners with capital investment of t 300,000, t 400,000 and
two,000 respectively. X and Y worked as Manager and Deputy Manager and are paid t W,000 and t
7,500 per month as salaries respiectively. After 6 months X took retirement but continued his fund in the
business. After thisY is promoted as Manager and Z is appointed the Deputy Manager, and are paid
salaries. In the year, the company earned a proift of1620,000. Find the total earning of Xfrom this
partnership business.
SOLUTION We have.
Salary paid to X = ^ (10,000 x 6) = ? 60,000
Salary paid to y = ^ (7,500 x 6 + 6 x 10,000) = 1105,000
Salary paid toZ = t (7,500 x 6) = ^ 45,000
Total amount paid in salaries = t (60,000 + 105,000 + 45,000) = t 210,000.
4.42 APPLIED MATHEMATICS-XII

Balance profit to be distributed = ? (610,000 - 210,000) = ? 400,000.


Now,
Ratio of capital investments = 300,000:400,000:100,000 = 3:4:1
Ratio of profits =3:4:1
Sum of the ratios = 3 + 4 + l= 8
f 3
X's share of profit = f — x 400,000 = ? 150,000
8

Hence, Total earnings of X from the business = ? (60,000 +150,000) = ? 210,000.


EXERCISE 4.4

1. X and y are partnership sharing profits and losses in the ratio of 4 : 5. Z is admitted as a
\th
(1 1 riY^'
partner for — share which he takes fromX and — fromY. Calculate new profit -
V 3 V 6

sharing ratio of the partners. Also, find the shares of each partner in a profit of ? 108000.

w
( 3
2. A and B are partners sharing profits in the ratio of 3:2. They admit C for -

F lo
profit which
V 7
\th \lh
(2 [1
he acquires from A and from B. Find out the new profit-sharing ratio.
7 7J

e
Fre
3. Ravi and Ravish were partners sharing profits in the ratio 7 : 3. Shikha was admitted on
' 2 /2 1 / ^
— share in profits which she took - from Ravi and
for from Ravish. Calculate new
7J ^ [7) 17
ratio of partners.
r
4. A and B are partners sharing profits and losses in the proportion of 7:5. They agree to
You
oks

\th

admitC, their manager, into partnership who is to get share in the profits. He acquires
eBo


ej

this share as
f 1
— from A and
1Y^'
from B. Calculate new profit-sharing ratio.
24) V oy
ad
our

( 1
5. A, B and C are partners sharing profits intheratioof 4:3:2. D is admitted for — share in
V 3
Re
dY

future profits. What is the sacrificing ratio ?


1
sth

in the ratio of 3 : 2. C is admitted for ^


Fin

6. A and B are partners sharing profits and losses

share. A and B decide to share equally in future. Find the sacrificing ratio and new profit -
sharing ratio.
7. A and B are partners sharing profits in the ratios 5 : 3. A surrenders - of his share and 6
5

surrenders —3 of his share in favour of C, a new partner. Calculate the new profit sharing
ratio and sacrificing ratio.
8. P and Q started a partnership business in which they agreed to share the profit in the ratio
3 : 4. A new partner R is admitted in the business who brought ? 42,000 as goodwill
premium to be distributed among P and Q. They finally agreed to share the profit in the
ratio 15 : 20 : 7. Find the sacrificing ratio of P and Q and the amount of premium each will
get.
4.43
PARTNERSHIP

9. X and Y are partners sharing profits and losses in the ratio 2:3 respectively. They admit Z as
a new partner, the new profit sharing ratio being 1:2:2 between A, B and C respectively. Z
pays ? 12,000 as premium for goodwill. Find the amount of premium shared by X and Y
individually.
10. A and B are partners sharing profits and losses in the ratio 5:3. C pays ^ 96,000 as premium
for the good will of the business and admitted for the fifth share. Show how this amount
will be shared by A and B, assuming that the share of C is contributed by A and B in their
profit sharing ratio. Find also the new profit sharing ratio.
11. A and B are partners sharing profits in the ratio 4\3.A and B surrender — of their share in
favour of C, a new partner. Calculate the new profit sharing ratio and sacrificing ratio,
12. A and B are partners sharing profits and losses in the ratio 1 : 2. They admit C as a new

w
partner, the new profit sharing ratio being 1:2:2 between A, B and C respectively. C pays
? 10,000 as premium for goodwill. How will it be shared between A and B.

F lo
13. A and B are partners sharing profits and losses equally They admit C as new partner who
pays premium for the goodwill, the profit sharing ratio being 5:4:3 between A, B and C

ee
respectively. Find out the amount of premium being brought in by C if B gets ? 20,000 more

Fr
than A. How will it be shared between A and B?
ANSWERS

1. 20 :25 : 9 ; ? 40,000, ^ 50,000, ? 18,000 2. 11:9:15


for
ur
3. 29 :11 : 30 4. 13:7:4 5. 4 : 3 : 2

6. Sacrificing ratio = 9:1, Profit-sharing ratio = 3:3:2.


s
ook
Yo

7. New ratio 2:1:1, Sacrificing ratio 1 :1


8. Sacrificing ratio 3 : 4; P's share = ? 18,000, Q's share = ? 24,000
eB

9. X's share = ? 6,000, Y's share = t 6,000


10. A's share = ? 60,000, B's share = ? 36,000
r

New profit sharing ratio =5:3:2.


ou
ad

11. Now profit sharing ratio =12:9:7, Sacrificing ratio = 4:3


12. A's share = ? 3,333.33, B's share = ^ 6,666.67
Y

13. A's share = ? 20,000, B's share = ? 40,000


Re
nd

4.9 PROFIT SHARING ON THE RETIREMENT OF AN EXISTING PARTNER


Fi

In a partnership firm a partner may withdraw from the firm either with the consent of all other
partners or by giving them a notice of his intentions to withdraw. Such a situation is generally
called retirement of a partner. Retirement of a partner results in reconstitution of the firm. The
retired partner can withdraw his capital invested and the profit earned on it.
At the time of retirement of a partner, the continuing partners acquire the share of outgoing
partner in their old profit sharing ratio. This results in the increase in the profit share of
continuing partners. The ratio of increase in the profit share of continuing partners is called the
gaining ratio. It is called gaining ratio because the continuing partners stand to gain by
acquiring the share of the retiring partner.
REMARK In case of retirement of a partner the profit sharing ratio is adjusted in such a way that the
continuing partners acquire the outgoing partner's share in their old profit sharing ratio. Thus, the new
profit sharing ratio of continuing partners remains the same as the ratio prior to the retirement of a
partner.
4.44 APPLIED MATHEMATICS-XII

ILLUSTRATIVE EXAMPLES

EXAMPLE 1 P,Q and R were partners sharing profit in the ratio 5:3 :2. R retired from the firm and P
and Q decided to share profits in the ratio 31 :19. Calculate the gaining ratio.
SOLUTION
Recall that the ratio of increase in the profit share of continuing partners on the
retirement of a partner is called the gaining ratio.
We have.

Old share of P = — , Old share of Q = —


10 "0
1
31 19
New share of P = —, New share of Q = —.
50 50
31 5 6
Increase in share of Q = 19 __3

ow
Increase in share of P = = —;
50 10 50 50 10 " 50
Hence, Gaining ratio = ~: — = 6 : 4 = 3 :2
50 50

1 1 7

e
EXAMPLE 2 X, y and Z are partners sharing profits and losses in the ratio —, - and Z retires

Fl
re
and X and Y decide to share the profits and losses equally. Calculate the gaining ratio.

F
SOLUTION We have.
ur
1

or
Old share of X = — Old share of X =
3 ' 5 f
1 1
New share of X = —, New share of Y =
ks
2 ' 2
Yo
1 1
1 ^_3
oo

Increase in the share of X = - -; Increase in the share of Y = -


2 3 6' 2 5 "lO
B

1 3
Hence, Gaining ratio = — : — -5:9
6 10
re

EXAMPLE 3 A, B andC are partners in a business and sharing the profits and losses in the ratio 2:3:4.
u

C took retirement from the partnership. Find the profit gaining ratio of A and B. Also, find the difference in
ad
Yo

the income from the profit of 45,000 before and after the retirement ofC.
SOLUTION We have,
d

Profit sharing ratio of A, B and C = 2 : 3 : 4


Re
in

Sum of the ratios = 2+ 3 + 4 = 9


F

Shareof/4=—, Share of B Share of C= —


9 ' 9 ' 9

On the retirement of C, his share is shared by A and B in the ratio 2 : 3.


2 4 (2 4'
Profit share gain by/I = —of — = — x —
5 9 l5 9j
f 3 A\ -X A A
Profit share gain by B = —of— = —x —= —
1,5 9j 5 9 15
8 4
Thus, profit gaining ratio = —; -— = 2:3.
^ * 45 45
We have. Total profit = ? 4,500.
(2 ^
Profit share of A before the retirement of C = ? -X 45,000 = ? 10,000
^9
PARTNERSHIP 4.45

Profit share of A after the retirement of C = ? - X 45,000 = ?18,000


V5

Hence, Difference in the income of A = ? (18,000 -10,000) = ? 8,000.


EXAMPLU4 A, Band Care partners and agreed to share the profit in the ratio 2: 3:4.At the end offour
months Btook retirement from the partnership. That time the profit of the firm was 11,200. B took out his
share of profit but continued to keep his capital in the firm till the final account is settled. He decided to
charge interest on his capital at the rate of 6% per annum for the period it is in use. Find the capital
invested by B if to total earnings at the end of the fiscal year is ? 850.
SOLUTION Let B's capital = .r. Then,
/ ^ 8
Interest on B's capital for 8 months = ? —
100
X

12
X .Y
25

We have,
Profit sharing ratio = 2:3:4
Sum of the ratios = 2+ 3 + 4 = 9

w
Total profit = ?1,200

Thus,
B's share ofprofit = ?^|x 1200 = ? 400.
Total earning of B = ? - + 400
(25

But, the total earning of B is given to be ? 850.


F lo for F
ree
—+ 400=850 =>—= 450=> .y=11,250
25 25

Hence, B's capital in the business was ? 11,250.


Your
ks
eBoo

EXERCISE 4.5

1. A, Band C were partners sharing profits in the ratio4:3:2. B retires from the firm and A and
C decide to share profits in the ratio 3 : 2. Calculate the gaining ratio.
ad
our

11 3
2. X, y and Z are partners sharing profits and losses in the ratio —, - and —. Z retires and X
and y decide to share the profits and losses equally. Calculate the gaining ratio.
Re

Ill
Y

3. Radha, Shyama and Sheela share profits and losses in ratio -:


2
—:
3 6
- respectively. Shyama
Find

4
retires and surrendes — of her share in favour of Radha and remaining in favour of Sheela.

Calculate the new profit sharing ratio and gaining ratio.


ANSWERS

1. Gaining ratio 7 : 8 2. Gaining ratio 0 :1. 3. 35:19; 4: 5.

HINTS TO SELECTED PROBLEMS

4 1 4,1
3. Share gain by Radha = — of —
^ ^ l9 3 9 3 27

(5 1^5 1-
Share gain by Sheela = g ^ = —9 x 3 ” 27
4.46 APPLIED MATHEMATICS-XIl

1 4 '^=5 1 5 19
New share of Radha = — + — = — , New share of Sheela = — + — = —
2 27 54 6 27 54
35 19
New profit sharing ratio = —: — = 35:19
^ ^ 54 54
4 5
Gaining ratio = — : — = 4:5
27 27

MULTIPLE CHOICE QUESTIONS (MCQs)


Each of the following questions has four choices (a), (b), (c) and (d), out of which only one is correct. Mark
the correct choice in each questioji.
1. A,B and C invested ? 45,000, ? 70,000 and ? 90,000 respectively to start a business. At the
end of 2 years, they earned a profit of ? 164,000. B's share in the profit is

w
(a) ? 36,000 (b) ? 56,000 (c) ? 64,000 (d) ? 72,000
2. A and B started a business investing X 85,000 and ? 15,000 respectively. In what ratio the

Flo
profit earned after 2 years be divided betu'een A and B respectively?
(a) 3:4 (b) 15:23 (c) 17:3 (d) 3:5

e
re
th
1
3. In a partnership, A invests — of the capital for —6 of the time, B invests of —3 of the capital for

rF
~ of the time and C invests the rest of the capital for the whole time. Out of a profit of
ur
? 4,600, B's share is
fo
(a) ? 650 (b) ?800 (c) ? 960 (d) ? 1,000
ks
Yo

4. A and B enter into a partnership with ? 50,000 and ? 60,000 respectively. C joins them after
oo

a: months, contributing ? 70,000 and B leaves .v months before the end of the year. If they
B

share the profit in the ratio 20 :18 : 21, then .r =


re

(a) 3 (b) 6 (c) 8 (d) 9


5. A, B and C are three partners. They altogether invested ? 14,000 in business. At the end of
u
ad

the year, A got ^ 33750, B ? 1,125 C ? 63750 as profit. The difference between the
Yo

investments of B and A was

(a) ? 2,200 (b) ? 3,200 (c) ? 4,200 (d) ? 5,250


nd
Re

6. A, B, C subscribe ? 50,000 for a business. A subscribes ? 4,000 more than 6 and ? 5,000 more
than C. Out of a total profit of ? 35,000, A receives
Fi

(a) ? 8,400 (b) ? 11,900 (c) ? 13,600 (d) ? 14,700


1 1 1
7. A, B and C are partners in a business. Their shares are in the proportion of —: :-. A
3 4 5

withdraws half of his capital after 15 months and after another 15 months, a profit of
? 4,340 is divided. The share of C is
(a) ?1,240 (b) ? 1,245 (c) ? 1,360 (d) ? 1,550
8. Anu is a working partner and Bimla is a sleeping partner in a business. Anu puts ? 5,000
and Bimla puts ^ 6,000. Anu receives 12.5% of the profit for managing the business and the
rest is divided in proportion to their capitals. What does each get out of a profit of ? 880?
(a) ? 400, f 480 (b) ^450, ? 430 (c) ? 460, ? 420 (d) ? 470, ? 410
1 2
9. X and Y are partners in a business. X — of the capital for 9 months and Y received - of the
3 5

profits. For how long was Vs money used in the business?


4.47
PARTNERSHIP

(a) 2 months (b) 3 months (c) 4 months (d) 5 months


10. The ratio of investments of two partners A and B is 11:12 and the ratio of their profits is 2:3.
If A invested the money for 8 months, then for how much time B invested his money?
(a) 11 months (b) 10 months (c) 9 months (d) 5 months
11. A, B and C enter into a partnership. Their capital contribution is in the ratio 21 :18:14. At
the end of the business term they share profits in the ratio 15 : 8 : 9. The ratio of time for
which A, B and C invest their capitals, is
(a) 81 : 56 : 90 (b) 90:56:81 (c) 56 : 90 : 81 (d) 81 : 90 : 56
ANSWERS

1. (b) 2. (c) 3. (b) 4. (a) 5. (d) 6. (d) 7. (a) 8. (c)


9. (b) 10. (a) 11. (b)

w
F lo
ee
Fr
for
ur
s
ook
Yo
eB
our
ad dY
Re
Fin
CHAPTER 5
RACES AND GAMES

5.1 INTRODUCTION

Intutively, a race is a competition beUveen two or more contestants trying to cover a specified
distance in the least amount of time. The concept of race involves physical quantities namely,
distance, time and speed about which we have learnt in earlier classes. Therefore, the present

w
chapter on races and games is an extension of the chapter on Time and Distance studied in class
XL All the formulae learnt in the afore mentioned chapter will be applicable to the problems in

F lo
this chapter also. Following are some important terms frequently used in problems on races and
games.

e
Fre
5.2 SOME USEFUL TERMS AND STATEMENTS

RACE A race is a competition in which the cojiipetitors have to cover a speciifed distance in the least
for
time.

A race may involve running, riding, sailing, rowing, driving etc.


r
RACE COURSE The ground or path on zuhich race takes place is called a race course.
You
oks

STARTING POl'--^ ^ The point from zohere a race starts is known as the starting point.
eBo

WINNING pfj:n The point set to hound a race is called the winning point. Winning point is also called
goal.
DEAD HEA I If all the competitors participating in a race reach the finishing point exactly at the same
our
ad

time, then the race is said to be a dead heat race.


Let A and B be two contestants in a race. Then, The following statements are commonly used.
A gives B a start of x meters: This statement means that before the start of race, A is at the starting
dY
Re

point and B is ahead of A by '.x' meters.


Fin

In a race of 100 meters when we say "A gives B a start of 15 meters". It means that A will have to
cover 100 meters while B will have to cover (100 -15) = 85 meters.
A beats Bbyx meters: This statement means that in the same time, zuhile A reached the ivinning point,
whereas, B is behind A by 'x' meters.
In a race of 100 meters the statements "A beats B by 10 meters" means that at the moment of
finish of the race A has covered 100 meters while B has covered only (100 -10) = 90 meters.
A can give B a start oft minutes: This statement means that A will start ‘f minutes after B starts from
the starting point and both A and B reach the finishing point at the same time.
A gives B X meters and t minutes: This meazzs that A and B start from the starting point at the same
instant, but zohile A reaches the finishing point, B is behind by 'x' meters and B takes t minutes more
compared to A to complete the race. So, B covers x zneters in extra time oft minutes.
Thus, if A gives B x meters and t minutes, then B covers x meters in extra time of t minutes. So,
X
speed of B is —t m/sec.
5.2 APPLIED MATHEMATICS-XII

ILLUSTRATIVE EXAMPLES

EXAMPLE 1 A can run 22.5 m while B runs 25 m. By luliat distance B beat A in a kilometre race?
SOLUTION B beats A by (25 - 22.5) m = 2.5 m in running 25 m.
Thus, in running 25 m, B beats A by 2.5 m.
2.5
In running 1 km i.e. ] 000 m, B beats A by x 'JOOO m=100m
25

ALITER We have.
Distance run by B _ 25 25 _ 50 _ 50x 20 _ 1000
Distance rtm by/4 22.5 45/2 45 45x20 900
Thus, when B runs 1000 m, A runs 900 m. Hence, B beats A by (1000 - 900) m = 100 m.

w
EXAMPLE 2 In a 50 m race, a can give a start of 5 m to B and a start of 14 m to C. In the same race how
much start can B give to C?
SOLUTION In the same time, A covers 50 m, B covers (50 - 5) m = 45 m and C covers

Flo
(50 - 14) m = 36 m.

e
Thus, when B runs 45 m, C runs 36 m.

re
36 4
When 6 runs 1 m, C runs — m = — m

F
45 5
4
When B runs 50 m, C runs — x 50 m= 40 m.
ur
5

Hence, B gives a start of (50 - 40) m = 10 m to C.


f or
EXAMPLE 3 In a kilometer race, A beats B by 100 m and B beats C by 150 m. In the same race, by hoiv
ks
many meters docs A beat C?
Yo
oo

SOLUTION In a kilometer race, A beats B by 100 m means: A runs 1000 m in the same time when
B runs 900 m. Therefore,
B

Speed of A : Speed of 6 = 1000 : 900


re

Similarly,
Speed of B : Speed of C = 1000 : 850 = 100 : 85 = 900 : 765
u

...(h)
ad
Yo

From (ii), we obtain


Speed of 6 : Speed of C = 900 : 765 ...(hi)
From (i) and (iii) we obtain
d
Re
in

Speed of A : Speed of B : Speed of C = 1000 : 900 : 765


=> Speed of A : Speed of C = 1000 : 765
F

Hence, in a kilometer race, A beats C by (1000 - 765) m = 225 m


EXAMPLE 4
In a 10 km race. A, Band C, each running a uniform speed get the gold, silver and bronze
medals respectively. If A beats Bbyl km and B beats Cbyl km, then by how much meters does A beat C?
SOLUTION In a 10 km race, A beats B by 1 km, means when A rims 10 km in the same time B
runs 9 km.

Speed of A : speed of 6 = 10 : 9 ...(i)


Similarly,
Speed of B ; Speed of C = 10 : 9 -(ii)
From (i) and (ii), we obtain
Speed of A : Speed of B = 100 : 90 [●●● 10 : 9 = 100 : 90]
Speed of B : Speed of C = 90 : 81 [v 10:9 = 9x10:9x9 = 90:81]
Speed of A : Speed of B : Speed of C = 100 : 90 : 81
=> Speed of A : Speed of C = 100 : 81.
5.3
RACES AND GAMES

Thus, in the same time when A runs 100 meters, C runs 81 meters.
In the same time when A runs 100 x 100 meters, C runs 100 x 81 meters
or. In the same time when A runs 10 km, C runs 8.1 km.
Hence, A beats C by (10 - 8.1) km = 1.9 km = 1900 meters.
EXAMPLE 5 hi a 800 m race, A heats B bi/ 74 m and in a 600 ni race, B beats C by 50 m. By hozv many
meters will A beat C in a race of 500 m?
SOLUTION In a 800 m race, A beats B by 74 m, means when A runs 800 m in the same time
B runs (800 - 74) m = 726 m
Speed of A : Speed of B = 800 : 726
In a 600 m race, B beats C by 50 m, means when B runs 600 m in the same time C runs
(600 - 50) m = 550 m.
550
While B runs 726 m, C runs X 726 m = 665.5 m
600

w
Thus, in a 800 m race, A beats C by ( 800 - 665.5) m = 134.5 m
134.5

F lo
Hence, in a 500 m race, A beats C by X 500 m = 84.06 m.
800

EXAMPLE 6 In a kilometer race, A beats B by 30 seconds and B beats C by 15 seconds. If A beats C by

ee
180 m. Find the time taken by A to run 1 kilometer.

Fr
SOLUTION Suppose A takes t seconds to run 1 kilometer race. Then, B takes (/ + 30) seconds and
C takes (f + 30 + 15) seconds = {t + 45) seconds. for
We find A beats C by (30 +15) seconds = 45 seconds and it is given that A beats C by 180 meters.
ur
C runs 180 meters in 45 seconds.
45
s
C runs 1000 meters in X1000 seconds = 250 seconds.
ook

180
Yo

f + 45 = 250 => f = 250-45 = 205.


eB

Hence, A takes 205 seconds to run 1 kilometer.


EXAMPLE? A runs 4 times as fast as B. If A gives B a start of 60 meters, hoiv far must the goal on the
course be so that A and B reach it at the same time?
r

race
ad
ou

SOLUTION Let the goal on the race course be x meters away from the starting point. It is given
that A gives B a start of 60 meters. This means that in the same time if A runs x meters, B runs
Y

(x-60) meters.
Time taken by A in covering .v meters = Time taken by B to cover (.r - 60) meters
Re
nd

X X - 60
.y=4x-240=> x = 80
Fi

4 1

Hence, the goal is 80 meters away.


AI.ITER A runs 4 times as fast as B. Therefore, ratio of speeds of A and B is4 :1. This means that
in a race of 4 m, A gains (4 -1) m = 3 m over B.
( 4
60 meters will be gained by A in a race of —3 x 60 m = 80 m.
Hence, the goal is 80 m away.
EXAMPLES In a mile race, Akshaycan be given a start of 128 m by Bharav. If Bharav can give Chinmay
a start of 4 m in a 100 m dash, then who out ofAkshay and Chinmay will win a race of one and a halfmiles,
and what will be the final lead given by the winner to the loser? One mile is 1600 m.
SOLUTION In a race of 1600 m, Bharav gives a start of 128 m to Akshay means that in the same
time while Bharave covers 1600 m, Akshay covers (1600 -128) m = 1472 m.
Speed of Bharav : Speed of Akshay = 1600 :1472 = 25 : 23
5.4
APPLIED MATHEMATICS-XII

In a 100 m dash Bharav give Chinmay a start of 4 m means that in the same time while Bharav
covers 100 m, Chinmay covers (100 - 4) m = 96 m.

Speed of Bharav : Speed of Chinmay = 100 :96 = 25 :24


Thus, we have.
Speed of Akshay : Speed of Bharav = 23 : 25
and. Speed of Bharav : Speed of Chinmay = 25 : 24
Speed of Akshay : Speed of Chinmay = 23 : 24.
1 n
1 - miles = — xl600 meters = 2400 meters

ow
2 2 )
Now,
Speed of Akshay : Speed of Chinmay = 23 : 24 = 2300 : 2400.
Thus in the same time while Akshay covers 2300 meters, Chinmay covers 2400 meters.

e
Hence, Chinmay wins the race by (2400 - 2300) meters = 100 meters.

re
EXAMPLE 9 In a kilometer race, A beats B b\j 40 meters or by 5 seconds. Find the time taken by A to
complete the race.

Flr
F
SOLUTION In a kilometer i.e. 1000 m race, A beats B by 40 meters means that A runs 1000 m,
B runs (1000 - 40) m = 960 m.
ou
A beats B by 40 meters or by 5 seconds. This means that B runs 40 meters in 5 seconds.

sr
i.e.
Time taken by B to cover 40 metres = 5 seconds

fo
f 5
Time taken by B to cover 960 metres = — x 960 = 120 seconds = 2 minutes.

Hence, time taken by A to complete the race is 2 minutes.


{40 k
oo
EXAMPLE 10 In a kilometre race, if A gives B, a start of 40 metres, then B wins by 19 seconds but if A
Y

gives B, a start of 30 seconds then B wins by 40 metres. Find the time taken by each to run a kilometre.
reB

SOLUTION Let the time taken by A and B to run 1 km be x and y seconds respectively.
If A gives B, a start of 40 metres, it means that in the same time A runs 1000 metres while B runs
uY

(1000 - 40) m = 960 m.


Now,
ad
do

B runs 1000 m in y seconds


i/ 24
B runs 960 m in X 960 seconds = y seconds
in

1000 25
24
Re

Thus, A and 6 reach the winning point in x seconds and — y seconds respectively.
F

25
But, B wins by 19 seconds.
24
y-x=19
25

25x-24y =-475
If A gives B, a start of 30 seconds, than A starts 30 seconds after B. Therefore, A runs for (y - 30)
seconds.
1000
In (y - 30) seconds, A runs (y - 30) meters.
X

It is given that B wins by 40 metres.


1000
1000- (y-30)=40
X

1000
X
(y-30) =960
RACES AND GAMES 5.5

25
(j/-30)=24
X

=> 24.v-25y=-750 ...(ii)


Adding (i) and (ii), we obtain
49.Y-49y=-1225 =>x-y=-25 ...(iii)
Subtracting (ii) from (i), we obtain
j: + y = 275 ...(iv)
Solving (iii) and (iv), we obtain x = 125 and y = 150
Hence, A and B take 125 seconds and 150 seconds respectively to run 1 kilometer.

5.3 GAMES

A game of 100 means that a person among the contestants who scores 100 points first is the
winner.

w
In a game of 100 points, if A scores 100 points, while B scores 80 points, then we say that A can
give B 20 points.

F lo
ILLUSTRATIVE EXAMPLES

a game of 100 points, A can give B 20 points and C 28 points. How many points can B

ee
give C?

Fr
SOLUTION If a game of 100 points, A can give B 20 points means that when A scores 100 points,
for
B scores (100 - 20) = 80 points.
Similarly,
ur
A can give C 28 points, means when A scores 100 points, C scores (100 - 28) = 72 points.
s

Thus, When B scores 80 points, C scores 72 points


ook
Yo

72
Wlaen B scores 1 point, C scores points
eB

80

^72
=> When B scores 100 points, C scores 100 points = 90 points.
80
our
ad

Hence, B can give C 10 points.


I At a game of billiards, A can give B 15 points in 60 and A can give C 20 points in 60. How
many points can B give C in a game of 90?
Y
Re

SOLUTION "A can give 15 points in 60" means that in a 60 points game A scores 60 points while
nd

B scores 45 points. Similarly, "A can give C 20 points in 60" means A scores 60 points while C
Fi

scores 40 points. Thus, when B scores 45 points C scores 40 points.


MO A
When B scores 90 points, C scores —x90 =80 points.
45

Hence, B can give C 10 points in a game of 90.

1. In a kilometer race. A, B and C are three participants. A can give B a start of 50 meters and C
a start of 69 meters. In the same race, how much start can B give to C?
A can run a kilometer in 4 minutes 50 seconds and B in 5 minutes. How many meters start
can A give 6 in a kilometer race, so that the race may end in a dead heat.
In a 100 meter race, A runs with 6 km/hr. If a gives B a start of 4 meters and still beats him by
12 seconds. What is the speed of B?
If A can run 1 kilometer in 190 seconds and B can run the same distance in 200 seconds, then
by what distance A can beat B?
5.6 APPLIED MATHEMATICS-XII

5.
A and B run 1 kilometer and A wins B by 1 minute. A and C also run 1 kilometer and A wins
by 375 meters. B and C run 1 kilometer and B wins by 30 seconds. Find the time taken by
each to run one kilometer distance.
6. In a 100 m race, A runs at 1.66 m/sec. If A gives B a start of 4 m and still beats him by
12 seconds, what is B's speed?
7.
In a race of 200 m, A can beat B by 31 m and C by 18 m. In a race of 350 m by how much
distance will C beat B?

8. In a racing over a distance 'd' at a uniform speed, A can beat B by 20 meters, 6 can beat C by
10 meters and A can beat C by 28 meters. Find distance d.
5
9. A runs
— times as fast as B. If A gives B a start of 80 m, how far must the winning post from

the starting point be so that A and 6 reach at the same time?


10. In a game of billiards, A can give 6 20 points in 60 and A can give C 30 points in 60. How

w
many points can 6 give C in a game of 100?

F lo
ANSiVBRS

1. 20 meters 2 — meters 3. 4.8 km/hr 4. 50 meters


3

e
5. 150 seconds, 210 seconds, 240 seconds 6. 1.33 m/sec 7. 25 m

Fre
8. 100 meters 9. 200 meters 10. 25 points for
H/NTS TO SELECTED PROBLEMS

2. A runs 1000 m in 290 seconds and B in 300 seconds. For the race to end in a dead heat, A and
r
B must reach the goal at the same time i.e. in 290 seconds. So, A must give 10 seconds start to
You
oks

1000 100
B. In 10 seconds, B runs xlO meters
eBo

300 3

60x60
3. Time taken by A to cover 100 m = X100 seconds = 60 seconds
6000
ad
our

B covers (100 - 4) = 96 m in (60 + 12) = 72 seconds


96 4 , 4
(4 60x60^
Hence, B's speed = — m/sec = — X km/hr
[72 3 l3 1000
Re
dY

1000
4. A beats B by 10 seconds i.e. X10 meters
200
Fin

5. It is given that A beats B by 60 seconds and B beats C by 30 seconds. Therefore, A beats C by


90 seconds. But, it is given that A beats C by 375 meters.
C covers 375 meters in 90 seconds.
90
C covers 1 km in X1000 seconds = 240 seconds.
375

8. A can beat B by 20 meters => When A runs d meters, 6 runs (d - 20) meters
6 can beat C by 10 meters => When B runs d meters, C runs {d -10) meters
A can beat C by 28 meters ^ When A runs d meters, C runs {d - 28) meters
Now,
When 6 runs d meters, C runs (d - 10) meters
(d-10
When B runs 1 meter, C runs meters
d J
RACES AND GAMES 5.7

f{d-10)
When B runs (d - 20) meters, C runs ^x(ii-20) meters
d

=> When A runs d meters, C runs -—^ X (rf - 20) meters


{ d J
d-10^
{d-2Q)=d-28 => d^-30d + 200 = d^-28d^d=100
d )

MUL TIPLE CHOICE QUESTIONS (MCQs)


Each offollowing questions has four choices (a), (b), (c) and (d) out of which o»/y one is correct. Mark the
correct choice.

1. In a race of 400 meters, A can give B a start of 20 meters and C a start of 39 meters. How

w
much start can B give to C in the same race?
(a) 20 meters (b) 15 meters (c) 18 meters (d) 25 meters

Flo
In a 2 kilometer race, A can give B a start of 200 meters and C a start 560 meters. In the same
race, how much start can B give to C?

e
(a) 500 meters (b) 350 meters (c) 300 meters (d) 400 meters

re
3. In a 100 m race, A can give B a start of 10 m and C a start of 28 m. How much start can B give

F
to C in the same race?
(a) 18 m (b) 20 m (c) 27 m (d) 9 m
ur
r
4.
fo
In a 500 m race, the ratio of speeds of two contestants A and B is 3 : 4. If gets a start of
140 m, then he wins by
ks
(a) 60 m (b) 40 m (c) 20 m (d) 10 m
Yo
oo

5. If in a 100 m race, A beats B by 10 m and C by 13 m, then in a 180 m race, B will beat C by


(a) 5.4 m (b) 4.5 m (c) 5 m (d) 6 m
eB

6. In a 100 m race A and B are two participants. If A runs at 5 kilometer per hour and A gives B
a start of 8 m and still beats him by 8 seconds, then the speed of B is
ur

(a) 5.15 km/hr (b) 4.14 km/hr (c) 4.25 km/hr (d) 4.4 km/hr
ad
Yo

7. If in a 600 m race, A can beat B by 50 m and in a 500 m race, B can beat C by 60 m. Tlren, in a
400 m race, A will beat C by
1
(a) 70 m (b) 77 m (c) 77 - m (d) 77-m
d
Re

3 2
in

8. In a kilometer race, A beats B by 50 meters or 10 seconds. What time does A take to complete
F

the race?
(a) 200 seconds (b) 190 seconds (c) 210 seconds (d) 150 seconds
9. A runs a 1000 m race in 4.5 minutes while B runs the same race in 5 minutes. How many
meters start can A give to 6 in a 1000 m race?
(a) 150 m (b) 125 m (c) 130 m (d) 100 m
in. In a 1000 m race, A beats B by 100 m and in a 800 m race B beats C by 100 m. By how many
meters will A beat C in a 600 m race?
(a) 57.5 m (b) 127.5 m (c) 150.7 m (d) 98.6 m

11. A is 2— times as fast as B. If A gives B a start of 80 meters, how long should the race course be
3
so that both of them reach at the same time?
(a) 170 meters (b) 140 meters (c) 160 meters (d) 150 meters
5.8 APPLIED MATHEMATICS-XII

12.
In a 1000 m race, A can beat B by 100 m. In a race of 400 m, B can beat C by 40 m. By how
many meters will A beat C in a race of 500 m?
(a) 85 m (b) 95 m (c) 105 m (d) 115 m
13.
In a 500 m race, the ratio of the speeds of two contestants A and B is 3 :4. If /I has a start of
140 m, then A wins B by
(a) 60 m (b) 40 m (c) 20 m (d) 10 m
14.
In a 400 m race, A gives B a start of 5 seconds and beats him by 15 m. In another race of
400 m, A beats B by 7^ seconds. Their respective speeds are
(a) 6 m/sec, 7 m/sec (b) 5 m/sec, 7 m/sec
(c) 8 m/sec, 7 m/sec (d) 9 m/sec, 7m/sec
15.
In a race of 200 m, B can give a start of 10 m to A and C can give a start of 20 m to B. The start
that C can give to 7l in the same race is

w
(a) 27 m (b) 29 m (c) 30 m (d) 25 m
16.
In a game of 100 points, A can give B 10 points and C18 points. Then, B can give C

17.
(a) 35:12 (b) 45:41

F lo
(c) 55 :25 (d) 35:41
In a game, A can give B 25 points, A can give C 40 points and B can give C 20 points. How

ee
many points make the game?

Fr
(a) 120 (b) 100 (c) 150 (d) 80
for ANSWERS

1- (a) 2. (d) 3. (b) 4. (c) 5. (d) 6. (b) 7. (c) 8. (b)


ur
9. (d) 10. (b) 11. (b) 12. (b) 13. (c) 14, (c) 15. (b) 16. (b)
s

17. (b)
ook
Yo
eB
our
ad dY
Re
Fin
CHAPTER 6
NUMERICAL INEQUALITIES
AND LINEAR INEQUATIONS

6.1 NUMERICAL INEQUALITIES

Let a,b are real numbers. Then, a>b,a>b,a<b anda<b are numerical inequalities. In this
chapter, we will learn about some properties of numerical inequalities.
6.1.1 ELEMENTARY PROPERTIES OF INEQUALITIES

w
PROPERTY-1 If a, b, c are real numbers such that a>b and b >c, then a>c.
More generally, ifa^, aj, ^3,...
then a-i > a„.
PROPERTY-2 If a, b are real numbers, then

Also,
a>b => a + c>b + c for all c sR
a>b => a-c >b-c forallceR.
F lo
a^ are real numbers such thata-^ ><72 >03 > ^4

for F
ree
1 >fl,,,

PROPERTY-3 If a, b are tzvo real numbers, then


a >b ac> be and - > -for all c> 0, c e K
Your

c c
oks

Also, ac <bc and - <-for all c <0, c ^ R.


eBo

c c

PROPERTY-4 //<7>l?>0,f/7tf« 7.
a b
ad
our

PROPERTY-5 Ifa^ >b^ , 02 >1’2 '^3 >V then


<?! + <?2 <73 + .... 4- 0)1 > bj -f-172 4- b^ 4-... 4" bji
Re

PROPERTY-6 //(7i, «2 ' b2 ,b„ are positive real numbers such that
dY

<7j>bj, <?2>b2,...., a„>b)), then a-iaja^ .... <^>bi 1)2 ^3... b„


Fin

PROPERTY-7 Ifa, b are positive real numbers such that a <band ifn is any positive rational number, then
- » - n l/)j <b l/»
(i) 77" <b (ii) a >b (hi) a

PROPERTY-8 If 0 < (7 < 1 and n is any positive rational number, then


(i) 0 <fl” <1 (ii) a~" >1

PROPERTY-9 Ifb > 1 and n is any positive rational number, then


- n
(i) b" >1 (ii) 0<b <1

PROPERTY-10 If0<a< \ and m, n are positive rational numbers, then


(i) m >n => tt”' < a' (ii) ?;i < 71 ^ fl"' > (t”
PROPERTY-11 If a >1 and rn, n are positive rational numbers, then
6.2 APPLIED MATHEMATICS-XII

m n
(i) m>n => a >a (ii) m <n => < a”
PROPERTY-12 (i) lfa>\andx>y>0,then log^-Y > log^, i/.
(ii) IfO<n<\ and x>y>0, then log^ x < log^ y.
PROPERTY-13 For real number a, we have -|fl|<i7<jrt|

PROPER^Y■^4 Ifr>Oandnisanyrealnumber, then |Y-fl|<r <=> a -r <x <a + r


PROPERTY-15 If a and b are positive real numbers, then
a + c a + c
a <b => >— foralloO and, a>b ^ - for all c>0

ow
<
b + c b + c b ^

PROPERTY-16 (i) If a is a positive real number, then a + ->2.


a

1
(ii) If a is a negative real number, then a + - <-2.

e
a

re
rFl
PROPERTY-17 If a, b are real numbers, then |i7 + &| <|fl|+|&|

F
More generally, we have
I ■+■ /?2 I —I 1 I ^2 I I I

r
ou
fo
THEOREM 1 (Arithmetic-Geometric mean inequality) Let a, b be distinct positive real numbers. Then,
ks
a + b
> 4^ i.e. AM >GM
2
oo

PROOF We know that


Y

(^/iI-^/t)^>0 for a^b


B

(Vi?)^ +{V^)^~2 4a 4b >0


re

b>24a 4b
ou

a +
Y
ad

a +b
>4^
2
d

AM > GM.
in
Re

REMARK 1 If a = b, then AM ^ GM. Thus, in general, AM > GM.


F

REMARK 2 If a, b, c are distinct positive real numbers, then ^● >(abc)


1/3 i.e. AM >GM.

ILLUSTRATIVE EXAMPLES

EXAMPLE 1 If a is any positive real number, prove that a + ->2.


a

SOLUTION Using the inequality AM ^ GM, we obtain


a + \/a 1 1 n 1
> ax a + - >l=>rt + ->2 [Multiplying both sides by 2]
2 a 2^ a a

1 4
ALITER Clearly, 4a - 4^ >0 > 0 for all Y € R]

1 1 1 1
a + —2 4a X >0=> a + — 2>0=c> a + >2.
a 4a~ a a
NUMERICAL INEQUALITIES AND LINEAR INEQUATIONS 6.3

F.XAMI’LE2 Ifa,b are positive real numbers such that ab = 1, then prove that (1 + a) (1 + Jj) > 4.
SOLUTION We find that

{\ + a) {1 + b) + a + b + ab = 1 + a + b +1 [V ab=\]
=2+a+b

Now,
a +b
> ^f^b [Using AM > GM]
2
a +b
>1 [v nb=\]
2
a+b >2

ow
2 + a+b>A [Adding 2 on both sides] ●●●(ii)
From (i) and (ii), we obtain
(1 +fl) (1 +;?) >4
l-A-
For amj real number x. prove that: +4 >4

e
EXAMPLES

re
SOLUTION Using the inequality AM > GM, we obtain
l-.r

rFl
F
4-’'+ 4 l-.r

2
l-.T 1-a:
4^+4 4^ + 4

r
l-.x
ou
> 2 => 4^ + 4 >4
2 2
fo
ks
2 2
EXAMPLE 4 If a, b, c are positive real numbers, prove that: a +b + c ~ >ab + be + ca.
oo

SOLUTION Using: AM > GM, we obtain


a^ + b^
Y
B

2 2 ^ 2
2 2
a^ +b^ b^ +
re

>ab, >bc and > ca


2 2 2
ou
Y

^+b'^>2ab, b^ + c'^>2bc and c^ + a^ > 2ca


ad

{a^ + b^) + {b^ + c^) + (c^ + a^) > 2ab + 2bc + 2ca
d

2 {a^ + b^ + c^) >2 (ab + be + ca)


in
Re

a ^ +b^ + c^ >ab +bc + ca


F

EXERCISE 6.1

1. Ifn>l, b >1, then prove that log/, (7 + logn ^2.


l-x
2. For any real number x, prove that 9'^ + 9 >6.

If a, b, c are non-zero real numbers of the same sign, prove that - + - + ->3.
b c a

4. If a, b, c are three distinct real numbers, prove that (a + b) (Ij + c) (c -f a) > 8 abc.

6.2 INEQUATIONS

In earlier classes, we have studied equations in one and two variables. An equation is defined as
a statement involving variable (s) and the sign of equality (=). Similarly, we define the term
inequation as follows:
INEQUATION A statement involving variable (s) and the sign of inequality viz, >, <,>or < is called an
inequation or an inequality.
6.4 APPLIED MATHEMATICS-Xn

An inequation may contain one or more variables. Also, it may be linear or quadratic or cubic
etc.

Following are some examples of inequations:


(i) 3.r-2<0 (ii) 2.V + 3 < 0 (iii) 5-T - 3 > 0
(iv) 4a + 5 > 0 (v) 2.V + 3y < 1 (vi) 5.r + 4y < 3
(vii) 4a - 6y >5 (viii) 2x + 5y > 4 (ix) 2a-^ + 3a' + 4 > 0
(x) a^ - 3a + 2 > 0 (xi) + 3.V + 2 < 0 (xii) x^ -5.a + 4 <0
(xiii) a^ - 6a^ + 11a - 6 > 0 (xiv) 4- 6,v^ + 11a: + 6 < 0
LINEAR INEQUATION IN ONE VARIABLE let a he a non-zew real number and x be a variable. Then
inequations of the form ax + b <0, ax + b <0, ax + b >0 and fl.r + /? > 0 are known as linear inequations

low
in one variable x.

For example, 9.v -15 > 0, 5a: - 4 > 0, 3j + 2 < 0 and 2a' - 3 < 0 are linear inequations in one
variable.

LINEAR INEQUATIONS IN TWO VARIABLES Let a, b be non-zero real numbers and a', y be variables.

ee
Then inequations of the form ax + by < c, ax + by < c, ax + by > c and ax + by>c are known as linear
rF
inequatio7js in two variables x and y.

Fr
For example, 2a:+3y<6, 3a'-2y>12, ar + y<4, 2a' + y>6 are liiaear inequations in two
variables x and y.
QUADRATIC INEQUATION Let a be a non-zero real number.
for
Then an inequation of the form
u
ax +bx + c <0, or ax +bx + c <0, or ax + bx + oO, or ax + bx + c>0 is knozon as a quadratic
ks
inequation.
Yo
o

For example, a*^ + a* - 6 < 0, - 3.t + 2 > 0, 2a'^ + 3.r +1 > 0 and x^ -5a' + 4 < 0 are quadratic
Bo

inequations.
re

6.3 SOLUTIONS OF AN INEQUATION

DEFINITION A solution of an inequation is the value (s) of the variable (s) that makes itatruestatement.
ou
ad

^ ., , . . 3 - 2a
< 4.
1

Consider the inequation —^—


Y

3
3 - 2a X
Left hand side (LHS) of this inequation is —^ and right hand side (RHS) is — -4.
nd
Re

Fora = 9, we have
Fi

3-2x9
LHS = 3 and, RHS = -
5 3

Clearly, - 3 < -1
LHS < RHS, which is true.

So, a = 9 is a solution of the given inequation.


For a = 6, we have
3-2x6
LHS = -- and RHS = --4 = -2
5 5 3
9
Because, - — < - 2 is not true. So, a = 6 is not a solution of the given inequation.

We can verify that any real number greater than 7 is a solution of the given inequation.
Let us now consider the inequation a + 1 < 0. We know that
a^ > 0 for all a € R
6.5
NUMERICAL INEQUALITIES AND LINEAR INEQUATIONS

+ I >l for aWxsR

,Y“ +1 4 0 for any x eR.


So, there is no real value of .r v^hich makes the given inequation a true statement. Hence, it has
no solution.

It follows from the above discussion that an inequation may or may not have a solution.
However, if an inequation has a solution it may have infinitely many solutions.
SOLVING AN INEQUATIO^! It is the process ofobtainmg all possible solutions of an inequation.
SOLUTION SET The set of all possible solutions of an inequation is knoivn as its solution set.
For example, the solution set of the inequation +1 > 0 is the set R of all real numbers whereas
the solution set of the inequation .r^ +1 < 0 is the null set (ji

w
6.4 SOLVING LINEAR INEQUATIONS IN ONE VARIABLE
As mentioned in the previous section that solving an inequation is the process of obtaining its all
possible solutions. In the process of solving an inequation, we use mathematical simplifications
which are governed by the following rules:
RULE 1

F lo
Same number may be added to (or subtracted from) both sides of an inequation without

ee
changing the sign of inequality.

Fr
RULE 2 Both sides of an inequation can be multiplied (or divided) by the same positive real number
xvithout changing the sign of inequality. However, the sign of inequality is reversed when both
for
sides of an inequation are multiplied or divided by a negative number.
Any term of an inequation may be taken to the other side with its sign changed xuithout
ur
RULE 3

affecting the sign of inequality.


s
ook

A linear inequation in one variable is of the form


Yo

(7Y +1 < 0 or, ax + < 0 or, ax +1? > 0 or, ax + b >0.


eB

We follow the following algoritlim to solve a linear inequation in one variable.


ALGORITHM
our
ad

STEPl Obtain the linear inequation.


STEP II Collect all terms involving the variable on one side of the inequation and the constant terms on
the other side.
Y

STEP III Simplify both sides of inequality in their simplest forms to reduce the inequation in the form
Re

ax <b, or ax <b, or ax >b, or ax >b


nd

STEP 1\ Solve the inequation obtained in step III by dividing both sides of the inequation by the
Fi

coefficient of the variable.


STEF V Write the solution set obtained in step IV in the form of an interval on the real line.
Following examples will illustrate the above algorithm.
ILLUSTRATIVE EXAMPLES

Type I SOLVING EQUATIONS OF THE FORM: n.v + b > cx + d, or, ax + b > cx + d,


or, ax + b < c.v + d or, ax + b < c.r + d
EXAMPLE 1 Solve the folloioing linear inequations:
(i) 2.r - 4 < 0 (ii) - 3y + 12 < 0 (iii) 4.t -12 > 0 (iv) 7y + 9>30
SOLUTION (i) We have,
2.y - 4 < 0

(2y-4) + 4 < 0 + 4 [Adding 4 on both sides]


2y<4
6.6 APPLIED MATHEMATICS-XII

lx A
< —
1 1
x<2

Hence, any real number less than or equal to 2 is a solution of the given inequation.
These solutions can be graphed on real line as shown in Fig. 6.1
— 00
2 + CO

Fig. 6.1

The solution set of the given inequation is (- co, 2]


(ii) We have,
3a: + 12<0

ow
-3:c<-12 [Transposing 12 on right side]
-3.V -12
>
-3 -3

a: > 4

e
Thus, any real number greater than 4 is a solution of the given inequation.

Fl
re
Hence, the solution set of the given inequation is (4, co). This solution set can be graphed on

F
real line as shown in Fig. 6.2
0 4
— 00
ur + 00

r
Fig. 6.2

(iii) We have.
fo
ks
4a--12>0
Yo
4.v>12 [Transposing 12 on RHS]
oo

=> i£>i? [Dividing both sides by 4]


B

4 4
.T> 3
re

.V e [3, oo)
u

Hence, the solution set of the given inequation is [3, co). This solution set can be graphed on real
ad
Yo

line as shown in Fig. 6.3.


- 00 0 3 + 00
d

Fig. 6.3
Re
in

(iv) We have,
F

7x + 9> 30
7x >30-9
7.v>21 [Transposing 9 on RHS]
7x 21
>
7 7
> 3

.T€{3, oo)
Hence, (3, oo) is the solution set of the given inequation. This can be graphed on real line as
shown in Fig. 6.4.
— 03 0 3 + 00

Fig. 6.4

liXA.viPLE 2 Solve: 5.V - 3 < 3x +1 when:

(i) .r is a real number (ii) X is integernumber (iii) x is a natural number.


NUMERICAL INEQUALITIES AND LINEAR INEQUATIONS 6.7

SOLUTION We have,
5.r - 3 < 3x + 1
5-y - 3.y < 3 + 1 [Transposing 3x on LHS and - 3 on RHS]
2x < 4
2x 4 1
< Multiplying both sides by -
2 2

Y < 2

(i) If X eR, then


Y <2 => Y €(-C0, 2)
Hence, the solution set is (- co, 2) as shown in Fig. 6.5.
— X 0 1 2 00

Fig. 6.5

w
(ii) If Y e Z, then
y<2 => Y = l,0,-l,-2,-3,-4
So, the solution set is {. -4,-3,-2,-1,0,1}
(iii) If Y 6 N, then

F lo
e
Y <2 => Y = 1

Fre
So, the solution set is (!}.
EXAMPLE 3 Solve tbefoUozviug equations:
for
(i) 3x + 17 < 2 (1 - y) (ii) 2(2y+3)-10 <6(y-2)
r
SOLUTION (i) We have,
You

3y + 17 <2(1 -Y)
s
ook

3y +17 < 2 - 2y

[Transposing - 2y to LHS and 17 to RHS]


eB

3y + 2y<2-17
5y<-15
5y^-15
our
ad

5 5
y<-3

Y e(-x, - 3]
dY
Re

Hence, the solution set of the given inequation is (- oo, — 3], which can be graphed on real line as
shown in Fig. 6.6.
Fin

- X -3 0 + »

Fig. 6.6

(ii) We have,
2(2y+ 3)-10<6(y-2)
4y + 6-10<6y-12
4y - 4 < 6y -12
4y-6y< -12 + 4 [Transposing - 4 to RHS and 6y to LHS]
- 2y < - 8
-2y -8
>
-2 -2

Y >4

Y e [4, co)
6.8 APPLIED MATHEMATICS-XII

Hence, the solution set of the given inequation is [4, cc) which can be graphed on real line as
shown in Fig. 6.7.
- oo 0 4 + oo

Fig. 6.7

EXAMPLE 4
Solve the following inequations:
2x - 4a: 5x-2 7x-3 X
(i) +9 >3 + (ii) >
4 3 3 5 4
1 (3 1 3{x-2) 5{2-x)
(iii) T .r + 4 > -(.r-6) (iv) >

21.5 5 3

SOLUTION (i) We have,

w
2a:-3 4a:
+ 9>3 + —
4 3
2a: - 3 4a: 4a:
>3-9 [ Transposing — to LHS and 9 to RHS]

Flo
4 3 3
3 (2a--3)-16a:

e
> -6
12

re
6x-9-16x

F
>-6
12
-9-lOx
ur
r
=> >-6
12 fo
9 -10.t>-72 [Multiplying both sides by 12]
ks
=> -10a: >-72+ 9
Yo

-10.v>-63
oo

=>
-10.t^-63
B

-10
re

63
a:< —
10
u
ad

=> . a: € (- 00, 63/10]


Yo

Hence, the solution set of the given inequation is{- oo, 63/10].This can be graphed on real line as
shown in Fig. 6.8.
d
Re

- CO 0 63 + 00
in

10
Fig. 6.8
F

(ii) We have.
5a:-2 7a:-3 X
> —
3 5 4

5(5.v-2)-3(7a:-3)^ a~
15 4
25a:-10-21.x+ 9 a-
> —
15 4
4a-1 .V
> —
15 4

4(4x-l) >15.x [Multiplying both sides by 60 i.e. 1cm of 15 and 4]


16x-4>15.y
16.v-15.y>4 [ Transposing 15a to LHS and - 4 to RHS]
A > 4

A e(4, oo)
NUMERICAL INEQUALITIES AND LINEAR INEQUATIONS 6.9

Hence, the solution set of the given inequation is (4, co). This can be graphed on the real line as
shown in Fig. 6.9.
- ® 0 4 + 00

Fig. 6.9

(iii) We have,

Ifl x + 4 >
2I5

w
\f3x + 20^ 1
2I 5
3a: + 20 x-6
>

e
10 3

ro
re
3(3.t + 20)>10(A:-6) [Multiplying both sides by 30 i.e. the km of 3 and 10]
9a: + 60>10.v-60

F
9x-10.r>-60-60 [Transposing IOa on LHS and 60 on RHS]
-A->-120

A <120

Fl
u [ Multiplying both sides by -1]

sr
=?■ A e(-00,120]

ko
Hence, the solution set of the given in equation is (- co, 120] which can be graphed on real line
o
as shown in Fig. 6.10.
of
+ CO
- 00
120
o
Fig. 6.10
Y
erB

(iv) We have.
3(a-2) ,5(2-a)
uY

5 3
3a-6 10-5a
>
5 3
ad
do

3(3a-6)> 5(10~5.a) [Multiplying both sides by 15 i.e. the l.c.m. of 5 and 3]


9a-18 >50-25a
in

9a + 25a >50+ 18 [Transposing - 25a to LHS and 18 to RHS]


34a > 68
Re

34a ^ 68
F

34 34
A >2

A e [2, x)
Hence, [2, x) is the solution set of the given inequation. This solution set can be graphed on real
line as shown in Fig. 6.11.
— CO + 00
0 2

Fig. 6.11

nxAMPLF 5 Solve thefollozuing inequations:


A + 1
<0 (ii) >1
A + 2

SOLUTION (i) We have.


1
<0
a-2
6.10 APPLIED MATHEMATICS-XII

a
x-2<0 < 0 and a>0 ^ b <0
b
x<2

:te(-o3, 2)
Hence, the solution set of the given inequation is ( - co, 2).
(ii) We have.
x + 1
>1
x+2

x + l

ow
-1 >0
x + 2

X + 1-x-2
>0
x+2
-1

e
=> >0

re
x + 2

Frl
a
x+2<0 V - >0 and a <0 => b <0

F
h
x<-2
ou
X e(-x, -2)

r
Hence, the solution set of the given inequation is (- oo, - 2).
so
Ti/pe II EQUATIONS OF THE FORM
kf
ax + b ax + b ax + b ax + b
> k, or >k, or <k, or <k
oo
CX + d cx + d cx + d cx + d
Y

In order to solve this type of inequation, we use the following algorithm.


eB

ALGORITHM

STEP I Obtain the inequatio)i.


ur
oY

STEP n
Transpose all terms on LHS.
STEP 111
Simplifi/ LHS of the inequation obtained in step U to obtain an inequation of the form
ad

px + ? px + q px + fj
>0, or >0, or <0, or <0.
d

rx + s rx + s rx + s rx + s
in

STEP IV Make coefficient x positive in numerator and denominator if they are not.
Re

STEP V
Equate numerator and denominator separately to zero and obtain the values ofx. These values
F

of X are generally called critical points.


5?rEP VI Plot the critical points obtained in step V on real line. These points ivill divide the real line in
three regions.
STEP \'H In the right most region the expression on LHS of the inequation obtained in step IV will be
positive and in other regions it will be alternatively negative and positive. So, mark positive
sigJ7 in the right most region and then mark alternatively negative and positive signs in other
regions.
STEP VIII Select appropriate region on the basis of the sign of the inequation obtained in step IV. Write
these regions in the form of intervals to obtain the desired solution sets of the given inequation.
EXAMPLE 6 Solve the following linear mequations:
(i)^>0 (ii)- ^>2
X -5 X + 5

SOLUTION (i) We have,


x-3
>0 ...(i)
X -5
NUMERICAL INEQUALITIES AND LINEAR INEQUATIONS 6.11

Equating x Band x-5 to zero, we obtain x = 3,5 as critical points. Plot these points on
real line as shown in Fig. 6.12. The real line is divided into three regions. In the right most region
the expression on LHS of (i) is positive and in the remaining two regions it is alternatively
negative and positive as shown in Fig. 6.12. +
+

+ cc
— oo 3 5

Fig. 6.12

w
Since the expression in (i) is positive, so the solution set of the given inequation is the union of
regions containing positive signs. Hence, from Fig. 6.12
X -5
^>0 => X g{-oo, 3) u(5, co)

e
Hence, the solution set of the given inequation is (-qo, 3) u(5, co) as shown in Fig. 6.12.

ro
re
(ii) We have.
X -
->2
X + 5

F
x-2

Fl
-2>0
x + 5

X - 2 - 2 (X -!● 5)

u
sr
=> >0
x+5

ko
o
X-2-2X-10
>0 of
x +5
-x-12
>0
o
Y
x +5
erB

x + 12
<0
'Multiplying by -1 to make coefficient of ...(i)
x +5 X positive in the expression in numerator
uY

On equating x + 12 and x + 5 to zero, we obtain x = -12, -5 as critical points. These points are
plotted on number line as shown in Fig. 6.12. The real line is divided into three regions and the
signs of LHS of inequation (i) are marked. Since the inequation in (i) possesses less than sign
ad
do

which means that LHS of the inequation is negative. So, the solution set of the given inequation
is the union of the regions ontaining negative sign in Fig. 6.13. Hence, the solution set of the
in

given inequation is (-12, -5). +


+
Re

-12 -5 -f OO
F

— CO

Fig. 6.13

EXA.MPl.E 7 Solve the following inequations:


2x + 4 X + 3
(i) >5 (ii) <2
x-1 x-2

SOLUTION (i) We have,


2x + 4
>5
x-1

2x + 4
-5>0
x-1

2x + 4-5(x-l)
>0
x-1
6.12 APPLIED MATHEMATICS-XII

2x + 4-5x + 5
>0
x-1

- 3.V + 9
>0
x-1

3x-9
<0 [Multiplying both sides by -1]
x-l

3{x-3)
<0
(^-1)
x-3
<0 [Dividing both sides by 3]

ow
x-l

l<x<3 [See Fig. 6.14]


A.*e(l, 3]
+ +

e
— CO 1 3 + GO

re
Fl Fig. 6.14

F
Hence, the solution set of the given inequation is (1, 3j.
(ii) We have.
ur
r
X + 3
x-2
<2
fo
ks
x + 3
Yo
-2<0
x-2
oo

x + 3-2x +4
<0
eB

x-2

-x + 7
<0
ur

x-2
ad

x-7
Yo

x-2
>0 [Multiplying both sides by -1]
d

X e{-oa, 2) u[7, oo) [See Fig. 6.15]


Re
in

+ +
-o-
— 00 + 00
2 7
F

Fig. 6.15

Hence, the solution set of the given inequation is (- co, 2) u [7, co).

Solve the following linear inequations in R.


Solve: 12r<50, when
(i) .Y e R (ii) Y e Z (iii) ysN
Solve: - 4y > 30, when
(i) Y e R (ii) Y e Z (iii) X sN

Solve: 4y - 2 < 8, when


(i) Y e R (ii) Y€Z (iii) Y eN
3y-7 >y + 1 y + 5>4y-10 3y + 9 > - Y + 19
6.13
NUMERICAL INEQUALITIES AND LINEAR INEQUATIONS

_ 3-t - 2 4a: - 3
7. 2(3-a)>-- + 4 8. < 9. - (a:-3)+ 4 <5-2a-
5 2

10. -<
3a-2 5a-3 11. 2(.v-l) ^3(2 + a:) 2 4 4
5 4 5 5 7
2a + 3 5-2a A

13.
A-1
+ 4< ^-2 3
15.
3
<--5
6
3 5 4
4 +2a 2a + 3 3(a:-2) 5a+ 8
16. >^-3 17. -2< 18. a-2<
3
3 2 5 5
6 a -5 2a- 3 3
19. 20. >0 21. <1
<0
4a+ 1 3a-7 A-2

.. 5a + 8 A 1
1 2'’..
22. <2 <2 > -

A-1 4-a A -5 2

ANSWERS

(ii) |...-3,-2,-l,0,l,2, 3,4) (iii) (1,2, 3,4}

w
1. (i) (-x,25/6)
2. (ii) (-co,-15/2) (ii) (. ....,-9,-81 (iii) (})
3. (i) (-co,5/2)
■ (4, co)
7. (-00,10/11]
(ii) i ,-2,-1, 0,1, 21
(-«=,5)
[11/14, x) F lo ■

(iii) 11, 2|
[5/2,0))
(-co,-2)

e
Fre
(-0), 2/9) [- 44, co) [3, co)
(-o),-13/2) (8,co)
(- CO,-50)
for
[-26, oo) (-1, co) ●● [-7,co)
(-l/4,5/6) (-00, 3/2)u(7/3, co) . ■ - (- co, 2) u (5, co)
r
You

(-00, 0) u(4, co) (-CO,-5) u(5, co)


oks

(-00, 1) U [3/2, co)


eBo

r
●SOLUTION OF S'"'" Of INEAR INEQUATIONS IN ONE Va -

In the previous section, we have learnt how to solve a linear inequation in one variable. In this
section, we shall use it to solve a system of linear inequations in one variable. Recall that the
ad
our

solution set of a linear inequation is the set of all points on real line satisfying the given
inequation. Therefore, the solution set of a system of linear inequations in one variable is the
intersection of the solution sets of the linear inequations in the given system.
We use the following algorithm to solve a system of linear inequations in one variable.
Re
dY
Fin

Obtain the system of linear inequations.


Solve each inequation and obtain their solution sets. Also, represent them on real time.
Find the intersection of the solution sets obtained in step U by taking the help of the graphical
representation of the solution sets in step 11.
The set obtained in step III is the required solution set of the given system of inequations.
Following examples will illustrate the above algorithm.
ILLUSTRATIVE EXAMPLES

Solve the following system of linear inequations:


3a-6 >0
4a-10 <6

SOLUTION The given system of inequations is


3a - 6 > 0 ...(i)
6.14
APPLIED MATHEMATICS-XII

4a:-10 <6
●●●(ii)
Now, 3a: ^>-6
3a: - 6 > 0 => 3a: > 6 ^ — => x>2
3 3

Solution set of inequation (i) is [2, oo)


and, 4a:-10 <6 => 4.\-<16 => .v<4

Solution set of inequation (ii) is (- oo, 4]


The solution sets of inequations (i) and (ii) are represented graphically on real line in Figs. 6.16

ow
(i) and (ii) respectively.
- 00 + 00
0 2 4

Fig. 6.16 (i)

e
re
— 30
0 2 4 + CO

Fig. 6.16 (ii)

Flr
F
Clearly, the intersection of these solution sets is the set [2,4].
ou
Hence, the solution set of the given system of inequations is the interval [2,4].
RXAMPLL2
Solve the following system of inequations:

sr
5x 3x 39

ko
— + — > —
4 8
2x-l X -1
<
3x +1
of
12 3 4
o
SOLUTION The given system of inequation is
Y

5x 3x 39
reB

— + >
4 8 8
...(i)
2x -1 X -1 3x + 1
uY

<
12 3 4
...(ii)
5x 3x 39
Now, — + >
4 8 8
ad
do

10x+3x 39
>
8
in

13x > 39
Re

X > 3
F

X e(3, oo)
So, the solution set of inequation (i) is the interval (3, oo).
2x -1 X -1 3x +1
and. <
12 3 4
(2x-l)-4(x-l) <
3x + l
12 4
-2x +3 3x +1
<
12 4

- 2x + 3 < 3 (3x + 1) [Multiplying both sides by 12 i.e. the l.c.m. of 12 and 4]


- 2x + 3 < 9x + 3
- 2x - 9x < 3 - 3
-llx<0
=> x>0

X e(0, oo)
6.15
NUMERICAL INEQUALITIES AND LINEAR INEQUATIONS

So, the solution set of inequation (ii) is the interval (0, oo). Let us now represent the solution sets
of inequations (i) and (ii) on real line. Tliese solution sets are graphed on real line in Figs. 6.17 (i)
and 6.17 (ii) respectively.
+ CO
- CO 0 3

Fig. 6.17 (i)

+ CO
3

ow
- CO 0

Fig. 6.17 (ii)

From Figs. 6.17 (i) and (ii), we observe that the intersection of the solution sets of inequations (i)
and (ii) is interval (3, <») represented by common thick line.

e
Hence, the solution set of the given system of inequations is the interval (3, <x).

re
EXAMPLr; 3 Solvc the follozving system of inequations:
, 2.r-3 4.V

Flr
F
+ 6^2 + —
2(2.v+3)-10<6(-.r-2), 3

SOLUTION The given system of inequations is


2 (2.V + 3) -10 < 6 {x - 2)
ou
sr
2x-3 4.V
...(ii)

fo
+ 6>2 + —
4 3

Now, 2(2x+ 3)-10<6(a:-2)


k
oo
4x + 6 -10 < 6x -12
Y
4.t-4<6v-12
reB

4x - 6x <4 -12
- 2x < - 8
uY

X >4

X e(4, oo)
So, the solution set of the first inequation is the interval (4, co).
ad
do

2x- 3 4x
and. + 6^2 +
4 3
in

2x - 3 + 24 6 + 4.V
Re

>
4 3
F

2x +21 4x + 6
>
4 3

3(2x + 21)>4(4x+6)
6x+63>16x + 24
6x-16x>24-63
-10x>-39
39
X < —
10

X < 3.9

xg(-oo, 3.9]
So, the solution set of inequation (ii) is the interval (- <», 3.9].
The solution sets of inequations (i) and (ii) are
graphed on real line in Figs. 6.18 (i) and (ii)
respectively.
6.16
APPLIED MATHEMATICS-XII

ii.y

— CO 0 4 + CO

Fig. 6.18 (i)

3.9

- X
0 4 + QO

Fig. 6.18 (ii)

We observe that there is no common solution of the two inequations. So, the given system of
inequations has no solution.
EXAMPLt 4 Solve: -11 < 4.V - 3 < 13
SOLUTION We have,

ow
-11 >4.v-3>13 o -11 >4x - 3 and 4a: - 3 >13

Thus, we have two inequations and we wish to solve them simultaneously. Instead of solving
these inequations by using the method discussed in first three examples, let us solve them
directly in a different way as given below.
We have.

e
re
-11 <4.v-3<13
-ll + 3<4.v-3+3<13+3

Fl [Adding 3 throughout]

F
- 8 < 4x < 16
ur 16

r
<r< —
4 4 [Dividing by 4 throughout]
-2 < X < 4
fo
ks
X e [- 2, 4]
Yo
Hence, the interval [ - 2,4] is the solution set of the given system of inequations.
oo

2 - 3x
EXAMPLE 5 Solve: - 5 < <9
eB

SOLUTION We have.
2-3x
ur

-5< <9
4
ad
Yo

2-3x
-5 X 4 < X 4 <9 X 4
4 [Multiplying throughout by 4]
d

- 20 < 2 - 3x < 36
Re

- 20 - 2 < 2 - 3x - 2 < 36 - 2
in

[Subtracting 2 throughout]
- 22 < - 3x < 34
F

- 22 - 3x >. ^
-3 -3 [Dividing throughout by - 3]
22 -34
— >x>
3 3
- 34 22
<x < —
3 3

xe[- 34/3, 22/3]


Hence, the interval [- 34/ 3, 22/ 3] is the solution set of the given system of inequations.
EXAMPLE b
Solve the system of inequations:
X
> i 6x
<
1

2x + l 4' 4x-l 2
SOLUTION The given system of inequations is
X 1
> —
2x + l 4
6.17
NUMERICAL INEQUALITIES AND LINEAR INEQUATIONS

6.Y .1 ●●●(ii)
< —
4.V-1 1
1
Now, > —

2.Y +1 4
Y 1
>0
2y + 1 4

4y-(2y + 1) >0
4 (2y +1)
2y-1
>0 [Multiplying both sides by 4]
2y + 1

ow
Y e(-oo, -1/2) u[l/2, oo) [See Fig. 6.19 (i)]
Thus, the solution set of inequation (i) is (-«>, -1/2) u [1/2, oo) ...(in)
6y 1
And, < —
4y-1 1

e
1

re
6y
4y-1
--<0
2

rFl
F
12y-(4y-1) <0
2(4y-1)

r
ou
8y + 1

2(4y-1)
<0 fo
ks
8y + 1
<0 [Multiplying both sides by 2]
oo

4y-1

[See Fig. 6.19 (ii)]


Y

Ye(-l/8,l/4)
eB

- 00 1 + «

2 2
r

Fig. 6.19(1)
ou
ad
Y

- CO 1 + 00
4
d

Fig. 6.19 (ii)


Re
in

Thus, the solution set of inequation (ii) is


F

(-1/8,1/4) ...(iv)

It is evident from Fig. 6.19 that the intersection of (iii) and (iv) is the null set.
Flence, the given system of equations has no solution.
EXERCISE 6.3

Solve each of the following si/stem of equations in R.


1. 2y+5<0, y-3<0 2. 5y-1 <24, 5y + 1 >-24

3. 3y-1>5, y+2>-1 4. 11 -5y>-4, 4y + 13<-11

5. 4y-1<0, 3-4y<0 6. y + 5>2(y + 1), 2-y<3(y + 2)


3y
8. 5y-7 <3(y+ 3), 1 > Y - 4
7. 2(y-6)<3y-7, 11-2y<6-y
7y-1 3y + 8
<- 3, + 11 <0
9.
4 - 2 > y - 6, 2 (2y + 3) < 6 (y - 2) + 10 10. 2 5
6.18
APPLIED MATHEMATICS-XI!

2.V + 1 x + 7
11. >5, >2 12. 0<—<3
7x-l -r-8 2

13. 10 <-5 (.r-2) <20 14. -5<2a--3<5


4 6
15. -^<3<-^,;i->0
x + l .t +1

ANSWERS
1. (-co,-5/2] 2. (-5,5) 3. [2, CO) 4. (-X, -6]
5. No Solution 6- (-1,3) 7. (5, ») 8. (-00,2]
0. No Solution 10. (-00, -21) 11. No Solution 12. (-6, 0)
13. (-2, 0] 14. (-1,4) 15. [1/3,1]

6.6 SOME APPLICATIONS OF LINEAR INEQUATIONS IN ONE VARIABLE

In this section, we shall utilize the knowledge of solving linear in equations in one variable in
solving different problems from various fields such as science, engineering, economics etc.

w
Following examples will illustrate the same,

F lo
ILLUSTRATIVE EXAMPLES

EXAMl’i.F. 1 Find all pairs of consecutive odd positive integers, both of which are smaller than 18, such
that their sum is more than 20.

ee
Fr
SOLUTION Let x be the smaller of tlie two consecutive odd positive integers. Then, the other
odd integer is x + 2. for
It is given that both the integers are smaller than 18 and their sum is more than 20. Therefore,
.Y + 2 < 18 and, y + (y + 2) > 20
r
You

Y <16 and 2y + 2 >20


s
ook

Y <16 and 2y >18

Y <16 and y >9 => 9 < y <16 => y =11,13,15 [ Y is an odd integer ]
eB

Hence, the required pairs of odd integers are (11,13), (13,15) and (15,17).
EXAMPLE 2
Find all pairs of consecutive even positive integers, both of which are larger than 8, such
our
ad

that their sum is less than 25.

SOLUTION Let y be the smaller of the two consecutive even positive integers. Then, the other
even integer is y + 2.
dY
Re

It is sgiven that both the integers are larger than 8 and their sum is less than 25. Tlaerefore,
Y > 8 and y + y + 2 < 25
Fin

Y > 8 and 2.v + 2 < 25


Y > 8 and 2.v < 23
23 23
Y > 8 and y < — ^ 8 < y < => Y = 10
[ Y is an even integer ]
2 2

Hence, the required pair of even integers is (10,12).


r;XAMPLi;3 The cost and revenue fuiictions of a product are given by C(y)=2y + 400 and
R{x) =. 6y + 20 respectively, where x is the number of items produced by the manufacturer. How many
items the manufacturer must sell to realize some profit?
SOLUTION We know that: Profit = Revenue - Cost. Therefore, to earn some profit, we must
have
Revenue > Cost
6y + 20 > 2y + 400
380
6y - 2y > 400 - 20 => 4y > 380 => y > = 95
4
Hence, the manufacturer must sell more than 95 items to realize some profit.
6.19
NUMERICAL INEQUALITIES AND LINEAR INEQUATIONS

MA
EXAMPL1I4 IQ of a person is given by the formula: IQ = X 100, where MA is mentnl age andCA is
CA

chronological age. If 80 <IQ< 140 for a group of 12 year children, find the range of their mental age.
SOLUTION We have: CA = 12 years
MA MA 25
IQ = xl00=> IQ = X 100 = — MA
CA 12 3

Now, 80 < IQ <140


80 < — MA<140
3
240 420
240 < 25 M/1 < 420 => < MA< ^ 9.6 < MA< 16.8
25 25

EXAMPLE s In the first four papers each of IQO marks, Rishi got 95, 72, 73, 83 marks. If he wants an
average ofgreater than or equal to 75 marks and less than 80 marks,find the range of marks he should score

w
in the fifth paper.
SOLUTION Suppose scores x marks in the fifth paper. Then,

F lo
95 + 72 + 73 + 83 + X
75 < <80
5
323 + A-

ee
75 < < 80 => 375 < 323 + .y < 400 => 52 < y < 77
5

Fr
Hence, Rishi must score between 52 and 77 marks.
EXAMPLE 6 A manufacturer has 600 litres of a 12% solution of acid. How many litres of a 30 /o acid
for
the resulting mixture will be more than 25% but less
solution must be added to it so that acid content in
ur
than 18% ?
of 12% solution of acid. Then,
s

SOLUTION Let Y litres of 30% acid solution be added to 600 litres


ook
Yo

Total quantity of mixture = (600 + .y) litres


30y
eB

X 600
Total acid content in the (600 + .y) litres of mixture =
100 100

It is given that acid content in the resulting mixture must be more than 15% and less than 18%.
our
ad

30y 12
15%of(600 + Y)< + - X 600 <18% of (600 +y)
100 100
15 30y 12 18
Y

X (600 + y) < + X 600 < X (600 + Y)


100
Re

100 100 100


nd

15 (600 + y) < 30y + 12 X 600 < 18 (600 + y) [Multiplying through out by 100 ]
Fi

9000 + 15y < 30y + 7200 < 10800 + 18y


9000 + 15y < 30y + 7200 and 30y + 7200 < 10800 + ISx
9000 -7200 < 30y -15.Y and 30y - 18y <10800 - 7200
=> 1800 < 15.Y and 12y < 3600
15y > 1800 and 12y < 3600
Y > 120 and y < 300
=> 120 < Y < 300

Hence, the number of litres of the 30% solution of acid must be more than 120 but less than 300.
EXA.MPl.E 7 /I man wants to cut three lengths from a single piece of board of length 91 cm. The second
length is to be 3 cm longer than the shortest and third length is to be twice as long as the shortest. What are
the possible lengths for the shortest board if third piece is to be at least 5 cm longer than the second?
SOLUTION Let the length of the shortest piece be x cm. Then, the lengths of the second and third
piece are x + 3 cm and 2x cm respectively. Then,
Y + (y + 3) + 2y < 91 and 2x > (x + 3) + 5
6.20
APPLIED MATHEfWIATICS-XII

=> 4.\- + 3 <91 and 2x>x + 8


=> 4:»: < 88 and a: > 8 => x <22 and .r > 8 => 8 < a: < 22.

Hence, the shortest piece must be at least 8 cm long but not more than 22 cm long.
.^EXERCISE 6.4
1. Find all pairs of consecutive odd positive integers, both of which are smaller than 10, such
that their sum is more than 11.
Find all pairs of consecutive odd natural number, both of which are larger than 10, such that
their sum is less than 40.

Find all pairs of consecutive even positive integers, both of which are larger than 5, such
that their sum is less than 23.
The marks scored by Rohit in two tests were 65 and 70. Find the minimum marks he should
score in tlie third test to have an average of at least 65 marks.
3. A solution is to be kept between 86° and 95°F. What is the range of temperature in deg ree
Celsius, if the Celsius (C)/Fahrenheit (F) conversion formula is given by f = — C + 32.

Fahrenheit?

F low
6. A solution is to be kept between 30°C and 35°C. What is the range of temperature in deg ee

7. To receive grade 'A’ in a course, one must obtain an average of 90 marks or more in five
papers each of 100 marks. If Shikha scored 87,95, 92 and 94 marks in first four papers, find
the minimum marks that she must score in the last paper to get grade 'A' in the course.

C = 300 + — a: and R
e
8. A company manufactures cassettes and its cost and revenue functions for a week are
for Fr
2 = 2.t respectively, where a: is the number of cassettes produced and
sold in a week. How many cassettes must be sold for the company to realize a profit?
9. Tlie longest side of a triangle is three times the shortest side and the third side is 2 cm
Your

shorter than the longest side if the perimeter of the triangles at least 61 cm, Find the
s
eBo k

minimum length of the shortest-side.


10. How many litres of water will have to be added to 1125 litres of the 45% solution of acid so
that the resulting mixture will contain more than 25% but less than 30% acid content?
ad
our

11. A solution of 8% boric acid is to be diluted by adding a 2% boric acid solution to it. The
resulting mixture is to be more than 4% but less than 6% boric acid. If there are 640 litres of
the 8% solution, how many litres of 2% solution will have to be added?
Re

12. The water acidity in a pool is considered normal when the average pH reading of three
daily measurements is between 7.2 and 7.8. If the first two pH reading are 7.48 and 7.85,
Y
Find

find the range of pH value for the third reading that will result in the acidity level being
normal.

ANSWERS
●1. (5,7), (7,9) 2. (11,13), (13,15), (15,17), (17,19) 3. (6, 8), (8,10), (10,12)
4. 60 5. Between 30°C and 35°C 6. Between 86°Fand95°F
7. 82 marks 8. More than 600 9. 9 cm
10. More than 562.5 litres but less than 900 litres
11. More than 320 litres but less than 1280 litres 12. Between 6.27 and 8.07

6.7 GRAPHICAL SOLUTION OF LINEAR INEQUATIONS IN TWO VARIABLES

If a, b, c are real numbers, then the equation (7r + fjy + c = 0 is called a linear equation in two
variables .v and i/ whereas the inequalities ax + by <c, ax + by >c,ax + by<c and ax + by>c are
called linear inequations in two variables x and i/.
6.21
NUMERICAL INEQUALITIES AND LINEAR INEQUATIONS

We have studied in coordinate geometry that the graph of theequation ax^●b\J=c\s a straight
line which divides the .ry-plane into two parts which are represented by ax + by < c and
+ by>c. These two parts are known as the closed half-spaces. The regions represented by
nx+by<c and ax + by>c are known as the open half spaces. In set theoretical notations, the set
{(.t, t/): (7.Y + 111/= c} is the straight line, sets {(x, y):ax +by <c} and ((y, y): fl.v + Oi/^ c} are
closed half spaces and the sets {(y, y): ax + by<c} and {(y, y): ax + I?y > c} are open half-spaces.
These half spaces are also known as the solution sets of the corresponding inequations.
In order to find the solution set of a linear inequation in two variables, we follow the following
algorithm.
ALGORITHM

STEP I Convert the given inequation, say ax + by <c, into the equation ax + by ~c which represents a
straight line in xy-p!ane.

w
STEP 11 Put y = 0 in the equation obtained in step 1 to get the point where the line meets ivith x-axis.
Similarly, put x = 0 to obtain a point where the line meets with y-axis.

Flo
STEP 111 join the points obtained in step I! to obtain the graph of the line obtained from the given
inequation. In case of a strict inequality i.e. ax + by <c or ax + by > c, draw the dotted line,

ee
othenuise mark it thick line.

Choose a point, if possible (0, 0), not lying on this line : Substitute its coordinates in the

Fr
STEP IV

inequation. If the inequation is satisfied, then shade the portion of the plane which contains the
chosen point; othenvise shade the portion which does not contain the chosen point.
for
ur
STEP V The shaded region obtained in step IV represents the desired solution set.
REMARK In case of the inequalities ax + by < c and ax + by > c points on the line are also a part of the
s
shaded region while in case of inequalities ax + by <c and ii.v + by > c points on the line ax + by
= c are
k
Yo
oo

not in the shaded region.


The following examples illustrate the above algorithm.
eB

ILLUSTRATIVE EXAMPLES

Solve the following inequations graphically:


r

EXAMPLE 1
ou
ad

(i) 2y + 3y < 6 (ii) 2y - y ^ 1 (iii) y>2 (iv) y ^ - 3


(i) Converting the given inequation into equation, we obtain 2y + 3y = 6.
Y

SOLUTION

Putting y = 0 and y = 0 respectively in this equation, we get y = 3 and y = 2. So, this line meets
Y-axis at /\(3,0) and y-axis at B (0,2). We plot these points and join them by a thick line. This line
Re
nd

divides the Yy-plane in two parts. To determine the region represented by the given inequality
Fi

consider the point O (0, 0). Clearly, (0, 0) satisfies the inequality. So, the region containing the
origin is represented by the given inequation as shown in Fig. 6.20. This region represents the
solution set of the given inequations.
y
y
//
V.

(0,2)B
X' X
6

^0)
X' X o
0
A
(3,0)
B(0,-I)

r y

Fig. 6.20 Fig. 6.21


6.22
APPLIED MATHEMATICS-XII

(ii) Converting the given inequation into equation we obtain 2.v - y = 1. This line meets x and
y-axes at A (1/2,0) and B(0, - 1) respectively. Joining these points by a thick line we obtain the
line passing through A and B as shown in Fig. 6.21. This line divides the Ay-plane into two
regions viz. one lying above it and the other lying below it. Consider the point 0(0, 0). Clearly,
(0, 0) does not satisfy the inequation 2.v-y>l. So, the region not containing the origin is
represented by the given inequation as shown in Fig. 6.21. Clearly it represents the solution set
of the given inequation,
(iii) We have .r > 2.Converting the inequation into equation, we obtain x = 2. Clearly, it is a line
parallel to y-axis at a distance of 2 units from it. This line divides the .ry-plane into two parts viz.
one part on the LHS of a: = 2 and the other on its RHS. We find that the point (0,0) does not satisfy
the inequation .r > 2. So, the region represented by the given equation is the shaded region
shown in Fig. 6.22. The shaded region is the required solution set of the given inequation.

w
y
y

Flo
X' X
X' X o

ee
o (2,0) y = -3

Fr
(0,-^)
<N
fl

for
ur
r y

Fig. 6.22 Fig, 6.23


k s
Yo

(iv) We have y < - 3. Converting the given inequation into equation we obtain y = -3. Clearly, it
oo

is a line parallel to .v-axis at a distance of 3 units below it. The line y = -3 divides the Ary-plane into
eB

two regions one below it and the other above it. Consider the point O (0,0). We fiiid that (0, 0)
does not satisfy the inequation y < - 3. So, the region represented by the given inequation is the
region not containing the origin as show in Fig. 6.23. Clearly, it is the solution set of the given
r
ou
ad

inequation.
FXAMPLE2
Solve the following inequations graphicall}/:
Y

(i) |x|<3 (ii) |y-.v|<3 (iii) |A:-y|>l


Re
nd

SOLUTION (i) Converting the given inequation into equation, we obtain a: = 3. This equation
represents a line parallel to y-axis at a distance of 3 units from it. The line given by .v = 3
Fi

divides the .A-y-plane into two regions. Clearly, the point 0(0,0) satisfies a:< 3. So, the graph of
AT < 3 is
as shown in Fig. 6.24. The shaded region represents the solution set of this inequation.
Y

X' (-3,0) (3,0) X


o

y'

Fig. 6.24
6.23
NUMERICAL INEQUALITIES AND LINEAR INEQUATIONS

(ii) We have, \\j-x\<3. This inequation is equivalent to


-3<y-.r<3 [●.● I XI <o -a <x<a]
- 3 <y -X and y -x < 3
^-y 3 < 0 and x -1/ + 3 > 0
The region represented by ] y - x| < 3 is the region common to the regions represented by
x-y - 3 < 0 and X - V + 3 > 0 as shown in Fig. 6.25. This shaded region represents the solution
set of the given inequation.

w
F lo y

ee
Fig. 6.26

Fr
(iii) Wehave, for
lx-yt>lo .Y-y>l or x-y<-l <=> x-y-l>0 or x-y + 1 <0
The required region is the union of regions represented by x - y -1 > 0 and x - y +1 < 0 as
r
shown in Fig. 6.26. The shaded region represents the solution set of the given inequation.
You
s
ook

EXERCISE 6.5
eB

Represent to solution set of each of the following inequations graphicalli/ in two dimensional plane:
2. X + 2y > 6 3. X + 2 > 0
1. x + 2y-4<0
our
ad

4. x-2y <0 5. - 3x + 2y < 6 6. X < 8 - 4y


7. 0<2x-5y + 10 8. 3y > 6 - 2x 9. y > 2x - 8
10. 3x-2y<x + y-8
dY
Re

6.8 SOLUTIONOF SIMULTANEOUSLINEAR INEQUATIONSIN TWO VARIABLE


Fin

In this section, we will discuss the technique of finding the solution set of simultaneous linear
inequations. Solving simultaneous linear inequations means finding the set of points (x, y) for
which all the constraints are satisfied. Note that the solution set of simultaneous linear
inequations may be an empty set or it may be the region bounded by the straight lines
corresponding to linear inequations or it may be an unbounded region with straight line
boundaries.

ILLUSTRATIVE EXAMPLES

Type} ON FINDING THE SOLUTION SET REPRESENTED BY SIMULTANEOUS LINEAR INEQUATIONS


EXAMPLE 1 Exhibit graphically the solution set of the linear inequations
3x + 4y < 12, 4x + 3y < 12, x > 0, y > 0
SOLUTION Converting the inequations into equations, the inequations reduce to
3x + 4y = 12, 4x + 3y = 12, x = 0 and y = 0.
6.24
APPLIED MATHEMATICS-XH

Region Represented bp 3x + 4y <12: The line 3a: + 4y =12 meets the coordinate axes at A (4, 0)
and B (0, 3). Draw a thick line joining A and B. We find that (0, 0) satisfies inequation
3.r + 4y < 12. So, the portion containing the origin represents the solution set of the inequation
3a: + 4y < 12.
Region Represented by 4x +3ij< 12: The line 4.v + 3y = 12 meets the .t and y-axes at Ai{3,0) and
Bi(0, 4) respectively. Join these two points by a thick line. Clearly, the region containing the
origin is represented by the inequation 4a: + 3y < 12.
Region Represented by x >0 and y > 0: Clearly, a' > 0 and y £ 0 represent the first quadrant.
Hence, the shaded region given in Fig. 6.27 represents the solution set of the given linear
inequations.
y

Bi(0,4)
4r + 3y = 12

w
(0,3) R
3x + 4y=12

X'
o

F lo
(3,0) /I
(4,0)
h X

e
Fre
r

Fig. 6.27
for
' '.iPLU: Exhibit graphically the solution set of the linear inequatiotis
r
x + y<5, 4a: + y>4, A: + 5y>5, a:<4, y<3
You
s
ook

SOLUTION Converting the inequations into equations, we obtain


AT + y=5, 4A: + y = 4, a:+5y=5, a: = 4, y = 3
eB

Region Represented by x + y <5: Tlie line a: + y = 5 meets the coordinate axes at/I(5,0) and B(0,5)
respectively. Join these points by a thick line. Clearly, (0,0) satisfies the inequality a: + y < 5. So,
our
ad

the portion containing the origin represents the solution set of the mequation .r + y <5.
Region Represented bij4x + y>4: The line 4a: + y = 4 meets the coordinate axes at Ai{l, 0) and
Bj (0,4) respectively. Join these points by a thick line. Clearly, (0, 0) does not satisfy the
dY
Re

inequation 4.v + y > 4. So, the portion not containing the origin is represented by the inequation
4.t + y > 4.
Fin

Region Represented by a: + 5y > 5: The line x + 5y > 5 meets the coordinate axes at A (5, 0) and
B2 (0,1) respectively. Join these two points by a thick line. We find that (0,0) does not satisfy
the inequation x + 5y > 5. So, the portion not containing the origin is represented by the given
inequation.

Region Represented by x < 4: Clearly, x = 4 is a line parallel to y-axis at a distance of 4 units from
the origin. Since (0,0) satisfies the inequation x < 4. So, the portion lying on the left side of x = 4is
the region represented by x < 4.
Region Represented f?y y < 3: Clearly, y = 3 is a line parallel to x-axis at a distance 3 from it. Since
(0, 0) satisfies y < 3. So, the portion containing the origin is represented by the given inequation.
The common region of the above five regions represents the solution set of the given linear
constaints as shown in Fig. 6.28.
6.25
NUMERICAL INEQUALITIES AND LINEAR INEQUATIONS

w
EXAMPLES Drazv the diagram of the solution set of the linear inequations 3x + 4y >12, y > 1, x>0.

not satisfy the inequahon 3.y + 4y > 12. So, the portion
by the inequation 3.v + 4y > 12.
F lo
SOLUTION Converting the inequations into equations, we get 3.t + 4y = 12, y = 1, =0
Region Represented bif 3.‘'i: + 4y >12; The line 3x- - 4y = 12 meets the coordinate axes at A (4,0)
and B(0,3) joining these points by a thick line we get the graph of 3.v + 4y = 12. Since (0, 0) does
not containing the origin is represented
for F
ree
Region Represented by y > I; The line y = 1 is parallel to x-axis at a unit distance from it. Since (0, 0)
does not satisfy the inequation y > 1. So, the region lying above the line y = 1 is represented by
Your

y>l.
oks

Region Represented byx> 0; Clearly, y > 0 represents the region lying on the right side of y-axis.
eBo

The solution set of the given linear constraints is the intersection of the above regions as
shown in Fig. 6.29.
ad

y
our

B (0,3)'
Re
dY
Fin

X’
0

Type U ON FINDING THE LINEAR INEQUATIONS WHEN THEIR SOLUTION SET IS GIVEN
EXAMPLE 4 Find the linear inequations for which the shaded area in Fig. 6.30 is the solution set.
SOLUTION Consider the line y + 2y = 8. We observe that the shaded region and the origin are
on the same side of the line x + 2y = 8 and (0, 0) satisfies the linear constraint x + 2y < 8. So, we
must have one inequations as x + 2y < 8.
6.26
APPLIED MATHEMATICS-XII

ow
e
re
Now, consider the line 2x + y = 2. We find that the shaded region and the origin are on the

Flr
F
opposite sides of the line 2x + y = 2 and (0, 0) does not satisfy the inequation 2.r + i/ > 2. So, the
second inequations is 2:r + y > 2.
ou
Finally, consider the line .r - y = 1. We observe that the shaded region and the origin are on the
same side of the line x - y = 1. We observe that the shaded region and the origin are on the same

sr
side of the line x - y = \ and (0,0) satisfies :r -y < 1. So, the third constraint is x~y < 1.
fo
k
We also notice that the shaded region is above .r-axis and is on the right side of y-axis. So, we
must have a: > 0 and y > 0.
oo
Thus, the linear inequations corresponding to the given solution set are
Y

x+2y<8, 2.r + y>2, x-y<l, x > 0, y>0


reB

1-XAMPLE 5 Find the linear inequations for which the shaded region in Fig. 6.31 is the solution set.
SOLUTION Consider the line 2x + 3y = 3. We observe that the shaded region and the origin lie
uY

on the opposite side of this line and {0, 0) satisfies 2x + 3y < 3. Therefore, we must have
2x + 3y > 3 as the linear inequations corresponding to the line 2x + 3y = 3.
Consider the line 3x + 4y = 18. Clearly, the shaded region and the origin lie on the same side of
ad
do

this line and (0, 0) satisfies the inequation 3x + 4y <18. So, we must have 3x + 4y <18 as the
linear inequations corresponding to 3x + 4y = 18.
in

Consider the line x - 6y = 3. It is evident from the figure that the origin and the shaded region lie
Re

on the same side of this line and (0, 0) satisfies x - 6y < 3. So, x - 6y < 3 is the corresponding
F

inequations.
6.27
NUMERICAL INEQUALITIES AND LINEAR INEQUATIONS

Consider the line -7.y + 4i/ =14. We find that the shaded region and the origin are on the same
side of this line and (0, 0) satisfies the inequations -7x+ 4i/ <14. So, the corresponding linear
inequations is - lx + 4y <14.
Also, the shaded region is in first quadrant only. So, we must have y > 0 and i/ > 0.
Thus, the linear inequations comprising the given solution set are
2y+3i/>3, 3y + 4v<18, -7y + 4i/<14, y-6i/<3, y>0, y>0
EXERCISE 6.6

1. Solve the following systems of linear inequations graphically:


(i) 2y + 3y < 6, 3y + 2i/ < 6, y > 0, y > 0 fii) 2y + 3i/ < 6, y + 4y < 4, y > 0, y > 0
(hi) Y-y<l, Y+2y<8, 2Y + y>2, y>0, y>0
(iv) Y + y>l, 7Y + 9y<63, y<6, y<5, y>0, y>0
(v) 2y + 3y < 35, y > 3, y > 2, y > 0, y > 0
2. Show that the solution set of the following linear inequations is empty set:

w
(i) Y-2y>0,2Y-y<-2,Y>0,y>0 (ii) Y + 2y<3, 3.Y + 4y >12,y>l, Y>0,y>0
3. Find the linear inequations for which the shaded area in Fig. 6.32 is the solution set. Draw

F lo
the diagram of the solution set of the linear inequations:
y

e
Fre
/ /
for
r
You

X
oks

X'
o
eBo

■3'
/ 6
r
our
ad

Fig. 6.32

4. Find the linear inequations for which the solution set is the shaded region given in Fig. 6.33.
5. Show that the solution set of the following linear in equations is an unbounded set:
dY
Re

Y + y>9, 3y+i/>12, y>0, y>0.


Fin

6. Solve the following systems of inequations graphically:


(i) 2y + y > 8, y + 2y > 8, y + y < 6
(ii) 12y + 12y < 840, 3y + 6y < 300, 8y + 4y < 480 Y > 0, y > 0
(hi) Y + 2y < 40, 3y + y > 30, 4y + 3y > 60, Y>0, y>0
(iv)5.Y + y>10, 2Y + 2y>12, Y + 4y>12, Y>0, y>0
7. Show that the following system of linear equations has no solution:
x + 2y<3, 3y + 4i/>12, y > 0, y>l.
8. Show that the solution set of the following system of linear inequalities is an unbounded
region2.Y + y>8, Y + 2y>10, y>0, y^O.
ANSWERS

3. 2y + 3y > 6, 4y + 6y < 24, 3y + 2y < 3, Y - 2y < 2, y > 0, y > 0,


4. Y + y < 4, y < 3, y < 3, y + 5y > 4, 6y + 2y > 8, y > 0, y > 0
6.28
APPLIED MATHEMATICS-XH

MULTIPLE CHOICE QUESTIONS (MCOs)


Mark the correct alternative in each of the following:
1- IfA'<7,then
(a) -X < -7 (b) -;c<-7 (c) -x>-7 (d) -x>~7
2. If-3i- + 17 <-13,then
(a) .r e(10, qo) (b) X € [10, oo) (c) Are(-00,10] (d) .Y e [-10,10)
3. Given that x, y and b are real numbers and x <y, b >0, then
(a) — < ^
^ ' b b (b)f.f
b b b b (d) fb .fb

w
4. If Y is a real number and | y| < 5, then
(a) y>5 (b) -5 < Y <5 (c) y<-5 (d) -5<y<5

lo
5. If Y and a are real numbers such that n>0 and | y| ><?, then

e
(a) Ye(-fl, go) (b) Y e [- CO, a] (c) xe{-n, a) (d) Y e(-oo, -a) u(a, oo)

re
6. If|Y-l| >5, then
rF
F
(a) yg(-4, 6) (b) yg[-4, 6]
(c) Y €(-co,-4) u(6, oo) (d) Y € (- 00, - 4) u [6, oo)

r
If|Y + 2| <9, then fo
u
(a) yg(-7,11) (b) Y€[-11,7]
ks
Yo
(C) Y €(-00,-7) U(ll, co) (d) Y e(-oo, -7) u[ll, oo)
oo

The inequality representing the following graph is


(a) |y| <3 (b) ]y|<3
eB

(c) |y|>3 (d) |y|>3


Y
ur
ad
Yo

X' X
O
d

0) (3/0)
Re
in
F

Y'

Fig. 6.34

The linear inequality representing the solution set given in Fig. 6.35 is
(a) |y| <5 (b) |y|>5 (c)|y1>5 (d) |y|<5
- CD
-5 5 CO

Fig. 6.35

The solution set of the inequation | y + 2| < 5 is


(a) (-7,5) (b) [-7, 3] (c) [-5,5] id) (-7, 3)
If
|y-2|
> 0, then
y-2

(a) Y € [2, co) (b) Y€(2, x) (c) YG(-X, 2) (d) Y€(-x,2]


NUMERICAL INEQUALITIES AND LINEAR INEQUATIONS 6.29

12. If|:c+31>10,then
(a) xe{-13,7] (b) :re(-13,7)
(c) X e(-oo, -13)u(7, oo) (d) X e(-QO,-13]u[7, oo)
13. Solution of a linear inequality in variable .r is represented on the number line as shown in
Fig. 6.36. The solution can also be described as
- 00 9 CO

Fig. 6.36

(a) .V e (- CO, 5) (b) ;t€(-x,5] (c) X e [5, x) (d) A'e(5, x)


14. The shaded part of the number line in Fig. 6.37 can also be represented as

w
f9 9\ 9
(a) a: e —, x (b) (c) :r e - x. (d) .ve -X,-
I 2' 2) 2

Flo
— oo 5 00

e
Fig. 6.37

re
15. The shaded part of the number line in Fig. 6. 38 can also be described as

F
ur
— oo
1 2 CO

r
Fig. 6.38 fo
(a) (-X, 1) u (2, x) (b) (-X, 1] u[2, x)
ks
Yo
(c) a, 2) (d) [1, 2]
oo

''FRS
eB

1- (c) (a) (a) . (b) (d) (c) (b) (b)


9. (c) 10. (b) II. (b) (d) 1? (d) (b) : -● (a)
ur

ri! i IN :HE BLANKS lESr:- Jl


ad
Yo

1. If X >-3, then 'A+ 5.... 2.

1 If -Y < - 4, then 2x

3. If —^ < 0, then .Y
d
Re

2.
in

.y-2

- If I -Y -11 < 2 then -1. .Y < 3.


F

5
If I 3.Y - 71 > 2, then .y — or, X .3.
3

6. If-4.Y ^ 2, then .y. -3.


3.Y
If- < - 3, then .y. 4.
4

^ If.Y >y and 2 <0, then-.Y2 -yz.


The solution set of the inequation | .y + 11 < 3 is
I! ■. The solution set of the inequation | .y + 21 > 5 is .
-Y-3
II. If > 0, then .Y belongs to the interval
x-31.
.y| +1
The solution set of the inequation < 0 is.
x| -1
6.30 APPLIED MATHEMATICS-XII

ANSWERS
1. > 2. > 3. < 4. <, < 5. <, > 6. < 7. > 8. >

9. (-4,2) 10. (-00,-7) u (3, co) 11. (3,oo) 12. (-1,1)

VERY SHORT ANSWER QUESTIONS (VSAOs)


Answer each of the following questions in one word or one sentence or as per exact requirement of the
question:
j
X‘“
1. Write the solution set of the inequation >0.

2. Write the solution set of the inequation .r + — > 2.

ow
3. Write the set of values of .r satisfying the inequation (.v -2x + 1) {.r - 4) > 0.
4. Write the solution set of the equation | 2 - .t | = :r - 2.
5. Write the set of values of a: satisfying | x -11 < 3 and ] -11 < 1.

e
6. Write the solution set of the inequation — - 2 <4.

re
rFl X

x + 2 1

F
7. Write the number of integral solutions of
x^ +1 2
.r + 2

r
8. Write the set of values of x satisfying the inequations 5:r + 2 < 3.t + 8 and <4.
fo
ou
.r-1

1
ks
9. Write the solution set of ;r + - > 2.
a:
oo

10. Write the solution set of the inequation | a: -1| > | a: - 31.
Y

ANSWERS
B

1. [2,«) 2. (0,00) 3. (-00, 4) 4. (2,cc)


e

5. [2, 4] 6. (-co,-l/2)u(l/6,oo) 7. 3
ur

8. (-oo,l)u(2, 3) 9. K-{-1,0,1} 10. [2, oo)


ad
Yo
d
Re
in
F
CHAPTER z
ALGEBRA OF MATRICES

7.1 MATRIX

ow
DEFINITION A set of mil numbers (real or imaginanj) arranged in the form of a rectangular array ofm
roxos and n columns is called an m x n matrix (to be read as'm by n' matrix).
An m X n matrix is usually written as
‘^12 ‘^13 ■■■ ■■■

e
^21 ^22 '^23 ●●● ^2/ ... U2,,

re
A =

Fl
F
‘^2 ^/3 ●●● ●●●
ur
r
a
A;fl %i2 ^it3 Ojjij ●●● mn

In compact form the above matrix is represented by A = x,i or, A =[Ojj].


fo
ks
The numbers ai2/--- t^tc. are known as the elements of the matrix A. The element belongs
Yo

the matrix A = [Ojj]. Thus, in the


oo

to row and column and is called the (/, element of


element Oj; the first subscript i always denotes the number of row and the second subscript j,
eB

number of column in which the element occurs.


Following are some examples of matrices:
ur

2 1 -l1
(i) A = 1^2 is a matrix having 2 rows and 3 columns and so it is a matrix of order 2 X 3
ad
Yo

such that = 2, (7-^2 = » ^21 =1/^22 ~ ^23 ~


d

sin Y COS X
(ii) B = is a matrix having 2 rows and 2 columns and so it is a matrix of order
Re

cos -Y - sm X °
in

= sin Y,
2x2 such that b 11 “ cos Y, bii - cos y, &22 “ “ sin y.
F

NOTE It is to note here that to define a matrix zue must define its order and its elements either by a general
formula (See illustration given below) or separately.
ILLUSTRATION 1
Construct rt 3 x 4 matrix A = [ojj] whose elements are given by
(i) +y (ii) Oij=i-j
SOLUTION (i) We have.
<?11 ai2 «13 <^14
A =
^21 ^22 ^23 ^24 ' '^here Ojj = i + j.
^31 ^32 ^33 ^^34 .

.'. = 1 +1 = 2, <7-j^2 =1 + 2 = 3, =1 + 3= 4, (7j,j=1 + 4=5.


Similarly, ^21 = 3, <122“^' ^23 (724“^ ^^31 =4/ ^32“^' ^33=6, ^34 =7.
'2345'
Hence, A = 3 4 5 6
4 5 6 7
7.2 APPLIED MATHEMATICS-XII

(ii) Proceeding as above, we obtain


0 -1 -2 -3'
1 0 -1 -2 .
2 1 0 -1

ILLUSTRATION 2 The unit cost of transportation of commodities from each of three factories to each of
the four zvarehouscs in hurdends of^ can be represented by a 3 x 4 matrix asfolloios:
Warehouse

Factory Wj W2 W3 W4
25 18 23 16

h 27 24 30 41
18 19 21 20
h

ILLUSTRATION 3 Processing time in hours of three products P, Q and R in fioo dijferezit production

w
departments D| and D2 may be represented by a 3x2 matrix as folloivs.

Product

P
Department
Dj
12
D2
10"
F lo
e
Fre
Q 9 6
R 3 4
for
I [.LUSTRATION 4 Vitamin A, B and C contents in tzvo types foods Fi and F2 may be represented by a
2x3 matrix as given belozv:
r
Vitamin
You
oks

Food A B C
eBo

■5 7 12'

Pi 4 8 10
ad
our

We may represent it as a 3 x 2 matrix as given below :


Food
Vitamin Pi Pi
Re

A ' 5 4'
dY

B 7 8
Fin

C 12 10

7.2 TYPES OF MATRICES

ROW MATRIX A matrix having only one rozv is called a rozu-matrix or a rozv-vector.
For example, /I = [1 2 -1 - 2] is a row matrix of order 1x4.
COLUMN MATRIX A matrix hazdng only one column is called a column matrix or a column-vector.
3
1
2
For example, i4 = 2 and B - g are column-matrices of order 3x1 and 4x1 respectively.
-1
4

SQUARE MATRIX A matrix in ivhich the number of rozvs is equal to the number of columns, say n, is
called a square matrix of order n.
ALGEBRA OF MATRICES 7.3

A square matrix of order n is also called a «-rowed square matrix. The elements aij of a square
matrix A for which i = j i.e. the elements rtn/^22' nn are called the diagonal
elements and the line along which they lie is called the principal diagonal or leading diagonal of
the matrix.

2 1 -1

For example, the matrix 3 -2 5 is square matrix of order 3 in which the diagonal
1 5-3

elements are 2,-2 and - 3.

DIAGONAL MATRIX A square matrix A = a diagonal matrix if all the elements, except
those in the leading diagonal, are zero i.e. Ojj - 0 for all i * j.
A diagonal matrix of order nxn having di, d2,..., djj as diagonal elements is denoted by

w
diag [dp d2,..., ●
10 0

is a diagonal matrix, to be denoted by A = diag [1, 2, 3].

Flo
For example, the matrix A = 0 2 0
0 0 3

e
re
SCALAR MATRIX A square matrix A = [fl,-,jpn
] xji is called a scalar matrix if

F
(i) Ojj = 0 for all i ^ j and, (ii) Ojj = c for all i, where ci^O.
In other words, a diagonal matrix in which all the diagonal elements are equal is called the scalar
ur
r
matrix. fo
1 - 21 0 0
2 0
ks
For example, the matrices A = , B = 0 1 -2/ 0 are scalar matrices of
0 2
Yo
0 0 1 -2/
oo

orders 2 and 3 respectively.


B

IDENTITY OR UNIT MATRI;: A square matrix A =[fly] is called an identity or unit matrix if
n xn
re

(i) Oij = 0 for all i ^ j and, (ii) a^i ^ 1 for all i


In other words, a square matrix each of whose diagonal element is unity and each of whose
u
ad
Yo

non-diagonal elements is equal to zero is called an identity or unit matrix.


The identity matrix of order n is denoted by
d

1 0 0
Re

1 0
,I^= 0 1 0 are identity matrices of orders 2 and 3
in

For example, the matrices I2 = 0 1


0 0 1
F

respectively.
NULL MATRIX A matrix whose all elements are zero is called a null matrix or a zero matrix.
0 0 0 0 0
For example. 0 0
and
0 0 0
are null matrices of orders 2x2 and 2x3 respectively.

UPPER TRIANGULAR MATRIX A square matrix A = [aij] is called an upper triangular matrix if ay = 0for
all i > j.
Thus, in an upper triangular matrix, all elements below the main diagonal are zero.
'12 4 3
0 5 1 3
For example, A = 0 0 2 9 is an upper triangular matrix.
0 0 0 5
7.4 APPLIED MATHEMATICS-XII

LOWER TRIANGULAR MATRIX A square matrix A = laq} is called a lower triangular matrix if aq = Ofor
all i < j.
Thus, in a lower triangular matrix, all elements above the main diagonal are zero.
■ 2 0 0
For example, = 3 2 0 is a lower triangular matrix of order 3. A triangular matrix A = [Ojj]
[453
n X n is called a strictly triangular iff ajj = 0 for all i = 1,2,n.
7.3 EQUALITY OF MATRICES

DEFINITION Tzuo matrices A = x„ nndB=:[bii],^s are equal if


(i) m =;● i.e. the number of rows in A equals the number of rows in B
(ii) n = s i.e. the number of columns in A equals the number of columns in B
(iii) Ojj =bjjfori =1,2, ...,mand j = 2,2,..., n.
If two matrices A and B are equal, we write A = B, otherwise we write A B.

The matrices A =

Matrices
[0 0
0 0
3
X

and
-1
2

\j 5

[ 0 0 0’
0 0
1

0
and B =
3
-1
1
F low2
0 5
1
1 ■

2
are equal if a: =-1,1/= 0 and 2 =4.

are not equal, because their orders are not same.


for Fre
ILLUSTRATION 1 If . ^ ~^ ^^
-1

’ yix -y 3 2 + re 0 ^ , find x, y, z, w.
13

SOLUTION Since the corresponding elements of two equal matrices are equal. Therefore,
-Y -1/ 2 .V + 2 -1 5
2 -Y -y 3 2 4- w 0 13
Your
eBo ks

x-y = -\, 2.y+2=5, 2.y-_i/=0, 32 + ze=13.


ad

Solving the equations x-y = -\ and 2y-i/ = 0 as simultaneous linear equations, we get
our

.Y = 1, ;/ = 2.
Now putting .Y = 1 in 2 Y + 2 = 5, we get 2 = 3. Substituting 2 = 3 in 3 2 + w' = 13, we obtain rr> = 4.
Thus, Y=l, i/ = 2, z = 3andrt;=4.
Re

ILLUSTRATION 2 Find the values ofx, y, z and a znliich satisfy the matrix equation
Y + 3 2 i/ + Y ~ 0 -7
Find Y

2-1 4a-6 3 2a

SOLUTION The corresponding elements of tw'o equal matrices are equal.


>+3 2i/ + Y]_'“0 -7’’
2-1 4i7-6 3 2a

.y+3=0, 2i/h-y = -7, 2-1 = 3 and 4a-6=2a.


Solving these equations, we gel: a =3, y=-3, i/ = -2, 2=4.

ILLUSTRATIVE EXAMPLES

EXAMPLE 1 A matrix has 12 elements. What are the possible orders it can have?
SOLUTION We know that if a matrix is of order m x n , then it has mn elements. Therefore, to
find all possible orders of a matrix with 12 elements, we will have to find all ordered pairs {a, b)
such that a and b are factors of 12. Clearly, all possible ordered pairs of this type are :
7.5
ALGEBRA OF MATRICES

(1,12), (12,1), (3, 4), (4, 3), (2, 6), (6, 2)


Hence, possible orders of the matrix are;
1 X 12,12x 1, 3x 4, 4x 3, 2x 6 and 6 x 2.
EXAMPLE 2 f/A = [Ojj] is a matrix given by
■4-2 1 3'
A=[<Ji:]= 5 7 9 6
[21 15 18 -25
write the order of A and find the elements r?24 / ^^34 ● ^^^ow that ^32= «23 '^24 ●
SOLUTION We observe that there are 3 rows and 4 columns in matrix .A. Therefore, it is of order
3x4.

ow
The element lying at the intersection of 2nd row and fourth column is 6.
a2i - 6
Similarly, the element lying at the intersection of third row and fourth column is — 25.
^34 = -25.

e
^32 ~ ^23 ~ ^ and ^24 ~ ^

re
«32 =

Fl
=9 + 6 = «23 + «24 ●

F
(i + 2;)^
EXAMPLES Construct a 2 x 2 matrix A =[aij]ivhose elemeitts are given by a^j
ur 2

r
2

SOLUTION Here Ojj = , 1 <i<2 and 1 <j<2. fo


ks
(1+2x1)2 _ (1 + 2)^ _ 9 (1 + 2 X 2f
Yo
25
a-12 -
oo

2 2 2' 0
2

(2+ 2x1)2 (2 + 2x2)^


eB

= 8 and ^22 “ = 18
^21 - 2
2
■9 25 1
ur

'^11 '’12
yl = 2 2
^21 ^22
ad

8 18
Yo

EXAMPLE 4 Construct (72x3 matrix A = [Ojj] whose elements are given by oij - 7^.
d
Re
in

SOLUTION We have, Ojj - -i + :,


l
1 < r < 2 and 1 ^ ^ 3. Therefore,
1
F

1 1 1
(7ii=0, fli2=-
3' ^13=--, (721- — , '^22-0 ^23“ g-
-
0 -- 1 _r
A = 3 2

1 0-i
3 5 J
X -y 2z + w 5 3
EXAMPLE 5 Find X, y, z and it» such that
2x-\j 2.v+iyJ~[12 15
SOLUTION We know tliat the corresponding elements of two equal matrices are equal.
.T - 1/ 2z + w 5 3

2x-y 2x-^w 12 15

A* - j/=5, 22 + TP = 3, 2a‘-_i/=12 and 2x + ze=15


Solving X -y =5 and 2.v - 1/ =12as simultaneous linear equations, we get x^7, y=2.
Putting A- =7 in equation 2x + w =15, we getzp =1.
7.6
APPLIED MATHEMATICS-XII

Puttingw = 1 in 2z + w = 3, we getz =1.


Hence, a:=7,i/ = 2,z = 1 andzi?=l.
EXAMPLE 6
Consider the following information regarding the number of men and women workers in
three factories 1,11 and 111.
Men workers Women workers
I 30 25
II 25 31
III 27 26

Represent the above information in the form of 3x2 matrix. What does the entry in the third row and
second column represent?
SOLUTION The given information can be represented in the form of a 3 x 2 matrix as follows:
Men workers Women workers

w
I 30 25
II 25 31
III 27

F lo
The entry in third row and second column represents the number of women workers in
26

e
factory III.

Fre
a +b 2 6 2
EXAMPLE 7 If ,find the values of a and b.
5 ab 5 8
for
SOLUTION Tine corresponding elements of two equal matrices are equal.
r
a +b 2 6 2
You
oks

5 ab 5 8
eBo

=j> a + b = 6 and ab = S
8
a + - = 6
a
[●.● ab = B^b = S/a]
ad
our

a'^ + 8 = 6a => a^ -6a + 8 = 0 => {a -4) {a -2) = 0 => a =2, i-


Now, a = 2 and ab =8 => b = 4
and. a = 4 and ab =8 => b =2.
dY
Re

Hence, a = 2 and b =4, or a = 4 and b = 2.


Fin

EXAMPl.E 8 For what values of x and y are the following matrices equal?
A =
2x +1
0
3y
, B=
X + 3
0
y^ + 2]
-6

SOLUTION The corresponding elements of two equal matrices are equal. Therefore,
2x + l
3y 1 \x+3 \?-+2
0 y^-5y " 0 -6

2x' + l=x:+3, 3y = 1/^ + 2 and y^-5y=-6


X = 2, y^ - 3y + 2 = 0 and y^ - 5y + 6 = 0
x = 2,(y-l)(y-2)=0and (y-2)(y = 3)=0
-V - 2, y =1,2 and y = 2, 3
x = 2,y = 2 [v y =1, 2 andy = 2, 3 => y = 2]
7.7
ALGEBRA OF MATRICES

EXERCISE .

1. If a matrix has 8 elements, what are the possible orders it can have? What if it has 5
elements?
2 3-5 2 -1
then find
; HA=[aij]= 1 4 9 and B = \bjj] = -3 4
0 7-2 1 2

(i) ^22 ^21 (ii) i7i| + ^22 ^22


3. Let be a matrix of order 3 x 4. If Kj denotes the first row of A and C2 denotes its second
column, then determine the orders of matrices R-[ and Cj-
4. Construct a 2 x 3 matrix A =[Ojj] whose elements Ojj are given by;
(i) Oij = ixj (ii) Oij = 2i-j (hi) a^j = i + j (iv) aij

w
2

5. Construct a 2 x 2 matrix A =
[Ojj] whose elements Ojj are given by:
(i)<^

F lo
■ (ii)
2

2/-3; I

ee
(iii) Oij 2 (iv) Oij 2

Fr
6. Construct a 3 x 4 matrix A = [a^j] whose elements Ojj are given by:
^ |-3i + ;l
(i) =/ + ;● (ii) =/ (iii) Ojj = 2i (iv) Oij
for
ur
7. Construct a 4 x 3 matrix A = ] whose elements c^j are given by:
s

I
ook

(i) Ojj =2i + ]- (ii) Oij=-—r (iii) Oij =/


Yo

} 1+)
■ 3.r + 4y 2 ;r - 2y 2 2 4
eB

8. Find x, y, a and b, if a + b 2a -b -1 5 -5 -1 ■

X 3-r-y 3 2
our

9. If , find X, y, 2, co.
ad

2x + z 3y - CO 4 7

j:-y 2 1 4
10. If findx, y,2, co.
Y

2.r - y CO 0 5 '
Re
nd

AT + 3 2 + 4 2y-7 0 6 3y-2
11. If 4.r + 6 a -1 0 2a: 3 2c+ 2
Fi

b-3 3b z + 2c 21)+4 -21 0

Obtain the values of a, b, c, x, y and 2.


xy 4 '8 w'
12. If then find the values of a:, y, 2 and w.
2+6 A: + y 0 6 '

13. Give an example of


(i) a row matrix which is also a column matrix
(ii) a diagonal matrix which is not scalar
(iii) a triangular matrix.
14. The sales figure of two car dealers during January 2013 showed that dealer A sold 5 deluxe,
3 premium and 4 standard cars, while dealer B sold 7 deluxe, 2 premium and 3 standard
cars. Total sales over the 2 month period of January-February revealed that dealer A sold 8
deluxe 7 premium and 6 standard cars. In the same 2 month period, dealer B sold 10 deluxe,
5 premium and 7 standard cars. Write 2x3 matrices summarizing sales data for January
and 2-month period for each dealer.
7.8 APPLIED MATHEMATICS-XII

15. For what values of x and y are the following matrices equal?
2.Y + 1 2y X + 3
, 6 =
0
/-5y 0 -6

3x + 4 3
16. Find the values of x and y, if .r +10 + 2y
0 -4 0
y^-5yj
a+4 3b 2rt + 2 h^+2
17. Find the values of rt and b if A =B, where A - rB =
8 -6 8 b^-lO

ANSWERS

ow
1. (i) 1x8, 8 X 1,2x4, 4x2 (ii) 1 X 5,5 X 1
2. (i) 1 (ii) 20 3. 1 X 4, 3x 1
1 2 3 1 0-1 2 3 4 2 9/2 8
4. (i) 2 4 6
(ii)
3 2 1
(iii) 3 4 5
(iv)
9/2 8 25/2

e
0 1/2' 1/2 2'

re
2 9/2 1/2 9/2
5. (i)
9/2 8
(ii)
1/2 0

rFl
(iii) 0 2
(iv)
1/2 1

F
2 3 4 5 1 2 3 4 2 2 2 2 1 1/2 0 1/2
6. (i) 3 4 5 6 (ii) 1 2 3 4 (iii) 4 4 4 4 (iv) 5/2 2 3/2 1

r
ou
4 5 6 7 12 3 4 6 6 6
fo 6 4 7/2 3 5/2
ks
3 5/2 7/3 0 -1/3 -1/2 111'
6 5 14/3 1/3 0 -1/5 2 2 2
7. (i) (ii) (iii)
oo

9 15/2 7 1/2 1/5 0 3 3 3


Y

12 10 28/3 3/5 1/3 1/7 4 4 4


eB

8. .V = 2, y =-l, (7 = 0, =5
9. .V = 3, y = 7,2 = - 2, to = 14 10. -v = 1, y = 2,2 = 4, to = 5
r
ou

11. /7 = - 2, b = - 7, c = -1, .Y = - 3, y = - 5,2 = 2


Y
ad

12. .Y = 2, y = 4,2 = -6, w = 4 or .y = 4, y = 2,2 = -6, w = 4


‘20 0 4 3 5
d

13. (i) [5] (ii) 0 3 0 (iii) 0 7 1


in
Re

0 0 -1 0 0 2

Deluxe Premium Standard Deluxe Premium Standard


F

Dealer A 5 3 4 Dealer A 8 7 6
14.
Dealer B 7 2 3 ' Dealer B 10 5 7

15. A and B cannot be equal for any value of y. 16. .r = 3,y = l 17. a=^2,b=2

7.4 ADDITION OF MATRICES

DEFINITION Let A, B be tivo matrices, each of order m x it Then their smn A + B is a matrix of order
in X n and is obtained by adding the corresponding elements of A and B.
Thus, if A -[OjAjjj and B =[bij]„, are two matrices of the same order, their sum A + B is
defined to be the matrix of order m x n such that

(A + B)ij = Ojj + bij for i — 1,2,..., m and j = 1, 2,..., n


NOTE The sum of two matrices is defined only zuhen they are of the same order.
12 3 6 5 4
If A , B = , then
4 5 6 3 2 1
ALGEBRA OF MATRICES 7.9

1 + 6 2+5 3 + 4 7 7 7
A + B =
4+ 3 5 + 2 6 + 1 7 7 7

-1 2 1
1 2 3
HA = ,B = 3 2 1 , then A + Bis not defined, because A and B are not of the
4 5 6
2 5-2

same order.

For the following pairs of matrices v4 + B is not defined because they are of different orders:
“1 2 ■
0 0 5
,B= ",3
1 -1
(ii) 71 = , B= 3 -1
(i) /I = 2 0 1 -1 0
4 5

7.4.1 PROPERTIES OF MATRIX ADDITION

w
THEOREM 1 (Commutativity) If A and B are two m x n matrices, then A + B = B A. i.e. matrix
addition is commutative.

PROOF LetA =
such that

F lo
B =[/j,y]be twom x nmatrices. Then,/l + Band B+A botharem x ;tmatrices

ee
(A + B),y = rt,y + bjj [By definition of addition]

Fr
(A + B),y = bjj + rt,y [By commutativity of addition of numbers]
(A + B),y = (B + A)iy [By definition of addition]
for
(7l + 6),y = (B + 7l),y for i = 1, 2,m and j = 1, 2,n
ur
Thus, A + B and B + A are two matrices such that their orders are same and the corresponding
oks

elements are equal. Hence, A + B = B + A.


Yo

Q.E.D.
o
eB

NOTE To prove that two matrices are equal it is required to proi’e that their orders are same and the
corresponding elements are equal.
THEOREM 2 (Associativity) If A, B, C are three matrices of the same order, then
our
ad

(A + B) + C = A + (B + C) i.e. matrix addition is associative.


PROOF Proceed as in theorem 1.
Y

THEOREM 3 (Existence of Identity) The null matrix is the identity element for matrix addition,
Re
nd

I.e. A + 0 = A=^0 + A.
Fi

PROOF Do yourself.
THEOREM 4 (Existence of Inverse) For every matrix A = Ky],„x« there exists a matrix [-Ojj] m x;i ’

denoted by - A, such that A+ {-A) = O = (-A) + /l.


PROOF Do yourself.
The matrix - A = [- i^y] m XII is called the additive inverse of the matrix A = [rtjy] m x/j-

ri -2 4 2 -4 -3
^ , then (- A) =
-1
IfA= 2 5 7 -4 -2 -5 -7 4 ●

THEOREM 5 (Cancellation laws) If A, B, C are matrices of the same order, then


A + B= A+ C=>B=C [ Left cancellation lazul
and, B + A= C + A=>B=^C IRight cancellation law}
PROOF Do yourself.
7.10 APPLIED MATHEMATtCS-XII

7.5 MULTIPLICATION OF A MATRIX BY A SCALAR (SCALAR MULTIPLICATION)


DEFINITION Let A = [cijj] be an mx n matrix and k be any number called a scalar. Then the matrix
obtained by multiplying every element of A by k is called the scalar tnultiple of A by k and is denoted by kA.
Thus,
kA
[k Oij] m x;i

1 2 5 3 6 15

w
For example, if A = -2 3 4 , then 3 A = -6 9 12
1 2 -1 3 6 -3

6 2 3
1
‘3 1 3/2
If A = 2 3-2 , then — A - 1 3/2 -1

e
2
2 4 1 1 2 1/2

re
ro
7.r - PROPERTIES OF SCALAR ..1ULTI!'-1.\TIO::

Various properties of scalar multiplication are stated and proved in the following theorem.

F
If A -[Ojj]„} , B - [bij]ffj xH are two matrices and k, I are scalars, then
oF
ul
(i) k(A+B)=kA + kB (ii) {k + [)A = k A + I A
(iii) (kl)A = k{lA) = l{kA) (iv) i-k)A = - (kA) = k{~A)

sr
(v) 1 A = A (Vi) (-l)A = - A

ko
Do yourself.
PlA':
of
For two matrices A and B of the same order, the subtraction of matrix B from matrix A is
o
Y
denoted by A - B and is defined as A - B = A + (- B).
B

-3 2 1 3 5 -2
For example, if A = and B = , then
Y

1 -4 7 -1 4 -2
er

-3 2 1 r-3 -5 2 -6 -3 3
u

A-B=A + (-S) = 1 -4 7
+
1 -4 2 2 -8 9
d
o
ad

2 3 4 3 0 5

f/A= 0 4 6,B=5 3 2 ,find3A-2B.


in

5 8 9 0 4 7
Re

SOLUTION We have, 3A - 2B = 3A + (- 2) B
F

6 9 12 ■ -6 0-10 0 9 2
3A-2B = 0 12 18 + -10 -6 -4 -10 6 14
15 24 27 0 -8 -14 15 16 13

ILLUSTRATIVE EXAMPLES

OM ADfinCt' AND SllRTRACTIC:- O" fvlATRiCES AND MULTiPUCATION OF A


^’^TR!X BY SCALAR

2 3 -5 0 5 1
e: ● 1 !
- ■ //A = 1 2 ^ and B ~ -2 7 3' ifnd A + B and A - B.
SOLUTION Clearly, A and B both are matrices of the same order 2x3. So, A + B and A - B
both are defined.
Now,
'2 3 -5 0 5 1 2+0 3+5 -5+1 2 8-4'
A + B = +
1 2 -1 -2 7 3 1-2 2+7 -1 + 3 1 9 2
7.11
ALGEBRA OF MATRICES

2 3-5 ■ 0 5 1 _ '2 3 -5 0 -5 -1
1 +(-l)
+
and. A-B = A+ {-B) = 1 2 -27 3 " 1 2-1 2 -7 -3

2 + 0 3-5 -5-1 2 -2 -6
1 + 2 2-7 -1-3 3 -5 -4

1 4
I and B=
2
3A - 2B.
EXAMPLE 2 1/A = 3 7 2

SOLUTION Clearly,
6 -3 -2 -8
3>1 =
9 3
and, (-2)8 = -14 -4

6 -3 -2 -8 6 + (- 2) - 3 + (- 8)
3A-2B = 3/1 +(-2) B = 9 3
+
-14 -4 9+ (-14) 3 +(-4)

w
4 -11 ■
-5 -1

F lo
EXAMPLES IfA=diag(l -1 2) and B = diag {2 3 -1),find A + B, 3A + 4B.
SOLUTION We have,

ee
’1 0 0’ 2 0 0

Fr
71 = 0-1 0 and, B = 0 3 0
0 0 2 0 0-1
for
ur
1 0 0 2 0 0 3 0 0
A + B = 0-10 + 0 3 0 0 2 0 = diag (3 2 1)
0 0-1 0 0 1
s
0 0 2
ok
Yo

3 0 0 8 0 0 11 0 0
o

and, 3A+4B = 0 -3 0 0 12 0 0 9 0 = diag (11 9 2)


eB

0 0 6 0 0-4 0 0 2
r

UEM.AHK !t is evident from the above example that if A = diag(i?;i A2 ^3 ■■■


ou
ad

B = diag(/?i ^2 b;^ ... b„) Then,


A + 8 = diag (rt-i + ^2+^2 ‘^3 ■■■
Y

7 O' 3 0
EXAMPU 1 Find X and Y, ifX + Y = andX-Y=
Re
nd

2 5 0 3 ●
Fi

SOLUTION We have,
7 0 3 0
x + y = and, X-y = 0 3
2 5

7 0 3 0
(x + y) + (x-y) = 2 5 0 3

7+3 0+0 10 0
2X =
2+0 5+3 2 8

1 10 0 5 0
X = -
2 2 8 " 1 4

7 0 3 0 7 0 -3 0
and, (x + y)-(x-y) = 2 5 0 3 2 5
+
0 -3

7-3 0+0 4 0
2y =
2 + 0 5-3 2 2
7.12 APPLIED MATHEMATICS-XII

1 4 0 2 0
y = -
2 2 2 1 1

5 0 2 0
Hence, X = and y -
1 4 1 1

2 3 3 -6
EXAMPLES
Find a matrix A, if A + -1 4 -3

2 3 3 -6
SOLUTION Let B = and C =
-1 4 -3 8 . Then, the given matrix equation is
A + B=C.

ow
Now, A + B = C

(A + B) + (-B) = C+(- B) [Adding -B on both sides]


A+{B + {-B)) = C + {-B) [Using associativity of matrix addition on LHS]
A+0 = C-B

e
A = C-B.

re
A =
-3
3 -6
8
+
-2
1
-3
-4

rFl
3-2
-3 + 1
-6-3
8-4 -2
1 -9
4

F
EXAMPLE 6 Find X, y,z, t, if 2
.V z
+ 3 10 -1
= 3 ■35]

r
2 4 6 ●
ou
SOLUTION The given matrix equation can be written as
fo
ks
2.r 2z 3 -3 r 9 15"
+
2y 2f 0 6 12 18
oo
Y

2a- + 3 2z - 3 9 15
eB

=>
2y 2t+6 12 18

2a + 3 = 9, 2z - 3 = 15, 2y = 12 and 2f + 6 = 18 [By definition of equality of addition]


r
ou

A = 3, z = 9, y = 6 and t = 6.
ad
Y

EXAMIM.E 7 Find non-zero values ofx satisfying the matrix equation:


f2A 21 [8 5a1 .x^ + 8 24
+ 2
d

A = 2
3 A 4 4a 10 6a
Re
in

SOLUTION We have,
F

2a 2 '8 5a A^ +8 24
A + 2 = 2
3 A 4 4a 10 6a

2a^ 2a 16 10a 2a^ +16 48


●) +
3a a“ 8a 20 12a

2a^+16 2a + 10a 2a^+16 48


3a + 8 A^ + 8a 20 12a

2a^ +16 = 2a^ +16, 2a + 10a = 48, 3a + 8 - 20 and x^ + 8a = 12a

12a = 48, 3a = 12 and x^ - 4a = 0


A =4 and A (a-4) =0
A = 4 and a = 0, 4
a =4
7.13
ALGEBRA OF MATRICES

-1 2 3 -2
EXAMPLES Find a matrix X such that 2A + B + X = 0, where A - 3 4 and, ^ = I 5 ■

SOLUTION We have,
2A + B + X = O

X = -2A-B

-1 2 3 -2
X = -2
3 4 1 5

ow
2 -4 -3 2 2-3 -4+2 -1 -2
=> X = +
-6 -8 -1 -5 -6-1 -8-5 " -7 -13

EXAMPLE 9 Find a matrix /I such that 2A - 3B + 5C = O, where


-2 2 0 2 0 -2'

e
B = and C =
3 1 4 7 1 6 ■

re
SOLUTION We have,

Frl
F
2A - 3B + 5C - O
=> 2A = 3B-5C
-2 2 0 2
ou0 -2'

sor
2A = 3 -5
3 1 4 7 1 6_
2A = ■-6 6 0] r-10 0 10' kf
.
9 3 12 J L
-35 -5 -30
oo
-6-10 6 +0 0 + 10"
2A =
Y

9-35 3-5 12- 30


B

-16 6 10
2A =
-26 -2 -18
re
oY

1 [-16 6 10 -8 3 5
u

A =
2 -26 -2 -18 -13 -1 -9 ●
ad

- 2'
-3 / =
d

EXAMPLE 10 Solve the matrix equation: 9 ■


2y
in
Re

SOLU'riON We have.
F

- 2'
2y 9
/
2
a: 3a: -2

6y 9
/
x^ - 3a: -2
9
y-61/
a:^ - 3a: = - 2 and i/^ - 6i/ = 9
- 3a: + 2 = 0 and -6y -9 = 0
6 ± ^36+36
(a:-1)(a:-2) = 0 and y = ■ 2

X = 1,2 and y = 3 ± 3 -J2.


7.14 APPLIED MATHEMATICS-XII

U ON APPLICATIONS OF MATRIX ADDITION AND SUBTRACTION


r.
':vc
IN BUSINESS AND
ECONOMICS

--- '● Two farmers Ram Kishan and Gurcharan Singh cultivate only three varities of rice namely
Basmati, Permal and Naura. The sale (in V of these varities of rice by both the farmers in the month of
September and October are given by the following matrices A and B
September sales (in
Basmati Permal Naura
10,000 20,000 30,000' Ram Kishan
A =
50,000 30,000 10,000 Gurcharan Singh
October sales (in

ow
Basmati Permal Naura
5,000 10,000 6,000 Ram Kishan
B =
20,000 10,000 10,000 Gurcharan Singh
Find:

e
(i) What were the combined sales in September and October for each farmer in each variety.

re
(ii) What was the change in sales from September to October?

rFl
(iii) If both farmers receive 2% profit on gross rupees sales, compute the profit for each farmer and for

F
each variety sold in October.
SOLUTION
(i) The combined sales in September and October is given hy A + B.

r
ou
Clearly,
Basmati Permal
fo
Naura
ks
10,000 + 5000 20,000+10,000 30,000+6000 Ram Kishan
A + B =
oo

50,000+20,000 30,000 + 10,000 10,000+10,000 Gurcharan Singh


Basmati Permal Naura
Y
B

15,000 30,000 36,000' Ram Kishan


A+B =
re

70,000 40,000 20,000 Gurcharan Singh


(ii) The change in sales from September to October is given by A-B.
ou
Y
ad

Clearly,
Basmati Permal Naura
d

10,000-5,000 20,000-10,000 30,000-6000 Ram Kishan


A~B =
in
Re

50,000-20,000 30,000-10,000 10,000-10,000 Gurcharan Singh


Basmati Permal Naura
F

'5,000 10,000 24,000 Ram Kishan


A-B =
30,000 20,000 0 Gurcharan Singh
(iii) The profit for each farmer and for each variety sold in October at the rate of 2% of gross sale
is given by
Basmati Permal Naura
'5,000 10,000 6,000 Ram Kishan
2% of B = — xB=0.02B = 0.02
100 20,000 10,000 10,000 Gurcharan Singh
Basmati Permal Naura
100 200 120 Ram Kishan
400 200 200 Gurcharan Singh
Thus, in October Ram Kishan receives ? 100, ? 200 and ? 120 as profit in the sale of each variety of
rice respectively, and Gurcharan Singh receives profit of ? 400, ? 200 and ^ 200 in each variety of
rice respectively.
7.15
ALGEBRA OF MATRICES

EXAMPi.h . The sales figures for two car dealers during January shoxved that dealer A sold 5 Luxury, 3
premium and 4 standard cars, while dealer B sold 7 luxury, 2 premium and 3 standard cars. Total sales
over 2-month period ofJanuary - February revealed that dealer A sold 8 luxury, 7 premium and 6 standard
cars. In the same 2-month period, dealer B sold 10 luxury, 5 premium and 7 standard cars. Write 2x3
matrices summarizing sales data for January and the 2-month periodfor each dealer. Hence, find the sales
in February for each year.
SOLUTION The sales for the month of January can be represented by the matrix
Luxury Premium Standard
Dealer A 5 3 4
P =
Dealer B 7 2 3

ow
The sales for the 2-month period can be represented by the matrix
Luxury Premium Standard
Dealer A 7 6
Q = 5 7
Dealer B 10

e
The sales for the month of February is equal to the sales for the 2-month period of

re
January-February minus the sales for the month of January. Thus, the sales in February is given

Frl
F
by Luxury Premium Standard
8 7 6 5 3 4 Dealer A 3 4 2
Q-P =
ou
or
10 5 7 7 2 3 Dealer B 3 3 4
kfs
In a certain city there are 25 colleges. Each college has 15 peons, 6 clerks, 2 cashiers, 1 typist
and 1 section officer. Express the given information as a column matrix. Using scalar multiplication, find
oo

the total number of posts of each kind in all the colleges.


SOLUTION The number of posts of each kind in a college can be represented by a 5 x 1
Y
B

column matrix given by


Peon 15
re

Clerk 6
oYu

A = Cashier 2
ad

Typist 1
d

Section officer 2
in

Using scalar multiplication, the total number of posts of each kind in all the college is
Re

15 Peon 375
F

6 Clerk 150

25A = 25 2 = Cashier 50

1 Typist 25

2 Section officer 50

EXERCISE 7.2

i Compute the following sums:


2 1 3 1 -2 3
[3 -2 -2 4
(ii) 0 3 5 2 6 1
W 1 4 + +
1 3
-1 2 5 0 -3 1

2 4 1 3 -2 5
2. LetA = ,B = -2 5
and C =
3 4
. Find each of the following:
3 2

(i) 2A-3B (ii) B-4C (iii) 3A- C (iv) 3A-2B+3C


7.16 APPLIED MATHEMATICS-XII

2 3 -10 2 -12 3
3. IfA = ,B = ,C = , find
5 7 3 4 1 2 1 0

(i) A + B and B+C (ii) 2B+3A and 3C -4B.


-10 2" 0-2 5 1 -5 2
4. Let A = , B = and C =
3 1 4 1 -3 1 6 0 -4 . Compute 2A - 3B + 4C.

5. If4=diag(2 -5 9),B = diag(l 1 - 4) and C = diag (-6 3 4), find


(i) 21-26 (ii) B + C-2A (iii) 2A+3B-5C
6. Given the matrices
‘2 11 9 7 -1 2-4 3
A = 3-10 , B = 3 5 4 and C = 1 -1 0

ow
0 2 4 2 1 6 9 4 5

Verify that (/4 + B) + C = /I + (B + C).


5 2 3 6
7. Find matrices X andY, ifX + y = and X - y =
0 9 0 -1

e
3 2 1 0

re
8. Find X, if Y =
1

Fl 4
and 2X + y =
-3 2 ●

F
6-6 0 3 2 5
9. Find matrices X and Y, if 2X - Y =
ur and X + 2Y =
-4 2 1 -2 1 -7

r
1 1 1 3 5 1
fo
ks
10. lfX-Y = 1 1 0 andX+Y = -1 1 4 , find X and Y.
Yo
1 0 0 11 8 0
oo

1 2 -1 9-14'
11- Find matrix 2l, if + 21 =
eB

0 4 9 -2 13'

9 1 1 5
12. IfA = , B= , find matrix C such thatSA + 3B + 2C is a null matrix.
7 8
ur

7 12
ad

2 -2 8 0
Yo

13. If A = 4 2 ,B = 4 -2 , find matrix X such tlrat 2A + 3X=5B.


-5 1 3 6
d
Re

1 -3 2 2 -1 -1
in

14. IfA = and, B =


2 0 2 1 0 - ^ , find the matrix C such that A + B + C is zero
F

matrix.

15. Find x, y satisfying the matrix equations


(i) Y" ?
3 -2 2 6 0 0
+
4 a: 6 1 0 -1 5 2x + 1/ 5

(ii) [x y + 2 2 - 3] + [y 4 5] = [4 9 12]
[21 [3
(iii) X ^ +y ^ + -11
=0

3 4 1 y 7 0
16. If2 g X
+
0 1 10 5 , find X and y.
10 2 1 2 3 4 4 10'
17. Find the value of X, a non-zero scalar, if X 3 4 5
+ 2
-1 -3 2 4 2 14 ●
7.17
ALGEBRA OF MATRICES

18. If X and Y are 2 x 2 matrices, then solve the following matrix equations for X and Y.
1 3 -2 2
2X+3Y =
4 0 ,3X+2Y= ^ -5

19. In a certain city there are 30 colleges. Each college has 15 peons, 6 clerks, 1 typist and 1
section officer. Express the given information as a column matrix. Using scalar
m
ultiplication, find the total number of posts of each kind in all the colleges.
20. Tl-ie monthly incomes of Aryan and Babban are in the ratio 3 : 4 and their monthly
expenditures are in the ratio 5 : 7. If each saves ? 15,000 per month, find their monthly
incomes using matrix method. This problem reflects which value?
ANSWERS

ow
3-16
1 2
1. (i) 2 7
(ii) 2 9 6
-1 -1 6

1 -1 9 -17 8 7 -2 21
(iii) (iv)

e
2. (i) (ii) -14 -11 6 2 22
12 -11

re
Fl -2 2 5 ’

F
3. (i) A + B does not exist, B + C = 5 5 1'
ur
1 6 1 2-14 -3

r
4.
(ii) 2B + 3A does not exit, 3C - 4B = -6 -13 -4 fo 27 n -11
ks
5. (i) diag(O -7 _17) (ii) diag (-9 14 - 18) (iii) diag{37 -22 -14)
Yo
4 4 1 -2 -1 -1 1
8.
oo

7. X = ,Y = -2 -1
0 4 0 5
eB

2 3 1 1 2 0
3 -2 1 0 2 2
9. X = ,Y = 10. X = 0 1 2 ,y = -10 2
-2 1 -1 0 0-3
6 4 0 5 4 0
ur
ad

12 4/3
Yo

8-3 5 -24 -10


11. 12. 13. 4 -14/3
-2 -3 -6 -28 -38
25/3 28/3
d

15. (i).v = | I (ii).t =!,}/ = 3,2=10


-3 4 -1
Re

(iii) x=l,y=2
in

14.
-3 0 -1
F

-2 0 2 1
16. x = 2, y=-8 17. X = 2 18. X = ,Y =
-1 -3 2 2

15 450
180
19. A = ?1 ,30A 30
20. ? 90,000, ? 120,000
1 30

H/NTS TO SELECTED PROBLEMS


7. We have.
5 2' '3 6
X + Y =
0 9 and X - Y = q
5 2 3 6

I and, (X + Y)-(X-y) =
5 2
(X+Y)+(X-Y) = 0 9
+
0 9 0 -1
7.18
APPLIED MATHEMATICS-Xll

=> 2X =
0 8
and 2Y = 0? -4
10

1 8 8 1 2-4 4 4
=> X = -
2 0 8
and y = —
2 0 10
=> X =
0 4
and y = 0^ -2
5

3 2 1 O'
8. We have, y = and 2X + y =
1 4

w
-3 2

3 2 1 0
2X +
1 4 -3 2

e
1 0 3 2 -2 -2 -2 -2
2X =
-3 2 1 4 -4 -2
x=l -1 -1
2 -4 -2 -2 -1

re
ro
7.7 MULTIPLICATION OF MATRICES

F
Let us first define the product of a row matrix and a column matrix.
oF
Let A = [rtj ^2 ... a„] be a row matrix and B =

ul be a column matrix. Then, we define

sr
ko
5
of
o
2 3 4] and B= t1 .Then,
Y
For example, if A = [1
B

-j
Y

5
er

AB=[1 -2 3 4] ^ = lx5+(-2)(-4) + 3xl + 4x(-2)=5 + 8+ 3- 8= S


u

-2
d
o
ad

Using the product of a row matrix and a column matrix, let us now define the multiplication
in

of any two matrices.


DEFINITION Tivo matrices A and B are conformable for the product AB if the number of columns in A
Re
F

(pre-multiplier) is same as the number of rows in B (post-multiplier).


Thusi/fA^[njj],„^„ andB=[bjj],^^p are two matrices of orders m x n and nx p respectively, then their
product AB is of orders mx p and is defined as
(AB)jj = [i^^ row of A) (/* column of B) for all / = !, 2,..., mand j = 1,2,..., p.

hj
hj
(AB)ij = [Oj^aj2 ... Oj„]
b^;
L J
n

(AB)ij +
... + 0j,b,j+...+ ni„b„j =^2^0i,b,i
If A and B are two matrices such that AB exists, then BA may or may not exist.
7.19
ALGEBRA OF MATRICES

2 1 3 1 -2

11 1 USlRA'nON I lfA = 3-2 1 andB= 2 1 , then A is a 3 3 matrix and B isa 3^ 2


-1 0 1 4 -3

matrix. Therefore, A and B are conformable for the product AB and it is of order 3x2 such that
{AB)ii = (First row of A) (First column of B)
1

(^B)ii =[213] 2 =2x1+1x2+3x4 = 16


4

{AB)i2 - (First row of A) (Second column of 6)


-2

(AB)i2 =[213] 1 = 2 x-2 + 1 X 1 + 3 x-3 =-12


-3

w
{AB)2\ = (Second row of A) (First column of B)
1

F lo
(AB)2i = [3-21] 2 = 3xl+(-2)x2 + lx4 = 3
4

ee
Similarly, we obtain

Fr
(A6)22 =-11,(^B)31 =3 and (ylB)32=-l.
'16 -12“ for
AB = 3 -11
ur
3 -1_
s
In this case BA does not exist, because the number of columns in B is not sajne as the number of
ook
Yo

rozvs in A.
eB

3
-2 3] ^ 2 . Find AB and BA and show that AB BA.
III 4 RA::
LetA= 2 2 andB= -14 -5
r
ou
ad

SOLUTION Here, A is a 2 x 3 matrix and B is a 3 x 2 matrix. So, AB exists and it is of order 2x2.
2 3'
2+2 + 12 3-4-15'
Y

1 -2 3 16 -16
-1 2
A6= 3 2 -1 6-2-4 9+4+5 0 18
Re
nd

4 -5

Again, B is a 3 x 2 matrix and A is a 2 x 3 matrix. So, BA exists and it is or order 3x3.


Fi

2 + 9 -4 + 6 6-3 11 2 3
2 ^ n -2 3' -1 + 6 2+4 -3-2 5 6-5
”^^32
BA =
-1
4 -5 ^ 4-15 -8-10 12 + 5 -11 -18 17

Clearly, AB BA.
7.7.1 PROPERTIES OF MATRIX MULTIPLICATION
THEOREM 1 Matrix multiplication is not commutative in general.
RROOl Let A and B be two matrices such that AB exists. Then it is quite possible that BA may not
exist. For example, if A is a 3 x 3 matrix and B is a 3 x 2 matrix, then AB exists but BA does not
exist. Similarly, if BA exists, then AB may not exist. Further, if AB and BA both exist, then they
may not be equal as shown in illustration 2 (given above).
Q.E.D.
Hence, in general, AB ^ BA.
7.20
APPLIED MATHEMATICS-XII

THEOREM 2 Matrix muitipUcatiou is associative i.e. (AB) C=A{ BC), whenever both sides arc deifned.
PROOl: LetA=la^j\„,,„ / S=I^y]/ixp andC ThenAB isanmx p matrix and so(AB) Cis
amxq matrix. Clearly, BC is of order nxq and so 7l (BC) is of order m q. Thus, (i4B) C and X

A (BC) are of the same order.


P
Now, ({AB)Ch = r I= ] (/IB),, (C)„-●'
V ( n P H
=>
({AB)Ch = I I c-= Z
' r
= l(^s = ] J ' r =1 s = l^((hshr)Crj
P II

((AB)C)ij r
S I (bs,. c,j) [By associativity of multiplication of numbers]

w
n ( P II

({AB)Ch‘ = s =I 1 Oi,
^ <^rj) % (®Qs/ =(^ (BC))jj for all/,/

F lo
Thus, (AB) C and A (BC) are matrices of the same order such tiiat their corresponding elements
are equal.

ee
Hence, {AB)C=A(BC).

Fr
Q.E.D.
THEOREM 3 Matrix multiplication is distributive over matrix addition i.e.
(i) v4(B + C) = AB + AC
for
ur
(ii) {A + B)C = AC + BC whenever both sides of equality are defined.
I'ROOF Do yourself.
ks
Yo

THEOREM 4 If A is an m x n matrix, then A = A = A I„.


oo

PROOF Do yourself.
eB

REMARK 1 The product of two matrices can be the null matrix while neither of them is the null matrix.
0 2' '1 O' 0 0
For example, if A = and B = thenAB =
r

0 0
0 oj' 0 0
while neither A nor B is the null matrix.
ou
ad

THEOREM 5 If A is m x n matrix and O is a null matrix, then


Y

A« xn O II Xp
= o
m xp (●0 Opxin Ah xh = O
pxn
nd
Re

i.e. the product of the matrix zvith a null matrix is alzvai/s a null matrix.
PROOF Do yourself.
Fi

REMARK 2 In the case of matrix multiplication if AB =0, then it does not necessarily imvly that
BA=0.

0 ] 1 0
For example, if A = 0 0
and B =
0 0
. Then, AB = O. But, BA = 1 0] '0 1 _ 0 1
* O.
0 0 0 0 “ 0 0
Thus, AB =0 zvhile BAj^O.

7.7.2 POSITIVE INTEGRAL POWERS OF A SQUARE MATRIX


For any square matrix, zve define
(i) A^ = A and, (ii) A H + 1 = A” ■ A, where n e N.
It is evident from this definition that/l“ =AA,A^ =A^ A ={AA) /I. etc.
It can be easily shown that
(i) A'” A" = A
m + n
and, (ii) (A'")"=A
iim
for all m, n e N.
7.21
ALGEBRA OF MATRICES

h-2
MATRIX POLYNOMIAL let f {x) = (7q V
(-1
+ aj X +...+ Ojj_\ X + anbea polynomial and let
A be a square matrix of order n. Then,
H-1 n-2
f(A) = + A + . + a„^i A + a,, I n

is called a matrix polynomial


matrix, then
For example, if / (x) = x^ - 3 x + 2 is a polynomial and A is a square
f{A) =2l^-3/l + 2/isa matrix polynomial.
ILLUSTRATIVE EXAMPLES

Type I ON MULTIPLICATION OF MATRICES

ow
a
s X

EXAMPLE 1 If A, B, Care three matrices such that A =[xyz],B^ h b f , C= Zy ,findABC.


S f
SOLUTION Since the product of matrices is associative. Therefore, we can find ABC either by

e
computing (AB) C or by computing A (BC). Let us compute A (BC).

Fl
re
Since B is a 3x3 matrix and C is 3x 1 matrix. Therefore, BC is of order 3 x 1.

F
'a h ^Ifxl rf7A: + /n/ + ^2
BC = h b f y = hx +by+ fz
ur
Now,
S f Ux + /i/ + cz_
or
sf
Clearly, A is of order 1x3 and BC is of order 3x1. Therefore, A (BC) is of order 1 x 1.
k
Yo
Now,
oo

ax + hy + gz
A (BC) = [xi/z] hx + by + Jx
B

_gx + Jy + cz_
re

A (BC) = X (ax 4- hy + gz) + y (hx + hy + fz) + z(gx + fy + cz)


A(BC) = ax^ + Ihxy + hx/ + cz^ + 2,^z + Igzx
u
ad
Yo

2 1
EXAMPLE 2 2 3 , B =
1 0
, prove that (A + B)^ A^ + 2AB + B^.
d

SOLUTION We have.
Re
in

2 1

2 = 1 0
F

-1 -4
-AA-\^
1 -1
^2
" ^ [ 2 3 2 3 8 7

r 1 -1 2 1 1 1
AB = 2 3 1 0 7 2

2 2
=> 2AB =
14 4

2 1 2 1 5 2
B-= BB =
1 0 1 0 2 1

2 1 3 0
A+B - 2 3
+
1 0 3 3

3 0 3 0 9 0
...(0
(A-B)^={A + B)(A + B) = 3 3 3 3 18 9
7.22
APPLIED MATHEMATICS-XN

-1 -4 2 2 5 2
Also, A^ + 2AB+B^ = + +
6 0
8 7 14 4 2 1 24 12 ●●●(ii)

From (i) and (ii), we obtain that (A + sf ^ + 2AB +


1 -1 a
LXAMl’Ll.
; ^ and {A + = A^ + , find a and b.
SOLUTION We have,
(A + Bf = A^+b'^
(A + B)(A + B) =
{A + B)A+(A + B)B = A^ + B^ [By distributive law]
A^ + BA+AB+b‘^ = + B^ [By distributive law]
BA+AB =0

w
a 1 1 -1 1 -1 a 1 0 0
+
b -1 2 -1 2 -1 b -1 0 0

£7 + 2
b-2
-£7 -1
-b+l
+
2a-h
a-b 2
3

F lo0
0 0
0

ee
2a-b + 2 -fl + 1 0 0

Fr
_2£7-2 -b + 4 0 0
for
2£7-& + 2 = 0, -£7 + 1=0, 277-2=0 and -fc + 4=0
£7=1, b = 4
r
a 0 1 O'
You

LXAMI’l.E4 If A = and B =
,fi7id the values of a for which A^ =B.
s

1 1 5 1
ook

SOLUTION We have,
eB

A^ = B
a 0 a 0 1 0
our
ad

1 1 1 1 5 1

a^ + 0 0+0 1 0
a+1 0+1 5 1
dY
Re

a 2 0 1 0
a +1 1
Fin

5 1

2
a =1 and a+1 = 5

a
= ± 1 and a = 4, which is not possible.
Hence, there is no value of a for which = B is true.

EXAMPLES LetA = 2 -^1 R 5 2


,C =
2 5
. Find a matrix D such that CD -AB = O.
3 4 7 4 3 8

£7 b
SOLUTION Let D = . Then,

CD-AB = O
CD = AB
2 5 a b 2 -lirs 2
3 8 3 4 7 4
y.
7.23
ALGEBRA OF MATRICES

2a + 5a: 2b + 5y ■ 10-7 4-4


3a + 8a 3b + 3y 15 + 28 6 + 16

2a + 5x 2fc + 5i/ 3 O'


43 22
3a + 8a 3b + 8y
+ 5a = 3, 3a + 8x = 43, 2b + 5i/ = 0 and 3b + 8ij = 22
Solving 2a + 5x = 3 and 3a + 8a = 43, we get: a = -191 and a -77.
Solving 2/j + 5y = 0 and Si’ + 8y = 22, we get: b = -110 and y = 44.

ow
a b -191 -no"
D =
77 44
^ y.
13 2 1
= 0.
F.XAMPLE(> Find the value of X such that: [1 a 1] 2 5 1 2

e
15 3 2 A

re
SOLUTION We have.

Frl
1 3 2 1

F
[1 A 1] 2 5 1 2 = 0
15 3 2 A
ou
r
7 + 2a

[1 A 1] 12+ A = 0
|_21 + 2a so
kf
7 + 2a + 12a + a^ + 21 + 2a = 0
oo

A^ + 16 a + 28 = 0
Y
eB

(a + 14)(a + 2) = 0 ^ A = -2 or -14.
I 0 1 0
,thenfindkso thatA^ =8/1 + kl.
ur

EXAMPLE? IfA = and 1 =


-1 7 0 1
oY

1 0
ad

SOLUTION We have, A = 1 7
d

1 0 1 0 1 0
A^ = AA =
in

-1 7 -1 7 -8 49
Re

1 0 1 0 8 0 k 0 8 + k 0
F

and. 8A + kI =8 + k +
-1 7 0 1 ~ -8 56 0 k -8 56+ k

A^ ^ 8A + kl
1 0 8 + k 0
-8 49 -8 56+ k

1 = 8 + cf and 56 + k = 49 ^ k = -7.
2 -1 -1 10

EXAMPLE 8 If 1 0 /I = 1-2 -5 ,fi7idA.


-3 4 9 22 isj
SOLUTION Since the product matrix is a 3 x 3 matrix and the premultiplier of A is a 3 x 2
matrix. Therefore, /4 is 2 x 3 matrix.
X y z
LetA =
^ ^ ^ ■ Then, the given equation becomes
7.24
APPLIED MATHEMATICS-XII

2 -1 -1 -8 -10
X y z
1 0 1 -2 -5
a b c
-3 4 9 22 15

2x-a 2y-b 2z-c -1 -8 -10


y 2 1 -2 -5
-3A: + 4fl -3j/ + 4fc -3z+4c 9 22 15

2:v-rt=-l, -3x + 4a = 9, 2 y -1 = - 8, y =-2,


-3y + 4:b=22, 2z-c=-10, 2 =-5, -32 + 4c=15
A'=l,rt = 3, y = -2, t=4,2 = -5 and c = 0
1 -2 -5'
Hence, A =
3 4 0

Ti/pc II ON MATRIX POLYNOMIALS AND MATRIX POLYNOMIAL EQUATIONS

r 2 0 1'
EXAMPLE9 Letf(x)==x^-5x + 6.Findf{A),ifA= 2 13.

SOLLmON First, we note thatby/(A) wemean the matrix polynomial

same as that of A.

Now, = AA=
2
2
0
1
1
3
- 5A + 6/3.Tlratis, to
obtain/(,4), x is replaced by A and the constant term is multiplied by the identity matrix of order

2
2
0
1
F low
1
3
|_1 -1 0

for Fre
e
1 -1 0 1 -1 0

4+0+1 0+0-1 2+0+0 5 -1 2


/1^= 4 + 2+ 3 0 + 1-3 2+3 + 0
Your

9-2 5
eBo ks

2-2+0 0-1+0 1-3+0 0 -1 -2

■(-5)x2 (-5)x0 (-5)xl -10 0 -5


ad

-521 = (-5)x2 (-5)xl (-5)x 3 -10 -5 -15


our

_(-5)xl (-5)x{-l) {-5)x0 -5 5 0

1 0 0 6 0 o'
Re

and. 61-1 = 6 0 1 0 0 6 0
0 0 1 0 0 6
Find Y

5-1 2 ■ -10 0 -5 6 0 0
f{A)=A^ -5A + ei^== 9-2 5 + -10 -5 -15 + 0 6 0

[0 -1 -2 -5 5 0 0 0 6

[1-1-3
f{A)=A^ -5A + 6l3= -1 -1 -10
-5 4 4

3 1
EXAMPLE 10 I/A = -1 2 , shozu that a"^ -5A +712=0.
3 1
SOl UTION We have, A -
[-1 2J
3 1
=

1 2
3 1
9-1 3+2]_ 8 5
-1 2
J . 3-2 -1+4 " 5 3
7.25
ALGEBRA OF MATRICES

-15 -5
-5/\ =
■(-5)x 3 (-5)xl
(-5)x(-l) (-5)x2 5 -10

7 0
7/2 = 7 10 ^1 0 7

8 5 -15 -5 7 0
A--5A+712 = -5 3
+
5 -10
+
0 7

8-15 + 7 5-5 + 0 0 0
A^-5A+7l2 = -5 + 5 + 0 3-10+7 0 0

EXAMPLE n if A is a square matrix such thatA^ =A, sboxo that{l +A)^ =7 A +1.
SOLUTION Using matrix multiplication, we obtain
{I+Af= {l+A){I+A)
^I(l + A) + A(l+A) [By distributivity of multiplication over addition]

w
= I^ + !A + AI+A^
= i + a + a + a'^

F lo
[v IA=AI=A]

= ] + 2A + A^
= 7 + 2A + A = I + 3/4 Iv/42=/4]

ree
(/ + /4)^=(/ + <4)^(/+/4) for F
(/+A)^-(/ + 3/l)(]+/4)
= 1{1+A) + 3A{1+A)
Your

= /2+ IA + 3{AI)-\-3{AA)
oks

= 1 +A + 3A+3A^ [●.● IA=AI= A]


eBo

= [ + A + 3A+3A = I+7A [v.42=A]

EXAMPLE 12 If A is a square matrix such that A^ =!, then find the simplified value of
ad
our

(/4-/)^+(A + lf-7A.
SOLUTION We have, /4^ = /
Re
dY

A^=A^A^!A=A
Fin

We know that

{A + E)^ =A^ + 3A^B+3AB'^ + and, (A -B)^ = A^ ^3A~B+3AB^


provided that AS = BA.
Sin.ee A1 - IA= A.

{A + lf=A^ + 3A^1 + 3AI~ + 1^ and{A-iy =A^ 3A^/ + 3A/2-P


=> (A + I)^ =A^ + 3A^ + 3A + I and(A-l)^ =A^ -3A~ + 3A-I
{A + lf+(A-lf=2(A^ + 3A)
{A + lf+{A-l)^ =2{A + 3A) [Using (i)]

{A + lf+iA~[f=3A
Hence, (/4 - /) ^ + (/4 + /)^ -7A = 8/4 -7 A = A.
7.26
APPLIED MATHEMATICS-Xll

l)/pc III MISCELLANEOUS PROBLEMS


LXAMPl.f
13 Under zoliat conditions is the matrix equation - {A- B) (A + B) is true?
SOLUTION We have,
A^ -B'^={A-B){A + B)
«■
A^-B^={A-B)A+{A-B)B By disributivity of matrix muliplication ’
over matrix addition

A^ -B^ =A^ -BA + AB-B^ By disributivity of matrix muliplication


over matrix addition
o 0=A^-BA + AB-B^-A'^+B^
<=> 0=- BA + AB

ow
o AB^BA

Thus the given matrix equation is true if the matrices A and B commute with each other.
I
\AMPU: 14 I/A is any inxn matrix such that AB and BA are both defined show that B is an nxm
matrix.

e
SOLUTION Since A is an m x n matrix such that AB exists. Therefore, the number of rows in B

Fl
re
should be equal to the number of columns in A. Thus, 6 has n rows. Further, BA exists, therefore
the number of columns in B should be equal to the number of rows in A. So B has m

F
columns.Hence, B is an n x m matrix.
ur
' XAMI’LE 1=; A,B are two matrices siwh that AB and
A + Bare both deifned; show tlwt A, B are square
matrices of the same order.
or
sf
SOLUTION Let A be an m x n matrix. Since A + Bis defined, therefore 6 is also an w x ft matrix.
Further since AB exists, therefore the number of columns in A is same as the number of rows in B
k
Yo

i.e. n = m. Hence, A and B are square matrices of the same order.


oo

^ \.\MPLL \u If A and B are square matrices of ordern, then prove that A and B will commute iff A - XI
B

and B-XI commute for every scalar X.


re

SOLUTION A - XI and B-XI commute


<=>
(A-XI){B-XI) = {B-XI){A-X1)
u

AB-XIA-XIB + 7? I^ = BA-XBI-XIA + )?l'^


ad

<=>
Yo

<=> AB-XA-XB + X^ I = BA-XB-XA + 7? I


<=> AB = BA
d
Re

<=> A and B commute.


in

I \ \MPi !
IfAB = A and BA = 6, then show that A^ =A,B^= B.
F

SOLUTION We have, AB = A and BA = B


Now, AB = A
(AB) A = AA [Multiplying both sides on right by A]
A (BA) = A^ [By associativity of matrix multiplication]
AB = A^ [●.● BA = B]
=> A = A^ [■.■AB=A]
Similarly, it can be proved that B^ = B.
i.XAMPLL IK
Give an example of two matrices A and B such that
(i) A O, B O, AB = O and BA ^ O
(ii) A O, B ^ O, AB = BA = O.
1 0 0 0
SOLUTION (i) If,4 = 0 0
and B = , then/I ^ O, B ^ O.
3 0
7.27
ALGEBRA OF MATRICES

1 Giro 0 0 0
But, AB = =0
0 0 3 0 0 0

0 oiri 0 0 0
and. BA =
3 0 0 0 3 0

1 1 1 -1
(ii) li A = and 6 = , then A ^ O, B ^ O.
1 1 -1 1

1 1 ' 1 -1 1-1 -1 + 1 0 0
=0
But, /1B = 1„1 _i+i 0 0
1 1 -1 1

1 -iiri 1 ‘1-1 1-1 0 0

ow
BA = =0
and. -1+1 -1+1 0 0
-1 1 1 1

Type IV ON APPLICATIONS OF MATRIX MULTIPLICATION IN BUSINESS AND ECONOMICS


FXAMIM.I-; U) There arc tzuofaniilics A and B. There are 4 men, 6 women and 2 children in family A and 2

e
men, 2 women and 4 children in family B. The recommended daily allowance for calories is: Man : 2400,

re
zooman :

rFl
: 1900, child: 1800 arid for proteins is : Man : 55 gm, zooman : 45 gm and child: 33 gm.

F
Represent the above information by matrices. Using matrix multiplication, calculate the total
requirement of calories and proteins for each of the tzvo families.

r
The members of the two families can be represented by a 2 x 3 matrix F given fo
ou
SOLUTION

below.
ks
M W C

A \4 6 2■
oo

f =
S 1_ 2 2 4
Y
eB

and the recommended daily allowance of calories and proteins for each member can be
represented by 3 x 2 matrix R as given below.
ur

Calories Proteins
ad

M r 2400 55
Yo

R = W 1900 45

C 1_1800 33
d
Re

The total requirement of calories and proteins for each of the two families is given by the matrix
in

multiplication FR as given below.


F

2400 55
FR
4 6 2
1900 45
_ A r 24600 556
2 2 4 " B 15800 332
1800 33

Hence, family A requires 24600 calories and 556 gm proteins and family B requires 15,800
calories and 332 gm proteins.
CXAMn.L 2(1 Lise matrix multiplication to divide ? 30,000 in tzoo parts such that the total annual
interest at 9% on the first part and 11% on the second part amounts ? 3060.
SOLUTION Let the two parts be ^ .v and ? (30000 - .v) respectively. Let /I be the 1 x 2 matrix
representing these two parts
Part I Part II

i.e. A = [.r 30000 - x]


Let R denote the 2 x 1 matrix representing the annual interest rates of interest on two parts
i.e.
7.28
APPLIED MATHEMATICS-XII

Part 1 r 0.09 '


R
Part II [ O.n
The total annual interest on the two parts is given by the matrix multiplication AR.
AR = 3060
0.09
I-v 30000 - x] = 3060
0.11

0.09 x +0.11 (30000-.V) = 3060

ow
9 11
Y +
(30000-.y) = 3060
100 100

9.V+ 330000-11 Y = 306000 => y = 12,000


Hence two parts of ? 30,000 are ? 12,000 and ? 18,000 respectively.

e
EXAMPLE 21 Three schools A, B and C organised a mein for collecting funds for helping the

re
rehabilitation offload victims. They sold hand made fans, mats and platesfrom recyled material at a cost of
f 25, ? 300 and ? 50 each. The number of articles sold are giiwn below:

Flr
F
School
A B C
Article

Hand-fans
ou
40

sr
25 35

fo
Mats 50 40 50
Plates 20 30 k 40
oo
Find the funds collected by each school separately by sailing the above articles. Also, find the total finds
collected for the purpose.
Y
reB

SOLUTION Three items sold by three schools are represented by the following 3x3 matrix Q
as given below.
uY

Hand-fans Mats Plates


A 40 50 20

Q = B 25 40 30
ad
do

C 35 50 40

The price matrix representing price of of three articles in ? is a 3 x 1 matrix given by


in

Hand-fan 25
Re

P = Mat 100
F

Plate 50

The funds collected by schools A, B and C separately by selling three articles are given by the
product matrix QP.
Hand-fans Mats Plates
A 40 50 20 25
QP = B 25 40 30 100
C 35 50 40 50

2l 40x 25+50x100-h 20x50' 7,000


QP = B 25 X 25 + 40 X 100 + 30 X 50 = B ? 6,125
C 35 X 25 + 50 X 100 + 40 x 50 C ?7,875

Hence, the funds collected by schools A, B and C are f 7,000, ? 6,125 and ? 7,875 respectively.
The total funds collected = ? (7,000 + 6,125 + 7,875) = ? 21,000,
7.29
ALGEBRA OF MATRICES

EXAMPLE 22 A baker makes bread, sweet patties and biscuits. He requires flour, egg, sugar, milk and
yeast for his preparations. The requirement of these basic items for making a bread, a sweet patty and a
biscuit is as follows :
Bread Sweet patty Biscuit

Flour 250 gms 50 gms 8 gms


1 1
Egg 0
2 4

Sugar 125 gms 100 gms 25 gms


0.05 litre 0.02 litre 0.01 litre

ow
Milk
1 1
cake cake 0
Yeast
4 8

The baker buys the flour for ^ 30 per kg. Sugar for 40 per kg, an egg for 110, milkfor ^ 30 per litre and
a

e
cake of yeast for ^ 20. matrix multiplication, find the cost of making a bread, a sweet patty and a

Fl
re
biscuit?

F
SOLUTION The requirement of flour, egg, sugar, milk and yeast for making a bread, a sweet
patty and a biscuit can be represented by a 3 x 5 matrix A given below.
ur
r
Flour Egg Sugar Milk Yeast fo
(inkg) (in kg) (in litre)
ks
Bread 0.25 0 0.125 0.05 0.25
Yo

A = Sweet Patty 0.05 0.5 0.100 0.02 0.125


oo

Biscuit 0.008 0.25 0.025 0.01 0


B

The cost of ingredients can be represented by 5 x 1 matrix B given by


re

Flour 30
u

Egg 10
ad
Yo

B = Sugar 40

Milk 30
d

Yeast 20
Re
in

The cost of making a bread, a sweet patty and a biscuit is given by the matrix product
F

30

0.25 0 0.125 0.05 0.25 10


Bread
0.05 0.5 0.100 0.02 0.125 40
/IB = Sweet Patty
0.008 0.25 0.025 0.01 0 30
Biscuit
20

Bread 7.5 + 0 + 5 +1.5 + 5 19

1.5 + 5 + 4 + 6 + 2.5 19
AB = Sweet Patty
Biscuit 0.24 + 2.5 + 1 + 0.3+0 4.04

Hence, the costs of making a bread, a sweet patty and a biscuit are ? 19, ? 19 and ? 4.04
respectively.
7.30
APPLIED MATHEMATICS-XII

! XAMiM.i- 2^ A firm produces chairs, tables and cupboards, each requiring three types ofrazo material -
timber, nails and varnish. The requirement of different raw materials for producing one unit of each
product is as given belozv:
Product
Timber (cubic feet) Nails (dozen) Varnish (litres)
Chair 0.7 2 1

Table 1 4 1.5

Cupboard 3.2 6 2

If the timber costs ? 2,000 per cubic feet, nails cost ^ 200 per dozen and Varnish costs ? 800 per litre, find
the cost of producing 500 chairs, 200 tables and 100 cupboards.
SOLUTION The matrix representing the requirement of different raw materials for producing
one unit of each product is

w
Chair Table Cupboard

F lo
Timber 0.7 1 3.2
A = Nails 2 4 6
Varnish 1 1.5 2

e
Fre
The matrix representing the requirement of the products i IS

Chair 500
for
B = Table 200
r
Cupboard 100
You
s
ook

The total quantity of each raw material required to produce 500 chairs, 200 tables and 100
cupboards is given by the matrix product.
eB

Chair Table Varnish


Timber 0.7 1 3.2 500 Timber
our
ad

C = AB = Nails 2 4 6 200 Nails


Varnish 1 1.5 2 Varnish
100
dY

Timber 350 + 200 + 640 Timber 1,190


Re

C = Nails 1,000+800 + 600 = Nails 2,400


Fin

Varnish 500+300 + 200 Varnish 1,000

The matrix representing the cost of different raw materials is


Timber Nails Varnish

D =[2,000 200
800]
So, the total cost of production is given by
Timber 1,190
Timber Nails Varnish
Nails 2,400
DC =[2,000 200 800] Varnish 1,000

DC = ? (2,380,000 + 480,000 + 800,000) = ? 3,660,000


Hence, the cost of production of requisite number of items is ? 3,660,000.
7.31
ALGEBRA OF MATRICES

EXAMPLE 24 The foUozving nidtrix gives the number of units of three products P, Q and R that can be
processed per hour on three machines A, B and C.
A B C

p flO 12 15'
X = Q 13 11 20
R 16 18 14

Determine bi/ using matrix algebra, how many units of each product can be produced, if the hours
available on the machines A, B and C are 54,46 and 48 respectively.
SOLUTION The number of hours available on machines/\, 6 and C can be represented by a 3 x 1
column matrix as given below.
A 54

Y=B 46

C 48

w
The number of units of each product that can be produced is given by the matrix product

F lo
A B C
P 10 12 15 A 54 P 540 + 552+720 P 1812

XY=Q 13 11 20 B 46 =Q 702 + 506 + 960 =Q 2168

ee
R 16 18 14 C 48 R 864 + 828 + 672 R 2364

Fr
Hence, 1812 units of product P, 2168 units of product Q and 2364 units of product R
are

produced in the stipulated times.


for
A firm produces three products A, B and C which it sells in two markets. Annual sales in
ur
EXAMPLE 25

units are given ns follows:


oks

Market Unit sold


Yo

A B C
o
eB

I 8,000 4,000 16,000


II 7,000 18,000 9,000
If the prices per unit of A, B and C are ?25, ?12.50 and ?15 respectively and the costs per iniit are
our
ad

H7, ?12 and ? 8 respectively, ifnd the total profit in each market by using matrix algebra.
SOLUTION The prices per unit and the costs per unit of A,B and C are given by the following
matrices.
Y

Price (in ?) Cost (in ?)


Re
nd

A 25 A 17
Fi

x = s 1250 , y =e 12

c 15 c

So, the profit per unit of A,B and C is given by the matrix
Price (in ?) Cost (in ?) Profit (in?) Profit (in ?)
A 25 A 17 A 25-17 A

P=X-Y=B 12.50 -B 12 = B 12.50-12 = B 0.50

C 15 C 8 C 15-8 C 7

Tl\e annual sales of A,B and C in two markets I and II are given by the matrix
Market A B C
I 8,000 4,000 16,000’
Q =
II 7,000 18,000 9,000

So, the total profit in each market is given by the matrix product
7.32 APPLIED MATHEMATICS-XII

Market A B C Profit (in?) Market


A
8
I 8,000 4,000 16,0001 I 64,000 + 2,000 + 112,000
QP = B 0.50
II 7,000 18,000 9,000 II 56,000 + 9,000 + 63,000
-■ C 7

Market
I ri78,000'
^ QP =
II [l 28,000
Thus, the profits in Markets I and II are of ? 178,000 and ? 128,000 respectively.
A transport company lias 3-types of trucks -T^, and to transport 3-types of Vehicles
V-y, V2 and V^. The capacity of each truck in terms of three types of Vehicles is given below:
y, ^2 V3
Ti 1 3 2

w
72 2 2 3
T3 3 2 2

has 10,20 and 30 trucks of type 7j, T2 and respectively.

F lo
Using matrix method, find the number of vehicles of each type ivhich can be transported if the company

ee
SOLUTION The capacity of each truck in terms of 3-types of vehicles can be represented by the

Fr
following 3x3 matrix.
71 h 73
y] 1 2 3
for
ur
A = V2 3 2 2

^3 2 3 2
s
ook
Yo

The number of trucks of each type wliich the company uses for transportation is given by the
eB

matrix
10

B = T2 20
our
ad

30

The total number of vehicles of each type which can be transported is given by the matrix
dY

product
Re

T2 T,
Fin

y1 1 2 3 10 V'l 10 + 40 + 90 y1 140

AB = V2 3 2 2 T’2 20 = y2 30 + 40 + 60 -
130

ys [2 3 2 30 V'3 20 + 60 + 60 ^3 140

Thus, the number of 3 types of vehicles -yj, y2 and y3 which can be transported are =140,
y2 =130 and y3 =140.
A firm produces three products Pj, P2 and P3 requiring the mix-up of three materials
Mj, M2 and M3. The per unit requirement of each product for each material is as follows:
M3 M2 M3
Pi 2 4 5

P2 3 2 4
P3 1 3 2

Using matrix algebra, find:


7.33
ALGEBRA OF MATRICES

(i) The total requirement of each material if the firm produces 100,200 and 300 units ofproducts
Pj, ?2 and P3 respectively,
(ii) The per unit cost ofproduction ofeach product if the per unit costs of materials Mj, M2 and M3
are ?10, ?15 and ?12 respectively.
(Hi) The total cost of production.
SOLUTION The matrix showing per unit requirement of materials Mi, M2 and M3 in
producing three products Pj, P2 and P3 is
Pi Pi P3
Ml 2 3 1‘
A = M2 4 2 3

M3 [5 4 2
(i) The matrix representing the requirements of products Pj, P2 and P3 is

w
Pi 100

Flo
B = P2 200

P3 [300

ee
So, the requirements of each material for producing the given quantities of three products 1
IS

Fr
given by the product
Pi P3
Ml '2 3 l' Pi 100 for
ur
4 2 3 Pi 200
AB = M2
M3 |_5 4 2j P3 [300
k s
Yo
oo

200 + 600 + 300 Ml 1100


Ml
AB = M2 400 + 400 + 900 = M2 1700
eB

M3 500+ 800 +600J M3 [l900


Thus, 1100 units of material Mi, 1700 units of material M2 and 1900 units of material M3 are
r
ou
ad

required to produce 100 units of Pi, 200 units of P2 and 300 units of P3.
(ii) The matrix representing per unit costs of materials Mi, M2 and M3 is as given below:
Y

Cost (in ?)
Re
nd

Ml 10

C = M2 15
Fi

M3 [12
The matrix exhibiting the materials Mi, M2 and M3 in three products Pi, P2 and P3 is
Ml M2 M3
Pi 2 4 5‘
D = P2 3 2 4

P3 [1 3 2
So, the per unit cost of each product is given by the matrix product
Ml M2 M3 Cost (in ?) Cost (in ?) Cost (in ?)
20 + 60 + 60 Pi 140
Pi 2 4 5 Ml 10
30+30 + 48 = Pi 108
DC = P2 3 2 4 M2 15 = Pi
P3 [1 3 2 J M3 [12 P3 [10 + 45 + 24 P3 79
7.34
APPLIED MATHEMATICS-XII

Hence, per unit cost of production of products P\,P2 and P3 are ?140, ?180 and ?79
respectively,
(iii) The total cost of product is given by the matrix product
Cost (in ?)
140
P: Pi
Pi 108 = ? (14,000 + 21,600 + 23,700) = f 59,300
[100 200 300]
P3 79

Hence, the total cost of product is ? 59,300.


EXAMPLE 28 ^ immifacturer produces three products X, Y mid Z which it sells in three markets I, II
and III. Monthli/ sales and sale prices are given as follows:
Market Items sold Sale Price (in ?)

w
X y z X y z

F lo
1000 1500 2000 20 30 40

II 2000 2800 3000 25 28 37

ee
III 3000 3500 4000 23 31 42

Fr
If the costs per item for three products X, Y and Z are ? 18, ?25 and ? 34 respectively, find by matrix
algebra his total profit. for
SOLUTION Let S|, $2 and S3 denote the sale price matrices of three products in three markets
ur
1, II and III respectively. Then,
s
X y Z X y z X y z
ook
Yo

Si =[20 30 40], So =[25 28 37], S3 =[23 31 42]


eB

LetC-i, C2 and C3 denote the cost price matrices and Pj, P2 and P3 denote the profit matrices of
products X, y and Z respectively in three markets. Then,
r

X y z X y Z X y z
ad
ou

Cl =[18 25 34], C2=[18 25 34], 03= [18 25 34]


X y z X y z X y z
Y

and.
^l=Si-Ci=[2 5 6],P2=S2-C2=[7 3 3], P3 = S3-C3 =[5 6 8]
Re
nd

Ql/ Q2 Q3 denote the matrices representing items X, y and Z sold in three markets.
Fi

Then,
X 1000 X 2000 X 3000

Qi=r 1500 . Q2=y 2800 ^Q3=y 3500


Z 2000 z 3000 z 4000

Total profit = t (P^Qj +P2Q2+ P3Q3)


1000 2000 3000

= ?(2 5 6] 1500 + ? [7 3 3] 2800 + ?[5 6 8] 3500


2000 3000 4000

= ? (2,000+ 7,500 +12,000)+? (14,000+8,400+ 9,000)+ ? (15,000 + 21,000+ 32,000)


= ? (21,500 + 31,400 + 68,000) = r 120,900
Hence, the total profit of the manufacturer is ? 120,900.
7.35
ALGEBRA OF MATRICES

EXAMPLE 29 The follozoirig matrix gives the proportionate mix of constituents used for the three
fertilizers:
Constituent

A B C D

7 0.5 0 0.5 0

Fertilizer n 0.2 0.3 0 0.5

III 0.2 0.2 0.1 0.5

(i) If sales are 1000 fins (of one kg each) per iveek each consisting of
20% being fertilizer /, 30% being
fertilizer II and 50% being fertilizer III, how much of each constituent is used?
(ii) Ifthecostofeachconstituentis^S, ? 6, ? SandX I0perl00gmsres pectwehj,hoiomiichdoesone
kilogram tin of each fertilizer cost?

w
(Hi) What is the total cost per week?
SOLUTION The matrix giving proportionate mix of constituents used for tliree fertilizers is as
given below;
1 II III

F lo
ee
A 05 0.2 0.2'

Fr
B 0 0.3 0.2

C 05 0 0.1 for
D 0 05 05
ur
The weekly sales of three fertilizers can be represented by 3 x 1 matrix given by
s

20
ook
Yo

I
100
200 I
eB

30
S=1000 II 300 II
100
500 III
50
our

III
ad

100

(i) The total quaiitity of each constiUient used is given by the matrix product:
Y

I II III
Re

A r05 0.2 0.2'


nd

A 260
200 I
B 0 0.3 0.2 B 190
Fi

PS = 300 II
C 05 0 0.1 c 150
500 111
D [0 05 05 D 400

Thus, 260 kg, 190 kg, 150 kg and 400 kg of constituents A, B, C and D are used to produce three
fertilizers,

(ii) The cost in ? (per kg) of each constituent can be represented by the following column
matrix:
5 A 50 A

6 B 60 B
R =10
8 C 80 C

10 D 100 D

Therefore, the cost of 1 kg tin of each fertilizer is given by the matrix product
7.36
APPLIED MATHEMATICS-XII

A B C D
/I 50
I 05 0 05 0 I 65
6 60
II 0.2 0.3 0 05 II 78
C 80
III 0.2 0.2 0.1 05 III 80
D 100

Hence, one kilogram tin of ferdlizers I, II and III costs ? 65, ? 78 and ? 80 respectively,
(iii) The total cost of three fertilizers, when 1000 tins (of one kg each) are sold per week, is given
by
I 65
I II III

ow
II 78 = ? (13,000 + 23,400 + 40,000) = ? 76,400
[200 300 500]
III 80

Hence, the total cost per week is ? 76,400.


EXERCISE 7.3

e
1- Compute the indicated products:

Fl
re
2 3 4 1 - 3 5
a b a -b 1 -2 1 2 3

F
(i) -b a (ii) (iii) 3 4 5 0 2 4
b a 2 3 -3 2 -1
4 5 6 3 0 5
ur
2. Show that AB ^ BA in each of the following cases:
or
sf
-1 1 0 1 2 3
5 -1 2 1
(i)A = and B = (ii) = 0 -1 1 and B = 0 1 0
k
Yo
6 7 3 4
2 3 4 1 1 0
oo

3. Compute the products AB and BA whichever exists in each of the following cases:
B

3 2
-2 1 2 3' 4 5 6
re

and B - (ii) A = -1 0 and B =


3 2 3 1 0 1 2
-1 1
u
ad

0
Yo

1 c b
(iii) A =[1 -1 2 3] and B = (iv) [n b]
3 d + [n b c d] ^
d

2 d
Re
in

4. Show that AB ^ BA in each of the following cases:


F

1 3 -r -2 3 -1 10 -4 -1 1 2 1
(i) ^ = 2 -1 -1 andS= -1 2 -1 (ii) A= -11 5 0 and B = 3 4 2
3 0-1 -6 9 -4 9 -5 1 1 3 2

5. Evaluate the following:


^ r 1 0 2 2
1 3 3 -2 1 3 5
(i) -1 -4
+
-1 1 (ii) [1 2 3] 2 0 1 4
2 4 6
0 12 6

1 -1
(iii) 0 2
1 [012
0 2 -| \

2 3 2 0 1J"[l 02
1 0 0 0 1
6. IfA = and C = , then show that=6^ =C^=I^.
[0 1 0 -1 1 0
7.37
ALGEBRA OF MATRICES

■ 0 4'
7. IfA = ^3 i and B =
2 -1 7
, find3A2-2B + /.

■ 4 2'
8. If .4 =
-1 1 , prove that {A - 21) {A -31) = O.
ri 1 ■ ri 2‘ 1 3
9. IfA = , show that A = and A3 =
0 1 0 1 0 1 '

ah \P-
10. IfA = , show that A^ =0.
-a^ -ah

w
2 -3 -5 -1 3 5
11. IfA = -1 4 5 and B = 1-3-5 , show that AB = BA=03x3-
1 -3 -4 -1 3 5

,2 ah ae

o
0 e -h

e
12. IfA = -e 0 a andB= ah h^ he , show that AB = BA = 03x3-

re
h -a 0 ac he

Frl
F
2 -3 -5 2 -2 -4l
13. IfA = -1 4 5 and B = -1 3 4 , show that AB = A and BA = B.
1 -3 -4 1 -2 -3
ou
or
-1 1 -1 0 4 3 kfs
14. LetA = 3-3 3 and B = 1 - 3 - 3 , compute A^ - B .
5 5 5 -1 4 4
oo

15. For the following matrices verify the associativity of matrix multiplication i.e.
Y

(AB)C=A(BC).
eB

1 0
■ 1 2 O' 1
(i) A = -10 1
, B = -1 2 and C = -1
ur

_ 0 3_
oY
ad

4 2 3 1 -1 1 1 2 -1
0 1 2 and C = 3 0 1
(ii) A= 1 1 2 , B=
d

3 0 1 2-11 0 0 1
in

of matrix multiplication over matrix


Re

16. For the following matrices verify the distributivity


addition i.e. A ( B + C) = AB + AC.
F

r-1 01 0 1

(i)A=[;
-1
, B= and C=
2 1 1 -1

2 -1 -1
(ii) A = 1 1 , B=
0
1
1
1
and C = \0 1 ■
[-1 2
■ 1 0-2' 0 5-4 1 5 2

17. If A = 3-1 0 ,B = -2 1 3 and C = -1 10, verify that


-2 1 1 -10 2 0 -1 1

A(B-C)=AB-AC.
18. Compute the elements ^43 and ^22 Oie matrix:
0 1 0
2 -1
2 0 2 0 1-1 2 -2
A = -3 2
0 3 2 3-3 4-4 0
4 3
4 0 4
7.38
APPLIED MATHEMATICS-XII

0 1 0
19. Ifyl
= 001 and / is the identity matrix of order 3, show that =pl + qA + rA^.
P ^ d
2 -3 -5]
20. IfA = -1 4 5 , show that A^ = A,
1 -3 -4

4 -1 -4l
21. If^ = 3 0-4 ,show that = fa.
3 -1 -3

1 0 2 1
22. (i) If [1 1 :c] 0 2 1 1 = 0, find:r.

w
2 1 0 1

2 3ir 1 -3 -4 6

F lo
(ii) If ,findx.
5 7 J[-2 4 -9 X

'2 1 2' X

ee
23. If [x 4 1] 1 0 2 4 = 0,findx.

Fr
0 2-4 -1

'3 -2 1 0 for
24. If ^ =
4 -2
and / =
0 1 , then prove that A^ -A + 2I =0.
ur
3 1 1 0
s

25. UA = and I = , then find Xso that A^ =5A + XL


ook

-1 2 0 1
Yo

3 1
eB

26. lfA =
-1 2 , show that A^ -5A+7I2 =0.
2 3
our

27. Show that the matrix A = satisfies the equation A^ -4A^ + A =0.
ad

1 2

3 -5
28. If /I = , find A^ -5A -14/.
Y

-4 2
Re
nd

3 1
29. IfA = , show that A^ -5A+ 71= O. Use this to find A^.
Fi

-1 2

3 -2
30. IfA =
4 -2 , find k such that A^ =kA- 2/2.
1 2
31. IfA =
2 1 and /(x)=x^-2x-3,showthat/(A)=0.
2 3 1 0
32. IfA =
1 2
and / =
0 1 , then find X, p so that A^ = X A+ p I

33. Find the value of x for which the matrix product


2 0 7 -X 14 X 7x
0 1 0 0 1
0 equal to an identity matrix.
1 -2 1 X -4x -2x
7.39
ALGEBRA OF MATRICES

34. Solve the matrix equations:


1 2 0 0
1 0 .V
2 = 0
(i)[-v 1] = 0 (ii) [12 1] 2 0 1
-2 -3 5
1 0 2 .r

1 0 2
1 2lr.t’
X

= 0 (iv) [2.V 3] = 0
(Hi) [.v-5-1] 0 2 1 4
-3 0 8
2 0 3 1

1 2 0

w
35. HA = 3-4 5 , compute -AA + 3/3.
0 -13

0 1 2
4 5 0
36. If /(.v) -2.Y, find f {A), where A =

o
e
0 2 3

re
1 0 2
, then show that A is a root of the polynomial / (.v) = X ^ -6x^ + 7x + 2.

Frl
F
37. If = 0 2 1
2 0 3J
ou
■ 1 2 2 1

sor
38. If A = 2 12 , then prove that -4/1-5/=0.
2 2 1 J kf
such that
39. Without using the concept of inverse of a matrix, find the matrix ^
oo
u
Y

5 -7 y -16 -6‘
B

-2 3 z ll 7 2 ●

40. Find the matrix A such that


re
oY

1 1 3 3 5' 12 3 -7 -8 -9
u

(i) A = (ii) A 4 5 6 2 4 6
0 1 1 0 3
ad

4 -4 8 4 -1 0 -1 1
d

-1 1 0 0 = A
(iii) 1 A = -1 2 1 (iv) [2 1 3]
in

0 1 1 -1
Re

3 -3 6 3
F

2 -1 -1 10 -7 -8 -9
1 2 3
1 0 A = 1 -2 -5 (Vi) A 2 4 6
(V) 4 5 6
-3 4 9 22 15 11 10 9

-2
41. Find a 2x 2matrix A such that A | 4 - ^^2-
rO --Y 0 1
42. If A = , B = and -Y^ =-l, then show that (A + B)^ =A^ + .
.Y 0 1 0

1 0 -3

43. If A = 2 1 3 , then verify that A^ + A = A (A + /), where I is the identity matrix.


0 1 1

3 -5
44. If A =
-4 2
, then find A^ -5A -141. Hence, obtain A
7.40
APPLIED MATHEMATICS-XII

X 0 0 a 0 0 xa 0 0

45. IfP= 0 y 0 andQ = 0 b 0 , prove that PQ = 0 yb 0 =QP


0 0 z 0 0 c 0 0 zc
2 0 1
46. If A = 2 1 3 , find A^-5A + 4I and hence find a matrix X such that
1 -1 0

A^-5A + 4Z+X = 0.
47. A matrix Xhasa + b rows and a+ 2 columns while the matrix Y has i? + 1 rows and a +3

ow
columns. Both matrices XY and YX exist. Find a and b. Can you say XY and YX are of the
same t)^e? Are they equal.
48. Give examples of matrices
(i) A and B such that AB ^ BA.
(ii) A and B such that AB = 0 but A 9^0, B5tO.

e
re
(iii) A and B such that AB = O but BA ^ O.
(iv) A, B and C such that AB = AC but B:;tC,A^O.

rFl
F
49. Let A and B be square matrices of the same order. Does (A + B)^ = A^ + 2AB + B^ hold?
If not, why?

r
ou
fo
50. If A and B are square matrices of the same order, explain, why in general
ks
(i) {A + Bf ji^A^+ 2AB + b‘^ (ii) (A-B)2 9^a2-2AB+b2
(iii) (A + B)(A-B)9iA2-B2
oo

51. If A and B are square matrices of the same order such that AB = BA, then show that
Y
B

(A + Bf =A^ + 2AB + B^.


re

52. Three shopkeepers A, B and C go to a store to buy stationary. A purchases 12 dozen


notebooks, 5 dozen pens and 6 dozen pencils. B purchases 10 dozen notebooks, 6 dozen
ou
Y
ad

pens and 7 dozen pencils. C purchases 11 dozen notebooks, 13 dozen pens and 8 dozen
pencils. A notebook costs 40 paise, a pen costs ? 1.25 and a pencil costs 35 paise. Use matrix
multiplication to calculate each individual's bill.
d

53. The cooperative stores of a particular school has 10 dozen physics books, 8 dozen chemistry
in
Re

books and 5 dozen mathematics books. Their selling prices are ? 8.30, ? 3.45 and ? 4.50 each
F

respectively. Find the total amount the store will receive from selling all the items.
54. In a legislative assembly election, a political group hired a public relations firm to promote
its candidates in three ways: telephone, house calls and letters. The cost per contact (in
paise) is given matrix A as
Cost per contact
40 Telephone
A = 100 House call
50 Letter

The number of contacts of each type made in two cities X and Y is given in matrix B as
Telephone House call Letter
B =
1000 500
5000] -^X
3000 1000 10000 y

Find the total amount spent by the group in the two cities X and Y.
7.41
ALGEBRA OF MATRICES

55 different types of bonds. The first


A trust fmid has f 30000 that must be invested in two
bond pays 5% interest per year, and the second bond pays 7% mterest per year.
matrix mulHplication, determine how to divide ^ 30000 among the two types of bonds. If
the trust fund must obtain an annual total interest of (i) ^ 1800 (ii) ? 2000.
56. To promote making of toilets for women, an organisation tried to generate
awamess

through (i) house calls (ii) letters, and (iii) announcements. The cost for each mode per
attempt is given below:
(i) ? 50 (ii) ?20 (iii) ^40
The number of attempts made in three villages X, Y and Z are given below:
(i) (ii) (iii)
X 400 300 100

low
Y 300 250 75

Z 500 400 150


matrices.
Find the total cost incurred by the organisation for three villages separately, usmg
57 There are 2 families A and B. There are 4 men, 6 women and 2 children in family A, and 2

e
men, 2 women and 4 children in family B. The recommend daily amount of calories is 2400

re
for men, 1900 for women, 1800 for children and 45 grams of proteins for men, 55 ^ams for
rF
women and 33 grams for children. Represent the above information using matrix. Using

F
matrix multiplicaHon, calculate the total requirement of calories and proteins for each of
people about the plaimed diet
r
the Uvo families. What awareness can you create among
fo
u
from this question?
58. In a parliament election, a political party hired a public relations firm to promote its
ks
candidates in three ways — telephone, house calls and letters. The cost per contact (in
Yo
oo

paisa) is given in matrix A as


ri40 Telephone
B

A = 200 House calls


re

150 Letters
u

The number of contacts of each type made in two cities X and Y is given in the matrix B as
ad
Yo

Telephone House call Letters


r 1000 500 5000 City X
B =
d

3000 1000 lOOOOj City y


Re
in

Find the total amount spent by the party in the two cities.
— party’s promotional activity
F

or
What should one consider before casting his/her vote
their social activities?
59 The monthly incomes of Aryan and Babbar are in the ratio 3 : 4 and their monthly
expenditures are in the ratio 5 : 7. If each saves ^ 15000 per month, find their monthly
incomes using matrix method. This problem reflects which value?
10% interest and
60. A trust invested some money in two type of bonds. The first bond pays
’ second bond pays 12% interest. The trust received ^ 2800 as interest. However, if trust had
interchanged money in bonds, they would have got ^ 100 less as interest. Usmg matrix
method, find the amount invested by the trust.
61. Three firms A, B and C supplied 40,35 and 25 truck loads of stones and 10,5, 8 truck loads
of sand respectively to a contractor. If the cost of stone and sand are ^ 1200 and f 500 per
truck load respectively, find the total amount paid by the contractor to each of these firms,
by using matrix method.
7.42
APPLIED MATHEMATICS-XII

62. A man buys 8 dozens mangoes, 10 dozens apples and 4 dozens of bananas. Mangoes cost
?200 per dozen, apples ?300 per dozen and bananas ?60 per dozen. Represent the
quantities bought by a row matrix and prices by a column matrix. Using matrix
multiplication, find the total amount spent by the man.

63. A firm produces three products Pj, ?2 and P3, requiring the mixup of four materials
Mj, M2, M3 and M4. The matrix below gives the amount of material needed for each
product:
M
1 M2 M3 M4
2 3 1 12‘
A=?2 7 9 5 20
P3 8 12 6 15

ow
Using matrix algebra, find:
(i) the total requirement of each material if the firm produces 70 units of Pj, 120 units of
P2 and 50 units of P3.
(ii) the per unit cost of production of each product if the per imit costs of materials
A^l/ M2, M3 and M4are ?10, ?12, ?15 and ?20 respectively,

e
Fl
re
(iii) the total cost of production.
64. Mr. Ashok went to a market to purchase 3 kg of sugar, 10 kg of wheat and 1 kg of salt. In a

F
shop near to Mr. Ashok's residence, these conunodities are priced at ?40, ?20 and ?16 per
ur
kg whereas in the local market these commodities are priced at ? 30, ?16 and ?12 per kg
or
respectively. If cost of travelling the local market is ?50, find the net savings of Mr. Ashok
using matrix multiplication.
sf
65. A company is considering which of the three methods of production it should use in
k
Yo
producing three goods A, B and C. The amount of each goods produced by each method is
oo

shown below:
Method A B C
eB

I [4 8 2
II 5 7 1
ur

in [5 3 9
ad
Yo

If the profit per unit on A, B and C are ?10, ?4 and ?6 respectively, using matrix
multiplication find which method maximizes total profit.
66 The manager of a car rental company is planning its maintenance programme for the next
d
Re

year and wants to determine the needs for certain repair parts and the expected costs. The
in

company rents out three types of cars: large, medium and small. The number of each size of
F

cars available for renting in the four regions is given below.


Region
I n III IV
Large 160 150 100 120
Medium 400 300 100 400
Small 500 200 150 300
Because of costs and frequency of replacement, the manager is concentrating on four repair
parts in particular: fan belts, spark plugs, batteries and tyres. On the basis of past records,
the average number of repair parts needed per car during a year are given below.
Large Medium Small
Fanbelts 1.7 1.6 1.5
Spark plugs 12 8 5
Batteries 0.9 0.75 0.5
Tyres 4 6.5 6
7.43
ALGEBRA OF MATRICES

If the costs (in ?) per unit for the four parts are as given below:
Fanbclt Spark plug Battery Tyre
80 300 350
Cost (in?) 125
Using matrix algebra, find:
(i) the total demand for each type of car.
(ii) the total demand for each repair part needed for the fleet,
(hi) the total combined cost for all repair parts.
67. A firm produces three products P2 and P3 requiring the mix-up of three materials
Ml, M2 and M3. The per unit requirement of each product for each material is as follows:
Material
Product Ml M2 ^3

ow
2 3 1

4 2 5
P2
2 4 2

Using matrix algebra find:


(i) the total requirement of each material if the firm produces 100 units of each product,

e
re
Fl
(ii) the per urut cost of production of each product if the per unit costs of materials
Ml, M2 and M3 are ? 50, ? 100 and ? 50 respectively,

F
(iii) the total cost of production if the firm produces 200 units of each product.
ur
68. In a certain city there are 50 colleges and 400 schools. Each school and college has 18 peons,

r
5 clerks and 1 cashier. Each college, in addition, has 1 section officer and 1 librarian. The fo
monthly salary of each of them is as follows:
ks
Peon Clerk Cashier Section officer Librarian
Yo
? 30,000 ? 50,000 ? 60,000 ? 70,000 ? 90,000
oo

Using matrix algebra find :


(i) the total number of posts of each kind in school and colleges taken together,
eB

(ii) the total monthly salary bill of all the school and colleges taken together.
ur

14 0 42
ad

0 7-2 5
Yo

1- (i) (ii) (iii) 18 -1 56


-7 10 3
0
[22 -2 70
d

■ 12 17 22 ' 1 14
Re

-3 -4 1
in

3. (i) AB = , BA does not exist (ii) AB= -4 -5 -6 , BA = -3 2


8 13 9 -4 -4 -4
F

0 0 0 0
-12 3
(iii) AB = [11],BA = I3-369 (iv) [a^ + b'^ + c'^ + d'^ + ac + bd\
2 -246
0 -1 1
4-20
6 16 26
(i) (ii) [82] (iii) 2 0-2 38 -10
-8 -18 -28
5 -2 -3

-2 -9 -1
cos 4 0 sin 4 0
3 26 3 ^43 “ ^22 “ ^
- sin 4 0 cos 4 0
35 15 34

.v=-2 x=-2,-1
0 0 ■ 39 55
-7 k=l
0 0 -55 -16
7.44
APPLIED MATHEMATICS-XII

1
32.
X = A,\i = -1 33. -
5
23
34. (i) X = - 3,5 (ii) a: = -1 (iii) .V = 4V3 (iv) .r = 0, —^
2
6 -14 10' 4 7 2
1 -4
35. -21 36 -25 36. 12 19 8 39.
3 -2
-3 5 -5 8 12 3
2 3 4 1 -2 -2 -5'
40. (i) 1 0 1
(ii) 2 0 (iii) [-1 2 1] (iv) [-4] (v) 3I 4 0
1 -2
4 2 187 -195
(vi) 2 0 41. 44.
-1 1 -156 148
-5 4

-1 -1 -3 1 1 3

w
46. -1 -3 -10 , 1 3 10 47. a = 2,b = 3, No 48. True when AB = BA

-5 4 2j [5 -4 -2
52. ? 157.80,? 167.40, ^281.40
55. (i) ? 15000 each (ii) ? 5000,? 25000

F lo
53. ?1597.20 54. ? 3400, ? 7200

ee
56. X: ? 30,000 Calories Proteins

Fr
y : ? 23,000 58. X : ? 9900
57. Family A : 24600 576
y: ? 21200
Family B: 15800
Z : ? 29,000 332
for
59. ? 90,000, ? 120,000 60. ? 10,000, ? 15,000
ur
61. Firm A ; ? 53,000; Firm B : ? 44,500 62. ?4840
s

FirmC:? 34,000
ook
Yo

63. (i) M^:1380, M2:1890, M3 :970, M4 : 3990 (ii) ?311, ?653, ?614 (iii) ?130,830
eB

64. ?24 65. Method III


66. (i) Large : 530, Small; 1200, Medium : 1150
(ii) Fan belts: 4546, Spark plugs: 21,710; Batteries: 1952; Tyres: 16,820 (iii) ?8,777,650
our
ad

67. (i) Mj: 800 units; M2 : 900 emits; M3 : 800 units (ii) : ? 450; P2 : ? 650, P3 : ? 600
(iii) ? 340,000
68. (i) Peons : 8100, Clerks : 2250, Cashiers : 450, Section officers : 50, Librarians : 50
Y

(ii) ? 390,500,000
Re
nd

HINTS TO SELECTED PROBLEMS


Fi

29. (ii) We have.


3 1
A =
-1 2

3 1 3 1 9-1 3 + 2 r 8 5
A^ = A/1
-1 2
1 2j " [-3-2 -1 +4j " [-5 3
8 5 -15 -5 7 0 8-15 + 7 5-5 + 0 0 0
A^-5A+7/ = + =0
-5 3 5 -10 0 7 -5 + 5 + 0 3-10+7 0 0

A^ = 5A -71

A^ = A^ A^ = (5A-71) (5A-71) = 5A (5A -7/)-71 (5A-71)


- 25 A^ -35A/ - 35/A + 49//
= 25A^ - 35A - 35A + 49/
7.45
ALGEBRA OF MATRICES

= 25A^-7QA+A9I
= 25 {5A-71)-70A+ 4:91
= 125A-1751 -70A +491
= 55^ -126/
-126 0 r 39 55
= 55 r -13 1]2 -126 1 0’ _ 165 55 +
0 1 " -55 no 0 -126 -55 -16

3 -2
30. We have, A = 4 -2

3-2 3-2 '9-8 -6 + 4 1 -2


A^ =
4 -2j [4 -2_ 12-8 -8 + 4 4 -4

ow
It is given that = kA-2l2
1 -2
= k [3 [1 O'
4 -4 4 -2\ [0 1
'1-2 3fc-2 -2k ^

e
4 -4 4k -2k-2

Fl
re
3Jc-2 =1, 4k = 4, -2k = -2 and -2k-2 = -4

F
k = 1
4
ur
1 2 0 0

34. (ii) [1 2 1] 2 0 1
[10 2
2 = 0 => [1 2 1]
or
.V =0 => 4 + 2a: + 2a: = 0 => x = -l
sf
X

10 2
k
Yo

37. We have, A = 0 2 1 and / (.i:) = - 6x^ + 7,v + 2


oo

2 0 3J
B

f{A) - A^ -6A^ +7A + 2I^


re

[10 2] 1 0 2 5 0 8

Now, A^ = AA 0 2 1 0 2 1 2 4 5
u

8 0 13_
ad

2 0 3 2 0 3
Yo

‘1 0 2
[5 0 8' 21 0 34
A^ = a'^ A = 2 4 5 0 2 1 ^ 12 8 23
d

8 0 13 2 0 3 34 0 55
Re
in

f{A) = A^ -6A'^ +7A + 2I2,


F

■7 0 14 2 0 0 0 0 0
[21 0 34 5 0 8
0 14 7 0 2 0 0 0 0 =0
/(A) = 12 8 23 -6 2 4 5 + +

8 0 13 14 0 21 0 0 2 0 0 0
34 0 55
3 2
Hence, A is a root of the polynomial f {x)=x -6x +7x+2.
A 2 3 -7 -8 -9'
(ii) We have to find a matrix A satisfying the equation A 4 5 6 2 4 6'

Clearly, the product of A with a 2 x 3 matrix is a 2 x 3 matrix. Therefore, A is a 2 x 2 matrix.


a b
Let A = . Then,
c d

A 2 3' 7 8 -9
A
4 5 6 2 4 6
L J
7.46
APPLIED MATHEMATICS-XII

a h 1 2 3 -7 -8 -9
c d 4 5 6 2 4 6

a + 4b 2a + 5b 3a + 6b' -7 -8 -9
c + 4d 2c + 5d 3c + 6d 2 4 6

a + 4b = -7, 2a + 5b = -8, 3a + 6b = -9
c + 4rf = 2, 2c + 5rf = 4, 3c + 6rf = 6
a = 1, b = -2,c = 2,d = 0
-2
0

ow
[5 01 0 0 1 0 0 0
48. (ii) A = 0 0
,B = (iii) A = ,B =
-1 0 0 0 2 1

2 0 3 0 ■-1 0]
(iv) A = ,B = , C =
0 0 0 0 0 0

e
55. Let ? X be invested in first bond and ^ i/ be invested in second bond. Let A be the investment

re
matrix and B be the interest per rupee matrix. Then,

rFl
F
5
100

r
A = [x y] and B =
ou
7 fo
ks
LiooJ
5
oo

100
Total annual interest =AB= [x y] = iL + Zl.
Y
B

7 100 100

LlOOj
re

Also,x + y = 30000
ou
Y

(i) If total interest is ? 1800. Then,


ad

5x 7^
100 100
= 1800 => 5x + 7y = 180000 ...(ii)
d

Solving (i) and (ii), we get: x = y = 15000.


in
Re

(ii) If total interest is ^ 2000. Then,


F

5x 7y
100 100
= 2000 => 5x + 7y = 200000 ...(iii)

Solving (i) and (iii), we get


X = 5000 and y = 25000
57. Let f be the family matrix and R be the requirement matrix. Then,
Men Women Children

^ Family A ’ 4 6 2

~ Family B [ 2 2 4

Calories Proteins
Men 2400 45
R = Women 1900 55
Children 1800 33
ALGEBRA OF MATRICES 7.47

Total requirement of calories and proteins of each of the two families is given by the matrix
product
Men Women Children Calories Proteins
Men 2400 45
Family A 4 6 2
FR = Women 1900 55
Family B 2 2 4
Children 1800 33

Calorics Proteins

Family A 24600 576


FR =
Family B 15800 332

ow
7.8 TRANSPOSE OF A MATRIX
T
DEFINITION U'fA=^] beanm II nmfrix. Then, the transpose of A, denoted by A
X or A', is mi n x m

matrix such that

e
Gji for all i = 1, 2. ...,m; j = l,2,...,n.

re
Frl
Thus, A^ is obtained from A by changing its rows into columns and columns into rows.

F
1 2 3"
1 2 3 4
T 2 3 2
For example, it A = 2 3 4 1 , then A
ou
or
3 4 1'
3 2 1 4
4 1 4
kfs
The first row of is the first column otA. The second row of /I ^ is the second column of A and
oo
so on.

7.8.1 PROPERTIES OF TRANSPOSE


Y
B

We shall now state and prove some properties of transpose of a matrix as theorems given below.
THEOREM 1 For any matrix A, (A^)^ =A.
re

T "T T
PROOF Let A = \ajj] be an m x n matrix. Then, A' is an n x m matrix and so (^ ) '
oYu

IS an m x n
ad

T' rr

matrix. Thus, the matrices A and {A‘) are of the same order such that
d

T T ^ T
{A‘) =

!‘ [By the definition of transpose]


in
Re

r
Ts
= (A)jj for all / = !, 2, and /=1,2,... r
F

Hence, by the definition of equality of tw'o matrices, we obtain


{A'^f = A. Q.E.D.
T
THEOREM 2 For any two matrices A and B of the same order, (A + B)
1>R001- Do yourself.
T T
THEOREM 3 If A is a matrix and k is a scalar, then (kA) = k{A^).

PROOF Do yourself.
THEOREM 4 1//4 and B are tzoo matrices such that AB is deifned, then {AB)
T
= B^A^.

PROOF Let A=[nij],„^ji and B=[bjj]„,,p be two matrices. Then AB is an mx p matrix and
therefore(AB)^ isa p x matrix. Since and B^ aren x »;and p x jjmatrices, therefore B^ A^
is a pxm matrix. Thus, the two matrices (AB) ^ and B^ A^ are of the same order such that
7.48 APPLIED MATHEMATICS-XII

[By the definition of transpose]

((ABf)ij■' = r Z= 1 '
[By the definition of matrix multiplication]
it

=>
{{ABf)^i= Z r = l
F [By commutativity of multiplication of numbers]

{{ABf)ij = Z (S’’),v (/l^) ■ [By definition of transpose]

{(AB)'')ij = (B^ [By definition of multiplication of matrices]


T
Hence, by the definition of equality of two matrices, we obtain {AB)
Q.E.D.

GENERALISATION If A, B, C are three matrices confirmable for the products (AB) C and A{BC), then

w
{A{BC)f = (C^B^)

F lo
REMARK The above law is called the reversal law for transposes i.e. the transpose of the product is the
product of the transposes taken in the reverse order.
ILLUSTRATIVE EXAMPLES

e
Fre
-1 for
'●●.AMPLEi ifA= 2 andB = [-2 -1 -4], verify that
3
r
SOLUTION We have.
You
oks

-1
A 2 andB = [-2 -1 -4]
eBo

-1 2 1 4
AB = 2 [-2 -1 -4] - -4 -2 -8
our
ad

3 -6 -3 -12

2-4 - 6‘
T
(AB) 1-2 -3 ...(i)
dY
Re

4 -8 -12
Fin

iT 2 -4 -6
-1 -2

Also, b”^ a'^ = [-2 -1 -4f 2 -1 [-1 2 3] = 1 -2 -3 ...(ii)


3 -4 4 -8 -12

From (i) and (ii), we observe that (AB)^ = 6^


’l 2 2
●i I
■ .
!fA= 2 1 2 is a matrix satisfying AA = 9/3, then find the values ofaand b.
a 2 b

SOLUTION We have.
1 2 2 1 2 a
A 2 1 -2 => = 2 12
a 2 b 2 -2 b

aa'^ = 9/3
7.49
ALGEBRA OF MATRICES

12 2 1 2 a 10 0
2 1-2 2 1 2 = 9 0 10
a 2 b 2 -2 b 0 0 1

9 0 a + 2b +4 9 0 0
0 9 2a+ 2-21) 0 9 0
0 0 9
a+ 2b + 4 2a+2-2b a^ + 4 + b^
=> a+2b + 4 = 0, 2a+ 2-2b =0 and a'^ +4+ b^ =9
a+2b + 4 = 0, a-b + \ =0 and a^+b^ =5
2 and b =-l.
Solving a + 2b + 4 = 0 and a-b + l = 0, we get: a =
0 2y 2
EXAMPLES Find the values of x,y,z if the matrix A = .r y -z satisfy the equation
X -y z_

w
T
A‘ A=I^.
SOLUTION

A =
We have,
■ 0
X
2y
y -z
2
=> /I
T
0

2y
F lo X

y -y
.V

e
2 -2 2
X -y 2

It is given that
Fre
for
A = h
0 X -t' 0 2y 2 1 0 0
r
2y y -y X y -2 0 1 0
You

0 0 1
oks

2 -2 2 X -y z
0
2.x2
eBo

0 1 0 0

0 6y^ 0 0 10
0 ‘O 32^ 0 0 1
ad
our

2x^=1, 6y^=l, 32^=1


1 1 1

^72'^=-76'^ = "T3
x=±
Re
dY

EXERCISE 7.4
Fin

2 1 O'
1. Let A =
-7
^5 and B - 2 -4 ' verify that

(i) (2A)'
T
= 2A
T
(ii) (A + = A^ +
(iii) {A-Bf = A^ -B”^ (iv) {ABf = A^
3
2. IfA=5 andB=[l 0 4], verify that (AB)^ = B^ a"^.
2

1 -1 0 1 2 3
3. Let A = 2 13 andB = 2 1 3 . Find A^, B^ and verify that
1 2 1 0 1 1

(i) (A + B)'^=A^ + b'^ (ii) (AB)^ = b”^ a'^ (iii) (2A)^ =2A^
7.50 APPLIED MATHEMATICS-XII

-2
4. IfA = 4
,B = [13 -6], verify that (AB)^
5

3 4
2 4-1
5. IfA =
-1 0 2 '^ = -1 2 , find (AB)^.
2 1

1 -1
2 1 3
6. (i) For two matrices A and B, A =
4 1 0
0 2 verify that {ASf =b'^ a'^.
5 0

1 3 1 4
(ii) ForthematricesAandB,verify that(AB)^ =b'^ A^,whereA =
2 4
, B=
2 5

w
3 4
r -1 2 1
7. If A -1 2 and B = find A^-B^.
12 3'
0 1

Flo
9. If Ij, nil, w’ =1/ 2, 3 denote the direction cosines of three mutually perpendicular vectors

e
r^i "'1 "ll

re
in space, prove that AA ^ =/,whereA= Ij nh ih ■

F
_h ”'3 "3.
ANSWERS
ur
r
5.
0 1
7. -3
4 3

0
fo
ks
15-2
-1 -2
Yo
oo

7.9 SYMMETRIC AND SKEW-SYMMETRIC MATRICES


B

SYMMETRIC MATRIX A square matrix A = lajjj is called a symmetric matrix, ifajj=aji for ail i,j.
re

'3-1 r
For example, the matrix A = -1 2 5 is symmetric, because
u
ad

!_ 1 5-2
Yo

^*12 = “ 1 = '^21' = ^31' ^^23 = '^32 ~ ^ji /-


It follows from the definition of a symmetric matrix that A is symmetric, iff
d
Re
in

~ ^ji for all i, j


<=>
{A)jj = (A^)ij for all
F

o A = a”^.
T
Thus, a square matrix A is a symmetric matrix iff A = A.
a h g 1 + i 1 3
T
Matrices A = h b f , B = 1 2 3 + 2/ are symmetric matrices because A =A
3 3+2/ 4
J f
and B'^ = B.

A square matrix A = (aij] is a skew-symmetric matrix ifoij = -ay for all i, j.


■ 0 2 -3'
For example, the matrix A = -2 0 5 is skew-symmetric, because
3-5 0

a-12 = 2, (721 = -2 => ^12 =“«2h =- 3,(731 = 3 => ^13 =“'^31/


and. (?23 =5,(?32 =-5 => (723 =“^32
ALGEBRA OF MATRICES 7.51

It follows from the definition of a skew-symmetric matrix that A is skew-symmetric iff


<=>
Ojj for all i,j
o
{A),! = - for all i, j
A=- a'^
T
<=> A = - A.
T
Thus, a square matrix A is a skezv-si/mmefric matrix iff A = ~A.

0 2 3 0 -3 5'
Matrices A = -2 0 4 ,B = 3 0 2 are skew-symmetric matrices, because
-3 -4 0 -5 -2 0
T T
A' =-AandB' =-B.

w
ILLUSTRATIVE EXAMPLES

F lo
LXAMPLE1 Show that the elements on the main dia^qonal of a skew-symmetric matrix are all zero.

ee
SOLUTION Let A = [ajj] be a skew-symmetric matrix. Then,
for all i, j [By definition]

Fr
-a;;
l>
% = - for all values of/

2% = 0
for
ur
Ojj = 0 for all values of /
- ^22 - ^33 - ●●● aW1 = 0.
ks
0 a 3
Yo

EXAMPLE 2 If the matrix A = 2 b -1 is skew-symmetric, find the values of a, b and c.


oo

c 1 0
eB

SOLUTION For a skew-symmetric A = [Ojj], we have


Ojj = - Ojj for all i ^ j and Ojj = 0 for all i
r
ou

To a 3
ad

Thus, if A = 2 b -1 is skew-symmetric, then A22 = 0, A-j2 = - A21 and A^^ =-A23.


Y

c 1 0
nd
Re

b =0, a = -2 and t: = - 3
Fi

'0 a 3'
ALITER If A - 2 b -I is skew-symmetric, then
c 1 0

a'^ =-a
-iT
0 a 3 0 rt 3

2 b -I 2 b -1

c 1 0 c 1 0

0 2c 0 -a -3

a b 1 -2 ~b 1

3-10 -c -1 0

2 = -a, c = -3 and b =-b => a = -2, c = -3 and 2/j = 0 => a = -2, b = 0 and c = - 3
7.52 APPLIED MATHEMATICS-XIl

EXAMPLE 3 Let A be a square matrix. Then,


T
(i) A + A is a symmetric matrix
(ii) A-A is a skew-symmetric matrix.
(iii) AA' and A A are symmetric matrices.
SOLUTION' (i) Let P = A + ^^.Then,
= {A + A'^f = a'^ + (A'^f [v (A + Bf =a'^ + b'^]
= a'^ + A [v (A'^f=A]

ow
P'^ = A + A^ = P [By commutativity of matrix addition]
P is a symmetric matrix.
(ii) Let Q = A - /l^.Then,
qT = (A - A'^f = a'^ - (aY [v {A + Bf =A^^ + b'^]

e
T T

re
Q' = A^ -A [v

qT =- {A-A'^) = -q

Frl
F
=> Q is skew-symmetric
(iii) We have,
ou
or
{AA'^f = (A'^f kfs [By reversal law]
= AA
T
[V {A'^)T = /I]
AA^ is symmetric
oo

Similarly, it can be proved that A^ A is symmetric.


Y
B

EXAMPLE 4 Prove that every square matrix can be uniquely expressed as the sum of a symmetric matrix
and a skew-symmetric matrix.
re

SOLUTION Let A be a square matrix. Then,


oYu

^(A + A^) + ^{A -A”^) = P + Q (say), where P = ^(A + A^) and Q = i(A -A^).
ad

A =

p^=fi{A + A^)lJ =i(A + A^)


d

Now, [v (kAf =kA'^]


,2 2
in
Re

P^ = i(A^+(A^)^) [v {A + Bf =a'^ + b"]


F

PT 1(a'^+A) [V (A^)^=A]

P'^ = -(A + a”^) - P


z
[By commutativity of matrix addition]
P is a svmmetric matrix.
1 7* 1
Ua-a^)
r
Also,
V ^
= ^(A-A'^f
T T
Q -A) = - - Q

Q is a skew-symmetric matrix.
Thus, A = P + Q, where P is a symmetric matrix and Q is a skew-symmetric matrix.
Hence, A is expressible as the sum of a symmetric and a skew-symmetric matrix.
Uniqueness: If possible, let A=R + S, where R is symmetric and S is skew-symmetric. Then,
ALGEBRA OF MATRICES 7.53

a'^ = (R+S)'^=R^ + S^
= R - S [■: R^ =R andS^ =-S]
Now, A - R + S and a'^ = R - S
R
~{A+a'^) = P, \(A-A^) = Q-

Hence, A is uniquely expressible as the sum of a symmetric and a skew-symmetric matrix.


LXAMPLE 5 If A and B are symmetric matrices, then show that AB is symmetric iff AB = BA i.e. A and
B commute.

SOLUTION AB is symmetric
o {ABf = AB

w
[v =B^ a'^]
T
o b'^ a'^ = AB
<=> BA = AB [●.● A and B are symmetric matrices A
T
= A,b'^ =B]

F lo
EXAMPLE 6 Show that the matrix B^AB is symmetric or skew-symmetric according as A is symmetric
or skeiv-symmetric.

ee
T
SOLUTION CASE 1 Let A be a symmetric matrix. Then, A = A.

Fr
Now, {B^ ABf = b'^ a'^ (B'^f [By reversal law]
(B^ AB)‘
T T T
B‘ A' B for [v (B^)’'=B]
ur
(B^ AB)'^ = B^ AB [v a'^ = A]
T
s
B AB is a symmetric matrix.
ok
Yo

r
CASE il Let /I be a skew-symmetric matrix. Then, A = ->l.
o
eB

Now,
(B'^ ABf = B^ a'^ [By reversal law]
(B^ ABf = b'^ A'^ B [V (B^)^=Bj
r
ou
ad

(b'^ ABf = B^(-A)B [V A^=-A]


(B”^ ABf = - b'^ AB
Y

b"^ AB is a skew-symmetric matrix.


Re
nd

EXAMPLE 7 Let A and B be symmetric matrices of the same order. Then, show that
Fi

(i) A + B is a symmetric ynatrix.


(ii) AB - BA is a skezu-symmetric matrix.
(iii) AB + BA is a symmetric matrix.
T T
SOLUTION Since A and B are symmetric matrices. Therefore, A =AandB =B.
(i) We have,
{A + Bf =a'^ + b'^ =A + B [v A^=A,B^=B]
A + B is symmetric
(ii) We have,
(AB-BAf = (ABf - (BA)
T

{AB-BAf = b'^ a'^ - A^ B^ [By reversal law]


{AB-BAf = BA - AB [v b"^ =6, A'^ =A]
{AB-BAf = - (AB-BA)
7.54 APPLIED MATHEMATICS-XII

AB - BA is skew-symmetric.
(iii) We have.
(AB + BAf ={ABf +{BA)
T

[By reversal law]


= BA+AB [vA'^ =A, b'^ =B]
=:AB+BA

AB + BA is symmetric matrix.
3 2 3

EXAMPLES Express the matrix A = 4 5 3 as the sum of a syimuetric and a skerv-si/wmetric


2 4 5

matrix.

SOLUTION We have,

w
3 2 3 3 4 2
T
A = 4 5 3 => A 2 5 4

F lo
2 4 5 3 3 5

3 2 3 3 4 2 6 6 5

ee
T
A + A 4 5 3 + 2 5 4 6 10 7

Fr
2 4 5 3 3 5 5 7 10

3 2 3 3 4 2 0 -2 1
for
T
and. 4-/1 4 5 3 - 2 5 4 = 2 0-1

|_ 3 3 5
ur
2 4 5 -1 1 0
s

3 3 5/2 0 -1 1/2
ook

-{A+a'^) = and, Q =1(4-4^) =


Yo

Let P = 3 5 in 1 0 -1/2
2
5/2 in 5 -1/2 1/2 0
eB

T
3 3 5/2 3 3 5/2
T
Then, P 3 5 7/2 3 5 7/2 = P
our
ad

5/2 7/2 5 5/2 7/2 5


iT
0 -1 1/2 0 1 -1/2 0 -1 1/2
T
Y

and, Q 1 0 -1/2 -1 0 1/2 1 0 -1/2 = -Q


Re

-1/2 1/2 Oj [1/2 -1/2 0 -1/2 1/2 0


nd
Fi

Thus, P is symmetric and Q is skew-symmetric.


3 3 5/2
0 -1 5/2] 3 2 3

Also, P + Q = 3 5 7/2 + 1 05/2 = 4 5 3 = 4


5/2 7/2 5 -1/2 5/2 0 2 4 5

Thus, 4 is expressible as the sum of a symmetric matrix P and a skew-symmetric matrix Q.


EXAMPLE 9 Show that aU positive integral powers of a symmetric matrix are symmetric.
SOLUTION Let 4 be a symmetric matrix and u e N. Then,
4” = 444 ... 4 upto n-times
(4")^ = (444... 4 upto »-times)^
(4'')^ = (4^ 4^ 4*^ ... 4^ upto Ji-times) [By reversal law]
(4")'^ = (4'^)" = 4 n
[V 4^=4]
Hence, 4" is also a syrrunetric matrix.
ALGEBRA OF MATRICES 7.55

EXAMPLE 10 Shozv thni positive odd intcgm! powers ofn skezv-spmnictric matrix are skeiv-symmetric
and positiiye even integral powers of a skew-symmetric matrix are symmetric.
SOLLJTION Let A be a skew-symmetric matrix. Then,/4 ^ = - A.
We have, (A")^=(A^)'' for all neiV. (See Example 6]
T
{A’Y = (-A) [v =-A]
It

{A’Y = (-1)"
II

n
ikT A if n is even
(A") II if J7 is odd
-A

ow
Hence, A” is symmetric if n is even and skew-symmetric if n is odd.
EXAMPLE n A matrix which is both symmetric as zoell as skew-symmetric is a null matrix.
SOLUTION
Let A = [ojj] a matrix which is both symmetric and skew-symmetric.
Now, A = [Ojj]is a symmetric matrix

e
re
for all /, j

Fl
F
Also, A = is a skew-symmetric matrix.
ur
Ojj = -aji for all/,/

r
fl;;
Ojj for all /, / fo
From (i) and (H), we obtain
ks
Ojj = - Ojj for all/,/
Yo
oo

2ajj = 0 for all /, /


Ojj = 0 for all /, /
eB

A = [Ojj] is a mill matrix.


EXERCISE 7.5
ur

2 3 7
ad

1. IfA = , prove that A - A is a skew-symmetric matrix.


Yo

4 5

2. IfA= 1? 4
, show that A-A
T
is a skew-symmetric matrix.
d

-1
Re
in

5 2 X

3. If the matrix A= y z -3 is a symmetric matrix, find x, y, z and t.


F

4 t -7

3 2 7
4. Let A = 14 3. Find matrices X and Y such that X + V = A, where X is a symmetric
-2 5 8

and y is a skew-symmetric matrix.


'4 2 -1^
5. Express the matrix A = 3 5 7 as the sum of a symmetric and a skew-symmetric
1 -2 1

matrix.

2 4
6. Define a symmetric matrix. Prove that for A = 5 6 , A + A^ is a symmetric matrix where
T'

A is the transpose of A.
7.56 APPLIED MATHEMATICS-XII

3 4 1
7. Express the matrix ^ ^ ^ as the sum of a symmetric and a skew-symmetric
matrix.
3 -2 -4

8. Express the matrix 3 -2 -5 as the sum of a symmetric and skew-symmetric matrix


-1 1 2

and verify your result.


2 3 T
9. For the matrix A = , find + .4 and verify that it is a symmetric matrix.
5 7

ow
ANSWERS

3 3/2 5/2 0 1/2 9/2


3. X = 4,1/ = 2, zeC,t = -3 4. X = 3/2 4 4 -1/2 0 -1
5/2 4 8 -9/2 1 0

e
re
5. Symmetric matrix = 5/2
4

rFl
5/2
5
0
5/2 , Skew-symmetric matrix = 1/2
0 -1/2
0
-1
9/2

F
0 5/2 1 1 -9/2 0

r
3 - 3/2 0 -5/2
7. Symmetric matrix = , Skew-symmetric matrix = 5/2
ou
-3/2 -1 fo 0
ks
3 1/2 -5/2 0 -5/2 -3/2
8. Symmetric matrix = 1/2 -2 -2 , Skew-symmetricmatrix = 5/2 0 -3
oo

-5/2 -2 2 3/2 3 0
Y
eB

4 8
9. A =
8 14
ur

MULTIPLE CHOICE QUESTIONS (MCQs)


ad

1 0 o]
Yo

1. If A = 0 1 0 , then is equal to
b -1
d

a
Re
in

(a) a null .matrix (b) a unit matrix (c) -/I (d) 71


T T
2. If A and B are symmetric matrices of the same order, then AB‘ -BA‘ is a
F

(a) skew-symmetric matrix (b) null matrix


(c) symmetric matrix (d) none of these
3. If A and 6 are two matrices such that AB = A and BA = B, then B is equal to

(a) B (b) A (c) 1 (d) 0


4. If AB = A and BA = B, where A and B are square matrices, then
(a) B^ = B and A^ = A (b) 6^ BandA^=A
(c) B^ = B (d) A^#A,B^5iB
5. If A and B are two matrices such that AB = B and BA. = A, then A + B is equal to
(a) 2 AB (b) 2 BA (c) A + B (d) AB
1 0 0

6. The matrix A = 0 2 0 IS

0 0 4
7.57
ALGEBRA OF MATRICES

(a) identity matrix (b) symmetric matrix


(c) skew-symmetric matrix (d) diagonal matrix
7. If the matrix AB is zero, then
(a) It is not necessary that either A =0 or, B =0 (h) A = O or B = O
(c) A = O and B = O (d) all the above statements are wrong
r 0 -5
8. Tl'ie matrix A = 5 0 12 is a

-8 -12 0

(a) diagonal matrix (b) symmetric matrix


(c) skew-symmetric matrix (d) scalar matrix

w
9. If A, B are square matrices of order 3, A is non-singular and AB = O, then B is a
(a) null matrix (b) singular matrix (c) unit matrix (d) non-singular matrix

Flo
n 0 0 ^2 ^3
10. If^= 0 n 0 andS= b-] ^3 / then AB is equal to
0 0 ^2 ‘^3.

ee
n

n
(d) A + B

Fr
(a) B (b) nB (c) B
A 3
2-1 3
n. ifA = and 6 = 4 -2 , then for
ur
-4 5 1
1 5
s
(a) only AB is defined (b) only BA is defined
k
Yo

(c) AB and BA both are defined (d) AB and BA both are not defined
oo

'l 2 X 1 -2 y
eB

12. IfA = 0 1 0 andB= 0 1 0 and AB=/3, then x + y equals


0 0 1 0 0 1
r

(a) 0 (b) -1 (c) 2 (d) none of these


ou
ad

1 -1 n 1
13. lfA =
-1 -1
and (A + B)^ A^ + B^, then values of a and b are
Y

l2
(a) a = A,b =1 {h) a = \,b = A (c) a = 0,b = A (d) a = 2,b = A
Re
nd

o. R o
14. IfA = ^ is such that A =/, then
Fi

LY -aj
(a) l + a^-KPY=0 (b) 1-a^+py = 0 (c)l-a^-Py=0 (d)l+a^-Py-0
15. If S^[sjj] is a scalar matrix such that s,7 = and A is a square matrix of the same order, then
AS = SA=?

(a) a’^ (b) k + A (c) kA (d) kS

16. If A is a square matrix such that A^ = A, then (/ +A)^-7A is equal to


(a) A (b) I-A (c) 1 (d) 3A
17. If a matrix A is both symmetiic and skew-symmetric, then
(a) A is a diagonal matrix (b) A is a zero matrix
(c) A is a scalar matrix (d) A is a square matrix
■ 0 5-7'
18. The matrix -5 0 11 is
7 -11 0
7.58 APPLIED MATHEMATICS-XII

(a) a skew-symmetric matrix (b) a symmetric matrix


(c) a diagonal matrix (d) an upper triangular matrix
19. If A is a square matrix, then AA is a
(a) skew-symmetric matrix (b) symmetric matrix
(c) diagonal matrix (d) none of these
20. If A and B are symmetric matrices, then ABA is
(a) symmetric matrix (b) skew-symmetric matrix
(c) diagonal matrix (d) scalar matrix
5 x' T
21. lfA = and A= A , then
y 0.

ow
(a) x = 0,y=5 (b) x + y=5 (c) x = \j (d) none of these
22. If is 3 X 4 matrix and B is a matrix such that A^B and BA^are both defined. Then, B is of
the type
(a) 3x4 (b) 3x 3 (c) 4x4 (d) 4 X 3

e
23. If A = [Ojj] is a square matrix of even order such that Ojj =

Fl then

re
(a) y4 is a skew-symmetric matrix and | | = 0

F
(b) A is symmetric matrix and | /I | is a square
ur
(c) A is symmetric matrix and | A | = 0

r
(d) none of these.
24. IfA and B are square matrices of the same order, then {A + B) (A -B) is equal to
fo
ks
(a) A^-B'^ (b) A^-BA-AB-B^
Yo
oo

(c) A^-B^ + BA-AB (d) A^-BA + B^+AB


■ 2 0-3'
B

25. U A= A 3 1 is expressed as the sum of a symmetric and skew-symmetric


re

-5 7 2

matrix, then the symmetric matrix is


u
ad
Yo

2 2-4 2 4 -5 4 4-8 1 0 0
(a) 2 3 4 (b) 0 3 7 (c) 4 6 8 (d) 0 1 0
d

-4 4 2 -3 1 2 -8 8 4 0 0 1
Re
in

26. Out of the following matrices, choose that matrix which is a scalar matrix:
0 0 0
F

0 0 0 0 0
(a) 0 0
(b) 0 0 0
(c) 0 0 (d) 0
0 0 0

27. The number of all possible matrices of order 3x3 with each entry 0 or 1 is
(a) 27 (b) 18 (c) 81 (d) 512
28. Which of the given values of x and y make the following pairs of matrices equal?
’3x + 7 5 0 J/-2
and.
y + 1 2 - 3:c 8 4

(a)x = -i,y=7 (b)y=7,;c = -| (c) x =


2
- (d) Not possible to find
5
0 3a
29. If A = 3 _4 andfcA = 2b 24 , then the values of k, a, b, are respectively

(a) -6,-12,-18 (b) -6,4,9 (c) -6,-4,-9 (d) -6,12,18


n
ALGEBRA OF MATRICES 7.59

B
30. If matrix ^ =^]2x2 'Where = jy' , then A is equal to

l
(a) I (b) /\ (c) O (d) -I
1 -5 7
31. The trace of the matrix >1 = 0 7 9 IS

f
11 9

(a) 17 (b) 25 (c) 3 (d) 12

u
32. If A = [fl^yjis a scalar matrix of order ?ix jj such that =^: for all/, then traceof Aisequal to
/ \ ”
(a) nk (b) n + k (d) none of these

i
0 0 4

33. The matrix A = 0 4 0 IS a

4 0 0

(a) square matrix (b) diagonal matrix (c) unit matrix (d) none of these
34. The number of possible matrices of order 3x3 with each entry 2 or 0 is
(d) none of these
F

(a) 9 (b) 27 (c) 81


e
2.t + y 4.V 7 7y-13'
35. If
5.V-7 4.\-
, then the value of + y is
y
o

(a) -Y = 3, y =1 (b) Y = 2, y = 3 (c) X = 2, y = 4 (d) .r = 3,y = 3


36. If A is a square matrix such that A^ =/, then(A -f)^ +(A +f)^ -7A is equal to
(a) A (b) I-A (c) f + A (d) 3A
37. If A and B are two matrices of order 3 x m and 3 x jj respectively and in = n, then the order of
5A-2Bis

(a) m X 3 (b) 3x 3 (c) ni.x n (d) 3xjj


38. If A is a matrix of order ?»x and 6 is a matrix such that A6^ and B‘ A are both defined, then
ad

the order of matrix B is

(a) mxn (b) ;ix n (c) nx m (d) mxn


... ^
If A and B are matrices of the same order, then AB - B A is a
Re

(a) skew-symmetric matrix (b) null matrix


(c) unit matrix (d) symmetric matrix
ANSWERS

1. (b) ■ (d) . (a) 4. (a) (c) 6. (d) 7. (a) . (c) ●. (a)


10. (b) II (c) ■■ (a) (b) : . (c) I-. (c) h. (c) (b) (a)
19, (d) (a) (c) 22. (a) (d) ■ (c) 2i:. (a) (a) (d)
- (d) (c) (a) . (a) (a) (d) (d) (b) (a)
(d) (d) (a)

If A and B are two matrices of orders nx 3 and 3xh respectively such that AB exists and is of
order 2x4. Tlien, (a, b) =
If P and Q are two matrices of orders 3xn and nxp respectively then the order of the matrix
PQ is
7.60 APPLIED MATHEMATICS-XII

-1 2 3x

3. = 2y 4 -1 is a symmetric matrix, then the value of 2;r + y is


6 5 0

4. If fl, 1; are positive integers such that <7 <1? and [^7 b] = 25, then {a, b) =
b

1
2 3 6
5. UA = 3 B = and AB = I, then x =
0 -1
0 2x-3

a:
nI 2
6. l(A = satisfies the equation A^ = O, then x =

ow
-1

7. If A is an m x n matrix and 6 is a matrix such that both AB and BA are defined, then the order
of B is
0 2 0 1 2 3

e
8. UA = 0 0 3 and B = 3 4 5 , then (/IB) 33 =

re
-2 2 0

Fl 5 -4 0

F
3 0 0
ur
9. UA = 0 3 0 , then/l'^=

r
0 0 3^ fo
ks
ni. [f/l=diag(2,-l, 3), B =diag(-l, 3, 2), then/1^6 =
Yo
oo

11. -1 [2 1 -1] =
eB

a: 1
ur

12. IfA = and A is the identity matrix, then x =


1 0
ad
Yo

1 1
13. Ife , then a: =
1 1
d

0 3£?'
Re

0 2
in

!4. IfA = and kA = then(fc, a, b) =


3 -4 2b 24 '
F

15. The negative of a matrix is obtained by multiplying it by


T T
16. If A is 3 X 4 matrix and B is a matrix such that A B and BA are both defined. Then the order
of B is
1

17. IfA = 2 , then


3
T —1
18. If A is a non-singular matrix, then (A ) =
19. If A, BandC aremx m,«x pand px g matrices respectively such that (BC) A is defined, then
m =

2 0 0

20. If A = 0 2 0 such that A^ = >.A, then A. =


0 0 2
7.61
ALGEBRA OF MATRICES

a b 1 1
21. If the matrices A = and B = commute with each other, then C =
c d 0 1

4 x+2
22. If A =
2x-3 x +\
is a symmetric matrix, then .r =
23. If A and B are two skew-symmetric matrices of same order, then AB is symmetric
if.

If A and S are matrices of the same order, then (3A - 2B) is equal to

w
24.

25. Addition of matrices is defined if order of the matrices is


26. If A and B are symmetric matrices of the same order, then AB is symmetric iff.
If A is symmetric matrix, then B AB is

e
2
.... matrix,
28. If A is a skew-symmetric matrix, then A is a

ro
re
matrix.
If A is a symmetric matrix, then A is a
->n

30. If A is a skew-symmetric matrix, then kA is a... .{kis any scalar).

F
If A and B are symmetric matrices of the same order, then
(i) AB - BA is a
(ii) BA - 2AB is a
uFl
sr
32. In applying one or more row operations while finding A ^ by elementary row operations,

ko
v/e obtain all zeroes in one or more row, then A~^
o
13, The product of any matrix by the scalar
of is the null matrix.
34. A matrix which is not a square matrix is called matrix.
o
matrix.
The sum of two skew -symmetric matrices is always
Y

A and B are square matrices of the same order, then


erB

(i) {ABf (ii) (kAf (iii) {k{A-B)f =


uY

where k is any scalar.


37. matrix is both symmetric and skew-symmetric matrix,
Matrix multiplication is over matrix addition.
ad
do

ANSWERS
in

3. 5 4.
(3, 4) 5. 1
(2,4) 3x p
Re

● ±1 / ● X m 8. 4 9.
F

2 1 -1

12. 0 13.
10. diag(-4, 3,18) V]. -2 -1 1 x = -l,y=-l
4 2 -2

1 2 3

3x4 2 4 6
(-6,-4,-9) -1
3 6 9

IvT 16 0 5
(A-^)
T T
AB = BA 3A' -2B same AB = BA

symmetric symmetric symmetric skew-syrmnetric


(i) skew-symmetric (ii) neither symmetric nor skew-symmetric
does not exist zero ' rectangular 35. skew-symmetric
7.62 APPLIED MATHEMATICS-XII

36. (i) b’'a^' (ii) kA


T
(iii) k{A'^ -B'^) 37. nitll matrix

38. distributive

VERY SHORT ANSWER QUESTIONS (VSAQs)


Anszoer each of the folloiving questions in one ivord or one sentence or as per exact requirement of the
question:
1. If A is an m x n matrix and Bisnx p matrix does AB exist? If yes, write its order.
'3 -l'
'2 1 4
2. If/I = and B= 2 2 . Write the orders of AB and BA.
4 1 5
1 3

4 3 4
3. UA = and B =
1 2 ^ , write6.
1

w
T
4. IfA = 2 , write/4/\' .

F lo
3

5. Give an example of two non-zero 2x2 matrices A and B such that AB =0.
2 3 T
6. IfA = , find A + A .

ree
5 7

0 a -3
for F
7. If the matrix A = 2 0-1 is skew-symmetric, find the values of 'a' and 'b‘.
b I 0
Your
ks

8. Let/I and Bbematrices of orders 3 x 2and2x 4respectively. Writetheorder of matrix/46.


eBoo

3 5
9. HA =
7 9
is written as A = P +Q, where as A = P + Q, where P is symmetric and Q is

skew-symmetric matrix, then write the matrix P.


ad
our

'1 0 0
10. It
y 5 "21 2 = i, where / is 2 x 2 unit matrix. Find .t and y.
1 -1
Re

11. If^ = , satisfies the matrix equation A = kA, write the value of k.
Y

-1 1
Find

1 1 4
12. If A = satisfies A = XA, then write the value of X.
1 1

-1 0 0
13. lfA = 0-1 0 ,fmdA^.
0 0-1

-1 0 0
14. If/1 = 0-1 0 ,find^^.
0 0-1

- 3
15. UA =
0 2 ,findi4‘^.
■16. If[l- 2] = 2, find A'
ALGEBRA OF MATRICES 7.63

17. \i A = [Ojj] is a 2 X 2 matrix such that a^j - i + 2j, write A .


2 3 3-6"
18. Write matrix A satisfying A + -1 4 3 8 ■

19. If A =[Ojj] is a square matrix such that Ojj =i -j, then write whether A is symmetric or
skew-symmetric.
20. For any square matrix write whether AA~^ is symmetric or skew-symmetric.
21. If A = [i^y] is a skew-symmetric matrix, then write the value of 2 cijj .
22. If A = [ajj] is a skew-symmetric matrix, then write the value of S I Oj: .
i j ’

ow
23. If A and B are symmetric matrices, then write the condition for which AB is also symmetric.
24. If B is a skew-symmetric matrix, write whether the matrix AB A^ is symmetric or skew-
symmetric.
Zr. If B is a symmetric matrix, write whether the matrix AB A^ is symmetric or skew-

e
symmetric.

re
Fl
26. If/4 is a skew-symmetric and^i e N such that(/4")^ =XA^\ write the value of X.

F
27. If/4 is a symmetric matrix and »eN, write whether A” is symmetric or skew-symmetric
ur
r
or neither of these two.

28. If /I is a skew-symmetric matrix and n is an even natural number, write whether A’' is fo
ks
symmetric or skew-symmetric or neither of these two.
Yo
29. If A is a skew-symmetric matrix and n is an odd natural number, write whether A^‘ is
oo

symmetric or skew-symmetric or neither of the two.


eB

30. If A and B are symmetric matrices of the same order, write whether AB - BA is symmetric
or skew-symmetric or neither of the two.
31. Write a square matrix which is both symmetric as well as skew-symmetric.
ur

1 3'
y 0] _ [5 6
ad

32. Find the values of x and y, if 2


Yo

-I-
0 -Y 1 2 “1 8 ●

.Y-H 3 4 5 4
d

33. If , find X and y.


y - 4 .Y + y 3 9
Re
in

2.Y-y 5 6 5 ■
34. Find the value of .y from the following: ^
F

y 3 -2 ●

35. Find the value of i/, if


r Y-y 2 2 2
X 5 3 5 ■

3a:+ y -y 1 2
36. Find the value of x, if
2y-x 3 -5 3 ●

37. Ifmatrix/1 =[1 2 3],writeAA^.


38. If 2.Y -I- y 3y _ 6 0 , then find .y.
0 4 6 4

1 2 T
39. IfA = , find A + A .
3 4

n +b 2 6 5
40. If 5 b 2 2
, then find a.
7.64 APPLIED MATHEMATICS-XII

41. If is a matrix of order 3x4 and 6 is a matrix of order 4x3, find the order of the matrix of
AB.
-1 0 -1 1

42. If [2 1 3] -1 1 0 0 = A, then write the order of matrix A.


0 1 1 -1

1 21T3 1 '7 11
43. If , then write the value of k.
3 4 2 5 k 23

44. If / is the identity matrix and A is a square matrix such that/4 = A, then what is the value of
(1+Af-3A7
1 2
45. UA =
0 3 is written as B + C, where B is a symmetric matrix and C is a skew-symmetric

w
matrix, then find B.
46. If A is 2 X 3 matrix and B is a matrix such that/I ^B and BA^ both are defined, then what is

F lo
the order of B?
47. What is the total number of 2 x 2 matrices with each entry 0 or 1?

ee
.Y 3 1
48. If , then find the value of y.

Fr
2.v + y 7 8 7

49. If a matrix has 5 elements, write all possible orders it can have. for
50.
Fora2x 2matrix4 =[rty] whose elements are given by ,write the value ofrt|2-
ur
J
oks

51. If x\l3 -1 10
find the value of .v.
Yo

1 5 '
o
eB

9 -1 4 1 2 -1
52. If = A + , then find matrix A.
-2 1 3 0 4 9
our
ad

n-b 2n + c 1 5'
53. If , find the value oft.
2n-b 3c+ d 0 13
Y

0 1 -2
Re

54. For what value of .y, is the matrix A = -1 0 3 a skew-symmetric mahix?


nd

.Y -3 0
Fi

'2 -2
55. If matrix A =
-2 2
and A =pA, then write the value ofp.

56. UA is a square matrix such that = A, then write the value of 7A -(/ + A) where / is the
identity matrix.
3 4 ^ '/ '7 0
3/ . If 2 +
, find X - y.
5 -Y 0 1 10 5

1 0
58. IflY 1] = 0, find -Y.
-2 0

a +A 3b 2n + 2 b +2
59. If , write the value of a-2b.
8 -6 (i-8b
GEFRAOFMATn r 7.65

Write a 2 X 2 matrix which is both symmetric and skew-symmetric.


‘ xy 4 8 w
If write the value of i/ + z).
2 + 6 .V +1/ 0 6'
-3/+/I
, if j
: Construct a 2 x 2 matrix A = whose elements ay are given by Ojj = ■ 2

{1+})^ ,ifi=j
2 1
If X + y
x-yj“[4 3 2 , then write the value of (x, y).
■ 0 2& -2“
Matrix/4 = 3 13 is given to be symmetric, find the values of rt and

ow
3fl 3 -1

I 1 Write the number of all possible matrices of order 2x2 with each entry 1, 2 or 3.
ANSWERS

e
Fl
re
12 3
-7
Yes, m X p ... 2x2 and 3x3 2 4 6

F
2
3 6 9
ur
or
2 0 0 0 '4 8'
. A = rB = . a = -2,b=3
3 0 2 -1 8 14
sf
3 6
, .Y = 0,y = -2 ;; 2
k
3x4
Yo
6 9
oo

‘81 0
- A or, !4. A
B

0 81
e

3 5 1 -9
1
-2
'● -2 4
I
skew-symmetric
ur

4 6
ad

: symmetric 0 0 AB = BA
Yo

;/
skew-symmetric symmetric (-1) symmetric
d

symmetric skew-symmetric ■ skew-symmetric ● I null matrix


Re
in

x = 3,y = 3 x = 2,y=7 x =2 y=l


2 5'
F

x = l 14 X = 3, y - 0 5 8

4 3x 3 ● 1x1 17

1 1
/ , 2x 3 16
■ 1 3

48. 2 4<.. 1 X 5,5 X 1 -● 1/2 x = 3


8 -3 5
b=l "I x =2 4 JO. -f
■ -2 -3 -6
0 0
57. x = 2,y = -S x=2 0 6C f)l. 0
0 0

■ 4 1/2 2 3
62. 6'.^. (-1,1) M. 6? 3^=81
5/2 16
CHAPTEi s
DETERMINANTS

8.1 DETERMINANTS

DEFINITION Ever]/ square matrix can be associated to an expression or a number which is known as its
determinant. If A = [a^j\ is a square jnatrix of order n, then the determinant of A is denoted by det A or,
\A \ or,

w
rtll a-12 ■■■ ■■■ 0^1

F lo
^21 «22 ●●● ●●● ^2»

Oj] Oj2 ■■■ ajj ... Oj„

e
Fre
^i2 cijjj
a nil

for
8.1.1 DETERMINANT OF A SQUARE MATRIX OF ORDER 1

If A =[f7|^] is a square matrix of order 1, then the determinant of A is defined as


r
A or.
=
^111 = «ii
You
oks

8.1.2 DETERMINANT OF A SQUARE MATRIX OF ORDER 2


eBo

fljl a-12
IfA = is a square matrix of order 2, then the expression a\\ 022 - ^21 is defined as
. ‘^21 ^^22 .
the determinant of A.
ad
our

i.e. A - fljj ^22 “^12 ^21


£?2i ^22

Thus, the determinant of a square matrix of order 2 is equal to the product of the diagonal
dY
Re

elements minus the product of off-diagonal elements.


ILLUSTRATION Evaluate:
Fin

5 4 1
1
-2 3
(ii) 1

2 2
x-\ 1
^ + -xy + y X +y
(hi) (iv)
X^ + .V + 1 X
2
-xy + y
2
x-y

SOLUTION By definition, we obtain


5 4
(i) 2 3
= 5x 3-4x-2=15 + 8=23

1 1
(ii) 1
= 1 - log^ a X logfl 1? = 1 -1 = 0 ●●● logt, ^ =

.T-1 1
(hi)
.Y^ + X + 1
= (a:-l) (x*^ -hx + l)-.Y^ =(.v^ =-l.
8.2 ,1A' ■ .lAlK;--.

2 2
●V + x\j + y x + y
(iv) 2 2 = {x^ +xy+i/) {x - y) - (.v^ - xy + i/) {x + y)
A' - .VI/ + y^
= (.v^-y^)-(.v^+y^)=-2y^
8.1.3 DETERMINANT OF A SQUARE MATRIX OF ORDER 3

^11 ^12 '^13


If /4 = <721 ^22 ^23 ^ square matrix of order 3, then the expression
. '^31 ^32 ^33 .

^11 ^22 ^33 '’12 '’23 '’31 '’13 '’32 '’21 “'’ll ”23 '’32 “'’22 '’13 '’31 “”l2 '’21 '’33
is defined as the determinant of A

'’ll '’12 '’13

w
i.e. A = <?2i <?22 ”23
”31 ”32 ”33

Flo
= <7|i <?22 ”33 + ”l2 ”23 ”31 + ”l3 ”32 ”21 “”ll ”23 ”32 “”22 ”31 ”l3 “”33 ”l2 ”21 -(’0
”11 ”12 ”13

ee
or. A I = ”21 ”22 ”23

Fr
”31 ”32 ”33

Ml = ”n (”22 ”33 “”23 ”32) “”12 (”33 ”21 “ ”23 ”3l) for ”l3 (”32 ”21 “ ”22 ”3l)
ur
A ”22 ”23 ”21 ”23 ”21 ”22
=
“”12 + ”13 [Using notation given in 8.1.2]
”32 ”33 ”31 ”33 ”31 ”32
s
”22 ”23 ”21 ”23 ”21 ”22
k

1+ 1 1+ 2 1+ 3
A] = (-1)
Yo

”11 + (-l) ”12 + (-l) ”13


oo

”32 ”33 ”31 ”33 ”31 ”32


eB

Thus the determinont of a square matrix of order 3 is the sum of the product of elements ay in first row
with (-1) ^ times the determinant of a 2x2 sub-matrix obtained by leaving the first row arid column
passing through the element.
r
ou
ad

The above expansion of | A | is known as the expansion along first row. For example, if
"3-2 4 1
Y

A = 1 2 1 is a square matrix of order 3, then


0 1 -1
Re
nd

3-2 4
Fi

M 1 2 1
0 1 -1

1+ 1 1 1 2
x3 +(-1)1+2(_2) q
1+ 3
A| = (-1) -1
+ (-l) 0 1

A[ = 3(-2-l) + 2(-l-0) + 4(l-0)=-9-2 + 4 = -7


There are three rows and three columns in a square matrix of order 3. The expression (ii) for the
determinant of a square matrix of order 3 can be arranged in various forms to obtain the
expansion of | A | along any of its rows or columns. Infact, to expand | A | about a row or a
column we multiply each element ay in row with (-1)'"^ i times the determinant of the
sub-matrix obtained by leaving the row and column passing through the element.
For example.
”11 ”12 ”13
! A I = <721 <7^2 ”23
”31 ”.32 ”33
8.3
DETERMINANTS

2+ 1 hi h3 2+2 <^11 '^13 2+ 3 ^11 hi


^1 = (-1) ^21 + (-l) ^^22 + (-l) ^23
^31 ^33 ^^31 ^^33 ^31 *^32

is the expansion of \A \ about second row.


The expansion of | ^ | about 2nd column is given as
1+ 2 ^21 ^13 2+2 ”n ”13 3+2 ‘’13
A|=(-l) ”12 + (-l) ”22 + (-l) ”32
^3\ ^33 ”31 ”33 ‘’21 ”23

2 3-2
ILLUSTRATION 1 Evaluate A = 1 2 3 bi/ expanding it along the second row.
-2 1 -3

SOLUTION By using the definition, of expansion along second row, we obtain

ow
2 3-2
A = 1 2 3
-2 1 -3

2+2 2 + 3 2 3

e
(3)
A =
3 (2) _2 _3 -2 1

re
3 -2 „ 2 ? -3
rFl
3 3

F
A = -
1 -3 -2 -3 -2 1

A = -(-9+ 2)+ 2 (-6-4)-3(2 + 6) = 7-20-24 - 37.

r
ou
ILLUSTRATION 2 Evaluate the determinant D =
2
1
3
2
-2fo3 by expanding it along first column.
ks
-2 1 -3
oo

SOLUTION By using the definition, of expansion along first column, we obtain


Y

2 3 -2
eB

D 1 2 3
2 1 -3
ur

(2) J f 4 2 ~3
1+ 1
D = (-1)
ad
Yo

=i» D = 2"1 ^ 3 -2
-2 23
d

-3 1 -3 3
Re
in

=> D = 2 (-6- 3)-(-9+ 2)-2 (9+ 4) - -18 + 7 - 26 = - 37.


Only square matrices have their deter7ninants. The matrices which are not square do not have
F

NOTH 1
determinants.

NOTE 2 The determinant of a square matrix of order 3 can be expanded along any row or column.
notes If a row or a colunm of a determinant consists of all zeros, then the value of the determinant is
zero.

8.1.4 DETERMINANT OF A SQUARE MATRIX OF ORDER 3 BY USING SARRUS DIAGRAM

The determinant of a square matrix of order 3 can be evaluated by the following procedure:
”11 ”12 ”13 ”11 ”12 ”13
Consider the determinant rt2i ”22 ”23 square matrix A = f?2i ”22 ”23
”31 ”32 ”33 ”31 ”32 ”33

In order to find the value of the determinant, we first enlarge the determinant by adjoining the
first two columns on the right and draw broken lines parallel and perpendicular to the diagonal
as shown below.
8.4
APPLIED MATHEMATICS-XI!

The value of the determinant is the sum of the products of elements in lines parallel to the
diagonal minus the sum of the product of elements in lines perpendicular to the diagonal.

a
12

^21 a-22 a-n «22


«31 X,3,X„33X„,X ^^32

*^11 ^^12 ^13


i.e.
^21 ^?22 ^^23 ='^11 ^^22 ^33 + ^12 ^23 ^31 + ^13 ^^21'^32
^31 ‘’32 ‘’33

'’13 ‘’22 '’31 “ ‘’ll ”23 '’32 ~ '’12 ”21 ”33


NOTE This method docs not work for determinants of order more than 3.

w
2-13

iLLUSTUATiON 1 Evaluate A = 4 12 by using Sarrus diagram.

F lo
1 -1 5

SOLUTION First we enlarge the determinant by adjoining the first two columns on the right
and then draw the broken lines parallel and perpendicular to the diagonal as shown below.

ee
To find the value of A, we find the sum of the products of elements in lines parallel to the

Fr
-1 for
1
ur
1^ -1 -1
s

diagonal and subtract from it the sum of the products of elements in lines perpendicular to them
ook
Yo

as given below.
eB

A=[2x 1 x5 + (-l)x 2x 1 + 3x 4x(-l)]-[3x 1 xl + 2x 2x(-l) + (-l)x 4x5]


A = [10-2-12]-[3-4-20]=(-4)-(-21)=17.
-1 6 -2
our
ad

ILLUSTRATION 1 Evaluate A = 2 1 1 by two methods.


4 1 -3

SOLUTION We have,
Y

■'I'
Re

. 1 1 2 1
A = -l ^ 3 +(-2) [Expanding along first row]
nd

-3 4 1
Fi

A = (-3-l)-6(-6-4)-2(2-4)
A = 4 + 60 + 4 = 68

To find A by a Sarrus diagram, first enlarge the determinant by adjoining the first two columns
on the right and then draw the broken lines parallel and perpendicular to the diagonal as shown
below.

Now, we find the sum of the products of elements in lines parallel to the diagonal and subtract
from it the sum of the products of elements in lines perpendicular to them as given below.
I 6s^2^ -1^6

4>^ 1>^3X4\ 1

A = [-lxlx-3 + 6xlx4 + -2x2xl]-[-2xlx4 + -lxlxl + 6x2x-3]


A ={3 + 24-4)-(-8-l-36) = 68.
DETERMINANTS 8.5

8.1.5 DETERMINANT OF A SQUARE MATRIX OF ORDER 4 OR MORE

To evaluate the determinant of a square matrix of order 4 or more we follow the same procedure
as discussed in evaluating the determinant of a square matrix of order 3.
For example.
1 2-13
2 1 -2 3
A =
3 1 2 1
1 -1 0 2
1 -2 3 2 -2 3
A = (-1)*+'(1)
1+2
=> 1 2 1 + (-l) (2) 3 2 1
-1 0 2 1 0 2

ow
2 1 3 2 1 -2
1+ 3 1 + 4
+ (-l) (-1) 3 1 1 + (-l) (3) 3 1 2
1 -12 1 -1 0

=> A = 1 (16)-2(12)+(-l)(-ll)-3(14)= -39.

e
RFMAKK It is evident from the above discussion that every square matrix A = [oij] of order n can be

re
rFl
associated to a number (real or complex) or an expression which is called determinant of the square j/iatrix
A. Thus, determinant may be thought as afimction from the set M of all square matrices to the set of all

F
numbers (real or complex).

r
8.2 SINGULAR MATRIX
ou
DEFINITION A scjuare matrix is a singular matrix if its determinant is zero. Otherwise, it is a fo
ks
non-singular matrix.
fl -2 3
oo

ILLUSTRATION! For wlmt value of X the matrix A = 1 2 1 is singular?


x 2-3
Y
eB

SOLUTION The matrix A is singular, if


A\ = 0
1 -2 3
r

1 2 1 = 0
ou

=>
ad
Y

A- 2-3

,2 1 1 1 1 2
^2 -3 X 3^3 = 0
d

Y 2
Re
in

(- 6 - 2) + 2 (- 3 -.y) + 3 {2 - 2x) = 0
-8-6-2y + 6-6y = 0^ -8y-8 =0=> .v =-1.
F

Y + 1 -3 4

ILLUSTR.ATION 2 Determine the values of x for which the matrix A= -5 y+2 2 IS

4 1 Y - 6

singular.
SOLUTION Given matrix /I is a singular matrix, if
A\ = 0
Y+1 -3 4
-5 y+2 2 = 0
4 1 y-6

y+2 2 -5 y + 2
(Y + 1) 1 y-6 ^ 4 y-6
+ 4
4 1
= 0

(Y + 1) 1(Y + 2) (y - 6) - 2| + 3 {-5y + 30 - 8) + 4 {-5 -4y - 81 = 0


(y + 1) (y^ - 4y -14) + 3 (- 5y + 22) + 4 (- 4y -13) = 0

Y (Y^ - 3y - 49) = 0 Y = 0, ^ ^ (3 ±
8.6 APPLIED MATHEMATICS-Xil

8.3 MINORS AND COFACTORS

MINOR Let A = [Ojj]be a square matrix of order n. The minor hAq ofaq in A is the determinant of the
square sub-matrix of order (n - 1) obtained by leaving i^^‘ rozu and f' column of A.
4 -7
For example, if/\ = -3 2 / then
M 11- Minor of fl-j-j = 2, M12 = Minor of = - 3,
M21 = Minor of fl2i = M22 - Minor of ^22 = 4
1 2 3 1
If A = ~ 3 2-1 , then
2-4 3

M 11 = Minor of a-^i
= Determinant of the 2x2 square sub-matrix obtained by leaving first row and

w
first column of A
2 -1
=> Mn = 2.
-4 3

Similarly, we obtain
M 12 = Minor of ai2
-3 -1

F lo 7, Mi3 = Minor of =
-3
' =8

ee
2 3 2 -4

Fr
2 3 1 3
M 21 = Minor of 021 - -4 3
= 18, M22 - Minor of a^i - 2 3
~ - 3 etc.
for
COFACTOfl Let A = [Ojj] be a square matrix of order n. The cofactor Cjj of ay in A is equal to (-1)' I
ur
times the determinant of the sub-matrix of order (n -1) obtained by leaving roiv and column of A.
s

It follows from this definition that


ook
Yo

Cjj = Cofactor of in A My, where M,y is minor of in A.


eB

Thus, we have
M:: if i + j is even

‘J 1-M^y if/ + j is odd


our
ad

4 -7 1
For example, if A =
-3 2 / then
Y

1+ 1 1+2
Cii=(-1) Mii=M;j^l=2, C;i^2=(“^) Mi2=-Mi2=-(- 3) = 3,
Re
nd

2+ 1 2+ 2
C21H-I) M21 =-M2i =-(-7) =7, and C22=(-l) M 22 = 4
Fi

1 2 3
If A = -3 2 -1 , then
2-4 3

1+ 1 2 -1
Cii = (-1) M 11 = M 11
-4 3

1+ 2 3 -1
Cl2= (-1) M 12 = -M 12
2 3
= 7

1+ 3 -3 2
Ci3 = (-1) M 13 = M 13
2 4

1
C23 = (-^)
2+3
M 23 = -M 23
2 -4
? = 8 etc.
ki:makk Some authors define the minors and cofactors for the elements of a determinant luhich is not
correct. Infact, minors and cofactors are defined for the elements of a square matrix.
8.7

ILLUSTRATIVE EXAMPLES

1 3
If A = 2 1 ,fiud the determmant of the matrix - lA.
1 3
SOLUTION We have, A =
2 1

1 3 1 3 T 3
A^ -2A -2
2 1 2 1 2 1

w
1+6 3+3 2 6 7 6 2 6 7-2 6-6 5 0
-2A =
2+2 6+1 4 2 4 7 4 2 4-4 7-2 0 5

5 0
-27l| = 0 5
= 25 - 0 = 25.

o
e
re
1 2
1 :
lfA = 4 2 , s//on’I 2A I = 4|A|.

Frl
F
SOLUTION We have,
T 2 2 4
A = => 2A
ou
4 2 8 4

sor
1 2 2 4
|A| = 4 2 = 2 - 8 = - 6 and ! 2A I = 8 4
= 8 - 32 = -24 = 4 X (- 6)
kf
Clearly, 12A| = 4|A|.
oo
X-2 -3
iXAMi’lL'- If = 3, find the values of X.
Y

3-r 2a;
B

SOLUTION We have,
re

a:-2 -3
= 3
oY

3a; 2a:
u

(.v-2)x2.v-(-3)x 3a: = 3
ad

2x(x-2) + 9x = 3
d

2a:^ - 4a; + 9a- = 3


in
Re

2a^ +5a-3 = 0 => (2.V-1) (a+ 3) = 0 =i- 2.V-1 =0 or, x+ 3 = 0 => j = -


2'
3.
F

3 1/ 3 2
I.XAMI’l I 4 Let
X 1 4 1 . Find possible values ofx and ij if x, \j are natural numbers.

SOLUTION We have,
3 1/ 3 2
A 1 4 1

3 - Ai/ = 3 - 8 => .ri/ = 8 => A = 1, y = 8; a = 2, y =4; a = 4, y = 2;a = 8, y =1


LXAMi’ir"' El’aluate the determinant A =
log 3 512 log4 3
logs 8 9
SOLUTION We have,

A =
logs 512 log4 3
logs 8 9
8.8 APPLIED MATHEMATICS-XII

3
A = logs 2^
logs 2^ 1o§22 3“

A =
9 logs 2
\ log2 3 log „ = - logrt 7n
31og3 2 |log2 3 p

A =
9 logs 2
3 logs 2
\ log2 3
log2 3

ow
1
^ = (9 log3 2) X (log2 3} - log2 3 (3 log3 2)
V2
3
^ = -9 (log3 2 X log2 3) - -2 (log2 3 X logs 2)

e
3
A = 9--
[●●● log/, (7 X log,, ^ = 1]

re
2
15

Frl
=> A =

F
2

1 3 -2
ou
EXAMru;6 Find the minors mid cofactors of elements of the matrix A =[oij]= 4-5 6 .

r
[352
so
kf
SOLUTION
Let Mjj and C,y denote respectively the minor and cofactor of element a^j in A. Then,
-5 6
M 11 = -10-30 = -40 => C 11
oo
5 2 = Mil = -40
Y

4 6
eB

Mi2 = 3 2
= 8-18=-10 => C 12 = -Mi2 =10

4 -5
ur

Mis - 3 5 = 20 + 15 = 35 => Cis = Mis =35


oY
ad

3 -2
M 21 = 6 + 10 = 16 => C21 - M21 =-16
5 2
d

1 -2
M22 - = 2 + 6 = 8 => C22 M 22
in

3 2
Re

1 3
F

M 23 = 5-9 = -4 => C23


3 5 = — M^s = 4
3 -2
M 31 = 18-10 = 8 => C 31 - ^31 - 3
-5 6

1 -2
M32 = 4 6
= 6 + 8 =14 ^ C 32 = - M 32 = -14

1 3
M 33 = -5-12 = -17 => C 33 = M 33 = -17
4 -5

EXERCISE 8.1

"■ Evaluate the following determinants:


-7 cos 0 - sin 0
(i) a:■! 5.y +1 (ii) sin 0 cos 0
DETERMINANTS 8.9

cos 15° sin 15° n + ib c + id


(iii) (iv)
sin 75° cos 75° -c + id a -ib

2 3 7 ^
2. Evaluate: 13 17 5
15 20 12

sin 10° - cos 10°


3. Show that =1
sin 80° cos 80°

2 3 -5

4. Evaluate 7 1 - 2 by two methods.


-3 4 1

w
2 5 4 -3
5. If^ =
2 1
and B =
2 5 , verify that| /IB| = | /111 B|.

lo
1 0 1

6. If/4 = 0 1 2 , then show that I 3 1 =27\A\.

e
0 0 4]

re
7. Find the values of x, if
rF
F
2 4 2.V 4 2 3 -T 3
(i) 5 1 6 X
(ii) 4 5 2.V 5

r
fo
u
3 X 3 2 3-r 7
(iii) .V 1 4 1
(iv) 2 4
= 10
ks
Yo
-t + l x-'l 4 -1 2.V 5 6 5
(V) (Vi)
oo

.v-3 x + 2 1 3 .V 8 3
B

.r 2 ;c 1
8. Find the integral value of .v, if 0 2 1 = 28.
e

3 14
ur
ad

q. For what value of x the matrix A is singular?


Yo

.r-1 1 1
l+x 7
(i) v4 = (ii) = 1 x-\ 1
d

3-.r 8
1 1 .r-1
Re
in

10. Write the minors and cofactors of each element of the first column of the following matrices
F

and hence evaluate the determinant in each case:


5 20 -1 4
(i) ^ = 0 -1
(ii) A = 2 3

1 -3 2 1 a be

(iii) = 4-12 (iv) A = 1 b ca

|_ 3 5 2 1 c ab

'0 2 6' a h g
(v) A = 15 0 (vi) A = h b f
3 7 1 g f c
2-1 0 1
-3 0 1-2
(vii) A = 1 1 -1 1
2-1 5 0
8.10 KLiCn i " MATICS-XM

4 NS VVERS

(i) 5x^ + 8-y (ii) 1 (iii) 0 (iv) +b^ + ■ 0 . -140

13
(i) ± (ii) 2 (iii) ± 2V2 (iv) 2 (v) 2 (vi) ± 3 2 (i) ^(ii) -1,2
15

Minors Cofactors
(i) M„=-1,M2|=20 Cjl =-l,C2i =-20

ow
(ii) M„ =3, M2i=4 C]i = 3, C21 = - 4
(iii) Mn =-12,M2i =-16,M3i =-4, =-12,C2i =16,C3i = -4
(iv) M^-y=a (/r - c^), M21 ^b{a^ - c^), C]-] =a{b^ -c\

e
M31 =C((7^ -b'^) C34=c(rt^-/7^)

re
(v) M„=5,M2i=-40,M3i=-30 =5, C21 =40,C3i =- 30

Flr
Cu=bc-f\C2, =fg-chX3i=hf-bg

F
(vi) M-^-^=bc-f-,M2\=hc-fg,
-h
(vii) Mn = - 9, M21 = 9, M31 = - 9, M41 = 0
ou C„ = - 9, C21 = - 9, C33 = - 9, C41 = 0

sr
fo
APPLICATIONS OF DETERMINANTS iN SC^.:NG A SYSTEM OF LINEAR EQUATIONS

Consider a system of simultaneous linear equations given by


rtj X + bi}/ + c-j z =
k
oo
(?2 X + b2]/ + CjZ = i^2
Y

173 X 4-/73 y + “^3 ^ = “^3


reB

A set of values of the variables x, y, z which simultaneously satisfy these three equations is called
uY

a solution set.

For example, x = 3, y = 4 and z = 6 is the solution of the system of equations


5x - 61/ 4- 4z = 15
ad
do

7x4-4y-3z = 19
in

2x 4- y 4- 6z = 46
Re

A system of linear equations may have a unique solution, or many solutions, or no solution at
all. If it has a solution (whether unique or not) the system is said to be consistent. If it has no
F

solution, it is called an inconsistent system.


If di=d2= ^3 = 0 in (i), then the system of equations is said to be a homogeneous system.
Otherwise it is called a non-homogeneous system of equations,
[i .● SCLUriONOF A NON-HOMOGENEOr''SYSTEM OF LINEAR EQUATIONS

We now intend to solve a system of simultaneous linear equations by Cramer's rule named after
the Swiss mathematician Gabriel Cramer.

THEOREf.; * (Cramer's rule) The solution of the system of simultaneous linear equations
rtj X 4- /?! y = Cl ...(i)
f?2 X 4- /?2 y ~ ^2 ...(ii)
D1 Di h _ h _ Cl
is given by x= y = where D = and D2 = provided that
D' D ’ «2 hi C2 b2 772 C2
D^O.
DETERMINANTS 8.11

h
PROOl We have, D =
(I2

.yD = .V
<7i‘ b-i‘ ^7l.V
-V &2
a-i X + y
.yD = [Applying q -> Cl+ yC2l
^2 ^2 y ^2

xD ^ q l>i = Di [Using (i) and (ii)]


C2 b2
Similarly, we obtain

w
yD = = D2
^2 ‘^2
D1 _ ^2
and provided that D 0. Q.K.D.

Flo
.Y y =
D D '

e
R-LMARK Here D = is the determinant of the coefficient matrix .

re
/?2 1?2

F
The determinant D-^ is obtained by replacing ifrst column in D by the column on the right hand side of the
given equations.
ur
r
The determinant D2 is obtained by replacing the second column in D by the right most column in the fo
given system of equations.
ks
THEOREM 2 (Cramer's Rule) The solution of the system of linear equations
Yo
oo

i?! -Y + l?i 1/ + 'cl 2 = di


02 X + b2 y C2 ^ — d-) ●●●(ii)
B

a^x + b^y + c^z = d^ ...(hi)


re

D\ _ C>2 and z =
is given by x = y = where _

D' D D '
u
ad

fli l^i Cl di l?i Cl fll tfi Cl ai bi


Yo

0=^2 ^2 C2 , Di = ^2 ^2 ^2 ' ^2 ~ ^2 ^2 ^2 0 = ^2 ^’2 ^2 '


<73 C3 ^3 ^3 ^^3 <73 (^3 C3 «3 ^3
d
Re

provided that D ^0.


in

PROOl We have.
F

(7i ^1 Cl
D = (72 ^^2 ^2 ■
(73 IJ3 C3
(?1 1^1 Cl (?i .Y &i Cl
.yD = .Y (?2 1^2 ^2 = (?2 X &2 ^^2
"3 ^3 ^3 (73 .Y 1)3 C3
7?1 .Y + I?! y + Cl 2 Cl
xD = (72 .Y + l)2y + ^2- 1)2 C2 [Applying Cl ^Ci +yC2 +2C3]
(73.Y + &3y + C32 &3 C3
dl ^1 Cl
xD = £^2 1^2 C2 = Di [Using (i), (ii) and (iii)l
^3 ^3 ‘^3
8.12 APPLIED MATHEMATICS-XII

Similarly, we obtain
<?! ci-i t’l rt] 1^1 di
yD = <?2 ^2 C2 =D2 and zD = U2 1?2 ^2 =^3-
<73 ^3 C3 ^3 &3 d-3,
D1 D2 D
A' =
y = and z = provided thatD ^ 0. Q.E.D.
D ’ D D'
REMARK Here D is the determinant of the coefficient matrix. The determinant Dj is obtained by
replacing the elements in first column ofD by dj, ^2, D2 is obtained by replacing the elements in the
second column of D by d^, d2, d^ and to obtain D3, replace elements in the third column of D by
d\> d2, dy

ow
In order to solve a non-homogeneous system of simultaneous linear equations by Cramer's rule,
we may use the following algorithm.
ALGORITHM

STLI’ I
ObfainD, D^, D2andD3.

e
re
SlUMl
Find the value ofD.

Fl
F
//'D 0, then the system of equations is consistent and has a unique solution. To find the
solution, obtain the values ofO^, D2 and D3. The solution is given by
ur
r
D1 _ C>2 and z =
X
D'
y =
D D fo
ks
// D = 0, go to step III
Yo
11 :’.ui
Find the values of D], D2, D^.
oo

If at least one of these determinants is non-zeiv, then the system is inconsistent.


eB

If Dj =D2 =03 =0, then go to step IV.


SI
1;L!.V Take any tivo equations out of three given equations and shift one of the variables, sayz, on the
ur

right hand side to obtain tzvo equations in x, y. Solve these two equations bi/ Cramer's ride to
ad
Yo

obtain x, y in terms of z. If these values ofx and y satisfy the third equation, then the system is
consistent and the values of x, y and z constitute a solution.
d

If the values of x and y do not satisfy the third equation, then the system is inconsistent.
Re
in

ILLUSTRATIVE EXAMPLES
F

Type I SOLVING A SYSTEM OF EQUATIONS BY CRAMER’S RULE


LXAMIM.i: I
Solve the following system of equations by Cramer's rule
2x~y = 17
3a: + 5y = 6
SOLUTION For the given system, we have
1
g = 2x5-(-l)x 3 = 13^0
17 -1
D1
g g =85 +6=91 and D2= 3 =12-51 =-39.
So, by Cramer's rule, we obtain
D1 _ 91
X = — =7 and 1/ = D2 _ -39 = - 3.
D 13 D 13

Hence, x = 7 and 1/ = - 3 is the required solution.


8.13
DETERMINANTS

EXAMI'U', 2 Solve the following system of equations using Cramer’s rule:


5-V-7i/+z=ll, 6.v-8i/-z=15 and 3.v + 2i/- 6z =7.
SOLUTION The given system of equations is
5x-7y + z = 11
6.Y-8y-z = 15
3.Y + 2y -6z =7
5 -7 1
1 = 5 (48 + 2) + 7 (- 36 + 3) +1 (12 + 24) = 250 - 231 + 36 =55 # 0

ow
D = 6
3 2-6

11 -7 1

Dj = 15 -8 -1 = ll(48+ 2) + 7(-90 + 7) + l(30+56)= 550-581 +86 = 55

e
7 2-6

re
5 11 1

D2 = 6 15 -1 = 5(_90 +7) -11 (- 36+ 3) + 1 (42 -45) = -415 + 363 - 3 = -55

Flr
3 7-6

F
5 -7 11
and C»3 = 6 -8 15 =5(-56-30) + 7{42-45)+ll(12 + 24) = -430-21 + 396 = -55
3 2 7
ou
sr
So, by Cramer's rule, we obtain
X = y =
Di _
D "
55
55
= -1 and z =
D,
D "
fo
k
55
55
= -l.
oo
D 55 '
Hence, y = 1, j/ = -1 and z = -1 is the solution of the given system of equations.
Y

The sum of three numbers is 6. If we multiply the third number by 2 a}td add the first
reB

● XAMI’LI 3

number to the result, zoe get 7. By adding second and third numbers to three times the first number we get
12. Use determinants to find the numbers.
uY

SOLUTION Let the three numbers be y, y and z. Then, from the given conditions, we obtain
Y + y+ z = 6 or. Y +y +z = 6
ad
do

y + 2z = 7 Y + Oi/ + 2z =7
3y + y + z = 12 3y + 1/ +2 = 12
in

1 1 1
Re

Here, D 1 0 2 ■ = l(0-2)-l(l-6) + l(l-0) = -2+5 + 1 = 4


F

3 1 1

6 1 1
D1 7 0 2 = 6(0-2)-1(7 -24) + l(7-0) = -12 + 17+7 = 12
12 1 1

1 6 1

D2 = 1 7 2 = l(7-24)-6(l-6)+l(12-21) = -17 + 30-9 = 4


3 12 1

1 1 6

and. Dr, = \ 0 7 = 1(0-7)-1(12-21)+ 6 (1-0) = -7 + 9 + 6 = 8


3 1 12

D1 12 C>2 = -
—^ 4 = ,1 and, 2 = ^
D3 = -8 = 2.
Y
D j = 3,y = D 4 D 4

Tlius, the three numbers are 3,1 and 2.


8.14
APPLIED MATHEMATICS-Xn

LXAMIM I- 4 Solve the following system of equations by Cramer's rule:


2,310_
- + - + — =-i,
4 6 5
+ - =1 and - +
6 9 20 2.
-V y z X y z X y z
1 1 1
SOLUTION Let = u, - = V and
written as
.V
y
~ Then, the above system of equations can be
In + 3i» + tOw = 4
4ii ~6v + 5iv =1
Oil + 9v- 20w = 2
2 3 10

ow
Here, D = 4 -6
5 = 2(120 - 45) - 3 (-80 - 30) + 10 ( 36 + 36) =150 + 330 + 720 =1200
6 9-20

4 3 10
D1 1 -6
5 = 4 (120 - 45) - 3 (- 20 -10) + 10 (9 + 12) = 300 + 90 + 210 = 600

e
2 9-20

re
2 4 10

Frl
F
D2 = 4 1 5 = 2 (- 20 -10) - 4 (- 80 - 30) +10 (8 - 6) = - 60 + 440 + 20 = 400
6 2 -20

2 3 4
ou
r
and, D3 = 4 -6 1 = 2(-12-9)-3(8-6)+ 4(36 + 36) = -42-6 + 288 = 240
o
6 9 2
kfs
D^ 600 1 1 1
oo

u
=> -V = 2,
D 1200 2 A' 2
Y
B

V = = ^00_ _ 1. 1
=
1
— =>!/ = 3,
D 1200 ~ 3 y 3
re

and. re ^ ^ _ 1 1 1
=> z = 5
D ” 1200 “ 5
oYu

z 5
ad

Hence, .v = 2,1/ = 3 and z = 5.


7 1//JI- // ON /iPPLICATiONS OF CRAMER’S RULE TO BUSINESS
d

AND ECONOMICS

A firm produces two products and ?2 pmssing through tzuo machines


in

LXAMIT I ?
and M2 befo re
Re

completion, M, can produce either 8 units o/Pj or 10 units of l\ per hour. M2 can produce 12 units of
F

either product per hour. Using determinants, determine: Production of P-^ andP2 f time available on two
machines is 33 hours and 25 hours respectively.
SOLUTION Suppose -V units of product P^ and y units of product P2 are produced in the time
available.

8 10
= 33 or, 5x + 4y = 1320
A
y
12 12
= 25 or, A + y = 300
For this system of equations, we obtain
5 4 1320 4
D = =5-4=1; Di = = 1320-1200=120
1 1 300 1

5 1320
and. D2 = = 1500-1320=180
1 300
8.15

Using Cramer's rule, we obtain


= 120, ij = P2 ^180
D 1 _ 120
.Y = = 180
D 1 D 1

Hence, 120 units of product and 180 units of product P2 are produced in the given time.
The prices per ton of zvheat mid rice are pi and p2 respective}}/. The inarlcet demand for
zvheat is given by Yj = 4 - lOpi + 7p2 ●''■2 3+ 7 Pi -5p2-The snppli/ of wheat is related to
prices by the relation y^ =7 + pi ~Pi and supply of rice by X2 = -27-pi + 2p2-Find the equilibnum
prices and quantities using Cramer's rule.
SOLUTION For equilibrium price of a commodity its demand and supply must be equal.
Therefore, for the equilibrium prices of wheat and rice, we must have
4-10pi+7p2 =7 + Pi-P2 3+7pi-5p2 =-27-pi + 2p2
llpi-8p2=-3
8pi -7p2 =-30

w
For this system of equations, we obtain

F lo
11 -8 -3
D = = -77+ 64 = -13; = = 21-240 =-219
8 -7 -30 -7

11 -3
= -330 + 24 = -306

e
and, D2 =

Fre
8 -30

Using Cramer's rule, we obtain


for
D| _ -219 _ 219 and P2 = Di -306 _ 306
Pl =
o’” -13 “ 13 D -13 13
r
Substituting the values of P| and P2 in y^ = 4 -lOp^ + 7p2 and Y2 = 3 + 7pi -5p2, we obtain
You
oks

2190 , 2142 1533 1530 _ 42


eBo

Yi=4- =— and Y2 = 3 + 13 ~ 13
13 13 13
219 306
Hence, the equilibrium price of wheat and rice are Pj - and p2 = respectively and the
13
ad

13
our

4 42
equilibrium quantities are y^ = — ana Y2 - respectively.
13

A company produces three products everi/day. Their total production on a certain day is
Re
dY

45 tonnes. It is found that the production of third exceeds the production offirst product by 8 tonnes while
the total production offirst and third product is tzvice the production of second product. Determine the
Fin

production level of each product using Cramer’s rule.


SOLUTION Let the production levels of first, second and third product on a certain day be x
tonnes, y tonnes and z tonnes respectively. It is given that
Y + y+2 = 45, 2-y = 8 and Y + z = 2y
Thus, we obtain the following system of equations
y+i/+2 = 45
-Y+0y+2 = 8
Y - 2_v + 2 = 0
1 1 1

D = -1 0 1 =l(0 + 2)-l(-l-l) + l(2-0) = 6


1 -2 1
8.16
APPLIED MATHEMATICS-XII

45 1 1
D1 - 8
0 1 =45(0 + 2)-l(8-0) + l(-16-0)=66
0 -2 1

1 45 1
-1
8 1 =l(8-0)-45(-l-l) + l(0-8)=90
1 0 1

w
1 1 45
and. ^3 = -1 0
8 =l(0 + 16)-l(0-8)+45(2-0)=114
1 -2 0

Using Cramer's rule, we obtain

e
D 1 _ 66 _ ^2 90 „ ,

ro
re
X =
— =11, i/ = — =15 and z - = 19
D 6 D 6 D 6

Hence, the company produces 11 tonnes of first product, 15 tonnes of the second product and

F
19 tonnes of third product everyday.

Fl
u
\ M PLE 8 The total sales (S) in thousands of rupees ofa firm selling tzoo products X and Y is given by
the relationship : S = a + bX + cY. Sales data from Jnnuary-March are given below:

sr
Month X y Total sales (S)

ko
January 2 3 12
o
February 6 2 13
of
March 5 3 15
o
Using determinant method, determine the sales in the next month when it sells 4 units ofX and 5 units
Y

ofY.
erB

SOLUTION The total sales (S) in thousands of rupees for products X and Y is given by
uY

S = a + bX + cY=U
In January, we have
X = 2,y = 3 and S = 12
ad
do

Substituting these values in S = ^7 + tX + cy, we obtain


a + 2b + 3c = 12
in

Similarly, for February and March months, we obtain


a + 6b + 2c=13
Re

[Putting X = 6, y = 2, S = 13 in (i)]
a + 5b + 3c =15
F

[PutHngX =5, y = 3, S =15 in (i)]


Thus, we obtain the following system of equations
a + 2b+3c=12
a + 6b + 2c =13
a + 5b+ 3c=15
1 2 3
D =
1 6 2 =l(18-10)-2(3-2)+3(5-6) = 3
1 5 3

12 2 3
D1 -
13 6 2 =12(18-10)-2(39-30) + 3(65-90) = 3
15 5 3

1 12 3

D2 = 1 13 2 =1 (39-30)-12 (3-2)+ 3 (15-13) = 3


1 15 3
8.17
DETERMINANTS

1 2 12

and, D3=1 6 13 =l(90-65)-2(15-13) + 12(5-6)=9


1 5 15

Using Cramer's rule, the solution is given by


a =
D1 _ 3 1 , ^2. 3 1
D D D 3

Substituting the values of n, b and cinS = a+bX + cY, we obtain


S=1+X+3Y

WhenX = 4 and Y =5, we obtain


5=1 + 4 + 15=20

ow
Hence, the sales in the month of April is of ? 20,000.
cxAMl’U-;9 A firm produces three products Pj, P2 and P3 processed on three machines Mi, M2 and
M3. Machine M| can process 25 units of Pi or, 50 units of P2 or, 75 units 0/P3 per hour. M2 can process
50 units of each product per hour. M3 can process 50 or, 25 or, WO units per hour of Pj,P2 and P3

e
respectively. The processhig hours available on the machines M^, M2 and M3 are 12, 12 and 13

re
rFl
respectively. Using determinants, find how many units of three products can be produced with the

F
available time fully used?
SOLUTION The available information can be arranged in the following form;

r
Pi P3 Hours available
Pt
fo
ou
25 50 75 12
Ml
ks
50 50 50 12
M2
50 25 100 13
M3
oo

Suppose X imits of product P^, y units of product P2 and z units of product P3 are produced.
Y
B

Then,
e

_ +^y
+ ^=12 or, 6.t+3y + 22=1800
X
ur

25 50 75

— + -^ + -^ = 12 or, X + 1/ + 2 = 600
ad

X
y
Yo

50 50 50

—+ X + _£_=i3 or, 2x + 4y + z=1300


d

50 25 100
Re
in

6 3 2
F

D=1 1 1 =6(l-4)-3(l-2)+2(4-2)=-ll
2 4 1

1800 3 2
D1 - 600 1 1 =1800(1-4)-3(600-1300) + 2(2400-1300) = -1100
1300 4 1

6 1800 2

D2 = 1 600 1 =6 (600-1300)-1800 (1-2)+ 2 (1300-1200) =-2200


2 1300 1

6 3 1800

Ds = 1 1 600 = 6 (1300-2400) - 3 (1300 -1200) +1800 (4 - 2) = -3300


2 4 1300
8.18

Pi _ -1100 =100,1/ = D2 _ -2200 P3 _ -3300


= 200,z = = 300
D -11 D ~ -11 D ~ -11
Hence, 100 units of product P^, 200 units of product P2 and 300 units of product P3 can be
produced in the available time.

XERCISE 8 2

Solve the following systems of linear equations by Cramer's rule:


X - 2y = 4 2x-y = l 2.Y-y = 17
- 3x + 5y = - 7 7x-2y=-7 3x + 5y = 6
Solve the following system of the linear equations by Cramer's rule:
3-y + y + 2 = 2 Y-4y-2=ll 6.Y + y - 3z = 5
2.V - 4y + 3z = -1 2x - 5y + 2z = 39 X + 3y -2z= 5

w
4x + y - 3z =-11 - 3x + 2y + 2 = 1 2.Y + y + 4z = 8
A salesman has the following record of sales during three months for three items
A, B and C which have different rates of commission.

F lo
Month
Sale of units Total commission drawn (in V
A B

ee
C

Jan

Fr
90 100 20 800
Feb 130 50 40 900
March 60 100 30 for 850
ur
Find out the rates of commission on items A, B and C by using determinant method.
An automobile company uses three types of steel S,, $2 and S3 for producing three types of
s
cars Cl, C2 andC3. Steel requirements (in tons) for each type of cars are given below:
ook
Yo

Cars .
eB

C1
Steel C2 C3
r

Si 2 3 4
ou
ad

S2 1 1 2

S3
Y

3 2 1

Using Cramer's rule, find thenumber of cars of each type which can be produced using 29,
Re
nd

13 and 16 tonnes of steel of three types respectively.


Fi

Given the following equations for two related markets A and B, find the equilibrium
conditions for each market and the price for each market by using the Cramer's rule :
.%V(A)=82-3p^+Pg , .^(A)=-5 + 15p^
-Vrf(B) = 92 + 2p^-4yg , =-6 + 32pg
where .y/ and denote quantity demanded and quantity supplied respectively.
A salesman has the following record of sales during three weeks for three items X, Vand 2
which have different rates of commission.
INeeks Units sold Total commission (in f)
X Y Z
I 40 30 20 270
II 50 50 40 450
III 60 30 10 260
Find out, using determinant method, the rates of commission on items X, Y and Z.
8.19
DETERMINANTS

11. A company earns before-tax profits of ^100,000. It is committed to making a donation to the
Red-Cross 10% of its after-tax profits. The central government levies corporate tax of 50% of
profits after deducting charitable donations and any local taxes. The company must also
pay local taxes of 10% of its profit less the deduction to the Red-Cross. Compute how much
the company pays in corporate tax, local taxes and as a donation to the Red-Cross, using
Cramer's rule.
__ANSWERS

1. a =-6, i/ = -5 2. x = -3,y = -7 3. x=7, ij = -3 4. .V = -1,1/= 2,2 = 3

5. .A = -1, y = - 5,2 = 8 b. x=l.ij = 2,z=l


7. ?2, ^4, ?11 H. Si: 2 tonnes : 3 tonnes S3 :4 tonnes
Pa =^'Pb = ^ 10. X:?2,Y:^3,Z:?5
900,000 8,100,000 1,800,000
11. Red-Cross:? , Corporate tax: ? , Local tax : ?
189 189 189

w
FILL IN THE BLANKS QUESTIONS (FBQs)

t. If A = diag (1, 2, 3), then \A\ =

2. If the matrix A = 2
fl

3
3
4

5
-Y + 2’

10

3. The set of real values of a for which the matrix A =


F lo
is singular, then y =

2
for F
2

4 ree
is non-singular is
r
4. IfA = and if det (A) = 2,then Y
You
oks

-InY 2
eBo

5. If 1 is the identity matrix of order 10, then determinant of I is


in A. Then
6. Let A=[0jj] be a 3 X 3 matrix such that i A [ =5. If Cjj = Cofactor of Ojj
flilCii +rti2 Ci2+'^13 ^13
ad
our

7. In the above question, C21 +'^12 C22 + *^13 *-23 “


1 2 3

8. The value of the determinant A = 4 5 6 is


Re

3y 6y 9y
dY

9. If A is a matrix of order 3x3, then the number of minors in A is


Fin

Y 3 7

10. If.v = -9 isarootof 2 y 2 = 0, then other two roots are


7 6 Y

ANSWERS

3. R-Ul 4. 5. 1
1. 6 2. 4

6. 5 7. 0 8. 0 9. 9 10. 2.7

VERy SHORT ANSWER QUESTIONS (VSAQs)

Answer each of thefoUozcing questions in one word or one sentence or as per exact requirement of the
question:
; If A is a singular matrix, then write the value of | A|.
8.20
APPLIED MATHEMATICS-XII

... For what value of X, the matrix 4


^ is singular?

2 3 4
.3. Write the value of the determinant 2x 3x 4x
5 6

2 3
4, State whether the matrix
6 4 is singular or nonsingular.

ow
1 2 1 0
..If 4= 3 -1
. and B =
-1 0 , find|/4B|.

l(A= [oij] is a 3 X 3 diagonal matrix such that = 1, ^22 = 2 and <733 = 3, then find | [.

e
If A = [Ojj]is a 3x 3 scalar matrix such thatajj =2, then write the value of |A|.

re
S. If fg denotes identity matrix of order 3x3, write the value of its determinant.

Flr
5x 2
9. If the matrix

F
-10 1 is singular, find the value of .v.
ou 2-3 5
10. Write the cofactor of in the matrix 6

sr
0 4
1 5 -7

11. If
2x + 5
5x + 2
3
9
= 0, find X. fo
k
oo

'=0
X
Y

Find the value of x from the following: 2 2x


reB

2 3 4
Write the value of the determinant 5
uY

6 8
6x 9x 12x
ad

0 2 0
do

14. V\Tiat is the value of the determinant 2 3 4 ■?

4 5 6
in

value of X is the matrix ^


-X 4
Re

■ For what
-X ^ singular?
F

5 3 8
Ih. lfA = 2 0 1 . Write the cofactor of the element a 32-
1 2 3

x+1 x-1 4 -1
ir. If
x-3 x+2 ^ ^ , then write the value of .x.
2x x+3 1 5
IS. If
2 (x + 1) x+1 - 3 2 / then write the value of .x.
3x 7 8 7
19. If find the value of .x.
-2 4 " 6 4 '
2x 5 6 -2
. If
X 7 ^ , write the value of X.
8.21
DETERMINANTS

P P+1
■' ■ Write the value of the determinant
P-1 P
.y+3 -2
2^. If a: e Nand = 8, then find the value of .v.
-3.V 2.T

ANSWERS

6. 6 8. 1
■ 0 2. 3 3. 0 i. Non-singular 5. 0

9.-4 1C. 46 11. -13 12. ±2 IT 0 14. 8 n. 2 16. 11

2 18. 1 14. -2 20. ± 6 1 22. ± 2

w
Flo
ee
Fr
for
ur
k s
Yo
oo
eB
r
ou
ad
Y
Re
nd
Fi
9
INVERSE AND
APPLICATIONS OF MATRICES

w
9.1 ADJOINT C A "^QUARE ■

ADJOINT LetA = [ajj] be a square matrix of ordern and let Cq be cofactor ofaq in A. Then the transpose of

e
the matrix of cofactors of elements of A is called the adjoint of A and is denoted by adj A.

ro
re
T
Thus, adj A = [C,y] => (adjA),^ =Cji = Cofactor of 77y, in A.

F
(7ii (7^2 '^13
C 11 C12 C 13 C 11 C21 C 31

Fl
c 12 c 22 C 32
A= 7721 '’22 '’23 ' ad)A= C21 C22 C23
If
(731 n32 7733 _

u c 31 c 32 c 33 c 13 ^23 ^33

sr
where Cq denotes the cofactor of aq in A.

ko
o
P ‘I
Mli-.ii<AiIi) Find the adjoint of matrix A = [Ojj] = r
of
s

SOLUTION We have.
Cofactor of t?! 1 = s. Cofactor of 7712 = - r, Cofactor of 7721 = - 7/ and. Cofactor of 7722 = P-
o
Y

-iT
erB

r s - r s -'7
adj A = -q
_^ p - r P
uY

RULE It is evidentfrom this example that the adjoint of a square matrix of order 2 can be easily obtained by
interchanging the diagonal elements and chatiging signs of off-diagonal elements.
-2 3" [4 -3
ad
do

7/A = -5 4 ' then by the above rule, we obtain adj ^4 = ^ -2 ■

1 1 1'
in

2 1 -3
lLl,UMKA‘Iin.\ . Find the adjoint of matrix A = [aq] =
Re

1 2 3
F

SOLUTION Let Cq be cofactor of Ojj in A. Then, the cofactors of elements2 of1 A are given by
1 -3 2 -3
C 13 =5
C 11 - 2 3 =^'
c
12--i_i 3 =-3' - -1 2

1 1 1 1 1 1
= -3
C21 -- = -l. C 22 - -1 3
= 4, <^23 -1 2
2 3

1 1 1 1 1 1
= -l
^33 -
C 31 -
1 -3
= -4, C
32-- 2 -31"^' 2 1

T
9-3 5 9 -1 -4
1 4-3 -3 4 5
adj A =
-4 5 -1 5 -3 -1

THEOREM 1 Let A be a square matrix of order n. Then, A (adj A) = j A | 7„ = (adj A) A.


fKOOl
Let A =[7T,‘y], and letC,y be cofactor ofTT^y in A. Then,
9.2
APPLIED MATHEMAT(CS-XII

(adj A)ij = Cy,' for =

Since y4 and adj A are botli square matrices of the same order nx n. Therefore, botlr,4 (adi^) and
(adj A) A exist and are of the same order nxn.
Now,
n

{A{ad]A)^ = (yl),v(adjAVy [By definition of multiplication of two matrices]


n
, if / = j
r ^= 1 % Qy‘ - 0 , if / y [By property 1 and 2 of section 9.4]

Thus, each diagonal element of (adj A) is equal to| ^ (and all non-diagonal elements are equal

ow
to zero.

'Ml 0 0 ... 0
0 Ml 0 ... 0
A (adj A) - 0 0 Ml 0 = MU n

e
re
0 0 0 A

Frl
F
Similarly, we obtain
u

((adj A) A)jj I= 1 {adjA)i, (^) ^.


ou
r

or
kfs
((adj A) A)ij 2
r = l
C.V
n
oo

A , if/=;
((adjA)Ak- = [By property 1 and 2 of section 9.4]
Y

0 / if /V /
B

Hence, A (adj A) =\A\I,, = (adj A) A.


re

14 5’
oYu

ILLUSTRATION S
Compute the adjoint of the matrix A given by A= 3 2 6 and verify that
ad

0 10

A (adj A) = |A U =(adj A) A.
d

SOLLTnON We have,
in
Re

I 1 4 5
Ml=|3 2 6 =l{0-6)-4(0-0)+5(3-0)=9
F

I 0 1 0

Let Cjj be cofactor of Ojj in A. Then, the cofactors of elements of A are given by
2 6 3 6 3 2
C 11 = -6, C 12
-
1 0 --
0 0
= 0, c 13 - = 3
0 1
4 5 1 5 1 4
C 21
1 0|”^' C22 - 0 0
= 0, c 23
0 1
= -l

4 5 1 5 1 4
C 31 - = 14, C 32 = 9, C 33 = = -10
12 6 3 6 3 2

■-6 0 3 iT -6 5 14’
adjA = 5 0 -1 0 0 9
14 9 -10 3 -1 -10
9.3
INVERSE AND APPLICATIONS OF MATRICES

1 4 5 -6 5 14 9 0 0 10 0

Now, A (adj A) = 3 2 6 0 0 9 0 9 0 = 90 1 0 =\A\I


[o 1 0 3 -1 -10 0 0 9 0 0 1

■-6 5 14 1 4 5 '9 0 0 [10 0


and. (adjv4)^ = 00 9 3 2 6 0 9 0 =9 0 1 0 = I 11
0 1 0 0 0 9 0 0 1

w
3 -1 -10

Hence, A (adj A) = \A \ 1 = (adj A) A.


9.2 INVERSE OF A MATRIX

e
INVERSE A square matrix of order n is invertible if there exists a square matrix B of the same order such

e
that AB = 1,1 = BA.

or
-1
= 6.
In such a case, we say that the inverse of A is 6 and we write, a

r
F
THEOREM 1 Every invertible matrix possesses a unique inverse.
PROOF Let A be an invertible matrix of order n x n. Let B and C be two inverses of A. Then,
oF
ul
AB = BA = 1 -(i)
and AC = CA = In ●..(ii)

s
r
Now, AB = I n

ko
C {AB) = C In [Pre-multiplying both sides by C)
(C>1) B = Cl of
[By associativity of multiplication]
[■.■ CA = In from (ii)]
InB = Cl
o
Y
B = C [●.● /„ B = B and C =C]
rB

Hence, an invertible matrix possesses a unique inverse.


eY

Q.E.D.
U-1
= A.
COROLLARY If A is an invertible matrix, then {A )
u

PROOF Since is inverse of


d

= I = A~'^ A
o
ad

-1
i.e. 21=(A-V^-
in

A is the inverse of A

THEOREM 2 A square matrix is invertible iff it is non-singular.


Re

PROOF Let A be an invertible matrix. Then, there exists a matrix 6 such that
F

AB = In = BA
AB\ = |/„1
A| 1 B| = I
1^1 0
^ is a non-singular matrix.
Conversely, let A be a non-singular square matrix of order n. Then,
A (adj A) =\A\In = (adj A) A [See
[ Theorem I on page 9.1]
●●● \A\^Q exists
A — adj A = I I! — — adj A A A

1
/l-l = adj A [By definition of inverse]

Hence, A is an invertible matrix.


o F n
9.4

This theorem provides us ci foriniih for finding the inverse of a non-singular snuare matrix.
The inverse of A is given bi/
-1 1
A ■ adj A

In order to find the iirverse of a square matrix, we may use the following algorithm.

Find! A I
lf{ A\= 0, then write "A is a singular matrix and hence not invertible".
Else write "A is a non-singular and hence invertible".
Calculate the cofactors of elements of A.
Write the matrix of cofactors of elements of A and then obtain its transpose to obtain adj A.

ow
-1 1
Find the inverse of A by using the formula: A adj A.

[2 -1
■ >rRATiONi Find the inverse of the matrix 4 ●

e
Fl
re
F2 -1
SOLUTION Let Al = ^ 4 -Then,

F
-1
ur
^ =8+ 3=11^0.
or
So, A is a non-singular matrix and therefore it is invertible. Let Q.- be cofactor ofojj in A. Then, the
sf
cofactors of elements of A are given by > I '
k
Cji = 4, C^2 = - 3, C21 = -(-1) = 1 and C 22
Yo
= 2.
oo

-iT

adjA = ^ 2
-3 4 r
-3 2
B

^ adj A -
-1 1 4 1 4/11 1/ir
Hence, A
re

11 -3 2 -3/11 2/11
u

8 4 2
ad
Yo

Find the inverse of the matrix A = 2 9 4


1 2 8
d

SOLUTION We have,
Re

I 8 4 2{
in

|A|=|2 9 4| = 8{72-8)-4(16-4) + 2(4-9)=454^0


F

I 1 2 8|
Thus, A is a non-singular matrix and therefore it is invertible.
Let Cjj be cofactor of Ufj in A. Then,
Cn - ' 9 4[ = 64, C 12 --
2 4
= -12, C 13 -
2 9
= -5
2 81 1 8 1 2 ~
4 2 8 2 8 4
C 21
2 8 ““ ^22 = 1 8
= 62, C 23
1 21“
= -12

4 2 8 2 8 4
C 31 - = -2, C 32 =-
9 41“ = -28, C 33 - = 64
12 4|“ 2 9
-iT
64 -12 -5 64 -28 -2
adj/1 = -28 62 -12 -12 62 - 28
- 2 - 28 64 -5 -12 64
9.5
INVERSE AND APPLICATIONS OF MATRICES

64 -28 -2
1
Hence, A ^ = —— adj A - 454
-12 62 -28
-5 -12 64

9.3 SOME USEFUL RESULTS ON INVERTIBLE MATRICES


In this section, we shall discuss some useful results on inverse of a matrix. VVe shall state and
prove these results as theorems given below.
THEOREM 1 (CaiKcUntion Laws) Let A, S, C be square inairices of the satiie order n. !f A is a

non-singular matrix, then


(i) AB = AC => B = C [Left cancellation lazu]
(ii) BA = CA ^ B = C [Right Cancellation laio]

w
-1
exists.
PKOOF (i) Since A is a non-singular matrix i.e.] A I 9^ 0. So.
Now, AB = AC

A“^ (AB) = A“^ (AC) [Pre-multiplying both sides by A

lo
(A“^ A)B = (A"^A)C [By associativity of multiplication]

e
re
[V
in B = i„C
rF /„ B = B and C =C]

F
B = C

Similarly, we can prove that 6A=CA=> B=C. Q.E.D.

r
A j ^ 0. Ofhenvise we can find matrices such
fo
u
lUMARK The result AB = AC B =C is true onh/ zohen |
that AB = AC but B^C as given below.
ks
r-2 O' 0 0
Yo
1 2
Let A = , B = ,C = . Then
oo

3 6 0 0 -1 0

■-2 0 1 2 0 0 -2 O'
ri 21 r-2 0
eB

AB = and AC = -6 0 ●
3 6 0 0 -6 0 3 6 -10

Clearly, AB = AC but B^C.


ur

THEOREM 2 (Reversal
i Law) If A and B are invertible matrices of the same order, then show that AB is
ad
Yo

invertible and (AB)


-1
= B"'* A"^
PROOl It is given that A and B are invertible matrices.
d

I A 15* 0 and | 619^ 0


Re
in

A| 1B|9^0
I AB| O [V 1 AB|=|A1 |Bll
F

AB is a invertible matrix.
Now,
(AB) (B“'‘ A“^) = A (6B“^) A
-1
[By associativity of multiplication]
-1 [V
(AB)(B-^“^) =(A/„) A
-1 [V AI„=A]
(AB) (B'^ A'^) = AA
[v
(AB)(B“^A^^) = /
and. (6“M“^)(AB) = B"^(A"M)B [By associativity of multiplication]
[V
(B-M-^)(AB) = B'^(i„B)
-1 [V I„B = B]
(B-^A“^)(AB) = B B

(B"'* A"^)(AB) = / II
[V b-'b = ;„]
Thus, (AB)(B“^A^^) = /„ = (B”^ A"^)(AB).
9.6
APPLIED MATHEMATICS-XII

Hence, (AB) ^ = B ^ A Q.E.D


RLMARK // A, B, Care invertible matrices, of the same order then (ABCF ^ = C~^ B~^ A^^.
THEOREM 3 If A is an invertible square matrix, then a"^ is also invertible and
FROOi Since A is invertible matrix.
|A| ^ 0
M^'l .. 0
T
A is also invertible
Now,
AA~'^ = 1„ = A~^A

w
(AA-y = (ly = (A-y) T
(A-y (a'^) = /„ = a'^ (A-y [By reversal law for transpose]

lo
= (A-y [By definition of inverse]

e
re
Q.E.D

rF
The inverse of an invertible symmetric matrix is a symmetric matrix.

F
1 1'. wJO I
Let ^ be an invertible symmetric matrix. Then, [/I I Q and = /I.
Now, {A-y - 1

r
T
[V A' = A]
fo
u
- 1 ;
A IS a symmetric matrix.
ks
\ LOIR Let A be a non-singular symmetric matrix. Then, A“
1
Yo
exists.
-1 1
Now, AA
oo

= / = A"‘A

{AA^y = {y = (A^^ A) T
B

(A~y a'^ = I = A^{A"y


e

{A-y a = I = A{A~y
ur

[V A^^A]
ad

-1
A = (A-y
Yo

[By definition of inverse]


-1 ●
A is symmetric.
Q.E.D
d

THEOREMS Let A be a non-singiflar Square matrix of order n. Then, |ad]A| = |A|"“^


Re
in

RROOl We have,
F

A (adj A) = I A I / n

'Ml 0 0 0
0 |A| 0 0
A(adj A) = 0 0 IA 0

0 0 0... |A
A| 0 0 0
0 I A| 0 0
l^(adjA)| = 0
0 |A| ... 0 = |A|
n

0 0 0 ... |A|
I A I [ adj A I = I A |M
[V |/iS|=Ml \ B\]
I adj A| = I A ^
Q.E.D
INVERSE AND APPLICATIONS OF MATRICES 9.7

ILLUSTRATION !fA is an invertible matrix of order 3 and\A\ =5, then find\adj A\.
SOLUTION Here A is an invertible matrix of order 3.
«-l
|adjA| =\Af [Using: |adjA|=[Aj ]

ad\A \ =5^ = 25 [v 1A| =5(given)]


THEOREM 6 If A and B are non-singular square matrices of the same order, then
adj AB = (adj B) {adj A)
PROOF Since A and B are non-singular square matrices of the same order. Therefore, AB exists
such that
AB A\\B\ 0 [●.● I AI 0,1 BI 0]

ow
We know that (AB) (adj AB) =jAB| In ...(i)

Also, (AB) (adj B adj A) = (A (B ■ adj B) adj A [By associativity of multiplication]


= (A|B|/„) adj A [v BadjB=|B|/„]

e
= |B|(A adj A) [■-■ A1„=A]

re
= 1B|(|A|W

rFl [●.* A adj A =1 A I i„]

F
= |A| 1B|I n

= I AB I Z [v |AB|=|A| IBj]

r
ou
Thus, (AB) (adj 6 adj A) =|A6|/ ...(ii)
From (i) and (ii), we get
n
fo
ks
(AB) (adj AB) = (AB) (adj B ■ adj A)
(AB)“^ ((AB) (adj AB)) = (AB)" ^ ((AB) (adj B . adj A))
oo
Y

((AB)"^ (AB)) (adj AB) = ((AB)-(AB)) (adj B. adj A)


eB

/(adjAB) = / (adj B. adj A)


adj AB = adj B, adj A Q.E.D
r

T T
ou

THEOREM 7 If A is an invertible square matrix, then adj A = (adj A) .


Y
ad

PROOt Since A is an invertible matrix.

|A| ^ 0
d

T
IA^l ^ 0 [● ● I A = MI]
in
Re

A^ is invertible.
F

We know that

A adj A = I AI / n
T
(A adj A)’’ = (|/l|J„)
(adj4)’'(A’') =\A\I„ ...(i)
Also,

(adjA^)(A^) = |A^|7 n

(adjA^)(A^) =\A\1 ji
...(ii)

From (i) and (ii), we get


(adj/l'^H/l^) = (adjAf(A^)
adj A = (adj^l)’'
T
[By right cancellation law]
Q.E.D
9.8 APPLIED MATHEMATiCS-XII

THEOREM 8 Prove that adjoint of a symmetric matrix is also a symmetric matrix.


T
EKOOF Let be a symmetric matrix. Then, A = /I

We know that

(adj A)'^ = (adj A^)


(adj.A)^ = adjv4 [■.● = /I]
adj ir is a symmetric matrix. Q.E.D.
ih-2 A.
THEOREMS If A is a non-shigitlar square ymtrix, then adj (adj/4) = \A
PitOOF We know that B (adj B) =| B\I„ for every square matrix of order n.
Replacing 6 by adj A, we get
(adj A) [adj (adj v4)] = | adj /I | I n
iii-l ii/-]
(adj ir) [adj (adj ir)] = 1 [: I adj I =1 >1

w
n

i?j-l
A l(adj A) (adj adj A)| = A [| ir h,\ [Pre-mulHplying both sides by A]
ii(-l

F lo
{A adj A) (adj adj /i) = | A [By associativity of multiplication]
iii-l
=>
I AI /„ (adj adj A) = \ A A [●.■ AI,i =A and AadjA=\A \ /„]

ee
|/l|{/„(adjadjv4)) = \A\ »-l^

Fr
|H-1 A
I AI (adj adj A) = \ A
1)1-2 1
adj adj A - \ A A. Multiplying both sides by
for
ur
Q.E.D
s
COROLLARY If A is a non-singular matrix of order n, then |adj(adj A)|=|A|*"
ok
Yo

PROOF We know that


o

n-2
adj (adj A) = |A|
eB

A
n-2
|adj(adj A)| = ||A|
|adj(adj A)| = |A| /i(h-2) 1^1
r

[v \kA\=k”\A\]
ou
ad

adj(adjA)| = |A|"^ -211+1


Y

ILLUSTRATION :
If A is an invertible matrix of order 3x3 such lhat\A\ = 2. Then, find adj (adj A).
Re
nd

SOLUTION Replacing n by 3 in the above theorem, we get


3-2
adj (adj A) = |A| A=|A|A=2A
Fi

ILLUSTRATIO.N ' If A is a square matrix of order 3 such that IA|=2, then write the value of
\adj{adjA)\.
SOLUTION If A is a square matrix of order )i, then |adj(adjA)| = |A| (»-l)^
Here A is a square matrix of order 3 such that | A | =2.
|adj(adjA)| = 2^3-1)^ ^2^ =16
'3 0 -1'
ILLUSTR.ATION-: //A = 2 3 0 , then find \adj(adjA)\.
0 4 1

SOLUTION Here,
3 0 -1

|A| = 2 3 0 = 3(3-0)-0(2-0)-l(8-0)=l
0 4 1
JNVERSE AND APPLICATIONS OF MATRICES 9.9

(«-l)2
If/I is a square matrix of order then I adj(adj A) I = |^|
So, for the given matrix, we obtain
|adj(adj/l)| = |/\|“ = 1
THEOREM 10 If the product of two non-null square matrices is a null matrix, shozu that both of them
must be singular matrices.
PROOi Let A and B be two non-null square matrices of the same order n x n. It is given that
AB-0 {null matrix). If possible, let Bbea non-singular matrix. Then, B~^ exists.

ow
AB = O

{AB) B~'^ = OB
-1
[Post-multiplying both sides by B
1
A(BB“‘) = O [By associativity of multiplication ]

e
-1
= AJ

re
=> Al„ = O [●●● BB
A = O.

F
Frl
But, i4 is a non-null matrix. Therefore, our supposition is wrong. Hence, B is a singular matrix.
Similarly it can be shown that /I is a singular matrix. Q.E.D.
ou -1
A-'
1
THEOREM 11 If A is a non-singular matrix, then prove that\ A = |A|-l i

sr
i.e.

PROOF Since | A | 0, therefore A


AA
-1
= I = A“^ A
-1
exists such that
kfo
oo
-1
AA I [Taking determinant of both sides]
Y
reB

-1
A| I A = 1 [v I AB| =1 A| I B| and | /| =1]
1
A-'I = [V |A|^0]
uY

Q.E.D.
ad
do

ILLUSTRATIVE EXAMPLES
in

1 i/;n- [ FINDING THE ADJOINT AND INVERSE OF A MATRIX


Re

a b
EXAMPLE 1 //A = c find adj A.
F

d '

SOLUTION Let Qy be the cofactor of in A = [fl,yj. Then,


C];! = Cofactor of =(-1)^“^ ^ d =^d, C^2 = Cofactor of =-(-1)^"^ ^ c = - c
2+ 1 2+2
C21 = Cofactor of ii2] = (“ 1) b --b and, C22 = Cofactor of /?22 ~(“ 1) a = a

-iT
Cn C 12 iT d -c d -b
adj A = C 21
C22J -b a - c a

31 -1 1
EXAMPLE 2
_ 2 / shozv that A = —A.
19

SOLUTION We have,

lAI-^
1^1-5 -2^ = _4_15=_19;±0.

Therefore, A is invertible. Let Cq be the cofactor of Ojj in A = [i^y]. Then,


9.10 APPLIED MATHEMATICS-XI!

Cii — ~ 2, C|2 ~ ~5, C21 ~ ~ 3 and C79 = 2.


-iT
-2 -5 -2 -3
adj A = -3 2 -5 2

1
Now, A-' adj A

A-1 j_r-2 -3]_ 1 r2 3 ^A.


-19|_-5 2j”l9[5 -2j 19
'1 3 3]
EXAMPl.E 3 Find the inverse of A =14 3 and verify that A~^ A =l2-
1 3 4

1 3 3

w
SOLLTFION We have, A = 1 4 3
1 3 4

F lo
|1 3 3|
|A|=1 4 3 = (16-9)-3(4-3) + 3(3-4)=7-3-3=1 0.
|1 3 4|

ee
Fr
So, A is invertible.

Let Cjj be the cofactor of Ojj in A = [^3iy]. Then, for


Cn=(-1)
l+l|4 3| = 7, Cl2=(-1)
l+2|l 3 _ -1, 1+ 3 _ -1
Ci3=(-1)
|3 4| |1 4
ur
|1 3
2+ 1 3 3| _ -3, C22=(-l) 2+2 1 3
2+ 3 |1 3
oks

C2i=(-1) = 1, C23=(-l) = 0
3 4|
Yo

1 4 |1 3
o
eB

3 3
C3l=(-1)
3+ 1
= -3, C32=(-l)
3+2 1 3
= 0, C33=(-l)
3+3 1 3|_
4 3 1 3 1 4|-^
-iT -iT
our

Cn C 12 C 13 7 -1 -1 7 -3 -3
ad

adj A - C21 C22 C23 -3 1 0 -1 1 0


C 31 c 32 c 33 -3 0 1 1 0 1
Y

^ adj A
Re

Now, A
Ml
nd
Fi

7 -3 -3 7 -3 -3
-1 1
=> A -1 1 0 -1 1 0
1
-1 0 1 -1 0 1

7 -3 -3 1 3 3 7- 3-3 21 -12-9 21 -9-12 1 0 0


-1
A A = -1 1 0 1 4 3 -1+1+0 -3+4+0 -3+3+0 0 1 0
-1 0 1 1 3 4 -1+0+1 -3+0+3 -3+0+4 0 0 1

[3 21 6 7
EXAMPLE 4 I/A = 7 5
and B =
8 9 '
verify that (AB)"^ = A“l
3 2
SOLUTION Clearly, | A | = 7 5
= 15 -14 = 1 0. So, A is invertible.

Let Ajj be the cofactor of elements a^j in A = [ojj]. Then,


1+ 1 1+ 2 2+ 1 2+2
Aii=(-1) 5=5, Ai2=(-1) 7=-7,A2i=(-1) 2 — — 2 and A22 — (~ 1) 3 = 3.
9.11
INVERSE AND APPUCATIONS OF MATRICES

A 12 iT
-iT
A 11 5 -7 5-2'
adj A = A 21 A 22 -2 3 -7 3

'5-2'
Hence, A ^ = ^ adj A =
A!
-7 3

6 7

w
We have, B =
8 9

6 7
B = 54-56 =-2^0.
8 9|

e
So, B is invertible.

e
Let Bjj be the cofactors of bjj in B= . Then,

or
r
1+ 2 2+ 1 2+2
1+ 1
Bll=(-1) 9=9, Bi2=(-1) 8=-8,B2i=(-1) 7=-7andB22=(-l) 6 = 6.

F
-,T
[B 11 B 12 9 -8 ■ 9 -7'
adj B =
oF
ul
B 21 6 22 -7 6 -8 6

1 1 9 -7

rs
Hence, B~^ = -—-
B
adj B = - -2-8 6

ko
We know that adj A6 =adj B. adj A.
adj AB -
r 9
-8
-7ir
6 -7
5 -2
3
■ 94
-82
-39
34
of
o
Y
We also know that | AB | = j .A | | B|.
B

|AB|=lx-2 = -2^0.
Y

So, AB is invertible.
er

94 -39 1 94 -39
Hence, (AB)~^ — adj (AB) = -
_2 -82 34 2 -82 34
...(i)
u

ABi
d

9 -7ir 5 -2 1 94 -39
●●●(ii)
o

Also,
ad

2 -8 6 -7 3 2-82 34
in

From (i) and (ii), we get


(AB)-'* = B”^ A“^
Re
F

note Students are advised 7wt to find the product AB and (A6)"^ by the usual technique.
Type I! FINDING THE INVERSE OF A MATRIX A WHEN IT SATISFIES SOME MATRIX EQUATION
f(A)^ O.
'2 -3'
EXAMPLE 5 Show that A =
3 4
satisfies the equation - 6a: + 17=0. Hence, find A *.
2 -3‘
SOLUTION We have, A =
3 4/
-5 -18
A^ = AA = '2 -3] [2 -3' '4-9
-6-12 ^
3 4 3 4 " 6 + 12 -9 + 16 18 7

'2 -3' -12 18 n 01 17 O'


-6A=(-6) and, 171=17 0 1 0 17 ■
3 4 -18 -24

-5 -18 -12 18' 17 0


A^ -6A +1712 = 18 7
+
-18 -24
+
0 17
9.12 APPLIED MATHEMATICS-XII

■-5-12+ 17 -18 + 18 + 01 0 O'


-6A + 17l2 = 18-18 + 0 7-24 + 17 0 0
= 0.

Hence, the matrix A satisfies the equation ,v“ - 6.r +17 = 0.


Now,

-6^+ 17/2=0
A^-6A =-17/2
A~^ {A^-6A)=A~^ i-l7l2) [Pre-multiplying both sides by/I ’]
A-'^ A~-6A-'' A=-J7(A-'^ I2)

ow
-1
A-6I2 =-17A
A-^ = :;^(A-6/2)=
17 ^17^
—(6/2-^) =—I
17 1
6
0
0
6
'2
3
-3'
4
1

17
4
-3
3
2

[3 11

e
EXAMl’l.F. b For the matrix A =
7 5 , find X andy so that A^ + xl = \/A.

re
Hence, find A \
Fl
F
3 1
SOLUTION We have, A =
ur
7 5 ●

r
A^=AA =
3 1 3 1 ■ 9 + 7 3+5 16 fo
ks
7 5 7 5 21 + 35 7 + 25 56 32
Yo
Now, + x] =yA
oo

16 ‘1 0 3 1
eB

+ .V
56 32 0 1 =y
7 5

16 + .V 8 + 0 _ 3y y
ur

56 + 0 32 + .Y ~ 71/ 5 [/
ad
Yo

=> 16 + Y = 3y, y = 8, 7y =56,5y = 32 + y


Putting y = 8 in 16 + Y = 3y, we get; y = 24 -16 = 8.
d

Clearly y = 8 and y = 8 also satisfy equations 7y = 56 and 5y = 32 + y.


Re
in

Hence, y = 8 and y = 8.
F

3 1
Now, \A\ = 7 5
= 8 0

So, A is invertible.

Putting Y = 8, y = 8 in /I^ + xl =\jA,we get


+ 8/ = 8A

=> A-'^ {A^ + 81) = A~'^ (8A) [Pre-multiplying throughout by A


-1
=> A A^ + 8A-'^ I = SiA-'^ A)
-1
=> A + 8A = 81 [-.-A A^ ={A~^ A)A=IA=^A,A~'^ I = A 1
and, A
1
A = I]
=> 8A~'^=81-A
[8 01 3 1
^i_rs-3 0-1 1 5 -1 5/8 -1/8'
=> A-^=-{8I-A)=-
8 0 8 7 5
~8 .0-7 8-5j"8[-7 3 -7/8 3/8
9.13
INVERSE AND APPLICATIONS OF MATRICES

EXAMPLf. -
For the matrix A = ^ ^ , find the numbers a and b such that + aA + bl - 0.
Hence, find A
■3 2
SOLUTION We have, A = 1 1

. . f3 2' 3 2' 11
■ A^
= ^ = [1 1 1 1 4 3

Now, A^ + aA+b 1 =0
T1 3 2 1 01 0 O'
+ a + b
4 3 1 1 0 1 0 0

ow
11+ 3a + b 8 + 2fl 0 0
A + a 3 + a +b 0 0

11 + 3a + b = 0,8 + 2a = 0, 4 + a = 0 and 3 + a + b=0


=> a = - 4 and b =1

e
Putting a = -4 and ^ = 1 in (i), we get

re
A^-4A + I =0

Fl
F
=> 4A~a'^ = I
ur
r
^ A(4I-A) = I
= 41 -A =
4
0
0
4
■3
1
2
1 -1
1 -2
3
fo
ks
Yo
‘1 2 2
oo

LXAN'iTLLS Shozv that the matrix A = 2 12 satisifes the equation A^-4A-51 ^ =0 and hence
2 2 1
eB

findA-\
12 2
ur

SOLUTION We have, A = 2 1 2
ad

2 2 1
Yo

1 2 2 9 8 8 '4 8 8 5 0 0
[12 2
d

^2=A^= 2 1 2 2 1 2 8 9 8 , 4A = 8 4 8 and, 5/3=05 0


Re

0 0 5
in

2 2 1 2 2 1 8 8 9 8 8 4
F

9 8 8 4 8 8 5 0 0

A^-4A-5l3= 8 9 8 8 4 8 0 5 0

[8 8 9 8 8 4 0 0 5

[9-4-5 8-8-0 8-8-0 0


0
0
0
0
0 = O
g_8_o 9-4-5 8-8-0
8-8-0 8-8-0 9-4-5 0 0 0

Now, -4/1-5/3=0
A^-4A=5l3
A
-1
-4A"^ A =5A"^ /3 [Pre-multiplying throughout by A"']
A -4/ =5A"^ [v A-^ A^ =(A~^ A) A = IA= A]
A'^=-(A-4/)
5
9.14 APPLIED MATHEMATICS-Xll

1 2 2 4 0 0 -3 2 2 - 3/5 2/5 2/5


A 1_1 2 1 2 0 4 0
1
2-3 '2
1
2/5 - 2/5 2/5
5
2 2 1 0 0 4
^1_ 2 2 -3j ^ 2/5 2/5 - 3/5

Type III FINDING THE INVERSE OF A MATRIX BY USING THE DEFINITION OF INVERSE

‘1 -1 1]
EXAMPLES 1/A=2 -1 0 , show that A~^=A^.
1 0 0

SOLUTION We know thata matrix Bis the inverse ofa matrix/I if =/ = 6/4. Here, we have to

ow
show that /4^ is the inverse of A. Therefore, it is sufficient to prove that A^ A = I or, A^ = I.
Now, A^=AA
1 -1 1 1 -1 1 1-2+1 -1+1+0 1+0+0 0 0 1
A^ = 2-1 0 2 -10 2-2+0 -2+1+0 2+0+0 0 -1 2

e
1 0 0 1 0 0 1+0+0 -1+0+0 1+0+0 1 -1 1

re
/4^ =A^A

Frl
and.

F
0 0 1 1 -1 1 0+0 + 1 0+0+0 0+0+0 1 0 0
= 0 -1 2 2-10 0-2 + 2 0+1+0 0+0+0 0 1 0 = I
ou
or
1 -1 1 1 0 0 1-2 + 1 -1+1+0 1+0+0 0 0 1

Hence, A^=A
kfs
Type IV ON SOLVING MATRIX EQUATIONS
oo

1 -2 6 0
EXAMPLE 10 Find a 2x 2matrix B such that B
Y

1 4 0 6 '
eB

‘1 -2 F6 01
SOLUTION Let A = and C = . Then, | /41 = 1
1 -2|_= 6 9^ 0.
1 4 0 6 41'
ur
oY

So, A is invertible.
ad

The given matrix equation is


1 -21 _ re 0"
d

6
1 4 " 0 6
in
Re

BA =C

{BA)A~'^=CA -1
F

[Post-multiplying throughout by A
BiAA)-'^ =CA~'^
Bl = CA~^
B=CA-\

Let Cjj be the cofactor


1+ 1
of Ojj in /4 = [Ojj]. Then,
■4=4, Ci2=(-1)
1+ 2 2+ 1
Cii=(-1) 1=-1, C2i=(-1) (-2)=2
2+2
and. C22=(-l) 1 =1.

rc 11 c 12 -|T 4 -1 -iT 4 2'


adj A =
C21 C22_ 2 1 -1 1

4 2'
Hence, A~^=-^adjA
Ml
= -6 -1 1
INVERSE AND APPLICATIONS OF MATRICES 9.15

Now,
B=CA~^
6 01 1 4 2 ir6 0 4 2' ir24+0 12 + 0' ■ 4 2'
B =
0 6 6 0 6 -1 1 6 0-6 0+6 -1 1
6 1
2 1 -3 2 1 0
EXAMPLE 11 Find the matrix A satisfying the matrix equation A
3 2 5 -3 0 1 ■

2 1 -3 2
SOLUTION Let B = and C = . Then,
3 2 5 -3

ow
2 1 -3 2
|B| = 3 2
= 4- 3= l#0 and, | C | = 5 -3
= 9-10=-l?^0.

So, B and C are invertible matrices. The given matrix equation is BAC = I.
Now, BAC = I

e
(BAC)C“^=B“^ IC
-1

re
B) A(CC"^) = B“^C“^

Frl
F
IAI = B~'^C~^
A=B“^ C"^
ou
r
Let Bjj be the cofactor of elements bjj in B=\bjj]. Then,
so
1+ 1 1+2 2+ 1
kf 2+2
%=(-!) 2 = 2, B^2=(-1) 3=-3, S2i=(-1) 1 = -l and, 622 ={-l) 2 = 2.
oo
-iT
B 11 ^12 2 -3 ■ 2 -1'
adj B = -1 2 -3 2
Y

B21 B22
eB

■ 2 -r
So, B"^=^adjB = -3 2 ●
[V |6|=11
1B|
ur
oY

Let Cjj be the cofactors of elements Cy in C = [Cy j. Then,


ad

1+ 1 1+ 2 2+ 1
Cii=(-1) (- 3)=-3,Ci2=(-1) 5=-5, C2i=(-1) 2=-2
d

2+ 2
and. C22=(-l) (- 3)=-3.
in
Re

nT
rc 11 C 12 iT -3 -5 -3 -2
adj C = -5 -3
.^21 ^22 -2 -3
F

-1 1 -3 -2 3 2
So, C adj C = - -5 -3 5 3
[V iC|=-l]
|Cl

Substituting the values of B ^ and C ^ in (i), we get


2 -lir3 2' 6-5 4-3' '1 1
A = B"^ C"^ -9 + 10 -6 + 6 1 0
-3 2 5 3

‘1 -4' -16 -6
EX.AMPLE 12 Find the matrix Xfor which 3 -2
X =
7 2

41 ‘-16 -6
SOLUTION Let A = \3 -2
and B =
7 2 . Then the given matrix equation is AX = B.

|A| = |3
|1 -4 = -2 + 12=10^^0.
-2
9.16 APPLIED MATHEMATICS-XII

So, A is an invertible matrix. Let Cjj be the cofactors of elements Ojj inA= Then,
Cn =(-!)' (-2)=-2,Ci2=(-1)
1+ 1 1+ 2 2+ 1
3=-3,C2i=(-1) (-4) =4
2+2
and. C22=(-l) 1 =1.

-iT

adj A =
-2 -3
_ r-2 4'
4 1
“ [-3 1
So, A~^^ = ad] A = j_r-2 4'
10-3 1 ●
1^1
Now, AX B

A~^{AX) = /I -1 B

{A-^A)X = A -]

low
B

=> IX = B

X = B.

1 -2 4ir-16 -6
_J_r32 + 28 12 + 8 6 2'

ee
X = —
10 -3 1 7 2j ~ 10 48 + 7 18 + 2 11/2 2 ●

F
Fr
0 1 3 1/2 -4 5/2'
EXAMPLE 13 -1
IfA = 1 2 .T and A -1/2 3 -3/2 , find X, y.
for
ur
2 3 1 1/2 y 1/2
SOLUTION We know that
ks

AA-^=l,
Yo
oo

0 1 3iri/2 -4 5/2 1 0 0
eB

1 2 .Y -1/2 3 -3/2 0 1 0

231 1/2 .1/ 1/2 0 0 1


r

1 3+ 3y 0
ou
ad

1 0 0
1 X 1 Y
—+- 2 + yi/ 0 1 0
2^2
Y

2 2
0 1 0 0 1
nd
Re

3+3y = 0 2 + .vy = l, l + i/ = 0
Fi

Y = l,y = -1
I’ypc VI MISCELLANEOUS PROBLEMS
rXAMPi.l T Let A be a non-singular matrix. Show that a'^A~^ is symmetric if/A^ =(A^)^.
SOLUTION First, let A~^A~^ be symmetric. Then,
{A'^A-y =A^A~^
(A~Y{A'^f =a'^A-'^
(a'^Y a=a'^a~'^ V (A-y

a"^ (A'^)-y A=A'^(A'^AyA

A‘T (A'^)~^ AA=(A'^A'^)iA-y)


INVERSE AND APPLICATIONS OF MATRICES 9.17

A-={A'^foT, {A'^f=A^
2 T 2
Conversely, let A be a non-singular matrix such that A'^ = {A ) . Then,
={A‘^f
aa=a'^a'^
T —1
Pre and post multiplying by {A )
(y4'^)“\AA) 71"^ ={A'^)~'^{A'^a'^) a
-1

and A~^ respectively


=>
{A-'^f A (AA-^)= (A'^)-'^A ]{A'^A~^)

low
{A~^f A I=1{A'^A~'^)

(A"^)'^A= A^A
-1

ee
rF
Fr
(A“^^(A^)^ = A^A“^
{A'^A-y = A^A^^
T —1
for
u
=S‘ A A is a symmetric matrix.
ks
EXERCISE 9.1
Yo

I. Find the adjoint of each of the following matrices:


o
Bo

-3 5 a b cos a sin a
(i) 2 4
(H) (iii)
c d sm a cos a
re

Verify that(adj A) A = | A | / = A (adj A) for the above matrices.


ou
ad

Compute the adjoint of each of the following matrices:


Y

T 2 2 1 2 5

(i) 2 1 2 (ii) 2 3 1
nd

2 2 1 -1 1 1
Re
Fi

Verify that (adj A) A = | A | / = A (adj A) for the above matrices.


'1-1 r
; For the matrix A = 2 3 0 , show that A (adj A) =0.
18 2 10

-4 -3 -3
4. IfA = 1 0 1 , show that adj A = A.
4 4 3j
-1 -2 -2]
5. If A = 2 1-2 , show that adj A =3A^ .
2 -2 1

1 -2 3

6. Find A (adj A) for the matrix A = 0 2 -1

-4 5 2
9.18 APPLIED MATHEMATICS-XII

7. Find the inverse of each of the following matrices:


a b
'0 r 2 5'
(i) 1 0
(ii) \ + be (iii) -3 1
c
a

8. Find the inverse of each of the following matrices.


'1 2 3 1 2 5 2 -1 1 1 0 0
(i) 2 3 1 (ii) 1 -1 -1 (in) -1 2 -1 (iv) 0 cos a sin a
3 1 2 2 3-1 1 -1 2 0 sin a - cos a

ow
9. Find the inverse of each of the following matrices and verify that A
'1 3 3 2 3 1
(i) 1 4 3 (ii) 3 4 1
1 3 4 3 7 2

e
-1
10. For the following pah's of matrices verify that (AB) = B“^ A~\

re
3 2 4 6 '2 1 '4 5
(i) A = 7

Frl
and B = (ii) ^ = and B =

F
5 ‘ 3 2 5 3 3 4

3 2 6 7
11. Let^ = and B = . Find (AB)-'^
ou
7 5 8 9

r
12. Given A =
■ 2 -3
,computeA ‘ and show that 2A ^=9/-A.
1
so
-4 7
kf
'4 5
oo

13. If A =
2 1 , then show that A - 3/ = 2 (/ + 3A ^).
Y
eB

a b
-1
14. Find the inverse of the matrix A =
1 + be
and show that aA = {a^ +bc + \) I -nA.
ur

c
oY

a
ad

5 0 4 1 3 3
15. Given A = 2 3 2 , = 14 3. Compute (AB)~\
d

1 2 1
1 3 4J
in

2 3 1 0 0 0
Re

16. If A =
1 2 , verify that A^ - 4 A + 7 = O, where I = 0 1
and O =
‘ 0 0
. Hence, find
F

A-^
3 1
17. IfA =
-1 2
, show that A^ - 5A +7 / = 0. Hence, find A
■4 3
18. IfA =
2 5 , find .r and y such that A^ -xA + i/I =0. Hence, evaluate A
3-2
19. If A =
4 -2
, find the value of Xso that A^ =XA - 21. Hence, find A“

■ 5 3
20. Show that A = satisfies the equation .r - 3.v - 7 = 0. Thus, findA“\
-1 -2

1 1 1
21. For the matrix A = 1 2 3 . Show that A ^ -6A^+5A + ll/3=0. Hence, find A
2-1 3
9.19
INVERSE AND APPLICATIONS OF MATRICES

1 0 -2
3 2
22. Show that the matrix, A = -2 -1 2 satisfies the equation, A -A 3^-/3 =0.
3 4 1

Hence, find A V
r 2 -1 1
23. lfA = -1 2 -1 . Verify that A^ - 6A^ + 9A-41 =0 and hence find A
1 -1 2

8 14
1
24. If = 4 4 7 , prove that 71 ^=A'^.

ow
9
1-8 4

■3 -3 4l
25. UA= 2 -3 4 , show that
0 -1 1_

e
'-1 2 0]

re
26. If-4= -1 1 1 , show that

rFl
F
0 1 0

27. Solve the matrix equation


■5 4
x-P
^ 1 3 , where X is a 2 x 2 matrix.

r
1 1
ou
28. Find the matrix X satisfying the matrix equation:
fo X _^
5 3 T4 7'
ks
2 7 7 ■

3 2' -1 1 2 -r
oo

29. Find the matrix X for which: X


7 5 -2 1 0 4 ●
Y
B

2 1 ■5 3 1 0
30. Find the matrix X satisfying the equation: X
5 3 3 2 0 1 ●
re

31. If tUs a square matrix of order n, prove that ] ,4 adj A | = | A\’\


ou

■ 1 2-2’
Y

3 -1 1
ad

32. If/4"^ = -15 6 -5 and B = -13 0 ,find(y4B)“^.


5-2 2 0 -2 1
d
in
Re

1 -2 3

33. If 71 = 0-1 4 ,find {A'^)~\


F

-2 2 1

-1 -2 -2

34. Find the adjoint of the matrix 7l = 2 1-2 and hence show that 7l (adj 7l) = | 7l | / 3.
2 -2 1

'0 1 1
-1
35. If = 1 0 1 , find A~^ and show that A 2
1 1 0

ANSWERS

' 4 -5' d -b cos a - sin a


1. (i) -2 -3
(ii) - C (7
(iii) - sm a cos a
9.20 APPLIED MATHEMATICS-XII

-3 2 2 2 3-13 3 -1 1
2. (i) 2-3 2 (ii) -3 6 9 (iii) -15 7 -5
2 2-3 5-3 -1
L 4 -2 2

1+bc
25 0 0 -b
6. 0 25 0
rO 11 a ..... 1 fl -5
7. (i) (ii)
0 0 25
1 0
-1' a
l7 3 2

-5 1 7 4 17 3 3 1 -1
8- (i) ^
18
1 7 -5 (ii)
27
1
-1 -11 6 (iii) T
1

4
1 3 1
7 -5 1 5 1-3 -1 1 3

ow
1 0 0 7 -3 -3 1 1 -1
1
(iv) 0 cos a sin a (i) -1 1 0 (ii) T -3 1 1
2
0 sin a - cos a -1 0 1 9 -5 -1

39 1 + be
-47 -b -2 19 -27

e
11. 2 14. n
15. -2 18 -25

re
41 -17 - c

rFl a -3 29 -42

F
'2-3' 1 2 -1
18. .r = 9, y=14 J_
Jh. 17. - 5 -3
-1 2 7 1 3 '14-2 4

r
ou
X = 1,A
-1 i_ r-2 2
2-4 3
20.
1
-
7 -1
2

-5
3
k
2!
sfo
11
1
-3
9 -1
4 5
-4
5 -3 -1
oo

-9 -8 -2
8 7 2
1 3 1 -1 -3 -14'
Y

- 1 3 1
4 17
B

-5 -4 -1
[-1 1 3
3 r ■-16 3' ■ 9-14
re

2S. 29. 30.


1 -2 24 -5 -16 25
ou
Y

-9 8 -2
ad

9-3 5
-2 1 0 33. 8 7 2
1 0 2 -5 -4 -1
d
in
Re

HINTS TO SELECTED PROBLEMS


F

15
We have tofind(/4B)“ ^ and we are given the values of/I and B~ \ But,(AB)“ ^ =B~^ A~^.
So, we need to find A~ \
Now,
5 0 4

A = 2 3 2 ^jA|=5(3-4)-0(2-2) + 4(4-3)=-5 + 4= -l7^0


1 2 1

-1
So, A exists.

Let Qj be cofactor of Oj-j in A = [ojj]. Then,


C 11 = 3-4 = -1,C 12
= -(2-2) = 0, Ci3 = 4-3 = l,C2i = -(0-8) = 8
C22 = 5-4 = 1, C23 = -(10-0) = -10,C3i = (0-12) = -12,
C32 = -(10-8) = -2,C33 = 15
INVERSE AND APPLICATIONS OF MATRICES 9.21

^12 ^13 -1 0 1 -1 8 -12

adj A = ^21 ^22 ^23 1 -10 0 1 -2

-12 -2 15 1 -10 15
c 31 C 32 C 33

1 12
1 1
So, A adj A = 0 -1 2
Ml -1 10 -15

1 3 3 1 12 -2 19 -27
-1
Hence, (AB) = 0-1 = 1 4 3 0 -1 2 -2 18 -25
1 3 4 -1 10 -15 - 3 29 - 42

ow
2 3
In. We have, A =
1 2

'2 3 '2 3 7 12
A^ = /l/l
1 2 1 2 4 7

e
7 12' [2 31 1 0 0 O'

re
a'^ -4A +I
4 7
-4
1 2

rFl
+
0 1 0 0
= O

F
A“’(7l^-4y4 + /) = A
-1
O [Multiplying both sides by A

r
ou
fo
ks
A-41+A“^ = O
4 O' '2 3 2 -3
oo

-1
A = 41 -A =
0 4 1 2 -1 2
Y
eB

■ 3 r
17. We have, A = -1 2
ur

3 1 3 1 8 5
A^ = AA =
ad

-1 2 -1 2 -5 3
Yo

5 3 1 0 '8-15 + 7 5-5 + 0 0 0
So,A^ -5A +7/ = -5 12 +7 =0
d

-5 3 -1 0 1 -5+5 + 0 3-10 + 7 0 0
Re
in

Now, A^ -5A + 7/ = O
F

-1
A'^(A^-5A+7/) = A O [Multiplying tliroughout by A
-1
A"^ A^ -5A“^ A +7A I = o
-1
A-51 +7 A = o

-1
=> 7A = 5/ - A

-1 '5 0 3 1 2 -1
7A
^05 -1 2 1 3

A
-1
1 \2 -r
7 [l 3_
'1 1 1
"* 5. We have, A = 1 2 -3
2-1 3
9.22 APPLIED MATHEMATICS-Xll

1 1 1 1 1 1 4 2 1 '
= AA = 1 2 - 3 1 2 -3 -3 8 -14
2-1 3 2-1 3 7-3 14

4 2 1 1 1 1 8 7 1
and, = A^ A = -3 8 -14 1 2-3 -23 27 - 69
7-3 14 2-1 3 32 -13 58

A^-6A^+5A+11/3

ow
■ 8 7 1 ■ 4 2 1 1 1 1 1 0 0
= -23 27 -69 -6 -3 8 -14 + 5 1 2 -3 + 11 0 1 0
32 -13 58 7 -3 14 2-1 3 0 0 1

8-24 + 5 + 11 7-12 + 5 + 0 1-6+5+0 0 0 0

e
-23 + 18+5 + 0 27 -48 + 10 + 11 -69 + 84-15 + 0 0 0 0 = O

re
32-42 + 10 + 0 -13 + 18-5 + 0 58-84 + 15 + 1 0 0 0

Now, A^-6A^+5A+11/3 =0

F
A~^{A^ -6A^ +5A+111^) = A O
-1

Frl [Multiplying both sides by A


A^ -6A +5/ +11A“^ = O
ou
sr
11A“^ = -A^ + 6A -51

llA"^ = - -3
r 4 2
8
1
-14 + 6
1
1
1
2
1
-3
kfo
-5
1
0
0
1
0
0
oo
7 -3 14 2-1 3 0 0 1
Y

-4+6-5 -2+6+0 -1+6+0 -3 4 5


reB

11A“^ = 3 + 6+0 -8 + 12-5 14-18 + 0 9 -1 -4


-7+12+0 3-6 + 0 -14 + 18-5 5 -3 -1
uY

-3 4 5 ■
1
A-1 = 9 -1 -4
11
ad
do

5 -3 -1

2 -1 1
in

23. We have, A = -1 2 -1
Re

1 -1 2
F

2 -1 1 2 -1 1 6-5 5
A^ = AA = -1 2 -1 -1 2 -1 -5 6 -5
1 -1 2 1 -1 2 5-5 6

6-5 5 2 -1 1 22 - 21 21
and. A^ = A^ A = -5 6 -5 -1 2 -1 21 22 - 21
5-5 6 1 -1 2 21 -21 22

A^-6A^ + 9A-4/
22 - 21 21 - 36 30 - 30 18-9 9 -4 0 0
-21 22 -21 + 30 - 36 30 + -9 18 -9 + 0-4 0
21 -21 22 -30 30 - 36 9 -9 18 0 0 -4

22-36 + 18-4 -21 + 30-9 + 0 21 -30 + 9 + 0 0 0 0


-21 + 30-9 + 0 22- 36 + 18-4 -21 + 30-9 + 0 0 0 0 = O
21 -30 + 9 + 0 -21 + 30-9 + 0 22-36 + 18-4 0 0 0
9.23
INVERSE AND APPLICATIONS OF A MATRICES

Now,A^-6A^ +9A-4I =0
-1
=> A^'^iA^-6A^+ 9A-4I) = A O [Multiplying both sides by A
-1
=> A^ -6A + 9I -4A = o

^ 4A"^ = A^-6A + 91
6-55' -12 6 -6 9 0 0
-1 0 9 0
=> 4A -5 6 -5 + 6 -12 6 +

-6 6 -12 0 0 9
5-5 6 _

ow
6_12 + 9 -5 + 6 + 0 5-6 + 0 ‘31-1
-1 : 1 3 1
=> 4A -5 + 6 + 0 6-12 + 9 -5 + 6+0
5-6 + 0 -5 + 6 + 0 6-12 + 9 -1 1 3

■ 3 1 -1'

e
-1 1
A 1 3 1

re
^-113

Frl
F
9.4 SIMULTANEOUS LINEAR EQUATIONS
Consider the following system of linear equations:
+ hiy-¥C-^ =0
ou
sor
rtz-v + b2)J + C2 = 0
SOLUTION A set of values of the variables which simiiltaneouslij satisfy all the equations is called a
kf
solution of the system of equations.
oo
For example, x = 2, y = -3 is a solution of the system of linear equations
Y

3a: + 1/ = 3
B

2-v + y=l
because 3 (2) + {-3) = 3 and, 2{2)+(-3)=l.
re

CONSISTENT SYSTEM If the system of equations has one or more solutions, then it is said to be a
Y
u

consistent system of equations, otherwise it is an inconsistent system of equations.


ad
do

For example, the system of linear equations


lx + 3y =5
4a: + 6y =10
in

is consistent, because a; =1, y =1 and a: = 2, y =1/3 are solutions of it.


Re

However, the system of linear equations


F

2a: + 3y =5
4a: + 6y = 9
is inconsistent, because there is no set of values of a:, y which satisfy the two equations
simultaneously.
HOMOGENEOUS AND 'JON-HOMOGENEOUS SYSTEMS A system of equations AX = B is called a
homogeneous system ifB = O. Otherwise, it is called a noji-homogeneous system of equations.
For example, the system of equations
2a: + 3y = 0
3a: - y = 0
is a homogeneous system of linear equations whereas the system of equations given by
2a:+ 3y =1
3A:-y=5
is a non-homogeneous system of linear equations.
9.24
APPLItD MATHEMATICS-Xil

9.5 SOLUTION OF SIMULTANEOUS LINEAR EQUATIONS BY ELIMINATION METHOD


9.5.1 METHOD OF ELIMINATION BY SUBSTITUTION
In this method, we express one of the variables in terms of the other variable from either of
the two equations and then this expression is put in the other equation to obtain an equation
in one variable as explained in the following algorithm.
ALGORITHM

Obtain the two equations. Let the equations be


a.vi+ljp/ + Ci =0
and, 02X+b^y + C2 = 0 ...11

Choose either of the tivo equations, say (i), and find the value of one variable, say y,in

ow
terms of the other, i.e. x.
Substitute the value of y, obtained in step ll, in the other equatio}i i.e. (ii) to get an
equation in x.
● 'TT' IV
Solve the equation obtained in step III to get the value ofx.

e
Substitute the value of x obtained in step IV in the expression for y in terms ofx obtained

re
'iJTPX I
in step II to get the value ofy.

rFl
The values ofx and y obtained in steps IV and K respectively constitute the solution of the

F
given system of two linear equations.
Following examples will illustrate tire above algorithm.

r
ou
fo
ks
ILLUSTRATIVE EXAMPLES

! \AMI’U-: 1
Solve the folloiving systems of equations by using the method of substitution:
oo

(i) 3x -5y = -1 (ii) x + 2y = ^l


Y
eB

.v-i/ = -l ,2.\--3i/=12
SOLUTION
The given system of equations is
r

3a'-5i/ =-l ●●●(i)


ou

.r-y =-l
ad
Y

...(ii)
From (ii), we get
y = -T +1
d

Substituting y = .v +1 in (i), we get


Re
in

3.V-5 (.Y + 1) =-l


F

^ -2.v-5 = -l
=> -2x = 4 => ,v = -2

Putting a: = -2 in y = -r +1 we get y = -1.


Hence, the solution of the given system of equations is .v = -2, y = -1.
The given system of equations is
,v+2y = -l -(i)
2.Y-3y =12 ...(ii)
From equation (i), we get
.v=-l-2y
Substituting a: = -1 - 2y in equation (ii) we get
2(-l-2y)-3y=12
=> -2-4y-3y=12
9.25
INVERSE AND APPLICATIONS OF A MATRICES

=> -7i/=14

Putting y = -2 in .V = -1 - 2y, we get


,y=-1-2x(-2) = 3
Hence, the solution of the given system of equations is y = 3,y = -2.
EXAMPI1 : Solve f/ic/o//owhi? systems of equations hy using the method of substitution:
(i) 2Y+3y = 9 (ii) +y=2
a b

3y + 4i/=5 ^-l=A
a b

ow
SOLUTION The given system of equations is
2y + 3y = 9
3y + 4y = 5 -..(ii)

From equation (i), we get

e
3y = 9-2Y^y = ^^

re
rFl
F
9 _2v
Substituting y = —^ in equation (ii), we get
3

r
9-2y
3y + 4 =5
ou
3 fo
ks
9.V+36-8Y ^
=0
=>
3
oo

y+36=15
Y

Y = -21
eB

9-2y
Putting Y = -21 in y = 3
, we get
^
r

9 + 42
= 17
ou
ad
Y

Hence, the solution of the given system of equations is y = -21, y = 17


(ii) The given system of equation is
d
Re
in

a b

1-1=4
F

a b

From equation (i), we get


y
2- = 2
^ 2y ^y=b, r.2 2y^
b a a
. \
2x
Substituting y=b' 2 —^
a j
= 4 in equation (ii), we get

= 4
a 11 a

^-23 = 4
a a

3y ,
— = b
a

3y = 6a
X = 2a
9.26
APPLIED MATHEMATICS-XII

Putting .t = 2a in equation (i), we get


-2b
b b

Hence, the solution of the given system of equations is .t = 2/7, y = -2b


9.5.2 METHOD OF EUMINATiON BY EQUATING THE COEFFICIENTS
In hiis method, we eliminate one of the two variables to obtain an equation in one variable
which can easily be solved. Putting the value of this variable in any one of the given equations,
the value of the other variable can be obtained.
Following algorithm explains the procedure.
ALGORITHM

STEP I
Obtain the tivo equations.
STEP II
Midtipli/ the equations so as to make the coefficients of the variable to be eliminated equal.
-.PEP HI
Add or subtract the equations obtained in step I! according as the terms having the same
coefifcients are of opposite or of the same sign.

w
SI'EIMV
Solve equation in one variable obtained in step 111.

F lo
Substitute the value found in step IV in am/ one of the given equations andfind the value oftl le
STEPV

other variable.

The values of the variables in steps IV and V constitute the solution of the given system of

e
equations.

Fre
Following examples will illustrate the above algorithm. for
ILLUSTRATIVE EXAMPLES
r
Type / SOLVING A SYSTEM OF EQUATIONS WHICH IS REDUCIBLE TO A SYSTEM OF
You
oks

SIMULTANEOUS LINEAR EQUATIONS


eBo

EXAMPLE
Solve the fotlozving system of equations:
1 1
1
2x y
ad
our

1 1
- + — = 8, zvhere x^Q.xi
X 2y
1
SOLUTION Taking — = u and = v, the given equations become
Re
dY

X
y
u
--v=-\ => u-2v = -2
Fin

2 ...(i)
v
and. 1/ + — = 8 2w +1; = 16
2 ...(ii)

Let us eliminate u from equations (i) and (ii). Multiplying equation (i) by 2, we get
2u-4v = -4
...(iii)
2u + v =16
...(iv)
Subtracting (iv) from (iii), we get
-5v = -20 => V = 4

Putting y = 4 in equation (i), we get


u-8 =-2=> u = 6
1 1 1 1
Hence, .t = - - and y = - = —
u 6 V 4
1 1
So, the solution of the given system of equation is x =
9.27
INVERSE AND APPLICATIONS OF A MATRICES

2 2 1
EXAMPI.I ' Solve:- + —
X 3i/ 6
3 2
- + -=0
^ .V

and hence find 'n'for which y = ax-A.


SOLUTION Taking -=u and - = v, the given equations become
X y
.2it 2 1
+ —v=-
3 6
12(( + 4i;=l
and, 3u+2v=0
Multiplying equation (ii) by 2 and subtracting from equation (i), we get
1

w
6u = 1 => u =
6

F lo
Putting Ii = -
6
in (i), we get
1
2 + 4i?=l=>i' = —

ee
4

Fr
1
Hence, .r = i = 6 and y =
u V
= -4

So, the solution of the given system of equations is x = 6, y = -4


for
Putting A- = 6, y = -4 in y = ax - 4, we get
ur
-4 = 6a-4=> a = 0
1 12 1
s

EXAMPLES Solve:
ook
Yo

2(2.v+3y) 7(3x-2y) 2
4
eB

7
+ = 2,
2a + 3y 3a - 2y
where 2x+3y *0 and 3x -2y ^0.
our
ad

1 1
SOLUTION Let = i;and - = v. Then, the given system of equations becomes
2a + 3y 3x-2y
Y

-H + —v= — => 7n + 24v =7 -(i)


Re

2 7 2
nd

and. 7u + 4v=2 ...(h)


Fi

Subtracting equation (ii) from equation (i), we get


20v=5=>v=-
4

Putting u = i in equation (i), we get


1
7u+6 =7 => » = -
7

i=> 2a+ 3y =7
1 1
Now, j/= —=>
...(hi)
7 2.v+3y 7
1 1 1
and. = -=> 3A-2y =4 ...(iv)
4 3a-2y 4
Multiplying equation (hi) by 2 and equation (iv) by 3, we get
...(V)
4a + 6y = 14
...(Vi)
9A-6y=12
9.28
APPLIED MATHEMATICS-XII

Adding equations (v) and (vi), we get


13.V = 26=^ X = 2

Putting ,Y = 2 in equation (v), we get


8 + 6i/=14=>i/=l
Hence, y = 2, y = 1 is the solution of the given system of equations.
. .1:'
Solve: 3 {2u + v) = 7uv
3 (i/+ 3v) = lluv
SOLUTION Clearly, the given equations are not linear equation in the variables, u and v but can
be reduced to linear equations by an appropriate substitution.
If we put 11 = 0 in either of the two equations, we getu = 0
So, H = 0, n = 0 form a solution of the given system of equations.
To find the other solutions, we assume that a 0, y

w
0.
Now, u^0,v=aO^uv¥=0.

On dividing each one of the given equations by uv, we get

V u

F lo
ee
'4=11

Fr
V u -(ii)

Taking -u = x and V- = y, the above equations become for


ur
3.V + 6y = 7 ...(iii)
9,Y+3y =11 ...(iv)
s
ok

Multiplying equation (iv) by 2, the above system of equations becomes


Yo

3.y + 6y = 7
o

●●●(V)
eB

18.r + 6y = 22 ...(vi)
Subtracting equation (vi) from equation (v), we get
-15.y = -15=> .v=l
r
ou
ad

Putting A* = 1 in equation (iii), we get


3 + 6i/=7=>y*^6
=i = -3
Y
Re

A=l^l = 1=>!( = 1
nd

Now,
u
Fi

2 1 2 3
and. ]/= — =>
3 V ' 3 2

Hence, the given system of equations has two solutions given by


(i) i/ = 0,u = 0 (ii) ii=l,v = 3/2
I'l/th'II EQUATIONS OF THE FORM,o AND''. J WHERE:?

To solve the above type of equations, following algorithm may be used.


Following algorithm explains the procedure.
ALGORITHM

STEl' I
Obtain the two equations.
Let the equation be ax + bi/ = c and bx + ai/ = d
^ II
Adding and subtracting the two equations, we obtain
c +d
(a + b) X + (a +b) y = c + d => X + ij = a +b
...(i)
9.29
INVERSE AND APPLICATIONS OF A MATRICES

c-d
A:-(r7-/j) y=c-d=> x-i/ = a-b

STIV IH Add and subtract equations (i) and (ii) to get the values of x and y.
LXAMI’l I. ~ Solve: 217j: + 131}/ =913
131:t +2171/= 827
SOLUTION We have,
217.v + 131i/ = 913 ...(i)
...(ii)
131.v + 217y = 827
Adding equations (i) and (ii), we get
...(iii)
348-v + 348y = 1740 => .r + y = 5
Subtracting equation (ii) from equation (i), we get
...(iv)
86.Y - 86y = 86 => .v - y = 1
Adding equation (iii) and (iv), we get
2x = 6 => Y = 3

Putting X = 3 in equation (iii), we get y = 2.

SOLUTION We have,
37x + 41y=70
41x+37y = 86
F low
Hence, x = 3 and y = 2 is the solution of the given system of equations.
rXAMm So/w;37x + 41y = 70
for Fre
...(i)

41x+ 37y = 86 ...(ii)

Adding equation (i) and (ii), we get


78x + 78y=156^ x + y = 2 ...(iii)
eBo ks
Your

Subtracting equation (i) from equation (ii) we get


...(iv)
4x -4y = 16 => X -y = 4
ad

Adding equation (iii) and (iv), we get


our

2x = 6 => X = 3

Putting X = 3 in equation (iii), we get y = -1.


Re

Hence, x = 3 and y = -1 is the solution of the given system of equations.


h/pelll EQUATIONS OF THE FORM
Find Y

^7|.x + b-^y + C| =
172-X + 1^2J/ ^2 ~ ^2
rtj.v + b^y + C3 — d^
To solve the above type of equations, following algorithm may be used.
ALGORITHM

MU' 1 Take any one of the three equations.


''UA’JI Obtain the value of one of the variable, say zfrom it.
VII I’MI Substitute the value ofz obtained in Step II in the remaining two equations to obtain two linear
equations in x, y.
STI r i\ Solve the equations in x, y obtained in Step III by elimi}iation method.
VI I ]■ ■Substitute the values ofx, y obtained in Step IV and step II to get the value of z.
Following examples illustrate the above procedure.
9.30
APPLIED MATHEMATICS-XII

b\AMl’LE9 Solve: 2x-i/ = 4:


ij~z = 6
a:-2=10
SOLUTION We have.
2a--i/ = 4
i/-z = 6
...(ii)
A'-2 =10
...(iii)
From equation (iii), we getz = a-10
Substituting the value of z in equation (ii), we get
i/-(.v-10)=6 ...(iv)
=> -A+l/ = -4
Adding equations (i) and (iv), we get

w
A =0

Putting .V = 0 equation in (i) and (iii) we get

F lo
y = -4 andz =-10
Hence, x = 0, y = —4, z = —10 is the solution of the given system of equations.

ee
EXAMPLE K) Solve: x + 2if+z=7

Fr
x+3z = ll

2.r-3y=l for
SOLUTION We have,
ur
.v + 2y+z=7 ●●●(i)
A+3z=n
...(ii)
s
ook

2A-3y=l
Yo

...(iii)
From equation (i), we get
eB

2=7 ~x-2\j
Substitutingz =7 - A-2y in equation (ii), we get
A+ 3(7-A-2y) =11
our

...(iv)
ad

=>
A + 21 - 3A-6y =11
-2.v-6y=-10
Y

Adding equation (iii) and (iv), we get


Re

9y = -9 => y = 1
nd

Putting y = 1 in equation (Hi) we set x = 2


Fi

Putting .r = 2, y = 1 in equation (i), we get


2 + 2+2=7=>z = 3
Hence, x = 2, y = l,z = 3

EXERCISE 9.3
1. i-+j^=3 2. -+3y=14
7x 6y x

1
i-=5 --4y = 23
2,t 3y X

3. ^=2 4.
2 3
- + —= —
9

A y Ay
^ =6 - + 2. = ?1
■\v X y Ay
9.31
INVERSE AND APPLICATIONS OF A MATRICES

22 15
xy _ 6 6. =5
x +y 5 x + \j x-y
55 45
= 6 = 14
y-x x+y x-y

7. x + y = 2xy 8. l{'iu-v)=5uv

^=6 2{u+ 3v) =5uv


xy

9.
2 3 _17 10. 152.v-378i/=-74
3x+2y 3x-2y 5

ow
5 1
-378-r + 152y=-604
= 2
3.r + 2y 3x-2y
11. 99:t + lOly = 499 12. 23.r-29y=98

e
101.v + 99i/=501 29.\--23y=110

re
13. .v-i/ + 2=4 14. .v-y+z = 4

Frl Y+y+z=2

F
.Y-2_v-22 = 9
2.V + y + 3z = 1 2.Y + y-3z = 0
ou
or
ANSWERS

1 1 1
1 1
kfs
3. .v = -
1. Y = — / V -- 2. x = -,y = -l
14 ^ 6 5'
oo

4. Y = l,y=3 5. Y =2, y = 3 6. Y = 8, y = 3
Y

-1 1
8. u = 2,v=\ 9. Y=l,y=l
B

7. Y =
2'^ = ,-
re

10. Y = 2,y=l 11. Y = 3,y=2 12. y = 3, y = -l


14. Y = 2,y=-l,z=l
oYu

13. Y = 3, y = -2,z = -l
ad

9.6 MATRIX METHOD FOR THE SOLUTION OF A NON-HOMOGENEOUS SYSTEM


Consider the following system of m linear equations in n unknowns:
d

^'i ^12 ^2 ●●●


in
Re

^21 '^1 ^22 '^'2 ^2>i


F

^h1^1 + ^h2^2+ ●●●+ m

This system of equations can be written in matrix form as


«!! a-12 ●●● (^1)1
^21 (^22 ■■■ ^2n ^2 h

^Hl ^n2 ■■■ a


mn
Yn bn

rtll n-12 '’in ^1 ^1


^21 ^22 ■■■ ^2n ^2 and B =
h
or AX = B, where A = ,X =
h
.’’ml %i2 ■■■ -1 m xn - /I X 1 " J fl X 1

The m x n matrix A is called the coefficient matrix of the system of linear equations.
9.32
APPLIED MATHEMATiCS-XII

ILI-USTKATION Express the following si/stem of simultaneous linear equation as a matrix equation:
2x+ 3y-z = l
AT + y + 2z = 2
2.V - y + 2 = 3
SOLUTION We have,
2x + 3y - 2 = 1
X + 1/ + 2z = 2
2x -}i +z= 3
This system of equations can be written in matrix form as

2 3 -1 X 1
1 1 2 y
9

2 -1 1 2 3

w
2 3-1 .Y 1
or, AX = B, where A = 1 1 2 , X= y and B = 2

F lo
2 -1 1 2 3

In this section, we shall discuss about a method for solving a system of non-homogenous
simultaneous linear equations in which the number of unknowns is same as the nmnber of

ee
equations. In this method, we will use the inverse of the coefficient matrix. So, it is also known as

Fr
matrix method.
for
THEOREM 1 If A is a non-singular matrix, then the system ofequations given by AX = Bhas the unique
solution given by X=A~ ^B.
ur
ryooi We have, AX = B, where | /41 0.
s

-1
ook

Now, 1^1 0. So, A exists.


Yo

-1
Pre-multiplying both sides ofAX = B by A
eB

, we get
A^\aX) = A-'^B
(A-'^A)X = A-^B
our
ad

IX = A~'^B
X = A~'^B
dY
Re

Thus, the system of equations/IX = B has a solution given by X = A~^B.


Fin

Uniqueness: If possible, let Xj and X2 be two solutions of/IX = B. Then,


AX-i = B and AX2 = B
AXi = /IX2
A~\aX^) = A-^{AX2)
=>
(A-'^A)X^ = {A-'^ A)X2
/Xj = /X2
Xj = X2.
Hence, the given system of equations has the unique solution given by X = A~^B.
Q.E.D.
In the above theorem, we have proved that a non-homogenous system AX = B ofn simultaneous
linear equations with ?z-unknowns has the unique solution -1
given hy X= A B, if /I is a
non-singular matrix. Now, a natural question arises, what happens when A is a singular matrix?
In order to answer this, let us consider the following system of equations:
9.33
INVERSE AND APPLICATIONS OF A MATRICES

2x + 1/ = 3
4.V + 2y = 6
This system of equations can be written in matrix form as
r2 11 [x 3 2 1 .V
and B-
f3'.
, or, AX = B, where A = ,X^
4 2 6 4 2 1/ o
y

Clearly, \A\ = 0. Also, the system of equations has infinitely many solutions as the two
equations represent coincident lines in .vi/-plane.
Now, consider the following system of equations:
2.V + 1/ = 3
4.V + 2y = 5
For this system of equations also the determinant of the coefficient matrix A is zero i.e. A is a
singular matrix. But, the system has no solution i.e. it is an inconsistent system of equations, as
the lines represented by the two equations are non-coincident parallel lines.
It follows from the above discussion that the system of equations AX = B may be inconsistent

w
or it may be consistent with infinitely many solutions when the coefficient matrix A is singular.

non-homogcnous system of linear equations.


ALGORITHM

STEP 1
STEPil
STEP HI
Find\A\.
F lo
We now state and prove the following criterion for the consistency or inconsistency of

Obtain the system of equations and express it in the matrix equation from AX = B.
for F
ree
a

!f\A\^ 0,then the given system of equations is consistent xvith uniquesolution. To obtain the
solution compute A“ ^ by using A~ ^ ^
A I
ndj A and use the formula X = A~^ B.
Your

STEP IV (f I AI = 0, then the given system of equations is either inconsistent or it has infinitely many
ks
eBoo

solutions. To distinguish these two proceed as follows:


Compute {adj A) B.
[f{adj A) B^O, then the given system ofequatiojis is inconsistent i.e. if has no solution.
ad
our

If {adj A) B = 0, then the given system of equations is consistent with infinitely many
solutions.

In order to find these infinitely many solutions, replace one of the variables by some real
Re

number. This will reduce the number of variables by one. Now, take any two out of the three
Y

equations and solve them by matrix method.


Find

Following examples will illustrate the above algorithm.


ILLUSTRATIVE EXAMPLES

Tl/ve I SOLVING THE GIVEN SYSTEM OF LINEAR EQUATIONS WHEN THE COEFFICIENT MATRIX 13
NON-SINGULAR

EXAMPLE 1 Use matrix method to solve the following system of equations:


5x-7y = 2,7x-5y = 3
SOLUTION The given system of equations can be written as
5.Y-7y = 2
7y-5i/ = 3
5 -7 X 2
or.
7 -5 1/ 3
9.34
APPLIED MATHEMATICS-XII

■5 -7 ■t r 2
or, AX = B, where A = , X = and B = „
7 -5 y 3
5 -7
Now, (A 7 -5
= - 25 + 49 = 24 0

So, the given system has a unique solution given by X = A~^ B.


Let Cjj be the cofactors of elements <7,y in A . Then,
Cii=(-l)^-^^-5)=-5, Ci2=(-1) 1+ 2
7=-7, C2i=(-1)
2+ 1
(-7)=7
2+2
and C22=(-l) 5=5.

ow
-5 -7 -iT -5 7
adj = 7 5 -7 5

So, /I
-1 1 „ 1 r -5 7

e
5

re
X=A-'B=±\-l 1 10 + 21 ■ '11/24 ■
24 -7 5

Frl 3 14 + 15 1/24

F
24

AT '11/24'
=>
y. 1/24
ou
r
11 1
.r
= —
24
and y =
so
24
kf
Hence, .v =11/24 and y =1/24 is the required solution.
oo

EXAMPLUZ
Use matrix method to solve the foUoiving system of equations:
Y

-V - 2y - 4 =0, - 3x + 5y + 7 =0
eB

SOLUTION The given system of equations can be written as


.v-2y = 4
ur

- 3-Y + 5y = - 7
oY

^1-2] [x^ 4
ad

or.
-3 5 -7
y.
'1-2
d

4
or. AX = B, where v4 = , X = and B -
-3 5 y -7
in
Re

1 -2
Now, IA 3 5
= 5-6 = -1 ^ 0
F

So, the given system has a unique solution given by X = A~^ B.


Let Cjj be the co-factors of elements in A =1 [ojj]. Then,
Cii = (-1) ' '5 = 5,Ci2 = (-1)
1+ 1 + 2+ 1
(-3) = 3,C2i=(-1) (-2) =2
2+ 2
and ^22 - (“^) 1 = 1.

5 3 iT 5 2
adj A = 2 1 3 1

-] 1 '5 2 ' -5 -2
So, A adj A = —
(-1) [31 -3 -1

X = A"^ B
X = r -5 -2 ]f 4 r-20 +14 1 -6
-3 -1 -7 12 + 7 -5
9.35
INVERSE AND APPLICATIONS OF A MATRICES

.V -6

y -5

.V = - 6 and y = -5
Hence, .v = - 6 and y = - 5 is the required solution.
EXAMPLE 3 Solve the follozuing system of equations, using matrix method:
x + 2y+z = 7, .V + 3z =11, 2.y - 3y = 1
SOLUTION The given system of equations is
.Y + 2y + z = 7
.Y + Oy + 3z = 11
2y - 3y + Oz = 1
1 2 1 ,Y 7

w
or. 1 0 3 y 11
2-30 2 1

Flo
1 2 1 .Y 7

1 0 3 r X = and B = 11
or, AX = B, where A = y
2 -3 0 1

ee
2

Fr
1 2 1 I
Now, |A1= 1 0 3|=l(0 + 9)-2(0-6)+l{-3-0)=9 + 12-3=18^i0
2 -3 0| for
ur
1
So, the given system of equations has a unique solution given by X = A
B.
s
Let Cq be the co-factors of elements Ojj in Ttien,
k
Yo

1 3
oo

1+ 1 0 3 1+2
= 9, Cl2 = (-1) = 6,
Cn = (-1) -3 0 2 0|
eB

1 0 2+ 1 2 1
1+ 3 = -3,
Ci3 = (-1) 2 -3
= -3, C21 = (-1) -3 0
r
ou
ad

2+2 1 1 2+ 3 1 2| = 7,
C22 = (-1) 2 0
= -2, C23 = (-1) 2 -3|
Y

1 1
3+1 2 1| =_ 6, C32 = (-1)
3+2
= -2,
C31 = (-1) 1 3
Re
nd

0 3|
3+ 3 1 2
Fi

= -2
and. C33 = (-1) 1 0

9 6-3 9-3 6
-3 -2 7 6 -2 -2
adj A =
6 -2 -2 3 7 -2

9-3 6
-1 1 1
A adj A = — 6 -2 -2
18
-3 7-2

Now, X = A"^ B
7 63 33+6 36 2
9-3 6 1
1
11 42 -22 -2 18 1
6 -2 -2
18 18 18
-21 +77 -2 54 3
-3 7-2 1
9.36
APPLIED MATHEMATICS-XII

.V 2
1
y => x = 2, y =1 and 2 = 3
2 3

Hence, a: - 2, y = 1 and z = 3 is the required solution.


Type H SOLVING A SYSTEM OF LINEAR EQUATIONS WHEN TH ~ INVEHS' : OF THE COEFFICIENT
MATRIX IS OBTAINED FROM SOME GIVEN RELATION
1-1 1 1
EXAMPLE 4 If A = 2
1
1
1
31 ,find A ^ and hence solve the system of linear equations
.Y + 2y + z = 4, - .V + y + 2 = 0, -Y - 3y + 2 = 2.
'1 -1 1
SOLUTION We have, A ^ 2 1-3
1 1 1

1 -1 1
1^1 = 2 1 -3

F low
= l(l + 3) + l(2+3) + l(2-l) = IOt^O
1 1 1

So, A is invertible.

Let Cij be the co-factors of elements a a in A [«,,]. Then,


-3
Cii - (-1) = 4, Ci2 = (-1) 1+2 12 - 3| _ -5,
|1 1

re
for F
Ci3 = (-1) 1+32 1| = h C21 - (-1)
2+ 1 1 1| =_ 2,
1 1| 1 1|
^^22 = (“1) 2+2 1 _ 2+3 1 -1
0, C23 - (-1) = -2
1
Your

1
ks

-1 1 1
eBoo

3+ 1 3+2 1
C31 = (-1) 1 -3
= 2, ^32 = (~1) = 5,
2 -3
3+ 3 1 -1
and. C33 = (-1) 2 1 ' ^
ad
our

4 -5 1
-|T
4 2 2
adj/l =2 0-2 -5 0 5
Re

2 5 3 1 -2 3
Y

.. 4 2 2
1
Find

A-1 = adj A = — -5 0 5 ..(i)


Ml 10
1 -2 3_,
Now, the given system of equations is expressible as

1 2 1 .Y 4

-I ll y 0
1-3 1 2 2

X 4
T
or. A X = B, where X = y and B = 0
2 9

Now, I I -1 A I = 10 0. So, the given system of equations is consistent with a unique solution
given by
X = B = (/4~y B [●●●
9.37
INVERSE AND APPLICATIONS OF A MATRICES

Y 4 2 2 1' 4
1
V -5 0 5 0 [Using (/)]
2
10
1 -2 3 J [2
Y
1
4-5 1 ir 4 ' 1
16 + 0+2 9/5
1/ 2 0-2 0 = 8 + 0-4 2/5
10 10
2 2 5 3 2 8 + 0 + 6 7/5

=> .Y = 9/5, 1/ = 2/5 and 2=7/5


Hence, y = 9/5,1/ = 2/5,2=7/5 is the required solution.
-4 4 4 1 -1 1

EXAMPLES Determine the product -1 1 3 1 - 2 - 2 i/s^ /f to solve the system


5-3-1 2 3

w
oj equations:
Y - 1/ + 2 = 4, Y - 2_v - 22 =9, 2y + !/ + 32 = 1.

Flo
'1-1 1 -4 4 4 l
SOLUTION Let A = 1 -2 - 2 andC = -7 1 3 . Then the given product is

ee
2 ] 3 5 -3 -1

Fr
-4 4 4 1 1 -1 1
CA = -7 1 3 1 -2-2
5 -3 -1 2 1 3
for
ur
-4+4+8 4-8 + 4 -4-8 + 12 8 0 0 1 0 0

=> CA -7+1+6 7-2+ 3 7-2 + 9 0 8 0 0 10 =8/0


s
5-3-2 -5+6-1 5 + 6-3 0 0 8 0 0 1
k
Yo
oo

1
^CA = /3
eB

8
n
=> A = Z3
^8
r
ou
ad

1
A
-1
= -C [By definition of inverse]
8
Y

-4 4 4
1
A
-1
-7 1 3 ●●●(0
Re
nd

5-3-1
Fi

The given system of equations can be written in matrix form as


r 1 -1 1 If Y 4

1-2-2 \j 9

2 1 3 z 1

1 -1 1 Y 4

1 -2 -2 , X = and B = 9
or. A X = B, where A = }/
2 1 3 2 1

The solution of this system of equations is given by


X = A"^ B
-4 4 4 4
1
X = - -7 1 3 9 [Using (i)l
8
5 -3 -1 1
9.38
APPLIED MATHEMATICS-XII

-16+36 + 4 24 3
1
y 28 + 9+3 -16 -2
8 8
2 20-27-1 -8 -1

X = >?, y = -1 and z = -1
1 2-3
I.XAMI’II f, Find A~'^, ivhere A = 2 3
2 . Hence solve the system of equations
_ 3 -3 -4
-Y + 2y - 3z = - 4, 2y + 3y + 2z = 2, 3y - 3i/ - 4z = 11.
SOLUTION We have,
“1 2 -3“

ow
A = 1 3 2 .
3 -3 -4

I 1 2-3
\A\ =\2 3 2 = -6 + 28 + 45 = 67 0

e
3 - 3 -4

re
So, A is invertible.

rFl
F
Let Cij be the co-factors of aq in A = [ajj]. Then,
Cll = (-1)
1+ 1 3 2| _ -6,
1+2 2 2
Ci2 = (-1)

r
= 14,
-3 -4j -
ou
3 -4

2 3 fo 2 -3| =_
ks
1+ 3 2+ 1
^13 - (-1) 3 -3
= -15, C21 - (-1) 17,
-3 -4|
oo

1 -3|
●^23 - (“^) 2+ 3 11
2+ 2 2
^22 - (“1) = 5, = 9
3 -4| )3 -3
Y
B

3+ 1 2 -3 3+ 2 1 -3
C31 = (-1) = 13, ^32 = (~1)
re

3 2 2 2

3+3 1 2
ou

and, C33 = (-1) = -1


Y
ad

2 3

hT
-6 14 -15 -6 17 13
d

adj A = 17 5 9 14 5-8
in
Re

13 -8 -1 -15 9 -1
F

-6 17 13
1
So, A-^ ad] A = — 14 5
Ml 67
-15 9 -1

The given system of equations is


Y + 2y -3z = - 4
2y + 3y + 2z = - 2
3y - 3y - 4z - 11
1 1-3 Y -4
or. AX = B, where A = 2 3 2 , X = y and B - 2
3 -3 -4 z 11

As discussed above A is non-singular and so invertible. The inverse of A is given by (i).


The solution of the given system of equations is given by
X ^ A-^ B
9.39
INVERSE AND APPLICATIONS OF A MATRICES

X -6 17 13 -4 24 + 34 +143 3
1 1
14 5-8 2 -56 +10 -2
y 67
67 11 60 +18 -11 1
z -15 9 -1

x = 3, y=-2 and z = 1 is the required solution.


Type III ON APPLICATIONS OF SIMULTANEOUS LINEAR EQUATIONS
EXAMPii " The sum of three numbers is 6. If we imiltiply the third number by 2 and add the first
number to the result, we get 7. By adding second and third numbers to three times the first number, we get
12. Using matrices find the numbers.
SOLUTION Let the three numbers be x, y and z respectively. Then,
X + 1/ +z = 6 [Given!
Also, X + 2z =7

ow
and. 3x + y+ z =12
Thus, we obtain the following system of simultaneous linear equations:
X +y+z = 6
X + Oy + 2z = 7

e
3x + y + z = 12

re
The above system of equations can be written in matrix form as
1 1 1 6

rFl
F
X

1 0 2 y 7

3 1 1 z 12

r
ou
AX = B, where A =
1

1
1

0
1

2,X = y
X

and
fo
B -
6
7
ks
or.
3 1 1 z 12
oo

|1 1 1]
Y

Now, 1A1=|1 0 2 = l(0-2)-(l-6) + l(l-0)= -2 + 5 + 1 = 4;^0


eB

|3 1 l|
So, the above system of equations has a unique solution given by X = A ^ B.
r
ou

Let Cjj be the cofactor of Oj; in A = [%]. Then,


ad
Y

Cii = -2,Ci2 = 5, Cj3 = 1,C21 = O/C22 = -2, C23 = 2,


c 31 = 2,C32 = -1 and C33 = -1
d

2 5 1 -2 0 2
Re
in

adj A = 0-2 2 5 -2 -1
2 -1 -1 1 2 -1
F

-2 0 2
A
-1 1
— adi
^
A = —
1 5 -2 -1
[A| ’ 4 1 2-1

Now, X = A"”^ B
-2 0 2 6 -12 +0 +24 3
1 1
X= - 5 -2 -1 7 30 -14 -12 1
4 4 2
1 2 -1 12 6 +14 -12 L J

X 3

y 1
2 2

X = 3, y = 1 and z = 2.
Hence, the three numbers are 3,1 and 2 respectively.
9.40
APPLIED MATHEMATICS-XII

EXAMPLE
8 An amount oft 5000 is put into three investments at the rate of interest of6%, 7% and 8%
per annum respectively. The total annual income is t 358. If the combined income from the first two
investments is ? 70 more than the income from the third, find the amount of each investment by matrix
method.

SOLUTION Let X, y andz ? be the investments at the rates of interest of 6%, 7% and 8% per
annum respectively. Then,
Total investment = ? 5000
x + y+z = 5000.
6x
Now, Income from first investment of x =?
100

7y
Income from second investment of ? y = ? 100

w
Income from third investment of ,
100

Total annual income = ?

F lo
Uoo 100 "^100,
6x 7y 8z = 358
100 Too Too [’.● Total annual income = ^ 358]

ee
Fr
6x + 7y + 8z = 35800.
It is given that the combined income from the first two investments is ? 70 more than the for
income from the third.
ur
6x 7y = 70 +
8z
100 ioo 100
=> 6x+7y-8z=7000.
s
ook
Yo

Thus, we obtain the following system of simultaneous linear equations:


x + y+z = 5000
eB

6x + 7y + 8z = 35800
6x + 7y-8z = 7000
r
ad

This system of equations can be written in matrbc form


ou

as follows:
11 1 X 5000
6 7 8 35800
Y

y
6 7-8 z 7000
Re
nd

1 1 1 X 5000
Fi

or. AX = B, where A = 6 7 8 ,X = y andB = 35800


6 7-8 z 7000
1 1 1

Now, |A| = |6 7 8 =l(-56 -56)-(-48 - 48) + (42 - 42)=-16 0.


6 7-8

So, A” ^ exists and the solution of the given system of equations is given by X = A
-1
B.

Let Cjj be the cofactor of Ojj in A^ = ^]. Then,


Cii = -112,Ci2 = 96,Ci3 = 0,C2i = 15, C22 = -14,
^23 = “1/^31 = 1^^32 = -2 and C33 = 1.
■-112 96 0l^ -112 15 1
adjA = 15 -14 -1 96 -14 -2
1 -2 1 0 -1 1
9.41
INVERSE AND APPLICATIONS OF A MATRICES
-112 15 1
1 1
-1 96 -14 -2
So, A (adj A) = - 16
1^1 0 -1 1

Hence, the solution is given by ■-560000 +537000 +7000


-112 15 1 5000
1 1
-1
96 -14 -2 35800 480000 -501200 -14000
X=A B=-
16 16 0 -35800 +7000
0 -1 1 7000

X 1000

y = 2200
z
|_1800
= 1000,1/ = 2200 and z - 1800

ow
X

Hence three investments are of ? 1000, ? 2200 and ? 1800 respectively.


EXAMPLE 9 A mixture is to be made of three foods A,yB, C. The three foods A, S, C contain nutrients
P,Q, R as shown below:
Ounces per pound of hhitrient

e
Food }
R

re
A

Fl p
1
Q
2 5

F
3 1 1
B
4 2 1
C
ur
Hozo to form a mixture which will have 8 ounces of P, 5 ounces of Q and 7 ounces ofR?

r
SOLUTION Let X pounds of food A, y pounds of food B and z pounds of food C fo be needed to
ks
form the mixture.
Since one pound of food A contains 1 ounce of nutrient P. So, x pounds of fo^od A will contain
X
Yo

ounces of nutrient P. Similarly, the amount of nutrient P my pounds of food B and z pounds of
oo

food C are 3y and 4z ounces respectively. Therefore,


Total quantity of nutrient P in x pounds of food A, y pounds of food B and z pounds of food C i:
eB

is

X + 3y + 4z ounces.
X + 3y + 4z = 8
ur

[For nutrient Q1
Similarly, 2x + y + 2z =5
ad

[For nutrient R]
5x + y + z = 7
Yo

and

The above system of simultaneous linear equations can be written in matrix form as
T 3 4
d

X
Re

2 1 2 5
in

y
5 1 1 z 7
F

'13 4 X

and B - 5
or. AX = B, where A = 2 1 2 , x y
5 1 1 z 7

1 3 4

Now, |A| - 2 1 2 = 1(1-2)-3 (2-10)+ 4(2-5)= -1+24-12 = 11 ^ 0


5 1 1

So, A" ^ exists.


Let Cjj be the cofactor of a^j in A = [a^j]. Then, = -19,
C]^ = = 8,Ci3 = -3,C21 ^ 1,C22
C23 = 14, C31 = 22,C32 = 6 and C33 = -5
-1
1 -19
-3f
14
-1
8 -19
1 2
6
adj A =
2 6 -5 -3 14 -5
9.42
APPLIED MATHEMATICS-XII

-1 1 2
-1 1
A
= ~ adj A = 8 -19 6
1^1 11
-3 14 -5

Thus, the solution of the system of equations is given by


1
"-1 1 21 rs^ -8 +5 +14 11 1
X = 7l
-1 1 1
B= — 8 -19 6 5
11
64 - 95 + 42 11 1
11 11
L-3 14 -5
L7J -24 +70 -35 11 1
X 1

y 1
2 1

ow
X = 1, y = 1 and 2 = 1.
Hence, the mixture is formed by mixing one pound of each of the foods A, B and C.

l/j, nnd V3. The capnaty of each truck in terms of 3 types of vehicles is given as follows:
“//n-cfaT], a„dT, to tramport 3 types of vehicles

e
re
1 ^3
7i 1 3 2

Frl
F
T, 2 2 3
7-3 3 2 2

of trucks of each type required to transport 85, 105


ou
and 110

r
veincles of v^, +2 and types respectively.
SOLUTION Siypose .r trucks of type Tj, y trucks of type T2
so
and 2 trucks of type T. are needed to
types Fespectively. Then ^
kf
transport 85,105 and 110 vehicles of V'j, V, and ^3
.r + 2y + 32 = 85
oo

3.t + 2y+22=105
Y

2x+3y +22=110
eB

Let us solve the above system of equations by matrix method. The given system can be written
in matrix form as c y
“12 3
ur

.V 85
oY

3 2 2 y 105
ad

2 3 2 7
110
d

1 2 3 .Y 85
in

or. Ax = B, where 2l = 3 2 2 , -V = y and B = 105


Re

2 3 2 2 110
F

Let Cjj denote the cofactor of a^j in ,4 = [Ojj]. Then,


C 11 - -|2 2| =_ -2, C - |3 2 . 3 2
3 2
Ci2--|2 2 =-2' C 13 -
2 3|“^
2 3 1 3 1 2
C 21 -- = 5, C 22 -
3 2 2 = -4,
2" C23 =- = 1
2 3
2 3 1 3
C 31 = 1 2
2 2
= -2, C 32-“ = 7, c 33 = = -4
3 2 3 2“

1^1 = ^11^11+^12 ^12+^13 C’ls = lx-2 + 2x-2 + 3x5=9


“-2 -2 S'f -2 5 -2
and adj A= 5 1 -2 -4 7
-2 7 -4 5 1 -4
9.43
INVERSE AND APPLICATIONS OF A MATRICES

1
ndj A
Ml
-2 5 -2
1
A-^ = -2 -4 7
9
5 1 -4

Now, AX = B
X=A~'^B
.V
-2 5 -2 85 -170 + 525 - 220 135 15
1 1 1
-2 -4 7 105 -170-420 + 770 180 20
y
2
9
5 1 -4 no
9
425 + 105-440 ^ 90 10

w
x=15, i/=20, 2=10
Hence, 15 trucks of type Tj, 20 trucks of type T2 and 10 trucks of type T3 are needed.
EXAMPLE 11 hi an engineering workshop there are 10 machines for drilling, 8 machines for turning and

Flo
7 machines for grinding. Three types of brackets are made. Type I brackets requircO minutes for drilling, 5
minutesfor turning and 4 minutes for grinding. The corresponding timesfor type ll and III brackets are 3,

e
3,2 and 3,2,2, minutes respectively. How many brackets of each type should be produced per hour so that

re
all the machines remain fully occupied during an hour? Solve by using matrix method.

F
SOLUTION The information provided can be summarized in the tabular form as follows:
Drilling Turning Grinding
ur
r
Brackets

Type / 0 5
fo 4

2
3 3
Type II
ks
3 2 2
Type in
Yo
oo

Time available 10x60 = 600 8x60 = 480 7x60=420


(in minutes)
B

Let.r, yandz denote the number of brackets produced of each type. Then, time taken by drilling
re

machine to produce x, y and 2 brackets of type I, II and III respectively, is Oat + 3i/ + 32 minutes.
But the time for which drilling machine is available is 600 minutes.
u
ad

0.V+3i/+3z = 600
Yo

Similarly, for turning and grinding machines, we obtain


5.V + 3i/ + 2z = 480 and 4x + 2y + 2z = 420.
d
Re

Thus, we obtain the following system of linear equations


in

O.r + 3y + 3z = 600
F

5.v+3}/ + 2z = 480
4r + 2i/ + 2z + 420
In matrix form the above system can be written as
'0 3 3] \x 600

5 3 2 y 480

4 2 2 2 420

0 3 3 .r 600

or, AX = B, where A = 5 3 2 ,X = y and B = 480

4 2 2 2 420

0 3 3

Now, 1A1 = 5 3 2 = 0(6-4)-3 (10-8) + 3(10-12) =-12 0


4 2 2
9.44
APPLIED MATHEMATICS-Xil

So, A is invertible.

Let C,y be cofactor oftyj in A =[(7^].Then,


Cii =2, Cj2 = -2, C|3 = -2, C21 = 0, C22 = -12, C23 = 12, C31 = -3, C32 = 15, C33 = -15
■ 2 -2 -2 r2 0 -3 ■
ndjA= 0 -12 12 = -2 -12 15
-3 15 -15
[-2 12 -15
'2 0 -3"
-1 1 1
Thus, adj A = -2 -12 15
Ml 12
-2 12 -15

ow
Now, AX^B
X = A~'^B
.Y 2 0 -3 600 -60 5
1200 +0 -1260
1 1
=>
y -2 -12 15 480 - -1200 -5760 +6300 -660 55

e
12 12 12
-2 12 -15 420 -1200 +5760 -6300

re
z

Y = 5,1/ = 55 and z = 145


rFl -1740 145

F
Hence, 5 brackets of Type-I, 55 brackets of Type-TI and 145 brackets of Type-III should be
produced to keep all machines fully occupied.

r
Given the foUoiving National Income model:
ou
EXAMPLE 12

C =a + bY (rt > 0, 0 < < 1)


fo
ks
I=d + eY (d>0,0<e<l)
y =/+c
oo

Solve the endogenous variables C, 1 and Y using matrix method.


Y
eB

SOLUTION The given system of equation can be written as


C + Ol-bY^a
0C + I-eY = d
ur

C+/-Y=0
ad
Yo

The above system can be written in matrix from as

1 0 -b C a
d

0 1 -c / d
Re
in

1 1 -1 Y 0
F

1 0 -b C a

or. AX = B, where A = 0 1 - e / , B = d
1 1 -1 y 0

1 0 -b

Now, |.A|= 0 1 - e
= 1 (-1 + e) -0 (0 +1-) -b (0 -1) = -1 + + £?
1 1 -1

U -1 + b + e ^ 0, then |/l| 9^ 0 and so A is invertible.


Let A = [Ojj] and Cjj be cofactor of Ojj in A. Then,
C|i =e-l, Cj2 =-e, Ci3 =-l, C21 =-b, C22 =~l+b, C23 =-l, C 31 -^'^32 -‘^'^33
e-l
iT
-e -1 e-l -b b

adj A = b -1+b -1 -e b-1 e


b e 1 -1 1 1
9.45
INVERSE AND APPLICATIONS OF A MATRICES

(?-l -b b
-1 1 1
Thus, A ndj A = -e b-1 e
1^1 b + e-l
-1 -11

Now AX = B
1
X=A~^B
'C' e-\ -b b a

1
/ -e b-I e d
b + e-\
y -1 -1 1 0

a-ae + bd
1 -b-e
ae-a-bd

low
C
1 d + ae-bd
/ -ae + bd-d
b + e-1 1 -b-e
Y -a-d
a +d
1-b-e

ee
a - ae + bd d + ae-bd a + d
C = ,1 =

F =

Fr
1-b-e 1-b-e 1-b-e

EXERCISE 9.2
for
ur
1. Solve the following system of equations by matrix method:
(i) 5;r + 7y + 2 = 0 (ii) 5x +2y = 3
ks

4.Y + 6i/ + 3 = 0 3.V + 2y = 5


Yo

2. Solve the following system of equations by matrix method:


oo

2 3 10 _ ,
(i) + 1 = 10 (ll) — H H — -1,
eB

X y z X y 2
1 1 1 4 6 5 ,
r

-V y 2 .V y z
ou
ad

3 1
- = 13 ^+ = y,z¥:0
Y

X y z

1 -1 0 2 2-4
Re
nd

3. IIA = 2 3 4 and B = -4 2-4 are two square matrices, find/4B and hence
0 1 2 2-1 5
Fi

solve the system of linear equations:


.Y - 1/ = 3, 2y + 3i/ + 4z =17,1/ + 2z =7
■'2-3 5' '
4. IfA= 3 2-4 , find A~^ and hence solve the system of linear equations:
1 1 -2

2Y-3y+52=n, 3y + 2i/-4z=-5, y + i/-22 = -3


2 2-4 '1 -1 O’

5. (i) Given A = -4 2 -4 , B = 2 3 4, find BA and use this to solve the system of


2-1 5 0 1 2

equations i/ + 2z =7, Y-y = 3, 2Y+3y + 4z=17


9.46
APPLIED MATHEMATICS-XII

2 3 1
(ii) lfA = 1 2
2 , find A ^ and hence solve the system of equations
-3 1 -1

2.V + y — 3z = 13, 3.T + 2y+z = 4, .v + 2y -2 = 8.


'1 -1 2l [-2 0 1
(iii) Use the product 0 2 -3 9 2 -3 to solve the system of equations
3 -2 4 6 1 -2

.r + 32 =-9,-.r + 21/- 22 = 4,2.V - 31/+ 42 = - 3.


6. The sum of three numbers is 2. If twice the second number is added to the sum of first and
third, the sum is 1. By adding second and third number to five times the first number, we
get 6. Find the three numbers by using matrices.
7. An amount of ? 10,000 is put into three investments at the rate of 10,12 and 15% per annum.

low
The combined income is ^ 1310 and the combined income of first and second investment IS
i:
? 190 short of the income from the third. Find the investment in each using matrix method.
8. A company produces three products every day. Their production on a certain day is 45
tons. It is found that the production of tliird product exceeds the production of first product
by 8 tons while the total production of first and third product is twice the production of

ee
second product. Determine the production level of each product using matrix method.
F
9. The prices of three commodities P, Q and R are ? a:, y and z per unit respectively. A

Fr
purchases 4 units of R and sells 3 units of P and 5 units of Q. B purchases 3 units of Q and
sells 2 units of P and 1 unit of R.C purchases 1 unit of P and sells 4 units of Q and 6 units of
for
R. In the process A, B and C earn ? 6000, f 5000 and ? 13000 respectively. If selling the units
ur
is positive earning and buying the units is negative earnings, find the price per unit of three
commodities by using matrix method.
ks

■Two factories decided to award their employees for three values of (a) adaptable to new
Yo

techniques, (b) careful and alert in difficult situations and (c) keeping calm in tense
oo

situations, at the rate of ? x, ? y and ? 2 per person respectively. The first factory decided to
eB

honour respectively 2,4 and 3 employees with a total prize money of ? 29000 The second
honour respectively 5, 2 and 3 employees with the prize money of
? 30500. If the three prizes per person together cost ? 9500, then
r

(i) represent the above situation by a matrix equation and form linear equations using
ou
ad

matrix multiplication. °
(ii) Solve these equations using matrices.
Y

11. A total amount of ^ 7000 is deposited in three different saving bank accounts with annual
Re
nd

interest rates 5%, 8% and 8—% respectively. The total annual interest from these three
Fi

accounts is ^ 550. Equal amounts have been deposited in the 5% and 8% savings accounts.
Find the amount deposited in each of the three accounts, with the help of matrices.
12. A shopkeeper has 3 varieties of pens 'A', 'B' and 'C. Meenu purchased 1 pen of each variety
for a total of ? 21. Jeen purchased 4 pens of 'A' variety, 3 pens of 'B' variety and 2 pens of 'C
variety for ? 60. While Shikha purchased 6 pens of 'A' variety, 2 pens of ‘B‘ variety and 3
pens of 'C variety for ? 70. Using matrix method find the cost of each pen.
13. Puransh has invested a part of his investment in 10% bond A and a part in 15% bond B. His
interest income during first year is ? 4000. If he invests 20% more in 10% bond A and 10%
more in 15% bond B his income during second year increases by ? 500. Find his initial
investments and new investments in bonds A and 6 using matrix method.
14. To control a crop disease it is necessary to use 8 units of chemical A, 14 units of chemical B
and 13 units of chemical C. One barrel of spray P contains one unit of A, 2 units of B and 3
units of C. One barrel of spray Q contains 2 units of A, 3 units of B and 2 units of C. One
barrel of spray R contains 1 unit of A, 2 units of 6 and 2 units of C. Using matrix method,
find how many barrels of each spray be used to just meet the requirement?
INVERSE AND APPLICATIONS OF A MATRICES 9.47

15. A firm produces two products P-\ and P2 passing through two machines and M2 before
completion. M^ can produce either 10 units of P^ or 15 units of P2 per hour. M2 can produce
15 units of either product per hour. Find daily production of -P] and P2 if time available is 12
hours on machine M^ and 10 hours on M2 per day using matrix inversion method.

rw
16. The equilibrium condition for three related markets is given by
Ilp^-P2-Ps=31
~Pl + ^Pl~^P3
-p^-2p2+7pj =24

e
Using matrix inversion method, find the equilibrium prices of each market.

e
ANSWERS

lo
7

r
(ii) .v = -l, y = 4
2' ^ 2

F
1 1

u
(ii) .Y = 2, 1/ = 3,2 = 5
XV - 2' -V - 3 '
2 =-
5
3. x = 2, y=-l, z = 4
oF
4. A=l, i/ = 2, 2 = 3
(iii) A = 36,1/=5,2 = -15

rs
5. (i) ,Y = 2,y=-l,2 = 4 (ii) .V = 1,1/ = 2,2 = - 3
6. 1,-1, 2 7. ? 2000, ? 3000, ? 5000

k
o
8. 11,15,19 9. A = 3000,1/=1000,2= 2000
10. A-= 2500, i/ = 3000, 2=4000 n. ? 1125, ^1125,? 4750
12. Variety A : ? 5, Variety B : ^ 8, Variety C : ? 8 of
o
Y
13. Initial investments: ^ 10,000 in A; t 20,000 in B
New investments: ? 12,000 in A; ? 22,000 in B
B
rY

14. P : 1 Barrel; Q : 2 Barrels; R : 3 Barrels 1 P^ : 60 units; P2 : 90 units


pj=4,p2=7,P3=6
ue

9.7 LEONTIEF INPUT-OUTPUT MODEL

The Leontief input-output model is named after its Creator Wassily Leontief (1906 -1999) who
od

received the Nobel Prize in Economics in 1973. It is a very valuable tool used for forecasting in
ad

economies of various countries in the World. Many models currently being used in the various
in

parts of the world are based upon this model. In this model there are n interdependent
industries producing n different commodities such that the commodity produced by each
Re

industry is partially consumed by the n industries and the rest is consumed by the external
F

bodies such as households, government and so on. The objective, in the model, is to find the
amount of production needed to meet out the inter-industry demand and the current demand of
external bodies.

In order to understand the model, let us consider an economy consisting of two industries as
described below:

An economy consists of two interdependent industries, say. Steel and Power. The steel industry
consumes 0.40 units of steel and 0.20 unit of power to produce each unit of steel. To produce
each unit of power, power industry consumes 0.25 unit of steel and 0.50 unit of power. The
economy has to fulfill external bodies demand of 80 units of steel and 40 units of power. How
many units of steel and power must be produced to meet the inter-industry demand and the
external demand?

In order to produce steel, the steel industry needs a little bit of steel (of course less than its
production) along with the power from power industry. Likewise the power industry needs
some steel from steel industry and power from itself to generate more power. We have to find
the amount of power and steel to be produced to meet out the inter-industry demand and the
external demand. Of course the amounts of commodities, the two industries will be producing.
9.48 APPLIED MATHEMATICS-XII

will be more than the amount needed to meet the external demand because some amount of the
steel and power produced goes back into the two industries to meet the inter-industry demand.
For both the industries, the quantity produced can be divided into three parts:
(i) Quantity needed to produce the same commodity,
(ii) Quantity needed to produce the other commodity,
(iii) Quantity needed to meat the external demand.
So, we obtain following equations.
Steel produced = [Steel required to produce more steel] + [Steel required to produce power]
+ [Steel required to meat external demand] ... (i)
Power produced = [Power required to produce steel]

w
+ [Power required to produce more power]
+ [Power required to meet external demand] ... (ii)
Let us now try to write these equations more mathematically. For this, we introduce two

Flo
variables x and y as follows:
X = Total amount of steel to be produced, y = Total amount of power to be produced.

e
It is given that

re
Amoimt of steel required to produce 1 unit of steel = 0.4 unit

F
Amoimt of steel required to produce x units of steel = OAx units
Amount of steel required to produce 1 unit of power = 0.25 unit
ur
r
Amount of steel required fo
to produce y units of power = 0.25y units
The external demand for steel is 80 units.
ks
Substituting these values in (i), we obtain
Yo

a: = 0.4 a: + 0.25y + 80
oo

Similarly, substituting the values in equation (ii), we obtain


y = 0.20a: + 050y + 40
B

Thus, we obtain the following system of equations for the production of x units of steel and y
re

units of power:
X = 0.4a: + 0.25y + 80
u
ad

y = 0.2a: + 050y + 40
Yo

This system of equations can be written more conveniently and quickly if we arrange the data
provided in the following tabular form which is generally caUed input-output table.
d
Re
in

INPUT-OUTPUT TABLE
F

Consumers (per unit input)


Steel Power External Demand

Producers (per unit output) Steel 0.40 0.25 80

Power 0.20 0.50 40

Entries in first column of input-output table give us the consumption of steel and power in
producing one umt of steel. Therefore, to produce x units of steel, the steel industry consumes
0.40x units of steel and 0.20x imits of power. Entries in the second column give us the
consumption of steel and power in producing one unit of power. Therefore, to produce y units of
power, it consumes 0.25y units of steel and 0.50y units of power.
Total consumption of steel = 0.40x + 0.25y + 80
and. Total consumption of power = 0.20x -f- 050y + 40
Using: Production = Consumption, we obtain
X = 0.40x + 0.25y + 80
y = 0.20x + 0.50y -i- 40
9.49
INVERSE AND APPLICATIONS OF A MATRICES

Writing these equations in matrix form, we obtain


rxi_ro.4xi ro.25y] rso’

■;c'l _r0.4x +0.251/1 fSO’


yJ"[o.2x +0.501/J’*'L40_
■xi_ro.4 0.251 rjci rso'
yj" 0.2 0.50
'0.4 0.25' [801
^D =

ow
X =AX + D, where A = ,X = 40
0.2 0.50 y

IX=AX + D
{1-A)X=D
If I / - A| ^ 0, then (/ - A) is invertible i.e. (i - A)”^ exists.

e
re
(/-A)X=D
(l-A)-^ {I-A)X={I-A)-^D

Frl
F
X = {1-A)-^D ...(iii)

ro.4 0.25'
ou
sor
Now, A =
0.2 0.50

ri 01 rO.4 0.25' r 0.6 -0.25 kf


J-A =
0 1 0.2 0.50 -0.2 0.50
oo
0.6 -0.25
= 0.3-0.05=0.25 5^0
Y

|/-A| = -0.2 0.50


B

(J - A) is invertible.
re

ro.5 0.251
Using: (J -A)-' “
oY

So,
0.25 [0.2 0.6 J
u
ad

Substituting the values of (7 -A)~^ and D in (iii), we obtain


d

1 ro.5 0.251 rso'


= 4 '40 + 101 _ [200'
0^|_0.2 0.6JL40.
X =
in

16 + 24 " 160
Re

xi_r2oo'
F

y “ 160
_ x = 200,1/=160
Hence, 200 units of steel and 160 units of power are to be produced by the steel and power
industries respectively to meet the inter-industry demand and external demand of 80 units of
steel and 40 units of power.
In the above discussion the matrix A is called the input-output coefficient matrix or the
technology matrix. The entries in the matrix A are called the input-output coefficients
or

technological coefficients.
Let us now introduce the Leontief input-output model in the formal way.
There are two types of Leontief input-output models viz. closed and open. In the closed modal,
the entire production is consumed by the participant industries i.e. the production is consumed
internally. The objective is to find the relative income of each participating industry. In the open
model, some of the production is consumed by the participating industries and the rest of the
production is consumed by external agencies such as households, government and so on. The
9.50
APPLIED MATHEMATICS-XII

objective is to find the amount of production needed to meet a forecasted demand when the
amount of production needed to meet the current demand is known. In this book we shall study
only open model. The discussion of closed model is beyond the scope of the book.
The general Leontief input-output open model can be described as follows:
Let there be an economy consisting of n industries /j, /2, 7„_i, /„ such that the commodity
or
commodities produced by each industry are partially consumed internally by the industries
and the rest are consumed by consumers such as households, government and so on for final use
called the final demand.

The inter-dependence of various industries is described in the following input-output table.


INPUT-OUTPUT TABLE

Consumers (Input)

w
Producers Industry Industry Industry Industry Fitral Total
(output) (h) (h) Ui) ih,) Demand output

Flo
Industry (h) ^11 ^12 Cl
Industry (72)

e
^21 X22 C2
52/ ●>^2» ^2

re
F
Industry (7,●) C;I
ur
r
%
fo
ks
Industry (7„)
Yo

■^1 C„ X,,
oo

In the above table industries /j, I2,..., //,..., I„ appear in rows and columns as well. The rows
B

exhibit outputs or supplies that each industry makes to other industries and the columns exhibit
re

the inputs (consumption) that each industry takes from other industries. All inputs and outputs
are in appropriate units, say, millions of rupees.
u
ad

The last entry in row 1 i.e. Xi represents that total output of industry /|.Out of this total output of
Yo

Xj units industry Ij supplies units to itself, X12 units to industry I2, ...,x^j units to industry I,●,
...xyj units to industry /„ and the restCj units to meet out the external demand.
d

=X|
Re
in

In order to produce x^ units industry consumes units of itself, X21 units of industry
I2, ●●●/ Xji umts of industry ... and x^j^ umts of industry Therefore, to produce 1 unit of
F

^11 ^21
output industry uses units of itself. unit of industry I2, ..., unit of industry
I://●●●/ ^1
unit of industry /„. Let = ^11
-,«21 =
£21 f ●
.., Oji -
5il . Then
^1 ^1
«11/ «21/ ^3V ●●●/ ^1 represent requirements per unit of output for industry /j.
hi row 2, the least entry X2 represents the total output of industry 12. Out of this total output
industry I2 supplies X21 units to industry fj, X22 units to itself,..., X2; units to industry/,.. ●/ X2„
umts to industry and the rest C2 units to meet out the external demand.
X2|+X22 + ... + .X2y + ... + X2,,+C2 =X2
To produce X2 units industry I2 consumes x^2 units of industry X22 units of itself, X32 units of
industry 13, units of industry 7,-, ...,x„2 units of industry 7„. Therefore, to produce one
^12 X22
umt of output industry I2 consumes unit of industry Ip unit of itself. ^32 unit of
^2 ^2 ^2
INVERSE AND APPLICATIONS OF A MATRICES 9.51

^2 Xn2
industry 13,..., unit of industry /,.. unit of industry Let
^2 ^2 ^2
^22 ^32 ^●2 ^2
«22 = ,fl32 = / '
● ●' ^‘2 ~ . Then, ^12/^22'^32'● ● ●'^2 represent require-
^2 ^2 ^2 ^2
ments per unit of output for industry /2-
Entries in other rows and columns can be interpreted in the same manner to obtain the
following equations
X11 + X12 + ●●●+ Xij +...+ Xy, H-Cj =Xi
X21+X22 + ●●●+ X2j +...+ X2JJ+C2 =X2

w
^●i + '-'i2 + ●●●+ +●●●+ ^hi+ Q=^’

Flo
^a + ^/2 + -+^v +---+x,j„+C„=x„
and the following matrix

ee
Fr
flu fli2 ...fliy ... ayj
fl2i fl22 ...«2y ●●● «2«

- [^j]n xn for
ur
Oil ^'2 ●●● ^7 ●●●
ks

%\ ^2 ●'●^7
Yo
oo

whose columns represent requirements per unit of output for industries I2, Ij ...//«
eB

respectively. This matrix is called the input output coefficient matrix or the technology matrix.
The entries in the matrix A are called the input-output coefficients or technological coefficients.
The technological coefficient Ojj represents the amount of input required from the industry to
r
ou
ad

produce one unit of output of industry. The technology matrix A =[fliy]MxM provides us the
Y

inter-industry consumption i.e. internal consumption when the total output of each industry in
the system to meet the final demand is given.
nd
Re

yi
Fi

V2

LetX = be the output matrix representing the production of each industry in the system to
Vi

Vn

A
d2
meet the final demand given by demand matrix D = . Then,
di

dn

Consumption = Production
Internal consumption + External demand = Total production
9.52 APPLIED MATHEMATICS-XII

'^iiyi+^i2y2 + ●●●+*/;■ +■●●+,v«+^^1 -yi


'’21’/! +^22?/2 + ■■■+ '^2/y/ +■■■+ ^2n 1/11+^2 =V2

<H\y\+^iy2 ^■■■+^jyj +■■■+ «,»y»+4- =yr

^»iyi+«;»2y2 + --+«,r/y/ +...+ a


IW y;i+^^
t =y,i

^11 ^12 ●●●^1/ ●●● ^Vi yi ■‘^1 yi

ow
^21 «22 ■■■^2/■■■ ^2jj '/2 ^2 y2

Oj2 ●●● (ijj ■■■ %, yi yi

e
(T,,2 .. . a
nn y» d, }ln

re
=> AX+D=X

Frl
IX~AX=D

F
(1-A)X=D
(/ -A)“^((/ -A) X) =(/ -Ay^D, provided that(7 -A)“^ exists
ou
r
X={l-Ar'^D
so
kf
This equation gives that the total output of each industry to meet the final demand. The outputs
of various industries are non-negative numbers, so the equation X={1 -A)~^D should give
oo

non-negative solutions. For this the system (7 -A)X =D must satisfy certain conditions which
Y

are known as Hawkins - Simon conditions for the viability of the system. The conditions given
eB

by D. Hawkins and H.A. Simon are as under: (i) the determinant 17 -A\ of the Leontief Matrix
must be positive, (ii) The diagonal elements:! -^22' ^ of the Leontief matrix I-A
ur

should all be positive, or in other words diagonal elements of matrix A should all be less than 1.
oY

9.7.1 EQUILIBRIUM PRICES


ad

Let us consider a hypothetical two sector economy producing only two commodities X and Y.
The two commodities serve as intermediate inputs in each others production.
d

X Y
in

X 0i2
Re

Let A = be the technology matrix for the given economy. This means thatiZi^ unit of
Y _<^21 ^22
F

X and unit of V arc used to produce one unit of commodity X, whereas a^2 of X and ^22
unit of Y are used to produce one unit of commodity Y. The product of two commodities also
require some primary input, say, labour. Suppose /j units and I2 units of primary input are
needed to produce each unit of X and Y respectively. Let the price ofprimary input be za per unit.
Let the equilibrium price per unit of the two commodities be p| and p2 respectively. Since each
unit of commodity X requires rt-] I unitofX,<72i unit of Y and /junitsof primary input. Therefore,
Cost of producing one unit of commodity X = -1- n2i Pi +
Similarly, Cost of producing one unit of commodity Y = a■^2 P\ + (^22 Pi
Since p^, p2 are equilibrium prices of the commodities. Therefore,
^llPl + '*2lP2 + ^1^^'=7h
^12 Pi ^^22 P2 ^2^^ ~ Pi
{i-z7n)Pi-rt2iP2
-rtl2 P]+(1-ZJ22) P2 “
This system of equations can be written in the following matrix form
INVERSE AND APPLICATIONS OF A MATRICES 9.53

1-^11 -^21 P\ w

-
1 -a 22 P2 I2 U’

(I-A) t\P}^
Pij h
-l r
T zo
Pi
= (/-/!)
Pi I2 ii’
T r
-1
Pi /■j Zt>
Pi
= \U-A) I2 W
Tliis gives the equilibrium prices of the two commodities.

w
If the gross output of the two commities X and Y are respectively .v units and y units to meet the
final demand. Then,
/i IV

lo
Value added = [.v y]
I2 w

e
re
rF
ILLUSTRATIVE EXAMPLES

F
Type I ON FINDING THE GROSS OUTPUTS
EXAMi’LL 1 A)7 economi/ produces otily coal and steel. The tioo commodities serve as intermediate

r
fo
inputs in each other's production. 0.4 tonne of steel and 0.7 tonne of coal are needed to produce a tonne of
ou
steel. Similarly, 0.1 tonne of steel and 0.6 tonne of coal are required to produce a tonne of coal. If the
ks
economy needs 100 tonne of coal and 50 tonne of steel, calculate the gross output of the two commodities.
SOLUTION The input-output coefficients may be arranged in the following tabular form:
oo
Y

INPUT-OUTPUT TABLE
eB

Input
Coal
ur

Steel
Output
ad

Steel 0.4 0.1


Yo

Coal 0.7 0.6


d

Let A denote the input-output coefficient matrix or the technology matrix. Then,
Re
in

0.4 0.1 ■
=
F

0.7 0.6

The Leontief matrix is given by


1 01 ro.4 O.T 0.6 -0.1
l-A =
0 1 0.7 0.6 -0.7 0.4

0.6 -0.1
\I-A\ = -0.7 0.4
= 0.24-0.07 =0.17

We find that 11 -A\ > 0 and diagonal elements of A are all less than 1. So, Hawkins-Simon
conditions are satisfied and hence the system is viable.
Let the gross output of steel and coal be x-[ units and .1*2 units respectively to meet the final
50 ■^T
demand given by the demand vector D = . Further, letX = .Then,
100 ■^‘2
1
X=(/-/\)"‘D
9.54
APPLIED MATHEMATICS-XII

Now,
0.6 -0.1
1-A =
-0.7 0.4

0.4 0.1
cidj {! -A) = 0.7 0.6

1 0.4 0.1

0.17 [0-7 0.6

Substituting the values of (/ -A)~'^ and D in (i), we obtain


30

ow
1 0.4 0.1 50 1 20 + 10 0.17 176.5
X =
0.17 0.7 0.6 100 0.17 35 + 60 95 558.8
0.17

'176.5

e
558.8 Xj =176.5, X2 =558.8
.^2

re
Fl
Hence, the gross outputs of steel and coal, for the given demand, are 176.5 and 558.8 tonnes

F
respectively.
ur
EXAMPLE 2 A two industries input-output relationship states that mdustry 1 requires 20 paisa worth of

r
fo
its own output and 50 paisa worth ofindustrij IIfor producing output of^ 1. Industry II requires 30 paisa
worth of its own output and 40 paisa worth of industry I for producing output ofK 1.
ks
(i) Write the technology coefficient matrix,
Yo

(ii) Determine the gross output required to satisfy the economy's demand of^ 180 crores and ? 270
oo

crores respectively.
eB

(Hi) Give the interpretation of its row sum and column sum, if any
SOLUTION (i) The input-output table is as given below:
ur

Input
ad
Yo

Industry I Industry 1!
Output
Industry I 0.20 0.40
d

Industry II 0.50
Re

0.30
in

Let A be the input-output coefficient matrix or, the technology matrix. Then,
F

'0.20 0.401
/I =
0.50 0.30

(ii) Let ^ Xj crores and ? X2 crores be the gross outputs of industries I and II respectively to meet
the demand of ? 180 crores and 1270 crores respectively. Then, the gross output matrix X =
X2
satisfies the equation
180
(I - A) X =D, where D = is the demand matrix.
270
0.20 0.40'
Now, A =
0.50 0.30
T 01 0.20 0.40' 0.80 -0.40
I-A =
0 1 0.50 0.30 -0.50 0.70

(7-^1 = 0.56-0.20=0.36^0
9.55
INVERSE AND APPLICATIONS OF A MATRICES

We find that ]/ -A\ > 0 and diagonal elements of A are all less than 1. So, Hawkins-Simon
conditions are satisfied and hence the system is viable.
0.70 0.40"
adj{J-A) = 0.50 0.80

1
adj (/ -y4)
\l-A\
0.70 0.40
0.36 0.50 0.80

Now, {I-A)X=D
1
X={I-A)-'D

ow
x=^
0.70 0.401 ri80' 1 ri26 + 108l_r650
0.36 0.50 0.80 270 0.36 90 + 216 “ 850

650

e
850
.■●^2

Fl
re
.Ti=650,a-2=850

F
Hence, the gross outputs of industries fj and II2 must be ? 650 crores and ? 850 crores
ur
respectively,

r
(iii) Row sum has no economic meaning First column fo
sum gives us the cost of intermediate
inputs to produce a rupee worth of output by iiidustry 1. Thus, to produce an output of worth ? 1
ks
the industry I requires input of worth ? (0.20 + 0.50) = ? 0.70 and the balance ^ 0,30 denotes the
Yo

value of addition per rupee. Likewise second column sum gives us the cost of intermediate
oo

inputs to produce a rupee worth of output by industry 11. Thus, to produce an output of worth
? 1 the industry II requires input of worth ? (0.40 + 0.30) = ? 0.70 and the balance ? 0.30 denotes
B

the value addition per rupee.


re

^ win A countn/ produces only two goods X and Y. The input-output coefficient matrix is
0.3 0.6’
u
ad

A = . The final demand for X is W and for Y is 5.


Yo

0.5 0

(;) Write down the open input-output model as a set of simultaneous linear equations,
(ii) Determine the output solution by finding the inverse of the relevant matrix.
d
Re
in

■'1
SOLUTION (i) Let X = be the gross output vector required to meet the final demand
.●^2
F

0.3 0.6'
D = . It is given that technology matrix is = 0.5 0 ■

Since production equals consumption. So, the balancing equations are


0.3 + 0.6.r2 +10 = A’l
0.5 A'-] + 0 ^2 + 5 = .a'2
(ii) In matrix notation, these equations can be written as
0.3 0.6 -■^1 10 ^1
+
0.5 0 5 .^2
.^'2
or. AX+D=X

X(I-A)-D
X - (I - A) D, provided tha 11 / - A10.
9.56
APPLIED MATHEMATICS-XII

‘1 0 '0.3 0.6' 0.7 -0.6


Now, I
0 1 0.5 0 -0.5 1

'1 0.6
/->i| = 0.7-0.3 = 0.4 and, ndj {I - A) = 0.5 0.7

-1 1 1 r 1 0.6
Hence, (/ -A) adj (/-A) =
1-A\ 0.4 0.5 0.7

Substituting these values inX =(/-A) ^D, we obtain


1 0.61 rioi 1 10+3 32.5
0.4 0.5 0.7 5 ^15+3.5 " 21.25
-v-i = 325, .1-2 = 21.25
\L1TER (ii) The equations obtained in (i) can be written as

w
07.V] -0.6a'2 =10
-05xi +X2 =5
This system of equations can be written, in matrix notation, as
■ 0.7 -o.6ir.vi]_rio'
F lo
ee
-0.5 5

Fr
■ 0.7 -0.6 10
or. AX = B, where A = ,X = and B =
-0.5 1
:^2
for
5

0.7 -0.6'
r
Now, A =
-0.5 1
You
s
ook

0.7 -0.6 1 0.6


|A| = -0.5 1 = 0.7-0.3 = 0.4 and, iidj A = 0.5 0.7
eB

-1 1 1 r 1 0.6
adj A
l^i 0.4 0.5 0.7
our
ad

Now, AX = B
1
X=A~‘B
dY

r 131
Re

1 1 0.61 rio
^1 j_rio+3 0.4 32.5
Fin

.^2 0.4 0.5 0.7 5 0.4 5+35 8.5 21.25

= 32.5, X2 =21.25
EXAMPLE 4 The inter-industry flow of the products of two industries is given ns under:
Consumption Consu^np^tion Final demand
industry! industry II Gross output

Production industry I 16 20 4 40

Production industry II 8 40 32 80

Determine the technology matrix. Find the gross output for each industry for the final demands of 36 and
88 units respectively.
9.57
INVERSE AND APPLICATIONS OF A MATRICES

SOLUTION Industry I consumes 16 units of output of itself and 8 units of output of industry 11
to produce 40 units of output. Therefore, it uses ^ unit of itself and — unit of industry II to
produce 1 unit of output similarly, industry 11 uses —
80
unit of industry 1 and ^
80
unit of itself to
produce 1 unit of output. Thus, the input-output coefficient matrix A is given by
ri6 ^ 2 1

40 80 5 4
71 =

w
40 1 1

,40 80 J L5 2j

LetX =
●^1
be the output matrix representing the gross output of each industry to meet the final

o
.-^2

e
36
. Then,

re
deman D -

rFl
F
(1-A)X=D
3
2 J_
1 0 5 4
5 4

or
ou
Now, I-A =
0 1 1 1 1 1_
ksf
.5 2 5 2

3 1 1 1
oo

2 4
^ 20^ =1>0 and (7tf/(/-A) =
Y

' ' l l 10 4 1 3
B

5 2 L5 5 J
re

1 1

4
nifJ(I^A)=4 ^1
1
oYu

(l-A)-^ =
ad

3
7-A|
5 5J
d

Now, (7-A)X=D
in
Re

X=(7-A)'^D
F

1 1
18 + 22 160
4 36
2
=4 = 4 36 264
1 3 88 + 240
.●^2 5 5 J
5 5

.V;i=160,.Y2=240
Hence, the gross output of two industries I and II for the final demand are 160 and 240 units
respectively.
EXAMPU s Suppose the inter-hictustryflow of the products of tioo industries is given as under:
Consumption Domestic Demand Gross output

X
40 50 120
Production X 30

20 10 30 60
Production Y
9.58
APPLIED MATHEMATICS-Xli

Determine the technology matrix and test Hawkins-Simon conditions for the viability of the system.
"80'
Compute the equilibrium level of output of the products when the domestic demand vector is
40 *

SOLUTION (i) To find the technology matrix, we divide the industry entries in each industry
column by the total output of that industry. Thus, we obtain the following technology matrix
~3o 401 2]
A = 120 60 ^ 4 3
20 10 1 1
.120 60 J Le 6.

ow
ri 2 1 r 3 2 1
1 0
I-A = 4 3 4 3

0 1J 1 1 _l 5
- 6 6. .6 6.

e
3 2

re
f1 3 3X 5

rFl 2 1
X — =— > 0
37 „

F
5 4 6 3 6 72
6 6

or
We also observe that the diagonal elements of the Leontief matrix / -
ou
are all positive.
Hence, Hawkins — Simon conditions are satisfied and the system is viable.
ksf
X
rsoi
Let X = denote the total vector when the domestic demand vector IS
oo

i .Then,
y 40
Y
B

Production = Consumption
AX + D=X =>(/-A)X=D ^X=(/-A)"^D
re

3 _2‘
oYu
ad

4 3
Now, I-A =
1 5
d

6 6
in
Re

■5 21 ■5 2'
F

adj{I-A)= f ^ -1 1 72 6 3
and A adj(l-A) =
\I-A\ 37 1 3
6 4 6 4

X = {I-A)-^D
5 2'
’2801 [67201
X
72 6 3 [801 72 3 37
37 1 3 40 37 130 3120
y \

6 4
. 3 J L 37 .
6720 3120
X =
37
and y =
37

6720 , 3120
Thus, the equilibrium level of output of the products is 37
and
units respectively.
37
INVERSE AND APPLICATIONS OF A MATRICES 9.59

Typjc II ON FINDING THE FINAL DEMAND WHEN GROSS OUTPUTS ARE GIVEN
EXAMPLE For two industries, input-output relationships (in units) are given below:

Input

Indiislry I Industry II Final Demand Gross output


Output Industry I 50 75 75 200

Industry II 100 50 50 200

'400'
If the gross output increases to 600 ' determine the final demand which can he satisfied.

w
SOLUTION To find the input-output coefficient matrix, we divided the industry entries in each
industry column by the total value of output for that industry. Thus, we obtain the following

Flo
input-output coefficient matrix
r 50 75 1 3

e
re
200 200 4 8

F
100 50 1 1

200 200 2 4
ur
r
A fo 400'
Let D = be the final demand vector when the gross output increase to 600 . Using
.^2
ks
production equals consumption, we obtain
Yo

71X + D=X => D=X-AX => D={I-A)X


oo

l_i _3 400 Ix400--x600 300-225 75


B

4 4 8
re

.^2 i 1-1 600


1
— x400 + -x600 -200 + 450 250
2 4 2 4
u
ad

=75 and ^2 =250


Yo

Hence the gross output of 450 units of industry 1 and 600 units of industry II is sufficient to meet
the final demand of 75 units of industry I and 250 units of industry II.
d
Re

Type III ON FINDING THE GROSS OUTPUTS. EQUILIBRIUM PRICES AND VALUE ADDED
in

EXAMIM.r." A two sector economy produces coal and steel. The two commodities serve as intermediate
F

input in each other's production. 0.4 tonne of steel and 0.7 tonne of coal are needed to produce a tonne of
steel Similarly, 0.1 tonne of steel and 0.6 tonne of coat are required to produce a tonne of coal. 2 and 5
labour days are required to produce a tonne of coal and steel respectively. If the economy needs 100 tonnes
of coat and 50 tonnes of steel, calculate the gross output of the two commodities and total labour reqidred.
Also, determine the equilibrium prices of the two commodities and the value added, if the wage rate is
? 1000 per man-day.
SOLUTION The input-output coefficients may be put in the following tabular form:
Input

Steel Coal

Steel 0.4 0.1


Output
Coal 0.7 0.6
9.60 APPLIED MATHEMATICS-Xli

’0.4 0.1’
Let A denote the input-output matrix. Then, A = 0.7 0.6
. The Leontief matrix is

I-A = T 0] ro.4 0.1] _r 0.6 -o.r


0 lJ"[o.7 0.6 “ -0.7 0.4
0.6 -0.1
\I-A\ = -0.7 0.4
= 0.24-0.07 =0.17 >0

We find that |J-y4|>0 and all the diagonal elements in {I-A) are positive. Hence,
Hawkins-Simon conditiorw are satisfied and the system is viable.
X
Let X = be the gross output vector for steel and coal to meet the final demand given by the
y_
r 501

low
vector D =
100 . Then, by using Consumption = Production, we obtain

AX + D =X => (7 -A) X =D => X =(/ -A)"^D


Now,
0.6 -O.r

ee
7-A = rF
-0.7 0.4

Fr
fo.4 O.r 0.4 0.1
adj(7-A) = and, (7 - A)"^ = adj (7 - A) = —
0.7 0.6 |7-A| for ^ 0.17 0.7 0.6

Hence,
u
X=(7-A)"^D
ks
Yo

r 3001
oo

X
^ rO.4 0.1] r 50 17
176.5
eB

0.17 9500
y. 0.7 0.6 100 558.8
L 17 J
r

x = 176.5 and y =558.8


ou
ad

Hence, the gross outputs of steel and coal, for the given final demand, are 176.5 and 558.8 tonnes
Y

respectively.
X T76.5
nd

Total labour days required = [/j l2\ ' = [5 2] = 2000.1


Re

y 558.8
Fi

Let ? Pi and ? p2 be the equilibrium prices per tonne of steel and coal respectively. Then,

P2 I2W
Pi _ 1 ro.4 0.7]^r5,000' [v/j=5,/2=2 and a; = ?1000]
P2 0.17 [0.1 O.bJ “[2,000
Pi _ 1 [3,400] [20,000'
P2 0.17 1/700 10,000

Pi = 20,000 and p2 =10,000


Hence, the equilibrium prices per tonne of steel and coal are ? 20,000 and ?10,000 respectively.
[1765 558.8] [5,000] = 2,000,100
lizu
Value added = [x y\ 2,000
I2 w
Hence, value added = ? 2,000,100
9.61
INVERSE AND APPLICATIONS OF A MATRICES

EXAMPLES A conntri/ produces only tivo goods X and Y. The txoo commodities serves as intermediate
inputs in each other's production. O.i unit of X and 0.55 unit of Y are needed to produce a unit of X,
whereas 0.4 unit of X and 0.2 unit ofYare needed to produce a unit ofY. In addition to this 4 units and
3 units of primary input are needed to produce each unit of X and Y respectii’el}/.
(i) if 240 and 140 units ofX and Y respectively are needed for final consumption, find gross output
levels of the two goods X and Y.
(ii) If the price of primary input is ^ 100 per unit, compute the prices of goods.
(iii) Compute the total value added.
SOLUTION (i) The information provided can be put in the following tabular form:
Consumer (Input)

ow
X Y Final Demand

Producers X 0.1 0.4 240

0.55 0.2 140


(outputs) y

Primary inputs 4 3

e
' 0.1 0.4

re
The input-output coefficient matrix is : A =
rFl 0.55 0.2

F
'240'
Let ^ be the gross output vector to meet the final demand D = 140 ■
Then, production

r
ou
equals consumption gives fo
ks
AX + D=X =>(/-A)X-D
0.1 0.4"
oo

Now, A =
0.55 0.2
Y
eB

■ 0.9 -0.4
(I-A) =
-0.55 0.8
ur

0.9 -0.4
/-Al = = 0.72-0.22 =0.5 ?^0
ad

-0.55 0.8
Yo

(/ - A) is invertible.
d

1 1 ro.8 0.4' T.6 0.8'


(7-/1)-' = adj(/ -A) = ^ 0.55
Re
in

\I-A 0.9 1.1 1.8


F

Now, (I-A)X=D
X=(/ -A)"^ D
.r 1.6 0.8 240 _r384 + 112l [496’
1.1 1.8 140 ^ 264-h 252 516
y.
.v=496, y=516
Hence, gross output levels of goods X and Y are 496 and 516 units respectively,
(ii) Let ^ Pi and ? P2 be the equilibrium prices per unit of goods X and Y respectively when
primary inputs of 4 units and 3 units are needed at the rate of ^ 100 per unit to produce each unit
of X and Y respectively. Then,
T r
Pi' _ -1 ll W
P2 I2 w
-iT ^
Pi _
n.6 0.8 4x100

Pi 1.1 1.8 3x100


9.62 APPLIED MATHEMATICS-XII

1.6 1.1 nT [4001 640+330 '970


=>
Pi
P2 0.8 1.8 300 320+540 860

Pi = 970 and p2 = 860


Hence, the equilibrium prices per unit of goods X and Y are ? 970 and ? 860 respectively.
"'ll w
(iii) Total value added = [.y y]
I2 w
'400
= [486 516] 300
=? 353,200

'0.5 0.3

ow
r-.XANJlM.F 9 For a two sector economy, the input-output coefficient matrix is: A = , If the
0.2 0.4

final demands of two sectors are 10 and 13, find the gross output. Land and labour are used as primary
Labour [0.4 0.31
inputs. Their coefficients for the tzuo sectors are given as Land 0.5 0.4 . If the wage rate and rent

e
re
are ? 400 and ? 1,000 respectively, find the equilibrium prices for the tzuo sectors.

rFl
F
SOLUTION Let X = 1/
be the gross output vector for the two sectors to meet the final demand
10
given by D = . Then, by using production equals consumption, we obtain

r
13
ou
AX + D=X => {I-A)X=D
fo
ks
Now,

1 01 [0.5 0.3 0.5 -0.3


oo

I-A =
0 lj“|_0.2 0.4 -0.2 0.6
Y
B

0.5 -0.3
I-A = 0.3-0.06=0.24^0
"1-0.2 0.6
re

So, inverse of I -A exists.


ou
Y
ad

'0.6 0.31
adj(/-A) =
0.2 0.5
d

1 1 0.6 0.3
adj(/ -A) =
in
Re

1-A\ 0.24 0.2 0.5


F

Now, (1-A)X=D
X=(/-/l)-^D
1,500
X 1 0.6 0.3 10 1 ‘15 24 62,5
V 0.24 0.2 0.5 30 0.24 17 70.83
1,700
24

Y=62.5 and y =70.83


Thus, the gross output for the two sectors are 62.5 and 70.83 respectively.
The costs of primary inputs for each unit of two sectors are given by
Land Labour] [ 400 Sector T 660
Sector I 0.4 0.5
Sector II 0.3 0.4 1,000 Sector II 520

Let t Pi and ? p2 be the equilibrium prices per unit for the two sectors. Then,
INVERSE AND APPLICATIONS OF A MATRICES 9.63

Tr
Pi -1 66 1 ro.6 0.21 r66 1 50 '208.33'
= (/-A)
P2 52 ^ 0.3 0.5 52 0.24 45.8 190.83

Pi =208.33 and P2=190.83


Hence, the equilibrium prices per unit of two sectors are 1208.33 and ? 190.83 respectively.
EXERCISE 9.3

1.
Two commodities A and B are produced such that 0.4 tonne of A and 0.7 tonne of B are
required to produced a tonne of A. Similarly, 0.1 tonne of A and 0.6 tonne of B are needed to
produce a tonne of B. Write down the technology matrix. If 68 tonnes of A and 102 tonnes of
B are required, find out the gross production of both of them.
Given the following transaction matrix, find the gross output to meet the final demand of
200 units of Agriculture and 800 units of Industry.

w
Producing sector Purchasing sector Final Demand

Agriculture industry
Agriculture
Industry
300

400

F lo 600

1200
100

400

ee
Suppose the inter-relationship between the production of two industries A and B in a given

Fr
year is: for
Consigners Current Demand Total output
r
A B
You
s

A 14 6 8 28
ook

B 7 18 11 36
eB

4.
In a two sector economy, it is known that industry I uses 10 paise worth of its own product
and 60 paise worth of commodity II to produce a rupee worth of commodity I; industry II
uses 30 paise worth of its own product and 50 paise worth of commodity I in producing a
our
ad

rupee worth of commodity II; and final demands are ? 1,100 billion worth of commodity I
and ? 2,200 billion worth of commodity II.
(i) Write down the input output matrix for this economy,
dY
Re

(ii) Find the gross outputs of two industries to meet the final demand.
A two industry input-output relationships are given below:
Fin

Industry—>■ I II Final demand Gross output


i

I 16 20 4 40

II 8 40 32 80

Labour day 80 120

Find;

(i) Gross outputs of the two industries to meet the final demand of 18 units of Industry I
and 44 units of industry II.
(ii) Total labour days required,
(iii) Total value additions, if wage is ? 40 labour day.
(iv) Equilibrium prices using the same wage rate.
9.64 APPLIED MATHEMATICS-XII

6. For two industries, input-output relationships (in units) are as follows:

lndustn/-> } II Finn! dcninnd Gross output


I

I 20 40 40 100

II 30 50 40 120

Capital (?) 2,000 3,000

Labour day 200 300

Find:

(i) Gross output required to satisfy the new final demand of 110 units and 220 units for
industry I and II respectively,

w
(ii) Total requirement of capital and labour days,
(hi) Total value addition, if cost of capital is 10% and the labour cost is ? 200 per labour
day.

F lo
7. A hypothetical economy produces only two commodities, X and V. The two commodities
serve as intermediate inputs in each other's production. To produce a unit of X, 0.6 unit of X

e
Fre
and 0.1 unit of Y are needed. Similarly, to produce a unit of Y, 0.7 unit of X and 0.4 unit of Y
are needed. Two and four labour days are required to produce a unit of X and a unit of V
respectively. The wage rate is ? 70 per man-day. The economy needs 100 units of X and 50
for
units of y for final consumption. Find:
(i) the gross output of each of the two commodiites.
r
You

(ii) the equilibrium prices,


oks

(iii) the total value added.


eBo

ANSWERS

1. A: 220 tonnes, 6:640 tonnes 2. Agriculture: 2000, Industry: 4000


3. A:72, B:96
our
ad

4. (i) Commodity I: ? 5666.67 billion Commodity II: ?8000 billion


5. (i) Industry 1:80 units. Industry II: 120 units (ii) 340
(hi) ? 13,600 (iv) ? 208 and ? 224 per unit
dY
Re

6. (i) Industry I: 375, Industry II: 570


(ii) Capital: ? 21,750, Labour days: 2175
Fin

(hi) The total value addition: ? 4,37,175


7. (i) X: 558.82 units, Y: 176.47 units (ii) X:?700, Y: ? 1400
(iii) ? 139,999.30

MULTIPLECHOICEQUESTIONS(MCQs)
1.
If A is an invertible matrix, then which of the following is not true
-1 1x2 -1 -1
(a) = (A-^) (b)|A =|A|
(c) (A^)-l = (A-‘)UT (d) I A I ^ 0
If A is an invertible matrix of order 3, then which of the following is not true
Ix-l
(a) I adj A I = | A |“ (b) (A-) = A

(c) If BA =CA, then B ^C, where B and C are square matrices of order 3
(d) (AB)“^ = A
-1
, where B = [bjj] 3^3 and | B | 0
INVERSE AND APPLICATIONS OF A MATRICES 9.65

[3 41 r-2
^ , then {A + B)
-1
3. IfA =
2 4 0

(a) is a skew-symmetric matrix (b)


(c) does not exist (d) none of these
a b
4. IfS =
c d
, then adj A is
d c
(b) M
-d -b -b d b
(d)
-c a a (0 , a b a

5. If A is a singular matrix, then adj A is

ow
(a) non-singular (b) singular (c) symmetric (d) not defined

6. If A, B are two nxn non-singular matrices, then


(a) AB is non-singular (b) AB is singular
(c) (ABr^=A"^ B~^ (d) (AB)“ ^ does not exist

e
re
'a 0 O'

rFl
F
7. If A = 0 a 0 , then the value of] adj A | is
0 0

r
(a) a
27
(b) (c) / (d)
ou
1 2-1
fo
ks
8. IfA = -1 1 2 , then det (adj (adj A)) is
oo

2 -1 1

(c) 142
Y

(a) 14^ (b) 14^ (d) 14


eB

9. If B is a non-singular matrix and A is a square matrix, then det (B ^ AB) is equal to


r

1 1
(a) Det(A-^) (b) Det(B-‘) (c) Det (A) (d) Det(B)
You
ad

'10 O'
10. For any 2x2 matrix, if A {adj A) = 0 10
, then I AI is equal to
d

(a) 20 (b) 100 (c) 10 (d) 0


/ for 1 < « < 4, then (/ - A) ^ equals
Re
in

11. If A^ = O such that A"


(a) a'' (c) I+A (d) none of these
F

(b)

12. If A satisfies the equation -5x^ + 4x + X = 0, then exists if


{a) X ^ 1 (b) 2 (c) X ^ -1 (d) ^ 0
13. If for the matrix A, A ^ = /, then A” ^ =
(a) a2 (b) A^ (c)^ (d) none of these

14. If A and B are square matrices such that B = - A~ ^ BA, then (A + B)^ =
(a)0 (b) a2 + b2 (c) a2 + 2AB + b2 (d) a + B
2 0 0]
15. If A = 0 2 0, then A^ =
0 0 2

(a) 5A (b) lOA (c) 16A (d) 32A


9.66
APPLIED MATHEMATICS-XII

16. For non-singular square matrix A, B and C of the same order {AB~ ^ C) -]
-1 -1
(a) eC (b) ic) CBA (d)
5 10 3

17. The matrix -2 -4 6 is a singular matrix, if the value of is


-1 -2 h

(a) - 3 (b) 3 (c) 0 (d) non-existent


18. Ifi/is the determinant of a square matrix^ oforder??, then the determinant of its adjoint is
n-\ II + 1
(a) d' (b) d (c) ci (d) d
Ti. If is a matrix of order 3 and | /41 = 8, then [ adj A \ =
(a) 1 (b) 2 (c) 2^ (d) 2^

w
20. If A^ - A + I =0, then the inverse of A is
(a) /I-2 (b) A+1 (c) I-A (d) A-I

Flo
21. If A and B are invertible matrices, which of the following statement is not correct,
-1 -1
(a) adj y4 = \ A \ A (b) det = (det A)

ee
-1 -1 -1

Fr
-1
(c) {A + B) = A + B (d) (AB) = B A

7: If A is a square matrix such that A^ =1, then /T ^ is equal to


for
ur
(a) A +I (b) (c) 0 (d) 2A

23. Let A = ^ andS =


1 0
and X be a matrix such that A = BX, then X is ●
s
0 2
k
Yo
oo

1 -2 4 . . 2 4
(b)-
2 3 -5 3 5
<")[3 -5 (d) none of these
eB

-b If ^4? 2 be such that A~^ = kA, then k equals


3 n
o
r
ou
ad

(a) 19 fb) 1/19 (c) -19 (d) -1/19


1 1 2
Y

2.3. IfA = - 2 1 - 2 satisfies A A=l, then x + i/ =


Re
nd

y.
(a) 3 (b) 0 (c)-3 (d) 1
Fi

1 0 1
2(k lfA =
0 0 1 . then rtf+M + 2 equals
a b 2

(a) /I (b) -/I (c) ab A (d) none of these


1 tan 0 1 tan 6
If
tan 0 1 - tan 0 1
-1
b
~^then
a

(a) <7 = 1, = 1 (b) = cos 2 0,1? = sin 2 0


(c) = sin 2 0, b = cos 2 0 (d) none of these
28. If a matrix A is such that 3 A^ + 2 A~ + 5 A + I = 0, then A -1 is equal to
(a) -{3A^ +2A+5) (b) 3a2 +2A+5
(c) 3A^-2A-5 (d) none of these
INVERSE AND APPLICATIONS OF A MATRICES 9.67

29. If A is an invertible matrix, then det (A is equal to


1
(a) det (A) (b) (c) 1 (d) none of these
det (71)
2 X -3
30. If/i = 0 2 5 , then A~^ exists if
1 1 3

(a) X=2 (b) 1^2 (c) (d) none of these

ow
ANSWERS

1. (a) 2. (c) 3. (d) 4. (b) 5. (b) 6. (a) 7. (c) 8. (a) 9. (c)


10. (c) 11. (d) 12. (d) 13. (a) I.’, (b) (c) 16. (d) 17. (d) IS. (b)
19. (d) 20. (c) 21. (c) 22. (b) 23. (a) 24. (b) 25. (c) 26. (d) 27. (b)

e
28. (d) 29. (b) 30. (d)

re
Frl
FILL IN THE BLANKS QUESTIONS (FBQs)

F
1. If A is a unit matrix of order n, then A (adj A) =
If A is a non-singular square matrix such that A ^
-1
2. = /, then A
ou
or
-1
3. If A and B are square matrices of the same order and AB = 31, then A kfs
1 a 2

4. If the matrix A = 1 2 5 is not invertible, then a =


oo

2 1 1
Y
eB

5. If A is a singular matrix, then A (adj A) =


6.
Let A be a square matrix of order 3 such that) A | = 11 and B be the matrix of cofactors of
ur

elements of A. Then, | B r =
oY

10 0 ■
ad

If A is a square matrix of order 2 such that A(adjA) =


0 10 '
then| A| =
d

8. If A is an invertible matrix of order 3 and | A | = 3, then | adj A | =


in
Re

9.
If A is an invertible matrix of order 3 and | A | =5, then adj (adj A) = ....
10.
If A is an invertible matrix of order 3 and | A | = 4, then | adj (adj A) | =
F

If A = diag (1, 2, 3), then [adj (adj A) | -

= ^, then | A
-1
12.
If A is a square matrix of order 3 such that | A |
13.
If A is a square matrix such that A (adj A) = 10/, then | A | =.
14.
Let A be a square matrix of order 3 and B = | A | A“^.If | A | = -5, then] B| =
15. If is a scalar and / is a unit matrix of order 3, then adj (kl) =
COS.Y Sin.Y k 01
16. IfA = and A (adj A) = , then k =
-Sm.Y COS.Y 0 k

17.
If A is a non-singular matrix of order 3, then adj (adj A) is equal to
18. If where
f i 4- j, if / / -1
,, then A
9.68 APPLIED MATHEMATICS-XIl

0 3
19. l(A =
2 0
and A ^ = X (adj A), then X =

20. If /I is a 3 X 3 non-singular matrix such that AA


T
= /I ^/I and B = A ^a'^, then
T
BB

2^. U A and B are two square matrices of the same order such that B--A ^BA, then
{A + Bf=
3 —1
22. If A is a non-singular matrix of order 3x3, then (/I ) =
23. If A be a square matrix such that | adj A | = | /I | , then the order of A is
X 5 2"

2!. IfA= 2 y 3 , .vi/z = 80, 3.v +2i/+ 10z = 20 and A adj A = W, then/:=.
1 1 z

w
ANSWERS

1(>.
I. A

3. 9

1
2. A^

17.
5A

lAIA
. Ib
3

’■.c. 4^^ 1
4. 1

18. -
6"^

1
F lo
0 3
Null matrix

5 for F
19.
10

-
ree
1
14.

20.
6. n**

25

I
7, 10

. k^l

21. A^ + B
9 3 1 6

K3
22. iA~^)
Your

3x 3 24. 81
ks
eBoo

VERY SHORT ANSWER QUESTIONS (VSAQs)

Answer each of the foliozoing questions i)i one word or one sentence or ns per exact requirement of the
ad
our

question:
-3 4
Write the adjoint of the matrix A = ^ -2 ●
Re

If A is a square matrix such that A (adj A) =5/, where / denotes the identity matrix of the
Y
Find

same order. Then, find the value of | A |.


IfAisasquare matrix of order3suchthat|A| =5, write the value of |adjA|.
4, If A is a square matrix of order 3 such that| adj A| =64, find lAj.
5. If A is a non-singular square matrix such that I A| =10, find 1A”^|.
” If A, B, C are three non-null square matrices of the same order, write the condition on A
such that AB - AC => B=C.

- 1
' If A is a non-singular square matrix such that A -2 -1 '

2'
If adj A = ‘^ ^ and adj 6 = _ ^ j , find adj AB.
9. If A is a symmetric matrix, write whether A ^ is symmetric or skew-symmetric.
INVERSE AND APPLICATIONS OF A MATRICES 9.69

10. If A is a square matrix of order 3 such that \ A\=2, then write the value of adj (adj A).
’ . If A is a square matrix of order 3 such that | A | = 3, then find the value of | adj (adj A) |.
12. If A is a square matrix of order 3 such that adj (2A) = k adj (A), then write the value of k.
T T
13. If A is a square matrix, then write the matrix adj (A )-(adj A) .

' Let Abe a 3 x 3 square matrix such that A (adj A) = 2 / , where / is the identity matrix. Write
the value of | adj A |.
If A is a non-singular symmetric matrix, write whether A~^ is symmetric or
skew-symmetric,

w
cos 0 sin 6 k 0
16. IfA = and A (adj A) = , then find the value of k.
- sin 0 cos 0 0 k

-1

lo
17. If A is an invertible matrix such that \A = 2, find the value of | A|.

e
5 0 0^

re
If A is a square matrix such that A (adj A) = 0 5 0 , then write the value of |adj A|.
rF
F
0 0 5

'2 3’ -1

r
= k A, then find the value of k.
19. IfA = g be such that A fo
ou
ks
''●V Let A be a square matrix such that A^ - A + / = O, then write A" ^ in terms of A.
'2 -3 5'
oo

. If Qy is the cofactor of the elementary of the matrix A = 6 0 4 , then write the value of
Y
eB

1 5 -7

'’SZ C 32-
ur

'3-2'
ad

Find the inverse of the matrix


-7 5 ●
Yo

cos 0 sin 0
Find the inverse of the matrix
d

sin 0 cos 0 ■
Re
in

1 - 3 a b 1 0
24. IfA= 2 Q / Write adj A. If A = / B = , find adj (AB).
F

c li 0 1

3 11
26. TfA = 2 _3 ' then find I adj A|.
3 I ..25
27. IfA = ? ^ in terms of A. Write A
- 1
for A = .
L5 - 2 , write A 1 3

7 ’. Use elementary column operation C2 ->C2 + 2Ci in the following matrix equation :
2 1 3 1 1 0
2 0 2 0 -11

30. In the following matrix equation use elementary operation R2 ^ R2+ arid the equation
thus obtained:
9.70 APPLIED MATHEMATICS-XII

‘2 3 1 0 8 -3
1 4 2 -1 9 -4

31. Ifis a square matrix with| A\=4 then find the value of| /I. (adj A) |.
ANSWERS

-2 -4 1
1. 2. 5 3. 25 4. ±8
-7 -3 10

'5 -2' -6 5
6. A must be invertible or| A15* 0 3 -1
8.
-2 -10 \ symmetric

10. 2A 11. 81 4 1.3. Null matrix 14. 4


1 1
I-.’, symmetric 16. 1 17. 18. 25 19. —
2 19

w
5 2 cos 0 sin 9
20. A ’ =(/-A) 21. no T>
7 3 sin 0 cos 0

F lo
■ 0 3' d -h
-11 27 . A'^=—A
-2 1 - c a
19

ee
-1 3 -5 '2 5 3 1 1 2 ‘2 31 ri 0 8 -3
28. A

Fr
30
-1 2 2 4 2 0 -1-1 3 7 2 -1

31. 4^<^) for


ur
s
ook
Yo
eB
r
ad
ou
Y
Re
nd
Fi
CHAPTER ID
INCREASING AND
DECREASING FUNCTIONS

10.1 INTRODUCTION

In this chapter, we shall study monotonicity of functions. A function / (x) is said to be a mono-
tonically increasing function on [a, b], if the values of / (.r) increase or decrease with the increase

w
or decrease in .v. If the values of / ex') decrease with the increase in the values of x, then / (a:) is
said to be a monotonically decreasing function. The monotonicity of functions in [a, b] is
strongly connected to the sign of its derivative in [a, I>]. The relation between the two will be

Flo
discussed in section 10.4. In determining the intervals of monotonicity of a function in its
domain, we shall be solving the inequations/' (a) > Oand/' (x) < 0. So, we shall first discuss the

ee
procedure of solving inequations in the following section.

Fr
10.2 SOLUTION OF RATIONAL ALGEBRAIC INEQUATIONS

The following results are very useful in solving rational algebraic inequations:
for
ur
(i) ab >0 => (fl > 0 and & > 0) or (a < 0 and b <0)
(ii) ab <0 => (fl > 0 and b <0) or (a < 0 and b > 0)
(iii) ab >0 and « > 0 => b >0
ks

(iv) rtl) < 0 and ^ < 0 => ^ > 0


Yo
oo

If P(a:) and Q (.r) are polynomials, then the inequations > 0 and
eB

Q(x) Q(x) Q(x)


P(x)
< 0 are known as rational algebraic inequations. These inequations can be solved by using
QW
r
ou
ad

the following algorithm.


Y

ALGORITHM

STEP I Factorize P (x) and Q (x) into linear factors.


nd
Re

STEP II Make coefifcient of x positive in all factors.


STEP 111 Equate all the factors to zero and ifnd the corresponding values ofx. These values are generally
Fi

known os critical points.


STEP IV Plot the critical points on the number line. Note that n critical points will divide the number line
in {n + 1) regions.
STEF V In the right most region, the expression will be positive and in other regions it will be
alternatively negative and positive. So, mark positive sign in the right most region and than
mark alternatively negative and positive signs in the remaining regions.
STEP VI Obtain the solution set of the given inequation by selecting the appropriate regions in step V.
Following illustrations will illustrate the above algorithm.
ILLUSTRATION 1 Solve: 4x^ - 24 x^ + 44x - 24 > 0.
SOLUTION We have,
4x^ - 24x^ + 44x - 24 > 0
=> 4(x^ -6x^ +11 x-6) >0
10.2 APPLIED MATHEMATICS-XII

Y ^-6-Y^+11x-6>0 [●.* 4 > 0 and > 0, rt > 0 => > 0]


(.y-1)(.v^-5.v + 6)>0
(y-1)(.v-2)(y-3)>0 -(i)
On equating all factors, on LHS of the inequation, to zero, we obtains =1, 2, 3as critical points.
Let us plot these critical points on the number line as shown in Fig. 10.1. These points divide the
number line into four regions. In the right most region the expression on LHS of (i) bears
positive sign and then alternatively negative and positive signs as marked in Fig. 10.1. Since the
expression in (i) is positive. Therefore, solution set of inequation (i) is the union of the regions
marked with + signs. Hence from Fig. 10.1, we obtain (1, 2) u(3, co) as the solution set.
^ -6x^ +11.Y-6 > 0 => (y-1) (y-2) (.y-3) >0 => y e(l, 2) u(3, co).

w
i.e. Y

Hence, the solution set of the given inequality is (1, 2) u (3, co).
+ +

o
- OC
1 2 3 oo

e
Fig. 10.1 Signs of(x- 1)(y-2)(.v- 3) for different values of y

re
1 4

rFl
F
ILLUSTRATION 2 Solve :
Y + 1 —^>0, x^-1, 2.
(2 + y)
SOLUTION We have.

r
ou
1 4 (2 + y)^-4(y + 1) fo
Y
2
ks
Y + 1 (2 + .xT (2 + x)2(a' + 1) (2 + xf (x + 1)
1
oo

-V + 1 (2 + y)
Y
eB

y2
>0
(2 + .y)^(y + 1)
r

2/
1
ou

Y
>0
Y
ad

2 + y / \
Y + 1

n2
1 Y
d

> 0 and Y 0 > 0 and if (7 > 0, then ab >0-^ b >0


Y + 1 2 + y
Re
in
F

Y + 1 >0 and x^O - >0 and a>0,=> b >0


b

Y > -1 and X =^0

Ye{-l,0)u(0, oo)
Hence, the solution set of the given inequality is (-1, 0) u (0, co).
l-Y^
ILLUSTRATION 3 Solve:
5y - 6 - y
2<0
SOLUTION We have,
1-.y2
5y - 6 - Y
2<0 + + +

- CO
-1 1 2 3 CO

o <0 (Y-1)(X+1)
-(.Y^ -5y + 6) Fig. 10.2 Signs of
(x-2)(x-3)
for different values of .r
INCREASING AND DECREASING FUNCTIONS 10.3

;c2-l
o <0
-5.T + 6

(.V-1) (.V + 1)

rw
<=> <0
(x-2) {x-3)

Equating all the factors to zero, we obtain .v = 1, -1, 2, 3 as the critical points.
Now, we plot these points on the number line as show in Fig. 10.2. These points divide the
number line into 5 regions. In the right most region the expression in (i) bears'+' sign and in the
other regions the expression bears alternate negative and positive signs as shown in Fig. 10.2.
Since the expression in (i) is negative, so solution set of the given inequation is the union of

r
luo
regions containing negative signs. Hence, from Fig. 10.2, we get e (-1,1) u (2, 3)
l-.v'

F
i.e.
5x - 6 - X
^<0 => :ce(-l,l)u{2, 3)

oF
8.r^ + 16.V -51
ILLUSTRATION 4 Solve: > 3.

rs
2:r^ +5a:-12

ok
SOLUTION We have,
+ + +

8a:^ +16.1' -51 3 5


> 3 - CO
-4 -3 oo

fo
2.1^ +5.1--12 2 2
(.Y+ 3) (2a--5) for different values of x
8.1-^ +163:-51 Fig. 10.3 Signs of
o (A+ 4)(2a- 3)
Y
-3>0
Y
2.i'^+5t-12
rB

8.1'^ + 16.1' -51 -6:r^ -15.1 + 36


>0
2.1^+51-12
2.1-^+1-15
ue

>0
d

<=>
2.1^ +51-12
no

2i^ +6i-51-15
ad

<i5> >0
2.y^ + 8i-3i-12
(1+ 3) (2x-5)
i

<=> >0 ...(i)


(1 + 4) (2i-3)
Re
F

Equating all factors to zero, we obtain i = - 4, - 3, 3/2,5/2


Now, we plot these points on the number line as shown in Fig. 10.3. These points divide the
number line into five regions. In the right most region the expression in (i) bears positive sign
and in all other regions it bears alternate negative and positive signs as shown in Fig. 10.3.
Since the expression in (i) is positive, so the solution set of the given inequation is the union of
regions containing'+'signs. Hence, from Fig 10.3, we get x €(—co, -4) u(- 3, 3/2) u(5/2, co).
8.1^ + 161-51
i.e. >3 => 1 € (- 00, - 4) u (- 3, 3/2) u (5/2, oo).
2.1^ +51-12
1^ - 2i + 5 1
ILLUSTRATION Solve:
SOLUTION We have,
3i^ -2i-5 ^2'
1^ - 2i + 5 1
> —

3i^ -2i-5 ’ 2
10.4 APPLIED MATHEMATICS-XII

~lx+ 5 1
<£=> ->0
3a:^-2.y-5 2
2 - 2x + 5) -(3.t^ - 2x -5) >0
2(3.t^ -2x-5)
- - 2x +15 + + +
<=> >0
2(3x^ -2x-5)
- oo
-5 -1 5 3 00

-(.v^ +2.r-15) >0


(x+5)(y-3)
<=>
Fig. 10.4 Signs of for different vaiues of x
2{3x^-lx-5) {.v+l)(3x-5)

ow
+ 2x-15 V P
<=> <0 >0 => -i-<0
2(3x^ -2x-5) (1

x^ + 2X-15 1
o >0 > 0
3x^ -2x-5 2

e
re
o
(x + 5) (x - 3)
(x + l)(3x-5)

Fl
<0

F
On equating all factors to zero, we get x = - 5, -1,5/3, 3. Plotting these points on number line
ur
and marking alternatively '+' and signs, we obtain as shown in Fig. 10.4.

r
fo
Since the expression in (i) is negative, so the solution set of the given inequation is the union of
regions marked withsigns. Hence, from Fig. 10.4, we get x e (- 5, -1) u (5/ 3, 3).
ks
x^ - 2x + 5 1
Yo
i.e. > - ^ xe(-5,-l)u(5/3, 3).
oo

3x^ -2x-5
x^ -2x+ 24
eB

ILLUSTRATION 6 Solve: <4.

SOLUTION We have,
X ^ - 3x + 4
ur

x^ - 2x + 24
<4
ad

+
x^ - 3x + 4
+
Yo

- X
2 4 00

x^ - 2x + 24 3
<=> -4<0
d

Fig. 10.5 Signs of (3.r + 2)(x- 4)for different values of .v.


x^ - 3x + 4
Re
in

(x^ - 2x + 24) - 4 (x^ - 3x + 4) <0


F

o
X 2-3X + 4
- 3.y^ + lOx + 8
<0
x^ - 3x + 4
3.x2-10x-8 .. L -P
<=> >0 <0 => >0
x^ - 3x + 4

<=>
(3x + 2)(x-4) >0
(x^ -3x + 4)
Disc, of X - 3x + 4 is - ve and coetT. of x is + ve
(3x + 2) (x-4)>0
.●. x^ - 3x + 4 > 0 for all x
2
<=> X < — or X > 4 [See Fig. 10.5]
3
10.5
INCREASING AND DECREASING FUNCTIONS

a: e(-oo, - 2/3] u [4, co)


.Y^ - 2a: + 24
Thus, <4 => .Y 6 (-00,-2/3 u [4, go).
- 3y + 4

ILLUSTRATION 7 Solve: — ^-4.v+7 ^2


.Y^
.2
-7y + 12 ”3
SOLUTION We have,
+
■Y^ - 4.Y + 7 < . —2
+ +

- 30 -3 1 3 4 00

,v^ -7.V + 12 ~ 3
jx+ 3)(.y-1) lor different values of .r
Fig. 10.6 Signs of
- 4y + 7 (.v-3)(a--4)
-<0
.t^-7.y + 12 3
3 (Y^ - 4y + 7) -2(y^ -7.Y + 12) <0
Y^ -7y + 12

w
Y^ + 2y - 3 (x+3) {x-1)
<=> <0 o <0

F lo
Y^ -7y + 12 (y-3)(y-4)

On equating all factors in (i) to zero, we get y = - 3,1, 3, 4 as critical points. Plotting these points
the number line and marking alternatively '+' and signs from the right most side, we

ee
on

obtain that the inequation in (i) has the signs as shown in Fig. 10.6.

Fr
Since the expression in (i) is negative, so the solution set of the given in, equation is the union of
the regions marked with signs. Hence, from Fig 10.6, we get x e [- 3,1] u (3, 4).
for
Y ^ -4y + 7
<- => Y€[-3,l]e(3,4).
ur
i.e.
Y^ -7y + 12 3
s

It should be noted that 3 and 4 are not included, because denominator becomes zero at y = 3 and
ook
Yo

y = 4.
eB

10.3 SOME DEFINITIONS

STRICTLY INCREASING FUNCTION Afunction f{x)is said tobeastrictl}/increasingfunctionon(a,b), if


x-[<X2 => f{x-i)<f{x2) foraUxj,X2eia,b)
our
ad

Thus, fix) is strictly increasing on {a, b) if the values of f(x) increase with the increase in the
values of y.

Graphically,/(y) is increasing on (rt, b) if the graph y = /( y) moves up as y moves to the right. The
Y
Re

graph in Fig. 10.7 is the graph of a strictly increasing function on (a, b).
nd

Y
Fi

/(^2)
M)

o a
Y, Y2 f? X

Fig. 10.7 Graph of an increasing function


10.6 APPLIED MATHEMATICS-XII

ILLUSTR.\TIO\ 1
Show that the function f{x) = 2x + 3 is strictly increasing function on R.
SOLUTION LetAT|,A'2 e i? and let .tj < a.^. Then,
,ri < X2 => 2xi < 2x2 2.Vj + 3 < 2x2 + 3 => f{x{) <f{X2)
Thus, Xi < X2 ^ fi^i) < f(x2) for all x-^, .V2 e R. So,/(.y) is strictly increasing function on R.
This result is also evident from the graph of the function shown in Fig. 10.8.
Y

ow
"3
X

e
re
■Cn
X' V: X

Frl
0 ATl X2

F
ou
r
so
r kf
Fig. 16.8 Graph of f(x) = 2x+ 3
ILLUSTRATION 2
Show that the function f{x) = x^ is strictly mcreasing function on [0,00).
oo

SOLUTION Let x^,X2 e [0, co) such that < .t2. Then,
Y

2
B

Xi < X2 => Yj < .Y-J ,Y2 [Multiplying both sides by y^]


2
again, .Vj < Y2 => .y^ Y2 < .Y2 [Multiplying both sides by .Y2I -(ii)
re
oY

From (i) and (ii), we get


u

.Yj < Y2 -^1 ^ 2^ => /(-^l) </('-^'2)


ad

Thus, Yi<Y2 => /(yi)</(.Y2) forall.Yi,.Y2 e[0, 00).


d

Hence,/(y) is strictly increasing function on [0, co) which is evident from the graph also.
in
Re

/(.v)=y2
F

/(^2)

o /(^l)
X' ^2 X
Y'

Fig. 16.9 Graph of/(A) = Y^,Y>0


II.I.USTKATION3
Show that the function f(x) =a^, a >1 is strictly increasing on R.
SOLUTION Let Y^, 'Y2 e R such that x^ < Xj- Then,
Yi <.Y2
10.7
INCREASING AND DECREASING FUNCTIONS

=> a < a i'2 [ ●.● rt > 1 -V] < X2 => ]


f{xi) </(-V2)
Thus, Xi < X2 => /(a'i) < f{x2) for all x-i, X2 e R.
Hence, is strictly increasing function on R. This fact is also exhibited in the graph of this
function as shown in Fig. 10.10.
y

fix) = a^>a > 1

A^x)
X’ o X

w
F lo r

Fig. lO.IOGraphof f{x) = a^,a>\

e
REMARK In the above example if lue replace a by e (=^2.71), then zve find that f{x) =e is also

Fre
increasing on R. for
STRICTLY DECREASING FUNCTION A fiiiiction f{x) is said to be a strictly decreasing function on
{a, b), if
r
X^<X2 => f{x\)>f{x2) forall x-[, X2 ^{a,b)
You

Thus, f{x) is strictly decreasing on (a, b) if the values of f{x) decrease with the increase in the
oks

values of x.
eBo

Graphically it means that f{x) is a decreasing function on {a, b) if its graph moves down as 'A
moves to the right. The graph in Fig. 10.11 is the graph of a strictly decreasing function.
y
our
ad

A^\)
dY
Re
Fin

X'
a 0 ^2 b X

iY
Fig. 10.11 Graph of a decreasing function

ILLUSTRATION 4 Slioiu that the function f{x) = - 3.r +12 is strictly decreasingfunction on R.
SOLUTION Let x■^ ,X2eR be such that .\'i < X2. Then,
Ai < -T2 => - 3a;i > - 3^2 => - 3.\'i + 12 > - 3a'2 + 12 => f{x^) >f{x2)
Thus, ●H <^2 /W >/(^2) ^'2 ^
So, /(a) is strictly decreasing function on R.
10.8 APPLIED MATHEMATICS-XII

This fact can also be observed from the graph of the function as shown in Fig. 10.12

w
Fig.10.12 Graphof/(.r) = -3.t+12

F lo
ILLUSTRATION 5
Show that the function f{x) ,0 <a < \ is strictly decrensing on R.
SOLUTION Let .Tj, X2 e R such that a'j < Then,
Ai<A2

ee
=> >a ^'2

Fr
a
[ ●.● 0 < < 1 .-. .Vj < .t2 => ]
/(ai) >/(.A2)
Thus,Ai < A2 => /(X]) >/(a'2) for all Xj, X2, £ R.
for
ur
Hence,/(x) is strictly decreasing fimction on R. This is also evident from the craph of f(x) as
shown in Fig. 10.13.
s

Y
ook
Yo
eB

fix) = «*/ 0 : rt < 1


our
ad

A>^2)
Y

X' o ^2 X
Re
nd
Fi

tv
Fig.10.13 Graphof/(i) = fl*,0<fl<l
REMARK Since0 <e ^ =
- <1, f/;t’re/ore/(x) =e~'^ isalsoastrictlydecreasingfiinctionon R.
ILLUSTRATION 6
Show that the function /(x) =x^ is a strictly decreasing function o«(-od, 0].
SOLUTION LetXj,X2 e(-x,0] besuchthatXi <x'9.Then,
2
Xj < X'2 => x^ > X| X2 ...(i)
and.
X| <X2 => Xj X2 >X^^ ...(ii)
From (i) and (ii), we obtain
Xi<X2 => Xj > X^ => f{x-])>f{X2)
Thus, ^1<-V2 => /U'l) >/(x2) forall Xi, X2 e(-x, 0].
Hence, /(x) is strictly decreasing on (- x, 0]. See also Fig. 10.14
INCREASING AND DECREASING FUNCTIONS 10.9

f(x)=x^

M)

X' ●^1 ^2 o X

w
Fig. 10.14 Graph of /(a-) = <0

Uptill now, we have been discussing about a strictly increasing or strictly decreasing functions.

Flo
But, it is possible that a function may neither be strictly increasing nor strictly decreasing on a
given interval. For example,/(.●r) in Fig. 10.15 is neither strictly increasing nor strictly decreasing

ee
on(fl, ^). However, it is increasing on the sub-intervals {a, {aj, ^3) and(fl4, b) and decreasing

Fr
on the intervals ^2) ^^id (<73, <74).
Y

for
ur
ks
Yo
oo

X' X
eB

a (1,0 ^7 «3 b
1
r
ou
ad

y.

Fig.10.15
Y

ILLUSTRATION 7 Show that the function f{x) = is neither strict!}/ increasing nor strictly decreasing
on R.
nd
Re

SOLUTION In illustrations 3 and 6 we have seen that/(.v) = .v^ is strictly increasing on [0, 00) and
Fi

strictly decreasing on (~ qo, 0]. Hence, it is neither strictly increasing nor strictly decreasing on R
i.e. (-03, co).
y

o a
«2 b X

Fig. 10.16
10.10 APPLIED MATHEMATICS-XII

Uptill now we were talking about strictly increasing and strictly decreasing functions. But, there
can be functions which are increasing (decreasing) but not strictly increasing (decreasing). For
example, consider the function whose graph is shown in Fig. 10.16. Clearly,/(x') is increasing on
{a, b) but it is strictly increasing only in the intervals (a, a{) and (^?2 / ^)- In this chapter, we shall be
studying only strictly increasing and strictly decreasing function.
NOTi: 'Prom now omvards, by an increasing or a decreasingfunctioti we shall mean a strictly increasing
or a strictly decreasing function.
MONOTONIC FUNCTION A function f{x) is said to be monotonic on an interval (a, b) if it is either
increasing or decreasing on (a, b).
DEFINITION A function f{x) is said to be increasing (decreasing) at a point .Yq if there is an interval
(.Vq - h, A'o + h) containing xg such that f(x) is increasing (decreasing) on (.Vg - h, Xg + h).

w
DEFINITION A function f{x) is said to be increasing oti [a, b] if it is increasing (decreasing) on (a, b) and
it is also increasing (decreasing) at x = a and x = b.

F lo
EXERCISE 10.1

1. Prove that the function/(y) = log^, x is increasing on(0, co).

ee
Prove that the function f{x) = log^ x is increasing on (0, c») if a > 1 and decreasing on (0, co),

Fr
if 0 < rt < 1.

Prove that f{x) = ax + b, where a, b are constants and « > 0 is an increasing function on R.
for
4. Prove that f{x) =ax + b, where a, b are constants and rt < 0 is a decreasing function on R.
ur
Show that f{x) = — is a decreasing function on (0, x).
s

X
ook
Yo

Show that f(x) = decreases in the interval [0, x) and increases in the interval (~ X, 0].
l + j2
eB

1
7. Show that f(x) = is neither increasing nor decreasing on R.
l + .v2
our
ad

8. Without using the derivative, show that the function f (x) = | -v | is


(a) strictly increasing in (0, x) (b) strictly decreasing in (-x, 0).
Y

9. Without using the derivative show that the function / (.v) = 7.t - 3 is strictly increasing
Re
nd

function on R.
Fi

HINTS TO SELECTED PROBLEMS

I. For any Xj, .V2 € (0, x), we have


A'l < ^2 => log^, X^ < log,. T2 => /(.Ti) </(X2).
So, / (.y) is increasing on (0, x).
2. CA^; I Whenrt>l: For any .Vj, X2 e(0, x)
X'l > .Y2 => log^ -v^ > log« -Y2 ^ /(.Yi) >/(X2).
So,/(x) is increasing on(0, x).
(. ASn n When a <1: For any .y-j, .Y2 € (0, x)
A'l > -^2 ^ '^1 < -'2 => /(^l) </(-t2)-
So, fix) is decreasing on (0, x)
INCREASING AND DECREASING FUNCTIONS 10.11

10.4 NECESSARY AND SUFFICIENT CONDITIONS FOR MONOTONICITY


In this section, we intend to see how we can use derivative of a function to determine where it is
increasing and where it is decreasing.
THEOREM 1 (Necessary Condition) Let f (x) be continuous on [a, b]and differentiable on (a,b).
(i) If f (x) is strictly increasing on (a, b),tbcnf'(x) >Oforallxs{a, b).
(ii) Iff (a:) is strictly decreasing on (a, b), tlienf (a:) < 0for all x s {a, b).
mtooi Let .Y be an arbitrary point in (a, b). Since f{x) is differentiable on (a, b). So, it is
differentiable at x.
f{x + h)-f{x)
/' (y) = lim ,h>Q exists.
h^O h

(i) If / (y) is strictly increasing on {a, b), th m


/ (y + h) > f (y) for all /i > 0
/(y + /Q-/(y) > 0 for all /j > 0

w
h

f(x + h)-f{x)

F lo
Urn >0
/i->0 h

/'(.Y)>0.

e
Since y is an arbitrary point of (r7, t). Therefore, /' (.;) > 0 for all y e {a,b).

Fre
(ii) If / (.y) is strictly decreasing on {a, b), then for
/(y + /i)</(y) for all//>0
f(x + h)-f(x) < 0 for all // > 0
r
=>
h
You
oks

lim
f{x + h)-f{x) <0
eBo

/'(Y)<0
Since y is an arbitrary point of (fl, b). Therefore, /' (.y) < 0 for all y e [a, b).
ad
our

Q.E.n.

UFMARK Iff (y) is an increasing function on (a, b), then as shown in Fig. 10.17, the tangent at every
point on the curve y =f (y) makes an acute angle 0 with the positive direction of x-axis.
dY
Re
Fin
10.12 APPLIED MATHEMATICS-Xll

// / (.y) is a decreasing function on {a, b), then as shoiun in Fig. 10.18, the tangent at every point on the
curve y=f (.y) makes an obtuse angle 0 luith x-axis.

tan 0 < 0 <0 or, /'(a:)<0 for all x&{a,b).


dx

THEOREM 2 (Sufficient Condition) Let f be a differentiable real function defined on an open interval
(a, b).
(i) If f' (.y) > 0/or all x e (a, b), then f{x) is increasing on (a, b).
(ii) If f' ( y) < 0 for all x e {a, b), then /( y) is decreasing on (a, b).
PKOOI Let X1^, Y2 ^(a,b) such that Y^ <.y'2. Consider the sub-interval [yj, Y2] ● Since f{x) is
differentiable on [a,b) and [x^, X2].c(a,b). Therefore, /(y) is continuous on [yj,Y2] and

w
differentiable on (y^, Y2). By the Lagrange's mean value theorem, there exists c 6 (yj, Y2) such
that

/(Y2)-/(Yi)

F lo
f'(c) =
Y2-Y1

ee
(i) Since /' (y) > 0 for all y 6 {a, b), so in particular, /' (c) > 0.

Fr
Now, /'(c)>0
/(Y2) -/(y^) >0 for [Using (i)]
Y2-Y1
ur
/(Y2) -/(Yj) >0 [●.● Y2 - Y-j > 0 when Y| < Y2]
s

/(Yj) >/(Yi) => /(Yj) </(Y2)


ook
Yo

Since Yj, Y2 are arbitrary points in {a, b). Therefore,


eB

Yi<Y2 => /(Yj) </(Y2) for allYi, Y2 e(<7,/7)


Hence, /(y) is increasing on (a, b).
r
ad
ou

(ii) Since / '(y) < 0 for all y e (a, b), so in particular, / '(c) < 0.
Now, /'(c) < 0
Y

/(^2)~M) < 0 [Using (i)]


Re
nd

Y2-Y1
Fi

=>
[●.● Y2 - Y| > 0 when y^ < Y2]
/ (-V2) < /(-Vl) => /(Yi) > /(Y2)
Since y^, Y2 are arbitrary points in {a, b). Therefore,
^ /(yi)>/(y2) forallYi,Y2 e(fl,Zi).
Hence,/(y) is decreasing on (a, b).
COROLLARY Let f{x) be a function defined on (a, b).
(i) If fix) > 0 for all x & {a, b) except for a finite number of points, where f'{x) =0, then f{x) is
increasing on ({a, b).
(ii) If f '(y) < 0 for all x e (a, b) except for a finite number of points, where f '(y) = 0, then f{x) is
decreasing on (a, b).
In order to find the interval in which a given function is increasing or decreasing, we may use
the following algorithm.
INCREASING AND DECREASING FUNCTIONS 10.13

ALGORITHM
STEP 1 Obtain the function and put it equal tof{x).
STEP 11 Findfix).
STEP in Put f '(x) > 0 and solve this inequation.
For the values ofx obtained in step III f{x) is increasing and for the remaining points in its
domain it is decreasing.
ILLUSTRATIVE EXAMPLES

Typ)e I ON FINDING THE INTERVALS IN WHICH A FUNCTION IS INCREASING OR DECREASING


EXAMPLE 1 Find the intervals in which f{x) = - - 2x +15 is increasing or decreasing.
SOLUTION We have,
f{x)=-x^-lx + 15
=> fix) = -2x-2 = -2(x + l)
For/(x) to be increasing, we must have

w
/'(x)>0
=> -2(x + l)>0
=>
=>
X + 1 <0
x<-l^xe(-oo,-1)
Thus, /(x) is increasing on the interval (- co, ~ 1).
For/(x) to be decreasing, we must have

=>
/'(x)<0
-2(x + l)<0
F lo [●.● -2 < 0 andab>0,a<0 => 6 <0]

for F
ree
=> x + 1 >0 [●.' - 2 < 0 and <7b < 0, fl < 0 => b >0]
=> X > -1 => X e (-1, Qo)
Your

So, fix) is decreasing on (-1, co).


ks

EXAMPLE 2 Find the intervals in which the function f(x) = 2x^ - 9x^ + 12x +15 is (i) increasing,
eBoo

(ii) decreasing:
SOLUTION We have,
fix) = 2x^ -9x^ + 12x +15
ad
our

=> fix) = 6x^-18x + 12 = 6(x^-3x+2)


(i) For/(.x) to be increasing, we must have
Re

/'(x)>0
Y

6 (x^ - 3x + 2) > 0
Find

=> x^ - 3x + 2 > 0 [v 6>0 6(x^ 3x + 2) > 0 => x^ - 3x + 2 > 0]


=> (x-l)(x-2)>0
x<lorx>2 [See Fig. 10.19]
=> -X € (- CO, 1) u (2, oo).
So, fix) is increasing on (- <», 1) u (2, oo).
- CO 1 2 CO

Fig. 10.19 Signs of/'(x) for different values of X

(ii) For/(x) to be decreasing, we must have


/'(.x)<0
=> 6 (x^ - 3x + 2) < 0
=> x^ - 3x + 2 < 0 [●.● 6 > 0 6(x^ - 3x + 2) < 0 => x^ - 3x + 2 < 0]
10.14 APPLIED MATHEMATICS-XII

(.v-1) (.r-2) <0


1 < -V < 2 => xe(l,2) [See Fig. 10.20]
So,/(a:) is decreasing on (1,2).
- 00
1 2 CO

Fig. 10.20 Signs of/'(.v) for different values of

liXAMl’Ll; .1 Fwd the intervals in which the function f{x) = 2x^ + 9x^ + 12x + 20 is (i) increasing:

ow
(ii) decreasing:
SOLUTION We have,
/(.y)=2.v^ + 9-.c^+12.y+20.
/ '{.y) = 6x^ +18x + 12= 6 {x^ + 3x + 2)

e
re
(i) For fix) to be increasing, we must have
/'(.v)>0

Flr
6 (.Y^ + 3.Y + 2) > 0

F
(.Y^ + 3y + 2) > 0 [■.■ 6 > 0 and 6 (y^ + 3y + 2) > 0 .-.y^ + 3y + 2 > 0]
{y + ])(.y + 2)>0
ou [See Fig. 10.21]

sr
Y<-2orY>-l
+

ko
Y e(-oo, -2) u(-l, Qo) - oo
of -2 -1 CO

So,/(y) is increasing on(-x, -2) u(-l, x) Fig. 10.21 Signs of/'(Y) for different values of Y
(ii) For/(Y) to be decreasing, we must have
o
/'(Y)<0
Y

6(y^ + 3y + 2) < 0
reB

Y^ + 3y + 2 < 0 [■■■ 6 > 0 and 6 (y^ + 3y + 2) < 0 .'. y^ + 3y + 2 < 0]


uY

(y + 1) (y + 2) < 0 [See Fig. 10.22]


-2<y<-1
ad
do

- ®
-2 -1 CO

So, fix) is decreasing on (- 2, -1). Fig. 10.22 Signs of/'(y) for different values of y
in

l:XAMPU-;4 Find the intervals in which fix) =(y + 1)^ (y - 3)"^ is increasing or decreasing.
Re

SOLUTION We have,
F

/(y)=(y + 1)3(y-3)3

fix) = |3{y + 1)2 A(.i: + l)j(Y-3)3+(Y + l)3


fix) = 3(y + 1)2(.y-3)^+3(y + 1)^(y-3)^
fix) = 3 (y + 1)^ (y - 3)^ (y + 1 + Y - 3)
fix) = 6 (x + 1)^ (y - 3)^ (y-1)
For fix) to be increasing, we must have
/'(.Y)>0
6(y + 1)2 (y-3)^(y-1)>0
Y -1 > 0 and Y -1, 3 [v 6(y + 1)^(y-3)^ >0forallY;t-l, 3]
=>
X > 1 and X ^ —1, 3 ^ y e(l, 3) w(3, x)
So, fix) is increasing on (1, 3) u (3, x).
For fix) to be decreasing, we must have
10.15
INCREASING AND DECREASING FUNCTIONS

/'(.r)<0
^ 6(a- + 1)^(:v-3)^(a--1)<0
=> A -1 < 0 and -1, 3 [v 6 (a +1)^ (a - 3)^ >0forall A7i-1, 3]
=> A < 1 and A -1, 3 => a e (-co,-1) u (-1,1)
So, /(a) is decreasing on(-oo, -1) u(-l, 1).
EXAMlM.ns Find the intervals in zvhich /(a) = (a -1) ^ (a - if is increasing or decreasing.
SOLUTION We have,
f{x)={x-lf{x-2f

fix) = 3(a-1)^|^(a-1)| (a-2)2+(a-1)^2x(a-2)|^(a-2)


fix) = 3(a-1)^(a-2)^+2(a-1)^(a-2)

w
=>

fix) = (a-1)^(a-2)(3a-6 + 2a-2) = (a-1)^ (a - 2) (5a - 8)

Flo
For/(A) to be increasing, we must have
f'ix)>0
=> (a-1)^(a-2)(5a-8)>0

ee
[●.● (a -1)^ > 0 for all A 9^ 1]

Fr
=> (a - 2) (5a - 8) > 0 and a ^ 1
=> 5{a-8/5)(a-2) >0 and a^I
=> (a - 8/5) (a - 2) > 0 and x*l for [V 5>0]
ur
=> A <8/5 or A>2 and x*\
^ A e(-co, 1) u{l, 8/5) u(2, <») [See Fig. 10.23]
k s
Yo

00
- CO 1
oo

Fig. 10.23 Signs of/'(x) for different values of a


eB

So,/(a) is increasing on (- co, 1) u (1, 8/5) u(2, co).


For/(.a) to be decreasing, we must have
r

/'(A)<0
ou
ad

{a-1)^{a-2)(5a-8)<0
[v (A-1)^ >0 for all A 9^1]
Y

=> (a - 2) (5a - 8) < 0 and a 9^ 1


5(a - 2) (a - 8/5) < 0 and a 1
Re
nd

^ (a - 8/5) (a - 2) < 0 and a 5* 1 [V 5>0]


Fi

=> Ae(8/5,2) and a 9^ 1 Ae(8/5, 2) [See Fig. 10.24]


CO
- CO 2
5

Fig. 10.24 Signs of/'(a) for different values of a


So,/(a) is decreasing on (8/5, 2). a3
RXAMIM.C f> Find the intervals in which the function /(a) = a“^ — is increasing or decrensmg.
SOLUTION We have.
3

fix) = 4a^-a^ = .a^(4a-1)


For fix) to be increasing, we must have
/'(.A)>0
10.16 APPLIED MATHEMATICS-XII

(4.v-l)>0
4a -1 > 0 and a 9^ 0 [v a-2>0]
4a >1 and a 9^ 0 => a > ^ ag(1/4, oo)
So, /(a) is increasing on (1 /4, co).
For /(a) to be decreasing, we must have
/'(A)<0
A^ (4a-1)>0
4a -1 < 0 and A 9^ 0
[●.● A^ > 0 for all A ^ 0]
4a<1 and A9^0 => a<1/4 and x^O => a e(-oo, 0) u (0,1/4)

ow
So, /(a) is decreasing on (-CO, 0)u (0,1/4).
2a . .
nXAMI’LET
Find the intervals in lohich the function /(A)=logp(l + a) - 2 + x
IS increasing or
decreasing.

e
Fl
lx

re
SOLUTION We have, /(a) = log^, (1 + a) - . Clearly, / (a) is defined for all a satisfying
2 +A

F
A + 1 > 0 i.e. A > -1. So, domain (/)=(-1 , oo).
ur
Now,

r
2a fo
/ (a) = log^ (1 + A) -
2 + a
ks
1
^(■Tll) + + 1 4
Yo
/'(-V) =
1 + x dx
(2 + .v)2
oo

(2 + a)^
(2 + a)^ - 4 (1 + A) r X f
B

a2 1
/'(X) =
(2 + a)2 (1 + A) (2 + a)2 (1 + A) 2 +A
re

A + 1

For /(a) to be increasing, we must have


u
ad

/'(x)>0
Yo

x2
A 1
>0
d

^2 + a A + 1
Re

^2
in

1 A
> 0 and A 9i 0 > 0 for all A 9i 0
A + 1
F

\
2 + a
A + 1 > 0 and A 9i 0

A >-l and A 9i^ 0

AS(-1,0)U(0, oo)
So, /(a) is increasing on (-1, 0) u (0, co).
4a^ +1 . .
EXAMPLES
Find the intervals in zvhichf{x) = is increasing or decreasing.
4a^ +1
SOLUTION We have, /(a) =
A

4a^ -1
Now, /(a) = 4x + -^
X
f'(x) = 4-T
r2 2
A

For /(a) to be increasing, we must have


/'(x)>0
10.17
INCREASING AND DECREASING FUNCTIONS

4a-^- 00
->0 - cc
-1/2 1/2
Fig. 10.25 Signsof/'(-T)fordifferentvaluesof a:
4,-c^ -1 >0
(2.v-1)(2.Y4-1)>0 [See Fig. 10.25]
(y-1/2) (x + 1/2)>0
x<-l/2 or, x>1/2
xe(-Go, -1/2) u{1/2, co)
So,/(x) is increasing on (-oo, -1/2) u(l/2, cc).
For/(x) to be decreasing, we must have
/'(.v)<0 - oc
-1/2 1/2
CO

4x^-1 Fig.10.26 Signsof/'{x)fordifferentvaluesof x


<0
x2

w
4.v^ -1 <0 [v -i2>0]
(2x-l)(2x + l)>0
1
— <x<-
2
1
2
F lo [See Fig. 10.26]

e
Fre
x6(-1/2, 1/2)
But, domain (/) = R - {0}. So, /(x) is decreasing on (-1 /2, 0) u (0,1 /2).
for
EXAMPLE 9 Determine the intervals in which the function /(.x) = x^ - 8x^ + 22.x^ - 24x + 21 is
decreasing or increasing.
r
You

SOLUTION We have,
oks

fix) = x^ - Sx"'^ + 22x^ - 24x + 21


eBo

/ '{x) = 4x^ - 24x^ + 44x - 24 = 4 (x^ - 6x^ + llx - 6)


fix) = 4(x-l)(x^-5x + 6)
our
ad

For/(x) to be increasing, we must have


/'(x)>0
4(x-l) (x^-5x + 6)>0
dY
Re

(x -1) (x^ -5x + 6) > 0 [v 4>0]


Fin

(x-1) (x-2) {x-3)>0


1 <x<2or,3<x<oo [See Fig. 10.27]
+
xe(l,2)u(3, od) oo
- 00
1 2 3
So, /(x) is increasing on (1, 2) u (3, co).
Fig. 10.27 Signs oif'ix) for different values of .x
For/(x) to be decreasing, we must have
/'(x)<0
4 (x -1) (x^ -5x + 6) < 0
=> (x-1) (x^ -5x + 6) <0 [v4>0]

(x-1) (x-2) (x-3) <0


2<x<3 or, x<l [See Fig. 10.28]
+

xe(2, 3)u(-oo,l) - OO 2 3 CO

So, fix) is decreasing on (2, 3) w (- co, 1). Fig. 10.28 Signs of/'(x) for different values of x
10.18 APPLIED MATHEMATICS-XII

liXAMl’i.l n
Find the intervals for which f{x) = 2x^ is increasing or decreasing.
SOLUTION We have,
f(x)=x‘^-2x^
fix) = 4.t:^-4.\-=4.y(.t^-1) - CO
-I 0 1 GO

For fix) to be increasing, we must have


Fig. 10.29 Signs of/'(y) for different values of x
/'W>0
4x -1) > 0
x‘ix^ -1) > 0 [V 4>0]
.r (x -1) (x +1) > 0
-1 < X < 0 or, X > 1 [See Fig. 10.29]
=>
X g(-1, 0) u(l, co)

w
So, fix) is increasing on (-1, 0) u (1, oo).
For/(x) to be decreasing, we must have

F lo
- CO
-1 0 1
/'(x)<0
00

4x(x^ -1) <0 Fig. 10.30 Signs of f'{x) for different values of .r

ee
=> x(x^ -1) <0 [v4>0]

Fr
x(x -1) (x +1) < 0
X < -1 or. 0 < X < 1 [See Fig. 10.30]
X e(-co, -1) u(0,1)
for
ur
So, fix) is decreasing on (- oo, -1) u (0,1).
s
x-2
ook

LXAMrLF n
Determine the values ofxfor zuhich fix) = , X ^ -1 is increasing or decreasing.
Yo

x + 1

SOLUTION We have,
eB

x-2
ff) = ,x^-l.
x + 1
r

(x + l)xl-(x~2)xl 3
ou
ad

/'(-t) = ,X¥^-l.
(X + 1)2 (X + 1)2
Y

Clearly, / '(x) = ^ 2 > 0 for all X e R - {-1].


(x + 1)--
Re
nd

So, fix) is increasing on R -{-!}.


Fi

2
Find the mtervals in loliich fix) - ^ + —— , X ^Ois increasing or decreasing.
I.XAMI’LE 12

SOLUTION We have,
c/ \ ^ 2

^ 1 2 x2-4
/ W=
2x^
For fix) to be increasing, we must have
/'(x)>0
X
'-4 >0. +

2x^
- CO
-2 0 2 oo

x^ - 4 > 0 Fig. 10.31 Signs of/'(x) for different values of x

(x-2)(x + 2) > 0
10.19
INCREASING AND DECREASING FUNCTIONS

.Y < - 2 or, Y > 2


Y e(-co, -2) u(2, cc)
So, /(y) is increasing on (- co, - 2) (2, oo).
For/(Y) to be decreasing, we must have
/'(Y)<0
y^-4 + +

- CO -2 0 2 00

2y-

Y^ - 4 < 0 Fig. 10.32 Signs oi/'(i‘) tor different values of .T

(y - 2) (Y + 2) < 0
X si-2, 2)
But, domain (/) = R - {0}. So,/{y) is decreasing on(-2, 0) u(0, 2).
2 1
^ , Y 0 /s

w
EXAMIM I- i' Find the intervals in which the function/given by f (y) = y Y

(ii) decreasing

F lo
(i) increasing
SOLUTION Clearly, domain (/) = R - {0}.
T 1
Now, /(y)=y‘^+^
Y^

ee
Z' i 2 1

Fr
Y + Y + I
f(x) = 3x^-\ Y Y^ y’
(y^-1)(y'^+y2+1) = 3 for Y^

(i) For/ (y) to be increasing, we must have


ur
+ +

/'(Y)>0 - CO
-I 0 1
00
s

Y 4 + y2+1
ook

(Y^-1)>0 Fig. 10.33 Signs of/'(.r) for different values of x


Yo

3
4
Y
eB

Y 4+y^+1
> 0, Y 0
(y^-1)>0 3
Y^
our
ad

{y -1) (y + 1) >0
Y e(-co , -1) u(l, co) [See Fig. 10.33]
dY
Re

So, / (y) is increasing on (- co , -1) u (1, cc)


Fin

(ii) For / (y) to be decreasing, we must have


/'{Y)<0
+ Y^ + 1
(Y^ -1) <0
Y^ + Y^ + 1
< 0
Y^ - 1< 0 3
Y^

(Y-1) (Y + 1)<0
=> Y6(-1,0)U(0,1) [See Fig. 10.34] [V y;^0]
+ +

CO
- 00 -1 0 1

Fig. 10.34 Signs of/'(Y) for different values of x

Hence, / (y) is decreasing an (-1, 0) u (0,1).


10.20 APPLIED MATHEMATICS-XII

EXAMPLE 1-; For which valuesofx, the function f{x) = — is increasing and for which values ofx, it
+1
is decreasing.
SOLUTION We have,

+ 1

/'W =
(.r^+l)xl-;c(2x + 0) 1-x^

w
{x^ + 1)^ (x-2+l)2‘
For f{x) to be increasing, we must have
/'(A-)>0

e
2>0
(a-2+1)

re
ro
:-x^>Q [V (j:2 + 1)2>0]

F
=> ~(x^ -1)>0

Ful
x^ -1 <0
- 00 -1 1 00

Fig. 10.35 Signs of/'(.v) for different values of x


(A-l) (A + l) <0

sr
-1 < A <1
[See Fig. 10.35]

ko
o
Ae(-l,l)
So, f{x) is increasing on (-1,1). of
For f{x) to be decreasing, we must have
o
/'(x)<0
Y
l-.r^
erB

2<0
ix^ +1)
uY

1 -x'^ <0 [v(.v2 + l)2>0]


-{x^ -1) <0
do

— 00
ad

- 1 > 0 -1 1 GO

Fig. 10.36 Signs off'(.x) for different values of x


(x-l) (a + 1)>0
in

=> A< -lorx>l


[See Fig. 10.36]
So, f{x) is decreasing on (- co, -1) u (1, oo).
Re
F

EXAMPLE 15 Find the intervals in which / (a) = 2 log^ (a - 2) - +4a + 1 is increasing or


decreasing.
SOLUTION Clearly, / (a) is defined for all x > 2.
Now, f{x) = 2 loge (x-2) + 4x + 1
- 2x^ + 8x - 6
fix) = ^
x-2
- 2x + 4 =
2 - 2x (x - 2) + 4 (X - 2)
x-2 x-2

- 2 (x^ + 4x - 3) -2(x-l)(x-3)
fix) =
x-2 x-2

For / (x) to be increasing, we must have


fix) > 0
_ -2(x-1)(x-3) > 0
x-2
10.21
INCREASING AND DECREASING FUNCTIONS

{x-l){x-3) < 0
x-2

a-3<0 ['.● a: e Domain (/)^ x>2^ .r-l>0 and, .v - 2 > 0]


X <3 => .Y e(2, 3) [V A->2]

So, / (.y) is increasing on (2, 3).


For / (x) to be decreasing, we must have
fix) < 0
-2(y-1) (.y-3) < 0
y-2

(A-1) (.y-3) > 0

w
y-2

Y - 3 >0 [●.● For Y > 2 , Y - 2 > 0 and y -1 > 0]


Y> 3

Flo
Y e(3, co)

e
So, / (y) is decreasing on (3, co).

re
Type II ON PROVING THE MONOTONtCiTY OF A FUNCTION ON A GIVEN INTERVAL

F
EXAMPLE 16 Prove tiuit the function f{x) =x^ - 3y^ + 3y -100 is increasing on R.
ur
r
SOLUTION We have, fo
/(y) = y^ - 3y^ + 3y-100
ks
^ fix) = 3y^-6y+3 = 3(y-1)^
Yo
oo

Now, Y e R => (y -1)^ > 0 / '(y) > 0.


B

Thus, /' (y) > 0 for all y e R.


re

Hence,/(Y) is increasing on R.
1 .
Let I be an intewal disjointed from 1-1,1]- Prove that the function /(y)=y + —Y is
u

EXAMPLE ]7
ad
Yo

increasing on I.
SOLUTION We have.
d
Re

f{x)=x + -
in

2-1
F

1 Y

/'(Y)=l—2
Y
= y2
Now, xe!

Yg[-l,l]
Y < -1 or Y > 1

y2>1
Y^-1>0
y2- [v Y^>1>01
i>0
y2
/'(Y)>0
Thus, / (y) > 0 for all xel. Hence, f{x) increasing on I.
10.22 APPLIED MATHEMATICS-XII

3
EXAMPLE 18
Show that the function fix) =-+7 is decreasing forxeR- {0}.
SOLUTION We have, f(x) = - + 7

,v2
1
Now, xeR,x 0 => ^ >0 => ~-y <0 => f'{x)<0.

Hence,/(.r) is decreasing for x sR, x ^0.


I XAMPl.I- I'’
Test whether thefimction f{x) =x^ -8 is increasing on [1, 2].
SOLUTION We have,

fix) = .v^ - 8 => / '(.t) = 3x^ => /' (.V) > 0 for all a: e [1, 2]
So,/(.v) is increasing on [1, 2].

w
Prove that thefimction fix) =
1.XAMPI.L2C
- x +1 is neither increasing nor decreasing on (-1,1).
SOLUTION We have, fix) =x^ - .t + 1

Now,
/'(x)=2:r-l=2(x-l/2).
-1 cr<l/2^ (x-1/2) <0=> 2(x-l/2) <0=>/'(x) <0 F lo
e
Fre
and. l/2<x<l^x-l/2>0=>2(x-l/2)>0=>/'(x)>0.
Thus,/ '(.r) does not have the same sign throughout the interval (-1,1).
for
Hence, fix) is neither increasing or decreasing on (-1,1).
i.\AMPLE:!
Determine the values ofx for which f (.v) = .v^ .y > 0 is increasing or decreasing.
r
You

SOLUTION Clearly, / (x) = x'^' is defined for x > 0. So, domain / = (0, qo).
oks
eBo

Now, / (x) = x'^


/(x) =
ad

/' (x) = ‘“S ^ (x log,, x)


our

dx

f'ix) = x-'‘ (1 + log^ x)


For / (x) to be increasing, we must have
Re
dY

/'W>0
Fin

=> X-'' (1 + log,, x) > 0


=> 1 + log,. X > 0 [●.■ x^ > 0 for X > 0]
=> log,, X > -1
-1 N
=> X >e
[●.● log,, x>N=> x>a for a>l. Here, e>l. So, log,, x > -1 => x >
=> xe(l/e, oo)
Thus, / (x) is increasing on (1/e, oo).
For / (x) to be decreasing, we must have
/'(x)<0
x' (1 + log^ x) < 0
1 + log,. X < 0 [●●● x'^’ > 0 for X > 0]
log,. X<-1
10.23
INCREASING AND DECREASING FUNCTIONS

-1
.Y < e

xs{QA/e)
Thus, / (y) is decreasing on (0,1/1’)-
Hence,/(x)isincreasingon(l/e,co)and decreasing on (0,1/c).
EXAMi’LC 22 Find the intervals in which f (y) - —^— is increasing or decreasing.
log,. Y

ow
SOLUTION Note that the domain of/ (y) is the set of all positive real numbers other than unity
i.e. (0,1) vj (1, 00).
Y
Now, / (y) =
log, Y

e
log^Yj^
=> fix) =

re
(log, y)^

Flr
For / (y) to be increasing, we must have

F
/'(Y)>0

(log, y)
ou
sr
(log, .y)^ > 0 for Y > 0 , Y 1]
=> log, Y -1 > 0
ko
of
=> log, Y>1
=> Y > logrt x>N X >a
N
forfl>l.Here,c>l log,Y>l => Y>t?^]
o
Y

z=> xe{e,co).
reB

So, / (y) is increasing on (e, co).


uY

For / (y) to be decreasing, we must have


fix) < 0
loge < 0
ad

=>
do

(log, Y)^
log, Y -1 <0 [●.● (log y)^ > 0 for y > 0 , y 1]
in

log, Y < 1
Re

1
F

x<e

Y6(0, e)-{\] [●.● / (.y) is defined for y > 0 , y 1]


So, / (y) is decreasing on (0, e) - {1}.
EXAMPLE 2.' Find the least value of 'a' such that the function f{x) = y + ax + 1 is increasing on [1, 2].
Also, find the greatest value of a'for which f(x) is decreasing on [1, 2].
SOLUTION We have, /(y) = Y"*^ +ax + l
/'(Y)=2Y + fl and f"{x) = 2forallY.
Now, /"(y) = 2 for all y e(l, 2)
/ "(y) > 0 for all Y € [1, 2]
=> / '(.y) is an increasing function on [1,2]
/'(I) and/' (2) are the least and the greatest values of/'(Y)on [1, 2],
As fix) is increasing on [1,2]
/'(y) > 0 for all Y g[1, 2]
10.24
APPLIED MATHEMATICS-XII

This is possible when least value of /' (x) i.e. /'(I) > 0.
Now, /'(1)>0 => 2 + fl>0=> a>-l
Thus, the least value of a is -2.
If f{x) is decreasing on [1, 2], then
f'{x) < 0 for all .t 6 [1, 2]
Greatest value of / '(.\-) < 0 for a: e [1, 2]
f (2)<0 [●.● fix) is increasing on [1, 2] /' (2) is the greatest value of f{x)]
=> 4 + <7<0=>ff<-4.
So, the greatest value of a is -4.

EXERCISE 10.2

1. Find the intervals in which the following functions are increasing or decreasing,

w
(i) fix) = 10 - 6a- - 2a^ (ii) f(^x) =x^ + 2a -5
(iii) /{a)=6-9a-a^ (iv) /(a)=2a^-12a^ + 18a + 15
(v) /{a) =5 + 36a+3,v^-2a^

Flo (vi) fix) =8+ 36y + 3a^ -2a^

ee
(vii) /(.r) =5a^-15.v^-120a+3 (viii) fix) = A^ - 6a^ - 36a + 2

Fr
(ix) /(a)=2a^-15a^ + 36a + 1 (x) fix)=2x^ + 9a^ + 12y + 20
(xi) /{a)=2a^-9a^+12a-5 (xii) fix) = 6 + 12a + 3a^ - 2a^
for
ur
(xiii) /(a)=2a^-24A + 107 (xiv) /(a)=-2y^-9.v^-12a + 1
s
(xv) f{x)=(x-l)(x-2)^ (xvi) fix) = A^ - 12.v^ + 36a +17
k
Yo

(xvii) fix) =2x^ - 24a + 7


oo
eB

(xviii) fix) = A 1 A^ - 3a^ + ^


5
y +11 (xix) /(a)=a^-4a
r

(xx) f(x)=— + ~:-^


A'" --Y^ -6a +7 (xxi) /(a) =Y^-4a^ + 4a^ + 15
ou
ad

(xxii) /(a)=5y3/2_3;^/2^^^q (xxiii) fix) = A® + 6a^


Y

(xxiv) /(a) =Y^-6a^ + 9a +15 (xxv) /(y)={a(y-2)}^


Re
nd

3
(xxvi) /(a) = 3.v'^-4y^-12.y^+5 -y^-4a^-45a^+51
Fi

2a
(xxviii) / (y) = log (2 + .y) - , A 6 R -A ^-5a^ + 24a + 12
2 +A

2. Determine the values of a for which the function fix) = y^ 6a + 9 is mcreasmg or


decreasing. Also, find the coordinates of the point on the curve y = a^ - 6a + 9 where the
normal is parallel to the line y = y + 5.
3. Show that fix) = is increasing on R.
4. Show that fix) = , x 0 is a decreasing function for all a :?= 0.
5. Show that /(a) = log^ a, 0 < a < 1 is a decreasing function for all a > 0.
6. Show that/{A) = y^ -15y^ + 75a -50 is an increasing function for all a e R.
7. Show that/(a) = (a -1) +1 is an increasing function for all a > 0.
INCREASING AND DECREASING FUNCTIONS 10.25

8. Show that the function -x + \ is neither increasing nor decreasing on {0,1).

9. Show that/(Y) + 4.v^ + 11 is an increasing function for all y 6 R.


10. Prove that the function/(Y) = - 6y^ + 12.y -18 is increasing on R.
n. State when a function f{x) is said to be increasing on an interval [a, b]. Test whether the
function/(.y) = - 6.y + 3 is increasing on the interval [4,6].
12. Find the intervals in which / (y) = log^ (1 + .t) - — is increasing or decreasing.
1 + Y

13. Find the intervals in which / (y) = (y + 2) e~ ^ is increasing or decreasing.


14. Show that the function / given by / (y) = lO'^' is increasing for all y .
15. Prove that the function / given by / (.v) = y - [.y] is increasing in (0,1)-

w
16. Prove that the following functions are increasing on R:
(i) /(y) = 3y''“’+40y^+240y (ii) /(y)=4y^-18y^ + 27y-27

F lo
17. Prove that the function / given by /(y) = y^ - 3y^ + 4y is strictly increasing on R.
18. Find the value(s) of a for which f{x)=x^ -/jYisan increasing function on R,

ee
ANSWERS

Fr
1. Increasing Decreasing Increasing
for Decreasing
(i) (-a=,-3/2) (- 3/2, 00) (ii) (-!-«>) (-co,-l)
ur
(hi) (-oo,-9/2) (-9/2, «=) (iv) (-00,1) u(3, co) (1-3)
(v) {-2, 3) (-co,-2)u(3, co) (vi) (-2,3) (-co,-2)u(3,co)
s
ook

(viii) (-00, - 2) (6, oo) (-2,6)


Yo

(vii) (- <x>, -2)<J (4, co) (-2, 4)


(ix) (-CO, 2) u(3, co) (2,3) (x) (-oo,-2)u(-l,cc) (-2,-1)
eB

(xi) (-CO, 1) u(2, cc) (1,2) (xii) (-1,2) (-CO, -1) u(2, co)
(xiii) (- 00, - 2) u (2, oo) (-2, 2) (xiv) (-2,-1) (-co,-2)u(-l,co)
our

(2,6)
ad

(xv) (-CO, 4/3) u(2, oo) (4/3, 2) (xvi) (- co, 2) u (6, co)
(xvii) (-00,-2) u(2, co) (-2,2) (xviii) (-2,1) u(3, co) (-«),-2) u(l, 3)
(xix) (l,oo) (-<»,!) (xx) (- 3, -1) u (2, oo) (-0), - 3) u {-1, 2)
Y

(xxi) (0,1) u(2, co) (-X, 0)u(l,2) (xxii) (0,1) (1,®)


Re
nd

(xxiii) (0/ oo) (-co,0) (xxiv) (-CO,-1) u(3, co) (1,3)


(-00,-1) u(0, 2)
Fi

(xxv) (0,1) U (2 , co) (-00,0) u(1,2) (xxvi) (-1, 0) u(2, co)


(xxvii) (-3, 0) u(5, co) (- CO ,-3) u (0, 5) (xxviii) (2, oo) (-CO, 2)
(xxix) (-3,2)u(4,co) (-qo,-3)u(2, 4)
2. Increasing on (3, oo) and decreasing on (-co, 3);(5/2,l/4) 11. Increasing

HINTS TO SELECTED PROBLEMS

1. (i) We have, /(.v) = 10-6y-2y^


=> /' (x) = - 6 - 4y = - 2 (2y + 3)
For / (y) to be increasing, we must have
3
/'(y) >0 => -2(2y+3)>0=> 2y+3<0=> x<-~

So, / (y) is increasing on (- co, - 3/2.


10.26 APPLIED MATHEMATICS-XIl

For / (:r) to be decreasing, we must have


f’{x)<0 => -2(2:c + 3)<0 => 2x+3>Q => ;c>--
2

So, / (x) is decreasing on (- 3 / 2, oo)


(ii) We have,/(x) = x^ + 2x-5
=> f'{x)=2x + 2 = 2{x + l)
For / (:r) to be increasing, we must have
/'(o:)>0=> 2(:r + l)>0=> x + l>0=?- x>-l => ^:e(-l, oo)
For / (x) to be decreasing, we must have
f (x) <0=> 2{x + l) <0=> x + 1 <0=> x<-l=> A:e(-oo,-1)

w
Hence, / (x) is increasing on (-1, oo) and decreasing on (- oo, -1).
(iii) Wehave, f{x) = 6-9x-x^ => f'{x) = -9-2x = ~{2x + 9)

F lo
For / (x) to be increasing, we must have
/'(x)>0 => -(2x + 9)>0 => 2x + 9<0 => x<-
I => A:e(-co,-9/2)

ee
Fr
For / (x) to be decreasing, we must have
/ '(x) < 0 => - (2x + 9) < 0 => 2x + 9 > 0 => X >-
|=> AT €(-9/2, 00)
for
ur
(xiv) We have, / (x) = - 2x^ - 9x^ - 12x +1
s

=> /'(x) = -6x2-18x-12 = -6(x2 + 3x + 2) = -6(x + 1)(x + 2)


ook
Yo

For / (x) to be increasing, we must have


eB

f'{x)>0 => -6(x + l)(x + 2) >0 => (x + l)(x + 2) <0 => xe(-2,-1)
For / (x) to be decreasing, we must have
r

/'(x)<0 => -6(x + l){x + 2)<0


ou
ad

=> (x + l)(x + 2)>0 => x<-2orx>-l=>xe(-oo,-2)u(-l,oo)


Y

Hence, / (x) is increasing on (- 2, -1) and decreasing on (- oo, - 2) u (-1, oo).


Re

(xviii) Wehave,
nd
Fi

lU t) 5

- /'W = ^x^-—x^-6x
^ 5
+ —=
5
-(x-l)(x^
5
-1-6) =
|(i-1)(at-3)(i + 2)
- 00
-2 1 3 00

Fig. 10.37 Signs of/'(x) for different values of x

For / (x) to be increasing, we must have

/'(x)>0 => |(x-)(x-3)(x + 2)>0 => (x-l)(x-3)(x + 2)>0 => x e(-2,1) u(3, oo)
So, / (x) is increasing on (- 2,1) u (3, oo).
For / (x) to be decreasing, we must have

/'(x)<0 ^ |(x-l)(x-3)(x + 2)<0 =>(x-l)(x-3)(x + 2)<0=>xe(- 00, - 2) u (1,3)


INCREASING AND DECREASING FUNCTIONS 10.27

.3/2 -3.V3/2
(xxii) We have, f{x)=5x

Y => Y < 1
Now, /' (a-) > 0 => — (1 “ .v) > 0 =?■ 1 -
(xxiii) Wehave, f {x) =x^ + 6x^ => f (x) = Ax (2x^ + 3)
/ '(.V) > 0 ^ Ax (2.y^ + 3)>0=> y>0 [●.● 2y^ + 3 > 0]
(xxv) We have, / (y) = y^ (y - 2)^
=> / '{y) = 2y (y - if + 2y^ (y - 2) = 2y (y - 2) (2y - 2) = 4 (y - 2) (y -1) y
For/ (y) to be increasing, we must have
/'(y)>0

w
=> 4(y-2)(y-1) Y>0
=> (y-2)(y-1) Y>0
X e (0,1) u (2, oo)

Flo
So,/(y) is increasing on(0,1) u(2, co).
For / (y) to be decreasing, we must have

ee
/'(Y)<0

Fr
0 1 2 OD
=> 4 (y - 2) (y -1) y < 0 =>
- 00

(y-2)(y-1) Y<0 Fig. 10.38 Signs of/'(y) for different values of y

or
ur
=> Y 6(-c0, 0) u(l, 2)
So,/ (y) is decreasing on (-CO, 0) u(l, 2).
f
3. We have, /(x) = => / '(y) = > 0 for all y e K /(y) is increasing on R.
ks
Yo

4. Since forallY^Oand -1/y^<0 for all y 0 .


oo

f'{x)=-{l/x^)e^^^<0 forallYii^O
B

3. Wehave, f(Y)=log„Y =>/'{y)=——.


re

Y log «
Since Q <a <1, therefore log /? <0.
u
ad

1 1
Yo

Now, Y > 0 >0=> <0 => /'(y) <0.


X Y log a
So, /(y) is decreasing for all y > 0
nd
Re

6. We have.
fix) = y^-15y^+75y-50
Fi

fix) = 3y^ - 30y + 75 =3(y-5)^ >0forallYeR.


So, / (y) is increasing function for all y e R.
7. We have.
fix) = (y -1) + 1 => / '(y) = Y e'^ > 0 for all y > 0.
So,/(y) is increasing for all y > 0.
9. We have.
fix) = Y^ + 4y^ +!!=>/' (y) = 9y® + 28y^ > 0 for all y € R.
So,/(y) is increasing for all y e R.
10. We have,
/(.v)=y^-6y^+12y-18 =>/'(Y) = 3(Y-2)^>0forallYeR.
So,/(.y) is increasing for all on R.
11. We have.
10.28 APPLIED MATHEMATICS-Xll

f(x)=x^-6x+3^ f'{x)=2{x-3).
Now, xe[4,6] => ,v > 3 => 2(jc - 3) > 0 => /'(:c) > 0
So,f{x) is increasing on [4,6].
■7 We have,/(a:) = .T^-3:r^ + 4a-
=> / '(:r) = 3x^ -6x + 4 = 3 {.y^ - 2x + 1) + 1 = 3 (y -1)^ + 1 > 0 for aU y € R.
Hence,/ (y) is strictly increasing on R.
18. If/(y) is increasing on R, then
/ '(y) > 0 for all y e => 3y^ - > 0 for all y € i? =s> a < 3y^ for all x eR.
But, the least value of 3y^ is 0 . Therefore < 0.

MULTIPLE CHOICE QUESTIONS (MCQs)

w
Mark the correct alternative in each of the following:
1. The function / (y) = y"'^ decreases on the interval
(a) (0, e) (b) (0,1)

Flo
(c) (0,1/c) (d) {l/c,c)

ee
2. The function / (y) = 2 log^ (y - 2) - y^ + 4y + 1 increases on the interval

Fr
(a) (1,2) (b) (2,3) (c) (1, 3) (d) (2, 4)
3. Let/(Y) = Y^-6.Y^+15Y+3.Then,
for
ur
(a) / (y) > 0 for all Y e R (b) / (y) >/ (y +1) for all Y € R
(c) / (y) is invertible (d) f(x) < 0 for all y e R
s
4. The function / (y) =x^ e~^ is monotonic increasing when
k
Yo
oo

(a) Y £ R - [0, 2] (b) 0 < Y < 2 (c) 2 < Y < 00 (d) Y < 0
5. Function / (y) =x^ -27 x + 5 is monotonically increasing when
eB

(a) Y < - 3 (b) I y| > 3 (C) Y<-3 (d) |y1>3


6. Function / (y) = 2 y^ - 9 y^ +12 y + 29 is monotonically decreasing when
r
ou
ad

(a) Y < 2 (b) Y > 2 (c) Y > 3 (d) 1 < Y < 2


Y

7. If the function f{x) —kx^ — 9 y^ + 9 .v + 3 is monotonically increasing in every interval,


then
Re
nd

(a) k<3 (b) k <3 (c) ^: > 3 (d) Ic > 3


Fi

8. The function/(y) = —^
1 +1 Y
IS

(a) Strictly increasing (b) strictly decreasing


(c) neither increasing nor decreasing (d) none of these
9. Function / (y) = a'^ is increasing on R, if
(a) >0 (b) <7 <0 (c) 0 <1 (d) fl >1
10. Function / (y) = log„ y is increasing on R, if
(a) 0 <a <1 (b) fl >1 (c) a<l (d) rt > 0
11. If the function / (.y) = y^ - for + 5 is increasing on [2,4], then
(a) cf£(2, x) (b) ke(-co,2) (c) k e (4, x) (d) Ic e(-x, 4).
12
. If the function/ (y) = y^ - 9k x^ + 27x + 30 is increasing on R, then
(a) -1 < 'A < 1 (b) /c < -1 or A: > 1 (c) 0 < cf < 1 (d) -1 < cf < 0
10.29
INCREASING AND DECREASING FUNCTIONS

13. The function / (x) = + 3.v^ + 64 is increasing on


(a) R (b) (-co,0) (c) (0, «>) (ci) Ro
i I. The interval on which the function f{x) = 2x^ + 9x^ + 12x -1 is decreasing, is
(a) (b) [-2,-1] (c) (-co,-2] (d) [-1,1]
15. y = .t (a: - 3)^ decrease for the values of v given by
(a) 1 < .V < 3 (b) -v<0 (c) .T>0 (d) 0<.v<|
ANSWERS

ow
2. (b) 3. (c) 4. (b) 5. (d) 6. (d) 7. (c) 8. (b) 9. (d)
1. (c)
10. (b) 11. (b) 12. (a) 13. (a) 14. (b) 15. (a)

e
HLL IN THE BLANKS QUESTIONS (FBQs)

re
2x^-1
1. The function/(.y) =

Frl
, .Y > 0, decreases in the inter\^al

F
.Y^
1
2. The function g{x) = ,x^0 decreases in the closed interval
ou
r
so
1
- decreases is
3. The largest open interval in which the function/(y)
kf
1+Y

4. The set of values of 'a' for which the f(x) = ax + bis strictly increasing for aU real y, is
oo

5. The largest interval in which f{x) = is strictly increasing is


Y
eB

ANSWERS

5. (l,e)
ur

1. (1,») 2. [-1,1] 3. (0, cc) 4. (0,co)


oY

VERY SHORT ANSWER QUESTIONS (VSAQs)


ad

Answer each of the follozoing questions in one word or one sentence or as per exact requirement of the
d

question:
in
Re

i. What are the values of 'a' for which / (y) = is increasing on R?


F

2. What are the values of 'a' for which / (y) = a^ is decreasing on R?


3. Write the set of values of ‘a' for which / (y) = log^ y is increasing in its domain.
4. Write the set of values of ‘a’ for which / (.y) = log^ y is decreasing in its domain.
5. Write the set of values of a for which the function / (y) - ax + b is decreasing for all y e R.
ANSWERS

1. a >1 2. 0 <n <1 3. rt>l 4. 0 <1 5. a €(->», 0)


CHAPTER u
MAXIMA AND MINIMA

11.1 INTRODUCTION

In the previous chapters, we have learnt about various applications of differentiation. In this
chapter, we will use differentiation to find the maximum and minimum values ot differentiable

w
functions in their domains. In the end of the chapter, we will discuss applications of maxima and
minima in solving some applied problems.

11.2

F lo
MAXIMUM AND MINIMUM VALUES OF A FUNCTION IN ITS DOMAIN
MAXIMUM Let f (.1-) be a real function defined 071 an interval [a, b]. Then, f {x) is said to have the

ee
Fr
maximum value in [a, b], if there exists a point c in [a, b] such that f (x) </(c) for all x e [a, (i).
the number/(c) is called the maximum value of / (x) in the interval [a, ii] and the
In such a case.
for
point c is called a point of maximum value of / in the interval [a, 1’].
r
You
o oks
eB
our
ad

.V=/-v)
/(c)
dY
Re
Fin

O b X
X’

Fig. 11.1 Maximum value of/(x) at .v= c

Consider the function / given by f (x) = - (l -1)^ + 10. Clearly, domain (/) = R = (- =o , co).
We observe that

-(a,* -1)^ < 0 for all xeR


-{x-1)^ +10 < 10 for all a: e R
/ (.v) <10 for all X gR
f (.v) </ (1) for all .Y e R [v / (I) = -(1-1)2 + 10=10]
11.2
APPLIED MATHEMATICS-XII

It follows from this expression that/ (1) = 10 is the maximum value of function/ and the point of
maximum value of / is v = 1. This fact is also evident from the graph of fimction / as shown in
Fig. 11.2.
y

(h 10)

X’
X

o (1,0)

/(jc) = -(x-l)2 + io

w
F lo
r

Fig. 11.2 Graph of/(r) = -(x-l)^ + u)


MINIMUM Letf (x) be a realfimction defined on an interval [a, b]. Thenf (x) is said to have the minimum

e
Fre
value m interval [a, b], if there exists a point c e [a, b] such that f (x) > f (c) for all x e [a, b].
In such a case, the number/ (c) is called the minimum value of / (.v) in the interval [a, b] and the
for
point c is called a point of minimum value of / in the interval [a, b].
r
Y
You
oks

fix)
eBo
ad
our

m
dY
Re

X' a O c X
b
Fin

Y'

Fig. 11.3 Minimum value oif(x) at x = c


Consider the function/ given by/ (.y) = y^ + 5. Clearly, domain (/) = R =(- oo , co).
We know that

Y^>0 foralLreR
x^ + 5 > 5 for all y e K
/(y) >5 for all x eR
/(y)>/(0) forallY eR
It follows from this expression and the above definition that the minimum value of function
/ (y) = Y +5 defined on R is 5 and the point of minimum value of / is y = 0.
This observation is also evident from the graph of/ (y) = + 5 as shown in Fig. 11.4.
MAXIMA AND MINIMA 11.3

= x'^ + 5

X' X

Fig. 11.4 Graph of f{x) = +5

In the above discussion, we have seen that the function / (x) = -(x -1)^ + 10, x e R has the

w
maximum value but it does not attain the minimum value, because - (x -1) +10 can be made as

F lo
small as we please, which is also evident from the graph (Fig. 11.2). The function /(x) = x +5
attains the minimum value 5 at x = 0, but it does not attain the maximum value at any point in its
domain. In fact, / (x) can be made as large as we please. From the graph of / (x) (Fig. 11.4), we
find that the values off (x) are increasing rapidly. That is why it does not attain the maximum

e
Fre
value. for
Now, consider the function / given by / (x) = x ^defined on (- 2, 2). Clearly, it is an increasing
function in the given interval. So, it should have the minimum value at a point closest to - 2 on
r
its right and the maximum value at a point closest to 2 on the left. In fact, it is not possible to
You

locate such points as shown in Fig. 11.5. Therefore, / (x) = x^ has neither the maximum value
oks

nor the minimum value in the interval (- 2,2).


eBo

It follows from the above discussion that a function f defined on an interval I.


Y
ad
our

'f(x)= x^
Re
dY
Fin

X' (-2,0) O (2,0) X

Y'

Fig. 11.5 Graph of/(a:) =

(i) may attain the maximum value at a point in I but not the minimum value at any point in /.
(ii) may attain the minimum at a point in I but not the maximum value at any point in /.
(iii) may attain both the maximum and minimum values at some points in I.
(iv) may not attain both the maximum and minimum values at any point in I.
Let us now discuss more examples on the maximum and minimum values of functions in their
domains.
11.4 APPLIED MATHEMATICS-Xil

ILLUSTRATIVE EXAMPLES

LXAMIM.E I Pind the maximum and the minimum values, if any, of the followmg functions
(^) f{x) = 3^^ + 6x + 8,xeR (ii) fix) = -1 X - 11 + 5for all x e R
(iii) f{x) = + 1 forallxeR (iv) fix) =\x+3\forallxeR
SOLUTION (i) We have,
f{x) = 3x^ + 6a: + 8
or. fix) = 3 + 2x +1) + 5 = 3 (x + 1)2 + 5.
Clearly, 3 (x + 1)2 >0 for all x e K
3(x + 1)2+5 >5 foraU xeR
fix) > /(-I) for all xeR. [V /(-I) =5]

w
Thus, 5 is the minimum value of fix) which it attains at x = -1.
Since fix) can be made as large as we please. Therefore, the maximum value does not exist

F lo
which can be observed from Fig. 11.6.
y

ee
Fr
/fix) = 3(.r + 1)2 + 5
for
L5)
r
You
s
ook

X' X
eB

(-U 0) o

y'
our
ad

Fig. 11.6 Graph of/(.Y) = 3(x+ 1)^ + 5


(ii) We have.
dY

/(.x) =-| X-11+5 for all xeR


Re

Clearly,
Fin

(L5)
I X -11 > 0 for all X € R
●b
-1 X -11 < 0 for all X e R
-1 X -1 I + 5 < 5 for all x e R
fix) < 5 for all x e R.
So, 5 is the maximum value of/(x). X' (-4,0) 0 (UO) (6, 0) X

Now,
r
/(x) =5=>-I X-11+5=5
Fig, 11.7 Graph of/(.r) = -1x - 1| + 5
|x-l|=0=>x=l.
Thus,/(.x) attains the maximum value 5 at x = 1.
Since /(x) can be made as small as we please. Therefore the minimum value of/(x) does not
exist (see Fig. 11.7).
MAXIMA AND MINIMA 11.5

(iii) We have, f{x) = +1, x g R.


Here, we observe that the values oif{x) increase when the values of x are increased andf{x) can
be made as large as we please by giving large values to x. So, f{x) does not have the maximum
value. Similarly, f{x) can be made as small as we please by giving smaller values to x. So f{x)
does not have the minimum value also. (See Fig. 11.8).

fix) = ,r3 + 1

w
X’ X

F lo
Fig. 11.8 Graph of f{x) = x^ + l

ee
Fr
(vi) We have, f{x) = | x + 31 for all x g R
Clearly, | x + 31 > 0 for all .v g R
for
ur
^ m > 0 for all X e R.
s
ook

So, the minimum value of f(x) is 0, which it attains at x = - 3.


Yo

Clearly,/(x) =| x + 31 does not have the maximum value. (See Fig. 11.9).
eB

r
our
ad

^i)=|x + 3|
Y
Re

X' (-3, 0) o X
nd
Fi

Fig. 11.9 Graph of f{x) = \ x+ 3|

EXERCISE 11.1

Find the nuiximiim and the minimum values, if any, without usmg derivatives of the followingfunctions:

1. fix) =4x^ -4x + 4on R _. fix) =-(x -1)^ + 2on R


3./(x)=[x + 2lonR /(x) = 2x^ +5 on R
5. fix) = -]x+l| + 3onR fix) = 16x^ - 16x + 28 on R
7. fix) = x^ -Ion R
11.6
APPLIED MATHEMATICS-Xll

ANSWERS
1, Min. = 3, Max. dose not exist. 2. Max. = 2, Min. does not exist.
3. Min. = 0, Max. does not exist. Max and Min. both do not exist.
5. Max. = 3, Min. does not exist. 6, Min. = 24, Max. does not exist
7. Max. and Min. both do not exist.

HINTS TO SELECTED PROBLEMS

2. We have,/(.v) xeR

Clearly, - (,y -1)^ <0 for al! x eR


=> - (-Y -1)^ -f 2 < 2 for all x sR

w
=> f{x) <2 for all y e R
So, /(y) attains maximum value 2 at y=1 and the minimum value does not exist

Flo
as

/ (y) can be made as small as we please.

e
5. We have, / (y) = -1 y + 11 + 3, xeR

re
We know that

F
-1 Y + 1 I < 0 for all X e R => -|y +1| + 3 < 3 for all xeR => /(y) < 3 for all xeR.
ur
r
So, the maximum value of / (y) is 3. As /(y) can be made as small as we please. So, the
minimum value of/ (y) does not exist.
fo
ks
Yo

11.3 LOCAL MAXIMA AND LOCAL MINIMA


oo

In the previous section, we have discussed about the greatest (maximum) and the least
(minimum) values of a function in its domain. But, there may be points in the domain of a
eB

function where the function does not attain the greatest (or the least) value but the values at
these points are greater than or less than the values of the function at the neighbouring points.
ur

Such points are known as the points of local minimum or local maximum and we will be mainly
ad

discussing about the local maximum and local minimum values of a function.
Yo

LOCAL MAXIMUM ^ function f{x) is said to attain a local maxinwm at x = a if there exists a
neighbourhood {a~b,n + 8) of a such that
d
Re

fix) < f{a) for all x e (a - 8, « + 6), y a.


in

fix) -fia) < 0for all X & in -5, a + 6),


F

or, X ^ a.

In such a case,/(a) is called the local maximum value of/(Y) at y = a.


LOCAL MINIMUM ^ function fix) is said to attain a local minimum at x = a if there exists a
neighbourhood (a - 8, a + 8) of a such that
fix) > fia) for alt x s (rt - 8, a + 8), y a
or, /(-■'■') -/(«) > 0 for all X sia-5,a + 6), x ^ a.
The value of the function at y = iz i.e. fin) is called the local minimum value of/(y) at y = a.
The points at which a function attains either the local maximum Values or local minimum values
are known as the extreme points or turning points and both local maximiun and local minimum
values are called the extreme values of /(y). Thus, a function attains an extreme value at .y = n if
fia) is either a local maximum value or a local minimum value. Consequently, at an extreme
point 'a', fix) - fia) keeps the same sign for all values of y in a deleted neighbourhood of a.
MAXIMA AND MINIMA 11.7

X
O

Fig.11.10

w
In Fig. 11.10 we observe that the ar-coordinates of the points A, C and E are points of local

F lo
maximum and the values at these points i.e. their y-coordinates are the local maximum values of
f(x). The ^-coordinates of points B and D are points of local minimum and their y-coordinates
are the local minimum values

NOTE By a local maximum (or local minimum) value of a function at a point x = a, we mean the greatest

ree
(or the least) value in the neighbourhood of point x = a and not the maximum (or the minimum) in the
for F
domain of the function. In fact a fimction may have any number of points of local maximum (or local
minimiun) and even a local minimum value may be greater than a local maximum value. In Fig. 11.10 the
minimum value at D is greater than the maximum value at A. Thus, a local maximum value may not be
the greatest value and a local minimum value may not be the least value of the function in its domain.
Your
ks

It follows from the above definition that if is a point of local maximum of a function f, then in
eBoo

the neighbourhood of a the graph of / should be as shown in Fig. 11.11. Clearly, / (x) is
increasing in the left neighbourhood (a - 6, a) of point a and decreasing in the right
ad

neighbourhood of x=a.
our

f'{x)>0 ior X e{a-d, a) and, f (x) <0 for x e(a, a + S)


Y
Re
Y
Find

O (7-5 a + 5 X

Fig. 11.11

This suggests that /' (fl) must be zero.


Similarly, if <7 is a point of local minimum of a function/, then in the neighbourhood of a the
graph of / should be as shown in Fig. 11.12. Here, we observe that / (.v) is decreasing in the left
neighbourhood (/? - 6, fl) of rt and increasing in the right neighbourhood (a, a + S) of a.
11.8 APPLIED MATHEMATiCS-X!!

f'{x)<0 for - 8, fl) and,/'(x) > 0 for a: g(a , a + 6).


Y

>0

w
O n-6 a (7 + 5 X

F lo
Fig. 11.12

This also suggests that /' (a) must be zero.

ee
In view of the above discussion we state the following theorem (without proof) which is known

Fr
as the necessary condition for points of local maximum or minimum.
THEOREM A necessary condition forf(a) to be an extreme value of a functionf(x)isthatf'{a) = 0,
for
in case it exists.
ur
RLMAKK : This result states that if the derivative exists, it must be zero at the extreme points. A
function may however attain an extreme value at a pomt without being derivable thereat. For example, the
s
ook
Yo

function f{x) =\x\ attains the minimum value at the origin even though it is not derivable atx = 0.
eB

y
our

y = -x y =
ad
Y

X' o X
Re
nd
Fi

Y'

Fig. 11.13Graph of/(Ar) = |x|


RLM.AKK.: This condition is only a necessary condition for the point x = atobean extreme point. It is
not sufficient i.e., / '(fl) = 0 does not necessarily imply that x = ais an extreme pomt. There are functions
for zuhich the derivatives vanish at a point but do not have an extreme value thereat. For example, for the
function f{x) = x^, /'(O) = 0 but at x ==0 thefunction does not attain an extreme value. In fact on the
left of x=^0, the curve is concave down and on its right the curve is concave up. That is, the concavity of
fix) changes x as increases through O. Such points are called points of inflection. If x = c is a point of
inflection of a function f(x), then /"(c) = 0 and f"{x) change its sign as x increases through 'c'.
ri;m . . . Geometrically the above condition means that the tangezit to the curve y =f(x) at a point
where the ordinate is maximum or minimum is parallel to the x-axis.
MAXIM.A AND MINIMA 11.9

f{x) = x^l

X' O X

Fig.11.14Graphof f{x) = x^
r:v'ark4 As discussed in Remark 2 that all xjor zvhich f'{x) = 0, do not give us the extreme values.

w
The values of xfor which f '(:e) = 0 are called stationary points or turning points and the corresponding
values off(x) are called stationary or turning values off(x).
RTMAKK S

A FIRST DERIVATIVE TEST FOR LOCAL MAXIMA AND MINIMA

F lo
The values of xfor which f (a:) = 0 or, f (a:) does not exist are known as critical points.

e
In the previous section, we have seen that an extreme poiiit (point of local maximum or

Fre
minimum) the derivative of the function either does not exist or in case it exists, it must be zero.
We have also seen that if a is a point of local maximum value of a function/, then there exists a
for
neighbourhood (rt - 6, + 5) of a such that
/' (a:) >0 for all a: E (tj - 5 , «) [See Fig. 11.11]
r
/' (a:) <0 for all a: e (fl , fl +
You

and.
oks

In case, fl is a point of local minimum value of function/, then there exists a neighbourhood
eBo

(fl - 6, fl + ^ of 4 such that


/’ (.t) <0 for all a: e (fl - 5, fl)
and. /' (a:) >0 for all a: £ (fl , (7 + 6) [See Fig. 11.12]
our
ad

In the light of these observations, we state the following theorem (without proof) for finding the
points of local maxima or local minima.
THEOREM 1
(First derivative test) Let f be a differentiable function defined on an interval I and let a e I.
dY
Re

Then,

(a) x = a is a point of local maximum value off, if


Fin

(i) /' (^)=0


and, (ii) /' (x) changes sign from positive to negative as x increases through a
i.e.f (at) > 0 at every point sufficiently close to and to the left of a, and f'{x) < 0 at every point sufficiently
close to and to the right of a.
(b) x = a is a point of local minimum value off, if
(i) /' («)=0
and, (ii) f'{x) changes sign from negative to positive as x increases through a
i.e. f (a:) <0at every point sufficiently close to and to the left of a, and f (a;) > 0 every point sufficiently
close to and to the right of a.
(c) Iff (a) = 0 and f (.t) does not change sign as increases through a, that is, f (a:) has the same sign in
the complete neighbourhood of a, then a is neither a point of local maximum value nor a point of local
minimum value. In fact, such a point is called a point of inflexion.
11.10
APPLIED MATHEMATICS-XII

The above theorem suggests the following algorithm to find the points to local maxima or local
minima of differentiable functions.

ALGORITHM

STF.P 1 Put y= fix)


STF.IMI Find it
dx

Put
STEP 111
dx
= 0 and solve this equation for x. Letc^, c^, C3,...,c„ he the roots of this equation. Points
●^1, ‘^2' ^3' —/ critical points (stationary values ofx) and these are the possible points where
the function can attain a local maximum or a local minimum. So, we test the function at each
one of these points.
STFP !V Consider X = Cl.

If ^ changes its signfrom positive to negative as x increases through Cj, then thefunction attains

w
a local maximum atx =

attains a local minimum at x = C|.


F lo
If ~ changes its sign from negative to positive as x increases through Cy then the function

If ~ does not change sign as x increases through Cp then x = Ci is neither a point of local

ree
for F
maximum nor a point of local minimum. In this case x = Cj is a point of inflexion.
Similarly, we may deal with other values of x.
Following examples will illustrate the above algorithm.
Your
oks

ILLUSTRATIVE EXAMPLES
eBo

EXAMPLE 1 Find all the pomts of local maxima and minima of thefunction f(x) =x'^ -6x^ + 9 x - 8.
SOLUTION Let y = f(x) = x^ ~ 6x^ + 9 x - S. Then,
ad
our

^ = f'(x) = 3:c^-12.v+9 = 3(x^-4.t+3)


dy
The critical points of f(x) are given by f'(x) - 0 or. = 0.
Re

dx
dY

dy
Now, = 0 => 3{x^ -4a: + 3) = 0 => a: = 1, 3.
Fin

dx

We have to examine whether these points are points of local maxima or local minima or neither
of them.

dy
We have, = 3 (x -1) (a: - 3)
dx

dy
The changes in signs of -f
dx
for different values of a: are shown in Fie. 11.15.
^

~Ch
— CO 00
3
dy
Fig, 11.15 Signs of for different values of at.
dx
dy
Clearly, changes sign from positive to negative as increases through 1.
MAXIMA AND MINIMA 11.11

So, A' = 1 is a point of local maximum,


Also
' dx
changes sign from negative to positive as x increases through 3.

So A = 3 is a point of local minimum.


EXAMl’iJ- 2 Find all the points of local maxima and local minima as well as the corresponding local
3 2
maximum and local minimum values for the function f{x) =(a-1) (a + 1) .
SOLUTION Lety=/(A)={A-1)^(A + 1)^. Then,
^
dx
= 3(a-1)^(a + 1)^ + 2{a + 1)(a-1)^

w
<^y
= (a-1)2 (a+ 1) {3{a + 1) + 2(a-1)}
dx

= (a-1)2{a + 1)(5a + 1).


dx

o
At points of local maxima or local minima, we must have

e
re
^y
dx

rFl
= 0 => (a-1)2 (a+ 1) (5a+ 1) = 0 => A = 1 or, A = -1 or, A = --

F
Now, we have to examine whether these points are points of local maximum or local minimum
or neither of them.

r
ou
+
-o- o-
+
■o
+
fo
ks
— 03 1 oo

5
oo

Fig. 11.16 Signs of fordifferentvaluesof .r.


dx
Y
eB

Since(A -1)2 is always positive, therefore the sign oi-^is same as that the(A +1) (5a +1).
r

The changes in signs of ^ for different values of a are shown in Fig. 11.16.
ou
Y
ad

dx

Clearly does not change its sign as a passes through 1. So a =1 is neither a point of local
d

dx
maximum nor a point of local minimum. In fact, a = 1 is a point of inflexion,
Re
in

Clearly ' dx changes sign from positive to negative as a passes through -1.
F

So, A = -1 is a point of local maximum.


The local maximum value of/(a) at a =-1 is/(-I) =(-2)^ (-1 + 1)2 = 0.
It is evident from Fig. 11.16 that ^
dx
changes sign from negative to positive as a passes through
-1/5. So, A = -1/5 is a point of local minimum.
■n3 / 1 ',2 3456
The local minimum value of/(a) at A = --is/ --
1
= i-1 -- + 1
5 5/ 5 / V 5 3125

Find all the points of local maxima and local minima of the function
/(a) =a^ -6a2 +12a-8.
SOLUTION Let y =/(a) = a^ - 6a2 + 12a - 8. Then,
^
dx
= 3a2-12a + 12 = 3 (a-2)2
11.12 APPLIED MATHEMATICS'XII

The critical points of y = f{x) are given by = 0.


dx

Now, dx
= 0 => 3(j:-2)^ = 0 => x = 2.
We observe that

^
dx
= 3 (.t-2)^ >0 for all x ¥^2.
+ +

- 00
2 00

dy
Fig. 11.17 Signs of for different values of x.
dx

Thus, ^ does not change sign as a: increases through :c = 2.

w
Hence, x = 2 is neither a point of local maximum nor a point of local minimum. In fact, it is a
point of inflexion.

Flo
l.XAMi’U 4 Show that the function f{x) =4x^ -18x^ +27x-7 has neither maxima nor minima.

ee
SOLUTION We have,

Fr
y = /(:r) =4.r^ ~18x^ + 27 x-7

^ = 12x^ - 36-r + 27 = 3 (4.v^ -12.v + 9) = 3 (2.r - 3)^ for


ur
The critical points of y =f(x) are given by ^
dx
= 0.
s
dy
k

= 0=> 3(2x--3)^ = 0=>2.v-3 = 0=> x = -


Yo

Now,
oo

dx 2
eB

+ +

- OO
3/2 00
r

dy
ou
ad

Fig. 11.18 Signs of -f-


dx
for different values of x.
Y

Clearly, ^
dx
= 3 (2x - 3)^ > 0 for all x ^ -2
Re
nd

"Thus, — does not change its sign as x increases through x = 3/2 as shown in Fig. 11.18. Hence,
Fi

a: = 3/2 is neither a point of local maximum nor a point of local minimum. In fact, it is a point of
inflexion.

I XAMt'i 1 Find the points of local maxima, local minima and the points of inflection of the function
fix) =xP -Sat** +5a:^ -1. Also, find the corresponding local tnaximiim and local minimum values
SOLUTION Lety =/(a:) =.yp -5x^ +5x^ -l.Then,
^=5a;‘* -20a:^ +15.a:^ =5a:^ {A:^-4A:-f 3) =5a.-^ (a:-1) (a:-3)
The critical points of y =f{x) are given by ^
dx
= 0.

Now,
^y
dx
= 0 => (a: -1) (a: - 3) = 0 => a: = 0, .Y = 1, a: = 3.
^y
Clearly, g does not change its sign as .y increases through 0. So, x = 0 is a point of inflection.
MAX!':-AND MINIMA 11.13

It is evident from Fie. 11.19 that ^


dx
changes its sign from positive to negative as a: increases
through 1. So, ;c = 1 is a point of local maximum.
+
o o o
oo
— 00 0 1 3

Fig. 11.19 Signs of for different values of x.


dx

The local maximum value of f(x) is/(I) =1 -5 + 5-1 =0.


We observe, from Fig. 11.19, that ^
dx
changes its sign from negative to positive as i increases
through 3. So, X = 3 is a point of local minimum.
The local minimum value of/(x) is/(3) = 3^ -5x 3^ + 5x 3^ -1 = -28.

w
1+ ■ :● 6 Find the points at which the function /given by f (x) = (x - 2)^ (x + 1)^ has

F lo
(i) local maxima (ii) local minima (iii) points of inflexion
SOLUTION We have,

f(x) = (x-2)*‘(x + lf

ee
f(x) = i(x-l)^ (x + \f ^i(x-2f{x + lf

Fr
=>

f(x) = (1-2)3(1 + 1)2(71-2) for


f'(x) = (x-2f(x*\f(x-2)(7x-2)
ur
The critical points of/(x) are given by f'{x)= 0.
s
ook
Yo

Now, /' (x) = 0 => X = 2, -


eB

Since (x - 2)^ (x +1)^ is always positive. So, sign of /' (x) depends upon the sign of
(x-2) (7x-2). The changes in signs of /' (x) as x increases through 2/7 and 2 are shown in
our

Fig. 11.20.
ad

+ +
-o-
— oo -1 2 2 00
Y

7
Re

Fig. 11.20 Signs of/'(x) for different values of x.


nd

Clearly, /' (x) changes its sign from positive to negative as increases through 2/7.
Fi

2
So, X = — is a point of local maximum.

We observe that/’(x) changes its sign from negative to positive as x increases through 2.
So, X = 2 is a point of local minimum.
There is no change in the sign of/' (x) as increases through -1. So, x = -1 is a point of mflexion.
EXERCISE 11.2

Find the points of local maxima or local minima, if any, of the following functions, using the first
derivative test. Also, find the local maximum or local minimum values, as the case may be:
fix) = (x-5)* . /(x) = x^ - 3x
f{x)=x^{x-l)'^ f{x)=ix-l) (a:+ 2)2
11.14
APPLIED MATHEMATICS-XII

1
5. f(x) =
+ 2
/(^) = - 6.r^ + 9a: +15

7. / (at) = .Y yJV-X , Y > 0 . / (y) = Y^ (2y -1)^


●^ 2
9.
/w = —^ + - , Y > 0Y

ANSWERS

Y = 5 is a point of local minimum, local minimum value = 0.


Y = -1 is a point of local maximum, local maximum value - 2
Y = 1 is a point of local minimum, local minimum value = - 2.
3. Y = 1 is a point of local minimum, local minimum value = 0
3 108
X =
— is a point of local maximum, local maximum value =
3125

w
4. Y = 0 is a point of local minimum, local minimum value = - 4

Local maximum at y = 0, Local maximum value =


F lo
- 2 is a point of local maximum, local maximum value = 0.
1

e
Fre
6. Y = 1 is a point of local maximum, local maximum value = 19
Y = 3 is a point of local minimum, local minimum value
for
= 15.

2 2
Local maximum at y = — , Local Maximum value =
3 9
r
You
oks

8. Minimum at y = — , Local Minimum value = —^


4'
eBo

512

9. Minimum at y = 2, Local Minimum value = 2


ad
our

HINTS TO SELECTED PROBLEMS


We have,

/(y) = Y^ - 3y
Re
dY

^ fix) = 3y^-3 = 3(y-1)(y + 1)


Fin

The critical points of/(.y) are given by/' (y) = 0.


Now, fix) = 0 ^ 3(y-1) (y + 1) = 0 => Y=l,-1.
The changes in signs of /' (y) for different value of y are as shown in Fig. 11.21.

+ +

- 00
-1 1 CO

Fig. 11.21 Signs of fix) for different values of y.

Clearly, /' (y) changes its sign from positive to negative as Y increases through -1. So,
Y
1 is a point of local maximum with the local maximum value given by
/(-I) = (-l)^-3(-l) = 2.
As/' (y) changes its sign from negative to positive as y increases through 1. So, y = 1 is a
point of local minimum with the local minimum value / (1) = 1 - 3 = - 2.
MAXIMA AND MINIMA 11.15

' We have.

/w=-^+ 2
-2x
=> fix) =
(x^ + 2)2
The critical points of/(x) are given by /' (x) = 0.
-2x
Now, /'(x) = 0 => = 0 => X = 0

+
-o-
— 00
0 X

low
Fig. 11.22 Signs of /'(.v) for different values of x.

The signs of f'{x) for different values of x are shown in Fig. 11.22. Clearly,/'(x) changes its
sign from positive to negative as increases tlirough 0. So, x = 0 is a point of local maximum
with the local maximum value/(O) =1/2.

ee
We have

/W = .#^,.>0^ /'(.V) = F = 2^
Fr
for
ur
At critical points of/(.x), we must have
2-3x
/'{.X) = 0^ = 0 => X = -
ks
3
Yo
oo

2
X -
2-3x
eB

Now, /' (x) = 3


271^
r

The changes in signs of /' (x) as x increases through 2/3 are shown below:
ou
ad
Y

- X
2/3 X
nd
Re

Fig. 11.23 Signs of f'(x) for different values of .y.


Fi

Clearly, /' (.x) changes its sign from positive to negative as x increases through 2/3. So,x = 2/3
/ 2 2
is a point of local maximum with the local maximum value / \3— J = 3v 3

11.5 HIGHER ORDER DERIVATIVE TEST

As we have seen in the previous section that finding the local maximum or local minimum by
first derivative test is very time consuming and of course tedious for beginners because it is
slightly difficult to determine the change in the sign of / '(.x) as x increases through the points
given by/' (x) = 0. We have another test known as the Higher order derivative test which enables
us to find the points of local maxima or local minima more easily and more quickly.
THEOREM (Higher Order Derivative Test) Let fbe a differentiable function deifned on an interval I and
let c be an interior point of I such that
» -1
(i) /'(c) =/"(c)=/'"(c) =...=/ (c) = Qand, (ii) f ” (c) exists and is non-zero.
11.16 APPLIED MATHEMATICS-XII

Then,

if n is even and / " (c) < 0 => x = cisa point of local maximum
if n is even and / ” (c) > 0 => x = cisa point of local minimum
ifn is odd, x = cis neithern point of local maximum nor a point of local minimum.
This theorem suggests the following algorithm to find the points of local maximum and local
minimum.

ALGORnHK

~ 11 Find f '{x)
-III
Put f '{x) =0 and solve this equation for x. LetC[,C2,-..,c^ be the roots of this equation. Points
Cj, C2,..., c„ are stationary values or critical points of f(x) and these are the possible points where
the function can attain a local maximum or a local minimum. So, we test the function at each one
of these points.
●. I i I 11
Find f "(x). Consider a: = Cj.

w
/// " (C]) < 0, then x=C] is a point of local maximum.
Iff "{c^)> 0, then x =Cj is a point of local minimum.

Iff
F lo
///"(Ci) =0, we must find f{x)and substitute in it c^for x.
" (^l) * 0, then a: = Cj is neither a point of local maximum nor a point of local minimum and
is called the point of inflection.

ree
= 0, we must find f^'^ (a:) and substitute in it c^for x.
for F
(cj) < 0, then x = Cj is a point of local maximum and if f
IV
(cj) > 0, then x = c^ is a
point of local minimum.
(c{) =0, we must findf ^ (x),and so on. Similarly, thepointsc2,c^,..., may be tested.
Your
ks

POINT OF INFLECTION An arc of a curve y = f{x) is called concave upward if at each of its points, the arc
eBoo

lies above the tangent at the point (see Fig. 11.24).


ad

y, y,
our
Re
Y
Find

o X o X

Fig. 11.24 Fig. 11.25

]/ =/(^) is a concave upward curve, then as a: increases,/'(.r) either is of the same sign and
increasing (see Fig. 11.24) or changes sign from negative to positive (see Fig. 11.25). In either case
/ '(x) is increasing and so/"(a:) > 0. Thus, for a concave upward curve / "(,v) > 0.
An arc of a curve y = f{x) is called concave downward if, at each of its points, the arc lies below the
tangent at the point.
If an arc of a curve y = f(x) is concave downward, then as a: increases, / '(x) either is of the same
sign and decreasing (see Fig. 11.26) or changes sign from positive to negative (see Fig. 11.27). In
either case/ '(at) is decreasing and sof'{x) < O.Thus, fora concave downward curve/ "(a:) < 0.
MAXIMA AND MINIMA 11.17

Y, Y.

o X O X

Fig. 11.26 Fig. 11.27

POINT OF iNFLEXiOi ^ A point of inflection is a point at luhich a curve is changing concave upward to
concave downivard, or vice-versa.
y,

y=/W

w
o
F lo /(c)
for F
X
ree
Fig, 11.28

A curve y =f{x) has one of its points t = c as an inflection point, if


Your
ks

(i) f " (c) = 0 or is not defined and


eBoo

(ii) /"(at) changes sign as :r increases through= c.


The later condition maybe replaced by f '"{c) ^ 0 when f '"{c) exists.
Thus, X = c is a point of inflection if f "(c) = 0atid f"'{c) ^ 0.
ad
our

PROPERTIES OF MAXIMA AND MINIMA

(i) If /(.t) is continuous function in its domain, then at least one maximum or one mmimum
must lie between two equal values of f{x).
Re

(ii) Maxima and Minima occur alternately, that is, between two maxima there is one
Y

minimum and vice-versa,


Find

(iii) If f(x) -> ooasx^ florband f'(x) = 0 only for one value of a: (say c) between fl and then
/(c) is necessarily the minimum and the least value.
If f^x) - coasxaorb, then f(c) is necessarily the maximum and the greatest value.

ILLUSTRATIVE EXAMPLES

FX wip: Find all the points of local maxima and minima and the corresponding maximum and
minimumvaluesofthefunctionf{x) = -- -r^+105.
2
SOLUTION We have.

fix) = ---a:^-8a:3- —A.-2+105


2

/ '(a:) = - 3.t^ - 24.v^ - 45a: = - 3a: {x^ +8x + 15)


The critical points of f(x) are given by /' (.v) = 0.
11.18
APPLIED MATHEMATICS-XM

Now, / '(.t) = 0 => - 3:<: (.T^ + 8 X + 15) = 0 => - 3 .t {.v + 3) {:c + 5) = 0 => jr = 0, - 3, -5
Thus, = 0, .r = - 3 and .r = -5 are the possible points of local maxima or minima.
Let us now test the function at each of these points.
Now, /' (x) = -3x^ - 24.v^ - 45-v
=> /"(x) = -9x^-48X-45
At X = 0; We have,
/"(0)=-45<0
So, X = 0 is a point of local maximum.
The local maximum value of /(x) at x = 0 is /(O) = 105.

ow
At x = -3: We have,
/"(- 3)=-9(- 3)2 -48(- 3)-45 = 18>0
So, X = - 3 is a point of local minimum.

e
The local minimum value of /(x) at x = - 3 is

Fl
re
/(-3)=-^{-3)‘‘-8{-3) ^ -(-3)2+105= —

F
4 2 4
At X = - 5; We have,
ur
r
/ ”(- 5) =- 9 (- 5)2 - 48(- 5) - 45 =- 30 <0 fo
So, X = - 5 is a point of local maximum.
ks
The local maximum value of/(x) at x =-5 is
Yo
oo

/(-5)=-|(-5)'^-8(-5)3 4^(-5)2
2 '
+ 105=^
4
B

tXAMi’U 2 all the poitits of local maxima ami minima and the corresponding maximum and
re

minimum values of the function f{x) = 2x^ - 21 x-2 + 36x - 20.


u

SOLUTION We have,
ad
Yo

/(x) = 2x^-21x2 + 36x-20


=> f'{x) = 6x2-42x+36
d
Re

The critical points of /(x) are given by /' (x) = 0.


in

Now, /'(x) = 0 => 6x2-42.y+36 = 0 => (x-l){x-6) = 0 => x = 1, 6.


F

Thus, X' = 1 and x = 6 are the possible points of local maxima or minima.

Now, we test the function at each of these points.


We have, /"(x) = 12x-42
Atx = l: We have,
/"(I) = 12-42=- 30<0
So, X = 1 is a point of local maximum.
The local maximum value is /(I) = 2-21 + 36-20 = -3
At X = 6: We have,
/"(6)=12(6)-42 = 30>0
So X = 6 is a point of local minimum.
The local minimum value is /(6) = 2(6)^-21(6)2 + 36x 6-20 = 128.
11.19
MAXIMA AND MINIMA

EXAMI’Ll ' Shozo that none of the follozuing functions has a local maximum or a local minimum:
(i) .Y^ + x^ + x + l (ii) (hi) logx

SOLUTION (i) Let/ (.t) = + .t ^ + ,Y +1. Then, /' (x) = 3x^ + 2.r +1.
At points of local maximum or niinimum, we have
/' (.v) = 0 => 3-V^ + 2.r + 1 = 0
But, this equation gives imaginary values of x. So, /' (a:) 0 for any real value of .v.
Hence,/ (.t) does not have a maximum or minimum,
(ii) Let / (.v) = Then, /' {x) = e'\ Clearly, /' (.v) ^ 0 for any value of ,v.
So,/ (x) =e^ does not have a maximum or a minimum,
(hi) Let/{x) = log X. Then, /' (x) = -. Clearly, / ' (x) ^ 0 for any value of x e Domain (/).

w
X

So, / (x) = log X does not have a maximum or a nunimum.


Find the maximum profit that a compamj can make, if the profit function is given

F lo
EXAMI’I I 4

P(x)=41 + 24x-18x^.

ee
SOLUTION We have,
P(x) = 41 + 24x - 18x^

Fr
2 /

—fp(x)'l
dx
= 24- 36X and -dx y V P (X) = - 36 for
ur
For maximum or minimum, we must have
s
d
P (x) = 0 => 24 - 36x = 0 => X = -
ook
Yo

dx 3

Also,
eB

v2 r 2
K PM - 36 < 0. So, profit is maximum when x = —.
dx^ 0
r

x=2/3
ou
ad

Maximum profit = (Value of P(x) at x = 2/3) =41 + 24 x (2/3) -18 (2/3) = 49


Y

liXAMIM I. At what points, the slope of the curve y = -x^ + 3x^ + 9x - 27 is maximum ? Also, find the
Re
nd

maximum slope.
SOLUTION The slope m of the curve y = -x^ + 3x^ + 9x - 27 at point (x,}/) is given by
Fi

m = ^ = -3x^ + 6x + 9 ...(i)
dx

dm dhn
= -6x + 6 and = -6
dx dx^
For maximum or minimum values of m, we must have
dm
= 0 => 6x + 6 = 0 ^ X = 1
dx

d^m
Clearly, = - 6 < 0 for all .v.
dx^
So, m is maximum at x = 1. Putting x = 1 in (i), we obtain m = 12.
Putting X = 1 in the equation y = -x^ + 3x^ + 9x - 27, we obtain y = -16.
11.20
APPLIED MATHEMATICS-XII

Hence, the slope of the given curve is maximum at the point (1, -16) and the maximum value of
the slope is 12.
' ^ ' ■ iff {x)=alog\x\+bx^ + X has extreme values ntx = -J and at x= 2, then find a and b.
SOLUTION We observe that f{x) is defined for all .v ^ 0.
Now,

fix) = a lo$\x\ + bx^ + x => f (.v) =


a
+ 2i)A 4- 1
A

It is given that f(x) has extreme values at a- = -1 and a = 2.


/'(-I) = 0 and f'{2) = 0
— a — 2b + I — 0 and — + 41? + 1 = 0=>a + 21j=l and a + Sb — —2
2

Solving these equations, we get: a=2 and b=-l/2.

w
\ \ vijm.l;
It is given that at x— 1, the function x^ — 62a^ + ax + 9 attains its maximum value on the

F lo
interval [0,2]. Find the value of a.
SOLUTION Let fix) = A^ - 62a^ + ax + 9. Tlien,
/'(A)=4.v^-124A + r7.

e
Fre
It is given that /(a) attains its maximum at a = 1.
/'(I) = 0=> 4 -124 + = 0 => = 120
for
ax -b
\.\MPLE8 //]/ = has a turning point P (2, - l),ifnd the values of a and b and shoio that
r
(A-1) (A-4)
You
oks

j/ is maximum at P.
eBo

SOLUTION We have,
ax -b ax -b
V =
.2
(a-1)(a-4) a^-5a+4
our
ad

^ _ (a^ -5a + 4) -jax-b) (2a -5)


...(h)
dx
(a^ -5a+4)2
dy _ (4-10 + 4) r?-(2n-I;)(4-5)
dY

b
Re

dx
(4-10 + 4)2 4
Fin

Since P is a turning point of the curve (i). Therefore,


dy = 0=^ = 0 => b = 0
dx 4 ...(iii)

ax - b
Since P (2, -1) lies on y = . Therefore,
(A-1) (A-4)
2a-b 2a ~b
-1 -1 => 2a-b = 2
(2-l)(2-4) -2 ...(iv)

From (hi) and (iv), we get <7 = 1, ft = 0.


Substituting the values of a and b in (ii), we get
dy ^ (.y2 -5a + 4) -a(2a -5) - a2 + 4
dx
(a2 -5a+ 4)2 (.v2 -5a + 4)2
11.21
MAXIMA /. ;v*A

d^y _ {x^ -5x+ 4)^ (- 2.V) - (- + 4) 2(.y^ - 5a: + 4) (2.y - 5)


dx^ {x^ -5x + Af
d^y _ -2x{x^ ~5x+ 4) + 2{x‘^ -4)(2x-5)
dx^ {x^ -5x + ^)^

Now,
dy = 0 and. d^y] (-2) (-4) = -1 <0
dx
A 2,-1) dx^ A 2,-1) (-2)3

So, y is maximum at P whenii=l and b = 0.


. '’^RCISEI.

Find the points of local maxima or local minima and corresponding local maximum and
local minimum values of each of the following functions. Also, find the points of inflection,

w
if any:
(i) /(a:)=:c'^-62.r^ + 120:t + 9 (ii) f{x) =x^ - 6x^ + 9x + 15
(iii) f(x)=(x^l)(x + 2f

F lo (iv) f{x)=2/x-2/x^,x>0

ee
(v) f(x)=xe^ (vi) f{x) =x/2 + 2/x, x>0

Fr
(vii) /(x)=(.t + l)(.v + 2)
1/3
,x>-2 (viii) f{x) =x -j32-x^, -5 < A' <5
for 2

(ix) f(x) =x^ - 2ax^ +a^ x,n>0,xeR (x) /{x)=.v + — , fl > 0, a: 0


ur
(xi) f{x)=x^2-x'^ -4l<x<4l
s

(xii) f{x) ~x + .r < 1


ook
Yo

Find the local extremum values of the following functions :


eB

(i) fix) =(x-l) (x - 2)2 (ii) fix) = x -V < 1 (iii) /(x) = -(a'-1)3(.v + 1)2
The functiony = <7 log + bx^ + x has extreme values at x~l and a: = 2. Find a and b.
our
ad

4. Show that
log^ has a maximum value at a: = e.
X
Y

4
and minimum values of the function fix) = —^
+ X.
Re

Find the maximum


nd

2 + bA; + c has a maximum at x = -1 and minimum at x = 3. Determine


Fi

lif(x)=x^ + ax

a, b and c.
AN,

(i) Local Max. at x = 1, Local Max. value = 68


Local Min. at x = 5, - 6; Local Min. values are - 316 and -1647.
(ii) Local Max. at x = 1, Local Max. value = 19
Local Min. at x = 3, Local Min. value = 15
(iii) Local Max. at x = - 2, Local Max. value = 0
Local Min at x = 0, Local Min value = - 4
(iv) Local Max. at x = 2, Local Max. value = 1/2
(v) Local Min. at x = -1, Local Min. value = - 1/e
11.22 APPLIED MATHEMATICS-XI!

(vi) Local Min. atx = 2, Local Min. value = 2


3
(vii) Local Min. at .t = -7/4, Local Min. value =-
44/3
(viii) Local Max. at .v = 4; Local Max. value = 16
Local Min. at .y = - 4; Local Min. value = -16
4«3
(ix) Local Max. at .v = a/3, Local Max. value =
27

Local Min. at .y = n, Local Min. value = 0


(x) LocalMax. at.Y=-fl, Local Max value = -2(7
Local Min. at .v = n, Local Min. value = 2a
(xi) Local Max, at :y = 1, Local Max. value = 1
Local Min. at .y = -1, Local Min. value = -1

w
(xii) Local Max. at y = 3/4, Local Max. value = 5/4
. .(i) Local Max. value = 4/27 at .r = 4/3, Local Min. value = 0 at y = 2

(ii) Local Max. value =


JTf
(iii) Local Max. value 3456/3125 at x = -1/5; Local
F lo
e
Min. value = 0 at x = - 1

Fre
3.a = -2/3,h=-\/6
Local Max. value = - 6 at x = - 4; Local Min. value = 2 at x = 0.
for
b. a = -3,b =-a, c e R
r
HINTS TO SELECTED PROBLEMS
You

4. We have.
s
ook

log X 1 - log -V
./■W = ^/'(.Y) =
x2
eB

At points of local maximum and minimum, we must have


/' (x) = 0 => Ij-logx = 0 => log X = 1 => X = c
our
ad

x2
1 - log X
Now, /' (x) =
x2
X ^(1-logx)
dY
Re

- 3 - 3
=> /”(x) = -2x (l-logx)-x -X
"^^(3-21ogx)
-3
Fin

=> f"{e) = -c"3(3-21ogc) = ■


^<0.
Hence,/ (.x) has a local maximum value at x = e.

11.6 MAXIMUM AND MINIMUM VALUES IN A CLOSED INTERVAL


Let 1/ =/(.x) be a function defined on [a, 1j]. By a local maximum (or local minimum) value of a
function at a point ce[n,b] we mean the greatest {or the least) value in the immediate
neighbourhood of .y = c. It does not mean the greatest or the maximum (or the least or the
minimum) of/(.x) in the interval [a, 1j]. A function may have a number of local maxima or local
minima in a given interval and even a local minimum may be greater than a local maximum.
Thus, a local maximum value may not be the greatest (the maximum) value and a local
minimum value may not be the least (the minimum) value of the function in any given interval
as shown in Fig. 11.29.
MAXiMA AND MINIMA 11.23

A Q

y=/(-v)

/(rt) D

ow
m

O a b X

Fig. 11.29

e
However, if a function/(.r) is differentiable and consequently continuous on a closed interval

re
\a, b], then it attains the absolute maximum (absolute minimum) at stationary points (points
where /' (.t) = 0) or at the end points of the interval [a, b]. Thus, to find the absolute maximum

Flr
F
(absolute minimum) value of the function, we ch.iose the largest and the smallest amongst the
numbers f(a), f{c{), /(c„), f{b) where .y = Cp c^,.... c„ are the stationary points.
ou
We may use the following algorithm for finding the maximum (absolute maximum) and the

sr
minimum (absolute minimum) of a function/defined on a closed interval [a, b].

ALGORITHM
fo
k
oo
^n-.r I Find f ’ (.t)
STM’II Put f (.r) = 0ami find values ofx. Let c/,C2,....c„ be the values ofx.
Y

’ Ml Take the maximum and minimum values out of the values f(a),f(ci). f(c2). f(b).
reB

sIM

The maximum and minimum values obtained in step HI are respectively the largest (or absolute
maximum) and the smallest (or absolute minimum) values of the function.
uY

ILLUSTRATIVE EXAMPLES
ad
do

i XAMPI I ] Find the maximum and minimum values off{x) =2x^ -24y + 107 in the interval [1, 3].
SOLUTION We have,
in

/(y) =2y^ -24.V + 107


Re
F

/' (y) = 6 Y^ - 24
Now, /' (y) = 0 => 6 Y^ - 24 = 0 ^ Y = ± 2
But, Y = - 2 «[1, 3]. So Y = 2 is the only stationary point.
Let us now compute the values of/(Y) at y =1, 2, 3.
/(1) = 2-24 + 107 =85,/(2) = 2(2)^ - 24 (2) + 107 =75
and. /(3) = 2(3)^ -24x 3 +107 =89
Clearly, largest of these values is 89 and the least is 75.
Hence, the maximum value of f{x) is 89 which it attains at x = 3 and the minimum value is 75
which is attained at y = 2.

liXAMPl r -? Find the maximum and tninimum values off(x) = y^^ - y^^ in the interval [0,1].
19
SOLUTION Let /(y) = Y
30 ..20
- Y .Then, /'(.y) =50y‘*^ -20 Y'^
At stationary points, we must have
11.24 APPLIED MATHEMATICS-XIl

f'{x) = 0
50:e^^ - 20x 19 = 0

/2\1/30
(50;c^° -20) = 0 => ;c = 0 or, 50a: 30 = 20 => a: = 0 or, a: =
V5
The values of f{x) at these points and at the end-points of the interval [0,1 ] are as given below.
xl/30 / 2 ^50/30 /2A20/30 (2 3 f 2^ ^
Now, /(0) = 0,/(|
\5 5; 5; 5;
--1
V5 5 ys
and, /(I) =1-1=0.

Of these values, the maximum value is 0 and the minimum value is —


3(2 ^
■ ■ 5I5.
Thus, the maximum value of f{x) in [0,1] is 0 and the minimum value of/(Ar) in [0,1] IS
i
3(2 ^

w
5 ys

X AMPLE1

F lo
F^nd the absolute maximum value and the absolute minimum value of the function
f{x)= \2^-x] +x^ in [-2,25].

e
SOLUTION We have.

Fre
(I
f{x) = --a: +a:, where Are [-2, 25].
y2
for
fix) = -2(l/2-x) + 3x^ = ~l + 2x+3x^
r
At the points of local maximum and local maximum, we must have
You
oks

/'(a:) = 0=> 3a:^ + 2a:-1 =0 => (3:r -1) (a: +1) = 0 a: = 1/3,-1
eBo

The values of/(a:) at these points and also at the end-points of the interval are computed as given
below.
x2 x2
1
l_i r 1V
+ i-2f=^ _7 1 1
/(-2)= \ ^2 + 2
ad
our

4'-^l3,
+
4 .2 3) 3j 36 ■ 27 108'
^2 X2
+ (-1)3=1 and,/(2.5)=fl-2.5
157
/(-i) = + (2.5)3 =
4 ^2 8
dY
Re

157 7
Of these values, the maximum value of / (a:) is 8
and the minimum value is
4
Fin

157 7
Thus, the absolute maximum - and, the absolute minimum = —
8 4
●-AMI'U- I
Fmd both the maximum and the minimum value of 3x‘^ - 8a:3 ^ ^2^^ - 48a: + lon the
interval [1,4].
SOLUTION Let fix) = 3t^ - 8a:3 + Ux'^ - A8x + 1. Then,
f'(x) = 12a:3-24,t:^ + 24a:-48 and f"(x) = 36a:^ - 48a: + 24
The critical points of/(a:) are given by/'(A:) = 0.
Now, fix) = 0
=> 12.a:^ - 2Ax^ + 24a: - 48 = 0
=> a:
^-2x^ + 2x-4 = 0^x\x-2) + 2ix-2)=0^ix-2){x^ + 2)=0=^x = 2 [vx^ + 2^0]
The values of f(x) at critical points and at the end-points of the interval are computed as follows:
MAXIMA AND MINM 11.25

/(2) = - 59, /(I) = - 40 and /(4) = 257.


Of these values the largest and the smallest values are/(4) = 257 and/(2) = -59.
So, the inmimum and maximum values oi f{x) on [1,4] are -59 and 257 respectively.
EXERCISE 77.4

I. Find the absolute maximum and the absolute minimum values of the following functions
in the given intervals:

(i) f(x)=4x--y in [-2,45] (ii) /(x)=(.r-l)2 + 3in [-3,1]


(iii) fix) = 3-r^ -Sx^ + 12.v^ - 48.r + 25 in [0, 3]
(iy) fix) ={x-2) [1,9]
Find the maximum value of 2.r 24.V +107 in the interval [1,3]. Find the maximum value

w
of the same function in [- 3, - Ij.
3. Find absolute maximum and minimum values of a function / given by
/(x)=m
4/3
-6.V
1/3
,A€[-1,1].

F lo
4. Find the absolute maximum and minimum values of a function / given by

e
fix) =2x^ -15x^ + 36x +1 on the interval [1,5].
Fre ANSWERS
for
1. (i) Absolute Maximum = 8 at : c = 4, Absolute Minimum - -10 at ;c - - 2
(ii) Absolute Maximum = 19 at x = - 3, Absolute Minimum = 3 at x = 1
r
You

(iii) Absolute Maximum = 25 at x = 0, Absolute Minimum = - 39 at x = 2


oks

2 4
eBo

(iv) Absolute Maximum = UV2 at x = 9, Absolute Minimum 3>/3


at X = —
3

'' Maximum value = 89 at x = 3 in [1,3], Maximum value = 139 at x = - 2 in [- 3, -1]


ad

9
our

3. Absolute Minimum, value = —-atx = -. Absolute Maximum value = 18atx = -1


4

4. Absolute Maximum value = 56 at X =5, Absolute Minimum value = 24 at x = 1


Re

‘NTS TO selected PROBLEMS


dY

x^
Fin

(i) We have, / (x) = 4 x 2

=> /' (.x) = 4 - X


f'{x)=0=>x=4:
Now,/(-2) = -8-2 = -10,/(4.5) = 18-10.125 = 7.875 and,/(4) = 16-8 = 8
Absolute maximum is 8 at x = 4.

(ii) We have, fix) = (x-1)^ + 3


/'(x)=2(x-l)
At critical points, we have fix) = 0
fix) = 0 => 2(x-l) = 0 X = 1.
Now,/(-3) = (-3-1)2 + 3 = 19 and/(1) = (1-1)2 +3 = 3.
Absolute maximum = 19atx = -3, Absolute minimum = 3 at x = 1.
11.26 APPLIED MATHEMATICS-XII

(iii) We have,
f{x) = 3x‘^-Sx^+12x^-^8x + 25^ f'{x) = 12.y^ -24;t^ + 24:t -48
At critical points, we have/'(a:) = 0.
■■■ f'(x) = 0
=> 12a:^-24x^+24a:-48 = 0
=> 12(a:^ -2x^ + 2x-4)=0 => (a:-2) + 2(.r -2) = 0 => (a: -2) (a:^ + 2) = 0 a: = 2.
The value of f{x) at critical points and at the end-points of interval are

/{O) = 25,/(2) = 48-64 + 48-96 + 25 = - 39

ow
and,/(3) = 243 - 216 + 108-144 + 25 = 16
Of these values the greatest and the least are /(O) = 25 and /(2) = -39 respectively.
Absolute maximum = 25 at .y = 0, Absolute minimum = - 39 at x = 2.

2. Let/(x) = 2y^-24x +107. Then,

e
fix) = 6x^ - 24 = 6 (.V - 2) [x + 2)

re
Frl
At critical points, we must have/'(x) = 0

F
fix) = 0=> 6(y-2)(.v + 2) = 0 => A' = -2, 2
If/(x) is defined on [1,3], Then/'(x) = Oatx = 2.
ou
or
The values of /(x) at critical points and at the end-points of the interval are:

/(I) = -85,/(2) = 75and/(3) = 89.


kfs
Absolute maximum = 89 at x = 3
If/(x) is defined on [-3,-1], then/'(x) = Oatx = -2.
oo

The values of /(x) at critical points and at the end-points if the interval are;
Y
B

/(-3) = 125, fi-2) = 139, and/(-I) = 129


So, the absolute maximum is 139 at x = - 2.
re

.3. We have,/(x) =12 x^^ ^-6 x^^^


oYu

2(8x-l)
/'(x) = 16x^/3-2x'2/3
ad

x2/ 3
d

At critical points, we must have f'(x) = 0.


2(8x-l) 1
in

/'{x)=0^
Re

= 0 => x =
^2/3
F

The values of /(x) at critical points and at the end points of interval are:

/(-I) = 12 + 6 = 18,/|-
K8 I and / (I) = 6
1
Absolute maximum = 18 at x = -1 and. Absolute minimum = - - at x
4 8

We have, / (x) = 2x^ - 15x^ + 36x + 1


/ '(x) = 6x^ - 30x + 36 = 6 (x^ -5x + 6)
At critical points, we must have /' (x) = 0.
f'(x) = 0 => 6(x^-5x+6) = 0 => x = 2, 3
The values of /(x) at these points and end points of interval are:
/(I) = 24, /(2) = 29,/(3) = 28 and/(5) = 56
Absolute maximum = 56 at x =5 and. Absolute minimum = 24 at x = 1
11.27
MAXIMA AND MINIMA

11.7 APPLIED PROBLEMS ON MAXIMA AND MINIMA


In this section, we will discuss some applied problems on maxima and minima for which
following results will be very useful,
(i) For a square of side x:
●y
Area = x , Perimeter = 4.r.

(ii) For a rectangle of sides x and y:


Area = xy. Perimeter = 2 (.r + y).
(iii) For a trapezium:
i (Sum of parallel sides) x (Distance between them),

w
Area =

(iv) For a circle of radius r:


Area = k , Circumference = 2n r.

o
(v) For a sphere of radius r:

e
re
4 2
Volume =—71
3
Surface Area =4Tur .

rFl
F
(vi) For a right circular cylinder of base radius r and height h:
Volume = nr^ h, Surface = 2tc rh + 2tc , Curved surface = 2n rh.

r
ou
fo
(vii) For a right circular cone of height h, slant height I and radius of the base r:
ks
Volume = (1/3) n rhi, Curved surface = n rl, Total surface = ti r 2 + KTl.
(viii) For a cuboid of edges of lengths x, y and z:
oo

Volume = xyz, Surface -2{xy + yz + zx).


Y
eB

(ix) For a cube of edge length .t;


Volume = .r^. Surface Area = 6 x^.
r
ou
Y

(x) Area of an equilateral triangle =—^{Side) .


ad

l/k
UF.MARK Ifkisapositivecxjnstant, tlienaJunctionoftJwfimnkfix), k + f{x), {f (:r)}^, {f{x)\ / log f{x)
d

will be maximum or minimum according as f(x) is maximum or minimum provided that f{x) > 0.
Re
in

ILLUSTRATIVE EXAMPLES
F

Find hvo numbers whose sum is 24 and whose product is as large as possible.
SOLUTION Let the numbers be x and y. Then,
x + y = 24 (given)
Let P be the product of these numbers. Then,
p = xy = x(24-x) [Using (i)l
=> P = 24x -

dP d^P
=> = 24 - 2.r and = -2
dx dx'^
dP
The critical points of P are given by dx
= 0.

dP
= 0 => 24-2.V = 0 => X = 12
dx
11.28 APPLIED MATHEMATICS-XII

Also, = - 2 < 0. So, P is maximum when x = 12.


dx^ x=l2

Putting .T = 12 in (i), we obtain y = 12. Hence, the required numbers are both equal to 12.
I x.AMPi.n:
Find two positive numbers x and y such that a: + y = 60 and xy^ is maximum.
SOLUTION Let P = .vy It is given that x + y = 60. Therefore, .t = 60 - y.
Now, P=xy^
P=(60-y)y3=60y3-y4
dP d^P
= 180y^ -4y^ and = 360y-12y^
dy^
dP
The critical points of P are given by = 0.

w
dP
= 0.

F lo
dy
180y 4y -0=i> 4y (45-y)-0=> y - 0, y = 45 => y = 45 [■.■ y = 0 is not possible]

ee
d~P
Now, = 360 X 45 -12(45)2 = 12 x 45 (30 - 45) = - 8100 < 0

Fr
w ^y = 45
for
So, P is maximum when y = 45. Putting y = 45 in a: + y = 60, we obtain a: = 15.
Hence, xy'^ is maximum when a: = 15 and y = 45.
ur
i ^ NMi’Li: 3 Find two positive numbers x and y such that their sum is 35 and the product x^ / is
s
ook
Yo

maximum.

SOLUTION Let P=a;2/. It is given that


eB

A; + y = 35^A: - 35- y
Putting a: = 35 - y in P = Af^y^, we get
our
ad

P = (35-y)2y5
dP
2(35-y)/+5(35-y)2 y'
Y
Re

dP
nd

=>
= (35-y)y^{-2y + 5(35-y)}
Fi

dP

^y
= y^ (35-y) (175-7y) = 7y^ (35-y) (25-y)
dP
The critical points of P are given by = 0.
dy
dP

dy
= 0 => 7y^^ (35-y) (25-y) = 0 y = 0, 25, 35
But, y = 0 and y = 35 are not possible. So, y = 25.
Now, ^
dy^
= 28y^ (35-y) (25-y)-7y^ (25-y)-7y^ (35-y)
d^P
= -7(25)^(35-20) = 7 (25)^ (10) <0
dy^ 7y = 25
11.29
MAXlMAAf ilNlMA

Thus, P has maximum when y = 25. Putting i/ = 25 in (i), we obtain x -10.


Hence, is maximum when x = 10, and y = 25.
. Wil’U-Amongst all pairs of positive numbers with product 156, find those zuhose sum is the least.
SOLUTION Let the required numbers be x and y. Then,
...(i)
xy = 256 (given)
Let S = X + y. Then,
256
S = X + [Using (i)]
X

dS 256 d^ S 512
= 1 - — and.

w
dx X dx^ x3
dS
The critical points of S are given ~

o
dS 256
— = 0 => 1 - = 0 => x^ = 256 ^ X = 16

e
dx x2

re
Now,
(d^S 512 1
= ->0

rFl
F
16
(16)3 8

Thus, S is minimum when x = 16. Putting x =16 in (i) we get y =16.

r
ou
Hence, the required numbers are both equal to 16. fo
ks
[●XAMi'LB 5 Find two positive numbers whose sum is lAand the sum of whose squares is minimum.
SOLUTION Let the numbers be x and y. Then,
oo

x + y = 14
Y

Let S be the sum of the squares of x and y. Then,


eB

S = x^ + y^
S = x^ + (14 -x)^ [Using (i)]
r

=>
ou

S = 2x^ - 28x +196


Y
ad

dS /s
=> — = 4x - 28 and = 4
dx^
d

dx
dS
Re
in

The critical points of S are given by — = 0.


F

dS
= 0 => 4x-28 = 0=> x=7
dx

d^S
Clearly, = 4>0
dx^
Thus, S is minimum when x = 7. Putting x = 7 in (i), we obtain y - 7.
Hence, the required numbers are both equal to 7.
iXAMPLEt Find the minimum value ofax + by, where xy = c^anda, b, care positive,
fj

SOLUTION Let2=flx + l7y,wherexy = c .Then,


2
bc'^ 2
z = ax + ...(i) V xy = c =>!/ = -
X
X

dz bc^ d^z 2bc^


— =a-
-^and—r
dx x^ dx^ x3
11.30 APPLIED MATHEMATICS-XII

dz
The critical points of 2 are given by —
dx
= 0.
dz
= 0
dx

bc^ bc^ b
a- = 0=>x^ = => -V = ± - c
x^ a a

At X = - c: We find that
a

^3
d^z a
= 2bc^
a
-X- = 2- - >0
dx^ b c c \b

w
So, 2 is minimum at x = c. - .
a

The minimum value of 2 is given by

Flo
fc + —
z=aj- bc^
Putting X = -cin(i)

e
a c

re
a

‘b

rF
At .V = - - c : We find that
a

d\
ur
a a
= 2bc^ -
a a

dx^ bc^ \b
= -2-
c \b
- <0 fo
ks
So, 2 is maximum at a: = c.
Yo
oo

I.XAMCLL 7
Show that all the rectangles with a given perimeter, the square has the largest area.
B

SOLUTION Let x and y be the lengths of two sides of the rectangle of fixed parameter P, and let
re

A be its area. Then,


P = 2(.v + y)
u
ad

and, A = xy
Yo

●●●(ii)
Now,
P = 2(.Y + y) ^ y=| -X
d

(P Px
Re

A - xy = X Y
in

V2 2

d^ A
F

^ _ P 2.V and = -2
dx ~ 2 dx^
dA
The critical points of A are given by rfY
= 0.

dA P
= 0 =>
dx --2y = 0=> P=4:X=> 2x + 2y=4x => 2x = 2y => x=y

d- A
Clearly, = - 2 < 0.
dx^
'x = y

Hence, A is maximum when x=y i.e. the rectangle is a square.


PXAMPl.L 8
Show that of all the rectangles of given area, the square has the smallest perimeter.
SOLUTION Let y and y be the lengths of two sides of a rectangle of given area A, and let P be the
perimeter.Then,
MAXIMA AND MINIMA 11.31

A = xy ●●●(i)
and, P = 2{x + y) ...(ii)

Now, A = xy => y = -
X

A
P = 2(x + y) = 2 x + -
X j

dP A d^P 4A
dx = 21 —2
X dx^ x-3
dP
The critical points of P are given by dx
= 0.

-■^=0 => 2 1- — = 0 => 1 - A = 0 => x^ = A => x^ = xy => X = y.


dP A

x'^

w
d^ P 4A
Clearly,
dx^ X
^ > 0 for all positive values of x.

F lo
Hence, P is minimum when x = y i.e the rectangle is a square.
EXAMl'LC 9 S}ww that of all the rectangles inscribed in a given circle, the square has the maximum area.

ee
SOLUTION Let ABCD be a rectangle inscribed in a given circle with centre at O and radius a. Let

Fr
AS = 2x and BC = 2y. Applying Pythagoras theorem in right triangle 0AM, we obtain
OA^ = AM^ + OM^ => = x^ + y^ => y = - x2 for
ur
Let A be the area of the rectangle ABCD. Then,
s

4 x^a'^ -x^
ook

A = 4 xy =
Yo

[Using (i)]
eB

dA x2 «2-2x2
= 4*^ D, C
dx - x“
r
ou
ad

0
dA 2y
The critical points of A are given by dx
= 0.
y
Y

A B
dA
Re
nd

= 0
dx
Fi

a
■ = 0 => - 2x^ = 0 => X =
" V2

dA
Now, 4 \ ■ >
dx

d^A
dx-2
= 4—
dx
I ((7^ - 2x^) (fl^ - x^) 1/2

d^A
= 4 -4x(rt^ - X V'’^^+(t72_2x^) (-1/2) (fl2_.v2)-3/2(_ 2x)
dx2
d^A -4x x{a^ - 2x^)
= 4-^
dx^
11.32 APPLiZI MA'rlEMATICS-Xil

A
= -16<0.
dx'^
x = al4.i
a a a
Thus, A is maximum when a: = ● Putting -V = in(i), we get ]/ = -.= ●
V2 V2 V2
Therefore, x=y = a/4l ^ 2x = 2y = ^a=> AB = BC => ABCD is a square.
Hence, area A is maximum when the rectangle is a square,
● AMI'I i
Show that the rectangle of maximum perimeter which can be inscribed in a circle of radius
a is a square of side 42a.
SOLUTION Let ABCD be a rectangle in a given circle of radius a with centre at O. Let AB = 2x
and AD = 2y be the sides of the rectangle. Applying Pythagoras theorem in AOAM, we get
Am2+oM^=OA^ => a
^ y = -x^ ...(i)
Let P be the perimeter of the rectangle ABCD. Then,

w
P = 4x + 4y
P = 4x +4 -Ja^ -x^
dP
dx
= 4-
4x

F lo
[Using (i)]

e
Fre
dP O
The critical points of P are given by dx
= 0.
for 2y
y
dP
= 0 A B
dx X
r
4x
You

= 0
o ks
eBo

4.V

-x^

-x^ = X => -x^ = => 2x^ =


our

a
ad

2
a => ,r =
42
dP 4x
Now, 4- .
dx
dY
Re

X (- x)
Fin

=>
-4.2
dx^ (.2_;,2)3/2

d^P -4.2 -8V2


<0
dx^ 2n3/2 a
^ x = a/4l a
2 a
2

Thus, P is maximum when x =


42
a a
Putting a: =
42 in (i), we obtain y = 42'
a: - y = a42 => 2x = 2y^ AB - BC ^ ABCD is a square.
11.33
MAXIMAANO' '"liMA

2n
Hence, P is maximum when the rectangle is square of side lx = V2 = V2fl.

AB is a diameter of a circle and C is any point on the circle. Show that the area of AABC is
maximum, when it is isosceles.
SOLUTION Let AB = la, AC = X and CB = y. Since AB is a diameter of the circle having centre
K

O andC is a point on the semi-circle ^C6. Therefore, ZACB = -.


Applying Pythagoras theorem in AACB, we obtain
ab'^ =AC^+CB^
(2af=x‘^+i/

ow
y =
Let A be the area of A ACB. Then,
1
A=-ACxCB =
1 i^y

e
re
-X
2
-x^

Fl [Using (i)J

F
.2
dA x^ la^ -.r
- ■ ^4a^ -
ur
r
dx 1

The stationary values of A are given by


dA
= 0.
fo
ks
dx
Yo
dA
oo

= 0
dx
eB

la^-x^
= 0 => la^ = a:^ => .T = 4la
4^ .2
ur

dA
Now,
ad

dx
Yo

^4(7“ -x"^ X -2-V -{la^ -x^) X


-Y

AZIZ x{6a^ -.Y^)


d

d^A
Re
in

d.Y2
Vi? -.Y
(4.2-..2)3/2
F

d^A
●2<0
dx'^ x=4la
Thus, A is maximum when .y - ^/2. and y = 4la.
Hence, the area of AABC is maximum when it is isosceles.
i:x AMPLH12 If the sum of the lengths of the hypotenues and a side of a right angled triangle is given,
show that the area of the triangle is maximum ivhen the angle between them is n/ 3.
SOLUTION Let ABC be a right angled triangle with base BC = x and hypotenues AC = y such
that x + y^k, where Jc is a constant. Let 0 be the angle between the base and hypotenues. Let A
be the area of the triangle. Then,
A = -BCxAC =
2
11.34 APPLIED MATHEMATICS-XII

.2
^2 =

/l2 =
^P-.v)2-a-2} [v x + y^k]

=
k^x^-2kx^
... 1
4

Differentiating with respect to x, we get


ciA 2k^x - 6kx^
2A
dx
●●●(ii)
4

dA k'^x - 3/c.y^
dx 4A

low
dA
The critical numbers of A are given by dx
= 0.

dA k^x- 3kx-
Now, = n0 => JC = —.
^
dx 4A 3

Differentiating (ii) with respect to .r, we get

ee
if —
\2
d^A
F
2k^-l2kx

Fr
+ 2A
dx rf.v2 4 ...(iii)

When ,v =
k dA dA k . ..... for
ur
3" dx = 0. Putting
- 0 and X = — in (ill), we get
dx
ks
<0.
dx^ 4A
Yo
oo

k
Thus, A is maximum when x - —. Putting .V = — 2k
in .V + y = k, we obtain y = —.
eB

3 -^3
BC X k/3 1 K
InAACB, cosO = => cos 6 = — => cos 0 = = - 0 = -
AB 2k/3 2
r

y 3
ou
ad

Thus, area of triangle ABC is maximum, when angle 0 between base BC and hypotenuse AB is
n/ 3.
Y

; XAMiM.l'. I,’? Prove that the area of right-angled triangle of given hi/potemies is maximum when the
Re
nd

triatigle is isosceles.
Fi

SOLUTION Let h be the hypotenues of the right-angled triangle, and let .vbe its altitude. Then,
Base of the triangle = -Jh^ -x^.
Let A be the area of the triangle. Then,
A

dA
1/2 d
dx 2 ^ dx

dA h^-2x^
I
1
=>
dx 2

A li
Fig, 11.34
MAXIMA AND MINIMA 11.35

dA
The critical numbers of .r are given by = 0.
dx

dA 1 h^-2x^ h
= 0 => ● = 0=> = 2x^ => X =
■ V2
— ●

dx 2

Now,
dA _l h~-2x^
dx^2\^2^
d^A 1 , . , 1
=2 + {/(^-2x")
- 3/2
Uh^-2c^)\
dx^ ^ (-4X-) \ 2) dx

d^A 1 X {h^ - 2.r^)

low
-4a-

dx^ - X 2^(/,2_,,2)3/2
f d~ A
=> = -2<0.
dx^ h

ee
-r-
■J2

F
Fr
h h h^] h
Thus, A is maximum when a = . Putting A = in (i), we get: Base = }?- 2 V2’
sf2 V2
for 1
ur
AB = BC=4
V2
k s

Hence, A is maximum when the triangle is isosceles.


Yo
oo

llXAMrLE i4 Show that the surface area of a dosed cuboid with square base and given volume is
eB

minimum, when it is a cube.


SOLUTION Let V be the fixed volume of a closed cuboid with length a, breadth a and height y.
r

Let S be the surface area of the cuboid. Then,


ou
ad

...(i)
\/ ^ x^y
Y

and. S = 2 (a^ + Ai/ + .ry) 2a^ + 4xy


Re
nd

Now, S = 2a^ + 4A-y


y ^ 7 V
Fi

S = 2a^ + 4a V y =A^ y .-. y = -^ A .


A A

4y
S = 2a^ +
A

dS 4y
— = 4a ...(iii)
dx a2
dS
The critical numbers of S are given by —
dx
= 0.
dS
= 0
dx
AV
4a = 0
a2
y = A^
11.36
’"OMATHEMATICS-XII

[■:V=x^y\
.T = y.

Differentiating (iii) with respect to .v, we get


d^S 8V Sx^y
= 4 + = 4 + = 4. + -d-
8y
dx^ .v3 ,v3 -T

f j2
d^S
= 12 > 0.
dx^
y = x

ow
Hence, S is minimum when length = .v, breadth = a: and height = .v i.e., when it is a cube.
' ■ M!’i.r i ● An open tank with a square base and vertical sides is to be constructedfrom a metal sheet
so
as to hold a given quantity of water. Show that the cost of the material will be least when depth of the

e
tank is half of its width.

re
SOLUTION
Let the length, width and height of the open tank be .v, ,r and y units respectively.
Then, its volume is .v^ y and the total surface area is x^ + 4xy.

Flr
F
It is given that the tank can hold a
given quantity of water. This means that its volume is
constant. Let it be V. Then,
= -v^ y
ou
sr
fo
The cost of the material will be least if the total surface area is least. Let S denote the total surface
area. Then,
S = .Y^ + 4Yy
k
oo
...(h)
We have to minimize S subject to the condition that the volume y is constant.
Y

Now,
reB

S = + 4x1/
4y
S - Y^ +
uY

Y
[Using (i)]
y
dS 4V d^s 8V
— = 2y-
Y and
ad
do

dx X dx-
y
V
JQ X
y
The critical numbers of S are given by— = 0.
in

dx
dS
Re

Now, — = 0
Y
dx
F

4V Fig. 11.35
2y- = 0
Y
.2

2y^ = 4V
2x^ = 4y^ 1/ [●●● y = Y^y]
Y = 2y
d^ S 8V
Clearly, = 2 +
^ > 0 for all Y.
dx^ X

Hence, S is minimum when y = 2y i.e. the depth (height) of the tank is half of its width.
UXAMlTt. li: A metal box with a square base and vertical sides is to contain 1024 cm^ of water, the
material for the top and bottom costs ? 5 per cnp- and the material for the sides costs ? 2.50 per cm^.Find
the least cost of the box.
11.37

SOLUTION Let the length, breadth and height of the metal box be .v cm, x cm and 1/ cm
respectively. It is given that the box can contain 1024 cm' of water.
1024
1024 = .v^i/=> i/ =
,v2
Let C be the total cost in ? of material used to construct the box. Then,
C = 5x^ + 5x^ + - X 4.VV
2

C = lO.Y^ + lO.vy
We have to find the least value of C.
1/
Now,

ow
C=10Y^+10Yy ■/
y

C^IOy^ + IO-Vx"^-^^ [Using (i)) X

X
u

10240 X
C =10.Y^ +

e
.V X

re
dC 10240 , d^c 20480 Fig. 11.36

rFl
= 20 +
dx
= 20-y 2“ dx^ .r3

F
x
dC
The critical numbers forC are given by = 0.
dx

or
ou
Now,
dC
10240 ^0^ .v^ =5012=> .Y^ =8^^ -y=8
ksf
= 0 => 20.V -
dx x~

d^C 20480
oo

Also,
dx^
= 20 +
^>0
Y
B

Thus, the cost of the box is least when y = 8. Putting y = 8 in (i), we obtain y = 16. So, the
re

dimensions of the box are 8x 8x 16.


Putting Y = 8 and 1/ = 16 in C = IOy^ + IOyi/, we obtain C = 1920.
oYu
ad

Hence, tlie least cost of the box is ?1920.


An open box with a square base is to be made out of a given quantify of card board ofarea c
d

cubic units.
square units. Show that the maximum volume of the box is y
6^3
in
Re

SOLUTION Let the length, breadth and height of the box y, y and 1/ units respectively, it is given
F

that the area of the card board is sq. units.


-.(i)
Y^ + 4.V1/ =
Let V be the volume of the box. Then,
...(ii)
V = Y^ y
V = Y^ [Using (i)]
4y

3
Y
u = Y -
4 4

dV d-V 3y
and
dx 4 4 dx^ 2
dV
The critical points of V are given by dx
= 0.
11.38
APPLIED MATHEMATICS-XH

dV 3v2
Now, = 0 => =0 ^ X = 4
dx 4 4 y[3
-3c
Clearly, < 0. Thus, V is maximum when a" = 4
dx^ 2V3 V3
^at = c/V3

Putting x = c/43 in (i), we obtain i/ = c/273


C C
Putting X = and y = in (ii) the maximum volume of the box is given by
c c3
cubic units
3 "" 2 V3 6 73'
EXAMPLE 18 The sum of the surface areas of a rectangular parallelopiped with sides x, 2x and - and a
3
sphere is given to be constant. Prove that the sum of the volumes is minimum, ifx is equal to three times
the radius of the sphere. Also, find the minimum value of the sum of their volumes.
SOLUTION Let}/ be the radius of the sphere and let S be the constant value of the sum of the

w
surface areas of the parallelopiped and the sphere. Then,

or. S = 6x^ +4h4


●' 4
— KU
S''
-3 + —XX
S = 2 X X 2.V + 2.r X —
3

3
J + 4jt4

Let V be the sum of the volumes of the sphere and the parallelepiped.
r.
+ XX 2xx —
X
3
F lo ree
for F Then,

^ 3 2 X..3
3 3
Your

/ \3/2
oks

4 S-6.r“ 2 3
V = —K + -.V^ S - 6x^
S = 6.v^ + 47U/“ => 1/2 _
eBo

3 471 3 4ti
1
V =
6^^K
(5-64)3/2+2^3
ad

3
our

dV 1

dx
=—J=X —
6fn
^(S-6.v2)1/2
2
(-12.v) + -x
3
3.v^
dV
=- (5 -6.4)^/2.y + 2.4
Re

...(ii)
dY

dx

dV
Fin

The critical numbers of V are given by dx


= 0.

Now,
dV
= 0
dx

yfn
4{S-6x^)^/2x + 24 = 0
3.V

Ttt
(S-64)’/2^24

4(s-64)'/2
Vt:
= 2.r

=>
9(S-64)=4tlv2 [Squaring both sides]
9 (4711/2) _ ^^^.2 [Using (i)j
MAXIMA AND MINIMA 11.39

9xf- =x^
x = 3y
^ 2 ^TtJ

Putting x = >?y or, y = -^ in (i), we obtain S = 6x + —


Differentiating (ii), we obtain
rfV 18x^ + 4x
dx^ Vtt Jn a/s - 6-i'^

When .t = 3i/ or, 1/ = ^ , we obtain


xl/2
d^V 3 f 4tzx^ 18x^ + 4x = -2.V +
27 X
+ 4.\- =
27 X
+ 2-t > 0
dx^ (2-^nx
r n

w
n
U

So, V is minimum when .r = 3y.

F lo
X 4 2 2..3
^X
, we obtain
PuttingA.- = 3yor, i/=^inP=—rcy +-
>j O w

fx~\^ 2 .V-3=1 .v3 1 + ^

ee
2n
V =-K +

Fr
3 I 3; 3 3 27

Hence the sum of the volume is minimum when .r = 3y i.e. x is equal to three times the radius
for
' .2 f 2n''
of the sphere and the maximum value of the sum of the volumes is P =—x 1-1- 27 ■
ur
i;XAMl’LE 19 A wire of length 36 m is to be cut info tzvo pieces. One of the pieces is to be made into a
s
ook

square and the other into a circle. What should be the lengths of the two pieces, so that the combined area of
Yo

the square and the circle is minimum?


eB

SOLUTION Let the length of a side of the square be .y metres and the radius of the circle be
y metres. It is given that the length of the wire is 36 m.
our
ad

4.Y + 2t: y = 36
2y + Tcy = 18
Let A be the combined area of the square and the circle. Then,
dY
Re

A = x^ + n y^ ...(ii)
Fin

18 - 2.V a2
A = x^ + 71 [Using (i)l
71

A = y^ + 1(18-2.y)^
71

dA d^A 4

dx
= 2x+1(18-2y)(-2) =
n
2.Y- - (18 - 2.y) and.
K dx-
= 2--(-2) =
71
2+ -
■K

dA
The critical numbers of A are given by = 0.
dx
36
dA
dx
= 0 => 2.Y -1 (18 - 2.y) = 7t
0 => -Y =
7: -f 4

d^A 8
Clearly, = 2 + ->0
dx^ 7 ,v = 36 / Ji + 4 K
11.40
'JEDMATHEMATICS-XII

36 36 18
Thus, A is minimum when x = . Putting X =
t: + 4 71 + 4
in (i), we obtain y =
71+4

So, lengths of the two pieces of wire are

36 144 18 36 7T
4.\- = 4x
71 + 4 71 + 4
m and 27uy = 2tix m
71+4 71 + 4

Hence, the combined area of the square and the circle is minimum when the lengths of two
144 3671
pieces are metres and metres.
71+4 7t: + 4

' WH’LEl
A figure consists of a semi-circle with a rectangle on
its diameter. Given the perimeter of
the figure, find its dimensions in order that the area may be maximum.
SOLUTION Let ABCD be a rectangle and let the semi-circle be described on side AB as diameter.
Let AB = 2x and AD = 2y. Let P be the perimeter and A be the area of the figure. Then,

w
P = 2x + 4y + 71 .r ... 1.
2

= (2.v)(2y) + ^
F lo
and, A
...(h)

TtX^
Now A = 4x1/ +

e
2

Fre
2 A B
. TtX
A = x(P-2x- 71 X) + [Using (i)] for
2
2</
.2
2 ^
A = Px - 2x^ - 71 X +
r
2 D Zr C
You

.2
oks

n X Fig. 11.37
A = Px-2x^ ~
2
eBo

dA d^A
= P - 4x - 71X and = -4-71
dx dx^
our
ad

dA
The critical numbers of A are given by dx
= 0.

dA P
= 0 => P - 4x - 71X = 0 => X =
dx
dY
Re

71+4

d^A
Fin

Clearly, - - 4 - 7i< 0 for all values of x. Thus, A is maximum when x = —~—


dx2 71+4
P p
Putting X = in (i) we get y
71+4 2(71+4)
So, area of the figure is maximum when dimensions of the figu re are:
2P p
Length = 2x = and Breadth = 2y = .
7T + 4 7t + 4

: \ \MI’1,L2. A square piece of tin of side 24 cm is to be made into a box without top In/ cutting a square
from each corner and folding up the flaps to form a box. Whatshoidd be the side of the square to be cut
off so that the volume of the box is maximum? Also, find this maximum volume.
SOLUTION Let x cm be the length of a side of the square which is cut-off from each corner of the
plate. Then, dimensions of the box as shown in Fig. 11.38 are Length = 24 - 2x, Breadth = 24 -2x
and height = x.
MAXIMA AND MINIMA 11.41

24-2.V.

I -T I
I I-*
H
X

I 24 cm
(N
●L.
r-Ts
I
.T 1

24 cm

Fig. 11.38

ow
Let V be the volume of the box. Then,
= (24 - 2x)^ X = 4.v^ - 96-V^ + 576.r
dV d~V
=> = 12.i:^-192.\+576 and = 24.V-192

e
dx dx~

re
dV
= 0.
The critical numbers of V are given by

Frl
dx

F
dV
= 0
dx
ou
12.v^-192.y + 576 = 0 ^ .y^-16.y + 48 = 0 => (.y -12) (.y - 4) =0 => y = 12, 4

r
But, Y = 12 is not possible. Therefore, y = 4.
so
kf
d^V
Clearly, = 24 X 4 -192 < 0. Thus, V is maximum when y = 4.
oo
dx^ /' .t«4
Y

Hence, the volume of the box is maximum when the side of the square is 4 cm.
eB

Putting y = 4 in V =(24-2.y)^y, we obtain that the maximum volume of the box is given by
V=(24-8)^x4=1024cm^.
ur
oY

● '.H’l.L 11
A rectangular sheet of fix perimeter with sides having their lengths in the ratio 8:15 is
ad

converted into an open ‘'rectangular box bi/folding after removing squares of equal area from all four
corners. If the total area of removed square is 100 square units, the resulting box has maximum volume.
d

Find the length of the sides of the rectangular sheet.


in

SOLUTION Let the sides of rectangular sheet be 8^ and 15rt units respectively. Let the length of
Re

each side of the squares of same size removed from each corner of the sheet be y units. Then, the
F

dimensions of the open box, formed by folding up the flaps, are;


Length = 15a - lx, breadtli = 8(7 - 2.v, height = y
●15fl-2Ar

I .V I

.V

.Y

.V I

15fl - Iv

Let V be the volume of the box formed. Then,


K = {15a - 2.y) (8(7 - 2y) y
=> V = 120(7^ Y - 46t7Y^ + 4y^
11.42
APPLIED MATHEMATICS-XII

dV
= 12Qa~ - 92ax + 12x^ and = -92fl + 24.v
dx dx'^
dV
The critical numbers of V are given by dx
= 0.

dV
= 0
dx

=> nOa'^ -92ax + 12x^ =0 => 30a^-23ax+3x^ = 0 => (5a-3a:) (6a-a:) =0 => x = 6a,x=~
3

Bu t a: = 6fl is not possible as for x = 6a bread th = 8« - 12rt =


-4a, which is not possible. So, .v = —,
When X = — 5a ^ - -92rt + 40fl - -52fl < 0. Thus, V is maximum when .r =
5a
3 dx^ 3

w
It is given that total area of four squares removed from each comer of the sheet is 100 sq. units.
4x^ =100^ x^ =25^?^=25:^ a^ =9=> a = 3

Flo
9

Hence, the dimensions of the sheet are 15f? = 45 and 8a = 24.

ee
EXAMPLE23
Show that a cijlinder of a given volume which is open at the top, has minimum total surface

Fr
area, provided its height is equal to the radius of its base
SOLUTION Let r be the radius and h be the height of a cylinder of given volume K. Then,
V ^ nrhi => /i = for
ur
nr

Let S be the total surface area of the cylinder which is open at the top. Then,
k s
S = 2nrh + nr^
Yo
oo

V 2
S = 2nr X
eB

+ nr
7tr
2 [Using (i)]
2V 2
S = + nr
r

r
ou
ad

dS 2V
dr r^
^ + 2;tr ...(h)
Y

JC

The critical numbers of S are given by — = 0.


nd

dr
Re

dS 2V
— = 0 => -
^ + 27ir = 0 => V = nr^ => nr^h = Tur^ => /j = r [v V = nrh]
Fi

dr

Differentiating (ii) with respect tor, we get


d^S 4V
— + 2n
dr^ r^
d^S 4V
+ 2ti: > 0.
dr^ h^

Hence, Sis minimum when h=r i.e., when the height of the cylinder is equal to the radius of
the base.

I-;XA MIM.I- 2; Show that the height of the closed cylinder of given surface and maximum volume, is equal
to the diameter of its base. ^
SOLUTION Let r be the radius of the base and h be the height of a closed cylinder of given
surface area S. Then,
MAXIMA AND MINIMA 11.43

S = 2nr^ + 2nrh
S-2%r^
h = ... 1

2nr

Let V be the volume of the cylinder. Then,


V = K rh

V = nr
2f S-27cr^ [Using (i)]
2k r

rS-2nr^ rS 3
V = -nr

ow
2 2
dV
=i> ^-3k
dr 2
dV
The critical numbers of V are given by = 0.
dr

e
dV
= 0 => - - 3nr^ = 0 => S = 6jir^ => 27tr^ + 2k rh = 6kt^ => h = 2r.

Fl
re
Now,
dr 2

F
d^V
Differentiating (ii) with respect to r, we obtain = - 6nr <0for all r.
dr^
ur
Hence, V is maximum when h=2r i.e., when the height of the cylinder is equal to the diameter
or
sf
of the base.

i-:\AMi’Ul 25 Show that the height of a cylinder, which is open at the top, having a given surface area and
k
Yo

greatest volume, is equal to the radius of its base.


oo

SOLUTION Let r be the radius and h be the height of a cylinder of given surface S. Then,
B

S = + 2k rh
re

S-Tir^ ●●● 1
h =
2k r
u
ad

Let V be the volume of the cylinder. Then,


Yo

Y = Tir^ h
d

2
[Using (i)]
Re

V = nr
2k7~
in

V
F

Sr-7cr^ Sr nr^
=> V =
2 2 2
dV S 3 2 ...(ii)
— Ttr
dr 2 2
dV
The critical numbers of V are given by dr
= 0.

dV
= 0 => =0 => S = 37cr^ => nr'^ + 2k rh = 3nr^ => r = h.
dr 2 2

Differentiating (ii) with respect to r, we get


dV
= -3nr < 0.
dr^
Hence, V is maximum when r —h i.e., when the height of the cylinder is equal to the radius of its
base.
11.44 M r L.&D MATHEMATICS-XI!

' \ !
A telephone compani/ in a town has 500 subscribers on its list and collects fixed charges of
^ 300 per subscriber. The company proposes to increase the annual subscription and it is believed that
every increase ofX 1 one subscriber will discontinue the service. Find what increase will bring maximum
revenue?

SOLUTION
Let the increase of ? .v in annual subscription of ? 300 maximize the profit of the
company. Due to this increase of ? x, x subscribers will discontinue the service. Therefore,
Number of subscriber using the service =500 - .v
Annual subscription of each subscriber = ? (300 + .v)
Let R be the total annual revenue of the company. Then,
R =(500-a-)(300 + :c)
R = 150000+ 200i'-.r2
dR d^R
= 200 - 2.t and = -2
dx^

w
dx
dR
The stationary values of R are given by = 0.

F lo
dx
dR
= 0
dx

ee
200-2.y=0 => a: = 100

Fr
d'^R
Clearly, = - 2 < 0 for all .y.
dx^ for
So, R is maximum when y = 100.
ur
Thus, the total revenue received will be maximum if annual subscription is increased by ?100.
s

■'i' : Find the point on the curve y^ =4x luhich is nearest


ook

to the point (2,1).


Yo

SOLUTION Let P{x, y) be a point on i/^ = 4y and A{2,\) be the given point. Then,
eB

AP^ = (.v-2)^ +(y-l)2


(2 '1^
our
ad

= iL_2 +(y-l)2 [vy^ =4y .-. Y=y^/4]


'v )
dY

Let Z =AP . Then, Z is maximum or minimum according as AP is maximum or minimum.


Re

(.1 n2
Fin

Now, Z =

dZ d^Z 3i/^
+2(y-l)=^ 2 and.
4 4 J ^ 4 dy^ 4

dZ
The critical numbers of Z are given by = 0.
^y
dZ y^
^y
= 0 =>
^ -2 = 0 => y^ 8 ^ y = 2

d^Z 3(2)^
Clearly, —:— = 3 > 0. Thus, Z is minimum when u = 2.
dy^ ^y = 2
4 ■'

Putting y = 2 in y^ = 4.y, we obtain y = 1. So, the coordinates of P are (1,2).

Hence, the point (1,2) on y^ = 4y is nearest to the point (2,1).


MAXir/ . ANDfwi'Nf'Mft 11.45

" A jct ofmi enemy is flying along the curvey = x'^ + 2. A soldier is placed at the point (3,2).
What is the shortest distance between the soldier and the jet?

SOLUTION Let P{x, y) be the position of jet and the soldier is placed at A (3, 2). Then, the
distance between the soldier and jet is given by

AP = .j(x-3f+{y-2f = ^(.v - 3f + .t
4
[vi/=a-2 + 2]
Let Z = AP^. Then, Z = {x - sf + x^
Clearly AP is maximum or minimum according as Z is maximum or minimum.
4
Now, Z = (.r - 3)^ + a:
dZ d^Z
dx
= 2(x-3) + 4.y^ and dx^ = 12x^ + 2

w
dZ
The critical numbers of Z are given by dx
= 0.

F lo
dZ
= 0
dx

2 {x - 3) + =0

ee
Fr
2y^ + .v-3 = 0
(x-1) (2.y^ + 2.V + 3) = 0 for
.Y=l [●.● 2.y^ + 2-y + 3 = 0 gives imaginary values of x]
ur
d^Z
s

Clearly, = 12 + 2 = 14 > 0.
ook

dx^
Yo

h=-[
eB

Thus, Z is minimum when.Y = 1. Putting y =1 in y = .y^ + 2, we obtain y = 3. So, the coordinates


of P are (1,3).
our

Hence, AP is minimum when jet is at the point (1,3) on the curve.


ad

Putting X = 1 and y = 3 in ylP = ^j{x- 3)^ + (y - if, we get


Y

AP = ->j{l - 3)^ + f = Vs. Hence, the shortest distance = VS.


Re
nd

EXERCISE 11.5
Fi

Determine two positive numbers whose sum is 15 and the sum of whose squares is
minimum.

Divide 64 into two parts such that the sum of the cubes of two parts is minimum.
How should we choose two numbers, each greater than or equal to - 2, whose sum is 1 /2 so
that the sum of the first and the cube of the second is minimum?
Divide 15 into two parts such that the square of one multiplied with the cube of the other is
minimum.

Of all the closed cylindrical cans (right circular), which enclose a given volume of 100 cm ,
which has the minimum surface area?
/ A . . (x \
Manufacturer can sell x items at a price of ? 5- each. The cost price is ? - + 500 .
lOOy \5 J
Find the number of items he should sell to earn maximum profit.
11.46 '^lED MATHEMATICS-XII

A wire of length 28 m is to be cut into two pieces. One of the pieces is to be made into a
square and the other into a circle. What should be the lengths of the two pieces so that the
combined area of the circle and the square is minimum?
A wire of length 20 m is to be cut into two pieces. One of the pieces will be bent into shape of
a square and the other into shape of an equilateral triangle. Where the wire should be cut so
that the sum of the areas of the square and triangle is minimum?
Given the sum of the perimeters of a square and a circle, show that the sum of their areas is
least when one side of the square is equal to diameter of the circle.
Find the largest possible area of a right angled triangle whose hypotenuse is 5 cm long.
Two sides of a triangle have lengths 'rt' and 'b' and the angle between them is 6. What value
of 0 will maximize the area of the triangle? Find the maximum area of the triangle also.

ow
I A square piece of tin of side 18 cm is to be made into a box without top by cutting a square
from each corner and folding up the flaps to form a box. What should be the side of the
square to be cut off so that the volume of the box is maximum? Also, find this maximum
volume.

e
Fl
A rectangular sheet of tin 45 cm by 24 cm is to be made into a box without top, by cutting off

re
squares from each comers and folding up the flaps. What should be the side of the square to

F
be cut off so that the volume of the box is maximum possible?
ur
’ ‘ A tank with rectangular base and rectangular sides, open at the top is to be constructed so
f or
that its depth is 2 m and volume is 8 m . If building of tank costs ? 70 per square metre for
the base and ? 45 per square matre for sides, what is the cost of least expensive tank?
ks
; =: A window in the form of a rectangle is surmounted by a semi-circular opening. The total
Yo
oo

perimeter of the window is 10 m. Find the dimensions of the rectangular part of the
window to admit maximum light through the whole opening.
B

1(1. A large window has the shape of a rectangle surmounted by an equilateral triangle. If the
re

perimeter of the window is 12 metres find the dimensions of the rectangle that will produce
the largest area of the window,
u
ad

r A rectangle is inscribed in a semi-circle of radius r with one of its sides on diameter of


Yo

semi-circle. Find the dimensions of the rectangle so that its area is maximum. Find also the
area.
d

18, Show that among all positive numbers a: and y with + y^ = r^, the sum .t + y is largest
Re
in

when x = i/=r/yf2.
F

Determine the points on the curve x^ = 4y which are nearest to the point (0, 5).
Find the point on the curve y^ = 4x which is nearest to the point (2,- 8).
Find the point on the curve x^ = 8y which is nearest to the point (2,4).
Find the maximum slope of the curve y = -x^ + 3x^ +2x - 27.
.2

The total cost of producing x radio sets per day is ? — + 35a:-I-25 and the price per set at
4
V

f
which they may be sold is ? V 50 - -2 J . Find the daily output to maximize the total profit.
ANSWERS

15/2,15/2 32, 32 (i- __


1

\2 VsJ' V3
MAX' 1A AND i.';:-: 11.47

1/3
50
J 6,9 The cylinder with radius —
y K J

240
28Tt
m.
112
m
80^f3 180

71+4 7T + 4 9 + 4V3' 9 + 4V3


25 2 71 . 1
cm -, Area = — ab 3 cm, 432 cm ^
4 2 2
20 10
5 cm ? 1000 Length = 71 + 4' Breadth =
71 + 4

12 18-6V3
lb. -Jlr, Area =r^ ' (±2^X3)

ow
6-V3' 6-V3 V2

(4,-4) -■ (4,2) 5 at (1,-23) 10 units

HINTS ro SELECTED PROBLEMS

e
Let r be the radius and h be the height of the closed cylindrical cane of volume 100 cm .

re
Then,

Fl 100

F
nr- h =100 => h = ...(i)
2
nr
ur
r
Let S be the surface area of the can. Then,
S = 2 nr h + 2nr^
fo
ks
200
+ 2nr^ [Using (i)
Yo
S =
oo

dS 200 d- 5 400
^ + 4 71 r and dr^ -^ + 4 7t
eB

dr r'^
dS
The critical numbers of S are given by ^ = 0.
ur

dr

/50 1/3
ad

dS 200
+ 4 Ttr = 0 => 4 71/-^ = 200 =>
Yo


dr
= 0 => -
^
r'^
r
V 71

die 50
d

Clearly, —J > 0 for all r. Hence, S is minimum when r -


Re
in

dr^ '',71/

Suppose -Y items are sold to maximize the profit P. Then


F

P - Revenue - Cost
/ X
=> P =x 5 - - -+500
looj V5
24 dP 24 Y d^P 1
=> P = — Y-^^ 500 => and
5 100 dx 5 50 dx^ 50

dP
The critical numbers of P are given by dx
= 0.

dP 24 Y
= 0 =>--—= 0 => Y = 240
dx 5 50

d^P
Clearly, — < 0 for all Y.
dx^ 50

Hence, profit P is maximum when 240 items are sold.


11.48 ,rs-.

Let r be the radius of the circle and a: meter be the length of each side of the square. Then,
14-2.V
2 nr + 4 X 28 ^ nr +2x =14^ r =
7C

Let A be the combined area of the circle and the square. Then,
2 2
A 71 + ,T

=> A
14-2.r^^ 9
rc +
[Using (i)]
{ TZ J
rfA 8 if^A 8
=> A = - (14 - 2x)^ + .v^ = -(7 - .v)^ + x^ ^
n dx
(7 - .v) + 2x and

ow
n n n

dA
The critical numbers of A are given by dx
= 0.

dA 8 28
= 0 => --(7-.v) + 2.r = 0=> a: =
dx n n + 4

e
d?-A

re
28
Clearly, = - > 0 for all a:. Hence, A is minimum when x =
dx^ K

rFl 71+4

F
112 112 28 n
The lengths of two partions are 4.y = meter and, 28 - m respectively.
71+4 7T+ 4 71+4

or
ou
Let .Y be the length of the each side of the square and y be the radius of the circle. Let S be the
sum of their perimeters. Then,
ksf
S-4y
S = 4.v + 27ry ^ y =
2tt
oo

Let A be the sum of the areas of the square and the circle. Then,
Y

A = + 7T
B

=> A = + -{S-4xf [Using (i)J


re

4 71
oYu

dA
= 2 Y-
ad

- (S - 4 y) and = 2 +
dx 7T dx^ 71

dA
The critical numbers of A are given by
d

= 0.
dx
in
Re

dA S
= 0 => 2y--(S-4y) = 0 => 71 Y - S + 4y = 0 => Y =
dx 7t n+ 4
F

d^A 8 5
Clearly, - 2 + — > 0 for all y. So, A is minimum when y = and for this value of
dx^ K 7T + 4

1 4S S
X the value of y is given by y = -(S-4y) = S-
2 7t 2 7U 7T + 4 2(71+4)
Clearly, y = 2y i.e. side of the square is equal to the diameter of the circle.
Hence, A is minimum when side of the square is equal to the diameter of the circle.
Let the length of the each side of the square which is cut from each corner of the tin sheet be
Y cm. By folding up the flaps, a cuboidal box is formed whose length, breadth and height
are 18 - 2.y, 18 - 2y and x respectively. Then, its volume V is given by
V =(18 - 2 y) (18 - 2 .v) Y = 324 Y -72 Y^ + 4 y^
dV d^V
= 324-144y + 12y^ and = -144 + 24.Y
dx dx^
11.49

The critical numbers of V are given by


dV
= 0 => 324 -144 .r +12 = 0 => .r^-12.v+27 = 0 => .t = 3,9.
dx

But, .r = 9 is not possible. Therefore, x = 3.


( j2
d^V
Clearly, . 144 + 72 = -72 <0.
I* A-=3
So, V is maximum when a: = 3 i.e. the length of each side of the square to be cut is 3 cm.
Let the length of a side of the square be .t cm and let 1/ be the volume of the box. Then,
V = (45 - 2 a:) (24 - 2a:) a:. Now, proceed as in Q. No. 10.
Let the length and breadth of the tank be a: and y meters respectively. It is given that the

ow
volume of the tank is 8m^ and height
4
is 2m.
2 A.-1/ = 8 => A-1/ = 4 => .V = - ...(i)

Let C be the cost of the tank. Then,


C = 70 x\j + 45 (2 X 2y + 2 X lx) = 70 xy + 180 y +180 a-

e
720
[Using (i)]

re
=> C = 280 + +180 .A
X

Frl
dC 720 d^C 1440

F
~ +180 and
dx X dx^ .a-3
dC
The critical numbers of C are given by = 0.
ou
r
dx

so
dC 720
= 0 => -^ + 180 = 0=> A-= 2
dx x~
kf
d^C
oo
Clearly, = 180 > 0. So, C is minimum when a = 2.
dx^ >x=l
Y
eB

720
Putting A = 1 in C = 280 + — + 180a, we get C = 1000.
Hence, the cost of least expensive tank is 11000.
ur

Ir. Let the width and height of window be 2a m and y m respectively. It is given that the
oY

perimeter of the window is 10 m.


ad

2a + 2y + 71 a = 10 => y = 5 - ^ (tt + 2)
d

Let A be the area of the window. Then


in

... ^ 1
= 2Ay + -A
Re

X X

= 10x-(Tt+2)A2+^A^
F

=> A [Using (i)]


dA d^A
- = 10 - 2a (rt + 2) + 71A and. = -2(K+2) + n = -n~4
dx dx^
dA
The critical numbers of A are given by = 0.
dx
dA 10
= 0 => 10 - 2a (71 + 2) + 77 A = 0 => A = 71 + 4 lY
dx

d^A Fig. 11.40


Clearly, - 71 - 4 < 0 for all a.
dx^
So, A is maximum when a =
10
andy = ^ 5(77+2) 10

77+4 (77+4) 77 + 4
20 10
Hence, the dimensions of the window are 2a = and y =
77+4 77 + 4
11.50
APPLIED MATHEMATICS-XII

● Slope m of the curve is given hym = ^


dx
=- + 6x + 2.
Now, m = -3.v^ + 6.v+2
dm dhn
= -6x+6 and = -6
dx dx^
dm
The critical numbers of w are given by dx
= 0.

dm
Now, = 0 => -6x + 6 = 0=>a: = 1.
dx
d^m
Clearly, = - 6 < 0 for all .y. So, m is maximumwhen x = l. Putting .y = 1 in the equation
dx^
of the curve, we get y = -23. ITius, slope is maximum at the point (1, -23). The maximum
value of slope is w =5.
Profit P is given by

w
.Y^ ,y2
P = Revenue - Cost - ? 50 - .Y - — + 35.Y + 25 -y^+15.y-25

F lo
2 4 4

ee
multiple choice questions (MCQs)

Fr
Mark the correct alternative in each of the following:
For the function / (,y) = y + — for
X
ur
(a) ,Y = 1 is a point of maximum (b) Y = -1 is a point of minimum
(c) maximum value > minimum value (d) maximum value < minimum value
s

Let / (y) = y^ + 3y^ - 9y + 2. Then, / (y) has


ook
Yo

(a) a maximum at y = 1 (b) a minimum at Y =1


eB

(c) neither a maximum nor a minimum at y = - 3 (d) none of these


3. The minimum value of / (y) = y^ - y^ - 2.y + 6 is
our

(a) 6
ad

(b) 4 (c) 8 (d) none of these


4. The number which exceeds its square by the greatest possible quantity is
1 1 3
2 ^^4 ^ none of these
Y
Re

5. het/(Y) =(y -a)^ + (y -/j)^ + (y -c)^. Then,/{Y) has a minimum at y =


nd

a + b + c
Fi

(a) (b)
3 (c) ^ - +
1 + -
1 (d) none of these
a b c
fi. The sum of two non-zero numbers is 8, the minimum value of the sum of their reciprocals is
i
^ ~8 none of these
. If Y lies in the interval [0,1], then the least value of y^ + y + 1 is
(a) 3 (b) 3/4 (c) 1 (d) none of these
. The least value of the function/ (y) = y^ - 18y^ + 96y in the interval [0, 9] is
(a) 126 (b) 135 (c) 160 (d) 0
The maximum value of/ (y) = ~—y on [-1,1 ] is
4 - Y + Y*^
1 1 1
(a)-j (b)-^
11.51
MAXIMA AND MINIMA

!{,. The point on the curve = 4.r which is nearest to the point (2,1) is
(a) (1,2V2) (b) (1,2) (c) (1,-2) (d) (-2,1)
II If r + y = 8, then the maximum value of xy is
(a) 8 (b) 16 (c) 20 (d) 24
The least and greatest values of / (x) = - 6x^ + 9x in [0, 6], are
(a) 3, 4 (b) 0, 6 (c) 0, 3 (d) 3,6
2 250 V
The minimum value of x + is
X ;

(a) 75 (b) 50 (c) 25 (d) 55

14. If / (x) = X + -
X
, X > 0, then its greatest value is

w
(a) -2 (b) 0 (c) 3 (d) none of these
1
If/(X) - , then its maximum value is
4x^ + 2x +1

.)|
Flo
(C) 1

ee
Let X, 1/ be two variables and x > 0, xy = 1, then minimum value of x + y is

Fr
1' 1
(d)3-
(a) 1 (b) 2 (c)2^
The function/(x) =2x^ -15x“ + 36x + 4 is maximum at x
for
ur
=
(a) 3 (b) 0 (c) 4 (d) 2
s
X
k

The maximum value of / (x) = yon I-l,l]is


Yo
oo

4 + X + x“

(d)i
(d)V
eB

14. Let/(x) = 2x^ - 3x^ -12x + 5on[-2, 4]. The relative maximum occurs at x =
r
ou
ad

(a) -2 (b) -1 (c) 2 (d) 4


"" The minimum value of x log^, x is equal to
Y

(a) e (b) \/e (c) -\/c (d) 2c (e) -e


The minimum value of the function / (x) = 2x^ - 21x^ + 36x - 20 is
Re
nd

(a) -128 (b) -126 (c) -120 (d) none of these


Fi

2
If X is real, the minimum value of x - 8x +17 is
(a) -1 (b) 0 (c) 1 (d) 2
(1
The maximum value of - i
IS
Vx;
^ye
(a) (b) (c) C
ye
(d)Y\e j

'' The function /(x) = x'^ has a stationary point at


(a) x = c (b) = (c) x=l (d) X = yfe
e

. Maximum slope of the curve y = -X ^ + 3x^ +9X-27 is


(a) 0 (b) 12 (c) 16 (d) 32
11.52
'■^MATHEMATiCS-XIl

The function f(x) = 2x^ ~ 3x^-12x + 4, has


(a) two points of local maximum (b) two points of local minimum
(c) onemaximumandoneminimum (d) no maximumno minimum

ANSWERS
(d) (b) ^ (b) ; (a) (a) (b) (c) (d) - (c)
(b) (b) (a) ■ (a) (d) ● (a) (b) ; (d) IH. (c)
(b) ■ (c) :i (a) ●: (c) (c) :4 (b) y-. (b) (c)

FILL IN THE BLANKS QUESTIONS (FBQs)


The positive real number x when added to its reciprocal gives the minimum value of the

ow
sum when, .Y =
The real number which must exceeds its cube is
The function/(.y) = ax + —, a,b, x > 0 takes on the least value at .v equal to

If y - alog.Y + Ij.v^ + X has its extreme values atx = 1 and X = 2, then (a,b) =,

e
Fl
re
The maximum value of f{x) = y is

F
.4 2
If the function/(y) = - 62y + (IX + 9 attains a local maximum at y = 1, then a
Y
=
ur
If the sum of two non-zero numbers is 4, then the minimum value of the sum of their

r
reciprocals is fo
If Y and y are two real numbers such that y > 0 and Yy =1. Then the minimum value of y + y
ks
IS
Yo

The niunber that exceeds its square by the greatest amount is


oo

Um andM respectively denote the minimum and maximum values of f{x) =(y-1)^ + 3 in
eB

the interval [-3,11, fben the ordered pair(/H, M) =


250 .
n.
The minimum value of/(Y) = y~ + :s
ur

The maximum slope of the curve y = -y^ + 3y^ -i- 9y -27


ad

is
Yo

X 2, , ,
The function /"(y) = - + - has a local minimum at y =
2 Y
d
Re

14.
The least value of the function f{x) = ax + - {a > 0, b > 0, x > Q) IS
in

i
Y
F

ANSWERS
1
■ 1 4. '^2 _l 1

. 3' 6 e

1
6, 120 1 ●. 2
2
10. (3,19)
75 ' 12 I... 2 ;2^Jab

VERY SHORT ANSWER QUESTIONS (VSAQs)


Answer each of the following questions in one word or one sentence or as per exact requirement of the
question:

Write necessary condition for a point y = c to be an extreme point of the function f (y).
Write sufficient conditions for a point y = c to be a point of local maximum.
11.53
'.lAXs:.

If / (.y) attains a local minimum at y = c, then write the values of /' (c) and/ (c).
Write the minimum value of / (y) = -V + — , y > 0.

Write the maximum value of / (y) = -V + — , y < 0.

Write the point where/ (y) = y log^. y attains minimum value.


Find the least value of / (y) =ax + —,
Y
where > 0, ^ > 0 and y > 0.

Write the minimum value of / (y) =

ow
Write the maximum value of / (y) = y ''.
logY
Write the maximum value of / (y) = , if it exists.
Y

e
re
/'(c) =0

rFl
/' (c) =0 and /" (c) <0 /' (c) = 0 and /" (c) > 0

F
/ 1 1
2 -2
\e ej

r
1
ou
,-l/^ ,l/e
2
e fo
ks
oo
Y
B
re
ou
Y
ad
d
in
Re
F
CHAPTER L2
APPLICATIONS OF DERIVATIVES IN
COMMERCE AND ECONOMICS

12.1 INTRODUCTION

In earlier chapters, we have studied derivatives and their applications in general. In this
chapter, we shall study applications of derivatives in Commerce and Economics. The same

w
requires the knowledge of some functions which occur very frequently in business and
Economics. So, we first introduce these function' in the following section.

12.2 FUNCTIONS RELATED TO BUSINESS AND ECONOMICS

F lo
In class XI, we have studied functions in detail. In this section we shall study some more

ee
Fr
functions which are used in business and economics.
COST FUNCTION // C is the total cost incurred in producing and marketing x units of a certain
for
commoditi/, then a function relating C and x is called a costfunction and is generally written asC =C (.v)
r
in the explicit form.
You

In the implicit form the cost function may be written as / (C, x) = 0.


s
ook

If C is the cost incurred in producing .v units of a commodity, thenC = 2 + 5a',C = 3 + 2.y + 5.v^,
eB

C = ax^ +bx + c etc. are cost functions.

The total cost of producing .r units of a commodity consists of two parts (i) Fixed cost, (ii)
our
ad

Variable cost.

FIXED COST It is the sum of all costs that are independent of the level of production.
dY

Fixed cost remains constant at all levels of production and it generally includes rent,
Re

depreciation, insurance, interest etc.


Fin

VARIABLE CC'.' It is the sum of all costs that are dependent on the level of production.
Variable cost varies as the level of production varies. It includes cost of lobour, material,
advertising expenses etc.
Clearly, Total cost = Fixed cost + Variable cost.
ILIIA’ ■ For each of the following cost fuiictions, find the fixed and variable costs:
(i) C (.r) = a + bx (ii) C (.v) = (7 + bx + cx^ (iii) C (.v) = a + bx + cx^ + dx^
SOLUTION Since the fixed cost remains constant at all levels of production. So, it is the cost
when there is no production i.e. x = 0. So, fixed cost = value of C (.t) at x = 0.
(i) Fixed cost = C (0) = a, Variable cost = hr
(ii) Fixed cost =C (0) Variable cost + ca:^
(iii) Fixed cost =C (0) =a, Variable cost = bx + c.r^ + dx^
12.2 APPLIED MATHEMATICS-XH

Tlie graph of the function C = C (.v) is called cost curve. It is evident from the cost curve the fixed
cost is determined by the intersection of cost curve with the cost axis.
C

v:
o
u
Cost curve
TJ
O
X
E

O .r

Fig. 12,1

w
DEMAND FUNCTION
Quantity demanded of a given commodity depends upon several variables such as

F lo
price of the commodity, prices of the substitutes and complements, disposable income, wealth, tastes,
habits etc. However, in elementary economic analysis, demand is considered to be dependent on the price of
the commodity. Throughout this chapter, we shall assume that the quantity demanded depe7ids on the

ee
price only.

Fr
It is a common phenomenon that corresponding to each price level of a commodity, there is a
for
certain quantity of that commodity which the consumers will demand during some time period.
An equation that relates price per unit and quantity demanded at that price is called the
ur
demand function.
s

If p is the price per unit of a certain commodity and t is the number of units of that commodity
ook
Yo

which consumers will demand during some time period at that price, then the demand function,
eB

in explicit form, is written as x -f {p).


In implicit form the demand function is written as F {x, p) = 0.
our
ad

In the demand function x =f (p), x is the dependent variable and p is the independent variable.
Also, .r > 0 and p > 0, because negative quantities and negative prices are meaningless.
Usually, as the price increases, demand decreases and due to decrease in price demand
dY
Re

increases. Therefore, the demand curve is always decreasing as shown in Fig. 12.2.
Fin

c
D
u

Q.

u
u
0.

Ci.
■v=/(p)
X
O
^(Quantity demanded) —►
Fig. 12.2

A relation befiveen the price per unit and the quantity supplied by the producers to
the market at that price is called the supply function.
APPLICATIONS OF DERIVATIVES IN C OlVlME " AND&' : 12.3

X = Sip)
c

a.

c.

Cw

.r

O -V (Quantity supplied)—►
Fig. 12.3

w
If p is the price per unit and .r is the corresponding quantity supplied, then the supply function
can be expressed in explicit form as .v (p)-

Flo
Clearly, .t > 0 and p > 0.
It is well known fact that an increase in price cau; es an increase in supply and a decrease in the

e
prices brings about a decrease in supply but it causes an increase in demand.

re
TOTAL REVENUE FUNCTION If R is the total revemie collected b\j a company when it sells x units of a

F
product at price p per unit, then R is given by R = px.
ur
This relation between R and x is called the total revenue function and is generally denoted by R (.r).

r

fo
If R {x) is the total revenue received and C (x) is the total cost incurred in the
production of X units of a commodity, then the function P {x) given by P{x) = R (x)-C {x) is called
ks
Yo
the proift function.
oo

ILLUSTRATIVE EXAMPLES
eB

A company finds its cost function to be C (x) = 100 + 50.v and its demand function to he
p (.t) =102 ~.r. Find: (i) the revenue function (ii) the profit function.
ur
ad

SOLUTION We have, C (x) =100 + 50a: and p(x) = 102-x


Yo

(i) R (x) = p (x) X =:> R (x) = (102 - x) X => R (x) = 102 x - x^


(ii) Let P (x) be the profit function. Then,
d
Re

P(x) = R(x)-C(x) P(x) = 102X-X ^-(100 + 50x) => P(x) = 52x-.x^-100


in

The denmndfunctionfor a certain commodity is given byp^l OOO - 15x - x , 0 < x < 25.
F

What is the price per unit and the total revenue from the sale of 2 units?
SOLUTION Let R (.x) be the revenue function. Tlien,
R(x) = px => R(.x) = 1000x-15.x^-x^
When X = 2, we get
p=1000-15x 2-2^ = 966 and, R =2000-15x 4-8=1932
Hence, from the sale of 2 units: Price per unit = 966 and. Revenue = 1932.
i ' . Suppose the cost to produce some commodity is a linear function of output. Find cost as a
function of output, if costs are ? 4000for 250 units and ? 5000for 350 units.
SOLUTION Let C denote the cost and x the output. It is given that the cost C is a linear function
of output .X.
C = ax + b, where rt, ^ are constants.
When X = 250, we have C = 4000
12.4 APPLIED MATHEMATICS-XII

4000 = 250 a+ b
When X = 350, we have C = 5000
5000 = 350 a+ b
●●●(ii)
Solving (i) and (ii), we get (7 =10 and & =1500. Substituting these values in C =<?:»: + & we get
C= 10a:+ 1500.
IAAMPLH
A company sells x tins of chocolate pozuder each day at ? 20 tin. The cost of
manufacturing these tins is ? 15 per tin plus a fixed daily overhead cost o/? 900. Determine the profit
function. What is the profit if 1000 tins are manufactured and sold per day ? Hoiu do you interpret the
situation if the company manufactures and sells 200 tins a day.
SOLUTION Let R (a:) be the revenue received by the company per day.

ow
Since the company sells a: tins of chocolate powder each day at ? 20 a tin. Therefore, R{x)= 20x
Thus, the revenue function is given by
R (x) = 20 X ...(i)
Let C (a:) be the total cost of the manufactured tins in a day. Then,

e
C (a:) = Fixed cost + Variable cost => C (a:) = 900 + 15 a:

re
Let P (,t) be the profit per day. Then,

rFl
F
P{x) = Rix)-C(x) P{x) = 20.v-(900 + 15a:) => P (x) = 5a:-900.
Thus, the profit function is given by

r
ou
P (:c) = 5a- - 900 ...(h)
fo
If 1000 tins are manufactured and sold in a day, the profit is given by
ks
P (1000) = 5 X 1000 - 900 = ? 4100.
oo

If the company manufactures and sells 200 tins a day, then


Y

P(200) = 5 X 200-900 = 100. [Putting X = 200 in (i)]


B

Thus, the company's profit is ? 100 when 200 tins are produced and sold in a day.
re

EX.AMPLE 5 A manufacturer can sell x items of a commodity at price of^ (330 -x) each. Find the
revenue function. If the cost of producing x items is ? (x^ + lO.r +12). Determine the profit function.
ou
Y
ad

SOLUTION Let p be the price per unit when x items are sold. Then, ^ = 330 - x[Given]
Clearly, it is the demand function.
d

Let R (x) be the revenue collected when x items are sold. Then,
in
Re

R{x) = xp ^ R (x) = X(330 -x) => R (x) = 330 x-x^


F

LetC (x) be the cost of producing x items. Then,


C(x) = x^+lOx + 12 [Given]
So, the profit function P (x) is given by
P (x) = R (x) -C (x) => P (x) = (330x - x^) -(x^ + lOx + 12) ^
P(x) = 320x-12-2x^.
EX.X.MPLE
A company sells its product of^ 10 per unit. Fixed costs for the company are ? 35000 and
variable costs are estimated to run 30% of total revenue. Determine the (i) total revenue function (ii) total
costfimction (Hi) quantity the company must sell to cover the fixed cost.
SOLUTION Suppose x units of the product are sold. Then, the revenue R (x) is given by
R(x)=10x
30
Variable cost = 30% of R (x) = X lOx = 3x
100
APPLICATIONS OF DERIVATIVES IN COMMERCE AND ECONOMICS 12.5

The fixed cost is ? 35000. Therefore, the cost function C (.v) is given by
C (a:) = Fixed cost + Variable cost = 35000 + 3x.
Suppose .V units of the product are sold to cover the fixed cost.
R (.v) = Fixed cost => 10a: = 35000 => a: = 3500.
Thus, 3500 units of the product must be sold to cover the fixed cost.
EXAMPLE" A company is selling a certain product. The demand function of the product is linear. The
company can set 2000 units when the price is ? 8 per unit and when the prices is ? 4 per unit, it can sell
3000 units. Determine

(i) the demand function. (ii) the total revenue function.


SOLUTION Let p be the price per unit when x units are sold and let the demand function be

w
p = ax + b ... 1

where a and b are constants,

Flo
(i) It is given that p = S when x = 2000 and p = 4 when x = 3000
2000rt + {;=8 and 3000fl + l?=4

ee
1
Solving these two equations, we get <? = - and b =16.
250

Fr
Substituting the values of a and b in (i), we get p = - +16 as the demand function,
for
ur
(ii) Let R (a:) denote the total revenue function. Then,
( X
R (a:) = pAT => R (x) 1-16 X => R{x) =~ + 16a:
s
V 250 J 250
ok
Yo

This is the total revenue function.


Bo

EXERCISE 12.1
re

1. The cost and demand functions of a commodity are given by C(x)=75 + 2x and
p (a) = 85 - a:. Find (i) the revenue function (ii) the profit function.
ou
ad

2. The demand fimction of a certain commodity is given by p=1000 25x + x , where


Y

0 < a: < 20. Find the price per unit and total revenue from the sale of (i) 2 imits (ii) 5 units.
A demand function for a commodity is known to be a linear function. The price for 5 units is
nd
Re

? 9 a unit and the price for 10 units is ? 12 per unit. Find the demand and revenue functions
Fi

and determine the total revenue when 50 units are sold.


A company sells .r tins of talcum powder each day at ? 10 per tin. The cost of manufacturing
is ? 6 per tin and the distributor charges ? 1 per tin. Besides these the daily overhead cost
comes to ? 600. Determine the profit function. What is the profit if 500 tins are
manufactured and sold in a day? How do you interpret the situation if the company
manufactures and sells 100 tins in a day ?
A company produced a commodity with ? 10000 fixed costs. The variable costs are
estimated to 25% of the total revenue received on selling the product at a rate of ? 6 per
unit. Find the total revenue, total cost and profit function.
6. A company produced a commodity with ? 20,000 fixed costs. The variable cost is estimated
to 35% of the total revenue when it is sold at a rate of ? 6 per unit. Find the total revenue,
total cost and profit functions of the revenue when it is sold.
7. The demand function is a: = ———, where x is the number of units demanded and p is the
3

price per unit. Find the revenue function R in terms of p.


12.6 APPLIED MATHEMATICS-XII

ANSWERS

(i) R = 85.V - .Y^ (ii) P-83.y-.v^-75


(i) /? = 954, R =1908(ii) p = 900, R=4500
5 3v^
.Y = - y -10, R (.v) =6.r + R =1800
3^ 5
P(x) = 3.V-600, P(500) = f 900, company incurs a loss of ?^300

ow
R (.r)=6.r;C (y)= 10000+ - .Y, P(.y) =-Y-10000
2 2
21 39
R(.v) = 6.y,C(.y)=20000 +
10
X, P (x) =—x- 20,000
10 R=8p-|p2

e
12.: BREAKEVEN/:

re
LetC (.y) be the totalcostof production and marketing of .y units of a commodity and R (.y) be the

Flr
F
total revenue obtained by selling these .y units. Then the profit function P (.y) from producing,
marketing and selling x units of the commodity is given by P (.y) = R (.y) - C (.y).
Now, three cases arise:
ou
sr
WhenC (.y) > R (.y) i.e. the total cost of producing and marketing x unitsofacommoditi/ismore
than the revenue R (.y) obtained b\/ selling these x units.
fo
k
In this case, the business is said to be in loss and the loss is equal to R (.y) -C (.y).
oo
WhenC (.y) < R (.y) i.e. the total cost of producing and marketing x units of a commodity is less
Y
than the revenue R (.y) obtained by selling these x units.
reB

In this case, the business is said to yield profits and the profit is equal to R (.y) - C (.y).
When R (.y) = C (.y) i.e. the total cost of producing and marketing x units of a comi7iodity is
uY

equal to the revenue obtained by selling these x units.


In this case, there is neither a profit nor a loss in the business. Such a situation is called business
break even.
ad
do

BREAK-EVEN POIt'
The break -even point is the level of production zuhere the revenue from sales is
equal to the cost of production and marketing.
in

At the break-even point the manufacturer is neither a loser nor a gainer.


Re
F

Graphically, the break-even point is the intersection of the total revenue and total cost
curves as shown in Fig. 12.4.
Y /
Cr /
/Profit

cO’’

3
Break-even point
C
01
V
01
.os;
c.e
Fixed cost curve
in
O
U

o Quantity ● X

Fig. 12.4
APPLICATi OF DERIVATIVES IN COMMER ' “ ' :iD ECONOMICS 12.7

In order to find the break even points, we may use the following algorithm.
ALGORITHM

-Ti.r Obtain the total cost function C (x) of producing and marketing x units of the commodity.
Obtain the total revenue fiinction R (.r) of selling x units of the commodity.
vrv\- III Equate the cost function C (a:) to revenue function R {x) i.e. C (.y) = R (y).
;lj;i‘: Solve the equation C (y) = R (y) for x. The value ofx obtained from C (.v) = R (y) determines
the level of production where the cost of production is equal to the revenue from sales i.e. the
break-even point.
Following examples illustrate the above algorithm.
ILLUSTRATIVE EXAMPLES

The total cost and total revenue functions of a commodity are given byC (y) =y + 40fl7id

w
R (y) = IOy - 0.2y^. Find the break-even point.
SOLUTION The break-even points are given by C (.v) = R (.y).
Now, C(y)=R(y)
Y + 40 = IOy - 0.2y^

F lo
0.2x^ - 9y + 40 = 0 => 2y^ - 90x + 400 = 0 =>

ee
x^ - 45y + 200 = 0

Fr
(x-40)(y-5) = 0 X = 5,40 for
Hence, the break-even points are x = 5 and x = 40
A television manufacturer ifnds that the total cost for producing and marketing x
ur
television sets is C (x) = 250x^ + 3250 x + 10,000. Each product is sold for ? 6500. Determine the
s
ook

break-even points.
Yo

SOLUTION Let R (.x) be the revenue received for selling x units. Then, R (.x) = 6500 .x.
eB

We have, C (x) = 250x^ + 3250x + 10,000.


At the break-even points, we must have
our
ad

R(x)=C(y)
6500 X = 250x^ + 3250X + 10,000 => 250x^ - 3250x + 10,000 =0
dY

X 2 -13X + 40 = 0 x=5,8
Re

Hence, the sale of either 5 or 8 television sets will give break-even point.
Fin

A television manufacturer finds that the total cost for the production and marketing of x
number of television sets is C (x) = 300x^ + 4200.x +13500. Each product is sold for ? 8400. Determine
break even points.
SOLUTION Let R (x) be the revenue received for selling x television sets. Then,
R (x) = px => R (x) = 8400x.
We have, C (x) = 300x^ -i- 4200x +13500. At the break even points, we have
R (x) =C (.X) ^ 8400X - 300x^ + 4200x + 13500 => 300x^ 4200x-f 13500 = 0=>

.x^ -14.x + 45 =0=i> X =5, 9


Hence, the sell of either 5 or 9 television sets will give break even points.
A company has fixed costs of^ 26,000. The cost of producing one item is ? 30. If this item
sells for ? 43, find the break-even point.
12.8 APPLIED MATHEMATiCS-Xll

SOLUTION Let .t be the number of items produced and sold. LetC (.v) be the total cost incurred
in producing a: items. Then,
C (.v) = Fbced cost + Variable cost =26,000 + 30.r
Let R (a:) be the total revenue received in selling x items. Then, R (a:) = 43 .r
At the break-even point, we have
C {x) = R (x) => 26,000 + 30 .v = 43 a => 13.t = 26,000 => x = 2000.
Hence, the break-even point is 2000 items.
[■XAMi'i !
A profit making company wants to launch a new product. It observes that the fixed cost of

ow
the neio product is ? 35,000 and the variable cost per unit is ? 500. The revenue function for the sale ofx
units is given by 5000 .v -100 .y“. Find (i) profit function (ii) break-even values and, (Hi) the values ofx
that result in a loss.

e
SOLUTION Let R (.r) and C (y) denote the revenue function and the total cost function

re
respectively. Then, we have
R (y) = 5000 Y -100 Y^ and C (y) = 35,000 + 500 y

Frl
F
- CO
10 35 oo
ou
Fig. 12.5 Signs of .v" - 10.v+ 35.

sor
(i) Let P (y) be the profit function. Then,
P(y)=R(y)-C(.v) kf
=> P (y) =5000 Y-100 Y^- 35,000-500y =-lOO.v^ + 4500y - 35,000.
oo

Hence, the profit function is P (y) = - IOOy^ + 4500y - 35,000


Y
B

(ii) For the break-even points, we have


P(-v)=C{Y)
re

=> P (.v) = 0 => - IOOy^ + 4500.V - 35,000 = O => - 45y


oY

+ 350 = O ^ y =10, 35
u

Hence, the break-even values are 10 and 35.


ad

(iii) For the values of y that result in a loss, we have


d

P (y) < 0
in

=> -lOO.Y^+4500X-35000 <0


Re

Y^ -45y+ 350>0 =>(y-10)(y-35) >0 => Y<10orY>35


F

Hence, the company will result in a loss if either y < 10 or, y > 35.
L'XAMPLE h
For a new product, a manufacturer sets up an infrastructure which costs him ? 140000.
The variable cost (labour, materials, etc) is estimated as ? 125 for each unit of the product. The sale price
per unit is fixed at ? 160. Write down the cost function, C (.v). Revenue function, R (,r), and Profit
function, P (.v),forx units of the product. How many number of units are to be produced in the first year of
production so that there may be no loss during that year ?
SOLUTION Here, we are given that
Fixed cost = ? 140000, Variable cost = ^ 125 per unit and. Selling price = ? 160 per
unit.

Let Y units of the product be produced. Then, Variable cost = 125 y


Cost function C (y) is given by
C (.y) = Fixed cost + Variable cost = 140000 -i-125 y
APPLICATIONS OF DERIVATIVES IN COMMERCE AND ECONOMICS 12.9

Since the selling price of one unit is ? 160. Therefore, Revenue function R (.v) is given by
R{x)=l60x
Now, P{x) = R(.v)-C(.v) => P(.Y) = 160y-(140000+ 125-t) => P (.y) = 35y -140000
Break-even point: At break-even point, we must have
P{x) = Q i.e. R(.v)=C(x) => 35-r-140000 = 0 => ,y=4000
Hence, the minimum number of units which must be produced in the first year of production so
that there is no loss in that year is 4000 units.
EXAMPLE ● A company sells its product at the rate of ^ 6 per wiit. The variable costs are estimated to

w
run 25% of the total revenue received. If the fixed costs for the product are ? 4500, ifnd
(i) total revenue f motion (ii) total costfiinction
(Hi) profit function (iv) the break even point
(v) number of units the company must sell to cover its fixed cost.

o
e
SOLUTION (i) Let R (.r) be the total revenue function for selling y units of the product. Then,

re
R (y) = 6y
(ii) LetC (y) denote the total cost function. Then,

Frl
F
C (y) = Fixed cost + Variable cost = 4500 + 25% of revenue received
25
= 4500 + X 6y = 4500 + -Y
ou
100 2

r
(iii) Let P (y) denote the profit function. Then,
9y
so
P(y) = R(y)-C(y) = 6y 4500 + - Y — - 4500
kf
2 2
oo

(iv) For break even points, we must have


Y

9y
C (.y) = R (y) or, P (.y) = 0 => — - 4500 = 0 => y = 1000
B

(v) In order to cover the fixed cost; total revenue received must be same as the fixed cost.
re

6y = 4500 => y=750


oY
u

EXERCISE 12.2
ad

1. A music-system manufacturer determines that its total cost for producing y units is given
d

by C (y) =500 Y“ + 4500.Y +10,000. Each unit can be marketed for ? 9000. Determine the
in

break-even point.
Re

ABC company Ltd. is planning to market a new model of shaving razor. Tlie fixed cost of
F

the company for the production of the razors is found to be ^ 27,000 and the cost for the
material and labour to produce each unit of razor is estimated to be ? 8 per unit. If each
razor can be sold in the market for ? 12.00, find (i) the cost function for producing y razors,
and (ii) the break-even point.
3. A company has fixed costs of ? 20,000 and the cost of producing one unit of their product is
? 5. If each unit is sold for ? 9, find the break-even point.
4. For the first year, the fixed cost for setting up a new electronic pocket calculators company
is ? 300,000. The variable cost for producing a calculator is ? 70. The company expects the
revenue from the sales of the calculator to be ^ 270 per calculator. Find (i) the cost function
(ii) the revenue function (iii) the break-even point, and (iv) the number of calculators
produced for which the company will suffer a loss.
J. A company decides to set up a small production plant for manufacturing electronic clocks.
The total cost for initial set up is ? 900,000. The additional cost for producing each clock is?
300. Each clock is sold at ? 750. Find (i) the cost function (ii) the revenue function (iii) the
profit function, and (iv) the break-even point.
12.10 ,;at(c? ■

A company sells its product for ? 4 per unit. Fixed costs for the company are ? 2800 and
variable costs are estimated to mn 30% of the total revenue. Determine (i) the total revenue
function (ii) the total cost function (iii) the break-even point, and (iv) the quantity the
company must sell to cover its fixed cost.
A text-book publisher finds that the production costs directly attribute to each book are
? 20 and that the fixed costs are ? 10,000. If each book can be sold for ? 30, determine (i) the
cost function (ii) the revenue function, and (iii) the break-even point.
The pricing policy of a company follows the demand equation p=D (.t),D (.v) being the
price per unit when .r units are demanded. After studying the market trends the company
determines the price function that is given by D (.v) = 2000 - 4x. If the product is to be
marketed the company will incur a fixed cost of ? 60,000 and will have to pay ? 600 for each
unit that is produced and placed in the store. At what sales level can the company expect to
recover its costs ?

C (.r) = 5.r-I- 350 and R{x)=5Qx-x^ are respectively

w
the total cost and total revenue
functions for a company that produces and sells .v units of a particular product. Find: (i) the

F lo
break-even values (ii) the values of A' that produce profit (iii) the values of .v that result in a
loss.

The fixed cost of a new product is ? 18000 and the variable cost per unit is ^ 550. If the

ee
demand function is p (.r) = 4000 - 150.t, find the break even values.

Fr
The fixed cost of a new product is ? 35000, and the variable cost per units is ^ 500. If the
demand function is p (.v) = 5000 - IOOa, find the break even values.
for
A company wants to launch a new product. It invested ? 37500 as fixed cost and ? 200 per
unit as the variable cost of production. The revenue function for the sale of a units is given
ur
by R (a) = 4825a - 125a^. Find the breakeven point (s).
s
ook

A company starts producing pens and finds that the production cost of each pen is 110 and
Yo

the fixed expenditure of production is ? 4500. If each pen is sold for ? 25, determine
eB

(i) cost function (ii) revenue function(iii) the breakeven point.


A company launched a new product with fixed cost of ? 25000 and the variable cost per
unit is ? 1500. The revenue received on the sale of a units given by 8500 a-400a^. Find:
our
ad

(i) profit function (ii) break-even points.


The price of selling one unit of a product when a units are demanded is given by the
Y

equation p = 4000 - 2a. The fixed cost of product is ? 20000 and ^ 1484 per unit are paid for
Re

the product to place in a store. Find the level of sales at which tlie company expect to cover its
nd

costs.
Fi

A company paid ? 16100 towards rent of the building and interest on loan. The cost of
producing one unit of the item is ? 20. If each unit is sold for ^ 27, find the breakeven point.
The fixed cost of a product is f 20000 and cost of production per unit is ? 75. If each unit is
sold for ? 100, find the break-even value. Also find the values of a for which the company
always results in profit.
A firm paid ? 25,000 as rent of its office and ? 15,200 as the interest of the loan taken to
produce a units of a commodity. If the cost of production per unit is ? 8 and each item is sold
at price of ? 75, find the profit function. Also, find the break-even point.
A calculator manufacturing company finds that the daily cost of procuring a calculators is
given by C (a) = 250a + 7500.
(i) If each calculatoris sold for ? 350, find the maximum number of calculators that must
be produced daily and sold to ensure no loss,
(ii) If the selling price is increased by ? 250. What would be the break even point?
APPLICAT .IVATIVRSli.! CONuMICf 12.11

A computer software manufacturer wants to start the production of floppy disks. He


observes that he will have to spend ? 2 lakhs for the technical know-how. The cost of setting
up the machine is ? 88,000 and the cost of producing each unit is ? 30. He can sell each
floppy at ^ 45. Determine (i) the total cost function for producing .v floppies, and (ii) the
break-even point.
; The fixed cost of a new product is f 30,000, and the variable cost per unit is ? 800. If the
demand function is p (.y) = 4500 -100 .r, find the break even values.
■\NSWERS

. A- = 4,5 (i) C(y)= 27,000-hS.Y (ii) 6750 Y =5,000

(i) C(.v)=70.y+ 300,000, (ii) K(.y) = 270.y (hi) y=1500


(iv) .y<1500 (i) C (.Y)= 900,000+ 300 Y (ii) R(y)=750.y
(hi) 450.V-900,000 (iv) 2000

w
6. (i) R(y)=4y
(ii) C (x) = 2800 + I Y (hi) 1000 (iv) 700

(hi) Y=1000

F lo
7. (i) C(y)=20y +10,000 (h) R(.y) = 30y
8. 50,300 (i) 10, 35 (ii) 10<y<35
(hi) Y>35orY<10 8,15 10, 35 12,25

ee
(i) C (Y) = 4500 + lO.Y, (h) R(y) = 25y, (ih) 300

Fr
14. (i) P(y) = 7000 Y-400 Y^-2500, (h) 5,12.5 l.T 1250, 8

16. 2300 :7. 800, Y> 800


for
P(.y) = 67y-40,200, 600
ur
19. (i) 75 (ii) 30
:n. (i) C(y)= 288,000+ 30.Y (ii) 19,200 .. y=12, 25
s
ook
Yo

12.4 AVERAGE AND MARGINAL COSTS

LetC = C (.y) be the total cost of producing ami marketing x units of a product, then
eB

AVERAGE COST
C
the average cost AC is defined as AC X
r
ad
ou

Note that the average cost is the cost of producing and marketing each unit of the product.
MARGINAL COST Let C =C ( y) be the total cost of producing and marketing x units of a product, then
Y

the marginal cost, MC, is defined as the rate of change of the total cost C zoith respect to x i.e. MC = .
Re
nd

Note that the marginal cost is the approximate cost of one additional unit of output.
Fi

RELATION BETWEEN AC AND MC Let C be the total cost of producing and marketing x units of a
commodity. Then,
AC [By definition]
Y

dC
-C

^(AC) = dx V Y, 1 r^_c 1
= -(MC-AC) ...(i)
dx y2 Y dx X ) X

Now three cases arise.

CASE I When MC > AC : In this case, we have


MC-AOO

— (AC)>0 [Using (i)]


dx
AC increases with y.
12.12 APPLIED MATHEMATICS-XI!

CASl: II \NhenMC = i4C : In this case, we have


MC~AC=Q

~{AC) = 0 [Using (i)I


dx

AC is constant i.e. average cost remains constant at all levels of output.


_ASi: I'l When MC < AC; In this case, we have
MC - AC < 0.

^{AC)<0
dx [Using (i)]
AC decreases with x.

w
ILLUSTRATIVE EXAMPLES

;c2
'!● If (1 manufacturer's total cost function C is given by C =— + 2x, find (i) the

o
25
average cost function (ii) the marginal cost function, and (Hi) the marginal cost when 5 units are

e
re
produced. Also, interpret the result.

rFl
F
SOLUTION (i) We have, ^ ~ average cost function AC is given by

r
AC =- or, AC = — + 2
ou
.V 25
fo
ks
(ii) The marginal cost function MC is given by
MC =
dC
=> MC = —
d (x^ 2x
+ 2.t => MC = — + 2
oo

dx dx 25 25
Y
eB

(iii) The marginal cost when 5 units are produced is (MC) x = 5 = — X 5 + 2 = 2.4
25
r

This means that, if the production is increased by 1 unit from 5 units to 6 units, then the cost of
ou

additional unit is approximately 2.4


Y
ad

I XAMI’i t
The total cost, C (x), associated with producing and marketing x units of an item is given
by:
d

C(x) = O.OOSx^-0.02x^ + 30X + 5000


Re
in

Find: (i) the average cost function (ii) the average cost of output of 10 units.
F

(iii) the marginal cost function (iv) the marginal cost tvhen 3 units are produced.
SOLUTION We have, C (.t) = 0.005.r^ - 0.02a:^ + 30.r + 5000
(i) The average cost function AC is given by
C(.v) 0.005.t^ - 0.02a:^ + 30x + 5000 5000
AC = = O.OOS.r^ - 0.02.r + 30 +
X X X

(ii) Putting .V = 10 in AC, we get


5000
= 10 = ^-005 ^ 10^ - 0.02 X 10 + 30 + 10
= 05 + 0.2 + 30 + 500 = 530.3

Hence, the required average cost = ? 530.3.


(iii) We have.
d
C (x) = 0.005.v^ 0.02x^ + 30a: + 5000 => (C (.r)) = 0.0] 5.\-^ - 0.04.V + 30
dx
APPLICATIONS OF DERIVATIVES IN COMMERCE AND ECONOMICS 12.13

Hence, the marginal cost function MC is given by


MC = —
dx
(C (x)) = O.OISa-^ - 0.04.V + 30.

(iv) Putting A' = 3 in MC, we get


(MC)_^.^3 = 0.015 X 9 - 0.04 X 3 + 30 = 30.045
Hence, the required marginal cost is ? 30.045.
A+ 7

ow
EXAVTFLE 3 if the total cost function C of a product is given by C = 3.v + 5. Prove (hat the
^a + 5
marginal cost falls continuously as the output increases.
SOLUTION We have,

e
fi t-,''
c - It ^ +7a: + 5

re
A +5

Flr
dC _ , f(A + 5)(2A + 7)-(.Y^+7A) ^

F
+0
dx
1 (-V + 5)2
dC ^ \ (2a^ + 17a + 35 - A^ - 7a)
ou ^ (.y^+10a+35) (A+ 5)^+ 10

sr
31 T— r = 3 < 5—
dx {x + 5f (A+5)^ (A+ 5)2 J
= 3n +
10 fo
k
oo
(x + 5f
Y

10
reB

Thus, the marginal cost MC is given by MC = 3 < 1 +


(.v + 5)2 ■
uY

In order to prove that the marginal cost MC falls continuously as the output increases, it is
sufficient to show that — (MC) < 0.
dx
ad
do

-(MC) = 3J0-
10 20 d 60
Now, MC = 3n + ● => ^ — (MC)=- 3<0
{x + 5f (x+5)3
in

dx dx (A+ 5)
Re

Hence, the marginal cost MC falls continuously as the output increases.


F

EXAMPLE 4 If the total cost function is given by C=a + bx + cx^ verify that
— {AC) =-{MC - AC).
dx X

SOLUTION We have, C = a + bx + cx^


C a dC
AC = = — + b + cx and, MC =
dx

dx
(<7 + bx + ca-2) = b + 2cx.
X A

-(MC-AlC) = -|a? + 2c.v)- V —A + b + cx


a 1 a a
Now, CA — c -
.2
...(i)
A A [ A A A

d , d (a , -a a
and. — (/IC) = — - + + CA 2+0= ●●●(ii)
dx dx\x A I'‘.A,
From (i) and (ii), we get — {AC) = -{MC-AC).
X dx
12.14 APPLIED MATHEMATICS-XII

36
EXAMPLE 5 The average cost function AC for a commodity is given by AC = x + 5 + in terms of
.V

output X, Find the total cost C and the marginal cost MC as the function ofx. Also, find the outputs for
which AC increases.

SOLUTION Let C be the total cost function. Then,


C 36
AC = ^ C = AC. x=> C = X+ 5 + .r => C = x'^ +5x + 36
X X J
+

— CO 00
-6 6

Fig. 12.6 Signs of (.r^ - 36)


Let MC be the marginal cost function. Then,
dC
— {x^ + 5.V + 36) => MC =

low
MC = => MC = 2x + 5
dx dx
36
4-(AC) =
36
Now, AC = .r + 5 + — => 1
X dx a-2
For AC to be increasing, we have

ee
— (AC)>0
dx

F
Fr
11 36 >0=>
^ -A x^ - 36
=> —~ >0

for
ur
- 36 >0 => (a - 6) (a + 6) > 0=> .V <-6 or a >6=> a > 6 IVA>0]

Thus, the average cost increases, if the output a > 6.


k s
EXAMPLE 6 IfC = ax^ + bx + c represents the total cost function, find:
Yo
oo

(i) Slope of the average cost curve (ii) Slope of the marginal cost curve
eB

SOLUTION We have, C = a^ +bx + c


(i) Tlie average cost function AC is given by
r

C ax^+bx + c
ou
ad

AL. — — => AC = ax + b + —
A A A
Y

d
The slope of the tangent to cost curve is (AC).
dx
nd
Re

-^{AC) =
c
— (AC) = — ax + b + — a :r
dx dx dx A-2
Fi

xj

c
Thus, the slope of the average cost curve is a —;T
X

(ii) The slope of the marginal cost curve is


d . d(dC^ d^C
dx dx \ dx J dx'^
'y dC d 7 d^C
Now, C - ax + bx + c ^ — {ax + bx + c) = lax + b ^ = ~{2ax + b) =2a
dx dx rfA^ dx
Thus, the slope of the marginal cost curve is 2a.
EXERCISE 12.3

1. A manufacturer's total cost function is C =1500 + 30a + a^, find : (i) the average cost
function, (ii) the marginal cost function, (iii) the marginal cost when 20 units are produced,
and (iv) the actual cost of producing twenty first unit. Give the economic interpretation to
these results.
APPLICATIONS OF DERIVATIVES IN CO? " 'ERCE AND ECONOMICS 12.15

5.y^
2. If the total cost function for a manufacturer is given by C = + 5000, find the
^/.v^ + 3
marginal cost function.
The cost of producing .v units of a product is given by C =7.v +130. Show that the marginal
cost is always constant.
4. The cost of producing and marketing x units of a certain commodity is given by
C = + 1500. Find the marginal cost when (i) .v = 4 (ii) .v = 8.
.V + 4

ow
5. IfC = 2.Y + 6 is the total cost of production of .v units of a certain product, show
x + 1

that the marginal cost falls continuously as the output .v increases.


x + b
■ ■ IfC = ax , t > c is the total cost function of producing .r units of a certain commodity,

e
yx + cj

re
show that average and marginal costs fall continuously as the output .y increases.
.T If the total cost C (y) of a firm is C (y) = 0.005 y^ - 0.02 y^ - 30y + 5000, where y is the

Frl
F
output. Determine (i) average cost (ii) marginal cost.
8. Show that the slope of average cost curve is equal to — (MC - AC) for the total cost function
ou
sor
C = ax^ + bx^ + CY + d.
8000
by AC = 0.0002y^ - 0.05y + 7 +
9. The average cost fimction for a product is given
kf Y
oo
where y is the output. Find the marginal cost Rmetion. What is the marginal cost when 100
units are produced? Interpret your result.
Y

10. The cost function of a firm is given by C (y) = 2y^ - 4.v + 5. Find (i) the average cost and,
B

(ii) the marginal cost, when y = 2.


11. The cost function C (y) of a firm is given by C(y)-3.y^-6y +5. Find: (i) the average cost
re
oY
u

and, (ii) the marginal cost, when y = 2.


12. The cost function C (y) of a firm is given by C (.v) = 2y" - 4y + 5. Find (i) the average cost,
ad

and (ii) the marginal cost, when x = 10.


d

.3
+ Y^ - 15.V + 3. Find
in

13. Given the total cost function for y units of a commodity as C (y)
Re

the (i) marginal cost function and (ii) average cost function.
F

14. The cost function of a firm is given by C = 3 y^ - 2y + 3. Find (i) the average cost and (ii) the
marginal cost, when y = 3.
15. The average cost function associated with producing and marketing x units of an item is
50
given by AC = 2y-11 + . Find the total cost function and marginal cost function. Also,
Y

find the range of output for which AC is increasing.


16. The total cost function for a production and marketing activity is given by
C(y)=—Y^
4
-7.Y + 27. Find the level of output (number of units produced) for which
MC-AC.

17. The average cost of producing y units of commodity is given by


y-2 Y 5000
AC = —-30 + . Find the marginal cost function and verify that
200 50 Y
MC-AC
^(AC)
dx
= X
12.16 APPLIED MATHEMATICS-Xl!

18. If the total cost function is given by C = 3 - 2.v + 5x"^, where x is the quantity of output,
MC -AC
show that — (AC) - V where MC and AC are the marginal cost and average cost,
dx X

respectively.
19. The total cost function is given by C = .v + 2x^ - 3.5x^. Find the marginal average cost
function {MAC). Also, find the points where the MC curve cuts the .r-axis and y-axis.
20. IfC (.y) = 0.05-Y - 0.2 x^ -5, find the level of output a: for which the average cost AC becomes
equal to the marginal cost.
21. If the total cost of producing and marketing a: units of a certain commodity is given by
-.x/3 MC -AC

w
C (at) = 3ae . Verify that the shape of AC curve is given by X
3
22. Given the total cost function for a: units of a
commodity as C (,y) = ^ + 3a:^-7,y + 16. Find

o
(i) the marginal cost (ii) the average cost. Also, show that the marginal average cost is given

e
a: MC - C (.v)
by

re
a-2

rFl
F
ANSWERS
1500
1. (i) AC = + 30 + a: (ii) MC = 30+ 2.Y (iii) MC =70 (iv) 71

r
.V
ou
2. MC =
5.V (.Y^ + 6)
4. (i) 48
fo
(ii) 192
ks
(,y2 + 3)3/2
15000
oo

7. (i) AC = 0.005.y3 - 0.02.Y - 30 + (ii) MC = 0.015-y^ - 0.04.V - 30


.Y
Y
eB

9. MC = 0.0008^3 - 0.10a: + 7, MC = 797


10. (i) 2.5 (ii) 4 11. (i) 2.5 (ii) 6 12. (i) 16.5 (ii) 36
r

.2
3
13. (i) + 2.V -15 (ii) — + .y -15 + -
ou

14. (i) 8 (ii) 16


Y
ad

3 .Y

15. C(.y)=2y^-11.y + 50, MC =4y-11,0<.y<5 16. 6 units

3.1-2
d

X (\
17. MC = 30 19. 4.Y -35,(0,1),(1,0) , -,0
Re
in

200 25 v6 )
x2
F

20. 5 22. (i) MC=.v^ + 6x-7 (ii) AC = —+3x-7


3

12.5 MINIMIZATION OF AVERAGE COST

In the previous section, we have introduced the concept of average cost. An economist generally
wants to find the level of output for which the average cost is minimum. In this section, we shall
discuss problems on minimization of the average cost.
The following algorithm will be helpful for the same.
ALGORITHM

STEP I
Obtain the average costfunction AC. If the cost function C=C{x) is given, then obtain AC by
using AC = —.
X

STEP 11 Find — (AC).


dx
APPLICATIONS OF DERIVATIVES IN COMMERCE AND ECONOMICS 12.17

STEP III Put — (AC) = 0and solve it to get the values ofx. Reject negative values, if any.
dx
d^
STEP IV Find
dx
2(^0-
.2

STEP V Find the values of (AC) for the values ofx obtained in step HI. The average cost luill be
dx^

minimum for those values of x for which —^


dx^
(AC) > 0.

ILLUSTRATIVE EXAMPLES

w
EXAMPLE 1 The manufacturing cost of an item consists 0/? 2000 as overheads, material cost ? 2 per
2

item and the labour cost — for x items produced. Find how many items may be produced to have the

o
90

e
average cost as minimum.

re
rFl
SOLUTION Suppose X items are produced to have the average cost as minimum. Let C denote

F
the total cost of producing .v items. Then, .2

C = Fixed cost + Variable cost = Fixed cost + Material cost + Labour cost = 1000+ 2.r+ —
90

r
ou
AC
C
= - ^ AC =
1000
+ 2 + —. fo
ks
X X 90
2
2000
-^{AC) =
1000 1
Now, AC =
1000 «
+ 2 + —
X
4-(AC) = + — and,
oo

X 90 dx .r^ 90 dx^
Y
eB

For AC to be minimum, we must have


d ,, 1000 1
= 0 => x^= 90,000 => .V = 300
— (AC) = 0 => =- + —
dx 90
r
ou

2000
Y
ad

For X = 300, we have


dx
j(AC) = (300) 3>0
Thus, AC is minimum when x = 300. Hence, AC is minimum when 300 items are produced.
d

-10^2+ '^^—.
Re
in

EXAMPLE 2 The cost function of a firm is given by C = 300 x Calculate the output at
3
F

zvhich the marginal cost is minimum. Will it be same at zvhich average cost is minimum ?
.3

SOLUTION We have, C = 300 .y - IOy^ + ^


dC
= 300 - 20y + y2 => MC = 300 - 20y + y^
dx
, .2

—(MC) = - 20 + 2y = 0 and, —^(MC)=2


dx dx

For MC to be minimum, we must have — (MC) = 0 =i> -10+lx => Y=10


dx

^2
For Y = 10, we have —;2 (MC)=2>0
dx

Thus, MC is minimum when Y = 10. Hence, MC is minimum when the output is of 10 units.
12.18 APPLIED MATHEMATICS-Xll

Now, C = 300.v-10a:^2 + —
3

c
AC=- = 300-10.r + —
a: 3
.2 >
.V 2.V
300-10.t + — =-10 + — and,
dx dx

w
3 3 dx^ 3

d 2t
For AC to be minimum, we must have -^(AC)=0
dx
=>-10 + —
3
= 0=> a:=15

.2
' 2

e
Clearly, —;2 (^C) = — > 0 for all X. Hence, AC is minimum when the output is of 15 units.
dx

re
o
Clearly, AC and MC are not minimum at the same level of output.

r
ALITER AC is minimum when MC = AC. Therefore, AC is minimum, when

F
y2 2.y^
MC = /4C => 300 - 20y + x^ = 300 - IOy + — -10.\: = 0 => 2.r^ -30.v=0 => y=15
oF
ul
3 3

EXAMPLE 3 The totnl cost function of producing and marketing x units of a commodify is given by

sr
C = 16 - 12.t + 2.v^. Find the level of output at which it is minimum.

o
k
SOLUTION We have,

C = 16-12.r + 2Y^ =>


dC
= 0 -12 + 4.Y and,
of
d^C
= 4
dx dx^
o
Y
For C to be minimum or maximum, we must have
rB

dC
= 0 => -12 + 4.V = 0 => .Y = 3
eY

dx

d^C
Clearly, = 4 > 0 for all x. Hence, C is minimumwhen the level of output is 3 units.
u

dx-
d

IXAMPLE 4 Given, the total cost function for x units of a commodity, C (.v) =ax^ +bx^- cx + d.
o
ad

wherea > 0,b <0, oO. Shoio that average variable cost and marginal cost curves intersect at minimum
in

average variable cost.


SOLUTION We have, C (y) =ax^ + bx'^ — cy + d. Therefore, Variable cost = ay^ + bx'^
Re

cx
F

Let AVC denote the average variable cost. Then,


Variable cost
AVC =
X

AVC = ax'^ +bx- c => — {AVC) = lax + b and. 2a


dx

For maximum or minimum value of A VC, we have


4-
dx
(AVC) = 0 => 2flY +1; = 0 => Y = - —
2a

Clearly, —;j (AVC) = 2(7 > 0 for all y


ri 1

dx
[V (7>0]

So, AVC is minimum when x = -b/2a.We have to show that/4 VC =MC at x=-b/2a.
dC
Now, C =ax^ + hx^ - CY + d => = 3ax^ + 2bx - c => MC = 3 ax^ + 2bx - c
dx
AVC=MC
12.19
APPLICATIONS OF DERIVATIVES IN COMMERCE AND ECONOMICS

ax:^+bx-c = 3ax^ + 2bx-c^ 2ax^+bx = 0=> .v = 0 and x = ~b!2a


Hence, AVC = MC when x = -b/2a. i.e. AVCisminimum.
EXAMPLE 5 The total cost function of a product is given bij C (a) = a^ - 315a^
+ 27,000.t + 20,000, where x is the number of units produced. Determine the number of units that should
be produced to minimize the total cost.
SOLUTION We have,
C {x) = x^ - 315x^ + 27,000a- + 20,000

dx
(C (a)) = 3a^ - 630a + 27,000 and, dx~
(C (a)) = 6a - 630

For maximum or minimum values of C (a), we must have

w

dx
{C (a)) = 0 ^ 3a^ - 630a + 27,000 = 0 ^ a^ - 210a + 9000 - 0 ^ a = 150, 60.

F lo
= 6 X 150 - 630 > 0. So, C (a) is minimum when 150 units are
Clearly,
dx
j(C (.a)
h = 150

ee
produced.

Fr
a3
+5a+ 3.Findthe
EXAMPLE 6 A firm produces a units of output per week at a total cost of^ — - a
output levels at which the marginal cost and average variable
for
cost attain their respective minima.
ur
SOLUTION Let C be the total cost of producing a units of output per week. Then,
s
.3 dC
ok

a
= a^ -2a+5 => MC = a^ -2a + 5
Yo

C = - a + 5a + 3 ^
3 dx
o

A
eB

Now, MC = .Y^ - 2a + 5 => — (MC) = 2a - 2 and.


dx
-%-(MC)
dx^
=2
r

For MC to be minimum, we have


ou
ad

d
(MC)=0 =>2a-2 = 0 =>a=l.
dx
Y

.2

Clearly, (MC) = 2 > 0 for all a. Tlius, MC is minimum at a = 1.


Re
nd

dx^
a3
Fi

Again, C = — - a^ + 5a + 3
a^ a^
=> Variable cost = — -a^ + 5a and, average variable cost (AVC) = — -a+ 5
3
Variable cost
Average variable cost = a

.2 j2 9
Now, AVC
X
= —-a + 5
3
— (AVC) = —-1 and,
dx 3
^(AVC)
dx^ =-3
For AVC to be minimum, we have
2a
f(AVQ=0^
dx g3 -1 =0 => a = -
2

A y 3
Clearly, (A VC) = - > 0 for all a. So, A VC is minimum when a =
dx^ 3
12.20
APPLIED MATHEMATICS-Xl)

liXAMPLE
A machine initially costs ? 6400 zvith no scrap value. The cost of operating is ? 500 in the
first year and increases by ? 800 in each successive year. Determine (i) the number of years it be operated
for minimizing total operating cost per year, and (ii) corresponding cost per year.
SOLUTION Suppose the machine is operated for .r years. Then,
Operating cost = 500+1300 + 2100 +... + a: terms

= |{2x500 + (.y-1)x 800} Using: =1 {2a + {n-l) d}


= -t(500 + 400.y-400) = 400.y^+100.t
It is given that: Capital cost = ? 6400
Total operating cost = 400x^ + lOO.v + 6400
400.t^ + lOO.Y + 6400
=^>
AC = Total operating cost per year =

w
X
6400
AC = 400.V +100 +
X
...(i)

Flo
d 6400 12800
-- (/4C) = 400 - — and.
dx x~ x^

ee
For minimum AC, we must have

Fr
6400
~ (AC) = 0 => 400 - = 0 => =16 => x = 4
dx .v2
d-^ for
ur
12800
At .Y = 4, we have (AC) > 0. Thus, AC is minimum when x = 4.
dx^ 64
ks
6400
Putting .Y = 4 in (i), we get: AC = 1600 +100 +
Yo

= 3300
oo

Hence, the total operating cost per year is minimum if the machine is operated for 4 years and
eB

the minimum cost is ? 3300.


EXA.MPLEs
The manufacturing cost of an item consists ofX 900 as overheads, the material cost is ^ 3
r
ou

per item and labour cost ? — for .Y items produced. How may items must be produced to have average
ad

100
cost minimum?
Y

,2
SOLUTION Let C (x) denote the cost of product of .y items. Then, C (x) = 900 + 3x + —.
nd
Re

100
C(x)
Fi

Now, AC =
X

AC =
900 -900 1 d^ A 1800
+ 3 + — => —(AC) = + and.
X 100 dx a-2 100 dx^ x3
For minimum AC, we must have
d 900 1
— (AC) = 0
dx
- +
= 0 ^ x^ = 90000 => X = 300 [●●● X > 0]
x^ 100
d^ A
Clearly, > 0 for X =
300. Hence, AC is minimum when 300 items are produced.
dx^
EXAMPLE 9 The cost function of a firm is C = 5x^ + 28x + 5, luhere C is the cost and x is the level of
output. A tax at the rate off 2 per unit output is imposed and the producer adds it to his cost. Find the
minimum value of average cost.
APPLICATIONS OF DERIVATIVES IN COMMERCE AND ECONOMICS 12.21

SOLUTION Itis given that the tax perunitofoutputis?2. Therefore,forxunits the tax added
to the cost is ? 2x. So, the total cost function C (.r) is given by
C (-Y) = 5.v^ + 28y + 5 + 2x
C (.v) = 5x^ + 30x + 5
d 10
/IC = = 5x + 30 + - ^ _(AC)=5- and, 2-(^C) = .r3
X X dx dx

For maximum or minimum value of AC, we must have

— (/4C) = 0=>5-4t=0=>.y = ±1 => X = 1 [v Y>0]

ow
dx X

Clearly, j{AC) = 10 > O. Thus, /' T is minimum when 1 unit is produced.


dx
x= 1

e
Also, Minimum average cost (MAC) = 5 + 30 + - = 40

re
1

hXA.MPLE 10

rFl
A manufacturer produces computers data stora'^efloppies at the rate ofx units per week

F
(x^
- lOx^ + 15x + 30 . Find the optimal
and his total cost of production and marketing is C (.x) = ^ —
o

or
ou
number of floppies produced per week at which the marginal cost and average variable cost attain their
ksf
respective minima.
SOLUTION We have.
oo

.3
C (x) = ~ -lOx^ + 15x + 30
Y

3
B

dC
x^ -20X + 15 => MC =x^ -20X + 15
re

dx
oYu

— (MC) = 2x - 20 and, 2(MC) = 2


ad

dx dx

For maximum or minimum value of MC, we must have


d

d
(MC) = 0=> 2x-20 = 0^ x=10
in
Re

dx

d^
F

Clearly, 2(MC) = 2 > 0. Therefore, marginal cost is minimum when 10 floppies are
dx
/x = 10

produced and marketed.


.3 x^
Again, C (x) =-3 lOx^ + 15x + 30 => Variable cost (VC) = —3 -lO.v^ +15.X and. Fixed cost
(FC) = 30

Average variable cost (AVC) =


VC
X
if ^ -lO.v^ +15x
X 3
x2
3
-lOx +15

■y

-f^(AVC) = \
X“ d 2x
Now, AVC = —-lOx + 15 => — (A VC) = —-10 and,
3 dx 3 dx ^

Maximum or minimum value of AVC, we must have


d 2x
-10=0 => x=15
dx (AVC)=0 =>y
12.22 APPLIED MATHEMATICS-XIt

^{AVC)
2
Clearly, — >0. Hence, AVC is minimum when 15 floppies are produced
dx
/x=15
and marketed.

EXERCISE 12.4

!. Given the cost fimction C = IOjc - + 6;r^, find the minimum average cost.
The total costC (a:) of a firm is:C (.y) = O.OOSa:^ - 0.02x~ - 30.y + 5000, where .y is the output.
Determine:

(i) the average cost (ii) marginal cost.


Given that the total cost function for .y units of a commodity.
C (y) =(1/3) Y^ + 3y^ -7y + 16. Find:

w
(i) the marginal cost (ii) the average cost (iii) the marginal average cost i.e. — (TlC)
dx

F lo
y(MC) -C
(iv) show that marginal average cost =
x2
given by C (.y) = yjax + b + c. Show that

ee
4. The total cost function for y units of a commodity

Fr
the marginal cost decreases as the output y increases.
The total cost function for x units of a commodity is given by
C =5000 + IOOOy -500y^ + (2/3) x"'*' Find:
for
ur
(i) the average cost function (ii) the marginal cost function (iii) the expression for the slope
of MC curve.
s

Y^
ook
Yo

6.
A manufacturer's total cost function is given by C = — + 3x + 400, where x is the number
eB

of units produced. At what level of output will average cost be minimum? What is this
minimum ?

Let the cost function of a firm be given by C = 300 x -10 x^ + (1/ 3) x^' where C stands for
r
ad
ou

cost and x for output. Calculate : (i) the output at which marginal cost is minimum, (ii) the
output at which average cost is minimum (iii) the output at which average cost is equal to
Y

the marginal cost.


Re

8. Given the cost functionC (x) ^x"^ -57x + 315x + 20. Find thelevelof output for which the
nd

cost is minimum.
Fi

0, If C = 0.01 X + 5x + 100 is a cost function, find the average cost function. At what level of
production, x, is there minimum average cost? What is this minimum?
x3 -
A firm has the following total cost and demand functions: C (.r) = ~ - 7x^ + lllx' + 50,
X = 100 - p. Find the profit maximizing output.
x3
+ x^ - 8x + 5
commodity as C (.v) = ^
11. Given that the total cost function for x units of a

Find: (i) the marginal cost fimction (ii) average cost function (iii) slope of average cost
curve.

The average cost function associated with producing and marketing x units of an item is
given by AC =2x-ll + — .Find :
X

(i) The total cost function and marginal cost function,


(ii) The range of values of the output x, for which AC is increasing.
APPLICATIONS OF DERIVATIVES IN COMMERCE AND ECONOMICS 12.23

_ANSWERS
5000
1. 74 (i) AC =0.005a.-^-0.02x-30 + .t
(ii) MC = 0.015.y^ - 0.04.V - 30
.2 2x
(i) MC = x~ + bx -1 (ii) AC = — + 3-i: -7
3 (iii) MAC = y + 3
5000
5. (i) AC = + 1000-500a: + -.v^
3
(ii) MC = 1000-lOOO+ 2at^
a:

(iii) Slope of MC curve = -1000+ 4.v 6. = 40, Min. AC = 23


(iii) .V = 0 or .v =15 8. 35
7, (i) 10 (ii) 15
9. 100, ? 7 10 .V = 11 11. (i) MC=a'^ + 2.y-8
.2

(ii)AC=^ + X - 8 (iii) ^{AC)Ax


dx 3 + \--.
x2

w
12. (i) C =2.y^-11.v + 50, MC=4.y-11 (ii) -v>5

F lo
12.6 TOTAL REVENUE, AVERAGE REVENUE AND MARGINAL REVENUE
In section 12.2, we have defined demand function of a commodity as a relation between the

ee
price per unit and quantity demanded at that price. If p is the price per unit when x units of a

Fr
commodity are demanded, then its demand functio i can be written as p =/ (x) or, x =/ (p).
In this section, we shall define total revenue, average revenue and marginal revenue.
for
TOTAL REVENUE Ifpisthepriccpcriirutarjdxisthemimberofmiitsofnn item sold at price p per unit,
ur
then the total revenue R is given by R = px.
AVERAGE REVENUE If R is the revenue received by selling x units of a commodity at price p, then the
s
ok

revenue per unit is known as the average revenue {AR).


Yo

px
o

i.e. AR =- => AR =
eB

X X

Thus, average revenue is same as the price per unit.


MARGINAL REVENUE The marginal revenue is defined as the rate of change of total revenue with respect
r
ou
ad

to quantity sold and is generally denoted by MR.


Thus, it R is the total revenue received by selling x units of a product, then
Y

dR d tip
MR = => MR = — (px) => MR = p + X -C =p
Re

f) dx)
nd

dx dx X

It is evident from the definition that the marginal revenue indicates the rate at which the total
Fi

revenue changes with respect to units sold and it can be interpreted as the approximate revenue
received from selling one additional unit of the product.
REMARK Generally, a firm operates under one of the following huo conditions: (i) Pure
competition (ii) Monopoly
(i) When a firm operates under pure competition, the basic assumptions are:
(a) There are many firms producing the homogeneous products,
(b) Price is determined by the market. Producer does not decide price,
(c) Price is independent of the output i.e. p is independent of x.
dp dp ^
p IS constant —=0
In this case, we have R
= px and MR=p + x-^ = p dx

(ii) When a firm operates under monopoly, the basic assumptions are:
(a) There is only one seller and there are no close substitutes for the product of the monopolists i.e.
the seller.
12.24 APPLIED MATHEMATICS-XII

(b) Demand depends on the price and vice-versa,


(c) The firm must lower its price to sell more items.
dp
In this case, we have R = px and MR = p + x
dx

NOTE Throughout our discussion we shall assume that p depends upon x, unless stated otherwise.
ILLUSTRATIVE EXAMPLES

EXAMPLE 1 The demand function of monopolist is given by p =100 - x -x'^. Find (i) the revenue
function (ii) marginal revenue function.
SOLUTION (i) Let R be the revenue function. Then,
R=px => R = (100 - X - A'^) X => R = 100a- - 'a^ - a-^
(ii) We have,

w
dR
R=100x-x^-x^ = 100 - 2x - 3a-^ i.e. MR = 100 - 2a - 3.y^
dx

EXAMPLE 2
function
F lo
The demand function for a monopolist is given by x = 100 - 4p. Find (i) total revenue

(ii) average revenue function (Hi) marginal revenue function (iv) price and quantity at which MR = 0.

e
Fre
SOLUTION We have, A=100-4p p for
(i) Let R be the total revenue. Then,
100-A- a2
r
R^px^ R = A- => R = 25a- - —
You

4 4
oks

2 'i
R 1
eBo

X X
(ii) AR = => AR = 25a-— =25--
A A 4 4

a-2 dR
(iii)
ad

R = 25a -
our

= 25 - i.e. MR =25--
4 dx 2 2

(iv) MR=0=>25-- = 0=>a=50


2
dY
Re

Putting A =50 in p =
100-A ^ ::
50 25
we get, p = - = 12.5
2
Fin

E'XAMPl.E 3 A firm knows tlmt the demand function for one of its products is linear. Italsoknoxvs
that it can sell 1000 units when the price is ? 4 per unit, and it can sell 1500 units when the price is ? 2 a
unit. Determine (i) the demand function (ii) the total revenue function (Hi) the average revenue function
(iv) the marginal revenue function.
SOLUTION (i) Since the demand function is given to be linear. So, let it be
X =ap + b ...(i)
where p is the price per unit and a is the quantity demanded at this price.
When p = 4, a- = 1000 and when p = 2, x -1500
1000 = 4a+ b and 1500 = 2a+ b

Solving these two equations, we get: a = - 250 and b = 2000


Putting the values of a and b in (i), we get
A = - 250p + 2000 ...(ii)
APPLICATIONS OF DERIVATIVES IN COMMERCE AND ECONOMICS 12.25

X
p = 8 ...(iii)
250

Hence, the demand function is .y = - 250p + 2000.


,v2
(ii) Let R be the total revenue function. Then, R =px=> R = 8.y - [Using (iii)]
250

(iii) The average revenue function is AR = — = p => AR = 8 - [Using (iii)]


dR X

(iv) The marginal revenue function is MR = = 8- [Using (iii)]


dx 125

EXAMPLE 4 The demand function for a certain product is represented b\/ the equation:
p=2Q + 5x- 3-v^, inhere x is the number of units demanded and p is the price per unit.

w
(i) Find the marginal revenue (ii) Obtain the marginal revenue when 2 units are sold.
SOLUTION We have, p = 20 + 5.v - 3.v^

Flo
Let R denote the total revenue. Then,

e
R=px => R = (20 + 5.Y - 3.y^) X => R= 20.r + 5.v^ 3.y^

re
F
(i) We have,
dR dR
R =20x + 5x^ -3x^ => = 20 + 10.Y - 9.v^ => MR == 20 + IOy - 9.y^ MR =
ur
dx

(ii) Putting .V = 2 in MR, we obtain: (MR) ^ = 2 = 20 + 10x2-9x2^ = 4.


f or dx
ks
Yo

Hence, the marginal revenue when 2 units are demanded is ? 4.


oo

EXAMPLE 5 For the demand fu}iction p = -c, where ab>c, shozo that marginal revenue
B

X + b
re

decreases with the increase ofx.


Let R be the total revenue. Then,
u

SOLUTION
ad
Yo

R = px
a
R = X - c
d

X + b
Re
in

ax + ab - ax ah -c
dR a ax a ax
- c
F

-c -c
dx X +b , (x + b)^ x + b (x + b)'^ (x + b)^ (.V + b)"^
ab -c dR
MR V MR =
dx
(x + b)^
2{ab-c) <0 [●.● ab >c => ab -c>0]
— (MR) =
dx {x + bf
Hence, MR decreases with the mcrease of .t.
EXAMPLE b The total revenue received from the sale of x units of a product is given by
R (.y) = 36-y + 3x^ + 5. Find
(i) the average revenue (ii) the marginal revenue
(iii) the marginal and average revenue ivhen .y = 5 (iv) the actual revenue from selling 50th
item.
12.26 APPLIED MATHEMATICS-XIl

SOLUTION We have, R (.v) = 36x + 3x^ + 5. Therefore,


R 5
(i) Average revenue = AR = — = 36 + 3.v + —
.Y -Y

ciR
(ii) Marginal revenue = MR =
dx
-^{36x+3x^
dx
+5) = 36 + 6-y
(iii) Wlien .v = 5, we get AR = 36 + 3 x 5 + - =52 and, MR = 36 + 6 x 5 = 66.
5

(iv) The actual revenue on selling 50th item


= (Revenue received on selling 50 items) - (Revenue received on selling 49 items)
= R (50) - R (49) = [36 X 50 + 3 (50)^ + 5] - [36 x 49 + 3 x (49)^ + 5]
= (1800 + 7500 + 5) - (1764 + 7203 + 5) = ? 33

w
nXAMPLE 7
A monopolist's demand function for one of its products is p (.y) = ax + b. He knows that he
can sell 1400 units zvhen the price is ? 4. per unit and he can sell ISOO units at a price of^ 2. per unit. Find
the total, average and marginal revenuefmctions. Also, find the price per unit zvhen the marginal revenue

F lo
is zero.

SOLUTION We have, p{x) = ax + b...{i)

ee
It is given that p = ? 4 when .y = 1400 and p = f 2 when .y = 1800

Fr
Substituting these values in (i), we get
4 = UOOa + b ...(ii) 2=1800n + ^ ...(hi)
1
for
ur
Solving these two equations, we get: a = - and b =11
200
s
Substituting the values of <7and& in (i), we get: v =—^ + 11 (iv)
ook
Yo

200

Let R (x) be the total revenue function. Then,


eB

R (x) = p x => R (x) = - + llx => Average revenue = AR = — - — =R =-


X X 200
r

-.y2 x2
dR ^ _d^
ou
ad

Now, MR + 11.Y -^+11 V R(x)=- + llx


dx dx 200 100 200
Y

MR=0=> - — + 11=0 => x = 1100


100
Re
nd

- X 1100
(iv) Putting X =1100 in ^ = +11, we get p = + 11=> p=?550.
Fi

200 200

EXAMPLE 8 For the demand function p = , show that the marginal revenuefimction is increasing
for allb <0, a > 0.
SOLUTION
We have, p= . Let R be the revenue function. Tlien,
a + X

X dR (a + x) b - bx ab lab
R =px => R = => MR — (MR) =
a + X dx (a + .Y)^ la + xf dx
(a + x)^
d - lab
Clearly, —(MR)>0. b <0 and<?>0 ab <0 =>
dx
(a + .y)
2>0
MR is increasing for all b <0, a >0.
12.27
APPLICATIONS OF DERIVATIVES IN COMMERCE AND ECONOMICS

EXAMPLE 9 hi the production unit of a firm it is found that the total number ofuni^s (.v) produced is
dependent upon the number of workers (n) and is obtained by the relation x = 25n {n + 36) , the
250
demand function of the product is p = X Determine the marginal revenue when n = 4.
+15

250
SOLUTION We have, p = Let R be the total revenue when x units are sold. Then,
.y + 15

250.V dR 250 {.V +15) - 250 ,Y 3750


=> A4R = ...(i)
R = px => R = =>
(,v +15)2
Y +15 dx (1 + 15)2
25 X 4
25» = 10
We have, x = Therefore, for n = 4, we obtain: y =
^64 +
yjn^ + 36
When » = 4 we obtain y = 10. Therefore, putting y = 10 in (i), we get

w
3750
MR = = 6
(10 + 15)^

Flo
_EXERCISE12.5

1. The demand function of a monopolist is given by p =1500 - 2y - y^. Find: (i) the revenue

ee
fimction (ii) the marginal revenue function (iii) the MR when y = 20 and interpret it.

Fr
2. The demand function of a monopolist is given by pY =100. Find the marginal revenue for
any level of output y.
3. A monopolist's demand function is given by /i = 200-5y. for Find the marginal revenue
ur
function. What is the relationship between the slopes of the average and marginal revenue
curves ? At what price is the marginal revenue zero ?
k s
5 where rt,!’ >0, show that the marginal revenue
Yo

4. For the demand function p =


oo

y+2 '
eB

decreases with the increase of y.


.Y"^
5. If R (y) rupees is the total revenue received from the sale of y tables, and R (y) = 600 ^
r
ou
ad

find
(i) the average revenue (ii) the marginal revenue when y = 25 (iii) the actual revenue from
Y

the sale of 26 tables.


.2
Y
nd

of a product is given by R(y) - 200 + —.


Re

6. The total revenue received from thesaleofx units

Find (i) the average revenue (ii) the marginal revenue (iii) the marginal revenue when
Fi

Y = 25.

7. A monopolist's demand function is p = 300 -5.v.


(i) Find the marginal revenue, (ii) At what price is the marginal
revenue zero ?
3 Y^
^
-7. Find
8. The total revenue fimction for y units of a commodity is given by R (.v) = y + —

the marginal revenue when the number of units of the commodity produced (y) is 20. Y
2

9. The total revenue received from the sale of y units of a product is given by R (.v) - 20y - 2

Find

(i) the average revenue (ii) the marginal revenue


12.28
APPLIED MATHEMATICS-XII

(iii) the marginal revenue when a: = 10 (iv) the actual revenue from selling
15th item.

jO. Find the relationship between the slopes of marginal revenue curve and average revenue
curve for the demand function x = where x denotes the number of units sold at the
a

price p per unit.


11. A monopolist's demand function for one of its products is p (a:) = aa' + b, where .v is the
number of units produced and p is the price per unit. If on selling 5 units the price is ? 1800
per unit and on selling 3 units the price is ? 2000. Find the total, average and marginal
revenue functions. Also, find the price per unit when the marginal revenue is zero.
U, In a factory it is found that the total number of units (a:) produced in a day depends upon the
5n
number of workers (n) and is obtained by the relation .v = . The demand function of
\
In+ 5
2
the product isp = —x. Determine the marginal revenue when n = 20.

w
The demand function for a commodity is given by p = - x/ 300 , where x is the quantity
f

F lo
a e

demanded and p is the price per unit. Given that the price is ? 7 per unit when 600 units of
the product are produced. Find the total, average and marginal revenue functions. Also,
find the price per unit when the marginal revenue is zero.

ee
. The price p per unit of a commodity and the number of units x of the commodity are related

Fr
linearly. If the consumer demands 50 units of the product when the price is ? 10 per unit
and 20 units when the price is ? 15 each. Find the demand function and the total, average
for
and marginal revenue functions.
ur
ANSWERS
s

I. (i) R{x)=1500x-2x^ -x^


ook

(ii) MR =1500-4.a:-3a:2
Yo

(iii) 220 0

3. MR=200-10.r, Slope of MR curve = 2 (Slope of AR curve), MR = 0 when u = 100


eB

x-2
●. (i) AR =600- — (ii) 525 (iii) ? 14896.96
25
our
ad

200 .A' 2.V


h. (i) AR = + —
(ii)MR=— (iii) 10 (i) MR = 300 -lO.r (ii) 3 units
A' 5 5

■ (i)AR=20-^ (ii) MR =20-A- (iii) 10 (iv) 5.5


1220
Y
Re

Id. SlopeofMR = 2(Slopeof AR)


nd

11. R (-Y) = 2300.t - lOO.t^, AR = 2300 - lOO.v, MR


Fi

= 2300 - 200a, ? 1150 12, ?40


2- ^ 2- — 2--
X

. R =7xe 300,/IK = 7e 300, mk = 7e 300 i_ ;7e


300
.2
.A
55-.A _ 55 X
, MR ,AR
6 3 3 6

12.7 MAXIMIZATION OF TOTAL REVENUE


In this section, we shall discuss some problems on maximization of total revenue. If p = / (a) is
the demand function of a firm's product, where p is the price per unit and a is the number of
units produced and sold, then the total revenue R, of the firm is given by
R =pA => R=xf (a)
Clearly, R is a function of a. To find the maximum value of R and the output level for maximum
value of R, we use the following algorithm.
12.29
APPLICATIONS OF DERIVATIVES IN COMMERCE AND ECONOMICS

ALGORITHM

STl:l’ I Obtain the total revenue R of the given firm.


STEP II Put ^
dx
= 0 and obtain the values ofx.
d^R
STL-J’ HI
dx^
d^R
SI I P IV From the values ofx obtained in step II, choose those values ofx for which dx
2<0-

sti:p \ Substitute the value ofx obtained in step IV in R to get the maximum value ofR.

w
Following examples illustrate the above algorithm.

ILLUSTRATIVE EXAMPLES

o
80 — V
-where .r is the number of

e
I XAMi’l F 1 The demand function for a manufacturer’s product is p = 4

re
rFl
units and p is price per unit. At what value ofx will there be maximum revenue ? What is the maximum

F
revenue ?

SOLUTION Let R be the revenue received by selling .r units. Then,

r
ou
„ (S0-x^ 1 7
- -A => R = 20.V - — .x~ fo
R =px ^ R =
I 4 4
ks
dR d^R _ 1
oo

= 20 - and,
dx 2 dx^ 2
Y
eB

For R to be maximum, we must have


dR X
= 0 => 20-- = 0 => -v = 40
r

dx 2
ou
Y
ad

Clearly,
d^R _ -1 < 0 for all .T. Hence, R is maximum when x = 40.
dx^ 2
d

Putting .V = 40 in (i), we get: R = 800 - 400 = 400. So, the maximum revenue is ? 400.
Re
in

-.r/3
r.XAMl’l I 2 The demand function for a particular commodity is p-\5e for 0 < .r < M > 8,
F

where p is price per unit and x is the number of units demanded. Determine the price and quantity for
which total revenue is maximum.

SOLUTION Let R be the total revenue. Then,

R =px=15e^''^^ X
dR -x/3
= 15 e-"-/3_£g-.v/3 = 5c (3-x)
dx 3

d^R d ,-x/3
(3 - .t) + 5 t'
dx 3

For R to be maximum, we must have


dR -.v/3
= 0 5 t? (3-.v)=0 => -t = 3.
dx
12.30 APPLIED MATHEMATICS-XII

Clearly, <0.
Tlius, R is maximum when x=3. Putting x=3 in
3
,v= 3

-.y/3
p =15 c , we get; p = 15e \ Hence, the total revenue is maximum when price per unit IS
i
15 e ^ and quantity demanded is 3 imits.
liXAMPLE 3
A T.V. compani/ has 1000 subscribers who are each paying ? 100 per month. The company
proposes to increase the monthly subscription and it is believed that for every increase of X 1, five
subscribers will discontinue the service. Find what increase will yield maximum revenue and what will
this revenue be ?

SOLUTION Let ? .T be the increase in monthly subscription. Then, New rate =100+ .r.

ow
Since, for each increase of ? 1, 5 subscribers will discontinue the service.
Total number of subscribers = 1000 - 5a:

Let R be the total revenue of the company. Then,


R = Ratex Number of subscribers

e
R = (100+ .v) (1000-5.Y) = 100000+ 500x-5.r2

re
=>

dR
rFl
d^R
... 1

F
= 500 -lO.Y and. = -10.
dx dx^

r
For R to be maximum, we must have
fo
ou
dR
= 0 500 - lO.Y = 0 =5> .Y = 50.
ks
dx

d^R
Clearly,
oo

= -10 < 0 for all X. So, R is maximum when x =50.


rfx2
Y
B

Puttingx = 50in(i)weget:R=10000+500x50-5x(50)^=112500
e

Thus, R is maximum when monthly subscription is increased by ? 50 and the maximum revenue
ur

is ? 112500.
ad

EXAMPLE 4 A company charges ^ 550 for a transistor set on orders of 50 or less sets. The charge is
Yo

reduced by ? 5 per set for each set ordered in excess of50. Find the largest size order company should allow
so as to receive a maximum revenue.
d
Re
in

SOLUTION Let x be the number of transistor sets ordered in excess of 50. Thus, the total number
of transistor sets ordered is (50 + x). Since for each set ordered in excess of 50, the charge is
F

reduced by ? 5 per set, therefore for each of x sets ordered in excess of 50, the charge is reduced
by 5x per set. Thus, the new charge per set is ? (550 -5x).
Let R be the total revenue of the company. Then,
dR d'^R
R={50 + x) (550 - 5.y) => R = 27500 + 300 x - 5x^ => = 300 -1 Ox and. = -10
dx rfx2
For R to be maximum, we must have
dR
= 0 => 300-10x = 0=> x = 30
dx

d^R
Clearly, < 0 for all X. Thus, maximum revenue is obtained when (50 + 30) = 80 sets are
ordered.
12.31
APPLICATIONS OF DERIVATIVES IN COMMERCE AND ECONOMICS

EXAMIM.E r Travel agenci/ has chartered a bus to Mussoorie for school children. The charge is ? WO
with an additional charge o/? 2.50 for each subsequent cancellation. The bus has a seating capacity of 50
children. Determine the total revenue R (.y), as afinidion ofx, the number of cancellations received prior
to the departure date. What is the value ofx for zuhich R (y) is maximum ?
SOLUTION Let x be the number of cancellations received prior to the departure of the bus.
Then,
Total number of children who will go to Mussoorie = (50 — y)
Amount to be paid by each child = (100 + 25x)
Total revenue = R (y) = (50 - y) (100 + 25y) = 5000 + 25y - 23y“
d d^R
Now,R(y)=5000 + 25y-25y^ => — dY
(R(-v)) = 25-5y and. dY-2
= -5

For R (y) to be maximum, we have


2

w
(K (.y)) = 0 => 25 -5y = 0 => y =5
dx^

F lo
2

Clearly, (R (y)) < 0 for all y. Thus, R (y) is maximum when y =5.
dx^
Hence, R (y) is maximum, if 5 cancellations are received prior to the departure.

e
Fre
EXAMPLE A steel plant is capable ofptyducingx tonnes per day of a low-grade steel and y tonnes per
n
40 -5y
day of a high-grade steel, where y = 10 - Y If the fixed market price of loiv-grade steel is half of that of
for
high grade steel, find the quantity of low-grade steel that should be produced per day for maximum
r
You

receipts.
oks

SOLUTION p per tonne. Then, the fixed


Let the fixed market price of low'-grade steel be ^
eBo

market price of high-grade steel is ? 2p per tonne. Let R be the total revenue. Then,
r40-5Y'l f-Y^ + 8o'
R = px + 2py =px + 2p 10 - Y
= P
10-Y
our
ad

dR
= P
(10-,Y) (-2y)+(-.y^ + 80)| ^ dR
=P i
X - 20y + 80

dx (10-y)2 I dx (10-y)2
dY
Re

For R to be maximum, we must have


dR
Fin

= 0
dx
x^ - 20y + 80
= 0
(10-.y)2
Y - 20y + 80 = 0

Y
20 ± J400 - 3^ Y = 10+2^5 => y=10±2x 2.236 .r = 14.472,5.528.
2

dR Y^ - 20y + 80 dR 20 d^R 40p


Now, = Pi 5—f = p\l -> =>
dx^ (10--y)^
dx
(10-.v)2 ) dx {W-xf
^d^R <0.
Clearly,
dx^ J, = 5.528

Hence, R is maximum when 5.528 tonnes of low-grade steel are produced per day.
12.32
APPLIED MATHEMATICS-XII

l-.XAMPLEy
The revenue function of a product is given by the relation y = 4,000,000 - (.r - 2,000)^,
where y is the total revenue and x is the number of units sold, Find:
(i) What number of units sold maximizes total revenue?
(ii) What is the amount of maximum rezKmue ?
SOLUTION (i) We have,

y = 4,000,000-(.r-2,000)2 y=4,00Qx-x^ ^ ^
dx
^ 4,000-2.t and, = -2
dj2
For maximum value of y, we must have
^=0
dx
4,000-2x = 0 => -v = 2,000
Clearly, - 2 < 0 for all values of .r. Thus, y is maximum when x = 2,000.
d.v2

w
Hence, 2,000 units must be sold for maximum revenue.

(ii) Putting ^ = 2,000 in y = 4,000,000 - (.v - 2,000)2, we get y^^^ = 4,000,000.


Hence, maximum revenue = ? 4,000,000.
EXAMPLE 8

F lo
If the demandfunction is p — yj6~x, find at what level of output, x, the total revenue will

ee
be maximum.

Fr
SOLUTION
Let R be the total revenue. Then, R =px=> R = x ^6 -x.
Now, R = X .J^-X for
12 - 3.r
ur
dR 3^[6^ +
^ .V
2 (6 - X) - X 12 - 3x
d^R _1
27^ 2 7^
and.
s

dx
24^ rf.\-2 2
ook

6-x
Yo
eB

For maximum value of R, we must have


dR 12 - 3x
= 0 => = 0 => 12 - 3.Y = 0 ^ .r = 4
2 fe-x
our

dx
ad

d^R -3
Clearly, < 0. Hence, R is maximum when x = 4.
dx2 2V2
Y

^.v = 4
Re
nd

1
EXAMPLE 9
The unit demandfunction is x = - (24 - 2p), wherexis the number of units demanded and
Fi

pisthepriceperiinit.Find (i) The revenuefunction R in lermsofpricep. (ii) The price and the number
of units demaridedfor zuhich the revenue is maximum.
1
SOLUTION We have, x =
3(24-2^)
= px => R =px^(24-2p) ^ R --(24y-2y2) ^ d^R _ 4
R dR
3 3
= — (24 - 4p) and.
dp 3 dy2 3
For maximum value of R, we must have
dR
= 0 =>
— (24 - 4p) = 0 => /? = 6
dp
d^R -4
Clearly, — < 0 for all p. Hence, R is maximum when p =6.
dy2
12.33
APPLICATIONS OF DERIVATIVES IN COMMERCE AND ECONOMICS

1
Putting p = 6 in .Y = -3 (24 - 2p), we get y = 4.
Hence, R is maximum when 4 units are demanded at the price of ? 6 per unit.
EXERCISE 12.6
121
1. If the demand function for a certain commodity isp = -1, where p is the price per unit
Y + 4

when Y units are demanded, find the output level at which the total revenue is maximum.
Also, find the maximum revenue.
2. For a monopolist's product, the demand function is y = 10,000 c ^ ■ Find the value of p
for which maximum revenue is obtained.
If the demand function is p = find at what level of output y, the total revenue will be
maximum and what will it be ?

w
4. A tour operator charges 1136 per passenger for 100 passengers with a refund of ^ 4 for each
10 passengers in excess of 100. Determine the number of passengers that will maximize the
5.
amount of money the tour operator receives.

F lo
A company notices that higher sales of a particular item which it produces are achieved by

e
lowering the price charged. As a result the total revenue from the sales at first rises as the

Fre
number of units sold increases, reaches a maximum and then falls off. This pattern of the
total revenue is described by the relation : R = 4,000,000 - (.v - 2000)^ where R is the total
for
revenue and y is the number of units sold. Find (i) What number of units sold to maximizes
r
total revenue ? (ii) What is the amount of maximum revenue ? (iii) What would be the total
You
oks

revenue if 2500 units are sold ?

A company charges ^ 15000 for a T.V. set on orders of 60 or less sets. The charge is
eBo

6.

reduced on every set by ? 100 per set for each set ordered in excess of 60. Find the largest
size order the company should allow so as to receive a maximum revenue.
180 - .-
our

where y is the number of


ad

7. The demand equation for a manufacturer's product is p = —^


units and p is the price per unit. At what value of y will there be maximum revenue? What is
the maximum revenue?
dY
Re

8. The revenue fimction of a product is given by the relationy = 600,000 - (.v - 2000)^, where i/
Fin

is the total revenue and y is the number of units sold,


(i) Find what number of units sold maximizes total revenue?
(ii) Wliat is the amount of maximum revenue?
9 A steel plant is capable of producing y tormes per day of a lower grade steel and i/ tonnes
5y-40
per day of a high grade steel, where y = . If the fixed market price of low-grade steel
Y-IO

is one-third of that of high-grade steel, find the level of output of low-grade steel per day
for maximum revenue.

10 The demand function for a product is given by p =15900 - 9y - 2Y^.Find the level of output
at which the total revenue is maximum.
11. A supplier charges packaging cost of ?. 620 for a digital diary on order of 50 or less set. The
charge is reduced by ^ 5 pre set if the order exceeds 50. Find the optimal number of digital
diary order, the supplier should allow so as to receive a maximum revenue.
12.34
APPLIED MATHEMATICS-Xli

ANSWERS

I. x=l8,R =81 2. 50 ^ 6,6^3 4. 220


5. 105 6. (i) 2000 (ii) 4,000,000 (iii) 3750000
x = 90, ?2025 8. (i) 2000 (ii) ? 6,000,000
9. 4.52 tonnes 10. 50 "’.27

12.8 MAXIMIZATION OF TOTAL PROFIT

In this section, we shall discuss some problems on maximization of total profit of a


manufacturer or company. Recall that ifC (x) denotes the total cost incurred in producing and
marketing x units of a commodity and R (x) denotes the revenue received in selling those x units,
then the profit P (.v) is given by P (x) = R (x) - C (x) or,P = R - C.
dP dR dC dP
dx dx dx dx
For P to be maximum, we must have

w
dP
= 0=> MR-MC =0 => MR=MC ...(i)
dx

F lo
dP d^P d^P
Now,
dx
= MR -MC =>
rfx2 dx dx dx
y<0, if dx
^(MR)<—(MC)
dx

ee
Thus, the total profit is maximum if MR = MC and — (MR) < — (MC).

Fr
dx dx
Graphically, if the slope of total revenue curve is equal to the slope of the cost curve and the
for
slope of the marginal revenue curve is less than that of marginal cost curve, then the total profit
ur
is maximum. ^
s
ook

ILLUSTRATIVE EXAMPLES
Yo

i-:\,AMPLE 1 A compamj has produced x items and the total cost C and total revenue R are given by the
eB

equations C =100 + 0.015x^ and R = 3x. Find how many items should be produced to maxhnize the
profit. What is this profit ?
r
ad
ou

SOLUTION
Let P be the profit function. Then,
P^R~C = 3x- (100 + 0.15x^) = 3x -100 - O.OlSx^
Y

dP d^P
- 3 - 0.03.x and. = -0.03
Re

dx
nd

For P to be maximum, we must have


Fi

dP
= 0 => 3-0.03.x = 0=s x = 100
dx
d^P
Also, - - 0.030 < 0 for all x. Thus, P is maximum when x = 100.
rf.X-2
Putting X = 100 in P = 3x -100 - 0.015x^, we get: P = 300 -100 - 0.015 x 10000 = 50

Hence, P is maximum when x =100 and the maximum profit = ? 50.


f “
i;
A given product can be manufactured at a total cost C (.x) = ? + lOOx + 40 , where x
\

is the number of units produced. The price at which each unit can be sold is given bu p = ? f 200 - -—1
I 400J'
Determine the production level x at which the profit is maximum. What is the price per unit and total
profit at the level of production?
12.35
APPLICATIONS OF DERIVATIVES IN COMMERCE AND ECONOMICS

SOLUTION Let the revenue and profit functions be denoted by R (-v) and P (x) respectively.
Then,

R(.v)=p.x= 200 400 L1.v^K(.v)=200x-^


.2
— + lOO.r + 40
Now, P(:r) = ^ W-C(--^-)= 200.V- 400 100

●●■(i)
P(.v)=100.r-^
^ 80
40
-1
— (P(.v))=100-^ and,

ow
dx 40 dx^ 40

For P (.v) to be maximum or minimum, we must have


A.
dx
(p (.v)) ^0 ^ 100 - 40 = 0 => A' = 4000

e
2

re
Clearly,
dx-
{P (X)) =
-1

40

Fl< 0 for all A. Therefore, P (.v) is maximum when a = 4000 i.e. when 4000

F
units are produced.
ur
V ^

r
Putting A = 4000 in p = ^ — and in (i), we obtain
200 - 400
4000
fo
ks
Price per unit = ? 200 - = ? 190 and. Maximum profit
400
Yo
oo

(4000)^ -40 = ? 199,960


= ? 100 X 4000 -
80
eB

EXAMIT.F.3 Find the profit maximizing output level, given x = 200 - lOp and AC -10 + where x
ur

represents the units of output, p the price and AC, the ni'erage cost.
ad
Yo

SOLUTION LetC (.a) and R (a) denote the total cost and revenue functions respectively. Then,
AC =10+—^ — +
d

25 A 25 25
Re
in

200^^ 20a-^'
. \ 200 - A
r 200 - A
V A = 200-10p=> P =
and, R = px => R = a 10
F

10 10
10

Let P be the profit function. Then,


-y ^
A“ ( 7a^
P =K-C = 20a - 10a+ — = 10a-
10 25 50
J V

dP 7a d^P -7
= 10 —^ and.
dx 25 dx^ 25

For P to be maximum or minimum, we must have


dP 7a 250
— = 0 => 10- — = 0=> A =
dx 25 7

250
d^P -7
Clearly, < 0 for all A. Hence, the profit maximizing output level is given by a - ^
dx^ 25
12.36
APPLIED MATHEMATICS-XII

f 2
i-.XAMPLE4 A radio manufacturer produces x sets per week at a total cost of Rupees- — + 3;t + 100

He is a monopolist and tlm demand ofhis product is x =75 - 3p, where pis the price in rupees per set. Show that
the maximum net revenue is obtained wheii about 30
sets are pvvduced per week. What is the monopohj price ?
SOLUTION
We have, C (.r) = — + 3.v + 100 and .y = 75 - 3p. Let K (x) be the total revenue

function. Then,
.2
75-Y^ Y
R (y) = pY => R (y) = .r = 25y- —

ow
I 3 3

Let P denote the net revenue received when y sets are produced and sold. Then,
Y-2 y2 - 28y- ..
P=K(y)-C(y) ^ P=25y ' 3y -100 => P = + 22y -100
3 25 75

e
re
dP _ 56y d^P _ 56
dx
+ 22 and.
dx^ 75

rFl
F
For maximum or minimum values of P, we must have
dP 22x75

r
= 0 => - ~x + 22 = 0 => Y =
ou
= 29.46
dx

d^P 56
75 56
k sfo
Clearly, - — < 0 for all Y.
rfY-2 75
oo

Hence, P is maximum when y = 29.46 i.e. when about 30


Y

sets
are produced per week. Putting
B

Y = 29.46 in y = 75 - 3p, we get


29.46 Y = 75 ~ 3p => 3p = 75- 29.46 ^ p = 15.18
re

Hence, the monopoly price is X 15.18.


ou
Y
ad

5 The relation between profit P of a ifrm and the selling


LX.AiVIPLE
price x of its goods is
P(.y) =2000 Y -50y^, 0 <y<40. For what range of selling price, is profit increasing ? For what
d

selling price is profit maximized ?


in
Re

SOLUTION We have, P = 2000y -50y^, where O < y < 40.


F

dP d^P
= 2000-100 Y and. = -100
dx
dx^
For P to be increasing, we must have
dP
— >0=> 2000-100y>0 => 100(20-y)>0 ^ 20-y >0 => y <20

So, the profit is increasing if the selling price y e (0, 20).


For profit to be maximum, we must have
dP
= 0 => 2000-100 Y = 0 => y = 20
dx

d^P
Clearly, < 0 for all .V. Thus, the profit is maximum, if the selling price is 20.
dx-
APPLICATIONS OF DERIVATIVES IN COMMERCE AND ECONOMICS 12.37

.3
EXAMPLE 6
Afirm has thefollowing total cost and demandfunctions ^ -7x^ + lll-v + 50 and

x=100~p.
(i) Write the total revenue function in terms ofx. (ii) Formulate the total profit function
P in terms ofx.
(Hi) Find the profit maximizing level of output x. (iv) What is the maximum profit ?
SOLUTION (i) Let R be the revenue function. Then,
R=p.x
R =(100 -:r) x [.■ .x = 100-;j=> p=m-x]
R =100x-.v^

w
=>

(ii) We have,
3 3
+ in.v + 50 => P=- — + 6.y^ -llY -50. ...(i)
X
P=R-C => P = (lOO.v-.x^)- —-7.v^
3 3

o
e
(iii) We have.

re
d^P
P=-
y3
+ 6y^ -ll.v-50 =>
dP
= -Y^+12.r-ll and.

rFl dx^
= -2y + 12

F
3 dx

For P to be maximum, we must have

r
— =0=> -.v^ +12.Y-11 =0 :=> -12.Y + 11 =0 => (Y-1)(Y-11)=0 => Y=l,ll
ou
dx fo
ks
d}p
Clearly, = -2 X 11 +12 = -10 <0. So, P is maximum when y =11.
dx^ Ay
oo

= n
Y

Thus, the profit maximizing level of output is 11 units.


eB

(iv) Putting Y =11 in (i), we get


(11)3 + 6x11^-11x11 -50 = 111.33. So, maximum profit =111.33.
r

P =-
ou

3
Y
ad

EXAMPLE 7 A radio manufacturer finds that he can sell x radios per week at ^ peach, where
\ Y^l
p = 2(100-y/4). His cost of production of x radios per week is ? ● 120 y + -)-. Show that his
d
Re
in

profit is maximum when the production is 40 radios per week. Find also his maximum profit per zveek.
F

SOLUTION Let C and R denote the cost and revenue functions. Then,
.2 2

C = 120 Y + - —2 and, R=pY=2fl00--lY=>


I 4J
R=200y-V2
Let P be the profit function. Then, \ /
.2
Y-2
P = R-C => P = 200y - 120 Y + — ^ P=80y-.y^ ...(i)
2 2
y \

dP d^P
— = 80 - 2y, and = -2
dx dx^
For P to be maximum, we have
dP
= 0=> 80-2y = 0 => y=40.
dx
12.38 APPLIED MATHEMATICS-XII

d^P
Clearly, = - 2 < 0 for all x. Thus, the profit is maximum when 40 radios are produced per
dx^
week.

Maximum profit = ? (80 x 40 - (40)^) = ? 1600. [Putting a: =40 in (i)]


l:\AMPLfI 8
The relationship betiveeu sales, TR (.y), and advertising cost, f x is given by
32,OOO.y
TR (X) = —. It is known that gross profit is 25% of the sales. Determine (i) the corresponding net
500 + -A

profit as a function of x, P (a), (ii) the value ofx -which maximizes P (a) and (Hi) maximized P(x).
SOLUTION (i) It is given that gross profit is 25% of the sales.
Gross profit = 25% of TR (a) = 25 32,000 A _ 8,000 a
100 "" 500 +A " 500 + A

low
Net profit P (a) is given by
8000 A
P (a) = Gross profit - Advertising cost = - A
...(i)
500 + A

ee
(ii) We have. rF
8000 A
fL(^P(x)) = (500 + -v) 8000 - 8000 a ^ _ 4,000,000 -1

Fr
P(.Y) = - A =5>
500 + A
' (500 + a)2 ~(500 + a)2
For P (a‘) to be maximum, we must have for
4-(P(.v))=0=> 4,000,000
-1 = 0 => (500 + a)^ = 4,000,000 => 500 + a = 2000
u
=> a = 1500.
dx
(500 + .y)^
ks
Yo

d 2
4,000,000
%('’(.v)) = -8,000,000 <0
oo

Now, ■^(Pix)) = for all A >0.


dx
(500 + xf dx^ (500+ .v)^
eB

So, P (.y) is maximum when advertising cost is X 1500.


(iii) The maximum value of P (a) is given by
r

8000 X1500
ou
ad

P{x) = -1500 = 4500.


[Putting A =1500 in (i)]
500 +1500
Y

So, maximum profit is ? 4,500.


nd
Re

-LXAMTLC9
For a monopolist's product, the demand function isp=~ and average cost function is
yjx
Fi

100
^ J
AC =0.50 +
Find the proift maximizing price and output. At this level, show that marginal revenue
is equal to marginal cost.
50
SOLUTION We have, p =

R=px^ R =50 fx
1000
It is given that AC = 0.50 + . Therefore, C = a AC => C = 0.50a + 1000.
A

Let P (.y) be the profit function. Then


P{x)=R-C= 50 ^/A-0.50a-1000 = 50Va---1000
2

d 25 1 25
^{P(x)) = r- — and,
dx VT 2 a3/2
For maximum profit, we must have
12.39
APPLICATIONS OF DERIVATIVES IN COMMERCE AND ECONOMICS

25
4-(P(.t))=0
dx
^ VI
-=0=> 50=V^=> .v=2500.
2
25
Clearly, 3/2
< 0. Thus, P (x) is maximum when .Y = 2500.
dx
/y=2500
(2500)

50 50

Putting .v = 2500inp = ^,weget ^=-^===1.


Thus, P (.v) is maximum when 2500 units are sold at ? 1 per unit.
MP= —= —(50VI) =
25
and, MC = =A (0.50 .Y +1000) =0.50
dx dx VI dx dx

w
When Y = 2500, we obtain
25
MR = , - - and, MC = 0.50 = -
V2^ 2 2

lo
TIyus, MR - MC when .y = 2500.

e
!-:XAMPLE 10 There are 60 newly built apartments. At a rent of ? 450 per month all will be occupied.

re
However, one apart7nent will be vncmU for each 115 increase in rent. An occupied apartment requires ? 60
rF
per month for maintenance. Find the relationship betzveen the profit and number of unoccupied

F
apartments. What is the number of vacant apartments for which profit is maximum ?
Let .Y be the number of vacant apartments. Then, number of occupied apartments

r
SOLUTION
fo
ou
= (60-.V).
Rent for each occupied apartment = ? (450 + 15.y)
ks
Let R be the revenue received from the occupied apartments. Then, R = (60 - .y) (450 + 15.v).
oo

Since each occupied apartment requires ? 60 per month for maintenance.


Y

Total maintenance cost for occupied apartments = C = (60 - .y) 60.


eB

Let P be the profit function. Then,


ur

P=R -C

p = (60 - ,y) (450 + 15-y) - 60 (60 - .y) = (60 - .y) (450 +15.y - 60) = (60 -.y) (390 + 15.y)
ad
Yo

= 23400+510.y-15.y^
dP d^P
d

= 510-30.y and, 30
dx^
Re

dx
in

For P to be maximum, we must have


F

— = 0 => 510 - 30-y = O => Y =17.


dx

d^P
Also, = - 30 <0 for all y. Hence, maximum profit is obtained when 17 apartments remain
dY-2
vacant.

EXERCISE 12.7

A firm has found from past experience that its profit in terms of number of units produced
Y^
+ 729 Y - 2500, 0 < Y < 35. Compute; (i) the value of y that
is given by P(-y) =—^
maximize the profit, (ii) the profit per unit of the product when this maximum level is
achieved.
12.40
APPLIED MATHEMATICS-XII

2. A company finds that it can sell out a certain product that is produced at the rate of ? 2 per
1 / \2
unit. It estimates the cost function to ? \ 1000 +
2l50j ■ , for X units produced and sold.
-

Find: (i) the expression for total profit, if x units are produced, (ii) the number of units
produced that will maximize the profit, and (iii) the amount of maximum profit.
3.
A manufacturer finds that his product can be assembled at a total cost C = ? (200 + 30x),
where .y is the number of units manufactured. Assume that the price at which he can sell
each unit is given by p = ? V 150 - 5 y . What level of product will maximize the total ^profit ?
What is the corresponding selling price per unit ? What is the total profit at this level of
production?
4.
A sitar manufacturer can sell .v sitars per week at ? p each, where 5a: = 375 - 3p The cost of
( x^ ]

w
production is ? 500 + 13.y + — . Find how many sitars should he manufacture for

5.
maximum profit and what is this profit?
3

If the total cost of a firm is C (.r) = ^ -


F lo
+ 30.v + 15, where C is the total cost and .r is the

ee
Fr
output, and price under perfect competition is given by 6, find for what value of x the profit
will be maximized?
6.
for
A monopolist firm has the following cost and demand functions :C =50.r + 3000,
p =100 - O.Ol.Y. Find out the profit maximizing output and price.
ur
7.
The demand and total cost function for a product facing a monopolist are given as:
s

p=5(2-x) and C =30+ 3x^ -2x^ respectively. Determine the optimum level price and
ook
Yo

output for profit maximization.


eB

8.
A monopolist has a demand function x=106-2p and the average cost funcHon
AC=5 + — ■
—, where p is the price per unit output and xis the number of units of output. If
our
ad

the total revenue is R


= p. X, determine the most profitable output and the maximum profit.
9. Find the profit maximizing output given the following revenue and cost functions:
i?(x)=1000x-2x^, C(x)=x^-59x" +13I5x+ 2000.
dY
Re

.2
10. A given product can be manufactured at a total costC
Fin

=? +100 X 4- 40 , where x is the


100

number of units produced. The price at which each unit can be sold is given by
p = ? 1^ 200 - — . Determine the production level x at which the profit is maximum. What
is the price per unit and the total profit at this level of production?
8000 X
11. The profit of a monopolist is given by P (x) 500 + X - X. Find the value of x for which P (x)
is maximum. Also, find the maximum profit.
12. Suppose a manufacturer can sell x items per week at a price p = 20 - 0.001 x rupees each
when it costs y = 5x + 2000 rupees to produce x items. Determine the number of items he
should produce per week for maximum profit.
l.->. The demand function and total cost function of a monopolist are given as p = 13 -15x and
C = - X + 3x respectively. At what level of price will there be maximum profit? What is
the maximum profit?
12.41
APPLICATIONS OF DERIVATIVES IN COMMERCE AND ECONOMICS

14. Given the demand function p = 21 - 4.r and the average cost function AC = 2. Determine
the profit maximizing output. What should be the impact of tax of ? t per unit of output on
profit?
15. If the cost function and revenue function of .r units of an item are given by C(.v) = .r
Y“
2

and R(.r) -50 x-x^. Find the number of items to be produced and sold to have maximum
profit. Find the maximum profit also. 600-p
16. A radio manufacturer produces .y sets per week and sells at ^ p per set, where y = 8
The cost of production of Y sets is 78 Y + 2000. Find the number of sets to be produced
and sold per week so as to have maximum profit. Calculate the maximum profit also.
ANSWERS

(ii) 5000 (iii) ?4'J00

w
1. (i) 27, (ii) ? 393.40
3. (i) ?300 (ii) ? 90 per unit (iii) r 17,800
Y = 30, Max. profit = ? 1180
5. 6 6. 2500, p =75

F lo
4.
8. 46 units. Profit =1107.68 9. 35 units
7. P=5,y=1
10. Y = 4000 unit, price = ? 190, profit = ? 199,960 n. 1500, ? 4500 12. 7500

ee
19 19-f
13. 1,?6 14. Y = units, Y = unit 15. 25 units, Profit = ? 8437.50

Fr
8

16. 29 sets, ? 5,569


for
miscellaneous exercise
ur
1. If the marginal revenue and marginal cost for an output y of a product are given as
MR = 3 + 2y and MC = 5 - 4y + 3.v^ and if the fixed cost is zero, find the profit function and
s
ook
Yo

the profit when output is y = 2.


The marginal revenue MR and marginal cost MC of a product are approximated as
eB

2.

MR = 16y - Y^ and MC = 81 - 20y + 2y^ respectively. If the fixed cost is zero, determine the
profit maximizing output and the total profit at the optimal output.
our

A travel agent arranges a tour from Delhi to Shimla and back. He has 60 seats in a special
ad

3.

bus at booking amount of ? 450 per seat provided all seats will be occupied. However, for
every increase of ? 15 in the booking amount one seat will remain vacant. He also plans ot
Y

provide a mineral water bottle and snacks costing ? 60 per seat. Find the relation ship
Re

between profit and number of seats remained vacant. What is the number of vacant seats
nd

for which profit is maximum?


Fi

4. The cost function of a product of a firm per-day for y units isi given by
Y^ the revenue function IS given by
C (y) = 3000 + 271.V + whereas
6 '
.3

R (.y) = 3300 + IOOOy - —


6
, 0 < y < 30. Calculate:
(i) the number of units that maximizes the profit,
(ii) the profit per unit when the maximum profit level has been achieved.
D. Find the relationship between the slopes of marginal revenue curve and average revenue
curve, for the demand fimction p = n - bx.
6. The cost function of producing y units of a product is given by C (.v) = ^nx + b,n and b are
positive. By using derivatives show that the average and marginal cost curves fall
continuously with increasing output.
12.42
APPLIED MATHEMATICS-Xll

7.
The total cost function of a product is C = 2x"^ —5.y^ + 7x, find the marginal average cost
function (MAC). Whether MAC increases or decreases with increase in outputs?
8.
A cable T.V. operator has 5000 subscribers, each of them pays ? 100 per month. The
operator proposes to increase the subscription and he found that for every increase of ?
0.50,10 subscribers will discontinue the service. Find what increase in the subscription rate
will increase maximum revenue and what will be maximum revenue?
9.
The total cost function of a product is C = a + bx + cx^, where a,b>0 and c > 0. Show that
the average and marginal cost are equal at minimum average cost.
10.
A chartered plane has 200 seats and charges of ? 3000 are taken per seat with an additional
charge of ? 150 for each subsequent cancellation. Determine the total revenue function in
terms of the number of cancellations before the departure of the plane. Also, find the
number of cancellations for which total revenue is maximum.
11.
A car is driven on CNG (compressed natural gas). The consumption of gas i/ kg per km, is

w
related to the speed .v km/hr at which the car is driven by the equation y = — + - Find
4y 2000

F lo
the speed at which the car be driven to minimize gas consumption.
12. The cost function C for a commodity is given by C = + 5y + 36, where .y is the number of

ee
units produced. Find the output for which average cost AC is increasing and the output for

Fr
which AC is decreasing, with increasing output. Also, find the marginal cost function.
13. The total cost of producing and marketing .y units of an item is given by C =x^ -22x + 160.
for
The government imposes a tax at the rate of ? 2 per unit. What will be the level of output for
ur
minimum
cost?
cost before and after tax? Why does the producer found it better to add tax in his
s
ook
Yo

ANSWERS
1. -64.y;2 2. 9,0
eB

3. P (.Y). = (60 - .Y) (390 + 15.y); 17 4. (i) 27 (ii) ? 497.11


r

5. Slope of MR curve = 2 (Slope of AR curve) 7. MAC = 4,y -5, Increase with output
ou
ad

8. ? 75, Max. revenue = 111500 10. R = 600000 + 27000.Y - 150y^, 90


Y

11. 50 km/hr
J2. More than 6 units. Less than 6 unit, MC = 2x +
Re
nd

13. 11 units (Before tax), 10 units (after tax)


Fi
CHAPTER 13
INDEFINITE INTEGRALS

13.1 PRIMITIVE OR ANTIDERIVATIVE


DEFINITION Afundion^ix) is called n primitive (or an mitideri- -^irvorau hitegml) ofa function f{x) if
= fix).

w
x‘^ .. r 3 , ^ ^ 3

For example, — is a primitive of x , because — ^


X .

-^{ii>(x) + Cl = ^'{x) = fix)


F lo
Let (l)(x) be a primitive of a function /(x) and let C be any constant. Then,
[V 4>'(v)=/(-v)l

ee
dx

Fr
<j)(x) + C is also a primitive of/(x). which
Thus, if a function /(x) possesses a primitive, then it possesses infinitely many primitives for
are contained in the expression <i)(.Y) + C, where C is a constant.
ur
4 4" 4
' 3
For example, —, — + 2, V " ^ etc. are primitives of x .
oks

4 4 4
Yo
o

13.2 INDEFINITE INTEGRAL


eB

DEFINITION Let fix) be a function. Then thefnmily of all its primitives (or antiderivatives) is called the
indefinite integral off(x) and is denoted by J /(x) dx.
our
ad

The symbol f / (x) dx is read as the indefinite integral of / (x) with respect to .x.
...(i)
f(i,{x) + c]=f{x) fix) dx = ^(x) + C
Y

o
Thus,
dx \ )
Re

where <{> (x) is primitive of /(x) and C is an arbitrary constant known as the constant of integration.
nd

Here, | is the integral sign, fix) is the integrand, x is the variable of integration and dx is the
Fi

element of integration or differential of x.


DEFINITION The process offinding an indefinite integral of a given function is called integration of the
function. _ _ .
It follows from the above discussion that integrating a function /(x) means finding a function
d f ^
(b(x) such that dx V
(x) / =/(x).

13.3 FUNDAMENTAL INTEGRATION FORMULAS


We know that

dx
{<t>ix)}=fix) « j /(x)dx=(l){x) + C,
obtain the following
Based upon this and various standard differentiation formulae, we
integration formulae;
13.2
APPLIED MATHEMATICS-XII

d n
11+ 1
(i) ^ = x ,n^~l r
X
II j
dx = —
X
+ C,
dx n +1 n + \

d 1 1
(ii) — log,, .V =- -i^.v = log,|.v| + C
dx[ X

(iii) ~{e^)=e'^
dx j dx =e^ +C
X
a X

(iv) = n^, a>Q, aitl => dx -


a
+ C
dx log,, a log,, a
Let us now
discuss evaluation of some integrals based upon the above formulae.

ILLUSTRATIVE EXAMPLES

w
rxAMPLE 1 Evaluate the following integrals:
(i) x'^ dx (iii)
1
dx

(iv)
.V
1
3 dx (V)

F lo
ree
4+ 1 5
SOLUTION (i) f dx =
X X
+ C = —+ C

F
4 + 1 5 for [Using formula (i)]

x-2
x'-'^dx = + C = 2,,3/2 + c
r
1
+ 1 3 [Using formula (i)]
You

2
oks

1
+ 1
eBo

(iii) ●’I -L x-^^^dx = X 2


fx
dx =
1
-+C=2.r^''^+C [Using formula (i)]
+ 1
2
our
ad

-3+1
(iv)
X
^ dx = X ^ dx = - 3 + 1 + C=-
lx
[Using formula (i)]
3+ 1

\a'°S‘>^^dx = { x^dx =
4
3 logfl .r dx. =
dY

(V) X
+ C=^ + C
Re

3 + 1
[V
4
Fin

^51oge J _^,4logp.v
tXAMiMh: Evaluate: j ,,3Iog,..t _^,2log,.x
dx

SOLUTION Since c"


X a
= x

^5 log,, .r _ ^4 log,, .r X ^-v4


(x-1) ^ .2 X^
■ e^log,. .Y _^21og,,.r ^
x^(x-l) X
J 3 _ ^,2
~ dx = f X^ dx = — + c
3 "J
) \ \Ml'LC3 If a >0and a^l evaluate the following integrals:
(i)
(ii) |■g«log,.Y dx

(iii) e'^ a^ dx (iv) f 2 ^ dx


SOLUTION (i) We know that ^^
INDEFINITE INTEGRALS 13.3

.V

gloge
a
dx = a'^ dx ~ + C
logt- «
(ii) We have.
a
rt + 1
a logg X dx
= I * =
a ,
.V fr.T = - + C
rt +1

(iii) We have.

dx = f (ae)^ dx = + C
log (ne)
l»g(j .V logfl a:
(iv) We know that x = y
log, 2+1 ^logi, (2c)
+ c = + c
logc 2 + 1 log, {2c)

w
EXERCISE 13.1

1. Evaluate each of the following integrals:

F lo
5/4 , 1
(i) I dx (ii) a: dx (iii) (iv)
^3/2
dx

ee
1
(v) 3^ dx (Vi) dx (vii) (viii) log Y A' dx

Fr
^6 log, X _ ^5 log, X for
2. Evaluate: J ^41og,.r _^31og, j:
dx
ur
1
3, Evaluate: dx
s

X , -T
a b
ook
Yo

plog
eB

4. Evaluate: dx
X

ANSWERS
our
ad

5
1
1. (i) +
5
+C 4 .9/4
(ii) - a: + C (iii) -
4a-
4"C (iv) -^ + C
Y

3
3^ 1/3
(v) + C (vi) 3.r + C (vii) —+ C (viii) Af + C
Re

log 3 3
nd

3
Fi

2. — + C 3. + C 4. 2-fx+C
3 - log, (^li’)
HINTS TO SELECTED PROBLEMS

1 6
^,6 log, -Y _ ^5 log, Y glogc a: glogc .T
6 .5
-A- , C 2 j X
3
^
3. dx - d.r = —; r- dv = -A dx - +C
^4!og,Y _^31og, Y ^log,Y‘* _^,log, X

13.4 SOME STANDARD RESULTS ON INTEGRATION

d
THEOREM (i) f{x)dx =f{x)
dx

i.e., the differei^tiation of an integral is the integrand itself or differentiation and integration are inverse
operations.
13.4 APPLIED MATHEMATICS-XIl

(ii) k f(x)dx=k f(x) dx, where k is a constant

i.e., the integral of the product of a constant and a function = the constants integral of the function.
(Hi)
{f(x) ± = J f(^) dx ± J g(x) dx
i.e., the integral of the sum or difference of a finite number offunctions is equal to the sum or difference of
the integrals of the various functions.
PROOF (i) Let f{x) dx = <j)(.T:). Then, by definition of an integral, we obtain
d f

dx V
4' ix) = m => 4-
dx
fix) dx = fix)

(ii) Let fix) dx = ij)(.r). Then, by the definition of an integral, we get

w
d f
-T- 4) (X) = fix)
dx V y

= k-f(Hx)) = kf(x)

F lo
dx
[Using (i)]
dx
k fix) dx = /c(|)(^) [By definition of an integral]

ee
k fix) dx = k fix) dx [v fix)dx = ^ix)]

Fr
0
COROLLARY If/(.v) = 1, then k-dx = k l-dx=k x dx = kx + C
Thus, integration of a constant k with respect to x is kx.
for
ur
(iii) Let j fix) dx = and g(.t) dx = v}/ (jt)
s
d d
ok

Then, -r(4>W)=/(^‘) "‘ind iy\>ix))=gix)


Yo

dx dx
o

4-Wx))±f(^(x)) = fix) ± six)


eB

dx dx

4~ (4>(-'f) ± ^ = fix) ± gix)


r

dx
ou
ad

\f(x)±gix)}dx = ^ix)±\\iix) [By definition of an integral]

] ifW±S(.v)lrf.v
Y

fix) dx± gix) dx [Using (i)]


Re
nd

GENERALIZATION The above results can be generalized to thefor7n


{kifix)±k2f2ix)±...±k„ffix)}dx = k-ij fffx)dx±k2\f2ix) dx ±... ± k„ \ f„ix) dx
Fi

i.e., the integration of the linear combination of a finite tuimber of functions is equal to the linear
conibination of their integrals.

ILLUSTRATIVE EXAMPLES

Type I PROBLEMS BASED ON j cf/(.v)ih L


EXAMPLE 1 Evaluate:

(i) 4.y^ dx (ii) J 3


.x+2
dx

SOLUTION Using k fix) dx = k /(.v) fr.Y, we obtain


5+ 1

(i) f 4.y^ dx = 4 { x^ dx = a - + C = -x^ +C = -x^ +C


5 + 1 6 3
INDEFINITE INTEGRALS 13.5

s’"
(ii) 3
.T+ 2
dx = ■ 3^ dx = 9 3 '^ dx = 9 + C
log 3

Ti'."'' II EVALUATION OF INTEGRALS BY USlNi

exa.:n;;\.i; - Evaluate:
.v^ + 5x^ + 4.Y +1
(i) \x^ +5a-^-4 + - ■ ^ dx
(ii) J X
2
dx

1
(iii) {l-x)^[xdx (iv) I X + dx

ow
2
(V) X + (Vi) dx
-2
A- >

e
(vii) dx
.Y-1

re
Fl +5jy^ -4 + -
2

F
SOLUTION (i) Let I = dx. TlYcn,
A' .Y
ur
Y r 9 f f 7 2

r
I = X dx + 5A dx - 4dx + - dx + dx
X
fo
ks
- 1/2 ,
I X
^ dx + 5 j x^dx-4 1 dx + 7 ~dx+2 X dx
Yo
X
oo

X-4 x^
I = — +5X 4.Y + 7 log I AI + 2 + C
eB

4 3 1/2

I = — + —3 A^ - 4a + 7 log^ I A I + 4 ^/A + C
ur

4
ad
Yo

r a^+5a^ + 4a + 1 dx. Then,


(ii) Let I = 2
A
d
Re

A + >=4
5 + - + —1,
in

/ = dx [Dividing each term by a^]


^ X^)
F

r r r 4 1
1 = 1 A frA + 5dx + - dx + dx
A ●>

I = X dx +5 Idx + 4 — rlA + f A ^ frA


A

2 -]

~ + 5a + 4 log I AI + —^
1 = + C

2 ,
1 =
^ + 5a + 4 log I A1 — + C A

(iii) Let 1 = f (1 - a) dx. Then,


3/2 . ^3/2 ^5/2
1 = (Va - A -Jx) dx={ yfx dx - A dx =
~372 ~'J/2 3 5
13.6 APPLIED MATHEMATICS-Xll

^2
1
(iv) Let I = dx. Then,
X

1 1 2
- dx + 2 j 1 ■ dx = ^
I = X + — +2 dx= X dx +
X
— + log I -t I + 2a: + C

2 1
(v) Let / = I X

-V
2
dx. Then,

1 =
+\
x^
+ 3x^ + ~]
a-2
dx
1 =I dx + A- ^ dx + 3 A^ dx +3 a ^ dx

ow
7
A--5 s'! -1 7
+ 3 ^ + 3 ^ +C = — ^ + A^ --+C
7 -5 3 -1 7 5 a' A

(-l+xf

e
(vi) Let I = dx. Then,

Fl
re
F
1 + 2a + A^
1 = dx
ur
I =
1
2 4x+x^^^ dx or
sf
A
k
I
Yo
oo

1/2
3/2 _^5/2
;>/2^4^3/2^2_^5/2
A A
/ = + 2x + C = 2a + C
B

1/2 3/2 5/2 3 5


re

A^-A^ + A-1
(vii) Let I = dx. Then,
A-1
u
ad
Yo

I
A^ (a -1) + (a -1) dx =
r (a^ + 1)(a-1) dx =
2 A^
A +1 dx = + A +C
A-1 A-1 3
d
Re
in

EXAMPLE 3 Evaluate: dx
A
F

SOLUTION Let / =
f a'^ + A^ + 1
dx. Then,
A 2-a-M
(-V^ + if ~^dx
2

I =
2
A- - A + 1

r (A^ + 1+a)(a^+1-a) 2

J (a^ + a + 1) dA = ^
=> 1 dA = + — + A + C
(A^ -A + 1) 2

Type III INTEGRATION OF EXPONENTIAL FUNCTIONS


EXAMPLE 4 Evaluate:

(i) ^xloga ^^aloQX + e


a log a dx
(ii)
X m
— + — + A
m
+ m
Y ,
dx
m X
INDEFINITE INTEGRALS 13.7

SOLUTION (i) Let/ = ^.v log a log i- log a dx. Then,


a a

^^logn
I =
I dx

I = (a^ + .v” + a“) dx


log?.
/ = f dx + f x^ dx + f dx e = X]
.Y fl + 1
a -V a
I + a“ x + C
log (7 a +\
r (X

m
+ — +X
m +m
X j T-,
dx. Then,
(ii) Let I =
m X

w
HI + 1 X
m
1
I X dx + m - dx + a:”’ dx + dx = — + m log I -TI + - + + C
m x 2m m+1 log m

Flo
EXAMPLES Evaluate:

l"" + 3^

ee
dx (ii) dx
(i)
5^ a^'

Fr
2^ + 3^
SOLUTION (i) Let/ = dx. Then,
5-^' for
ur
I = ^+ 1^ dx
2f
+
(3 ■ dx = (2/5)-^' ^ (3/5)-^ + C
s
5Y 5-y 5J 5 log,. (2/5) log,, (3/5)
k
V - /
Yo
oo

(«■'■+
(ii) Let I = dx. Then,
eB

X I X
a b
lx lx
a +b + la^ b''
r

1 = dx
ou
ad

{a/bf ^ (b/af
Y

a
I = + - +2idx = + 2x + C
a' a
\ - / log,, (a/b) log,, (b/a)
Re
nd
Fi

TupcIV MISCELLANEOUS PROBLEMS


EXAMPLE 6 Evaluate:

(i) I {xUx^+l)d{x^) (ii) I d(x^)


SOLUTION (i)Let.v^=L Then,
I = {x'^ + + 1) d{x^)
,3 ,2 6 ..4
I (/^ +1 + 1) dt — 1 ht+C =
Y
+
Y
+ Y
2 ^
+ C [●●● t = -y2]
3 2 3 2
3
(ii) Let e'^ -1. Then,
3
I dt = / + C = e'^ +C
13.8 APPLIED MATHEMATICS-XM

EXAMPLE "
///' (,v) = - 4 «nd f (1) = 0, find f (x).
X

SOLUTION We have,
/W = f fix) dx
fix) = - -

3
a:

-2

ow
=>
fix) = 3 ^3 -2
-2
+ C

1
fix) = x^ + 2^^
.V

e
/(I) = 1 +1 + C [Replacing a: by 1]

re
0 = C + 2
[■●7(1) = 0]
=> C = -2

Flr
F
fix) = x^+^-2 .Y
[PuttingC =-2 in (i)]
ou
sr
EXERCiSE 13.2

Evalunte the following integrals (1-44):


(3 x ^fx + 4 fx +5) dx 2-^' + ^- fo
k
5 1
dx
oo
X
7/3
Y

"x iax^ + bx + c) i dx (2-3a:) (3 +2a:) (1 -2A:)rfA-


reB

fn x X HI 1
—h — + m + X + mx dx
uY

n. n. ‘x - dx
V .t m X

(l + .t)^ (eY
ad

J ■ x^ + e logx
do

dx + ■ dx
V? V
2
*- /
in

3 2
ix'^ + e^ + e*^) dx X X dx
Re

X)
F

II, 1+1 dx
a:^+1
dx
a:
x^ +1

13.
+ Yx + 2 dx (1+V^)2 dx
xT 4x~
(,v + l) (x-2)
■v/x (3 - 5a:) dx dx

+ 2
2
dx (3:c + 4)^ dx
X

2a-^ + 7x^ + 6 .v^ 5 7 + 12 a:^ +


i x^ + 2x
dx
■)
X~ + X
dx

(x^ + S)(x-l) 3 - 3.t


o 2 +5x
(- -7 + X 2 a .y
^ I
X
dx dx
X ^ - 2x + 4 2x^
INDEFINITE INTEGRALS 13.9

23. Uf'{x)=x-^
.1'
and/(I) =-^,
^
find/(.v).
lif'ix)=x + b,f(l) =5,/(2) =13, find/ (x).
ZZ. If /' (x) = 8 - 2x,/(2) = 8, find /(x).
1
Write the primitive or anti-derivative of / (x) = X +
X

ANSWERS

2"' ., 3 +C
1. 2. + 5 loe X —
5 3 log 2 ^2
3/2 o3 X 4 4 3 17 2
3. + C . + - X .x^ + 6 X + C
7 5 3 3 2
2 .r HJ + 1 2 2
X m X jnx

5. m log 1 XI +
2 log m
+

m +1 2 - + C 6. ^ - 2x + log 1 XI + C

w
6 X 5/2 2 X 7/2 + C
^
3
X
.2
1 r f
7. 2Vx + 2x^^^ + - ' + - 8. ^ + — + + C

F lo
2
5 7 3 2 , ^
log[2
c+ \
X
+ + e'^ X + C 10.

e
9.

Fre
e + 1 9
5 3

2Jx- ^+C 12. —~^ + x + C


X 5 3
for
i/3
14. 2Vx+2x + -x^^^ + C
r
3x
7 3
You

2_^5/2_2^.3/2_4^^(.
oks

3/2 .5/2
15. 2x -2x + c
5 3
eBo

4 -3

17.
X X
2 +C I ●3
i(3x + 4)^+C
4 3 X

-tt). 2 _^.3_^
our
ad

3 2 3 2

1 x^ 7
.\

X x^ UC
_ + ^ _ 2x + C — ^ 3x + 5 log X + — +
2 2 ^ X l0g^7
dY

3 2
Re

2 2
X 1 X 13
Fin

—+ --1 . _ + —x-2
2 X 2 2

^ 2x' - X ^ -20 26. _2 ^.3/2 + 2x


.1/2 + C
3

13.5 GEOMETRICAL INTERPRETATION OF INDEFINITE INTEGRAl


In order to understand the geometrical meaning of an indefinite integral, let us consider a
function/given by/(x) = -2.x.
Clearly,
/ (x) dx = - x^ + C, where C is the constant of integration.
Let us now consider the family of curves given by y = J / (^') or, y = - x + C.
Clearly, y = - x^ + C represents a family of parabolas having their common axis of symmetry
along y-axis as shown in Fig. 13.1.
13.10 APPLIED :VJATHEMAT1CS-XII

V
ix = a
\

P3

X'
A X

p,
y = -x^ + 2
ij = -x^ + 1
y = “A:2 ^
T

w
V'

Fig. 13.1

F lo
In other words, each integral of/(.v) =-2x represents a parabola with its axis of symmetry along
y-axis.

ee
Let us now consider the points of intersection of each of these parabolas with a line parallel to

Fr
y-axis (a line orthogonal to the axis representing the variable of integration). Let x = a bea line
for
parallel to y-axis which cuts the parabolas y = - ,y=- +1, y = -.V:^ + 2, y = -:r^ + 3 etc.
ur
respectively at points P\ , P2 ^ P3 > P4 stc. At each of these points, we have
2a
s

dx
ook
Yo

That is the slopes of the tangents to the parabolas at , P2 > P3 > ^4 same. Consequently,
eB

tangents at -P\ , P2 'P3 , P4 etc. are parallel.


Thus, the indefinite integral of a function may be interpreted geometrically as follows:
our

The iudefinite integral of a function represents geometrically a family of curves having parallel tangents at
ad

their points of intersection zvitli the lines orthogonal to the axis representing the variable of integration.
Y

13.6 CCMPARJSCM BET'AcEN DIFFEREMTIATIOM AND .NTEGRATION


Re

Following points will help us to imderstand the difference between the differenUahon and
nd

integration.
Fi

(i) The operations of differentiation and integration are defined on functions.


(ii) The derivative of a function, when it exists is a unique function whereas the integral of a
function is not unique. In fact there are infinitely many integrals of a function such that
any two integrals differ by a constant.
(iii) The derivative of a function at a point (if it exists) is meaningful but the integral of a
function at a point does not have any sense.
(iv) Every function is not differentiable. Similarly, every function is not integrable.
(V) The derivative of a function at a point determines, the slope of the tangent to the
corresponding curve at that point. The integral of a function represents a family of curves
having parallel tangents at the points of intersection of the curves of the family with the
lines orthogonal to the axis representing the variable of integration.
(vi) The processes of differentiation and integration are inverse of each other.
(vii) The operations of differentiation and integration are operations on functions.
INDEFINITE INTEGRALS 13.11

(viii) Both the operations are linear.


i.e.
4-
dx
{/ (-V) + X (X)) =-^(f(x))
dx
+ 4-
dx
(.? (.V))
and, if {x) + g (x)) dx = f (x) dx+ g (x) dx
(ix) The constant can be taken out side the differential as well as integral sign.

i.e. -^{kf
dx
{x)) = k-^
dx
if (x)) and. k f (x) dx ~ k f (.x) dx

(x) Differentiation and integration both are processes involving limits.


13.7 METHODS OF INTEGRATION

We have the following methods of integr.. tion:

ow
(i) Integration by substitution
(ii) Integration by parts
(iii) Integration of rational algebraic functions by using partial fractions.
We shall now discuss these methods in the following sections:

e
Fl
re
13.8 INTEGRATION BY SUBSTITUTION

F
In section 13.4, we have considered the problems or integration of functions in standard forms
ur
and the problems involving combinations of these functions. Integrals of certain functions
or
cannot be obtained directly if they are not in one of the standard forms given in section 13.4, but
sf
they may be reduced to standard forms by proper substitution. The method of evaluating an
integral by reducing it to standard form by a proper substitution is called integration by
k
Yo
oo

substitution.

If (|)(.v) is a continuously differentiable function, then to evaluate integrals of the form


B

/((t>(x)) (x) dx, we substitute iti(x) =/ and (j>' (.v) dx = dt.


re

This substitution reduces the above integral to | / (0 dt. After evaluating this integral we
u
ad

substitute back the value of t.


Yo

13.8.1 INTEGRALS OF THE FORM | /(/t.v ●


d
Re
in

THEOREM 1 // f(x) dx = ^{x), then f{ax+ b) dx =-<^{ax+b)


F

rROpi Let i = / {ax + b) dx.

Let ax + b = t. Then, d (flx + b) - dt ^ a dx = dt => dx = - dt


a

1
Substituting (7X + b =f and dx = -a iff, we get
1 1 1
I
I f{ax + b) dx = a
fit) dt = a
4>(0 = - ^{ox + b)
a
■ )
H + 1
(l7X + b)
THbOPEr.i ■ Prove that (ax + b)” dx = + C, ll¥^ -1
a {n +1)

RRiX jI Let ax + b = t. Then,


13.12 APPLIED MATHEMATICS-XII

d {ax + b) = dt ^ a dx = dt => dx = - dt.


a

Putting ax + b = t and dx = -a dt, we get


H + 1
1 t {ax + b)
(rt.Y + b)’^ dx = - t’‘ dt = — X - + C = + C
●’ (7 ●' a n + 1 a {n +1)
1 Q.E.D.
THEOREMS Prove that ●'\ —-—dx
ax + b
= log I a
+ 61 + C

ow
1
t i<OOF Let ax + b =t.Then. d (ax + b) = => adx-dt-^ dx = - dt.
a

Putting ax + b = t, and dx = -a dt, we get


1

e
1 1 1
dx = =
-a log I f I + C = -a log I ii.v + & I + C

re
●* ax + b a ■’ t

rFl Q.E.D.

F
On comparing these three results with
.71 + 1
n , X 1
X dx = - + C, n^-1, —.r dx = log I .r I + C and. sin X dx = - cos x + C

r
« +1
ou
respectively we find that if x is replaced by ax + b, then the same formula is applicable but we fo
ks
must divide by the coefficient of x or derivative of (flx + b) with respect to x i.e., a.
oo

Thus, in any of the fundamental integration formidas given in section 13.3 if in place of x ive have ax + b,
then the same formula is applicable but toe must divide by coefficient of x or derivative of (ax + b) i.e. a.
Y
B

We give below a list of some of them which are frequently used.


re

H+ 1
{ax + b)
(i) {ax + b)" dx - + C,n?i-1
a {n +1)
ou
Y
ad

1 1
(ii) rfx = - log I rtx + b I + C
ax + b a
d

a.x+ b
(iii) e + C
in
Re

bx+ c
F

bx+ c 1 a
(iv) a dx = - ■ + C, a>0 and a^l
b log a
Following examples illustrate the applications of these formulae to evaluate integrals.

ILLUSTRATIVE EXAMPLES

■ EVALUATION np intfGRALS DlP'^nrt y BASED UPON ABOVE GIVEN FORMULAE

Evaluate:

1 1 1
(i) - (2x - 3)^ + ^ 3x + 2 ● dx
(7x-5)^ ■-f - 4 ^ 2-3x
+

2.V-3 3.V+ 2
(ii) e dx (iii) a dx
INDEFi^4ITE INTEGRALS 13.13

SOLUTION (i) Let


1 1
I = ■ {2x - 3f + _
x-4
-— + J3x + 2 ■ dx. Then,
2-3x ^

-1/2
I = (2x-3fdx+{ (7x-5)~^dx+ (5 a:-4) dx + -— dx + \j3x + 2 dx
J 2 - 3 .r J^
1/2 f 3/2
/ =
(2,v-3)^ +
(7.Y-5)~^ +
(5l-4)

1
log|2-3.T| +
(3.V + 2) + c
2x6
■ -

5 X
1
/ 3x3
2 2

ow
/=T(2,t-3)‘^-T(7^^5)-^+|^ ilog|2-3*|+|(3.v + 2)
3/2
-4 - + C
12 14

2.V- 1 2 j:- 3
(ii) e ^dx = — X + C
2

3a-+ 2 1 3.V+ 2

e
(iii) a dx = X a + C

re
3 log a

rFl
F
Evaluate:
1 1
(i) dx (ii) ' dx

px + 4-^3y + 1 ■ Z^ + ,/3^

r
ou
1
dx. Then,
fo
ks
SOLUTION (i) Let 1 =
^3x+4-px + l
oo

I =
^3 X +~i + ^3y 4-1 dx
Y
eB

\ f

^3.v + 4 + ^3.V4Tj|^^3.v + 4
^3.y + 4 + ^3y + 1
r
ou

I =
ad

(3y + 4)-(3.v + 1)
Y

1
/ + 4 +
^3y +1} dx
d
Re
in

I
—3 ^3.t + 4 fr.Y + —3 ^3y + I fr.r
F

3/2 3/2
(3y + 1)
=>
(3.Y + 4) i- + —
1
+ C = A i(3x + 4)^/^ + (3y + 1)^^^) + C
3
3x3 3
3x3 27
2 2

1
(ii) Let I = rf.v. Then,

2.t
/ = rf.Y
+ V^^) (Vr^-^3-2.Y)
/ = rf.Y
(1 -2y)-(3-2y)

^1 - 2.V ^ ^3 - 2x i/x
1
/
2
13.14 APPLIED MATHEMATICS-Xn

3/2 3/2
I = -
1 (1-2X) 1 (3-2.V) 1
!■ + -
2 3 2
-2x -2x ^ 6 6
2 2

1 + A- l-.r
+ 4
r.XAMPLE 3 Evaluate: dx
2-v
1+ A 1-A-
+ 4
SOLUTION Let I = rf.r. Then,
2-''
23x+3_^2^ 2-v
/ = dx

ow
2X
2^x + 3 22- 3x 2'^x + 2 ,^2 — 3.V
i 22a + 3 2^ - 3-'-' dx = + + C =
2
+ c
2 log 2 (- 3) log 2 log 2 3 log 2

e
EXERCISE 13.3

re
(2x - 3)^ + px + 2 dx
Fl 1 1

F
1. dx
{7x-5f V^x-4
ur
1

r
1 .Y+ 3
+ rf.Y 4.
2-3.Y
(T + l)
fo
ks
1 1
5. dx 6. rf.v
Yo
+ 1 + Vy ^2y + 3 + ^2.y — 3
oo

2y 1
yrf-v 8. iiY
eB

(2y + 1) ^x + a + b
+1)^ frY .r
ur

9. in. e + dx
X
e ;
ad
Yo

1
n. dx
d
Re

ANSWERS
in

{2x-3f 2
F

3/2
1. +
-(3..+ 2) + C 2. +C
12

ilog|2-3,v| + |^^37^ + C
1
3. - 4.
2(y + 1)^ 3(y + 1)
2 1

|(2y+3) -(2y-3)
5. - + C 3/2 3/2
6. + C
3 18
1

3{a-b) |(y + a)
2 3/2 3/2
7. -2 log I 2y + 11 + + C 8. -(Y + i7) + c
2 (2y +1)

9.
3
+1)^+C Ml. + 2x~-e~^^ + C
2 2
2

{(-t+3)
3/2 3/2
1 t
+ (y+2) + c
3
INDEFINITE INTEGRALS 13.15

r(.v)
13.8.2 EVALUATION OF INTEGRALS OF THE FORM It
,n~N , WHERE /»(r) IS A POLYNOMIAL
(iix + i>)

In order to evaluate this type of Integrals, we may follow the following algorithm.
STEP 1 Check whether degree of P (x) > or <n.
STEP li If degree of P (x) < n, express P (:r) in the form
n-1
Aq + {ax + b) + A2 (rt-v +b) +... + A,, _ ^ (rt.v + b)
P(A-) ^0 A2 Ai-1
STEP 111 Write ns + + ... +
It n » - 1 n-2 r?.v + b
{ax + b) (fl.r + b) {ax + b) (ax + b)
STEP IV Evaluate
1
f P{x) 1
dx + A 1 dx +... + Ajj _ 2 ^ dx
n dx = A() n H - 1 ax + b
(ii.t + b) {ax + b) {ax + b)
P{x)
If degree of P{x)>n, then divide P {x) b\j {ax + bf and express

w
as
STEP V n
{ax + b)

F lo
R{x)
Q ix) + 71 ' where degree ofR (.t) is less than n.
{ax + b)
R {X) dx
Use step II and III to evaluate

e
STEP VI

Fre
II
{ax + b)

Following examples will illustrate the above procedure.


for
ILLUSTRATIVE EXAMPLES
r
You
oks

EXAMPLE I Evaluate:
eBo

3 4
x-1
(ii) I
x
(i) dx dx
x + l

3
ad
our

SOLUTION (i) Let / = f ——r dx. Then,


(.v+2r
{{x^2)-2]^ dx
Re

I
dY

.ix + 2)*
Fin

(i-+ 2) ^ - 6 (l-+ 2) ^ +12 (l + 2)j^ dx


I
= 1 {x + 2)^
1 6 12
=> I = ● dx
x +2
(x+lf (x + lf (I+ 2)“
6 6 8
/ = log I .X + 21 + 3"^
.x + 2
(x + 2)^ 3(x+2)
x-1 f dx. Then,
(ii) Let
x +l

I =
l(* + l)-21^ dx
(x + 1)^
13.16 APPLIED MATHEMATICS-XII

=>
^=1 (■v + l)^-^Ci (y + l)^x2+^C2 (a- + 1)^x2^-^C3(x + 1)x2^+^C4 (2)^ dx
(^+1)“
(^r + l)‘'-8(a: + l)^ + 24(A: + l)^-32(.t + l) + 16
’-I dx

8 24 32 16
=> I = n- ● dx
x + 1
{x + lf {x + lf ix + l)‘'
24 16 16
1 =:»:-8 logi x + 1 \-
AT + l
(.t + 1)^ 3(:f + l)
i.L - Evaluate:
ax + b x + 2 r 2 +x + x^ 2x-l
(i) 2 (ii) dx (iii)

w
~2 2
(CA + d) A-^ (2 + a:) (a- + !)●
ax + b
SOLUTION (i) Let / = dx

F lo
(c.r + d)^
Let
ax + b = X {cx + fr) + li. On equating coefficients of like powers of x, we get

ee
a be - ad
a = Xc and b = Xd + n => X = and p =

Fr
c c
flA* + b
I = dx
{cd + df
for
ur
I =
(CA + d) + )i dx
(CA + d)^
s
ook
Yo

—^ dx + ii
1
I = X dx
■' CA + fl
{cx + d)^
eB

X
a . I r, {be- ad) 1
=> I
Iog|CA + d| - — + C = log |CA + d[ + C
c
c (CA + d) c cx + d
our
ad

A LITER Let
rtA + b
I = dx. Then,
(CA + d)^
Y

b
Re

A +
nd

I = a a_ dx
[Making coefficient of a unity in the numerator]
●* (CA + ^0^
Fi

be
cx +
a
a
I = - dx
[Making c as the coefficient of a in the numerator]
c
(CA + li)^
a
{cx + d) +
} a_ dx
[Adding and subtracting d in the numerator]
c
{cx + d)^
{be - ad) 1
I
-I -— dx + dx
[Separating the integrals]
c *' (CA + d) c
(CA + d)^
I
a {be - ad) 1
log I CA + d| - 2
X + C
c c (CA + d)
(ii) LetA + 2 = X(A + l) + p.
On equating the coefficients of like powers of a on both sides, we get
13.17
INDEFINITE INTEGRALS

X = 1 and 2 = X + i.i=> X = l,)i =1


.\- + 2 X(.T + 1) + l-l X
! = dx ~ dx = + dx

(x + 1)^ {x + lf X + 1
(:v + l)^
1
I = X
i- dx + n J
X + 1 (x + 1)
2 dx = X log|x + l|- x ^+ 1 + C = log I '.c +11 - x + 1
+ C

1 1
x +2 (x + l) + l 1
ALI ILK / = 2^.v = dx - -Irfx =log|x + l|- X +1
+C

(x + 1) (.r + 1)^ X + 1
(X + 1)

2 + X + x^ 2x -1
(iii) Let / = dx. Then,
i ~7 ^ 2
x^ (2 + X) (X + ir
(2 + x) + x^ 2 (x +1) - 3 1 2 3
/ =
x‘ (2 + x)
+

(.v + 1)^
■ dx = j 4+ 2 + X X
+
X +1 (.v + 1)^
dx

w
3

EXAMPLE 3

(i)
/

^ {x + lf
X
3
-X + log [2 + ^] + 2 log IX +1|

Evaluate:

dx

X
3
(ii)
+

{a + bx)^
F lo
2
x + 1

dx
+ C

for F
ree
(iii)
(.v + 1)^
dx

SOLUTION (i) Let/ =


- (x + 1)
Your

Using long division method, we obtain


ks

_ 3x+2 3 (X +1) - 3 + 2
eBoo

= X -2 + X -2 +
(x + lf (.v + 1)^ {x + lf
3
X 3 1
= X-2 +
ad

(x + lf {x + lf
our

x + 1

3
3 ] 2 I
X
- 2 + I frx = ^—2x+3log|x
^ + l|+ — +C
I =
y frx = x + 1 (.V + 1)^ 2 x + 1
(.t+1)
Re
Y

2
X

I (« +/?x)^ dx
Find

(ii) Let/ =

Using Long division method,2awe get^ a


X -
2
X 1 b

(a + bx)'^ b^ (bx + a)~


2
X 1 a (2bx + a)
{a + bxf b^ b^ (bx + af
2 2
1
X 1 a 2{bx + a)-a 1 2a 1 a

{a+bxf b^ b^ 1 (fcx + rt)^ /,.v + rt // (bx + a)


2
X
/ dx
(fl + /?x)^
13.18
APPLIED MATHEMATICS-Xn

2
1 2a 1 a 1
=> I dx
i,^ bx + a + -^X
{bx + ay
2
1 2a 1 1
J
a
I 1 ● dx - dx + dx
b^ bx + a
b^ ^ (bx + af
2 2
X 2a a 1 1 a
1
^
b
^
b
log [ bx + a\- [} 3 — X

bx + a
+ C = -^ <bx -2a\og\bx + a\ - >+C
b^ bx + a

Ai .riTK We have,
2
X
I = dx
{a + bx)^
r b^x^

w
1
dx
b^ (a + bx)^
(b^ x^ + 2abx + a^)-(2abx + a^)

Flo
1
/ dx
b^ {bx + a)^

ee
I
1
{bx + a)^ - [2a {bx + a) - 2a^ + a^\

Fr
dx
b^ {bx + a)^
/ =
1 {bx + a)^ - 2a {bx + a)+ a^ dx for
ur
b^ {bx + af
2
2a
s
a
1 - ■ dx
k

+
Yo

bx + a
{bx + a)^
oo

2
eB

r 1 2a . . 2
a a
I =-T-<x —-- log I bx + rt|- UC = ~ ■ bx~ 2a log \bx-ha\- + c
hb^ b {bx + a) bx + a

x^ + 1
r
ou
ad

(iii) Let I = dx. Then,


{x + lf
Y

I =
x^ +1+ 2x - 2x
dx
{X +1)^
Re
nd

● {x + \f -2x
Fi

I dx
(x + lf
2x
I = dx
(x + lf
X
=> 1 = \-dx-2 dx
{x + lf
=> I l-dx-2
(x + D-i
dx
(x + lf
1 1
I = 1-dx-2 dx
x + 1
(x + lf '
1 1 2
I = 1 ● dx - 2 dx + 2
x + 1 dx = - 2 log IX +1) + C
x + 1
13.19
INDEFINITE INTEGRAL?

ILXAMI’LI - Evaluate:
7 6
3 .V
X X
dx (iii) dx
(i) dx (ii) J x-l
J x + 2 x + \

SOLUTION (i) Using long division method, we obtain


3
x 8
- = - 2.V + 4 -
x + 2 x + 2
3
8 a: 3 ,
dx = ^ - 2x + 4 - ● dx = — + 4.V - 8 log I a: + 21 + C

ow
.V
a + 2 A-+ 2 3

3
X
Ai rri R Let 1 = dx. Then,
J a + 2

e
I
(a^ + 2^)-2^ dx

re
x + 2

(A + 2) (A^ - 2a + 4)

Flr
8

F
I = . dx
a+2 ou a + 2

I = < A ^ - 2a + 4 - j
dx A ^ + 4a - 8 log I A + 21 + C

sr
a + 2 3

(ii) Using long division method, we have


7
3
^-A+1-
1 fo
k
oo
A
A -A + A
A + 1 A + 1
Y
7
A j r 6 5 4 „3 + A
reB

dx = A - A + A - A —A + 1 —— ● dx
A + 1 A + 1

7 6 5 4 .3 .2
uY

7
A A A A A A A
dx = + + + A - log IA + H + C
A + 1 7 6 5 4 3 2
7
ad
do

A
Ai.niR Let/ = dx. Then,
A + 1
in

7
(A' +D-1 dx
1
Re

A + 1
F

A^+1 1
I
A + 1 dx - j A + 1
dx

-(-1/
7
A 1
I = dx- dx
A-(-l) A + 1
II »
X -a n-1 . .11-2
= A + A a
1
-A^+A^ - A^ + A^ - A + 1) frA - dx x-a
A + 1
II- 3_2 n-2 II-1
+ A a +... + X a + a

5 4 3 2

^ + x-log|A + ll+C
A A
I
7 6 5 4 3 2
6
A
(iii) Let I = dx
A-1
13.20
APPLIED MATHEMATICS-XII

X 6 _|6
1 dx
x-\

X 6 _^6 1
/ =
1 x-\
+
x-l
dx

>1
.V -a 11-1 11-2 II-3 2
5 4 ■2 9 1 = .Y + .Y a + X a
1 -Y + .Y + X + Y“ + ,Y + 1 + ■ dx X - a
x-l
II-2 . 1
+ ... + X a + a

6 5 4 3 .2
X X X .Y
I

EXERCISE 13.4
2 3
Y +5y+2
1

w
.V
1. dx 2. dx
x + 2 y-2

3. I
0.
r
x^ + .Y + 5
3x+2

+ 3.Y - 1

(x + lf
dx

dx F lo 4.

6.
r
2.Y+ 3

(X-l)
2.Y-1

(x-1)^ for
jdx

F
dx

ree AIVSiVERS
2 3

1. ^ + 3.Y - 4 log I .Y + 21 + C
r
^ £
+ Y^ + 4.Y + 81oglY-2[ + C
You
oks

Y^ 1 43
eBo

3^ + 2j+C 4. 21og|.Y-l|-
x-1

5. X .; + log(Y + l|+^-
-Y + 1
+ C 6. -
1
+ 2 log|.Y -1| +C
ad

x-l
our

(IX + h
13.8.3 EVALUATION OF INTEGRALS OF THE FORM I (rt.v ' h) \
C.Y +
li f/.YAND f d.x
. \ ,'CY * d
Re
dY

In order to evaluate this type of integrals, we may use the following algorithm:
ALGORITHM
Fin

Si FF I
Write {ax + b) in terms of{cx + d) as follows:
(<7.Y + I>) = A.{c.Y + d) +g
sTur n
Find X and g bi/ equating coefficients of like powers ofx on both sides.
^rEPm Replace ax + b Inj X (cy + d) + g in the given integral to obtain

{ax + b) ^cx + d dx = J |?, (cy + tO + g| ^ 3/2


CY + d dx

= X (cY + d) dx + p I ^cx + d dx
2X 5/2 2g 3/2
= — {cx + d) + — (t'Y + d) + c
5c 3c
ax + b X (cY + d) + g
1 dx
= J dx
.Jcx + d ■J<^x + d
13.21
INDEFINITE INTEGRALS

1
= X
^cx + d dx + p dx

= — (c.v + d) 3/2^^ ^cx^d) 1/2 + C


2X

3c c

Following examples will illustrate the above procedure:


ILLUSTRATIVE EXAMPLES

● -rii Evaluate:

(i) X +~2 dx (ii) j (7.V-2) .^3x + 2 dx


SOLUTION (i) Let J = j .r ^x+ 2 dx. Then,
Iv X = (x+2)-21
1 = f [(x + 2)-21 ^x + 2 dx

w
I (.V + 2) I (..2)
5/2 3/2
I = [{x+2f^^ -2{x + 2)'^^^}dx = + C

F lo
(ii) Let / = I (7-.C-2) ^/37T2 dx
Let 7x-2 = X{3x + 2]+\i

ee
On equating the coefficients of like powers ofj on both sides^20we get

Fr
7
3X = 7 and -2 = 2X + [i ^ X = - and u =
3 3

/ = [ (A.{3a: + 2) + p) ^3^+2 dx
for
ur
1/2
I = {?.(3x+2)^^^+p {3x + 2) } dx
s
ook

5/2 3/2
Yo

14 40
I
, (3a-+2) (3.r + 2) + C= — (3x + 2)
5/2
(3.v + 2)^^^+C
3x3 45 27
eB

- X 3
2 2

'III I K Let I = (7a-2) ^3a + 2 fix. Then,


our
ad

9 \ ,
I = 7 X- dx
Y

7 (
3.t-|] /37T2
Re

I dx
nd

3 I
6^
Fi

' —1
3 J 3X + 2-2-- .J^+2 dx

}d
20
(3a + 2) - — X + 2 dx
3 J 7

13/^}
7 3/2
7 (3x + 2) dx
3

5/2 3/2
7=^ (3a+ 2) 20 X
(3a+ 2)
3
+ C —
14
(3a + 2)^''^-|?
27
(3a+2)^^^+C
3
3x5 7
2
X 3
45
2
X
IXAMIM.I. .r Evaluate: dx.
yjx + a + ^x +T
13.22
APPLIED MATHEMATICS-Xll

-Y
SOLUTION Let I = dx . Then,
+ a + yjx + b
Y 'y + a X X + n

=J
I = dx dx

^JTTa +7^}{V x + a
a-b

1
I =
l{W
a ~b Y ^Y + & I dx
X + a -

/
^ J {(●v+«-")V (x + b-b) ^Y + cfj X + a - dx

j(Y + rt)
3/2
^ + (J - (y + b) + b^Y + &I
/ 3/2
- i7 dx
rt -i.

w
1

|(y + .)
=> / = 5/2 2rt 3/2
a-b -(y./.) + + C

F lo
EXAMPLES Evaluate:
8y +13 Y + 1
(i) dx (ii) dx
X
(iii) dx

ee
^/4FT7 7FT2

Fr
8y +13
SOLUTION Let/ = dx.

for
ur
Let 8y + 13 = ;.(4Y + 7) + (a
On equating the coefficients of like powers of x on both sides, we get
s
8 = 4A., 13 = 7X + \i =:> X = 2 and |a = -1
ok
Yo

Replacing 8y 4-13 by /, (4y + 7) + p in the given integral, we get


o
eB

A.(4y + 7)+p
I =
^/4TT7
dx =
- X^4x + 7 + dx
r

1
= ?. j ,^4y + 7 frY + p
ou
ad

/ dx
Y

3/2 1/2
(4y + 7) (4y+7)
1 (4-7) 3/2 1
Re

/ = 7.
^ (4y + 7)^''^+C
nd

- +p + C
3
4x 4x -
Fi

2 2

8y +13
AUTER Let / = dx. Then,
T44T7
13
Y + --

I 8
dx
VST7
13 ^13 1
4y + (4x + 7)+ -7
8 . (4y + 7)-:
/ = - = dx = 2 Jf _ki rfY = 2 2 dY
4 ●' ^ + 7 ^/4JT7
/ = 2 rfY
^ 2 ^4y + 7
INDEFINITE INTEGRALS 13.23

3/2 1/2
(4x + 7) 1
/ = 2 ^
(4.x-+ 7)
3
1
1
I + C = - (4.V + 7)
3/2
-2 (4.v + 7)^^^ +C
2 3
4x 4x ^
2 2
X +1
(ii) Let / = rf.r. Then,
J

3
2.V + 2
dx = -
1
* = 1 .^/2T::t + dx
2 2
V2^ 2 ●’

1 1
1 dx
2

ow
3/2 1/2
1 (2-V-l) 3 (2-r-l) 1
(2.V-1)
1/2
+ C
!> + - + C = (2.x--1)
2
2x
3 2
2x ^ 6 2

2 2

e
.X

(iii) Let I = j rfx. Then,

re
^JoT+2

Fl
F
(x + 2)-2 2
I
=1 dx = c/.v
ur
J7T2

r
3/2 1/2
fo
ks
(.X- + 2) (.t+2) 2 3/2
-4(.x + 2)^^^ +C
I = -2 ● + C — — (x + 2)
Yo
3 1 3
oo

2 2

EXERCISE 13.5
eB

x + 1
X ^x + 2 frx
ur

1. t/x
72^T3
ad
Yo

x-1
rfx 4.
(x + 2) ^3x+^ lix
d

2x + l [■ 3x + 5 rfx
rfx
Re

5.
/TT?
in

/3?T2
r 2-3x
F

X
£^X
J /nr
7. dx 8.

X
dx
q.
(5x + 3) /x -T frx in.
^x + n--yjx + b
ANSWERS

2 5/2 4 3/2
1. i(2x+3)^''^-l(2x+3)
6 2
1/2
+ C 7. r(.x' + 2)
5
--(X.2) + C

2
3. -(x + 4)
3/2
-10(x + 4)^^^+C ^
135
(9x + 20)(3x + 5) 3/2
+ C
3

5.
2

27 (6x +1) ^3x+ 2 + C 6.


^ /m (7x + 17)+C
49

7. -3 (x - 8) ^x + 4 + C 8. I (8 - 3x) + 3x + C
13.24 APPLIED MATHEMATICS-Xll

1 3/2
-(3a-+ 4) (2.V-1) + C

1 2, -5/2 2n , 3/2 +. 2
(I. 5/2 2(7
-{x + a) -—{x + a) ^{x + h) + C
a-b I 5 3 0

13.8.' "^ITEGRALSOFTHE FOf- ' i

THcUr^-.nl 1 fix)
dx = log \f(x)} + C
' fix)
Let I = fix) dx.
^ fix)

ow
Putting / (a) = t and /' (x) dx = dl, we get
/ = I i frf = log f + C = log 1/ (x) I + C :.D.

It followsfrom the above theorem that if the numerator in integrand is exact dijferential of the

e
Fl
re
denominator then its integral is logarithm of the denominator.

F
I EVALUATl ● i.irSGRALS UPC"
ur
In order to evaluate this type of integrals, we may use the following algorithm:
or
sf
vl.GORir ,
k
Yo
/'(-V)
Obtain the integral, let it bel =j dx
oo

fix)
B

1
Put f(x)=t and replace f' (x) dx by dt to obtain I = dt
t
re

Evaluate integral obtained in step II to obtain / = log |t[ + C


u

Replace f by f (x) in step III to get I = log \f (a) | + C


ad
Yo

Following examples will illustrate the above procedure.


d

Evaluate:
Re
in

X
2x + 5 e -e
- X

(i) dx (ii) dx
F

x^ +5x~7 e^ + e
-1 1
(iii) dx (iv) dx
1 + e
2x + 5
SOLUTION (i) Let I = dx
-v^ +5.V-7

Let + 5x - 7 = t. Then, d (x^ + 5x - 7) dt=>{2x + 5)dx = dt=>dx =


dt
2x + 5
dt
Putting x^ + 5x - 7 = t and dx = 2x +5 '
we get

2x + 5
/
x^ +5X-7
-dx = f -t dt = log|t|+C = log)x^+5x-7|+C
■'
13.25
INDEFINITE INTEGRALS

.V - .V
e -e
(ii) Let 1 = dx.
X - X
e + e

dt
Let/+e""' = LThen, ff(e'' + c ^') = t^f=> (t?"‘-t* dx = dt^dx = e
X
-e
- X

dt
Putting + e ^ = t and dx = e -e
— / we get

X - X
dt
= log|fl + C = logl^-'^ + e -'l + C
e -e
J = dx =
X - A-
e + e
t

(iii) We have.
3

w
x
I
j e^^^^^-{xUlf\ix = \ .v“ + 1
dx =
/+i
dx

dt

F lo
Let .Y** + 1 = t. Then, d (y^ +1) = dt => 4y^ dY = dt => dx =
4,v^
dt

ee
Putting Y^ +1 = t and dx = 4y^ ' we get

Fr
/=ljldt
4J I = i log 111 + C = ^ log (y"^ + 1) + C for
ur
A
1 1 e

(iv) Let I =
l + e~
dx =j 1 +
rdY = e'"' +1
dx
s
A
ook

e
Yo

dt
Let +1 = t. Then, d + 1) = dt dx = dt => dY = A
eB

dt
Putting 1 + = t and dx = — , we get
e
r
ou
ad

f £1-^
X

- dt = log 111 + C = log 11 + I +C


e
I = dx =
/+! J t f
Y

EXERCISE 13.6
Re
nd

Evaluate the foUoiuing integrals:


Fi

3a 1
e dx
dx
e'
3a
+1 J Y(3+logY)

+1 1
dx dx
+ Y X log Y
1
a dx
dx h.

b + ce’'^
2a 1
- I dY
e
dx
e
2a
-2 X log Y log (log Y)

9.
10 Y^ + lO'"' log,. 10 dx
Y + 1
dx
10^' + Y
10 Y (y + log Y)
13.26 APPLIED MATHEMATICS-XII

●V - 1 e-I
1 e + X
II. dx 12. dx

ANSWEilS
1 3at
1.
iog k + 1| + C - log| 3 + logA-|+C
3. log[ + ,r| + C logi log.vl+C

ow
J>.

^ log \be^ ^ + c I + C (>■ -log[ 1 + \+C


1 2x
7.
- logic -2| +C 3- log {log (log a:)}+ C

log|10'%.r^‘^|+C

e
111. log |a: + log a:1 + C

re
n. 21og| + 1|+C 12.
-e log) (+C

Flr
F
13.8.5
INTEGRALS OF THE FORM \ \f{ vi]" / '(v) l.\
ou
»+ 1

lf{x)ff'{x)dx = m)

sr
THEOREM
, ;i -1
?i + l

I'KOOf Let/=| {f{x)ff'{x)dx. fo


k
oo
Putting f{x) = f and /' (x) dx = dl, we get
Y

H + l n+ 1
+ c _ m}
t
reB

I =
J = j (''rff= n + 1 =
n + 1
+ C,fi^ -1.
Q.E.D.
uY

ILLUSTRATIVE EXAMPLES
ad
do

EXAMl’i.i i Evaluate:

(i)
3x +1
(log -v)^
(3x^ + 2x + l) 3ri.V
in

(ii) dx
X
Re

3x +1

^ (3.x^ + 2x + l) y dx. Let 3x^ + 2x + 1 = f. Then,


F

SOLUTION (i)Let/ =

d (3x^ + 2x 4-1) = dt => (6x -f 2) dx = dt =3- dx = df

2{3x + l)
dt
Putting 3x^ 4- 2x +1 = f and dx = , we get
6x4-2

3x -i-1 dt
= 1
4/
/ = t
''2(3x + l)
+ c
2 2

1
/ = -
4f
4(3x^4-2x + 1)
(logx)^
(ii) Let/=j X
dx.
IMDEFIMITE INTEGRALS 13.27

1
Let log .Y Then, (log .y) = dt=> — dx = cit=> dx = x dt
Putting log Y = t and dx = y dt, we get
(log xf
I
r.- X dt = t^dt=\J — + c
4
=
4
+ c

EXAMPLil i Ez’alunte:

(i)
(1 + log xf dx (ii) if {ax + b)f f' {ax + b) dx ,n ^ -1

ow
Y

SOLUTION (i) Let


(1 + log y)^ dx
Y

- dx = dt=> dx = Y dt
Let 1 + log Y = t. Then, d{l + log y) = dt=> X

e
Putting 1 + log Y = f and dx = = x dt, we get

re
I = — X x dt = t^dt = + C =
(1 + log y)^ + C

Frl
F
Y 3 3

(ii) Let/=| [f {ax + b)]" f' {ax + b) dx


ou
Putting f{ax + b) = fund f'{ax + b).adx = dt, we get

sor
u + 1 n+ 1
1 t {/ {ax + &)}'
1 - \ t"df = - + C = + C,u^
kf
a a n + 1 a {n +1)
oo
EXAMPLE? Evaluate:
Y
B

(i) 2 2^ 2^ dx (ii) 5
dx
Y
re

22-^ 2^"
oY

.,-V
SOLUTION (i) Let / = 2^ 2^ 2-'‘ dx. Let 2 = t. Then,
u
ad

2-X 2'^
2^
2^
= frf=> 2 2^'"" 2-''(log 2)^ dx = dt
d

d 2
in
Re

2^2X 2^2-r
--V

Putting 2 = t and 2 2^ 2''’ (log 2)^ dx = dt, we get


F

2-V
1 1
t +C = + C
I =
jdt =
(log 2)^
3
(log 2) (log 2)

1
1/4
Y 1
dx = (Ly
(ii) We have, I = 5 5 4
Y Y Y
Y

3 y'^
Let 1 - \-
.3
= t. Then, d 1 - = dt => — dx = dt=> dx - — dt
4 X
Y Y X

1 Y^
Putting 1-^ =fand,rfY=—rff,weget
Y
13.28 APPLIED MATHEMATICS-XII

x5/4
1 1
a J 4
1/4 4
=lf5/4 + C = i 1- 1
3
4-C
3 15 15 .V

1
I.XAMPLE4 Evaliinte: J dx

^(x-lf(x + 2f
1
SOLUTION Let dx. Then,
^(.v-1)^(,v + 2)5
X- 3/4
1 1 r x-1 1
dx - dx - X dx
f
-t-1
x3/4 x + 2
(T+2)^
\ A-+ 2
(A + 2) (A+ 2)2 .Y+ 2

w
-Y-1 3
Let = t or, 1 - = t. Then,
-r + 2 .r + 2

d 1 -
x + 1
3
= dt=>
3
j dx = dt=>

F lo 1

(A+ 2)2
dx = - dt

ee
(x + 2) 3

Fr
x-1 1
Putting - f and -^dx = ~dt, vve obtain
(x + 2) 2 3
x +2

n1/4
for
ur
1
I t
-3/4
dt = if’/-* + C= 2:
4 x-1
+ C
3 J 3 3 x + 2
s
ook
Yo

■‘H’LLa Evaluate:

(i) x'^(l + logx) dx (ii) x2'^ (1 + log x) frx


eB

SOLUTION (i) Let x’^ = t. Then,


r

d (x'’^) =di => d ^ .vlogx


(logx +1) dx = dt=> x''(l + logx) dx = dt
ad
ou

/ = x’^ (1 + log) dx= dt = t + C ~x^ +C


Y

(ii) Let/=Jx2-'(l + logx)rfx


Re
nd

Putting .Y' -1 and x'^(l + log x) dx = dt, we obtain


Fi

tdt=lt^ +C=-(x-'')2 + C = -2 x2-’^ + C


.V 1
I = X
x'^ (1 + log x) dx = 2 2
X
Mf'l 1 Evaluate: dx
.V-/7 -1
X
SOLUTION Let / = dx. Then,
x-V7^
] =
X X + 'Jx^ -1
X dx
X -
x + ^jx‘^-l
(x + Va2-1) dx =
i-r- x2-(x2-l) ' x2 + X 2
x“ -1 dx
13.29
IMDEFINIT6 INTECaALS

/ = A^rfA + -1 A' dx

=> I =
A
3 ir ^ dt, where f = a -1
3 2^

,=£!3 + lt3/2+c
3

/= —+
3
i(A^-l)^^^+C
3

E.^CJSE 13.7

ow
Evaluate the following integrals:
1
log 1 +
1.
logA-- dx dx
A A (1 + a)

e
(1 + V^)" 1 + dx

re
Fl
dx

F
.t
e
dx h
- (log A)^ dx
X.2
ur X
{i + n

r
3
7.
A

^dx (4 A + 2) -^1?+ A + 1 dx
fo
(:c^ +1)
ks
Yo
4a+ 3 1

1 dx 1 dx
oo

1 + Jx
^2 + 3a + 1
eB

log r.A+i 2
dx (a + log a) dx
X
X
ur

2x
2
13. f A e^ dx ‘Ir dx
ad

I
Yo

1 + e^'

A + ,/a + 1 i;5'^ V
^ dx 5^ 5- 5" dx
d

15.
Re

a + 2
in

5
X
4 A^ yjs - x^ dx
F

dx
3
+ A

1 1
dx dx
3/4
Va + A A^ (.V-^+1)

(log x)'^
|(1 + +C
1 1
+ c — < log 1 + — ( +c
2 2 ^ A
2 a^3/2
+ c
1
+ C ^(logA)^
3
+C
3 (1 + e’^)
(1 + 2a^) . -{x^ +x+D
3/2
+ C
2 yjlx^ + 3a + 1 + C
2-C
4 (1-^+1) 3
13.30
APPLIED MATHEMATIOS-XII

10. 2V^-21og|l+V^|+C 11. (log.T)^+C 12.


■^(.T + log.r)^ +C
13.1/.C
2 14. - log (1 + +C

5^
15
. (x +1) + 2 + 1 - 2 log IX + 21 - 2 tan ^ ^x + 1 + C 16.
3"^
(loge 5)
17. + C 4 /c 2. 5/2 20 .. 2,3/2
18. -{5-x )
9 3 ^ D y(5-.v‘) + c

1/4
1
]9. 2Iog|l + V7|+C 20. + C
X

w
Hwrs TO SELECTED PROBLEMS
8. Let I =
J (4x + 2) ^x^ + x + 1 dx. Then, I = 2 J (2x + 1) ^ + x + 1

F lo
dx
Let x^ + X + 1 = f. Then, d (x^ + x + 1) = dt => (2x +1) dx = dt

ee
= 2 =1/3/2
|(x^ + x + l)
/ + C = 3/2
+ C

Fr
●' 3
X T
+1i 2
12. Let I
— (x + log x) dx. Let X + log x for
- /.Then,
ur
1 x + 1
d(x + log x) =dt => 1+ dx = dt => dx = dt
s

xj X
ook
Yo

dt =1/3+c = l(x + logx)3+c


eB

1 1
"T Let 7=1
1
our

dx - dx. Let 1 + — = t
ad

x^x% 1)3/4 3/4


X
4

X
Y

1
4 \dx = --dt
Re

Then, fr 1 + — = dt => - = dt =>


nd

.4
X X 4
Fi

1 1 1/4 xl/4
I dt == - 1 r t -3/4 dt = -
1 /
+ C= - + c
4 J ^ 3/4 4 4. 1/4 X

13.9 SOME SPECIAL INTEGRALS

Let us discuss problems on evaluation of integrals by making above substitutions.


THEOREM For any constant a, prove the following:
(i) 1 . 1 , X: - a
+ C
1 a + X
(ii) :r2 frx = —
2a log^
2
+ C
X
a -X f? -X

1
(iii) log X + + x^
It 2^ 2^'^' " - a^ +C
i/x = + C
2 2
+ X
X -a

(i) Clearly,
13.31
INDEFINITE INTEGRALS

1 1 1 1 1
2 2
X -a {x - a) (x + a) la x-a X + a

1 1 1
I =
x
^dx =
-a
2a x-a X + a
■ dx

1 1
I = — dx- dx ■
2a X-a X + a

1 , X:-a
I
= -^|loglx-;Tl-log|x + (?|| + C = + C
-

2a X + a

(ii) Clearly,

w
1 1 1 1 1
+
2 2
a -X {a - x) (a + x) 2 a a + X a-X

Flo
1 1 1 1
dx + dx
I
2 jdx = 2 a ●’ +X a- x

e
a -X

re
a + X

log|fl-x|J
1 + C
I = a + X + c = r-log

F
2a 2a n-x

1 ( I 2 2
ur
dx. Let log X + V <7 + X =t. Then,

r
(iii) Let / =
a
2.
+ X
2 fo
ks
d log x + -Ja^+ X
Yo
oo

1 ,[
= d x + -\Ja
fl +x 2 = dt
B

=>

x + 4?^x 2
re

1 X
1 + dx = dt
u

I 2 2
i^ + x 2
ad

X +V +X
Yo

ri 2
■\ja +x +x 1
1
d

- dx s:dt ^ dx = dt
Re

Vfl^x^
2
in

x +
F

1 = 1. frf = t + c = log X 4- ^fa^ + X ^ +C

(iv) Proceed as in (i).


ILLUSTRATIVE EXAMPLES

EXAMPLE 1 Evaluate:
1 1
(i) dx (ii) 2
9x^ - 4 16 - 9x

1
SOLUTION (i) Let / = (ix. Tlien,
9x^-4
13.32
APPLIE" :.^ATHEMATICS-XII

2
.V -
1 1 1 3a:-2
I 3
yrf.V = -X + c = + c
9
:c^-(2/3)" 5
a: +
2 12 3a:+ 2
3 3

1
(ii) Let I =
16-9a-
2 dx . Then,
1 1 1 1
I dx = — dx
9J 16
9
- A'
2 9 J /
4)^ ~x
2
3j

ow
4
+ A-
/=i X —— ' X
log
3
+ C =
4 + 3a-
+ C
9 _r4 4 24 4 - 3a-
2 -A

3j 3

e
^rU:2 Evaluate:

re
(i)/
1
rfA

Fl (ii)
1
rfA

F
+ 25
l/4-v^-9
ur
r
1
SOLUTION (i) Let I = dx . Then,
^16a-^ + 25
fo
ks
Yo
1 1
dx = -4 log x+ x^ + 51^ Vl6?+^
oo

!
/ =- 1 4a-+
4 + C =- log + C
2 5^' u 4 4
eB

A +

4J

/=^log 4a + Vi6a^+25 - ^log4+C =^log 4a + Vi6a^ + 25 +C],


ur

4
ad
Yo

1
where Cj = —log
4
4+C
d

1
(ii) Let / =
Re

rfA. Then,
in

t/4.v^-9
F

ii
1
^=-1
2 J 2 9
dx dx
A
4
2 31"
2;

7 =
ilog
2
x + Jx^ 2j
+ C = i2 log A + x^ - -4 + C = 21 log 2x + ^[Z^
2
+ 9
+ C

I =ilog 2x + ^[4x^-9 ~^\og2+C = hog 2x + y[^ -9 +q,


1
where = —2 log 2 + C
13.33
INDEFINITE INTEGRALS

EXERCISE 13.8

Evaluate the follozving integrals:


1 1
1 dx
1. dx 2. -dx
(7 2 -h,22
X Jl+4.v^
1 1
1 dx
M dx dx
.n +h, 2 X 2
Ja J(2-xf+^
ANSWERS

ow
a +bx 1 ax - b
1 + C
1. log + C
1
● - - log /7,V + b
lab a -bx

3. ^ log I 2.V + ^4x^ + 11 + C 4.1b log bx + '


+C

e
re
5. - log (2 - .r) + +1 +C

rFl 6.-log 2-X- + ^J(2-xf-l +C

F
HINTS TO SELECTED PROBLEMS

r
ou
1
dx -
1
dx = log 2x + fo 4x +C
ks
j.

^7^
oo

5. ,
1
dx = -
1
= d(2-x) = log (2-x) + ^(2-.t)^+1 +C
Y

/(2-xf+^
2

p-xf+l
B
re

6.
1
dx = -
1
= fr(2-x)=-log 2-:.x +
p-xf -1 +c

p-xf-1^ ^ p-xf-l
ou
Y
ad

1
13.9.1 EVALUATION OF INTEGRALS OF THE TYPE f
d

-;v I ●
in
Re

To evaluate tMs type of integrals we express ax^ + bx + c as the sum or difference of two
F

squares by using the following algorithm.


ALGORITHM

STEPl Make the coefficient ofx^ unity, if it is not, by multiplying and dividing by it.
STEP II Add and subtract the square of the half of coefifcient of x to express ax +bx + c in the form a

x + —
b 7 4ac -b^
2(7

STEF 111 Lise the suitable formula from the following formulas:
^ log
x-a
a +X ^ .... r 1
P^^x = I log
+ C.
(i) - +C (ii) 2 ~ 2(7
a - X ^ X -a X + a
a X
13.34
APPLIEID MATHEMATICS-Xil

ILLUSTRATIVE EXAMPLES
I XAMI’LE 1 Evaluate
1 1 1
(i) dx
2.v^+x-l
(ii)
3 + 2a; -
jdx {iii) J 3a-“ +13a:-10
dx

1
SOLUTION (i) Let / = j 2a:^ + .t -1
dx . Then,

/=i 1
dx

2 2
1
/
- dx
x^ + x/1 + (1/4)^ - (1/4)^ -1/2

w
1
^dx
2
(,V +1/4)2-(3/4)

F lo
1 1 -r +1/4-3/4 1
/ =-x -v-1/2 1 2.t-l
log + = '
+ C=-log + C
2 2(3/4) -T + 1/4+ 3/4 3 a:+ 1 3 2{.r + l)

ee
1
(ii) Let / =
-J dx. Then,

Fr
3 + 2.Y-.V
1
1
I =
~(x^ ~2x-3)
dx
for
ur
1
I = dx
-(.v2-2.y + 1-1-3)
s
ook
Yo

1
I = dx
eB

|(Ar-l)^-2^j
1
2 + (x-l)
our

=i> /
= 1 y frx =
-V+1
ad

+ C =
T^Og + C
2^-(.r-l) 2(2) 2-(.t-l) 4 3-.V

1
(iii) Let I =
Y

dx. Then,
3a:2+13a:-10
Re
nd

1 1
dx
Fi

.2^13 X -
10
3 3
1 1
dx
^ X
2 + 13■x+ (13']^

13 10
3 I 6 6 j
1 1
/
1 y dx
3- /
.t +
13^ [17^
6 J I 6 j
13 17
a: + -
1 1 1
/ 6 6 x-4/6 3.Y - 2
3 13 17 +C =—
17
log + C = 17 log + C
2 A- + — + x +5 3 (x + 5)
I 6 6 6
INDEFINITE INTEGRALS 13.35

EXERCISE 13.9

Evaluate the following integrals:


1 1 1
dx dx
1.
4x^ + 12.x + 5
2.
1 + .T - X
jdx ^ 2x^~x-l
ANSWERS

2x + l ^/5 -1 + 2x
I log
1

2x + 5
+ C
■ V5
log
^/5 +1 - 2x
+ C 3.
3
log ^
2x +1
+ C

13.9.2 INTEGRALS REDUCI3LE TO THE FORM f

Following examples will illustrate the procedure of evaluating the above type of integrals.
ILLUSTRATIVE EXAMPLES

w
LXAMri-. ; Evaluate:

F lo
X e
X
2x^
dx (ii) dx (iii) 8
dx
xU.x^-1 e^'^ + 6e^ + 5 4-x

X X

e
SOLUTION (i) Let I = dx = dx

Fre
X 4 + X 2 -11 (.xV+x^-1
dt
Let .x^ =f. Then, fr(x^) = dt=> 2xdx = dt=> dx = —
for
2x
X dt
r
I X

2x
You
oks

1 1
I = - dt
eBo

2 t^t-1

f +
1 V5
1 1 1 1
our

2 1 +C
ad

I — X
log
2
t +
V5 2
V5 f+ l_^V5
2 2 2
2j 2
dY
Re

1 2t+l-f5 1 2x^+1-^/5
I = log + C = log +C
2^/5 2t + l+f5
Fin

2V5 2x^+1+^’5
X X
e e

(ii) Let / = J + 6e^ + 5


dx -
+ 6 e'' +5
dx

Let (’■' = f. Then,d(c^) = dt ^ e^ dx = dt


dt 1 f + 3-2 + 1
+ C=ilog
1
+ c
/
^dt = 2x2 log
t^ + 6t + 5 {t+ sf-2 f + 3 + 2 4 /+5

2x' 2x^
(iii) / = dx = dx
8
4-x 2^-(x^)
dt
= f.Then,d(x^) = dt=>4x^ dx - dt^dx =
4
Let X
4.X-3
13.36 APPLIED MATHEMATICS-Xll

4
2x^ df 1 2 +t 1 2 +x
I = log + C =-log 4"^
4-r 4.v^ ^ 2 -t 2x2x2 2-t 8 2-x

{■;XAMPLE2 Evaluate:
- X
1 e
(i) dx (ii) dx
-2.T
X {6 (log .v)^ + 7 log .V + 2} 16-9e
1
SOLUTION (i) Let / = dx
X {6 (log x)^ + 7 log .V + 2}
Let log X = t. Then, d (log .r) = dt — dx = dt^ dx = x dt
X
1
I dt
6f^+7t + 2
/ =if I

w
dt
1
t +
6 3

F lo
1 1
I = - dt
6 ●’
t + 7 1 _ ^
12 J 3 144

ee
Fr
1
=>
6 ^ f 71^ for
t +
12 12
r
7 _ 1
You

t +
s

1 1 12 12 2t + l 2 log X +1
ook

I — X
log 7 1 + C = log + C = log + C
6
2
1
f +
3f + 2 3 log X + 2
eB

12 12 12

- X - X
e e
(ii) Let / = dx = dx
our

-2.V
A^-(3e
ad

16-9e

dt
Let 3e ^ = f. Then,rf(3e - dt^-3e ^ dx = dt=>dx = -
3e ^
dY
Re

dt
Putting 3e ^=fand dx = - —, we get
Fin

3e
- X
dt 1 dt 1
^dt
e
I =
16
o
3 e
- X
3-^ 16 3
(4)2-(2
1 1 4 + 1 4+3e ^
/
3
X
2x4
log
4-t
+ C =-^log
24 4-3e
+C

EXAMPLES Evaluate:
1 1
(i)J x(x” + 1)
dx (ii)
X (x^ +1)
dx

SOLUTION We have.
n-\
1 X
I = dx = dx
X (X ' + 1)
, n , n ,■,
x (x + 1)
INDEFINITE INTEGRALS 13.37

ji- 1 dt
Let .T' +1 = f. Then, d (,t" + 1) = dt=> n x dx = dt^ dx =
M - 1
It X

1 1 1
I dt = - dt [v A-"+l=f x' =t-\]
nx t n-'

1 dt 1
}
jdt
n
r-t n ■’
-t + 1/4-1/4 (t-l/2f-{1/2)
n
t-1/2-1/2 t-1
+ C = i log —
1 1 1
+ C=-log + C
I =-x log
n 2(1/2) f-1/2 + 1/2 n
f 1 n

4
1 .V

ow
(ii) / dx = dx
i-(.r=+l) /(/+i)
dt
Let A'^ + 1 = f. Then, (a^ +1) = dt=> Sa'"^ dx = dt=> dx =
5x^
1 1
^dt = -1
1
dt = -

e
I =
S*" tx 5 5J l(f-l) ^ r-t

re
Fl 1
dt-li
1

F
5 l^-l + l/4-l/4 (f-1/2)^-(1/2)^
ur
5

r
1 1 f-1/2-1/2 1 1-1 1 X
/ =-x
5 2(1/2)
log
1-1/2 + 1/2
+ C=-log
5 t
fo + C=-log
5 .r^ + l
+ C
ks
Yo
EXERCiSE 13.10
oo

Evaluate the following integrals:


eB

X 3x 2

● I -6^*
e e
1. dx dx
+5 e^ + 6 4^6a_9 X - a
ur

X
1 X e

I
ad

dx dx dx
^ Sa**-18a:^+11
Yo

a: (a^ +1) H+e^){2 + e^)


ANSWERS
d

3x 3
Re

X
in

e'^ + 2 2e -3 1 a

log + C ~ log + C
■ z: 3 -3 3
e
X
●%
+ 3
o
36 2c^^ + 3 6a A + a
F

X ^-3- ^ 1 + e^
1 V3 V3 + c + C
log — log
T>Og-^
+ C
4
’ 6 X +1 ■ 48 .r" - 3 + 2 + e-’'
V3

70 SELECTED PROBLEMS
X
e
b. Let / = dx. Let = t. Then, dx = dt
(1 + e^) (2 + e^)
1 1 1
dt = dt = dt
(1+1) (2 + 1) (t^ + 31 + 2) 1 +
3f rif
2
13.38 APPLIED MATHEMATICS-XII

3 1
t +
2 2 t + l + 1
=> I = log + C = log + C = log + C
2x X 3 1 t + 2 + 2
2 2 2

13.9.3 INTEGRALS OF THE TYPE r-- ;/v


fl.v" - bx + c
\

In order to evaluate this t) pe of integrals, we may use the following algorithm.


ALGORITHM

2
STEP I Make the coefficient ofx unity, if it is not.
STEP II Find half of the coefficient ofx.

low
^2
1
STEP 111 Add and subtract — Co(ff. of x inside the square root to express the quantity inside the
K2
bf 4ac-b^ 4ac -b^ bf
square root in the form x + — + or, X +

ee
2.
rF 4.2 4.2 2.

Fr
STEP IV Use the suitable formula from the following formulas:
\ I 2 ^
,
.2 + A.-2
dx = log X + Ja + X + C,
X
for
-a
dx = log x + ^ 2 +C
u
ks
ILLUSTRATIVE EXAMPLES
Yo
o

EXAMPLE 1 Evaluate:
Bo

1 1
(i)f dx (ii) dx
^-l){x~2) ^2^2 + 3x - 2
re

1
SOLUTION (i) Let i = J dx. Then,
ou
ad

P-l) (1-2)
Y

1 1 1
I =
^2 - 3a* + 2
dx =
1 2 9 9
dx dx
x‘^-3x+ --- +2
nd
Re

4 4
2) 2
Fi

3^1 3^^ 3l
I = log a — + , a - + C = log -T - + -3x +2 + c
2j f 2) 2
V “ y
2

1
(ii) Let / = J dx. Then,
^2a2 + 3a - 2
1
I = da = 4 1
dx
a2 + ^ a-1 42 3'i2 9
V 2 a + -1
4j 16

/
1 1
dx = ^log a + — 3 'i +,a ri 3
+ — a-1 r + C.
42 42
a +
3^^ f5V 4 2

4 4j
INDEFINITE INTEGRALS 13.39

EXAMPLE 2 Evaluate:
1 1
(i) dx (ii) dx

^(x-a) (x-b) |v^-4.v + 2


1
SOLUTION (i) Let 1 = dx. Then,
J ^(x-a){x^
1
I = dx

Jx~ - X (a + b) + ab
1
I dx

^ - a:(« + b) +
a +b]^ f a + b]^ + ab

r I 2 2

1
I dx

a-b)^

w
a + b
■ X -

[ = log
1
X -

lx -a-b
a + b
2
2

)■ + . < X-

1
2

2
F lo
a+b
2
a-b
2
+ C

ree
for F
I = log 2
+
^{x-a){x-b) + C
Your

(.\--rt) + (x-&) + 2^{x-a){x-^


ks
eBoo

+ C
I = log 2

a2
ad

/ = log (
our

=> - log 2 + C

/ =
2 log ^x - a + -b + C, where = C - log 2
Re
Y

1
Find

(ii) Let I = dx. Then,


X ^ -4x+2
1 1
I dx = dx

Jx~-4X + 4-4 + 2
I =
log (x-2) + ^{x-2f ~{42f +C = log x-2 + .j? -4x + 2 +C

EXERCISE 13.11

Evaluate thefollowmg integrals:


1 1
1. dx 2. dx

■Jsx^ + 5a: + 7 ^5x^ -2x


13.40 APPLIiD MATHEMATICS-X:i

ANSWERS

l.i Ilog x + -5 + Jx
\ 2 +—X
5 + —7 +C^ 1
log
5x~]
+
-lx
+ C
V3 ^ 6 V 3 3 V5 5 75

13.9.4 JNTEGF .1LS REDUCIBLE TO THE FORM

Following examples will illustrate the procedure of evaluating this type of integrals:
ILLUSTRATIVE EXAMPLES

' ● AMPLE ! Evaluate:

w
2
sec X 1
(i) dx
(ii) j dx

ijl6 + tan^ .Y x.^(log xf -5


X
2

F lo X
2

ee
SOLUTION (i) Let / = dx = dx
6

Fr
-X

dt
LetA:^ = t. Then, d(A:^) = dt=> 3x^ dx = dt=^dx = for
3x^
ur
1 dt 1 . -1
I = -
3 ●’
— sm
(f)+C = -3 sin \a'^)+C
s
ok
Yo

2 2
o

sec ,v sec X
(ii) Let, I = dx = dx.
eB

■^|l6Vtan^^ ■^4^ + tan^ a:


^ dt
r

Let tan x = t. Then, d (tan .v) =dt^ sec x dx = dt=> dx -


ou
ad

2
sec X

dt dt
log t + ^4^ + t^ + C = log tan x + ^16 + tan^ x + C
Y

r=i
Re
nd

1
(iv) Let / = J dx
Fi

X ^og xf -5

Let log a: = f. Then, fr (log a:) = df=>-d.v = dt=> dx - X dt


a:

1
/ =
= = \og\t + yjr-5\+C=\og logr + -J(log.r)^-5 +C
i/A-(75)
EXAMPLE .r Evaluate:
X
a lx X
(i) dx (ii) dx (iii)
1 + n
2x 4
+ X
3 frT
X + X

X X
a a
SOLUTION (i) Let I = dx = dx.

i/p+(»')
2-y
1 + a
13.41
INDEFINITE INTEGRALS

(it
Let = t. Then, d {a'^) = frf => logg a dx = dt^ dx =
log,, a
X
a dt 1 dt
I
a'^ log a logfl

— log t + C = logn— log + Vl + n


2x
I = +C
log a
2x 2x
(ii) Let / = =4 dx = J\ dx.

-Jl^x^ + x Jl-x^+(x^)

ow
dt
Letx^ = t.Then,d{x'^) = dt^2xdx = dt=>dx = 2x

1 1 1
= dt= { dt

e
1 =
2 J
2-f + l) _1 + 1

re
1-f+t
4 4

Frl
F
1 1
I
I dt^ dt

3
ou
t- + - +

r
2J
f 2
4 2

so
kf
+ ^f^-f + 1 =log x^-- +x[^-x'^ + l +C
1
I = log t--
2)
oo
Y

(iii) Let / = I 3 frx =


eB

3
fl + a:
a
3/2^^ + -V
3/2^^
ur
oY

3/2 3/2.
Let X = t. Then, d {x = dt=> dx = dt=> dx =
2 3yfx
ad

dt = -^log t + *3 log ,r=’^^V«'+.r"+C


2 1
d

3
3/2^^
in

a
Re
F

EXERCISE 13.12

Evaluate the following integrals:


X
e X
X
dx dx dx

X
4
4- a
4
16+ e
lx
f77
1 1
a dx dx

xJ4+9(logxf
ANSWERS

^ log x^ + ^x‘^ + a^ + C log e^ +-J^e


lx
+C

3.-^ log x^ + Vl +A'^ +C


13.42 APPLIED MATHEMATICS-XJI

5. 3 log ^ -4 +C

px + q
13.9.5 INTEGRALS OF THE FORM \ ;/.r

To evaluate this type of integrals, we use the following algorithm.


ALGORITHM

STEP 1
Write the numerator px + q in the following form:
px + q = X
(ax^ +fj.T + c)| 4- g i.e. px + q = X{2ax + b) +g

ow
STEP II
Obtain the values ofXand g by equating the coefficients of tike powers ofx on both sides.
STEP 111
Replace px + q by X (lax + b) +p in the given integral to get
px + (i 2i?.v + b 1
I 2
dx = X dx + g dx

e
2 2
ax + bx + c ax )- bx + c ax +bx + c

re
STEP IV

rFl
Integrate RHS in step III and put the values ofXand g obtained in step II.

F
Following examples illustrate the above algorithm.

r
ou
ILLUSTRATIVE EXAMPLES
fo
ks
EXAMPLE 1 Evaluate:
oo

X 4.X + 1
(i) dx (ii) dx
x'^ + x~l X^ 4- 3x 4- 2
Y
eB

2x-3
(iii) dx
+ 3.Y-18
r
ou
ad

SOLUTION (i) Let x = X ~ (x^ + x + 1) + g. Then, x =


Y

(2x + 1) 4- g

Comparing the coefficients of like powers of x, we get


d

1
1 = 2X and A.4-g = 0=>A.= — and g =
Re
in

2 ^ 2
X
F

I = dx
X^4-X-l
I =
1/2 (2x4-1)-1/2
dx
X^ 4-X-l
2x 4- 1
/=if dx -
1
dx
2 x^ 4- X -1 X
2
4-X-l

2x4-1 1 1
dx - dx
2 J 777^1 2
X
2
4- X 4-
1 5
4 4

2x4-1 1 1
^ = -1
2 J .V^ 4-X-l
dx-
2-' /
dx

X 4- -
V5
2> 2
INDEFINITE INTEGRALS 13.43

( 1] -J5
I 2) 2
I =
|log|AT^+^-l|-^X ( x + -1) + —
V5
+c

2 2 2

2x-hl-V5
=> / =
ilog|x2 + x-l|-^log 2x + l-^ +C

(u) Let 4jc + 1=X —(:c^


dx
+ 3:c + 2) + n.Then, 4x +1 = A.(2jc + 3) + n

Comparing coefficients of like powers of x, we get

ow
2 A, = 4 and 3A, + p =1=>X = 2 and p = -5
43: +1
I = dx
x^ + 3x +2

e
j ^ j 2(2:c-h3)-5
Fl
re
dx
X + 33: + 2

F
2x+ 3 1
^=^12- dx-5 ac
ur
a: + 3a: + 2 X + 3a: + 2

1
I = 21og|3r^ + 3:c + 2|-5 j a:^ + 3a: + (9/4)-(9/4)+ 2 dx or
k sf
Yo
1
oo

=>
I = 21og|3:^ + 33: + 2|-5 j (jr+3/2)^-(l/2) 2^X
eB

3 1
x +

I = 21og|x^ + 3x + 2|-5x.j^log
2 2
+ C
ur

3 1
X + - + -
ad

2 2
Yo

X + 1
I = 2 log I a:^ + 3a: + 21 -5 log + C
d

a: + 2
Re
in

(iii) Let 2a: - 3 = ^— (a:^ + 3a: -18) + p.Then, 2a: - 3 = >.(2a: + 3) + p.


F

dx

Comparing coefficients of like powers of x, we get


2 A, = 2 and 3X, + p =-3=>A, = l and p = - 6
2x-3
I = dx
x^ + 3x-18
2X+3-6
dx
x^ + 3x-18
2x+3 1
'=It2
x" + 3x-18
dx-6
x^ + 3x-18
dx

1
I = log|*^ + 3j;-18|-6| -
x^ + 3X + ----18
dx

4 4
13.44 APPLIED MATHEMATiCS-XII

1
I = log I .Y^ + 3a: -181 - 6 n2 Y dx
X +
3 (9^
2) 12
3 9
Y +
1
I = logi Y^ + 3Y-18j-6x log
2
3
2
9
+ C
2 Y + — + -
2 2 2

;-3
/ = log|Y^ + 3Y-18|-^3 log -
Y + 6
+ C

i.XAMl’i 1 : Evaluate:
3
r Y + Y 1
(i) rfY (ii) dx
Y^ -9 j «
2e
2a:
+ 3 £?
~ a:
+ 1
,

w
r Y + Y
SOLUTION (i) Let 1 = dx. Then,

/1 -
=

Y
Y

Y^ -9
Y

^ -9
3

3
dx +

dx and I2 =
Y ^-9

Y^ -9
X
dx =

Y ^-9
Y
F lo
dx.
+ /2 (say), where
for F
ree
4 3
Putting Y -9 -t and 4y dx = dt, we get
Your

^ dt
ks

\j i* = jlogM =ilog|x“-9|
Y 1
eBoo

= X

t 4y^
Putting Y^ = t and 2xdx = dt, we get
ad
our

X X
h = 4
dx =
^dx
y’ -9 - 3

dt y^-3
Re

1 1 f-3 1
h - = — X
log = 7T log
Y

t^ -3^ 2 2x3 t+3 12 y2 + 3


Find

2
-3
Hence, I = /1 + /2 = Tlogl^^"9|
4
+ :^log
12
-Y^ + 3
+ C

(ii) We have.
-2x
1 1 e
/ =
2e^^ + 36>-^ + 1 rfx = I ^ +
3
+ 1
rfY =
2 + 3c + ^
-2a:
dx

-2a: -X
e e

dt
Let e ^ =LThen, ^) = dt=> -e ^ dx = dt=> dx = - - X
e

-tdt t
I
1 2+3t + t^ I t^ + 3t +2
dt

Let t = X (2^ + 3) + )i.


INDEFINITE INTEGRALS 13.45

Comparing the coefficients of like powers of t, we get


2X = ], 31 +\i = 0=> X = 1/2, p = -3/2
I
X (2t + 3) + p dt
+ 3f + 2

■ + 3 1
1 = rft - p dt
+ 3f + 2 + 3f + 2

1 f +3 3 1
1 = -
2^ f^ + 3f + 2
dt + —
^dt
2 ●'
(f+ 3/2)^-(1/2)
3 1
f +
2 2 + C
I -log
2
+ 3f + 2 + —2 X —r
1
log 3 1

w
2 f +
2 2
2J

F lo
^ +1
+ 3e ■' + 2| + —2 log ^"-'
-2x
+ C
2 + 2

ee
3
AT

Fr
li - -MPI.L
Evaluate: | + 3x^ + 2
dx

3 2

SOLUTION (i) Let I = dx. Then, 1 =


1 for .V
2.Y dx.
ur
+ 3-y^ + 2 2
{x^f + 3.v^ + 2
Let x^ = t. Then, d (y^) = dt or, 2y dx = dt
s
ook
Yo

1 t
I =- dt
2-' t^ + 3t + 2
eB

2f+3 3 1
1 = if =l - dt — dt
4-’ r + 3f + 2 4^ r + 3t + 2 ^^'r + 3+2
r
ad
ou

1 r 2f+ 3 3 1
=> [ =- dt -
jdt
4 ■’ t^ + 3f + 2 4
3f 1
Y

t + -
2 U
Re
nd

3 1
t +
Y^+1
+ C =ilog|Y‘* + 3Y^ + 21-|log
Fi

1
/ =
4
log] t^ + 3f+ 2|-—log
4 . 3
2 2
1 y^.-2
+C
t — +
2 2

EXERCISE 13.13

Evaluate the following integrals:


Y + 1 y-3
I
Y
dx dx dx
Y^ + 3y + 2 Y^ + Y + 3 Y^ + 2 Y - 4
2y 2y + 5 y+5
4. dx dx
2 + Y -Y
2
Y 2-Y-2 3y^+13y-10
y +7 3y + 5 ^-3yY
dx dx 9. dx
3y^ + 25y + 28 y^ + 3y-18 x^+2x^-A
13.46 APPLIED MATHEMATICS-Xll

ANSWERS

ilog];r^ + 3j: + 2|-|log


.Y + 1 2x4-1
2. i log I x^ + X 4- 31
1 -1
1. + C 4- tan + c
X4- 2 Vn Vn
i-Vs
^ log I + 2.V - 4|-|log
X 4-
3. 4-C
X 4- 1 4-Vs
1 4-X

-log|2 + x~x^i + -^log


x-2
4.
2-x
+ C 5. log I x^ X - 21 4- 2 log X4-1
+ c

3x-2 1
6.
-log|3x^+13x-10|
6
+-log
6
+C 7. -log|3x4-4l4-C
3(x + 5)

w
I log I ,Y^ + 3.V -181 +
1 x-3
8.
log 4-C
18 X4-6

9.
^Iog|x^+2x“
2 , -v^+l-Vs

F lo +C

ee
P(.v)
13.9.6 INTEGRALS OF THE FORM f ,

Fr
.fv, WHERE /VxllS A POLYNOMIAL OF DEGREE
TWO OR MORE ● n + />.v ●(

To evaluate this type of integrals \ve divide the numerator by the denominator and express the
for
integrand as
ur
K (.V)
Q(-V)4- , where R (x) is a linear function of x.
s

2
ook

nx +bx + c
Yo

P{x) R{x)
I dx = Q (x) dx 4-
eB

dx
2 2
ax + bx + c ax +bx + c

Now, to evaluate the second integral on RHS apply the method discussed earlier.
our
ad

Following examples will illustrate the procedure.


Y

ILLUSTRATIVE EXAMPLES
Re
nd
Fi

LXAMi’LLl Evaluate:

x^ + X + 1 ■ x^ 4- 5x -i- 3
{i)J x^-1
dx (ii)
x^ + 3x + 2
dx

X^ 4- X 4- 1
SOLUTION (i) Let / = I - X
2
-1
dx. Then,

2x +1
/ X + dx
7^
2x 1 x^ 2 1 x-1
{ X dx 4- dx 4- dx =
y4-l0g|x^-l| + -l0g x + 1
+ C

X^ 4- 5x 4- 3
(ii) Let I = dx. Then,
x^ 4- 3x + 2
INDEFINITE INTEGRALS 13.47

2.Y + 1
I 1 + dx
+ 3y + 2

2y +3-2
I 1 ● frY + dx
X + 3y + 2

r 2y+ 3 1
=> I - 1 ■ dx + dx - 2 dx
^ Y^ + 3y + 2 Y^ + 3y + 2
1
I = Y + log 1 Y^ + 3y + 21 - 2 ^dx
Y + -
2> 2)

low
3 1
Y +
1 2
I = Y + log I Y^ + 3y + 21 - 2 X

^log 3
2 Y + - + -
2 2
2)

ee
rF Y + 1
/ = Y + log I Y^ + 3y + 21 - 2 log + C

Fr
y + 2

EXERCISE 13.14

Evaluate the following integrals:


for
u
f Y^ + Y + 1 Y^ + Y - 1
ks
1. dx 2. dx 3. dx
Yo
2
Y
2
- Y Y + Y -6 Y (1 - 2 y)
o
Bo

2
Y
4. r dx 5. rfY
y^-5y + 6 Y^ + 7 Y + 10
re

ANSWERS
ou
ad

Y-1 y-2
1. Y + logi Y^ - y| + 2 log + c 2. Y + log + c
Y

Y Y + 3

3. ^ Y + log I Y I -1 log [ 1 - 2 Y I + C 4. Y - 5 log IY - 21 + 10 log IY - 31 + C


nd
Re
Fi

7 T 29 y + 2
5. Y 2
log I -Y + 7 Y + 10 I + —
6
log y+5
+ c

HINTS TO SELECTED PROBLEMS

4. We have.
Y^+l 5y-5 Y-1
I dx = dx = f 1 ● ^^Y + 5 [ -y dx
X
2
-5y + 6 Y -5y+ 6 ^ ’ y^-5y+6
5 2y-2 5 r 2Y-5+3
I Y + — dx = Y + - dx
2 Y ^-5y + 6 2 Y ^-5y + 6
5 2y-5 15 1
1 Y + - dx A
n2 2
2 Y ^ -5y + 6 2
5 ^1^
Y -

2; {2)
13.48 APPLIED MATHEMATICS-XH

5 1
X —
5 , 2 c z 15 1 2 2
/ = x + - log X - 5x + 6 + — X log + C
2 5 1
^ 2 X — + -
2 2
2;
5
/ = x + -
2
log^ x^ -5x + 6 + — log -—- + C
2 ® x-2

3.9.7 i"TEGRALSOFTHEFOP*-

In order to evaluate this type of integrals, we use the following algorithm:


■ CORITK'.

Write the numerator px + q in the following form:


px + q = X ■—
dx
{ax^ +bx + c) + p i.e. px + q = X {2ax + fj) + p

w
Obtain the values ofX and p bi/ equaling the coefficients of like powers of x on both sides.

F lo
Replace px + qbyX {2ax + &) + p in the given integral to get
aI
2ax + b 1
1 dx = X dx + p. dx

^ax^ +bx + c +bx + c


■Jax^ +bx + c

e
Fre
n r i Integrate RHS in step III and put the values ofXandp obtained in step II.
Following examples will illustrate the above algorithm.
for
ILLUSTRATIVE EXAMPLES
r
You
oks

'I L i Evaluate:
eBo

r 2x+3 x + 2 fTT7
(i) dx (ii) dx (iii) dx

yj^X^ + 4x + 1 yjx^ + 5x + 6 V
d 9
ad
our

SOLUTION (i) Let 2x + 3 = X — (x + 4x + 1) + p. Then,


dx
2x + 3 = X(2x + 4) +p.
Comparing the coefficients of like powers of x, we get
Re
dY

2X = 2 and 4X + p = 3^X = 1 and p = -1


2x+ 3
Fin

I = dx

yj.x^ + 4x + 1
(2x + 4)-l
I = dx

yjx^+ 4x + 1
2x + 4 1
I =
dx-j dx

yjx'^ + 4x + 1 yjx^ + 4x + 1
dt 1 2
I = dx, where t -x + 4x + 1
^+2f-iff3f
/ =
2^-log (x + 2) + -^x^ + 4x + 1 +C
I
2 yjx^ + 4x + 1 - log I X + 2 + ^ + 4x + 1 I + C
INDEFINITE INTEGRALS 13.49

(ii) Let;c + 2 = X^(:c^+5A: + 6) + |x.Then, a: + 2 = >.(2:c+ 5) + |x


Comparing the coefficients of like powers of x, we get
1 1
l=2A,and5A. + p= 2 => k = - and \i = --
x + 2
I dx

^x^+5x + 6
1 1
i(2x + 5)--
2^ 2
Idx

w
2x + 5 1
/=if dx- dx
2 J
^x^ +5x +6 yjx^ + 5x + 6

Flo
1
/ = _ if dx, where f = x^ + 5x + 6
~ 2' -Ji 2 f (lY

e
re
\yv ^■'2 "2
v-^y

F
J = Vf-ilog x + — + yjx^ + 5x + 6 +C
ur
r
fo
/ = +5x + 6 - - ^ log X + — + ijx^ +5x + 6 + C
ks
V
Yo
oo

(iii) We have.
1 + X 1+x 1+x 1 + x 1 + x
dx =j dx
B

dx =
X
X ^1 + X
^X{l + x) -J? + x
re

Let x +1 = 1 — (x^ + x) + |Li. Then, x +1 = X (2x +1) + p.


u
ad

dx
Yo

Comparing the coefficients of like powers of x, we get


1 = 2X and A, + |i =1=>A. = ^, p = ^
d
Re
in

II + X
I = dx
F

x +1
/ =
2 dx = f 2
dx

^x +x yjX +X
2x + l 1
/ = i dx + i dx
2-'

1
, = if 1 dx, where f = x^ + x
2^7^ 2J 17 1^^
i 2j 2J
I 1 1 ^

I = Vf + ^ log x + - + ^x^ + X +C Jx^ + x + -log


2 \
x + -
Zj
+^x^ + x+C
^yZ V
13.50 APPLIED MATHEMATICS-Xn

EXERCISE 13.15

Evaluate the foUozoing integrals:


X 2x-hl 6x-5
1. dx 2. dx .t. dx
+ 6j; +10
^?+2.y-1 yjsx^ -5y + 1
.v + 2 y + 2 x-1
dx S. dx dx

yjx^ + 2x-l
Y Y + 1 2y + 5
7. dx dx 9. dx
Y^ + Y +1
2y + 1
Y^ + 1
■Jx'^ + 2y + 5
in
2y+ 3 5y + 3
dx dx 12. dx
Y^ + 4y + 3 + 4y + 5 + 4x + 10

w
y+2

yjx^ + 2y+ 3

F lo
ANSWERS

■jx^ + 6y + 10 - 3 log I (y + 3) + ^Y^ + 6y + 10 | + C

ee
1.

Fr
2 ^x^ + 2x-l - log I Y + 1 +
^y^ + 2y-1|+C
2^3? -5y + 1 +C
for
ur
■ ^?+2y-1 +l0g|(Y + l) + .jY^ + 2Y-] I+C
s
ok
Yo

5. -1 + 2 log I Y + ^Y^ -11 + C


o
eB

6. ^/?+T - log I Y + ^Y^ + 1 I + C


2y +1
'y^+y^I- ■ log 1
■Jy^ + Y + 1
r

+ c
ou
ad

2 2

+ 1 + log I Y + yjx^ + 1 I + C
Y
Re
nd

2 + 2y + 5 + 3 log I Y +1 + .^Y^ + 2y + 5 | + C
Fi

2 ^Y^ +4y+ 3 - 3 log IY + 2 + ^Y^ + 4y + 3 [ + C


2^x^ + 4y + 5 - log ^Y^ + 4y + 5 +C
Y+ 2 +

5 -^Y^ + 4y + 10 - 7 log Y + 2 + ^Y^ + 4y + 10 + C


+ 2y + 3 + log (y +1) + >/y^ + 2y+ 3 + C
H/A/7S TO SELECTED PROBLEMS

? We have.
y+2 Y 1
/ = dx = rf.v + 2
I rfY
13.51
INDEFINITE INTEGRALS

1 1
=> I = 12 J. ^
rr
dx + -
2
dx

=> / = 1x2 +“ log x + ^ r-1 +c =


yjx^ ^ ^ log .V + ^-1 +C
6. We have.
;r-l 1
/ = dx =
f (/.t - rf.r

■J?+T .v“ +1

1
2.r
=> / = - ii.T- rf.v
2

=> / = 1x2 - log -T + ^.v^+1 +C = Jx^ + 1 - log .r + ,Jx^ +1 + C

w
13.10 INTEGRATION BY PArV-
THEOREr2 If u and v are two functions ofx, then
(r
tivdx = u\\ V fr.T -J
j 1 r f

F lo
f V dx ]■ dx
J ^

ee
tegral of the product of two functions = (First function) x (Integral of second function)

Fr
i.e. The in

- Integral o/| (Diff. offirstfunction) x (integral of secondfunction) |


for
nuH't For any two functions / (x) and g (a:), we have
ur
£ 1/ (*) ■ s (*)l = / (●':) ■ f Is +
s

4- Is (i-)l + i s
ook
Yo

fix)- dx dx
eB

fix)- ^ Ig (-t)1 I dx +
dx
g{x)--^{f{x)}\dx=f{x)g{x) dx
our
ad

^{fix)\\dx
{/(.v)-£teWI ■dx=f{x)-g{x)~ S ix) ■ dx
Y

Let f (x) = u and — \g (.v)} = u so that g (x) = V dx. Substituting these in (i), we get
Re

dx
nd

du
uv dx = £< -I V dx ■ - V dx> dx
Fi

dx

Proper choice offirst and second function —


I

Integration with the help of the above rule is called the integration by parts. In the above ride there _
are two

terms on RHS and in both the terms the integi-al of the second fimctwn is involved. Therefore in the
product of two functions if one of the tivo functions is not directly integrable (e.g., log x), we take it as the
first function and the remainingfunction is taken as the second function. If there is no otherfunction, then
unity is taken as the second function. If in the integral both the functions are easdy integrable, then the
first function is chosen in such a way that the derivative of the function is a simple function and the
function thus obtained under the integral sign is easily integrable than the original function,
■ SO I t We can also choose the first function as thefunction which comesifrst in the word LATE, where

L - stands for the logarithmic functions


A - stands for the algebraic functions
T - stands for the trigonometric functions
£ - stands for the exponential functions
13.52
APPLIED MATHEMATICS-Xll

ILLUSTRATIVE EXAMPLES

LXAMi'i i ‘ Evaluate:

(i) X dx
(ii) I x^e^dx (iii) -Y log X dx
SOLUTION (i) Here, both the functionsviz. y and e'^ are easily integrable and the derivative of y
is one, a less complicated function. Therefore, we take y as the first function and as the second
function.

I = { X dx

ow
I II

1 = Y e^dx dx

I =X -j e^dx

e
re
I^xe^-e'^+C

Flr
l=ix-l)

F
(ii) Let / = f dx. Then,
I II
ou
sr
/ X f c dx ■ T(-v^)x dx dx

fo
dx

=> 2.V dx = x^ - 2 ●'f YI e"'II dx


k
oo

I = x^e^-1 X
I -_ ■ (y) X I dx ■ dx
Y
reB

I = x^ -2 \x e-'^- dx = Y
,.2 eT
-2(xe^~e^)+C
uY

(iii) Let I = X log Y dx. Then,


II I
ad
do

! = log Y ● Y dx
|^(log.r)x j Yrf.vj dx
in
Re

x^ , X- 1
F

/ = (log — dx - — loe Y — Y dx
2 ^ 2J
( ■i\
[ = 1 £2 + C =
Y^ 1 2
Y "2 — log Y - - Y + C
2 2* 4
EX.AMri.h: Evaluate:

(i) log Y dx
(ii) I (log y)^ dx
SOLUTION (i) Let / = log Y ● 1 dx
I II

I = log Y ●
dx
(log y) ● f 1 dx I dx
1
I = (log y) y - -Y ● Y dx = x (log y) - 1 ● dx = X (log .y) - Y + C
13.53
INDEFINITE INTEGRALS

(ii) Let I = (log ● II1 dx. Then,


I

J = (log xf ■ 1 ■ dx ■ -
[ dx A(iogx)^-J 1-rfxj dx
/ = (log x)^ X - J 2 log X ■ ■ X t/x = X (log xf - 2 log X ● 1 dx
I
II

I = x(logx)^ -2 (logx) ■ 1 ● dx — (log x) 1 ● frx I dx


dx

/ = x(logx)^ 2|(log,v).v-J Lid-vj = X (log x)^ - 2 (X log X - x) + C


EXAMl’l I ' Evaluate:

'Y * (ii) f log(x^-l)rfx

w
(i)
X

SOLUTION (i) Let I =

1
logx
X
Y~ dx. Then,

F lo 1 1

e
dx
- log X dx

Fre
I =

●' X X
X I V
II for
I - log X + f x~^ dx = - - log X - - + C = - - (1 + log x) + C
r
log (x^ -1) dx
You

(ii) Let I =
oks

1
log (x^ -1) ■ 1 dx = X log (x^ -1) - ● 2x X dx
eBo

=> I
I
II X 2-1
2 X 2 -1 + 1
dx = X log(x2-l) -2
X
dx
I = X log (x2 -1) - 2 ^
ad
our

X 2-1
X 2-1
x-1
1 = X log (x2 -1) - 2 f 1 + X —-
-1
dx = T log (x2
-l)-2 ■ x + ^log x +1
^ +c
Re
dY

x-1
= X log (x2 -1) - 2x - log + C
Fin

I
x +1

^ Evaluate:
3 X

(i) X log (1 + x) dx (ii) x^log2xdx (ii) X e dx

SOLUTION (i) Let I =


●’
XII log (x + 1) frx. Then,
I
2 2
1
— frx
I = log (x + 1) x + 1
X

2
2 2 -1+1
^log(x + l)-|
X

y log(x + l)-^
X X
rfx = rfx
I = x + 1
x + 1

x2 1 x2-l 1
rfx
y l0g(X + l)--
+
/
x+1 x+1
13.54
APPLIED MATHEMATICS'XII

2
1 1
y Iog(.r + 1) - 2 I +
I dx
x + l

2
X
— loe(.t + 1) — i — - a: + log I a: +11 j- 4- C
2 ^ 2,2
(ii) Let / = I log 2.V dx. Then,
1 x‘^ 4 4 .4
^log 2.V-14

ow
/ = (log 2x) — - _ X 2 X — fr.v =
dx = — log 2a: - — + C
4 2a: 4 4 4 16
(ill) Let / = x^I II
dx. Then,
r 3 X
I = X e 3x^ e'^ dx = a:^ x^ dx

e
I II

re
I = e"" - 3 \x^ 2.y dx ■ = X
3
e
X
-3\ -2 X e ' dx
I II

Frl
F
I = x^ e"" ~ 3 x^/'-2
{xe"-jl..-^rf.r}
ou
/ = A:^e"^-3<!x^^-'' x^

sor
2 xe^ -e + C
V

^ e’^ - 3x^ + 6x -6e^ +C = (x^ - 3x^ + 6x -6)e^ +C


I = X
kf
oo
EXAMPLES Evaluate: log (1 + x^) dx
3
Y

X
B

SOLUTION Let 1 = r iog(i + .T^) rfx. Then,


3
re

X
oY

1
1 = I log(l + x^)x"'^ dx = ~ 2x^ log (1 + x^) + ■
X dx
u

I II x^ (1 + a-2)
ad

=> / =
—K ^og (1 + X^) + 1 r dt 2
d

where t = x
2x^ 2 J /(l+O'
in

Kr log (1 + X^) + i2 1 ]_
Re

I = - 1 2 1
dt = -
2x^ t t + 1 -^log(l + x ) + -'^logt-log(t + l) +c
F

X 2

2
/ =
^ log(l + A-2) + i2
2x^
log
X
+ C
x^ +1

EXAMPLE 6 Evaluate: | + 1 I log (x^ + 1) - 2 log x| 4


dx
X

SOLUTION Let

/ =
I ^|x^ +'i I log (x^ + 1) - 2 log x|4
dx
X

= f
I
X
4 ■ log (x^ +1) - log x^ ● dx
13.55
INDEFINITE INTEGRALS .

f T~ f 1 1 1
I
■’ ' X -Y J X

Let 1 + i- = f. Then, d f I + —^1 ^ = dt - ^dx


C
= dt X'
dx = idt
^
X ^ V X ^ X

/ - -Jt log t dt
2*’ II I

}=-ij|(,ogO<
1 2 3/2 _lf 3/2 4,3/2 + C
dt
I (log 0 t 3J f 9
2 3
3/2
2 1 L 1 1 2
4 t,3/2 +C.
/ = -
3
logf 3
+c =- 3 — In—
r2 log 1 + - -3 Y

EXERCISE 13.16

Evaluate the following integrals:


3. I Y^ e ^ dx

F low
2.V
1. xe^ dx 2. xe"' dx

5.
log (log Y) dx b. y” log Y dx
4. Y log Y frY Y

/.
logY
n
dx 8.
<●
2y-^
3
dx 9. (log y)^ y dx
X

log (y + 2)
re
for F logio Y dx
I ix + 2)^
V-V n. dx 12.
10. dx

logY 14. (y +1) log (xe^) dx 15. (Y + l)l0gY(fY


13.
2 Clx
(Y + 1)
ANSWERS
Your
s
eBook

Y 1 'l e lx +, ^
C . -r^'(Y^+2Y + 2) + C
1. (Y-1)/ + c 2 4
M + 1 n + 1
4 4
ad

Y
Y , Y
5. log Y [log (log .y) -11 + C 6. -JI log Y - 2-C
our

4. log Y + c
+ 1 {n +1)
4 ® 16

y |(log y)^ - log Y + ^l


1- II \-n .2
7. ^ logY-
Y
+ C 8./ (Y^-l)+C 9. + C
Re

1 -n (1-M)^
Y

1 log(Y + 2) + C
Find

10. H. -
(y+2) (y+2)

12.
log 10
1

|,-(log,-l)}+C 13. -^ + log +C 14. xe^


I log (yc^) -l| +c
2^ 2'\
V , X ^
15. X + log Y - Y + —-
4
+C
2

13.10.1 INTEGRALS OF THE FORM

THEOREM Prove that:


e"" |/(^) + /' (■■^■)} ^
rKOOi- We have.

lf{x) + f{x)\dx = f{x) dx + fix) dx


II 1
13.56
APPLIED MATHEMATICS-XII

J + /' (.y)1 dx = f{x) ■ - f' {x) dx + J e'^ f (x) dx + C


' e^{f(x)+fix)] dx = e^f(x) + C Q.Ii.D.

This theorem suggests the following algorithm to evaluate integrals of the form
I e"" j/(.T) +/'(x) I dx.
ALGORITH'-

Express the integral as the sum of two integrals, one consisting off (x) and other containing
fix).
i.e.
s'" ● fix) +/' (x)| dx = J e-f (x) dx + I (y) dx
II Evaluate the first integral on RHS usmg integration by parts bij taking as the second
I f.i'

w
function. The second integral on RHS zuill cancel out from the second term obtained bu
evaluating the first integral.

F lo
kx
The above theorem is also true if we have e in place of e^.
I.e.
j/: / (y) + /' (y)I dx = e^'" f{x) + C

ee
Fr
ILLUSTRATIVE EXAMPLES

● Evaluate:
for
ur
X, 1 1 Y
(i) e (ii) e
X
—2 jdx
s
X
ix + l)
ook
Yo

y-3
logY
(iii) I e'^ ^dx (iv) 2 dx
eB

(Y-1) (1 + log y)

Y 1 1
r

SOLUTION (i) Let / = ^ ~ dx. Then, I =


ou
ad

X x^ r
1 f
Y

Y 1
/ = e
- dx -1 e^ ^ dx
■1 ^
Re
nd

II
Fi

1 1
=> dx-i
Y
X
●-yrfY
Y
+C
1
\-e^dx-\ +C =
I e
Y
+
1
- +C
Y
Y Y Y

Y Y
(ii) Let / = e ●

(-T + 1)
Y Y + 1-1
jdx
(Y + 1)

1 -1
+ dx
Y + 1
(x + lf
/ /'
13.57
INDEFINITE INTEGRALS

■t 1 (-1)
I e dx + f ^dx
II x + l (x + l)
I
-1 .t (-1) 1
1 /+C
/ = e j dx + e ^dx + C = x + l
.v + 1 (X + 1)“ (x + l)
x-3
(iii) Let / = j / (x-1)
- dx. Then,

_xix-l)-2 X 1 (-2) dx
I = dx = e
^ 3
(x-1)^ (.v-1)^ (x-1)^

w
/ /'
1 -2
dx + j X (x-1)^
X
I e X
dx
II
(X-1)^

lo
I

e
1 -2 -2
X
— xe^dx+ X jdx + C

re
I
ix-lf' (x-1) (x-1)
rF
F
X

/ -J-—dx + C
(x-1)^ ^ (x-1) {x-iy

r
fo
u
X

=> I =
^^+C
ks
(X-1)^
Yo

logx
oo

(iv) 1 = dx. Let log X = f.Then, .X = e ^ dx = d{e) = e dt


(1 + log x)
B

I
t e 0 + iM / dt
e

1
^dt = (t + i)^
(f + 1)
ur
ad
Yo

1 -1 t
I ■ e dt
f + 1
(t + lf
d

/
f
Re
in

1 t,,
I — / it + ■'f -^e dt
F

f+1 (f + 1)-
I II
-1
1 =
1
e
t
dt + - eUt+C
f + 1
(f + 1) (^ + l)
t
X
I = — + C = + C
f + 1 (log x + l)
CXAMiM.r:: Evaluate:
1
1 1 ● dx
(i) ● ix (ii) ■ log (log x) +
logx (logx)^ (log X)^
1
^ I ix. Putting log^x=f or x = e and dx=e^ dt,
SOLUTION (i) Let 1 =
^ 1 logx (logx)^
we obtain
13.58
APPLIED MATHEMATICS-XII

1 . (-1) i t 1
I edt = e^-dt-
e ^dt
-+
t t t
f f II I

1 t t 1
I - e -irx e dt - e X dt + C
t r t
1 f
I =
t
e‘ +C
7^
log .V + c
1
(ii) Let / = J ■ log (log "a) +
■ dx. Let log .V = f. Then, a = => dx = d (c‘) = e dt.
(log a)^

= J logf + i
t
t e dt
t^ I

1 1 t
I ● log t + - “H e dt
t t 1^

I
/ =

=
log t + -
/

e log f frt + f e' --dt +


II I
t
f

1
●'
dt 4-

(
I
1
-t + 4 e dt
f r

eII i-l/t)
I
F low
dt+\^ e‘\dt
r-n
r e
for Fr
e
/ = (log t) e - -■e‘ dt + e --dt +
-j dt + e' —^dt
t t
I f t
+C
I = e .\ogt~~e* +C = X log (log a) ^+C
Your

f log A
s
eBo k

I'XAMPLi '● Evaluate:

(i) e
X x^+1 ,
jdx (ii) ,-v dx
ad

ix + 1)
our

SOLUTION (i) Let i = e


X x^ + 1
2^X
(x + 1)
Re

2a
Y

I = 1- dx =
' ? dx~2 J e ’' -7:2^^
Find

(x + lf (A + 1)
I
= e^-2j +
ix + 1)
2

1 -1
/ = e^-2 e
.Y
< + dx
A + 1
(A + if
/ /'

1
! e ^ -2 e
X
■ dx - e
X

A + 1 -—2
II I (A + 1)
13.59
!NDEr!MlTE INTEGRALS

1 1 1
■Ux-\e^ (-V + 1) jdx
X
I = -2 e
2
.v + 1 (-V + 1)^

^ dx\+C
X f X x
I = e ~2< — ■ e + I t’ ● e ■

x + 1 (T + 1) (.T + 1)

;c 2 e"' + C
/ e
x + 1

ow
X
(ii) Let / = e' dx. Then,
2^2
(1 + t")

I e
,-l-2.v + .r^ dx
/I 2.2

e
(l + .v )

re
=> } e
, (l+x^) + (-2.v) dx
2.2
(1 + -V )

Frl
F
.T 1 (-2.Y)
ou ● dx
I

sor
e
2.2
1 + x^ (1 + x")
f r kf
a: 1 X i-2x) dx
I dx + e
oo
2 2.2
(1 + -v^) (1 + -y")
II I
Y

1 (-2.V) (-2.Y)
B

I =
X

2.2
e^'dx + f 2.2
dx + C
(1 + x ) (1 + .V-)
re

X
oY

e
!
j +C
u

1+x
ad

EXERCISE 13.17
d

Evaluate the follozving integrals:


in

.Y-1
x( 1 2 .V ^ dx 3. e
.X
dx
Re

1. e 2. e 3
U'
2
Y . 2a-2 ; (Y + 1)
F

2-y X 1 +Y ( 1 ' dx
idx 6. log Y + -
4.
2 dx u.

(2 + Y) I yJ
(1-Y)

7.
e^ logY + -y dx
1
8. — \x (log y)^ + 2 log yI^y
Y
9. e
(Y-2)
Y >

ANSWERS

X
X
X
^ /- 2. — + C 3.
1.
-2+C
Y
2y (Y+ir
2

X X

b. e'^ log x + C
e e
4. + c v^- + C
1 -Y y + 2
X

7. e "f logY-- X
+ C 8. (log y)^ + C 9.
(Y-2)^
2
13.60
APPLEED MATHEMATICS'Xll

HINTS TO SELECTED FROBLEF/S

r .x\x-l 1 (-1) 1 1
dx = ^ — + - dx = e
X

rfx + J dx
Ix- 2x 2x^ 2x 2x^
f /’ 11 I

/ = ^ f X 1
dx + e
X 1 1
— e - \ e - dx + C = — +C
2.V J ^ 2 2x
^ 2x

X x-1 (x + l)-2 X 1 -2
jdx = dx = e + [ dx
(x + 1) (-■^ + 1)^ {x + \f ' (x + lf

w
/ /'
1 (-2)
2 dx + jdx

Flo
(-V + 1) (x + 1)
II I

e
re
1 (-2)
X
X
X (-2)
/ =
2 ^ j dx + e ■rdx + C=—^—-+c
{x + l)2

F
(.t + 1) (-t + 1) ^ (x + lf
ur
r
5. 1 = e
X x+2-1
dx = e
X
<
1
+
(-1)
dx
fo
ix + 2f x+2
(x + 2f
ks
f r
Yo
oo

x( . 1
eB

1 1 t X 1
/ = e log X + —
X

x
— +
X
dx = e log X + - dx + e — dx
x

/ /' / /■
ur
ad

\ (log xf + - (log x) dx
Yo

h. 1 =
f ^ f
d
Re

13.11 SOME IMPORTANT INTEGRALS


in

In this section, we will prove three formulae which will be used in evaluating integrals of the
F

form
yjax^+bx + c dx and J {px + q) -Jax'^ +bx + c dx.
IHEOREf/ Prove that:

(i) + x^ dx=ix-^a
2 2 1 2 /~2 T
+x +-a log x + ^fl 4-x +C
(ii)
^jx^^ dx=^Xyj 2 X - a
2 1 2 ,
~~a log,x+yx
I r~2 -a ^ +C
l’KO(
(i) Leti =1 -Ja^+x^ dx. Then,/ Ja^ + x^ 1 ● dx
^ I “
Integrating by parts, we obtain
l~2 2
1 a + X X -
(0 + 2x) X dx
13.61
INDERMlTEI.NTEOaALS

I = X Ja
I 2 +, X 2 X
dx
2 2
a + x

I = X
rr,~+ .r 2 + X^) dx
2 2
a + X

1
7 2
I = xJn + a:
2 a
2
X ^ dx + dx
2 2
(J + X

ow
2
^-I + log I A' + + -V
2
I X ^ U7 + X

=> 21 = X Ja^ + x^ + Cl ^\og\x + y + x^\

e
1 'y l~2
logi A + ^rt + A ^ I + C
1 2 2
- X Ja + X +

re
I =
2 2

^ 1 - dx
(ii) Let / = -Jx^ - dx. Then, / =

Frl
F
-a

l 11

Integrating by parts, we obtain


ou
sor
[-^ 2 1
I A -a -x- {lx)xdx
J 2
2
kf
{ = xJx^
X
dx
oo
2 2
X -a
Y

(a^ - a^) + a^
B

I 2 2 dx
I = A A -a
2 2
X -a
re
oY

1
^dx-a^
u

2 2 2
1 aJa -fl X -a dx
2 2
ad

A -a
d

^ -I -a^ logl A + -^
2 2
-a
[ = xjx -a
in

^ log I A + -J?
Re

2 2 2
21 X X -a -a - a
F

/
1
- A A 2 -2_l,2 log I A + -j? - 1 + C
-a
2 2

13.11.1 INTEGRALS OF THE FORM j v


In order to evaluate the above type of integrals, we use the following algorithm.
ALGORITHM
,■ 2 c
2
STFl' I Make coefficient ofx as one Inj taking 'a' common to obtain x + -a a + -.
a

b f in. 2 b c , . b ^ Aac-b^
● 'iPii Add and subtract — X + - A + - to obtain a + -— +

2a a a 2a 4a^ ■

After applying these tzvo steps the integral reduces to one of the folloiving three forms:
^A^ - a^ dx.
2
a + X ^ dx,
13.62
APPLIED MATHEMATICS-XII

STEP m
Use the appropriate formula.
ILLUSTRATIVE EXAMPLES
EXAMPLE 1 Evaluate:

(i) + 9 dx
(ii) ■Jx^ + 2x + 5 dx
SOLUTION (i) Let / = J ^4x^ + 9 r^xThen,
I = 2 + - dx = 2 x
2
+
31^ dx
4 I2

I = T2-i-.rJx
1 I 2 +-+-
9 1 I'sf log x+lx +-
2 9
't+C

w
2 4 2 2 4

=> 1
^ + 9 + ^ log I 2.y + ^x^ + 9 I + C
(ii) / = .v^ + 2x + 5 dx

Flo
ee
Fr
2
/ X
^(x +1)^ + 2^ dx
1 ~ ~{x + l) + 1)^ + 2^ + i (2)^ log (x +1) + ^x + 1)^ + 2^ + C for
ur
1
/ = -
(x + l)^x^ + 2x+5+2Iog (.Y + 1) + + 2x + 5 +C
s
2
ok
Yo

/TT7
Bo

IXAMPI.E 2 Evaluate: X dx
V.Y-1
re

1 + .Y
SOLUTION Let / = .Y dx. Then,
.Y-1
ou
ad

2
I =
x (1 + x) X X
dx =
Y

dx + dx
-1 -1
nd

(.Y^-l) + l
Re

1 2x
1= -
dx + j dx
V?
2 J
Fi

-1

I = V? 1 + f V? -1 dx +
1
dx
4x^-1
/=V7 -1 +
^xVx^-1 +ilog x +4x^-1 +log x + 4?? +C
1=4? -1 + — X
1
-1 + log X + y[? -1 +C

EXERCISE 13.18
Evaluate the following integrals:
1.
4?4x4l dx 2. e
X
e
2x
+1 dx 3.
j ^16x^ + 25i?x
4.
^4x^ -5 dx 7 J ^x^ + 3x + 4 dx 6.
X 4?44 dx
13.63
INDEFINITE INTEGRALS

7.
Jl6 + (log -v)^ dx 8. ■\jx^ -2x dx
X
ANSWERS

Jx^ + X +1 + - log (2.\- +1) + -Jx^ + -V +1 + c


2x +1
1.
4 * 8

1 .V 2.Y
+ l +C
2. -e e +1 +
2

. 2 25 25 , (2 . 25 ^ r
3. 2x. .V + -7 + — log .v + J.v +— +C.
M 16 8 ^ \ 16

4. .V
2 55 5,log a: + j.\Q -75 ^+C^
4 4^ V 4
4x + 3 23^/2 , f 3 ^+lx+2 +C
^2x^ + 3x + 4 +

w
5. log \x + - +
2
32

6. -4

8.
V
+ 74 tog

7. ^ log X ^(log x)^ +16 + 8 log I log X + ^(iog x) ^+16|+C


i(.v-l)V? -2x- — log (x-1) +-yx^ ^2.v +C
+

F lo
+C

for F
ree
13.11.2 INTEGRALS OF THE FORM j 0>.v ; q) * l-.v + r ,1v
Your

In order to evaluate this type of integrals, we use the following algorithm.


oks
eBo

ALGORITHM

STEP I Express px + q as
— (flx^ + bx + c) + p i.e. px + q = X{2ax + /’) + p
ad

px + q = X
our

dx

STEP 11 Obtain the values ofX and p Inj equating the coejficients of x and constant terms on both sides.
STEP IH Replace px + qhy X{2ax + b) + p in the integral to obtain
Re
dY

(px + q) ^ax^ +bx + c dx = X {2ax + b) ■J ax ^ + bx + c dx + p .^ax^ +bx + c dx


Fin

n+ 1
1/Wl
STEF IV To evaluate first integral on RHS, use the formula |(/ (x))" /' (x) dx » + l

Evaluate second integral on RHS by the method discussed in the previous section.

ILLUSTRATIVE EXAMPLES

EXAMPLE 1 Evaluate:

(i) I (x -5) tJx^ 4- a: dx (ii) j (3x-2)-J?+x + l dx


SOLUTION Let (x -5)=X~
dx
(x^ + x) + p i.e. x -5 = X (2.y + 1) + p
Comparing coefficients of like powers of x, we get
13.64
i^PPLlEDMATHEMATICS-XII

1 11
1 = 2 X and = -5=>>. = and n = -
2 2

I {x -5) +x dx
f 1
yjx^ + dx
11
I {2x + l)-~
2 2 J
I x^ + X dx - — + X dx

\! (2-V + l)^ -T
2 + X dx
j 11
2 ^jx^ + X dx

ow
12 Jf VFrf/-li2 Jf X + — dx, where t = x +x
2j 2)
3/2
/
1 t 11 if .V l] f
1

e
- X - + - a: + -
2 3/2 2 2 1} I 2

re
Frl
F
1 1 {

log A+ - + a: + — + C
2U 2; 2/ 2
ou
j _ 2. ^3/2 11 2a- + 1 f 1
or /A* 2
=> 2

-glog
3
a: + A-
kfs + X > + C
2 4
oo

11 2a- + 1 .2 1
=> / A- + X -
2
X + - X + X
3 2 4
Y

2/
B

(ii) Let3x-2=>. ^ (at^ + x +1) + i.e. 3x - 2 = /.(2x + l) + ^


re

Comparing the coefficients of like powers of x, we get


oYu

2 X. = 3 and X + p =-2=^X. = — and ii = - ^


ad

2 2

(3x -2) ^x^ + x + 1 dx


d

7
in

3 7
Re

I = -(2a
2
+ 1)-42 + A + 1 dx
F

3
(2a+ 1)^ ^A^ + A + 1 dx
7
/ dx —
2 2 J

3 if r Vsf
7
j A + — + dx, where f = x + x + 1
2
1 2j 2

=> 7 = t
3/2 7
X + —
11
A^ + A + 1 + ^/3f log X + i +A + 1 ^ +c
4 2. 2

n
7 =(a2 + ah-1)3/2 22 A + -

2J aP"+a + 1 +|log x + ^+^X^ + A + I ^ +c


A
' \ s Evaluate: dx
A-V? -I
13.65
INDerlNlTElNTEG.RALS

SOLUTION Let,
x +
X
dx = .V dx
I = ^ I'
X- X-
) \

.V -t +

-dx = j .v^ + x-\~^ dx = dx + X -1 dx

I = -
X' dx + — f Ji dt, where t = .r;2-l
2J
x^
— + -(x -1)
1/2 IV3/2 + C

w
3 3 3 3
EXERCISE 13.19

Flo
Evaluate the following integrals:
i. f (.Y +1) X +1 dx
7
(X +1) yflx^ + 3 dx

ee
4. j (Y + 2) -J? + Y + 1 frY

Fr
3. I {y - 3) V?+ 3y-18 dx
(4.V + 1) j? -X -2 dx 6.
' (y - 2) -px^ -6x + 5 dx
for
ur
7. (x +1) + Y + 1 dx 8. f (2y + 3) y? + 4y + 3 frY
s

(2y-5) -Jy^ -4y+ 3 dx |y ^ + X dx


k

iO.
Yo

9.
oo

answers
eB

3/2
(y^ -Y + 1) + -(2y-1) Jx^-x-^ + :^log X- 2J Jy^ + y + 1 +C
1
+

3 8 * io V
r
ou
ad

3s/2 f2x + -Jlx^ + 3


^(2x^ + 3)
3/2 + C
log
V3
Y

6 4
Re
nd

3^1
^y^+3x-18 +C
243
3. i(Y^ + 3x-18)
3/2
j(2x+3)f
4 ''
3X-18 +
8
log Y + —
2
+
Fi

4. 1(y^
3
+ x + 1) 3/2 +
3 (2x +
^^Y^ + X + l + ^lOg I X + —
2
+
.Jx^ + X + 1 +C
J.
4 , 2 -x-2) 3/2 + -(2y-1)y/x^Tx-2-^
4 8
log
27
X—
2j
+
^Y^ -X -2 +C
6. —6 (2x^ - 6x + 5)
3/2 1 j 2y - 3 5
^-3x + - 1 ,
+ - log
2x - 3
+ . X 2-3y + ^ > + C

”V2 I 4 ^
X
2 8^ 2 \ 2

1 f 2x + 1 Y^ + X + 1 +
7. 13 (y^ + X +1) 3/2 + —
2 4 |iogU+iW Y^ + X + 1 I +C

8.
I (Y^ + 4y + 3)
3/2
■ -2 (y + 2) Jx^
'
+4.X+ 3 ilog (x + 2)+-j?+4Y+3 .+0
13.66
APPLIED MATHEMATICS-XII

3/2
- 4.V + 3) - 4.Y + 3 -
^log x~2 + ^ -4.V+ 3 +C

50..1IJ?
10. 1 1 2
+ a: + — log .V + — + X + X + C
16 2;

13.12 INTEGRATION OF RATIONAL ALGEBRAIC FUNCTION' r PARTIAL


FRACTIONS
fix)
PARTIAL FRACTIONS If /(a:) and a:) are hvo poly)ioinials, then defines a rational algebraic
function or a rational function of x. g(x)
fix)
If degree of f{x) < degree of ^(a:), then is called a proper rational function.
g(A-)
fix)
If degree of f(x) > degree of g(Ar), then is called an improper rational function.
^(-v)
/(-V)

w
If
gix) is an improper rational function, we divide f{x) by g(.r)so that the rational function IS

gix)

F lo
V (^)
expressed in the form (})(Ar) + where ^ (at) and (.v) are polynomials such that the degree of
gix)
fix) .
V(/ (at) is less than that of g{x). Thus ' ^^pressible as the sum of a polynomial and a proper

e
Fre
rational function. for
fix)
Any proper rational function can be expressed as the sum of rational functions, each having
a simple factor of ^(.v). Each such fraction is called
3 partial fraction and the process of obtaining
r
You

them is called the resolution or decomposition of - ’' into partial fractions.


oks

gix)
eBo

The resolution of fix)


gix)
into partial fractions depends mainly upon the nature of the factors of ^(.r)
as discussed below.
ad
our

^ ■'.( I When denominator is expressible as the product of non-repeating linear factors.


Let^(A:) =(a‘ - a-^) {x - 02) (a* Then, we assume that
fix) A1 Ai A
Re

+
dY

+ ... +
c?(-v) X -aj X -U2 x-a,
Fin

where A■^, A2A„ are constants and can be determined by equating the numerator on RHS to the
numerator on LHS after taking LCM on RHS and then substituting a'
3.V + 2
II11 ● r.A'i It Resolve —
into partial fractions.
A -6x^ + 11A--6
3a- + 2 3a + 2
SOLUTION We have.
a^-6a^+11.y-6 (A-1) (A-2) (A-3)
3a + 2 A B C
Let
. Then,
(a--1)(a-2) (a-3) -1 A - 2 A - 3
3a + 2
_ ^ A(a-2)(a-3) + B(a-1) (A-3)+CfA-ll (x-2)
(a-1)(a-2) (a-3) (^1)(a-2)(a-3)
3a + 2
= A (A-2) (A-3) + B(a-1) (a- 3) + C (a-1) (a-2) ...(i)
Putting A -1 == 0 or, a = I in (i), we get 5 = A (I - 2) (I - 3) A = -
2
INDEFINITE INTEGRALS 13.67

Putting a: - 2 = 0 or, X = 2 in (i), we obtain 8 = B (2 -1) (2 - 3) => B = - 8


11
Putting X - 3 = 0 or, X = 3 in (i), we obtain 11=C(3-1)(3-2)=>C=
^
3x + 2 3x + 2 5 8 11

X ^ -6x^ +11X-6 (x-1) (x-2) (x-3) 2(x-l) x-2 2(x-3)

RLMAHi: In order to determine the values of constants in the numerator of the partial fraction

w
corresponding to the non-repeated linearfactor px + q in the denominator of a rational expression, we may
proceed as follows:
Replace X = - q/p (obtained by putting px + q = Q) everywhere in the given rational expression except in
the factor px + q itsel.f For example, in the above illustration the value of A is obtained by replacing x by 1

e
3x+ 2
in all factors of except (x -1).

ro
re
(x-1) (x-2) (x-3)
3x1 + 2 5
i.e. ^ =

F
(1-2) (1-3) 2

Fl
3x + 2
Similarly, B is obtained by putting x = 2 in all factors of

u (x-l)(x-2)(x-2)
except (x-2) in

sr
denominator.

ko
3x2 + 1
o
B = = -8
(2-2) (2-3)
3x + 2
of
To find C, we put x = 3 in all factors of except (x - 3) in denominator.
(x-1) (x-2) (x-3)
o
Y
3x3 + 2 11
erB

C =
(3-l)(3-2) 2
uY

^ - 6 x^ +10 X - 2
ILLUSTRATION 2 Resolve — into partial fractions.
x^ -5x + 6
ad
do

SOLUTION Here, the given function is an improper rational function. On dividing, we get
X ^-6x^+10x-2 (-X + 4)
— X — 1 + —2
in

X ^ -5 X + 6 X -5 X + 6
Re

- X + 4 -x + 4
Now,
F

X ^ -5 X + 6 (x-2) (x-3)
-x + 4 A 6
So, let +

(x-2) (x-3) x-2 x-3

-x + 4 = A (x - 3) + 6 (x - 2) ...(ii)
Putting X - 3 = 0 or, X = 3 in (ii), we get
1=B(1)=>B-1.
Putting X - 2 = 0 or, X = 2 in (ii), we get
2=A(2-3)=>A=-2
-x + 4 -2 1
+

(x-2) (x-3) x-2 x-3

-6x^ +10X-2
Hence, - = x-l-^ +
1

x^-5x + 6 x-2 x-3


13.68 APPLIED MATHEMATICS-XII

CASE n
When the denominator g{x) is expressible as the product of the linear factors such that some of
them are repeating.
Let g{x) = (a: - a)^ {x - a{) (a: - ^2) ● ● ● )● assume that
fix) A1 +
A2 + +... + +
B1
?2 +... +
Br
six) x-a
(x-a)'^ (x-a)^ X -a
X-U2 (X-Or)

i.e., corresponding to non-repeating factors we assume as in Case I and for each repeating factor
(at “ rt) , we assume partial fractions
A1 A2
+ +... + , where.Aj, A2, A^ are constants.
ix-a)^
2
x~a
(x-a) ix-a)
Now, to determine constants we equate numerators on both sides. Some of the constants are

w
determined by substitution as in case I and remaining are obtained by comparing coefficients of
equal powers of a: on both sides.

F lo
Following illustration illustrates the procedure.
3.r-2
ILLUSTRATIONS Resolve into partial fractions.
{x-lf (,v + l)(jr + 2)

ee
Fr
SOLUTION Let
3x-2 A1
+
Ai ^3 I ^4
{x-lf{x + l)(x + 2) x-1
(x-l)2 a: +1 X + 2

3.V - 2 = {x -1) {x + 1) (X + 2) + /I2 (X +1) (X + 2)


for
ur
+ /43(x—1) (x + 2) + (x — 1) (x + 1)
s
ook

Putting x-1 = 0 or, X = 1 in (i), we get


Yo

1 = A2 (1 + 1) (1 + 2) => A2 = -6
eB

Putting x + 1 = 0 or, X = -1 in (i), we get


-5 = A3(-2f(-l + 2)^A3 =
r
ou
ad

Putting X + 2 = 0 or, X = - 2 in (i), we get

Ai{-3f{-l)^A^ = ^
Y

-8 =
Re
nd

Now, equating coefficient of x on both sides, we get


8 ^ 13
Fi

0 = A| + Ag + A^ ^ A| = — A 2 — A4 = — 9 " 36
3x-2 13 1 5
+ +

fx-l)^x + l)(x+2) 36(x-l) 6(x-1)^ 4(x + l) 9(x + 2)


CASE HI
When some of the factors of denominator g{x) arc quadratic but non-repeating.
2
Corresponding to each quadratic factor ax +bx + c, we assume partial fraction of the type
Ax + B
2
, where A and B are constants to be determined by comparing coefficients of similar
ax + bx + c

powers of X in the numerator of both sides. In practice it is advisable to assume partial fractions
A (2flx + b) B
of the t^^pe 2
+
2
ax + bx + c ax + bx + c

Following illustration illustrates the procedure.


13.69
INDEFINITE INTEGRALS

2.T-1
ILLUSTRATION 4 Resolve into partial fractions.
(x + l){x^+2)
2x-l A Bx+C
SOLUTION Let + . Then,
(x + l)(x^+2) x + 1

2x-l A {x^ + 2) + {Bx + C) (at + 1)


(X + 1) (x^ + 2) (x + 1) (x^ + 2)
2x-l = A(x^ + 2)+(Bx + C)(x + l)

ow
Putting x + l= 0or, x = -lin (i), we get
-3 = A{3}=> A=-l.
Comparing coefficients of like powers of x on both sides of (i), we get
A + B = 0, C + 2A=-1 and C + B = 2
-1 + B = 0, C - 2 = -1 [Putting A = -1]

e
B = 1, C = 1.

re
2x-l 1 x + 1

Frl
+

{x + l)(x'^ + 2) x^ +2

F
x + 1

CAST IV When some of the factors of the denominator g (x) are quadratic and repeating.
ou
or
bx + c)^, we assume 2 k partial fractions of
2
For every quadratic repeating factor of the type {ax +
kfs
the form

Aq {2ax + b) A1 Ai {2ax + b) A2
● + *1
oo
+ -
2 2
ax
2
+bx + c ax^ +bx + c {ax^ + bx + c)^ {ax + bx + c)
Y
B

^2k-l (2flx + b) ^2k


+ ... + ● +

{ax^ +bx + cf (flx^ + bx + c)^


re

Following illustrations will illustrate the procedure.


oYu
ad

2x-3
ILLUSTRATION 5 Resolve - into partial fractions.
(x-l)(i-^+l)
d

2x- 3 A Bx + C Dx + £
2 ● Then,
in

SOLUTION Let
Re

(x-1) (x^+1)^ x-1 x^+1 (x^+1)'


F

2x “ 3 = A (x^ + 1)^ + (Bx + C) (x -1) (x^ + 1) + (Dx + E) (x-1)


Putting X = 1 in (i), we get

-1=A(1 + 1)^=> A=-^


Equating coefficients of like powers of x, we get
A + B = 0, C-B=0, 2A + B-C+D=0, C+E-B-D=2and A-C-£ = -3

Putting ^ ^ solving these equations, we get


B = — = C, D = — and £ = -
4 2 2
2x-3 -1 x + 1 x +5
+

4(x-l) 4(x^+l) 2(x^+l)^


2x
ILLUSTRATION 6
Resolve —^ into partial fractions.
X"^ -1
13.70 APPLIED MATHEMATiCS-XIJ

lx lx
SOLUTION We have.
(x-1) (x^ + x + l)
2.V A Bx + C
So, let + . Then,
(a:^ + a: + 1) x-1 X + X + ,1 2

lx = A(a'^ + 3; + 1)+(Bx + C)(x-1}


Putting X -1 = 0 or, X = 1 in (i), we get
2 =3A^ A = -
3
Putting X = 0 in (i), we get
1
A -C = 0=> C = A
3
Putting X = -1 in (i), we get
- 2 = A + 1B-1C=> - 1 ~-IB
3 3 3
lx 2 1 113 x +1/3 lx 1 1 2 x + 1

w
+ ■ or. + — ●
3 x-1 X^ + X + 1 x^-l 3 x-1 3 x^ + X +1

functions.

ILLUSTRATIVE EXAMPLES

F lo
We shall now use partial fractions in evaluating integrals containing rational algebraic

e
Fre
Type 1 WHEN THE DENOMINATOR IS EXPRESSIBLE AS A PRODUCT OF DISTINCT LINEAR FACTORS

rXAMIMJ: 1 Evaluate:
for
x-1 3
2x-l X
(i) dx (ii) dx (iii) dx
J (x + l)(x-2) (x-1) (x+2) (x-3) J (x-l)(x-2)
r
You

x-1 A B
oks

SOLUTION (i) Let ...(i)


(x + l)(x-2) x+1 x-2
eBo

=> x-1 = A(x-2) + B{x + 1) ...(ii)


Putting X - 2 = 0 or, X = 2 in (ii), we get
ad

1 =3B B=l/3
our

Putting X + 1 = 0 or, x = -1 in (ii), we get


-2 = - 3A=:> A =2/3
dY
Re

Substituting the values of A and B in (i), we get


x-1 2 1 1 1

(x + 1) (x-2) 3’x + l"^3'x-2


Fin

x-1
I dx
(x + 1) (x-2)
2 f 1 . 1 1
/
3 J X +1 3^ x-1
dx =
|log|x + l| + ilog|x-2|+C
2x-l A B C
(ii) Let + +
...(i)
(x-1) (x + 2) (x-3) x-1 x+2 x-3

2x -1 = A (x + 2) (x - 3) + B (x -1) (x - 3) + C (x -1) (x + 2) ...(ii)


Putting X + 2 = 0 or, X = - 2 in (ii), we get
-5 = B(- 3) (-5)=> 6=-l/3
Putting X - 3 = 0 or, X = 3 in (ii), we get
5=C(2)(5) => C=l/2
INDEFINITE INTEGRALS 13.71

Putting A' -1 = 0 or, T = 1 in (ii), we get


l=A{3) (-2)^ A=-l/6
Substituting the values of A, B and C in (i), we obtain
2.V -1
1 _3__1 ^ i 1
(x-1) (x + 2) (x-3) 6 x-l s'x + 2 2 .r-3
2x-\
I dx
(.v-1) (:r + 2) (.x-3)
I = -
1 ^ dx
— ^ —If — dx + -
1
dx
6-’x-l 3 ●’x + 2 2 J .T - 3

I
~ log 1X -1| - ^ log IX + 2| + ^ log IX - 3| + C
6 3 2

(iii) Here, the degree of numerator is greater than that of denominator. So, we divide the
numerator by denominator to obtain

w
3
X 7x-6
= X + 3 + ...(i)

F lo
(x-1) (x-2) (x-1) (x-2)
7x-6 A B
Now, let + ...(ii)
(x-1) (x-2) x-1 x-2

e
Fre
=> 7x-6 = A(x-2) + 6(x-l) ...(iii)

Putting X - 2 = 0 or, X = 2 in (iii), we get: B = 8


for
Putting X -1 = 0 or, X = 1 in (iii), we get: A = -1
Substituting the values of A and B in (ii), we obtain
r
7x-6 1
You
oks

(x-1) (x-2) x-1 x-2


eBo

3
X 1 8
= X + 3 - [From (i)j
(x-1) (x-2) x-1 x-2
ad
our

3
X
I dx
(x-1) (x-2)
2
1 8
^ + 3x -log|x -1| + 8 logjx -2| +C
dY
Re

I = X + 3 - + dx = -
x-1 x-2
Fin

2x
EXAMPLE 2 Evaluate: dx
(x^ + 1) (x^ + 2)
2x
SOLUTION Let 1 = dx. Putting X = t and 2x dx = dt, we get
(.v^+l) (x^ + 2)
dt
I =
(t + 1) (f + 2)
1 A 6
Let + ...(i)
(t + 1) (f + 2) f+1 f + 2
1 = A (f + 2) + B (f + 1) ...(ii)
Putting f = - 2 in (ii), we obtain: B = -1
Putting f = -1 in (ii), we obtain: A = 1.
Substituting the values of A and B in (i), we get
13.72 APPLIED MATHEMATICS-XII

1 1 1

{t + l){t + 2) t +1 t+2
1
I = dt
(i + 1) (t + 2)
1 1
I = dt
t+1 t+2

I = log I f +11 - log 11 + 21 + C = log I +11 - log I +2[+C


EXAMPLE 3 Evaluate:
ix-l){x-2) (x-3) dx.
J (x-4)(x-5)(x-6)
SOLUTION Let
{x-l){x-2) {x-3) = 1 + A B C
+
(x-4) (x-5)(x-6) (x-4) x-5 x-6
Then, (x-1) (x-2) (x - 3) = (x-4) (x-5) (x-6) + A(x-5) (x - 6)

w
+ B (x - 4) (x - 6) + C (x - 4) (x -5) ...(ii)

F lo
Putting X = 4,5 and 6 successively in (ii), we obtain
A = 3, B = -24 and C = 30
Substituting values of A, B and C in (i), we obtain

ee
(x-1) (x-2) (x-3) ^ 24 30

Fr
(x-4) (x-5) (x-6) x-4 x-5 x-6

I =
(x-l)(x-2)(x-3) dx
for
(x-4) (x-5) (x-6)
r
1 f 1
I = j 1-dx + 3 dx-24 dx + 30
You

x-4 X -5 x-6
s
ook

/ = X + 3 log I X - 41 - 24 log I X -51 + 30 log I X - 61 + C


eB

Type II WHEN DENOMINATOR CONTAINS SOME REPEATING LINEAR FACTORS

EXAMPLE 4 Evaluate:
our

x^ +1
ad

3x + l
(i) dx (ii) dx
{x-2f(x+2) (x-l)2(x+3)
3x + l A B C
dY

SOLUTION (i) Let +


...(i)
Re

(x-2)2(x+2) x-2 (^-2)^ x+ 2


Fin

3x +1 = A (x - 2) (x + 2) + B (x + 2) + C (x - 2)^ ●●●(ii)
Putting X - 2 = 0 i.e. x = 2 in (ii), we get
7 =4B=> B =-
4

Putting X + 2 = 0 i.e. x = - 2 in (ii), we get


-5=16C => C=- —
16

Comparing coefficients of x on both sides of the identity (ii), we get


/I +C = 0=> A = -C=>A = ~
16

Substituting the values of A, B and C in (i), we get


3x + l 5 1 7 1 5

{x-2f{x + 2) 16 x-2
i(x-2f 16 (X + 2)
13.73
INDEFINITE INTEGRALS

3x + \
I = dx
(x-2)^ (x + 2)
1 1
1=^ 1 dx .. + —
7 ^ dx dx
16 ^ ;c - 2 4-' {x-2) 2 16 x + 2

— log 1 X + 21 + C
16

(ii) We have,

ow
x' + l dx
/ =
(x-l)^ (x+3)
A B C

e
+
Let +

(x-1)^ (x+3) x-1


(x--l)^ x+3

re
+ 1 = A (x -1) (x + 3) + B (x + 3) + C (x -1)^ ●●●(ii)

Flr
F
Putting X -1 = 0 i.e. x = 1 in (ii), we get
2 = 4B => B = -
2
ou
sr
Putting X + 3 = 0 i.e. x = - 3 in (ii), we get

fo
10 =16C => C
8

Equating the coefficients of x^ on both sides of the idenUty (ii), we get


k
oo
1 = A+C=> A = 1-C = 1-^ ^
Y
8
reB

Substituting the values of A, 6 and C in (i), we get


x^ + 1 ^
2+|(^+3)
3 1 1 1
uY

(.f-l)^ (x+3) 8 x-1 7 (x-1)


x^+1 1 ^ 5 1
dx
ad
do

=> I = —7 dx + -
8 ●’ X + 3
(x-1)^ (x+3) 8 ●'x-1 ^ (x-1)
1
in

/ =
|log|x-l|- 2(x-l) + — log I X + 31 + C
8
Re
F

X^ + X + 1 dx.
t'XAMPLES Evaluate:
(x-1)^
X^ + X + 1
SOLUTION We have, f = | ^ dx. Putting X -1 = f and dx = dt, we get
(x-1)

I =
(f + l)^+(f + l)+l dt =
+ 3t+ 3
dt=
1
- +
3 3.^*
t^ t r r

' =losm-7-^ + C =log|x-l|-^^


NOTE This sum can also be done by using partial fractions. We write
x^ + X +1 A
+
B
+
C

(x-1)^ x-1
(x-1)^ (x-1)^
13.74
APPLIED MATHEMATICS-XII

2
X
nxAMFM'6 Evaluate: dx
(x-lf(x + l)
SOLUTION We have.
2
X
dx
(x-lf(x + l)
2
X A B C D
Let + + +
x-1
{x-lf ' (x-lf x + 1

x^ = A {x -1)^ (x + 1) + B {x -1) {x + 1) + C (a: + 1) + D (x -1)^ ...(ii)


Putting .V -1 = 0 i.e. a: = 1 in (ii), we get
1=2C => C=-

w
2

Putting a: + 1 = 0 i.e. at = -1 in (ii), we get

F lo
1
1 = -8D => D=-
8

Putting a: = 0 in (iii), we get

ee
0=/l-B + C-D=> A~B = -~

Fr
8

Putting a: = 2 in (ii), we get for


4 = 3/l + 3B+3C+D=> 4 = 3{A + B + C)+D=>A + B = -
ur
8

Now, A - B = - — and A + B = —=>A= — and B = —


s
ook

8 8
Yo

o 4
1 3 1 1
Thus, we have /I = —, B =—,C =- D=~
eB

8 4 2 8

Substituting the values of A, B, C and D in (i), we get


our

1 3 1
ad

1
+ +

{x~lf{x + l) S(x-l) 4(a:-1)^ 2(a:-1)^ 8 (a: + 1)


2
Y

X
I dx
Re
nd

1
^ dx
^ + —3 f 1 1 .4 1 r 1 dx..
I
■y dA: + -2 Jr
Fi

— dx--
8 ●' a: -1
4 ●’ (.v-1) 2 (:^-l) 3 8 J a: + 1
1 1
I =
8
log I a: -1|
^ ' 4(a:-1)
—— ^8 log|A: + 1| + C
4(.r-l)2
1 a:-1 3
! log
a: + 1
4(^-1) 4(a--1)^
Type III THE DENOMINATOR CONTAINS IRREDUCIBLE QUADRATIC FACTORS

IMPORTANT NOTE If a rational function contains only even powers of x in both the numerator and
denominator, then to resolve it into partial fractions, we proceed as follows:
Put X =y in the given rational function.
Resolve the rational function obtained in step 1 into partial fractions.
INDEFINITE INTEGRALS 13.75

SJ liP Ml Replace y by x^.


EXAMlM.h 7 Evaluate:
2
a: +1
(i) dx (ii) dx
{x^ + 1) {x^ + 4) (jr^ + 2)(2.v^+l)
SOLUTION Let x^ = y. Then,
2
.r
y

{x^ +1) {x^ + 4) {y + 1) (y + 4)


y A B
Let +

(y +1) (y + 4) y +1 y + 4
y = A(y + 4) + B(y + l) ●●●(ii)
Putting y = -1 and y = - 4 successively in (ii), we get

w
A = and B = —.
3 3

Substituting the values of A and B in (i), we obtain

(y + 1) (y + 4)
J/
3(y + l)
1 4

F lo
3(y + 4)

ree
2

F
Replacing y by a: , we obtain
2
a: 1
+
4
for
(a-2 + 1)(a-2 4-4) 3(a:^+1) 3{x^ + 4)
r
2
You

1 4 1 f 1 jdx 4 f 1
oks

Y
dx = dx = - - + - dx
● 2 2
(Y^+l) (a-2+4) 3(a:^+1) 3{a:^ + 4) 3 + 1 3 ●' +4
eBo

2. -if A-'
/ \
1
=> I, 1 tan
. -1 x + —
4 X —
1 tan
. -1 A -1
= — + C = — tan " A + — tan + C
3 3 2 3 3 2
our
ad

(ii) Let A^ = y. Then,


A^ +1 y+1
(a^ + 2)(2a^+1) (y + 2)(2y + l)
dY
Re

Let y+1 . /I B
-(i)
Fin

(y + 2)(2y + l) y+2 2y + l
=> y+ 1 = yl(2y + l) + B(y + 2) ...(ii)
Putting y + 2 = 0 i.e. y = - 2 in (ii), we get
1
-1 = 3A^A=-
3

Putting 2y + 1 = 0 i.e. y = - in (ii), we get


3 1
1=B - => B =
2 [2) 3
Substituting the values of A and B in (i), we obtain
y^ + 1 _ ]_ 1 1 1
(y + 2) (2y + l) “ 3'y+2"'3(2y + l)
13.76 APPLIED MATHEMATICS-XII

2
Replacing yhy x , we get
1_ 1 +
1

{/ + 2) {Ix^ + 1) 3 4- 2 3{lx^ +1)


1 1 1 1
I dx = — dx + — dx
{x^ + 2) {Ix^ + 1) 3 x^ + 2 3^ ,(42xf+l
1 1 , -1 a: 1 1 -1 a:
=> I =-
- X -:= tan —p tan ^ {yl2x) + C = tan tan ^ -Jl a: ● + C
3 ^/2 ,V2j 3V2 3^2 V2
EXERCISE 13.20

Evaluate the following integrals:

w
2x + l 1
1
●J (x + l){x-2)
dx
J x(x -2){x- 4)
dx

F lo
3.
X
2
+ X -1
.
3 + 4x-x^
dx 4. dx
2
X + X -6 (x + 2) (x-1)

ee
x^ + 1 2

Fr
X
5. dx 6. dx
X 2-1 (x-l)(x-2)(x-3)

5x x2+l
for
ur
dx 8. dx
{x + l) {x'^-4) x{x^-l)
s

3
ook

2x-3
Yo

‘>■1 (x2-1)(2x+3) dx 10.


(x-l)(x-2)(x-3)
dx
eB

1 2x
n. dx 12. dx
x(x^-l) (x2 + 1) {X^ + 3)
our
ad

1 2x + l
13. dx 14. dx
X log X (2 + log x) (x-2) (x-3)
Y

2
ax +bx + c
Re

15. dx, where a, b, c are distinct.


nd

{x-a){x-b) (x-c)
Fi

1 1
16.

X (x^ + 1)
dx
17. J (x-1) (x + 1) (x + 2)
dx

18.
X
2
5x2
dx 19. dx
(x2 + 4) (x2 + 9) x(x-l) (x + 1)

20.
x2 + 6x - 8 1
dx 21. dx
x^ -4x x(x^ + 8)
1 1
22. dx 23. dx

X j 6 (log x)2 + 7 log X + 2| x(x" + 1)


2
X X +X + 1
24. dx 25. dx
(x2-i;2) (X +1)2 (x+2)
INDEFINITE INTEGRALS 13.77

+ x + l 3x-2
26. dx 27. dx
:c2-l {x^\f- (x+3)
2x + l
dx
28.
2 dx 29.
{x-l)^(x+3)
{x + 2) (x - 3)
2
X X
30. dx 31.
jdx
(x-l)2 (x + 2) ix-l) (x + 1)
X^ +X-1 2x^+7x-3
32. dx
33. I x^ (2x +1) rfx
(x + 1)^ (x + 2)
5x^ + 20x + 6 3x + 5

ow
34. dx 35. dx
x"^ + 2x^ + X x^-x^-x + 1
2x
36.
2
(x^+l)(x^ + 2)
ANSWERS

e
re
1. |loglx + ll + |log|x-2|+C
Fl 1
2. - log
X (x - 4) + C

F
8
{x-2f
ur
3-x-log x+3| + logi x-2j+C 4. - x+ 3 logI X + 2] + 2 log! x-1] + C

r
5. X + log
x-1

x + 1
+ C fo
ks
Yo

6. ^ log I X -11 - 41og 1 X - 21 + ^ log I X - 31 + C


oo

(x + 1)^ (x-2) ^-1


eB

7.|,og (x + 2)^ 8. log ^


+ C + C
ur

9.
I log I X + 11 - ^ log I X -11 - y log I 2x + 31 + C
ad
Yo

10. X + ^ log I X -11 - 8 log |x - 2| + y log I X - 31 + C


d

X "-1
x^+l
1, r
Re
in

11. -
12. + c
x^ + 3
F

7
1 log^ (x-3)
+ C 14. log
13.
2^og 2 + logx (x-2)
5

2
a ^ + ^?b + c flb^ + b^ + c flc + be + c
15. log|x-fll + log|x-b| + log|x-cl + K
(fl-b) (fl-c) (b - fl) {b - c) (c-fl)(c-b)
1 ( 4 1 {x + lf + C
+C
jlog
17.
16. -log
4 x^ +1

18. —tan
2,
5
-IX—+ —tan
2
3^ -IX
5
^
—+ C
3
19. log|x(x^ -1)^ 1 + C
x\x-2) + C 21. -log|x|-—log x^+8
20. log
ix + lf 8 24
13.78 APPLIED MATHEMATICS-XII

>i

22. log [ 2 log .r +11 - log I 3 log x + 21 + C 23.


- log — + C
« ^ x‘+l
2 2
1 a: -a 1
24.
iog —2 + C 25. -21og|:c + l|- x + 31og |a: + 2| +C
2{a^-b^) X -b1.2 +1

2 1 x-1

y+ logU “l| + “log


26. : + C
x + 1

11
27. — log I -V + 1 I + log [ X + 31 + C
4 ' 2{x + l) 4

ow
28. -
log I X + 21 + — log I ar ~ 31 + C
25 ' 25 ' 5(x-3)

I log I X + 31 + I log I X - 21 - +C 2, X-1 1


20, 30. + C

e
5 5 x-2 x+2 3 (x-1)

re
31.
^log|x-l| + |log|x+l| + 2(x + l)
rFl 1
+ C
32. ^ ^ + log IX + 21 + C

F
33.
— + 13 log|x| -12 log|2x + 1| + C 34. 6 log I XI - log I X + 11 — + C

r
^ {x + 1)
ou
fo
ks
1 x +1 x^ + 1 1
35. -log —+ C
36. log + +C
x-1 x-1
x^ + 2 x^2
oo
Y

HINTS TO SELECTED PROBLEMS


B

7. We have.
re

5x 5x
1 dx = dx
(x + 1) (x^-4)
ou

(x + l)(x + 2) (x-2)
Y
ad

5x A B C
Let
(x +1) (x + 2) (x - 2) x+1 x+2 x-2
d

Using Cover-up method, we get


in
Re

= ^,B =
-10
A - and C = 10 _ 5
3 12 " 6
F

4 2
5x
1 =
(x + l)(x + 2)(x-2)
dx =
I log I X + 11 -1 log I X + 21 + I log I X - 21 + C
23. We have.
H - 1
1
I = dx=f— dx
J n /
x(x"+l) x' (x" +1)
Let x” = t. Then, n x” dx - dt
rfl 1 1
^ = n' \*’ t(t + l)
1
^t=i
n
{t t+l
dt =
^jlog f-Iog (t + l)|+C
n
t
I = -log + C=-log — +C
n t + l » ^ x"+l
INDEFINITE iNTEGRALS 13.79

+ X + 1
25. Let/ = dx.
(x + l)^ (x + 2)
x^ + x + 1 A B C
Let + 2
{x + lf{x + 2) x +1
(x + lf X + 2

x'^ + x + l = A{x + l){x + 2) + B{x + 2) + C(x + lf (i)

Puttings = -1, - 2 and 0 successively in (i), we get


1 = B, 3=Candl = 2A + 2B + C => A = -2,B = 1,C = 3
2
X +j: + 1 -2 1 3
+ 2
ix + lf{x + 2) x + 1
{x+ir x + 2

X^ + X + 1
— dx+\ — dx+3 \ — dx

w
dx = -2
{x + lf (x + 2) x +1 ^ {x + lf ^^ + 2
= -21og|:c + l|--^^^+31og|a: + 2l+C
30. Let/ = I {x-lf {x + 2)
1
dx

F lo
ee
Fr
1 A B C
Let + 2 + . Then,
(x-1)^ (x + 2) x-1
(x-ir x+ 2
for
1 = A{x-l){x + 2) + B{x + 2)+C{x-l)^
ur
(i)

Putting X = 1,-2 and 0 successively in (i), we get


s

B = -,C = i9 and 1 = -2A + 2B + C => B = 13' C i A = -1


ok

9'^ 9
Yo

3 9
o

1 1 1 1
eB

{x-l)'^{x + 2) 9(ar-l)'^ 3(x-1)2'*^9(x + 2)


1 j +-
dx 1 fI —1 dx
dx =
r

/ =
9 ^ x+2
(a'-l)^(;c + 2) 9 ^ x-1 3 \x-l)^
ou
ad

x + 2 1
+ C
Y

=> / = -
? ?‘°8 x-1 3{x-l)
Re
nd

3x + 5
35. Let / = f dx. Then,
Fi

-X

3:c + 5 3x + 5 3x + 5
I = dx =
dx = j (x-1)^ {x + 1) dx
{x-l)-{x-D {x-l){x^-D
3x + 5 A B C
Let . Then,
{x-l)^{x + D x-1
{x-D^ x + 1

3;c + 5 = A(x-l)(x + l) + B(:c + l) + C(x-l)^ .(i)

Putting X =1,-1 and 0 successively in (i), we get


8 = 2B,2 = 4C and5 = -A + B + C=>B=4, C =1,A
2'
= -I2
3.T + 5 1
^ ___ + 4
+1
(x-l)^(a: + l) 2(a:-1) {x-1)^ 2 (x + 1)
13.80 APPLIED MATHEMATICS-XII

3x + 5
ilog|x-l[
1
dx = - +
- log I a: +11 + C
{x-lf (x + 1) 2 X-

MULTIPLE CHOICE QUESTIONS (MCQs)


Mark the correct alternative in each of the following:
1.
(.r - \) e~^ dx is equal to
(a) -xe^ + C (b) +C {c)-xe ^ + C (d) A:e~^ + C
l/x
2. If dx = kl + C, then k is equal to
-Y

(a) L

w
(b) - log, 2 (c) -1
log, 2
' 3
I a: I dxis equal to

F lo
4 4

(a) +C (b)- + C
(c)^ + C (d) none of these

ee
4 4

Fr
r x+3
4.
(y + 4)
2^^ dx =
X X for X
ur
1
(a)
Y + 4
+ c (b)^ + c (c) 2^^ (d) —^ +C
Y + 3
(y + 4) (a:+ 4)^
ks

2
Yo

dx
oo
eB

- X
- e 1 -1 1
-

-^C (b)-- —+ C (c) 2"^ (d)- + c


(e^ + 1)
- X
e + e e + e e -e
r
ou
ad

1
Y+ -
1
6- The primitive of the function / (y) = 1 —^ a ^ > 0 is
Y

1 1 1
Re

x+ - 1 x+ -
nd

X + -
a X x+ -
a
X
a X
(a) (b) log, a-a X (c) loge (d) Y
log,
Fi

X
log, a
1
7. The value of dx is
Y + Y log X
(a) 1 + log Y (b) Y + log Y (c) Y log (1 + log y) (d) log (1 + log y)
8- j e’^\f{x)+f'(x)]dx =
(a)e^f(x)+C (b)e^+f(x) + C (c) 2e^/(Y)+C (d) e^-/(Y)+C
9
Y
9.
dx is equal to
(4y^+1)^
-5
1 1
v5 1 1
^-5 -5
1 1
+ C (b) ^- I 4 + -^ + C (C) + C (d) 2+4 +c
X J Y IOyI^y 10 Ly
13.81
INDEFINITE INTEGRALS

m.
X
dx = a {1 + +b Vl+? +C, then
(c) fl = - i ^^=-l
1

(a) «=|-i>=l (b) a= — ,b =1 (d) -,b = -1


3 3 3

11. dx is qual to
x + l
3 2 .3

(a) :c + y + y-logll-xl+C (b)^ + y ^--log|l-.t| + C o

log 11 + a: I + C
(c) A:-y-y-logll + x| + C (d) :v-y + 3

X ~x

~——dx ax+ b loge 1 4e^' +5e~^\ +C, then

w
12
4e^ + 5 e ^

(b)a = if = ^ (d)a4,i^=|

F lo
(a) a=--,b=- (c) fl=- -,b=~-
' ^ 8 8 8 8

^2

ee
.Y l-X
dx is equal to
U+x

Fr
Y Y
Y
e e

(b) --^ +c (d) - r +C


(a) -S- +C (c) +c

1+x l +x (1+0
2x2 for (l + x )
2x2
ur
ANSWERS
s
ok
Yo

1. (c) 2. (a) 3. (d) 4. (a) 5. (a) 6. (a) 7. (d) 8. (a)


o

9. (d) 10. (d) 11. (d) 12. (c) 13. (c)


eB

FILL IN THE BLANKS QUESTIONS (FBQs)


r

1. The value of the integral ^21ogY


ad
ou

4- e IS

X
f x+3 X

^e^dx = ^dx =
Y

2. 3. e

ix + 4.) ix + l)
Re
nd

X^+l /log,,Y_^4!ogeY dx =
dx =
g3iogeY_^2!oge7
D.
Fi

4.
.Y 2-1

6. dx = 7.
^dx =
.r 2-y + I e
3y

X
e
dx =
8. Y^(l +logY) dx = 9.
e^+1

10. (y+3) (y2 + 6y + 10)^ dx =


answers
X
X
3 ^Y
X 2 —+c 3. r+C
!. — + +c
y +4 Y +1
3 log 2
13.82
APPLIED MATHEMATICS-XII

3
x-1
9. -Y+log +C e. -x^ +-x'^ + x+c
-

-t + 1 3 3 2
1 -3.v^
7. --e +C 8. ;c^ + C 9. log I +11 + C
6

— {x^ +6a: + 10)


10
10. + c
20

VERY SHORT ANSWER QUESTIONS (VSAOs)


Answer each of the following (juestions in one luord or, one sentence or, as per exact rec^uirenient of the
question:
n

i. Write a value of f x“^ dx. 7- Write a value of log X dx.


X
n

3. Write a value of (log x) dx.


1
4. Write a value of dx.

w
X
●’ 1 + e"-'
1
5. Writea value of j dx.
6. Write a value of log^. x d.x.

F lo
l + 2e^

7. Write a value of a^ e^ dx. 8. Write a value of


C2x^ + \nx
dx.

e
Fre
X
9. Writea value of a \oge X a
+ e ) dx. 10. Write a value of dx.
for ●’ 3 + n^'
11- Write a value of ● 1 + log a: - dx.
1
12. Write a value of dx.
●* 3 + .T log a: -V (log .V)
n
r
x( 1 1
You

13. Writea value of e ' Write a value of e


ax
oks

X —2 f ix) + f' (x)} dx.


X J
eBo

15. Write a value of + x^ dx. 16. Write a value of dx.


2 2 .
X
17. Evaluate: X + 4x
dx 18. Evaluate: dx
1 + x^ x^ + 6x^ +5
our
ad

19. Evaluate: (1 + log xf dx 20. Evaluate: log^' dx


X X

^-1
dY

21. Evaluate: f 2^ dx
Re

22. Evaluate: J ^ dx
Fin

23. Evaluate: f—
x-1
dx
24. Evaluate: J (1 - a:) -fx dx
x-1
25. If
X J
dx=f(x)e^+C, then write the value of/(a:).
1 1
26. Write the anti derivative of 3fx + 27, Evaluate: dx
●' -t(l + logA-)
ANSWERS
8
n+ 1
l.^+C 3 (log X)
8 2. |(logx)2+C n + 1
+ C

4. - log (1 + e~ +C 5. -log[2 + t-"'VC 6.x {logs a: -1) + C


X (7+1
7. + C a X
8. - +C 9. + C
log (mO 4
loge a a +1
13.83
INDEFINITE INTEGRALS

1-H
1 .. Oog.t) + c
10. log (3 + +C 11, log (3 + a: log a:) + C 12.
1 -fi
log a
14. + c
X
2 1
X ^9 + ^ log x + ^
1
15. + 'A ^ +C 16. -X -9 +C
2 2

(1 + log x)^
1- llog|l+A^l + C 18. —3 log I + 6x^ + 51 + C 1-!,
3
+ C

2
2^ X 1
20. -2 (log -v)^ + C 21.
loge 2
+ C . H- + - + C
2 .t
3 1
— + -i + c
3 5

w
A

26. 2 (A . log(l + logA)+C

F lo
ee
Fr
for
ur
s
ook
Yo
eB
r
ad
ou
Y
Re
nd
Fi
CHAPTER 14
DEFINITE INTEGRALS

ow
14.1 FUNDAMENTAL THEOREM OF INTEGRAL CALCULUS
STATEMENT Let (j){x) be the prhnitive or antiderivative of a continuous function f(x) defined on [a, E?]
le. — 14)(x)l =f{x). Then the definite integral off{x) [a, b] is denoted by f f{x) dx and is equal to

e
over

dx

re
a

Frl
F
b
i.e. f(x)dx = <l)(b)
ou
or
a

The numbers a and b are called the limits of integration, 'a' is called the lower Umit and b the
Upper limit. The interval [a, b] is called the interval of integration,
kfs
If we use the notation (x) to denote <j> (1j) - (|> {a). Then,
oo

-I a
Y

b -I b
B

[From (i)]
f{x) dx = <t) (x) J a
re

b / f
oYu

f{x)dx = Valueof «l)(x)at x j-j^Value of <t)(x) at x =


a
ad

b
d

=5- f{x) dx = (Value of antiderivative atx=b) -(Value of antiderivative at x fl)


in
Re

RljMAKK I In the above statement it does not matter zohich anti-derivative is used to evaluate the
F

definite integral, because if f{x) dx = <j>(x) + C, then


b -\b
fix) dx = (j) (x) + C (J)(6) + C }■ - U(fl) + C [ = 4>(b)-<t)(iJ)
a

In other words, to evaluate the deifnite integral there is no need to keep the constant of integration,
Ri MARK fix) dx is read as the integral off{x)from a to b" or integral o//(x) over [a, b].
a

14.2 EVALUATION OF DEFINITE INTEGRALS


b

To evaluate the definite integral j /(x) dxof a continuous function/(x) defined on [a, b], we may

use the following algorithm.


14.2
APPLIED MATHEMATICS-XII

ALGORITHM

Find the indefinite integral ^ f{x) dx. Let this be (}) (.v). There is no need to keep the constarit of
integration.
!' Evaluate ^(b) and (l)(rt).
● 111 Calculate ij) (b) - (j) (tr).
b

The number obtained in Step III is the value of the definite integral f f(x) dx.
a

ILLUSTRATIVE EXAMPLES
rXAMPiE I Evaluate:

w
2
-1

(i) j x^ dx (ii)
1
- dx
1
-4"'

Flo
1
1
(iii) I
1
dx

e
(iv) dx
+ X + fx i 2x - 3

re
0
0

SOLLITION We have.

F
2
ur
x^dx =

r
(i)
3
1
3 3 3 3 ^3 fo
ks
1 j 1

/ -rf-Y=l0g|.T(
Yo
(ii)
log |-l|-log|-4| =logl -log4 = 0-log4 = -log4
oo

-4 J -4
eB

(iii)
1
dx = / + .r
dx
\ /
0 0 + X +
^fx Jl + X ~fx
ur

J\''
ad
Yo

+ X

0
d
Re

j(l.x)
3/2
in

3
0
F

I (1.1) |(l + 0)3''"-|(0)


3/2 3/2 3/2

2 ^23/2
f(]-0) =1(272-2] =
-1 4 f ^ ^
3V O 3 \ /
1 nl

(iv) I , dx
1
=
●*0 2.V-3 2 'og (2-3- - 3) 0

~ log hl|-log)-3[ = I(^logl-log3j=l(0-log3) =


EXAMPLE 2
If ^ (3.V + 2x + k) dx = 0,ifndk.
0
14.3
DEFINITE INTEGRALS

SOLUTION We have.
1

(3.r + 2x + cf) t/.t = 0 => X + -V + kx = 0=>(l+l + cf)-0=0=>/c=-2


0 Jo

EXAMPLES If f (3.v^ + lx +1) dx =11, find real values of a.


SOLUTION We have,

(3/ + 2.t + l) dx = ll
1
a

3 2
.r + a: + X = 11

w
Jl

=> (i7^ + + il) — (1 + 1 + 1) — 11

F lo
a ^ +a^ + a-3 =11
^ + a -14 = 0

ee
a

Fr
((j - 2) (/+ 3(7 + 7) = 0 => rt = 2 ●: a 2 + 3(7 + 7 ^ 0 for any aeR

^ 3 r 2 2 for
ur
EXAMPLE 4 If rf.T = 0 and if X dx = — , find a and b.
a a
s
SOLUTION We have,
ook
Yo

dx = 0
eB

X
4f
r

= 0
ou
ad

4
■ia
Y
Re

_/=0 => {b^ a^) {b^ + 0^) = 0 ^ b^ ^ = 0 => = [v b^a]


nd

- a -a
Fi

b ry
Now, f x'^ dx = -
3
a

(
X 2

3 3
'a

3 3

b^ -a^ = 2 => (-(7)^ -a^ = .2 [v b=~a]

=> -2a^ = 2 => (7^ = -1 =i> (7 = -1


b = -a => b = 1
Hence, (7 =-1 and b =1.
14.4
APPLIED MATHEMATICS-XII

EXAMPLE 5 Evaluate:
4 4
1
(i)|
X 2x
dx (ii) dx (iii) dx
0 + 2x + 3 2 x^ -hi 0 5x^ + 1
4
1
SOLUTION (i) Let I = dx. Then,
0 ^|x^ -h2x+ 3
4
1
/
=/ dx
0
^x^ + 2x +3
4
1
I = dx

w
0

l4

/ = log (a + 1) + ^{x + lf + i^f

Flo
0

h4

ee
/ = log X + 1 + + 2x + 3

Fr
0

/ =
log (5 + ^16 + 8 + 3) - log (1 + ^3)
for
ur
5+ 3^3
/ = log (5 + 3 V^) - log (1 + ^3) = log
1+^3
s
4
ok
Yo

(ii) Let I = ^iAr. Then,


: +1
Bo

/ =i| 2x
dx
re

x^ -hi
-l4
ou
ad

=>
^ ^ log + 1) J2
Y

^ [log (4^+1)-log (2^+1)] = ^


I =
1 1 17 ^
- (log 17-log5) = - log —
nd
Re

■4 z 5 ,
2x
Fi

(iii) Let / = dx. Then,


0 5x^+1
1 il
10 X
dx =\5 log {Sx^ + 1)
1
I

0 5x'^ +1 5
(log 6 - log 1) = 5i log 6
0

r;XAMP{.E6 Evaluate:

(i) y dx ■ logy
(ii) dx (iii) y log (1 + 2y) dx
0 X
1 0

SOLUTION (i) Let/ = y fry.Then,


0

'll nl -il
I =
^ ={e~0)-{e~e^)=l
X
X dx = 1 ● fry = xe e
I II Jo
0 0 Jo JO
14.5
DEFINITE INTEGRALS

■ log a: dx. Then,


(ii) Let / =
a:

I f,
= I log X ■ \dx
-
^ X
^
=
(
(logx) --- T f-
x)[~{ 'n ^
[Integrating by parts]
II

if
2 N.
1 1
1 1
/ --logx -- = -Mog2 +(lx logl)- ^2
L X UJi I 2
1

^
I =
+ i = ^(->°S2 + 1) = l(-log2 + loge) = -log^-^
(iii) Let J = j ;c log (1 + 2a:) dx. Then,
0 “ 1

w
1
2 2
2 ^ J

F lo
— dx
J = ylog(l + 2x)
X

1 +2x 2
JO 0

2
X

i log 3-0 -I

ee
I = dx
1 + 2x

Fr
V ^ / 0

1 I X 1 1
● dx
for .. ^=^-1+ 1

I =-log3-
2
\ 2 4J 4(1 + 2x)
■ l + 2x 2 4 4(l + 2x)
0
r
1
You

1 1 1 3
= ilog3-
s

^-i + hog(l + 2x) -log3 -0\ =-rlog3


+
ook

I = 4 4 8
2
Jo
eB

7 Evaluate:
3
1
2
5x^
(ii) 1 ^ (x +1)- dx
our
ad

(i)J x^ + 4x + 3
dx
1
1
2 e 2
5x
dx. Then,
dY

SOLUTION (i) Let I =


{I x^ + 4x + 3
Re

2 (
Fin

2 2 4x+ 3
X
dx = 5 1 - dx
I
1
x^ + 4x + 3 1 I x^ + 4x+3^
2
2
? 4x + 3 r 2(2x + 4)-5 dx
dx = 5 l-dx-5
/ = 5 1 ● frx - 5
x^ + 4x + 3 1 1 ^^ + 4x + 3
1

I =5
2

l-dx-5
I I 2(2x + 4) 5
● dx
x^ + 4x + 3 x^ + 4x + 3
=>

1
2
2 1
^ 2x + 4 rfx
1 = 5 ldx-10 dx + 25
■ x^ + 4x + 3 x^ + 4x + 3
2
^ ^^ + 4 1
/ = 5 ldx-10 dx + 25 j (x +2)^-1^ dx

1
x^ + 4x + 3 1
14.6
APPLIED MATHEMATICS-XII

i2 i2
-|2 1
I = 5 X -10 log(.T^+4.v+ 3) + 25x
log :
●V + 2 -1
Jl 2(1) x+2 + 1
1

^ - 5 (2 -1) -10 log 15 - log 8 25 , (3 f2


T 5 iJ
1 = 5-10 log —
15 25 , (3
— log — X -
4'l 15
25, (6^
= 5-10 log — +
+

1
I 8 2 ^(5 2J 8 T'°n5
A Bx + C
(ii) Let +

x^ (a: +1) x + 1 X
2 ...(i)

Then, 1 = Ax^ + (Bx + C) (a: + 1)

w
●●●(ii)
Putting x = 0,x = -l respectively in (ii), we get: C = 1 and A = I

Equating coefficients of on both sides of (ii), we get: 0 = A +B => B = -A = -1

Flo
Substituting the values of A, B and C in (i), we obtain

e
1 1 -a: + 1 1 1

re
+
.V
1__1
x^ (x + 1) x+1 .T
2
X +1 2
^ +1 x~^

F
.r X
3
1 3 f
dx =
1 1
-X + ^
1 ^
dx =
if
ur
^og\x + l\-\og\x\--

r
1 f (a: +1) 1 V
a: +1 v2
.V fo X
1
^ 1
= log 4 - log 3 — -(log 2-logl -1) =log4-log3-log2-i3 + l
ks
V 3
Yo
oo

,
= log —
4 1
-- + 1 . (2] 2
2x3 3 ® 3 3
eB

EXAMPLES Evaluate:
2
1 2
ur

1
(i) dx
(ii) j dx
ad

(x + 1) (x+2)
i x(l+x^)
Yo

1 A B
SOLUTION (i) Let +
(at + 1) (a:+ 2) a: + 1 a: + 2 ...(i)
d
Re

Then, I = A (x + 2) + B (x + 1)
in

●●●(ii)
Putting a: + 2 = 0 or, a: = - 2 in (ii), we get: B = -1
F

Putting a; + 1 = 0 or, a: = -1 in (ii). we get: A = 1


Putting the values of A and B in (i), we get
1 1 1

(a: + 1) (a: + 2) a:-i-1 a: + 2


2 2 2
1 1 1
1 dx = dx - dx
(X +1) (X + 2) 1
x + 1 x + 2
1
l2 -,2
1 = log (x+1) - log (x + 2)
Jl J]

/ = (log 3-log 2)-(log 4-log 3) =2 1og3-log2-log4 = log9-log8=log[^'


.8
14.7
DEFINITE INTEGRALS

1 A Bx + C
(ii) Let 2 +
,v(l + x^) X 1 +x^
Then, 1 = A{1+x^) + (B.t + C)x ...(ii)

Putting :t = 0 in (ii), we get A = 1.


2
Comparing the coefficients of .v and x in (ii), we get
A + B = 0 and C = 0=>B = -1 and C = 0 [V A=l]
Substituting the values of A, B and C in (i), we get
1 1 X

X (1 + x^) -●t 1 + x^
2
? -dx--
1 , 1 r 2X 1 r 21 ^

ow
I
1 ^f/x = X 2 ●' 1 + x
^dx = log X J 1
- log(l
1 X (1 + X ) 1 1

/ = (log 2 - log 1) - 1 (logs-log 2) = log 2logs + I log 2 = I log 2-^ logs

e
EXERCi€E14.1

Fl
re
Evaluate thefollozvmg definite integrals (1-17):

F
3
1 1
dx
ur
1. dx
4
X

or
sf
oo
3
X
dx 4. e ^ dx
k
3.
Yo

2 X^ + 1 0
oo

1
1 -X
6.1
B

X
D. dx dx
1 + X
0
re

e X

" logx 8. — (1 + X log x) dx


u

7. dx
ad

■ (AT + l)^
Yo

1
2
e
1 1
f ● dx
d

9. dx
log^ (logi)^
Re

X
in

4 .2
? X+ 3
F

X + X
11. dx 12. dx
X (x + 2) 1

2 r
1
X-1^
— e X,dx
13. X (1 - x)^ dx 14.

0 1 V -V
2
1 X
dx 1 r, dx
15.
1
(x + 1) (x + 2)
2 k
1 1
r 2x 1 dx ● If dx = ~ , find the value of k.
e

^ 2x^J 0
2 + 8x^ 16

If [ 3x^dx = 8, find the value of a.


0
14.8 APPLIED MATHEMATICS-XII

ANSWERS
1
I. 2 2. log 2 3- T
2
log 2 4. 1
/ \
3
5. log ^ 6. 21og2-l 7. -log3-log2 8.
2)
2
9.1 57-yI
2
10. -
2
-e
11. — log 6 12.
2 5
2
32 2
13. — 14. ~—e
42 2
15. log — 16.
27 3
17. K 18. 2 19. V2

HINTS TO SELECTED PROBLEMS

w
} x+1-1 1 /'
1 n 1
5. / =
— dx = J( 1 - dx= A:-log(A:+l) = l-log2

F lo
=

i x + 1 x + 1 x + 1 -I 0
0 0 0 V

n 1
1 -X : 2-(l + x) 2
6. 1 = dx = dx -

ee
-1 dx ~ 2 log (x +1) - X
il+x 1 + X 1 + x

Fr
0 0 0 0

=> / = (2 log 2-1)-(2 log 1-0) =2 log 2-1 for


2 2 2
-i(? 2 2
e e e e
r
1 1 1
1-2
X
10. / = ● 1 frx -
2rfx = jxdx-
You

e (log^) logxj^ X (log x)


s

e e (logx)
ook

I II

2 2 2
eB

=> /=-^ e e e

log loge 21oge loge 2


our
ad

1 ; + X +
!-. / = dx = dx - + x) + rfx
0
+ X -
0
(1 + x) - X 0

ll
dY

2 3/2
+ 1x^/2 hH - 4 72
Re

=> / = - (1 + x)
3 3
Jo 3 3 3 ^
Fin

2 2 /
X 1 2
U). /
=1 (x + l)(x+2)
dx -
x+1
+
x+2
dx
1 1 V

21^
=> / =
- log (x +1) + 2 log (x + 2)
t2
{x + 2f 16 9 32
= log = log — - log -2 = log —
J 1 x + 1 3 27
1

2
17. /
=1 e 1
lx
1
X
-l_
2x^
dx =
1
e -
x
dx -
1
e

2x
^dx
II I

-|2 2
1 2.V 1 lx 1 4 1 2 e 4 ~2e
^2
dx - e
^dx -
— e + — e — e
2x
1 2x .4 2 4
14.9
DEFINITE INTEGRALS

14.3 EVALUATION OF DEFINITE INTEGRALS BY SUBSTITUTION

Consider the integral J/(g(x)]g'(*) &■ Let gW=t, then g(x)dx^it Also, when
a

X = a, t= g{a) and t = g{b) for x=b.


b gib)
f six) g'ix)dx = fit) dt
a gia)
in terms of new
Thus, if the variable in a definite integral is changed, then the substitution i
variable is effected at three places :
(ii) in the differential, say, dx (iii) in the limits
(i) in the integrand

w
Also, limits of the new variable f are simply the values of t corresponding to the values of the
original variable a;, and so they are obtained by putting the values of v m the subsHtutional
relation between a: and t.

Flo
We may use the following algorithm to evaluate definite integrals by making some substitution.

e
ALGORITHM

re
b

Obtain the definite integral and express it in the form 1=] fig {x)g' (3^) dx.

F
STHI’ I
a
ur
r
STEP 11 Put t =gix) which gives g’ix) dx = dt. fo
STEP 111 Putx=a (the lower limit for x) in t = g(ac) to get t = g(a) as the lower limit for t.

b (the upper limit for x) in t = to get t = g{b) as the upper limit for t.
ks
Putx =

S'IUI‘ IV Substitute g’ix) dx = dt and replace old limits of integration bp the new limits to get
Yo
oo

gib)I
^ fit) dt
eB

gU)
Evaluate this integral by using standard methods of integration discussed in section 14.1.
ur

Following examples will illustrate the above procedure.


ad
Yo

ILLUSTRATIVE EXAMPLES
d
Re

EXAMPLE 1 Evaluate:
in

4 2x
1 dx
(i) dx (ii)
F

x + ^|x 0
5x^+1
0

SOLUTION (i) Letx=f^. Then, frx = => dx=2tdt.


When -
x = 0, x = =t^=^ f2=o=>f = 0. When x=4,t^=x=>f^=4=>f = 2
^i^ =2 ] — frf =2['log it + l)
4

I =
1
dx = == 2 [log 3 - log 1] = 2 log 3
0
X + ^fx 0
t^+t t+10
Jo

(ii) Let 5x^+ 1 =L Then, fr (5x^+1) = dt => 10 xdx = dt 2


When x = 0,t=5x^+l=> t=l. Whenx=l,f=5x +l=>f = 6
6
2x
dx =
f 2x
— X
dt 1 f 1
- dt =
I logf i(log6-logl)
5
= -log6
^
0
5x^+1 t lOx
5 i t D
Jl
14.10
APPLIED MATHEMATICS-XII

EXERCISE 14.2

Evaluate the following integrals:


4 2 2
1
1.
I 2- f jdx 3. f
3x
dx
2 X^ + 1 1 a: (1 + log .V) 1 9x^ -1
X 2
4.
2 2
dx 5. ●ve"' dx 6. -V 2 dx
0 a + X
0 0
1
I-a-2 ■ 24
12

4
a: + a:^ + 1
2
dx 8.
2x4
dx 9.
x{x-4)^^^ dx
0 0 (1 + ^ ) 4

ow
1f 11 -AT 2
:n. dx n. ●V^ +1 dx 12.
1

0 a+A'V 2^x
-1 1 -v(l + logA:)
9
13. r dx

e
: (so-x^^Y

re
rFl
F
ANSWERS
1 (17 log 2 1
1. - log — ● - (log 35 - log 8) 4. fl(V2-1)

r
2 ° S
log le 6
ou
16
6- 7z(2 + V2)
„ 1
fo
ks
2 15 2
log 3 S. 1

9. Z20 1
4V2
oo

10. - 11. 12. log 2


7 2 3
1 4- log 2
Y
eB

19
13. —
99
ur

WW7S TO SELECTED PROBLEMS


ad
Yo

4 i4
1 i 2x
dx = -2 log {x^ +1 = i(logl7-Iog5)=^Iog ~
1. I = — dx =
i x^+i 2 2 AT^+1
d

2
^ V ^
Re

2
in

6. 1 =
x ^x + 2 dx. Let a: + 2 = Then, dx = 2tdt
F

Also, .V = 0 => = 2 => ^ = V2 and, a: = 2 => = 4 => f = 2

2
2t^f
I
j {i^-2)4^2tdt = 2 {t^-2t^)dt=2 ~
3 3
v'2 V2
^V2
/ = 2 4V2 4^2 = 2 ^+M 32 +16 V2
5 3 5 3 15 15 15

a; 1 / I 1
/ =
y fr.T. Let a: + — = /. Then, d a' + — = dt => 1 —- dx = dt.
1 X A
0 A
A +

Xj
14.11
DEFINITE INTEGRALS

Clearly,,v = 0 => i = cc and .t =1 => t=2.


2
1 l1^ 1
f t L*
I J GO
2
CO

n. Let / = 5 + 1 dx and let + 1 = t. Then, d {x^ +1) = dt 5 dx = dt


-1

Also, X = -1 => t = Oand, X = 1 => t = 2.


? 2 3/2^ 4^/2
Mrf<=3
0
f
-0 3 3

9
= t. Then, — = dt.

w
13. I (/x. Let 30
3/2^2 2
4 (30-x )

X = 4=> f = 22 and, x = 9 => t = 3

F lo
Now,
2rif 19
-I 3 U 22

ee
3 1 3 t J22 99
22

Fr
14.4 AREA AS A DEFINITE INTEGRAL
THEOREM Let fix) be a continuous function defined on [a, b]. Then, the area bounded by the curve
for
ur
y =/(x), the x-axis and the ordinates x=^aand x=b is given by
b b
s
I fix) dx or. ydx
ook
Yo

a a

IMtOOF Let AD be the curve y -f(x) between the ordinates BA (x - rt) and CD (x - b). Then, the
eB

on the curve and


required area is the area of region ABCD. Let P(x,y) be any point
Q(x +Ax, y + Ay) be a neighbouring point on it. Draw ordinates PL and QM. Then, PL - y,
r

QM =y + Ay and LM = Ax.
ou
ad

Let A denote the Area BLPA, and let A + AA be the Area BMQA. Then, AA = Area LMQP
Y

Y
D
Re
nd

y=/W‘\
Fi

R Q, >

Ns
A| P
x = b

x = a
AH A/\

B L M c X
O

Fig. 14.1

Also, Area LMNP = y Ax, and Area LMQP = (y + Ay) Ax.


Clearly, Area of rectangle LMNP < Area LMQP < Area of rectangle LMQR
14.12
APPLIED MATHEMATICS-XII

y < A A < (y + Ay) A;t


A A
<y +Ay
Aa:

AA
=?●
lim y < Urn ^ lim (y + Ay)
Aa:-)-0 4a: -> 0 Ax Ay^O
dA
y-^-y
dx
dA

? frA b

J dx
dx= y dx [Integrating between the limits a and b]

w
a a

-\x=b b
A ydx

Flo
x=a
a

ee
(Area A when x=b) -(Area A when a: = fl) = ydx

Fr
a

Area ABCD - 0 =
When x = a, PL coincides with AB
ydx
for So, area ABLP = 0 when x = a '
ur
(1

Area ABCD = y dx = f{x) dx


k s
Yo

a a
oo

iUi\ I AJiK 1, If the curve y =f(x) lies below x-axis, then the area bounded by the curve y -fix), the x-axis
eB

b h
and the ordinates x =
a and x = bis negative. So, area is given by J \f(x) \ dx or, \y\ dx.
r

a a
ou
ad

RLMAKKA
The area bounded by the curve x-f{y), they-axisand the abscissae y = candy = d is given by
d d
Y

|/(y)| dy or, | |a:( dy


Re
nd

c c
Fi

C
y = d
D

Q(a: + Ax,y + Ay)


M
A/1
.6
P(x,y)
'^A
x=fiy)
B ●A
y = c

O X

Fig. 14.2
14.13
DEFINITE INTEGRALS

14.5 AREA USING VERTICAL STRIPS


In order to find the area of the region bounded by the curve y =f{x), x-axis and the ordinates
X and x-h, we may use the following algorithm.
ALGORITHM

^y\'v\ Make a sketch of the curve and identify the region whose area is to he found.
Slice the region into vertical strips. Take an arbitrary point P{x. y) on the curve and construct a

w
representative strip of width dx having two ends of its base on x-axis at pomts\^x, 0 jand

e
dx
x + — 0 and (x, 0) as the mid-point of its base.

ro
2 '

re
srnp.ni Construct an approximating rectangle whose base is same
as that of the representative strip

F
and height equal fo | y 1 = 1 /(●^) 1 ●
y

Fl
u
sr
y =fix)

ko
o
of
P(x,y)
o
Y
erB

x = b

II
uY

!-!

O ‘dx X
ad
do

Fig. 14.3
in

STI-P IV Find the area of the approximating rectangle as y I dx =1 f(x) 1 dx.


Find the values ofx, say, x=aandx=b zvithin which the approximating rectangle can
move
Re

STl-P V
F

horizontally in the given region and form the integral j | / (:t} | dx


a
or,
a
\y\dx.

STHPVl Evaluate the integral obtained in step V. The value of integral so obtained is the required area,
RHMARKi If the curve y = f{x) lies above Xraxis on interval [a, 1?], then the area of the region bounded
by the curve y =f{x), x-axis and the ordinates x=aand x=b is given by
b b
/(y) > 0 for all X € [fl, Ij] \f(x) \-f {x)
j 1 fix) \dx = fix) dx^\ ydx
a a
a

REMARK2 If the curve y= fix) lies below x-axis on interval [a, b], then the area of the region bounded
by the curvey =fix), x-axis and the ordinates x=a and x=b is given by
14.14
APPLIED MATHEMATiCS-XII

b b

j \f{x)\dx=- ' f{x)dx = - y dx ●●● / (.r) < 0 for aU .v e [a, b] \ f{x) \ = -f (x)
a a a

KK
Iff{x) is a conthwous function defined on [a, b]and c e{a, b) such that the curve y = f(x)
hes above x-axis on [a, c] below x-axis on [c, b] as shown in Fig. U.4. Then, area A of the region bounded
y X-axis, X = aand x = b is given by
y

w
1/ =/(-v)

Flo
e
re
F
a c dx b
O dx
●●
X
ur
r
P(x,ij)-^
fo
Fig. 14.4
ks
Yo
b
A= \f{x)\dx = I fix) \ dx + J I fix) \dx = l f ix) dx +
oo

-/ (-V) dx
a a c
eB

a c

ILLUSTRATIVE EXAMPLES
ur

hXAMf'Mii Find the area bounded by the parabola y^=4ax and its latus rectum.
ad
Yo

SOLUTION A rough sketch of the parabola i/ = 4ax is shown in Fig. 14.5. Let S(rt, 0) be the focus
and L SL' be the directrix of the parabola y^ = 4ax. The required area is LO L' L Since the curve
d
Re

is symmetrical about jc-axis.


in

So, required area = 2 (Area LO SL).


F

Here, we slice the


Fig. 14.5, we have
area LOSL into vertical strips. For the approximating rectangle shown ii n

Length = \y\, Width = dx and so Area = \y\dx


Since the approximating rectangle can move between ;r = 0 and
X = a.. So, required area A is given by
A = 2 (Area LOSL)
a

^=2 \y\dx
0

=> A=2 ydx [V y>0 :.\y\=y]


0
PEFINTE INTEGRALS 14.15

A =2 ■J4ax dx ●/ P{x,y) lies on i/^ = 4ax y = yjAax

A = 4^ ^fx dx

1 0
^3/2
A =4 4a
3/2
J 0

2 3/2 8 2
A = 4 -Jfl X — (a -0)=- a
sq. units
^ 3
EXAMI’l.l ; Using integration, find the area of the region bounded betzveen the line x = 4 and the
2

w
parabola y =\6x.

SOLUTION The equation y^ = 16x represents a parabola with vertex at the origin and axis of

F lo
symmetry along the positive direction of .v-axis as shown in Fig. 14.6. Clearly, x = 4 is a line
parallel to i/-axis. The region is the shaded portion shown in Fig. 14.6. Since y =16.r is
2

ee
Fr
symmetrical about .v-axis.
y
.-. Required area = 2 (Area OCAO) for
Here, we slice the area above v-axis into vertical P{x,y) A
r
strips. For the approximating rectangle shown in
You

.v = 4
s
ook

Fig. 14.6, we have X'


}/
X

Length=|i/|, Width = d.v and Area=|y| dx. O dx C(4, 0)


eB

The approximating rectangle can move between B


V = 0 and x = 4.
our

1/2 = 16v
ad

So, required area A is given by r

Fig. 14.6
A = 2 (Area OCAO)
dY

4 4
Re

A =2 11/1 = [v i/>0 .-.\y\=y]


Fin

0 0

A =2 f P(.v, y) lies on y^ = 16.v .’. y = Vl6.v


0

A =8 Vv dv

-I 4
^.3/2 16 m3/2_q3/2. 16 128
A =8 — sq. units
3/2 3 3
J 0

l.XAMPLE 3 Sketch the region bounded byy = 2x-x^ and x-axis and find its area using integration.
SOLUTION The equation of the curve is y = 2.v - .v . Clearly, it represents a parabola opening
downward which cuts .v-axis at (0,0) and (2,0). The sketch of the curve is as shown in Fig. 14.7.
14.16 APPLIED MATHEMATICS-XK

The required region is the shaded region in Fig. 14.7. Here, we slice this region into vertical
strips. For the approximating rectangle shown in Fig. 14.7, we have
Length = \y\, Width = dx and. Area = |i/| dx y

The approximating rectangle can move from .y = 0 to y = 2.


So, required area A is given by V)
2 2

/1=J \ij\dx=\ ydx [v j/>0 .-.11/1=1/]


0 0 y
2
o (2,0)
A = (2y-.y^) dx ●: P (y, y) lies on y = 2x -
, /
dx X

w
0

y = Zx- x~
3l" .84
V'

A= x^
X
= 4 — =— sq. units. Fig. 14.7

Flo
3 3 3 ‘
0

e
re
EXAMPLE 4 If the area above x-axis, bounded by the curves y = 2^ and y = 0 and x = 2 is , then
log, 2

F
find the value of k.
ur
r
kx
SOLUTION The graphs of y = 2'''', y = 0 and y = 2 are shown in Fig. 14.8 and the region bounded
fo
by these curves in first quadrant is shaded in Fig. 14.8. Let us slice this region into vertical strips.
ks
The approximating rectangle shown in Fig. 14.8, has length =|y|, width = dx and area =|i/| dx.
Yo

Clearly, it can move horizontally between y = 0 and y = 2.


oo
eB

Y
Y = 0

y = 2^
ur
ad

P{x,y)
Yo

x = 2
d
Re
in

o dx X
X'
F

Fig. 14.8
2 2

Area of the shaded region = J | j/| dx = J y dx [●.● y > 0 .-. I y I = y]


0 0
3
But, it is given that the area of the shaded region is .
log, 2
2

|y| dx=^
0 log, 2
2

1/ dx = [v y>0 .■.|y[=y]
0 log. 2
DEFINITE INTEGRALS 14.17

2
2
kx 3 2
kx
3 ^2k 1 3
I T' dx =
loge 2 k log,, 2 log,. 2 k log,, 2 k log,. 2 log^. 2
0 Jo

4*^-1
= 3
k

Clearly, k =1 satisfies this equation. Hence, k =1.


EXERCISE 14.3

Using integration, find the area of the region bounded between the line .v = 2 and the
2
parabola y =8.\-.
Using integration, find the area of the region bounded by the line y -1 =x, the .r -axis and

w
the ordinates .r = - 2 and x = 3.
2
j. Find the area of the region bounded by the parabola y = 4a.v and the line ,v = a.

Flo
2
4. Find the area lying above the Ar-axis and under the parabola y = 4.v- .y .
2

ee
5. Draw a rough sketch to indicate the region bounded between the curve y = 4.y and the line

Fr
.Y = 3. Also, find the area of this region.
2
= 4 Y and the
6. Find the area of the region in the first quadrant bounded by the parabola y
for
ur
lines .V = 0, y = 1 and y = 4.
7. Find the area of the region bounded by x^ = 16y, y = 1, y = 4 and the y-axis in the first
k s
quadrant.
Yo
oo

ANSWERS
eB

32 17 8 2 32
1. — sq. units — sq. units . — a~ sq. units — sq. units
3 ^ 2 ^ 3 3
r
ou
ad

7 56
5. 8-JS sq. units 6. — sq. units
3
-. — sq. units
3 ^
Y
Re
nd
Fi
mnn

DIFFERENTIAL EQUATIONS

15.1 SOME DEFiNITiONS

ow
DIFFERENTIAL EQUATION An equation containing an independent variable, dependent variable and
differentia! coefficients of dependent variable zvitb respect to independent variable is called a differential
equation.
For instance.

e
(i)
dy
= 2xy (ii) = 4.y (iii) ^ = sin .v + cos x

re
dx dx^ dx

rFl 1 3/2

F
2 ,2
d'^y r- /■ 2
(iv) + 2xy = x^ (V) —^-5^ + 6i/ = y (Vi) 1 +
dx dx^ dx dx

or
ou
2 3
a2
ksf
^y
(vii) y = Y — + 1 +
dx dx
(viii) {x^ + i/^) dx - 2.V1/ dy = 0 (ix)
j 3
+ 1 +
dx
= 0
dx
oo

are examples of differential equations.


Y

The order of a differential equation is the order of the highest


B

ORDER OF A DIFFERENTIAL EQUATION


order derivative appearing in the equation.
re

ILLUSTRATION 1 In the equation + 3 dx + 2y = e^, the order of highest order derivative is


oYu

dx
ad

n2
d^y Jdy - 4y = 0 is of order 3,
2. So, it is a differential equation of order 2. The equation
dx^ dx
d

because the order of highest order derivative in it is 3.


in
Re

NOi I The order of a differential equation is a positive integer.


DEGREE OF A DIFFERENTIAL EQUATION The degree of a differential equation is the degree of the
F

highest order derivative, when differential coefficients are wade free from radicals and fractions.
In other words, the degree of a differential equation is the power of the highest order derivative
occurring in a differential equation when it is written as a polynomial in differential coefficients.
d^\
ILLUSTRATION 2 Consider the differential equation— — - 4i/ = 0.
dx

In this equation the power of highest order derivative is 1. So, it is a differential equation of
degree 1.
-3 4

ILLUSTRATION 3 Consider the differential equation .v


. d^ +
dy
+ y 2=0
j 3
dx dx
V

In this equation, the order of the highest order derivative is 3 and its power is 2. So, it is a
differential equation of order 3 and degree 2.
15.2 MATHEMATICS-Xll

f .
dy
n.I.USTRATlON 4 Tlie differential equation ]/ = ;r + 1 + when expressed as a
dx
1 dx

(dif
polynomial in derivatives becomes {x~ -1) —
dx
- 2.ry ^
dx
+ {\/ -1) = 0. In this equation, the

power of highest order derivative is 2. So, its degree is 2.


2l ,2

ILLUSIRATION ' Consider the differential equation ● 1 + = 1C U

ow
dx dx^

The order of highest order differential coefficient is 2. So, its order is 2. To find its degree we
express the differential equation as a polynomial in derivatives. When expressed as a

e
2 3
^2
polynomial in derivatives it becomes k.2 d^y

re
1 + = 0. Clearly, the power of the
dx^ dx

Frl
F
highest order differential coefficient is 2. So, its degree is 2.
2 2
ou
n LUSl RATION f)
The differential equation {x +y ) dx-2xydy = 0 may be written

r
as

. So, it is a differential equation of order 1 and degree 1.


so
dx 2xy
kf
oo
I [.LUSTRATION "
Consider the differential equation y = px + ''
+ b^, where u = ^
dx
. The
Y

order of the highest order derivative is 1. So, its order is 1. To determine its degree we express it
B

as a polynomial in differential coefficients as follows:


re

1/ = px +
oY
u

iy-pxf = a^p^+b^
ad

=>
p^(x^ -a^)-2xyp + y^ = 0
d
in

_2 2_j2 ^ 0
Re

dx dx
V / \ J
F

Clearly, the power of highest order differential coefficient is 2. So, its degree is 2.
^2
dy
ILLUSTRATION s Consider the differential equation + sin — = 0. We observe that the
dx^ V
dx
/

highest order derivative present in the differential equation is d^y So, its order is 2. Since the
dx^'
differential equation cannot be expressed as a polynomial in differential coefficients. So, its
degree is not defined.
LINEAR AND NON-LlMcAR DIFFERENTIAL EQUATIONS A differential equation is a linear differential
equation if it is expressible in the form
ji -1 n-2
+ p1 d y d y dy
II-1 + P2 n-2 + ●●■+ 1 “T + y
dx dx dx dx

where Pq,P^,P2,...,P„ -1' P„ and Q are either constants or functions of independent variable x.
DIFFERENTIAL EQUATIONS 15.3

Thus, if a differential equation when expressed in the form of a polynomial involves the
derivatives and dependent variable in the first power and there are no product of these, and also
the coefficient of the various terms are either constants or functions of the independent variable,
then it is said to be linear differential equation. Otherwise, it is a non linear differential equation.
It follows from the above definition that a differential equation will be non-linear differential
equation if
(i) its degree is more than one.
(ii) any of the differential coefficient has exponent more than one.
(iii) exponent of the dependent variable is more than one.
(iv) products containing dependent variable and its differential coefficients are present,
d y
f ^ d y
f .2

w
ILLUSTRATION'' The differential equation -4i/ = 0, is a non-linear
j 3 j 2
dx dx
/ \

differential equation, because its degree is 3, more than one.


1 / n2

o
d^y dy
+ 9y = X, is a non-linear
ILLUSTRATION 10 The differential equation 1+2

e
dx dx

re
differential equation, because differential coefficient
rFl has exponent 2.

F
dx

L l USTRATlON 11 The differential equation [x^ + y^) dx - 2.vi/ dy =0 is a non-linear

r
differential equation, because the exponent of dependent variable y is 2 and it involves the
ou
product of 1/ and —.
dy fo
ks
dx
2

-5 ^ -I- 6y = sin .v. This is a linear


oo

1-LUfTRATlON 12 Consider the differential equation


dx^ dx
Y
eB

differential equation of order 2 and degree 1.


ILLUSTRATIVE EXAMPLES
r
ou

LXAMI’LE 1 Determine the order and degree of each of the following differential equations. State also if
Y
ad

they are linear or non-linear,


3/2

dyf
d

1 +
Re
in

dx
= K (ii)
d^y =1
(i)
^ dx
F

d^y
dx^
dy
(iv) y + ^ = jl ydx
c

(iii) i/=-f +
dx dy/dx dx 4 ●'

SOLUTION (i) The given differential equation when written as a polynomial in derivatives
becomes
3
f 2 f
K
2 y 1 +
dx^ dx

d^y
The highest order differential coefficient in this equation is dx ^ and
its power is 2. Therefore, the
given differential equation is a non-linear differential equation of second order and second
degree.
15.4 APPLIED MATHEMATICS-XII

(ii) The given differential equation when written as a polynomial in derivatives becomes
f 2 . \2
d\j + 1=0
dx^ dx dx^ dx^ dx
Clearly, it is a non-linear differential equation of second order and second degree,
n2
(iii) The given differential equation when written as a polynomialin^ is ^ dy
-y^ + c = 0.
dx dx dx

Clearly, it is a non-linear differential equation of order 1 and degree 2.


(iv) We have.

ow
2
, d\/ d u 1
y + y dx => ^ + —4r [ On differentiating with respect to x ]
dx 4 dx rf.v2 4-^
Clearly, this is a differential equation of order 2 and degree 1. Also, it is a linear differential
equation.

e
EXAMPLE :
In each of the following differential equations indicate its degree, wherever possible. Also,

re
give the order of each of them.

rFl fy

F
dy
(i) — + sin dy 0 (ii) + e
dx dx

or
' ■:} 2
ou
(iii)
d^y + sm —^ = 0 d^y dy
(iv) + cos = 0
dx^
ksf
dx^ \
dx

dy
oo

SOLUTION
(i) The highest order derivative present in the differential equation IS
i .So, it is of
dx
Y

cannot be expressed as a polynomial in ^ .


B

order 1. Clearly, LHS of the differential equation


dx
re

So, its degree is not defiired.


5
oYu

(ii) The highest order differential coefficient present in the differential equation is So, it is
ad

dx^
of order 5. We observe that the LHS of the differential equation is not expressible as a
d

polynomial in dx
. So, its degree is not defined.
in
Re

(iii) The highest order derivative present in the given differention equation is 4, so the order of
the given differential equation is 4. As it is not expressible as a polynomial in differential
F

coefficients. So, its degree is not defined,


(iv) The order of the highest order derivative present in the given differential equation is 2. So,
its order is 2. The given differential equation is not expressible as a polynomial in differential
coefficients. So, its degree is not defined.
EXERCISE 15.1
Determine the order and degree of each of the following differential equations. State also zvhether they are
linear or non-linear.
r . ^2
- fl .V d x dx
= e
t
- d^y + 4y = 0
dt
dx^
^2 ^2 ( 2
1
3.
dx
+ = 2 '1 + % _ ^d xj
dy/dx dx
DIFFERENTIAL EQUATIONS 15.5

r^.
dx
-f .VI/ = 0 6. 3 py _ dx
1 3/2
0 n2
c +
dy 8. .V + 1 +
dx"^ \
dx dx
1 dx

d^x 2 dh dt
9.y = 1/2+1 10. s
■^ + st ds
= S

dx/ ds

( d,1 y x4
/y /y dy
.T 2
dy 4
12. + 4 1/ = sin .V
11. + y + 1/ =0 +

dx dx^ dx^ dx

13. {x\/ + x) dx + (y-x^y) dy = {i ! 1.


^1 -1/2 dx + - x~ dy = 0
n2

w
2/3
15. d^y _(dy l(v 2
rf2y^-+ 3 1
dy
^ -y=0
dx 2 1 dx
rf,+ 1

F lo
dx
3/2
2

17. 5 /l- 1 + 4/ 18.


dx/
= x — + n
.
1 +
dy

e
dx^ dx dx
1 dx

Fre
r~2 2 ^ dy 20. + = 0
I'l. y = px + Jn p +b~, where p dx dx
for
4
^2
a2 ^2
...' X 2dSzr d'^y
r
dy d~y dy
=●! 1 + + X sin
You

dx^ ■ dx^ dx dx^


oks

dx
eBo

f , \
23. (y")^+(y')^+siny = 0 2-I
dx^
^ + 5x dx
-6y = logx
^2
d^x/ d^y dy . „ 2fK^P/ = .v2 log d^y
our
ad

dy
2d. —^ ^ + -^ + y sin y = 0
dx^ rfx2 dx dx rfx2
\3 ^2
dy dy
dY
Re

-4 + 7y = sin X
dx dx
Fin

ANSWERS

Order Degree Linear/Non-lincar


I
3 1 Non-linear
D 2 1 Linear
3 1 3 Non-linear
4 2 2 Non-linear
D 2 1 Non-linear
6 2 2 Non-linear
/ 4 2 Non-linear
8 1 2 Linear
9 2 1 Linear
'0 2 1 Non-linear
I ] 2 3 Non-linear
12 3 1 Non-linear
13 1 1 Non-linear
14 1 1 Non-linear
15.6 APPLIED MATHEMATICS-XI!

15. 2 3 Non-linear
16. 2 2 Non-linear
17. 2 2 Non-linear
18. 1 2 Non-linear
19. 1 2 Non-linear
:n. 1 1 Non-linear
21. 2 1 Non-linear
22. 2 Undefined Non-linear
23. 2 2 Non-linear
2 1 Non-linear

ow
3 1 Linear
f 2 Undefined Non-linear
1 3 Non-linear

H/Wrs ro SELECTED PROBLEMS

e
re
f
The differential equation

rFl
I+5.V - 6y = log .V has highest order differential

F
dx dx

coefficient d'^y
which is of order 2 and its exponent is 1. Hence, it is of order 2, degree 1 and

or
rf.v^
ou
it is non-linear.
ksf
The order of the highest order differential coefficient in the differential equation is one and
its highest exponent is 3. So, the given differential equation is of order 1, degree 3 and it is
oo

non-linear.
Y
B

15.2 FORMATION OF DIFFERENTIAL EQUATIONS

Consider the family of curves given hyy=A , where A is the parameter. For different values
re

oi A, we obtain different members of the family.


oYu

Differentiating y = Ae^ with respect to .y, we get = Ae\


ad

dx

On eliminating the parameter A between y =A e'^ and — =Ae^, we get — = y. This is the
d

dx dx
in
Re

differential equation of the family of curves represented by the equation]/ = Ae'^.


Thus, by eliminating one arbitrary constant, a differential equation of first order is obtained. In
F

other words, one parameter family of curves is represented by a first order differential equation.
Now, consider a two parameter family of curves given by
y = /I cos 2.Y + B sin 2.v ...(i)
where A and 6 are arbitrary constants.
Differentiating (i) with respect to .y, we get
— = - 2A sin 2.Y + 2B cos 2y ...(h)
dx

Differentiating (ii) with respect to .y, we get


^2
—- = -4A cos 2 .Y - 4B sin 2.y ...(hi)
dx^
Eliminating/I and B from equations (i), (ii) and (iii), we get

4y =>
d^y
I + 4y = 0.
_

dx^ dx
15.7
DIFFERENTIAL EL. ..A7I0NS

Here, we note that by eliminating two arbitrary constants, a differential equation of second
order is obtained. In other words, a two parameter family of curves is represented by a second
order differential equation.
Similarly, one can see that by eliminating three arbitrary constants differential equation of
a

third order is obtained or, three parameter family of curves is represented by a third order
differential equation.
Thus, from the examples cited above it can be concluded that if an equation involves « arbitrary
constants, a differential equation of nth order can be obtained by eliminating these n arbitrary
constants. In other words, an n-parameter family of curves is represented by an nth order

ow
differential equation.
Formulating a differential equation from a given equation representing a family of curves
means finding a differential equation whose solution is the given equation. If an equation,
representing a family of curves, contains n arbitrary constants, then we differentiate the given
equation n times to obtain n more equations. Using all these equations, we eliminate the

e
i
constants. The equation so obtained is the differential equation of order n for the family of given

re
curves.

Frl
F
In order to formulate a differential equation from a given relation containing independent
variable (.r) dependent variable (y) and some arbitrary constants, we may follow the following
ou
algorithm:

r
so
ALGORITHM

Write the given equation involving independent variable x (say), dependent variable ij {say) and
kf
the arbitrary constants.
oo
-ui-.i' i Obtain the number of arbitrary constants in Step I. Let there be n arbitrary constants.
Y

Differentiate the relation in step 1 n times zvith respect to x.


B

Eliminate arbitrary constants with the help of n equations involving differential coefficients
obtained in step III and an equation in Step I. The equation so obtained is the desired differential
re

equation.
oY
u

The following examples will illustrate the above procedure.


ad

ILLUSTRATIVE EXAMPLES
d

2 3
nx AMPLE I Form the differential equation of the family of cuives represented c {y + c) =x ,zvherecis
in
Re

a parameter.
SOLUTION The equation of the family of curves is
F

c{y + c)^ = ...(i)

Clearly, it is one parameter family of curves, so we shall get a differential equation of first order.
Differentiating (i) with respect to .v, we get
2c(y + c)^
dx
= ...(ii)

Dividing (i) by (ii), we get


c{y + c)^
3
.r 2x dy => c 1/
3 dx
dy 3.r
2c (y + c) dx

Substituting this value of c in (i), we get


\r
2
2.Y dy X
3
-y
~Jdx \
3 dx
/
15.8 APPLIED MATHEMATICS-XIl

2/-
4 ^ -y X

9[dxj 3 dx
2
4 ^ y = .V => 8.V- it -12y ^ = 27.V
27 dx 9 fr.Y dx dx
\

This is the required differential equation of the family of curves represented by (i).
i;XAMPLF.2
Form the differential equationof the family of curves represented byy =c{x - c)^,where c
is a parameter.
SOLUTION The equations of the family of curves is
ij = c(x~cf ...(i)
This equation contains only one parameter. So, we differentiate it only once. Differentiating (i)
with respect to x, we get

w
^ = 2c(.v-c) ●●●(ii)
dx

F lo
From (i) and (ii), we get
y_ _ c{x-cf
^.y 2c {X - c)

ee
dx

Fr
y x-c

dy 2 for
dx
ur
X -
2y
^ = A
dy
s
ook

dx
Yo

2y
c = x--f-
eB

dy
dx

Substituting this value of c in (i), we get


our
ad

n2

2y 2y
y =
dy
Y
Re

K dx){dx)
nd

dy .if dy . .
Fi

y
dx
= 4y x-^~
dx
ly
3
f|y .dy
dx
= 4i/ .Y -—
dx
2y , which is the required differential equation.

EXAMPLE-' Form the differential equation corresponding to y~ =a {b ~ x) {b + x) by eliminating


parameters a and b.
SOLUTION The equation of the family of curves is
y-=a(lr-x^)
Clearly, there are two arbitrary constants in this equation. So, we shall differentiate it two times
to get a differential equation of second order.
Differentiating (i) with respect to .v, we get
DIFFERENTIAL EQUATIONS 15.9

^ dy - dy ...(ii)
2i/— = -lax => y^ = -ax
' dx dx

Differentiating (ii) with respect to x, we get


n2 ,2 f dy
j
d^y dy d 1/
...(iii)
V + -a a
- >/—y+ j-
dx^ dx dx^ [dx

Substituting the value of a obtained from (iii) in (ii), we get

d^y +
dy
= y -2- ^ which is the required differential equation.
dx ‘ dx

2 2 2
EXAMPLE 4 Form the differential equation corresponding to y =m{a -x ) by eliminating

w
parameters m and a.
SOLUTION The equation of the family of curves is
y^ = m(a^ -x'^)

F lo
...(i)
This equation contains two parameters. So we shall differentiate it two times to get a differential
equation of second order.

ee
Differentiating both sides of (i) with respect to x, we get

Fr
2y
dx
= m{-2x) => y^
dx
= -mx for ...(ii)

Differentiating both sides of (ii) with respect to .r, we get


ur
2

h. dy ...(iii)
s
-m
y
ook

dx^ dx
Yo

Putting this value of -m in (ii), we get


eB

y
d^y + dy]\ = y
dx'
which is the required differential equation.
dx^
our

dx
ad

EXAMPLE 5 Find the differential equation of all circles touching the


(ii) y-axis at the origin
Y

(i) x-axis at the origin


SOLUTION (i) The equation of the family of circles touching .v-axis at the origin is
Re
nd

(.Y - 0)^ + (y - af = Y^ + y^ - 2fiy = 0 ...(i)


Fi

where n is a parameter.
Y

X' O X

Fig. 15.1
15.10 APPLIED MATHEMATICS-XII

This equation contains only one arbitrary constant. So, we differentiate it once with respect to x,
so that

lx + 2i/ '^-2f?^ = 0=> a = x-\-\j{dy/dx)


...(h)
dx dx dy/dx
Putting the value of n from (ii) in (i), we get
X + y (dy/dx)
+ / = 2y
dy/dx
2 2
(.r -1/ )-=- = Ixy, which is the required differential equation.

ow
dx

(ii) The equation of the family of circles touching y-axis at the origin is
(x-af+(lj-0f a => x^ + y^ - 2nx = 0
2
...(i)
where a is a parameter.

e
y

re
rFl
F
or
o
ou
X' («<0) X
ksf
oo

Y'

Fig. 15.2
Y
B

This equation contains only one arbitrary constant. So, we differentiate it only once with respect
re

to -V, so that
dy <^y
lx + 2y - la = 0 a = x+y
oYu

...(ii)
dx dx
ad

Putting the value of a from (ii) in (i), we get


d

2
.V +y ^-2.v x+ = 0
in

dx
Re

dy
F

or. y -x= 2xy


dx '
which is the required differential equation.
I \
Find the differential equation of all the circles in the first quadrant which touch the
coordinate axes.

SOLUTION The equation of the family of circles in the first quadrant which touch the
coordinate axes is

(x - a)^ + (y - a)^
2
a ...(i

where i? is a parameter.
This equation contains one arbitrary constant, so we shall differentiate it once only to get a
differential equation of first order.
Differentiating (i) with respect to .y, we get
2(x-a)^2(y-a)^
dx
=0
DIFFERENTIAL EQUATIONS 15.11

x-a + iy -a)^
dx
=0
a (a, a)
x+ij
dx a
0

1 +
dy
X’ O X
dx
-y + py dy
n , where p =
1 +p dx
V'
Substituting the value of a in (i), we get Fig. 15.3
/
2
r

ow
X + py ^ + py
+ y-
\ +p 1 + p 1+p

{xp - p\fr + {y- x)^ = (.V + pyf


(.V - y)^ p^ + {x - \jf = (-Y + py)^

e
(Y-y)^(p^+l) = (Y + py)^

re
^2

rFl a2

F
rfy ^y
(Y-y)^ 1 +
dx
Y + y -r
dx ^
, which is the required differential equation.

r
ou
Form the differentinl equation offamily of parabolas having vertex at the origin and axis
liXAMPI I ●

along positive y-axis. fo


ks
SOLUTION The equation of the family of parabolas having vertex at the origin and axis along
oo

positive y-axis is
2
= 4<?y , where/7 is a parameter. ...(i)
Y

.V
B

This is a one parameter family of curves. So, we differentiate it once only.


re

Differentiating wtih respect to .v, we get


_2.V = ^ ^
dy .Y
ou

<7
Y
ad

dx
dx

Substituting the value of a in (i), we get


d

.Y 2 =4x
x Jy
= 2i/, which is the required differential equation.
in
Re

vxy^Y
dx
2 ^
F

dx

liXAMPLI H Form the differential equation of the family of ellipses havingfoci on y-axis and centre at the
origin.
SOLUTION The equation of the family of ellipses having centre at the origin and foci on y-axis is
2 2
X y
—-2 + —- - 1, where b > a
a b^
This is two parameter family of ellipses. So, we differentiate (i) twice with respect to.Y to obtain a
differential equation of order 2.
Differentiating with respect to -y, we get
...(ii)
a

Differentiating (ii) with respect to x, we get


15.12 .APPLIED MATHEMATICS-XII


1
+ - f^’/f y = 0
...(iii)
a dx^
Multiplying throughout by x, we get
f , ,2
X
A- di, xy dy
~2 ^ ...(iv)
a dx dx^ ■
Subtracting (ii) from (iv), we get
^2 ^2
1
x{^ + xy
d^y ~y = 0 => .V. ^y n
= 0,
+ xy
b^ dx dx^ dx
\
dx dx

which is the required differential equation.

w
Form the differential equation not containing the arbitrary constants and satisfied by the
equation y = ae^^^, a and b are arbitrary constants.

F lo
SOLUTION The given equation is
bx
y = ae

ee
Fr
Clearly, it contains two arbitrary constants. So, we shall differentiate it two times and get a

differential equation of second order.


Differentiating (i) with respect to .v, we get for
ur
■b
dx
s
ook
Yo

^
dx
= by [By using (i)] ●●●(ii)
eB

Differentiating (ii) with respect to .v, we get


d^y
...(iii)
r

dx
ad
ou

1 dy
From (ii), we get/? = -f-. Substituting this value of b in (iii), we get
y
Y

2 r \ ^2
Re

d y 1 dy dy
nd


rf.v^ = - 7^ ^ => y dx^
, which is the required differential equation
y dx J dx dx
Fi

Show that the differential equation representing one parameter family of curves
-y^)=c{x^ +y~)'^is(x^-3xy‘^)dx = {y^~3x^y)dy
SOLUTION The given equation of one parameter family of curves is

...(i)
Differentiating (i) with respect to .v, we get
dy
2x-2y
dx
= 2c(a^ + y^) 2a + 2y dx

<iy - , 2 2v dy
^-yff- = 2c(a + }/ ) A + l/^ ...(ii)
dx dx

On substituting the value of c obtained from (i) in (ii), we get


15.13
DIFFERENTIAL EQUATIONS

X + y -f-
dx (.v^ + i/“) dx

dx = 2(/-/) -^' + y£
^y
x(x^ +y^)-2x{x^ -i/) dx
2y (x'^ - ih + y {x^ + }h

{3x/-x^) = ||(3/y-y’)
(x^ - 3x:/) dx = (i/^ - 3x^1/) dij, which is the given differential equation.

w
rXAMPLE I i Represent the folloxving family of curves by fowling the corresponding dijferentia!
equations (a, b are parameters):
(ii)
X
IX = 1 (iii) (}/-&)"= 4 (x-fl)

o
= 1
a b a

e
The equation of the family of curves is

re
SOLUTION

=1

rFl ...(i)

F
a b

where a, h are parameters,


two parameter family of curves. So, we will differentiate it twice with respect to x.

r
It is a
ou
Differentiating (i) with respect to x, we get fo
ks
= 0
...(ii)
a b dx
oo

Differentiating (ii) with respect to x, we get


Y

1 d^y d^y = 0, which is the required differential equation.


eB

= 0 =>
b dx^
(ii) The equation of the family of curves is
r
ou

2 2
...(i)
Y
ad

=1
a ^ b^
It is a two parameter family of curves. So, we will differentiate it twice to obtain the differential
d

equation.
Re
in

Differentiating with respect to x, we get


X + XiL = 0 ...(ii)
F

Differentiating (ii) with respect to x, we get


= 0
a x[dx] }P- dx^
Multiplying both sides by .x, we get
2 ,2
X
. X dy. +£y^ = o ...(iv)
a

Subtracting (ii) from (iv), we get


1
— 'U ^
(dy^ +xy—
d^y dy
—^ = 0
dx
b^ dx
15.14
APPLIED MATHEMATICS-Xil

^2 2
X
dx
+ xy
~y~^ ~ which is the required differential equation.
(iii) The equation of the family of curves is
{y-bf = 4(,V-^I) ...(i)
It is a two parameter family of curves. So, we will differentiate it twice with respect to .v.
Differentiating (i) with respect to x, we get
2(y-b)^ = 4=> (y-i)i'y
dx
=2 ...(h)

Differentiating with respect to x, we get


^2
y dy
[y-b)~ + = 0
...(hi)
dx^ dx

w
2
From (ii), we get i/ - & = —
dy'

F lo
dx

Substituting this value of y-h in (iii), we get

ee
\2 \3
2 d\j dy

Fr
= 0 => 2
^y dx^ \
dx dx^ dx = 0, which is the required differential equation.
dx for
LXAMI'LF. 12
Obtain the differential equation of all circles of radius
ur
r.
SOLUTION The equation of the family of circles of radius r is
{x-af +{y-b)^ = r^
s
ook

-(i)
Yo

where a and b are a parameters.


eB

Clearly equation (i) contains two arbitrary constants. So, let us differentiate it two times with
respect to .t.
Differentiating (i) with respect to .r, we get
r
ad
ou

2(x-a) + 2(y-b)^
dx
=0
Y

(x-a) + (y~b)^ =0 ...(h)


Re

dx
nd

Differentiating (ii) with respect to :r, we get


Fi

l+(y-/;)
A = 0
dx^ tix ...(hi)
j

y-b = -
1 + {dyidxf
...(iv)
d^y/dx^
Putting this value of (y - &) in (ii), we obtain

1 + dy dy
dx dx
\

X - a

Substituting the values of (.v - a) and (y - b) in (i), we get


15.15
DIFFERENTIAL EQUATIONS

2 2
^2
ay dy 1 +
dy 3
1 + n2
dx dx dx
2
+ r 1 +

(d^yldx^) dx rf.v^
(d^y/dx^)
This is the required differential equation.
I \AMPLE13 Shoiv that the differential equation that represents the family of all parabolas having their
axis of symmetry coincident uhth the axis of x is yy2 + = 0-
SOLUTION The equation that represents a family of parabolas having their axis of symmetry
coincident with the axis of x is

w
i/ = 4a (x - h)
where a and h are parameters.
This equation contains two parameters a and h, so we will

o
differentiate it twice to obtain a second order differential

e
equation.

re
X
Differentiating (i) with respect to .v, we get

rFl X'
o

F
= 4«
dx

dy _

r
y-r = 2a ...(ii)
ou
dx
Differentiating (ii) with respect to x, we get
fo 1/2 = 4rt{.v - h)
ks
^2
d^y dy = 0
oo

+
y
dx^ dx
Y

yyi y?' ~ which is the required differential equation.


eB

EXAMPLE I- Find the differential equation of


(i) all non-horizontal lines in a plane (ii) all non-vertical lines in a plane ● i -v
r

SOLUTION The general equation of a line in a plane is ax + by =l.If it is parallel to .x


axis, *
ou

is oMhe form y Iconstant. So, 1. ^ 0 and « = 0. Also, if «.v by = 1 represents a line paralle to
Y
ad

y-axis, then it should be of the form x ^constant. So, ^ 0 and b = O.Thus, ax + by -1 represents
family of non-horizontal lines in a plane, if ^ 0 and a family of non-verhcal lines, i
d

(i) The equation of the family of non-horizontal lines in a plane is ax + by = 1, where a * 0 and b
Re
in

can take any real value. It is a two parameter family of curves with a 0 and f? e K.
Now, iix + by
F

Differentiating both sides with respect to y, we obtain


dx
a^ + b =0
dy
Differentiating both sides with respect to y, we obtain
d^x d^x = 0
[V a^O]
a = 0 =>
dy^ dy^
This is the required differential equation. , , n j p

(ii) The equation of the family of non-vertical lines in plane is flx + by = 1, whereb ^ 0 and e K.
It is a two parameter family of curves with b ^ 0 and a eR.
Now, ax + by = 1
Differentiating both sides with respect to x, we obtain
a + b^ = 0
dx
15.16
APPLIED MATHE.MATICS-XII

Differentiating both sides with respect to x, we obtain


= 0 => = 0
dx^ [●●● b 0]

Tliis is the required differential equation.


EXERCISE 15.2

!● Form the differential equation of the family of curves


represented by i/^ = (:r - c)
2. Form the differential equation corresponding to y = by eliminating m.
^ r°bSary^ differential equations from the following primitives where constants are
(i) =4ax (ii) y = c.v + 2c^ + c^ (iii) .ry=^7
2
(iv) y = ax^ +bx + c

low
■1. Find thedifferentialequationof the family of curves y=Ae
2.V ~2x
+ Be , where A and B are
arbitrary constants.
>. Form the differential equation representing the family of curves y = e 2.r (a + bx), where a and
b are arbitrary constants.
6, Form the differential equation corresponding iox/= a {b- x^) by eliminating a and b.

ee
rF
7. Form the differential equation corresponding to y^ - 2rty + x^

Fr
2
by eliminating a. = a

8. Form the differential equaHon corresponding ot (x~nf+(y ~bf by eliminating


and b.

r
fo
9. Find the differential equation of all the circles which pass through the origin and whose
u
centres he on y-axis. °
ks
10. Find the differential equation of all the circles which pass tlirough the origin and whose
Yo

centres he on .r-axis. o a
oo

11. Assume that a rain drop evaporates at a rate proportional to its surface area. Form a
B

differential equation mvolving the rate of change of the radius of the rain drop.
pSalieUo'x equation of all the parabolas with latus rectum '4a' and whose axes are
u re

13. Show that the differential equation of which y = 2(.v^ -1) +


ad

is a solution, is
Yo

^ + 2.V1/ = 4.v^.
dx
nd

14. Form the differential equation havmg y=(sin ^xf+Acos \+B, where A and B
Re

are

arbitrary constants, as its general solution.


Fi

d°JparamttSy^*^^^‘^^ equaUon of the family of curves represented by the equation (a being


(i) (2.V + + y2 = (ii) (2x - a)^ - \/
^ (iii) {x-af + 2x/ =
= a

16. Represent the following families of curves by forming the corresponding differential
equations {a, b being parameters): °
(i) x^ +y^ = a^ (ii) .Y^ - y^ 2
(iii) t/2 = 4rt.v
(iv) .Y^ + (y-bf = 1
2

(v) {x-af-y y
^ = 1
●y

(vii) y“ =4a(x-b) (viii) y = ax^


b^
(ix) .Y^ + y^ = ax ,«.Y

Sd"^ocron?a"xr/‘^' equation representing the family of ellipses havmg centre at the origin
18. Form the differential equation of the family of hyperbolas having foci on Y-axis and centre
at the origin. vr &
15.17
DIFFERENTIAL EQUATIONS

19. Form the differential equation of the family of circles in the second quadrant and touching
the coordinate axes.
i).r+ 5
, where a and b
20. Find the differential equation representing the family of curves y~ae
are arbitrary constants.
ANSWERS

1. 8
dy
dx
= 27.1/ 2. .t^
dx
= i/logy
\3
Jy ^y
/
<^y d\/
3. (i) 2.r + (iii) 1/ + -.C ^ =0 (iv) = 0
dx dx dx dx dx^

4.^ = 4y 5. ^-A^
dx^ dx
+ Ay = 0
dx^
^2
, dy ^y 7. {x^ - l\h - 4^-^/ - = 0, p = dx
6. y^-x +
dx

8. (1+p )
2s3
dx

^2 ^
dx^
dx

\2
dy
dx
F low
9. (Y^ -1/^) ^
dx
= 2.Yy forFe
dr
10. {x^-i/) + 2xy dx
= 0 11. —=-k
dt

2^ ^^y x^-2 = 0
12. 2fly2 + y/ = 0 14. (1-Y )
dx^ dx

(iii) 2y"-^^=4.vy^
eBoks

15. (i) y^ -4y^ -2Yy^ = 0 (ii) 2y ^ = 4.y^ + i/


Your

dx
ad

16. (i) Y + y ^ =0 (ii) Y-y^ = 0


dy (iii) y - 2y ^ =0
our

dx dx
dx

/ , \2
Re

2 dy
(iv) Y- 1 + (v) y -J/ ^ =1
dx dx dx
FindY

\2
(Vi) -V y
d^y = y-r
dy
(vii) y
d~ y +
dy' = 0 (viii) Y ^ = 3y
dx
■ dx^ rf.Y
y
dx dx^ dx

(ix) Y^ + 3y^ = 2Yy £ (x) Y ^


dx
= y log y
a2
17. xy
d^y + Y
'dy^ -y"^
dy
=o P. Yy
d^y + Y
dx
dy
-y^
dx
=o
dx^ dx dx

\2 a2
dy dy d^y J dy
19. Y + y
dx ,
= {x + yf\l + dx
20. y
dx^ dx
15.18
APPLIED MATHEMATICS-Xll

HINTS TO SELECTED PROBLEMS

1. The equation of the family of curves is i/^ = (.v - c)


Differentiating with respect to a:, we get

rw
2i/ iJi = 3(x~cf
=> =27 (x~cf
=> 8y^t//=27(yV [v (.Y-c)^ = l/]
3
=> 8i/j = 27y, which is the required differential equation.

r
luo
The equation of the family of curves is y = e'”^.

F
Differentiating with respect to .%●, we get
mx
\Jl =me => y^ =wy.

oF
Now, logy
y =c => log y = mx => m =

rs
X

Substituting this value of m in yi = im/, we get .vyi =ylogi/, which is


i the required

ok
differential equation.
. (i) The equation of the family of primitives is =4ax.

fo
Differentiating with respect to .v o
1 tfi/
Y
2y = 4a => a = -y ~
Y
dx 2 ‘ dx
rB

Substituting this value oin in 4 ax, we gety^ 4^ f ~}/~


1 di/ ....
^y
or y=2-f^ as the required
1^2 dx dx ^
differential equation,
ue

(ii) The equation of the family of primitives is


d

i/ = cx + 2c^ ^ =c
no

dx
ad

Substitute this value of c in y - cy + 2c^ + c^, to get the required differential equation,
(iii) The equations of family of primitives is xy=a^.
i

Differentiating with respect to y, we get


Re
F

dy
X -2-
dx + y = 0, which is the required differential equation.

(iv) Differentiate y three times to get d y = 0 as the required differential equation.


dx^

Differentiatey two times to get d^y = 4y as the required differential equation.

7. The equation of one parameter family of curves is y^ - 2rty + x^ = n^.


Differentiating with respect to y, we get
Y +y
,^y
2y ^
dx - 2fl ^
dx + 2y = 0 ^ (7 = ^y
dx

dx

Substituting the value of a in y^ -2ay + x^ =a^, we


obtain the required differential
equation.
15.19
DIFFERENTIAL EQUATIONS

12. The equation of the family of parabolas is (y -k)^ = 4a {x- h), where h and k are arbitrary
constants.
Differentiate this relation twice and eliminate h and k to get the differential equation.
15. (iii) The equation of the one parameter family of curves is
(x - af + ly^ = a^
Differentiating with respect to x, we get

2(j:-<l) + 4y^ = 0=> x-fl = ^


Substituting the value of fl in (i), we get
/ \2 / A ^
4y
2{dy
dx
^2/ + => 2y^-x^ = 4xy^
This is the required differential equation.

w
17. The equation of the family of ellipses having centre at the origin and foci on x-axis is
^+ =1
a 0

It is a two parameter family of curves.


F lo
ee
Differentiating (i) twice with respect to x, we get

Fr
= 0
for
^2 dx ^2 \^dx) ^^2
ur
= 0 ...(iii)
...(ii) and.
b'^[dx) dx^
= 0
«^\2 dx
s
ook
Yo

Multiplying (iii) by x and subtracting from (ii), we get


eB

n2
/ - \2
~^y
d^y = 0 => xy + X
dy dy
-y-r = 0
^2 dx l^dxj dx^ dx^ dx
/
dx
our
ad

This is the required differential equation.


18. The equation of the family of hyperbolas having foci on x-axis and centre at the origin is
Y
Re

= 1
...(i)
nd

/ Ip-
Fi

Differentiating this twice with respect to x, we get


= 0 and l_Xf 2y d^y = 0

P P dx P P U^J P dx^
= 0
i/jyf y d^ y = 0 ...(iii)
^ a ^ b^dx 2 b'^[dxj
a b^ dx^
Multiplying (iii) by x and subtracting from (ii), we get
1 dy d^y = 0 xy d^y + X >1 ^ dx
= 0
T -y-r:
dx
+ X
dx
+ xy
dx^ dx^ [dxj
This is the required differential equation.
15.20
APPLIED MATHEMATICS-XII

* equation of the family of circles in second quadrant and touching the coordinates axes

is (x + rt) + (y - af- = , where (T > 0 or, + \/ + lax - lay + a^ = 0


Differentiating (i) with respect to .v, wc get
lx + 2y
dx
+ la-la^ = 0
dx

^y dy
=> x+y^ + a 1 = 0 => a _ ^ + py where p =
dx V
dx ^ dx

Substituting the value of in (x + a)~ +{ij-a)^ = a


2
, we get
2 2 2

X +
X + py ^ + py _ x + pij
+ y-
p-i P-1 p-i

^ (x + i/f p^+(-y-xf = (,v + pijf ^ (,v + ijf (1 + p^) = (x + pijf^ wherep=^

w
dx
This is the required differential equation.
15.3 SOLUTION OF A DIFFERENTIAL EQUATION

F lo
SOLUTION The solution of a differential equation is a relation between the variables involved which

ee
Fr
satisfies the differential equation.
Such a relation and the derivatives obtained therefrom when substituted in the differential
for
equation, makes left hand, and right hand sides identically equal.
For example, y = c is a solution of the differential equation ^ = y.
r
You
s

Consider the differential equation


ook
eB

+y = 0
dx~ -(i)
Also, consider the relation y = A cos x + S sin x ●●●(ii)
our
ad

where A and B are arbitrary constants.


Differentiating (ii), with respect to x, we get
^y
dY

= - A sin X + 6 cos x
Re

dx

Differentiating this with respect to x, we get


Fin

d^y =_ - A cos X - B sin x => rf2y d^y


dx^ = -y =i> + y=0
rfx-2 rfx-2
This shows that
y = A cos X + B sin X satisfies the differential equation (i) and hence it is a
solution of differential equation given in (i).
It can be easily verified that y = 3 cos x + 2 sin x, y = A cos x, y - B sin x etc., are also solutions
of differential equation given in (i).
We find that the solution y = 3 cos x + 2 sin x does not contain any arbitrary constant whereas
solutions y = A cos .y, y = 6 sin x contain only one arbitrary constant. The solution y = A cosx
+ 6 sin X contains two arbitrary constants, so it is known as the general solution of (i) whereas all
other solutions are particular solutions.
GENER AL SOLUTION The solution which contains as mamj as arbilranj constants as the order of the
differential equation is called the general solution of the differential equation.
15.21
DIFFERENTIAL ECUATICNS

For example, y = A cos + B sin x is the general solution of the differential equation
+ i/=0.
dx^
But, y = A cos .Y is not the general solution as it contains one arbitrary constant.
PARTICULAR SCLJ7I0N Solution obtained by giving particular values to the arbitrary constants in the
general solution of a differential c(]uation is called a particular solution.
For example, ^ = 3 cos.Y + 2sin .y is a particular solution of the differential equation (i).
ILLUSTRATIVE EXAMPLES
B
siiozo that y = Ax + - , ,Y 0 is a solution of the differential equation

w
EX.^MPLE 1
^ X
2 d^y
X —^ + x-^-y = 0
dy f.
dx^ dx

o
SOLUTION We have,

e
re
1/ = A.y + — , .y 0
.Y

rFl
F
Differentiating both sides with respect to .y, we get
^ = A- — ... (ii)

r
dx
ou
Differentiating with respect to .v, we get fo
ks
d^y _ ^3 ...(hi)
oo

.Y

^ ^ in x^ + -V ^ - y, we get
Y

dy
eB

Substituting the values of y, dx^ dx


dx-
,2
B B 2B B B
id—^y . ^y 2 2B + Ax Ax-- = 0
r

X + Y /4.Y +
-y = 3 Y Y
ou

dx^
x
dx X X !
X
Y
ad

,2
B _1 d y = 0.
Thus, the function i/ = Ay + - satisfies the differential equation y dx
d
Re
in

Hence, y = Ax + —X is a solution of the given differential equation.


F

3.V d^y 6 + 9y = 0
Shoxv that the function y ={A + Bx)e is a solution of the equation
dx^ dx

SOLUTION We have,
3.V ...(i)
y = (A + By) e
Differentiating (i) with respect to y, we get
^
dx
{A + By) ...(ii)

Differentiating (ii) with respect to .y, we get


d^y = 6Bc’^-'' + {A + By) ...(hi)
dY^
. ^ 6^?^ + 9t/= {66e 3.r + 9e 3-v {A + Bx)\- 6 {Be^^ + 3c 3-r (^ + 6y)1+ {9 {A + Bx)e^'‘}^0
■■ dY^ dx
15.22
APPLIED MA7HEMATICS-XII

3.r
Thus, 1/ =(/l + Bx)e satisfies the given differential equation. Hence, it is a solution of the given
differential equation.
2.V
J XAMPLH3 Show that y = ae + be ^ is a solution of the differential equation
dx^ dx
SOLUTION We have,
2.V
+ be
...(i)
Differentiating (i) with respect to x, we get
^
dx
= 2ae-^-be- ...(h)

Differentiating (ii) with respect to x, we get


d^y =_ 4ae
2x
+ be~^
dx^

w
= {4ae^’^ + be '^)-(2rtc 2x -be '^)-2{ae 2x + be "')=0

F lo
dx dx
2.V
So, y = ae + be ^ satisfies the given differential equation.
Hence, it is a solution of the given differential equation.

e
I XAMPLi: 4 Show that \J = cx + -isa solution of the differential equation
Fre \/ = x^
a
+
^ ' dx dy
for
dx
SOLUTION We have,
a
r
y = cx + -
You
oks

Differentiating with respect to .x, we get


eBo

^=c ...(h)
dx
ad

dy
our

a a
x-^ + —
dx dy
xc + -
c Putting ^
dx
=c
dx
a
Re

dx ^ dy =y
dY

[Using (i)]
dx
Fin

This shows that y - c.x + - is a solution of the given differential equation.


2
rxAMPLE s Show that xy = ae'^ + be~^ ^ + X is a solution of the differential equation
2
d y ry dW 2
x^ + 2-f--Xi/ + x -2 = 0.
dx^ dx '
SOLUTION We have,
xy = ae''‘ + be~ + x^ ... (i)
Differentiating with respect to .x, we get
dy V , -r
x~+y = ae -be ^ + 2x
dx ■

Differentiating this with respect to .x, we get


15.23
DIFFERENTIAL EQUATIONS

+ +^ = ae + bc~^ + 2
dx^ dx dx

d^y - -^
<^y = ae X + be
I -Y +2
, T ...(ii)
dx^
Using (i)
x^
dx^
+ 2^-xy + x^-2={ ae^ + be~''^ + 2I-( ne^ + be^ + -y^ / \
+ Y ^ -2 = 0
and (ii)

2 .
Thus, xy = cie^ + be '^ + y' is a solution of the given differential equation.
EXERCISE 15.3

2.V . d\j 3^+2y = 0.

low
I. Show that y = be^ + ce IS a solution of the differential equation,
dx^ dx

Verify that 1/ = 4 sin 3y is a solution of the differential equation dx


|. + 9y = 0.
d'^y dy - 2y = 0.

ee
2x
Show that 1/= f7i’ + be ^ is a solution of the differential equation
rF dx^ dx

Fr
4. Show that the function y = >\ cos y + B sin y is a solution of the differential equation
-i + y=o-
r
dx
fo
u
For each of the following differential equations verify that the accompanying function is a
ks
solution:
Yo

Function
oo

Differential equation
B

}/ =
dx
re

1 2 2
(ii) x + y^ = 0
= + a - Y
1/
dx
u
ad

a
Yo

(ill) Y ^
dx
+y = y y =
Y + a

3^ = 1
1
nd

1/ = ax+ b + ^
Re

(iv) Y ■’ 2y
rfY^
Fi

x2
^y 1 / _L_ ^2
(V) y = y = 74 (Y ± a)
dx
1 / ,

Bx ■ y _ M ^y
6. Show that y=Ae is a solution of the differential equation
dx^ y dx

2 dy = 0.
7. Verify that y = -- + & is a solution of the differential equation dx^
+ -
x dx

dy dx

8. Verify that y^ = 4rt.v is a solution of the differential equation y =


Y —
dx
+ « — ●
^y
,2 f j
dy
d y , ^
9. Show that Ax'^ + By^ = 1 is a solution of the differential equation y ■ y dx^ dx J .
=y-r-
dx
15.24
APPLIED MATHEMATICS-XIt

3 2 /V ^
10.
Show that y = ax + bx + c is a solution of the differential equation ^^ = 6a.
dx^
-X
11.
Show thaty = — is a solution of the differential equation (1 + x^) — + (1 + y^) = 0.
1 + cx dx
2
12.
Show that y = e ' +ax + b is solution of the differential equation ^ = 1.
dx^
2
13.
Verify that y - c.t + 2c“ is a solution of the differential equation 2 ^ + x^-y = 0.
dx
J4.
Verify that y = - x - ] is a solution of the differential equation (y — .v) dy - (y^ - x“) dx = 0.
2
2
Verify thaty =4a (.r + is a solution of the differential equations = 2.t.^y
15.
y 1 -

w
dx dx

F lo
2
16. Show that the differential equation of which y = 2 {x^ -1) + ce~ is a solution is

^+2.vy
dx = 4.v^

ee
^2

Fr
i 7. Verify that y ^ log x + ^ + a
2
satisfies the differential equation
for
, 2 , 2> d^y + .v!^'=0.
(a + A- ) —^
r
dx^ dx
You
s
ook

15.4 METHODS OF SOLVING A FIRST ORDER FIRST DEGREE


In this section, we shall discuss several techniq of obtaining solutions of following types of
eB

ues

differential equations.
(i) Differential equations of the form ^ =/{x).
our
ad

dx

(ii) Differential equations of the form ^


dx
=/(y).
(iii) Differential equations in variable separable form.
dY
Re

(iv)
Differential equations reducible to variable separable form.
Fin

(v) Homogeneous differential equations.


(vi) Linear differential equations.

15.4.1 DIFFERENTIAL EQUATIONS OF THE TYPE

To solve this type of differential equations we integrate both sides to obtain the general
as discussed below. ®
solution
We have.
^y
— =f{x) o dy = /(x) dx
Integrating both sides, we obtain
^y J /(t) dx + C or, y-j f(x) dx + C , which gives the general solution of the
differential equation.
Following examples will illustrate the procedure.
15.25
DIFFERENTIAL EQUATIONS

ILLUSTRATIVE EXAMPLES
EXAMPLE 1 Solve:

(ii) dx
=(/-<>-')
+1

(Hi) (x+2) ^ = .v^ +4x-9 ,x*-2


dx

SOLUTION (i) We have,


^ - X

dx .v^+l

w
X
dy = dx
x“ + l

Integrating both sides, we get

o
X
’ dy^ dx

e
x^^l

re
dy = -
1 2x
dx

rFl
F
2 ^ .v^+l

i log 1 +11 + c

r
y =
ou
fo
ks
Clearly, y = log | + 11 + C is defined for all .y e R.
oo

Hence, i/ = - log | + 11 + C , xeR is the solution of the given differential equation.


Y
eB

(ii) The given differential equations is


, V — X\ dy , X — i\
{e + e ) = (^ - ^ )
dx
r
ou

X - X

dy _ e - e
Y
ad

X - X
dx e + e

X - X
d

-e
, e dx
dy = -X
Re
in

- X
e + e
F

Integrating both sides, we get


X - X
e -e
dx
dy = .r - .V
' e'' + e

dy = j ^ , where e^ + e
y = log 1 f I + c
y = log \e^ + e I + C.
Clearly, y = log [ + e ^ | + C is defined for all xeR.
Hence, y = log | + e 1 + C , Y e R is the solution of the given differential equation.
(iii) We have.
(.Y + 2) ^
dx
= Y^ + 4y - 9
15.26
APPLIED MATHEMATICS-Xll

d\j _ + Ax ~9
dx x + 1 [v x.^-2]

x^ + 4.Y - 9
dxj = dx
.r + 2

Integrating both sides, we get


+ 4x - 9
J x + 2
dx

13
d\j ^ x + 1- dx
x + 1

w
.V
y =
y + 2x-131og|x + 2|+C
Clearly, it is defined for all x^R, except x = - 2

Flo
:,.2
Hence, y - y + 2.T-13 log | x + 2 [ + C, x e R - (2) is the solution of the given differential

ee
equation.

Fr
dy _ 3c^-^‘ + 3c
4,t
I XAMI'LI 2 Solve:
dx e^ + e ^
for
ur
SOLUTION We have,
^ _ 3e^-'~ + 3c
4.V
s
dx + e~^
k
Yo

d^ ^ 3c^-' (1 +1 e^^) ^ 3c^-^' (1 + _ 3c^'‘' (1 + c^^)


oo
eB

dx 2.x
c-% ^ e + 1
.X
e .X
e

^y =_ 3.V
r

3c
ou
ad

dx
3.V
dy = 3c dx
Y

rfy = 3 c^"' dx
[Integrating both sides]
Re
nd

3.T

y = 3 ^ + C
Fi

3.r
y - +C , which is the required solution.
fXAMi’Lt: “ Solve the initial value problem =x + 1; y(0) =5.
SOLUTION We are given that
.v + 1

^ = log (x + 1)
dy = log (x + 1) dx
Integrating both sides, we get
j 1-dy log (x + 1) ● 1 dx
1 II

^ J
y = X log (x + 1) - dx
x + 1
15.27
DIFFERENTIAL EQUATIONS

rf.v
y = A- log (a + 1) - A + 1

=> y = A log (a + 1) - 1-^ dx


A + 1

●●■(i)
\J = X log (a + 1) - -V + log (a +1) + C
It is given that y(0) = 5 i.e., when a = 0, we have i/ = 5.
5 = log 1 - 0 + log 1 + C [Substituting a = 0, y =5 in (i)]

ow
C = 5

Substituting the value of C in (i), we get


y = A log (a + 1) - A + log (a + 1) + 5
We observe that a log (.v + 1) - a + log (.v + 1) + 5 is define- '.'r all a e {-1 , ^).

e
Hence, y = x log (a + 1) - a + log {a + 1) + 5, where a e (-1 , x) is the solution of the given initial

re
value problem.

Flr
EXERCISE 15.4

F
Solve the following differential equations (1-8):
ou ^
2.
dy
— = A
5,22
+ A , A 0

sr
dx A dx X

3. ^ + 4. (.y + 2)^=a^ + 3a + 7
fo
3a-

dx dx
k
oo
(). + A dy + A dx = 0
dx
Y

7. -^ = A log^ A 8. (a^+a^+a + 1)^=2a^+a


reB

dx
dx
Solve the following initial value problems: (9-12)
uY

=x + l; y(0) = 3 10. C’(a)=2 + 0.15a;C(0)=100


dy
+ 1 = 0;yC-l) = 0 12. a(a^-1)^^=1;j/(2)=0
ad

n. A
do

dx
dx
ANSWERS
in

3 2 6 ..3
Re

X
1. l/= —
A
+ i_-logl-^'l+C 2. 2 log I a| + C
3
F

3
2
2 1 3.V , 4. t/ = — + A + 5 log 1 A + 21 + C
1^ + .V = —3 e +C 2
2 , 3/2
5. 1/ = A (log A -1) +C 0. y + -ia.x) 2a ^a + X =C
2

/.
'
y =-
^ A^ logA-^ + C
2

8. y = log [a + 1| + -^ log (a^ + 1) - — tan a + C 2

9.1/= (A + 1) log (a + 1) - A + 3 10. C (a) =2a + (0-15) ^+ 100


f 2 'l
. 4 A- -1
11. y = - log I A 12. i/ = log- 2
A
15.28
APPLIED MATHE^iATICS-Xll

.HINTS TO SELECTED PROBLEMS


We have.

(.V + a-~a)
dy=-
a
rf.v => d\/ = dx = - 'i? + -V + dx
(7+ A* 'a + A- 'a + X

Now, integrate both sides to obtain the general solution.


8. We have.

(x^+x^ + x + 1) ^
dx
= 2x^+x
=> (a' + 1) (.v^ + 1) ^
dx
= 2x^ + -V

2a'^ + A'

w
dx
(a- + 1)(a-^+1)
Integrating both sides with respect to .v, we get
y =
2A'^ + A
■'(a + 1)(a-2+1)
dx

F lo
e
Fre
3 1
- A -
1
- . = ! 2(a + 1) + 2
A^ +1
2 d.v for [Using partial fractions]
1 1 r3A-l
r
dx + — dx
A + 1
2 ●*77^
You
oks

1 r 1 3i-
=■ y = X dx
o

2 ●’a + 1 T
2
. '7
A- + 1 2
eB

^2 log|x + l)+|4
2.V
=> y = d.v-i 1
dx
●’ x^ +1 2 ■’ A^ + 1
our
ad

1 I I i I 3 2 1 -1
- log I A + 1 + -
log I A + 11 - 2 'X+C
=> y =
2 4
dY
Re

MA.2 DIFFEREMTKI. EQU/'TIONf or TH; TYPE ''


Fin

To solve this type of differential equations we integrate both sides to obtain the general solution
as discussed under;
^y
xT =
dx 1
provided that f {y)^0
fiy)'
1
dx = ^y
fiy)
Integrating both sides, we obtain
1
J fiy)
dy +C or, A = dy + C
●’ fiy)
Following examples will illustrate the procedure.
DIFFERENTIAL EQUATIONS 15.29

ILLUSTRATIVE EXAMPLES

CXAiMFL' : Solve:
. ^1/
+ y=l
dx
SOLUTION We have,

-r+
dx
y = 1

dx

w
dx 1

<iy 1-1/
1
dx = dy
i-y

o
e
re
Integrating both sides, we get
dx =
■:^^y

Frl
F
1-y
x = -log|l-y|+C, which is the required solution.
ou
::XAMTLE2 Solve the initial value problem ^ + 2y^ =0 , y (1) =1 and find the corresponding

sor
dx L...

solution curve. kf
SOLUTION We have,
oo
dy ^ 2 dy n 2 _ dx 1
^+2y =0=>^=-2y => —
dx dx dy
Y
B

Integrating both sides with respect to y, we get


1 ,
re

dx = ~^dy
oY
u

1
ad

X = —+ c ... 1

2y
d

It is given that y (1) = 1 i.e. y = 1 when .y = 1.


in

Putting Y = 1 and y = 1 in (i), we get


Re

1
1 = - + C=> C =
F

2 2

Putting C = 1/2 in (i), we get


1 1 1
which is the required solution curve.
2y 2 ^ 2y-1 '

EXERCISE 15.5

Solve the following differential equations:


1. = 0 ^ dy _l + i/ 3
dx dx
y y

ANSWERS

1. Y +
I log 11 + y^ I =C 2. Y =
15.30 APPLIED MATHEMATICS-XII

15.4.3 EQUATIONS IN VARIABLE SEPARABLE FORM

If the differential equation can be put in the form f{x) dx = g(y) dy, we say that the variables are
separable and such equations can be solved by integrating on both sides. The solution is given
by f{x) dx = dy + C, where C is an arbitrary constant.
\OTE There is no need of introducing arbitrary constants of integration on both sides as they can be
combined together to give just one arbitrary constant.
Following examples will illustrate the procedure.
ILLUSTRATIVE EXAMPLES

EXAMPLE 1 Solve: {.Y + 1) ^2xy


dx

SOLUTION We have,

w
(y + 1) ^
dx
= 2xy

Flo
(y + 1) dy - 2xy dx
dy _ 2y dx

ee
y x + 1

Fr
1 f ^ J
-dy = 2 dx
[Integrating both sides]
y Y + 1

r Y + 1 -1 for
ur
= 2 dx
y Y + 1
s
1 1
k

-dy = 2 1 - dx
Yo
oo

y Y + 1
eB

logy =
2 I Y-log| Y + 11| + C, which is the solution of the given differential equation.
EXAMPLE 2 Solve the differential equation x (1 + y^) dx - y {I + x^) dy = 0, given that y = 0, when
r
ou
ad

Y = l.
Y

SOLUTION The given differential equation is


x{l + y‘^'^ dx-y{\ + x^) dy = 0
Re
nd

Y (1 + y^) dx = y (1 + Y^) dy
Fi

5- dx =
1 + Y^ 1 +y
2y
jdx = dy
1 + Y l+y2
Integrating both sides, we get
2y 2y
1 + Y
2* = I dy

log I 1 + Y^ = log U + I + log c


1+.V"
log = logC
1+y^
DIFFERENTIAL EQUATIONS 15.31

1 + X^
= c
1 + y^
(l + x^) = (1+/)C
It is given that when x = \,y = Q. So, putting x = \ and y = 0 in (i), we get
(1 +1) = (1 + 0) C ^ C = 2
Putting C = 2 in (i), we get
(1 + X- ) = 2 (1 + y ), which is the required solution.
EXAMPLE 3 Solve the following differential equations:
dy .... dy 2 dy
(i) = l + x + y + xy (n) y-x^
dx
= a y +-f-
dx
dx

w
SOLUTION (i) We are given that
dy
= l + x + y + xy

F lo
dx

dy
-f-
dx
= (1 + x) + y (1 + x)

ee
dy
= (l + x)(l+y)

Fr
dx

^— fry = (1 + x) dx
1 +y
for
ur
1
dy = (1 + x) dx [Integrating both sides]
1 +y
s
ook

2
Yo

log |1 + y\ =x + ^ + C, which is the general solution of the given differential equation,


eB

(ii) The given differential equation is


^y _ 2 dy
our

y-x^ = a y +-^
ad

dx dx

y-ay = -f{a + x)
Y

dx
Re

(y - fly^) dx = (a + x) dy
nd

dx dy
Fi

2
a + X
y-«y

^dy
1
dx = [Integrating both sides]
a + X
y-^y

1 1 a
dx = — +
dy [By using partial fractions on RHS]
a + X y i-fly
log|x + fl| = log|yl-log|l-tjy| + logC
(x + a) (1 - ay)
log = logC
y

(x + fl) (1 - ay) = C
y

(x + fl) (1 - ay) = Cy, which is the general solution of the given differential equation.
15.32 APPLIED MATHEMATICS-XH

liXAMPl.r-; Solve:

{x^ - ]/x^) dy + {y^ + x'^ y^) dx = 0


SOLUTION (i) The given differential equations is
x^ (1 ~y) dy + y^ (1 + ,v^) dx = 0
x^ (1 - y) dy = - (1 + .y“) dx
1-y
t^y = 2— dx, if X, y ^0
V X

1 1 ^
-^ +1 dx

w
2 =
y yU X

--X = \ -y+ 1 (ix


1
[Integrating both sides]

o
X

e
re
1 1
log I '/ I + - =
1/

rFl
— + Y + C, which is the general solution of the differential equation.

F
liXAMPLE 5 Solve:

x-y
+ x^ e y (ii) ^y=e^'^y ^y

r
(i) -f!-=e (iii) log — = AY + by
ou
dx dx
fo dx
ks
SOLUTION (i) We have,
+ e-y
oo

dx

+ x^ e y) dx
Y

x-y
dy = {e
eB

ey dy = ((?■' + Y^) dx
{e'^ + Y^) dx
r

ey dy = [Integrating both sides]


ou
Y
ad

3
Y
—+ C, which is the required solution.
d

(ii) The given differential equation is


^ = cX+y
Re
in

dx
F

it = ,y
dx

dy = e^ ey dx
e y dy = dx
e'~ y dx = j dx [Integrating both sides]
-e y = + C, which is the required solution.
(iii) We are given that
( d\ '
log — = AY + by
[dx)
dy _ + by
dx
DIFFERENTIAL EQUATIONS 15.33

dx
ax
dij = e ■ dx

dy = dx
[Integrating both sides]
h a

This is the required solution.


LX AMI’LE 6 In a bank principal increases at the rate of5% per year. In how many years ^ 1000 double
itself

w
SOLUTION Let P be the principal at any time f. Then,
dP _ 5P
dt “lOO

o
e
dt 20

re
1 1
- dP=— dt
P 20

rFl
F
Integrating both sides, we get

J' - dP = J\ — dt

r
p
ou
20

^ogP=j^l + logC
fo
ks
=>
oo

^og- = — t
20
C
Y

t/20
eB

P^Ce

It is given that P =1000 when t =0.


Substituting these values in (i), we get
r
ou

1000 =c
Y
ad

Substituting C = 1000 in (i), we get


f/20 ...(ii)
P=1000 ?
d

Let years be the time required to double the principal i.e. at t P = 2000.
Re
in

Substituting these values in (ii), we get


F

2000 =1000

^ i=log,2 ^ fl=201og,2
Hence, the principal doubles in 20 logf. 2 years.
EXERCISE 15.6

Solve the following differential equations:


dy _ (1 + .v^) dy = xy dx
(,V-1) = 2xy
dx

V f = + 1)!/
dx
(x-l)f- = 2x^j
dx
r dy X 4
xy (y + 1) dy = + 1) dx b. 5^=e y
dx
/I 2v dy
y(l-x ) dx
+ X

dx
15.34 APPLIED MATHEMATICS-XIt

2
9. a:
dx
+ 3/=y 10. xy dy ={y -1) (a: +1) dx
- x + y
n.
dx
= 1 -x + y-xy 12. + e
dx

13. In a bank principal increases at the rate of r% per year. Find the value of r if ? 100 double
itself in 10 years (log^. 2 = 0.6931).
14. In a bank principal increases at the rate of 5% per year. An amount of ? 1000 is deposited
with this bank, how much will it worth after 10 years = 1.648).
15. In a culture the bacteria count is 100000. The number is increased by 10% in 2 hours. In how
many hours will the count reach 200000, if the rate of growth of bacteria is proportional to

ow
the number present.
ANSWERS

1. 2a: + 2 logi x-11 = log y + C 2. y=cjl77

e
2 3
3. logi y| =e^ +_x + C ^ + 2A: + 21og| a:-1| +C

re
4- log|y|=-■ X + X

3 2 2

rFl
F
sJ- V X , , . ^
6. -
3y3

or
ou
7. (l+y^)(l-x^)=C 8. -e~>=e^+^+C
ksf
3
9. y -1 =C A:y 10. y-J: = log|A:|-log|y-l|+C
2
oo

11. log(l+y) = x-^+C U. e~^ -e~^ = +C


Y
B

13. 6.931% 14. ?1648 15.


2 log 2
11
re

log
10
oYu

HINTS TO SELECTED PROBLEMS


ad

13. Let P be the principal at any instant t. It is given that


d

dP dP rt
— P=>
dt 100 — dt => log P = + C ...(i)
in

P 100
Re

100

Initially i.e. at t = 0, let P = Pq. Putting t = 0 and P = Pq in (i), we get


F

log Pq = C
Putting C = log Pq in (i), we obtain
rt p rt
logP = + log Pq => log — ...(ii)
100 100

Substituting Pq = 100, P = 2Pq = 200 and f = 10 in (ii), we get


log2 = => r = 10 log 2 = 10 X 0.6931 = 6.931
10

14. Let at any point of itme f, the principal be P.


It is given that the principal increases at the rate of 5% per year.
dP 1 , „ t
— = — dt ^ log P = 77 + log C -(i)
dt 100 P 20 ® 20
Initially i.e. at f = 0, it is given that P = ? 1000
log 1000 = log C
Substituting the value of log C in (i), we get
DIFFERENTIAL EQUATIONS 15.35

t
log P = — + log 1000
B 20 ^
Putting i = 10, we get
, P P
log = 05 => = => P = 1000 X 1.648 = 1648
* 1000 1000

15, Let at any time the bacteria count be N. It is given that


dN dN
ocN => = XN —dN = Xdt
dt dt

On integrating, we get
log N = X t + log C

w
It is given that at t = 0,N = 100000
logC = log 100000
Putting the value of log C in (i), we get

o
log N = Xt + log 100000

e
re
It is also given that at t = 2,N = 110000. Putting t = 2, W =110000 in (i), we get

rFl
F
= X
log 110000 = + log 100000

Substituting the values of log C and X in (i), we get

r
ou
11

log N = ^ log 10
t + log 100000 fo ...(ii)
ks
WhenN = 200000, let f = T. Substituting these values in (ii), we get
oo

2_l2^11
log 200000 = I log ^ + log 100000 => log 2 = I log ^ => T =
Y
eB

log
10
r

15.4.4 LINEAR DIFFERENTIAL EQUATIONS


ou

A differential equation is linear if the dependent variable (y) and its derivative appear only in ifrst degree.
Y
ad

The general form of a linear differential equation offirst order is


+ Py =Q ...(i)
d

dx
Re
in

where P and Q are functions ofx (or constants)


F

For example,
dy
(i) ^
dx
+ xy =x^, (ii) x^
dx
+ 2y = x^, (iii) dx
+ 2y = sin x etc. are linear differential equations.

This type of differential equations are solved when they are multiplied by a factor, which is
called integrating factor, because by multiplication of this factor the left hand side of the
differential equation (i) becomes exact differential of some function.

Multiplying both sides of (i) by e


1 Pdx
, we get
Pdx Pdx
dy
e- + Py = Qe'
dx

d f
= Qe
I Pdx

dx
15.36 APPLIED MATHEMATICS-Xll

On integrating both sides with respect to .v, we get


Pdx Pdx
ye' Q dx + C ...(ii)
This is the required solution, where C is the constant of integration.
Here, e 1 Pdx
is called the integrating factor.
The solution (ii) in short may also be written as
y(I.F.) = J Q(l.F.)dx + C
Following algorithm may be used to solve a linear differential equation,
ALGORITHM

<iy
Write the differential equation in the form -^
dx
+ Py = Q and obtain P and Q

w
Pdx
●●,1 I’ll
Find integrating factor (I.F.) given by l.F. = c

F lo
Multiply both sides of equation in Step 1 by l.F.
-!i-:r I-
Integrate both sides of the equation obtained in step III with respect to x to obtain
y(i-F.) = ■ Q (l.F.) dx + C, which gives the required solution.

ee
Fr
Following examples will illustrate the procedure

ILLUSTRATIVE EXAMPLES
for
ur oks

LY.XMPLEl
Solve the differential equation
Yo

= 2x^
o

dx x
eB

SOLUTION We are given that


-Iv = 2x^
our

+
...(i)
ad

dx X

Clearly, it is a linear differential equation of the form


Y

^+ = Q, whereP = --andO = 2x^


Re

dx Y
nd

1
Pdx dx -1
Fi

-1 1
Now, l.F. = e e' X
X
x

1
Multiplying both sides of (i) by l.F. = —
X
, we get
Idy 1
X dx .V
2 y = 2a-
Integrating both sides with respect to .v, we get
/ - \
1
2.V dx + C
y
X
[Using :y (l.F.) = Q(I.F.)(i.r + C]

3
y = X + Cx, which is the required solution.
DIFFERENTIAL EQUATIONS 15.37

I-.a.v.M!' Solve the differential equation:


^ + i = 3.^
dx 2x

SOLUTION The given differential equation is


dy 1 o3.V 2
dx 2.V

This is a linear differential equation of the form


-^ + Py
dx
= Q, where P = —
lx
and Q = 3x^
Pdx (l/2.r)rfA: 1/2
1/2
I.F. = e e- ^ ^{l/2)logx ^^log.r = .v

Multiplying both sides of (i) by I.F. = we get

w
dy 1 5/2
+ y = 3x
dx

F lo
Integrating both sides with respect to .x, we get
5/2-
yy,fx=j 3x dx + C [Using:.!/(I.F.) = Q (I.F.) d.v + C]

ee
1/1

Fr
+ C
7/2

r- ^ VI 63^ -1/2
for
xj-p = - x' +C=> XJ=-X +C.X , which is the required solution.
ur
Solve the differential equation:
s

HXAMPLE3
ook
Yo

dy
.Ylogx^
dx
+ y = -logx
X
eB

SOLUTION Tine given differential equation is


dy 2,
X log .X ^ + 1/ = -.X log a:
our
ad

dx

dy 1 2
h y = -
dx X log X X
Y

This is a linear differential equation of the form


Re
nd

1 2
^ + py = Q, where P = and Q = —
Fi

dx X log X X

1
dx -dt
Pdx
t
I.F. = e' e , where f = log x

I.F. = = t = log X
Multiplying both sides of (i) by I.F. = log x, we get
1 dy 1
log.x^
d.x + -y
X = \^ogx
X

Integrating both sides with respect to .x, we get


2
ylogx = —^ log xdx + C [Using : y (I.F.) = Q (I.F.) dx + C]
x^
c —2
y log X = 2 log X X dx + C
I II
15.38 APPLIED MATHEMATICS-XII

-1 -1
1 X
ylogx = 2 dx + C
X -1

log^ X ^ dx ■ + C
y log AT = 2 ● -
X

logy 1
y log X = 2 ● - + C
x X

2
y log = — (1 + log x) + C, which gives the required solution.
EXAMPLE 4 Solve the differential equation:
(x'^-l)il + 2xy =
1

w
dx -1

SOLLmON The given differential equation is

(^2_i)^^2:ry = x^-1
F lo
ee
lx 1

Fr
+
y =
dx x^~\
This is a linear differenhal equation of the form for
lx 1
ur
dx
+ Py = Q, where P = and Q =
s

2x/(.t^ - l)dx
ook

Pdx
Yo

I.F. = e'
eB

Multiplying both sides of (i) by I.F. = {x -1), we get

(x^-l)^ + 2xy =
1

dx
r

-1
ad
ou

Integrating both sides with respect to x, we get


1
y(y^-i) = dx + C
[Usmg:y(I.F.)=J Q(I.F.) dx+C]
Y

x^-1
Re
nd

x-1

y(^^-i) = ^log + C, which is the required solution.


Fi

x + 1

EXAMPLES Solve: (x^ +1) ^


dx
+ Ixy = Jx^
V
+4.
SOLUTION We are given that

(x^ +l)^
dx
+ 2xy = Jx^
V
+ 4 => ^
dx
2x
y = -(i)
+1 x2^1
This is a linear differential equation of the form ^
dx
+ Py = Q , where
2x
P = and Q =
x^ +1 x^ + 1
Pdx
I.F. = e = (y^+1)
DIFFERENTIAL EQUATIONS 15.39

2
Multiplying both sides of (i) by I.F. ={x +1), we get

{x^ +l)^ + 2xy = Jx^ + 4


Integrating both sides with respect to x, we obtain
y (:c^ + 1) = J + 4 tix + C [Using :y(J.f.)= Q{LF.)dx+C]

=> y(x^ + l) = ^x i (2)^ log 1 X + ^x^ +4 | + C


=> y{x^ + \) =^x yjx^ +"4 + 2 log I X + ^x^ + 4 | + C, which is the required solution.

ow
EXAMPLE 6 Solve: (I +x^)^
dx
+2xy - 4x^ = 0 subject to the initial condition y{0) = 0.

SOLUTION The given differential equation can be written as

^4. 2x y = 4x^ -(i)

e
dx 1 +x^ 1 +

re
This is a linear differential equation of the form
rFl -^ + Py = Q, where

F
2x 4x^
P =
j and Q = 1 + x^

r
1+X
ou
2x/i\+x^)dx
fo
ks
Pdx
I.F. = c €●

Multiplying both sides of (i) by I.F. = (1 + x^), we get


oo
Y

{1 + x^)^ + 2xy = 4x^


eB

dx

Integrating both sides with respect to x, we get


ur

y (1 + = f 4x^ dx + C [Using :y (I.F.) = Q (I.F.) + C]


ad
Yo

/i 2v 4x^
‘tX ...(ii)
=> y(l + x ) = — + C
d

It is given that y = 0, when a: = 0. Putting a: = 0 and y = 0 in (i), we get


Re
in

0 = 0 + C=>C = 0
4x^
F

Substituting C = 0 in (ii), we get y = , which is the required solution.


3 (1 + a:^)
EXAMPLE 7 Solve each of the follozving initial value problems:
(i)
^y
dx
-y = /,y(0)=l (ii) X
dx
+y = X log X, y (1) = ^
SOLUTION (i) We have,
-y = e
X
-(i)
dx

This is a linear differential equation with P = -1 and Q = e^.


' -Idx - X
I.F. = e e

Multiplying both sides of (i) by e we get


15.40 APPLIED MATHEMATICS-XII

- X
-ye e ■ e
dx

Integrating both sides with respect to x, we get


- X
.e~^ dx + C [Using :y(I.F.)= Q (I.F.) fr.r + C]
- X
ye = x^C -(ii)
It is given that i/(0) =1 i.e. i/=1 when ;c = 0. Putting .r = 0, i/=l in (ii), we get
1 = 0+C => C = 1

Putting C = 1 in (ii), we get


ye~ ●^ = 'X +1 => y = (x +1)

w
Hence, y = (x +1) is the required solution,
(ii) We have, x —
dx
+ y = x log x

o
dy 1
-f
dx + ~y = log a:

e
X ●●●(i)

re
This is linear differential equation of the form

rFl
^y 1
+ Py=Q with P = — and Q = log x.

F
dx X
I
dx
I.F. = e X
= = I
[vx>0]

r
ou
Multiplying both sides of (i) by I.F. = x, we get fo
ks
^y 1
x^
dx
+ y = Xlog X
oo

Integrating with respect to .v, we get


Y

yx = X log X dx [Using :y (I.F.) = Q(I.F.)dx + C]


eB

II
I
2
1 r
yx =
y(log:r)-i xdx
r
ou

2 2
Y
ad

xy =
y (log X) - y + C ...(ii)
d

It is given that y (1) = — ie. y ~~ when x =1. Putting x = 1 and y = — in (ii), we get
Re
in

1 1 1
= 0-- + C => C = -
F

4 4 2
1
Putting C = — in (ii), we get
2 2
1 1 , X 1
xy =
|-(logx)-^ + - -=>!/
2
= — X log X — +
2 ® 4 2x
1 X 1
Hence, y =
- X log X - ^ + y is the solution of the given differential equation.
i-.XAMl'U; 8 Solve each of the follozaing initial value problems:
^-8 2x
y =
1
,y(0) = 0
dx x^ +1
SOLUTION We have.
<^y +
2x 1
y = ...(i)
dx x2 +1 (x^ +1)2
15.41
DIFFERENTIAL EQUATIONS

2x 1
This is a linear differential equation with P = and Q =
x'^ + 1
lx
dx

l.F. = e
x^ + 1 e logd^+l) ^ / + 1
Multiplying both sides of (i) by l.F. = -s-1, we get

(/+l)|/+2.vi/
dx = x^+1
Integrating both sides with respect to .r, we get

ow
1
y(/ + l) = dx + C [Using:y (l.F.) = Q (l.F.) dx +C]
2 -1
=> 1/ (.\- +1) = tan A' + C ...(ii)

It is given that y (0) = 0 i.e. y = 0 when v = 0. Putting ,v = 0, y = 0 in (ii), we get

e
re
0=0+C=>C=0

Frl
Putting C = 0 in (ii), we get

F
-1
tan
y {x^ +1) = tan“ ^ .y => y = —, which gives the required solution.
ou
or
ih
15.4.5 LINEAR DIFFERENTIAL EQUATIONS OF THE FORM : Ry
kfs
I. ■

dx
Sometimes a linear differential equation can be put in the form + Rx = S, where R and S are
oo

dy
Y

functions of y or constants.
B

Note that here y is independent variable and y is a dependent variable.


re

The following algorithm is used to solve these types of equations.


oYu

ALGORITHM
ad

dx
Write the differential equation in the form — + Rx =S and obtain R atid S.
dy
d

R dy
in

Find l.F. by using l.F. = c-


Re

STLIMII Multiply both sides of the differential equation in Step I by l.F.


F

1
Integrate both sides of the equation obtained is Step III with respect to y to obtain the solution
I ■

given !?y y(I.F.) = S(I.F.)dy + C, where C is the constant of integration.


Following examples will illustrate the above procedure.
ILLUSTRATIVE EXAMPLES

EXAMPLE 1 Solve: y dx-{x + 2y^) dy = 0


SOLUTION The given differential equation is
y frY -(y + 2y^) dy = 0
^ y
dx
x + 2,/
dx X + 2i/
dy y
15.42 APPLIED MATHEMATICS-XII

dx 1
— + — X = 2y -(i)
iy y)
This is a linear differential equation of the form
dx
+ Rx = S, where R = -- and S = 21/
y

I.F. = €■
Rdy
e
y
= g-iogy = giogj/ ^ = y
-1

Multiplying both sides of (i) by I.F. = we obtain


1 dx 1
= 2
y dy

ow
Integrating both sides with respect to y, we get
1
XX — 2dy + C [Using : X (I.F.) = S(\.F.)dy + C]
y ●'
X

e
- = 2y + C , which is the required solution.

Fl
re
y

EXAMPLE 2 Solve: ydx + {x~y^)dy = 0

F
SOLUTION The given differential equation is
ur
y dx + (x-y^) dy = 0
dx X or
sf
2

dy'^ y ^ ...1
k
Yo
oo

This is a linear differential equation of the form — + Rx = S, where R = - and S =


dy y
B

I.F. = = e ^
re

= y
=
u

Multiplying both sides of (i) by I.F. = y, we obtain


ad
Yo

dx 3
y~T^x = y
dy
d

Integrating both sides with respect to y, we get


Re
in

^y = y^dy + C [Using :x (I.F.) = f S(I.F.)rfy + C]


F

xy = ^+C, which is the required solution.


EXAMPLES Solve: (x + 2y^)dy = y dx.
SOLUTION The given differential equation can be written as

dx _ x+ 2y^
r . \
dx
-- x = 2y^
dy y dy y

This is a linear differential equation of the form


~
dy
+ Rx = S, whereK =-- and S = 2y^
y

We have, I.F. = eJ I Krfy = e I -'/>


^ = g-iogy = giogy
-1

= y
1

y
DIFFERENTIAL EQUATIONS 15.43

1
Multiplying both sides of (i) by I.F. = — , we get
y

1 ^_JL X = 2y
y dy
Integrating both sides with respect to y, we get
1
X 2t/ dy + C [Using: a: (I.F.) = S(I.F.)dy + C]

- = + C => a: = y^ + Cy, which is the required solution.


y

w
y dx
CX.\MrLE4 Solve: — = 1.

o
SOLUTION We have.

e
re
y dx

y[x ^ dy
_ .= 1

rFl
F
dy _ y

r
dx y[x
ou
fo
ks
dx X
oo

This is a linear differential equation with P = ^ and Q =


Y


eB

Pdx
r

I.F. = e =
ou
Y
ad

Multiplying both sides of (i) by I.F. = , we get

1
+11
d

dx
Re
in

Integrating both sides with respect to x, we get


F

y^ 2^x _ f>2-\/^ dx + C [Using ;y (I.F.) = Q (I.F.) dx + C]

y^
l^x ~dA: + C

= lyfx+C
-2Jx
y = {2+Jx +C)e , which gives the required solution.
EXERCISE 15.7

Solve the following differential equations:


dy -3x
^ dy + 2y~e3x 2.4 ^ + 8y =5 e
dx
dx

4. ^ +y =c
-2x
3.^
dx
+ 2y = 6e^ dx
15.44 APPLIED MATHEMATIC3-XII

,S.
= x + \j 6. ^ + 2y=4x
dx dx
If dy X ■ iL 4x 1
'■ x^^\f.=xe + —
y + = 0
dx dx X- +1 {x^ +1)^
x^
dx
+ y = X log® X ■ ^ ~ -y ={x ~\)
dx
n. ^ + l = x'^ 12.
2 dx 1
y —+x--=0
dx X
y

■ r (1 + x^) ^ - 2x\j = {x^ + 2) (x^ + 1) ■4. (x^-i)^ + 2(x + 2)y = 2(x + l)

w
dx dx
^y jr
1(1. ^y 4a-
^-y = x(? + 2y = xt’
dx dx
17.
Solve tlie differentia] equation (x + 2y^) —

o
= y, given that when x = 2, y = 1

e
dx

re
Find the general solution of the differential equation x -^ + 2y = x^.
18.
dx

Frl
F
19.
Solve the differential equation (y + 3x^) — = x.

^y
ou
ANSWERS

r
5,-3.v
so
-2.r -2.V
+Ce + Ce
^ = -4
kf
lx
3. ye = 2r>^-^' +C -2.r
+ Ce~^
oo
■ . y = -e

5- ^ = log I XI + C -lx
Y

X
■■ y = (2x-l)+Ce
B

x-ll
'● y = 8.
y{x^ + l)^=-x + C
re

X X
oY

4xy = 2.t^ log|x|-x^ + C


u

in.
y = e^ + Cx
ad

Ih 5.xy =x^ +C 12.


y=
d

L y
in

-1 2(.v + l) { 2 1
X + C) (x^ + 1)
Re

13.
y =(x + tan 1. y =
^|x -6x + 81og(x + l)|+C
F

2
15. — +C e 10.
X 4a- 1 4.V -2a
e + Ce
36

x = 2y^ 18. ^ + Cx-2 19. y


= 3x + C
^=1 X

HINTS TO SELECTED PROBLEMS


We have.

X^
dx
+ 2y = x^ dy
-f
dx + ~y
x
2
= X
This is a linear differential equation of the form ^
dx
+ Py = Q, where P = -X and Q = x.
’ ^dx
l.F. = £?■ A
15.45
DIFFERENTIAL EQUATIONS

Multiplying both sides of (i) by a", we get


3
.Y
dx

Integrating with respect to x, we get


Y + ^C=>l/= X^ +C.Y
^ -1
4
3 j ^ 2
= ±_
2
A dx + C => yx 4 -^ 2

15.5 APPLICATIONS OF DIFFERENTIAL EQUATIONS


In this secHon, we shall discuss some problems on the applicaUons of differential equations in
Science and Engineering. We shall also discuss problems on applications to other disciplines.
ILLUSTRATIVE EXAMPLES

w
Tup-' ' APPLICATIONS ON GROWTH AND DECAY
■ \
A population grows at the rate of 8% per year. How long does it takefor the population to

Flo
double? Use differential equation for it.
Let Pq be the initial population and let the population after t years be P. It is given

e
SOLUTION

re
that

rF
dP 8P

dt 100
ur
dP 2P fo
dt 25
ks
dP
^dt [By separating the variables]
Yo
oo

P 25

irfP =— [Integrating both sides]


25 Jf l.dt
B

P
re

logP = — f+ C
25
u

Initially i.e. at f = 0, we have P = Pq ■ Putting t = 0 and P = Pq in (i), we get


ad
Yo

2x0
logPo = + C => C = log Pq
25
d

Substituting C = log Pq in (i), we get


Re
in

25 , P
logP = ^ f + log Pq => log j — t=>t =
F

25 ^ 0f 25

WhenP = 2Pq, we get


2Po 25
25 ,
t =
y log Po y 10g2
25

Thus, the population is doubled in -y log 2 years.


iAAMPi.r :■ Suppose thegroivth of a population is proportional to the number present. If the population
of a colony doubles in 25 days, in hoio many days xoill the population beconie triple?
SOLUTION Let Po be the iniHal populahon and Pbe the population at any timef.lt is given that
dP
ocP
dt
15.46
APPLIED MATHEMATICS-Xll

dP
= XP, X is a constant
dt

dP
^ Xdt
P

-dP
p = ?.J dt
logP = XI+C
At t = 0, we have P = Pq. Putting t = 0 and P = Pq in (i), we get
^08^0 = 0 + C ^ C = log Pq
Putting C = log Pq in (i), we get

ow
logP=^t + logPo

...(ii)

e
It is given that P = 2Pq when t = 25 days. Putting t = 25 and P = 2Pq in (ii), we get

re
log 2 = 25A.=> A. = — log 2
25 ^

rFl
F
Putting ^ ~ ^ ^08 2 in (ii)/ we get

r
/ \
ou
, P r 1 "i
= — log2 f
[25
fo ...(in)
ks
Suppose the population is tripled in days. i.e. P = 3 Pq when t = t^.
oo

Putting P = 3Pq and I = fj in (iii), we get


Y
B

r 1
log 3 = ~ log 2 t-i => = 25 logs days
re

log 2
ou

Hence, the population is tripled in 25 logs days.


Y
ad

log 2

LXAMPLE 3 It IS ktiozvn that, if the interest is compounded continuously, the principal changes at the
d

rate equal to the product of the rate of bank interest per annum and the principal.
in
Re

(i) If the interest is compounded continuously at 5% per annum, in how many years will ^ 100 double
F

itself?
(ii) At what interest rate will ? 100 double itself in 10 years? (log^ 2 = 0.6931).
(iii) How much will ? 1000 be worth at 5% interest after 10 years? (e— 05
= 1.648).
SOLUTION If P denotes the principal at any time t and the rate of interest be r % per annum
com
pounded
dP
continuously,
Pr
then according to the law given in the problem, we get
°
dt 100
dP
— dt
P 100

J■’ ^dP
P = 100 J dt
rt
logP = + C
100 ...(i)
15.47
DIFFERENTIAL EQUATIONS

Let Pq be the initial principal i.e. at f = 0 , P = Pq ●


Putting P = Pq in (i), we get
log Pq = C
Putting C = log Pq in (i), we get
rt
logP = + log Pq
100
/ \
rt -(ii)
log — 100

(i) In this case, we have


r = 5,Po=^100 and P=?200 = 2Pq
Substituting these values in (ii), we have

w
log 2 = — t => ^ = 20 log. 2 = 20 X 0.6931 years = 13.862 years.
^ 100

Flo
(ii) In this case, we have
Pq = ?100, P=?200 = ZPoandt = 10 years.

e
Substituting these values in (ii), we get

re
lOr
=» r = 10 log 2 = 10 X 0.6931 = 6.931
log 2 =

F
100

Hence, r = 6.931% per annum.


ur
r
(iii) In this case, we have fo
Pq = ?1000,r=5 and t=10
ks
Substituting these values in (ii), we get
Yo

logf^
5x10 1 p
oo

05
= 05 => — e => P = 1000 X 1.648 = 1648
^ 1000 100 2 1000
eB

Hence, P=^1648.
EXAMPLL 4 It is given that the rate at which some bacteria multiply is proportional to the instantaneous
ur

number present. If the original number of bacteria doubles in tzvo hours, in how many hours will it hefive
ad

tltTJCS ^
Yo

SOLUTION Let the original count of bacteria be Ng and at any time t the count of bacteria be N.
It is given that
d

dN
Re

az N
in

dt
dN
F

= XN, where A. is a constant


dt
dN
= Xdt
N

— dN = A. f dt
J N
...(i)
log N = A. f + C
We have, N = Ng at f = 0. Putting t = 0 and N = Ng in (i), we obtain
log Ng = 0 + C => C = log Ng
PutUng C = log Ng in (i), we get
log N = A. f + log Ng
N
= Xt
...(ii)
=;> log
N0
15.48
APPLIED MATHEMATICS-XII

It is given that the original number of bacteria doubles in 2 hrs.


That is when t = 2 hours, N = 2Nq . Putting t = 2 and A/ = 2Nq in (ii), we get
2 N0 1
log„ =2X^ X =
\
N
Nq ^ J
2 Jog 2
Putting ^ = 2 2 in (ii), we get
N ' 1
log = -log 2 I
N,
^^''0

^log
N
f =
log 2 N, 0 ...(iii)

ow
Suppose the count of bacteria becomes 5 times i.e. 5 Nq in f j hours. Putting f = fi and N = 5 Nn i
(iii), we get ^
2 , 5iV,0
h = -rlog ^(log5) hours.

e
log 2 N, 0 log 2

Fl log 2

re
i-XAMi’LC 0 It is given that radium decomposes at a rate proportional to the amount present. Ifp % of the

F
original amount of radium disappears in I years, what percentage of it will remain after 21 years?
ur
SOLUTION
Let An be the original amount of radium and A be the
time t. Then, the rate of decompose of radium isi
dA
or amount of radium at any
sf
dt
. It is given that
dA
k
Yo
oc A
dt
oo

dA
= -XA, where is a positive constant
B

dt
dA
re

= ~Xdt
A
u

log A = -Xf + C
ad

-(i)
Yo

At t = 0, we have A = Aq . PutHng f = 0 and A = Aq in (i), we get


log Aq = 0 + C => C = log Ag
d

Putting C = log Aq in (i), we get


Re
in

log A = - A. f + log Ag
F

A
log = -Xt
A0 ...(ii)

It IS given that p % of the original amount of radium disintegrates in / years. This means that the

amountof radium presentatf =;is(^ylo-i. Aoj -ij/!„. Putting/( = fl-ijand


f = / in (ii), we get
log 11 - 7?
100 = -XI => X = —/ log 1 -
1 P
100

Substituting the value of X in (ii), we get


= ^ log f1 -
A
log
A0 / 100 ...(iii)
15.49
DIFFERENTIAL EQUATIONS

Let A be the amount of radium available after 21 years.


Putting t = 2/ in (iii), we get
A
log ■= 21og 1- 100
^
^0
n2
A

^0 100

— xlOO = xlOO [Multiplying both sides by 100]


100
^0

w
A
xlOO = 10--^
10
^0

o
2
P

e
Hence, required percent =

re
rxAMri.i h A radioactive substance dismtegrates at

rFl a rate proportional to the amount of substance

F
present. If 50% of the given amount disintegrates in 1600 years. What percentage of the substance
( -log2 'l

r
160 = 0.9957
ou
disintegrates in 10 years? Take e
fo
ks
V ^

SOLUTION Let A denote the amount of the radioactive substance present at any instant t and
oo

let Aq be the initial amount of the substance.


Y

It is given that
eB

dA dA -.(i)
a A ^ = ‘XA
dt dt

?. > 0. Here, negative sign indicates that A


r

where is the constant of proportionality such that


ou

decreases with the increase in t.


Y
ad

Now,
dA
= -lA
d

dt
Re
in

1
— dA = -Xdt
A
F

LdA
A
= -x ^\ l.dt
..●(ii)
log A = - ^ f + C
Initially i.e. at ( = 0, we have A = Ag. Putting t = 0 and A = Ag in (ii), we get
log Ag = 0 + C => C = log Aq
Putting C = log Ag in (ii), we get
log A = -Xt + log Ag
A
= ~xt
...(iii)
log A0

It is given that A = 2 ^ atf =1600 years. PutHng A = ^2 and t = 1600 in (iii), we get

1
= -1600 7. ^ X = L: log 2
log^~ 1600
15.50
APPLIED MATHEMATICS-Xn

Substituting the value of >.in (iii), we get


,log (a] f I , 1
A 1600 ^
log 2 f
A
- e 1600

log 2 /
1600
^ = Aq 't
Putting t =10, we obtain the amount of the radioactive substance present after 10 years and IS
i
given by ^

ow
log 2
A = Aq (0.9957) e 160
= 0.9957

Amount that disintegrates in 10 years = Aq - A = Aq ~ 0.9957 Aq = 0.0043

e
Hence, percentage of the amount disintegrated in 10 years

Fl 0.0043A 0

re
X100 = 0.43
^0

F
Hence, 0.43% of the original amount disintegrates in 10 years.
ur
LXAMP
LE 7 The rate at which radioactive substances decay is known to be proportional to the number of
such nuclei that are present at the time in a given sample. or
sf
(i) In a certain sample 10% of the original number of radioactive nuclei have undergone
disintegration m a period of 100 years. Find what percentage of the original radioactive nuclei will
k
Yo

remain after 1000 years.


oo

(ii) IflOOgramsofaradioactivesubstanceispresentl year after the substance was produced and


B

75 grams is present 2 years after the substance was produced, how much radioactive substance was
produced?
re

SOLUTION (i) Let there be N radioactive nuclei in a sample at any time t and let Nq be the
u

miH^number of radioactive nuclei. Then, the rate of decay of the radioacHve substance is given
ad
Yo

dt
dN
d

It is given that xh/


Re
in

dt
dN
F

= -XN , where K > 0 is a constant


dt
dN
= -Xdt
N

JJ N = -XfJ df [On integrating]


log IV = -Xt + C
At f = 0, we have JV = Nq . Putting f = 0 and N = Mq in (i), we get
log Nq = 0 + C ^ C = log Nq
Putting C = log Nq in (i), we get
logN = -Xf + IogNo
N
log = -Xt
N0 ...(ii)
15.51
DIFFERENTIAL EQUATIONS

It is given that 10% of the original number of radioactive nuclei have undergone disintegration
in a period of 100 years. Therefore, number of radioactive nuclei available at f = 100 is given by
10 9Nq
N = No- 100 xNo = 10

9Nq and f = 100 in (ii), we get


Putting N = 10

,ogA = -ioo^^
Putting the value of X in (ii), we get
log
N f 1 , 9^ t ...(iii)
^100 10^

ow
No
We have to find the value of N at t = 1000 years. Putting t = 1000 years in (iii), we get
N f 9^
log = 10 log —
No ^10 J

e
Fl
re
N
log = log
N, 0 10 J

F
V

10
(9)
ur
N

No 1,10 J
or
sf
10
N 9
xlOO = X100 = -
k
.io.
8
Yo
No 10
oo

10
N 9
xl00=-
Percentage of radioactive nuclei that remain after 1000 years = 8 ●
B

No 10
e

qlO
1 % of radioactive nuclei will remain after 1000 years.
ur

Hence,
10 I®
ad

(ii) Suppose Nq grams of radioactive substance was produced and at any time f, Njrams of
Yo

substance is present. Then, the rate of decay of the radioactive substance is given by —.
d
Re

dN
in

ocN
It is given that dt
F

dN
= - A, N, where A, > 0 is a constant
dt
dN
= -Xdt
N

Jf —dN = -X dt
N
...(i)
=> logN = -Xt + C
At f = 0, we have N = Nq. Putting t = 0 and N = Nq in (i), we get
log Nq = 0 + C C = log Nq
Putting C = log Nq in (i), we get
logN = -A.f + logNo
N
= -Xt
...(ii)
log
No
15.52
APPLIED MATHEMATICS-Xii

It is given that after 1 years 100 grams of radioactive substance was present, i.e.
N = 100 grams at f = 1. Putting f = 1 and W = 100 in (ii), we get
100
log = -X
...(hi)
N.

After 2 years, 75 grams of radioactive substances was present i.e. at / = 2, N = 75 grams Putting
W=75gramsandf =2in(ii), weget °
75
log = -2-k
N ●●■(iv)
1^0 J
Eliminating X from (iii) and (iv), we get
75 100
log = 2 log
N, 0 N0

w
75
^2 x2

log
100 75 100 100^ 400
N.0
= log => .N0 => ,N0 — grams.

F lo
N, 0 N, 0 N.0 75
400
Hence,
— grams of radioactive substance was produced.

e
Fre
MPLE8
^ <^oUege hostel accommodating 1000 students, one of them came in carrying a flu virus
then the hostel zvas isolated. If the rate at zvhich the virus spreads is assumed to be proportional to the
product of the number N of infected students and the number ofnon-infected students, and if the number
for
oj infected students is 50 afterA^ijears, then shozv that more than 95% of the students ivill be infected aft ei'
10 days. ■’ ●'
r
SOLUTION At any time t, we have
You
oks

Number of infected students = N


Number of non-infected students = 1000-N
eBo

It is given that at any time t, the rate at which the virus spreads i.e. dN .
IS proportional to the
dt
product of number of infected and the number of non-infected students.
our
ad

dN
dt
oc
N(IOOO-N)
dN
dY

X N (1000 - N), where X is the constant of proportionality.


=>
Re

dt
1
Fin

dN = Xdt
N(IOOO-N)
1
dN = X dt
J N(IOOO-N) [On intperating]
1 1
1000 J 1000-N
+^
N
dN = \ dt
XI+C

1 N
=i>
log = Xt + C
1000 1000 - N

It is given that initially one student was carrying a flu virus. That is at t = 0, we have N = 1.
Putting f = 0 and N = 1 in (i), we get
15.53
DIFFERENTIAL EQUATIONS

1 1log 1 = 0 + C => C = 1 1 f 1 1 log 999


1000 999 1000 1000

Substituting the value


/
of C in\ (i), we get
N
= Xt-
log 999
—— log
1000 ^ 1000-N 1000

N
+ —log 999 = Xt
1000 1000-N 1000 ^

999 N ...(ii)
^log
1000 1000-N
= Xt

After 4 years the niunber of infected students is 50. That is at f - 4, N -50.

w
Substituting these values in (ii), we get
1 f49950^ = AX
1000
X =
950 J
-— log
( 49951 = ——log ( 999 "i
4000 ^ 19
F lo
ee
4000 ^ 1, 95

Fr
Putting the value of A, in (ii), we get
^ 999N ^ ( 999]
^log
for
^log t
1000 1000-N
14000 19 J
r
You

999N 999"! t ...(iii)


s

= log
ook

4 log 1000 - N 19 J
eB

We have to find the number of infected students after 10 years. So, putting f = 10 in (iii), we get
999'
/
999N
= 10 X log
4 log
Il9 J
our
ad

1000-N

999N 5 '999'
= «log
log
1000-N 2 [l9 J
dY
Re

-5/2
flOOO-N 5 999] 999]
Fin

log --log = log 19


999N 2 119 j
-5/2
1000-N _ f 999'
999N " t 19 ,
-5/2
1000 1 _f 999'
999N 999 ~ t 19 ,
-5/2
1000 ^ 1 ^ (999'
999N 999 [ 19 ^
1000
N
= l+(999)"^^^xl9®/^
.1000
=> N = = 952 approximately
l + (999)"^^^xl9^/^
15.54
APPLIED MATHEMATICS-XII

Thus, percentage of infected students after 10 years is given by


N 952
xl00 = X100 = 95.2.
1000 1000

Hence, more than 95% students will be infected after 10 days,


nx AMPLE 9 Assume that a spherical rain drop evaporates at a rate proportional to its surface area. If its
radius originally is 3 mm andl hour later has been reduced to 2 mm, find an expression for the radius of
the rain drop at any time.
SOLUTION Letr be the radius, V be the volume and Sbe the surface area of the rain drop at any
time f. Then, ^
V = .1 Tcr^ and S = 47rr^

w
We are given that the rate of change of volume of the rain drop is proportional to surface area.
dV
i.e. ●X S

F lo
dt

dV
= kS, k is the constant of proportionality

ee
dt

Fr
=> d_( —4 Kr
^ = k{4Tzr^)
dt 3 )
for
ur
4k r^ ~
dt
= k (4k r^)
s
dr
ook

= k
Yo

dt
eB

dr = k dt

Integrating both sides, we get


r

dr = k dt => r = kt+C
...(i)
ou
ad

We are given that r = 3atf=0andr = 2atf=l


Y

3 = cf(0) + C and 2 = k + C
Re
nd

C = 3 and cf = -1

Putting the values of C and k in (i), we get r - 3 - t, where


Fi

0 < f < 3.

EXERCISE 1S.8
1.
A population grows at the rate of 5% per year. How long does it take for the population to
double?
2.
The rate of growth of a population is proportional to the number present. If the population
of a city doubled in the past 25 years, and the present population is 100000, when will the
3.
city have a population of 500000? [Given log^ 5 = 1.609, log^ 2 = 0.6931.]
In a culture, the bacteria count is 100000. The number is increased by 10% in 2 hours In how
many hours will the count reach 200000, if the rate of growth of bacteria is proportional to
the number present?
If the interest is compounded continuously at 6% per annum, how much worth ? 1000 will
4.

be after 10 years? How long will it take to double ? 1000? [Given e


0.6
=^1.822]
15.55
DIFFERENTIAL EQUATIONS

5. The rate of increase in the number of bacteria in a certain bacteria culture is proportional to
the number present. Given the number triples in 5 hrs, find how many bacteria will be
present after 10 hours. Also find the time necessary for the number of bacteria to be
= 9]
the number of initial present. [Given log^ 3=1.0986, e
6 The DODulation of a city increases at a rate proportional to the number of inhabitants
pres^t^at any time t. If the population of the city was 200000 in 1990 and 250000 in 2000,
what will be the population in 2010? dC
= 2 + 0.15 :c.
7. If the marginal cost of manufacturing a certain item is given by C' (x) dx

Find the total cost function C (;t), given that C(0) = 100.
8. A bank pays interest by continuous compounding, that is, by treating the mterest rate as the
instantaneous rate of change of principal. Suppose in an account interest accrues at 8 /o per
year, compounded continuously. Calculate the percentage increase in such an account
[Takec
over
=1.0833]
one year. '■
9. The decay rate of radium at any time t is proportional to its mass at that time. Find the time

w
when the mass will be halved of its initial mass.
10.

F lo
Experiments show that radium disintegrates at a rate proportional to the amount of radium
present at the moment. Its half-life is 1590 years. What percentage will (disappear in one
[Use; =0.9996]

e
year?

Fre
11 The rate of increase of bacteria in a culture is proportional to the number of bacteria present
' and it is found that the number doubles in 6 hours. Prove that the bacteria becomes 8 times
for
at the end of 18 hours.
12 Radium decomposes at a rate proportional to the quantity of radium present. It is foun
r
that in 25 years, approximately 1.1% of a certain quantity of radium has decomposed.
You
oks

Determine approximately how long it will take for one-half of the original amount o
radium to decompose? [Given log^ 0.989 = 0.01106 and log^ 2 = 0.6931]
eBo

ANSWERS

2 log 2 hours
2. 58 years
ad

1. 20 log 2 years
our

11
log
10

5 log 10 6. 312500
5. 9 itmes
4. f 1822,12 years
Re
dY

' log 3
8. 8.33%
7. C (x) = 0.0753:^ +2x + 100
Fin

n. 0.04%
10. -k log 2, k is the constant of proportionality
12. 1567 years.
HINTS TO SELECTED PROBLEMS

6. Let P be the population at any time t. It is given that


dP
00 P
dt
dP
= XP, where is a constant of proportionality
dt
dP
= Xdt
P
...(i)
=> log P = X. f + log C
Initially i.e. at t = 1990, P = 200000 and at t = 2000, P = 250000
15.56
APPLIED MATHEMATICS-XIl

log 200000 = 1990 l+logC ...(i) and, log 250000 = 2000 ?.+log C ●●●(ii)
=> log 200000 - log 250000 = lO^i [On subtracting (ii) from (i)]
^ X = 1 (4

1 (4'
Putting — log - in (i), we get
10 1^5y

log 200000 = 1990 X 1 f4^ + logC


5

log 200000
= 199 log ^ + log C
= log 200000-199 log I

w
=> logC

Putting X =
^ I / log C = log 200000 -199 log ^ and t = 2010 in (i).

Flo
we get
1 4 i A
logP = — log - 2010 + log 200000 -199 log -

ee
lU 5 ^ 5

Fr
201

^ log P = 1log h)
-
5V^"
+ log 200000 X -
15J 4
for
ur
201 199
f5 sf 25
s
X 200000 X - X 200000 = —X 200000 = 312500
k

5 4
u
Yo

V y 16
oo

MULTIPLE CHOICE QUESTIONS IMCOs)


eB

Mark the correct alternative in each of the following:


1 The integrating factor of the differential equation {;e log :r) ^ + y = 2 log ;r, is given by
r
ou
ad

(a) log (log a:) (b) (c) log a; (d) a:


Y

The general solution of the differential equation ^ ^X is


Re
nd

dx
(a) logy = kx (b) }/ = kx (c) xy = k (d) y = k log a:
Fi

The degree of the differential equation dy 3 .


1/ / IS
dx^ dx

(a) 1/2 (b) 2 (c) 3 (d) 4


2
f 2 \
The degree of the differential equation ] 5 +
= X
5 d^y1- , IS
dx dx^
(a) 4 (b) 2 (c) 3 (d) 10
The equation of the curve whose slope is given by ^
dx —; a: > 0, y > 0 and which passes
through the point (1,1) is
(a) = 1/ (b) y^ a:
(c) AT^ = 2y (d) y^ = 2a-
15.57
DIFFERENTIAL EQUATIONS

6. The solution of the differential equation ~ ^ */ (1) -1 given by


1 1
(b) X = — (c) X = - (d) 1/ = -
a:
y
X y
^y
7. The solution of the differential equation 2x —
dx
-1/ - 3 represents

(a) circles (b) straight lines (c) ellipses (d) parabolas

The differential equation x^-y


dx
= x^, has the general solution
(a) i/-x^ =2cx (b) 2y-x^ =cx (c) 2y + x^ = 2 cx (d) y + x^ = 2 cx

w
9. Which of the following is the integrating factor of (x log ~^ ^
(a) X (b) (c) log X (d) log (log x)

o
^3
d^y dy + 1 = 0, is

e
dy
10. The degree of the differential equation
+ + sin

re
dx

rFl dx

F
(a) 3 (b) 2 (c) 1 (d) not defined
2 d^y 3^ + y = 0, is
11. The order of the differential equation 2x

r
dx^ dx
ou
(a) 2 (b) 1 (c)0 fo (d) not defined
ks
12. The number of arbitrary constants in the general soluHon of differential equation of fourt
order is ^
oo

(a) 0 (b) 2 (c) 3 (d) 4


Y

13. The number of arbitrary constants in the particular solution of a differential equation of
eB

third order is
(b) 2 (c) 1 (d) 0
(a) 3
r

dy == 2,r^
14. The integrating factor of the differential equation x ~y
ou

dx
Y
ad

(b) ^ ^ (c) 1/A- (d) X


(a) e
_ ANSWERS
d

6. (a) 7. (d) 8. (b) 9. (c)


Re
in

J. (c) 2. (b) 3. (b) 4. (c) '● (a)


10. (d) 11. (a) 12. (d) 13. (d) 14. (c)
F

FILUN THE BLANKS QUESTIONS (FBQs)


2
= Aax
1. The order of the differential equation representing the family of parabolas y
is.

f
dy = 0is
2. The degree of the differential equation — dx^
\^ax

3. Tlie number of arbitrary constants in a parhcular solution of the differential equation


tan X dx + tan y dy = 0 is
dy
4. The integrating factor of the differential equation a— -y - sm x is
dy
~^ = e
x-u-^1is

5. The general solution of the differential equation dx
15.58
APPLIED MATHEMATICS-XII

6. The general solution of the differential equation—


dx
+ ^X = lis
7. The differential equation representing the family of curves y = Asmx + Bcosx is
8. The linear differential equation ^ ^ = 1, a: ^ 0 when written in the form
^^x -yfx dx

^ + Py=QfthenP=
9. The order of the differential equation representing the family of ellipses having centre at

ow
origin and foci on a:-axis is

10
. The degree of the differential equation 1 + = ^ + xi
V dx^ dx

e
. The integration factor of the differential equation
11
-y = xcosx is

re
dx

Frl 2 dy

F
12
. The degree of the differential equation^ + edx =0is
dx^
ou
or
13. The degree of the differential equation 1 = xis kfs
14. order
The number of arbitrary constants in the general solution of the differential equation of
oo

three is ^
Y

. The general solution of the differential equation of the type ^ + l?x = S, where i? and S are
15
B

dy
functions of y, is
re

16. The integrating factor of the differential equation ^ + y = 1+y:


oYu

IS
dx X
ad

17
. The general solution of the differential equation x^
dx
+ 2y^ = x^ is.
d

18
● The solution of the differential equation y dx + (x + xy) dy = 0 is
in
Re

19. The order of the differential


equation representing the family of circles
ar^+(y-fl)^=/ is
F

20. The number of arbitrary constants in the particular solution of a differential equation of
order two is ^

. The integrating factor of all differential equation (x^ +1) ^ + 2xy^ = x^ -lis
21

dx
/ . n2 / \2
22.
The degree of the differential equation y=x ^ _ is
[^y
23. The order of the differential equation representing all circles of radius r is
24. The degree of the differential equation representing the family of curves y = Ax + A^,
where A is arbitrary constant, is
2a. The general solution of the differential equation — + ^ = 0 is
X
y
15.59
DIFFERENTIAL EQUATIONS

4
+ 2 = y are
26. The order and degree of the differential equation dx^ dx^ dx
/

and respectively.
27. The differential equation for which y = flcosjc + efsinx is a solution, is
28. The curve for which the slope of the tangent at any point is equal to the ratio of the abscissa
and ordinate of the point, is
29. Family y = Ax+A^ of curves will correspond to a differential equation of order
and degree
30. The differential x dy + y fry = 0 equation represents a family of

ow
31. The differential equation of the family of curves + y^ -lay = 0, where a is arbitrary
constant, is
ANSWERS

4.1

e
2. 2 3. zero
5. e^'=/+C
1. 1

re
2

rFl 1

F
10. 1

"7^
8. 9. 2
6- + y =0
Jx
1

or
12. not defined 13. 2 14. 3
ou
11. -
X
ksf
1 2 -2
15. I =J(Se>“!')dy + C 16.
.V
17. y = -X
^ - +Cx
4
oo

y 19. one 20. zero 21. x^ + 1 22. 4


18. xy=Ce
d^y + y=0
Y
B

27.
24. 3 25. xy=C 26. 3, 2
23. 2 dx^
re

30. circles
29. One, Three
28. a rectangular hyperbola
oYu

31. {x^ -y^) dy = 2xy dx


ad

VERY SHORT ANSWER QUESTIONS (VSAQs)


d

Answer each of the following questions in one word or one sentence


or as per exact requirement of the
in
Re

question:
1. Define a differential equation.
F

2. Define order of a differential equation.


3. Define degree of a differential equation.
4, Write the differential equation representing the family of straight lines y^Cx+5, where C
is an arbitrary constant.
5. Write the differential equation obtained by eliminating
2 2 2
the arbitrary constant C in the
equationX -y =C .
6. Write the differential equation obtained eliminating the arbitrary constant C in the
equation xy =C^. 1/4

2d^y_ 1 +
dy
7. Write the degree of the differential equation a dx
15.60
APPLfED MATHEMATICS-XII

(. (.2 f
8. Write the order of the differential equation 1 + 1^1 =7 Ll
dx dx^
n2

9. Write the order and degree of the differential equation y = x^


dx
+ a V|l+f^
.2 fn)
Write the degree of the differential equation ^ + X ^ =2.^ log .
K)

dx^ \
dx

IJ. Write the order of the differential equation of the family of circles touching X-axis at the
origin. °
Write the order of the differential equation of all non-horizontal lines in a plane.
12

w
1.3. If sin X is an integrating factor of the differential equation — + Py=Q, then write the value
dx
ofP.

F lo
14. Write the order of the differential equation of the family of circles of radius r.
i5. Write the order of the differential equation whose solution isy = a cos x+ b sin x + c e~

ee
16. Write the order of^the differential equation associated with the primitive

Fr
y ^ + ^3 ^ whereCj, C2, C3, C4 are arbitrary constants.
1 / ● What is the degree of the following differential equation? for
ur
5x

dx dx
|-6y = logx
s
ook
Yo

18. Write the degree of the differential equation ^ + 3x = 0.


dx dx^
eB

( 2 r - n4
19.
Write the degree of the differential equation X —^ ^=0.
r

+y + X
ad

dx
ou

\ /

20. constant.
Write the differential equation representing family of curves y =mx, where m is arbitrarv?
Y
Re

f 2 )
nd

Write the degree of the differential equation x^ —^


21.
= 0.
dx^
Fi

dx
\

r ^ f^2 V
. Write the degree of the differential equation 1 + ^ =
22

dx dx^
y

2 x2

23. Write degree of the differential equation ^


dx^ + 3^1 =a2ios *2
x2
24.
Write the degree of the differential equation
dx^ 7
4^
I*
= xsm ● (^y)
dx
^
2 X \ 1/4

Write the order and degree of the differential equation 1-2+(l*.l


^
25.
+ x’^==0.
DIFFERENTIAL EQUATIONS 15.61

dy/dx = 0.
2b.
Write the degree of the differential equation 4-e

27.
How many arbitrary constants are there in the general solution of the differential equation
of order 3.
3
28. Write the order of the differential equation representing the family of curves y -=ax + a .
3 ,2
^y ^ y
2y. Find the sum of the order and degree of the differential equation y = X —
dx ■^7^-
dx

.30.
Find the solution of the differential equation xjl + i/^ dx + y -Jl + x^ di/ = 0.

w
31. Form the differential equation representing the family of curves \j = Asin x, by eliminating
the arbitrary constant A.
ANSWERS

o
dy "4 8.2

e
4. X -y + 5 = 0 .r dx -y dy = 0 6. X dy + y dx ~ 0
dx

re
9. 1,2 in. Not defined

rFl
11. 1 12. 2 13. cot X

F
14. 2 15. 3 3 1 18.1

19. 3 20. 21. 2 22. 2

r
dx
ou
23. Not defined 24. Not defined
fo
25. order 2 degree not defined
ks
26. Not defined 27. 3 2^v one 29.3

^l + x'^ +^l + y^ = C 31.


|+y =0
oo

30.
dx
Y
r eB
ou
Y
ad
d
Re
in
F
CHAPTfR 16
APPLICATIONS OF INTEGRATION

ow
16.1 INTRODUCTION
In chapter 12, we have leamt about some applications of differentiation in determining
marginal cost functions and marginal revenue functions when we were given cost and total
revenue functions. In the following sections, we shall investigate the applications of integration
in obtaining the total cost and total revenue functions when the marginal cost and marginal

e
re
revenue functions are given.
16.2 DETERMINING COST FUNCTION AND AVERAGE COST FUNCTION WHEN MARGINAL

Frl
F
COST FUNCTION IS GIVEN

We know that the marginal cost function is the derivative of the cost function. Therefore, if C
ou dC
denotes the total cost function then the marginal cost function MC is given by MC =

sor
dx

Integrating both sides with respect to a:, we get kf


C = J MC dx + k, where k is the constant of integration.
oo
The constant k can be determined, if we are given the fixed cost i.e. costs involved when .t = 0 or
Y

we are given the cost of production of a specific number of units of the commodity.
B

NOTE The constant k ahvays represents the fixed cost.


C
re

After obtaining the total cost function C, we can also find the average cost by using =~
oY
u

Following examples will illustrate the procedure of obtaining the cost function and average cost
ad

function from the given marginal cost function.


d

ILLUSTRATIVE EXAMPLES
in
Re

2
EXAMi’i L 1 The marginal cost function of manitfacturing x units of a commodity is 6 + lO.v -6.v . The
F

total cost of producing one unit of the commodity is ? 12. Find the total and average cost functions.
SOLUTION We are given that Marginal cost = 6 + lO.v 6x\
i.e. MC = 6 + 10.T - 6.\-2
dC
dx
= 6 + 10.V - 6.y^ => C = (6 + lO.Y - 6.r^) frx + /c => C = 6-y + 5.y^ - 2.y^^ +k

It is given that the cost of producing one unit of the commodity is ? 12 i.e. when .v =1, C =12
Substituting x = 1 and C = 12 in (i), we get
12 = 6+5-2 + A:=> cf = 3

Putting k = 3 in (i), we get; C = 6x + 5.x^ - 2x^ + 3. Hence, the total cost fimction is
C = 6x + 5x^ - 2x^ + 3.
C
AC =- = 6 + 5x - 2.y^ + -.
X X
16.2 APPLIED MATHEMATICS-XII

2
EXAMPLE 2 The marginal cost function MCfor a product is given bi/MC = and the fixed cost
V2FT9
is ? 14, find the average cost for 8 units of output.
2 dC 2
SOLUTION We are given that MC =
fix + 9 dx fix + 9
Integrating both sides with respect to x, we get
2 -1/2+1
C = d.v + => C =
2 {2.V + 9)
+ k => C = 2 ^2x + 9 + A: ...(i)
V2VT9 2
1
+ 1
V 2
It is given that the fixed cost is ? 14. i.e. when .v = 0, C = 14. Putting x = 0, C = 14 in (i), we get
U=lflx0 + 9 +k => U^6 + k => k==8

low
Substituting ^: = 8 in (i), we get: C = 2 fix + 9 + 8
C
Now, /1C=- => /1C=-
.V -V .Y

5 9
Wlien ,Y = 8, we obtain AC = —8 fix 8 + 9 H = h 1 — —. Hence, average cost for 8 units of

ee
4
9

F
Fr
output is f .

EXAMIM.F 3 Given that the marginal cost MC and average cost AC for a product are equal. Shozv that for
ur
the total cost C is a linear function of number of units (x) produced.
SOLUTION It is given that MC=AC
s
dC C
ook

— , where .v is the number of units produced.


Yo

dx X

dC dx
eB

=>
C X

Integrating both sides, we get


r
ou

log C = log .Y + log k => log C = log (kx) => C = kx


ad

Hence, the total cost is a linear function of x.


Y

.Y
LX'AMl’LE 4 The marginal cost function of a product is given by MC = ●. Find the total cost
^x^ +400
Re
nd

function and the average cost function if the fixed cost is X 900.
Fi

SOLUTION We have,
X dC X
MC =

yjx^ + 400 dx
/y^ + 400
Integrating both sides with respect to .y, we get
X
C = dx + k
^x^ +400
Let .Y“ + 400 = t^. Then, Ixdx = ltdt xdx = tdt dx =— dt
X

C = f
*' f^2 X
.-dt + k=>C = { I .df + k
J
=> C = f + /c=>C = y[x^ + 400 + Jc
It is given that C = 900 when x = 0. PutHng x = 0 and C = 900 in (i), we get
900=Vi00 + A: => lc=880.
APPLICATIONS OF INTEGRATION 16.3

Substituhng the value of k in (i), we get: C = + 400 + 880, which is the required total cost
function.

C + 400 880
Now, AC=-=> AC = . which gives the average cost function.
X .V X

LX AM I’LE 3 Given that the marginal cost MC and average cost AC of a product are directly proportional
to each other. Find the total cost function so that the cost of producing 2 units is tS and of producing
4 units is ? 84.
SOLUTION It is given that the marginal cost MC and average cost AC are directly proportional
to each other.
MC = >.i4C, where X is the constant of proportionality.
rfC = A., —
C ^ dC = A, —
dx

w
dx X c X

Integrating both sides, we get


log C = log (b:^) ^ C=kx ...(i)

Flo
log C = X log X + log A: => log C = log + log cf =>
It is given that C = 8 when y = 2 and C = 64 when y = 4. Putting y = 2, C = 8 and y = 4, C - 64

ee
respectively in (i), we get

Fr
2k
kx4^
S=kx 2^ and 64 = /c x 4^ =>
»
- ^=> 8 = -2 k => 2^ = 2^ => A. = 3
kx2 ^
for
ur
Putting X = 3 in 8 = A: X 2^, we
get: 8=fcx8=> k=l. Putting A: = 1 and ^ = 3 in (i), we get:
C=.y3.
k s
Yo

.3
Hence, the total cost function C is given by C = y
oo

CXAMl’Lt 6 The marginal cost MC of a product is given to be a constant multiple of number of units (x)
eB

produced. Find the total cost and the average cost function if the fixed cost is t 1000 and the cost of
producing 30 units is ? 2800.
r

SOLUTION It is given that the marginal cost MC is a a constant multiple of the number of units
ou
ad

(y) produced.
Y

MC = k Y, where X. is a constant.
dC
= kx => dC = k xdx
Re
nd

dx
Fi

Integrating both sides with respect to y, we get


C = k^ + k
2

where A: is a constant.

It is given that the fixed cost is ? 1000. This means that C = 1000 when y = 0. Substituting these
values in (i), we get:
1000 = k

Putting k =1000 in (i), we get


>.y2 -(ii)
C + 1000
2

= 30.
It is also given that the cost of producing 30 units is ? 2800. i.e. C = 2800 when y
Substituting y = 30 and C = 2800 in (ii), we get
2800 = 450 + 1000 => 450A = 1800 => ?. = 4
16.4
APPLIED MATKEMATICS-XII

Putting - 4 in (ii), we get C = + 1000. This is the required total cost function.
C 2x^ + 1000 1000
The average cost function is given by AC = —
X X
= 2x +
X

CXAMIM.L 7 If the marginal cost function at x units of output is given bpMC = ^ r, where p, q are
■Jpx + q
consta7its and the fixed cost of production is zero, find the total cost function.

ow
SOLUTION We have,
MC=-7 ^ dC p => dC= . ^
=> —=-j=^ dx
fpx + q dx fpx + q fpx + q
Integrating both sides, we get

e
/ ^ dx + k => C = 2 Jpx + q + k,

re
fpx+q '

Flr
where k is the constant of integration.

F
It is given that the fixed cost of production is zeroi.e.C =0 whenA: = 0. Putting^' = 0, C = Oin (i),
we get
0 = 2^+ k => k=-2^q
ou
sr
Putting k=- 2^ in (i), we find that the total cost function is given by C = 2
ko
X +

EXAMPLE 8 The ?nargmal cost function of a firm is MC


of
= (log .r)^ Find the total costfunction when the
cost of producing one unit is ? 20.
o
SOLUTION We have,
Y
erB

dC
MC = (log .v)^ => dx = (log.v)^ =i> dC = (log.v)^dx
uY

Integrating both sides, we get


(log X)
C =
(\ogxf dx + k=^j (logx)^
I
.1I' dx + k=x(logxf -2 X
. xdx + k
ad
do

=> C-x(logx)^-2 log X. 1 frx + A: = x(log x)^ - 2 {x log X-x} + A-


in

=> C =x(log x)^-2xlog x+2x +it -(i)


Re

It is given that the cost of producing one unit is ? 20. That is C = 20 when x = 1. Putting x = 1 and
F

C = 20 in (i), we get
20 = 2 + A: => lc=18
[v log 1=0]
Putting /: = 18 in (i), we get
C = X (log x)^ - 2x (log x) + 2x + 18, which is the total cost function.

500
EXAMPLE 9
The marginal cost fimction of a product is given by MC ●. IfC is in rupees,
72x725
determine the costs involved to increase production from 100 units to 300 units.
SOLUTION We have.
500 dC 500 500
MC = dC — dx
7^+25 dx fix + 25 72.x + 25
We have to find the cost involved to increase production from 100 units to 300 units. That IS,
i we
have to find the value of C (300) -C (100).
16.5
APPLICATIONS OF INTEGRATION

Clearly,
300 300 n 300
dC 500
C (300)-C (100) = dx = dx = 500 ^2x + 25
100
dx ^2x + 25 100
JlOO

= 500 (V^ - V2^) = 500 (25 ~ 15) = 5000


Hence, required cost = ? 5000.
_ EXERCISE 16.1

I. The marginal cost function of manufacturing units of a product is given by


MC =1000 - 200.V + x^. The fixed cost of production is f 9000. Find the total and average
cost functions.

2 The marginal cost function of manufacturing units of a product is given by


MC = 3-v^ - lO.r + 3. The total cost of producing one unit of the product is ? 7. Find the total

w
and average cost functions.
If the marginal cost function of a product is given by MC = 4 - 2x + x^, find the total and

F lo
average cost functions given that the fixed cost is? 100.
The marginal cost function of a firm is MC = 33 log x. Find the total cost function when the
cost of producing one unit is ? 11.

e
Fre
5. Tine marginal cost of producing a: units of an electronic appliance is given by MC = x^ + 1.
The cost of producing 3 units of the appliance is ? 7800. Find the cost function.
for
6. The marginal cost function of a product is given by MC = 2000 - 40.y + 3-V^, where y is the
number of units produced. The fixed cost of production is ? 18000. Find the total cost
r
You

function and the average cost function.


oks

14000
and the fixed
7. The marginal cost function MC of a commodity is given by MC =
eBo

cost is ? 18000, find the total cost and the average cost of producing 3 units of the output.
> The marginal cost function of a product is given by MC = 10 - 0.01 y + 0.0009y“. Find the total
ad
our

cost function and the average cost function if the cost of producing 10 units is ? 105.
9. The marginal cost of production is MC = 20 - 0.04y + 0.003y", where y is the number of
dY

units produced. The fixed cost of production is ? 7000. Find the total cost and average cost
Re

functions.
Fin

10. A manufacturer's marginal cost function is given by MC =0.2y+ 3. If C is in rupees,


determine the cost involved to increase the production from 60 to 70 units.
3
U. If the marginal cost of a product is given by MC = and the fixed cost is ? 2. Find the
^^
average cost for 7 units of the output.
12. The marginal cost function of a firm is given by MC = 3000 +50, where y is the
number of units produced. If the fixed cost is ? 80000, find the total cost function.
13. The marginal cost (in lakhs) of producing y units of a product is given by
MC = — e~ + 5y^ + 4. Find the total cost of production when y = 2, if the fixed cost is ? 8
2
lakhs.

14. A manufacturer's marginal cost function is given by MC =150 + , where y is the


number of units produced, If y increases from 200 to 400, find the total increase in cost.
16.6
APPLIED MATHEMATICS-XM

O.OOlx
15. The marginal cost function of a product is given by MC = 2e . Find the total cost
function if the fixed cost is X 2000.

16. The marginal cost fimction MC for a product is given by MC = ^ and the fixed cost
^4a* ^ 9
is ? 2000. Find the total cost and the average cost of producing 4 units of the out put.
17. Assuming that the marginal cost (in thousand rupees) of producing :c units of a commodity
is given by MC = + 5. Find the total cost function and the cost of production when
X = 5, if fixed cost is X 7000.

ow
ANSWERS

1. C =1000;c-10.Y^ ^ 9000
+ — + 9000, AC = 1000-10.T + — +
3 3 X

e
2. C=x^-5x^ + 3x + 7,AC=x^-5x+3 + -

re
X

3. C =4x-x^ x^ 100
AC = 4 - .V + — +

Frl
F
3' 3 X

116888
4. C = 33 (y log x~x) + 44 5. C = — {x + l)^^^{3x-2) +
ou 15 15

sor
18000
6. C = 2000y - 20x^ + + 18000, AC = 2000 - 20x + x^ +
.V

7. C = 4000 ^7x + 4 + 10000, AC 4000 /=


— J7x + 4 + 10000
kf
X '' X
oo

8. C = lOx - O.OOS.r^ + 0.0003x^ + 5.2, AC = 10 - 0.005X + 0.0003x^


5.2
Y

+
B

9. C = 20x - 0.02x^ + O.OOht^ + 7000, AC = 20 - 0.02x + O.OOlx^ +


7000
re

X
oY

0.3 :t
10. ?160 11. X- 12. C= 10000 e + 50x + 70000
u

7
ad

185 3 T ^
13. C = ? -e~^
— e lakh 14. C = 300 + 2e^ -2e 15. C=2000e
O.OOlx
d

6 2 /
0.3x 15
in

'i', ? 2002, ? 500.50 17. 10 e + 5x + 6990,10 e + 7015


Re

16.3 DETERMINING THE TOTAL REVENUE FUNCTION AND THE DEMAND FUNCTION
F

FROM A GIVEN MARGINAL REVENUE FUNCTION


In the earlier sections, we have learnt that the marginal revenue function MR is the derivative of
dR
the revenue function R . i.e. = MR , where x is the number of units sold.
dx
dR
Now, = MR R = MR .dx + k
dx [Integrating both sides]
where k is constant of integration.
When X = 0, we have R = 0. Putting x = 0, R = 0 in (i), we get Jc = 0. Substituting it = 0 in (i), we
get:
R=j MRdx
We know that: R=px, where p is the price per unit when x imits of a commodity are sold,
p = -
X ●●●(ii)
APPLICATIONS OF INTEGRATION 16.7

Substituting value of R in (ii), we obtain the demand function.


Following examples will illustrate the procedure of obtaining the revenue and demand
functions when marginal revenue function is given.

ILLUSTRATIVE EXAMPLES
ab
- c. Prove that the total
V. ]
The marginal revenue function of a ifrm is given by MR -
ab ix-bf a

revenue function and the demandfunction are given byR = b-x -cx-aandp = b-x - c respectively.

ow
SOLUTION We have.
ab dR ab
MR = -c => -c

{x-bf dx {x-bf

e
Integrating both sides with respect to x, we get

re
ab ab
R= i - c \ dx + k => R = - -cx + k

Frl
F
(x-bf X ~b

When X = 0, we have R = 0. Putting x = 0 and R = 0 in (i), we get


ou
sor
0=fl + ^:=> k =-a

Putting k = -ain (i), we get kf


ab ab
R =- -cx -a => R = - cx - a, which is the total revenue function.
x-b b-x
oo

Let p be the price per unit when x units of the product are sold. Then,
Y
B

ab a
R=px => p = — => p = -c —
x{b -x) X
re
oY

a
- c, which is the required demand function.
u
ad

J XAMt’l f -
If the marginal revenue function of a commodity is MR = 9 - 6x^ + 2x, find the total
d

revenue and the corresponding demand fimetion.


in

SOLUTION We have,
Re

dR
MR = 9 - 6x^ + 2x => = 9 - 6.x^ + 2x
F

dx

Integrating both sides with respect to x, we get


R =9x -2.r^ +x^ +k ...(i)

When X = 0, we have R = 0. Putting x = 0 and R = 0 in (i), we get k=0.


Putting cf = 0 in (i), we get: R = 9x - 2x^ + x^ which is the required total revenue function.
If p is the price per unit when x units are sold, then
R = px => p= — X
p = 9 - 2x^ + X, which is the corresponding demand function.
a

A firm has a marginal revenue function given by MR = x + b - c, where x is the output

^ . . , a , x+b
and a, b, c are constants. Shozv that the demand function is given pi/ p = - log — -c.
X V ^
16.8
APPLIED MATHEMATICS-XIt

SOLUTION We have.
a dR a
MR = -c => -c
x +b dx x +b

Integrating both sides with respect to x, we get: R = <? log (.r + b) - cx +


We know that R=0 when .v = 0. Putting .y = 0 and R = 0 in (i), we get:
0 = a\ogb + k => k =-a \ogb
Substituting the value of k in (i), we get
R =alog{x + b) - cx -alogb
D 1 (x+b
R=fllog —— - cx, which is the total revenue function.
I b
If p is the price per unit when y units are sold. Then, the demand function is given by
R a , 'xY +
4 b^
=> P=-log --

w
p = — c
Y Y I b

F lo
A manufacturer's marginal revenue function is given by MR = 275 - x - 0.3y^. Find
LXAMPI.E4

the increase in the manufacturer's total increase in revenue if the production is increased from 10 to 20
units.

e
Fre
SOLUTION We have,
dR
MR = 275-y-0.3y^ => = 275-y-0.3.y^
for
dx

We have to find the total increase in revenue if the production is increased from 10 to 20 units.
r
That is, we have to find R (20) - R (10).
You
oks

Clearly,
20 20 l20
eBo

dR 2
R(20)-R(10) =
dx dx=j (275-y-0.3y^) frY= 275y-^^—O.Iy^
2
= 4500-2600=1900
10 10 10

Hence, increase in revenue = ? 1900.


our
ad

EXERCISE 16.2
Find the total revenue and demand functions for each of the following marginal revenue function (1-9)
dY
Re

i- MR = X + 1
-3 T MR = 100 - 9x^ 3. MR = a + — ^
X + b
(x + b)'^
Fin

ab 4 y'i
4. MR - c 5. MR -1 6. MR = 1 - —
(x + b)^ (2y 4 3)2 10

MR = log (y + 1) 8. MR = 2 - 6y 9. MR = 9 - 4y^
4
10. Given that the marginal revenue function MR = -1, show that the average
(2y + 3)2
4
revenue function is u = 1.
6y 4- 9

a
11. A firm has the marginal revenue function given by MR - c, when y is the output
(Y4l))2
a
and a, b, c are constants. Show that the demand function is given by x -b.
b{p + c)
APPLICATIONS OF INTEGRATION 16.9

12. If the marginal revenue function for output x is given by MR = + 5, find the total
(x + 2)
revenue function and the demand function.
ax
13. A firm's marginal revenue function is given by MR = - - + c. Show that the
^ ^ x + b (x + b)
a
demand fimction is p = + c.
x +b

14. If the marginal revenue function is given by MR =5 + — , find the total revenue
{X + 2r
function. Also, find the total revenue earned when the number of units produced is 4.

w
15. If the marginal revenue function of a commodity is MR = + 2, find the total
(:r + l)-
revenue function. Also, find the total revenue when the price is ? 2.20.

Flo
16. If the marginal revenue and marginal cost for an output a: of a product are given as
MR = 3 + 2:c and MC =5-^x+Sx"^ and if the fixed cost is zero, find the profit function and

ee
the profit when output is x = 2.

Fr
17. The marginal revenue MR and marginal cost MC of a product are approximated as
MR = 16x - x^ and MC = 81 - 2Qx + 2x^ respectively. If the fixed cost is zero, determine the
for
ur
profit maximizing output and the total profit at the optimal output.
18. A travel agent arranges a tour from Delhi to Shimla and back. He has 60 seats in a special
ks
bus at booking amoimt of ? 450 per seat provided all seats will be occupied. However, for
Yo

every increase of ? 15 in the booking amount one seat will remain vacant. He also plans to
o
Bo

provide a mineral water bottle and snacks costing ? 60 per seat. Find the relation ship
between profit and number of seats remained vacant. What is the number of vacant seats
re

for which profit is maximum?


19. The cost function of a product of a firm per-day for X units is given by
ou
ad

x^
C(x) = 3000 + 27\x + whereas the revenue function is given R{x) = 3300 + lOOOx -—,
Y

0 < X < 30. Calculate;


nd
Re

(i) the number of units that maximizes the profit,


Fi

(ii) the profit per unit when the maximum profit level has been achieved.
20. Find the relationship between the slopes of marginal revenue curve and average revenue
curve, for the demand functionp = a-bx.
21. The cost function of producing x units of a product is given by C (x) = ^jax + b, a and b are
positive. By using derivatives show that the average and marginal cost curves fall
continuously with increasing output.
22. The total cost function of a product isC =2x^ -5x^ +7x, find the marginal average cost
function (MAC). Whether MAC increases or decreases with increase in outputs?
23. A cable T.V. operator has 5000 subscribers, each of them pays ? 100 per month. The
operator proposes to increase the subscription and he foimd that for every increase of ?
0.50,10 subscribers will discontinue the service. Find what increase in the subscription rate
will increase maximum revenue and what will be maximum revenue?
16.10
APPLIED MATHEMATICS-XIl

:The total cost function of a product isC=a + bx + cx^, where a,b>0 and c > 0. Show that
the average and marginal cost are equal at minimum average cost.
25. A chartered plane has 200 seats and charges of ? 3000 are taken per seat witli an additional
charge of ? 150 for each subsequent cancellation. Determine the total revenue function in
terms of the number of cancellations before the departure of the plane. Also, find the
number of cancellations for which total revenue is maximum.

26. A car is driven on CNG (compressed natural gas). The consumption of gas y kg per km, ic
related to the speed x km/hr at which the car is driven by the equation y=— + . Find
4x 2000

the speed at which the car be driven to minimize gas consumption.


27.
The cost function C for a commodity is given byC =x^ +5x+ 36, where .x is the number of

w
imits produced. Find the output for which average cost AC is increasing and the output for
which AC is decreasing, with increasing output. Also, find the marginal cost function.
28.

F lo
The total cost of producing and marketing x units of an item is given by C = x^ - 22x +160.
The government imposes a tax at the rate of ? 2 per unit. What will be the level of output for

e
minimum cost before and after tax? Why does the producer found it better to add tax in his

Fre
cost?
for
ANSWERS
r
9 0 9x
R = log (x + 1) - 3x, p = _3 -X^,
2. R =100x-- n=100- —
You

2 ^ 2
oks

X 1 ax a
3. R
eBo

ax -
p=a- A. R = -cx,p = -c
X +b ' X {b + x) x + b x +b

2 2 4
i? = - - X + — , V = -1 6. R = 20x e x/10 -x/10
2x+3 3 ^ ,p=20e
our

6x + 9
ad

log {x +1)
7.
R = X log (x + 1) - X + log (X +1), p = log (X + 1) - 1 +
.X
dY
Re

4
8. R=2x- 3x^, p = 2 - 3x 9. R =9x--
Fin

3x C 3
12. R = + 5x, V = + 5 14. 5x —+ 3, ? 22
x + 2 x + 2 x + 2

15, ^ + 2x + l, ?8.80
x + 1
16. - 64x;2
2

9,0
18. P(x) ={60-x) (390 + 15x); 17
h>. (i) 27 (ii) ? 497.11 '!r>. Slope of MR curve = 2 (Slope of AR curve)
■ MAC = 4x - 5, Increase with output ? 75, Max. revenue = 111500
2“ R =600000 + 27000x-150.r^, 90 50 km/hr

More than 6 units. Less than 6 unit, MC = 2x + 5


11 units (Before tax), 10 units (after tax)
appl'Cations of integration 16.11

16.4 DETERMINING MAXIMUM PROFIT WHEN MARGINAL REVENUE AND MARGINAL


COST FUNCTIONS ARE GIVEN

If C, R and P denote respectively the cost, revenue and profit functions, then
P=R-C

dx dx dx
dP
= MK-MC ...(i)
dx
dP
Thus, if MR and MC are given, we obtain by using
dx
dP

w
= MR-MC
dx

Integrating both sides with respect to x, we obtain


P = ■ (MR-MC)dx + k,

o
e
where k is the constant of integration which can be determined if we are given fixed cost or loss

re
at zero level of output.
For maximum profit, we must have
rFl
F
dP
= 0
dx

r
[Using (i)]
ou
MR-MC = 0
MR =MC
fo
ks
Solve this equation to get the value of .r.
From (i), we obtain
oo

d^P
= A(mR)-—(MC)
Y
eB

dx^ dx dx

d'^P ..d'^P
Substitute the value of x to check the sign of ^.if dxy is negative, then profit is maximum
r

dx
ou
Y

at that particular level of output.


ad

We may use the following algorithm to find the maximum profit.


d

ALGORITHM

srj;r i Obtain MR and MC.


Re
in

dP dP
by using = MR-MC.
sill'll
Let P be the profit function. Then, find
F

dx dx
dP
-111’111 Put
dx
= 0 i.e. equate MR and MC and solve the equation obtained to find the values ofx.
dP
si 1 r I
Differentiate both sides of- dx = MR -MC to obtain

d^P
dx^

dx
(MR)--f-(MC)
dx

d^P
sn;r v Substitute the values ofx obtained in step III in — one by one. The value of x for which
dx
d^P
dx
^ < 0 gives the level of output at which the profit is maximum. Tofind the maximum profit,
dP
integrate both sides of = MR - MC to get the expression for P.
dx
16.12
APPLIED MATHEMATICS-XII

.-.TEP VI
To find the constant of integration in step V. Put .v = 0 and P = -Fixed cost in the expression
for P. Substitute the value of constant of integration in the expression for P to get the final
expression for P.
Following examples will illustrate the above algorithm.
ILLUSTRATIVE EXAMPLES

tXAMPi.E 1 A company has approximated the marginal cost and marginal revenue functions for one of
its products by MC = 81 - 16a' + x^ and MR = 20.r - 2.t^ respectively. Determine the profit maximizing
output and the total profit at the optimal output, assuming fixed cost as

w
zero.

SOLUTION Let P be the profit fimction. Then,


dP
= MR-MC
dx
dP
= {20x-2x^)-{81M6x + x^}

o
e
dx

re
dP
= -81 + 36x-3x-

Frl
dx

F
dP d^P
= -81 + 36a: - 3.v^ and = 36 - 6.V
dx dx^
ou
sor
For P to be maximum, we must have
dP
= 0 kf
dx

-81 + 36a:-3.y^ =0
oo

-3(a:^-12.v + 27)=0
Y
B

3 (a:-3) (.r-9) = 0 => a: = 3 or, .a =9


We find that
re
oY
u

= 36-6x 9 =-18 < 0.


dA.'^
ad

2x=9

Thus, the output a: = 9 gives maximum profit.


d

From (i), we obtain


in

dP
=-81+ 36.T-3.1-2
Re

d.v
F

Integrating both sides, we obtain


P=-81x + 18x^-x^ + k ...(h)
When .r = 0, fixed cost = 0 i.e. there is no profit. So, putting x = 0, P = 0 in (ii), we obtain
P=-81x + 18a-2-.v^
Putting X = 9, we obtain
P=-81x9 + 18x92-9^=0
Hence, there is no profit when 9 items are produced.
Rl;MARK As the maximum profit is zero, so the company runs in loss.

EXAMPLE 2 The marginal cost and marginal revenue functions of a product are MC =20 + — and
20
MR =
30 respectively. The fixed cost is 200. Determine the maximum profit and number of items
produced for this profit.
APPLlCATiONSOFlNT.- 16.13

SOLUTION Let P denote the profit function. Then,


dP
= MK-MC
dx
dP
= 30- 20 + —
dx 20
dP
= 10-—
dx 20

Integrating both sides with respect to .r, we get


.v2
P =10x-—+ /c
40

ow
where k is the constant of integration.
It is given that the fixed cost is ? 200.
P =-200 when .V = 0

Putting -T = 0 and P = -200 in (i), we obtain k = -200.

e
Putting k = -200 in (i), we obtain

re
v2
P=10.Y-—-200

rFl ...(ii)

F
40

dP d^P 1
= 10-— and

or
rf.X-2 20
ou
dx 20

For maximum or minimum values of P, we must have


ksf
dP X
= 0 =>10-—= 0 => .x=200.
dx 20
oo

d^P 1
Y

Clearly, < 0 for all .Y.


B

dx^ 20

Hence, profit P is maximum when y = 200 i.e. 200 items are produced.
re

Putting Y = 200 in (ii), we obtain


(200)^
oYu
ad

P=10x200- -200=800
40

Hence, profit P is maximum when 200 items are produced and the maximum value of profit is
d

?800.
in
Re

V M I’LE A compnm/ suffers a loss of^ 1,000 fits product does not sell at all Marginal revenue and
Marginal cost functions for the product are gwen by MR = 50 - 4y and MC = - 10 + y respectively.
F

Determine the total profit function, break-even points and the profit maximization level of output.
SOLUTION Let P denote the profit function. Then,
dP
= MR-MC
dx

dP
= (50-4.v)-(-10 + y)
dx

dP d^P
= 60 -5y and = -5
dx dx^
For maximum value of P, we must have
dP
= 0=> 60-5y = 0=> y = 12.
dx
16.14
APPLIED MATHEMATICS-XII

d^P
Clearly, = -5 < 0 for all at.
dx^
So, profit P is maximum when 12 units are produced. Thus, the profit maximizaHon level of
output is 12 units.
dP
Now, = 60-5a:
dx

P = J (60-5a:) dx+k [On integrating]


P =60x-~x^ + k ...(i)
2

where k is the constant of integration


It is given that the company suffers a loss of ? 1000, if its product does not sell at all i.e. P 1000
at a: = 0. Substituting these values in (i), we obtain k = -1000. Putting k = -1000 in (i), we obtain

w
P=60a---a-^-1000
2

This is the total profit function. For break-even points


P = 0 60a: x^ -1000 = 0

F lo
5x^ -120a: + 2000 = 0 => - 24.v + 400 = 0

ee
This equation does not give real values of a:. So, there is no break-even point.

Fr
I.XAMI’I (
4 A convpanxj suffers a loss of^ 121.50 if none of its products does not sell at all. Marginal
revenue (MR) and Marginal cost (MC) functions for the product are given by
for
MR = 30 - 6a: and MC = -24 + 3a:
ur
Determine the total profitfunction, the break-even points and the total profit between break-even points.
s
SOLUTION
Let P denote the profit function. Then,
ook
Yo

dP
-=MR-MC
dx
eB

dP
dx
= (30-6a:)-(-24+3a)
r

dP d}p
ou
ad

=54-9x and = -9
dx dx^
Y

For maximum profit, we must have


dP
Re
nd

= 0=>54-9x = 0=>x = 6
dx
Fi

d^P
Clearly, - -9 < 0 for all x. Thus, the profit is maximum when 6 units are sold.
dx^
This is the profit maximization level of out put.
Now,
dP
= 54-9x
dx

P=J (54-9x)dx [On integrating both sides]


P^54x--x- + K
2

It is given that the company suffers a loss of ? 121.50 if none of its products is sold. Therefore,
P = -121.50 when x = 0. Substituting these values in (i), we obtain k = -121.50.
Putting k = -121.50 in (i), we obtain
16.15
APPLiCATiONS OF INTEGRATION

P =54.r--.r^-121.50
2

For break-even points, we must have


P -0

54,y--.v^-121.50 = 0
2

9.t^-108.v+243=0
-I2x + 27 = 0 => (X - 3) (.V -9) = 0 => X = 3, 9
The total profit P between the break-even points is given by
9
Pdx =
U 5^x--x^
9 2 121.50 dx
2
3 3

w
= 27.v^-^.v 3 243 1^ -V
L 2 2 J3
243 243
= f27x9^--x9^

F lo
x9 - 27x3^-~x3^ X 3
2 2 2
V 2

= 27(92-3^)-|(93-33)-^(9-3)

ee
Fr
o 243
x6=162
= 27xl2x6--x6x(81+27
2
+ 9)-^
2

Hence, the total profit between the break-even points is ? 162. for
ur
, '.AMPLE - The marginal cost (MC) of producing x units of a commoditi/ in a day is given as
MC = 16 -a -1591. T/je selling price is fixed at ^9 per unit and the fixed cost is ? 1800 per day. Determine:
ks

(i) Cost function (ii) Revenue function (Hi) Profit function, and (iv) Maximum profit that can be obtained
Yo
oo

in a day.
(i) Let P, C, and R be the profit function, the cost function and the revenue function
eB

SOLUTION

respectively. It is given that


MC =16.t-1591
r
ou
ad

dC
= 16.t-1591
dx
Y

Integrating both sides with respect to a:, we obtain


nd
Re

C = f (16.v-1591)dx
Fi

C=8.\-^-1591.r+K
where K is an arbitrary constant.
It is given that the fixed cost is ? 800 i.e. C = 1800 when x = 0. Substituting these values in (i), we
obtain K = 1800. Tlierefore, the cost function is given by
C=8.y^-1591x + 1800
(ii) The selling price is Hxed at ? 9 per unit. So, the revenue function R is given by R = 9x.
(iii) the profit function P is given by
P=R-C

p^gx-(8x^ -1591X + 1800)


P =-8.v^+1600y-1800
(iv) We have.
P = -8y^ + 1600y-1800
16.16
APPLIED MATHEMATiCS-XIl

dP ifp
= -16.t + 1600 and = -16
dx dx^
For maximum profit, we must have
dP
= 0 ^ -16.V +1600 = 0 => .V = 100
dx

d^P
Clearly, = -16 < 0 for all .t.
dx^
Thus, P is maximum when x = 100. The corresponding profit is given by

ow
P =-8x100^ +1600x100-1800 =78,200
Hence, maximum profit = ^ 78,200.
Suppose when x units of a commodihj are produced, the demand is p{x) =58 -.v^ rupees
V \MI"

e
re
per unit, and the marginal cost is MC=6 + Assume that there is no overhead. Find the (i) total
levcnue and marginal revenue, (ii) the value ofx that maximizes profit.

Flr
F
SOLUTION (i) P and R denote respecHvely the profit function and the revenue function. Then,
R =xp{x) => R =58.v-x^
dR
ou
sr
MR = = 58-3.^2.
dx
(ii) We know that
dP fo
k
oo
= MR-MC
dx
Y
dP
= (58-3.t^)- V 6 + —x~
●>
reB

dx 4

dP d^P 13
= 52- — and
uY

=>
X
dx dx^
4 2

For maximum profit, we must have


ad

dP
do

= 0=>52-—x^=0^ =52^x2 = 16 ^ X = 4
dx 4 4

d^P 13
in

When X = 4, x4<0.
rfx2
Re

2
F

Hence, profit is maximum when x = 4.

EXERCISE 16.4
1.
The marginal cost (MC) and marginal revenue {MR) of a firm are given as MC = 4 + 0 08x
and MR = 12, where x denotes the level output. Compute the maximum profit of the firm
given that the initial fixed cost is ? 100.
1
The marginal cost of production is found to be MC = 1000 - 20x + x“, where x is the number
of units produced. The hxed cost of production is ? 9000. Find the cost function If the
rnanufacturer fixes the price per unit at ? 3400, find the profit function and the sales volume
that yields maximum profit.
3.
At the production level x, the marginal cost of production of a 11
firm is? 4 + —X and the
10
marginal revenue is ? 15. The overhead cost is ? 100. Find the maximum profit the firm can

earn and the level of product for this profit.


16.17
APPLICATIONS OF INTEGRATION

10
4. A manufacturer estimates marginal revenue (M.R.) to be ^ per unit when the level of
2
production is .v units. The marginal cost (MC) has been found to be ? — per unit. Suppose
the manufacturer's profit is X 520 when the level of production is 16 units. What is
manufacturer's profit when level of production is 25 units?
5. The marginal cost and marginal revenue of a firm are given as MC = 14+ x and
MR = 36-24.V+ 3.v“. Determine the maximized profit using definite integration, assuming
fixed cost is zero.

6. The marginal cost and marginal revenue of a firm are given as MC = 3.r -118.y +1315 and
MR =1000 -4.V. Determine the maximized profit using definite integration, assuming fixed
cost is X 595.
ANSWERS

; Max profit = X 300 and .v = 100 .3


: C = IOOOy -lO.v^ + — + 9000, Profit = 2400-v + lO.v^ -9000, Y = 60

w
3 3
6. X 15,330

F lo
Max Profit = ? 450 at.Y =100. 4. T466.20 ?10.5

16.5 CONSUMER’S AND PRODUCER S SURPLUS


Let us recall that the consumer's demand function is the relation p =D{x) between price p per

ree
of a commodity and the quantity .y of the
unit that a consumer is willing to pay for .y units for F
commodity. The graph of p=D(.y) is known as the consumer's demand curve as shown in
Fig. 16.1. In the consumers's demand function price per unit i.e. p is the independent variable
and demand .Yis the dependent variable. It is to note here that in Fig. 16.1, the horizontal axis is
used for the dependent variable .y and the vertical axis for the independent variable p which
is in
Your

contrast with the usual practice of drawing the graph of a function in mathematics. The purpose
oks

is to be consistent with the convention that most economists use. The demand function p=D (.y)
expects to pay less per unit for large
eBo

is usually a decreasing function since the consumer


quantiHes of the commodity. The area of the region under the demand curve p =D(x) from y = 0
to Y = -Yq represents the total amoimt that the consumers are willing to spend to get Yq units of
ad

the commodity. The point where the demand curve cuts the vertical axis determines the price at
our

which the consumers are unwilling to buy the commodity at all.


P
Re
Y

t
Find

Amount consumers are


a.
willing to pay to get -Xq
c units of commodity
o
a. 6.<?.
01
U
C
Ch P=D(x)
X
0 ^0
1 Quantity Demanded (x)
Fig 16.1 Demand curve

The producer's supply function p - S (y) is the relation between the price p per unit that the
producer is willing to accept for selling y units of the commodity and the quantity Y of the
commodity supplied or sold by the producer. The graph of p = S(y) is caUed the producer s
supply curve or simply supply curve as shown in Fig 16.2. It is usually an increasing function
since a higher price per unit is an incentive for the producer to make more units available for
16.18
APPLIED MA THEMAHCS-XII

sale. The point where the supply curve meets the vertical axis determines the price at which the
producer is not willing to sell the commodity at all. The area of the region under the supply
curve p = S(j;) from a: = 0 to x = Xq represents the total amount the producers receive when ^0
units of the commodity are supplied or sold.
P

t
P=S{x)
Total amount
e-
c producers receive
u
when .In xmits are
Cl supplied
<j
u
CL.

-T=0

w
0 ●^0 X

Quantit)--Supplied (.y)
Fig. 16.2

The equilibrium point (.\>, p^) is the intersection

Flo
e
point of the consumer's demand curve and

re
producer's supply curve. It is the point at which sellers and buyers come together and purchases
and sales actually happen. The area of the region under the demand curve p =D(x) and above

rF
the horizontal line p = p, represents the consumer's surplus at equilibrium price p, and the area
of the region enclosed by the supply curve p = S(.v), horizontal line p = p, and the vertical axis
ur
represents the producer's surplus at equilibrium price pg as shown in Fig. 16.3. Let
discuss these two in detail.
fo us now
ks
Yo
oo

f
B

p=S(.v)
S
re

Consumers
c
surplus
u
ad

a>
Q.
P=Pe
Yo

^Pe
c-
Prodiicer's
surplus -V=A-, p=D(x)
d
Re

O
in

Quantity (.r)
F

Fig. 16.3

16.5.1 CONSUMER’S SURPLUS (CS)


In order to understand the concept of consumer's surplus, let us take an example of a person X
? 90,000 for the T V set in dining lobby, ? 70,000 for master bed room and ? 50,000 for the
second bedroom. If the market price is ? 60,000, then he buys only two sets and would spend
only ? 2 X 60,000 - ? 120,000. This amount is less than ? 90,000 + ? 70,000 = ? 160 000 that he
saves'? 160,000-
^ 12U,UUU = ? 40,000 which is consumer's surplus of Mr. X.
In a compeati^ve economy, the total amount that consumers actually spend on a commodity is
usually less tlian the total amount they would have been willing to spend. The difference
between the two amounts can be thought of as a savings realized by consumers and is known as
the consumer s surplus.
Thus,
16.19
APPLICATIONS OF INTEGRATION

Consumer's surplus = (Total amount consumers are willing to spend)


- (Actual amount spent by consumers)
Let us assume that in a competitive economy the market is at equilibrium having (aq, Pq) as
equilibrium point. This means that there is demand of Xq units of a commodity at a price
of ? Pq per unit. In Fig. 16.4, the area of the region under the demand curvep =D{x) from a; ~ 0 to
.Y = .Yq represents the total amount that the consumers are willing to spend to get Yq units of the
commodity.
P

Sl.

3
Total \
01
&. amount ' (^o,Po)
consumers

w
u
IH
are willing
to spend p=D{x)
for Iq units

F lo
.r
O ●'O
Quantity
Fig. 16.4 Total willi ngness to spend

ee
Fr
The rectangle in Fig. 16.5 has area PqXq which represents the actual consumer expenditure for Yq
units of the commodity at price ? Pq per unit. for
P
ur
\
s
ook
Yo

c
eB

Ci
Cl. Po
01 Actual
u
u
ixpendituK
our
ad

by
consumers

X
o ^0
Quantity (.r)
Y

Fig. 16.5 Actual expenditure


Re
nd

The difference between the area in Fig. 16.5 and Fig. 16.6 i.e. the area of the region be^een the
Fi

demand curve p=D{x) and the horizontal line p=pQ represents the consumer s surplus.
P

\
Consumer's
c
y surplus
1-
0>
D.
01

L-
Ch

X
O ^'O
Quantity (x)
Fig. 16.6 Consumer's surplus
16.20
APPLIED MATHEMATICS-Xn

Thus,
Consumer's surplus (CS) = Total amount consumers are willing to spend for Xq units)
- (Actual consumer expenditure for Xg units)
●vp
Consumer's surplus (CS) = pdx-pQXQ
0

y
Consumer's surplus (CS) = D(x)dx~pQXQ. [V p = D(x)]
0

w
CONSUMER'S SURPLUS (CS) Ifp=D(x) is the demand function for a
commodity. Then the
consumer s surplus for the point (xg, pg) is defined as
●^0 ●^0
CS =
pdx-pQXQ or, CS= J D(x)(/x-PqXq

o
0 0

e
re
II.LUSTRATION I The demand function for a commodity is given by p=20-2x -X Find the
consumer's surplus (CS) when market demand is 3.

Frl
F
SOLUTION It is given that market demand is .Xg = 3. Substituting x = 3 and p = pg in the demand
function p = 20 - 2x - x^. We obtain
ou
r
Po =20-2x 3-3^ =5.

I prfX-pgXg so
CS =
kf
oo
0
3
Y

CS=f (20-2x-x-2)dx-5x3
B

3]^
re

X
CS= 20x-x^~ — -15 =(60-9-9)-15 = 27.
oY
u

0
ad

Hence, the consumer's surplus is 27.


d

ILLUSTRATION :
The demandfunction for a commodity is given byp = 100 -8x. Find the consumer’s
surplus corresponding to the market price pg = 4.
in
Re

SOLUTION The demand function is p = 100 -8x. It is given that pg = 4. Substituting p = 4 and
X - Xg in p = 100 - 8x, we obtain
F

4=100-8Xg=>Xg=12.
cs =
pdX-pgXg
0
12
2 ll2
=> CS = (100-8x) frx-4xl2 = 100x-4x -48 =(100x12-4x144)-48 =576
0 0

II.LUSTRATKJN
The demand function of a commodity is p =10 Find the consumer's surplus
when the market price is p = 1.
SOLUTION The demand function is p=10t--^. It is given that pg=l. SubstituHng p = 1 i in

p =10 e~^, we obtain


1 = 10(?"-'^ = 10 => X = log^. 10
16.21
APPLICATIONS OF INTEGRATION

.vo = log,,10
The consumer’s surplus is given by
XQ
CS = pdx-pQXQ
0

log, 10
CS = lOC fr.Y-lxlog^lO
0

_.v]log,10 -log, 10 -l)-log,10


CS =-10 6’ -log,10 = -10 {e
Jo
1
CS=-10 ---1 -log,10=9-!og,10.

w
10

16.5.2 PRODUCER'S SURPLUS (PS)

The producer's supply function p = S {x) is the relation between the price per imit the producer

Flo
modity. If .Yq units of a commodity are sold at
is willing to accept for selling .v units of the con
the supply curve p = S(x) represents the
price of po per unit, then the area of the region under

e
amount producer is willing to accept for selling .Yq units of the commodity. The area of the

re
rectangular region formed by the lines .y=.Yo and p=po represents the actual consumer

F
expenditure for xq units i.e. the actual amount the producer receives for selling Yq units. The
producer's surplus is the difference between these two as defined below.
ur
r
P
fo
ks
Yo
oo

P=S{x)
Prodiiciir's
eB

surplus
f’O
/
U
CJ
a
ur

CJ Total amount
producers are
ad

Cu
willing to accept
Yo

for selling .Vq units


.Y
o ●^0
Quantity
d
Re

Fig. 16.7
in

PRODUCER S SURPLUS (PS) Let p = S{x) be the producer's supply function for a commodity. If Xq units
F

of commodity are sold at a price ofpQ per unit, then the producers surplusfor the point (yq, p^) is defined
as
■H)
●Yp
PS=P()Yo- P dx or, PS = Po Yq - S(.Y) dx.
0 0

ILLUSTRATION I The supply function for a commodity is p = 4 +y. Determine producer s surplus if
12 units of the goods are sold.
SOLUTION We have, p = 4 + y and Yq = 12.
Substituting .Yq = 12 and p = pg in p = 4 + y, we obtain pg = 16.
-Y'O

PS = PqXq- \ pdx
0
16.22
APPLIED MATHEMATICS-XII

12
PS = 16xl2- (4 + x) dx
0

1
PS=192- 4x + ~ = 192- 4x12 + xl2^ = 192-120=72
2 2
0

Jl.l USi KA I ! '


The supply function for a commodity is lOOp =(a: + 20)2. Find the producer's
surplus zvhen the market price is ? 25.
SOLUTION ThesupplyfunctionisgivenbylOOp =(x + 20)2 Itis given that po = 25. Substituting
Po = 25 and x = .tq in the supply function, we obtain

ow
100 X 25 = (xq + 20)2 ^ 20)2 = 2500 ato + 20 = 50 => Xq = 30.
^0
PS = po.Vo- p dx
0

30

e
PS = 25x 30-
(a:+ 20)2

re
Fl
dx
100
0

F
l30
1_ (Xq+20 ^3 1 { 3 3]
ur
35^{(30.20) -20 [
PS = 750- = 750-

r
100 3

PS =750-
1
0
fo
ks
(125000 - 8000) = 750 - 390 = 360
300
Yo
oo

(I l.USIKM ● .
■ The supply function of a producer is given by p = -5 xuhere x denotes thousand
eB

imits. Find producer's surplus zvhen sales are 2000 imits.

SOLUTION The supply function is p = —


5
e^^.
ur

When sales are 2000 units i.e. .Yq = 2, we obtain


ad

2 4
Yo

[Putting Y = 2 in p = —
3
^2^]
XQ
d

PS = pqXq- pdx
Re
in

0
F

0 2 .
=> PS=-e^x2-f -g2-v dx
5 i 5
0

PS = -e'^-- .2^f 2(e^_l 3


— e
4 1
+-
5 5 2 5 5 2 2 5 5
0

ILLUSTRATIVE EXAMPLES

The demand and supply functions for a commodity are p=x^ -6x+ 16 and
1 2 4
^=3
= —Y +
—Y + 4 respectively. Find each of the follozving assuming Y <5:
(i) The equilibrium point,
(ii) The consumer's surplus at the equilibrium point.
(Hi) The producer's surplus at the equilibrium point.
16.23
APPLICATIONS OF INTEGRATION

SOLUTION The demand and supply funchons are p=D(x) and p=S(x), where
o 2 4 , 1
D(.v) = -6i' + 16and S(.v) =—^“^ ++ —X + A. 3

(i) The equilibrium point {xq, Pq) is the point at which the demand supply curves intersect.
Therefore, the equilibrium point is obtained by setting D (:e) = S(a:).
Now, D(.r)=S(Ar)
n 1 T 4
x^-6x + \6=—x^ +—X + A
3 3

~—x + 12=0 => x^-lU- + 18=0 => (x-2)(3:-9)=0=> j: = 2 [v ;r<5]


3 3

Putting .r = 2 either in p = D (x) or in p = S{x), we obtain p = 8. Thus, Xq = 2 and po - 8.

w
Hence, (2,8) is the equilibrium point,
(ii) The consumer's surplus (CS) at the equilibrium point (2, 8) is given by

F lo
^0
CS= D{x)dx-pQXQ

ee
0

Fr
CS = (.x^-6a: + 16) fr.r-8x2
0

2
for
ur
X 3 , 20
CS = --3.r^ +16.r -16= --12+32 -16 = —
3 13 3
s
Jo
ook
Yo

(iii) The producer's surplus (PS) at the equilibrium point (2, 8) is given by
eB

●"P
PS = PqXq- S(x) dx
0
r
ad
ou

■ f1 0 4
PS = 8x2- — x'^ +—X + 4 dx
3
Y

3 2 2 + Ax
.f = 16-
Re
nd

A- 40
PS=16 - + —X - +- +8
9 3 9 3 9
Fi

Jo

LXAMIM 1. : A tyre manufacturer estunates that x (thousand) radial tyres will be purchased i.e.
demanded by wholesalers when price is p=D{x)=90~ thousand rupees per tyre and the same
number of tyres zvill be supplied wheiz the price is p = S(a:) =^x 2 ^_;j. + 50 thousand rupees per tyre (i)
Find the equilibrium price and the quantity supplied and demanded at that price,
(ii) Determine the consumer's and producer’s surplus at the equilibrium price.
SOLUTION The equilibrium point (xq / Po) point at which the demand and supply curves
intersect. Therefore, the equilibrium point is obtained by setting D(x) = S(x).
Now, D(.t)=S(x)

90-—= —+ X + 50
10 5
16.24
APPLIED MATHEMATICS-XII

3 2 2
=>
—X +.t-40 = 0=> 3.r^+10.v-400 = 0=>(.v-10)(3.v + 40) = 0=t>.v-10=0=^ x = 10.

Putting .T = 10 in p = D (x) or p = S(x), we obtain p = 80.


Thus, the equilibrium point is (xg, pg) = {10, 80) i.e. the equilibrium occurs at ? 80,000 per tyre,
when 10,000 tyres are supplied and demanded,
(ii) We have, .Vg = 10 and pg = 80. The consumer's surplus (CS) is given by
An

CS =
D(x)dX-pgXg
0

10 r .2 Olio
CS =
x'^
90-— frx-80xl0 = 90x- -800
10 30
0 0
1000 200
CS= 900-

w
-800 = = 66.667
30 3

Since x is in thousands, so the consumer's surplus is ? 66667.

F lo
(ii) The producer's surplus (PS) is given by
"j?
PS = PgXg- S(x) dx

ee
Fr
0
10
1 2 ^
PS = 80xl0- -.x‘"+x + 50 dx for
^5
0
ur
llO
x3 .2
PS = 800- —+ 50x
s

15 2
ook
Yo

- flMO , 100
200 550
PS = 800
eB

+ 500 =800- +50+500 = 183.333


15 2 3 3

So, the producer's surplus is ? 183333.


our
ad

I XAMPLE3
Parts for a piece of heavy machinery are sold in units of 1,000. The demand for the parts (in
rupees) is given by p=U0-x. The total cost is given byC = x^-25.v^+2x+30.
(i) For what value ofx is the profit maximized?
Y

(ii) Find the consumer s surplus that corresponds to the maximum profit.
Re
nd

SOLUTION (i) Let P denote the profit function. Then,


Fi

P ^xp-C
P = X (110 - x) - (x ^ - 25x^ + 2x + 30)
P = -x^ + 24x^ + 108x - 30
= -3x^+48x + 108 and d^P
dP
= ~6x + 48
dx
dx^
For maximum profit, we must have
—-0 =>-3x2 + 48x + 108=0 => x^-16x-36 =0=> (x-18) (x + 2) = 0 => x = 18
d^P
Clearly, = -6xl8 + 48=-60<0
[dx^ Lt=18
Hence, profit is maximum when x = 18 i.e. when 18000 pieces are sold.
When X = 18, p = 110 -x gives p = 92
16.25
APPLICATIONS OF INTEGRATION

(ii) The consumer's surplus at x = 18 is given by


18

CS= pcix-92xis
0
18

CS= (110-:r)dA:-1656
0
,-|18
X 18^
CS = IIO.T 1656= 110x18- -1656=162
2 2
Jo

Hence, the consumer's surplus that corresponds to the maximum profit is of ? 162.
EXAMPLE 4 Suppose that the demand function for a certain commodity is p = 20-4.r" and the
marginal cost is MC=2x + 6, where x is the number of units produced. Find the consumer's surplus at

w
the sales level .to where profit is maximized.
SOLUTION Let P be the profit function and R be the revenue function. Then,

F lo
dR
R =px = 20x - 4.r ^ and MR = = 20-12.v^
dx

Now, P=R-C

ee
^_^_dC

Fr
dx dx dx
dP for
=MR-MC
dx
ur
dP
dx
= {20-12x^)-{2x + 6)
s
ook

d^P
Yo

dP
=> = 14-2.y-12a:^ and 2-24.Y
dx dx^
eB

For maximum value of P, we must have


dP
= 0
our
ad

dx

14-2x-12x^ =0
6x^ + x-7 =0
Y

6x^ +7x -6x -7 = 0 => (6x -7) (x -1) = 0 => x -1 = 0 => x = 1.


Re
nd

d^P
Fi

Clearly, = -2-24 = -26<0.


dx^ ^x=\

Hence, the profit is maximum when x -1. Therefore, Xq =1. Putting Xq = 1 in p = 20 -4x^, we
obtain Pq =16.
The consumer's surplus at xq = 1 is given by
CS= pdx-pQXQ
0

CS = (20-4.x^) frx-16xl
0

4 4 8
=> CS = 20x--x^ -16 = 20---16 =
3 3
3 Jo
16.26
APPLIED MATHEMATICS-XII

i \ ^
Suppose when x units of a commodity are produced, the demand is p = 45- rupees per
unit, and the tnarginal cost is MC =6 + —x^. Assume there is no overhead i.e. C(0) = 0. Find:

(i) the total revenue and the marginal revenue.

(ii) the value of x (to the nearest unit) that maximizes profit.
(Hi) the consumer's surplus at the value of x where proift is maximized (use the exact value ofx).
SOLUTION (i) Let R be the total revenue. Then,
R =px
R =(45-x^)x
dR
l^=45.v-.r^ and = 45-3.t^
dx

R=45x~x^ and MR=45-3x^

w
(ii) Let P be the profit function. Then,
dP
= MR-MC

F lo
dx
dP

dx
= (45-3x^) 6.^-xA
4 )

ee
dP d^P 13
= 39 x^ and

Fr
a:
dx 4 dx^ 2

For maximum profit, we must have for


dP
= 0 => 39-^at^=0 => .t^ =12 => x = 2f3=3.46-3.5
ur
dx 4

d^P
s

13
ook

Clearly, X 3.5 <0


Yo

dx^ 2
Lt=35
eB

Hence, P is maximum when


a: - 3.5. Putting a: = 2V3 in p = 45 -.v^, we obtain p = 45 -12 = 33.
Thus, we obtain pp = 33 and a:o = 2f3.
our
ad

(iii) The consumer's surplus at a-q = 2^^3 is given by


CS= J pdx-pQXQ
Y

0
Re
nd

2>/3
CS = (45-a:^) dx-33x2j3
Fi

a:-^
CS - 45 a: -
3 66^3 = (90V3 - 8^3 - 66V3) = 16^3
0

Hence, the consumer's surplus at ATf, = 2^3 is ? l6^^3 * ? 28.

.EXERCISE 16.5

Find the consumer's surplus for the demand function p = {x -5)“ when a: = 3.
Find the consumer's surplus for the demand function p=x^~6x-h 16 when .y = 1.
Find the producer's surplus for the supply function p=x^ + x + 3 when y = 3.
L The supply function for a commodity is p = f9 + x and the quantity sold is 7 units. Find the
producer's surplus.
16.27
APPLICATIONS OF INTEGRATION

3. In each of the following ((i) - (viii)) D(a:) is the price in rupees per unit that the consumers
are willing to pay for .y units of an item, and S(.r) is the price in rupees per unit that
producers are willing to accept for x units. Find:
(a) the equilibrium point
(b) the consumer's surplus at the equilibrium point,
(c) the producer's surplus at the equilibrium point,
(i) D(y)=--y
6
+ 9, S{x)=^x+1
2
(ii) D(y)=-3y + 7, S(.v)=2y + 2

(iii) D{x)={x-4f, S{x) =x~+2x + 6 (iv) D{y) =1000-10.v, S(y) =250+5y


1800
(v) D (y) = 8800 - 30y, S(y) = 7000 + 15y (Vi) D(y) = , S(y) = 2Vy + L

Y^
(vU) D(Y)=56-Y2,S(Y)=8 + y (viii) D(y) =18-2y-y^, S(y) =2y-3

w
.2
Y
and
6. For a particular commodity, the demand and su
pply functions are p = D ( y) = 16 - 100

F lo
— + 6 respectively. Determine to the nearest rupee the consumer's surplus and
Y
p = S(y) = 400

ee
producer's surplus if market equilibrium prevails,

Fr
. The consumer's demand fimction for a certain commodity is p=D{x), where
D(.v) =50-3y-y^ rupees per unit Find the number of units that will be bought if the
for
market price is ^ 32 per unit. Also, find the consumer's surplus when market price is ? 32 per
ur
unit.
s
ANSWERS
ook
Yo

45 10
4. —
1. 27 3
2
eB

5. (i) (a) (6, 4) (b)? 15 (c) (ii) (a) (1,4) (b) ^|(c) n
r

(iii) (a)(l,9)(b)?^ (0? I


ou
ad

(iv) (a) (50, 500) (b) ?12,500 (c) ?6250 (v) (a) (40,7600) (b) ? 24,000 (c) ? 12,000
Y

53824
(vii) (a) (6,20) (b) ? 144 (c) ^ 48
(vi) (a) 899,60 (b) ?50460 (c) ? -y-
Re
nd

CS = ^ 150.40, PS = ? 37.60 3 units, ^ 31.50


(viii) (a) (3, 3) (b) t 27 (c) ? 9
Fi
1 Z
DISCRETE RANDOM VARIABLE
AND ITS PROBABILITY DISTRIBUTION

17.1 INTRODUCTION
associate a real number. This
Corresponding to every outcome of a random experiment, we
can

correspondence between the elements of the sample space associated to a random experiment
and the set of real numbers is defined as a random variable. If a random variable assumes
countable number of values, it is called a discrete random variable. Otherwise, it is known as
continuous random variable. In this chapter, we will study discrete random variable and its

w
probability distribution.
17.2 DISCRETE RANDOM VARIABLE
DEFINlTIO^i
F lo
Let S be the sample space associated with a given random experiment. Then, a real vahied
function X which assigns to each event w eStoa unique real number X {w) is called a ra7idom variable.

e
Fre
In other words, a random variable is a real valued function having domain as the sample space associated
with a random experiment.
for
Thus, a random variable associated with a given random experiment associates every event to a
r
unique real number as discussed below,
You

random experiment of tossing three coins. The sample space of eight possible
oks

Consider a
outcomes of this experiment is given by
eBo

S = {HHH, HHT, HTH, THH, HTT, THT, TTH, TTT)


Let X be a real valued function on S, defined by
X (le) = Number of heads in ze € S.
our
ad

Then, X is a random variable such that:


X (HHH) = 3, X (HHT) = 2, X (HTH) = 2, X (THH) = 2
X(HTT) = 1,X(THT) = 1,X(TTH) = 1, and X (TTT) = 0
dY
Re

Also, if u? denotes the event "getting two heads", theme = {HTH, THH, HHT} and, X(ze) =2.
Fin

Similarly, X associates every other compound event to a unique real number.


that X is a random
For the random variable X, we have range (X) = {0,1, 2, 3} and we say
variable such that it assumes values 0,1, 2,3. This random variable can also be descnbed as the
number of heads in a single throw of three coins.
Now consider the random experiment of throwing an unbiased die. Let Y be a real valued
function defined on the sample space S = (1, 2, 3, 4,5, 6[ associated with the random
experiment, defined by
1, if the outcome is an even number
Y{w) =
- 1, if the outcome is an odd number

Clearly, Y is a random variable such that:


y(l) = .-l,Y(2) = i,y(3)=-l,Y(4)=l,Y(5)=-landY(6)=l
random variable such that it assumes
Here, range (Y) = {-1,1}. Therefore, we say that Y is a
values -1 and 1.
17.2
APPLIED MATHEMATICS-XIl

ILLUSTRATION 1
Consider a random experiment of tossing three coins. Let X be a real valued
function defined on the sample space S = {HHH, HHT, HTH, THH, HTT, TTH, THT, TIT) such that
X (w) = Number of tails in w e S.
Then, X is a random variable such that
X(HHH) = 0,X{HHT) = 1,X{HTH) = 1,X{THH) = 1,X(HTT) = 2
X(THT) = 2,X(TTH) = 2 and X{TTT) =3
Clearly, range ofX is {0,1, 2, 3}.
ILI.USTRATION 2 Consider a random experiment of throwing a six faced die. Let X denote the number
on the upper face of the die. Then,
X(l) = 1,X(2) = 2,X(3) = 3,X(4) = 4, X(5) = 5andX(6) = 6

X*^^{f^2 3 variable which assumes values 1, 2, 3, 4, 5, 6 i.e. range of


nil's I RATION 3 Let there be a bag containing 5 white, 4 red and 3 green balls. Three balls arc drawn
If X denotes the number of green balls in the draw. Then, X can assume values 0, 1, 2, 3 Clearlu X is a

w
random variable with its range = {0,1, 2, 3}.

F lo
FRATii )\ A pair of dice is thrown. IfX denotes the sum of the numbers on two dice, then X
assumes values 2,3, 4,..., 12. Clearly, X is a random variable with its range {2, 3, 4, ,12}.
17.3 PROBABILITY DISTRIBUTION

ee
In the previous section, we have defined random variable. Now, consider a random experiment

Fr
in which three coins are tossed simultaneously (or a coin is tossed three times). Let X be a
random variable defined on the sample space for
S = {HHH, HTH, THH, HHT, THT, TTH,HTT, TTT} such that
ur
X (w) ~ Number of heads in re e S.
Clearly, X assumes value 0,1,2,3.
s
ook

Now, 1
Yo

P(X = 0) = Probability of getting no head = P{TTT) =


8
eB

P(X = 1) = Probability of getting one head = (HTT or THT or TTH) = -


8
our

P (X = 2) = Probability of getting two heads = P {HHT


ad

or THH or HTH) = —
8
and.
P (X = 3) = Probability of getting 3 heads = P {HHH) = -
8
Y

These values of X and the corresponding probabilities can be


Re

exhibited as under:
nd

X: 0 1 2 3
Fi

1 3 3 1
P(X):
8 8 8 8

This tabular representation of the values of a random variable X and the corresponding
probabilities is known as its probability distribution.
The formal definition of the probability distribution of a random variable is as given below.
^ ^2 with respective
probabihtiespi,p2^... ,p„,then i, ^ « p
X: ^‘1 -■^2 ^3
P(X): Pi P2 P3 Pn

is known as the probability distribution ofX.


Thus, a tabular description giving the values of the random variable along with the
corresponding probabilities is called its probability distribution.
DISCRETE RANDOM VARIABLE AND ITS PROBABILITY DISTRIBUTION 17.3

Rl-MARKl The probability distribution of a random variable X is defined only when we have the various
values of the random variable e.g. x-^, X2, Xjj together with respective probabilities p^, P2> ■■■ > Pn
n

satisfying X Pi = 1-
i~l

RI-'MAKK 2 IfX is a random variable with the probability distribution


X: ●^1 ^2
P(X): Pi Pi P.r

Then,
P{X<Xi) ^ P(X ^ xf)^P{X = X2)+ ....+P(X = Xi) = P1+P2+ + P<
P{X<Xi) = P(X = a:i) + P(X = a-2)+ + P(X = ^_i) = P1+P2+ Pi-1
P{X>Xi) = P{X = Xi)^P{X = + ....+P(X = .1^,) = Pi+Pi^i+.... + Pn

w
P{X>Xi) = P(X = Xi^ ^) + PiX = Xi^2) + ""+ Pi+ , +Pj^2 + -- + Pn
Also, P (X > Xi) = 1 - P (X < .t, ), P (X > Xi) = 1 - P (X < Xi),

F lo
P{X<Xi) = l~P{X>Xj) and, P{X<Xi) = l-P(X>.t:,-)
P(.V^<X<.ry) = P(X = .Y,-) + P(X = .V;+;i)+....+ P(X=A-)

ee
P{Xi<X<Xj) = P(X = Xi^^) + P{X = Xi^2) + --+^i>^ =

Fr
The graphical representation of a probability distribution is as follows:
P(X) for
ur
s

I
ook
Yo

Pi
P„
i
eB

Pi
Ps

0 -■^1 ●'2 Y. X

I
our

Fig. 17.1
ad

ILLUSTRATIVE EXAMPLES 39
Y

Determine lohich of thefollozoing can be probability distributions of a random variable X


Re

EXAMPLE 1
nd
Fi

2 (ii) X : 0 1 2
(i) X 0 1

P(X) 0.4 0.4 0.2 P(X): 0.6 0.1 0.2

(iii) X 0 1 2 3 4

P(X) 0.1 0.5 0.2 -0.1 0.3

SOLUTION (i) Clearly,


P (X = 0) + P (X = 1) + P {X = 2) = 0.4 + 0.4 + 0.2 =1.
Hence, the given distribution of probabilities is a probability distribution of random variable X.
(ii) We have, P (X = 0) + P (X = 1) + P (X = 2) = 0.6 + 0.1 + 0.2 = 0.9 1.
Hence, the given distribution of probabilities is not a probability distribution,
(iii) We have.
P (X = 0) + P (X =1) + P (X =2) + P (X = 3} + P (X =4) = 0.1 + 05 + 0.2 - 0.1 + 0.3 =1
But, P(X = 3) = -0.1 <0
So, the given distribution of probabilities is not a probability distribution.
17.4 APPLIED MATHEMATICS-Xn

iiXAMi’1.1-. _ An unbiased die is rolled, [/the random variable X is defined as


1, if the outcome w is an even number
X (ly) - ●
0, if the outcome wis an odd number
Find the probability distribution ofX.
SOLUTION In a single throw of a die either we get an even number or we get an odd number.
Thus, the possible values of the random variable X are 0 and 1.
Now,
3 1
P(X = 0) = Probability of getting an odd number = —
6 2
3 1
P(X=1) = Probability of getting an even number = —
6 2

Thus, the probability distribution of the random variable X is given by

w
X; 0 1
1 1
P{X): 2

F lo
2

l'\AMPLE3 The random variable X has a probability distribution P (X) of the followingform, where k is
some number:

ee
k , if X = 0

Fr
2k ,if X =1
P(X = .v) =
3k ,if X = 2 for
0 , othenoise
ur
(i) Determine the value ofk (ii) Find P (X < 2), P (X < 2) and, P (X > 2).
s

SOLUTION (i) The probability distribution of X is


ook
Yo

X: 0 1 2
eB

P(X): k 2k 3k

The given distribution of probabilities will be a probability distribution, if


r

P (X = 0) + P (X = 1) + P (X = 2) = 1 ^ + 2^: + = 1 6Jc = 1 ^ Ic = -
ad
ou

6
1
(ii) P(X<2) = P(X = 0)+P(X = 1) = ^+21t = 31c = -
Y

6 2
(iii) P(X<2) = P(X = 0) + P(X = 1) + P(X = 2) = jt + 2-t+31c = 61c = 1
Re
nd

(iii) P(X>2) =1-P(X<2) = i2 = -2


Fi

■ xAMi'i ;■ -
Let X denote the number of hours you study during a randomly selected school day. The
probability that X can take the value x has the follozving form, where k is some unknozun constant.
0.1 , if x = 0
kx , if x=l or 2
P{X^x) =
k {5 -x) , if x=3or 4
0 , othenoise
ii) Find the value of k
What is the probability that you study (ii) At least Izuo hours? (iii) Exactly tivo hours? (iv) Almost
two hours?
SOLUTION
The probability distribution of X is
X: 0 1 2 3 4

P(X): 0.1 k 2k 2k k
DISCRETE RANDOM VARIABLE AND ITS PROBABILITY DISTRIBUTION 17.5

(i) The given distribution is a probability distribution.


P (X = 0) + P (X = 1) + P (X = 2) + P (X = 3) + P (X =4) =1
=> 0.1 + cf + 2fc + = 1 => 6/c = 0.9 k = 0.15
(ii) Required probability = P (X > 2) = P {X = 2) + P (X = 3) + P (X = 4)
= 2k + 2k+ k = 5k = 5 xQ15 = 0.75

(iii) Required probability = P {X = 2) ~ 2k = 2x0.15 = 0.3


(iv) Required probability = P (X < 2) = P (X = 0) + P (X = 1) + P (X = 2)
= Ql+k + 2k = 0.1 + 3k = 0.1 + 3 x 0.15 = 055

LXAMI’LE 5 A random variable X has the following probability distribution:


X: 0 1 2 3 4 5 6 7

ow
P(X): 0 k 2k 2k 3k 2k^ 7k^ + k

Find each of the following:


(i) k (ii) P(X<6) (iii) P (X > 6) (iv) P(0<X<5)
SOLUTION (i) Since the sum of all the probabilities in a probability distribution is always

e
unity. Therefore,

re
P(X = 0) + P(X=1) +

Fl
+ P(X=7) = 1

F
0+ k + 2k+ 2k + 3k+ k^ + 2k'^ + 7k'^ + k = 1
ur
IQk^ +9k-\ = 0

r
{10k-l){k-hl) = 0 fo [●.● cf > 0 +1 0]
ks
=> lOit-1 = 0
1
Yo
k =
oo

10

(ii) P (X < 6) = P(X =0) + P (X =1) + P (X =2) + P (X = 3) + P (X =4) + P (X =5)


eB

P (X < 6) = 0 + k + 2k + 2k + 3k + k^
P (X < 6) = k'^+ 8k
ur

( 1
ad

P (X < 6) + [v /c=l/10]
Yo

10 10

81
d

P (X < 6)
100
Re
in

(iii) P (X > 6) = P (X = 6) + P (X =7)


F

P (X > 6) = 2k^ + 7k^ + k


P (X > 6) = 9k^ + k
9 1
P (X > 6) Iv cf=l/10]
100 10
19
P (X > 6)
100

81 _ 19
P (X > 6) = 1 - P (X < 6) = 1 -
100 ~ 100
(V) P(0<X<5) = P(X = 1) + P(X = 2) + P(X = 3) + P(X =4)
P (0 <X <5) = k + 2k +2k + 3k
P(0<X<5) = 8k
8 4
P(0<X<5) = — [v /c=l/10]
10 5
17.6 APPLIED MATHEMATICS-XII

EXAMPLE b A random variable X can take all non-negative integral values and the probability that X
takes the value r is proportional to a'' {0<a< 1). Find P (X = 0).
SOLUTION We have,
P(X=r)cca' => P(X=r)=Xa^f■=0,l,2,....
Since sum of all the probabilities in a probability distribution is 1.
P (X = 0) + P (X = 1) + P (X = 2) +.... = 1
= 1
+
[v P(X=r)=Xa^ (given)]
X(1 + a + +....) = 1
1
= 1
ll-«

ow
=> A, = 1 - a

P(X = r) = (l-a)a^ r - 0,1,2,....


Hence, P (X = 0) = (1 - a) = (1 - a).
EXAMPLE 7 Find the probability distribution of X, the number of heads in two tosses of a coin (or a

e
Fl
re
simultaneous toss of two coins).
SOLUTION
When two coins are tossed, there may be 1 head, 2 heads or no head at all. Thus, the

F
possible values of X are 0,1, 2.
ur
Now,

P(X = 0) - P (Getting no head) = P (TT) = — or


sf
4
1
k
P(X=1) = P(GetHng one head) = P(HT or TH) = -
Yo
4 2
oo

P (X = 2) = P (Getting two heads) = P{HH) = —


B

Thus, the required probability distribution of X is given by


e

X: 0
ur

1 2
1 1 1
ad

P(X):
Yo

4 2 4

EXAM PLE 8 Three cards are drawn from a pack of 52 playhtg cards. Find the probability distribution of
d

the number of aces.


Re
in

SOLUTION Let X denote the number of aces in a sample of 3 cards drawn from a well shuffled
pack of 52 playing cards. Since there are four aces in the pack, therefore in the sample of 3 cards
F

drawn either there can be no ace or there can be one ace or two aces or three aces. Thus, X can
take values 0,1,2, and 3.
Now,
P (X = 0) = Probability of getting no ace = Probability of getting 3 other cards
^ _ 4324
52
5525
C3
4 48

P (X = 1) = Probability of getting one ace and two other cards =


Cj X C2 1128
52
5525
C3

P (X = 2) = Probability of getting two aces and one other card = ‘^C2 X % 1 72


52
5525
C3
4^
^3 1
and. P (X = 3) = Probability of getting 3 aces = 52
5525
C3
DISCRETE RANDOM VARIABLE AND ITS PROBABILITY DISTRIBUTION 17.7

Thus, the probability distribution of random variable X is given by


X: 0 1 2 3
4324 1128 72 1
PfX}:
5525 5525 5525 5525

It is to note here that the sum of the probabilities is 1 which is the condition for a distribution to
be a probability distribution.
tXAMPl.E 9 An iirn contains 4 luhiteand 6 red balls. Four balls are drazvn at random from the urn. Find
the probability distribution of the number of white balls.
SOLUTION Let X denote the number of white balls drawn from the urn. Since there are 4 white

ow
balls, therefore X can take values 0,1,2,3 and 4.
Now, p (X = 0) = Probability of getting no white ball = Probability that 4 balls drawn are red
4 _
1
10 14
C4

e
X _ _8

re
Fl
P(X=1) = Probability of getting one white ball = 10
C4 21

F
C2 X ^C2 _ 6
4

P (X = 2) = Probability of getting two white balls


ur = 10 14
C4

r
P (X = 3) = Probability of getting three white balls
fo =
ks
10 35
C4
Yo
^4 1
oo

and. P (X = 4) = Probability of getting 4 white balls = 10 210


C4
eB

Thus, the probability distribution of X is given by


ur

X: 0 1 2 3 4
1 8 6 4 1
ad

P(X):
Yo

14 21 14 35 210

EXAMPLE 10 Four bad oranges are mixed accidently with 16 good oranges. Find the probability
d

distribution of the number of bad oranges in a draw of two oranges.


Re
in

SOLUTION Let X denote the number of bad oranges in a draw of 4 oranges drawn from group
of 16 good oranges and 4 bad oranges. Since there are 4 bad oranges in the group, therefore X can
F

take values 0,1 and 2.


Now, P (X = 0) = Probability of getting no bad orange = Probability of getting 2 good oranges
16
C2 _]2
20 19
C2
4 16
C,x Cl 32
P(X=1) = Probability of getting one bad orange = 20 95
Cz

and. P (X = 2) = Probability of getting two bad oranges =


^C2 _ _3
20 95
C2

Thus, the probability distribution of X is given by


X: 0 1 2
12 32 3
P(X): 19 95 95
17.8
APPLIED MATHEMATICS-XII

F-XAMPLE 11 An unbiased die is thrown twice. Find the probability distribution of the number of sixes.
SOLUTION Let X denote the number of times six occurs i.e. the number of sixes. Since the die is
thrown twice, X can take values 0,1 and 2.
Let Sj denote the event that a six occurs on the die in /th throw and denote the event that the six
does not occur in the ith throw. Then,
P (X = 0) = Probability of not getting six in both the throws
= P (Pi and F2)
= P(fi^P2)
= P(h) PiPl) [v Pj, P2 are independent events]

ow
_ 5 5 _ 25
6 6 36

P (X = 1) = Probability of getting one six in two throws

e
= P [(Pi and S2) or (Sj and P2)]

re
= P[(PinS2)u(SinP2)l
= P(PinS2) + P(SinP2)

Frl
F
[By addition Theorem]
= P(Pi) P(S2) + P(Si) P(p2) [By multiplication tlieoremfor independent events]
5 115 10 _ 5_
ou
or
— X —I X — =
6 6 6 6 36 " 18 kfs
and. P (X = 2) = Probability of getting sixes in both the throws
= P(Sin^)
oo

= P(Si) P(S2) [By Multiplication Theorem]


Y

= -X- = 1-
B

6 6 ~ 36
Thus, the probability distribution of X is
re

X: 0 1 2
oYu
ad

25 5 1
P(X):
36 18 36
d

EXAMPLE 1: Tivo cards are drawn successively with replacement from a well-shuffled pack of 52 cards.
in
Re

Find the probability distribution of the number of kings.


SOLUTION Let X denote the number of kings. Then, X can take values 0,1 or 2.
F

Let Sj denote the event of getting a king in the i^^ draw and ,F denote the event of not setting a
king in the draw. Then,
P (X = 0) = Probability of not getting a king in the two draws
= P (Not a king in 1st draw and not a king in second draw)
= P(PinP2) = P(Pi)P(P2) ['.● Pi and P2 are independent]
48 48 144
— X —

52 52 169

P (X = 1) = Probability of getting one king in the two draws


= P ((Si n P2) or (Pi n S2)) - P (Si n P2) + P (Pj n S2)
= P(Si) P(P2) + P(Pi)P(S2) [By multiplication theorem]
4 48 48 4 _ 24
— X

52 52 52''52 ~ 169
DISCRETE RANDOM VARIABLE AND ITS PROBABILITY DISTRIBUTION 17.9

and. P (X = 2) = Probability of getting kings in both the draws


= P(SiOS2) = P(Si)P(S2) [By multiplication theorem]
_ _4 4^1
“ 52 52 “ 169
Thus, the probability distribution of X is
X: 0 1 2
144 24 1
P(X): 169 169
169

nXAMl’Lli 13 A coin is tossed until a head appears or the tail appears 4 times in succession. Find the
probability distribution of the number of tosses.
SOLUTION Let S be the sample space associated with the given random experiment. Then,

ow
S = {H,TH,TTH,TTTH,TTTT}
Let X denote the number of tosses. Then, X can take values 1, 2,3 and 4
Now,
1
P(X=1) = P(H) = i, P(X = 2) = P(TH) = P(T)P(H) = L
1

e
2 4

Fl
re
P(X = 3) = P(TTH) = P(T)P(T)P(H) = |

F
and. P(X = 4) = P{TTTHorTTTT) = P {TTTH) + P {TTTT)
ur
1
P (X = 4) = P{T) P(T) P(T) P(H) + P(T) P(T) P(T) P(T) =
or
sf
Thus, the probability distribution of X is given by
k
Yo
X: 1 2 3 4
oo

1 1 1 1
B

2 4 8

Find the probability distribution of the number of


re

EXAMPLE 14 An um contains 4 white and 3 red balls.


red balls in a random draw of three balls.
u

When three balls are drawn, there may be all red, 2 red, 1 red or no red ball at all.
ad

SOLUTION
Yo

Thus, if X denotes the number of red balls in a random draw of three bails. Then, X can take
values 0,1,2,3.
d

Now,
Re

P (Getting no red ball) = P (Getting three white balls)


in

P(X = 0) =
_ 4x 3x2 4
F

7^ 7x6x5 35
^3
^Ci X _ 18
P(X=1) = P (Getting one red and two white balls) = 7^ 35
^3
12
P (X = 2) = P (Getting two red and one white ball) =
^C3 35

^3 1
p(X = 3) = P (Getting three red balls) =
^C3 35

Thus, the probability distribution of the number of red balls is giv en by


X: 0 1 2 3

4 18 12 1
P(X): 35 35 35 35
17.10
APPLIED MATHEMATICS-XII

EXAMPLE 15 Tzoo cards are draum witiwut replacement from a well-shuffled deck of 52 cards.
Determine the probability distribution of the number offace cards (i.e. jack, Queen, King and Ace).
SOLUTION Let X denote the number of face cards in two d raws. Then, X can take values 0,1,2.
Let denote the event of getting a face card in draw. Then,
P (X = 0) = Probability of getting no face card
P(X = 0) = P(^ n
P(X = 0) = P(^)P(^/^) [By multiplication theorem]

w
=>
P(X = 0) = 36 35 _ 105
52 "" 51 ~ 221
P(X = 1) = Probability of getting one face card and one other card

e
P(X = 1) = P((P,n^) u

ro
re
P(X=1) = P{f^n^) + P(^nP2) [By addition theorem]
P(X=1) = P(Pi)P(VFi) + P(^)P(P2/^) = — x~ 36 X 16
— 96

F
52 52 51 221

Fl
P(X = 2) = Probability of getting both face cards
P (X = 2) = P {P^ n f2)

u
sr
P(X = 2) = P(Pj) P(P2/Pi) = 16 ^ 15 20
52 ^ M m

ko
o
Hence, the required probability distribution is of
X: 0 1 2
105 96
o
20
Y
P(X):
221 221 221
erB

EXAMPLE 16
Find the probability distribution of the number of green balls drawn when 3 balls are
nmm,
one by one, without replacement from a bag containing 3 green and 5 zvhite balls.
uY

SOLUTION
Let X denote the total number of green balls drawn in three draws without
replacement. Clearly, there may be all green, 2 green, 1 green or no green at all. Thus, X can
assume values 0,1,2, and 3. LetG/ denote the event of getting a green ball in draw.
ad
do

Now,
in

P (X - 0) - Probability of getting no green ball in three draws


P{X = 0) = P(^nG^nQ= P (Gf) P P nG^) =^x-x-
Re

3 5
F

8 7 6 28
P (X = 1) = Probability of getting one green ball in three draws

P(X = 1) = pf(Gin^n^) u
(Gj n G2 o G3) u (Gi n G2 n G3)

P(X = 1) = P(Gin^nQ+^(^nG2nQ + P(^n^nG3)


P (X = 1) = P (Gi) P (G2 /Gi) P (G3 /Gi n G2) + P {^) P (G2 /^) P n G2)
P (Gj) P (G^/G^) P (G3/^ n
+

lP('Y_i\_3 5 —4 + -X—X-
5 3 4 + -X-X
5 4 —3 15
fA-i; - —x-x
0 / 6 8 7 6 8 7 6 28

P(X=2) =P (Gj nG2 nG3) n(Gj nG2 nG3) ^fG^n^oGg)


PfX = 2) = P(Gi)P(G2/Gj)P(G3/GinG2) + P(G^)P(G2/G^)P(G3/G];nG2)
+
P (Gi) P (^/Gi) P (G3/G1 nG^)
DISCRETE RANDOM VARIABLE AND ITS PROBABILITY DISTRIBUTION 17.11

3 2 5 5 3 2 3 5 2 15
P(X = 2) = —X—X — + — X—X — + — X—X — = —
8 7 6 8 7 6 8 7 6 56
and.
3 2 1 1
p (X = 3) = P (Gi n G2 n G3) = P {G{) P (G2/G1) P (G3/G1 n G2) = 8 "" 7 "" 6 “ 56
Thus, the probability distribution of the number of green balls is given by
X: 0 1 2 3
5 15 15 1
P(X): 28 28 56 56

EXAMPLE 17 From a lot of 10 items containing 3 defectives, a sample of 4 items is drawn at random. Let

ow
the random variable X denote the number of defective items in the sample. If the sample is drawn
randomly, find
(i) the probability distribution ofX (ii) P(X<1)
(iii) P(X<1) (iv) P(0<X<2)
SOLUTION (i) Clearly, X can assume values 0,1,2,3 such that

e
7
C4 1

re
Fl
P (X = 0) = (Probability of getting no defective item) = 10
C4 6

F
^CjX^Cg _ 1
p (X = 1) = (Probability of getting one defective item)
ur = 10 2
C4

r
P (X = 2) = (Probability of getting two defective items)
fo =
^C2X^C2 _ _3
ks
10 10
C4
Yo
oo

and. P (X = 3) = (Probability of getting three defective items) = 10 30


C4
eB

Hence, the probability distribution of X is


X: 0 1 2 3
ur

1 1 3 1
P(X):
ad

10 30
Yo

6 2
1 1 2
(ii) P(X<1) = P(X = 0) + P(X = 1) - + —
2 3
d
Re
in

(iii) P(X<1) = P(X = 0) = -


6
F

(iv)
P(0<X<2) = P(X =1) = ^
tXAMI'l E18
We take 8 identical slips of paper, zurite the number 0 on one of them, the number 1 on three
of the slips, the number 2 on three of the slips and the number 3 on one of the ships. These slips are folded,
put in a box and thoroughly mixed. One slip is drazon at random from the box. IfX is the random variable
denoting the number zvritten on the drawn slip, find the probability distribution ofX
SOLUTION Clearly, X takes values 0,1, 2,3 such that
P (X = 0) = (Probability of getting a slip written 0 on it) = —
8
3
P (X =1) =(Probability of getting a slip written 1 on it) = —8
3
P (X = 2) = (Probability of getting a slip written 2 on it) = —8
1
and, P (X = 3) = (Probability of getting a slip written 3 on it) = 8
17.12 APPLIED MATHEMATICS-XIl

Hence, the probability distribution of X is


0 1 2 3
X:
1 3 3 1
P(.v): 8 8 8

LXAMPLt 1^/
A coin is biased so that the head is 3 times as likely to occur as tail. If coin is tossed twice,
find the probability distribution for the number of tails.
SOLUTION Let p be the probability of getting a tail in a single toss of a coin. Then, probability of
getting a head is 3p.
Since "Getting head" and "Getting tail" are mutually exclusive and exhaustive events in a
single toss of a coin.
1
P{H) + P{T) = 1 => p + 3p = 1 ^ u = —
^ 4

w
P (H) = - and P (T) = -
4 4

F lo
Let X denote the number of tails in two tosses of a coin. Then, X can take values 0,1,2.
Now,

P (X = 0) = Probability of getting no tail = Probability of getting both heads

ee
= P{HH)

Fr
= P{H)x P (H) [■.● Two trials are independent]
3 3 9 for
4 4 16
ur
P (X = 1) = Probability of getting one tail and one head
= P (HT) + P (TH)
s
ook
Yo

= P{H)P{T) + P{T)PiH) =^xl


4 4
+ lx-
4 4
= ^8
eB

1
P (X = 2) = Probability of getting both tails = P (TT) = P (T) P {T) = - x-
4 4 16
our

Hence, the probability distribution of X Is


ad

X: 0 1 2
9 3 1
Y

P(X):
16 8
Re

16
nd

nxAMPi.E 20 A die is loaded in such a way that an even number is twice likely to occur as an odd
Fi

number. If the die is tossed tzoice,find the probability distribution of the random variable X representing
the perfect squares in the txvo tosses.
SOLUTION Let p be the probability of getting on odd number in a single throw of a die. Then,
probability of getting on even number is 2p.
We have.
P (1) + P (2) + P (3) + P (4) + P (5) + P (6) = 1 [●.● Sum of the probabilities = 11
p + 2p + p + 2p + p + 2p =1
9p = 1 = i
9
Now,
Probability of getting a perfect square i.e. 1 or 4 in a single throw of a die
- 3 1
= P (1) + P (4) = p + 2p = 3p = — = -
^ 9 3
DISCRETE RANDOM VARIABLE AND ITS PROBABILITY DISTRIBUTION 17.13

Since X denotes the number of perfect squares in two tosses. Then, X can take values 0,1,2 such
that
2 2 4
p (X = 0) = Probability of not getting perfect squares in both the tosses = ^ ^ ^ 9

p (X = 1) = Probability of getting perfect squares in one of the two tosses


1 2 2 1 4
= — X - + — X — = —
3 3 3 3 9
1
P (X = 2) = Probability of getting perfect squares in both two tosses = -^ x 9

Hence, the probability distribution of X is

ow
X: 0 1 2

4 4 1
P(X): 9 9 9

e
EXAViPLl-ZI Two biased dice are throivn together. For the first die P(6) = —, other scores being equally

re
Fl 9

likely while for the second die, P{1) = - and other scores are equally likely. Find the probability

F
ur
distribution of'the number of ones seen'.

r
SOLUTION
For the first die, it is given that P(6) = ^ and other scores are equally likely.fo
ks
i.e. P(l) = P(2) = P{3) = P(4) = P{5) =pi (say).
Yo
1
P(l) + P{2) + P( 3) + P(4) + P(5) + P{6) = 1 => 5pi +i
oo

10
eB

So, for the first die, we have

P{1) = P(2) = P(3) = P(4) = P(5) = ^ and P(6) = ^


ur

2
For the second die, it is given that P(l) = - and other scores are equally likely.
ad
Yo

I.e. P(2) = P( 3) = P(4) = P(5) = P(6) = P2 (say).


2 3
d

P(l) + P(2) + P( 3) + P(4) + P(5) + P(6) = 1 => - + 5p2 = 1 => P2 = ^


Re
in

So, for the second die, we have


F

P(l) = - and P(2) = P( 3) = P(4) = P(5) = P(6) = ^


5 40

When two dice are thrown, there may not be one on both the dice or one of the dice may show
one or both of them show one. This, if X denotes 'the number of ones seen'. Then, X can take
values 0,1 and 2 such that
P(X = 0) = Probability of not getting one on both dice
= (Probability of not getting one on first die) x (Probability of not getting
one on second die)

11 1 Wi
X 1 —2 _9_ 3^^
10 / \ s^'io^'s’so
P(X = 1) = Probability of getting one on one die and other number on the other die
1 f. 2] ( 1 X —
2
=
21

10 5) [ 10 5 50
17.14
APPLIED MATHEMATtCS-XII

P(X = 2) = Probability of getting one on both dice = — x-= —

10 5 50
Thus, the probability distribution of X is as given below.
X: 0 1 2

P(X): ^ ^
50 50 50

EXERCISE 17.1
1. Wliich of the following distributions of probabilities of a random variable X are the
probability distributions?

ow
(i) X
3 2 1 0 -1 (ii) X: 0 1 2

P (X): 0.3 0.2 0.4 0.1 0.05 P (X); 0.6 0.4 0.2
(iii) X: 0 1 2 3 4 (iv) X : 0 1 2 3
P(X): 0.1 0.5 0.2 0.1 0.1

e
P (X): 0.3 0.2 0.4 0.1

re
2. A random variable X has the following probability distribution:

rFl
F
Values of X : -2 -1 0 1 2 3

P(X): 0.1 k 0.2 2k 0.3 k

r
ou
Find the value of k.

3. A random variable X has the following probability distribution: fo


ks
Values of X: 0 1 2 3 4 5 6 7 8
oo

P(X}-. a
3fl 5^ 7a 9a 11a 13a 15^ 17<?
Determine:
Y
B

(i) The value of a (ii) P (X < 3), P (X > 3), P (0 <X <5).
●L The probabilit)' distribution function of a random variable X is given by
re

Xi‘. 0 1 2
ou
Y
ad

Pi- 3c3 4c-10c^ 5c-l

where c> 0
d

Find: (i) c (ii) P(X<2) (iii) P(1 <X<2)


in
Re

3. Let X be a random variable which assumes values Xi,X2,x^,x^ such that


2P (X = x^) = 3P (X = X2) =P(X=x^)=5P{X = j;^). Find the probability distribution of X.
F

(). A random variable X takes the values 0,1, 2 and 3 such that:
P(X = 0)=P(X>0)=P(X<0);P(X=-3)=P(X=-2)=P(X = -1);
P (X = 1) = P (X = 2) = P (X = 3). Obtain the probability distribution of X.
7 Two cards are drawn from
distribution of the number of
a well shuffled pack of 52 cards. Find the probability
aces.

Find the probability distribution of the number of heads, when three coins are tossed.
Four cards are drawn simultaneously from a well shuffled pack of 52 playing cards. Find
the probability distribution of the number of aces.
!(-. A bag contains 4 red and 6 black balls. Three balls are drawn at random. Find the
probability distribution of the number of red balls.
11, Five defective mangoes are accidently mixed with 15 good ones. Four mangoes are drawn
at random from this lot. Find the probability distribution of the number of defective
mangoes.
DISCRETE RArmCl^yl VARIABLE AND ITS PROBABILITY DISTRIBUTION 17.15

I Two dice are thrown together and the number appearing on them noted. X denotes the sum
of the two numbers. Assuming that all the 36 outcomes are equally likely, what is the
probability distribution of X?
13. A class has 15 students whose ages are 14,17,15,14,21,19,20, 16,18,17,20,17,16,19 and 20
years respectively. One student is selected in such a manner that each has the same chance
of being selected and the age X of the selected student is recorded. What is the probability
distribution of the random variable X?

14. Five defective bolts are accidently mixed with twenty good ones. If four bolts are drawn at
random from this lot, find the probability distribution of the number of defective bolts.
13. Two cards are drawn successively with replacement from well shuffled pack of 52 cards.
Find the probability distribution of the number of aces.

ow
16. Two cards are drawn successively with replacement from a well shuffled pack of 52 cards.
Find the probability distribution of the number of kings.
17. Two cards are drawn successively without replacement from a well shuffled pack of 52
cards. Find the probability distribution of the number of aces,
i 3. Find the probability distribution of the number of white balls drawn in a random draw of 3

e
Fl
re
balls without replacement, from a bag containing 4 white and 6 red balls.
19. Find the probability distribution of Y in two throws of two dice, where Y represents the

F
number of times a total of 9 appears.
ur
20. From a lot containing 25 items, 5 of which are defective, 4 are chosen at random. Let X be

or
the number of defectives found. Obtain the probability distribution of X if the items are
sf
chosen without replacement.
2j. Three cards are drawn successively with replacement from a well-shuffled deck of 52
k
Yo
cards. A random variable X denotes the number of hearts in the three cards drawn.
oo

Determine the probability distribution of X.


eB

22. An urn contains 4 red and 3 blue balls. Find the probability distribution of the number of
blue balls in a random draw of 3 balls with replacement.
23. Two cards are drawn simultaneously from a well-shuffled deck of 52 cards. Find the
ur

probability distribution of the number of successes, when getting a spade is considered a


ad
Yo

success.

24. A fair die is tossed twice. If the number appearing on the top is less than 3, it is a success.
d

Find the probability distribution of number of successes.


Re
in

25. An urn contains 5 red and 2 black balls. Two balls are randomly selected. Let X represent
the number of black balls. What are the possible values of X. Is X a random variable?
F

26. Let X represent the difference between the number of heads and the number of tails when a
coin is tossed 6 times. What are possible values of X?
27. From a lot of 10 bulbs, which includes 3 detectives, a sample of 2 bulbs is drawn at random.
Find the probability distribution of the number of defective bulbs.
28. Four balls are to be drawn without replacement from a box containing 8 red and 4 white
balls. If X denotes the number of red balls drawn, find the probability distribution of X.
29. The probability distribution of a random variable X is given below:
X: 0 1 2 3
k k k
P(X): k
2 4 8

(i) Determine the value of k (ii) Determine P (X ^ 2) and P (X > 2)


(iii) FindP(X<2) + P(X>2)
17.16 APPLIED MATHEMATICS-XII

30. Let X denote the number of colleges where you will apply after your results and P(X = a-)
denotes your probability of getting admission in x number of colleges. It is given that
kx , if = 0,1
2kx /dx = 2
P{X = x) =
k (5-j), if :r = 3 or 4
0 ,if.v>4
where kisa positive constant. Find the value of k. Also find the probability that you will get
admission in (i) exactly one college (ii) at most 2 colleges (iii) at least 2 colleges.
ANSWERS
1 .... 1 8

ow
(iii) and (iv) 2.k ^ 0.1 3- (i) « = — (ii) -. -
81 9 9
1
(i) ^
3 (ii) i (iii) I
X:
'■^2 ^3 .T4

e
15 10 30 6

re
P{X):

rFl
61 61 61 61

F
X: -3 -2 1 0 1 2 3
1 1 1

or
1 1 1 1
ou
P(X):
9 9 9 3 9 9 9
ksf
X: 0 1 2
oo

188 32 1
P(X):
221 221 221
Y
B

X: 0 1 2 3
re

1 3 3 1
P(X) :
oYu

8 8 8
ad

9. X: 0 1 2 3 4
d

48 48
C4 C3 X 48c, 4C3 X 48c 1 4p
in

L4
Re

P(X): 52 52 52 52
C4 C4 C4
52
F

1(1 X: 0 1 2 3
1 1 3 1
P(X):
6 2 10 30

X: 0 1 2 3 4
91 455 70 10 1
P(X):
323 969 3^ 323 969

X: 2 3 4 5 6 7 8 9 10 11 12

P(X): — A J_ A A A A A A 2 1
36 36 36 ^ 36 36 36 36 36 36 36
17.17
DISCRETE RANDOM VARIABLE AND ITS PROBABILITY DISTRIBUTION

X: 14 15 16 17 18 19 20 21

2 1 2 3 1 2 3 1
P(X) : 15 15 15 15
15 15 15 15

X: 0 1 2 3 4

969 114 38 4 1
P(X): 253 2530
2530 253 253

X: 0 1 2

ow
144 24 1
P(X): 169
169 169

X: 0 1 2

e
144 24 1

re
P(X) :
169

Fl 169 169

F
ur 1 2
X: 0

r
1
P(X) :
188
221
32
221 221
fo
ks
Yo
oo

X: 0 1 2 3

5 15 9 1
eB

P(X): 30 30
30 30

Y: ●^3
ur

I
'^1
20
C4-r
ad

64 16 1 '-r
, r = 0,1, 2, 3, 4.
Yo

P(X): i P(X = r) = 25
^ 81 81 Q
d

0 1 2 3
X:
Re
in

27 27 9 1
P(X): 64 64 64 64
F

0 1 2 3
X:
64 144 108 27
P(X): 343 343 343 343

1 2 0 1 2
0 X;
X;
13 2 4 4 1
19
P(X): 34 34 34 P(X): 9 9 9

0,1,2. Yes -6,-4,-2, 0, 2, 4,6


0 1 2 X: 0 1 2 3 4
X;
7 7 1
P(X):
1 32 168 ^ 7^
P(X): 15 15 15 495 495 495 495 495
7
● (i) (iii) 1 k = 48 (i) 48 (») 48 (iii)
15 ' ^ 15 15 8
17.18
APPLIED MATHEMATICS-XII

HINTS TO SELECTED PROBLEMS


5. Let P(X = X3) = k. Then,
P(X = x^) =~,P{X = X2) = = X4) = -
2 '

P(X =Xi) + P(X = X2) + P{X=x^) + P(X = x^) = 1


=>
k k
- + - + k + - = 1
, k 30
=> it =
2 3 5 61
6. Let P(X = 0) =k. Then,
P(X = 0) = P(X>0) = P{X<0) => P(X>0)=fcandP(X<0)=Jc.
Now, P{X = 0) + P(X < 0) + P(X > 0) =1 => 3k =1 => k = -
3
P (X < 0) = k

w
=> P{X = ~1) + P (X = - 2) + P (X = - 3) = k
=> 3P(X = -1) = k

F lo
=> P(X = -1) = P(X=-2) =P(X = -3)=-=-
3 9

ee
Similarly, P(X > 0) = /c => P(X = 1) = P(X = 2) = P{X = 3) =-
9

Fr
13. We observe thatX takes values 14,15, 16,17,18,19, 20 and 21.

P(X=14) = A,p(x=15)
15
= l,p(x
15
= 16) = A,p(x
15
= 17) = 1,
15 for
ur
P{X=18) = ^,p(X=19)
15
= :^,P{X
15
= 20) = A^p(x
15
= 21) = 15 —
ks

Hence, the probability distribution of X is


Yo
oo

X: 14 15 16 17 18 19 20 21
eB

_ J_ A ^ ^ 1
15" 15 15 15 15 15 15 15
27. Let X denote the number of defective bulbs in a sample of 2 bulbs drawn from a lot of 10
r
ou
ad

bulbs containing 3 defective and 7 non-defective bulbs. Then, X can take values 0,1 and 2
P(X = 0) = 10C2 _ 7 P(X = 1) = ^C2 _ 1
Y

15' 10 -,P(X = 2) = 10
C2 C2 15' 15
C2
Re
nd

Hence, the probability distribution of X is as follows:


X: 0 1 2
Fi

7 7 1
P(X): — —
15 15 15

17.4 MATHEMATICAL EXPECTATION


DEFINITION IfX IS a discrete random variable zuhich
assumes values x-^, X2, x^,x„ with respective
probabilities Pi, p2> P3,, p,„ then its mathematical expectation or expected value or mean value is the
zveighted average of ail possible values where weights are the probabilities associated with the
corresponding values. The expected value or the mathematical expectation ofX is denoted bp E(X) orX.
Thus,

E(X) = Pl^l + P2X2 + P3X3 + ... + p,jX^,


Pl + P2+P3+...+ Pn
E(X) = + P2X2 + P3.Y3 + ... + PjjXji [v Pi+P2+P3+--.+Pn='^]
17.19
DISCRETE RANDOM VARIABLE AND ITS PROBABILITY DISTRIBUTION

n n

£(X) = X Pi ^i or, X = X Pi ^i
f = 1 1 = 1

RI-MAUk 1 In case of a frequency distribution x-J f ; / = 1, 2,n the mean X is given by


1
^ = Tt(/i
N
■●'^1 ■‘■/2 +

x=A .Vi /2
+ ^ 'A2 + N
N N

k
X = Pi + P2^2 + a:,, , where p,- = N

w
Thus, if we replace — by p,- in the definition of mean, we obtain the mean of a discrete random
H

variable. Consequently, the term 'mean' is appropriate for the sum S^p, Xj.

o
e
re
t^illj The mean of a random variable means the mean of its probability distribution.

rFl
In a single throw of a die, ifX denotes the number on its upper face. Find the mean or

F
lU.US I RATION 1

expected value of X.

r
SOLUTION Clearly, X can take the values 1, 2, 3, 4, 5, 6 with probabilities - each.
ou
fo
ks
So, the probability distribution of X is as given below:
oo

X : 1 2 3 4 5 6
Y

1 1 1 1 1 1
eB

P(X): ^ 6 6 6 6 6

1
1 1 1 1
1x6
r

£{X)=- X 1 + X 2 + x3 + — x4 + -x5 +
ou

o 6 6 6 6 6
Y
ad

6(6 + 1) _ 7
£(X)=lx{l
6
+ 2+3 + 4+5 + 6) = lx
6 2 2
d

u! I'S IRAI ION : If a pair of dice is throivn and X denotes the sum of the numbers on them. Find the
Re
in

probability distribution of X. Also, find the expectation ofX.


SOLUTION In a siiigle throw of a pair of dice the sum of the numbers on them can be 2,3,4,
F

12. So, X can assume values 2, 3, 4,..., 12. The probability distribution of X is as given below:

X: 2 3 4 5 6 7 9 10 11 12

1 2 3 A A A _L A A A A
P(X): ^ 36 ^ 36 36 36 36 36 36 36

£(X) =
1
X 2 + AX 3+AX4+—
36
X5+A
36'
X 6 + ■—
36
X7+—
36
X8+—
36
X 9
36 36 36
3 2 1
+ _xl0 + — xll+—xl2
36 36 36

252
= 7.
£{X) = — [2 + 6 +12 + 20 + 30 + 42 + 40 + 36 + 30 + 22 + 12] = 36
36
17.20
APPLIED MATHEMATICS-XII

A dealer in refrigerators estimates from his past experience the probabilities of his
selling refrigerators in a day, which are as follows:
No. of refrigerators sold 0 1 2 3 4 5 6
in a day:
Probability: 0.03 0.20 0.23 0.25 0.12 0.10 0.07

Find the expected value of the mwiber of refrigerators sold in a day.


SOLUTION Let X denotes the number of refrigerators sold in a day. Then, the probability
distribution of X is
X;I : 0 1 2 3 4 5
● 6
Pi- 0.03 0.20 0.23 0.25 0.12 0.10 0.07
£(X) = 0.03 X 0 + 0.20 X 1 + 0.23 x 2 + 0.25 x 3 + 0.12 x 4 + 0.10 x 5 + 0.07 x 6

w
£(X) = 0.20 + 0.46 + 0.75 + 0.48 + 050 + 0.42 = 2.81.
A salesman wants to knozv the average number of units he sells per sales call. He

F lo
checks his past sales records and comes up zuith the follozuing probabilities:
Sales (in units): 0 1 2 3 4 5

ee
Probability:

Fr
0.15 0.20 0.10 0.05 0.30 0.20

What is the expected value of the number of units he sells per sale call ?
SOLUTION Let X denote the number of imits. Then, X is a random variable with the following
for
ur
probability distribution
s
0
ook

Xj.- 1 2 3 4 5
Yo

Pi- 0.15 0.20 0.10 0.05 0.30 0.20


eB

£(X) = 0.15 X 0 + 0.20 X 1 + 0.10 X 2 + 0.05 x 3 + 0.30 x 4 + 0.20 x 5


£{X) = 0.20 + 0.20 + 0.15 + 1.20 + 1.00 = 2.75
r
ad
ou

Thus, the expected value of the number of units he would sell per sale call is 2.75.
Y
Re
nd

If X is a discrete random variable which assumes values x^, X2, x^,...., x,^ zvith the
respective probabilities p^, p2, then variance ofX is defined as
Fi

Var (X)=£(X-£(X))2
II
or. Var(X)=£(X-X)^ [V X=£(X) = 2p,.^,.]
/ = !

IfX is a random variable and a, b are real numbers, then prove that
(i) E(aX+b) =aE(X)+b

(ii) Var (flX + fj) Var (X)

Suppose the random variable X takes values .V|, X2, .1:3 ..., x,j with corresponding probabilities
P\'P2‘P3'---'Pn respectively. Then, the random variable aX + b takes values axi+b, ax2+b.
ax^+b,..., ax,j+b zuith the respective probabilities pj, p2, £3 ..., p„.
17.21
DISCRETE RANDOM VARIABLE AND ITS PROBABILITY DISTRIBUTION

(i)E(aX + b) =2 PiK+ft)
j =l

n n

= a
D PiXi+b'Zpi
i=l i=l

n n

= fl£(X) + fo V £ Pi =1 and ^ Pi Xi=E{X)


1 =1 i=l

(ii) Var(flX+b) =E (flX + b)-£(flX+b)^


n2

ow
=£ (aX + b -aE(X)-h
^2
= £ a(X -£(X))

e
r 2 2"i

Fl
re
=£ a (X-£(X))

F
= fl^£(X-£(X))2 [V £(flX)=fl£(X)l
ur
= Var (X),
or [V Var(X) = £(X-£(X))2]
sf
If X is a random variable which assumes values Xi, X2/---/Xfj with respective probabilities
k
Yo
£1/P2/-/Pn-Then,
oo

Var(X) = i =I 1 Pi(Xi-xf
e B

Var(X) = X+X^)
ur
ad

/
«
Yo

Var(X) = ;c,.^-2X S pfXf +X^


f=l J
d

n
" 2
Re

2X.X+X^ V I =Z 1 Pi=l
in

Var(X) =
F

Var(X) = X^PiXi^-2X^+X^
Var(X) - i^pjX.^-X^
/ n2
«

Var(X) = 2^. Pi ^
^/ = 1
n2
n
n 2
Thus, Var (X) = Pj X/
i = l

or. Var(X) = E(X^)-ff(X)}^


17.22
APPL lED MATHEMATICS-XII

ILLUSTRATIVE EXAMPLES
»AAMI’I.L )
A discret random variable X has thefoUozving probability distribution-
X: 1 2 3 4 5 6 7

P(X): c 2c 2c 3c 2c^ 7c^+c


Find the value of c. Also, find the mean of the distribution.
SOLUTION Since X is a random variable taking values 1,2,.. .,7. Therefore,
P(X = 1) + P(X = 2) +...+ P(X=7)=l
c + 2c + 2c + 3c + + 2c^ + 7c^ + c = 1
10c^ + 9c-l=0

w
(c + 1) (10c-l)=0
10c-1 =0
1
[v P(X=l)=c>0 c + l?^0I

o
c = —
10

e
re
Now, X=Yx^P(X = Xi)
/= 1

Frl
F
7

or
ou
( = 1
kfs
l^c + 2x 2c + 3x 2c + 4x 3c+ 5 X
c^ + 6x2c^+7x(7c^ + c)
66 366
X = 66c^ + 30c = + 3 = = 3.66 1
oo

100 100
10
Y

liXA.MPLE:
The probability distribution of a random variable X is
B

viven below
X:: 0 1 2 3 4
P(X): 0.1 0.25
re

0.3 0.2 0.15


fX
oYu

Find (i) Var fX) (ii) Var —


l2
ad

SOLUTION (i) Computation of mean and variance


d

2
in

-Y;I
Pl = P(X = Xj)
Re

Pi ^i Pi
0 0.1 0 0
F

1 0.25 0.25 0.25


2 0.3 0.6 1.2
3 0.2 0.6 1.8
4 0.15 0.6 2.40

I Pi Xi = 2.05 .T pj =5.65

Thus, we have I pj Xj =2.05 and I pj =5.65


Var (X) = I Pi Xj^ -(I Pi Xjf =5.65 -(2.05)“ = 1.4475
(ii) We know that Var {aX + b)=a^ Var (X).
Var
rx'i 1 1
- =-Var(X) =-xl.4475 = 0.361875
V 2 y 4 4
17.23
DISCRETE RANDOM VARIABLE AND ITS PROBABILITY DISTRIBUTION

IXAMI’tl A random variable X has the following probability distribution:


X;! : 2 -1 0 1 2 3

0.1 k 0.2 Ik 0.3 k


Pi-
(i) Find the value ofk. (ii) Calculate the mean of X. (iii) Calcidate the variance ofX.
SOLUTION (i) The sum of the probabilities in a frequency distribution is always unity.
0.1 + it + 0.2 + 21c + 0.3 + cf = 1 => 0.6 + 4/c = 1 => 41c = 0.4 => /c = 0.1
Calculation of mean and variance
2
Pi Pi ^i Pi T,.

w
-2 0.1 -0.2 0.4

-1 0.1 -0.1 0.1

Flo
0 0.2 0 0

1 0.2 0.2 0.2

ee
0.3 0.6 1.2

Fr
2

3 0.1 0.3 0.9

for Sp,x.^=2.8
ur
Z Pj Xj = 0.8
2
Thus, we have I p, Xj = 0.8 and Z pj Xj = 2.8
k s
Yo
oo

Mean = Zpj Xj - 0.8


Variance = Zp/ x^'^ - (Z p; Xj)'^ = 2.8 - (0.8)^ = 2.8 - 0.64 = 2.16
eB

and.

n\ \MI’U- 4 The probability distribution of a random zmriable X is given as under:


r

kx^ for X =1,2, 3


ou
ad

P{X = x)=-2kxforx-4,5,6 ; lohere k is a constant,


Y

0 otherwise
Re
nd

Find (i) P (X > 4) (ii) £(X) (iii) £(3X^)


Fi

SOLUTION The probability distribution of X is as given below:


1 2 3 4 5 6

10k 12k
P{X=Xi): k 4fc 9k 8k
Tlie sum of the probabilities in a probability distribution is 1.
P (X = 1) + P (X = 2) + P (X = 3) + P (X = 4) + P (X = 5) + P (X = 6) = 1
k + 4:k + 9k + 8k + 10k+12k=l => 441:=1 => k=^
44
30 15
(i) P(X>4)=P(X = 4) + P(X=5) + P(X = 6)=8k + 10k + 12k = 30k=^
(ii) £(X) = 1 P(X = 1) + 2 P (X = 2) + 3 P(X = 3) + 4 P(X = 4) + 5P(X = 5) + 6 P(X - 6)
190 _ 95
£(X) =k + 8k + 27k+ 32k + 50k + 72k = 190k = 44 " 22
17.24
APPUED MATHEMATICS-XH

(iii) The probability distribution of 3X^ is as given below:


3 12 27 48 75 108
Pi: 4fc 9k 8k 10k 12k
E (3X^) = 3 X /c +12 X 4^r + 27 X 9^: + 48 X 8/: + 75 X lOfc + 108X12A:

£ {3X2) ^ ^ 243k + 384k + 750k + 1296fc = 2724fc = ^


44 II

The probability distribution of the discrete random variables X and Y are given below:
X; 0 1 2 3 y: 0 1 2 3

P(X): I
2 1 1 1 3 2 1
5 5 5 5 PiY): ^
o 10 5 10

Prove fhat:E(Y^) =2£(X).

w
SOLUTION We have,
£(X)=0xl5 +lxf+2xi+3xi=-
2 1 1 7

2£(X)= —
14

5
5 5 5 5

F lo
ee
Fr
The probability distribution of Y^ is as given below.
y,'2: 0 1 4 9 for
1 3 2 1
Pi-
r
5 10 5 10
You

£(y2) =0x1 + 1 xA + 4x 2 + -If


s
ook

5 10 5 10 10 " 5
Clearly, E(Y^) = 2 £(X).
eB

I'.x AMI’LL h
The random variable X can take only the values 0,1, l.Given that P(X = 0) =P(X = 1) =v
and that E(X^) =E{X),find the value ofp.
our
ad

SOLUTION Clearly,
P(X = 0) + P(X=l) + P(X=2)=l=>p + p+P(X = 2)=l=>P(X=2)=l-2p.
So, the probability distribution ofX is as given below:
dY
Re

xi: 0 1 2

Pi- P P l-2p
Fin

£(X) = 0xp + lxp + 2(l-2p)=2-3p


and. E(x2) =0^xp + l^xp + 22(1 -2p)=4-7p
It is given that
£(x2)=£(X) ^ 4-7p-2-3p=> p = l
The probability distribution of a discrete random variable X is given as under:
X: 1 2 4 2A 3/4 5/4
1 1 3 1 1 1
P(X):
2 5 25 10 25 25
Calculate:

(i) Thevalueof A, if E{X)= 2.94 (ii) Variance of X.


SOLUTION (i) Using the definition of £(X), we have
E{X)=ZxjP{X=Xi)
17.25
DISCRETE RANDOM VARIABLE AND ITS PROBABILITY DISTRIBUTlOk

£(X)=lx- + 2x
1
-1 + 4 X — + 2A X -?- + 3A X 25
h 5A x —
25
, 5 25 10
2 12 A 3A A 69 , 13A
+ —= — +
2'^5'^25'^J^ 25 5 50 25
It is given that £(X) = 2.94
A =2.94 => 69 + 26A=147 => 26A=78 => A = 3.
50 25

(ii) The variance V(X) of random variable X is given by


Var(X)=£(x2)-(£(X))2
Now,

w
E{X^)=i:xi^P{X=Xi)
1
E(x^) =l^xi + 2^xi + 4^x^ + (2A)^ X ^ + (3A)^ x ^ +(5A)^
5 25 10 25
X —

25

o
1 4 48 2A^ 161 44
+ — X 9
„ 953 [vA = 3]
E(X^) =-2 + -5 + — + —5

e
25 25 50 25 50 25 50

re
Var(X)=£(x2)-(£(X))2 =
953

rFl
- (2.94)^ = 19.06 - 8.6436 = 10.4164.

F
50
rXAMPLES Let X be a discrete random variable luhose probability distribution is defined as follows:
k{x + l) for X =1,2, 3,4

r
ou
p(X = -y) = -j 2kx for Y = 5, 6,7 fo
ks
0 otherwise

, where k is a constant.
oo

Find: (i) k (ii) £(X) (iii) Standard deviation ofX.


Y

(i) The probability distribution of X is as given below.


eB

soLirriON

4 5 6 7
.Y,-: 1 2 3
5k m 12^: 14ic
3k 4fc
r

Pi- 2k
ou

The sum of the probabilities in a probability distribution is 1.


Y
ad

P(X =1) + P(X = 2) + P(X = 3) + P(X = 4) + P(X =5) + P(X = 6) + P{X =7) =1
2k+3k + 4k+5k + 10k + 12k+14k=l => 50k=l=>k= —
d

£(x) =1 P(X =1) + 2P (X = 2) + 3 P(X = 3) + 4 P(X = 4) +5 P(X =5) + 6 P(X = 6) +7 P(X =7)
Re
in

=> x^l2k + 7 x 14k


£(X) = lx2k + 2x3fc+3x4k + 4x5k + 5xl0k + 6 260
F

= 5.2
=> £(X) = 2k + 6k + 12k + 20fc + 50k + 72k + 98k = 260k = 50

(iii) We have,
Var (X) = £ (X^) - (£(X))^ and a (X) = JVar W
2
So, let us first compute £ (X ).
£(x2)=i2 P(X=l) + 22p(X = 2)+32p(X = 3) + 4^P(X=4)+52p(X=5)
+ 6^P{X=6)+7^P(X=7)
E(X^) =1^ X 2k + 2^ X 3fc + 3^ X 4k + 4^ X 5k +5^ x 10k + 6^ x 12k +7^ x 14k
1498
= 29.96
E(X^) = 2fc + 12k + 36k + 80k + 250k + 432k + 686k = 1498k 50

Var (X) = £(X^) - {E{X)f = 29.96 - (5.2)^ = 2.92


Hence, o (X) = V292 = 1.708
17.26
APPLIED MATHEMATICS-XI!

EXAMPLE 9 A random variable X has the following probabilitij distributio7i:


Xi‘. -2 -1 0 1 2 3
Pi- 0.1 k 0.2 2k 0.3 k

(i) Fmd the value ofk. (ii) Calculate the mean ofX. (iii) Calculate the variance ofX.
SOLUTION (i) Since sum of the probabilities in a frequency distribution is always unity.
0.1 + k+ 0.2 + 2k + 0.3 +k = 1 => 0.6 + 4A: = 1 = 0.4 => k = 0.1

Calculation of mean and variajice


Xi! 2
Pi Pi^i Pi^i

ow
-2 0.1 -0.2 0.4
-1 0.1 -0.1 0.1
0 0.2 0 0
1 0.2 0.2

e
0.2

re
2 0.3

rFl 0.6 1.2

F
(13 0.9

2 Pi Xj = 0.8 S Pi = 2.8

r
ou
Thus, we have S Pi X; = 0.8 and S p,- x.^ = 2.8 fo
ks
Mean = 0.8
oo

and. Variance x.^ -(S p,- Xjf = 2.8 -(0.8)^ = 2.8 - 0.64 = 2.16
Y
eB

EXAMPLE 10 The random variable X can fake values 0,1,2,3. Given that P (X = 0) = P(X = 1) =p and
P {X = 7) = P{X = 3) such that E (X^) = 2 E {X),find the value of p.
ur

SOLUTION LetP(X = 2)=P(X=3)=X.


It is given that X is a random variable taking values 0,1,2, and 3.
ad
Yo

P(X = 0) + P(X=1) + P(X=2) + P(X = 3)=1


P+P+X+X=l
d
Re
in

2 ^
Tlius, the probability distribution of X is
F

Xi : 0 1 2 3
1 1
Pi- P P
r” 2-p
It is given that
£(x2)=2(£(X))
^ S pd=^'LPi^i
=> p X 0^ + p X 1^ +
~~p]x2^+U-py3^ =2|px0+pxl+ — -p / x2+ 12
4 7 \2 J I \ 2
fl
p X 3

=> P + 2-4p+^-9p
2
= 2fpV + l-2p + -2 -3p
fl3 '] f5 ^
=> y-12p j =2 \A--4p j => 13-24p=10-16p => 8p = 3=>p=-
\ ^ ^8
DISCRETE RANDOM VARIABLE AND ITS PROBABILITY DISTRIBUTION 17.27

EXAMPLE 11 Fmd the mean and variance of the number of heads in the tioo tosses of a coin.
SOLUTION Let X denote the number of heads in the two tosses of a coin. Then, X can take
values 0,1 or 2 such that
1
P (X = 0) = (Probability of getting no head) = P (TT) = —

1
P(X=1) = (Probability of getting one head) = P(HT or TH) = -4 2

and. P (X = 2) = (Probability of getting both heads) = P(HH) = i


Thus, the probability distribution of X is as given below:

ow
X: 0 1 2

1 1 1
P(X):
4 2 4

e
Computation of mean and variance

re
Fl 2

F
X;I Pi=P{X = x^) Pi^ Pi Xi
ur
0

r
0 1/4 0

1 1/2 1/2 fo 1/2


ks
2 1/4 1/2 1
Yo

I,piX.^=3/2
oo

I Pi Xj =1
eB

Thus, we have

I Pi Xi= 1 and S Pi X- ^ = |
ur
ad

1
= Mean = ^Pi Xi = 1 and, Var(X) = S p,-x-^-(I p,-x,-)^ = |-1 = ^
Yo

X
1
d

Hence, Mean = 1 and Variance = -


Re

2
in

EXAMPLE 12 Find the mean, variance and standard deviation of the number of heads in a simultaneous
F

toss of three coins.


SOLUTION LetX denote the number of heads in a simultaneous toss of three coins. Then, X can
take values 0,1,2,3.

Now, P(X = 0) = P(TTT) =


1

8
, P(X=1) = P(HTT or TTH or THT) = |8
p (X = 2) = P (HHT or THH or HTH) = -8 and, P (X = 3) = P {HHH) = ]-8
Thus, the probability distribution of X is given by
X: 0 1 2 3

1 3 3 1
P(X): 8 8 8 8
17.28 APPLIED MATHEMATICS-XII

Computation of mean and variance


2
X;!
Pi=P{X = X^) Pi^i
Piffl
1
0 0 0
8

3 3 3
1
8 8 8
3 6 12
2
8 8

w
3 1 3 9
B. 8 1
V 3
^Pi^=- .T Pi x.^ = 3

o
e
Thus, we have

re
3 2
S Pi Xi = - and Z Pi X. = 3

Frl
F
3^^ 3
X - Mean = I pi Xj = and, Var(X) = Zp,x'-(Zp,^.)2 = 3- I 4
ou
V 2

or
^ V3
Standard deviation = fWar{X) - - 2
= 0.87 kfs
3 3
Hence, Mean = -, Variance = — and. Standard deviation = 0.87
2 4
oo

rxAMPLE 13 Two dice are thrown simultaneously. IfX denotes the number of sixes, find the expectation
Y

and variance of X.
eB

SOLUTION Clearly, X can take values 0,1 and 2.


We have.
ur

25
P (X = 0) = (Probability of not getting six on any dice)
oY

36
ad

P (X = 1) = (Probability of getting one six) = —


36
d

P (X = 2) = (Probability of getting two sixes) = —


in

36
Re

Thus, the probability distribution of X is given by


F

X: 0 1 2
25 10 1
P(X):
36 36 36

Computation of mean and variance


2
X-I
Pi=P{X=Xi) Pi^ Pi
25
0 0
36 0
10 10 10
1
36 36 36
1 2 4
2
36 36 36.
12 14
^ Pi
36. 36
17.29
DISCRETE RANDOM VARIABLE AND ITS PROBABILITY DISTRIBUTION

Thus, we have
12 7
^Pi^=T7 — and I Pi X; ^
36 3 ' 18
_ 1=A
E{X) = .Ipi Xi =-3 and, Var (X) = S pj x- ^ -{I.piXjf=^^18 9 18

Hence, E (X) = — and Var(X)=—.


3 18

LXAMI’LL I f Two numbers are selected at random (ivithout replacement) from the first six positive
integers. Let X denote the larger of the two numbers obtained. Find £ (X) and Var (X).
SOLUTION We observe that X can take values 2,3,4,5,6 such that
P (X = 2) = Probability that the larger of two numbers is 2
P (X = 2) = Probability of getting 1 in first selection and 2 in second selection or
getting 2 in first selection and 1 in second selection

w
P(X=2)=i.Uixl=A
^ ^ 6 5 6 5 30 = l
15

F lo
p (X = 3) = Probability that the larger of two numbers is 3
p (X = 3) = Probability of getting a number less than 3 in first selection and 3 in

ee
second selection or getting 3 in first selection and a number less than 3

Fr
in second selection.
. 2 112 4 2
P(X = 3)= ±x- + -x- = — = —
^ ^ 6 5 6 5 30 15 for 3 1 1 3 6 1
ur
p (X = 4) = (Probability that the larger of two numbers is 4) = -D x -^- 4- — X — =
6 5 30
= —
15
5 1 1 5 10 _ 1
s
4 1 1 4 8
ook

P(X=5)=-x- + -x- = — = — and, P (X = 6) =


Yo

' ^ 6 5 6 5 30 15 30 3

Thus, the probability distribution of X is


eB

X: 2 3 4 5 6
r

1 2 1 4 1
ad
ou

P(X): 15 15 5 15 3

1 70 14
Y

■■■
— X 6 =—
15 3
Re
nd

; -
LXAMPLL i In a meeting 70% of the membersfavour a certain proposal, 30% being opposed. A member
is selected at random and lelX=0 if he opposed, andX = l if he is infavour. Find E (X) and Var (X).
Fi

SOLUTION It is given that 30


P (X = 0) = Probability that a member opposed a certain proposal = 100
70
p (X = 1) = Probability that a member favoured a certain proposal = 100
The probability distribution of X is
X; 0 1

30 70
P(X): 100 100

30 70 30 70
£(X) = X 0 + xl = — and, E (X^) = 100 X 0^ + 100 10
100 100 10

~
Var(X)=£(x2)-[£(X)f 10 49 ^ 21
100 " 100
17.30
APPLIED MATHEMATICS-XII

7 21
Hence, £ (X) = — and Var (X) =
100
lA Wli’LE 1(1
A class has 15 students zvhose ages are 14,17,15,14, 21,17,19, 20,16,18, 20, 17,
16,19 and 20 gears. One student is selected in such
a manner that each has the same chance of being
choosen and the age X of the selected student is recorded. What is the probability distribution of random
variable X? Find mean, variance and standard deviation ofX.
SOLUTION We observe thatX takes values 14,15,16,17,18,19, 20 and 21 such that

P(X=14)=^,P(X=15)=^,P(X
15 15
= 16)=—,£(X
15
= 17)= —15

w
P(X=18)=T,p(x=i9)=A,p(x
15
= 20)=Tp(x
15
= 21) = 1

15

So, the probability distribution of X is as given below;

o
X: 14 15 16 17 18 19 20 21

e
2 1 2 3 1 2

re
3 1
P(X):
15 15

Frl
15 15 15 15 15 15

F
Computation of mean and variance
-V;I
Pi=P{X=Xi) 2

or
Pi Xi
ou
Pi X;
2 28 392
14
kfs
15 15 15
1 15
oo

15 225
15 15 15
Y
B

2 32 512
16
15 15 15
re

17
3 51 867
oYu

15 15 15
ad

1 18
18 324
15 15 15
d

2 38
19 722
in
Re

15 15 15
F

3 60 1200
20
15 15 15
1 21
21 441
15 15 15
263 2 4683
Z Pi Xi = ZpiX. =
15 15
We have.
263 2 4683
I Pi Xi = and I Pi X- =
15 15
263
Mean = Z pi Xj = 15
= 1753

Variance = Z Pi x. ^ -(Z p. Xj)^ = 4683


15 I 15
^ 70245 - 69169 1076
225 225
17.31
DISCRETE RANDOM VARIABLE AND ITS PROBABli !-"v disi RIBUTION

32.80
Standard Deviation = VVariance = = 2.186
~ 15 15

LXAMl-LL i: Find the probability distribution of the number of successes in two tosses of a die, where a
success is defined as 'getting a number greater than 4'. Also, find the mean and variance of the
distribution.
SOLUTION Let X denote the number of successes in two tosses of a die. Then, X can take values
0, 1, 2.
;th
Let Sj = Getting a success in i toss and, ,F = Getting a failure in toss.Tlren,
2 1
P(Sl) = Probability of getting a number greater than 4 in first toss = - 3
1
Also,
^(%)= 3

ow
P(Fl) = P(f2) = I
Now, P (X = 0) = Probability of getting no success in two tosses of a die

e
P(X = 0) = P(Fi nP2)

re
=> P(X = 0) = P(Fi)xP(P2)

P(X = 0) = I
Fl
2 _ 4
[By multiplication theorem]

F
X —
3 ” 9
ur
p (X = 1) = Probability of getting one success in two tosses of a die.

r
P{X = 1) = P{(SinF2) u (FinS2)l fo
P(X^l) = P(Si nF2) + P(FinS2) = P(Si)P(F2) + PCFi)P(S2)=|x| + |xl = ^
4
ks
Yo
oo

and. p (X = 2) = Probability of getting two successes in two tosses of a die


= P(S^nS2) = P(Si)P(S2)
eB

Thus, the probability distribution of X is given by


ur

X: 0 1 2
4 4 1
ad
Yo

F(X): 9 9 9

Computation of mean and variance


d
Re
in

2
X;I Pi=P{X = Xi) Pi ^i Pi
F

4
0 0
0
9
4 4 4
1
9 9 9
1 2 4
2
9 9 9
_ 6 V 2 8

Thus, we have
6 2 , _ 2 8
S Pi Xj - = — and S Pi X. = -
9 3 ‘ 9
2^^ 4
X = Mean = I p/ x,- = — and, Var (X)
3 = ZpiX.^-ilpiXif - ^ 9
17.32
APPLIED MATHEMATICS-XU

Hence, Mean = — and, Variance = —.


3 9

t.XAMi’LL 18 Find the probabiliti/ distribution of the number of sixes in three tosses of a die. Find also
the mean and variance of the distribution.
SOLUTION Letx denote the number of sixes in three tosses of a die. Then, X can take values
0,1,2,3. Let Si denote the event of getting a six in toss, 2 = 1, 2, 3. Then,
P {^) = 76 and P {^) = -,
6
/ = 1, 2, 3.
Now, P(X = 0) = P(Si n S2 n S3)
P(X = 0) = P(^)P(^)P(%) [v S^, $2, S3 are independent events]

w
5 5 5 125
P(X = 0) = — X — X —

6 6 6 216
\

P(X=1) = P|^(S,n^nS3)u(Sin^nS3) uCSjoS^nSg)

o
e
P(X=1) = P(Sin^nS3)ji-P{S^n%n%) + P{^n^nS3)

re
P(X=1) = P (Sj) P (%) P (S3) + P{Si)P (S2) P(^) + P (S^) P (5) P (S3)

Frl
F
=>
P(X = 1) = ix|x.^ + ixlx^ + ^x^xl
666666666 216

or
ou
P(X = 2) = P|^(SinS2nS3)n(Sin^nS3) n(SinS2nS3) kfs
P(X = 2) = P(Sin%n5)^P(^n^nS3) + P(Sin%nS3)
P (X = 2) = P (Si) P (S2) P (S3) + P (^) P {S2) P (S3) + P (Si) P (5) P (S3)
oo

P(X = 2)=lxlx^ + ^xlxi + ix:^xi = il


Y

666666666
B

216
and.
P(X=3) = P(SinS2nS3) = P (Sj) P (S2) P (S3) = lxlxi=J-
re

6 6 6 216
Thus, the probability distribution of X is given by
oYu
ad

X: 0 1 2 3

125 75 15 1
d

P(X):
216 216 216 216
in
Re

Computation of mean and variance


F

-V;
I P(X = a:,-)=p^ Pi^ Pi
2

125
0 0 0
216

1 75 75 75
216 216 216
2
15 30 60
216 216 216
1 3
3 9
216 216
216

S Pi Xi = 108 ^ 1 144
216 ~ 2 216
17.33
DISCRETE RANDOM VARIABLE AND ITS PROBABILITY DISTRIBUTION
108 1 j ^ 2 144
Thus, we have S p, Xj = - and S Pi -X
216 2 ' 216
1
Mean = X = ^ Pi Xi = -
144 nf 90 _ 5
and, Var(X) = .rpiX.^-(YpjXif = 216 2) 216 12

Hence, Mean = —
— and Variance = —
2 12

EXAMrt.n i' A die is tossed twice. A "success" is "gettmgau odd number" on a random toss. Fmd the
variance of the number of successes.
SOLUTION Let X denote the number of successes in two tosses of a die. Then, X can take values
0,1,2.

ow
.●th
toss. Then,
Let Si and denote the success and faUure respectively i in/

3 1
.●th
P(£j) = Probability of getting an odd number in/ toss
6 2
1
;lh 1 --
p (]]) = Probability of not getting an odd number in /
toss
and,

e
2J 2

re
Now,

Fl
P (X = 0) = Probability of getting no success in two tosses of a die

F
P(X=0) = P(finf2)
P(X=0) = P(fi)P(f^) [By Multiplication Theorem
ur
V P (Pi) = P (P2) = ^
r
1 1 1
P(X = 0) =
2 2 4 fo
p (X =1) = Probability of getting one success in two tosses of a die
ks
Yo
P(X-l) = p((Si nP2) u (fj 062)
oo

11 11 1
P(X=1) = P(Si np2) + P(Pi ^ %) = P(^) ^ ^
= —X — + — x — = —
eB

2 2 2 2 2

and. P (X = 2) = Probability of getting two successes in two tosses of a die


P(X=2) ^ P (Si n S2) = P(Si) P(S2) =\^\
ur

4
ad

Thus, the probability distribution of X i.e. the number of successes in two tosses of a die, is given
Yo

by
X: 0 1 2
d

1 1
Re

1
in

P(X)t 2 4
4
F

Computation of variance
2

X;l Pi=P(X^Xi) Pi Xi Pi^i


1 0
0 0
4
1 1 1
1
2 2 2
1 1
1
2
A 2

Hpi Xi =1

1 J V 2 3
Thus, we have Z p, xi = 1 and Hpi X.

Var(X) = ^PiX^-{Y.piXif=- 1=12


17.34
APPLIED MATHEMATICS-XII

f-.XAMi’LE 2t) Tivo cards are drawn successively zvith replacement fr. om
a ivell-shuffled deck of 52 cards.
Find the mean and standard deviation of the number of aces.
"lit" are

P(Aj) = Probability of getting an ace indraw = -*"1 = ^ 1

52 13
12
and. P(A) = i-P(A) = 1- — i = 1, 2.
13 13'
Let X denote the number of aces in two draws. Then, X can take values 0,1, 2.
Now, P {X = 0) = Probability of getting no ace in two draws
P (X = 0) = P{Al n = P (4^) p (A2) = 1312 12 _ 144
13 169
P (X = 1) = Probability of getting an ace in either of the two draws

P(X=1) = P (A-^ 0^2) (^i ^ A2)

w
F lo
P(X = 1) = P(/li n
A2) + P {Ai 0/^2)
1 12 12 1
P(X = 1) = P Ml) P (^) + P (^4^) P (^2) = 24
=>
— X — + X — =

13 13 13 13 169
and.
P(^ = '2.) = Probability of getting ace in each draw

e
Fre
1 1 1
P(X = 2) = PMi n >12) = PMi)PM2) = 13 ^ 13
=>

for 169
Thus,
the probability distribution of X is given by
X: 0 1 2
r
144 24 1
You

P(X):
s

169
ook

169 169
144 24 1 26
S Pi Xj = 0 X + 1 X
eB

+ 2x
169 169 169 169
144 24
and, Xp,x.^ = Ox
169
+ lx + 4 X
1 28
169 169 169
our
ad

Hence, X = Mean = Z p, a^- = = A


169 13
f 2
Var(X) = lpiX.^-(I.p.Xif = ■169
and, 28 24
dY
Re

13 169

=J
^ 2V6
Fin

S.D. =
169 13

Hence, Mean = — and S.D. =


13 13

rXAMEM f;21 Two cards are drawn successively zvithout replacement from a
well-shuffled deck of
52 cards. Compute the variance of the number of aces.
setting an ace in /th draw, where / = 1, 2. Further let X
denote the number of aces in two draws. Then, X can take values 0,1, 2.
Now,
P (X 0) - Probability of getting no ace in two successive draws
48 47 564
P(X = 0) = PMi n ,42) = PMi) P M2M1) = — X

P(X=1) = Probability of getting an ace in one of the two draws

P(X = 1) = P {A-inA2) u {A-i n A2)


DISCRETE RANDOM VARIABLE /● "^D ITS PROBABILITY DISTRIBUTION 17.35

P(X=1) = FiAinX2) + P(A^ n A2)


48 48 4 96
P(X=1) = P{A^)P{2X/A-i) + PiX)P{A2/X) = J-x
52 51
+
52
X —

51 663

p (X = 2) = Probability of getting an ace in each draw


3
P (X = 2) = P (^1 n A2) = P (A-y) P {A2/A-1) = ;^
52
X -51 663

Thus, the probability distribution of X is given by


X: 0 1 2
564 06 3
P(X): 663 663 663
564 96 102
I pj Xj = 0 X 663
+ 1 X
663 663 663

w
564 96 3 108
and, 2 Pi X. ^ 663
X 0 +
663
X 1 +
663
X 4 =
663

108 X 663 - (102)^ _ 61200 ^ 400

F lo
108 (102
Hence, Var(X) = 2 pj x. ^ - (I pj .v,)^ = 663 663 (663)^ 663 X 663 2873

ee
.V . ■ ^ From a lot oflO items containing 3 defectives, a sample of4 items is drawn at rmuioin. Let

Fr
the random variable X denote the number of defective items in the sample. If the items in the sample ore
draiun one by one ivithout replacement, find
(i) The probability distribution ofX (ii) Mean of X (iii) Variance of X
for
ur
SOLUTION (i) Clearly, X can assume values 0,1, 2, 3 such that
^^4 _ 1 P(X = 1) =
1
s
P(X = 0) =
“ 6' 10
ook

10 2
c4 C4
Yo

^C3 X _ _l
!%i!^=A,andP(X =
eB

P (X = 2) = 3) = 10
IOq 10 C4 30

So, the probability distribution of X is as given below.


r
ad
ou

X: 0 1 2 3
1 1 3 1
P(X):
Y

6 2 10 30

Computation of mean and variance


Re
nd

2
P{X^Xi)=Pi Pi Pi
Fi

1
0 0
0 6
1 1 1
1 2 2 2
3 3 6
2 5 5
10
1 1 3
3 30 10 10
12
^Pi^i=7F
10 Ip,..v."=2
12
Thus, we have I p,- Xj = —
10
-5 and Ip, x^' =2
6
X = Mean - I p; .V/ =
5
17.36 ;e[ ■THEMATICS-''

6^2 14
and. Var(X) = (LpiXif = 2 - 15^ 25 25

Hence, Mean = - and Variance = —.


5 25
3 1
tiX.Mvil'l.i; 23 A coin weighted so that P {H) = — and P (T) = — is tossed three times. Let X be the random
4 4
variable which denotes the longer string of heads which occurs. Find the probabiliti/ distribution, mean
and variance ofX.
SOLUTION The random variable X is defined on the sample space S given by
S = {TTT,HTT,THT,TTH,THH,HTH,HHT,HHH}

ow
Note that the string of heads means the sequence of consecutive heads.
Since X denotes the longest string of heads. Therefore,
X(TTT) = 0,X{THT)^1,X{HTT) ^1,X{TTH) =\,X{HTH) =1,
X (HHT) = 2, X {THH) = 2 and X (HHH) = 3.

e
1 1 1
Now, P (X = 0) = P (TTT) = P (T) P (T) P (T) = -
4 "" 4 ^ 4 ~ 64

re
P (X = 1) = P (THT u HTT u TTH u HTH)

rFl
F
P (X = 1) = P (THT) + P (HTT) + P (TTH) + P (HTH)
P (X =1) = P (T) P (H) P (T) + P (H) P (T) P (T) + P (T) P (T) P (H) + P{H) P (T) P (H)
3 ll 3 1 h —3 = 18

or
ou
P(X=1) = 3 —- X—X— + — X
U 4 4J 4 4 4 64
ksf
P{X = 2) = P (THH u HHT)
p (X = 2) = P (THH) + P (HHT)
oo

p (X = 2) = P (T) P (H) P (H) + P (H) P (H) P (T) = 2 l4


f 1 x -4 x -3^ 18
Y

4 64
B

■ 3 3 27
and. P (X = 3) = P (HHH) = P (H) P (H) P (H) = —
- X — X — = —
re

4- 4 4 64

So, the probability distribution is as follows:


oYu

0
ad

●V;I : 1 2 3
1 18 18 27
Pi-
d

64 64 64 64
in
Re

Calculation of mean and variance


F

2
-Y;;
Pi Pi ^i Pi
1
0 0 0
64

18 18 18
1
64 64 64
18 36 72
2
64 64 64
27 81 243
3
64 64 64
135 333
2 Pi =
64 64
135 333
Thus, we have I pj .Xj = 64 and I pj Ay ^ 64
DISCRETE RANDOM VARIABLE AND ITS PROBABILITY DISTRIBUTION 17.37

135
Mean = I pj Xj = = 2.1
64
2 2 333
and, Variance = Ipj x. -(Mean) 64
(2.1)^ = 5.2-4.41 = 0.79
Hence, Mean = 21 and Variance = 0.79.

> i :s Afair coin is tossed until a head or five tails occur. If X denotes the number of tosses of the
coin, find the mean ofX.
SOLUTION The sample space related to the gp-^n random experiment is given by
S = {H,TH,TTH,TTTH.TrTTH,TTrTr'

ow
Clearly, X assumes values 1,2,3,4, 5 such that

P(X = 1) = P(H) = ^
p (X = 2) = P (TH) = P (T) P (H) = ^

e
re
P(X = 3) = P(TTH) = P(T)P(T)P(H) = ixixi ^

rFl
F
1 1 1 1 1
P(X = 4) - PiTTTH) = P(T)P(T)P(T)P(H) = — X—X—X
2 2 2

2
-
16

or
ou
and, P(X=5)=P(TrTTH u TTTTT)
P (X =5) = P (TTTTH) + P {TTTTT)
ksf
^5
ri
(T) P (T) P (T) = f ^
1
P(X=5)=P{T)P (T) P (T) P (T) P (H) + P (T) P (T) P +
oo

V 2 2) 16
Y

So, the probability distribution of X is given by


B

1 2 3 4 5
.t,-:
re

1 1 1 1 1
Pi¬ 4 8 16 le
2
oYu
ad

1 1 1 1.1c
X 2 + — X 3 + _x4 + — x5
31 - 1.9.
= — , „
Mean = Z Pj at,- = — x 1 + 4 16 16 16
d

: - ● I I There are 5 cards numbered 1 to 5, one number on one card. Two cards are drawn at
in
Re

random without replacement. Let X denote the sum of the numbers on tivo cards drawn. Find the mean
and variance.
F

SOLUTION Clearly, X can take values from 3 to 9 such that


p (X = 3) = Probability of getting 3 as the sum
P(X = 3) = P KGetting 1 in first draw and 2 in second draw) or (Getting 2 in first
draw and 1 in second draw)!
I l l 1
P(X = 3) = 1■ -X —h-.x
4 5

4
= —
10

p (X = 4) = P {(Getting 1 in first draw and 3 in second draw) or (Getting 3 in first


draw and 1 in second draw)}
1 1 1 1 1
P(X=4)=-x- + -x- =
^ 5 4 5 4 10
17.38 APPLIED MATHEMATICS-Xll

nrv 11111111 1
P (X =5) = —X h—X h —X I--X — —
5 4 5 4 5 4 5 4 5

D /V 11111111
P (X =6) =-x — + -X — + -X — + -X — =
1

5 4 5 4 5 4 5 4 5

Dfv -7\ 11111111


P (X =7) = —X —f — X —h-x —h — X — =
1

5 4 5 4 5 4 5 4 5

P(X = 8) = 5ixl4 + ixl


5 4
= -1
10
and P(X = 9) =-5- 1
— X —h-x —
4
1
5
1
4
- —
1
10

w
Thus, the probability distribution of X is as given below:
X: 3 4 5 6 7 8 9
1 1 1 1 1 1 1
P(X):
10 10 5 5 5 10 10

o
e
Computation of mean and variance

re
2
a:,-I
Pi=P{X = Xi) Pi Xi Pi

Frl
F
1 3 9
3
10 10 10
ou
1 4 16

r
4
10

so
10 10
1 5
kf 25
5
5 5 5
oo
1 6 36
6
5 5
Y

5
B

1 7 49
7
5 5 5
re

1 64
oY

10
u

10 10
1 9 81
ad

9
10 10 10
d

60 390
^ Pi ■■f/ = TT = 6 = 39
in

Total Ipi = 1 10 10
Re

Thus, we have, I pj Xj = 6 and Sp,- Xj = 39


F

X = Mean = LpjXj = 6 and, Var(X) = S p,-.y-^-(I p, .v/)^ = 39 - 6^ - 3


' a Person is paid ^5 if he gets all heads or all tails when three coins are tossed, and
he will pay X 3 if either one or tzuo heads show. What can he expect to win on the average per game?
SOLUTION Let X be the amount received by the person. Then, X can take values 5 and - 3 such
that

P (X - 5) - Probability of getting all heads or all tails when three coins are tossed
= i = i
~ 8 “ 4
6 3
P (X = - 3} = Probability of getting one or two heads ^ -
8 4

Expected amount to win, on the average, per game = X = Z p. .y,- = 5 x - + - 3 x — = -l


4 4

Thus, the person will, on the average, lose ? 1 per toss of the coins.
DISCRETE RANDOM VARIABLE AND ITS PROBABILITY DISTRIBUTION 17.39

EX.MVHM.r.2'' Lei X denote the number of vowels in word selected at random from this sentence. Find the
expected value and standard deznation of the random variable X. (Consider Xasa word ivith one letter).
SOLUTION There are 12 words in the following sentence.
"Find the expected value and standard deviation of the random variable X".
Clearly, X can take values 0,1, 2, 3,4,5 such that
P (X -0) = P (Selecting a word containing no vowel) = P (Selecting X) 12

ow
p (X = 1) = P (Selecting a word containing one vowel)
5
P(X=1)= P (Selecting a word from the words'The','Find','and', of'nae') 12

P (X = 2) = P (Selecting a word containing two vowels)

e
2

re
= P (Selecting a word from the words 'standard', 'random') = 12

Flr
p (X = 3) = P (Selecting a word containing three vowels)

F
2
= P (Selecting a word from the word 'expected', 'value')
ou =—

P (X = 4) = P (Selecting a word containing four vowels)

sr
= P (Selecting the word 'variable') = ^
ko
P (X =5) = P (Selecting a word containing five vowels)
of
= P (Selecting the word 'deviation') = ^
o
Y
reB

So, the probability distribution of X is as given below:


uY

X: 0 1 2 3 4 5

1 5 2 2 1 1
P(X): 12
12 12 12 12 12
ad
do

£(X)=0x —+ lx 12^ 2 + 3x
+ 2x — - —2 + 4x
, —+5X 1 1
0+5+4+6+4+5_^
12 12 12 12 12 12
in

E (X^) =0^x — + l^x — +2^x—+3^x — + 4^x — +5^x^


Re

12 12 12 12 12 12
F

0 + 5 + 8 +18 + 16 + 25 72
E (a ) — ^ ^ “ t)
12 12

Var (X) = £ (X“) - [£ (X)]^ = 6 - 2“ = 2.


=>
Standard deviation of X = .^Var (X) =
Hence, £ (X) = 2 and Standard deviation = V2.
EXAMPLE 28 In a game a man wins a rupee for a six and looses a rupee for any other number when a fair
die is thrown. The man decided to throzuadie thrice but to quit as and when he gets a six. Find the expected
value of the amount he loins/looses.
SOLUTION The man may get six in the first throw and then he quits the game. He may get a
number other than six in the first throw and in the second throw he may get six and quits the
game. In the first two throws he gets a number other than six and in third throw he may get a six.
He may not get six in any one of three throws.
Let X be the amount he wins/looses. Then, X can take values 1, 0, -1, - 3 such that
17.40 APPLIED MATHEMATICS-Xn

P (X = 1) = P {Getting six in first throw) = -


6

P (X = 0) = P (Getting an other number in first throw and six in second throw)


= -x

6 "" 6
- = —

36
P{X = -1) = P (Getting numbers other than 6 in first two throws and a six in third
throw)
5 5 1 25
— X — X — = —
6 6 6 216

P(X = - 3) = P (Getting a number other than six in first three throw) = - x ^ ^


6 6 6 216
Thus, the probability distribution of X is as given below:
X: 1 0 -1 -3

w
1 5 25 125
P(X):

F lo
6 36 216 216
1 25
£ (X) = 1 X - + Ox — + (-l)x + (- 3)x 125 36 + 0 - 25 - 375 364 _ 91
6 36 216 216 216 216 " 54

ee
RX AMPLE 29 In a group o/30 scientists working on an experitnent, 20 never commit error in their ivork

Fr
and are reporting results elaborately. Two scientists are selected at random from the group. Find the
probability distribution of the number of selected scientists ivho never commit error in the work and
for
reporting. Also, find the mean of the distribution. What values are described in the question?
ur
SOLUTION Let X denote the number of selected scientists who never commit error in the
work and reporting. Clearly, X can take values 0,1,2.
s

Now,
ook
Yo

P (X = 0) - Probability that two scientists selected commit error either in the work
eB

or in reporting
_ 3
30
our

29
C2
ad

P(X = 1) - Probability that one out of two scientists selected does not commit error
in the work and reporting while the other is not so
Y

20
Cixl^Cj _40
Re
nd

30
C2 87
Fi

P (X = 2) — Probability that two scientists selected do not commit error in the work
and reporting
20
C2 _ 38
30 87
C2
The probability distribution of X is as given below:
X: 0 1 2
3 40 38
P(X):
29 87 87

LetX be the mean of the distribution, llien,


3 40 38 116
X =0x hlx + 2x — = 1.33
29 87 87 87

This means that on an average out of two selected scientists one scientists will not commit error
in the work and reporting.
17.41
;SCRET£nANDO:.. ■ .

-■●'.MI’LEIm; A biased die is such that P(4) = — and other scores being equalhj likely. The die is tossed
10

twice. IfX is the 'number offours seens'Jind the variance of the random variable X.
1
SOLUTION It is given that P{4) = and other scores being equally likely. So, let
10

P{1) = P{2) =P(3)= P(5) = P(6) =p.


P(l)+P(2) + P(3)+P(4) + P{5) + P(6)=l +

Thus,P{l)=P(2)=P(3)=P(5)=P(6)-^ and
When die is tossed twice, there may be no four or one of the two throws may result in a four or
both the throws produce 4. So, X can take values 0,1 and 2.
Now,
P(X = 0) = Probability of not getting a four in both the throws

w
1 L¥i J_l-A i-=il
lOA ToJ~io^io"ioo
P(X =1) = Probability of getting a four in one of the two throws
-
1
X 1
1 ●\
+
/
1
1 A
X

F lo
1 1
X
9
h
9 1 18

e
10 10 J 10 10 10 10 10 100

Fre
10 y V

1 1
P(X = 2) = Probability of getting a four in both the throws = ^
for 10 x —
10 = 100

Thus, the probability distribution of X is as given below.


r
X: 0 1 2
You

81 18 1
s

P(X):
ook

100 100 100


eB

Calculation of variance
2
Pi Pi ^i Pi
our
ad

81
0 0
0
100
dY

18 18 18
Re

1
100 100 100
Fin

1 2 4
2
100 100 100

20 ■c- 2 22
I Pi Xi = I Pi X.
100 100

Thus, we have
20 1 , _ 2 22 _11
I Pi Xi =
100
-5 and I Pi Xj 100 "50
11 if 9
Var {X)=lpiXi^-{I.PiXif = 50 V5J
— = 0.18
50

Find the probability distribution of the maximum of two scores obtained when a die is
throzvn twice. Determine also the mean of the distribution.
SOLUTION The sample space associated to the random experiment of throwing a die twice
consists of 36 elementary events and is given by
17.42 APPLIED MATHEMATICS-XII

S = {(1,1), (1,2),...,(1,6), (2,1), (2, 2), (2, 6),..., (6,1),(6, 2),..., (6, 6)}
Let X denote the maximum of the two numbers obtained in two throws of a die. Then,X can take
values 1,2,3,4, 5 and 6.

P(X=1)=F(1,1)= —
36

P(X = 2) = P ((1, 2) (2,1), (2, 2)) = A


36

P{X = 3) =P ((1, 3) (3,1), (2, 3), (3, 2), (3, 3)) --


36

ow
P(X = 4) = P ((1, 4) (4,1), (2, 4), (4, 2), (3, 4), (4, 3), (4, 4)) = —
36

P(X = 5) = P ((1,5) (5,1), (2,5), (5, 2), (3,5), (5, 3), (4,5), (5, 4), (5,5)) =

e
36

re
rFl
11
P(X = 6) = P ((1, 6) (2, 6), (3, 6), (4, 6), (5, 6), (6, 6), (6,1), (6, 2), (6, 3), (6, 4), (6,5))

F
36

So, the probability distribution of X is as given below:

or
X: 1
ou
2 3 4 5 6

P(X): i A A _Z_ A 11
ksf
36 36 36 36 36 36
oo

1 3 73
X=lx + 2x —+3x — + 4x — + ^5x —
9 + 6x11 = —
36 36 36 36 ' 36 36 “ 36
Y
B

LXAMI’LE 32 There are 5 cards numbered 1 to 5. one number on one card. Two cards are drawn at
re

random without replacement. Let X denote the sum of the numbers on two cards drawn. Find the mean
and variance ofX.
oYu
ad

SOLUTION The sum X of the numbers on two cards drawn without replacement can take
values 3,4,5,6, 7,8,9.
d

The sum can be three if one of the cards drawn bears number 1 and other bears number 2.
in
Re

P(X = 3) = P((l, 2), (2,1)) = 5lx -4 + -X - = 20


1 11 2
5 4
F

Similarly,
11 11 2
P(X = 4) = P((1, 3), (3,1)) = -X h —X — =
5 4 5 4 20

P(X=5)=P((1,4), (4,1), (2, 3),(3,2))=lxl + lxl + lxl + lxl = 4


54545454 20

P (X = 6) = P ((1, 5), (5,1), (2, 4), (4, 2)) = A


P (X ^7) = P ((2, 5), (5, 2), (3, 4), (4, 3)) ^ A
P(X = 8)=P((3,5),(5, 3)) = A
P(X=9)=P((4,5),(5,4))= —
20
DISCRETE RANDOM VARIABLE AND ITS PROBABILITY DISTRIBUTION 17.43

Thus, the probability distribution of X is as given below.


X: 3 4 5 6 7 8 9

P(X): 2 ^ A ^ A A A.
20 20 20 20 20 20 20

Calculation of variance
2
X;
I Pi Pi Xi Pi

w
2 6 18
3
20 20 20
2 8 32
4
20 20 20

o
e
4 20 100
5

re
20 20 20
4 24 144

Frl
6

F
20 20 20
4 28 196
7
20
ou 20 20

sor
2 16 128
8
20 20 kf 20
2 18 162
9
oo
20 20
120 2 780
Y

I Pi Xi = = 6 ZpiX. = = 39
20 20
B

Thus, we have
S Pi Xi =6 and I pi x^ = 39
re
oY
u

Mean = I Pi xi = 6 and. Variance = I pj Xj^ -(£ p, Xj)^ = 39 - 36 = 3


ad

EXERCISE 17.2
d

1. Find the mean and standard deviation of each of the following probability distributions:
in

(ii) Xi 1 3 4 5
(i) .v^: 2 3 4
Re

0.4 0.1 0.2 0.3


Pi : 0.2 0.5 0.3 Pi
F

(iii) -5 -4 1 2 (iv) Xj 1 2 3 4

0.4 0.3 0.2 0.1


Pi-A/A 1/8 1/2 1/8 Pi

(vi) Xi 0 1 3 5
(v) xp 1 2 3 4
0.2 0.5 0.2 0.1
Pi : 0.3 0.1 0.1 0.3 Pi

0 1 2 (viii) Xi -3 -1 0 1 3
(vii) X.f -2 -1
0.1 0.05 0.45 0.20 0.25 0.05
Pi: 0.1 0.2 0.4 0.2 Pi

0 1 2 3 4 5
(ix)
1 5 2 1 1 J_
Pi-
6 18 9 6 9 18
A discrete random variable X has the probability distribution given below:
X: 05 1 15 2

P(X): k 2k^ k
(i) Find the value of k. (ii) Determine the mean of the distribution.
17.44 ■>PL1ED MATl iEMATICS-Xli

3. Find the mean variance and standard deviation of the following probability distribution
Xi‘. a b

Pi- P
where p + q = 1.
4. Find the mean and variance of the number of tails in three tosses of a coin.
5. Two cards are drawn simultaneously from a pack of 52 cards. Compute the mean and
standard deviation of the number of kings.
!). Find the mean, variance and standard deviation of the number of tails in three tosses of a
coin.

7. Two bad eggs are accidently mixed up with ten good ones. Three eggs are drawn at random
with replacement from this lot. Compute the mean for the number of bad eggs drawn.
8. A pair of fair dice is thrown. Let X be the random variable which denotes the minimum of
the two numbers which appear. Find the probability distribution, mean and variance of X.

w
9. A fair coin is tossed four times. Let X denote the number of heads occurring. Find the
probability distribution, mean and variance of X.

F lo
10. A fair die is tossed. Let X denote twice the number appearing. Find the probability
distribution, mean and variance of X.
11. A fair die is tossed. Let X denote 1 or 3 according as an odd or an even number appears.

ee
Find the probability distribution, mean and variance of X.

Fr
12. A fair coin is tossed four times. Let X denote the longest string of heads occurring. Find the
probability distribution, mean and variance of X.
for
13. Two cards are selected at random from a box which contains five cards numbered 1,1,2,2,
and 3. Let X denote the sum and Y the maximum of the two numbers drawn. Find the
ur
probability distribution, mean and variance of X and Y.
s

14. A die is tossed twice. A 'success' is getting an odd number on a toss. Find the variance of the
ook
Yo

number of successes.
15. A box contains 13 bulbs, out of which 5 are defective. 3 bulbs are randomly drawn, one by
eB

one without replacement, from the box. Find the probability distribution of the number of
defective bulbs.
16. In roulette. Fig. 17.2, the wheel has 13 numbers 0,1, 2,...., 12 marked on equally spaced
our
ad

slots. A player sets ? 10 on a given number. He receives ? 100 from the organiser of the game
if the ball comes to rest in this slot; otherwise he gets nothing. If X denotes the player's net
gain/loss, find £ (X).
Y
Re
nd
Fi

i 7. Three cards are drawn at random (without replacement) from a well shuffled pack of 52
cards. Find the probability distribution of number of red cards. Hence find the mean of the
distribution.
I An um contains 5 red 2 black balls. Two balls are randomly drawn, without replacement.
Let X represent the number of black balls drawn. What are the possible values of X? Is X a
random variable? If yes, find the mean and variance of X.
Two numbers are selected at random (without replacement) from positive integers 2,3,4,5,
6 and 7. LetX denote the larger of the two numbers obtained. Find the mean and variance of
the probability distribution of X.
DISCRETE RANDOM VARIABLE AND ITS PROBABILITY DISTRIBUTION 17.45

In a game, a man wins 15 for getting a number greater than 4 and loses ? 1 otherwise, when
a fair die is thrown. The man decided to throw a die thrice but to quit as and when he gets a
number greater than 4. Find the expected value of the amount he wins/lose.
ANSWERS

1
(i) 3.1, 0.7 (ii) 3,1.7 (hi)-1,2.9 (iv) 1,1.5

(v) Mean= 2, S.D = 1 (vi) Mean= 1.6, S.D = 1.497


(vii) Mean= 0, S.D = 1.095 (vui) Mean= -0.2, S.D = 1.249
35 35
(ix) Mean =— , S.D. =

ow
(vui) Mean =— S.D. = 18
18 18 18
1 23
2. (i) k = (ii) X =
3 18

Mean = ap + bq Var = pq{a-b) o


\a-b\^

e
34 400

re
4. Mean = 1.5, Var = 3/4 5. Mean = , Var =

rFl 221 2873

F
6. Mean =15, Var = —, S.D. = 0.87
3
7.1
4' 2

or
ou
8. 1 2 3 4 5 6 Mean = 2.5
Xi‘.
11/36 9/36 7/36 5/36 3/36 1/36 Var = 2.1
ksf
Pi-

3 4 Mean = 2
oo

0 1 2
9. Xj:
1/16 4/16 6/16 4/16 1/16 Var = l
Pi-
Y
B

10. .X;J : 2 4 6 8 10 12 Mean = 7


re

1/6 1/6 1/6 1/6 1/6 1/6 Var = 11.7


Pi-
oYu
ad

1 3 Mean = 2
11. x^ ■
Pi- 1/2 1/2 Var = l
d
in
Re

12. 0 1 2 3 4 Mean = 1.7


Xi ■
1/16 7/16 5/16 2/16 1/16 Var = 0.9
Pi-
F

2 3 4 5 Mean = 3.6
13. Xi:
0.1 0.4 0.3 0.2 Var = 0.84
Pi ■

1 2 3 Mean = 2.3
.V/ :
0.1 0.5 0.4 Var = 0.41
Pi-

. 1/2

X : 0 1 2 3

28 70 40 5
P(X): 143 143
143 143

30
E(X) = -
13
17.46 APPLIED MATHEMATICS-XII

X: 0 1 2 3

703 741 117 11 997


P(X): ; Mean =
1700 1700 850 1700

^,Var(X) =
50
18. 0,1, 2,; Yes; Mean =
147
17 14
19. Mean = - , Var (X) = - 20.
3 9 9

MULTIPLE CHOICE QUESTIONS (MCQs)


Mark the correct nltermtive in each of the following:
1. If a random variable X has the following probability distribution:

w
X: 0 1 2 3 4 5 6 8

a 3a 5a 7a 9a Ua I3a 15rt I7a

Flo
then the value of a is
7
(b)l81 (C) ^

ee
i^) —
81 81
(d)l
81

Fr
2.
A random variable X has the following probability distribution:
X: 1 2 3 4 5 for
ur
6 7 8

PJ2Cfi 0.15 0.23 0.12 0.10 0.20 0.08 0.07 0.05


k s
For the events £ = {X: X is a prime niunber}, f = pC: X < 4}, the probability P{EuF) is
Yo
oo

(a) 050 (b) 0.77


(c) 0.35 (d) 0.87
eB

3. Arandom variable X takes the values 0,1, 2, 3 and its mean is 1.3. If P(X = 3) = 2 P(X-l)
and P (X = 2) = 0.3, then P {X = 0) is
r

(a) 0.1 (b) 0.2 (c) 0.3 (d) 0.4


ou
ad

4.
A random variable has the following probability distribution:
Y

X=:t:,- : 0 1 2 3 4 5 6 7

P(X = Xi) : 0 2p 2p 3p 2p~ 7 p^ 2p


Re
nd

The value of p is
Fi

(a) 1/10 (b) -1 (c) -1/10 (d) 1/5


5.
If X is a random-variable with probability distribution as given below:
X = -Xf : 0 1 2 3

P(X = x,-): k 3k 3k k

The value of k and its variance are


(a) \ 13,22117 (b) 1/8,23/27 (c) 1/8,24/27 (d) 1/8,3/4

6. The probability distribution of a discrete random variable X is given below:


X: 2 3 4 5

P(X): 5/k 7/k 9/k n/k


The value of E(X) is
(a) 8 (b) 16 (c) 32 (d) 48
DISCRETE RANDOM VARIABLE AND ITS PROBABILITY DISTRIBUTION 17.47

. For the following probability distribution:


X: 4 -3 2 -1 0

P(X): 0.1 0.2 0.3 0.2 0.2

The value of E(X) is


(a) 0 (b) -1 (c) -2 (d) -1.8
For the following probability distribution:
X: 1 2 3 4

F(X): 1/10 1/5 3/10 2/5

ow
The value of E(X^) is
(a) 3 (b) 5 (c) 7 (d) 10
Let X be a discrete random variable. Then the variance of X is

e
(a) £(X^) (b) £(x2)+(£(X))^ (c) E(X^)-(£(X))2 (d) ^E{X^)-{E{X)f

re
rFl
Let X be a discrete random variable. The probability distribution of X is given below:

F
X: 30 10 -10

r
3 1
ou
1
P(X):
5 10 2 fo
ks
Then E(X) is equal to
(a) 6 (b) 4 (c) 3 (d)-5
oo
Y

NSWERS
B
re

(d) (b) (d) . (a) (d) ●● (c) (d) ■ (d)


(c) (b)
ou
Y
ad

HLi. IN SLANKS QUESTIONS (FBQs)


d

If X is a random variable with the following probability distribution:


in
Re

X: -V n
F

P(X): Pi P2 Pn

Then, Mean (X) =


In Q. No. 1, Var (X) =
A discrete random variable X has the following probability distribution:
X: 1 2 3 4 5 6 7

P(X): c 2c 2c 3c c^ 2c^ 7c^+c


Then,P(X<2)=
: If a discrete random variable X has the following probability distribution:
1 3
X: 1 2
2 2

P(X): c c2 2c2 c

The, c = ....
17.48 APPLIED MATHEMATICS-XII

If a random variable X has the following probability distribution:


X: 0 1 2 3
1 3 2 1
?(X):
5 10 5 10

Then, E(X^) =
ANSWERS
;/ n n2
3 1 14
I. Z Pi 10
i=] i=l 3 5

QOlSTIONS ' /S/ ●

ow
Answer each of the following questions in one zvord or one sentence or as per exact requirement of the
question:
Write the values of 'a' for which the following distribution of probabilities becomes a

probability distribution:

e
re
Fl
X=x-:
f
-2 -1 0 1
1 -^7 1+la 1 -2fl 1 + <7

F
P(X = .x^-):
4 4 4 4
ur
For what value of k the following distribution is a probability distribution?

r
X=Xi : 0 1 2 fo 3
ks
P{X = Xi)-. Ik^ 3k^-5k^ 2k - 3k^ 3k
Yo

IfX denotes the number on the upper face of a cubical die when it is thrown, find the mean
oo

ofX.
eB

If the probability distribution of a random variable X is given by


X = Xji 1 2 3 4
ur

P{X = Xi): 2k 4k 3k k
ad

Write the value of k.


Yo

Find the mean of the following probability distribution:


d

X = -X:
I
1 2 3
Re
in

1 1 5
P{X=x^):
4
F

If the probability distribution of a random variable X is as given below:


X=Xi: 1 2 3 4

P{X=Xj)-. c 2c 3c 4c

Write tlie value of P (X < 2).


A random variable has the following probability distribution:
X=.r,-: 1 2 3 4

P{X = Xi): k 2k 3k 4k
Write the value of P (X > 3).
A NS ,: zRS
1 1 1 19 7
--<a< 35 k = 0.1 0.3
2 2 2 8 10
18

BINOMIAL DISTRIBUTION

18.1 . joucTior:

In the previous chapter, we have studied about discrete random variable and its probability
distribution. In this chapter, we shall study a particular type of probability distribution, known

w
as binomial distribution. It was discovered by James Bernouli in the year 1.700 and was first
published posthumously in 1713. This distribution has been used to describe a wide variety of

F lo
processes in business and social sciences as well as other areas. In binomial distribution a
random experiment is performed repeatedly under identical conditions and the distribution
determines the probability of occurrence of one set of dichotomous alternatives i.e. success or

ee
failure.

Fr
18.2 BERNOULLI TRIALS

In our day to day life we come across many experiments which are dichotomous in nature i.e.
for
ur
they produce one of the two possible outcomes in a trial. For example, a tossed coin shows a
'head' or 'tail', a manufactured item can be 'defective' or 'non-defective' etc. In such type of
experiments, it is customary to call one of the outcomes a 'success' and the other not success , or
s
ook

'failure'. If this type of experiments are repeated under identical conditions, then the outcome of
Yo

any trial is independent of the outcome of any other trial. In each of such trials, the probability of
eB

success or failure remains constant. Such independent trials which have only two outcomes
usually referred as 'success' or 'failure' are called Bernoulli trials as defined below.
BERMOULLI TRIALS Trials of a random experiment are called Bernotdii trials, if they satisfy the
r
ou
ad

follozving conditions:
(i) They are finite in number.
Y

(ii) They are independent of each other.


(iii) Each trial has exactly tivo outcomes: success or failure.
Re
nd

(iv) The probability of success or failure remains same in each trial.


Fi

For example, if four balls are drawn successively with replacement from a bag containing 7 red
and 6 black balls, then the probability of getting a black ball in each trial remains same equal to
4:.
13
So, trials are Bernoulli trials. If the balls are drawn successively without replacement, then
the probability of getting a black ball in first trial is —
13
and in second trial it is —, if the first ball
12

drawn is black or , if the first ball drawn is red and so on. Clearly, the probability of getting a
12
black ball is not same for all trials. So, the trials are not Bernoulli trials.

18.3 BINOMIAL DISTRIBUTION


Consider a random experiment and an event A associated with it. If the experiment results in the
event A, let us call it a "success", denoted by S. If on the other hand, the event A does not occur
let us say that the experiment has resulted in a "failure", denoted by F. Let P(S) = p and P(F) = q.
Then, p + q = l. Suppose the experiment is performed 4 times under identical condition and we
18.2 APPLIED MATHEMATICS-XH

want to find the probability of 3 successes. Three successes in 4 trials can occur in mutually
exclusive ways as given below
SSS f, S FSS, F SSS, SS FS
By addition theorem of probability, we have
Probability of 3 successes in 4 trials = P (SSS F) + P (S FSS) + P (F SSS) + P (SS FS)
Since 4 trials are independent. Therefore, by multiplication theorem for independent events,
we have

w
P(SSSf) = P(S)P(S)P(S)P(F) =p^q =
Similarly, we have
P(SFSS) = p^q*-^,P(FSSS) = p^ q^'^ and P(SSFS)=p^ q*~^

e
Thus,

ro
re
Probability of 3 successes in 4 trials = ^ +p^ q^ ^ +p^ q^~^ +p^ q^~^
= *C3p3,4-3

F
Fl
Proceeding on the same lines we can easily show that the probabilities of 0,1,2 and 4 successes
are given by
'^Cq P^ q^
u
p^ q^ \ ^€2 p^ q^ ^ and ^€4 p^ q^ ^ respectively.

sr
ko
Now, if X denotes the number of successes in 4 trials, then X can take values 0,1,2,3 and 4 such
o
that of
P(X = 0) = ^CoP%^-“,P(X=l) = *C,p^q*-\p(X = 2)=*C2P^q^-^
P(X = 3) = %p^q^-^ and P(X = 4) = %p*q*-*
o
Y
erB

In other words, we have


P(X=r) = =0,1,2,3,4.
uY

This result can be generalised to the case where the experiment is repeated n times under
identical conditions. The probability of r successes in n independent trials in a specific order, say
SSSFSFFSS... SF is given by
ad
do

r - successes,(n-r) failures
in

P (SSSFSFFSS...SF) = P(S) P(S) P(S) P(F) ...P(S)P(F) [By mulHpUcatton theorem]


r successes,(n-r) failures
Re
F

= pppq...p q = f cp "

But, r successes in n trials can occur in ”Cr mutually exclusive ways and the probability of each
such way is p^cf . So by addition theorem of probability, the probability of r successes in n trials
n

in any order is given by cr/r^


Let X denote the random variable which associates every outcome to the number of successes i m

it. Then, X assumes values 0,1,2,..., n such that


-r
P(X=r) = , r — 0,1, 2,..., w.
The probability distribution of the random variable X is therefore given by

X: 0 1 2 r n

Cip' 4"-'
n
P(X): CiP^q"-^
« n n
Crp’’ q
n-r n-n
18.3
BINOMIAL DISTRlBliTlor^'

From the probability distribution of the random variable X, we observe that the probabilities of
the random variable taking values 0,1,2,/i are given by the terms in the binomial expansion
of (q + pf. That is why we say that the probability distribution of the random variable X is the
binomial distribution or, that X is a Binomial random variable.
DEFirs>'iON(BINO'Vl'i TRI; -:ot A random variable X zvlikh takes values 0,1,1,n is said to
follow binomial distribution if its probability distribution function is given by
p(X=r) = ”Cr p’^ ^ = 0,1,2....,n,wherep,q>Osuchthatp + q = l.
The two constants n and p in the distribution are known as the parameters of the distribution.
The notation X B (n, p) is generally used to denote that the random variable X follows binomial
distribution with parameters n and p.

w
We have.
i-l
C-i pV
; - H
I - 0 , n
P(X = 0) + P(X=l) + ... + P(X=Ji) = "Cq p^ if + +... + Cn P” ^
= (q + pf = 1 = 1

Flo
Thus, the assignment of probabilities to the random variable X is permissible.

e
Ifn trials constitute an experiment and the experiment is repeated N times, then thefrequendes of

re
0,2, 2,..., n successes are given by

F
N-P(X=0), N-P(X=1), N-P(X=2) N-P(X=n).
ur
r
ILLUSTRATIVE EXAMPLES

THE pr
fo
ks
'S OF'
Yo

A die is thrown 6 times. If "getting an odd number" is a "success”, what is the probability of
oo

(i) 5 successes (ii) at least 5 successes (Hi) at most 5 successes (iv) at least one success (v) no success.
eB

SOLUTION Let p denote the probability of getting an odd number in a single throw of the die.
Then,
ur

u = -16 = 12 and q = 1-p = 1 - 1-1


ad

^ 2 ” 2
Yo

Let X denote the number of successes in 6 trials. Then, X is a binomial variate with parameter
jj =6 and p =1/2.
d
Re
in

The probability of r successes is given by


6-r
P(X=r) = (1/2) (1/2/, where r = 0,1,2,..., 6
F

-(0
or, P(X=r) = (1/2)1 where r = 0,1,2,... 6

/1 3
(i) Probability of 5 successes = P(X=5) = ^C5 - [Using (i)l
32

(ii) Probability of at least 5 successes =P(X>5)


= P(X=5) + P(X = 6)

= T + ‘-e T = (6 + 1)/64 ^
64
[Using (i)l
\2J \2

(iii) Probability of at most 5 successes = P (X < 5) = \61 - P (X > 5) = 1 - P (X = 6)


1 1 63
= 1 6(—
^6 [Using (i)]
2j 64 64
18.4
:s-x

(iv) Probability of at least one success = P (X > 1) = 1 - P (X = 0)


= 1 -
0 = 1 - J- = ^ [Using (i)]
1) 64 64

1
(v) Probability of no success = P (X = 0) = 0 [Using (i)]
U 64

A coin is tossed 5 times. What is the probabilih/ of getting at least 3 heads.


SOLUTION Let p denote the probability of getting head in a single toss of a coin. Then,
P = - and so, n = ^
2 ' 2
Let X denote the number of heads in 5 tosses of a coin. Then, X is a binomial variate with
parameters » = 5 and p = 1/2 such that
P{X=r) = ^C,- (1/2)^ ' (1/2)^^ = ^C,- (1/2)^, where r = 0,1,2 5

w
Now,
Probability of at least 3 heads = P (X > 3)

F lo
= P(X = 3) + P(X = 4)+P(X=5)
/I
' fif ^,v fil'Y^

ee
[Using (i)]
=
3

Fr
■^C3^=C,.=C3}(1J
1 1
= (10 + 5 + 1) X =
for 32 2
ur
CXA.V1PLE3
A pair of dice is throum 7 times. Ifgetting a total of 7 is considered a success, what is the
probability of
s
(i) no success? (ii) 6 successes? (Hi) at least 6 successes? (iv) at most 6 successes?
ok
Yo

SOLUTION Let p denote the probability of getting a total of 7 in a single tlirow of a pair of dice
o

Then, ^
eB

6 1
The sum can be 7 in any one of tlie ways:
I

P = —
36 6
(1,6), (6,1), (2,5), (5, 2), (3, 4), (4, 3)
r
ad
ou

1 5
^ = 1-p = 1-
6 6
Y

Let X denote the number of successes in 7 throws of a pair of dice. Tlie X is a binomial variate
with parameters n = 7 and p = 1 /6 such that
Re
nd

lY/sf
P(X=r) =
Fi

u , r = 0,1,2,..., 7
-

\6y ...(i)

(i) ' 1 nO / 5 \7 - 0 rsf


Probability of no success = P(X = 0) = 0 [Using (i)]
.6/ l6

(ii) Probability of 6 successes = P (X = 6) = (5^


. 6 y I 6,
= 35(1/6)^ [Using (i)]
(iii) Probability of at least 6 successes = P {X > 6)
= P(X = 6) + P(X=7)
n {5 V ^7,- flf fsT
'C6 . 6 / 1,6 UJ lu [Using (i)]

fl X’’ S') if 1 r 35 1 if
= 7 + 1—
(>) \ 6 6 / I 6 y V 6 6 6/
18.5
BINOMIAL DISTRIBUTION

(iv) Probability of at most 6 successes =P (X < 6)


= 1 - P (X > 6)
= 1 - P(X=7)
0
7r if
^
= 1 - [Using (i)]
V D V 6 6)

An urn contains 5 zuhite, 7 red and 8 black balls. Iffour balls are drawn one by one zoith
replacement, zohat is the probability that
(i) all are white? (ii) only 3 are ivhite? (Hi) none iszohite? (iv) at least three arc zvhite?

SOLUTION Let p denote the probability of drawing a white ball from an urn containing 5 white.
7 red and 8 black balls. Then,
5C 1 5 1 c 3
p = — = — .So, q = l^p
4 4
20 c 1 20 4

w
Let X denote the number of white balls in 4 draws with replacement Then, X is a binomial
1

F lo
such that
variate with parameters n = 4 and P - ^
4-r
nY r3
; r=0,l,2, 3,4
P{X=r) = Probability that r balls are white = - / V 4j

e
Fre
Now,
4-4
if
4 nff3
for [Using (i)]
(i) Probability that all are white = P (X = 4) = ^4 |^- y- 4j
r
3
/ ^ if psf “ 1
You

= 3 [Using (i)l
Probability that only 3 are white = P (X = 3) = C3 -
oks

(ii) .4y U
eBo

/ 1 '>0 / 3 -\4 3f [Using (i)l


(iii) Probability that none is white = P(X = 0) = Cg 4j 4; 4j
our
ad

(iv) Probability that at least three are white = P (X > 3) = P (X = 3) + P (X - 4) 0

=
4^C ●! nf
— + - - [Using (i)l
H4 ) V 4
dY
Re

'if
Fin

= 13
.4,

A man takes a step forward zvith probability 0.4 and backivards with probability 0.6. find
the probability that at the end of eleven steps he is just one step azimjfrom the starting point.
SOLUTION Let p denote the probability that the man takes a step forward. Then, p = 0.4
g = 1 -p = 1 - 0.4 = 0.6
Let X denote the number of steps taken in the forward direction. Since the steps are independent
of each other. Tlierefore, X is a binomial variate with parameters n = 11 and p = 0.4 such that
11-r .
; r = 0,1,2,..,,11
...(i)
P(X=r) = (0.4)'' (0.6)
Since the man is one step away from the initial point. Therefore, he is either one step forward or
one step backward from the initial point at the end of eleven steps. If he is one step forward, then
he must have taken six steps forward and five steps backward and if he is one step backward,
then he must have taken five steps forward and six steps backward. Thus, either X = 6 or X - 5.
18.6
APPLIED MATHEMATICS-XII

Required probability = P[(X=5) or (X = 6)]


= P(X=5) + P(X = 6)
11
C5 {(Uf (0.6) 11-5 , nCe (0.4)« (0.6) 11-6
+
[Using (i)]
11
C5 (0.4)5 ^ [■-■ '^5 =
= 462(0.4)5(0.6)5 = 462(0.24)5
An uni contains 25 balls of which 10 balls beam mark ‘A'and the remaining 15 balls bear a
mark B . A ball is drawn at random from the urn, its mark is noted down and it is replaced. If 6 balls are
drawn in this wa\/,find the probabilih/ that
(i) all-will bear'A'mark
(ii) not more that 2 will bear 'B' mark
(iii) the number of balls with 'A' mark and 'B' mark will be equal

w
(iv) at least one ball will bear 'B' mark
SOLUTION Let p denote the probability of drawing a ball which bears mark '/I' Then

Flo
10 2
p = —
25 5

ee
Let X denote the number of balls which bear mark in 6 draws. Then, X is a binomial variate with

Fr
parameters n = 6 and p = -.
5

Also ‘? = l-p=l-r^=1
for
ur
5 5

Now, P (X =r) = Probability of getting r balls bearing mark 'A'


ks
_ 6r _ .
Yo

" u 5J
. r = 0,1,2,...,6 ●●●(i)
oo

/ \

(i) Probability that all balls bear'A' = P(X = 6)


B

= 2'i^r3
f
6-6
re

' Is y \ 5J
= (2/5) [Using (i)]
(ii) Not more than 2 balls will bear '6' mark means that there can be either no ball or one ball or
ou
ad

two balls of 'B' mark. This implies that there can be either 6 or 5 or 4 balls of 'A' mark.
Y

Required probability = P (X > 4)


= P(X=4) + P(X=5) + P(X=6)
nd
Re

f3f + Cc fz
2f 3^ ' f 3')^ 2']^
tjJ l5j
— — = 7
Fi

\5 5(^5 5] ^l5 15 15

Let p denote the probability that a ball drawn bears mark 'S'. Then, p 15 _ 3
25 ~ 5
Let y denote the number of balls which bear mark 'B' in 6 draws. Then, Y is a binomial variate
with parameters n = 6 and p = 3/5 such that
3Y /2
P(Y=r) = ^Cr , r=0,l,2,...,6 ●●●(ii)
5j l5
Required probability = P (Y < 2)
= P(Y=0) + P(Y=1) + P(Y=2)
6^ f3)^f2)^ + ^C / 3 ^ 2 Y
^0 \5J J 15r 1
. 5 yV5 /
+ 5c2 .5] - [Using (ii)]
= 7(2/5)^
18.7
BINOMIAL DISTRIBUTION

f f ^ = 20
(2 ^ 3
[Using (i)]
(iii) Required probability = P (X = 3) = -
V 0 / V S) . 5 y ^ 5;

(iv) Probability that at least one ball will bear 'B' mark = P (Y > 1)
= 1 - P (y = 0)
/ 3 \0 / 2
= 1 - ^C 0 [Using (ii)]
\5) \b
/
2x6
5;
In a hurdles race, a plai/er has to cross 10 hurdles. The probability that he will clear each

w
EXAMPLE 7

hurdle is -. What is the probability that he will knock doiun fewer than 2 hurdles?
6

SOLUTION Let X denote the number of hurdles knocked down by the player. Then, X follows

o
5 1 , 5
binomial distribution with /? = 10, p = 1 - — — and q .

e
6 6 '6

re
lO-r
flf f5

rFl
10
P(X=r) = ;r = 0,1,2,10

F
y V 6 y

Required probability = P (X < 2) = P (X = 0) + P (X =1)

or
ou
10
f5^ 1
+ X - X
6) 6 V6 6y 6 6 2x6^
ksf
EXAMPLES Five dice are thrown simultaneously. If the occurrence of an even number in a single dice is
oo

considered a success, find the probability of at most 3 successes.


Let X denote the number of successes in 5 throws of a die and let p be the probability
Y

SOLUTION
B

of getting an even number in a single throw of a die. Then,


3 1 .
re

V = — = - and q = 1 - p - 1 -
^ 6 2 ' 2 2
oYu

The probability of r successes in five throws of die is given by


ad

^ 1Y f 1 ~ 1Y
P(X=r) = -
= -
, where r = 0,1,2,..., 5
2J H2
d

V zy V

Required probability = P (X < 3) = 1 - P (X > 3)


in
Re

l-lP(X=4) + P(X=5)i
F

= 1 - 5r f’f
1 ^4 \ TZ
= 1 -
5
L] - ^
32 32J ' 32 " 16

J ^[2)

EXAMPLE 9 An unbiased die is thrown again and again until three sixes are obtained. Find the
probability of obtaining 3rd six in the sixth throw of the die.
SOLUTION Let p be the probability of obtaining a 'six' in a single throw of the die. Then,
1 j -1 1 5
P = — and q = i —
6
6 6

Obtaining third six in the sixth throw of the die means that in first five throws there are 2 sixes
and the third six is obtained in sixth throw. Therefore,
Reouired probability = P (Getting 2 sixes in first 5 throws) P (Getting 'six' in sixth throw)
625
23328
18.8
APPLIED MATHEMATICS-XII

i:\AMm iH
A lot of 100 watches is known to havelO defective watches. If 8 watches are selected (one bij
one with replacement) at random, what is the probabilihj that there will be at least one defective zuatch?
SOLUTION LetX denote the number of defective watches
in 8 draws and letp be the probability
of selecting a defective watch in a draw. Then, X follows, binomial distribution with parameters
«=8andp=^^
100
= —^ such that
10

1 V f 9
P(X=r) =^C ,r = 0,l,2,...,8
''uoj llO

Required probability = P(X > 1) = 1 - P(X = 0) = 1 -®Cq [UO


— 9 1 f^

ow
10 10
LXAMPl.i' I :
For 6 trials of an experiment, let X be a binoznial variate which satisfies the relation
9 = 4) = P(X = 2). Find the probability of success.
SOLUTION Let p be the probability of success and q that of failure in a single trial. Tlien,

e
P{X=r)=%fq^~'^ [●.●« = 6 (given)]

re
It is given that

Fl
F
9 P (X = 4) = P (X = 2)
ur
9x^C4 ^4

r
9p2=^2 fo
ks
3p=q
Yo
1
oo

3p=l-p => p=_ [●●● P + q = l]


eB

lypc II PROBLEMS ON FINDING THE PROBABILITY DISTRIBUTION


lAAMPTE 1*
Find the probability distribution of the number of heads luhen three coins are tossed.
ur

SOLUTION Let X denote the number of heads obtained in three tosses of a coin. Then, X can
ad

take values 0,1,2 and 3.


Yo

Let p denote the probability of obtaining a head in a single toss of a coin. Then
1
d

= 1-1 1
P - 2 ^ ^ ~P
Re
in

2 2

Since three trials are independent, therefore X is a binomial variate with parameters =3 and
F

p =l/2such that
3-r
\ fl
(
P(X=r) =
''U / V 1) 2)
, r = 0,1, 2, 3.

Thus, the probability distribution of X is given by


X: 0 1 2 3

P(X) ; 3r i 3C 1 fi n
'C2 l2^ 3p
^3
fi
V 2
^ y l2 l2
or.

X: 0 1 2 3
1 3 3 1
P(X):
8 8 8
18.9
SIN--. 1ST

, A bug confuins 3 red and 4 black balls. One bull is druum mid then replaced in the bag and
the process is repeated. Every time the ball drawn is red we say that the draw has resulted in success. Let X
be the number of successes in 3 drnxos. Assuming that at each drazu each ball is equally likely to be selected,
find the probability distribution ofX.
SOLUTION Let p denote the probability of success in a draw. Then,
3
p = Probability of getting a red ball in a draw = —

q = l-p^q=\-
3^4
7 7

Since the ball drawn in each draw is replaced in the bag,


therefore three trials are independent.

ow
Consequently X, the number of successes, can take values 0,1,2 and 3 and is a binomial variate
with parameters n = 3 and p = 3/7 such that 3-r
(3^' (4 , r = 0,1,2, 3.
p(X=r) = Probability of r successes = 7 / V 7

e
re
Thus, the probability distribution of X is given L y

rFl 2 3

F
X: 0 1

V
4^3 3^ f3f f4V /33^
2 7 ^3
7) [7

r
PfX) .- \ / J V/ 1 7
ou
fo
ks
or.
1 2 3
oo

X: 0

'4']^ 9(4
n2
ur 3f f 3
Y

7 I7
B

P(X): 7)
.7, 7{7
re

LXAMPLF. M Two cards are drawn successively zvith replacement from a zvell shuffled pack of 52 cards,
Find the probability distribution of the number of aces.
ou
Y
ad

SOLUTION Let p denote the probability of getting an ace in a draw. Then,


1 12
p = — = —. So, q = l-p=> q =1-
d

52 13 13 13
in
Re

Let X denote the number of aces in 2 draws. Then, X can take values 0,1 and 2. Smee two cards
drawn successively with replacement. Therefore, X is a binomial variate with parameters
F

are

n = 2 and p =1/13 such that


1 V r 12 \2-r
P (X = r) = ,r = 0,1,2
13 13

12^ _144 P(X = 1) = ^Ci


0
1 V12'i 24
2r n
V I3J ”1^'
P (X = 0) = 13 } V 13 169
U3 )

2/123 2-2 1
and. p (X = 2) = 2C2 I 13J 13/ 169

Thus, the probability distribution of X is given by


0 1 2
X:
24 1
144
P(X): 169 169
169
18.10
APPLIED MATHEMATICS-XII

I:XAM pi .F ; Find the probabiliti/ distribution of the number of doublets in 4 throws of a pair of dice.
SOLUTION Let p denote the probability of getting a doublet in a single throw of a pair of dice
Then, ^
1 5
P = — = — and, q - 1-p
36 6 6 6

Let X denote the number of doublets in 4 throws of a pair of dice. Then, X is a binomial variate

w
with parameters n = 4 and p =1/6 such that
P(X=r) = Probability of getting r doublets
or. P(X=r) = 4^, ,r = 0,1,2, 3,4

e
l6 / V 6y

e
1 5^4
2fsf

or
P(X = 0) = 0 - ,P(X = 1) =
aUJ'

r
U/ V. 6y 6J '■ 6 jl.6.

F
1 ^3 f5
P(X = 2) = 4^
^2 71 , P (X = 3) =
1

^ I 6 J lb 3 U
=

V 6 6/ 6 1,6

oF
ul
and.
n^rs^ 0 rn'
P(X = 4) =
He 6y 16

rs
/ \

Thus, the probability distribution of X is given by

ko
X: 0 1 2
P(X):
(5/6)^ (2/3)(5/6)''’
of
(l/6){5/6)2 (10/3)(l/6)^
3 4

(1/6)^
o
Y
Pppc HI ON FINDING THE NUMBER OF TRIALS WHEN PROBABILITY OF OCCURRENCE OF CERTAIN
B

EVENT IS GIVEN
Y

‘Y of a man hitting a target is 1/4. How many times must he fire so that the
er

probability of Ins hitting the target at least once is greater than 2/3?
u

SOLUTION Suppose the man fires n times and let X denote the number of Hmes he hits the
target. Then,
od
ad

\r
rsY'""
P(X = r) = ,r = 0,1,2,...,;;
U; u
in

It is given that
P(X>1) > -
Re
F

1 - P(X
= 0)>|
l-"Co if 2
14JI4
>
3
/3Y' 2
1 - >
4/ 3

3Y' <
1
u 3

Clearly, | > - 1 /3^^ 1_ (3')^ >-,


1 but 81 1
3'UJ ^3'U 3 4J 256
<
3
3Y' 1
.4j 3 n = 4, 5, 6,...
<

Hence, the man must fire at least 4 times.


BINOMIAL DISTRIBUTION 18.11

EXAMl’Lt How many dice must be thrown so that there is a better thnn even chance of obtaining a
six?

SOLUTION Let n dice be thrown, and let X denote the nunaber of sixes. Then,
a Y rsY
P{X = r) = % V -6 , r = 0,l,2,...,n.
/ V 6/

We have to find the smallest value of n for which P (X > 1) > —

i.e.
l-P(X = 0)>i or, P(X=0)<^.
1
Now, P(X = 0) < i => ^67
<

2
S 1 1 (5'\^ 1 625 1
Clearly, - - <t-, but - < —

6 2 2 6) 2 U 1296 2

^5Y' < . 1
P(X = 0) < ^ ^ ej 2
^ ?j = 4,5,....

Thus, at least 4 dice must be thrown.


EXAMPLE
probability of hitting the target at least once is more than 90%.
SOLUTION

trials. Then,
F low
The probability of a man hittmg a target is 1/2. Hozo many times must he fire so that the

Suppose he fires n times. Let X denote the number of times he hits the target in n
forFe
1Y 1Y ~ ^ n n
, r = 0,1, 2,...,n
11
P (X = r) = Cr - Cr -
H2 / V 2J '^12
90
Now, P (X > 1) >
100
eBoks

90
Your

1 - P (X =0) >
100
ad

90
our

P (X = 0) < 1 -
100

(if 1
Re

n
C0 <
2) 10

n Y <
1

2)
FindY

10

Clearly,
'1Y j_ ri_Y , i but 1Y < —
1
10 '[iJ lO'UJ ^ 10' {2 10

n 1
< ^ n = 4,5, 6,...
2 10

Thus, man must fire at least 4 times.


Ivr. i DETERi ' JGTHEFREGUE

t Assuming that half the population are consumers of chocolate, so that the chance of an
individual being a consumer is 1/2, and assuming that 100 investigators each take ten individuals to see
whether they are consumers, how many investigators would you expect to report that 3 people or less zvere
consumers?
18.12
'flCS-XII

SOLUTION Let X denote the number of consumers in a group of ten individuals. Then, X
follows binomial distribution with n = 10, p = l/2and q = 1/2.
10-r 10
P(X = r) =
10
Q f-1 f- [ij l2
_ lOp f 1 , r = 0,1, 2,..., 10.

P (X < 3) = P (X = 0) + P (X = 1) + P (X = 2) + P (X = 3)
10 10 10
n n 1
P(X<3) = % 0 ^lOc ] +
10
^2 -
yi) 2) ^l2 ^2
1^10 10 10 1 176
P(X<3) = - Cn0 + + '^2 + C3 = (1 +10 + 45 + 120) = - 10
l2 2^^ 2
Number of investigators reporting that there are 3 persons or less are consumers.
= 100P(X<3) =100 X 126 _ 17600
5= 17 approximately.
2IO “ 2^0

w
Six dice are throzvn 729 times. Hou> many times do you expect at least three dice to shou ’a

F lo
five or s?.v.
SOLUTION Let X denote the number of dice showing five or six in a set of six dice. Then, X
follows binomial distribution with n = 6,p = probability of getting 5 or 6 in a single throw of a

ee
die= f- = —^ and● q = —.
2

Fr
6 3 '3
6-r
P(X=r) = (1/3)''(2/3) , r=0,l,2, 3,...,6.
for
Now,
ur
P (X > 3) = 1 - P (X < 3)
P(X>3) = 1-IP(X = 0) + P(X=1) + P(X=2)]
s
ook

P(X>3) = l-[^Co(2/3)^+^Ci(l/3)(2/3)^
Yo

+
'’C2 (1/3)2 (2/3)'*]
eB

2 \6 ' 15 \6 / 3n
P (X > 3) = 1 - 1 + 3 + — 1-f^ = 1 -
64x 31
= 1 - 496 ^ 233
3) 4 V3 / \ 4 729 X 4 729 ~ 729
our
ad

Thus, the frequency that at least three dice show five or six when six dice are thrown 729 times
233
= 729x P(X>3) = 729 X = 233
729
Y
Re
nd

There are 6% defective items in a large bulk of items. Find the probability that a sample of 8
Fi

items will include not more than one defective items.


A coin is tossed 5 times. What is tlie probability of getting at least 3 heads?
A coin is tossed 5 times. What is the probability that tail appears an odd number of times?
A pair of dice is thrown 6 times. If getting a total of 9 is considered a success, what is the
probability of at least 5 successes.
A fair coin is tossed 8 times, find the probability of
(i) exactly 5 heads (ii) at least six heads (iii) at most six heads.
Find the probability of 4 turning up at least once in two tosses of a fair die.
A coin is tossed 5 times. What is the probability that head appears an even number of times?
The probability of a man hitting of target is 1 /4. If he fires 7 times, what is the probability of
his hitting the target at least twice?
Assume that on an
average one telephone number out of 15 called between 2 P.M. and
3 P.M. on week days is busy. What is the probability that if six randomly selected telephone
numbers are called, at least 3 of them will be busy?
18.13
BINOMIAL DISTRIBUTION

10. If getting 5 or 6 in a throw of an unbiased die is a success and the random variable 'X'
denotes the number of successes in six throws of the die, find P (X > 4).
11. Eight coins are thrown simultaneously. Find the chance of obtaining at least six
heads.
12. Five cards are drawn successively with replacement from a well shuffled pack of 52 cards.
What is the probability that
(i) aU the five cards are spades? (ii) only 3 cards are spades? (iii) none is spade?
13. A bag contains 7 red, 5 white and 8 black balls. If four balls are drawn one by one with
replacement, what is the probability that
(i) none is white? (iii) any two are white?
(ii) all are white?
A box contains 100 tickets each bearing one of the numbers from 1 to 100. If 5 tickets are

ow
14.

drawn successively with replacement from the box, find the probability that all the tickets
bear numbers divisible by 10.
15. A bag contains 10 balls each marked with one of the digits 0 to 9. If four balls are drawn
successively with replacement from the bag, what is the probability that none is marked

e
with the digit 0?

Fl
re
16. There are 5 percent defective items in a large bulk of items. What is the probability that a

F
sample of 10 items will include not more than one defective item?
The probability that a bulb produced by a factory will fuse after 150 days of use is 0.05. Find
ur
17.

the probability that out of 5 such bulbs


(ii) not more than one or
(iii) more than one
sf
(i) none
(iv) at least one will fuse after 150 days of use.
k
Yo
18. Suppose that 90% of people are right-handed. What is the probability that at most 6 of a
oo

random sample of 10 people are right handed?


If four balls are drawn one by one with
B

19. A bag contains 7 green, 4 white and 5 red balls.


replacement, what is the probability that one is red?
re

20. A bag contains 2 white, 3 red and 4 blue balls. Two balls are drawn at random from the bag.
If X denotes the number of white balls among the two balls drawn, describe the probability
u
ad
Yo

distribution of X.
21. An um contains 4 white and three red balls. Find the probability distribution of the number
of red balls in three draws, with replacement from the urn.
d
Re

Find the probability distribution of the number of doublets in 4 throws of a pair of dice.
in

22.

Also, find the mean and variance of this distribution.


F

23. Find the probability distribution of the number of sixes in three tosses of a die.
24. A coin is tossed 5 times. If X is the number of heads observed, find the probability
distribution of X.
25. An unbiased die is thrown twice. A success is getting a number greater than 4. Find the
probability distribution of the number of successes.
26. A man wins a rupee for head and loses a rupee for tail when coin is tossed. Suppose that he
tosses once and quits if he wins but tries once more if he loses on the first toss. Find the
probability distribution of the number of rupees the man wins.
27. Five dice are thrown simultaneously. If the occurrence of 3, 4 or 5 in a single die is
considered a success, find the probability of at least 3 successes,
28. The items produced by a company contain 10% defective items. Show that the probability
. 28x9^
of getting 2 defective items in a sample of 8 items is 10 8 ■
18.14
APPLIED MATHEMATICS-Xll

A card is drawn and replaced in an ordinary pack of 52 cards. How many times must a card
■‘Q

be drawn so that (i) there is at least an even chance of


drawing a heart, (ii) the probability of
drawing a heart is greater than 3/4?
30.
The mathematics department has 8 graduate assistants who are assigned to the same office.
Each assistant is just likely to study at home as in the office. How many desks must there be
in the office so that each assistant has a desk at least 90% of the time?
31.
An unbiased coin is tossed 8 times. Find, by using binomial distribution, the probability of

ow
getting at least 6 heads.
32.
Six coins are tossed simultaneously. Find the probability of getting
(i) 3 heads (ii) no heads (ii) at least one head
33. Suppose that a radio tube inserted into a certain type of set has probability 0.2 of

e
functioning more than 500 hours. If we test 4 tubes at random what is the probability that

re
exactly three of these tubes function for more than 500 hours?
34.
The probability that a certain kind of component will survive a given shock test is —. Find

Flr
F
4
the probability that among 5 components tested
ou
(i) exactly 2 will survive (ii) at most 3 will survive
35.
Assume that the probability that a bomb dropped from an aeroplane wilt strike a certain

sr
target is 0.2. If 6 bombs are dropped find the probability that
(i) exactly 2 will strike tlie target.
ko
(ii) at least 2 will strike the target.
of
36. It is known that 60‘/o of mice inoculated with a serum are protected from a certain disease. If
5 mice are inoculated, find the probability that
o
(i) none contract the disease
Y
(ii) more than 3 contract the disease.
reB

37. An experiment succeeds twice as often as it fails. Find the probability that in the next 6 trials
there will be at least 4 successes.
uY

38. In a hospital, there are 20 kidney dialysis machines and that the chance of any one of them
to be out of service during a day is 0.02. Determine the probability that exactly 3 machines
will be out of service on the same day.
ad
do

The probability that a student entering a university


39.
will graduate is 0.4. Find the
probability that out of 3 students of the university:
in

(i) none will graduate, (ii) only one will graduate, (iii) all will graduate.
Re

40.
Ten eggs are drawn successively, with replacement, from a lot containing 10% defective
F

eggs. Find the probability that there is at least one defective egg.
41.
In a 20-question true-false examination suppose a student tosses a fair coin to determine Iiis
^swer to each question. If the coin falls heads, he answer 'true'; if it falls tails, he answers
'false'. Find the probability that he answers at least 12 questions correctly.

= ^ ● Show thatX = 3 is the most


42.
SupposeX has a binomial distribution with n=6 and F
likely outcome.
4.3.
In a multiple choice examination with three possible answers for each of the five questions
out of which only one is correct, what is the probability that a candidate would get four or

more correct answers just by guessing?


44.
A person buys a lottery ticket in 50 lotteries, in each of which his chance of winning a prize
IS
100 . What is the probability that he will win a prize
(i) at least once (ii) exactly once (iii) at least twice?
18.15
BINOMIAL DISTRIBUTION

\5. The probability of a shooter hitting a target is - How many minimum number of times
must he/she fire so that the probability of hitting the target at least once is more than 0.99?
How many times must a man toss a fair coin so that the probability of having at least
one
41'.

head is more than 90%.

How many times must a man toss a fair coin so that the probability of having at least
one

head is more than 80%?

A pair of dice is thrown 4 times. If getting a doublet is considered a success, find the
probability distribution of the number of successes.
From a lot of 30 bulbs which include 6 defectives, a sample of 4 bulbs is drawn at random

ow
4^'

with replacement. Find the probability distribution of the number of defective bulbs.
Find the probability that in 10 throws of a fair die a score which is a multiple of 3 will be
obtained in at least 8 of the throws.
A die is thrown 5 times. Find the probability that an odd number will come up exactly three

e
re
times.

rFl
The probability of a man hitting a target is 0.25. He shoots 7 times. What is the probability of

F
his hitting at least twice?
1
A factory produces bulbs. The probability that one bulb is defective is — and they are
50

or
ou
packed in boxes of 10. From a single box, find the probability that
ksf
(i) none of the bulbs is defective (ii) exactly two bulbs are defective
(iii) more than 8 bulbs work properly.
oo

A box has 20 pens of which 2 are defective. Calculate probability that out of 5 pens drawn
Y

one by one with replacement, at most 2 are defective.


B
re

1 1 49 7 37 247
oYu

1.42 X (0.94)^ (i) (ii) (iii)


ad

256
2 2 96 32 256

11 1 4547 14^^ 6 ri4f 1 14


d

1 -
’^15115
+

36 2 8192 15 15 U5
in
Re

1 45 243
73 37
(ii) (iii)
F

1
256 1024 512 1024
729

81 1 27 nf /9V
(i) (ii) (iii) 10
256 256 128 10

ri9 V ( 29 /19 6/19 6/19^‘‘ /19f


(iii) 1-T (iv) 1 -
20 y V 20
(i)
20
(ii)
5 I20 5I2O 20

10 5/H
10 10-r
1-Z C, (0.9)'' (0.1) 4U6
y =7

1 2
X: 0
21 14 1
P(X): 36 36
36
18.16
APPLIED MATHEMATICS-XIl

X: 0 1 2 3
64 144 108 27
P(X):
343 343 343 343

X: 0 1 2 3 4
625 500 150 1
P(X): 20
1296 1296 1296 1296
1296
10 5
Mean = —, Variance = —
3 ' 9
X: 0 1 2 3
125 25

w
P(X): 5 1
216 72 72 216

Flo
X: 0 1 2 3 4 5

e
1 5 10 10 5 1
P(X):

re
32 32 32 32 32 32

F
X: 0 1 2
ur
r
P(X):
4

9
4 fo 1
9 9
ks
Yo
oo

X: 1 0 -2
1
B

1 1
P(X):
2 4
re

4
1
3, 5 37
6
2
u

256
ad

5 1 63
Yo

(i) ^ (ii) (iii) ^ 0.0256


16 64 64
(i) 0.0879 (ii) 0.3672 (i) 0.2458 (ii) 0.34464
d
Re

496
(i) 0.0778 (ii) 0.087
in

0.0065
729
F

(i) 0.216 (ii) 0.432 9IO


(iii) 0.064 1 -
10
10
20
c 12
20 11
220 243
' 99 \50 (99)^^ X 149
(i) 1- (ii) f1 ( 99 (iii) 1 , 4
-100 2
100^®
4<'.. 4 47. At lest 3 times.

X: 0 1 2 3 4

2 25 5 1
PiX):
. 6> 3 U. 216 324 1296
18.17
:;r-i

0 1 2 3 4
X: 201 5
256 256 96 16 1
P(X): 3IO 16
625 625 625 625 625

4547 49
10
45 (49)
8
59 (49)^ ^f8
(iii)
8192
(i)
1,50
(ii)
(50)
10
(50)
10 27 19

1
1 - Here, n=4,p = Probability that the drawn ticket is marked with the digit zero 10

^ -1 ^ -1

ow
^nd/ —i —
' 10 10
1 V9f
Required probability = P (X = 0) = ^Cq 10 y V. 10

We have,

e
90 _ 9

re
p = Probability that a person is right-handed = 100 "10

rFl
F
1
■■■ =

Let X denote the number of right-handed persons in a sample of 10 persons. Then, X follows

or
ou
binomial distribution with parameters= 10, p =— and ‘I =
ksf
10-r
V ( 1
P(X=r) = lOp'-r r ,,9 ,r=0,l,2,...,10
oo

UO 10
Y

Required probability = P (X < 6)


B

= 1-P(X>6)
re

= 1 -[P (X =7) + P (X = 8) + P (X =9) + P (X =10)]


10 lO-r
10
9 Y n
oYu

10
1- X P(X=r) = 1- X Cr
ad

10 / \ 10
r=7 r=7

37. Let p denote the probability of succeeding in the experiment. It is given that
d

2
in
Re

p = 2 (1 - p) => p = -
Let X denote the number of successes in 6 trials. Then, X follows binomial distribution with
F

2 1
n=b,p = — and (7 = — such that
3 ' 3
6-r
f2Y n
P(X=r) = ^Q 8) 8)
; r = 0,1, 2,..., 6

Required probability = P (X > 4) = P (X - 4) + P (X - 5) + P (X 6) 4


a. Cc 2f II
2^ 31
^ 6r X

^“3 3r HsJ U
~ +
3J 3y ( 3; 9

40. Let X denote the number of defective eggs in a sample of 10 eggs drawn successively with
re
placement. Then, X follows binomial distribution with parameters li =10,
, 9
and q = —.
10
18.18
APPLIED MATHEMATICS-XII

10-r
10 1 V r 9 ^
A P(X=r) = Q — , r = 0,1, 2,..., 10
"uoj uo;
Required probability = P (X > 1) = 1 - P (X < 1) = 1 - P (X = 0)
10
I f g
= 1 - ‘“C0 = 1-^ 10
uoj lio. 10

ii. Here,« = 20, p = land, ^ = 1


\20
A P(X = r) =
20
C fi

ow
20 \20
Required probability = P (X >12) = ^ u
-
r = 12
1 99
! A Here,p =
100 ' n = 50 and, q = 100

e
50 ' 1 Y f 99

re
A P(X = r) = C
iooj li^
'-r

rFl
F
99 ^50
(i) Required probability =P(X>1)=1-P(X = 0)=1-
UOOJ

or
ou
x50
(ii) Required probability = P (X = 1) = x — x f—
Moo Uoo;
ksf
(iii) Required probability = P (X > 2) = 1 - P (X = 0) - P (X = 1)
x50
oo

99 n49
= 1 - 50/- 1 99
- Cj X X
uooj 100 uoo
Y
B

i - Let the shooter firen times and letX denote the


number of times the shooter hits the target.
re

Then, X follows binomial distribution with p = -4 and q = -4 such that


oYu

/ Q V 1t
ad

P(X=r)="Q
u;
It is given that
d

P(X>1)>0.99
in
Re

=> 1-P(X = 0)>0.99 => 1-i- >0.99 => —<0.01 => 4”>-l- => 4" >100
F

4" 4” 0.01
pie least value of n satisfying this inequality is
4 times.
4. Hence, the shooter must fire at least

4h, Suppose the man tosses a fair coin n times and X denote the number of heads in n tosses
Then,
/I \it
P(X=r) = 'Cr 1
n

●●● P
1

= ^ = 2
It is given that
P(X>1)>0.9
=> 1-P(X = 0)>0.9
^lY* n 1
1 -”C 0 - > 0.9 => ir. < => 2
n
> 10 => n = 4,5,6,....
2J \ 2J 10
Hence, the man must toss the coin at least 4 times.
BINOMIAL DISTRIBUTION 18.19

49, LetX denote the number of defective bulbs in a sample of 4 bulbs drawn successively with
1
replacement. Then, X follow binomial distribution with parameters n = 4, p = ~ = - and
30 5
1 4
(7 = 1 — = — such that
5 5
4-r

P(X=r) = -
(- ,r=0,l,2, 3, 4.
- ) V5/
Tlie probability distribution of X is
X: 0 1 2 3 4

4f ^4'\^ 1 16 96
4, X —1 4 16
P(X): 6 X — X X —

5) \5) 25 25 625 125 5 625 V5

w
18.4 MEAN AND VARIANCE OF BINOMIAL DISTRIBUTION

Let X be a binomial variate with parameters n and p. Then,

F lo
P{X=r) = "Q P'' r = 0,l,2,...,n.
a

Mean I r.P(X=r) [■.● Mean = I p,-

ee
r = 0

Fr
II

Mean L rxX.p'cp-''
r = Q

u
n ,1-1 -1
for " n-l
Q-iP-p' ●●● "Q = Cr-l
ur
Mean I r X -
r = 0 r r
s
n
H-l
ook

Mean - np S
Yo

r = 0
eB

a-1
Mean = np iq + p)
Mean = np [●●● p + t? = 1]
our

Variance =E (X^) -(£ (X)f


ad

and. Variance = I P(X=r)l-{Mean)2


Y

r =0
Re

,1

Variance = r =S Q (r(r-1)+r] "Q p'^ rt" -{np)^ ^ =r(r -1) + r and Mean =np]
nd

r
Fi

Variance = I r{r-1) "Cj. p'^ cf '■+ r =Z 0 r"Cr p'cp-'-inpf


I -

r = 0

a
n n-1
Cr-lP^-P' ^ f/' ^+»p-(np)^
a-2
Variance = I r(r-l)-.
r = 0 r r -1

^ ^ +np-n^p^
2
Variance = n(n-l) p S «-2 Q_2P r -

,r = 0
a-2 2..2
Variance =n («-1) p {q + p) + np - n p
2 2 2 2 2
Variance = n p - np + np - n p [●●● q + p =
Variance = np-np = ?ip(l-p) = npq

Hence, mean and variance of a binomial variate with parameters n and p are np and npq
respectively.
18.20 APPLIED MATHEMATiCS-XII

ILLUSTRATIVE EXAMPLES

EXAMPLE 1 Prove that the mean of a binomial distribution is always greater than the variance.
SOLUTION Let X be a binomial variate with parameters and p. Then,
Mean = np, and Variance = npq
2
Mean - Variance = np-npq = np{l-q) = np
2
Mean - Variance > 0 [●.● 71 e N and p >0 np^ > 0]
Mean > Variance.

EXAMPLE 2 A die is thrown 20 times. Getting a number greater than 4 is considered a success. Fi}td the
mean and variance of the number of successes.
SOLUTION Clearly, the distribution of the 'number of successes' is a binomial distribution with
n = 20 and,
2 1

w
p = (Probability of getting a number greater than 4) = —
6 3

F lo
1
rt=l-n = 1l 1 2
= —
3 3

Now, Mean = np and. Variance = npq

e
1

Fre
Mean = 20 x — = 6.66 and. Variance = 20 X - X - = 4.44
3 3 3

Hence, Mean = 6.66 and Variance = 4.44.


for
EXAMPLES The mean and variance of a binomial distribution are 4 and 4/ 3 respectively, find P (X ^ 1).
r
SOLUTION Let X be a biiiomial variate with parameters n and p. Then,
You
oks

Mean = np and Variance = npq


eBo

np = 4 and npq = Mean = 4, Var (X) = (Given)

4/3 1
npq
=> q = — => p = 1-1 = 2 [■■ p = 1-q]
4 3 3 3
our

np
ad

Putting p =2/3 in = 4, we get


2
nx— =4=>n = 6
dY

3
Re

Thus, we have
Fin

^
77 = 6, p = — and a ~ —
2 , 1
^ 3 ' 3
P(X=r) = "C,/
/ 2Y n
P(X=r) i ,r=0,l,2,..., 6.
V 3 3J
0
1.6
Now, P (X > 1) = 1 - P (X < 1) = 1 - P (X = 0) = 1 - ^Co f-
V 3 \3)
= 1 -
3)
=1 - 1 ^728
729 " 729
EXAMPLE4 A die is tossed thrice. Getting an even number is cojisidered as success. What is the variance
of the binomial distribution?
SOLUTION Here,;? = 3
p = Probability of getting an even number in a single throw of a die
3 1
'’=6 = 2
BINOMIAL DISTRIBUTION 18.21

1
and. q =\-p =
2 2
. 1 1 3
Variance = npq = 3 X — X — =
2 2 4

EXAMPLE 5 Find the binomial distribution for which the mean is 4 and variance 3.
SOLUTION Let X be a binomial variate with parameters n and p. Then,
Mean = 4, Variance = 3
np = 4 and npq - 3
npq = —3 => = -3

ow
np 4 ' 4

p =l-q = 1 4 " 4
1 .
Putting p = in Jip - 4, we get: n = 16

e
re
Then, we have
1 ^ 3

Frl
F
n = 16, p = —4 and q' = —4

Hence, the binomial distribution is given by


ou
or
16-r
16
P (X = r) = Cr - ,r = 0,1, 2,..., 16.
U 4
kfs
i:XAMPLE6 If the sum of the mean and variance of a binomial distribution for 5 trials is 1.8, find the
oo

distribution.
Y

SOLUTION Let n and p be the parameters of the distribution. Then,


B

Mean = np and Variance = npq


re

It is given that
n = 5 and. Mean + Variance = 1.8
oYu

Now, Mean + Variance = 1.8


ad

np + npq = 1.8
d

5p + 5pq = 1.8
5p + 5p (1 -p) = 1.8
in
Re

5p^ -lOp + 1.8 = 0


F

p2 - 2p + 0.36 = 0 (p - 0.2) (p -1.8) - 0 p = 0.2 [V 11

Thus, we have
n = 5, p = 0.2 and q = 0.8.
Therefore, if X denotes the binomial variate, then
5 -r
P(X=r) = ^Cr (0.2)" (0.8) ,r = 0,l,2, 3, 4,5.

This is the required binomial distribution.


example: The sum and the product of the mean and variance of a binomial distribution are 24 and 128
respectively. Find the distribution.
SOLUTION Let n and p be the parameters of the given binomial distribution. Then,
Mean = np and Varaince = npq, whereq=l -p
Now, Mean + Variance = 24 and. Mean x Variance = 128
np + npq = 24 and np x npq = 128
18.22 APPLIED MATHEMATICS-XII

np {1 + q) = 24 and P^ q = 128
24 128
lip = and p^
1+q

24 f 128
1 +q
=>
576q = 128 (1 + i/)^
9q - 2{1 +2q + q^)

2^ -5q+ 2 = 0 ^ {2q-l){q-2) = 0 => q = ^ [●●● q ^ 2]

ow
, 1
P = 1 -'/ = 2
Putting p = i , ly = i in»p + Jip;/ = 24, we get

e
re
- + -
2 4
= 24 ^ H = 32

Fl
F
Let X be the binomial variate. Then, the probability distribution of X is given by
ur 32-r
rn
P{X=r) = 32(2

r
r - ; r =0,1, 2,..., 32.
2) 2) fo
ks
EXAMPLE 8 The sum of mean mid variance of a binomial distribution is 15 and the sum of their squares
Yo
is 117. Determine the distribution.
oo

SOLUTION Let ii and p be the parameters of the distribution. Then,


Mean = np and Variance = npq.
eB

It is given that
Mean + Variance = 15 and, (Mean)^ + (Variance)^ = 117
ur

Now, np + npq = 15 and, n^p^ + n^p^q^ = 117


ad
Yo

np(l +q) = 15 and, = nj


d

nV(l+<7)^ = 225and/iV(l+'7^) =
Re
in

nVa + qf 225
F

117

(1 + qf ^ 225
{1 + q^) 117

1 + q^ +2q 225
117
1+r

=> 1 +
2^7 225

1 + q^ 117

27 108

1 + q^ 117

27 12

1+q^ 13
BINOMIAL DISTRIBUTION 18.23

1+q^ 13

^‘1 12

-l+q^ + 2q _ 13 + 12 (Applying componendo and dividendo]


l+q^-2q 13-12
^2
1 +q = 25 =>
1 +q ,
= 5 => 6r/ = 4
2
=> q = - = =1-1 1

l-q 1-q 3 " ' 3 3

1 2 .
Putting p = q = — in np + npq = 15, we get
3 3
n 2n ._ 5n ,^ _ o-?
3 9 9
1 2
Thus, 11 = 27, V = — and q = —
^ 3 3

w
Hence, the distribution is given by
27 -r
/ 1 Y f 2')

F lo
27
P(X=r) = Cr - , r = 0,1,2,..., 27.
"l3 / V 3j
EXAMPLE 9 If the probability of defective bolts is 0.1, find the mean and standard deviation for the

ee
distribution of defective bolts in a total of 500 bolts.

Fr
SOLUTION We have,
n = 500 and p = 0.1 for
Mean = up = 500 X 0.1 = 50
ur
And, S.D. = ^Variance = ^npq = -J50Q x 0.1 x 0.9 = 6.71
EXAMPLE 10 IfX follows binomial distribution xvith mean 4 and variance 2, find P (| X - 4 ] < 2).
s
ook
Yo

SOLUTION Let n and p be the parameters of the binomial distribution. Then,


eB

Mean = 4 and. Variance = 2


=>
np = 4 and npq = 2
npq _ 2 1 1
our

= -^ q = - => p = -
4 ' 2 2
ad

np

Putting p = l/2inMp = 4, we get: n = 8.


Y

Thus, X is a binomial variate with parameters /i = 8 and P ~


Re
nd

8-r
1 n
,r = 0,1, 2,..., 8.
Fi

P (X = r) =
2J U
Now,
P(|X-4|<2) = P(-2<X-4<2)
P(|X-4|<2) = P(2<X<6)
P(|X-4|<2) = P(X = 2) + P(X = 3) + P(X = 4) + P(X=5) + P(X = 6)

P(iX-4[<2) =
fl M
+ «C3^[2]
flv
- + ^C4H2-
8 fl 8. rnL
C2 -
^[2
8

8C2 + **C3 + ''C4 + '*C5 + «cJ[L


119
P(|X-4|<2) = 128

EXAMPLE 11 A perfect cubic die is thrown a large number of times in sets of 8. The occurrence of 5 or 6 is
called a success. In what proportion of the sets would you expect 3 successes.
18.24 APPLIED MATHEMATICS-XII

SOLUTION Let there be n sets of 8 dice.


2 1
We have, p = Probability of getting 5 or a 6 with six faced die = —
6 3
2

3 3

Let X denote the number of successes in one set of 8 dice. Then,

^ 1 y / 2 '\® “ ^
P(X = r) = ,r = 0,1, 2,..., 8.
H3 / V 3J

1792
P(X = 3) = ^3 -
^l3 3) 6561

ow
/ V

1792
Total number of sets in which we get 3 successes =N-P {X = 3) = N
6561
1792 1
So, percentage of 3 successes in 100 sets = Nx — X 100 = 27.31%.
6561 N

e
re
Fl
Find the expectation of the number of heads in 15 tosses of a coin.
CXAMl’Li; 11

F
SOLUTION
Let p be the probability of getting a head in a single toss. Then, P ~
ur
r
Clearly, the distribution of the number of heads is a binomial distribution with n = 15, P “
fo
ks
Expectation = E (X) - np = 15 x — = 7.5
2
Yo
oo

EXAMPLE 1-
In a game, a man wins a rupee for a six and loses a rupee for any other number when a fair
dieis thrown. The man decided to throw a die thrice but to quit as and lohen he gets a six. Find the expected
eB

value of the amount he wins/loses.


SOLUTION Let n denote the number of throws required to get a six and X denote the amount
ur

won/lost.
ad
Yo

The man may get a six in the very first throw of the die or in 2nd throw or in the third row (as he
has decided to throw a die at most thrice).
d

Thus, we have the following probability distribution for X.


Re
in

Number of throws (n): 1 2 3 3

Amoimt won/lost (X): 1 0 -1 - 3


F

1 5 1 5 5 1 5 5 5
Probability(P(X)): — X — — X —X — X — X

6 6 6 6 6 6 6 6 6

£(X) = Ixi6 + Ox —+(-l)x


25 125 364
+ (- 3)
36 216 216 216

EXAMPLE 1-- If tluo dice are rolled 12 times, obtain the mean and the variance of the distribution of
successes, if getting a total greater than 4 is considered a success.
SOLUTION Let X denote the number of successes in 12 trials. Tlien, X follows binomial
distribution with parameters « = 12 and p.
We have.
p = Probability of getting a total greater than 4 in a single throw of a pair of dice.
p - 1- Probability of getting a total less than or equal to 4
5
p == i-A
36 6
18.25
BINOMIAL DISTRIBUTION

q = 1-p = 1- 5^1
6 6
S 5 1 5
Mean — xl2 =10 and, Variance = npq = 12x—x- 3
6 6

EXERCISE 18.2

1. Can the mean of a binomial distribution be less than its variance?


2. Determine the binomial distribution whose mean is 9 and variance 9/4.
3. If the mean and variance of a binomial distribution are respectively 9 and 6, find the
distribution.
4. Find the binomial distribution when the sum of its mean and variance for 5 trials is 4.8.

ow
5. Determine the binomial distribution whose mean is 20 and variance 16.
6. In a binomial distribution the sum and product of the mean and the variance are
25
— and — respectively. Find the distribution.
3
7. The mean of a binomial distribution is 20, and the standard deviation 4. Calculate

e
Fl
re
parameters of the binomial distribution.
8. If the probability of a defective bolt is 0.1, find the (i) mean and (ii) standard deviation for

F
the distribution of bolts in a total of 400 bolts.
. n ● 10
ur
9. Find the binomial distribution whose mean is 5 and

or
variance ^ .
10. If on an average 9 ships out of 10 arrive safely to ports, find the mean and S.D. of ships
sf
returning safely out of a total of 500 ships.
k
Yo
11. The mean and variance of a binomial variate with parameters n and p are 16 and 8
oo

respectively. Find P (X = 0), P (X = 1) and P (X > 2).


12. In eight throws of a die 5 or 6 is considered a success, find the mean number of successes
B

and the standard deviation.


e

13. Find the expected number of boys in a family with 8 children, assuming the sex distribution
ur

to be equally probable.
ad

14. The probability is 0.02 that an item produced by a factory is defective. A shipment of 10,000
Yo

items is sent to its warehouse. Find the expected number of defective items and the
standard deviation.
d

a throw. Find the mean and variance of the


Re

15. A die is thrown thrice. A success is 1 or 6 in


in

number of successes.
F

16. If a random variable X follows binomial distribution with mean 3 and variance 3/2, find
P(X<5).
17. If X follows binomial distribution with mean 4 and variance 2, find P(X >5).
18. The mean and variance of a binomial distribution are — and - respectively. Find P (X > 1).
3 9 .. . 10
19. If the sum of the mean and variance of a binomial distribution for 6 trials is —, find the

distribution.

20. A pair of dice is thrown 4 Hmes. If getting a doublet is considered a success, find the
probability distribution of number of successes and hence find its mean.
21. Find the probability distribution of the number of doublets in three throws of a pair of dice
and hence find its mean.
22. From a lot of 15 bulbs which include 5 defective,a sample of 4 bulbs is drawn one by one
with replacement. Find the probability distribution of number of defective bulbs. Hence,
find the mean of the distribution.
18.26 APPLIED MATHEMATICS-XII

23. A die is thrown three times. LetXbe 'the number of twos seen'. Find the expectation ofX.
24. A die is tossed twice. A 'success' is getting an even number on a toss. Find the variance of
number of successes.

25. Three cards are drawn successively with replacement from a well shuffled pack of 52 cards.
Find the probability distribution of the number of spades. Hence, find the mean of the
distribution.
26. An urn contains 3 white and 6 red balls. Four balls are drawn one by one with replacement
from the urn. Find the probability distribution of the number of red balls drawn. Also, find
the mean and variance of the distribution.
27. Five bad oranges are accidently mixed with 20 good ones. If four oranges are drawn one by
successively with replacement, then find the probability distribution of number of bad

ow
one

oranges drawn. Hence, find the mean and variance of the distribution.
28. Three cards are drawn successively with replacement from a well shuffled pack of 52 cards.
Find the mean and variance of number of red cards.

e
ANSWERS

re
12-r

2. P{X=r) =
12
c
^ 4
n
4;
rFl
,r = 0,l, 2,...,12

F
27 -r
27 /1Y / 2 'i
3. P{X=r) = C. ,r=0,l, 2, ...,27

r
3J 3)
ou
4Y n
fo
ks
4. p(X=r) = - ,r=0,l,2,...,5
V 5 y V 5)
oo

100-r
100 1 4
Y

5. P(X=r) = , r=0,l,2,...,100
eB

5J K5
15-r

6. P(X=r) =
15 r'-r (^y ,r = 0,l,2,...,15 7. n = 100, p = -
1
T “ 8. (i) 40 (ii) 6
r

3J 3) ^ 5
ou

xl5-r
ad
Y

\r
15 n (2
9. P(X=r) =
"^3 3)
, r=0, 1,2,...,15 10. Mean = 450, S.D. = 6.71
32 x27
d

1 33
11.
,1 - 12. 2.67,1.33 13. 4 14. 200,14
Re
in

2) 2) 2.32
93 65
15- Mean = 1, Variance = 2/3 16. —
F

17. 18.
64 256 81

f If 2y~''
19. P(X=r) = ^Q r =0,1,2,..., 6
3) [3
20. P{X=r)=-^Cr V -6/ I -6 ;r=0,l, 2, 3,4 Mean = -
3
21. X: 0 1 2 3
125 75 15 1
P(X): Mean = 1/2
216 216 216 216'
\4-r
(1 (2
22. P(x=r)=-*C, ; r = 0,1, 2, 3, 4; Mean = — 23. i 24. i
\3) 3 2 2

25. P(X=r) = 3c ; r = 0,1, 2; Mean = —


' 4J u 4
BINOMIAL DISTRIBUTION 18.27

4-r
8
26. P{X=r) = -^C, ; r=0,l,2, 3,4; Mean = —, Variance = —
. 3J V3 3 9

'1 4 16
27. P(X=r) = ^C, ; r = 0,l,2, 3,4; Mean Variance =
^5 J v5/ 25
3 3
28. Mean = — , Variance =
2 ' 4

HINTS TO SELECTED PROBLEMS

23. Clearly, X follows binomial distribution with n = 3 and p = probability of getting two in a
single throw = —
^ 6

ow
£(X)=«p = 3xl=i.
24. Let X denote the number of successes. Then, X follows binomial distribution with
parameters 11 = 2, p = - .

e
Fl
1

re
Variance = »P‘7 = -

F
3-r
13 1 3 a M ^^/■ 3^
25. Here, n-3, p =
52
-and
4
£7^4
= -. P{X=r)=\. U- ,r = 0,l, 2.
ur
/ \ 4)

Mean -up =^. f or


ks
VERY SHORT ANSWER QUESTIONS (VSAQs)
Yo
oo

Anszver each of the following questio)is in one zvord or one sentence or as f>er exact requirement of the
question:
B

1. In a binomial distribution, if n = 20, q 0.75, then write its mean.


re

2. If in a binomial distribution mean is 5 and variance is 4, write the number of trials.


3. In a group of 200 items, if the probability of getting a defective item is 0.2, write the mean of
u

the distribution.
ad
Yo

4. If the mean of a binomial distribution is 20 and its standard deviation is 4, find p.


5. The mean of a binomial distribution is 10 and its standard deviation is 2, write the value of
d

q.
having a binomial distribution are 4 and 2
Re

6. If the mean and variance of a random variable X


in

respectively, find P (X =1).


7. If the mean and variance of a binomial variate X are 2 and 1 respectively, find P (X > 1).
F

8. If in a binomial distribution n = 4 and P (X = 0) = —, find q.


81
9. If the mean and variance of a binomial distribution are 4 and 3 respectively, find the
probability of no-success.
10. If for a binomial distribution P (X = 1) = P (X = 2) = a, write P (X = 4) in terms of a.
11. An unbiased coin is tossed 4 times. Find the mean and variance of the number of heads
obtained.

12. If X follows binomial distribution with parameters n =5, p and P (X = 2) = 9 P (X = 3), then
find the value of p.
ANSWERS

1. 5 2. 25 3. 40 4. 1/5 5. 0.4 6. 1/32

7. 11/16 8. 2/3 9.
3 V'' 10. a/3 11. Mean = 2, Variance = 1
u
12. 1/10
18.28 APPLIED MATHEMATICS-XI!

MULTIPLE CHOICE QUESTIONS (MCQs)


Mark the correct alteniafive hi each of the following:
1. In a box containing 100 bulbs, 10 are defective. What is the probability that out of a sample
of 5 bulbs, none is defective
z' 9 f1
(a)
10
(b)4
10
(c) 10-5
(d) 2
16

2. If in a binomial distribution n = 4, P (X = 0) = ^ , then P (X = 4) equals


1 1 1 1
(a) (b) (c) (d) T
16 81 27 8
3. A rifleman is firing at a distant target and has only 10% chance of hitting it. The least
number of rounds, he must fire in order to have more than 50'/o chance of hitting it at least

w
once is

(a) 11 (b) 9 (c) 7 (d) 5


4. A fair coin is tossed a fixed number of times. If the probability of getting seven heads is

F lo
equal to that of getting nine heads, the probability of getting two heads is
13
(a) 15/2' (b) 2/15 (c) 15/2 (d) none of these

ee
5. A fair coin is tossed 100 times. The probability of getting tails an odd number of times is

Fr
(a) 1/2 (b) 1/8 (c) 3/8 (d) none of these
6. A fair die is thrown twenty times. The probability that on the tenth throw the fourth six
for
ur
appears is
20

(a)
C,ox5^ (b)
120x5^
(c)
84x5^
(d) none of these
s
6^0 6^0 6l0
ook
Yo

P(X=r) .
7. If X is a binomial variate with parameters n and p, where 0 < p < 1 such that IS
eB

P(X=»-r)
independent of?? and r, then p equals
(a) 1/2 (b) 1/3 (c) 1/4 (d) none of these
r
ou
ad

8. Let X denote the number of times heads occur in n tosses of a fair coin. If P (X =4), P (X =5)
and P (X = 6) are in AP; the value of n is
Y

(a) 7, 14 (b) 10, 14 (c) 12, 7 (d) 14,12


Re
nd

9- One hundred identical coins, each with probability p of showing heads are tossed once. If
0 < p < 1 and the probability of heads showing on 50 coins is equal to that of heads showing
Fi

on 51 coins, the value of p is


(a) 1/2 (b) 51/101 (c) 49/101 (d) none of these
10. A fair coin is tossed 99 times. If X is the number of times heads occur, then P (X =r) is
maximum when r is

(a) 49, 50 (b) 50,51 (c) 51, 52 (d) none of these


11. The least number of times a fair coin must be tossed so that the probability of getting at least
one head is at least 0.8, is
(a) 7 (b) 6 (c) 5 (d) 3
12. If the mean and variance of a binomial variate X are 2 and 1 respectively, then the
probability that X takes a value greater than 1 is
(a) 2/3 (b) 4/5 (c) 7/8 (d) 15/16
13. A biased coin with probability p, 0 <p <1, of heads is tossed until a head appears for the
first time. If the probability that the number of tosses required is even is 2/5, then p equals
BINOMIAL DISTRIBUTION 18.29

(a) 1/3 (b) 2/3 (c) 2/5 (d) 3/5


14. If X follows a binomial distribution with parameters n = S and p =1/2, then P (| X - 4| < 2)
equals
118 119 117
(a) (b) (c) (d) none of these
128 128 128

15. If X follows a binomial distribution with parameters n = 100 and p=l/3, then P (X = r) is
maximum when r =
(a) 32 (b) 34 (c) 33 (d) 31
16. A fair die is tossed eight times. The probability that a third six is observed in the eight throw
is

^C2x5^ (b)
^C2x5^ (c)
^C2x5^ (d) none of these
(a)
6^ 6« 6^

w
17. Fifteen coupons are numbered 1 to 15. Seven coupons are selected at random, one at a time
with replacement. The probability that the largest number appearing on a selected coupon

F lo
is 9, is
/ o \7 ^7 n7
r 8
I (b) ^ (c) (d) none of these

ee
U5/ 15/

Fr
18. A five-digit number is written down at ranc 3m. The probability that the number is
divisible by 5 and no two consecutive digits are identical, is
1/9 for
I
9
ur
(b) - — (d) none of these
5 UoJ \0 /
s
19. A coin is tossed 10 times. The probability of getting exactly six heads is
ook
Yo

512 105 100


(a) (b)
512
(c)
153
(d) '"Q
513
eB

20. The mean and variance of a binomial distribution are 4 and 3 respectively, then the
probability of getting exactly six successes in this distribution, is
r

●1 \6 /
\ 10 / 3
ou
ad

4/ W ‘”^6 V4/
I 7
U/
Y

/1 \6
\6
3^^
(d) -c, i 4/
Re
nd

21. In a binomial distribution, the probability of getting success is 1/4 and standard
Fi

deviation is 3. Then, its mean is


(a) 6 (b) 8 (c) 12 (d) 10
22. A coin is tossed 4 times. The probability that at least one head turns up, is
14 15
(a)
16
(b) A
16
(c)
16
(d)
16

23. For a binomial variate X, if n = 3 and P (X = 1) = 8 P (X = 3), then p =


(a) 4/5 (b) 1/5 (c) 1/3 (d) 2/3
24. A coin is tosed n times. The probability of getting at least once is greater than 0.8. Then, the
least value of n, is
(a) 2 (b) 3 (c)4 (d) 5
25. The probability of selecting a male or a female is same. If the probability that in an office of n
persons (n -1) males being selected is 10 , the value of n is
(a) 5 (b) 3 (c) 10 (d) 12
18.30 APPLIED MATHEMATICS-XII

26. A box contains 100 pens of which 10 are defective. What is the probability that out of a
sample of 5 pens drawn one by one with replacement at most one is defective?
/ q \5 If 9^^ 1 f 9 \5 ( 9 ^ +— 1 f 9
(a) UO
^ (b) (c) - — (d)
2U0; 2I10J 10 2Uo;
27. Suppose a random variable X follows the binomial distributionwith parametersn and v,
, n 1 P(-X=r)
where 0 <p <1. If is independent of n and r, then p equals
P{X = n~r)
1
(a)- (b)i (d) r
1

7
28. Tlie probability that a person is not a swimmer is 0.3. The probability that out of 5 persons
4 are swimmers is

(a) '^C4(0.7)^{0.3) (b) (0.7) (0.3)^ (c) ^C4 (0.7) (0.3)'^ (d) (0.7)^(0.3)
29. Which one is not a requirement of a binomial distribution?

w
(a) There are 2 outcomes for each trial
(b) There is a fixed number of trials

F lo
(c) The outcomes must be dependent on each other
(d) The probability of success must be the same for all the trials.

ee
30. The probability of guessing correctly at least 8 out of 10 answers of a true false type

Fr
examination is

(b) -?
45
(a)
64 128
(c)
1024
for (d)4
41
ur
ANSWERS
s

1- (a) 2. (b) 3- (c) 4. (c)


ook

5. (a) 6. (c) 7. (a) 8. (a) 9. (b)


Yo

10. (a) n. (d) 12. (d) 13. (a) 14. (b) 15. (c) 16. (b) 17. (d) 18. (c)
eB

19. (b) 20. (b) 21. (c) 22. (d) 23. (d) 24. (b) 25. (d) 26. (d) 27. (a)
28. (a) 29. (c) 30. (b)
our
ad
Y
Re
nd
Fi
CHAPTER 12
POISSON DISTRIBUTION

ow
19.1 INTRODUCTION
Poisson distribution, similar to the Binomial distribution, is a discrete probability distribution
and is very widely used in statistical works where the number of trials is very large and the
probability of occurrence of an event in a trial is very small. It was discovered by the French
mathematician and physicist Simeon Denis Poisson (1781-1840) in 1837. This distribution is

e
used to describe the behaviour of rare events.

re
rFl
F
19.2 POISSON DISTRIBUTION

Poisson distribution is the limiting case of binomial distribution under the following conditions:
(i) n, the number of trials is indefinitely large, i.e., n cc.

r
ou
(ii) p, the probability of success for each trial is indefinitely small, i.e. ^
(iii) =»i (say) is finite.
0. fo
ks
Let X be a binomial variate with parameters n and p. Then,
oo

P{X=r)=Xr/ cf
I - r
Y

PiX=r) = ”Cr / (l-p)”"'' [●.● p + q = l q = l-p]


eB

n
P(X=r)=''Q (1-p)
ur
ad

m
Yo

If « 00 and p —> 0 such that iip =m, then p = —.


n
d

n\ P n
P(X=r) = (1-p)
Re
in

(ii -r) !r l[l - p


\r
F

m
V'
fi(n-l) (;i-2)...(n-(r-l)) II
m
1 --
P (X =r) = in
r! >2
1 -
n J

2V
1--
IV
1 1 - 3^
n / V n / V n n in V
1
P{X=r) =
rl rn Y n
1 -
H

'Yaking limit as « -> oo such that np - in, we have


-W/ 2\f ( r-l'i
1 - ^ ... 1-
n)\ n / V n n
n{ m

lim P (X = r) = lim 1 --
JI —» 00 n —» CO r! in Y nj
1 -
n
19.2 APPLIED MATHEMATICS-XII

r
m - m m') m
lim P(X=r) =
-m
e lim 1 — =e and, lim 1-- =1
H ^ CO r\ n-*ra \ n n-*oo 71

- m
Hence, lim P (X = r) = — e ,r = 0,l,2, ....,00.
/7 -> X r!

This is the required probability distribution function of the Poisson distribution.


DEFINITION A random variable X is said to follow a Poisson distribution if it assumes only
non-negative integral values and its probability distribution is given by
n{ e
- m

P(X=r) = , r = 0,1, 2,...., oc.


;●!

Here, m is known as the parameter of the distribution and m » 0.

w
REMARK 1 Poisson distribution is a probability distribution, because
X X
n{ g-'”
Z P(X=r)= Z

F lo
r = 0 r =0 r!

- m

ee
= e
/- = 0 r!

Fr
2 3
- HI , m m~ m
1 + — + — + — +....
1! 2! 3! for
ur
- HI ,r2
= e . e ●f h ....
s
1! 2!
ook
Yo

REMARK :
If a sequence of independent trials constitute an experiment and the experiment is repeated
eB

N times, then the frequencies ofO, 1, 2,3,... successes are given by


/(0)=A/.P(X = 0),/(1)=N.P(X=1),/(2) = N.P(X = 2),
r
ad

19.3 ROLE OF THE POISSON DISTRIBUTION


ou

The Poisson distribution is used in practice when there are events which do not occur as
Y

outcomes of a definite number of trials of an experiment but which occur at random points of
time and space. In other words, Poisson distribution is used in problems where there are
Re
nd

infrequently occurring events with respect to time, area, volume or similar units. In general, the
Fi

Poisson distribution is used to explain the behaviour of those discrete variables where the
probability of occurrence of the event is small and the total number of cases is large.
Following are some instances where Poisson distribution may be used :
(i) Number of printing mistakes at each page of the book,
(ii) Number of air accidents in some unit of time,
(iii) Number of suicides reported in a particular city,
(iv) Number of cars passing a crossing per unit of time during busy hours of a day.
(v) The omission of radioactive particles,
(vi) The number of deaths in a district in a given period of time by rare disease such as heart
attack, cancer etc.
(vii) Number of telephone calls received at a particular telephone exchange in some unit of
time,

(viii) The number of defective material in a packing manufactured by a good concern.


19.3
POISSON DISTRIBUTION

ILLUSTRATIVE EXAMPLES

EXAMPLE 1 Assume that the cimncc of an individual coahnmer being killed in a mine accident during a
1
year is . Use the Passion distribution to calculate the probabilily that in a mine employing 350
1400
-0.25
miners, there loill be at least one fatal accident in a year. (Use e = 078).

SOLUTION Let p be the probability that an individual coalminer will be killed in a mine
accident during a year and n be the total number of coalminers. Then,
1
n = 350 and p = [Given]
1400

Since p is very small and n is large, therefore, we shall use Poisson s distribution.
1
Webave,m=np ^ m = 350x = 0.25
1400

w
Let X denote the number of individuals killed in a mine accident during a year. Then, X is a
Poisson variate such that

F lo
-HI

P(X=r) = , r = 0,l,2. 00

r!

ee
(0.25)''
, r=0,l,2,

Fr
P(X=r) = CO

r!
0 0.25
(0.25)^' c -0.25
= 0.78 for
P (X = 0) = = e
ur
0!

Now,
s
Required probability = Probability of at least one fatal accident
ok
Yo

= P(X>1)=1 -P(X<1)=1-P(X = 0)=1 -0.78=0.22


o
eB

EXAMPLE 2 ff the probability that an individual suffers a bad reaction from an injection of a given
serum is 0.001, determine the probability that out of 2000 individuals (i) exactly 3, (ii) more than 2,
individuals suffer from bad reaction. (Given e ^ ■= 0.13534).
r
ou
ad

SOLUTION Let p be the probability that an individual suffers a bad reaction from an injection
of a given serum and n be the total number of individuals who took the injection.
Y

It is given that n = 2000 and p = 0.001


Re
nd

Since p is very small and n is large, therefore we shall use Poisson's distribution.
We have, m = np m = 2000 x 0.001 = 2.
Fi

Let X denote the number of individuals who suffer from bad reaction. Then, X is a Poisson
variate such that
- Ill
n{ e
P(X=r) = ,r=0,l,2. , X
r!

2'' c-2
P(X=r) = ,r=0,l,2. / ^
r!

(i) Required probability = P (X = 3)


o3
-1^. -
3!
4
= ix 0.13534 [●.● e" 2 = 0.13534]
3 3
054136
= 0.18044
3
19.4
APPLIED MATHEMATICS-XII

(ii) Required probability = P {X>2)


= 1 -P{X<2)
= 1 - P {X = 0) + P {X=l) + P {X = 2)]
2 e~^~\
-2
= 1 - c -2+2^ + 2^.
1! 2!

M 2 2
=1
\
,2
e e 2 ,2)
= 1 - ^ = 1 - 5 X 0.13534 = 1 - 0.67670 = 0.323

EXAMPLE 3 A mmiufnclurer knows that the razor blades he makes contain on the average 0.5%
defectives. He packs them in packets of 5. What is the probabiliti/ that a packet picked at random contains 3

w
or more fault}/ blades. (Given e~ = 0.9753)

SOLUTION Let p be the probability that a blade is defective. Then,

Flo
05
p = = 0,005
100

e
Since p is very small, therefore we should use Poisson's distribution. We have, n = 5.

re
m = up => /« = 5 X 0.005 = 0.025

F
Let X denote the number of defective blades in a packet of 5. Then, X follows Poisson
distribution with m = 0.025 such that
ur
r
P(X=r) =
nf e~
rl
m
fo
ks
(0.025)''
Yo
- 0.025
P(X=r) = X e , r = 0,1,2, 3,....
oo

r!

Now,
B

Required probability = P (X > 3)


re

= 1 - P (X < 3)
=1 -
P (X = 0) + P (X = 1) + P (X = 2)]
u
ad
Yo

=1 - 0.025 +, (0.025)2 ^-0.025


2!
d

= !- (?■ ‘’■^25 q_q25 + 0.0003125]


Re
in

=1 0.025 ^ J Q253
F

= 1 - 0.9753 X 1.0253 = 0.00002491.

EXAMPLE 4 At a busy traffic intersection, the probability p of an indwidual car having an accident is
very small, say p = 0.0001. However, during a certain part of the day, a large number of cars, say 1000,
pass through the intersection. Under these conditions, what is the probability of two or more accidents
occurring during that period ? (Use e~ ^ = 0.9048).
SOLUTION We have,
p = Probability p of an individual car having an accident = 0.0001
and. n = number of cars passing through the intersection = 1000.
Since n is very large and p is very small, so we can use the Poisson distribution to compute the
required probability. Let the random variableXdenote the number of accidents. Tlien, X follows
Poisson distribution with parameter m given by
w=;ip=1000x 0.0001 =0.1
19.5
POISSON DISTRIBUTION

The probability distribution of X is given by


-m
n{ c
P(X=r) = , r = 0,l,2,...
r!
-0.1
(0.1)'' ^ , r = 0,l,2,.... ...(i)
or. P(X=r) =
r!

Now,
Required probability = P (X > 2)
= 1-P(X<2)
= 1-{P (X=0) + P(X=1)
,-0.1
(0.1)
=1 e
-0.1
+ [Putting r = 0 and 1 in (01

w
1!

=1 (1+0.1)
-0.1
= 1 -1.1x 0.9048=0.0047

Flo
= l-l.lxc’

There are 50 telephone lines in an exchange. The probability that any one of them will be

e
EXAMPLE 5

busy is 0.1. What is the probability that ail the lines are busy ?

re
Let p be the probability that a telephone line will be busy and n be the total number

F
SOLUTION
of lines. Then,
ur
p = 0.1 and n=50.
m=np=5
Since p is very small, therefore we should use Poisson's distribution.
f or
ks
Let X denote the number of busy lines. Then, X is a Poisson variate such that
Yo
oo

-111
nf e
P(X=r) = , r=0,l, 2,
r!
B

S'-
, r=0,l,2,....
re

P(X=r) =
r!

Now,
u
ad

50 ,-5
Yo

5 X t

Required probability = P (X =50) = - 50!

one of the numbers from 1 to 200. 20 tickets are


d

EXAMPLE (> A box contains 200 tickets each bearing


Re

Find the probability that at most 4 tickets bear


in

drawn successively loith replacement from the box.


numbers divisible by 20.
F

SOLUTION Let p denote the probability that a ticket drawn bears a number which is divisible
by 20. Then,
10
P = = 0.05
200

Let n be the total number of tickets drawn from the box. Then, n = 20.
Since n is large and p is very small, so we can use the Poisson distribution to compute the
required probability. Let the random variable X denote the number of tickets which bear
numbers divisible by 20. Then, X follows Poisson distribution with parameter m given by
m =np = 20 X 0.05 =1
The probability distribution ofX is given by
-111
nf e
P(X=r) = , r = 0,l,2.
r!
19.6
APPLIED MATHEMATICS-XII

- 1

P(X=r)=.^ r =0,1,2, [v m = ll
Now,
Required probability = P (X < 4)
= [P (X = 0) + P(X=1) + P(X=2) + P(X = 3)+P(X=4)]
e~^ e~^ c~^ e~^
+ + + + 2+1 H
^ H
1 'l
0! 1! 2! 3! 4! 2! 3! 4!j
EXAMI’LE 7
The probability that a man aged 35 years will die before reaching the age of 40
years may be tnkeii ns 0.018. Out of a group of 400 men, now aged 35 years, what is the approximate
probability that 2 men will die within next 5 years ? (Use e~ = 0.000747).
SOLUTION Let p denote the probability that a managed 35 years will die before reaching the

w
age of 40 years. Then,
p = 0.018
We have, >7 = 400.

F lo
Since p is very small and n is very large, so we can use Poisson's distribution to compute the
required probability. Let the random variable X denote the number of persons now aged 35

ee
years and will die in the next 5 years. Then, X follows Poisson distribution with parameter

Fr
= 400x 0.018 =7.2

The probability distribution of X is given by for


ur
nf c-'"
P(X=r) = ,r=0,l, 2
r!
s
(7.2)'' e~^-^
ok
Yo

P(X=r) = ,r = 0,l,2,....
r!
o
eB

Now,
Required probability = P (X = 2)
_ (7.2)2
-7.2
r

X e 51.84 X 0.000747
ou
ad

= 0.01936
2! 2
Y

EXAMPLE 8 An insurance cojnpanyhas discovered that only about 0.1% of the population is involved
in a certain type of accident each year. If its 10000 policy holders were randomly selected from the
Re
nd

population, what is the probability that not more than 5 of its clients are involved in such an accident next
year ?
Fi

SOLUTION
Let p = probability that a person is involved in an accident
0.1 0.1
= 0.001 V 0.1% =
100 100

We have, n = Total number of policy holders = 10000


Since n is large and p is very small. Therefore, Poisson's distribution can be used to find the
required probability. Let the random variable X denote the number of policy holders who are
involved in some accident.

Then, X is a Poisson variate with parameter


777 =»p= 10000 X 0.001 =10
The probability distribution ofXis given by
?n'^ e - Ill
W X c"
P(X=r) = [V 777 = 10] ,..(i)
7-! r!
19.7
POlSSON DISTRIBUTION

Now,
Required probability =P(X< 5)
P (X = 0) + P (X =1) + P (X = 2) + P (X = 3) + P (X = 4) + P (X -5)
5 ^
5
= I P(X=r)= I = e
-10
I^ 1°:
r! = 0 r\
r = 0 r = 0 I '■

KXAMi-ii Amanufachirerofscreiosknowsthat4%of his product isdefectivcJfhi’sellstlH’^


boxes of 100 and guarantees that not more than 5 screivs wilt be defective, what is the probabditij that a box
will fail to meet theguranteed qualiti/ ?
chosen is defective. Then,
SOLUTION Let p be the probability that a screw

4
P = — = 0.04
100

We have, n = number of screws in a box = 100.

w
j?i = jip = 100 X 0.04 = 4
Let X denote the number of defective screws in a box of 100 screws. Since p is very small and n is

F lo
large, therefore, X follows Poisson^s distribution such that
-III
. nf e

ee
P(X=r) = ,r = 0,l, 2, 3,
r!

Fr
-4
4'' e
=> P(X=r) = ,r = 0,l,2, 3,
r!
for
ur
Now,
Required probability = P (number = P (of defective screws is greater than 5)
s
= P(X>5)=1-P(X<5)
ok
Yo

5
=1 - Z P(X=r)
o

r = 0
eB

5
-4 4''
=1 - Z e
r = 0 r:I
r
ou
ad

■ \

-4
f 5 4'
= 1 -c Z
= 0 r!
I''
Y

42 4^ 4*^ 45^
Re
nd

=1 -
Fi

43 44 45^
= 1 -c + h H
2! 3! 4! 5!

, N xi 1-1 I .A manufacturer of cotter pins knows that 5% of his product is defective. If he sells pins m
boxes of 100 and guarantees that not more than 4 pins will be defective, what is the approximate
probabilih/ that a box will fail to meet the guaranteed qualihj? (Given e = 0.0067.)
SOLUTION Let p be the probability that a pin is defective. Then,
5
= 0.05
100

We have, n = 100. m - np -100 X 0.05 = 5.


Therefore,
Let X denote the number of defective pins in a box of 100 pins. Then, X follows Poisson s
distribution such that
nf
P(X=r) = e-’’‘;r =0,1,2, 3....
r!
19.8
APPLIED MATHEMATICS-XII

Now,
Required probability = P (X > 4)
= 1 - P (X £ 4)
=1 -
P(X = 0) + P(X=1) + P(X=2) + P(X = 3) + P(X = 4)]
2 3 4
-in -m m m m m
= 1 - C +1’ + e
- m
+ c
-w
+ e
- Ill

1! 2! 3! 4!

2 3 4>
-III m m m
= l-e !+;?; + + +
2 6 24
-in
e
=1 - [24 + 24w + 12/??^ +
24
-5

w
e
= 1 -
24
[24 +120 + 300 + 500 + 625]
1569
= 1 -(0.0067) ^ = 0.0067]

F lo
I-.' e
24

= 1 - 0.437979 = 05620.

ee
EXAMPLE 11 Suppose 2% of the items nnule In/ n factory defective. Find the probability that there are

Fr
are

3 defective items in a sample of 100 items. (Given e~ ^ = 0.135)


SOLUTION Let p be the probability that an item is defective. for
ur
Then, p = — = 0.02.
100
s
We have, »= 100.
ok
Yo

Since p is small, so we use the Poisson's distribution with m = np = 100 x 0.02 = 2.


o

Let X denote the number of defective items. Then,


eB

nf
P(X=r) = e“'”,r = 0,l,2
r!
r
ou
ad

/'●=0,1,2,
r!
Y

Now,
2^ A
Re
nd

Required probability = P (X = 3)=~. e~^ =-x 0.135 = 0.180.


3! 3
Fi

EXAMPLE 12 The probability that a man aged 50 years will die loithin a year is 0.01125 What is the
probability that out of 12 such at least 11 will reach their fifty first birth day ? (Given
men

^-0.135 = 0.87371).
SOLUTION Since the probability of death is very small, so, we use the Poisson distribution.
Here,
p = probability that a man aged 50 years will die within a year
= 0.01125 and n = 12.
So, m=np =12 X 0.01125 = 0.135.
Let X denote the number of men aged 50 years who will die within a year. Then, X follows
Poisson's distribution such that
nf e
-III

P(X=r) = , r=0,l,2,
r!

(0.135)''
r = 0,l,2.
r!
19.9
POISSON DISTRIBUTION

Now,
Required probability = Probability that at least 11 men will reach their fifty first birthday.
= Probability that at most one man dies within a year
= P(X<1)
= P(X = 0) + P(X = 1)
- 0.135
= + 0.135 X e (Using (i)]
0.135
= 1.135 X c

= 1.135 X 0.87371 =0.99166.

EXAMPLE 13 Records show that the probabiUti/ is 0.00002 that a car will have aflat tyre while driving
over a certain bridge. Use the Poisson's distribution to determine the probability that among 20,000 cars
driven over the bridge, not more than one will have a flat tyre. ( Given e ' = 0.670).
SOLUTION We have,
p = probability that a car has flat tyre = 0.00002. and h = 20,000.

w
m = np = 20000 X 0.00002 = 0.4
Then, X follows Poisson's distribution such

F lo
Let X denote the number of cars having flat tyres.
that
nf - m ...(i)
P(X=r) = e ,r=01,2,....

e
r\

Fre
Now,
Required probability = P (X < 1)
for
= P(X = 0) + P(X=1)
-m -m
[Using (i)]
r
- e + m e
You

= e-"' (m + 1)
oks

= c" (0.4 4-1)


eBo

= 0.670x1.4 = 0.938

EXAMPLE !4 Suppose 220 misprints are distributed randomly throughout a book of200 pages. Find the
probability that a given page contains (i) no misprints, (ii) one misprint, (Hi) 2 misprints, (iv) 2 or more
our
ad

misprints. (Given e~ =0.33287).


SOLUTION Let p be the probability of selecting a page out of 200 pages. Then,
dY
Re

u = — = 0.005.
^ 200
Fin

Since p is small, we use the Poisson's distribution.


Here, n = Total number of misprints = 220.
Average number of misprints in a page= n p
m =n p=> m = 220 x 0.005 = 1.1
Let X denote the number of misprints in a page. Then, X follows Poisson's distribution such that
p(X=r)=Xr'",
r!
r = 0,l,2, 3
,-1-1 , r=0,l,2,...
r!

(i) Required probability = P (X = 0)


= =0.33287

(ii) Required probability = P (X =1)


19.10
APPLIED MATHEMATICS-XIl

~e
1!
^-^=1.1x 0.33287 =0.366157
(iii) Required probability = P (X = 2)

2!

(1.1)^ X e
n

2
1.21 X 0.33287
2
= 0.20138.
(iv) Required probability = P (X > 2)
= 1 - P (X < 2)

w
= 1 -[P(X = 0) + P(X = 1)]
= 1 -10.33287 + 0.366157] [Using (i) and (ii)j
= 0.300973.

Flo
HX.AM I’l }■; 1 ^ Suppose a book of585 pmgcs contains 43 typographical errors. If these errors are randoinli/
distributed throughout the book, what is the probability that 10 pages, selected at random will befree from

ee
errors ? (Use e~ = 0.4795).

Fr
SOLUTION Let p be the probability that a misprint appears on a page. Then,
^ _ Number of misprints 43 = 0.0735 for
ur
Total number of pages 585
We have,« =10
s
w=,i^=10 x 0.0735 = 0.735
k
Yo

Since p is very small. Therefore we use the Poisson's


oo

distribution.
Let X denote the number of misprints in 10 pages. Then, X follows Poisson's distribution such
eB

that

- m
P(X=r) = e , r = 0,l,2, 3....
r

rl
ou
ad

(0.735)'' -0.735
P(X=r) = e
,r=0,l,2, 3
Y

r!
Now,
Re
nd

- 0.735
Required probability= P (X = 0) = e [Puttingr =0 in (i)j
Fi

nxAMl’Li-: ifa Ten percent of the tools produced in a certain manufacturing process turn out to be
defective. Find the probability that in a sample of 10 tools chosen at random, exactly two will be defective
by using (i) the binomial distribution, (ii) the Poisson distribution.

SOLUTION Let p be the probability that a tool is defective. Then,


10
P = = 0.1.
100

(i) We have, n = 10, p = 0.1 and q = l -0.1= 0.9


Let X denote the number of defective tools. Then,
P{X=r)= (0.1)^ (0.9) ^ r = 0,1, 2,.... 10
Required probability = ’'’C2 (0.1)^ (0.9)^ = 45 x 0.01 x (0.9)
8
= 0.194.

(ii) We have, p = 0.1 and n=10.


m=np =10 X 0.1 =1
Let X denote the number of defective tools. Then,
19.11
POISSON DISTRIBUTION

-1
e
-m
V .e
P(X=r) = , r=0,l,2,...
r:
I ●I
ri r\
-1
£ 1
— = —= 0.184.
Now, Required probability = P (X = 2) = - 2! 2e

EXAMIM.E 17 Suppose the probability that an item produced by a particular machine is defective is 0.2. If
10 items produced by this machine are selected at random, what is the probability that not more than one
defective item is found ? Use the binomial and Poisson distributions and compare the ansiuers (Use
0.1353).

Lot p denote the probability that an item produced by a particular machine i


IS
SOLUTION
defective. Then, p ~ 0.2 (given).

ow
Using binomial distribution : We have, )i = 10, p = 0.2 and ly = 1 - p = 0.8
Let X denote the number of defective items in a sample of 10 items. Then, X follows binomial
distribution such that
lO-r
P(X=r) = %, (0.2)'' (0.8) , r=0,l,2....

e
re
Required probability = P (X < 1)
= P (X = 0) + P (X = 1)

rFl
F
= (0.2)" (0.8)^" + '"Ci (0.2)1 (08)^
= (0.8)l"+10x0.2x(0.8)^

r
ou
= (0.8)^ (0.8 + 2)=2.8x(0.8)‘l = 0.3758 fo
ks
Using Poisson's distribution: We have, p = 0.2 and »=10.
m-np => m=^ 2.
oo

Let X denote the number of defective items in a sample of 10 items. If X follows Poisson s
Y
eB

distribution, then we have


- m
n( e
P(X=r) = - , r=0,l,2,
r

r!
ou

2''
ad
Y

-2
= — e ,r=0,l,2,....
rl

Required probability = P (X < 1) = P (X = 0) + P (X = 1)


d

2" I 2x
Re
in

OT^ 1!
= ^ (1 + 2) = 3 X ^
F

= 3 X 0.1353 = 0.4059

EXAMI'Lt 18 If X is a Poisson variate such that 3P(X = 2)=2 P(X=1). Find (i) P(X=0), (ii)
-4/3
P (X = 3). (Given e = 0.264).

SOLUTION Let the probability distribution of X be


n{ - m
P(X=r) = e , r=0,l,2,....
r!

where m > 0.
Now, 3P (X=2) = 2P(X=1)
m\-"'
- Ill
m e
3. = 2.
2! 1!
4
-m'^=2m => 3m^-4m = 0 => w(3m-4)=0=> m = - [v m>0]
3
2
19.12
APPLIED MATHEMATICS-XII

4 .
Putting m = - in (i), we get

4 ,-4/3
P(X=r)=^ ,r = 0,l,2....
r!
Now,
-4/3
(i) P(X = 0)=e = 0.264

4^3 ,-4/3
I 3
(ii) P(X = 3) = :^,-4/3
3! 81

EXANtiM.r. 19 //X /I£7S a Poisson distribution such that P (X = l)=P{X = 2), ifnd P (X = 4) (Given
C’"^= 0.1353).

w
SOLUTION Let the probability distribution of X be given by

F lo
n{ e -m
P{X=r) = ,r=0,l,2, 3,
r\

ee
Now, P(X = 1)=P(X=2) ni e

Fr
2!
2m = n? m [m - 2) = 0 => m=2
for [v m>0]
Putting w = 2 in (i), we get
ur
2^ e-2
P(X=r) = ,r=0,l,2,...
s
r!
ok
Yo

24 ,-2 2
P(X = 4) =
o

- X 0.1353 - 0.0902
4! 3
eB

CXAMi-U 20 In a certain factor}/ turning out blades, there is 0.2% probability for any blade to be
defective. Blades are supplied in packets of 10. Using Poisson distribution, calculate the approximate
r

number of packets containing (i) no defective (ii) one defective (Hi) two defective and (iv) 3 defective blades
ou
ad

consignment of 10000 packets, it being given that e~


in a
= 0.9802.
Y

SOLUTION Let p denote the probability that a blade is defective. Then,


0.2
Re
nd

P = = 0.002
100
Fi

We have, n = number of blades in a packet = 10.


m=np=> w = 10 X 0.002=0.02

Let X denote the number of defective blades in a packet of 10 blades. Then, X follows Poisson's
distribution such that the probability of r blades being defective i a packet is given by
in

nf e c 0-02(0.02) = 0.9802 X (0.02)''


- m

P(X=r) =
r! r! rl ...(i)

denote the number of packets, each containing r defective blades, in a consignment of


10000 packets. Then, °
fir) =10000 X P(X=r) [Using ;/(r) = N.P(X=r)]
f(r) =10000 X 0.9802 x
/●! [Using (i)]
(0.02)
fir) = 9802
rl -(ii)
19.13
POISSON DISTRIBUTION

(i) Putting r = 0 in (ii), we get


/(O) =9802x 1 =9802
Thus, the number of packets containing no defective blade is 9802.
(ii) Putting r = 1 in (ii), we get
0.02
/(I) = 9802 X = 196.04
1!

Thus, the number of packets containing one defective blade is 196.


(iii) Putting r = 2 in (ii), we get
(0.02)^ = 9802 X 0.0002=1.9604
/(2)=9802x 2!

Thus, the number of packets containing 2 defective blades is 2.


(iii) Putting r = 3 in (ii), we get
(0.02)^ = 0.013
/(3)=9802 3!

w
Thus, there is no packet containing 3 defective blades.

F lo
EXAMPLE 21 After corresponiiing 100 pages of a book, the proof reader finds that there are, on the
average, 4 errors in 10 pages. How many pages would one expect tofind with 0,1,2,3 and 4 errors in WOO
pages of the first print of the book 7 (Use e = 0.6703).

ree
4
The average number of errors per page = — = 0.4
F
SOLUTION
for
Thus, if m is the mean of the Poisson's distribution, then m = 0.4.
Let X be the random variable denoting the number of errors per page.
r
-0,4
h/c”'" (0.4)''e (0.4)^
You

= 0.6703 X ... 1
oks

Then, P(X=r) = r\
r! r\
eBo

Let/ (r) denote the number of pages, each containing r errors, in 1000 pages.
Then,
/ (r) =1000 xP(X=r)
ad
our

(0.4)''
= 1000 X 0.6703 X [Using (i)]
r!

= 670.3 X
(0.4)'' ...(i)
Re
dY

r!

Putting r = 0,1, 2, 3 and 4 in (ii), we get


Fin

0
(0.4)
= 670.3
/(O) = 670.3 0!
1
(0.4) = 268.12
/(I) = 670.3 X 1!

(0.4)2 = 53.624
/(2) =670,3 2!

(0.4)3 = 7,1498
/(3)= 670.3 3!

(0.4)4 = 0.71498
/(4) =670.3 X 4!

Hence, the number of pages containing 0,1,2,3 and 4 errors are 670,268,54,7 and 1 respectively.
19.14
APPLIED MATHEMATICS-XII

19.4 MEAN OF POISSON DISTRIBUTION


Let a random variable X follows Poisson's distribution with parameter m. Then,

P(X = r) =
- m
.e r r = 0,l,2.
r!
Now,
CO

Mean = I r.P{X = r) Mean = Z pj x)


r = 0 I

r
m - m
1 r. — ^
/- = 0 r\
cc r-1
r .m. m
- m
1 e
r
= 0 r (r-1)!

1 ~^

w
- m
= m e

r = i (r-1)!

F lo
2 3
- m w m m
= m e 1 + + + +....
1! 2! 3!

ee
- m m
= m e . e ' =m.

Fr
Tluis, the mean of Poisson distribution with parameter m is parameter m itself.
19.5 VARIANCE OF POISSON DISTRIBUTION
for
ur
Let a random variable X follow Poisson distribution with parameter m. Then,
r
m
P {X=r) =
s

. e , r=0,l,2, 3,
ook

r!
Yo

Now,
eB

CO

Variance = r =I 0 r^ . P (X =r)-(Mean)“ V Var(X) = Ip^.v,-2-(Sp. .v,.)2


our
ad

n{ e
- in

= I r^. -7U
2
[●.● Mean =»/]
r = 0 rl
Y
Re

00

I [r(r-l) + r] — -m'^
- m
nd

=-e
r = 0 r!
Fi

- m
CO
,7/ CO
2
= e
I r(r-l) + I r. -m
r = 0 r! r =0 rl

CO CO -1
- in
2
= e Z r(r-l). + Z r. m. -m
r = 0 r(r-l) (r-2)i r = 0 r(r-l)!

- in 2 2
-e m I + m Z -m
r =2 {r-2)l r = i (r-1)!
- in
— e

2 2
= m + 7U - 771

= m.

Thus, the variance of a Poisson variate with parameter m is the parameter m itself and the
standard deviationis ^/m.
19.15
PCISSCN DISTRIBUTION

19.6 ,^?^CURREMCE FORMULA FOR THE PR08A^I-.I TIES OF POISJOH DISTRIBUTION


Let X be a binomial variate with parameter m. Then,
- HJ r+1 -HI
rji'" e m

P(X=r) = and P(X=r + l) =


r! (r +1)!
-HI
r!
P(X=r + l) m
X

P(X=r) (r +1)! m'- r'”


P(X=r + l) m

P(X=r) r + 1
m
P(X=r + l) = P{X=r), r=0,l,2, 3,...
r + 1

This is called the recurrence formula for the probabilities of the Poisson distribution. Using this

w
formula, we can find P (X = 1), P (X = 2), P (X = 3),if we are given P (0) = t’
If m is known, then P (X = 0) = e
- HI
can be found and the values of P (X = 1), P (X = 2),.... can be

F lo
found by putting r = 0,1, 2, 3 .... in (i) as explained below.
Putting r = 0,1, 2, 3,.... in the recurrence formula (i), we have

ee
P(X = 1) = m.P(X = 0)

Fr
2
ni

P(X==2)=^ P(X=l)=^x)»PfX = 0) = P (X = 0) 2
2!
3 for
in m

P(X = 3)=|P(X = 2)='^ P (X = 0) = P (X = 0)


ur
X

2! 3!
3 4
m
s
in

P(X = 4)=?^P(X = 3)=^^ P(X- = 0} = P (X = 0) etc.


ook

X
Yo

3! 4!
eB

ILLUSTRATIVE EXAMPLES

EXAMPLE 1 If a rnndoin wricihle X follows Poisson s distribution such that


r

P(X =2) =9. P(X = 4) + 90 P(.T = 6).


ad
ou

Find the mean and variance ofX.


of random variable X. Since X follows Poisson s distribution.
Y

SOLUTION Let m be the mean


Re

therefore
nd

i:f m
P(X=r) = e ,r=0,l,2.
Fi

r\

Now,
P (X = 2) = 9. P(X = 4) + 90. P (X = 6)
2 4 6
m m - HI
m - HI
-HI
e = 9. e + 90 e

2! 4! 6!

111 2 90 in^
+
2! 4! 6!
,1 9 2 90 4
=— in + m
12 360
.!= —3 2
/«+- m
1 4
4 4

in^ + 3ni^ -4=0


0»2 + 4)(hi^-1)=O
19.16
APPLIED MATHEMATICS-XII

m^-1 =0
+ 4 0]
m = 1.
[v m>0]
Hence, the mean and variance of X are each equal to 1.
EXAMPLE 2 IfX is Poisson variate such that P(X = 1) = 2 P (X = 2), find (i) mean (ii) variance (Hi)
P (X = 0).
SOLUTION
Let m be the mean of Poisson variate X. Then,
P(X=r) = e"'",r=0,l,2
r!

Now, P(X = 1) = 2P(X = 2)


2
m

ow
-III 2
m^ -m = 0
m
m e = 2 e m =m
2!

m{m-l)=0 => m=l


[v m>0]
Since =1. Therefore, mean and variance of X are each equal to 1.
m^e -m 1

e
(iii) P(X = 0) = - e
-m

re
0!

rFl
e

EXAMPLES If the variance of a Poisson's distribution is 2, find thedistributionforr = 1,2,3,4,5 from

F
the recurrence relation of the distribution. (Use e~ “ = 0.1353).
SOLUTION
Let m be the parameter of the given Poisson's distribution.

r
ou
We have. fo
ks
Variance = 2=> m = 2
LetX denote the Poisson variate. Then,
oo

nf e - in

P(X=r) =
Y

r! r!
eB

P(X=0)=c~ 2 =0.1353 [Given]


Putting r = 0,1, 2, 3, 4 in the recurrence formula
r
ou

ni
P(X=r + l) = . P (X =r), we get
ad
Y

r + 1

P (X = 1) = 7H. P (X = 0) = 2 X 0.1353 = 0.2706


d

P (X = 2) =
^ P (X = 1) = IX 0.2706 = 0.2706
Re
in

m 2
P (X = 3) = - P (X = 2) = - X 0.2706 = 0.1804
F

3 3
111 2
P (X = 4) = - P(X = 3) = - X 0.1804 = 0.0902
4 4
in 9
P (X =5) = - P (X = 4) = - X 0.0902 = 0.361
5 5

EXAMPLE 4 Six coins are tossed 6400 times. Using Poisson distribution obtain the approximate
probabilit}/ of getting 6 heads x times.
SOLUTION
Let p be the probability of getting 6 heads in a throw of six coins. Then,
1
p =
\2) 64
Here, n = 6400.
1
Mean = m = np= 6400 x = 100.
64
19.17
POISSON DISTRIBUTION

Let X denote the number of times 6 heads occur in 6400 tosses of six coins. Then,
- 100
/j/c
- m
(100)'' c
P(X=r) = , r = 0,l,2,....
I
ri r!
100
(lOO)'-"' c
P(X=.Y) =
.v!

EXAMPLE 5 Suppose that in key rucking of 80 cohinw IBM cards, the arithmetic mean number of
mistakes per card is 0.3. Wliat percent of cards xoitl have (i) no mistake, (ii) one mistake, and (w) two
mistakes. IGiven e~ = 0.7408]
SOLUTION We have, m = 0.3.

ow
Let X denote the number of mistakes. Then,
-m
nf e
P(X=r) = ,r=0,l,2,....
I
r:

(0.3)'' c" ^ ,r = 0,l, 2,


P(X=r) =

e
r!

re
(i) We have P (X = 0) =
(0.3)° r ^ = c

rFl
2 = 0.7408

F
0!

Therefore, 74% of the cards punched will have no mistake.

r
(0.3).
ou
(ii) We have, P (X =1) = 1! fo
ks
= (0.3) X 0.7408=0.22224.
This means that 22% of the cards punched will have one mistake.
oo

(0.3)^e“ ^ _ 0.09 X 0.7408 = 0.0333


Y

(iii) We have, P (X = 2) =
eB

2! 2

This means that 3% of the cards punched will have 2 mistakes.


number of phone calls per minute coming into
ur

EXAMPLE 6 Between the hours 2 and 4 p.m. the average


the switchboard of a company is 2.5. Find the probability that during one particular minute there wdl be
ad
Yo

no phone cal! at all. (Given e ^ = 0.13435 a?id e = 0.60650).


This is a problem of Poisson distribution such that m - mean = 25.
d

SOLUTION

Let X denote the number of phone calls received during a minute. Then,
Re
in

- m
nf e
F

P(X=r) = 1
r;
-25
(25)'' ^ , ;-=0,l, 2
r!
-15 -2 05
Now, Required probability = P (X = 0) = f X e

= 0.13534 X 0.60650 = 0.0821.

::
EXAMPLE 7 III a certain factory turning out razor blades, there is a small chance 1/500for any blade to
be defective. The blades are supplied in a packet of 10. Use Poisson distribution to calculate the
approximate number of packets containing (i) no defective, (ii) one defective, (in) two defective (w) 3
defective blades, in a consignment of 10,000 packets.
SOLUTION We have, p = probability that a blade is defective = —^ , »=10.

1
m = np = xl0 = — = 0.02
500 50
19.18
A PPLIED M ATHEMA7ICS-X:i

LetX denote the number of defective blades in a packet of 10 blades.


Then,
v{ e-"’
P(X=r) =
r!

(0.02)''
, r=0,l,2,....
rl
-0.02
(i) We have, P{X = 0)=e = 0.98019

Number of packets containing no defective blade in a consignment of 10,000 packets


= 10,000 xP(X = 0)
= 10,000 X 0.98019 = 9801.9 = 9802, approximately.
(ii) We have.
-0,02

w
P(X=l)=0.02xf = 0.02 X 0.98019 = 0.0196038

Number of packets containing one defective blade in a consignment of 10,000 packets

F lo
= 10000 X P(X=1)
= 10000 X 0.0196038

ee
= 196.038

Fr
= 196, approximately.
(iii) We have.
(0.02)^p 0.02
0.0004 X 0.98019
for
ur
P(X = 2) = = 0.000196
2! 2

Required number of packets = 10000 x P (X = 2)


s
ok
Yo

10000 X 0.000196 = 1.96 = 2, approximately.


(iv) We have.
o
eB

P (X = 3) =
(0.02)^c~°-“- = 0.0000013
3!
r

So, required number of packets =10000 x P (X = 3) = 0.013 = 0, approximately,


ad
ou

i: -y ^ Suppose that a manufactured product has 2 defects per unit of product inspected. Using
Poisson distribution, calculate the probabilities offinding a product (i) ivithout any defect, (ii) 3 defect^
Y

(iii) 4 defects. (Given e~'^ = 0.135).


Re
nd

SOLUTION
It is given that the average number of defects is 2. This means that the mean of
Poisson distribution is 2 i.e. m = 2.
Fi

LetX denote the number of defects. Then,


2'' ^ nf e~"'
P(X=r) = , r = 0,l,2,... Using: P (X =r) =
rl
rl

(i) Required probability = P (X = 0) = ^ = 0.135


2
(ii) 4-2 4
Required probability = P (X = 3) = — if ^ —
- X 0.135 = 0.180.
3! 3 3
2
(iii) Required probability = P (X = 4) = - -X 0.135 = 0.09
4! 3

EXERCISE 19.1

Derive Poisson distribution as a limiting form of binomial distribution.


Show that the mean and S.D. of a Poisson distribution are equal.
19.19
POISSON DISTRIBUTION

3. Explain the meaning of Poisson distribution and state the conditions under which this
distribution is used.
4. If a random variable X has a Poisson distribution such that P (X = 1) = 2 . P (X = 2). Find
P (X = 0).
a.
It is given that 3% of electric bulbs manufactured by a company are defective. Using the
Poisson distribution, find the probability that sample of 100 bulbs will contain (i) no
defective, (ii) exactly one d-'^'cetive.
6. Certain mass- produced articles of which 0.5 % are defective, are packed in cartons each
^ free from defective articles and what
containing 100 articles. What proportion of cartons are
proportion contains 2 or more defectives ? (Given e = 0.60650).
7. Suppose on an average 1 house in 1000 in a certain district has fire during a year. If there are
2000 houses in that district, what is the probability that exactly 5 houses will have a fire

w
during the year ?
8. Suppose 300 misprints are distributed randomly throughout a book of 500 paps. Find the

Flo
probability that a given page contains (i) exactly 2 misprints, (ii) 2 or more misprints,
9. Suppose 1% of the items made by a machine
defective. Find the probability that 3
are
or

e
re
more items are defective in a sample of 100 items.
Suppose 2% of the people on the average are left-handed. Find the probability of 3 or more

F
10.

left-handed among 100 people. , . x fmnnnn


Suppose there is an average of 2 suicides per year per 50,000 population. In a city of 10000
ur
or
11.

find the probability that in a given year there are (i) 0, (ii) 1, (iii) 2, (iv) 2 or more suicides,
f
12. An insurance company found that only 0.017o of the population is involved in a certain type
ks
of accident each year. If its 1000 policy holders, were randomly selected from the
Yo
than two of its clients are involved in
population, what is the probability that not
more
oo

such an accident next year? (Given e~ = 0.9048).


B

road of Delhi is 0.001, on how


13. Assuming that the chance of a traffic accident in a day on a accident (ii) more than three
many days out of a total of 1000 days can we expect: (i)
re

no

accidents, if there are 1000 such roads in the whole city.


u

14. An office switchboard receives telephone calls at the rate of 3 calls per minute on
an
ad
Yo

minute interval, (ii) at the


average. What is the probability of receiving (i) no calls in a one
most 3 calls in a one minute interval?
-product articles of which 0.5 per cent are defective, are packed m cartons each
d

15. Certain mass


Re

containing 130 articles. What proportion of cartons are free from defective articles and what
in

proportion contains 2 or more defectives? t


F

16. A manufacturer knows that the condensers he makes contain on the average 1 o o
defectives. He packs them in boxes of 100. What is the probability that a box picked at
random will contain 4 or more defective condensers?
17. Out of 1000 balls 50 are red and the rest white. If 60 balls are picked at random, what is the
' than 3 red balls in the sample?
probability of picking up (i) 3 red balls (ii) not more

Assume Poisson distribution for the number of red bells picked up in the sample, where
c” ^ = 0.0498.
call is wrongly connected
18. In an automatic telephone exchange the probability that any
one

is 0.001. What is the minimum number of independent calls required to


ensure a

probability of 0.09 that at least one call is wrongly connected?


19. The probability that an item, produced by a certain machine will be defective is aOl By
applying Poisson's distribution, show that the probability that random sample of 100 items
selected at random from the total output will contain not more than one defective item is
27o.
19.20
APPLIED PMTHEMATICS-XII

20. A manufacturer, who produces medicine bottles, finds that 0.1% of the bottles are
defective. The bottles are packed in boxes containing 500 bottles. A drug manufacturer
buys 100 boxes from the producer of bottles. Using Poisson distribution, find how many
boxes will contain : (i) no defective, (ii) at least two defectives. [Given c" = 0.60650]
21. A telephone exchange receives on an average 4 calls per minute. Find the probability on the
basis of Poisson distribution, of (i) 2 or less calls per minute (ii) up to 4 calls per minute (iii)
more than 4 calls per minute. (Use : ^ = 0.01832)
^e number of accidents in a year attributed to taxi
22.
drivers in a city follows Poisson
distribuhon With mean 3. Out of 1000 taxi drivers, find approximately the number of
drivers with (i) no accident in a year, and (ii) more than 3 accidents in a year. [Given
e = 0.3679, e 0.1353, e~ ^ = 0.0498]
Red blood cell deficiency may be determined by examining a specimen of the blood under a
microscope. Suppose a certain small fixed volume contains on the average 20 red cells for
normal persons. Using Poisson distribution, obtain the probability that a specimen from a

w
normal person will contain less than 15 red cells. ^

F lo
The probability of getting no misprint in a page of a book is
24.
What is the probability that
a page contains more than 2 misprints?
A manufacturer of bulbs knows that on an average 4% of his products are defective. He

e
sells bulbs in boxes of 100 pieces and guarantees that not more than 3 bulbs will be defective

Fre
m a
box. What is the probability that a box selected at random will meet the guarc.ntee?
2Eight percent of the bolts produced in a certain factory turn out to be defective.
for Find the
probability using Poission distribution, that in a sample of 25 bolts chosen at random,
(i) exactly 3 bolts will be defective,
r
You

(ii) more than 3 bolts will be defective. (Take = 0.135)


oks

27. Twelve percent of the bolts produced in a certain factory turn out to be defective. Find the
eBo

probability that in a sample of 25 bolts chosen at random (i) exactly 3, (ii) more than 3 bolts
will be defective, using Poission distribution. (Take e~^ = 0.049)
28.
A manufacturer of screws known that 4% of his product is defective. If he sells the screws m
our
ad

boxes of 50 and guaranteed that not more than 2 screws will be defective, what is the
probability that a box will fail to meet the quality? (Take e~^ = 0.1353)
29.
Assuming that the chance of a traffic accident in a day in a street of Delhi is 0.001. On how
dY
Re

many days out of a total of 500 days can we expect (i) no accident (ii) more than 3 accidents,
Fin

if there are 1000 such streets in the whole city. (Use e~^ = 0.3679)
30.
There are 500 boxes each containing 1000 ballot papers for election. The chance that a ballot
paper is defective is 0.002. Assuming Poisson's distribution, find the number of boxes
containing at most two defective ballot papers. (Use e~^ = 0.1353)
31.
For a Poisson's distribution, 3P (X = 2) = P(X = 4). Find, P(X = 3) (Take = 0.00248)
^ airline accepts reservations for the seats on a particular flight of a 98 seater aircraft It is
32.

Imown from past experience that 3% of the persons who reserve seats do not turn up and so
the airline has a policy of a allowing 100 persons to book seats on the fight. What is the
probability of more than 98 persons turning up for the flight?
33.
If the probability that an individual suffers from reaction by an injection is — find the
1000'
probability that out of 5000 individuals given that injections (i) exactly 5 will suffer from
reaction (ii) no one will suffer from reaction. (Given e~^ = 0.00672)
POISSON DISTRIBUTION 19.21

ANSWERS
4. 1/e 5. (i) 0.05 (ii) 0.15 6. 60.65%, 9% 7. 0.036

8. (i) 0.1 (ii) 0.122 9. 0.080 10. 0.325

11. (i) 0.0183, (ii) 0.0732, (iii) 0.1464, (iv) 0.909 12. 0.9998

13. (i)369 (ii) 984 14. (i) ^ (ii) 13 ^


15. 0.30325,0.09025 16. 0.019 17. (i) 0.2241 (ii) 0.6470 20. (i) 61, (ii) 9
21. (i) 0.2379 (ii) 0.6825 (iii) 0.3715 22. (i) 50, (ii) 355
r
14
-20 (20) -4
(ii) 0.325
23. I e 24. 8 c 25. 0.43357 26. (i) 0.18
r = 0 r\

27. 0.5835 28. 0.3235 29. (i) 184 days (ii) 9 days 30. 338

ow
31. 0.08928 32. 0.2 33. (i) 0.175 (ii) 0.00672
_HINTS TO SELECTED PROBLEMS
3
5. We have : p = probability that a bulb is defective ~

e
re
»/= = 100 X 0.03 = 3.

rFl
Let X denote the number of defective bulbs in a sample of 100 bulbs. Then,

F
P{X=r) = r = 0,l,2,....
r\

r
05 05
ou
6. We have, p = ,» = 100 :.m = np = X 100 =05
100 100
fo
ks
LetX denote the number of defective articles. Then,
(05)'* e
-w -05
e
oo

P (X = V) = ,r = 0,1,2,...
r! r!
Y

1
B

7. We have, p = , /I = 2000 :.m =np = 2.


1000
re

Let X denote the number of houses having fire during the year. Then,
v{ e
- Ill
2^ c-2
ou

P(X=;-) = ,r=0,l, 2,....


Y
ad

r! r!

25.c -2
Required probability = P (X =5) = 0.036
d

5!
in

1
Re

8. Here, p = ,n = 300=> m=np = 0.6


500
F

1
9. Here, p = n =100 => m=np =1
100'
20. Wehave, p = 0.001, fi= 500 => m=np=05
-03
(05)'' c
=> P(X=r) = ,r=0, 1,2,...
r!

(i) No. of boxes containing no defective out of 100 boxes = 100 x P (X = 0)


(ii) No. of boxes containing at least two defectives =100 [1 - P (X = 0) -P(X = 1)
21. We have, JH =4. So, if X denotes the number of calls per minute, then P (X =r) = r!

nf c
- m

24. Let X denote the number of misprints in a page. Tlien, P (X = r) = ,r = 0,l, 2,...
rl

Now, P (X = 0) = c”
- Ill
- e “ ^ ^ m=4.
19.22 APPLIED MATHEMATICS-XII

4^e-4
P(X=r) =
r!

4\e-^
Hence, Required probability = P (X = 2) = = 8e-4.
2!
25. We have,
4
p = — and n = 100
100

m=np=4
Let X denote the number of defective bulls. Then,
n{ e -m

ow
P(X=r) = ,r=0,l,2,...
rl

^r^-4
rl

Required probability = P (X < 3)

e
= P(X = 0) + P(X = l) + P(x + 2) + P(X = 3)

re
rFl
8
26. Here,« = 25 and p =

F
100

.'. m = np = 2
Let X denote the number of defective bolts. Then, X follows Poisson's distribution such that

r
ou
nf e -m
P(X=r) = fo
ks
rl
2''c-2
=> P(X=r) = ,r = 0,l,2,...
oo

rl
Y
B

(i) Required probability = P (X = 3) = = 0.18


3!
re

(ii) Required probability = P (x > 3)


= l-(P(x<3))
oYu

= l-{P(X = 0) + P(X=l) + P(X + 2)}


ad

= 1- e~'^ + 2e~^ + 2e~^ =0.325


d

28. See Example 9.


in
Re

29. Here, n= 1000,^=0.001


F

m=np=l
LetX denote the number of accident in a day in a street. Then,
n{ e -m
P(X=r) = ,r=0,1, 2,...
rl
-1

=> p(x=r)=- r = 0,l,2,...


rl '
(i) We have,
-1
P(X = 0)=- = c-'
0!

.●. Required number of days = 500 P (X = 0)


=500 X e~^ =500 X 0.3679 =184 days
(ii) We have,
P(x>3)=l-P(X<3)
POISSON DISTRIBUTION 19.23

= 1-{P(X = 0) + P(X=1) + P(X=2) + P(X = 3)}


= l-ie"^ + e"^ + 6-1
2! 3!

. 8
= 1 —-X e -l=l--x 0.03679 =0.0189
3 3

.*. Required number of days =500 x P (X > 3) =500 x 0.0189 = 9 days


31. Here, p = 0.002, n = 1000 :.m=np = 2
Required number of boxes =500 x P (X < 2)
32. We have, m =np= 100 x = 3. Find P(X = 0) + P(X = 1)

ow
e
Fl
re
F
ur
or
k sf
Yo
oo
eB
ur
ad
Yo
d
Re
in
F
CHAPTER

NORMAL DISTRIBUTION

20.1 INTRODUCTION

ow
In the previous two chapters, we have studied discrete probability distributions. In this chapter,
we shall study the probability distribution of a continuous random variable. The normal
distribution or normal probability distribution is the most useful distribution of a continuous

e
variable. It was first discovered by English mathematician De-Moivre in 1733 as a limiting case of

re
rFl
binomial distribution. It was also known to Laplace not later than 1774, but through a historical error

F
it has been credited to Gauss, who first made reference to it in 1809.

r
Since normal distribution is the distribution of the probabilities of a continuous random
ou
fo
variable. Therefore, it becomes very essential to introduce the notion of a continuous random
ks
variable. The same is discussed in the following section.
oo

20.2 CONTINUOUS RANDOM VARIABLE AND ITS PROBABILITY DENSITY FUNCTION


Y
eB

In Chapter 5, we have studied about discrete random variables. A random variable X is said to
be a continuous random variable if it can take all possible values between certain limits.
r
ou

PROBABILITY DENSITY FUNCTION OF A CONTINUOUS RANDOM VARIABLE Let X be a real number.


ad
Y

dx dx
Consider the small interval x -
2 , X + -^ of length dx around the point x. Iff (.v) is a continuous
d
Re
in

function of x such thatf{x) dx. represents the probability that the random variable Xfalls in the interval
F

^ .then f{x) is known as the probability densityfunction of random variable X.


dx
X , .r +
2

Thus, if f (.r) is the probability density function of a random variable X, then


dx dx
P X-—<X<Y +
2 ^ J

If we draw the graph of the function/ (;c) as shown in Fig. 20.1, then/ (x). dx represents the area
bounded by the curve y =f (a), x-axis and the ordinates at the points x - and x + Thus,
dx dx
P X-—<X<x + — = Area enclosed by y=/(.v),.v-axis and the ordinates at the points
^ ■ . dx
2
dx
X - and .Y + —.
2 2
20.2 APPLIED MATHEMATICS-XII

0 .T
X -

NOTE The probdbilihj density function of a random variable X is also known as the density function.

w
If / (.y) is the probability density function of a random variable X, then

F lo
P{a<X<b)-^ f{x)dx.
a

REMARK The probability density function f (x) of a random variable X has thefollozving properties:

e
(i) f (x) > o/for all X s R

Fre
oo
b
(ii) fix) dx = 1, ifX assumes all real values. IfX assumes all values in [a, bl then f f{x) dx = 1. for
- 00
a
r
ILLUSTRATIVE EXAMPLES
You
s

EXAMPLE 1 Verify that f (x) = -^ . 1 +^ ^ V - CO < .Y < 00 /s the probability density function.
ook
eB

SOLUTION For / (.r) to be a probability density function, it should satisfy the following
conditions:
our
ad

00

(i) / (.v) > 0, for all x eR (ii) fix) dx = 1


- CO

Clearly, / (.v) > 0 for all xsR i.e. for - 00 < .T


dY

< CO.
Re

00

'f 1 1 1 1
-ICO

Now, fix)dx= f =l yd.Y = -K tan ^ X


Fin

— 00 — 00
^ 1 + .Y“ K 1 + J— CO
— CO

1 1
-1
- tan ^ (- oo) I =
71 TC
tan CO = 1.
7T K 2 I 2J
Hence,/ (.y) is a probability density function.
EXAMPLE 2 Prove that
(1 .2
, if 0 < X < 1

fix) =
|[x-2-3(.y-1)2],//1<.y<2
1 [y2 _ 3 (Y -1)2 + 3 (Y -2)2], (f2<x<3
is the probability density function of a continuous random variable X in the range (0,3).
NORMAL DISTRIBUTION 20.3

SOLUTION For f (.r) to be the probability density function , it should satisfy the following
conditions
3

(i) f{x) > 0 for nil x e (0, 3) (ii) /(.v)d.v = l.


0

We shall now verify these two properties,


(i) For 0 < A' < 1, we have

fix) M ^ ^ fo*" all ^ ^ (0,1]

w
Fori < A <2, we have

/(.V) = i [a^ - 3 (a -1)2] =1 (- 2a2 + 6a ~ 3)


/' (a)=1(-4a + 6) = -2a +

o
3

e
re
^
- 3 => A < —
3
/'(a)>0=> -2a+3>0=> -2a>

Frl
2

F
Thus, / (.a) is increasing on [1,3/2] and decreasing on [3/2,2].
1 1
ou
We have, /{!)=- and /(2)=-

r
Since / (a) is increasing on [1, 3/2] and / (1) = —. Therefore,
so
kf
oo
/ (a) >/ (1) for all A E [1, 3/2) => / ^ 2 ^‘^**^*^ ^
Y

1
B

Also, / (.a) is decreasing on [3/2, 2) and / (2) =

/ (a) >/(2) for ail ae[3/2,2)=> /(A)>iforall ae[3/2,2)


re
oY
u

Hence,/ (a) > 0 for all a e [3/2, 2).


ad

For 2 < A < 3, we have


d

fix) = i [a2 - 3 (a -1)2 + 3 (a - 2)2] = 1 [x2 6a+ 9] =1(a-3)2


in
Re

fix) > 0 for all A E [2, 3).


F

Hence, / (a) > 0, for all a e (0, 3)


3
1 1 1
(ii)
0
fix)dx = J - a2 dx +
0 ^ 2
[a2 - 3 (a -1)2] dx +
6'*’ -3(a-1)2+ 3(A-2)2]rfA
2 n3
3 .3

/ (A) dx = 2
^-(A-1)^
3
JO -12
0

1 1 /27 /8 1 2 :
-1 + - —-8 + 1 --1 + 0 1 1

u 3 2 3 ^3 6 3 6
0

Hence, / (.a) is the probability density function.


EXAMPLES The probability density function f (.a) of a continuous random variable X is given by
/(a)=A.a2,0<a<1
Find: (i) A (ii) P(0.2<X<05) (Hi) P(X<0.3)
20.4
APPLIED MATHEMATICS-XII

SOLUTION (i) Since / (.v) is the probability density function of random variable X.
-|1
■) A
/ (x) dx = 1 => \ A x~dx = l=^ A — = 1=>—= 1=>A = 3
0
3
0 0

0.5 nO.5
0-5 [3
(ii) P(0.2<X<05) = f{x)dx= { Ax^dx = A — = -(0.2)^^] =-3 [0.125 -0.008]
’’ 3 3
0.2 0.2 L 0.2

= 0.125-0.008 = 0.117. [■■■ ^ = 3]


(iii) P(X<0.3)=P(0<X<0.3)
0.3 i0.3
A 2 J A 3
Ax dx = —
3
X
= [(0.3)^-0^1 = 0.027. I-.- 71 = 3]
0 Jo

w
EXAMPLE 4 The amount of bread (in hundreds of kilograms) X that a certain bnkenj is able to sell in a
day is found to be a numerical valued random phenomenon with a probability function spiecified by the
probability density function f (.y), given by

Flo
Ay, for 0 < X < 5
/ (.y) = ^ /I (10 - X), for 5 < X < 10

e
re
0, otherunse

F
Find the value of A such that f (x) is the probability density function. What is the probability that the
number of kilograms of bread that will be sold tomorrow is
ur
r
(i) more than 500 kgs (ii) less than 500 kgs (iii) between 250 kgs and 750 kgs? fo
SOLUTION For / (.y) to be the probability density function, we must have
ks
00

/ (x) dx = 1
Yo
oo

— 03

0 5 10
eB

=>
/ (.y) dx + f / (x) dx + f / (x) dx + f / (x) dx = 1
— X 0 5 10
ur

0 5 10 X

j 0dx+ Ax dx + J (10 - .y) dx +


ad

0 dx = 1
Yo

CO 0 5 10

r,.2f .2
nlO
25 25'i
d

X X
A 4-/1 lOx - —
Re

= 1=> A+A = 1=> /I =-


in

2 2 2 2 ; 25
0 J5
F

X
for 0 < X <5
25'

/(-V) = ~ (10 - x), for 5 = X < 10


25

0, otherwise

Now,
X 10 X 10 X
1
(i) Required probability = P (X >5) = f/(x) dx = f/(.Y)dx4- f/(x) dx = (10-x) dx+ Odx
5 10 5^5 10

1
2-jlO
25 ^ 1 25 1
lOx-- - (100-50)- 50- —
25 2 25 2 J 25 I 2 2
5
NORMAL DISTRIBUTION 20.5

5 0 5 0 5

(ii) Required probability = P{X <5) = f{x)dx = f{x) dx + f{x) dx = 0 dx + — dx


25
- X — cc 0 - X 0

' ..2 T
1
J_ - 1
25 2 25 I 2 J " 2

w
Jo
75 75

(iii) Required probability =P(25<X<75)= f{x)dx- f {x) dx + f (x) dx


2.5 25 5

e
75
5 75
1 1 ,Y 1 X^

e
- dx + — (10 -x) dx = H lO.Y-
2

or
25 2 25
25
5 25 J25 J5

r
2.5

(75)2

F
1 r25 6.75 1 25
75- -50-
25 L 2 2
+ —
25 2 2)

oF
ul
1 18.75 _ 3
(9.375 + 46.875 - 375) =
25 "4

rs
25

ko
20.3 NORMAL DISTRIBUTION
The normal distribution is a continuous distribution and it was first discovered by the French
mathematician De Moivre in 1733 as the limiting case of the binomial distribution under the of
o
following conditions:
Y
(i) n, the number of trials is indefinitely large i.e. n 00
B

and, (ii) neither p (probability of success in a trial) nor q (probability of failure in a trial) is very
Y

small.
er

The mathematical derivation of this result is beyond the scope of this book.
u

The second condition means that the probability of success is close to In fact, the histogram of
od
ad

the binomial distribution approaches to the normal probability curve (Fig. 20.2) imder these
in

conditions.
Re
F

v=ji-5o x=h~2<3 x-h-(j l=ti .v=n*o x=n+2o .r=n+3o

Fig. 20.2

DEFINITION A random zmriable X is said to have a normal distribution with parameters p (called
"mean") and (called "variance"), if its probabiliti/ distribution fwiction is given by
(y-p)2
1
/(.V) = 2a^ ,-oo<.Y<oo, - oo<p <», a>0.
a

where ^/2n= 25066 and e= 2.7183.


A random variable is said to be a normal variate if it follows a normal distiibution.
If a random variable X follows a normal distribution with mean p and variance a , then we say
2 2
that X is a normal variate with mean p and variance and we write X ~ iV (p, a ).
20.6 APPLIED MATHEMATICS-XII

The graph of / (a*) is a bell shaped curve as shown in Fig. 20.2 such that
P(p-cr<X<|j+a) = Area enclosed by y =/ (x), .Y-axis and the ordinates at p - a
and p + a
M + a
1
p + a _ i x - M_)^
f{x)dx^ c fr.Y = 0.6826.
a
p-a p “ a

P(p-2a<X<p + 2a) = Area enclosed by y = / (x), .v-axis and the ordinates


at p - 2a andp + 2a

ow
p + 2o p + 2a (.y-p)2

p - 2a
/w*=-4r
a V27T I 2ct^ dx = 0.9544
p - 2a

and.

e
P {p - 3a <X <p + 3a) = Area bounded by the curve y =/(x), x-axis and the

re
ordinates at p - 3a and p + 3a

Frl
F
p + 3o p + 3a
1
fix) dx = c 2a^ fr.v = 0.9973.
a
ou
p - 3a

or
p - 3a

20.4 PROPERTIES OF THE NORMAL DISTRIBUTION AND NORMAL PROBABILITY CURVE


kfs
The probability distribution function of a normal variate with mean p and variance a^ is given
oo

by
Y
B

1
fix) = 2a^ - CO < X < 00
re

The curve y =/ (.y) is known as the normal probability curve and it has the following properties:
oYu

(i) The curve is bell shaped and it is symmetrical about the line x = p i.e. if the curve is folded
ad

along the vertical line x = p, the halves coincide.


(ii) Mean, median and mode of the distribution coincide.
d
in
Re

(iii) As X increases numerically,/(x) decreases rapidly, the maximum value of/(x) occurs at
x = p.
F

(iv) X-axis is an asymptote to the curve i.e. it touches the curve at x = ± co.
(V) = 0 and P2 - 3.
(vi) Odd order central moments are zero and even order central moments are given by
P2r = 1-35 ... (2r -1) a^, r = 0,1,2,....
(vii) The area under the normal curve is distributed as follows :

(a) Mean ± acovers 619.27%area, i.e. P(p - a <X<p + a) = 0.6827.So, 34.135% area will
lie on either side of the mean,
(b) Mean ± 2a covers 95.45% area, i.e. P (p - 2a < X < p + 2a) = 0.9545.
(c) Mean ± 3a covers 99.73% area, i.e. P (p - 3a < X < p + 3a) = 0.9973.
(ix) The area
bounded by the curve y=/(x) and X-axis is one unit i.e.

j fix) dx = 1.
- CO
20.7
NORMAL DISTRIBUTION

20.5 STANDARD FORM OF THE NORMAL DISTRIBUTION

The probability distribution function f{x) of a normal variate X with mean |j and standard
deviation a is given by

/(-v)=^-j=
2ct-
e , - 00 < 'X < X
Jlii
a

Clearly, f (.t) gives the ordinates of the curve i/ = / (.v) for different values of .y. However, we are
usually interested in the area under the normal curve i/ = / (x) instead of its ordinate, because the

ow
area under the normal curve determines the probability that the normal variate lies between two
given numbers.
The normal curve y=f{x) depends upon mean (=|i) and standard deviation (=<t) and for
different values of p and a, we obtain different normal curves. This means that if we want to

e
tabulate the area under normal curves, we will have to prepare separate tables for each pair of

re
values of p and o. This is an impossible task. To resolve this difficulty, we first standardize the
given normal variate X by using the transformation

Frl
F
i.e. X=p + aZ
CT
ou
This transformation transforms the probability den ity function of X to the probability density

sor
function of variable Z, given by

1
kf
MZ) = -p= e 2 , - CO < Z < X
oo
Y

The proof of this is beyond the scope of this book.


B

The random variable Z has mean equal to zero and variance unity and is known as the standard
normal variate, written as Z ~ N(0,1).
re
oY

The probability density function of standard normal distribution is, therefore, given by
u
ad

2 , —X < Z <X
d
in

The advantage of the above function is that it does not contain any parameter wliich enables us
Re

to compute the area under the normal probability curve by making use of standard tables.
F

The graph of the above function is as given in Fig. 20.3.

z = -3 z = -2 : = -l : = 0 z= 1 J = 3

Fig. 20.3

20.6 AREA UNDER THE STANDARD NORMAL PROBABILITY CURVE

The probability density function of the standard normal variate Z is given by


Z^
, -X <Z <x
20.8 APPLIED MATHEMATICS-XII

Zl
The curve y = ^{Z) is known as the standard normal curve. The definite integral f isf(Z) dZ is
0

known as the normal probability integral and gives the area under the standard normal curve
between the ordinates Z = 0 and Z = Zj. These areas have been tabulated for different values of
Z|, at intervals of 0.01 and are shown in the table at the end of the book. Since the standard
normal curve is symmetrical about Z = 0, therefore areas under the normal curve are tabulated
only for positive values of Z. The total area under the normal curve is equal to 1 square unit.
Since P (0 < Z < Z-j) is equal to the area under the normal curve between the ordinates at Z = 0
and Z = Z^. Therefore,
Zi
P(o<z<Zi)= 4)(Z)rfz

low
0

2/2

ee
j = -3 2 = -2 rF 1 = 0 :=1 2=2 2=3

Fr
Fig. 20.4

Also, from the symmetry of the standard normal curve about the line Z =0, we obtain the
following results: for
(I) If Z^ > 0, then P (0 < Z < Z|) is equal to the area enclosed by the standard normal curve
u
between the ordinates Z = 0 and Z = Z-\, and it is obtained from the table given in the end
ks
Yo
of the book.
o

(II) P{Z <0) = P{Z >0)=05


Bo
re
ou
ad

2=0
Y

Fig. 20.5
(III) For any Z^ > 0, we have
nd
Re

P(-Z|<Z<0)=P(0<Z<Zi).
Fi

-2] 2=0

Fig. 20.6
(IV) For any Zj » 0, we have
P(Z>2^)=05-P(0<Z<Zi)

2=0

Fig. 20.7
NORMAL DISTRIBUTION 20.9

(V) IfZi>0,then
P (Z < Z|) = P (Z < 0) + P (0 < Z < Z^)
P (Z < Z;i) = 05 + P (0 < Z < Zi)

2=0 2=2^
Fig. 20.8

(VI) If Zi < 0, then

w
P(Z<Zi)=P(Z>|Zi|) = 05-P(0<Z<|Zi|)

P(z^l2ll)

Flo
e
re
2l 2=0 ‘■I

rF
Fig. 20.9

(VII) If Zp Z2 > 0 such that Z^ < Z2, then


ur
P (Zi < Z < Z2) = P (0 < Z < Z2) - P (0 < Z < Zi). fo
ks
Yo
oo
eB

2=0 2-] 22
Fig. 20.10
ur

(VIII) If Zi < 0 and Z2 > 0, then


ad
Yo

P(Zi <Z <Z2) =P(Zi <Z <0) + P(0<Z <Z2)


= P(0<Z<|Zi|) + P(0<Z<Z2)
nd
Re
Fi

-1 2=0 IZ]IZ2
Fig. 20.11

(IX) If Z-, < 0, Z2 < 0 and Z] < Z2, then


P (Zt < Z < Z2) = P (0 < Z <1 Zjl) - P (0 < Z < I Z2I)

2i 22 2=0 1221 2]
Fig. 20.12
20.10 APPLIED MATHEMATICS-XII

ILLUSTRATIVE EXAMPLES

LXAMI'U-: i Let Z ben standard normal variate. Find:


(i) P(0<Z<1.42) (ii) P(-0.73<Z<0) (iii) P(Z> 1.13)
(iv) P(Z>-1.28) (v) P (0.81 <Z <1.94) (vi) P(-0.46<Z <2.21)
(vii) P(\Z\<05) (viii) P(|Z[>05) (ix) P(|Z-1|<05)
SOLUTION
(i) P (0 < Z < 1.42) is equal to the area under the standard normal curve between
the ordinates Z = 0 and Z = 1.42. Thus, in the table given in the end of the book, proceed down
the column marked Z until entry 1.42 is reached, and then proceed right to column marked 2.
The entry is 0.4222. Hence, P (0 < Z < 1.42) = 0.4222

w
F lo
2=0 2=1.42

Fig. 20.13

(ii) By symmetry of the standard normal curve about Z = 0, we have

ee
P (- 0.73 < Z < 0) = P (0 < Z < 0.73)

Fr
= Area under the standard normal curve between Z = 0 and Z = 0.73
= 0.2673.
for [From the table]
ur
s
ook
Yo
eB

2=-0.73 2=0

Fig. 20.14

(iii) P (Z > 1.13) = (Area to the right of Z = 1.13)


our
ad

= (Area to the right of Z = 0) - (Area between Z = 0 and Z = 1.13)


= P(Z >0)-P(0<Z <1.13)
Y

= 05-P(0<Z <1.13)
Re
nd

= 05-0.3708 = 0.1292 [From the table]


Fi

2=0 2=1.13

Fig. 20.15

(iv) P(Z >-1.28)


= Area bounded by the standard normal curve on the right of Z = -1.28
= (Area between Z = -1.28 and Z = 0) + (Area to the right of Z = 0)
= P(-1.28<Z<0)+ P(Z>0)
= P(-1.28<Z<0) + 05 [●●● P (Z > 0) = 05]
= P(0<Z<1.28) + 05
= 0.3997 + 05 =0.8997 [From the table]
20.11
NORMAL DISTRIBUTION

z=-1.28 z=0
Fig. 20.16

(v) P(0.81 <Z <1.94)


= Area bounded by the standard normal curve between Z = 0.81 and Z = 1.94
= (Area between Z = 0 and Z =1.94) - (Area between Z = 0 and Z = 0.81)
= P(0 < Z < 1.94) - P (0 < Z < 0.81)

w
= 0.4738 - 0.2910 = 0.1828 [From the table]

o
e
2=0 \ 2=1.94

re
rFl Fig. 20.17
2=0.81

F
(vi) P(-0.46<Z <2.21)
= (Area between Z = - 0.46 and Z = 0) + (Area between Z = 0 and Z = 2.21)

r
ou
sfo
= (Area between Z = 0 and Z = 0.46) + (Area between Z = 0 and Z = 2.21)
= P (0 < Z < 0.46) + P (0 < Z < 2.21)
k
= 0.1772 + 0.4864 = 0.6636. [From the table]
oo
Y
r eB

2=-0.46 z=0 r=2.21


You

Fig. 20.18
ad

(vii) P(|Z|<05)
d

= P(-05<Z <05)
Re
in

- Area between Z = - 05 and 2 = 05


F

= 2 (Area between Z = 0 and Z = 05)


= 2P(0<Z <05)
z=-0.5 2=0 2=0.5
= 2(0.1915) [From the table]
Fig. 20.19
= 0.3830.

(viii) P(|Z| >05)


= P (Z > 05 or Z < - 05)
= P(Z >05) + P(Z <05)
= (Area to the right ofZ=05) + Area to the left ofZ=-05)
= 2 (Area to the right of Z = 05)
= 2[P(Z >0)-P(0<Z <05)1
= 2 [05 - 0.1915] = 2 (0.3085) = 0.6170. 2=A).5 z=0 2=0.5
Fia. 20 20
20.12 APPLIED MATHEMATICS-XII

ALITER P(|Z|>05)=1-P(|Z|<05)
= 1 -0.3830 [See (vii)]
= 0.6170

(ix) P(|Z-1|<05) = P(-05<Z-1<03)


= P(05<Z<15)
= P{0<Z<15)-P{0<Z<05)
= 0.4332-0.1915 [See Table]
= 0.2417.

ALGORITHM TO FIND THE PROBABILITY THAT A NORMAL VARIATE X LIES BETWEEN .v AND .t,
STEP 1
Obtaht the mean and standard deviation of the given normal variate. Let^ be the mean and g be

w
the standard deviation of the given normal variate X.
X-^i
STEP 11 Define the standard normal variate Z given by Z =

F lo
a

STEP 111 Obtain the values ofZ corresponding toX = and X = X2- Let the values ofZ be Zj and Z2

ee
respectively.

Fr
STEP IV Use the result P (xj <X <.V2) = P (Z^ <Z <Z2)
STEP IV Compute P (Zj < Z < Z2) to get the required probability.
Following examples illustrate the above algorithm. for
ur
EXAMPLE 2 IfX is a normal variate with mean 12 and S.D. 4. Find:
ks
(i) P (X > 20) (ii) P(X<20) (iii) P(0<X<12)
Yo

SOLUTION We have, p = 12, a = 4.


oo

X-p
Z=^
eB

z =
a 4

(i) When X = 20, we obtain


r

Z=^ = 2.
ou
ad

4
Y

P (X > 20)
= P(Z>2)
nd
Re

= P (Z > 0) - P (0 < Z < 2)


Fi

2=0 2=2
= 05 - P (0 < Z < 2) [vP(Z >0)=05] Fig. 20.21
= 05 - 0.4772 [See table]
= 0.0228

(ii) When X = 20, we obtain

z=^ = 2.
4

P (X < 20)
= P (Z < 2)
= P (Z < 0) + P (0 < Z < 2)
= 05 + P (0 < Z < 2)
= 05 + 0.4772 [See Table] 2=0 2=2
= 0.9772. Fig. 20.22
20.13
NORMAL DISTRIBUTION

12
^ 0-12
= - 3; When X =12, Z=—-
(iii) WhenX = 0, Z=—^—
P(0<X<12)
= P (- 3 < Z < 0) 2=-3 2=0 2=3

[See Table] Fig. 20.23


= P (0 < Z < 3) = 0.49865
EXAMPLl: 3 }fX is a Honml variate with wean 30 and S.D. 5. Find:
(i) P(26<X<40) (ii) P (X > 45) (iii) P(jX-30|>5)
SOLUTION Wehave,)a = 30 and u=5.
Let Z be the standard normal variate. Then,

w
X-30
z=^ Z =
5
a

^ 26-30 = -0.8
(i) When X = 26, we obtain : Z = —-—

o
e
40-30 ,,

re
When X = 40, we obtain: Z = —-— = 2.

Frl
F
P(26<X<40)
= P (- 0.8 < Z < 2) 2=-0,8 2=0 z=2
ou
= P (- 0.8 < Z < 0) + P (0 < Z < 2) Fig. 20.24

or
= P (0 < Z < 0.8) + P (0 < 2 < 2) kfs
= 0.2881 + 0.4772 = 0.7563 [See Table]
(ii) When X = 45, we obtain
oo

Z=i^ = 3
Y

5
eB

P (X > 45)
= P(Z >3) 2=0 2=3
ur

= 05-P(0<Z<3) Fig. 20.25


oY

= 05-0.49865 [See Table]


ad

= 0.00135
d

(iii) P(|X-30|>5)
in

= 1-P(|X-30|<5)
Re

= l-P(30-5<X<30 + 5)
F

= 1-P(25<X<35)
25-30 35-30
Now, X = 25 => Z = = -1 and, X = 35 => Z =
5

P(|X-30|>5)
= 1 -P(-l <Z <1)
= 1-2.P(0<Z<1)
= 1 - 2 X 0.3413
= 1 -0.6826 2=-l 2=0 2=1
Fig. 20.26
= 0.3174

(iv) We have.
P(|X-24l<8) = P(24-8<X<24 + 8)=P(16<X<32)
16-30 ^ 32-30 = 0.4
Now, X=16=> Z = = - 2.8 and, X = 32 ^ Z —
5 5
20.14
APPLIED MATHEMATICS-XII

P(|X-24[<8)
= P(-2.8<Z<0.4)
= P (- 2.8 < Z < 0) + P (0 < Z < 0.4)
= P (0 £ Z < 2.8) + P (0 < Z < 0.4) z=-2.8 2=0 2=0.4

= 0.4974 + 0.1554 = 0.6528 Fig. 20.27

EXAMl’Lt 4 Let X denote the number of scores in a test. IfX is normally distributed with mean 100 and
standard deviation 15, find the probability that X does not exceed 130.
SOLUTION
It is given that X is normally distribnted with mean p = 100 and standard deviation
CJ
-15. Let Z be the standard normal variate. Then,
X-100
z =
a 15

w
130-100
When X = 130, we obtain : Z = = 2
15
2=0 2=2

Flo
Required probability = P (X < 130) = P (Z < 2) Fig. 20.28

= P (Z < 0) + P(0 < Z < 2) = 05 + 0.4772 = 0.9772.

ee
EX.AMl’LE 5 Students of a class were given a mechanical aptitude test. Their marks were found to be

Fr
normally distributed lo ' mean 60 and standard deviation 5. What percent of students scored
(i) more than 60 marks ? (ii) less than 56 marks ? (Hi) behueen 45 and 65 marks ?
for
ur
SOLUTION Let X denote the marks scored by the students. ThenXis normally distributed with
mean p = 60 and standard deviation a = 5. Let Z be the standard normal variate Then
^_X-6Q
ks
Yo

(7 5
oo

60-60
(i) When X = 60, we obtain : Z = = 0.
eB

P(X>60)=P(Z>0)=05 2=0
r

Thus, 50% of the students scored more than 60 marks. Fig. 20.29
ou
ad

56 - 60
(ii) When X = 56, we obtain : Z = = -0.8
Y

P(X<56)
nd
Re

= P(Z<-0,8)
= P (Z > 0.8)
Fi

2=-0.8 2=0
= 05 - P (0 < Z < 0.8) = 05 - 0.2881 = 0.2119 Fig. 20.30
Thus, 21.19% of the students scored less than 56 marks.
45-60
(iii) When X = 45, we obtain : Z = = - 3
5
65 — 60
When X = 65, we obtain : Z = = 1
5

P(45<X<65)
= P(-3<Z<1)
2=-3 2=0 2=1
= P(-3<Z<0) + P(0<Z<1) Fig. 20.31
= P(0<Z <3) + P(0<Z<l)
= 0.4986 + 0.3413 = 0.8399
Thus, 83.99% of the students scored between 45 and 65 marks.
20.15
NORMAL DISTRIBUTION

LX AM VI V (. The marks obtained by a group of students in a final examination in Statistics have a mean
of 58 and standard deviation of 20.5. Assuming that these marks were approximately normalhj
distributed, what percentage of the students can be expected to have obtained marks from 60 to 69, both
inclusive ?

SOLUTION Let X denote the marks obtained by the given group of students. Then, X is
normally distributed with meanp =58 and standard deviation o = 85.
Let Z denote the standard normal variate. Then,
X-58
=> Z =
CT 85
60-58
VVlien X = 60, we obtain : Z = = 0.24
85 0

w
69-58 2=0.24
When X = 69, we obtain : Z = = 1.29
85 Fig. 20.32

P(60<X<69)

Flo
= P(0.24<Z <1.29)
= P (0 < Z <1.29) - P (0 < Z < 0.24) = 0.4015 - 0.0948 = 0.3067

ee
Thus, 30.67% of the students obtained marks from 60 to 69, both inclusive.

Fr
LXAMi’i [; 7 It is known from the past experience that the number of telephone calls made daily in a
certain community between 3 PM and 4 P.M has a mean 352 and a standard deviation of 31. What
for
ur
percentage of the time will there be more than 400 telephone calls made in this community between 3 P.M
and 4 P.M.
s
SOLUTION Let X denote the number of calls made daily in a certain community between 3 PM
ok
Yo

and 4 PM. Then, X is normally distributed with mean p = 352 and standard deviation ct = 31.
Bo

Let Z be the standard normal variate. Tlien,


X-352
re

Z =
a 31
ou

400 - 352
ad

When X = 400, we obtain : Z = = 1548 «155


31
Y

2=0 2=1.55
P(X>400) Fig. 20.33
nd

= P(Z >155)
Re

= 05 - P {0 < Z < 155) = 05 - 0.4394 = 0.0606


Fi

Thus, in (0.0606 x 100) = 6.06% of the time there will be more than 400 telephone calls made m
the given community betv\'een 3 PM and 4 PM.
\ win \ The lifetimes of certain kinds of electronic devices have a mean of 300 hours and standard
deviation of 25 hours. Assuming that the distribution of these lifetimes, which are measured to the nearest
hour, can be approximated closely with a normal curve,
(i) find the probability that any one of these electronic devices will have a lifetime of more than 350
hours,

(ii) what percentage will have lifetime of 300 hours or less?


(Hi) what percentage zvill have lifetimes from 220 to 260 hours?
SOLUTION Let X denote the lifetimes of the given electronic devices. Then X is normally
distributed with mean p = 300 and standard deviation a = 25.
Let Z be the standard normal variate. Then,
20.16
APPLIED MATHEMATICS-XII

Z=^ ^ _ X - 300
c 25

(i) When X = 350, we obtain : 2 = — =2


25

P (X > 350)
= P(Z>2)
= 05 - P (0 < Z < 2) = 05 - 0.4772 = 0.0228
300 - 300
(ii) When X = 300, we obtain : Z = = 0
25

P (X < 300) = P (Z < 0) = P (2 > 0) = 05

w
Thus, 50 % of the electronic devices will have lifetimes of 300
hours or less,
r=-3.2 z=-1.6z=0
220 - 300 Fig. 20.35

Flo
(iii) When X = 220, we obtain : Z = = -3.2
25

e
260 - 300
When X = 260, we obtain : Z =

re
= -1.6
25

F
P(220<X<260)
= P(- 3.2<Z <-1.6)
ur
r
z=-3.2 z=-l,6z=0
= P(1.6<Z<3.2) fo Fig. 20.36
= P (0 < Z < 3.2) - P (0 < Z <1.6) = 0.4993 - 0.4452 = 0.0541
ks
Thus, 5.41% of the electronic devices will have lifetimes from 200 to
Yo
260 hours.
oo

EXAMPLE 9 Suppose the temperature during June is normally distributed ivith mean 20°C and
B

standard dez’iation 3.33°C. Fmd the probability p that the temperature is between 21.11°C and 26.66°C.
re

SOLUTION Let X denote the temperature in June. Then, X is a normal variate with mean
p = 20°C and standard deviation
- 3.33®C. Let Z be the standard normal variate. Then,
a
u
ad

^_X-20
Yo

a 3.33

21.11 -20 1.11


When X = 21.11°C, we obtain : Z =
d

= 0.33
Re

3.33 3.33
in

26.66 - 20 6.66
When X = 26.66°C, we obtain : Z =
F

3.33 3.33
Required probability :=0 ■2
= P (21.11 <X <26.66) 2=0.33

= P(0.33<Z <2) Fig. 20.37

= P(0<Z<2)-P(0<Z<0.33)
= 0.4772-0.1293 = 0.3479
[See Table]
EXAMPLE 10 If the diameters of ball bearings are normally distributed with mean 0.6140 inches and
standard deviation 0.0025 inches, determine the percentage of ball bearings with diameters (i) between
0.610 and 0.618 inches inclusive, (ii) greater than 0.617 inches, (iii) less than 0.608 inches (iv) equal to
0.615 inches.

SOLUTION Let X denote the diameters of ball bearings. ThenX is a normal variate with mean
|4 - 0.6140 inches and standard deviation a = 0.0025 inches.
20.17
NORMAL DISTRIBUTION

Let Z be the standard normal variate. Then,

^ X-|i_ -0.6140
a ^ 0.0025
0.610 - 0.6140
= -1.6
(i) When X = 0.610, we obtain : Z = 0.0025

0.618-0.6140
When X = 0.618, we obtain : Z = = 1.6
0.0025

.-. P (0.610 <X< 0.618)


z=-1.6 z=0 2=1.6
= P(-1.6<Z <1.6) [By symmetry] Fig. 20.38
= 2.P(0<Z<1.6)

w
= 2 X 0.4452

= 0.8904 [See Table]

F lo
Thus, the percentage of ball bearings with diameters between 0.610 and 0.618 inches is
0.8904 X 100 = 89.04.

ee
0.617-0.6140

Fr
= 1.2
(ii) When X = 0.617, we obtain : Z = 0.0025

P(X> 0.617) for


ur
= P(Z >1.2)
= P(Z >0) -P(0 <Z <1.2) 2=0 2=1.2
s
ook

Fig. 20.39
Yo

= 05-0.3849=0.1151

Thus, the percentage of ball bearings with diameters greater than 0.617 inches is
eB

0.1151 X 100 = 1131


0.608-0.6140
r

= -2.4.
(iii) When X = 0.608, we obtain : Z =
ou
ad

0.0025

P (X < 0.608)
Y

= P(Z <-2.4)
Re
nd

= P (Z > 2.4) z=-2.4 2=0


Fi

Fig. 20.40
= P(Z>0)-P(0<Z<2.4)
= 05 - 0.4918 = 0.0082.

Thus, the percentage of ball bearings with diameters less than 0.608 inches = 0.0082 x 100 - 0.82
0.615-0.6140
= 0.4
(iv) When X = 0.615, we obtain : Z 0.0025

P(X= 0.615)

= P(Z =0.4)
2=0 2=0.4

= 0.1554 Fig. 20.41

Thus, the percentage of ball bearings with diameter equal to 0.615 inches = 0.1554 x 100 =1534.
20.18
APPLIED MATHEMATICS-XI!

I \AMiM.l; 11 The mean inside diameter of a sample of500 washers produced by a machine is 5.02 mm

and Hie standard deviation is 0.05 mm. The purpose for which these ivashers are intended allows a
maximum tolerance in the diameter of 4.96 to 5.08 mim otherwise the washers are considered defective.
Determine the percentage of defective zvashers produced by the machine, assuming the diameters are
normally distributed.
SOLUTION Let X denote the diameters of washers. Then, X is a normal variate with mean
—5.02 mm and standard deviation a = 0.05 mm. Let Z be the standard normal variate. Then
X-5.02
Z = => 2 =
a 0.05

WhenX = 4.96, we obtain :


4.96-5.02
= -1.2

ow
0.05

WhenX =5.08, we obtain ;


_ 5.08-5.02 = 1.2 2=-1.2 2=0 2=1.2
0.05 Fig. 20.42

e
Probability that a washer is non-defective

Fl
re
= P (4.96 <X <5.08)

F
= P(-1.2<Z <1.2)
ur
= 2. P (0 < Z <1.2) = 2x 0.3849 = 0.7698
Thus, the percentage of non-defective washers = 0.7698 x
Hence, the percentage of defective washers = (100 -77) = 23. or
100 = 76.98 a 77.
f
ks
I.X
\MPl.E 12 If a set of measurements are normally distributed, zvhat percentage of these differfrom the
Yo

mean by (i) more than half the standard deviation, (if) less than three quarters of the standard deviation ?
oo

SOLUTION LetXdenote the measurements. Then X is given to be a normal variate with mean p
B

(say) and standard deviation cr (say). Let Z be the standard normal variate. Then,
X
re

Z =
a ...(i)
u
ad

We have to find the percentage of measurements which differ from the mean by more than half
Yo

the standard deviation i.e. |X -p| > —.


d

Now,
Re
in

a (T G a
<=>X- p < — or X P >— X<p - — or X >p + —
F

2 2 2
G
p — —p
1
When X = p - —, we obtain : Z = 2
2' G 2
a

When X =
O’
we obtain : Z = P + 2-^ 1
O 2 — 1 2=0 1
' ~2 ^"2
pf|X-p|>^
G O
= P X<p - — or X > p + — Fig. 20.43
z 2 2
f 0l f fT^
= P X<p-- +p x>p+-
V 2) \ 2,
1 n
=p z<— +p z>-
2) 1)
20.19
NORMAL DISTRIBUTION

= 2P Z>-]=2P{Z>0)-p{o<Z
2J V
<\2 = 2 [05 - 0.1915] = 2 [0.3085] = 0.6170

Hence, required percentage = 0.6170 x 100 = 61.7


(ii) We have to find the percentage of measurements which differ from the mean by less than
three quarters of the standard deviation.
3a 3a 3 3o 3o
i.e. <X- a [a - — <X <u + —
4 4 4
3a
3a
When X = u + — ,
X-p "" 4 3
we obtain : Z = 4
4 o a

3a

ow
3a X-p 4 3
When X = p - — , we obtain : Z = a a 4

3a
...

e
Fl
re
F
z=-0.75 z=0 z=0.75
= P(|Z|<0.75) Fig. 20,44
= 2 P (0 < Z < 0.75) = 2 X 0.2734 = 05468
ur
Hence, required percentage = 05468 x 100 =54.68
or
sf
EXAMPLE Suppose the zoaist measurements X of 800 girls are normally distributed zvith mean 66
cm and standard deviation 5 cm. Find the number N of girls ivith zvaists (i) betzoeen 65 and 70 cm, (u)
k
Yo

greater than or equal to 71 cm.


oo

SOLUTION It is given that X is a normal variate with mean p = 66 cm and standard deviation
B

a = 5 cm. Let Z be the standard normal variate. Tl-ien,


e

X-66
z=^ => Z =
ur

a 5
ad

65 - 66
= -0.20 and, X=70 => Z
Yo

(i) X = 65 => Z =
5

P(65<X<70)
d
Re

= P (- 0.20 < Z < 0.80)


in

= P (- 0.20 <Z<0) + P(0<Z< 0.80)


F

0.8
= P (0 < Z < 0.20) + P (0 < Z < 0.80) -0.2
Fig. 20.45
= 0.0793 + 0.2881=0.3674

Hence, N = 800. P (65 < X < 70) = 800 x 0.3674 = 294.


72-66
= 1.2
(ii) WhenX =72, we obtain : Z =—-—

P(X>72)
= P(Z £1.2) 2=0 z=1.2
= P (Z >0) - P (0 <Z <1.2) =05-0.3849 = 0.1151. Fig. 20.46
Hence, N = 800 P (X > 72) = 800 x 0.1151 = 92.
mean 68.0 inches and
EXAMPLE 14 If the heights of 300 students are normalhj distributed zvith
standard deviation 3.0 inches, hozv many students have heights (i) greater than 71 inches, (ii) less than
equal to 64 niches, (Hi) betiveen 65 and 71 inches inclusive.
20.20
APPLIED MATHEMATICS-XII

SOLUTION Let X denote the height. It is given that X is a normal variate with mean g = 68.0
inches and standard deviation a
- 3.0 inches. Let Z be the standard normal variate. Then,
7 _ X - g _ ^_X-68
a 3

72-68 4
(i) X=72=> Z = -=1.33
3 3

P(X>72) 2=0 1.33

= P(X> 1.33) = 05 - P (0 < X <1.33) = 05 ~ 0.4082 = 0.0918 Fig. 20.47

Thus, the number of students having heights greater than 72 inches = 300 . P (X > 72)

ow
= 300 X 0.0918 = 2754
Hence, there are 27 students whose height is greater than 72 inches.
64-68 4
(ii) X = 64 => Z = - = -1.33
3 3

P (X < 64)

e
re
= P(Z <~1.33)
= P(Z >1.33)
rFl
F
-1.33 2=0
= 0.0918. [See (i)] Fig. 20.48

r
Thus, the number of students having heights less than or equal to 64 inches = 300 x 0.0918
ou
fo = 2754.
ks
Hence, there are 27 students whose height is less than or equal to 64 inches.
65 - 68
oo

71 -68
(iii) X = 65 => Z = —;— = -l and, X=71 => Z = = 1
3 3
Y
eB

P(65^X<71)
= P(-1 <Z <1)
ur

= 2.P(0<Z <1)
ad

2=]
Yo

= 2 X 0.3413 = 0.6826 Fig. 20.49

Thus, the number of students having heights between 65 and 71 inches


d

= 300 X P (65 <X <71) = 300 x 0.6826 = 204.78


Re
in

Hence, 205 students have their heights between 65 and 71 inches.


F

EXAMPLE 15 Lcf Z be a standard normal variate. Determine the value oft, if


(i)P(0<Z <0 = 0.4236 (ii) P(Z <0 =0.7967 (iii) P(f < Z < 2) = 0.1000.
SOLUTION (i) In the table given in the end of the book we find that the entry 0.4236 appears m
i
the row corresponding to 1.4 and under the column 3. Hence, t = 1.43.

2=0 2=/

Fig. 20.50

(ii) Since P(Z < f) = 0.7967 and P (2 < 0) = 05. Therefore, t must be positive.
Now, P (Z <t)
= P(2<0) + P(0<Z<f)
20.21
NORMAL DISTRIBUTION

0.7967 =05 + P(0<Z<0

P(0<Z <0=0.2967
t = 0.83 [From Table] 3=0 Z=t

(iii) We have. Fig. 20.51

P{t<Z<2)= 0.1000
P(0<Z <2)-P(0<Z <0 = 0.1000
0.4772 - P {0 < Z < 0 = 0.1000
P(0<Z <0 =0.3772
3=0 t 2
t=U6
Fig. 20.52

EXAMPLE 16 IfX is n noruuil vnrinble with mentt 11 and standard deviation 1.5, find the tnnnber a
such that p (X >a) = 0.09.

w
SOLUTION Let Z be the standard normal variable. Then,

F lo
X-p x-n
z = z =
a 15
rt-11 ... 1
Let Z Z] whenX - a i.e. Z^ =

e
15

Now,
Fre
for
P (X >(7) = 0.09
P(Z >Zi)=0.09
r
P(Z >0)-P(0<Z <Z0=0.09
You
oks

05-P(0<Z <Z0=0.09 '1 3=0


eBo

Fig. 20.53
P (0 £ Z < ZO = 05 - 0.09
P(0<Z <Z0=0.41
Z| =1.34 [From Table 2]
ad
our

a-n
= 1.34 [Using {i)l
15

=> (7=13.01
Re
dY

EXAMPLE 17 In a distribution exactli/ normal, 7% of the items are under 35 and 89% are under 63.
What are the mean and standard deihation of the distribution?
Fin

SOLUTION Let X be the normal variate representing the given distribution. Letp be the mean
and a be the standard deviation of the distribution. It is given that: P(X<35)=0.07 and
P(X<63)=0.89.
X-p
Let Z be the standard normal variate. Then, Z =

35 -p
X = 35 => Z =
a
= 2j (say) ; X = 63 => Z =

P (X < 35) = 0.07 2=0 -2

P(Z <Z|)=0.07 Fig. 20.54

P(Z <0)-P(Z^ <Z <0)=0.07 [■.■ P (Z < Zj) is less than 0.5 .■. Z| < 0]
05-P(0<Z <|Z;il)=0.07
20.22
APPLIED MATHEMATICS-XII

P(0<Z<|Zi|)=0.4300
=>
IZ^ I =1.48 [See Table]
Zi =-1.48 [v 2i<0]
35
= -1.48 ●●● Z1 -
35-m'
C7
a

|.i -1.48 a =35


And,
P(X< 63) = 0.83
=>
P(Z<Z2)=0.83
P(Z<0) + P(0<Z<Z2) = 0.89 [●●● P (Z < Z2) = 0.83 Z2 > 0]
05 + P(0<Z<Z2) = 0.89

w
P(0<Z<Z2)=0.39
Z2 =1.23 [See Table]

F lo
63 -fi
= 1.23
c

|i + 1.23a = 63 ...(ii)

ee
Solving (i) and (ii), we obtain ji =50.3 and a = 10.33.

Fr
Hence, the mean of the distribution is 50.3 and standard deviation is 10.33.
for
EXAMPI.1; 18 The diameter of shafts produced in a factory confirms to normal distribution. 31% of the
shafts have a diameter less than 45 mm and 8% have more than 64 mm. Fmd the mean and standard
ur
deviation of the diameter of shafts.
s

SOLUTION Let X denote the diameter of shafts. Let |.i be the mean and a be the standard
ook
Yo

deviation. It is given that X follows normal distribution.


eB

Let Z be the standard normal variate. Then, Z =


a

45-^1 64 -p.
our

X = 45 => Z = = Z] (say) and, X = 64 => Z =


ad

a
a
= Z2 (say).
Now,
Y

P(X<45) = 0.31
Re

P(Z <Zi) = 0.31


nd

P(Z<0)-P(2i<Z<0)=0.31 [●.● P (Z < Zj) is less than 0.5 .●. Z-^ < 0]
Fi

05-P(Zi<2<0)=0.31
P(Zi <2 <0)= 0.19
P(0<Z <|Zi|)=0.19
=>
|Zi| =05 [See Table]
-Z, =05 [V Zi<0.-.|Zi[=-Zi]
Zi=-05
45-M 45-M
= -05 Z1 - 2] 2=0
a
a
Fig. 20.55
M - 05 a = 45
...(i)
And,
P(X>64)=0.08
P(Z >Z2) = 0.08
20.23
NORMAL DISTRIBUTION

P{Z >0) -P(0<Z <Z2)=0.08


05-P(0<Z<Z2)-0.08
P(0<Z<Z2)=0.42
Z2=1-41 z=0 22
64-|i = 1.41 Fig. 20.56
a

...(ii)
|.i +1.41 a = 64
Solving (i) and (ii), we get (j = 10 and [x = 49.9.
Hence the mean and standard deviation of the diameter of shafts are 49.9 mm and 10 mm.
respectively.
EXAMPLE 19 In a certain examination the percentage of passes and distinction were 46 and^ 9
respectively. Estimate the average marks obtained by the candidate, the minimum pass and distinction
mai ■ks being 40 and 75 respectively (assume the distribution of marks to be normal).

w
SOLUTION Let X denote the marks obtained by the candidates. Let p be the mean and a be the
Standard deviation of the normal distribution.
Let Z be the standard normal variate. Then,

Now,

Now,
a

X = 40 => Z =
40-p
G
F lo
= Zi (say) and, X =75 => Z =
for 75-M

Fa

ree
= Z2 (say)

[Given]
P (X > 40) = 0.46
r
You

P(Z >Z^)= 0.46


oks

'v P(Z>Z|)<05
eBo

P(Z>0)-P(0<Z<Zi)=0.46 .●. Z^>0

05-P(0<Z<Zi)=0.46 2=0 ^
ad

-1
our

P(0<Z <Zi)=0.04
Fig. 20.57
Z| =0.1
40-p
= 0.1
Re
dY

...(i)
^ + 0.1 a = 40
Fin

And,
P(X>75)=0.09
P(Z >Z2)=0.09
v P(Z ^Z2) <05
P (Z > 0) - P (0 < Z < Z2) - 0.09 .●.Z2>0
05-P(0<Z<Z2)=0.09 2=0 ~2

P(0<Z<Z2)=0.41 Fig. 20.58


Z2 =1.34
75-p
= 1.34
a

...(ii)
p + 1.34 a =75
Solving (i) and (ii), we get p = 37.18 and a = 28.22.
Thus, the average marks obtained by the candidates is 37.
20.24
APPLIED MATHEMATICS-XM

EXAMPLE 20 The income distribution of loorkers in a certain factory tonsfound to be normal zoith mean
? 500 and standard deviation equal to X50. There zvere 228 persons getting above ? 600. How many
workers were there in all ? (Area under the standard normal curve between height at 0 and 2 is 0.4772).
SOLUTION LetXdenotetheincomeofworkers.Then,Xisanormalvariatewith meanfi = ^500
and standard deviation a = ? 50. Let Z be the standard normal variate Then
^_X-5Q0
50

X=600 => Z=-®5^^=2.


50

Now, P(X> 600)


= P(2>2)
2=0

w
■^2
= 05 - P (0 < 2 < 2) Fig. 20.59
= 05 - 0.4772
= 0.0228

Flo
Let the total number of workers be;i. Then,

e
Number of workers getting above ? 600 = /i. P (X > 600) = nx 0.0228

re
228
228 0.0228 => /? = = 10,000.

F
0.0228

Hence, the total number of workers =10,000.


ur
r
fo
EXAMPLE 21 15,000 students appeared for an examination. The mean marks zvere 49 and standard
deviation of marks was 6. Assuming the marks to be normally distributed, what proportion of students
ks
scored more than 55 marks?
Yo

If in the same examination. Grade 'A' is to be given to students scoring more than 70 marks
oo

what proportion of the students will receive grade 'A' ?


B

SOLUTION Let X denote the marks of the students. It is given that X is a normal variate with
re

mean p = 49 and standard deviation a


= 6. Let 2 be the standard normal variate. Then,
X-49
i.e. 2 =
u
ad

a 6
Yo

55 - 49
X=55 => 2 = = 1.
6
d

Now, P(X>55)
Re
in

= P(2>1)
F

= 05 - P (0 < 2 < 1) = 05 - 0.3413 = 0.1587.


Thus, the proportion of students scoring more than 55 marks is 0.1587 or 15.87%
70-49
Now, X=70 => 2 = = 35
6

P (X > 70) = P (2 > 35) = 05 - P (0 < 2 < 35) - 05 - 0.4998 = 0.0002


Thus, the proportion of students scoring grade 'A' is 0.002 or 0.02%.
EXAMPLE 22 The income of a group of 70,000 persons was found to be normnlli/ distributed with mean
I'fOp.ni. and standard deviation ? 50. Show that of this group about 95% had income exceeding
< 668 and only 5% had income exceeding ? 832. What was the loioest income among the richest 100 ?
SOLUTION Let X denote the income. Then X is normally distributed with mean p = ? 750 and
standard deviaHon a = ? 50. Let 2 be the standard normi variate Then
2 =
X-p X-750
or. 2 =
CT 50
20.25
NORMAL DISTRIBUTION

668-750 82
When X = 668, we obtain: Z = = -1.64
50 50

Now, P{X> 668)


= P(Z >-1.64)
= P (-1.64 <Z <0) + P (Z >0) = P(0 <Z <1.64) + 05 = 0.4495 + 05=0.9495
Thus, 94.95% persons had income exceeding f 668.
S32-750
When X = 832, we obtain : Z = = 1.64
50

P (X > 832)
= P(Z >1.64)
= P (Z > 0) - P (0 < Z < 1.64) = 05 - 0.4495 = 0.0505
Thus, 5.05 % persons had income exceeding X 832.
100

w
= 0.01
Now, probability of selecting a person out of richest 100 persons = 10000

F lo
hi order to find the lowest income among the richest 100, we have to find the value k of X such
that P{X>k)= 0.01
k -750
= Z-, (say)

e
When X = ic, we obtain Z =

Fre
50

Now, P(X>/c)=0.01 for


P(Z >Zi)=0.01
05-P(0<Z<Zi)=0.01
r
P(0<Z<Z-i)=0.49
You
oks

Zi =2.33
eBo

k -750
= 2.33 => cf=750 + 50x 2.33 => k = S665
50

Hence, the lowest income among the richest 100 was ? 866.50.
our
ad

EXERCISE 20.1

1. Define normal distribution. Draw a rough sketch of the standard normal curve and write
down the salient features of the curve.
dY
Re

2. Write the conditions under which binomial distribution tends to normal distribution.
Fin

3. If Z is the standard normal variate, find :


(i) P(053<Z <2.03) (ii) P (Z < 0.73) (hi) P(-0.81<Z <1.13)
(iv) P(|Z|<0.25) (v) P(0<Z <1.2) (vi) P(- 0.46 < Z < 2.21)
(vii) P(Z >-1.64) (viii) P(-1.96<Z <1.96) (ix) P(|Z1>1)
4. Let X be normally distributed with mean 8 and standard deviation 4. Find :
(i) P(5<X<10) (ii) P(10<X<15) (hi) P(X>15) (iv) P(X<5)
5. Let Z be the standard normal variate. Determine the value of t if
(i) P (0 < Z < 0 = 0.3770 (ii) P (Z < 0 = 0.8621 (hi) P (-15 <Z <0 = 0.0217
(iv) P(Z >0=0.84 (v) P(-f<Z <0=0.9.
6. If X is normally distributed with mean 5 and standard deviation 2, find P(X > 8).
7. In a sample of 120 workers in a factory the mean and S.D. of the wages were? 11.35 and
? 3.03 respectively. Find the percentage of workers getting wages between ? 9 and ? 17 in
the whole factory assuming that the wages are normally distributed.
20.26
APPLIED MATHEMATICS-Xll

8. The mean I.Q. of a large number of children of age 14 was 100 and the standard deviation
16. Assuming that the distribution was normal, find
(i) What percentage of the children had I.Q. under 80?
(ii) Between what limits the I.Q.'s of the middle 40% of the children lay?
(iii) What percentage of the children had I.Q.'s within the range |a +1.96 ci?
9. The breaking strength Xof a cotton fabric is normally distributed with mean 16 and S.D. = 1.
The fabric is said to be good if X > 14. What is the probability that a fabric chosen at random
is good,

ow
10- In a large group of men, it is found that 5% are under 60 inches and 40% are between 60 and
65 inches in height. Assuming the distribution to be exactly normal, find the mean and S.D.
of height.
11. In a distribution exactly normal, 7% of the items are under 35 and 89% are under 63. What

e
are the mean and standard deviation of the distribution?

re
' 15000 students appeared for an examination. The mean marks were 49 and the standard
deviation of marks was 6. Assuming the marks to be normally distributed, what proportion

Frl
F
of students scored more than 55 marks.
]In an intelligence test administered to 1000 children, the average score is 42 and standard
deviation 24.
ou
sor
(i) Find the number of children exceeding the score 60, and
(ii) Find the number of children with score lying between 20 and
kf 40.
14. Assume the mean height of soldiers to be normally distributed with mean 68.22 inches and
oo
variance of 10.8 inches. How many soldiers in a regiment of 1,000 would expect to be over
six feet tall?
Y
B

15. 1000 light bulbs with a mean life of 120 days are installed in a new factory; their length of
life is normally distributed with standard deviation 20 days. How many bulbs will expi re in
re

less than 90 days?


oY

Ih. Suppose the weights of 2000 gorillas are normally distributed with mean 155 kg and
u

standard deviation 20 kg. Find the number of gorillas with weights (i) less than or equal to
ad

100 kg, (ii) between 120 and 130 kg, (iii) between 150 and 175 kg (iv) greater than or equal to
d

200 kg.
in

I. The mean weight of 500 male students at a certain school is 151 lb and the standard
Re

deviation is 15 Ib. Assuming that the weights normally distributed, find how many
are
F

students weigh (i) between 120 and 155 lb, (ii) more than 185 lb.
'H. In an examination 15% of the candidates got first class (60% marks or above) while 40%
failed (securing below 40%). Assuming the marks to be normally distributed, find the mean
and standard deviation.
1 In a certain examination 20% of the candidates scored 60 or more marks and 30 % scored 40
or less marks. Find the mean and standard deviation of marks, assuming that the marks are
normally distributed.
Suppose that the height of all cakes baked with a certain mix have a mean of 5.3 cm and a
standard deviation of 0.75 cm. Assuming that the distribution of the heights of these can be
approximated closely with a normal curve, find
(i) the percentage of cakes which have a height of 4.4 cm or less,
(ii) the percentage of cakes having height from 5 cm to 6.2 cm.
(iii) the height below which we may find the flattest 20% of the cakes.
20.27
NORMAL DISTRIBUTION

w
21. In a sample of 120 workers in a factory the mean and standard deviation of wages were
X 11.35 and X 3.03 respectively. Find the percentage of workers getting wages between X9
and ? 17 in the whole factory assuming that the wages are normally distributed.

r
22. The length of telephone calls received by the office of a particular firm is normaUy
distributed with a mean of 5.3 minutes and a standard deviation of 2.8 minutes. If a single
call is selected at random, compute the following probabilities:
(i) the call will be 2 minute, or less in length

u
(ii) the call will last between 3 and 5 minutes,

o
sF
(iii) the call will be 7 minutes or more in length.
23. The mean life time of 60 watt light bulbs produced by a company is 20 hours. It is known
that the standard deviation is 20 hours. Assuming that the lift times of light bulbs are
normally distributed, what are the probabilities that a single 60 watt light bulb extracted

F
from the production lot will:

o
k
(i) bum out between 180 hours and 210 hours ?

l
r
(ii) bum out at a time greater than 250 hours ?
24. In a sample of 1000 items, the mean weight is 450 kg with a standard deviation 15 kg.

o
Assuming the normality of the distribution, find the number of items weighing between

Y
40 and 60 kg.
25. An aptitude test for selecting officers in a bank was conducted on 1000 candidates the

f
o
Y
average score is 42 and the standard deviation of score is 24. Assuming normal distribution
for the scores, find :
B
(i) the number of candidates whose scores exceed 60,
(ii) the number of candidates whose scores lie between 30 and 60.
r

Assuming that the height distribution of a group of men is normal, find the mean and the
d

26.

standard deviation; given that 84% of the men have heights less than 65.2 inches and 68%
u
e

have heights between 65.2 and 62.8 inches.


n

27. The results of a particular examination are given below in a summary form:
io

Result % of candidates
ad

10
(i) Passed with distinction
60
(ii) Passed
F

30
(iii) Failed
Re

It is known that a candidate gets plucked if he obtains less than 40 marks (out of 100) while
he must obtain at least 75 marks in order to pass with distinction. Determine the mean and
standard deviation of the distribution of marks assuming to be normal.
28. 1000 light bulbs with a mean life of 120 days are installed in a new factory, their length of
life is normally distributed with standard deviation 20 days. How many bulbs will expire
m

less than 90 days ?


If it is decided to replace all the bulbs together, what interval should be allowed between
replacement if not more than 100% should expire before replacement ?
29. Delhi Vidut Board installed 2000 bulbs in the streets of Delhi. If these bulbs have an average
life of 1000 burning hours, with a standard deviation of 200 hours, what number of bulbs
might be expected to fail in the first 700 burning hours ?
30. In a distribution exactly normal, 10.03% of the items are under 25 kilogram weight and
89.97% of the items are under 70 kilogram weight. What are the mean and standard
deviation of the distribution?
20.28
APPLIED MATHEMATiCS-XII

ANSWERS
3. (i) 0.2769 (ii) 0.7673 (iii) 0.6618 (iv) 0.1974 (v) 0.3849
(vi) 0.6636 (vii) 0.9495 (viii) 0.9500 (ix) 0.6826
4. (i) 0.4649 (ii) 0.2684 (iii) 0.401 (iv) 0.2266
5. (i) 1.16 (ii) 1.09 (iii) -1.35 or -1.69 (iv) - 0.995 (v) ± 1.645
6. 0668 7. 75.1 8. (i) 10.56% (ii) 91.6,108.4 (iii) 0.95%
9. 0.9772 10. M =65.41, a = 3.29 11. =50.3, <7=10.33
12. 0.3413 13. (i) 227 (ii) 289 125 15. 67
16. (i) 6 (ii) 131 (iii) 880 (iv) 24
17. (i) 300 (ii) 5 IS. Mean = 43.94,5.0 = 15.5

w
19. Mean = 47.74, S. D. = 14.6 (i) 11.51% (ii) 54.03% (iii) 4.794 to 5.806
21. 75.1% 22. (i) 0.1271 (ii) 0.2573 (iii) 0.2611

Flo
23. (i) 0.5328 (ii) 0.0062 24. 471 25. (i)227 (ii) 533

ee
26. =64, a = 1.14 27. M =50.17, 0 = 19.4 28. 67,94 days

Fr
29. 134 30. IX =475, 0=1758

for
ur
k s
Yo
oo
eB
r
ou
ad
Y
nd
Re
Fi
CHAPTER
ij 1

INFERENTIAL STATISTICS

21.1 POPULATION AND SAMPLE

If a grain merchant wishes to buy wheat from a farmer, he assesses the quality of wheat by
taking a handful of it from a bag and then decides to purchase or not. Wheat in the bag is
considered as a population and handful of wheat taken for inspection is called a sample.

w
Similarly, in a factory producing electric lamps, some lamps are picked up randomly to check
the quality by the quality control department. Lamps chosen for inspection form a sample and

F lo
the totality of the lamps manufactured is called the population.
Tlie following is the formal statistical definition of population.

ee
POPULATION The population is an aggregate of objects, animate or inanimate, under stud}/.

Fr
Objects (animate or inanimate) in the population are also called statistical individuals.
The population may be finite or infinite according as the number of objects in it is finite or
for
infinite.
ur
In the above discussion wheat in the wheat bag form an infinite population whereas the totality
of electric lamps manufactured by the factory over a period of time form a finite population.
s
ook
Yo

If the population is infinite or the number of statistical individuals (objects) in the population is
eB

extremely large, then for any statistical investigation complete enumeration of the population is
not possible. Even if the population is finite, complete enumeration is impracticable because of
administrative and financial implications, time factor etc. So, for any statistical investigation, we
our
ad

take the help of sampling. Sampling is quite often used in our day-to-day practical life.
SAMPLE A finite subset of statistical individuals (objects) in a population is called a sample.
SAMPLE SIZE The number of statistical individuals (objects) in a sample is called the sample size.
Y
Re

The process of selecting samples from a population is called sampling. The purpose of sampling
nd

is to draw inference about the population by examining the sample. For the purpose of
Fi

determining population characteristics, instead of enumerating the entire population, the


individuals in the sample are observed to determine sample characteristics. These sample
characteristics are used to estimate the population characteristics. For example, by examining
some samples of certain items, we may arrive at a decision of accepting or rejecting those items.
However, the acceptance or rejection based on the characteristic s of the sample or samples give
rise to an error called the sampling error.The sampling error is inherent and unavoidable in any
and every sampling scheme.
21.2 PARAMETERS AND STATISTICS

PARAMETERS The Statistical constants or measures of the population, like mean(\.i), variance (g^) etc.
are called the parameters of the pjopulation.
STATISTICS The statistical measures or constants computed from the sample observatio7is alone, like
mean (X), variance (s ) etc. are called statistics.
21.2 APPLIED MATHEMATICS-XII

<y
In practice, values of population parameters viz. p, cr etc. are not known and the corresponding
values of statistics obtained from samples are used for the analysis of the population. However,
the statistics based on different samples can vary from one sample to another sample. One of the
fundamental problems of sampling theory is to find out whether these variations in the statistic
obtained from different samples are significant or insignificant.
21.2.1 SAMPLING DISTRIBUTION

If we select a number of independent random samples of a definite size from a given population
and calculate some statistic (like mean, mode, median, standard deviation etc) from each
sample, we shall get a series of values of the statistic. These values obtained from the different
samples can be put in the form of a frequency distribution as given below.

Sample Number 1 2 3 4

w
n

Statistic (say mean) X1 ^2 ^3 ^3 X,

Flo
The distribution so formed of all possible values of a statistic is called the sampling distribution
or the probability distribution of that statistic.

e
Thus, if we draw 150 random samples from a given population and calculate their means, we

re
shall get a series of 150 means which would form a frequency distribution. This distribution is

rF
called the sampling distribution of the means.
In general, if , $2, S3 3re values of a statistic S (like mean, variance etc.) obtained from
ur
n independent random samples of a definite size chosen from a_ given population, then
fo
S^, $2, S3,..., form a sampling distribution of statistic S.The mean(S) and variance of statistic
ks
Sare given by
Yo
1 1
S = -n : ^ Sj and Var (S) Z (Si-s)^
oo

n
j = l j = l
B

STANDARD ERROR (S.E.) The standard deviation of the sampling distribution of a statistic is known as
re

its standard error.


u
ad

21.3 STATISTICALINFERENCES
Yo

The principal objective of statistical inference is to draw inferences about a population


parameters from the analysis of a sample drawn from that population. Statistical inference
consists of two major areas (i) estimation and (ii) testing of hypothesis. We shall study about
d
Re

testing of hypothesis as the theory of estimation is beyond the scope of this book. Hypothesis
in

testing begins with an assumption called null hypothesis that we make about a population
F

parameter. The null hypothesis asserts that there is no significant difference in the sample
statistic and the corresponding population parameter or between two sample statistics. It is a
hypothesis of no difference. Null hypothesis is usually denoted by Hq. In case of a single
statistic, Hq will be that the sample statistic does not differ significantly from the hypothetical
parameter value and in the case of two sample statistics, Hq will be that the sample statistics do
not differ significantly.
Having set up the null hypothesis, we compute the probability P that the deviation between the
observed sample statistic and the hypothetical parameter value might have occurred due to
fluctuations or sampling. If the deviation comes out to be significant (as measured by a test of
significance) null hypothesis is rejected at the particular level of significance adopted and if the
deviation is not significant, null hypothesis may be retained at that level.
Any hypothesis which is complementary to the null hypothesis is called an alternate
hypothesis, usually denoted by Hj. For example, if we want to test the null hypothesis that the
population has a specified mean p q (say) i.e. Hq : p = p q , then the alternative hypothesis could
be:
INFERENTIAL STATISTICS 21.3

(i) Hi :p (ii) Hj :m >Po (iii) Hi :|i <|iQ


The alternative hypothesis in (i) is known as a two-tailed alternative and alternative in (ii) and
(iii) are known as right-tailed and left-tailed alternatives respectively. The setting of alternative
hypothesis is very important since it enables us to decide whether we have to use a single-tailed
(right or left) or two-tailed test.
21.3.1 SIGNIFICANCE LEVEL, CONFIDENCE LEVEL AND CONFIDENCE INTERVAL

The significance level also called the alpha level is a term used to test a hypothesis as defined
below.
SIGNIFICANCE LEVEL
In a hypothesis test, the significance level, a, is the probability of making wrong
decision xohen the null hypothesis is true.
For example, significance level 0.05 indicates a 5% risk of concluding that a difference exists
(between the population parameter and sample statistic or between statistics of two samples)
when there is no actual difference. In other words, if we use the same sampling method to such

w
different samples, then 5% of the samples drawn do not include the population parameter.

F lo
Thus, significance level a means that 100a% of the samples drawn do not include the
population parameter and so (100 - 100a) % = 100 (1 - a) % of the samples drawn include the
population parameter.

e
Fre
To graph a significance level of 0.05 in a two-tailed distribution, we need to shade the 5% of the
distribution, that is furthest away from the null hypothesis. In the following graph two shaded
areas are equidistant from the null hypothesis value and each has a probability of 0.025, for a
for
total of 0.05. These shaded areas are called the critical region for a two-tailed test. The critical
region defines how far away our sample statistic must be from the null hypothesis value before
r
You

we can say it is unusual enough to reject the null hypothesis. If sample statistic falls within the
s
ook

critical region, it indicates that it is statistically significant at the 0.05 level. In Fig. 21.1, the
sample statistic does not fall within the critical region representing 0.01 level of significance. So,
eB

we cannot reject the null hypothesis at 0.01 level of significance. This comparison tells us why
we need to choose the significance level before accepting or rejecting the null hypothesis.
our
ad dY
Re

|.i (Null hypothesis


Fin

Sample statistic
value)
Fig. 21.1

CONFIDENCE LEVEL A confidence level refers to the percentage of all possible samples that can be
expected to include the true population parameter.
For example, a 95% confidence level means that in 95% of the samples drawn include the
population parameter and remaining 5% samples drawn do not include the population
parameter.
Clearly, Confidence level = 1 - Significance level
CONFIDENCE INTERVAL A confidence interval is a range that could be expected to contain the
population parameter of interest.
Confidence intervals are intrinsically connected to confidence levels. When we say that
confidence interval for population parameter is [a, b] with a 95% confidence level, it means that
in 95% of the samples drawn the population parameter falls within the confidence interval.
21.4 APPLIED MATHEMATICS-Xil

21.4 STUDENT'S f-DISTRIBUTlON

In 1905 Sir William Gossett gave a test popularly known as Mest. Gossett was employed by the
Guiness Brewery in Dublin, Ireland, which did not permit employees to publish research
findings under their own name. So, Gossett adopted the pen-name 'Student' and published his
findings under this name. Thereafter, the f-distribution is commonly called Student's
f-distribution or simply Student's distribution.
The ^distribution is used when sample size is 30 or less and the population standard deviation
is unknown.

Let .Vj, .\'2,..., x„ be a random sample of size n from a normal population with mean and variance a"^.
Then Student's t is defined b^ the statistic
f =
X-p
s/Vm

w
where X = - ^ Xj is the sample mean and — S
n-^
unbiased estimate of

F lo
n
/ = ! /=!

population variance.
It follows Student's /-distribution with v =(??-!) degrees of freedom with probability density

ree
function

F
v+r

f{t)=C 1+- 2 - 00 <f < OD. for


V
ur
where C is a constant required to make the area under the curve equal to unity.
s

REMARK 1 The number of degrees offreedom of a statistic is the number of independent varieties used to
ook
Yo

compute that statistic. For example, if a small sample has n observations with m constraints on these
values i.e. m values are already available, then the number of degrees offreedom is v = n-m.
eB

REMARK 2 //.Vj, are n obseivations_ in a sample, then for computing sample mean X, we use
all the values Xi, X2,...,x„.Therefore, the mean X has n degrees offreedom. Since the standard deviation of
r

the sample depends on the mean, therefore the standard deviation has (n -1) degrees offreedom.
ad
ou

GRAPH OF t-DISTRIBUTlON The probability density function of a random variable X folloiving


t-distribution is given by
Y

v+ T
Re

2^
nd

t . 2 ,
m=c i-i-- ,-CO </ <00
Fi

We observe that/(-/) =/(/), so the probability curve is symmetric about the line / = 0.
As f —»■ CO, /(f) 0 rapidly. So, f-axis is an asymptote of the curve.
The curve resembless the standard normal probability curve and is bell shaped as shown in
Fig. 21.2.
Standard normal m
distribution \
j /-distribution for y = 15
2
/-distribution for i’ = 8
3
3 3
●2-
2 1
1 O

Fig. 21.2 t-distribution


INFERENTIAL STATISTICS 21.5

As the number of degrees of freedom increases, the f-distribution curve moves closer to the
standard normal probability curve.
PROPERTIES OF f-DlSTRiBUTlON (i) The variable t of t-distribiition ranges from - co fo oo.
(ii) The probability curve is symmetric about the line f = 0 and it resembles with the standard
normal probability curve and is bell shaped as shoW' i in Fig. 21.2. As tlie number of degrees of
freedom increases, the f-distribution curve moves closer to the standard normal probability

ow
curve,

(iii) The variance of f-distribution is greater than one, but approaches one as the number of
degrees of freedom and therefore the sample size becomes large.
The values of t- distribution have been tabulated extensively. The values of fy(a) (two tailed)

e
have been tabulated for a =0.10, 0.05, 0.025, 0.01, 0.005 and v=l, 2, 3,..., 29, 30 where a) is

re
such that the area to its right under the curve off-distribution with v degrees of freedom is equal

Flr
to a (see Fig 21.3). That is i^ia) is such that if the random variable f has f-distribution with v

F
degrees of freedom, then
P(|fl>fv(a))=a
ou
sr
The table does not contain values of fy(ct) for a>0.50, because 4(1-a) =-fy(ct) as the

fo
probability density function/(f) is symmetrical about f = 0. When vis more than 30 probabilities
k
related to f-distribution are usually approximated witli the use of normal distributions.
oo
Y

Rejection Rejection
reB

Accep ance
region a/2 region (1-a) region a/2
uY

0
-fy(a) fi,(a)
Fig. 21.3 Critical values of f-distribution
ad
do

CONFIDENCE OF FIDUCIAL INTERVALS FOR POPULATION MEANp 7/4(0.05) is the tabulated value of t
in

for v = ?j -1 degrees offreedom at 5% level of significance, then


Re

f \ f \
P |f|> 4(0.05) =0.05 =>P I fl <4(0.05) =0.95
F

Thus, the confidence interval for population mean p with a 95% confidence level is given by
I f| <4(0-05)
X-p
<4 (0.05)
S/4n

p-X
<4(0.05)
Sf-sln

|p-Xl<^4 (0-05)
X -A
sfn
fy (0.05) < p < X + 4= fy (0.05)
n
21.6
APPLIED MATHEMATICS-Xn

Hence, the confidence interval with a 95% confidence level or at 5% level of significance is
X-A^^(0.05), X+^

fv(0.05) , where is the tabulated value of t for v=(»-l)
degrees of freedom.
Similarly, the confidence interval with a 99% confidence level or at 1% level of significance is

^ ~^^v(0-01)/ ^ +^fv(0.01) , where tv(O.Ol) is the tabulated value of t for v=(n-l)


degrees of freedom.

If instead of S the sample variance s = — ^ (x; -X)^ is given then confidence interval at 5%

ow
level of significance X -
=^fv(0.05),
y/fJ-l X +-^== (0.05) . Similarly, the confidence
intervals at 1% level of significance is X - = fv(0-0i), x + ^i=t^(0.0i).

e
Vn-1 Vn-1

re
Frl
21.5 APPLICATIONS OF f-DISTRiBUTION

F
The f-distribution has a wide range of applications in Statistics.
In this chapter, we will discuss the following:
ou
or
(i) To test if the sample mean (X) differs significantly from the hypothetical value p of the
population mean,
kfs
(ii) To test the significance of the difference between two sample means.
oo
21.5.1 TEST OF SIGNIFICANCE OF THE DIFFERENCE BETWEEN MEAN OF A RANDOM SAMPLE
AND POPULATION MEAN
Y
B

In order to determine whether the meanXof a small random sample X2, X3,...,x„ drawn
from a normal population deviates significantly from the hypothetical value p of the population
re

mean, when standard deviation ct of the population is known, we proceed as follows.


De/me Null hypothesis Hq : There is no signiifcant difference between the sample mean X and the
oYu

STEP!
ad

population mean p i.e. The sample has been drawn from the population with mean p
STEP II Define the statistic
d

t =
X-p
in

S/ffn
Re
F

where,
1 ^
^ sample mean, p = Hypothetical mean of the population, n = Sample size

2
1 If
n-1 n
i=l

^2
2 1
■, di = Xj - A, A is the assumed mean.
n \

Sample statistic follows t-distribution with v = (n-l) degrees offreedom.


sicpjn Find (f\ and tabulated value oft with (n-l) degrees offreedom at a certain level of significa nee.
INFERENTIAL STATISTICS 21.7

If calculated | f | < tabulated (0.05), the null hypotl^sis Hq may be accepted at 0.05 level of
significance and we say that the difference between X and p is not significant and hence the
sample might have been drawn from a population with mean p.
If calculated 111 < tabulated t,,_i (0.01), the null hypoUiesis Hq may be accepted at 0.01 level of
significance and we say that the difference between X and p is not significant.
If calculated \t\> tabulatedf„_i (0.05), the null hypothesis Hq may be rejected at 0.05 level of
significance and we say that the difference between X and p is significant at 5% level.
If calculated | f | > tabulated (0.01), the null hypothesis Hq may be rejected at 0.01 level of
significance and we say that the difference between X and p is significant at 1% level.

ow
REMARK Let s^be the sample variance, then

n
i=l

e
n
=>

re
n-l

rFl
,=i

F
n
s'^ =S^, where is the unbiased estimate of population variance.
H-1

S2

r
s S
ou
n-\ n yjn-\ 4n
fo
ks
X-p X-p
t = => f =
S/i^ s/^Jn-i
oo

Thus, we have
Y
B

X-p X-p
t = or, t =
S/i^ s/^ln-1
re

1
where, S^=^— Y (x'-X)^ and
ou
Y
ad

n ■
/=1 i=l

ILLUSTRATIVE EXAMPLES
d

[iXAMPLE 1 A company has been producing steel tubes of mean inner diameter of 2 cm. A sample of 10
in
Re

tubes gives an inner diameter of 2.01 cm and a variance of 0.004 ctn^. Is the difference in the values of
F

itieans significant? (Given fg (0.05) = 2.262)


SOLUTION It is given that:
p = Population mean = 2 cm, X = Sample mean = 2.01 cm
«= Sample size = 10 and. = Sample variance = 0.004 cm^
We define.
Null Hypothesis Hq: The difference between the sample mean X and the population mean p is
not significant.
Alternate hypothesis H.y The difference between the sample mean X and the population mean p
is significant.
Let t be the test statistic given by
X-p (X - p) ^^n^
t = or, t =
s/fn-1 s
21.8
APPLIED MATHEMATICS-Xl!

t =
(2.01 - 2.00) 0.01 X 3 10
— = 0.476
V0.004 0.063 21

The test staristic't' follows Student's f-distribution with (10 -1) = 9 degrees of freedom. We shall
now
compare this calculated value with of [ / [ the tabulated value of t for 9 degrees of freedom
and at a certain level of significance. It is given that f9(0.05) =2.262.
Clearly, 111 </9 (0.05) i.e. calculated 111 < tabulated U) (0.05).
So, the null hypothesis Hq is accepted at 5% level of significance. Hence, the difference in the
values of sample mean and population mean is not significant.
EXAMPLE; 2 A machinist is making engine parts with axle cliaf7ieter of 0.7 inch. A random sample of 10
parts shows mean diameter 0.742 inch with a standard deviation of 0.04 inch. On the basis of this sample,
would you say that the zuork is inferior? (Given (0.05) = 2.262)

ow
SOLUTION It is given that:
|j = Population mean = 0.7, X = Sample mean = 0.742
n = Sample size = 10 and. s = Sample standard deviation = 0.04
We define.

e
Fl
re
Null Hypothesis Hq: There is no significant difference between sample mean X and the
population mean p or, the product is not inferior.

F
Alternate hypothesis Hp The difference beh\'een the sample meanXand the population meanp is
ur
r
significant i.e. p X or the product is inferior.
Let t be the test statistic given by fo
ks
t = or, t =
Yo
sZ/tt-l s
oo

(0.742-0.7) 0.042 126


VT^ = X 3 = = 3.15
B

0.04 0.04 40
e

The test statistic't' follows Student's f-distribution with (10 -1) = 9 degrees of freedom. We shall
ur

now
compare this calculated value with the tabulated value of t for 9 degrees of freedom and at a
certain level of significance. It is given that tg (0.05) = 2.262.
ad
Yo

We observe that

|f| = 3.15>2.262 =t9 (0.05)


d

Calculated | f | > Tabulated tg (0.05)


Re

I.e.
in

So, the null hypothesis is rejected at 5% level of significance or the alternative hypothesis is
F

accepted at 5% level of significance. Hence, sample mean X differs significantly from population
mean p i.e. the work is inferior.

nXAMl'I.t 3 A soap manufacturing company was distributing a particular brand of a soap throng}, 2 a
large number of retail shops. Before a heavy advertisement campaign, the mean sales per week per shop
zvas 140 dozens. After the campaign a sample of 26 sfiops was taken and mean sales was found to be
147 dozens with standard deviation 16. Can you consider the advertisement effective'^ (Given
t25 (0.05) =2.06)
SOLUTION It is given that
p = Population mean = 140, X - Sample mean = 147
n = Sample size = 26 s =16.

We define.

Null Hypothesis Hq: There is no significant difference in the mean sales before and after
advertisement.
K'JFERENTIAL STATISTICS 21.9

Alteruate hypothesis There is significant difference in the mean sales before and after
advertisement.

Let t be the test statistic given by


X-^i X-M
t = or, t =
S/i^ s/^ln-1

t = (l^^V2iPr=^.5=^
16 16 16
= 2.187
The sample statistic't' follows student's f-distribution with v = (26-1) = 25 degrees of freedom.
We shall now compare this calculated value with the tabulated value of t for 25 degrees of

ow
freedom and at a certain level of significance. It is given that ^25 (0.05) = 2.06.
We observe that

|f|=2.187>2.06=f25 (0.05)
i.e.
Calculated | f | > Tabulated ^25 (0.05)

e
re
rFl
So, we reject the null hypothesis and accept the alternate hypothesis. Hence, we conclude that
advertisement is effective for sales.

F
i-;x.AMri !● ; A random sample of size 16 has 53 as mean. The sum of the squares of the deviations taken
from mean is 150. Can this sample be regarded as taken from the population having 56 as mean? (Given

r
(0.01) = 2.95)
fo
ou
SOLUTION We have,
ks
n = Sample size = 16, X = Sample mean = 53, |i = Population mean - 56
oo

16

and, ^ (x, -X)^ =150, where Xj, X2, ^re sample observations
Y
eB

/ = 1

1 16 _ o 1
S2 = Y (.^i-X)^=—Xl50=10
ur

(16-1) ^ 15 i=l
ad
Yo

We define,

Null Hypothesis Hq: The sample is drawn from tlie population having 56 as mean.
d
Re
in

Alternate hypothesis Hp The sample is not drawn from the population having 56 as mean.
Let the sample statistic f be given by
F

X -p 53-56 -3x4 12V1O -12x3.162


t - f = = -3.794
S/4n ViO/16 VlO 10 10

The sample statistic follows Student's f-distribution with v = (16 -1) = 15 degrees of freedom.
We shall now compare this calculated value with the tabulated value of f for 15 degrees of
freedom at a certain level of significance. It is given that t^^ (0.01) = 2.95.
Calculated\t\ = 3.794 >2.95 = (0.01)
i.e. Calculated | f | > Tabulated (0.01)
So, we reject the null hypothesis. Consequently, the alternate hypothesis is accepted at 0.01 level
of significance. Hence, the sample is not taken from the population having 56 as mean.
i-.XAMPU; s A random sample of 17 values from a normal population has a mean of 105 cm and the sum of
the squares of deviations fi-om this mean is 1225 cm^. Is the assumption of a mean of 110 cm for the normal
populatio)! reasonable? Test under 5% and 1% levels of significance. Also, obtain the 95% and 99%
confidence limits. (Given (0.05) = 2.12 and (0.01) = 2.921).
21.10 APPLIED MATHEMATICS-XII

SOLUTION We have,
|i = Population mean = 110, X = Sample mean = 105
17 _ „
n = Sample size = 17 and. £ (Xi -X)2 =1225.
/=1

5^=1” 1 £
=1

= 72.0588 => s = V72.0588 = 8.4887


17

We define.

w
Null Hypothesis Hq: There is no significant difference between sample mean and population
mean i.e. assumption that mean of the population is 110 cm is valid.
Alternate hypothesis Hj: Assumption that mean of the population is 110 cm is not valid.

Flo
Let t be the test statistic given by

e
X-p 105-110 -5x4
=> t = xVi^ = = -2.3561

re
s/Vn-1 8.4887 8.4887

F
I f| =2.3561
The sample statistic follows Student's f-distribution with v = (17 -1) =16 degrees of freedom.
ur
r
fo
We shall now compare this calculated value with the tabulated value of t for 16 degrees of
freedom at 5% and 1% levels of significance.
ks
At 5% level of significance: It is given that (0.05) = 2.12. We find that
Yo

Calculated | f | = 2.3561 > 2.12 = (0.05)


oo

i.e.
Calculated | f | > Tabulated fjg (0.05)
B

So, we reject the null hypothesis at 5% level of significance. Hence, the assumption that the
re

population has mean 110 cm is not correct.


The confidence limits at 5% level of significance are
u
ad
Yo

X-
VM—1
(0-05) and X +
^*>6(0.05)
8.4887 8.4887
d

or. 105- x2.12 and 105 + x2.12


Re

4 4
in

or. 105 - 4.499 = 100501 and 105 + 4.499 = 109.499


F

The confidence interval is [100501,109.499]


At 1% level of significance: It is given that (0.01) = 2.921.
Clearly, calculated 111 < tabulated (0.01).
So, we accept the null hypothesis at 1% level of significance. Hence, the assumption that the
mean of the population is 110 cm is valid.
The confidence limits at 1% level of significance are
X -
(is (0.01) and X + (16(0.01)
8.4887 8.4887
or. 105- X 2.921 and 105 + X 2.921
4 4

or. 105-6.199=98.801 and 105 + 6.199=111.199

The confidence interval at 1% level of significance or at 99% confidence level is [98.801,111.199].


INFERENTiAL STATISTICS 21.11

Ten students are selected at random from a college and their heights arefound to be 100,104,
●Mi’i.l-i'

108,110,118,120,122,124,126 and 128 cms. In the light of these data, discuss the suggestion that the
mean height of the students of the college is 110 cms (Given (0.05) = 2.262).
SOLUTION We define

Null Hypothesis Hq: There is no significant difference between the sample mean and hypothe
tical population mean 110 cm.
Alternate hypothesis Hj: The sample mean is not same as the population mean.
Let the sample statistic t be given by
t =
X-p

ow
S/-Jn
Let us now compute the sample mean (X) and S.
Computation ofX and S

.r,--X

e
X;
!

re
100

Fl -16 256

F
104 -12 144
ur
108 -8 64

r
110 -6 fo 36
ks
118 2 4
Yo
120 4 16
oo

122 6 36
eB

124 64

126 10 100
ur

128 12 144
ad

10
Yo

ZXf =1160 £ (x--Xf=864


/=!
d
Re
in

X-M
t =
S/4n
F

116-110
=> f = X Vl0=-^x 3.162=1.94 [v 1.1 =110]
9.798 9.798
10

We find that ^ = 1160, and n -10.


j = 1

1 10 1160
X=- = 116
10
” /=1
10

From the table, we find that ^ (Xj -X)^ = 864


j = l

1 10 1 1 29.393
=
£ (Xi-X)2=>s2=-x864^S = 3 3
= 9.798
^1 = 1 ^
21.12 APPLIED MATHEMATICS-XI!

Tlie sample statistic follows student’s f-distribution withv =(10 -1) = 9 degrees of freedom. We
shall now compare tliis calculated value with the tabulated value of t for 9 degrees of freedom at
a certain level of significance. It is given that (0.05) = 2.262.
Calculated 11 \ =1.94 < 2.262 = tg (0.05)
i.e.
Calculated | f | < tabulated tg (0.05)
So, we accept the null hypothesis. Hence, the sample mean is same as the population mean.
Consequently, the mean height of the students of the college is 110 cm.
EXAMPLE: 7 A random sample of 10 boys had the following LQ's: 70,120,110,101,88,83,95,98,107,
100. Do these data support the assumption of a population mean I.Q. of 100? Find a reasonable range in

ow
which most of the mean I.Q. values of samples of 10 boys tie. (Given tg (0.05) = 2.262)
SOLUTION We have,

p = Population mean = 100, n = Sample size = 10


We define

e
Null Hypothesis Hq: Tlie data are consistent with the assumption of a mean I.Q. of 100 in the

re
population.

rFl
F
Alternate hypothesis H.p Tlie mean I.Q. of population ^ 100.
Let the sample statistic t be given by

r
ou
«

t =
X-p
S/fiT
, where -
1
Z (.V, -X)^ fo
ks
n-l ■
/ = 1

Let us now compute X and S^.


oo

Computation ofX and S


Y
eB

X;
I dj = Xj - 90
r

70 -20 400
ou
ad
Y

120 30 900

no 20 400
d

101 11 121
Re
in

88 -2 4
F

83 -7 49

95 5 25

98 8 64

107 17 289

100 10 100

I.dj =72 Z di^ = 2352


Here, if =Xj - 90
1 1
X=90 +
10
'£f=90 + —
10
= 972 Using:X=A+-J]4 n

(72)2
=1^ 2352-
1833.6
S2 = = 203.73
10 9
INFERENTIAL STATISTICS 21.13

X 97.2-100 -2.8 -2.8


t = ^=>t = = -0.62
S/4ii 203.73 "V2037 4514
10

=> \t\= 0.62


The sample statistic t follows student's f-distribiition with v = (10 -1) = 9 degrees of freedom. It is
given thatf9 (0.05) = 2.262.
Calculated\t\< tabulated tg (0.05)
So, the null hypothesis may be accepted at 5% level of significance.
Hence, the assumption of a population mean I.Q. of 100 is valid.
The 95% confidence limits within which the mean I.Q. values of samples of 10 boys will lie are

X
Jn
fq (0.05) and X + -^
V/i
to (0.05)

w
203.73 20373
X 2.262 and 97.2 + X 2.262
V 10
or, 97.2-
V 10
or.

or,
97.2 - 4514 X 2.262 and 97.2 + 4514x 2.262
97.2-10.21 and 97.2 + 10.21

F lo
e
Fre
or. 86.99 and 107.41

Hence, the required 95% confidence interval is [86.99,107.41]. for EXERCISE 21.1
r
1. Ten cartons are taken at random from an automatic filling machine. The mean net weight of
You

the cartons is 11.8 kg and the standard deviation 0.15 kg. Does the sample mean differ
oks

significantly from the intended weight of 12 kg? (Given fq (0.05) = 2.262).


eBo

2. A machine is designed to produce insulating washers for electrical devices of average


thickness of 0.025 cm. A random sample of 10 washers was found to have an average
thickness of 0.024 cm with a standard deviation of 0.002 cm. Test the significance of the
our
ad

deviation. (Given fq (0.05) = 2.262).


3. A random sample of size 25 from a normal population has the mean 47.5 and standard
deviation 8.4. Does this information refute the claim that the mean of the population is 42%.
dY
Re

(Given f24 (0.05) = 2.06).


4. A process of marking certain bearings is under control if the diameter of the bearings have
Fin

the mean 0.5 cm. What can we say about tliis process if a sample of 10 of these bearings has a
mean diameter of 0.506 cm and standard deviation of 0.004 cm? (Given fq (0.05) = 2.262).
5. A machine is supposed to produce washers of mean thickness 0.12 cm. A sample of 10
washers was found to have a mean thickness of 0.128 and standard deviation 0.008. Test
whether the machine is working in proper order at 5% level of significance. (Given
fq (0.05) =2.262).
6. A random sample of 16 values from a normal population showed a mean of 41.5 and sum of
squares of deviations from mean equal to 135. Can it be assumed that the mean of the
population is 43.5? (Given fj5 (0.01) = 2.95).
7. A sample of size 9 from a normal population X = 15.8 and = 10.3. Find 99% confidence
interval for population mean. (Given fg (0.01) = 3.335).
21.14
APPLIED MATHEMATICS-XII

8. A random sample of size 16 has 53 as mean. The sum of the squares of deviations taken
from mean is 150. Find 95% and 99% confidence intervals for population mean. (Given
tl5 (0.01) = 2.95 and (0.05) = 2.13).
9. A random sample of 16 values from a normal population showed a mean of 41.5 inches and
the sum of squares of deviations from this mean equal to 135 square inches. Show that the
assumption of a mean of 43.5 inches for the population is not reasonable. Obtain 95% and
99% confidence intervals for the same. (Given (0.05) = 2.131 and (0.01) = 2.947).
1 (). The annual rainfall at a certain place is normally distributed with mean 45 cm. The rainfall
during the last five years are 48 cm, 42, cm, 40 cm, 44 cm and 43 cm. Can we conclude that
the average rainfall during the last five years is less than the normal rainfall? (Given
f4 (0.05) = 2.132).
r ■. The heights of 8 males participating in an athletic championship are found to be 175 cm, 168
cm, 165 cm, 170 cm, 167 cm, 160 cm, 173 cm and 168 cm. Can we conclude that the average

w
height is greater than 165 cm? (Given tj (0.05) = 1.895).
\ 2. The mean weekly sales of chocolate bar in general stores was 146.3 bars per store. After an

F lo
advertising the mean weekly sales in 22 stores for typical week increased to 153.7 bars and
showed a standard deviation of 17.2. Was the advertising campaign successful? (Given
/2i (0.05) =2.08).

ee
13. The foreman of ABC mining company has estimated the average quantity of iron ore

Fr
extracted to be 36.8 tonnes per shift and the sample standard deviation to be 2.8 tonnes per
for
shift, based upon a random selection of 4 shifts. Consider a 90% confidence interval around
this estimate. (Given tg (0.1) = 2.353).
ur
14. A random sample of size 20 from normal population gives a sample mean of 42 and
s

standard deviation of 6. Test the hypothesis that the population mean is 44. (Given
ook
Yo

fl9 (0.05) = 2.09).


eB

1 - . A random sample of 10 boys had the following I.Q.s.: 70,120,110,101,88,83,95,93,107,100


Do these data support the assumption of a population mean I.Q.s of 100? (Given
fg (0.05) = 2.262). Also, find the 95% confidence interval for the population mean.
our
ad

1 f,. The manufacturer of a certain make of electric bulbs claims that his bulbs have a mean life
of 25 months with a standard deviation of 5 months. A random sample of 12 such bulbs
gave the following values:
Y
Re

Life in months: 24 26 32 28 20 18 23 27 29 34 20 28
nd

Can you regard the producer's claim to be valid at 1% level of significance?


Fi

_ANSWERS
1. Yes 2. Not significant Yes 4, Process is not under control
5. No 6. Yes 7 [11.99,19.61]
8. [51.32,54.68], [50.67,55.33] 9. [39.902, 43.098], [39.29, 43.71]
10. No ■1. Yes 12. Yes )2. [34.5,39.10]
14. True 15. Yes; [87.494,107.906] 16. Yes

21.5.2 TEST OF SIGNIFICANCE OF THE DIFFERENCE BETWEEN THE MEANS OF TWO


RANDOM SAMPLES (INDEPENDENT SAMPLES)
Suppose two independent random samples .Vj, X2,...x,, and t/j, \/2'--^¥n2 sizes and ;i2
respectively have been drawn from the same normal population. Let and X2 be the means
and let and S2 be their standard deviations. In order to test whether the two sample means X 1
and X2 differ significantly or not, we calculate the statistic t given by
INFERENTIAL STATISTICS 21.15

t =
X1-X2 or, t = Xi-X2^. »1»2
1 1 S fl-j + II2
S I- +
V»1 «2

, where S =
+ »2S2^
^ n-i + »2 - 2
The statistic t follows i-distribution with v = +112 - 2 degrees of freedom.

KLMAHK Wc/jrtW —"li ^= l (Xj -X-if rtJirf S2^ ^£(./,--X2)^


”2/=]

ow
II'
"1

Z (-Vi-Xi)" + IVi-Xlf
1 =1 i =l

+«2“2

e
are taken from assumed means. In such cases, S is

re
When the actual means are infraction the deviations

Frl
given by

F
\2 \2
”1 1 fll "2
1 2 1
S2 = 1
+ I I

”1 +”2 “ 2 ;_i ”1 1=1 n


/ =! ,i = l
ou
;/ i = 1,2,... ,n-[ and dj = i/j - A2 i = 1, 2,..., 112
or
xvhere diI = X;I - A 1
kfs
ILLUSTRATIVE EXAMPLES
oo

EXAMl’Ui ! for the follozoing data examine if the means of two samples differ significantly:
Y

Mean Standard deviation


B

Size
6 40
Sample I:
re

5 50 10
Sample II:
oYu

(Given t^{0.05) =2.262)


ad

SOLUTION We have,
H-j = 6, X| = 40, = 8, ?i2 =5, X2 =50 and $2=10
d

"1^1^ +”2^2^
in
Re

^ H-] + }t2 ~ 2
F

6x 64 + 5x100 884 29.732


S = = 9.910
6 + 5-2 9 3

We define

Null hypothesis Hq : The difference in the means of two samples is not significant.
Alternate hypothesis Hi: Means of two samples differ significantly.
Let f be the sample statistic given by
X1-X2-X «1”2
t = —i
s ill + '^2

t =
40-50 ,, 675 -10 30 -10x 1.651
= -1.666
9.910 ^\6+5~ 9.910 V 11 9.910
The sample statistic t follows f-distribution with v=(6 + 5-2)=9 degrees of freedom. We shall
now compare this calculated value of f with the tabulated value for 9 degrees of freedom at a
given level of significance. It is given that tg (0.05) = 2.262.
21.16
APPLIED MATKEMATICS-Xn

We find that; calculated 11 \ =1.666 < tabulated fg (0.05). So, we accept the null hypothesis at 5%
level of significance. Hence the difference in sample means is not significant.
‘ AMI-U-,: Tzvo batches of the same product are tested for their mean life. Assuming that the lives of the
product follow a normal distribution with an unknozun variance, test the In/pothesis that the mean life is
the same for both the branches, given the follozuing information:
Batch Sample size Mean life in hrs Standard deviation
Batch I 10 750 12
Batch II 8 820 14
(Use t^(,{0.05)^ 2.2120)
SOLUTION We define

Null In/pothesis Hq : Mean life is same for both the batches.


Alternate hypothesis : Two batches have different mean

w
lives.

The sample statistic t is given by

F lo
t = X
, where S =
s + "2 +772-2

ee
We have.

Fr
77] -10,7?2 - 8, Xj =750, X2 =820,S|=12 and $2=14
2 2
"1^1 +”2^2
^ 77| + 772 ~ ^
for
ur
10x144 + 8x196 3000
S = = VIM =13.711
s
=>
ook

10 + 8-2 16
Yo

750-820 10x8 -70 80 -70 -140


-Vio =
eB

t = X
X 3.162= -10.762
13.711 VlO+8 13.711 V18 13.711 3 41.133

The sample statistic follows Student's f-distribution with v =(77i+?72-2) =16 degrees of
our

freedom. Let us now compare the calculated value of t with the tabulated value of f at a given
ad

level of significance. It is given that fjg(0.05) =21.20.


We observe that Calculated | f| =10.762 > 2.120 = Calculated (0.05). So, we reject the null
Y

hypothesis at 5% level of significance and hence accept the alternate hypothesis. Hence, the
Re
nd

mean life for both the batches is not the same.


Fi

liXAMPM :
Samples oftivo types of electric light bulbs were tested for length of life andfolloiving data
were obtained:

Type I Type II
Sample size 77j = 8 7?2 =7
Sample means Xi = 1234 hrs X2 =1036 hrs
Sample S.D's = 36 hrs S2 = 40 hrs
Is the difference in the means sufficient to zvarrant that type I is superior to type II regarding length of
life? (Given ti3 (0.05) = 2.216)
SOLUTION We define

Null hypothesis Hq : Two types I and II of electric bulbs are identical.


Alternate hypothesis : Type I bulbs are superior to type II bulbs.
Let the sample statistic t be given by
21.17
INFERENTIAL STATISTICS

2 2

t =
X1-X2 X
»1»2
, where S =
”l^l +''2''^2
s \ }J| + H2 \ + ;i2 - 2
Now,
8x 36^+7x40^ I10368 + 11200 = 1659.076=40.731
^ /i-j + JJ2 ”2 ^ 8+7-2 M 13

1234-1036 8x7 198


t = X 1.932 = 9.391
40.731 8+7 40.731

The sample statistic 't' follows Student's f-distribution with v=(8 + 7~2) = 13 degrees of
freedom. Let us now compare this calculated value with the tabulated value of f at a given level
of significance. It is given that f|3 (0.05) = 2.16.
We find that Calculated | f | = 9.391 > tabulate (0.05). So, the null hypothesis is rejected at 5%

w
level of significance. Hence, the two types of electric bulbs differ significantly. Further, since
is much greater thanX2, we conclude that type 1 bulbs are definitely superior to type 11 bulbs.

Flo
Samples of sales in similar shops in towns A ami B regarding a new product yielded the
following information:

ee
For town A : X-i = 3.45 IX;I = 38 = 228 11-1=11

Fr
For town B: Xi1 =4.44 Zy,-=40 = 222 112 - 9

is there any evidence of difference in sales in the two towns? (Given f-jg (0.05) - 2.10)
for
ur
SOLUTION We define

Niillln/pothesis HQ-.H-xereisno significant difference in sales in similar shops in towns/I and S.


s
Alternate hypothesis : Sales in similar shops in towns A and B are not same.
k
Yo

The sample statistic t is given by


oo

2 2
Xi -X2= niii2
eB

t = —I X , where S =
s + )l2 /?! +^2 -2
We have, = 11, X^ = 3.45, 1.^^- = 38, = 228
r
ou
ad

= 9, X2 = 4.44, Ii/,- = 40, = 222


Y

S^ = + ?l2 ^2^
JI-] + ^2 “ 2
Re
nd

Z(j;-X,)^+I(y,--X2)^ lAr;^-HlXi^ + I.y-K2X2^


Fi

?j-j +112 -2 n^ +112-2


^2 228-ri X (3.45)^ + 222 - 9x (4.44)^ 450 -130.93 -177.42 141.65 = 7.869
® " 11+9-2 18 18

S = V7.869 =2.805
X1 X2 »i»2
f = —

S n^ +112

3.45-4.44 11x9
f = ^ 1 =-0.353 X 2.22 =-0.784
2.805 Vll+9
-2=11+9-2 = 18
The sample statistic 't' follows Student's /-distribution with v=«i +JI2
degrees of freedom. Let us now compare the calculated 11 \ with the tabulated value of / at a
given level of significance. It is given that fjg (0.05) = 2.10. We find that the calculated ] f | is less
than the tabulated fig (0.05).So, the null hypothesis is accepted a 5% level of significance. Hence,
there is no evidence of difference in sales in the two towns.
21.18 APPLIED MATHEMATICS-XII

LXAMiM I 5 Two different types of drugs A and B were tried on certain patients for increasing weight, 5
persons were given drug A and 7 persons were given drug B. The increase in weights in pounds is given
below:

Drug A: 8 12 13 9 3
Drug B: 10 8 12 15 6 8 11

Do the two drugs differ significantly with regard to their effect in increasing weight (Given
ilO (0.05) =2.23) ^ ^
SOLUTION We define

Null hypothesis Hq : Two drugs do not differ significantly with regard to their effect of
increasing weight.
Alternate hypothesis Hj: Two drugs differ significantly with regard to their effect of increasing

ow
weight.
The sample statistic t is given by
X1-X2
|z(^-X,)2 + S(K-X2)2j
«1«2 1
t = —

, where =
S +«2 +ri2-2

e
Fl
re
Let us now compute Xj, X2 / ^ -^1)^ and S (y,- -X2)^.

F
Drug A
ur
DrugB
Xi-X^=Xi-9 (^- -Xi)^ y/ or
yi~^2 -10 (» -X2)2
sf
8 -1 1
k
10 0 0
Yo
oo

12 3 9 8 -2 4
B

13 4 16 12 2 4
re

9 0 0 15 5 25
3 -6 36 6 -4 16
u
ad
Yo

8 -2 4

11 1 1
d

IXi =45 Z(:^-Xi)=0 Z(:ti-Xi)2=62


Re

^Vi=70 2(y,-X2)=0 =54


in
F

1 45
1 ^ 70
— =10
n2 7

S^=
«1 +«2 -2 |z(^;-X,)2 +Z(k.-X2)2|
S2 =
5^1-^(62+5.4)=11.6
S = VlL6 =3.406

f = ~^2
S y«j+n2
9-10 5x7 1 35 17078
f = ^ ^ ^ __ = -0501
3.406 V5 +7 3.406 V12 3.406
21.19
INFERENTIAL STATISTICS

The sample statistic 7' follows Student’s f-distribution with v =(5 + 7-2) =10 degrees of
freedom. Let us now compare the calculated 11 \ with the tabulated value of t at a given level of
significance. It is given that (0.05) =2.23. We find that the calculated | f | is less than the
tabulated t^Q (0.05) = 2.23. So, the null hypothesis is accepted at 5% level of significance. Hence,
the null hypothesis Hq holds true i.e. the drugs A and B do not differ significantly with regard to
their effect in increasing weight.
RIINIAKI- In the above example and X2 come out to be integral values
_ and hence the direct method of
computing I (.r,- -X^)^ and Z (pj -Xj)^ is used. In case X^ and (or) X2 out to be fractional, then
2
the step deviation method is convenient for computmg S .
EXAMlM I !■ The heights in inches of 6 randomly chosen sailors and 10 randomly chosen soldiers are given

as under:
Sailors: 63 65 68 69 71 72

Soldiers: 61 62 65 66 69 69 70 71 72 73

Do these figures show that the soldiers are on an average shorter than sailors? (Given t-^^ (0.05) = 2.15)

w
We define

F lo
SOLUTION

Null hypothesis Hq : Soldiers are not on average shorter than sailors.


Alternate hypothesis : Soldiers are on an average shorter than sailors.
The sample statistic t is given by

ree
t = X »1»2 ^
for F
s 111 + »2'
n2 n2

f u-)
1 ”1 1 1
r
where = di
You

"2^/ = l
oks

■'■”2 1 = ] "1
eBo

, where dj = Xj - A-i; i =1, 2 iij and dj =yj - Aj }i = 1, 2 U2


— — 2
Let us now compute X^, X2 and S .
ad
our

Sailors Soldiers

X-t i1j=Xi-A-^=Xi-68 Vi 4'=i/,--A2=y,--66


Re
dY

63 -5 25 61 -5 25
Fin

65 -3 9 62 -4 16

68 0 0 65 -1 1

69 1 1 66 0 0

71 3 9 69 3 9

72 4 16 69 3 9

70 4 16

71 5 25

72 6 36

73 7 49

Z^- =0 Zd,-^ =60 ZtV =18 Zd:^ =186


I
21.20
APPLIED MATHEMATICS-XII

Wehave,nj=6 and U2=10.


From the above table, we find that
/4 1
= 6S, A2=66,Tcij =0,Tcij^ =60.Idj' =18 and I^^:^=186
1 18
X-[ = /\i H— Z dj = 68 H— = 68, Xt = + I 4' = 66 + = 67.8
6 ^ »2 10

1
S2 =
//-] + ~2 ”1 ”2

=> S2 =
1 0
● 60 — +186
(18)^ 1 213.6
6 + 10-2 6
(60 + 186-32.4) = = 15.2571
10 14 14

ow
S = Vl 5.2571 = 3.9060
f = ^1 ^^2 X
”1»2
s ”l+”2

e
68-67.8 6x10 _ 0.2
t = X 1.9365 = 0.0991

re
3.9060 6 + 10 ~ 3.9060

rFl
F
The sample statistic 't‘ follows Student's f-distribution with v =«-j+«2-2 =(6 +10-2) =14
degrees of freedom. Let us now compare this calculated value of | f | with the tabulated value for
14 degrees of freedom at a given level of significance. It is given that

r
(0.05) = 2.15. Clearly, the
ou
fo
calculated value of|f| is much less than the tabulated value. So, the null hypothesis Hq is
accepted at 5% level of significance. Hence, on an average soldiers are not shorter than sailors.
ks
EXERCISE 21.2
oo

1. The mean life of a sample of 10 electric bulbs was found to be 1456 hours with standard
Y
eB

deviation of 423 hours. A second sample of 17 bulbs chosen from a different batch showed a
mean life of 1280 hours with standard deviation of 398 hours. Is there a significant
difference between the means of the two batches?
ur

2. Strength tests carried out on samples of two yarns spun to the same count gave the
ad

following results:
Yo

Size of sample Smnple mean Sample size Sample Variance


Yarn A
d

4 50 42
Re

Yarn B
in

9 42 56

The strengths are expressed in kg. Is the difference in mean strengths significant of real
F

difference in the mean strengths of the sources from which the samples are drawn?
3. Samples of two types of electric bulbs were tested for length of life and the following data
were obtained :

T\/pe I Type II
Number of bulbs in the sample 8 7
Mean (in hours) 1134 1024
Standard deviation (in hours) 35 40

Is the difference in the sample means significant? (Given (0.05) = 2.16).


4. I.Q. test on tw'o groups of boys and girls gave the following results:
Girls: Xi=78 =50 s, =10
Boys: X2=73 =100 S2=15
Is there a significant difference in the mean scores of boys and girls? (Given ^23 (0.05) = 2.07)
21.21
INFERENTIALSTATISTICS

5. Two methods of performing a certain operation are compared. The following data are
obtained:
Xj =505 2 =95
n-j =15
»,=12 X2=57.2 S2^=5.7
Is there a significant difference in the means of the two methods at 5% level of significance?
(Given f25 (0.05) = 2.06).
6. The meansof the random samples of sizes 9 and 7 are 196.42 and 198.42 respectively. The
are 26.94 and 18.74
sum of the squares of deviations from the respective means
respectively. Can the samples be considered to have been drawn from the same normal
population?

ow
7. The I.Q.'s (intelligence quotients) of 16 students from one area of a city showed a mean of
107 with a standard deviation of 10 while the I.Q.’s of 14 students from another area of the
city showed a mean of 112 with a standard deviation of 8. Is there a significant difference
between the I.Q.’s of the two groups at (i) 1% and (ii) 5% level of significance?
8. Below are given the gain in weiights (in kg^) of pigs fed on two diets A and B.

e
Gain in weight

re
Diet A: 25 32

Fl
30 34 24 14 32 24 30 31 35 25

F
DietB: 44 34 22 10 47 31 40 30 32 15 18 21 35 39 22
ur
Test, if the two diets differ significantly as regards their effect on increasing in weight

r
((25 (0.05) = 2.06).
9. In a certain experiment to compare two types of animal foods A and
fo B, the following results
ks
of increase in weights were observed in animals;
Yo
oo

2 3 4 5 6 7 8
Animal number 1
47 50 52 53
eB

Food A 49 53 51 52
Increase

weight in lb Food B .52. 55 52 53 50 54.. _ 54 _ 53

Assuming that the two samples of animals are independent, can we conclude that food B is
ur
ad

better than food A?


Yo

lO. The marks obtained by two groups of students in Mathematics test are given below:
Group A Group B
d

15 11
Number of students:
Re
in

42 38
Mean Marks:
15
F

Standard deviation of Marks : 10

On the basis of this data, can it be concluded that there is a significant difference in the
mean marks obtained by two groups? (Given (14 (0.05) = 2.064).
11 . Two kinds of fertilizers were applied to 15 plots of one acre, other conditions remaining the
same. The yields in quintals are given below:
14 20 34 48 32 40 30 44
Fertilizer I:
18 22 28 40 26 45
Fertilizer II: 31

Examine the significance of difference between the mean yields due to the use of difierent
kinds of fertilizers. (Given (^3 (0.05) = 2.16)
12. Two different types of drugs A and B were tried on certain patients for increasing weight. 6
persons were given drug A and 8 persons were given drug B. The increase in weight in
pounds is given below:
7 10 13 12 4
Drug A:
6 18 16 9 3
12 8
Drug A:
21.22
APPLIED MATHEMATICS-XII

Do the drugs differ significantly with regard to their effect i


in
increasing weight? (Given
fl2 (0.05) =2.16).
13. A group of 5 paHents treated with medicine A weigh 42,39,48,60 and 41 kg; second group
of/patients from the same hospital treated with medicine B weigh 38,42,56,61,68,69 and
62 kg. Do you agree with the claim that medicine B increases the weight significantly^
(Givenfjo (0.05) =2.281). & y-
ANSWERS
1. No
-■ Not significant 3. Sigiiificant

ow
4. Significant
>- Significant 6. No
9. Yes
7. Not significant 8. Do not differ significantly
10. Not significant n ● No significant difference
12. No 13. Yes

e
re
F
Frl
ou
sr
kfo
oo
Y
reB
uY
ad
do
in
Re
F
21.23
INFERENTIAL STATISTICS

Table I

Significant values (a) of t-distribution (Two-Tail areas)


P[|fl>fy (a)] = g
degrees of Probability (Level of Sigruficance)
freedom 0.50 0.10 0.05 0.02 0.01 0.001
M
12.71 31.82 63.66 636.62
1 1.00 6.31
2.92 4.30 6.97 6.93 31.60
2 0.82
3.18 4.54 5.84 12.94
3 0.77 2.35

ow
2.78 3.75 4.60 8.61
4 0.74 2.13
2.57 3.37 4.03 6.86
5 0.73 2.02

1.94 2.45 3.14 3.71 5.96


0.72

e
6
3.50 5.41

re
7 0.71 1.90 2 37 3.00

1.86 2.31 2.90 3.36 5.04

Frl
8 0.71

F
2.26 2.82 3.25 4.78
9 0.70 1.83
2.23 2.76 3.17 4.59
10 0.70 1.81
ou
or
2.20 2.72 3.11 4.44
11 0.70 1.80 kfs
1.78 2.18 2.68 3.06 4.32
12 0.70
2.16 2.65 3.01 4.22
0.69 1.77
oo
13
1.76 2.15 2.62 2.98 4.14
14 0.69
Y

2.95 4.07
eB

15 0.69 1.75 2.13 2.60

2.12 2.58 2.92 4.02


ur

16 0.69 1.75
oY

1.74 2.11 2.57 2.90 3.97


17 0.69
ad

1.73 2.10 2.55 2.88 3.92


18 0.69
1.73 2.09 2.54 2.86 3.88
0.69
d

19
1.73 2.09 2.53 2.85 3.85
20 0.69
in
Re

2.52 2.83 3.83


F

21 0.69 1.72 2.08


2.07 2.51 2.82 3.79
22 0.69 1.72
1.71 2.07 2.50 2.81 3.77
23 0.69
1.71 2.06 2.49 2.80 3.75
24 0.69
1.71 2.06 2.49 2.79 3.73
25 0.68

2.06 2.48 2.78 3.71


26 0.68 1.71
1.70 2.05 2.47 2.77 3.69
27 0.68
1.70 2.05 2.47 2.76 3.67
28 0.68
2.05 2.46 2.76 3.66
29 0.68 1.70
1.70 2.04 2.46 2.75 3.65
30 0.68
1.65 1.96 2.33 2.58 3.29
00 0.67
CMPM

INDEX NUMBERS

22.1 INDEX NUMBERS

w
Consider the following data :

Year 1994 1995 1996 1997 1998

Flo
Price per

e
metre (in

re
Cotton Cloth 10 12 15 18 20

F
35 39 42 45
Polyster Cloth 30
ur
r
We find that the rate of cotton cloth has increased fo
by ? 10 over the period 1994-1998 and the rate
of polyster cloth has increased by ? 15 over the same period of time. From this It appears that the
ks
increase in price of polyster cloth is more than that of cotton cloth. However, if we compute the
Yo
oo

rise in the percentages taking 1994 as the base year, we find that
B

Percentage rise in the price of polyster cloth ~ ^ ^


re

10
= _ X 100 = 100
Percentage rise in the price of cotton cloth 10
u
ad
Yo

This shows that percentage rise in the cotton cloth is more than that of polyster cloth. If we
consider ? 100 as the price of both types of cloths in 1994, than we find that the prices of cotton
and polyster cloths in 1998 are ^ 200 and ? 150 respectively. In statistical language we say that
d
Re
in

the index numbers for cotton and polyster cloths are 200 and 150 respectively when 1994 is taken
as the base year. Thus, index numbers are indicators which measure percentage changes in a
F

variable or a group of related variables over a specified period of time. If we say that the index
number of wholesale prices is 143 for Jan. 1998 compared to Jan. 1997, it means that there is net
increase of 43% in the prices of wholesale commodities during the year. Similarly, if the index of
industrial product is 115 in 1997 compared to 1996, it means that there is a net increase in the
industrial production to the extent of 15%. Thus, index numbers relate a variable or variables in
a
given period to the same variable or variables in another period, called the base period.
Some prominent definitions of index numbers are given below :
DEFINITION 1 'An index number is a statistical measure desi^ied to show changes in a variable or a
group of related variables ivith respect to time, geographic location or other characteristics such as income,
profession etc." Spiegel
DEFINITION 2 " Index numbers are used to measure the changes in some quantity zuhich we cannot
observe directly". Dr. A.L. Bowley
22.2 APPLiED MATHEMATICS-XII

DEFINITION 3 "In its simplest form, an index number is nothing more than a relative number, or
relative zvhich expresses the relatiotiship between two figures, where one of thefigures is used as a base
Morris Hamburg
DEFINITION 4 "An index number is a device which shows bp its variation the changes in a magnitude
zvhich is not capable of accurate measurement in itself or of direct valuation in practice". Wheldon
DEFINITION 5 "Index numbers are devices for measuring differences in the magnitude of a group of
related variables . Croxton and Cowden

ow
22.2 CHARACTERISTICS OF INDEX NUMBERS
Following are some important characteristics of index numbers:
1. EXPRESSED IN PERCENTAGE Index numbers are expressed in terms of percentage to measure
the extent of relative change. However, the percentage sign (%) is never used.

e
2. ABSOLUTE NUMBERS Index numbers are free from units. Index numbers describe in one

re
single figure the percentage increase or decrease in a group of related variables under study.
The group of variables may be the prices of a specified set of commodities, the volume of

Flr
F
production in different sectors, etc.
3. INDEX NUMBERS ARE SPECIALISED AVERAGES As discussed in the chapter on Measures of
ou
Central Value, an average is a single figure representing a group of figures. Averages are used to

sr
compare two or more series which are expressed in the same units. We cannot compare two

fo
series by computing their averages if they have different units. However, it is not so with index
k
numbers. If two or more series are expressed in different units or they are composed of different
types of item, their index numbers can be compared as they are specially designed for the
oo
purpose of comparison in situations where two or more series are expressed in different units.
Y

Therefore, index numbers are called speciali2ed type of averages.


reB

4. INDEX NUMBERS MEASURE THE EFFECT OF CHANGE OVER A PERIOD OF TIME Index
numbers are used for measuring changes over a period of time. Thus, we can compare change in
uY

prices of different commodities, wages, exports, imports, industrial production, etc., at two
different times.
ad
do

5. INDEX NUMBERS MEASURE CHANGES NOT CAPABLE OF DIRECT MEASUREMENT Index numbers
measure changes in magnitude of a variable or a group of variables which are not capable of
direct measurement due to composite and complex character of the phenomenon. Examples of
in

such phenomena are cost of living, price level, business activity, etc. The changes in business
Re

activity in a country are not capable of direct measurement but it is possible to study relative
F

changes in business activity by measuring the variations in the values of those factors which
affect business activity, and which are capable of direct measurement.
6. INDEX NUMBERS ARE TOOLS TO MEASURE RELATIVE CHANGE Index numbers by their very
nature measure rela tive change in the values of a variable or a group of variables over a period of
time or between places. If wholesale price index number of 2007 is 190 as compared to 2000, it
means price level of 2007 is increased by 90%.
22.3 USES OF INDEX NU^^PERS

The significance and uses of index numbers can be best appreciated by the following points.
1 INDEX NJP^DF^.'' '●● ●
r*’- AS ECONOMIC BAROMETERS Just as barometers in physics measure

atmos- pheric pressure, index numbers in statistics measure the level of business and economic
activities of a country. That is why index numbers are termed as 'economic barometers', or
barometers of economic activity'. Index numbers measure the pulse of economy of a country
and measure the ups and downs in general economic condition of a country. Indices of prices,
industrial production, agriculture production, foreign exchange, reserve bank deposits, etc..
22.3
INDEX NUMBERS

determine the level of business activity of a country and these indices can be combined into a
composite index which could act as an economic barometer.
2. INDEX rJUMBERS HELP US IN FRANtiNG SUITABLE POLICIES Most of the economic and business
policies are framed with the help of index numbers. Index numbers work as tools for the
management of any organisation for efficient planning and formulation of business policies. For
example the increase in dearness allowance of the employees is decided on the basis of the cost
of living index. Also, relative whole-sale and retail price index numbers and industrial and
agriculture product index numbers help in economic and business policy making.
conditions, there are other fields also
In addition to the evaluation of business and economic
where index numbers are used. For example, health authorities prepare indices to display
changes in the adequacy of hospital facilities. Also educational research organisations have
devised formulae to measure changes in effectiveness of school systems. Similarly,
psychologists measure intelligence quotients which essentially index numbers comparing a

w
persons's intelligence score with that of an average of his (or her) age.
3. INDEX NUMBERS ARE HELPFUL IN DETERMINING TRENDS AND TENDENCIES Since index

F lo
number measure changes over a period of time. Therefore, measuring the changes for different
intervals of time one can study the general trend of the phenomenons under study. For example,
by examining the index number of industrial production for the last few years, we can say about

ee
the trend of production whether it is increasing or decreasing. With the help of index numbers of

Fr
prices, wages, income, demand etc., a business executive can decide about whether a new
product should be launched or whether there is scope for exploring new markets or whether
for
existing pricing and production policies need a change. Also, by examining the trend of
ur
phenomenon under study we can determine the effect of seasonality.
4. INDEX NUMBERS ARE USEFUL IN DEFLATING Index numbers are very useful in deflating
s

(adjusting) i.e. they are used to adjust the original data for price changes, or to adjust wages for
ook
Yo

cost of living changes and thus transform nominal wages into real wages. They are also used in
adjusting national income on the basis of constant prices, to enable us to find whether there is
eB

any change in real income of the people. With the help of appropriate index numbers nominal
mcome can be transformed into real income and nominal sales into real sale.
r

5. INDEX NUMBERS ARE USED TO MEASURE THE PURCHASING POWER OF MONEY Index numbers
ad
ou

are useful in finding out the intrinsic worth of money as contrasted with its nominal worth. We
read in newspapers that the purchasing power of the Indian rupee in 2008 is only 10 paise as
Y

compared to its purchasing power in 1985. This means that a person who was having yearly
Re

income of ? 12000 in 1985 should have yearly income of ? 12000 in 2008 to maintain the same
nd

standard which he was maintaining in 1985. This helps in determining the wage policy of a
Fi

country.
22.4 TYPES OF INDEX NUMBERS

The various types of index numbers used in business and economics are:
(i) Price Index (ii) Quantity Index
(iii) Value Index (iv) Special Purpose Index
Each index number is designed for particular purpose. Price index numbers measure changes in
prices between two points of time and they are also used for the comparison of the prices of
certain commodities. Generally, when we speak of index numbers, it refers to price index
numbers. Throughout this chapter the term 'index number' will mean price index number.
Quantity index numbers measure changes in physical volume of goods produced or consumed.
A value index measures the change in actual values between the base and the given period.
Besides these, there are some special purpose index numbers. The discussion of such index
numbers is beyond the scope of this book.
22.4
APPLIED MATHEMATICS-XII

22.5 PROBLEMS IN THE CONSTRUCTION OF INDEX NUMBERS


Following are some problems which frequently occur in the construction of index numbers.
These problems must be well defined and one should give a careful thought to these problems
before the construction of index numbers.
(i) Definition of purpose (ii) Selection of base period (Hi) Selection of number of item
(iv) Selection of source of data (v) Price quotations (vi) Choice of an average
(vii) Selection of an appropriate method
Wo shall now have detailed discussion on the above problems.

ow
1. DEFINITION OF PURPOSE Before starting construction of index number the purpose of
constructing index numbers must be very clearly decided. There is no all purpose index number
as every index number has its own particular uses and limitations. So, it is important to know
what is to be measured and how these measures are used. For example, if a price index is to be

e
constructed for measuring consumers 'price, it should not include wholesale prices. Likewise, if

re
a price index is constructed to measure the cost of living of middle class families in a particular
city or region, one should not include items or areas which relate to the labour or agriculatural

Flr
F
worker.

2. SELECTION OF BASE PERIOD The base period of an index number (also called the reference
ou
period) is the period against which comparisons are made. It may be a year, a month or a day.

sr
Wlienever index numbers are constructed a reference should be made to some base period. The

fo
index for base period is always taken as 100. For making comparison over a period of time it
k
should be noted that base period (reference period) should not be too far from the period of
current year. It should be fairly recent period. Also the base period should be a normal one i.e. it
oo
should be free from abnormalities like wars, earthquakes, famines, booms, depressions, etc.
Y

3. SELECTION OF NUMBER OF ITEMS While constructing index numbers it is not possible to


reB

include all commodities or items. Therefore, one has to select a sample. In selecting a sample, the
following points must be considered:
uY

(i) The total number of commodities in the sample should be neither too small nor too large,
(ii) The items selected should relevant to the purpose of the index,
(iii) Only standardized or graded items must be included in the sample so that they can be
ad
do

easily identified after a time lapse.


4. SELECTION OF SOURCE OF DATA Since the sources of data are scattered over a large
in

geographical region, therefore, we have the problem of reliability and comparability of data. If
Re

the data is not selected carefully, there is always a chance of getting misleading results.
F

Therefore, generally data is obtained from the reports published by business houses, chamber of
commerce and government agencies, etc.
5. PRICE QUOTATIONS After selecting the commodities for the construction of data the next
problem is to obtain price quotations for these commodities. The prices of many commodities
vary from place to place and even from shop to shop in the same market. It is not possible to
obtain price quotations form all places where a particular commodity is available. Therefore, a
selection of representative places and of representaHve persons has to be made. Tlie places
should be those which are well known for trading for that particular commodity and is
purchased or sold in large quantities. After the selection of places from where the price
quotations are to be obtained, the next thing is to appoint some persons or institutions who
would supply the price quotations from time to time. One must be very careful regarding the
unbiasedness of the price reporting agency. In order to check the accuracy of price quotations
supplied by an agency, quotations should be obtained from more than one agency. Information
published in reliable journals or magazines about prices ruling in various places may be
utilized.
22.5
INDEX NUMBERS

There are two methods of quoting prices: (i) money prices, and (ii) quantity prices. In case of
money prices, prices are quoted per unit of commodity, for example, rice at ? 2000 per qumtal
(100 kg). While in case of quantity price — prices are quoted per unit of money, for example, nee
1/20 kg for one rupee. The former method is convenient and free from any confusion. So, it is
generally adopted while quoting prices.
Another considerahon is regarding to the type of quotation, whether the wholesale prices
or

retail price are required. The choice would mainly depend upon the purpose of the index
number. A wholesale price index or general index requires wholesale price quotations. But for
constructing a cost of living index number real price quotations will be desirable.
6. CHOICE OF AN AVERAGE In the construction of an index number any average such as
arithmetic mean, geometric mean, harmonic mean, median and mode can be used. So, a
decision has to be made about the choice of a particular average for constructing index number.
Median, mode and harmonic mean are almost never used in the construction of index numbers
because of their being erratic. Basically a choice has to be made between arithmetic and
geometric mean. The geometric mean is difficult to calculate in comparison to AM. So, generally

w
AM is used. Theoretically geometric mean is the best average in the construction of mdex

F lo
numbers.
7. SELECTION OF AN APPROPRIATE METHOD There are various methods of constructing index
numbers. The problem is that of selecting the most appropriate method. No single method is

ee
appropriate for all types of index numbers. So, the choice of the method mainly depends upon

Fr
the purpose of index number and on the data available.
22.6 METHODS OF CONSTRUCTING UNWEIGHTED INDEX NUMBERS
for
The following are the methods of constructing unweighted index numbers:
r
(i) Simple Aggregative method (ii) Simple Average of price relatives method.
You
s

We shall now discuss these two methods one by one.


ook

22.6.1 SIMPLE AGGREGATIVE METHOD


eB

This is the simplest method of constructing index numbers. In this method, the total of current
year prices for the various commodities is divided by the total of base year prices and the
our

quotient is multiplied by 100.


ad

Symbolically,
I Pi X 100
Pin =
dY

^ Po
Re

where. I Pi = Total of current year prices of various commodities,


Fin

I Pq = Total of base year prices of various commodities and,


?0i = Current year Index Number
We may use the following algorithm to compute index numbers by simple aggregative method:
ALGORITHM

STEP 1 Compute I Pq i.e. add the base year prices for various commodities.
STEP 11 Compute I pi i.e. add the current year prices for the same commodities.
step hi Divide ^p-y by .I Pq and muUiply the quotient by 100 to obtain Pqi i.e. compute
I Pi X 100.
^01 -

note 1 The subscript 1 is used for the current year and the subscript Ofor the base year.
note 2 pQi denotes the price index for current year with respect to the given base year.
Following examples will illustrate the above algorithm.
22.6
APPLIED MATHEMATICS-XII

ILLUSTRATIVE EXAMPLES

three
process. The prices of the materials were as shown raze
below: materials A B and C in its manufacturing
^ ^
Commodities
Price in ? in the year 2005 Mcejn ? in the year 2015
A 4 5
B 60 57
C 36 42

Using 2005 as the base year, calculate a simple aggregate price index for 2015.
SOLUTION We prepare the following table for the computation or
construction of price index:
Commodities
Price in ? in the year 2005

w
Price in ? in the year 2015
iPo) Ih

F lo
A 4 5
B 60 57
C 36

ee
42

Fr
S Pq =100 =104

Wehave, and I A = 104 for


104
P,01 _

xlOO = X 100 = 104


r
100
You

Hence, the price index for 2015 taking 2005 as the base year is 104.
s
ook

iAAMPl 1.2 "


2 From thefollowing data, construct price index numberfor 2017 taking 2015 as the base u ear:
eB

Commodity Price in 2015 (in Price in 2017 (in


A
our

50
ad

90
B 40 70
C 80 120
dY
Re

D 110 150
£
Fin

20 30

^LUTION Construction of price index number


Commodity Price in 2015 (in Price in 2017 (in
Po Pi
A 50 90
B 40 70
C 80 120
D 110 150
E_ 20 30

Spo = 300 = 460

We have, I Pq = 300 and Zp^ = 460


22.7
INDEX NUMBERS

^Pl 460
X 100 = X 100 = 153.33
Pot 300
^Po

Thus, the price indexnumber of 2017by taking 2015 as the base year is 153.33, it means that there
is net increase in the price of commodities in the year 2017 to the extent of 53.33 % as compared
to 2015.

l-XAMl’LE 3 Using the simple aggregative me ?thod calculate the price index number for 2016 taking 2010
as the base year from the following data :

Commodities Prices in 2010 (in Prices in 2016 (inX)

ow
50 80

40 60
B

10 20

e
C

re
D

rFl
5 10

F
2 6
£

r
Construction of price index number
ou
SOLUTION
fo
ks
Price in 2010 Price in 2016
Commodities
PO Pi
oo
Y

80
eB

A 50

40 60
B
r

20
ou

C 10
ad
Y

5 10
D
d

2 6
E
Re
in

Zpo =107 Z Pi =176


F

We have, .TpQ=107 and Spi-176


I Pi 176
xl00 = X 100 = 164.48
Pot = 107
^PO

Hence, the price index for 2016 is 164.48 by taking 2010 as the base year, it means that there is net
increase in the prices of commodities in the year 2016 to the extent of 64.48 % as compared to
2010.

EXAMl’LE4 Compute the index numbers for the years 2008 to 2015 by taking 2007 as the base yearfrom
the following data:
Year : 2007 2008 2009 2010 2011 2012 2013 2014 2015
7 8 10 9 10 11
5 6
Price of Commodity 'X' 4
22.8
APPLIED MA-^HEMATICS-XII

SOLUTION
Construction of index numbers taking 2007 as the base year
Year
Price of Commodity 'X' Index Numbers

2007 4 100

2008 5 5/4x100 = 125

2009 6 6/4 X 100 = 150

2010 7 7/4 X 100 = 175

2011
8/4 X 100 = 200

2012 10 10/4 X 100 = 250

w
2013 9 9/4 X 100 = 225
2014

2015

F lo
10

11
10/4 X 100 = 250

11/4 X 100 = 275

e
Fre
[AA\:i'i ' :
For the data given below, compute the index number by taking 2011 as the base year.
for
Year:
2004 2005 2006 2007 2008 2009 2010 2011 2012
Price of Commodity ‘X‘ 4 5 6 7 8 10 9 10 11
r
SOLUTION Computation of index numbers taking 2011 as the base year
You
oks
eBo

Year
Price of Commodity 'X' Index Numbers Taking 2001
as the base year
ad
our

2004 4 4/10x100 = 40
2005 5 5/10 X 100 = 50
Re
dY

2006 6 6/10x100 = 60
Fin

2007 7 7/10x100 = 70

2008
8/10x100 = 80

2009 10 10/10x100=100
2010 9 9/10x100=90

2011 10 10/10x100 = 100

2012 11 11/10x100 = 110

r.'.AMn' i "
For the data given in Example 5, compute index numbers for various years by taking 2004 to
2006 as base period.
INDEX NUMBERS 22.9

SOLUTION Taking 2004 to 2006 as base period means that we have to take an average of prices in
4 +5 + 6
2004,2005 and 2006 as the base price. The average price is 3 " ■
Index numbers taking 2004 to 2006 as the base period

Year Price of Index Numbers (2004


Commoditi/ 'X' to 2006 as base period)

2004 4 4/5 X 100 = 80

2005 5 5/5x100 = 100

2006 6 6/5 X 100 = 120

7/5x100 = 140

w
2007 7

2008 8/5 X 100 = 160

F lo
2009 10 10/5 X 100 = 200

2010 9 9/5 X 100= 180

ee
Fr
2011 10 10/5x 100 = 200

2012 11 11/5 x 100 = 220


for
ur
EXAMPLE 7 The index numberfor the year 2020 taking 2015 as base yearfor thefolloiuing data zuasfound
to be 125. Find the missing entries in the data if I. pQ = 360.
s

£ F
ook

Commodities: A B C D
Yo

Prices (in f) in 2015 40 60 20 50 X no


eB

Prices (in X) in 2020 55 70 40 no 115


}/

SOLUTION The given data can be presented in the following form :


r
ad
ou

Commodity Price in 2015 (in X) Price in 2020 (in X)


Y

/I 40 55
Re
nd

B 60 70
Fi

C 20 40

D 50 y

£ X 100

f no 115

Z Pq = 280 4- X lpi = 380 + y

It is given that: 2 po = 360 => 280 + x = 360 ^ .t - 80


Zpl xlOO
Now, Poi =

125 =
380+ y xlOO [●.● Fqi = 125 (given)]
360
450 = 380 + 1/ => 1/ = 70
Hence, .v = 80 and y = 70
22.10 APPLIED MATHEMATICS-XII

EXAMPLE
Using 2015 as base year, the index numbers for the price of a commodity in 2016 and 2017
are 118 and 125. Calculate the index numbers for 2015 and 2017, if 2016 is taken as the base year.
SOLUTION Let Pq , Pi and p2 be the prices in 2015,2016 and 2017 respectively.
It is given that the index numbers for the price of a commodity in 2016 and 2017 are 118 and 125
respectively.
Pi
^ X 100 = 118 and ^ x 100 =125
Po Po

We have to compute:
(i) Index number for 2015 taking 2016 as the base year i.e. ^ x 100
Pi

ow
P2
(ii) Index number for 2017 taking 2016 as the base year i.e. xlOO
Pi

(i) Wehave,-^xl00 = 118 => ^xlOO = 100


X 100 = 84.75
Po Pi 118 Pi 118

e
Hence, index number for 2015 taking 2016 as the base year is 84.75.

re
(ii) We have.
Pi
^xlOO = 118 and, ^xlOO = 125
rFl
F
Po Po
P2
X 100

r
ou
Po 125 P2 125 Pt 125
Pi 118
^=
118
=> i^xl00 = fo X 100=105.93
ks
X 100 Pi Pi 118
Po
oo

22.6.2 SIMPLE AVERAGE OF PRICE RELATIVES METHOD


Y

PRICE RELATIVE A price relative is the number obtained by expressing the price for the current period as a
B

percentage of the price of the base period.


re

Thus, if Po and pi denote the commodity price during the base period and the current period
respectively, then
ou

Price relative = Pi xlOO


Y
ad

Po

In the method of simple average of price relatives, first of all price relatives are computed for the
d

various commodities and then average of these relatives is obtained by using any one of the
in
Re

measures of central value, i.e., AM or GM or HM or Median or Mode. Although any one of these
measures of central value can be used to obtain the price index, but generally either AM or GM
F

are used to compute the average of price relatives.


If n is the number of items (commodities) in the list, then
Pi
Z XlOO
P.01 Pi)
n
(When AM is used)

Pi
Z log XlOO
{Po
and. ^01 = antilog (When GM is used)
n
22.11
INDEX NUMBERS

Z , where P=—x 100


Pi
or, Poi = antilog n Po

We may use the following algorithm.


ALGORITHM
Pi xlOO
STI2P I Compute price relatives for different commodities bp using the formula Po

STEP 11 Compute the sum of price relatives for different commodities computed in step I i.e. compute
Y ^xlOO
^ Pi
Pi xlOO
Z

w
iPo
STEP III Use of the formtda Poi = zvhere n is the nuinber of commodities.
n

Following examples will illustrate the above algorithm.

Flo
ILLUSTRATIVE EXAMPLES

e
re
EXAMPLE 1 Construct the consumer price index number for 2020 taking 2019 as the base pear, and

F
using simple average of price relative method, for the following data:
ur
r
Commodities Price in 2019 {in ?) Price in 2020 {in ?)

Butter 20
fo 21
ks
Yo
Cheese 16 12
oo

Milk 3 3
B

2.80
Eggs 2.80
re

SOLUTION Construction of index number by using AM of price relatives by simple aggregative


u
ad

method:
Yo

Commodities Price in 2019 (in Price in 2020 (in Price relatives — x 100
d

Po Pi Po
Re
in

21 21
Butter 20 — X 100=105
F

20
12
Cheese 16 12 xlOO =75
16

Milk 3 3 -xlOO =100


3
2.80
2.80 2.80 xlOO =100
Eggs 2.80

Pi X100 = 380
Po

Thus, we obtain
Pi xlOO
n = Number of commodities = 4 and, Z =380
Po
22.12
APPLIED MATHEMATiCS-XII

.^1
I xlOO
[Po 380
^01 = => .P01 = 95
n 4

Thus, the price index number for 2020 taking 2019 as tlie base year is 95.
EXAMPLE 2 From thefolloiving data, compute price index bp using simple average of price relatives:

Commodity and unit Price in 2019 (in Price in 2020 (in

Butter (kg) 20.00 21.00

Cheese (kg) 15.00 14.00

Milk (It.) 3.00 3.00

w
Bread (1) 2.80 2.80

F lo
Eggs (doz.) 6.00 8.00

Ghee (1 tin.) 250.00 260.00

ee
SOLUTION Construction of index number by using simple average of price relatives by taking

Fr
1989 as the base year.

Commodity and unit Price in 2019 (in ?)


for
Price in 2020 (in ?) Pi
Price relatives — x 100
ur
Po Pi Po
s

21
Butter (kg)
ook

20.00 21.00 — X 100=105


Yo

20
eB

14
Cheese (kg) 15.00 14.00 — X 100 = 93.33
15

Milk (It.) 3.00 3.00


our

-X 100 =100
ad

3
2.80
Bread (1) 2.80 2.80 X 100 =100
2.80
Y
Re

Eggs (doz.) 6.00 8.00 -X 100=133.33


nd

6
Fi

260
Ghee (1 tin) 250.00 260.00 X 100 =104
250

Pi
Z X 100 = 635.66
Po

Thus, we have

Pi
n =
Number of commodities = 6 and, ^ ^xlOO =635.66
IPO
Pi
I X 100
Po 635.66
P01 = 105.94
n 6

Hence, the price index for 2020 taking 2019 as the base year is 105.94.
22.13
INDEX NUMBERS

EXAMPLE 3 The price quotations offive different commodities for years 2015 and 2016 are as follows :
Commodities Price in 2015 jin fi Price in 2016 {in ?)
5 8

B 2 6

C 6 8

D 6 9

2 4
E

Calculate the price index number for the year 2016 loith 2015 as the base year, using simple average of

ow
price relatives.
SOLUTION Construction of index number

Price in 2016 (in pi Pi


Commodities Price in 2015 (in Pq Price relatizyes — x 100
lo

e
re
A 5

rFl -X100 =160


5

F
13 2 6 -xl00 = 300
2

r
ou
C 6 fo
8 — X 100=133.33
ks
6

D 6 9 -xl00=150
oo

6
Y
B

E 2 4 -X100 = 200
2
re

Y, ^xlOO = 843.33
ou
Y
ad

Number of commodities = 5 and, ^ ^xlOO =843.33


d

Now, =
Po
in
Re

I ^^xlOO
F

iPo 843.33
= 168.66
Poi = n 5

Hence, the price index for 2016 is 168.66. It means that there is net increase in the prices of
commodities in the year 2016 to the extent of 68.66% as compared to 2015.
EXAMPLE 4 Using simple average of relatives method, the price index for 2011, taking 2001 as base year,

was found to be 127. //I Po = 263,//??rf .v and yfrom thefollowing data :


C D E F
Co7nmodities: A B

50 18 25
Prices (in t) in 2001 80 70 .r

61 22 y 32.50
Prices (in ?) in 2011 100 87.50

SOLUTION We have,
Ipo = 263 ^ 243 + -V = 263 => -v = 20
22.14
APPLIED MATHEMATICS-XII

Construction of index number


Commodities Price in 2002 {in ?) Price in 2011 (in ?) Price Relatives — x 100
Po Pi Po

A 80 100 100
xlOO = 125
80
B 70 87.50 8750
X100 = 125
70
C 50 61 61
— xlOO = 122
50

D 20 22 22
—XlOO = no
20

w
E 18 y y ^ 50y
18x100 9

F lo
F 25 32.50 3250
XlOO = 130
25

ree
50]/
I ^xlOO = 612 +

F
Po 9

We have.
for
r
n
= Number of commodities = 6 and, Y ^ x 100 = 612 +
You

■'
Po 9
oks

Poi = 127
eBo

Pi
z XlOO
612+ 50y
;
Po
= 127=> ^ = 127=> 612 + ^=762 => 50y
-^=150 => y
^ =27
our

9
ad

n 6 9
Thus, -V = 20 and y = 27
SIMPLE AVERAGE OF PRICE RELATIVES BY USING GEOMETRIC MEAN
dY
Re

In order to compute index number by simple average of price relatives when geometric mean is
used as an average, we may use the following algorithm:
Fin

ALGORITHM

STEP 1
Compute price relatives for different commodities by using the formula P=^x 100.
Po
STEP 11
Compute logarithms of price relatives obtained in step II.
STEP 111
Obtain Z log P.

Use the formula: Pq^ = antilog S log P 'j


STEIMV
n
, where n is the number of commodities.

Following example illustrates the above algorithm.


EX.-WIPL: .
From the following data construct an index for 2017 taking 2015 as the base by the simple
average of price relatives method using geometric mean for averaging the price relatives
22.15
INDEX NUMBERS

Commodities Price in 2015 (in Price in 2017 (in

A 50 70

B 40 60

C 80 90

D no 120

E 20 20

SOLUTION Construction of index number

ow
Commodities Price in 2015 Price in 2017 Pi logP
Price Relatives P = — xlOO
(in Pq (in Pi Po

70 70 2.1461
/\ 50 — X100 =140

e
Fl
50

re
60 2.1761

F
B 40 60 — X100 =150
40
ur
or
90 90 2.0512
C 80 — xl00=1125
80
sf
120 2.0378
k
120
Yo
D no X100 =109.1
oo

10

20 2.0000
B

£ 20 20 — X100 =100
20
re

^ log P =10.4112
u
ad
Yo

We have,

11=5 and S log P =10.4112


d
Re

I log P' 10.4112 ^


in

^01 = antilog P,01 - antilog = antilog 2.0822=120.9


n 5
F

Hence, the index for 2017 is 120.9. It means that there is net increase of 20.9% in the prices of
commodities over the period of 2 years 2015 - 2017.
I.XAMPLE i> The wholesale price index (or price relative) of rice in 2018 compared to 2015 is 150. If the
cost of rice was ? 24 per kg in 2015, calculate the cost in 2018.
SOLUTION Let the cost of rice be ? Pq and pi per kg in 2015 and 2018 respectively.
We have, Pq = ? 24 and Pqi = 150
Poi = X 100 => 150 = ^
24
X 100 ^ Pi = 36
Po

Hence, the price of rice in 2018 is ? 36 per kg.


REMARK The purchasing power of a rupee in a certain year, relative to some base year is as follows.
Index number for base year
Purcimsingpower =
Index number for current year
22.16
APPLI'i MATHEMATICS-Xll

EXAMPLE 7
Net monthly income of an employee zvas ? 6000 per month in 2010 and ? 9000 per month
in 2015. If consumer price index in 2010 was 150, fmd the consumer price index for 2015, given that net
income of the employee linked to consumer price index.
SOLUTION Let the consumer price index in 2015 be .v. It is given that the net income of the
employee is linked to the consumer price index which means that the standard of living does not
change. So, the value of money remains same.
6000 9000
=>.v = 225
150 .V

Hence, the consumer price index in 2015 is 225.


EXAMPLE During a certain period, the cost of living index number goesfrom 110 to 200 and the salary
of a loorker is also from ? 3250 to f 5000. Does the worker really gains or loses, and bi/how much amount
in real terms?

w
SOLUTION We know that:
Actual wage
Real wage = xlOO

F lo
Cost of living index
52S0
Real wage of ? 3250 = ? x 100 = ? 295450 and,

ee
no

Real wage? 5000 = ? 5222

Fr
X 100 = ? 2500
200

So, the worker actually loses? (295450 - 2500) = ? 45450 for


ur
EXAMPLE
■ With price index of 2011 ns 100, the cost of living index for 2016 is 160 and 2017 is 192. The
salary of an employee increasedfrom ? 20.000 in 2016 to ? 21,000 in 2017. Find whether the real income in
s
2017 has increased or decreased as compared to 2016. Also, calculate if any extra dearness allozoance
ok
Yo

should be paid to him tocompzensateforloss. Also, calculate purchasingpower of rupee in 2016 and2017.
o

SOLUTION The price relative for 2017 compared to 2016 is


eB

192
Rp = — X 100 = 120
160
r

This means that ? 120 in 2017 are equivalent to ? 100 in 2016 i.e. if an article costs ? 120 in 2017,
ou
ad

then its cost in 2016 was ? 100.


100
Y

Wages of ? 21000 in 2017 are equal to wages X 21000=? 17500 in 2016.


120
Re
nd

Tluis, the real income in 2017 has decreased compared to 2016.


Fi

Suppose ? .V are paid as dearness allowance to compensate this loss. Then,


Value of ? (21,000 + .y) in 2017 is same as the value of ? 20,000 in 2016.
21,000+ Y 20,000
192
=i> 21,000+ .Y = 24,000 => Y =3000
160

Hence, ? 3000 are paid to the employee to compensate the loss.


Purchasing power of rupee in 2016 compared to 2011 = Index number for base year
Index number for current year

^ ^ 100
160 ~ 160 X 100 paise = 625 paise~ 62 paise
Purchasing power of rupee in 2017 compared to 2011 = Index number for base year
Index number for current year
= ? 100 _100
180 "l^ X 100 paise =5555 paise - 56 paise
22.17
INDE): NUMBERS

It follows from this that due to increase in price index i.e. price rise, the purchasing power of
rupee decreases.
EXERCISE 22.1

What is an index number ? Explain the uses and limitations of index numbers.
Discuss the problems involved in the construction of index numbers.
Explain simple aggregative method for the construction of index numbers.
Explain the method of simple average of price relatives to construct index numbers.
State the uses of index numbers. What considerations are taken into account while selecting

ow
the base year for the preparation of ii dex number ?
Describe index numbers.
7. Index numbers are economic barometers. Explain the statement.
Construct index number for 2014 with 2012 as base from the following prices of

e
commodities by simple aggregative method.

re
Conunodities

Fl Price ill 2012 {in ?) Price in 2014 {in ?)

F
80
/I
ur 50

r
40 60
B
fo 20
ks
c 10
Yo
D 5 10
oo

£ 2 6
eB

Construct the index number for 2015 taking 2013 as base by price relative method using
AM.
ur
ad

Price in 2013 {in ?) Price in 2015 {in ?)


Yo

Commodities
I

A 10 13
d

17
Re

B 20
in

C 30 60
F

D 40 70

10. Compute the index number for 2019 with 2018 as base from the following prices of the
commodities by simple aggregative method.

Commoditi/ and unit Price in 2018 {in ?) Price in 2019 {in ?)

20.00 22.00
Butter per kg
3.00 4.50
Milk per litre
2.00 3.80
Bread per kg
4.00 4.50
Eggs per dozen
18.00 19.80
Cheese per tin
22.18 APPLIED MATHEMATICS-XII

11. Calculate index number for 2020 from the following data using simple average of price
relatives.

Item Price in 2019 (in ?) Price in 2020 (in ?)


A 200 320

B 400 420

C 100 120

D 40 60

ow
£ 20 28

J 2. Construct index numbers from the following data for 2019 and 2020 taking 2018 as base by
using the method of simple average of price relatives :

e
re
Group Price in 2018 {in ?) Price in 2019 (in ?) Price in 2020 (in

Fr l
F
A 20.00 24.00 22.00

B 1.25 1.50 1.00

or
ou
C 5.00 8.00
kfs 8.00

D 2.00 2.25 2.12

13. Compute index numbers from the following data by simple aggregative method taking
oo

prices of 2016 as base:


Y
B

Commodities Price in 2016 (in ?) Price in 2017 {in ?) Price in 2018 {in ?) Price in 2019 (ip ?)
re

A 0.30 0.33 0.36 0.39


oYu

B 0.25 0.24 0.30 0.30


ad

\
C 0.20 0.25 0.28 0.30
d

D 2.00 2.40 2.50 2.60


in

14. Calculate index number for 2018 on the base prices for 2011 from the following by average
Re

of price relative method.


F

Items Price in 2011 {in Price in 2018 {in ?)


Bricks 10 16
Timber 20 21
Plaster Board 5 6
Sand 2 3
Cement 7 14
15
The index number by the method of aggregates for the year 2019, taking 2010 as base year
was found to be 116. If Z /?q = 300, find the missing entries in the data:
Commodities: A B C D E F
Prices {in ?) in 2010: 56 X 27 76 110 17
Prices {in ?) in 2019: 64 20 y 84 124 20
22.19
INDEX NUMBERS

16. Calculate the price index number using simple average of relatives method for the year
2019 taking 2014 as base year

Commodity Price in 2014 {in ?) Price in 2019 {in ?)

A 40 55

B 60 70

C 20 40

D 50 70

£ 80 100

ow
F no 115

17. From the following data, calculate price index number for the year 2017, taking 2009 as base
year, by simple average of relatives method, using arithmetic mean:

e
Commodity Price in 2009 {in Price in 2017 {in ?)

re
A

rFl 80 100

F
B 60 90

C 50 70

r
22
ou
D 16

E 150 fo 175
ks
F 40 70
oo

G 150 225
H 120 160
Y
eB

18. Consider the following data:


Price (in
r
ou

Items ibiits
ad
Y

In 2014{po) In 2018 {pi)

5.60 7.20
Wheat 1 kg
d

24.80
Re
in

Rice 1 kg 17.20

36.00 44.00
Pulses 1 kg
F

Milk 1 It. 24.00 30.00

199.00 130.00
Clothing 1 m

Using 2014 as the base year, calculate the index for 2018 correct upto one decimal place
using (i) Simple aggregative method (ii) Simple average of price relatives method.
19. Taking 2011 as the base year, with on index number 100, compute an index number for
2016, based on (i) Simple aggregative method (ii) Simple average of price relatives derived
from the table given below:
A B C D
Commodity:
20 10 25 40
Price per unit 2011:
24 20 30 40
Price per unit 2016:
22.20 APPLIED MATHEMATICS-Xtl

20. Based on year 2010 as base, the index numbers for 2010,2011,2012,2013 and 2014 are 100,
110,120, 200 and 400. Taking 2014 as base year, calculate index numbers for years 2010,
2011,2012,2013 and 2014.
21

rw
● Using 2014 as the base year, the index numbers for the price of vegetable oil in 2015 and
2016 are 125 and 140 respectively. Compute the index numbers for 2014 and 2016, taking
2015 as the base year.
The index number for the year 2018 taking 2013 as base year for the following data was
found to be 125. Find the missing entries in the data, if Ipo = 360.

e
Commodity: A B C D E F

r
Price (in V in 2013:

luo
40 60 20 50 AT 110
Price (in in 2018: 55 70 40

F
100 115

23- Construct the index number for 2011 taking 2010 as thebaseyearby simple average of price
relative method:

oF
rs
Commodity: A B C D E

ok
Price (in in 2010: 100 80 160 220 40

Price (in V in 2011: 140 120 180 240 40

fo
o ANSWERS
Y
8- 164.48 9. 147.5, 10. 116.47 135
Y
17. 128.12,112.75 13. 117.1,125.1,127.3,130.5 1- 147
rB

15.
.Y = 14,1/ =36
16. 137.28 17. 140.94 '3- (i) 129.1 (ii) 130.00 '
1^- (i)114 (ii) 135 70. 25, 27.5, 30.00, 50.00,100
80,112 22.
A- = 80,1/ = 70 23. 122.32
due

22.7 METHOD OF CONSTRUCTING WEIGHTED INDEX NUMBERS


In the previous section, we have learnt about the construction of un-weighted index numbers. In
no
ad

the true sense these index numbers are not unweighted. In fact, we have assigned equal
importance to all the items included in the index.That is why they are in reality weighted as the
i

weights are implicit rather than explicit. In this section, we shall study the construction of
weighted index numbers by assigning each commodity (item) a weight in accordance with its
Re
F

importance in the total phenomenon that the index is supposed to describe.


Weighted index numbers are of two types:
(i) Weighted aggregate index numbers. (ii) Weighted average of price relatives.
Let us now learn about these methods:
22.7.1 WEIGHTED AGGREGATE METHOD

By using this method, we construct weighted aggregative index numbers. These indices are of
the simple aggregative type with the fundamental difference that the weights are assigned to
the various items included in the index. There are various methods of assigning weights. But,
we
will be discussing problems where weights will be given. If along with the base prices Pq and
current prices of a number of items, the weights w of each item are given, then the index
number based on weighted aggregates is given by
Pm
Z XV
xlOO
Z pq w

Sometimes, we take quantities in the base year as weights and in such cases the formula for
constructing the weighted index number becomes
22.21
INDEX NUMBERS

Z P\% X 100
^01
Z Po %
Following examples will illustrate the above method.
ILLUSTRATIVE EXAMPLES

EXAMPLE 1 Cnlculnte the index number for the year 2018 with 2013 as the base year by weighted

aggregate method from the following data:

ow
Price in ?
Commodity Weights
Year 2013 Year 2018

e
140 180 10
A

re
B 400 550 7

Flr
250

F
c 100 6

D 125 150 8

£
ou
200 300 4

sr
Computation of the index number for the year 2018 with 2013 as the base year by
fo
SOLUTION

using weighted aggregate method:


k
oo
Commodity Price in ? in the Price in ? in the Weights (w) Pq
year 2000 (pq) year 2005 ip-])
Y
reB

10 1800 1400
A 140 180

7 3850 2800
B 400 550
uY

250 6 1500 600


c 100

8 1200 1000
D 125 150
ad
do

4 1200 800
£ 200 300

J^p^ w = 9550 Zfo “ 6600


in
Re
F

Thus, we have Z £o ^ 6600, Z £l ^


Z w
xlOO =
9550
X 100 = 144.69
P()l
Z Po w 6600

Hence, the index number for the year 2018 with 2013 as the base year is 144.69.
EXAMPLE 2 Construct index number for price for the year 2017 xoith 2015 as the base year from the
following data by taking quantities in the base year as weights.
2015 2017
Commodity
Price Quantity Price Quantity
/I 2 8 4 6

B 5 10 6 5

C 4 14 5 10

D 2 19 2 3
22.22 APPLIED MATHEMATICS-XII

SOLUTION Computation of weighted price index for the year 2017 with 2015 as the base year by
weighted aggregate method taking quantities in the base year as weights:
Commodity 2015 2017

Quantity P\% Po%


Price Price Quantity
(^’o) i^l) (Po) (^l)

A 2 4 6 32 16
B 5 10 6 5 60 50
C 4 14 5 10 70 56
D 2 19 2 3 38 38

Z Pi %= 200 2 Po‘Jo =160


We have, P] % = 200 and Z Po % = 160

F low
n _ Z P] -70 xlOO = 200 X100 = 125
‘ 01 - ^
Z Po ‘Jo 160

22.7.2 WEIGHTED AVERAGE OF PRICE RELATIVE METHOD

Similar to the simple average of price relative method, we compute the weighted average of
price relatives. In this method, first of all price relatives are computed for various commodities
re
and then weighted average of these relatives is obtained by using either arithmetic mean or
for F
geometric mean. Generally weights of various commodities are taken as the values of the
commodities in the base year. Thus, if pg and denote respectively the price and quantity of a
commodity in the base year, then its weight is taken as pg %■ Thus, we have the following
formula:
Your
ks

Pi
Z Po ‘Jo X 100
eBoo

[Po
Poi [When AM is used]
Z Po %
ad
our

We may use the following algorithm to compute the weighted aritlimetic mean of price
relatives.

ALGORITHM
Re

Pi
^11 '■ I Compute price relatives i.e. X
100 for each commodity.
Y

Po
Find

Compute base year values pg qo/or each commodity and consider them as zveights, if zveights
are not given.
"II iMIi
Multiply price relatives obtained in step I for each commodity by the corresponding zveights
Pi
obtained in step I! i.e. compute Pq qQ X
100 for each commodity.
Po

hJILi' Add the result obtained in step III to get Z Po 'Jo ^xlOO
iPo
Sll i'V
Divide the sum obtained in step IV by the sum of the zveights obtained in step II to get the
required index number Poi '>●
Pi xlOO
Z Po%
Po
% =
Z Po ‘Jo
22.23
INDEX NUMBERS

Instead of using arithmetic mean, we may use the geometric mean for averaging price relatives
and the formula for computing the index is
Pi xlOO
Z Po%
[PO
Pqi = anti log
Z Po %

We may use the following algorithm to compute the weighted geometric mean of price
relatives.

ow
ALGORITHM

'll I Compute price relatives i.e. — x 100 for each commodihj.


Po

Find the logarithm of each price relative computed in step II.


Multiply the logarithms computed in step 11 by the weights assigned i.e. compute

e
II

re
(v \
Po fjo log — xlOO for each commodity.
[Po )
rFl
F
.. Add the values obtained in step III.
Divide the total obtained in step IV by the sum of the -weights.

r
ou
s:ri';r vi fo
Find the antilogarithm of the quotient obtained in step IV to get the weighted geometric mean of
ks
price relatives. The number so obtained is the required index.
oo

ILLUSTRATIVE EXAMPLES
Y

For the following data compute price index by applying zueighted average of price relatives
B

liXAMl’l i

method, using:
re

(i) Arithmetic mean (ii) Geometric mean


ou

Commodity Price in 2014 (in V Price in 2015 (in Quantity in 2014


Y
ad

Sugar 3 4 20 kg
40 kg
d

Flour 1.5 1.6


1.5 10 It
in
Re

Milk 1.0

(i) Computation of index number using weighted arithmetic mean of price


F

SOLUTION
relatives.

Commodity Price in 2014 Price in 2015 Quantity Po % -^x 100 Pi xlOO


(in in 2014 Po %
(in V Po Po
Po Pi %

Sugar 3 4 20 60 ixlOO 8000


3
1.6
Flour 1.6 40 60 X 100 6400
1.5
15
15
xlOO 1500
Milk 1.0 1.5 10 10
1.0

130 15900
22.24 APPLIED MATHEMATICS-XII

Thus, we have
P\
T Po% = 130 and Po qo 100 = 15900
Po

Pi
Z ^'0 qo xlOO
Po 15900
^01 = => R01 - = 122.31
Z Poqo 130

Tlie means that there has been a 22.3% increase in prices over the base level,
(ii) Computation of index number using geometric mean of price relatives.
Commodity Price in 2014 Price in Quantih/in Pi
P = ILL X 100
(inV 2015 (in V 2014 Po
Po Pi (qo)

w
Sugar 3 4 20 60 -X 100 =133.33 127.494

F lo
3

1.6 2.0252
Flour 1.5 1.6 40 60 X 100 =106.7 121.692

ee
15

Fr
15
Milk 1.0 1.5 XlOO =150.0 2.1761
10 10 21.761
1.0

for
ur
130 270.947
Thus, we have
s
ook

Z Po ^0 = 130 and pq log ^ x 100 = 270.947


Yo

{Po
eB

Pi
Z Po qo log
.POxlOO, 270.947
Pgi = ‘inti log - anti log = anti log 2.084=121.3
r

Zpo«7o 130
ou
ad
Y

This mean that there has been a 21.3% increase in prices over a period of one year.
Re

example; 2
The quotations for four dijferent commodities for the years 2010 and 2015 are given beloiu.
nd

Calculate the index number for 2015 with 2070 ns the base year by using the weighted average of price
Fi

relatiz’es method.

Commodity Weight Price in ? in the year Price in f in the near


2010 2015

A 5 2.00 4.50

B 7 2.50 3.20

C 6 3.00 4.50

D 2 1.00 1.80

SOLUTION Computation of the index number for the year 2015 with 2010 as the base year by
using the weighted average of price relatives method.
22.25
INDEX NUMBERS

Price relatives PV
Commodity Weight Price in ? in the Price in ? in the
(V) year 2010 year 2015 p=^xlOO
Po P\ Pu

2.00 4.50 450 1125


A 5 xlOO = 225
2.00

7 2.50 3.20 3.20 896


B xlOO = 128
250

ow
4.50 450 900
C 6 3.00 xlOO = 150
3.00
1.80 360
D 2 1.00 1.80 xlOO = 180
1.00

e
re
S^=20 Y, PV = 3281

Flr
F
P^

Foxioo 3281
^01 =
ou
^ P01
20
= 164.05

sr
This means that there has been an increase of 64.05% in the prices over a period of 5 years.

I \ \\li’l 1 Calculate the index number of the following: fo


k
oo
Relative index: 181 116 110 157
Y
12 3 7
Weight: 4
reB

SOLUTION Computation of index number by using weight arithmetic mean of price relatives.
uY

PV
Relative index P Weight V
4 724
181
ad
do

116 12 1392

330
in

no 3

10_99
Re

157 7.
F

YV =26 XPV = 3545


Y PV _ 3545 = 136.34
^01 26

EXAMPLE 4 Calculate the index number for the year 2019 with 2010 as base from the follozving data

using zveighted average of pric,e relatives:


Price in ?
Commodity Weights
2010 2019

A 22 2.50 6.20

48 3.30 4.40
B

17 6.25 12.75
C

13 0.65 0.90
D
22.26
APPLIED MATHEMATICS-XII

SOLUTION Computation of index number by using the weighted average of price relatives.

Commodity Weight Price in ? Price relatives PV


(V) P\
2010
P =-^^xlOO
2019
Po
Po Pi

A 22 2.50 6.20 6.20


xlOO = 248 5456
250

B 48 3.30 4.40 4.40


xlOO = 133.33 6399.84
3.30

ow
C 17 6.25 12.75 12.75
XlOO = 204 3468
6.25

D 13 0.65 0.90 90
xlOO = 138.46 1799.98

e
0.65

re
2 V=100

Fl X P7= 17123.82

F
ur
r
Pi
Zv
Po X 100
Z PV 17123.82
fo
ks
Poi = 01 = 171.2382
Z^ 100
Yo
oo

Calculate weighted index number for 2019 from the following data:
eB

Item A B C
Quantity (units): 20 15 10
ur

Price in 2018 (in ?); 200 100 20


ad

Price in 2019 (in 320 120 28


Yo

SOLUTION Computation of weighted index number by using weighted arithmetic mean:


d
Re
in

Item Quantity Price in Price in 2019 V=pQqQ Price relatives PV


% 2018 (in V (in V Pi
F

P =-^x 100
Po ^1 Po

A 20 200 320 4000 320


X 100 = 160 64000
200

B 15 100 120 1500 120


X 100 =120 180000
100

C 10 20 28 200 28
— X 100=140 28000
20

2^=5700 5^ py =848000
^01
Z _ 848000 = 148.77
Y. y 5700
22.27
INDEX NUMBERS

iiXAMl'l.t 6 Taking 2025 as the base year, with an index number 100, calculate an index number for
2019, based on weighted average of price relatives from the table given below:
B C D
Commodity: A

15 25 30
Weight: 30
5 40
Price per unit in 2015 20 10
30 40
Price per unit in 2019: 24 20

The weights are noxo changed so that the iveight for A is 40 and C is 10 and the total weight is 100. If the
value of the index number in 2019 with the changed weight is 182. Calculate the weights applied to B
and D.

(i) Computation of index number for 2019 taking 2015 as the base year by weighted

ow
SOLUTION

average of price relatives:


PV
Commodity Price per unit Price relatives for 2019 Weights V
2015 2019 P=h.xl00
Po

e
Po Pi

re
A 20 24 24
— X100 =120

rFl 30 3600

F
20

6 10 20 20 15 3000
— X 100 =200

r
10
ou
C 5 30 30
— X 100 = 600
fo25 15000
ks
5

40 30 3000
oo

D 40 40 X 100 =100
40
Y

2^ y = 100
B

py = 24600

X PV _ 24600
re

= 246
% =
IV 100
ou
Y
ad

(ii) Let the weight for 6 be .x. It is given that the weights for A and C are 40 and 10 respectively
and the total weight is 100.
Weight for D = 100 - (40 + x + 10) =50 - .y.
d

Computation of index number by weighted average of price relatives


in
Re

PV
Commodity Price per unit Price relatives for 2019 Weights V
F

2015 2019 P =-^xl00


Po
Po Pi

24 24 4800
A 20 — X 100 =120 40
20

10 20 20 200 Y
B X 100 = 200 Y
10

5 30 30 6000
C — X 100 = 600 10
5

40 40 40 5000 - IOOy
D — X 100 =100 50-Y
40
100 15800 +100Y
22.28
APPLIED MATHEMATICS-Xn

Thus, we have

X ^ = 10 and py = 15800+ 100.V


p.01 _^PV => 182 =
15800+ 100.Y
=> 182 = 158+ .V => .r = 24 [vPoi=182 (given)]

100

Hence, the weights for B and D are 24 and 26 respectively.


EXAMi’i.R 7 The price relatives and weights of a set of a commodities are given below:
Commodih/: A B C D
Price relative:

w
125 120 127 119

Weight: .T 2.V y y +3
If the sum ofxueights is 40 and the index for the set is 122, find the numerical values ofx and i/.

o
SOLUTION It is given that the sum of the weights is 40.

e
re
-V + 2.V + 1/ + 1/ + 3 = 140 ^ 3.V + 2}/ = 37

rFl ...(i)

F
Computation of index number by weighted average of price relatives.

Commodity Weight (V) Price relatives (P)

r
PV
ou
/I 125
fo
ks
.r 125 .r

B 2x 120 240 X
oo

C y 127 127 y
Y
eB

D V + 3 119 119 y+ 357


X = 3.x + 2y + 3 'ZPV = 365x + 246 y + 357
r
ou
Y
ad
d

122 = 365.Y + 246y + 357


Re
in

40 [vPoi=122 andsy =40]


F

4880 = 365.x + 246y + 357 => 365.x + 246y = 4523 ...(ii)


Solving (i) and (ii), we get X = 7 and y = 8
EXAMPLE 8 An index of retail prices is the mean of the five other price index numbers which are weighted
as follows:
A B C D E
Food Rent
Household goods Fuel and Light Miscellaneous
348 88 106 65 393

The original index of retail prices are established at 100. If the following percentage increases in the
various indices have occurred since that time, find the index of retail prices }10W.
A: 55%; B: 180%; C; 64%; D: 45%; E: 90%.
INDEX NUMBERS 22.29

SOLUTION It is given that the base year prices for commodities A, B,C,D and £ are 100 each
/ N

and the price relatives P = — x 100 are respectively 155,280,160,145,190.


Pq

Thus, we have the following table:

Price relatives PV
Conwiociity Weights (V)
p = El xlOO
Po

Food 348 155 53940

Rent 88 280 24640

w
16960
Household goods 106 160

F lo
- 145 9425
Fuel and light 65

Miscellaneous 190 74670

ee
393.

Fr
2 l/=1000 Y, PV =179635

X PV _ 179635 = 179.635
for
ur
Required index = 1000
s
ook
Yo

22.8 LIMITATIONS OF INDEX NUMBERS

Though the index numbers are very useful for measuring relative changes in prices, quantity
eB

produced (consumed), value, etc., but they have some limitations and the reader must also be
aware of their limitations so that he (she) may use them properly. The chief limitations of index
r
ad
ou

numbers are :

1. Since index numbers are generally based on samples. Therefore, it is not possible to take
Y

into account each and every item in the construction of index members.
Re
nd

2. In the construction of index numbers samples are not chosen randomly because random
Fi

sampling is practically not convenient. In fact, index numbers are constructed from
deliberately selected samples which may introduce errors.
3. Index numbers are only approximate indicators of the relative level of phenomenon and
should be interpreted as such.
4. In the construction of index numbers it is generally assumed that the qualities of
commodities remain same over a period of time, because differences in quality would
mean differences in price also. But in modern times when qualities of various products
undergo quick changes, so it makes comparisons over long periods less reliable.
5. A large number of methods are designed for constructing index numbers and different
methods of construction give different results. Generally, the selection of an appropriate
formula creates problems. Also, there is no method of constructing index numbers which
is suitable from all points of view.
22.30 APPLIED MATHEMATICS-XII

EXERCISE 22.2

1 ● Calculate cost of living index for the following data:


Commodity Weight Price relatives

A 22 248

S 48 133.30
C 17 204

D 13 138.50

Calculate the cost of living index from the following table of prices and weights:
Commodity Weight Price index

Food 35 108.5

w
Rent 9 102.6

F lo
Clothes 10 97.0

Fuel 7 100.9

e
Miscellaneous 39 103.7

Fre
● Construct index number for the following data:
Commodity Butter Bread
for Tea Bacon
r
Relative index 181 116 no 152
You
s

Weight
ook

12 3 7

4. The following table shows the prices per unit in 2010 and 2014 with weights of commodities
eB

71, B, C, D:

Commodity Weights Price per unit in 2010 Price per unit in 2014
our
ad

A 20 25 30

B 25 20 30
dY
Re

C 15 50 70
Fin

D 40 5 10

Taking 2010 as base year with index number 100, calculate the index number of 2014 based
on weighted average of price relatives.
.‘5. From the following data compute price index by applying weighted average of price
relative method using arithmetic means:
Commodity Po Bo. Pi ('»
Sugar 9 20 kg 15
Flour 5
40 kg 10
Milk 6 10 lit 15

Find the consumer price index for 2014 on the base of 2008, from the following data, using
the method of weighted average of price relatives:
INDEX NUMBERS
22.31

Item Food Rent Clothing Fuel Miscellaneous

Price in 2008 (in ?): 200 100 150 50 100

Price in 2014 (in ?): 280 200 120 100 200

20 10 20
Weight: 30 20

7. Find the consumer price index number for 2019 on the base of 2018 from the following data,
using the method of weighted average of price relatives:

w
Item Quantity Price in 2018 (in ?) Price in 2019 {in ?)

>4 20 units 200 320

420

o
B 14 units 400

e
C 15 units 100 120

re
D 18 units

rFl 40 60

F
£ 10 units 20 28

Taking 2005 as the base year with an index number 100, calculate an index number for 2015

r
8.
ou
fo
based on the weighted average of price relatives derived from the following table:
ks
A B C D
Commodity
oo

5 40
Price per unit in 2005: 10 20
Y
eB

10 80
Price per unit in 2015: 30 35
r

10 40
Weight: 20 30
ou
Y
ad

9. Calculate an index number for the second year, taking the first year as base, taking into
account the prices of four commodities (in Rupees per kg) and the weights given below:
d
Re
in

A B C D
Commodity
F

28 36 28
1st year: 30

45 42
2nd year: 42 35

6 25
Weight: 24 14

10. Construct the consumer price index number for 2016 on the basis of 2006 from the
following data:
A B C D £
Commodity
5.12 2.00
2006 Price per unit (in ?): 16.00 40.00 0.50

6.25 1.50
2016 Price per unit (in ?): 20.00 60.00 0.50

5 20 10
Weight: 40 25
22.32
APPLIED MATHEMATICS-Xn

Compute price index for the following dahi by applying weighted average of price relative
method, using (i) Arithmetic mean (ii) Geometric mean:
Item Price in 2018 (in f) Price in 2019 (in ?) Qmniihj in 2018
Wheat 2.00 2.50 40 kg
Sugar 3.00 3.25 20 kg
Milk 1.50 1.75 10 It.

11:.
The price of various commodities are given in the following table;
Commodity Base Price (in ?) Current price (in ?)
Rice

w
30 35
Wheat 22 25
Fish

F lo
54 64
Potato 20 25

ee
Coal 15 18

Fr
If a Bengali family buys quantities of rice, wheat, fish, potato, and coal in the ratio 3:1:3:2;2.
Find the price index using weighted average of price relatives.
for
A manufacturer uses raw materials A, B, C, D in the production of a certain commodity.
ur
Masses of raw materials consumed are in the ratio 2 : 3 : 4: 1. The prices in rupees, of the
materials per kilogram in the years 2018,2020 are given in the following table:
s
ook
Yo

A B C D
2018 12 6 18
eB

2020 9.50 13 7.50 20


Calculate the index number for the total cost of raw materials used for the manufacturer of
r

the commodity in 2020, using 2018 as base year.


ou
ad

If the commodity is sold for ? 5.75 in 2018, calculate the selling price in 2020 on the
assumption that selling prices are directly proportional to the cost of raw material,
Y

Taking 2000 as the base year, with an index number 100, calculate an index number for
Re
nd

2005, based on weighted average of price relatives derived from the table given below:
Fi

Commodity A B C D

Weights 30 15 25 30

Price per unit in 2000 20 10 5 40

Price per unit in 2005 25 20 30 40

The weights are now changed so that the weight for A is 40 and C is 10 and the total weight
is 100. If the value of index number in 2005 with changed weight is 182, calculate the
weights applied to B and D.

ANSWERS
171.23 104.4 135 162.5 193.18 158 137.27 . 212.5
139.27 124.41 (i) 117.74 (ii) 117.4 . 119.02 . 114.9,? 6.60
Pqi = 246 New weights for B and D are 24 and 26 respectively.
U'Miliiil

TIME SERIES

23.= SERIES
A set of statistical observations taken at different points of time (usually at equal intervals) is

w
known as a time series i.e. if we record the data relating to population, production, prices, sales,
per capita income, etc., for the last 5, 10,15, 20 years or some other time period, the series so

F lo
formed is called a time series.

The product of sugar in India over last 8 years as shown in the following table is a time series.
Year: 2010 2011 2012 2013 2014 2015 2016 2017

ee
Sugar Production 54 50 75 80 77 82 85 84

Fr
(Lakh tonnes):
Similarly, the sale of godrej refrigerators from 2015 to 2020 as given in the following table is a
for
time series.
ur
Year: 2015 2016 2017 2018 2019 2020
40
Sale of refrigerators 75 85 60 58 62
s
ook

(in Lakh units):


Yo

It is evident that in a time series time is the most important factor, because the variable
eB

(production, sales, population, income etc.) is related to time which may be either year, month,
week, day, hour or even minutes or seconds.
In a time series values of the variable i/ (say) depend on time t. So, mathematically y is a function
our
ad

of t i.e. 1/ = F (t).
There can be changes in the values of the variable recorded over different points of time due to
various forces. Analysing the effect of all such forces on the values of the variable is generally
Y

known as the analysis of time series. Broadly there can be four types of variations or changes in
Re
nd

the values of the variable as discussed below:


Fi

(i) Variations which generally occur due to general tendency of the data to increase or
decrease. These variations are known as secular variations or secular movements,
(ii) Variations which occur due to change in climate, weather conditions, festivals etc.
(iii) Variations which occur due to booms and depressions. These are known as cyclic
variations,

(iv) Variations which occur due to some unpredictable forces like floods, famines,
earthquakes, etc. These are known as irregular variations. These variations are also
known as movements or components of a time series.
SECULAR MOVEMENTS The word Secular is derived from the Latin word Saeculum which means
generation or age. Therefore secular trend movements are considered as long term movements
and are attributable to factors such as population change,
technological progress and large-scale
of trends (movements) in the values of the
shifts in consumer tastes. There are various types
variable. Some series increase slowly and some increase fast, some remain constant for long
periods of time and some after a growth or decline, reverse themselves and enter a period of
decline or growth. These trends or movements are divided into two groups.
23.2
APPLIED MATHEMATICS-XII

(i) Linear or straight line trend. (ii) Non-linear trends.


In this book we shall be studying linear trends only.
SEASONAL VARIATIONS Those periodic movements in business activity which occur regularly
every year are known as seasonal variations. These variations can be predicted fairly accurately
as
they repeat during a period of 12 months. These variations are of periodic nature and their
repeating cycles are of relatively short duration. Seasonal variations occur due to change in
i”
climate and weather conditions. Customs, traditions and habits also have their impact on
seasonal variations.

CYCLIC VARIATIONS Cyclic variations are also periodic variations but their repeating cyclic is
usually more than a year. The amplitude and length of these variations are not same in every
cycle. Cyclic variations are long-term movements that represent consistently recurring rises and

w
declines in activity.
iFiREGULAR VARIATIONS Those variations in business activity which do not repeat in a definite

F lo
pattern. These variations occur due to some unpredictable forces like floods, earthquakes,
famines, etc. In fact, Irregular variations include all types of variations other than secular,

ee
seasonal and cyclic movements. Irregular variations are random in nature, because they are

Fr
results of chance factors.

The basic objective in this chapter is to determine the nature of the trend or movement whether
for
it is increasing or decreasing. Because by determining the nature of trend we can ascertain the
ur
growth factor, rate of change and tentative estimates concerning the future can be made.
s
There are various methods to determine trend. But, we shall be discussing only one of them
ook
Yo

known as moving average method.


eB

23.2 METHOD OF MOVING AVERAGES

In order to determine a trend by the method of moving averages, the average value for a number
r

of years (or months or weeks) is computed and it is taken as the normal trend value for that year
ad
ou

(or month or week) which lies at the middle of the period covered in the computation of the
average. The period in moving average method may be an even number or an odd number. So,
Y

we will discuss following cases:


Re
nd

: WHEN PERIOD OF fwOVING AVERAGE IS AN ODD NUMBER; In this case to determine the
Fi

trend values we may use the following algorithm:


ALGORITHM

Si'l-I’ ! Obtain the time series.

Select a period of moving average such as 3 years, 5 years etc.


Compute moving totals according to the length of the period of moving average.
If the length of the period of moving average is 3 i.c. 3-yearly moving average is to be calculated,
compute moving totals as follows:
a + b + c, b + c + d, c+d + e, d + e+f,
For 5-yearly moving average, moving totals are computed as follows:
a + b + c + d + e,b + c + d + e+ f,c + d + e + f + g,....
Place the moving totals at the centre of the time span from zuhich they are computed.
TIME SERIES 23.3

-j':. Compute moving averages by dividing the moving totals in step 111 by the length of the period of
moving average and place them at the centre of the time span from which the moving totals are
computed.
These moving averages are also called the trend I’alues.
By plotting these trend values (if desired) one can obtain the trend curve zvith the help of which
we can determine the trend xvhether it is increasing or decreasing.
If needed, one can also compute short term fluctuations by subtracting the trend values from
the actual values.

Following examples will illustrate the above algorithm.

ILLUSTRATIVE EXAMPLES

EXAMPLE 1 The number of letters, in hundreds, posted in a certain city on each day for a zveek is given as

w
follows:
35, 70 36, 59,62, 60, 71

F lo
Calculate the three days moimig averages and display these graphically.
SOLUTION In order to calculate three days moving averages, we first calculate three days
moving totals as follows:

ee
Number of letters, in hundreds, posted on Monday, Tuesday and Wednesday

Fr
= 35 + 70 + 36 = 141.

Place this value against the middle day i.e. Tuesday as shown in the following table.
for
Now, compute the number of letters, in hundreds, posted on Tuesday, Wednesday and
ur
Thursday. This number is 70 + 36 + 59 = 165.
Place this value against the middle day i.e. Wednesday.
s
ook

Calculate now the number of letters, in hundreds, posted on Wednesday, Thursday and
Yo

Friday. This number is 36 + 59 + 62 = 157. Write this number against the middle day i.e.
eB

Thursday.
Continue this process to calculate three days moving totals and place them against the centre
our

(middle) day as shown in the following table.


ad

Divide each one of three days moving totals by 3 to get 3 days moving averages as shown in
fourth column of the following table.
dY

Computation of three days moving averages


Re

Number of letters posted Three days Three days


Fin

Day (in hundreds) moving totals moving averages

Monday 35

141 47
Tuesday 70

165 55.00
Wednesday 36

157 52.33
Thursday 59

181 60.33
Friday 62

193 64.33
Saturday 60

Sunday 71

These averages are displayed on the graph paper as shown by dotted graph in Fig. 23.1
continuous curve is the curve obtained by plotting actual values.
23.4 APPLIED MATHEMATICS-XII

Actual data
Three days moving
80 ● averages

70 ●

■§ 60 .
●c

c 50 .
T3
3

^40.
a.

4>
30 ●

w
o

^20 ●
a

Flo
z
10 .

e
re
0 Monday Tuesday Wednesda)' Tluirsday Friday Saturday
Sunday

F
Days—B.
Fig. 23.1
ur
r
i:x A M I’Ll. 2 The average number, in lakhs, of working days lost in strikes during each year of the period
fo
2001-2010 was ' <^ . j r
ks
2001 2002 2003 2004 2005 2006 2007 2008 2009 2010
Yo
1.5 1.8 1.9 2.2 2.6 3.7 2.2 6.4 3.6 5.4
oo

Calculate the three-yearly moving averages and draw the moving averages graph.
B

SOLUTION
In order to calculate three yearly moving averages, we first compute three yearly
re

moving totals and place each total against the middle year of the three year span from which the
moving totals are calculated. These moving totals are given in the third column of the following
u
ad

table. From these three yearly moving totals, we calculate three yearly moving averages by
Yo

dividing each moving total by 3 as shown in the following table.


Year V^orking days lost Three yearly
d

Three yearly
Re

in strikes moving
in

moving
(in lakhs) totals averages
F

2001 1.5

2002 1.8 5.2 1.73

2003 1.9 5.9 1.96


2004 2.2 6.7 2.23
2005 2.6 8.5 2.83
2006 3.7 8.5 2.83
2007 2.2 12.3 4.1

2008 6.4 12.2 4.06


2009 3.6 15.4 5.13

2010 5.4
23.5
TIME SERIES

The graph of these moving averages is shown in Fig. 23.2.


Actual value
Tliroe yearly
t 1* moving averages

2 6*
ej
>
tc
00 :>●
c

I 4-
a
g 3*
J

I
[-
2*
!●

w
21X)5 2006 2007 21K)8 2009 2010
2001 2002 2003 2004
Years

F lo
Fig. 23.2

The production of soft drink compnin/ in thousands of litres durin;^ each month of a year is

ee
EXAMPl-E 3

Fr
asfoUozvs:
Nov. Dec.
Jan Feb March April May June July August for Sept. Oct.
2.4 2.6 3.0 3.6 2.8 1.9 3.4
1.2 0.8 1.4 1.6
ur
Calculate the five monthly moving averages and shoiv these moving averages on a graph.
s
ook

Calculation offive monthly moving averages:


Yo

SOLUTION
eB

Month Production of soft drink 5 monthly 5 monthly moving


(in thousands litres) totals averages
our
ad

January 1.2

February 0.8
Y

6.8 1.36
Re

March 1.4
nd

8.0 1.6
April 1.6
Fi

9.8 1.96
May 1.8

2.4 11.4 2.28


June
13.4 2.68
July 2.6

3.0 14.4 2.88


August
13.9 2.78
September 3.6

October 2.8 14.7 2.94

November 1.9

December 3.4

These moving averages are plotted on the following graph;


23.6
APPLIED MATHEMATICS-XII

3 ●

T3: 2 ●
c

3
c
£
-S

ow
●o

s
c
o
●c
(J

T3

e
c.

re
Fl
F
c
lU
ur 3
ao
c.
3

r
U. Z
Cu 3 o
R
< s < Z Q
s
Month — fo
ks
Fig. 23.3
Yo
I.XAMIM.H4 The table given below shows the daily attendance in thousands at a certain exhibition
oo

over a period of two weeks:


Week 1 52 48 64 68 52 70 72
eB

Week 2 55 47 61 65 58 75 81
Calculate seven days moving averages and illustrate these and original information on the same graph
using the same scales.
ur

SOLUTION
Calculation of seven days moving averages
ad
Yo

Days Attendance 7 days moving 7 days moving


(in thousands) totals averages
d

Week 1 Monday
Re

52
in

Tuesday 48
F

Wednesday 64

Thursday 68 426 60.86


Friday 52 429 61.28

Saturday 70 428 61.14

Sunday 72 425 60.17


Week 2 Monday 55 422 60.28
Tuesday 47 428 61.14

Wednesday 61 433 61.86


Thursday 65 442 63.14
Friday 58

Saturday 75

Sunday 81
TIME SERIES 23.7

Seven days moving averages and original data are plotted on the graph paper in Fig. 23.4.
These points are joined by line segments to obtain the graphs to illustrate the trend.
Actual values
7 days moving
averages

ow
e
re
rFl
F
Week 1 Week 2

Fig. 23.4

or
EXAMI’lI: : Consider thefolloiuing data:
ou
Year: 2003 2004 2005 2006 2007 2008 2009 2010 2011 2012 2013 2014
ksf
Production: 137 140 134 137 151 121 124 159 157 169 172 150
oo

Calculate a suitable moving average and show on a graph against the original data.
SOLUTION In order to find which moving average will be appropriate, we will have to
Y
B

estimate the length of the cycle of the above data. We observe that the data has the pattern (137,
140,134), (137,151,121), (124,159,157), (169,172,150). Thus, we have cycle length of 3. So, we
re

will calculate 3 yearly moving averages as shown in the following table.


oYu

Calculation ofS-ijearly moving averages


ad

Year Production 3-yearly moving 3-\jearly moving


d

totals averages
in
Re

2003 137
F

2004 140 411 137.00

2005 134 411 137.00

2006 137 422 140.67

2007 151 409 136.33

2008 121 396 132.00

2009 124 404 134.67

2010 159 440 146.67

2011 157 485 161.67

2012 169 498 166.00

2013 172 491 163.67

2014 150
23.8 APPLIED MATHEMATICS-XII

These moving averages and the original data are plotted on the graph paper to obtain the
following graph.

Actual data
3 yearly moving
180 averages
170
160
150
140
130
f 120
Clio
■i 100
3 90
"B 80

ow
£ 70
60
50
40
30
20
10

e
re
2003 2004 2005 2006 2007 2008 2009 2010 2011 2012 2013 2014

rFl
Year ►

F
Fig. 23.5

I -XAMPU.i
Using three-yearly moving averages, compute the trend values and short term fluctuations

r
ou
for the following data :
fo
ks
Year: 2008 2009 2010 2011 2012 2013 2014 2015 2016 2017
Production (Thousand 21 22 23 25 24 22 25
oo

26 27 26
tonnes):
Y
B

SOLUTION Computation of trend values


re

Year Production 3-yearly moving 3-yearly moving Short term fluctuations


(Thousand tonnes) totals
ou

averages Yc (Y-Y,)
Y
ad

2008 21
d

2009 22 66 22.00 0
in
Re

2010 23 70 23.33 -0.33

2011 25
F

72 24.00 1.00
2012 24 71 23.67 0.33

2013 22 71 23.67 -1.67


2014 25 73 24.33 0.67

2015 26 78 26.00 0.00


2016 27 79 26.33 0.67
2017 26

1-XAMPLH7
Construct 5-yearly moving averages from the following data :
Year 2005 2006 2007 2008 2009 2010 2011 2012 2013 2014 2015 2016 2017

Production 105 107 109 112 114 116 118 121 123 124 125 127 129
TIME SERIES 23.9

SOLUTION Cnlciilation of moving averages

Year Production Y 5-yearly totals 5-yearly moving averages


2005 105

2006 107

2007 109 547 109.4

2008 112 558 111.6

2009 114 569 113.8

ow
2010 116 581 116.2

2011 118 592 118.4

2012 121 602 120.4

2013 123 611 122.2

e
re
2014 124 620 124.0

2015 125

rFl 628 125.6

F
2016 127

or
2017 129
ou
ksf
EXAMPLE 8 Construct 5-yearly moving averages from the following data of the number of industrial
failures in a country during 2003-2018:
oo

Year No. offailiires Year No. of Failures


Y
B

2003 23 2011 9
re

2004 26 2012 13
oYu

2005 28 2013 11
ad

2006 32 2014 14

12
d

2007 20 2015
in

2016 9
Re

2008 12

2009 12 2017 3
F

2010 10 2018 1

SOLUTION Computation of moving averages

Year No offailures 5-yeariy moving totals 5-yearly moving averages

2003 23

2004 26

2005 28 129 25.8

2006 32 118 23.6

2007 20 104 20.8


23.10 APPLIED MATHEMATICS-XII

2008 12 86 17.2

2009 12 63 12.6

2010 10 56 11.2

2011 9 55 11.0

2012 13 57 11.4

2013 11 59 11.8

2014 14 59 11.8

2015 12 49

2016 9 39 7.8

2017 3

w
2018 1

EX.^MPLE 9

Year: 2010

F lo
Construct 3-yearly moving averages from the following data:
2011 2012 2013 2014 2015 2016

e
Imported cotton consumption in

Fre
India (in '000 bales): 129 131 106 91 95 84 93
SOLUTION
Construction of 3-yearly moving average
for
r
Year
Imported cotton 3 - yearly moving 3-yearly moving
You

totals
oks

consumption in India (in ‘000 bales) averages


eBo

2010 129

2011 131 366 122.00

2012 106 328 109.33


our
ad

2013 91 292 97.33


2014 95 270 90.00
dY

2015 84 272 90.66


Re

2016 93
Fin

CASE 11 WHEN PERIOD OF MOVING AVERAGE IS AN EVEN NUMBER

When period of moving average is even (4-years, 6-years etc.), we compute the moving averages
by using the following algorithm:
ALGORITHM

STEPl Obtain the time series.

STEP 11
Obtain the length of the period of moving average. Let the length of the moving average period
be 4-years.
STEP III Compute 4-yearly moving totals and place them at the centre of time span. The four-yearly
moving totals are computed as follows :
a + b + c + d,b + c + d + e, c + d + e + f,
TIME SERIES 23.11

STEP IV Computed-yearly moving average and place them at the center of the time span. Note that this
placement is inconvenient, because the moving average so placed xvould not cohicide zuith
original time period.
STEP V Take two-period moving average of moving averages and place them at the middle of the
periods. This process is called centering of moving averages.
Following examples will illustrate the above algorithm.
EXAMPLE 10 The table below gives details of the electricity generated in million kiloivatt hours in each
quarter for the year 2017 to 2019.
Year Quarter

1st 2nd 3rd 4th

2017 8 7 6 9

w
2018 10 7 7 10

F lo
2019 11 7 10

Calculate the four quarterly moving averages and show these moving averages on a graph. Comment on

ee
the general trend.

Fr
SOLUTION The calculation of four quarterly moving averages is shown in the following table;

Quarter Electricity generated Four quarterly


for
Four quarterly Four quarterly centred
ur
(in million kilozvatt) moving totals moz’ing averages moving averages
oks

2017 1st 8
Yo
o

2nd 7 30 7.5
eB

3rd 6 7.75
our
ad

32 8

4th 9 8

32 8
Y

2018 1st 10 8.125


Re
nd

33 8.25

2nd 7 8.375
Fi

34 8.5

3rd 7 8.625

35 8.75

4th 10 8.75

35 8.75
2019 1st 11 8.875

36 9

2nd 7 9

36 9

3rd 8

4th 10

We obtain the following graph from the above data.


23.12 APPLIED MATHEMATICS-XII

The graph shows four quarterly cycle but there is steadily upward long term trend.
Actual data
4 Quarterly
moving
averages

ow
5

e
2

re
1

rFl
F
1st 1st 1st
2017 2nd 3rd 4th 2018 2nd 3rd 4th 2019 2nd 3rd 4th

r
Quarter 1
ou
Fig. 23.6 fo
ks
EXAMPLE 11 Coded monthly sales ifgures of a particular brand ofT.V.for 18 months commencing
January 1,2017 are as follows:
oo
Y

Year 2017: Jan Feb March April May June July Aug Sept Oct Nov Dec
eB

18 16 23 27 28 19 31 29 35 27 28 24

Year 2018: 24 28 29 30 29 22
r
ou

Calculate six monthly moz>ing averages and display these and the original figures on the same graph,
ad
Y

using the same axes for both.


SOLLITION The following table shows the computation of six monthly moving averages.
d
Re
in

Month No. ofT.V. Six monthly Six monthly Six monthly centred
F

Sets Sold moving totals moving averages moving averages


2017 Jan 18

Feb 16

March 23

131 21.83

April 27 22.92

144 24.00

May 28 25.08

157 26.17

June 29 27.17

169 28.17
23.13
TIME SERIES

31 28.17
July
169 28.17

29 28.17
August
169 28.17

35 28.59
Sept.
174 29.00

Oct. 27 28.42

167 27.83

Nov. 28 27.75

ow
166 27.67

Dec. 24 27.17

160 26.67

24 26.92
2018 Jan
163 27.17

e
re
Fl
Feb 28 27.25

164 27.33

F
March 29 27.17
ur
162 27.00

r
April 30 fo
ks
May 29
Yo
June 22
oo

Actual data
eB

Six Quarterly
moving
averages
ur
ad
Yo
d
Re
in
F

2017 2018
Jan. Feb Mar Apr May Jun July Aug Sept Oct Nov Dec Jan Fab Mar Apr May Jun.
Fig. 23.7

CXAMrU-12 From the follozvirig data compute 4-yearly moving averages and determine the trend
values. Also, find the short-term fluctuations.
Year: 2006 2007 2008 2009 2010 2011 2012 2013 2014 2015

Value: 50 36.5 43.0 44.5 38.9 38.1 32.6 41.7 41.1 33.8
23.14
APPLIED MATHEMATICS-XII

SOLUTION Conipiitatiott of4-yearly moving averages.


Year Value 4-yearly moving 4-yearly centered Short term fluctuations
totals moving averages V'-Vc
2006 50.0
2007 36.5
<- 174.0 43.5
<■ 42.1125 0.8875
2008 43.0
162.9 40.725
<■ 40.925 3.575
2009 44.5

ow
<- 164.5 41.125
39.825 0.925
2010 38.9
<- 154.1 38.525

e
<■ 38.175 - 0.075

re
Fl
2011 38.1
<- 151.3 37.825

F
ur <■ 38.1 -5.5

r
2012 32.6
<■ 153.5 38.375 fo
ks
<■ 37.8375 3.8625
Yo
2013 41.7 <■ 149.2 37.3
oo

2014 41.1
2015 33.8
eB

EXERCISE 23.1
ur

1. The production of cement by a firm in years 1 to 9 is given below:


Year: 1 2 3 4 5 6
ad

7 8 9
Yo

Production 4 5 5 6 7 9 10
(Tonnes):
d

Calculate the trend values for the above series by the 3-year!y moving average
Re
in

method.

2. Calculate 3-yearly moving averages for the following data:


F

Year: 2005 2006 2007 2008 2009 2010 2011 2012


Annual sales 3.6 4.3 4.3 3.4 4.4 5.4 3.4 2.4
(in 0000 f):
3. Consider the following data:
Year: 2007 2008 2009 2010 2011 2012 2013 2014 2015 2016 2017
Annual sales 3 7 2 6 4 8 3 7 5 9 4
(in laks of^):
(i) Calculate 2-yearly centred moving averages,
(ii) Verify that 3-yearly moving averages with weights 1, 2,1 are same as above,
(iii) Draw original data and above moving averages on a graph,
(iv) Calculate 4-yearIy centred moving averages,
(v) Verify that 5-yearly moving averages with weights 1, 2,2,2,1 are same as above,
(vi) Draw original data and above moving averages on a graph.
TIME SERIES 23.15

(vii) Compare the two graphs. What is the length of cycle? What is the long term trend?
4. In an influenza epidemic the number of cases diagnosed were
Date (March): 1 2 3 4 5 6 7 9 10 11 12 13 14

Numbers: 2 0 5 12 20 27 46 30 31 18 11 5 0 1

Calculate 3-days moving averages and display them and the original figures on the same
graph.
5. Calculate five yearly moving averages of the number of students who have studied in a

w
school given below:
Year: 2003 2004 2005 2006 2007 2008 2009 2010 2011 2012

No. of students: 442 427 467 502 512 515 520 527 515 541

o
6. Daily absence from a school during 3 weeks is recorded as follows:

e
re
Monday Tuesday Wednesday Thursday Friday
Weekl 23

Frl 28 21 33 40

F
Week 2 38 52 43 58 63

or
ou
Week 3 52 54 61 51 51

Draw a graph, illustrating these figures. Calculate 5 day moving averages and plot them on
kfs
the same graph.
oo

7. Calculate the 5 yearly moving averages of the following time series of steel production:
Y

Year: 2006 2007 2008 2009 2010 2011 2012 2013 2014 2015
B

Production (in tonnes): 351 366 361 362 400 419 410 420 450 500
re

8. Assuming a four yearly cycle, calculate the trend by the method of moving averages from
oYu

the following data:


ad

Year: 2005 2006 2007 2008 2009 2010 2011 2012 2013 2014
d

Value: 12 25 39 54 70 87 105 100 82 65


in
Re

9. The profits of a soft drink firm in thousands of rupees during each month of a year were:
Feb March April May June July Aug Sept Oct Nov Dec
F

Jan
1.2 0.8 1.4 1.6 2.0 2.4 3.6 4.8 3.4 1.8 0.8 1.2

Plot these on a graph.


10. Calculate four-monthly moving averages and plot on the same graph. Comment on the
general trend.
2014 2015 2016 2017 2018 2019

252 261 288 299 312 312


January
284 258 312 322 306 324
April
254 261 312 274 306 284
September
Draw the graph of these figures. Calculate a set of moving averages using the most number
of observations. Give reasons of your choice. Plot the moving averages on the same graph
paper and comment in the general trend.
23.16 APPLIED MATHEMATICS-Xll

11- Construct 4-yearly moving averages from the following data:


Year Value Year Value
2003 12 2010 100
2004 25 2011 82
2005 39 2012 65
2006 54 2013 49
2007 70 2014 34
2008 87 2015 20
2009 105 2016 7

12. Compute 4-yearly moving averages from the following data :

w
Year: 2010 2011 2012 2013 2014 2015 2016 2017
Earnings f?. Lakhs): 38 40 65 72 69 60 87 95
13. Compute 5-yearly moving averages for the following data :

Flo
Year: 2005 2006 2007 2008 2009 2010 2011 2012 2013 2014 2015 2016 2017

Production: 105 107 109 112 114 116 118 121 123 124 125 127 129

ee
14. Calculate four yearly moving averages for the following data. Also, compute short term

Fr
fluctuations.
Year: 2006 2007 20082009 2010 2011 2012 2013 2014 2015 2016 2017

for
ur
Sugar Production 37.4 31.1 38.7 39.5 47.9 42.6 48.4 64.6 58.4 38.6 51.4 84.4
(Lakh tonnes):
1 The number of traffic offences committed in a certain city over a period of 3 years is given in
s
ok
Yo

the following table:


Jan-March April-June Jul^-Scpt Oct-Dec
Bo

2016 74 56 48 69
2017 83 52 49 81
re

2018 94 60 48 79

Draw a graph illustrating these figures. Calculate suitable moving averages and plot them
ou
ad

on the same graph. Comment on the result.


16. Using the following data, find out the secular trend by using four quarterly moving
Y

average
nd
Re

Quarter Qi Q2 Q4
Fi

Year

2014 29 37 43 34

2015 90 42 55 43

2016 47 51 63 53

2017 45 49 60 48

ANSWERS

4.67,5.33, 6,7,8, 8.33,9 4.067,4, 4.03, 4.40, 4.40, 3.73


(i) 4.75,4.25, 4.5, 5.5, 5.75, 5.25, 5.5, 6.5, 6.75
(iv) 4.625, 4.875, 5.125,5.375, 5.625, 5.875, 6.125
(vii) Long term increasing trend with a four yearly cycle.
I- 2.3,5.6,12.3,19.6,31.0,34.3,26.3,20.0,11.3,5.3,2.0 470,484.6,503.2,515.2,517.8,523.6
TIME SERIES 23.17

● > 29,32,36.8,41.2,46.2,54.8,53.6,54,57.6,56.2,53.8 368,381.6,390.4,402.2,419.8,439.8


8. 39.75, 54.75, 70.75, 84.75, 92, 90.75 1.4,1.7, 2.1,2.8, 3.4,3.5, 3.1, 2.3

Profits pick up from March and go on increasing till September and then go down during
winter months.

I 3 term averages are: 263.3,266.3,257.7,260,269,287,304, 307.7,311,298.3,302.7,297.3,308,


308, 314, 306.7

We observe that there is a steadily increasing long term trend through there are some
seasonal variations.

’ ■■. 39.75, 54.75, 70.75, 84.75, 92.00, 90.75, 81.00, 65.75, 49.75, 34.75
15. 62.875, 63.500, 63.125, 64.750, 67.625, 0, 70.875, 70.500

This shows long term increasing trend.


16. 37.125, 39.125, 41.25, 43.875, 45.875,47.875, 50, 52.25, 53.25, 52.75, 52.125, 51.125

w
This shows a long term steady increase and then tapering off.

F lo
e
Fre
for
r
You
oks
eBo
ad
our
Re
dY
Fin
CHAPTER 24

FINANCIAL MATHEMATICS-I

24.1 RECAPITULATION

We have learnt about present value and o dinary annuity in class XI. Let us recapitulate some
important points to understand about perpetuity and sinking fund.
The present value provides an estimate of what we should invest today to have a certain amount

w
of money at a specific point in future.

F lo
PRESENT VALUE [/money is zuroth i per period, then present value P of mnoiint S due n periods hence is
given by P = S (1 + 0"”-
An annuity is a sequence of equal payments made at equal intervals of time. In an ordinary

ee
annuity the first payment is made at the end of the fir -t payment period. The term of an ordinary

Fr
annuity begins with the beginning of first payment period and ends on the day of last payment.
The amoimt of future value of an ordinary annuity is the sum of the compound amoimts of all
for
the payments accumulated at the end of the term.
ur
AMOUNT OF AN ANNUITY The amount or future value S of an ordinary annuity of^ R per period for n
s

periods at the rate i per period is given by


ook
Yo

S = R
{i+iT-i
eB

The present value of an ordinary annuity is the sum of the present values of all payments.
our
ad

PRESENT VALUE OF AN ANNUITY The present value P of an ordinary annuity of ^ R per payment
period for n periods at the rate ofi per period is given by
-II
Y

i-(i+0
P=Rl
Re
nd
Fi

24.2 SINKING FUND

t is an annuity created for accumulating money that can be used for paying off a financial
obligation at some future predecided date.
For example, a person may decide to accumulate money for his child's higher education
expenditure to be done after 10 years by setting aside a certain sum of money every year which
will earn him compound interest.
Sometimes an individual or a company accumulates money, probably by periodic deposits,
either to repay the principal of a loan in one installment or for the expansion of business etc. In
such cases, we say that a sinking fund is created. In this book, we shall assume that any sinking
fund considered is created by investing equal periodic payments.
AMOUNT IN A SINKING FUND Thc amount in a sinking fund at any time is the amount of the annuity
formed by the payments. Thus, the amount S in a sinking fund at any ti)ne is given by
24.2 APPLIED MATHEMATICS-XII

S = R
(1+/)"-!
or. S = R
i

R = Periodic deposit or payment


n = Number of periodic deposits or payments
i = Interest per period
SINKING FUND PAYMENT The periodicpni/meut of^ R required to accumulate a sum ofX S over n periods
with interest charged at the rate i per period is given by
iS
R = or, R =
(1+0" -1 ^rii

w
ILLUSTRATIVE EXAMPLES

r -viru. I
Hoiv much should a company set aside at the end of each year, if it has to buy a machine

F lo
expected to cost ^ 200,000 at the end of 6 years and rate of interest is 10% per annum compounded
annually? {Given (1.1)^ = 1.771),

ee
SOLUTION Let ? K be set aside at the end of each year. Since the company wants ? 200,000 at the

Fr
end of 6 years. Therefore,
10
S = 200,000, n = 6, / = = 0.1
for
100
ur
iS
R =
(i+0"-i
s
ook
Yo

0.1 X 200,000
R =
(1.1)^-1
eB

20,000
R =
1.771-1
r
ad
ou

20,000
R = = 25,940.33
0.771
Y

Thus, to accumulate ? 200,000 after 6 years the company should keep aside ? 25,940.33 every
Re

year at 10% per annum compounded annually.


nd

A sinking fund is created for the redemption of debentures of^ 100,000 at the end of 25
Fi

years. How much money should be provided out of profits each year for the sinkingfund, if the investment
can earn interest 4% per annum ? {Use (1.04)^^ = 2.6658).
SOLUTION Suppose ? R are provided out of profits each year for the sinking fund. Then,
iS
R = , where S = 100,000, n - 25 and i = = 0.04
II
(1 + iT -1 100

0.04 X 100,000 4,000 4,000 _ 4,000


R = = 2401.25
{imf^-l' (1.04)25-1 “2.6658-1 “1.6658
Thus, ^ 2401.25 are provided out of profits each year for the sinking fund.
Ml-. A company establishes a sinkingfund to providefor payment oft 250,000 debt, maturing
in 4 years. Contributions to the fund are made at the end of every year. Find the amount of each annual
deposit if the interest is 18% per annum. (L/sc(1.18)^ = 1.9387)
SOLUTION Let each annual deposit in the sinking fund be of ? R. Since the company requires
? 250,000 at the end of 4 years. Therefore,
FINANCIAL MAThEMATICS-l 24.3

18
S = 250,000,« = 4 and / = = 0.18
100

R =
iS
=>R =
0.18 X 250,000 45,000 _ 45,000 = 47938.64 -(i)
Now,
(I.IS)"^ -1 1.9387 -1 " 0.9387

Hence, the required amount = ? 47,938.64


i;XAMPI.E- i A conipanif intends to create a sinkingfund to replace at the end of 20th year assets costing
? 500,000. Calculate the amount to be retained out of profits every year if the interest rate is 5%. {Given
(1.05)^° =2.6532)
SOLUTION Let ? R be retained out of profits every year for 20 years to accumulate f 500,000.
Then,

ow
iS
R = , where 5=500,000,/= — = 0.05 and n = 20
100

R = 0.05 X 500,000 _ 25,000 ^ 25,000 = 15,122.18


(1.05)^° -1 2.6532-1 ~ 1.6532

e
re
Thus, ? 15,122.18 are retained out of profits every year for 20 years to accumulate ? 500,000.
EXAMPLES

rFl
A ifrm anticipates a capital expenditure oft 50,000for a new ecjuipnient in 5 years. How

F
much should be deposited quarterly in a sinkingfund carrying 12% per annum compounded quarterly to
provide for the purpose ? (Given (1.03)^® = 1.8061)

r
ou
SOLUTION Let ? R be deposited quarterly to accumulate 150,000 in 5 years. Then,
iS fo 12
ks
R = , where S = 50,000, n = 5 x 4 = 20 and i = = 0.03
4x 100
(1 + /■)" -1
oo

0.03 X 50,000 1500 1500


R = = 1,860.81
(1.03)^° -1 1.8061-1 0.8061
Y
B

Hence, the required amount = 11860.81.


re

EXAMPLE 6 A person has set up a sinking fund in order to have ? 100,000 after 10 years for his
children's college education. How much amount should be set aside bi-annually into an account paying
ou
Y

20
ad

5% per annum compounded half-yearly ? (Given (1.025) = 1.6386)


SOLUTION Let ? R be set aside bi-annually for 10 years in order to have 100,000 after 10 years.
d

Then,
5
in
Re

5=100,000, =10x2 = 20 and / = = 0.025


2x100
F

/5 0.025x100,000 2500 2500


R = = 3,914.81
(1+0" -1 (1.025)^*^-1 1.6386-1 0.6386

Hence, the required amount = 13,914.81.


EXAMPLE ■ A machine costs a company 152,000 and its effective life is estimated to be 25 years. A
sinking fund is created for replacing the machine by a new model at the end of its life time, when its scrap
realizes a sum ofX 2500 only. The price of the neio model is estimated to be 25% more than the price of
present one. Find what amount should be set aside at the end of each year
25
out of the profits for the sinking
fund, if it accumulates at 3.5% per annum compound? (Given (1.035) = 2.3632)
SOLUTION Let ? R be the amoimt set aside each year. Since the cost of the new machine is 25%
more than the cost of the present one. Therefore,
25
Cost of the new machine = ? 52,000 + X 52,000 = ? 65,000.
100

Scrap value of the present machine = ? 2,500.


24.4 APPLIED MATHEMATICS-XII

So, net amoimt required at the end of 25 years to purchase the new model = ^ (65,000 - 2,500)
= ? 62,500.
Now,
iS 35
R = , where S - 62,500, ii = 25 and i = = 0.035
100

0.035x 62500 21875 2187.5


R = = 1,604.68
(1.035)^^-! 2.3632-1 1.3632
Thus, ? 1,604.68 are set aside each year out of the profits to purchase the new model of the
machine.

EXERCISE 24.1

How much money must be set aside each year so as to replace a machine that wilt cost
? 15,000 after 8 years ? The rate of interest being 12% per annum compounded annually.
(Use (1.12)^ =2.4759)

w
For his son's education, a man sets aside ? 4,000 at the end of every year for 8 years. If the

(Given (1.15)*^ = 3.0590).

F lo
rate of interest is 15% per annum what is the value of this sinking fund ?

3, A person has set up a sinking fund in order to have ? 20,000 in 10 years for his children's

e
Fre
college education. How much should he set aside each quarter into an account paying 10‘/o
compounded quarterly ? (Use (1.015)'*'^ = 1.8140) for
A machine costs ? 98,000 and its effective life is estimated to be 12 years. If the scrap value is
X 3,000 only, what should be retained out of profits at the end of each year to accumulate at
r
compound interest at 5% per annum for depreciation so that a new machine can be
You
oks

purchased at the same price after 12 years. (Given (1.05)^^ = 01.7958).


eBo

To save for a child's education, a family decides to invest ? 3000 at the end of each six month
period in a fund paying 8% per year compounded semi-annually. Find the amount of the
investment at the end of 18 years. (Given (1.04)^^ = 4.1039)
our
ad

- A firm anticipates a capital expenditure of ? 10,000 for a new equipment in five years. How
much should be deposited quarterly in a sinking fund earning 10% per year compounded
quarterly to provide for the purchase ? (Use (1.025)“^ = 1.6836)
dY
Re

Suppose a machine costing ? 50,000 is to be replaced at the end of 10 years, at that time it
will have a salvage value of ? 5,000. In order to provide money at that time for a new
Fin

machine costing the same amount a sinking fund is set up. The amount in the fund at that
time is to be the difference between the replacement cost and salvage value. If equal
payments are placed at the end of each quarter and the fund earns 8% compounded
quarterly, what should each payment be? (Use (1.02)^*^ = 2.2080)
8. Mr X plans to set his son for higher studies abroad for 10 years. He expects the cost of the
studies to be ? 200,000. How much must he set aside at the end of each quarter for 10 years
to accumulate this amount, if money is worth 6% compoimded quarterly.
(Given (1.015) =1.8140)
A machine bought for ? 50,000 is depreciated at 10% for 8 years. To make up the loss due to
depreciation a sinking fund is created by setting aside a sum of money every year. What is
the value of each payment if the accumulated amoimt in the fund is equal to total
depredation, and if the interest is 5% compounded annually? (Use (1.05)® = 1.4774)
FINANCIAL MATHEMATICS-1 24.5

10. A sinking fund is created from the redemption of debenture of ? 100,000 at the end of 20
years. How much money should be provided out of profits each year for the sinking fund, if
the investment can earn interest 8% per annum compounded annually.
(Use (1.08)^^ =4.6609)
11. Mr X, finance manager of ABC Reliable products, wants to establish a sinking fund of
^ 200,000 in five years for expanding the plant capacity of liis company. The UVW Trust
Company specialising in handling investment portfolios, advised him that Mr X's annual
deposits with the trust could earn, under the most conservative investment policy, 5.5%
compounded per annum. How much from the earnings of his firm should Mr X contribute
to the fund annually in order to accumulate f 200,000 in five years ?
(Use(1.055)^ = 1.2762)

w
12. A sinking fund is created for redemption of debentures of ? 2,500,000 at the end of 20 years.
How much money should be provided out of the profits of each year for the sinking fund if

Flo
the investment can earn interest at 8% p.a.
13. Mr. Ranbir plans to build his house after 5 years at a cost of ? 6,000,000; send his son to U.S.

e
for higher studies 8 years hence, for which he anticipates the cost at that time will be

re
? 2,400,000. How much should he save in the beginning of each year to accumulate the

F
required amount at the end of 5 years and 8 years if rate of interest is 8% p.a. effective?
A machine costs a company ? 52,000 and its effective life is estimated to be 12 years. A
ur
14.

r
fo
sinking fund is created for replacing the machine by a new model at the end of its life time,
when its scrap realizes a sum of 15,000 only. The price of new model is estimated to be 25%
ks
higher than the price of the present one. Find what amount should be set aside at the end of
Yo

each year, out of the profits, for the sinking fund, if it accumulates at 10% effective.
oo

15. Mr. Shyam plans to send his son to Australia for higher education 10 years hence. He
B

anticipates the cost at the time will be ? 1,700,000. How much should he save in the
beginning of each year to accumulate this amount at the end of 10 years if rate of interest is
e

9% p.a. effective.
ur
ad

16. A machine is bought for ? 320,000. Its effective life is 8 years, after which its salvage value
Yo

would be ? 25,000. It is decided to create a sinking fund to replace this machine at the end of
its effective life by making half yearly payments that will earn an interest of 8% p.a.
compounded half yearly. If it is known that the cost of machine increases by 5% p.a.
d
Re

Calculate the amount of each payment to the sinking fund.


in

17. A house is sold for ? 50,000 down and 10 semi-annual payments of ? 5,000 each, the first due
F

3 years hence. Find the cash price of the house if money is worth 6% p.a, compounded
semi-annually.
18. A person borrows ? 200,000 at the rate of 6% p.a. and agrees to pay 20,000 from the end of
3rd year for next 5 years. How much he should pay in equated amuial installments for the
next 5 years to pay off the amount borrowed together with interest.
Ai^SWERS

1. ? 1219.59 2. ? 54,906.67 3. ? 368.55 4, ? 5968.83

5. ? 232,792.50 6. ? 365.71 7. ? 745.03 8. ? 3,685.50

9. ? 2,982.40 10. ^2,185.25 11. ^39,826.21 12. ? 54,630.52

13. 1 - 5 Years: ? 1,155,901.99; 6-8 Years: ? 208,921.69, 14. ? 2,805.80

15. ? 102,655.19 16. ? 20,517.54 17. ^8,6791.14 1?^. ? 44,626.77


24.6 APPLIED MATHEMATICS-XII

HINTS TO SELECTED PROBLEMS

8
(1.15)°-1 4,000(3.0590-1) 8236
S=4000- - >
= 54,906.67
0.15 0.15 0.15

iS
Take S = 20,000, /; = 10 x 4 = 40 and / = —-— = 0.015 and use the formula R =
4x 100 (i + 0"-l
4. Net amount required after 12 years = Total cost of machine - Amount realised from scrap
= ? (98,000 - 3000) = ? 95,000.
iS
Take S = 95,000, n =12 and i = = 0.05 and use R =
100 (1+0”-1

w
8
Take R = 3,000, =18 x 2 = 36, i = = 0.04 and use S = R ■
2x 100 2

F lo
10 iS
Take5 = 10,000,« =5X 4=20,2 = = 0.025 and use R =
4x100 (l+i)"-l

ee
8

Fr
Here, S = 50,000 - 5,000 = 45,000,72 = 10 x 4 = 40 and i = = 0.02.
4x100

Now, use R =
iS for
ur
(l + 0”-i
6 iS
s
8. Here, 5 = 200,000,72 =10x 4 = 40,2 = . Use R =
ook
Yo

4x 100 (1 + 2)” - 1
eB

10 'l
4. Value of machine after 8 years = ? ■ 50,000 1 - = ? 21,524
100 j
r
ad
ou

Total depreciation = ? 50,000 - ? 21,524 = f 28,476.


iS
Now, take 5 = 28,476,2 = and 22 = 8 to find R by using the formula R =
100 ^ ^
Y

(i+0"-i
55
Re

iS
nd

Here, 5 = 200,000,2 = = 0.055 and 72=5. UseR =


100 (1+0"-1
Fi

24.3 PERPETUITY

In class XI, we have learnt about an ordinary annuity in which periodic payments are made at
the end of each payment period and the number of periodic payments is finite. In this section,
we will study ordinary annuities not having finite term. Such an annuity is called a perpetuity as
defined below.

PERPETUITY A perpefuity is a special form of an ordinary annuity in which period payments continue
forever.
In other words, a perpetuity is an ordinary annuity without any end.
Clearly, the term of a perpetuity is not finite.
The concept of perpetuity is often used in financial theory, like the Dividend Discount Model
etc. Following are some examples of perpetuity:
Fixed coupon payments on permanently invested money is an example of perpetuity.
24.7
FINANCIAL MATHEMATICS-I

Another example of a perpetuity is consol bonds. Consol bonds are fixed income bonds without
a maturity.
If a buyer obtains a property and then chooses to rent it out, he/she receives an infinite cash flow
as long as he/she owns the property (assuming that the property always on rent). This is also an
example of perpetuity.
The "amount" of a perpetuity is undefined, since it increases beyond all bonds as time goes on.
There are two types of perpetuities;
Type 1: Perpetuity payable at the end of each payment period.
Type n: Perpetuity payable at the beginning of each payment period.
Let us find the present values of these two types of perpetuities.
24.3.1 PRESENT VALUE OF PERPETUITY PAYABLE AT THE END OF EACH PAYMENT METHOD

w
Consider a perpetuity of ^ K payable at the end of each period, the first payment due one period
hence Let the money be worth i per period i.e. / is the interest per rupee per payment period.
The present value of this perpetuity is defined to be that sum of money which, invested now at

F lo
the rate / per period, will yield ^ Rat this end of each period forever. The present value of the
perpetuity is the sum of the present values of all payments.

ee
The present value of the first payment = R (1 +/) ^

Fr
_2
The present value of the second payment = R (1 + 0
The present value of the third payment = R (1 +1) and so on.
for
ur
So, the present value P of the perpetuity is given by
s

p = R (1 + 0"’ + ^ (1 + 0“^ + ^ (1 + 0"^ + +


ook
Yo

Clearly, RHS is an infinite G.P. with first term R(1 +/)"^ and common ratio (1 +/)“ .
eB

-1 2 a

P=AM -1 Using :a + ar + ar +...


1-r
i-(i + 0
r
ad
ou

R
=> p
P R/(l+0
^—
R
1
1- (1 + 0-1 i
Y

(1+0
Re

the present value of ? R payable at the end of each payment period when money is
nd

Hence,
Fi

worth t per period is —.

Thus, R is the cash equivalent of an unlimited number of periodic payments of ^ R each, the first being
due one period hence.
24.3.2 PRESENT VALUE OF PERPETUITY PAYABLE AT THE BEGINNING OF EACH PAYMENT
PERIOD

Let us consider a perpetuity of ^ R payable at the beginning of each payment period, the hrst
payment due now. Let the money be worth / per period. In this type of annuity, we find that
Present value of first payment = R
-1
Present value of second payment = R (1 + 0
Present value of the third payment = R (1 + 0
“3
Present value of the fourth payment = R (1 + 0 so on.

So, the present value P of the perpetuity is given by


24.8
APPLIED MATHEMATICS-XII

-1
P=R + R(\+i) + R(l+i)~^ + R{l+i)-^ + +

R R R
P = = R+-
-1 1
1-(!+/) 1- i I I
1 +i

Hence, if money is worth i per period, then the present value of a perpetuity of ^ R payable at
the beginning each period is K + —.
i
I
This type of perpetuity can be considered as an initial payment of^ R followed by a perpetuity

ow
of^R payable at the end of each period. Therefore present value of this type of perpetuity is R + present
value of perpetuity o/T R payable at the end of each period i.e. R +
24.3.3 PRESENT VALUE OF A GROWING PERPETUITY

e
A growing perpetuity is a perpetuity in which periodic payments do not remain fixed rather

re
these payments keep on growing at the same constant rate of growth. If the rate of growth of the
periodic payment is x %, each payment will be x % more than the payment received before it.

Frl
F
Consider a growing perpetuity of ? R, which grows at the rate of g per rupee per period,
payable at the end of each payment period. Let the money be worth i per period. Then,
ou
Present value of first periodic payment = R{1 +/)“\

osr
Since periodic payments grow at the rate of ^ per rupee per period.
Second periodic payment = R+Rg = R{l+g) kf
Present value of second periodic payment = /?(!+ (1 +
oo

Third periodic payment = R (I + g) + R {1 + g)g = R {1 + gf


Y
B

Present value of third periodic payment = R (1 + y)^(l + i)~^


and so on.
re
Y

So, the present value P of the perpetuity growing at the rate g is given by
u

P = R(1 + i)-^ + R(l +g) (1 +i)-2 + R (1 + gf(i + ,^-3 +


ad
do

-1
P =
R(l+0 R(l+0"^_ R
-1
l-(l+,y)(l + 0 1-^t^ i-g
in

1+i
Re

Hence, the present value of a perpetuity of ? R growing at the rate of g per rupee per period,
F

payable at the end of each payment period, when money is worth i per period is
i-g

ILLUSTRATIVE EXAMPLES

I !//■!' / PERPETUITY PAYABLE AT THE END OF EACH PAYMENT PERIOD

EXAMI-U: I Find the present value of a perpetuity oft 5,m payable at the end of each year, ifmoneu is
xvorth 5% compounded annually.
SOLUTION Let P be the present value of the perpetuity. It is given that: R =5,000 and
i - = 0.05.
100

= ? 100,000
I 0.05

Hence, the present value is ? 100,000. This means that a sum of ? 100,000 invested now at 5%
compounded annually will yield X 5,000 at the end of each year forever.
24.9
FINANCIAL MATHEMATICS-1

I'X AMii r: Find the present vniueofa sequence of payments oft 8,000 made at the end of each 6 months
and continuing forever, if money is worth 4% compounded semi annually.
SOLUTION Let P be the present value of the given perpetuity. It is given that R = 8,000 and
4
i = — = 0.02.
200

P=^=> 400,000
i 0.02

Hence the present value of the given perpetuity is t 400,000. It means that a sum of t 400,000
invested now at 4% compounded semi annually will fetch t 8,000 semi annually forever.
l-XAMn.P 3 At 6% coiwerted quarterly, find the present value of a perpetuity oft 4,500 payable at the
end of each quarter.
SOLUTION Let P be the present value of the given perpetuity. It is given that R =t 4,500 and
— = 0.15.
400
R 4,500
P=— => P=? = t 300,000
0.015

w
1

Hence, the present value of? 300,000.


liXAMPLl- ●' Salim holds a perpetual bond that generates
believes that the borrower is credit worthy and that
Compnite the present value of this perpetuity.
SOLUTION

100
F lo
Let P be the present value of the perpetuity. We have,
= 0.08.
and annual return oft 50,000 each year. He
an S% interest rate wilt be suitable for this bond

for F
ree
R =? 50,000 and
.

P=-^ P = ^ ^ 625,000
I 0.08
Your

It means that if a company issues bonds each of t 625,000 at 8% per annum. The purchaser will
ks
eBoo

receive 150,000 at the end of each year forever.


rxAMl’Li; .s Your grandfather is retiring at the end of next year. He zuould like to ensure that his heirs
receive payments oft 75,000 a year fortwer, starting when he retires. Hozv much does he need to invest in
ad

the beginning of this year to produce the desired cash flow, if money is zoorth 6% compounded annually.
our

= 0.06.
SOLUTION It is given that R = t 75,000 and /
The amount to be invested by your grandfather is the present value of a perpetuity of ? R when
Re

money is worth 6% compounded annually.


Y
Find

Let P be the present value. Then,


75,000
p=:^=> P=? = ? 1,250,000
I 0.06

Hence, your grand father has to invest ? 1,250,000.


rxAMi’LtIf money is zvorth 5% compare the present value of a perpetuity of t 2,000 payable at^ the
end of-tooeach year zui'th that of an ordinanf annuity of t 2,000 per year for 100 years. (Given
(1.05) ■■■ = 0.0076)

SOLUTION Let P be the present value of a perpetuity of ? 2,000 payable at the end of each year
when money is worth 5%. It is given that
i = — = 0.05 and R = 2,000.
100

P=-^
i 0.05
24.10
APPLIED MATHEMATICS-Xll

Let Pj be the present value of an ordinary annuity of ? 2,000 per year for 100 years. Then,
-n

P,=R.
i

We have, R = 2,000, i = — = 0.05 and » =100


100
-100
1-(1.05) 1 -0.0076
Pi =? 2,000
0.05
● =? 2,000 ^ 2,000 X 0.9924 = ? 39,696
0.05 0.05

We observe that the present value of the perpetuity is more than that of ordinary annuity.

ow
i i>l'c II ON THE PRESENT VALUE OF PERPETUITY PAYABLE AT THE BEGINNING OF EACH PAYMENT
PERIOD

nXAMPl.r ●
Find the present value of a peiyetiiity of ? 3,120 payable at the beginning of each year, if
money is ivorth 6% effective.

e
re
SOLUTION We have, R = 3,120 and i =

rFl
= 0.06. Let P be the present value of the given

F
perpetuity. Then,
R 3120'!
P = P+— => P=? 3120 +

r
= ? (3120 + 52,000)=? 55,120
ou
/ 0.06

Hence,the present value is ? 55,120.


k sfo
EXAMPLi .
How much money is needed to endure a series of lectures costing ? 2,500 at the beginning of
oo

each year indefinitely, if money is worth 5% compounded anmialh/?


Y
B

SOLUTION Here we have to find how much money should be invested now that would
provide for an unlimited number of payments of ? 2,500 each year, the first due now. So, it is a
re

perpetuity of ?2,500 payable at the beginning each year, if money is worth 5% compounded
ou

annually. Thus, we have


Y
ad

R =2,500 and/=— = 0.05


100
d

Let P be the present value of this annuity. Then,


in
Re

R 2,500
P = P+—=> P=? 2,500 +
F

= ? 52,500
/ 0.05

Hence, required sum of money is ? 52,500.


I lire ni ON FINDING THE RATE OF INTEREST. IF THE VALUE AND THE PRESENT VALUE OF A
PERPETUITY ARE GIVEN

EXAMPLI
If the cash equivalent of a perpetuity o/? 1,200 payable at the end of each quarter is
? 96,000, find the rate of interest convertible quarterly.
SOLUTION Let the rate of interest be r% converted quarterly. Then, / = It is given that the
present value of a perpetuity of ? 1,200 payable at the end of each quarter is ? 96,000.
r
i.e.
P=? 96,000 R= ? 1,200 and / =
400
24.11
FINANCIAL MATHEMATICS-I

1,200 1,200x400 1,200x400


P=4- => 96,000 = r/400
i
96,000 =
r
r =
96,000

Hence, the rate of interest is 5% convertible quarterly.


hfiu- /I ON FINDING THE PRESENT VALUE OF A PERPETUITY
The present value of a perpetual income of ^ Rat the end of each 6 months is ^ 144,000.
Find the value ofR if money is worth 6% compounded semi-annually.
SOLUTION It is given that the present value of a perpetuity of R payable at the end of each 6
months is ? 144,000, if money is worth 6% compormded semi-annually. Thus,we have
P = ? 144,000, / = -It
200
= 0-03
R
P =4- => 144,000 = 0.03 R =4,320

low
I

Hence, R =? 4,320.

fy/v ’ ON FINDING THE PRESENT VALUE OF A GROWING PERPETUITY


EXAMm Aanishi purchased a number of stocks of Reliance. At the end offirst year, she received a
11

ee
payment oft 6,000, which grows at a rate of 4% per year and continues forever. If the discoimt rate
IS

F
Fr
10%, find the present value ofAarushi’s investment.
SOLUTION We have, R =6,000,/= 10 and ^=4. Let P be the present value of Aarushi’s
investment. Then, for
ur
R 6000
p = => p=t = t 100,000
10-4
ks
Yo

Hence, the present vale of stocks Aarushi purchased is 1100,000. This is useful for Aarushi
oo

because she can compare the present value of perpetuity to the amount she paid for the stock. If
eB

the present value is higher than she paid, it was a wise investment for her.
EXERCISE 2^.2
r
ou
ad

Using an interest rate of 6% per annum, compute the present value of the receipt of
t 27,000 at the end of each year in perpetuity.
Y

What sum of money invested now would establish a scholarship oit 6,000 to be awarded at
Re
nd

the end of each year to a deserving candidate, if money is worth 4% compounded annually?
Find the present value of a perpetuity of t 7,800 payable at the end of each year, if money is
Fi

3.

worth 6% effective.
4, At 6% converted quarterly, find the present value of a perpetuity of 145,000 payable at the
end of each quarter?
Find the present value of a sequence of payments of t 14,000 made at the end of each 6
months and continuing forever, if money is worth 7% converted semi annually.
If money is worth 5% compare the present value of a perpetuity of ? 2,500 payable at the
end of each year with that of an ordinary annuity of t 2,500 per year for 200 years. (Given
(1.05)^°° = 0.00005782).
7. What sum invested at 8% compounded semi annually will provide payments of t 1,000
each at the end of each 6 months forever?
H. Find the present value of a perpetuity of ^ 7,800 payable at the beginning of each year, if
money is worth 6% effective.
24.12
APPLIED MATHEMATICS-Xll

9. If the cash equivalent of a perpetuity of ^ 300 payable at the end of each quarter is ? 24,000.
Find the rate of interest convertible quarterly.
10. The present value of a perpetual income of R at the end of each 6 months is ? 36,000. Find
the value of R if money is worth 6% compounded semi-annually.
I i. At what rate converted semi annually will present value of a perpetuity of ^ 450 payable at
the end of 6 months be ? 20,000?
12. Vinod wants to retire from his job and get hold of ? 30,000 per months. He wants the money
to go to the future generation after his death. He will earn interest rate of 8% compounded
annually. What is the total amount he will need to achieve the perpetuity goal?
An investor plans an investment where the cash flow payments are ? 50,000 per year. If the

ow
required rate of return is 8% and the cash flow payments grow by 3% every year and paid
indefinitely, find the present value of investment.
Find the amount to be deposited
- into an endowment fund that iIS to be compounded
annually at the rate of 12% p.a. to provide for an annual scholarship of ? 36,000 for an

e
indefinite period of time.

re
rFl
15. How much money is needed to ensure a series of lectures costing ? 4,500 at the beginning of
each year indefinitely, if the money is worth 6% compoimded armually.

F
In. How much money is needed to ensure a monthly pension of ^ 80,000 at the beginning of
each month forever, if the money is worth 10.5% p.a. compounded monthly.

r
17.
The principal amount demanded by a bank to guarantee a perpetual annuity is reduced by
ou
fo
25% after rate of interest compounded annually in increased by 3%. Find out of the rate of
ks
interest allowed by the bank after reduction in principal amount.
: How much money is needed to ensure a pension of ? 42,000 at the end of each year forever
oo

that grows at the rate of 3% p.a., if the money is worth 10% p.a. compounded yearly.
Y
B

ANSWERS

1. ^450,000
re

2. ?=150,000 3. ^ 130,000 4. ^3,000,000


5. ? 400,000 6. ^50,000,^49,997 r 25,000 8. ? 137,800
ou

T 5%
Y

HI. ^1080
ad

11. 4.5% 12. ? 4,497,750 13. ^ 100,000


● ? 300,000 15. ? 79,500 16. ^9,222,857 17. 12% p.a. IS. ^600,000
d

24.4 BONDS
in
Re

Companies and corporations need money to run their business operations. Also, municipalities
and governments require lot of money for infrastructure development and to run various
F

welfare schemes for general public. To raise money these origanisations issue bonds for
investors willing to lend money for a certain period of time. Thus, bonds are loans made to large
organisations like municipalities, national governments and companies. In finance, a bond is an
instrument of indebtness of the bond issuer to the bond holders. We may also describe the term
'bond' as follows.

BOND A bond is a debt instrument that provides a periodic stremn of interest payments to investors (or
buyers) while repaying the principal sum on specific maturity date.
A bond's terms and conditions are contained in legal contract between the buyer and the issuer,
known as indenture.

The issuer of bonds agree to pay back the bond holder the face value of the bond on a specific
date (normally called maturity date) and periodic stream of interest payments.
A bond is characterised by the following terms:
MATURITY A bond's maturity is the time period until the principal is scheduled to be repaid.
24.13
FINANCIAL MATHEMATICS-1

FACE VALUE The face value (also known ns par value) of a bond is the price at which the bond is sold to
investors (or bu\/ers) at the time of issue.
It is also the price at which the bond is redeemed at maturity.
REDEMPTION PRICE The redemption price of a bond is the amount the bond issuer pays at maturity.

rw
The redemption price is usually equal to the face value, in which case the bond is said to be
redeemed at par. Sometimes, the bond may be redeemed at a price higher than the face value, in
such cases, the bond is said to be redeemed above par. The redemption price is stated on the
bond.

The statement "a bond is to be redeemed at 110" means that the redemption price of the bond is

e
110% of its face value.

luo
r
DIVIDEND RATE The rate at which a bond yields interest is called the dividend rate or the nominal

F
interest rate.

The dividend is always computed on the face value of the bond.


The issuer pays the interest at pre-determined intervals (usually semi-annually or annually) and
returns principal on the maturity date.

oF
rs
COUPON RATE A bond's coupon rate denotes the annua! interest rate paid by the bond issuer to the

k
bond holder.

o
Coupon rate is determined as a percentage of the bond's face value or par value.

o
f
Bonds are bought in the open market at a price usually different from the redemption price
or

face value. This is due to the fact tliat the purchaser or investor expects to realize an income from
o
Y
his investment at a rate which is usually higher than the dividend rate. The price at which an
Y

investor purchases a bond is called the purchase price. The purchase price may be higher or
rB

lower than the face value of the bond. The return an investor realizes on a bond is called the
bond yield. Note that bond yield is the annual net profit that an investor earns on an investment
whereas dividend is the total interest earned on investment in a bond.
ue

The owner of a bond is entitled to receive all future dividends and the redemption price at
od

maturity.
ad

FAIR PRICE The fair price of a bond is the open market value that is acceptable to both the buyer and the
in

seller.

24.5 VALUATION OF BONDS


Re

Bond valuation is a technique for determining the theoretical fair value or fair price of a
F

particular bond. The fundamental principle of bond valuation is that its fair value is equal to the
sum of present values of its expected cash flows. Cash flow is the cash that is estimated to be

received in future from investment in a bond. There are only two types of cash flows that can be
received from investments in bonds (i) coupon payments i.e. periodic interest payments (ii)
principal payment at maturity i.e. payment received on redemption. The coupon payments are
received at regular intervals as per the bond agreement, and final coupon payment plus
principal payment is received at the maturity. There are different approaches to determine the
fair value or fair price or purchase price of bonds. In this book, we will study the two main
approaches viz. (i) Present value approach (ii) Relative price approach.
24.5.1 PRESENT VALUE APPROACH

In the present value approach, we first calculate the present value of each expected cash flow
and then we add all the individual present values to obtain the value or fair value of purchase
price of a bond. Let there be a bond with the
Face value = F

Redemption price or. Maturity value = C


24.14
APPLIED MATHEMATICS-XII

Yield rate or market interest rate per period = /


Number of periodic payments or. Number of cash flows = n

Periodic interest (dividend) payment or. Coupon payment = R


Value of bond or Purchase price of bond or. Market price of bond = V
Then,
Bond value = Present value of first periodic payment + Present value of second
periodic payment +...+ Present value of nth periodic payment
+

R
PresentRvalue of Maturity value or. Redemption price
R C
Bond value = +...+
l+i
(i+if (1+0" (1+0
n

1 f
O-
R l+i C
Bond value =
1+/ 1 II
1- (1+0
1+iJ

w
-It
i-(i+0

F lo
Bond value = R ■ ■ +C(l+0
-It

If a bond is redeemed at part, then C = F


Bond value = R ■
'l-(l+0 -II

■ +F(1+/)■''.

ree
i
for F
24.5.2 RELATIVE PRICE APPROACH

In relative price approach, the bond is priced relative to yield to maturity (YTM) of a benchmark,
usually a government bond of similar maturity or duration. This required return is then used to
Your

discount the cash flows, by replacing i in the present value approach, to obtain the market price
ks
eBoo

of the bond.

ILLUSTRATIVE EXAMPLES
ad

A company ABC Ltd has issued a bond having a face value ofX 10,000 paying annual
our

dividends at 8.5 The bond will be redeemed at par at the end of 10 years. Find the purchase price of this
bond if the investor wishes a yield rate of 8% (Given (1.08)"^° = 0.46319349).
Re

SOLUTION It is given that:


Y

f = Face value of the bond = ? 10,000


Find

n = Number of periodic dividend payments = Period of redemption = 10


Q

7 = Annual yield rate = 0.08


100

rs.5
R = Annual dividend = 8.5 % of face value = ? X 10,000 =?850
Uoo
The bond is redeemed at par.
C = Redemption price or Maturity value = Face value = ? 10,000.
Let V be the purchase price of the bond. Then,
-n
i-(i+0
V=R ■+C(l+i)
-It

1
24.15
FINANCIAL MATHEMATICS-!

-10
1-(1+0.08) -10
850- + 10,000 (1 + 0.08)
0.08

85000 10
U=? — {1-(1.08)-'°1+10,000 (1+1.08)
8

V=t {10,625 (1 - 0.46319349) +10,000 x 0.46319349}


V = ? (10,625 X 0.53680651 +10,000 X 0.46319349)
V (5703.57 + 4631.93) = ? 10,335.50
Hence, the purchase price of the bond is ? 10,33550.
A cot?ip{inif has issued a bond having face value of ^ 100,000, cani/ing an annual divided

ow
lXAMIM.

rate of 7% and maturing in 15 years. If the prevailing market rate of interest is 9%, and the bond is
redeemed at par, ifnd the bond value. Given (1.09)~^^ = 0.27453804).

SOLUTION We have,

e
F = Face value of the bond = ? 100,000

re
n

rFl
= Period of redemption = Number of annual dividends = 15

F
9
i = Annual yield rate = = 0.09
7

r
R = Annual dividend = 7 % of face value = ? X 100,000=? 7,000
ou
100
fo
ks
The bond is redeemed at par.
C = Redemption price or Maturity value = Face value = ? 100,000.
oo

Let V be the bond value. Then,


Y
B

-H
l-(i + 0 -n
U =R + C(l+0
i
re

-15
1-(1+0.09) -15
ou

V/ =? 7,000- 1+100,000 (1 + 0.09)


Y
ad

0.09

700^ 1 -(1.09)"^^1 +100,000 (1.09)


-15
d
in
Re

U =?
1700^000 (1 _ 0,27453804) + 100,000 X 0.27453804|
F

^700,000
V =? X 0.72546196 + 27453.804 = ? (56424.819 + 27453.804) = ? 83878.62
. 9

Hence, the bond value is ? 83,878.62.


EXAM IT..-, - A company has issued a bond having the face value o/? 100,000 C(7rri//n^rt coupon rate of
87o to be paid semi-annually and
-10
maturing in 5 years. If the prevailing market rate of interest is 77o,find
the bond value. G/wfi (1.035) = 0.70891881).
SOLUTION We have,
F = Face value of the bond = ? 100,000, n = Number of periods = 2x5=10
7
i = Semi-annual yield rate = = 0.035

R = Semi-annual dividend = ? 100,000 x = ? 4,000


200
24.16
APPLIED MATHEMATICS-XII

The bond is redeemed at par. Therefore,


C = Redemption price or maturity value = Face value = f 100,000.
Let V be the bond value. Then,
~}J

V =R
l-(l+0 -II

I
^+C(i+o

-10
1-(1 + 0.035) -10
V 4,000 ●+100,000(1 + 0.035)
0.035

4,000,000 -10
V/ =?
35
{1 -(1.035) +100,000 (1.035)

ow
4,000,000
v/ = ? (1 -0.70891881) +100,000x 0.70891881
35

V
4,000,000
35 X 0.29108119 + 70891.881 = ? (33266.421 +70891.881) = ? 104158.30

e
Hence, the bond value is ? 104158.30.

re

rFl
^ 10,000 nmi matures in 15 years at part. The nominal interest is

F
7%. What
is the price of the bond that will yield an effective interest of 8%? (Given
(1.08)“^''' =0.31524170).

r
SOLUTION We have,
ou
F = Face value = ? 10,000, n =Number of periods = 15 fo
ks
/ = Annual yield rate = = 0-08, R = Annual dividend = ? — X 10,000 = ? 700
100 100
oo

C = Maturity value = Face value = ? 10,000.


Y

Let V be the purchase price of the bond. Then,


B

V^R\
'l-(l+0 -n
-H
^+C(1+/)
re

I
ou

-15
1-(1 + 0.08)
Y
ad

700 -15
0.08
+ 10,000 (1 + 0.08)
d

70,000 -15
-15
V=f 1 -(1.08) + 10,000(1.08)
in
Re

70,000
V=^
F

=:>
(1-0.31524170)+10,000 x 0.31524170
8

70,000
l/ = ?
8 X 0.6847583 + 3152.4170 = ? (5991.6351 + 3152.4170) = ? 9144,05

Hence, the purchase price of the bond is ? 9144.05.


I XAMri 1
Find the purchase price ofa^ 20,000 bond, redeemable at the end of 10 years at 110, and
paying annual dividends at 4%, if the yield rate is to be 5% effective. (Given (1.05) -10 = 0.61391325).
SOLUTION It is given that
n = Number of period = 10, / = Yield rate = — = 0.05
100

R = Annual dividend = 4% of face value = ? — X 20,000 =?800


100
24.17
FINANCIAL MATHEMATICS-1

The bond is redeemed at 110. Therefore, the redemption price of the bond is 110% of its face
value. Thus,
110
C = Redemption price = ? 20,000 x = ? 22,000
100

Let V be the purchase price of the bond. Then,


-II
i-(i+0 -II
V =R\ ● +C(i+o
I

-10
1-(1+0.05) -10
V=^ 800 + 22,000 (1 + 0.05)
0.05

-10
K = ? 16,000 {1 -(1.05)"^“) + 20,000 (1.05)

w
V {16,000 (1-0.61391325) + 22,000 x 0.61391325}

Flo
V =t (16,000 X 0.38608675 + 22,000 x 0.61391325)
y = ? (6177.388 +13506.0915) = ? 19683.48

ee
EXAMPLE 6 Find the purchase price of a ? 50,000, 6% bond, dividends pai/nble semi-muiualhj,

Fr
redeemable at par in 10 i/ears, if the yield rate is to be 5% compounded semi-annually. (Given
-20
(1.025) = 0.61027094).
for
ur
SOLUTION It is given that
f = Face value of bond = ? 50,000, i = Yield rate = = 0.025
s
200
k
Yo
oo

R = Semi-annual dividend = ? — x50,000 = ^ 1500


eB

200

n = Number of periodic payments = 2 x 10 = 20


r

The bond is redeemable at par. Tlierefore, C = Redemption price = ^ 50,000


ou
ad

Let V be the purchase price of the bond. Then,


Y

-II
l-(l+0 -II
V =R + C(l+0
i
Re
nd

-20
1-(1+0.025) -20
Fi

=> V = ? 1500 { + 50,000(1+0.025)


0.025

-20 -20
60,000 {1-(1.025) + 50,000 (1.025)

=> V = ? [60,000 (1 - 0.61027094) + 50,000 x 0.61027094]


V (60,000x 0.38972906 +50,000x 0.61027094) - ? (23383.7436 + 30513.547) = ^ 53897.29
EXAMPLE? Find the purchase price of ? 70,000 bond, redeemable at th end of 10 years at ? 21,000 and
paying annual dividends at 4%, if the yield rate is to be 5% per annum effective. (Given
-10
(1.05) = 0.61391325)
SOLUTION We have,
F = Face value of the bond = ? 10,000
C = Redeemable value of the bond = ? 11,000
24.18
APPLIED MATHEMATICS-XII

R = Annual dividend = ? — X 10,000 = ?400


100

n = Number of periodic payments (dividends) = 10


i = Yield rate = = 0.05
100

Let V be the purchase price of the bond. Then,


-n
l-(l + 0
V =R\ }+C(l+i)
-n

-10

ow
V 400
1-(1 + 0.05) -10
●+11,000(1+0.05)
0.05

-10
8,000 <11-(1.05)
-10
+ 11,000 (1.05)

e
re
V [8,000(1 -0.61391325) +11,000x 0.61391325]

rFl
V = ^ (8,000 X 0.38608675 + ll,000x 0.61391325)

F
V = ? (3088.694 + 6753.04575) = ? 9841.73975 = ? 9841.74

r
ou
EXERCISE

fo
A f 100,000 bond paying annual dividends at 3.5% will be redeemed at par at the end of 6
ks
years. Find the purchase price of this bond, if the investor wishes a yield rate of 4.5% (Given
(1.045)“^ = 0.76789574).
oo

A bond has face value of ^ 10,000 and maturity period of 10 years. The nominal interest rate
Y

is 6%. What should be the price of the bond to yield an effective interest rate of 8%? (Given
B

(1.08)"^° =0.46319349).
re

An investor intends purchasing a 12 years ? 1,000 face value bond having nominal interest
rate of 6%. At what price the bond may be purchased now if it matures at par and the
ou
Y
ad

investor requires a yield rate of 4.5%? (Given (1.045)"^^ = 0.58966386)


A bond with face value of f 5,000 matures at par in 12 years. The nominal rate of interest on
d

bond is 12% per annum paid annually. What should be the price of the bond so as to yield
in
Re

an effective rate of return equal to 10% per annum? (Given (1.1)“^“ = 0.31863082)
A bond has a face value of ?1,000 and matures at par in 15 years. The nominal interest rate ISi:
F

9%. What is the purchase price of the bond to yield an effective interest rate of 8%? (Given
(1.08)"*^ = 0.31524170)
Find the purchase price of a ? 500,4% bond dividends payable semi-annually, redeemable
at 104 in 5 years, if the yield rate ISi to be 6% compounded semi-annually. (Given
(1.06)'^ =0.74725817)
Find the purchase price of ? 1,000 bond, redeemable at the end of 20 years at 105, and
paying annual dividends at 5%, if the yield rate is to be 8% effective. (Given
(1.08)'^^ =0.21454821)
Find the purchase price of a ? 25,000, 5% bond dividends payable semi-annually,
redeemable at par in 5 years, if the yield rate is to be 6% compounded semi-annually (Given
(1.06)"’‘^ =0.55839478).
An investor is considering purchasing a new issue of 5-year bonds of ? 100,000 par value
and an annual fixed coupon rate of 12% while coupon payments are made semi-annually.
24.19
FINANCIAL MATHEMATICS-!

The minimum yield that the investor would accept is 6.75%. Find the fair value of the bond.
-10
(Given (1.0675) = 0.52038068)
10. A bond has a face amount of 1 1,000 and maUires in 12 years. The nominal interest is 9%.

rw
What is the price of the bond that will yield an effective interest of 87o?
11. Find the purchase price of a ? 1,000 bond, redeemable at the end of 12 years at 157o
premium and paying annual interest at 8%, if the yield rate is to be 10/o per annum
effective.
12. A bond has a face value of 1,000 and maturity period of 15 years. The nominal interest rate
is 10%. What should be the price of the bond to yield an effective interest rate of 87o?

e
in 12 years. The nominal rate of interest on
13. A bond with a face value of 5,000 matures at par
bond is 97o p.a paid annually. What should be the price of the bond so as to yield an

r
luo
effective rate of return equal to 117o per annum?

F
U. A coupon bond has a ? 1,000 face value and provide.-. .i 10.57,. .semi-annii.d coupon for
14 years. The appropriate effective (or discount) rate is 9% per annum. What is the value of
the coupon bond?

oF
A zero coupon bond has a ? 1,000 face value with a 25 year life. The appropriate discount (or

rs
15.

effective) rate is 97o p.a. What is the value of the zero coupon bond? Find out what should

ok
be the value of this bond after 8 years?
ANSWERS

fo
1. ? 94,842.02 2. ? 8,658 ■ ? 1,136.78 ?5,681.40 ? 1,085.59
9. ^94,671 10. ? 1,075.36
o
8. ? 23,931.50
Y
6. ? 472.18 7. ? 716.13
Y
n. ?911.53 12. ? 1,171.19 i.. ?4,350.76 14. ?1,118.07 15. ? 115.97,? 231.05
rB

24.6 EFFECTIVE RATE OF INTEREST


An investment of ?100 at 107o compounded semi-annually is made by a person for a year. After
/ 1o n2
ue

10
= ? 110.25. This means that on an
year he/she receives the amoimt ? 100 1 +
d

one
200

investment of ^ 100 for a year he/she gets ? 10.25 as interest. In order words his rate of return is
no
ad

10.257) per annum whereas the stated rate of return, at the time of investment, is 107o per
annum. Thus, we find that the actual rate of return is more than the stated rate of interest. In
general, in transactions involving compound interest, the stated annual rate of interest is less
i

than the actual annual rate of return. The stated rate of interest, in transactions involving
Re
F

compounded interest, is called the nominal rate of interest and the actual rate of return is called
the effective rate of interest. When the conversion period is a year, the effective rate of interest and
nominal rate of interest are same.
rem.\r:-. The effective rate of interest is the actual rate compounded anniiallij.
Let fg denote the effective rate corresponding to the nominate rate r, compounded m times in a
year. Then, rate per conversation period is i=—. Let P be the principal. Since there are
m

conversion periods in a year.


In one year, at the rate i, the principal P amounts to P (1 + /) ●
In one year, at the effective rate rg, the principal amounts to P (1 + r,.).
m
Thus, P (1 -f Vg) = P (1 + 0
n/H r

P (1 + r,.) = P f 1 + - mj
m

}■ V"
I+rg= I+ -
m
24.20
APPLIED MATHEMATICS-XII

-1 or, r^. =(1 +/)"' -1


m)
( r V”
Hence, Effective rate of Interest = ] + — -1

rw
\ mj

This formula gives us the effective rate r,, equivalent to nominal rate r compounded w times in a
year.
m
The formula r^. =(!+/) -1 gives the effective rate equivalent to the rate i per conversion
period.

e
If the interest is compoimded continuously, then m —> oo and the effective rate r corresponding
to the nominal rate r compounded continuously is given by

r
luo
\m
- lim f-,1 r
+ — -1

F
III CO mJ
V"
r^, = lim -1

oF
III -> CO \ mJ

rs
lim '' Kill

/ =eJll 00 >» -1

ok
r, =/ -1.

fo
Hence, the effective rate i\, equivalent to the nominal rate r compounded continuously is given
byr^, =/ -1. o
Y
FORCE OF INTEREST The nomiml rate r compoimded continuously equivalent to a given effective rater,
Y
is called the force of interest. ‘
rB

Now, ^r. = -]
e'" = 1 + r^,
ue

r =log,(l +/-,)= log 10 (1 +/;.) ■ log, 10 =(2.3025) logjQ (1 + r,)


d

Thus, Force of interest = 2.3025 logio(l + r,).


7"/*^ effective rates are used to compare different rates. The rate having larger effective rate
no
ad

yields more interest.


ILLUSTRATIVE EXAMPLES
i

EXAMPLE 1 Find the effective rate that is equivalent to a nominal rate of 8% compounded:
Re
F

(i) semi-annually (ii) quarterly (Hi) continuously


8
SOLUTION (i) We have, r = = 0.08, m = 2
100

V"
re = 1+1 -1
nij

0 08
r, = 1 + -f-
4 J
-1 =(1.04)2-1 =1.0816-1 =0.0816
Hence, the effective rate is 8.16% which means that
the rate 8.16% compounded armually yields
the same interest as the nominal rate 8% compounded semi-annually,
(ii) We have, r = = 0.08, m = 4
100
yti
re = (i.L m
-1
24.21
FINANCIAL MATHEMATICS-1

r.,= 1 +
0.08 f -1 =(1.02)‘* -1 =1.08243216-1 =0.08243216 [See Table]
e
4 )

Hence, the effective rate of interest is 8.24% which means that the rate 8.24% compounded
annually yields the same interest as the nominal rate 8% compounded quarterly,
(iii) We have, r = = 0.08
100

= -1 => r^. = t -1 =1.0833-1 = 0.0833

Hence, the effective rate is 8.33%. This means that the rate 8.33% compounded annually yields

ow
the same interest as the nominal rate 8% compounded continuously.
EXAMPLii 2 A woney-lender charges 'interest' at the rate of 10 rupees per 100 rupees per half-year, payale
■nnm?
in advance. What is the effective rate of interest does he charge p
SOLUTION It is given that the money-lender charges interest at the rate of ).0 rupees per 100

e
rupees per half-year, payable in advance. This means that ?10 is the interest on ^ 90 for half year.

re
So, the interest rate per half-year ^ annual interest rate is - compounded

Frl
F
half-yearly. Thus, we have
2
r = — and m - 2.
ou
sor
9

Let r^. be the effective rate of interest. Tlien,

’’e ~
V"
-1 => r.(■ = 1 + -
1 ^2
l=f—f
kf
19
oo
in J 9J 9 J 81

19
Y

So, the effective rate is — x 1007o = 23.45%


B

81

Hence, the money-lender charges at 23.45% per annum.


re

EXAMPLE 3 A money-lender charges “interest" at the rate of 10 paise per rupee per month, payable in
oY
u

advance. What effective rate of interest does he charge per annum?


SOLUTION The money-lender charges interest at the rate of 10 paisa per rupee per month,
ad

payable in advance. This means that 10 paise is the interest on 90 paise for one month. So, the
d

, . . 10 1
mterest rate per month is i = — = -.
in
Re

12 4
Thus, the annual interest rate is r = — = - compounded monthly. Consequently, we have
F

r = — and m = 12
3

Let j;, be the effective rate of interest. Then,


r, = 1+1 -1
inj
12 12 12
=
1 +
4
-1= 1 + - l=f — -1
3x12) 9) 9 ;

12
no
r,+1= —
^ I 9
log {r^, + 1) = 12 (log 10 - log 9) [Taking logarithm of both sides]
=> log (r,. + 1) =12(1 -0.9542) =12 x 0.0458 = 0.5496
24.22
APPLIED MATHEMATICS-XII

1+r^. = antilog (0.5496) => 1+/;. = 3.545 => =2.545


Hence, the effective rate of interest is 25.45%.
EXAMrLL4
Which gives better yields :9.1% compounded semi-annually or 9% compounded monthly?
SOLUTION The effective rate corresponding to 9.1% compounded semi-annually is
0.91 x2 9.1
r, = 1 + -1 = 0.91 and m = 2
2 100

=i>
r, = (1.0455)^ -1 = 1.09307025 = 0.09307025
So, the effective rate is 9.3% per annum.
The effective rate corresponding to 9% compounded monthly is

ow
12
0.09 9
V= 1 + -1 r = — = 0.09 and ?m=12
12 100
12
r/=(1.0075) -1 = 1.09380609.1 = 0.09380609

So, the effective rate is 9.38'^ per annum.

e
Hence, the investment with second option gives better yields.

re
Fl
rXAMn.l 5 To what sum will ^6,000 accumulate in 8 years if invested at an effective rate of8%?

F
SOLUTION We know that an effective rate is the actual rate compounded annually. So, the sum S
ur
accumulated is the compound amount of the sum ?6,000 invested at 8% compounded annually.

r
We have, P = 6,000, / =
8
100
= 0.08 and n = 8 fo
ks
Yo
II
S = P (1 + 0
oo

S = 6000 (1 + 0.08)^ = 6000 (1.08)® = 6000 x 1.85093021 =11105.58


eB

Hence, required sum = ^ 11,105.58.


EXAMPLE6 Find the amount to which ^12,000willaccumulateat the effective rate of 3% for 10 years, 4%
ur

for 4 years and 5% for 2 years.


ad
Yo

SOLUTION The effective rate is the actual rate compounded annually. Therefore, the required
sum S is given by
10
d

S = 12000fl + 3 'i
l.J 1 +
5 f
Re
in

I loo; V looj V looj


S =12000 (1.03)^° (1.04)'* (1.05)^
F

5=12000(1.34391638) (1.16985856) (1.1025) = 20800.10


Hence, the amount is ^ 20,800.10.
EXAM!
7 Hozv many years will it take for a sum of money to treble at the effective rate of 4% ?
SOLUTION Let the sum of money be ^ P. Since the effective rate is the actual rate compounded
annually. Therefore, we have

Principal = P, Compound amount (S) = 3P and i = ~ = 0.04


100
II
S = P (1 + i)

3P = P (1 + 0.04)” 3 = (1.04)” log 3 = n log 1.04 => n =


log 3 _ 0.4771 = 28.06
log 1.04 ~ 0.0170
Hence, it will take 28.06 years.
FINANCIAL MATHEMATICS-1 24.23

EXAMPLE 8 Find the force of interest corresponding to the effective rate 8%.
SOLUTION The force of interest r corresponding to the effective rateis given by
r =(2.3025) log (1 + r,.)
8
Here, r^ = = 0.08
100

r = 2.3025 log (1 + 0.08) = 2.3025 log 1.08 = 2.3025 x 0.0334 = 0.0769


Hence, the force of interest is 7.69% which means that 7.69 % compounded continuously and 8%
effective are equivalent.
EXAMPLE 9 What annual rate compounded continuoiisli/ is equivalent to an effective rate ofW %?
SOLUTION We know that the annual rate r compounded continuously equivalent to the

w
effective rate is given by
r =(2.3025) log (1 + r,,)

Flo
10
Here, ^c = = 0.1
100

ee
r = (2.3025) log 1.1 = 2.3025 x 0.0414 = 0.0953

Fr
Hence, the annual rate is 9.53%.
EXAMPLE 10 A banker credits the fixed deposit account of a depositor on a continuous basis. As a result,
for
ur
the effective rate of interest earned bi/ a depositor is 9.43%. Find out the rale of interest that is allowed by
the banker. What is the effective rate of interest if it is compounded on quarterly basis?
9.43
s
SOLUTION Let the rate of interest allowed by the banker be r. It is given that r^ = = 0.0943
k
Yo

100
oo

r = 2.3025 log (1+r,,)


eB

r = 2.3025 log (1.0943) =2.3025x 0.0391 = 0.0900


Thus, the rate of interest allowed by the banker is 9% compounded continuously.
r

If the mterest is compounded quarterly, then


ou
ad

r=0.09,m = 4
V"
Y

-1
nij
Re
nd

n no
— 1 + ^ -1 =(1.0225)"^ -1 =1.0930-1 =0.0930
Fi

Thus, the effective rate of interest is 9.3%


EXERCISE 24.4

Find the effective rate that is equivalent to a nominal rate of 6% compounded:


(i) semi-annually (ii) quarterly (iii) continuously.
2. Find the effective rate that is equivalent each of the following nominal rates:
(i) 6% compounded monthly (ii) 12% compounded monthly (iii) 5% compounded
continuously (iv) 7% compounded continuously
3. Which yields more interest: 7.8% compounded semi-annually or 8% effective?
4. Find out the effective rates of interest when the bank credits the fixed deposit account of the
depositors at the rate of 9% per annum compounded alternatively by (i) half yearly
(ii) quarterly (iii) monthly.
24.24 APPLIED MATHEMATICS-XII

Which is better from the stand point of the investor, 6.2% compounded semi-aiinually or
6% compounded monthly?
6. To what amount will ^ 12,000 accumulate in 12 years if invested at an effective rate of 5%?
7. How many years will it take for money to double at the effective rate of 6%?
8. A money lender charges “interest" at the rate of ^ 5 per ? 100 per quarter, payable in
advance. What effective rate does he charge per annum?
9. A money lender charges interest at the rate of 5 paise per rupee per month, payable in
advance. What effective rate of interest does he charge per month?
10. To what sum will ^ 20,000 amount in 8 years if invested at 6% effective rate for the first two
years at 6% compounded semi-annually for next three years, and at 6% compounded

ow
continuously thereafter,
n. HDFC Bank offers an interest rate of 6% compounded annually whereas ICICI bank
compounds it continuously. What rate should ICICI bank offer so that the effective rates of
interest for the two banks are equal?

e
12. Mr. X has two investment options either at 10% per annum compounded quarterly or 9.8%

re
rFl
per annum compounded continuously. Which option is preferable and why?
13. A depositor is entitled to receive 12% per annum as interest on the fixed deposit made with

F
a nationalize bank. What is the effective rate of interest on fixed deposit, if the bank credits
the interest on fixed deposit account after (i) every 4 months (ii) every 2 months?

r
ou
14. A money lender borrows money at 5% per annum on simple interest and lends at 6%
fo
compounded semi-annually. He makes a profit of f 220.26 after three years. Find the initial
ks
money borrowed by him and the effective rate charged by him.
15. Mr. X took a loan of ^3,000. Find the effective rate of interest charged by the lender if while
oo

lending, the lender deducts in advance (i) ? 200 as interest for 6 months (ii) ? 100 as interest
Y

of 3 months.
B

16. The effective rate of interest earned by a depositor on his fixed deposit with a bank is found
to be 12.6825%. Find out the rate of interest that is allowed by the bank to credit the fixed
re

deposit account on a monthly basis.


ou
Y
ad

ANSWERS

1. (i)6.09% (ii) 6.13% (iii) 6.18%


d

2. (i)6.16% (ii) 12.68% (iii) 5.13% (iv)7.25%


in
Re

3. 8% effective 4. (i) 9.20% (ii) 9.31% (iii) 9.38%


F

5. 6.2% compounded semi-annually 6. ^21,560 7. 11.9 years 8. 22.8%


9. 8.5% ? 32,122.70 11. 5.82%

12. First option, effective rate in first option is more than that of second option
13. (i) 12.497o (ii) 12.62% 14. ? 5,000,6% 15. 14.87op.a., 14.527o p.a.
16. 127o

24.7 NOMINAL RATE OF RETURNS

There can be two basic kinds of returns on investment viz. Real rate of return and nominal rate
of returns. The actual rate of return realized on an investment after accounting for the effects of
inflation, taxes and expenses is known as the real rate of return. The real rate of return is
considered a more accurate measure of an investor's earnings after income taxes have been paid
and the rate of inflation has been adjusted for. These two factors impact the gains of an investor
and so must be accounted for. This is in contrast with the nominal rate of return on an
investment.
FINANCIAL MATHEMATICS-; 24.25

«OMINAL RATE OF RETURN The nominal rate of return is the total rate of return on an investment before
adjusting for am/ deductions and premiums such as investment fees, tax expenses, inflation etc.
Note that the real rate of return is lower than the nominal rate of return. The nominal rate of
return helps investors in comparing various investments with different tax treatments. It also
helps investors to gauge the performance of their portfolio by stripping out outside factors such
as taxes and inflation that can affect performance.
Tlie nominal rate of return on an investment can be computed by using the following formula:
Nominal rate of return percent
Current market value of investment-Original investment value xlOO
Original investment value
We may use the following algorithm to compute nominal rate of return on an investment.

w
ALGORITHM

STEPf Obtain original hwestment amount or principal amount invested and denote it bi/ OlV.
Find the current market value of the investment or the value of the investment at the end of

lo
STEP II

investment period and denote it by CMV.

e
Find the difference: CMV- OIV.

re
STEP III

STEP L\ Use the formula:


rF
F
CMV-OIV
Nominal rate of return percent = xlOO,
OIV

r
ILLUSTRATION 1 Ronit made an investment of ^ 225,000 in a no-fee fund for one year. At the end of the
fo
u
year the value of the investrnent increases to ? 250,000. Find the nominal rate of return percent on his
ks
investment.
Yo
SOLUTION It is given that
oo

Original investment value (OIV) = 1225,000


B

Current market value {CMV) = ?250,000


CMV-OIV
e

Nominal rate of return percent = X100%


OIV
ur

250,000-225,000 25,000
ad

X100% = X100% ==11.11%


Yo

225,000 225,000 ’

ILLUSTRATION 2 Mr X purchases 100 shares of a company that cost ^ 250 each. After one year the price
d

of each share rose to ? 300. Assuming that there no trading costs and no dividends,find the nominal rate of
Re
in

return on the investment.

SOLUTION We find that


F

Original invesitment value (OIV) = ? (250 x 100) = ? 25,000


Current market value (CMV) = ? (300 X 100) = ? 30,000
CMV-OIV
Nominal rate of return percent X100%
OIV

30,000-25,000 5,000
X100% = X100% = 20%
25,000 25,000

Hence, the nominal rate of return on investment is 20°/< O.

24.8 COMPOUND ANNUAL GROWTH RATE (CAGR) ,


Consider an investment of ? V^ which grows at the rate iper rupee per year. Let V„ be the value of
the investment at the end of the n years. Then, by using the formula for compound interest, we
obtain
24.26 APPLIED MATHEWIATICS-XII

^11 ~ O"/ where V„ = Ending value (£ V) and Vp = Beginning value (6 V)


(1+0"=^
Vo
\l/»
V.n
1 +/ =
V.0 J

1/H
1= -1
\
V,
' 0 y>

1/>I Un
Ending Value (EV
1 = 1 or, / -1.
Beginning Value BV

ow
The value of / given by the above formula is known as the compound annual growth rate
(CAGR) of the investment. It represents the consistent rate at which an investment would have
grown had the investment compoimded at the same rate each year. This is widely used as a
measure of growth of investments that can flactuate over a period. Compound annual growth

e
rate (CAGR) measures the average yearly growth of an investment over a certain time period.

re
rFl
In order to calculate CAGR of an investment, we may use the following algorithm:

F
ALGORITHM

STEP I
Find the value of investment at the beginning of the period i.e. Beginning value (BV)

r
ou
STEP n
fo
Find the value of the investment at the end of the investment period i.e. Ending value (EV)
ks
EV
STEP in Find
BV
oo

STEP IV
Find the investment period i.e. n.
Y

\t/n
eB

EV
STEP V
Compute by using scientific or financial calculator.
BV
ur

EV^ l/»
STEP VI
Use the formula: CAGR = -1
ad

BV J
Yo

Consider an investment of ? 10,000 whose annual values are as given below:


d

Jan 1,2015 Jan 1,2016 Jan 1,2017 Jan 1,2018


Re
in

? 10,000 ? 14,000 ? 14,700 ? 22,050


F

The year-to-year growth rate of the investment are as follows:


Jan 1,2015-Jan1,2016 Jan 1,2016-Jan 1,2017 Jan 1,2017-Jan 1,2018
40% 5% 50%

We can see that on annual basis, the year-to-year growth rates of the investment are unevenly
fluctuating. So, it is difficult to comment on the annual rate of growth of the investment over the
period Jan 1, 2015 - Jan 1, 2018. The CAGR dampens the effect of volatility and provides
smoothed growth rate per year. We find that
Beginning value (BV) = 10,000, Ending value (EV) = ^ 22,050 and, n ^ Number of years = 3.
1/3 1/3
EV 22,050
CAGR = -1 = -1 =1.3016-1 =0.3016
BV 10,000.
CAGR% =0.3016x100 = 30.16%
FINANCIAL MATHEMATICS-! 24.27

Hence, the compound annual growth rate of the above investment is 30.16%.
REMARK 1 The compound annun! grozotli rate is not a true growth rate, but it is a representational

figure. It gives the rate at zuhich an investment zuould have grown if it had the same rate even/ year.
REMARK 2 TheCAGRcaii be used to compare the performance of tzoo investments that are uncorrelated.
24.9 EMI CALCULATION

w
In this section, we will learn about the fundamentals of EMI loan system, which refers to
Equated Monthly Installments.
EMI An equated monthly installment (EMI) is a fixed pmyment made by a borrozuer to a lander on a

e
specified date of each month.
EMI consists of interest and principal both and the borrower has to repay both so that over a

e
or
specified number of years/months, the loan is paid in full.

r
EMI depends on the following factors:

F
(i) The amount of loan (ii) The loan tenure (iii) The interest rate

oF
ul
EMI can be calculated by two methods:
(i) Flat Rate Method (FRM) (ii) Reducing Balance Method (RBM)

rs
FLAT RATE METHOD In fiat rate method the principal amount remains same throughout the tenure and
the interest is charged on it at a constant rate throughout the loan tenure. Suppose an amount oft P is

ko
borroived at fiat rate of r per rupee per month for a period ofn months. Then,
Interest = Pin
Principal + Interest
of
o
EMI =
Y
n
B

P + Pin 1
EMI = = P /■ + -
Y

n n nj
er

ILLUSTRATION Mira takes a loan oft 300,000 at an interest of 10% compounded annually for a period
of 3 years. Find her EMI by using flate rate method.
u

SOLUTION We have, P = Principal = t 300,000


d

10 _ 1
o
ad

/ = Rate of interest per rupee per month = 1200 " 120 , n = Number of installments = 36.
in

( 1 1
.-. EMI=?P /+ - = ? 30,000 — | = ? (250+833.33)=? 1083.33
n 120 36
Re
F

Hence, Mira's EMI is of? 1083.33.


REDUCING BALANCE METHOD (RBM) In reducing balance method principal paid back gets
deducted from the outstanding load amount and the interest for the subsequent year is charged
on the remaining deducted balance and not on the entire loan amount unlike the flat rate

method.

Let the amount P is borrowed on which interest is payable at the rate of r per rupee per
installment period. This amount is to be paid back along with the interest in n equal installments
of ? E each. Each installment comprises some part of the principal P that is to be paid back, the
rest is the interest on the amount that was outstanding for that installment period. Let P„ denote
the principal amount left (outstanding amount) at the end of n^^ month.
Initially i.e. at the beginning of the tenure: We have
i = 0 and Pq=P = Principal amount
First month: We have.
Principal = P, Installment paid = E, Interest accrued = Pi
Outstanding amount = (P + P/) - £ = P (1 -ri) -E
Pi =P (!+/)-£
24.28 APPLIED MATHEMATICS-XII

Second month: We find that


Principal = Outstanding amount at the end of first month = Pj
Installment paid = E, Interest accrued = PiZ.
Outstanding amount = (Pj + Pj!) -£
= Pl(l+0-£
= {P(l+0-£}(i+0-£ [Using (i)]

w
= P(l+0^-£{(l+0 + l}
P2=P(1+0^-£{(1+0 + 1} ...(h)
Third month: We have.

e
Principal = Outstanding amount at the end of second month = P2

ro
Installment paid = £, Interest accrued = P2 i

re
Outstanding amount = (P2 + P2 0 ~ £
= P2(1+0-£

F
Fl
= P (1+0^-£{(1+0 + 1} (l + 0-£ [Using (ii)]

= P(l+0^-£|(l+0^ +(l+0+l| u
sr
...(hi)

ko
o
P3=P(l+0^-
£|(l+0^ + (1+0 +1} of
th
Continuing in this manner, we find that the outstanding at the end of n month is given by
o
Y

P„=P(1+0"-£|(1+0
n-1 n-2
+...+ (1+0^ +(1+0 + 1
erB

+ (1+0
uY

As the loan is fully paid in n months. Therefore,


Outstanding at the end of «th month is zero i.e. P„ = 0.

P(l+0”-£|(l+0
M-l n-2
+ (l+r) +...+ (1+z)^+(1+0 + 1
ad

=0
do

£ jl +(l+0 + (l+0^ + .
in

n-2 n-1
..+ (1+0 + (1+0 = P(l+0‘
Re

g[(l + 0"-l
F

= P(l+0‘
1 (1+0-1
^f(i+0"-l = P(l+0'

^ ^_Pi (i+0»_ Pi

(l+0"-i i-(l+0
-n

This gives the formula for calculating EMI, where


£ = Equated Monthly Installment
P = Principal or Loan amount
i = Interest rate per rupee per month (the annual interest rate percent divided by 1200)
n = Number of monthly installments
24.29
FINANCIAL MATHEMATICS-!

ILLUSTRATIVE EXAMPLES

EXAMI'I! i Ainan borrowed a home loan amount of ^ 5,000,000froiii a bank at an interest rate of 12%
per annum for 30 years. Find the monthli/ instaliment amount Aman has to pay to the bank. (Given
-360
(1.01) = 0.02781668).

SOLUTION We have,
12
P = Principal = ? 5,000,000, i = 1200 - 0.01 and n = 12 x 30 = 360.

Let E be the monthly installment Aman has to pay to the bank. Then,
Pi
E =

ow
-?i
1-0+0
5,000,000 X 0.01 50,000 50,000 50,000
£=? = ? 51,430.63
l-O + 0.01)”^^° 1 -(1.01)
-360 1 - 0.02781668 ~ 0.97218332

Hence, Aman's EMI is ? 51,430.63.

e
Fl
re
EXAMPLE 2 Avni takes a loart of ^ 500,000from a bank at an interest rate of 6% p.a. for 10 years. She
wants to pay back the loan in equated monthly installments. Find her EM! by using (i) Flat rate method

F
-120
(ii) Reduced balance method. (Given (1.005) = 05496327334)
ur
SOLUTION We have,
or
sf 1
, 6
P = Principal = 500,000, i = Interest rate - — = 0.005, » =12x10 =120
1200 200
k
Yo
oo

Let £ be the amount of EMI.


/ 1
(i) Flat rate method: Using£ = P / + - , we obtain
B

nj
re

1
£ = ? 500,0001— = ^ (2,500 + 4,166.67) =? 6,666.67
200 120J
u
ad

Pi
Yo

Reducing balance method: Using £ = -n


-, we obtain
i-(i+0
d

500,000 X 0.005 2,500 2,500


= ^5,551.02
Re

E=? =^ =?
in

-120 1-05496327334 0.4503672666


1 -(1 +0.005)
F

We find that EMI by using reducing balance method is less than EMI by using flat rate method.
Hence, reducing balance method is more cost effective to Avni.
EXAMPLE 3 Rohit buys a car for zvhich he makes down payment of ? 150,000 and the balance is to be
paid in 2 years by monthly installment of^ 25,448 each. If the financer charges interest at the rate of 20%
-24
^61
p.a, find the actual price of the car. (Given = 0.6725335725)
60

SOLUTION Let P be the amount finaced by the financer. Then, the actual price of the car is
? (P + 150,000). In order to pay back the amoimt financed by the financer Rohit is paying
monthly installments of ? 25,448 for 2 years. The financer charges interest at the rate of 20% p.a..
Thus, we have
Principal = P, n = Number of installments = 2 x 12 = 24
20 1
i =rate of interest = = — and, £ = Monthly installment = ^ 25,448.
1200 60
24.30 APPLIED MATHEMATICS-Xll

Pi
E =
-)i
l-(l+0

£0-(!+/)
=> p =
I

-24
25,448 1 'I
=> P=? 1- 1+ —
1/60 60

n-24

=> P =^1,526,880 —
60

=> P = ? 1,526,880 (1 -0.6725335725) =? l,526,8S0 x 0.3274664275 = ? 500,001.93 ^ 500,002


So, the financer paid ^ 500,002 and hence the actual cost of the car is

w
^ (150,000 + 500,002) = ? 650,002.
EXAMPl.h4 Mahesh purchased a house from a compaj^y for ^ 700,000 and made a down payment

F lo
of ? 150,000. He repays the balance in 25 years by monthly instalments at 9%. Compound
monthly: (i) What are monthly payments? (ii) What is the total interest payment? (Given
(1.0075) = 0.1062878338)

ee
Fr
SOLUTION We have,

P=^ (700,000-150,000)= ? 550,000, J! = 25x 12 = 300 and / = — for = 0.0075.


1200
ur
(i) Let E be the monthly instalment. Then,
Pi
s
E =
ook

-n
i-(i+0
Yo

550,000 X 0.0075 4,125 4,125


eB

E =^ = ? = ? 4,615.58
1 -(1.0075)“^^’° 1 -0.1062878338 0.8937121662

(ii) We have, £ = ^ 4,615.58, n = 300 and P - ^ 550,000


r
ad
ou

Total Interest

= jiE - P = ^ (300 X 4,615.58 -550,000) = ? (1,384,674 - 550,000) = ^ 834,674


Y

EXAMPLE 5
A person buys a house for which he agrees to pay ? 5,000 at the end of each month for 8
Re

years. If money is zvorth 12% converted monthly, zvhat is the cash price of the house? (Given
nd

(1.01)“^^ =0.3847229701)
Fi

12
SOLUTION It is given that: EMI =? 5,000, a = 12x 8 =96 and / = = 0.01.
1200

Let P be the cash price of the house. Then,


Pi
E =
-/f
l-(l+0

P = -ll-(l + 0
i

5,000 -96
P=^ 1 -(1.01)
0.01

P = ^500,000(1-0.3847229701)=? (500,000 X 0.6152770299) = ? 307,638.51


Hence, the cash price of the house is ? 307,638.51.
FINANCIAL MATHEMATICS-) 24.31

EXERCISE 24.5

1. Ravish borrowed a home loan of ^ 10,000,000 from a bank at an interest rate of 11% per
annum. If the loan is to be paid back in 20 years, find the amount of EMI Ravish has to pay
240
n2ii
to the bank (Given = 8.935015349).
U200

Raman has taken a personal loan ? 200,000 for 2 years at an interest rate of 20% p.a. which is
to be paid pack in equal monthly instalments. How much monthly installment Raman will

ow
pay?
Rohan bought a car in 2018 worth ? 5.95 lakh. He made a down payment of X 150,000 and
took an auto loan for the rest of the ar .ount at 12% interest p.a. for four years. The loan is to
be paid in equated monthly installments. Find the amount of each EMI.

e
4. Mrs. Sood took a housing loan of ? 800,000 to be paid in 10 years by equal montlily

re
installments. The interest charged is 10.5% compounded monthly. Find her monthly
-120
instalment. (Given (1.00875) = 0.3515416)

F
Frl
Mr. X borrowed ? 1,000,000 from a bank to purchase a house and decided to repay by
ou
monthly equal instalments in 10 years. The bank charges interest at 9% compounded

osr
-120
monthly. Find his EMI and total interest paid. > Given (1.0075) = 0.407937305).

6. A person sells his old car for ^ 100,000 to buy a new car for ^ 250,000. He pays ? .r in cash and
kf
balance in 18 instalments of ? 7,000 at the end of each month. If interest is reckoned at 6%
oo
18
per annum compounded monthly, find the value of .v. (Given (1.005) = 0.9141361599).
Y

7. A person amortizes a loan of ? 150,000 for a new home by obtaining a 10 years mortgage at
B

the rate of 12% compounded monthly. Find : (i) the monthly payment (ii) the total interest
paid.
re
uY

ANSWERS
ad
do

1. ^ 103,218.84 2. ^10,179 3. ^ 11,718 -4. ^10,795

5. ^ 12,668, ^ 520,160 6, ^ 29,790.62 7. (i) ? 2,157.65 (ii) ? 108,918


in
Re

24.10 DEPRICIATION
F

The dictionary meaning of the word "Depriciation" is devaluation or reduction in the value of an
asset overtime due to wear and tear. Every business enterprise has some fixed assets such as
buildings, machinery and equipment, furniture etc. The values of these assets decrease over
time as the asset gets older or as wear and tear occurs or the decline of one currency in relation to
other. Such a reduction or decrease in the value of an asset is called depreciation and the asset is
called a depricibale asset. Formally, we may define depriciation as follows.
DEPRICIATION The decrease in monelary value of an asset over time due to use, wear and tear or
obsolescence is called its depriciation.
USEFUL LIFE The useful life of an asset is an accounting estimate of the number of years it is likely to
remain in service for the purpose of cost effective revenue generation.
The Internal Revenue Service (IRS) employs life estimates to determine the amount of time
during which an asset can be depriciated.
SCRAP VALUE The value of a depriciable asset at the end of its useful life is called its scrap value.
24.32 APPLIED MATHEMATICS-XII

TOTAL DEPRICIATION The difference betzveen the original value of an asset and the scrap value is the
total depriciation or total depriciated value or zvearing value.
BOOK VALUE The book value of an asset on a given date is the original value of the asset minus the
accumulated deprication at that time.
●^4.10.1 METHODS OF COMPUTING ANNUAL DEPRECIATION

The computation of annual depriciation of an asset depends on the following factors:


(i) Original value of the asset (ii) Useful life of the asset (iii) Scrap value of the asset.
The original value of an asset is always known. However, the useful life and scrap value are
determined by valuation experts. There are many methods to calculate the annual depriciation
amount.

w
Following are some commonly used methods:
(i) Straight-line method or Linear method of depriciation.

F lo
(ii) Reducing Balance Method
(iii) Sum-of-the-years-digits method

ee
(iv) Double declining Balance Method.
The following are commonly used notations/symbols in these methods:

Fr
C The original cost of the asset
S The estimated scrap value of the asset
for
ur
W The total depriciation
D The annual depriciation
ks
n
Useful life in years of an asset
Yo
oo

We will study only the straight-line method or Linear method of depriciation as study of the
remaining methods is beyond the scope of this book.
eB

24.10.2 LINEAR DEPRICIATION METHOD


r

This is the simplest and most commonly used method of computing depreciation. It is also
ou
ad

called the straight line method of depreciation, In this method the depreciation amount is the
same for every year over the useful life of the asset, that is, the depreciation amount is charged
Y

until the asset gets reduced to zero value or its salvage value at the end of its useful life. If the
Re

useful life of an asset is plotted on horizontal axis and the book value on vertical axis, we find
nd

that book values lie on a straight line. Also, if the accumulated depreciation is plotted against the
Fi

accounting period, we obtain a straight line. That is why this method is called straight line
depreciation method or linear depreciation method.
In linear depreciation method the annual depreciation of an asset is found by dividing the total
depredation by the number of years in its estimated useful life. Thus, we the following formula;
Annual depreciation of an asset
_ The original value of the asset-Salvage value of the asset
Useful life of the asset
C-S W
or. D = or, D = —
n n

Uie straight line depreciation rate can be calculated as follows:


Annual depreciation
Straight line depreciation rate =
Cost of the asset - Salvage values of the asset
We may use the following algorithm to calculate linear depression.
FINANCIAL MATHEMATICS-1 24.33

ALGORITh;.

■: Detennhw the original value (C) of the asset and its salvage value fSJ.
Determine the useful life (n) of the asset.
Compute annual depreciation (C - S).
C-S
Compute the annual depreciation (D) bi/ using the formula D = n

ILLUSTRATIVE EXAMPLES

MIM i A machine costing ? 50,000 has a useful life of 4 years. The estimated scrap value is
^ 10,000. Using the straight line method, find the annual depreciation and construct a schedule for

ow
depriciation. Also, find the depreciation rate percent.
SOLUTION We are given that;
C = Original value = ? 50,000, S = Salvage value = ^ 10,000, n = Useful life = 4 years.
The annual depreciation D is given by

e
C-S 50,000-10,000

re
D =
n
=> D=?
4

rFl= ? 10,000.

F
At the beginning of the first year, the book value of the machine is ^ 50,000. At the end of the first
year, the accumulated depreciation is ? 10,000; hence the depreciation charge for the first year is

r
^ 10,000. The book value at the end of the first year or in the beginning of the second year is
ou
^ (50,000 - 10,000) = t 40,000.
fo
ks
At the end of the second year, we have:
oo

Accumulated depreciation = ? 20,000.


Y
eB

Depreciation charge = ? (20,000 - 10,000} = 10,000.


Book value at the end of the second year = ? (40,000 - 10,000) = ^ 30,000.
r

At the end of third year, we have :


ou
ad

Accumulated depreciation = ? 30,000.


Y

Depreciation charge = ? (30,000 - 20,000) = ? 10,000.


d

Book value at the end of the third year = ^ (30,000 - 10,000) - ^ 20,000.
Re
in

At the end offourth year, iue have:


Accumulated depreciation = ? 40,000.
F

Depreciation charge = ? (40,000 - 30,000) = ^ 10,000.


Book value at the end of the third year = ^ (20,000 - 10,000) = ? 10,000.
Clearly, this is the salvage value of the machine.
These values can be presented in the following tabular form which is known as the depreciation
schedule of the machine:
Depreciation Schedule
Year Book value (Beginning of year) Depreciation Book value (End of year)
1 X 50,000 ? 10,000 ? 40,000

2 ? 40,000 ? 10,000 ? 30,000

3 ? 30,000 ? 10,000 ? 20,000

4 ^ 20,000 ? 10,000 ^ 10,000


24.34 APPLIED MATHEMATICS-XII

We find that

Annual depreciation amount = ^ 10,000.


Cost of the machine - Salvage value of machine = ? (50,000 -10,000) = ? 40,000
10,000
Depreciation rate per cent = xlOO =25%
40,000

A mainframe computer whose cost is ^ 500,000 will depreciate to a scrap value ofX 50,000
in 5 years. Using linear method of depreciation find the book value of the computer at the end of third year.
SOLUTION We have,
C = Original cost of computer = ? 500,000, S = Scrap value of computer = ? 50,000

ow
n = Useful life of computer = 5 years.
Let D be the annual depreciation. Then,
c-s 500,000-50,000
D = => D = ? = ? 90,000
n 5

e
The book value at the end of third year = ? (500,000 - 3 x 90,000)

re
= ? 500,000 - ? 270,000 = ? 230,000

Frl
F
EXERCISE 24.4

t.
A piece of machinery costing ? 100,000 is expected to have a useful life of 5 years and scrap
ou
or
value? 20,000. Find the annual deprecation charge using the linear method of depreciation.
Also, find the depreciation rate percent. kfs
An asset costing? 150,000 is expected to have a useful life of 5 years and scrap value of
? 30,000. Find the annual depreciation charge and the depreciation rate by using the linear
oo

depreciation method.
A machine costing ? 30,000 is expected to have a useful life of 13 years and a final scrap
Y
B

value of? 4,000. Find the annual depreciaton charge using the linear depreciation method.
ANSWERS
re

1 ? 16,000,20 % 2- ? 24,000,20% 3. ?2000


oYu
ad
d
in
Re
F
CHAPTER-

FINANCIAL MATHEMATICS-II

ow
25.1 INTRODUCTION

In this chapter, we shall discuss about the shares, debentures and stocks. Most of the private
limited or public limited companies raise their capital either by issuing or by borrowing money

e
through debentures. Public invest money in shares for profit which is also called dividend.

re
Money invested in debentures earns interest. Sometimes, the government requires money to
meet the expenses of a certain plan or big work of public utility. In such cases, government raises
loan from the public at the fixed rate of interest and issues Bonds as acknowledgment of the

F
Frl
debt. If a person purchases a bond of ? 100 at which 10% interest has been fixed by the
government, then the holder of such a bond is said to have a ? 100 stock at 10%'.
ou
osr
25.2 JOINT STOCK COMPANY

To start an industry or a big business, a large amount of money is required. If an individual does
kf
not have sufficient money, then some parsons associate together and prepare a detailed plan of
the project with the help of some experts in that particular field. They also frame some rules and
oo
regulations regarding its functioning. These are then registered under the Indian Companies
Y

Act. The company so formed is called a joint stock company. The total amount of money
B

required for the project is called the capital. In the following sections we shall discuss how a joint
stock company raises its capital through shares and debentures.
re
uY

25.3 SHARES

In this section, we shall discuss how a joint stock company raises its capital through shares. A
ad
do

joint stock company divides the required capital into equal small units. Each imit is called share.
Generally, the value of a share is ? 10 or ? 100. Let us assume that the capital required is ? 8 crore
and the value of a share is taken to be ^ 10, then the whole capital consists of 80 lakh shares of
in

? 10 each. If, however, the value of a share is taken to be ? 100, then the whole capital is divided
Re

into 8 lakh shares of ? 100 each. The company then issues a prospectus, explaining the plan of the
F

project and invites the general public to invest money in the proposed project by purchasing the
shares of the company. Those persons who accept the terms and conditions of the company and
consider the investment profitable, apply for these shares. The company reserves the right
whether to allot shares to a person or not. This situation generally arises when the number of
applicants is very large in comparison to number of shares. When one is allotted shares by the
company and has paid the money prescribed by the company for the shares, the company issues
certificate (s) indicating the number of shares allotted to the person and the value of each share.
These certificates are called share certificates. This is to show that he owns these shares. The
person who subscribes in shares is called a share holder.
25.4 SOME IMPORTANT TERMS
(i) DIVIDEND After collecting sufficient money through shares from the general public a
company starts working. If it makes a profit during a financial year, a part of the profit is kept in
reserve fund, depreciation account etc. to pay government taxes etc. and the rest is distributed
amongst the share-holders in proportion to the number of shares held by them, This distributed
profit is known as dividend. Dividend may be given as per share or as a percent. When we say
25.2 APPLIED MATHEMATICS-XII

that a dividend of ? 7 per share, this means that a person having 100 shares will get ? 700 as
dividend. A '30% dividend' means that on a share of ? 100, the share-holder gets ? 30 and on a
share of ? 10 the share-holder gets ? 3.
It follows from the above discussion that dividend is the part of the profit of a company which is
distributed amongst the share holders,
(ii) FACE VALUE The value for which a share is issued by a company is called the face value of the share.
The face value of a share is printed on the share certificate and is also known as the nominal
value or par value of the share,
(iii) MARKET VALUE Like any other commodities shares can be sold and purchased in a market, called
stock exchange. When we sell a share in the market, it may fetch a value more than the face value or less
than the face value depending upon market conditions and so many other factors affecting the market.
The value for which a share is available in the market is called the market value of the share.

Note that the market value of a share may vary from day to day whereas its face value always

w
remains same.

A share is said to be above par or at a premium, if its market value is more than its face value.
If the market value of a share is less than its face
a discount.

F lo
value, then the share is said to be below par or at

ee
If the market value of a share is same as the face value, then the share is said to be at par.

Fr
ILLLSJ KA I l:
If f 100 share is quoted at 75 premium, then its market value is
? (100 + 75) = ? 175. for
iLLUSTltA’l'lON .i If ^ 100 share is quoted at 10 discount, then the market value of the share is
ur
? (100-10) = ? 90.
s

● \Kk I
Even/ company declares dividend on the face value of its shares.
ook
Yo

The statement “ 15%, ? 100 shares at ? 120", means that


eB

(i) the face value of one share is ? 200;


(ii) the market value of one share is ? 120;
(iii) the dividend earned on one share is 15% of its face value;
our
ad

(iv) an investment of ^ 120givesan income of^ 15.


(iv) BROKERAGE

As discussed earlier shares are sold and purchased in a market, called stock exchange. The sale
dY
Re

and purchase of shares is, generally, done through agents called share-brokers or simply
brokers. Brokers charge commission from buyers as well as sellers. The broker's commission is
Fin

called brokerage. While purchasing a share from the market, a buyer has to pay market value to
the share-holder and brokerage to the broker. The seller of the shares receives the market value
of the share but has to pay brokerage to the broker. Brokerage is calculated on the market value
of the share.

We shall now illustrate the points discussed above with the help of following examples.
25.5 TYPES OF SHARES

There are two types of shares : (i) Preferred shares (ii) Common shares or ordinary shares.
EFERRED SHARES
In preferred slrnres, the share holders receive a specified percent of the profits as
dividend.

Note that preferred share holders are paid dividend only if the company has profits after paying
working expenses and taxes.
CC . ' on ORUinary shares Common or ordinary shares are those shares which are paid dividend
only when profits are left after preferred share holders have been paid dividend at specified rate.
FINANCIAL MATHEMATICS-11 25.3

The rate of dividend on common shares depends upon the amount of available profit.
Sometimes common share holders do not get any dividend because no profit is left for them
after paying for working expenses, taxes and preferred share holders.

ILLUSTRATIVE EXAMPLES

[iXAMPLE I A company issued shares at 10% premium. Satish applied for 1000 shares but was allotted
500 shares of this company. Find his investment, if the face value of a share is 1100.
SOLUTION Face value of a share = ? 100.
Premium percentage = 10%
Premium on one share =?10.

ow
Market value of one share = t (100 + 10) = ? 110.
Price of 500 shares = ? (500 X 110) =? 55000.
EXAMPLE 2 Find the investment in buying 525 shares oft 100 each at 12 premium.
SOLUTION Face value of a share = 1100.

e
Premium on a share = 112.

re
Market value of a share = till.

rFl
F
Investment in buying 525 shares = t (525 x 112) = X 58800.
EXA.MPL1-; 3 Find the investment in buying 450 shares oft 100 each at 5 discount.

r
SOLUTION Face value of a share = 1100.
ou
fo
Market value of a share = Face value - discount = t (100 -5) = t 95.
ks
So, investment in buying 450 shares = t (450 x 95) = ? 42750.
EXAMPLE 4 Find what a buyer zuould have to pay for 500 shares oft 10 each quoted at t 74. What
oo

should be th.e gain to the share-holder, if he had purchased the share at par?
Y
eB

SOLUTION Face value of a share = 110.


Market value of a share = t 74.

Amount paid by the buyer for 500 shares = ^ (74 x 500) = t 37000.
ur

Gain from one share = t (74 -10) = t 64


ad

Gain from 500 shares = t (64 x 500) = t 32000.


Yo

EXAMPLES Anil invested 10846 in buying the shares of a corzipanyat 117 each. If the face value of each
share be 110 and company paid 15% dividend at the end of the year, ifnd the dividend earned by him.
d
Re
in

SOLUTION Market value of one share = t 17


Total money invested = 110846
F

10846
Number of shares bought = = 638
17

Face value of 638 shares = t (638 x 10) = t 6380


Rate of dividend paid by the company = 15%
X 6380 = t 957.
Dividend received by Anil = 15% of t 6380 = 100

EXAMPLE 6 A company declared an annual dividend of 10%. Find the annual dividend of Rohan
owning 1500 shares of the company of par value 110 each.
SOLUTION We have.
Total par value of 15 shares = t (1500 x 10) = ? 15000
Rate of dividend = 10% per annum
10
Toal dividend of Rohan = t 15000 x = 11500
100
25.4 APPLIED MATHEMATICS-XII

EXAMl’l FT
A company issued 50000 shares of par zyaliie ? 10 each. If the total dividend declared by the
company is ? 62500, find the rate of dividend pwid by the company.
SOLUTION We have.
Number of shares =50000
Par value of a share = ? 10

Total par value of 50000 shares = t 500000


Total dividend = ? 62500
Let the rate of dividend be r% per annum. Then,
Total dividend = ? I 500000 x — = ? 5000 r
100

But, total dividend = ? 62500

w
625
5000 r = 62500 => r = = 125
50

Hence, the rate of divident paid by the company is 12.5% per annum.

Flo
EXAMPU; Shalu had 50 preferred shares and 400 common shares of par value t 100 each. If the
dividend declared on preferred shares is 10% per annum and a semi-annual dividend of 7.5% is on

e
re
common shares, find the annual dividend received In/ Shalu.
SOLUTION Total par value of 50 preferred shares = ? (50 x 100) = ? 5000

F
Rate of dividend on preferred shares = 10% per annum.
ur
r
5000
Dividend on preferred shares = ? 10 x = 500
fo
100

Total par value of 400 common shares = ? (400 x 100) = ? 40000


ks
Yo
Rate of dividend on common shares = 7.5% semi-annual = 15% annual
oo

15
Dividend on common shares = ? X 40000 = ? 6000
100
B

Hence, total dividend received by Shalu = ? (500 + 6000) = ? 6500


re

. \AMI’l I
■ ■ Ravi bought 500 shares of a company quoted at ? 280. Find the amount spent by him on
this purchase, if the brokerage be 1%.
u
ad
Yo

SOLUTION Market value of one share = ? 280.


Market value of 500 shares = ? (500 x 280) = ? 140000
Since the brokerage is paid on the market value. Therefore,
d
Re

1
in

Brokerage paid by Ravi = 1% of ? 14000 = ? X 140000 =?1400.


100
F

Total amount spent by Ravi = ? (140000 + 1400) = ? 141400


How many shares of market value ? 112.50 each can be purchased for ? 113625, if the
brokerage paid is 1%?
SOLUTION Market value of a share = ? 112.50.
Brokerage percentage = 1%.
Brokerage on one share = 1% of ? 112.50 = ? — X 11250 = ? 1.125.
100

Net cost on the purchase of one share = ? (112.50 + 1.125) = ? 113.625


113625
Number of shares bought = = 1000.
113.625

Ankur invested ? 4444 in the shares offace value ? 100 each of a company. At the end of
the year, the company declared dividend at 15% which gave him an income of^ 600. At what price ivas the
share quoted if the brokerage zvas 1%?
25.5
FINANCIAL MATHEMATICS-i:

SOLUTION Suppose the share was quoted for X x i.e. the market value of the share was ? x. Let
the total number of shares bought be y.
Face value of a share = X 100
Dividend on one share = 15% of X 100 = ^ 15
Dividend on y shares = X\5y
It is given that the income i.e. dividend is X 600.
15i/ = 600 => i/=40.
Thus, the number of shares bought = 40.
Brokerage percentage = 1%
Market value of 40 shares = ? 40 x.

ow
f40x^ 4x
Brokerage = 1% of X 40.v = X 100 j = ?—.
10

Total investment = X 40x + —


4x
^ 4Q4y
10 10

e
But the total investment is given to be X 4444.

re
404 4444x10
- =110.

rFl
F
X = 4444 => .V =
10 404

Hence, the share was quoted at X 110.

r
Amit buys ^ 20 shares of a company which pays 18% dividend to him. The market price is
ou
I XAMl’in ii;

such that he gets 24% on his money. At what price did he buy it?
fo
ks
SOLUTION Suppose Amit bought a share for X x.
oo

Face value of a share = X 20.


18
Y

Dividend on one share = 18% of ? 20 = X X 20 =X 3.60


eB

100

It is given that Amit gets 24% profit on the market price.


ur

24.x-
= 3.60 => 24 .V = 360 => .r=15.
ad

100
Yo

Hence, Amit bought the share at X 15 per share.


Renii buys 200 shares, each of par value X 10 of a company which pays annual dividend of
d

rxAMPi r i:-
Re

15% at such a price that he gets 12% on his investment. Find the market value of a share.
in

Par value of 200 shares = X (200 x 10) = ? 2000.


F

SOLUTION
15
X 2000 = X 300
Dividend received by Renu = 15% of X 2000 = X 100

Let the market value of a share be X x.


Then, market value of 200 shares = X 200.T.
{ 12
X 200.x- = ? 24 .r.
Amount earned by Renu on her investment = (12% of X 200 .v) = X 100

24.x-= 300 => x= 125

Hence, the market value of a share = X 12.50.


1'^ - iPi I ! The capital of a company is made up of50,000 preferred shares with dividend of 20/o and
20,000 common shares, the par value of each type of share being XIO. The company had a total profit of
XISOOOO out of ivhich X 30000 were kept in reserve fund, and the remaining distributed to share holders.
Find the dividend percent paid to the common shareholders.
25.6
APPLIED MATHEMATICS-XII

SOLUTION We have.
Total profit of the company = ? 180000
Amount kept in reserve fund = ? 30000
Net amount paid as dividend to shareholders = ? (18000 - 30000) = ? 150000
Number of preferred shares = 50000
Par value of 50000 preferred shares = ? (50000 x 10) = ? 500000
Rate of dividend on preferred shares = 20% per annum
20
Dividend paid by the company on preferred shares = ? X 500,000 =? 100,000
100

Dividend to be paid to common shareholders = ? (150000 -100000) = ? 50,000


Number of common shares = 20000
Par value of 20,000 common shares = ? (20000 x 10) = ? 20000

w
50000
Hence, dividend percentage paid on common shares = xlOO %=25%
200000

F lo
I XAMPLt 15
Moliit invested ? 3333 in the shares offace value f 100 each of a compam/. At the end of
the year, the company declared dividend at 15%. He got an incoine of^ 450. At zvhat price was the share

ee
quoted, the brokerage being 1%?

Fr
SOLUTION Suppose the share was quoted x and the total number of shares bought be y.
Dividend on one share = 15% of f 100 = ? 15.
for
ur
Dividend on y shares = ? 15 y.
It is given that Mohit got an income of ? 450.
15y = 450 => y = 30
ks
Yo

Thus, Mohit bought 30 shares.


oo

Brokerage on one share = 1% of ? .v = ?


eB

100
f X
Brokerage on 30 shares = ? 30 x —
100 10
r
ou
ad

3;r ^
Total investment = ? xy + = ? 30.V +
3.Y
^ f 3003 X
[● ● y = 30]
100 100 100
Y

3003
-t = 3333
100 [●.● Total investment is ? 3333 (given)]
nd
Re

3003a: = 3333 x 100 => .t =110.


Fi

Hence, the market value of share was ? 110.


I-XAMPLE ir. Fi)id the price of 15% share luith face value ? 100, that gives 12% income.
SOLUTION Let the market value of each share be ? .v.
The given statement means that dividend on a share of ? 100 is 115. Therefore, income = ? 15.
This is 12 % of the investmentof ? .v.
12.Y 1500
= 15 => .v = = 125.
100 12

Hence, the market value of each share is ? 125.


EXAMPLE 17
100-rupee shares of a company are sold at 20% discoimf. If the return on investment is
15%, find the rote of dividend declared.
SOLUTION Let the rate of dividend declared be x %.
We have.
Face value of a share = ? 100
25.7
FINANCIAL MATHEMATICS-!'

Market value of a share = ? 80 [●.● shares are sold at 20% discount]


Dividend on a share = ? a:
Thus, on the investment of ? 80, dividend received is ? .v.
r
- X 100.
Return percentage 80

But the return percentage is 15% (given).


— xl00=15=>.r=12.
80

Heiv"’, the rate of dividend declared = 12%


Since the shares are sold at 20% discount and the return on investment is 15%.
Therefore,
Investment of ? 100 yields income of ? 15
15
X 80 = ? 12.
Investment of ? 80 will yield income of ? 100

w
Hence, the rate of dividend declared = 12%.

F lo
rx AMPLE T - 100-rupee share ofacompam/ is quoted at 1899 in the market. How much doti,.vianish pay
to purchase 500 such shares, if the brokerage paid by him is 1% ? Wiat is his gain percent on ihis investment, if
the compajiy pays a dividend of 25% ?
of a share. It is given that the market

e
SOLUTION The brokerage is paid on the market value

Fre
value of a share is ? 899.

Brokerage on one share = 1% of ? 899 =


for
100 J
899
r
x500 =?4495
Total brokerage on 500 shares = t 100
You
oks

Dividend on one share = 25 % of ? 100 = ? 25


eBo

Dividend on 500 shares == ? (500 x 25) = ? 12500.


Total investment = ? (500 x 899 + 4495) = ? 453995
12500
X 100 =2.75%.
Gain percentage =
ad
our

453995

■ ' 1 ■ A company has a total capital of ? 5,00,000 divided into 1000 preferred shares, of 6%
dividend, with par value of ^ 100 each and 4000 ordinary shares of par value of ? 100 each. The company
declares on annual dividend of t 40,000. Find the dividend received by a person having 100 preferred
dY
Re

shares and 200 ordinary shares.


Fin

SOLUTION The par value of 1000 preferred shares = ? (1000 x 100) = ? 1,00,000
Since dividend on preferred shares is paid at the rate of 6 /o
Total dividend paid to preferred share holders = ? 100 X100000 = ? 6000.

Since there are 1000 preferred shares.


6000
= ? 6
Dividend for one preferred share = ^ 1000

So, dividend on 100 preferred shares = ? (6 x 100) = ? 600


Remaining dividend = Total dividend - Dividend paid to preferred shares
= r (40,000 - 6000) = ? 34,000.
34000
= ? 850.
Dividend for one ordinary share = ? 4000 ;

Dividend on 200 ordinary shares = ? (850 x 200) = ^ 1700.


25.8
APPLIED MATHEMATICS-XII

Total dividend received by the person = Dividend received on preferred shares


+ Dividend received on ordinary shares
= ? (600 +1700) = ? 2300.
MI’LE2u
The capital stock of a company is ? 4400000, which is divided into 4000 preferred shares,
which yield an annual dividend of 5%, and 40000 ordinary shares. Both the preferred and ordinary shares
have a par value o/? 100 each. If the net profit shozun by the company is ? 450000 out of which ? 300000 is
distributed as dividend, ivhat dividend will Asha get if she holds 150 preferred shares and 300 ordinary
shares.

SOLUTION Since dividend is paid at the rate of 5% to preferred share holders.


Total dividend paid to preferred share holders = ? ( 4000 x — x 100 = ? 20000.
100

Remaining dividend = Total dividend - Dividend paid to preferred share holders


= ? (300,000 - 20,000) = ? 280,000

w
Dividend for 40,000 ordinary shares = ? 280,000.
280,000
Dividend for one ordinary share = ? = ?7.

F lo
40,000

Dividend received by Asha on 300 ordinary shares = ? (300 x 7) = ? 2100.


Total dividend paid to 4000 preferred share holders = ? 20,000.

ee
20,000 ^

Fr
Dividend for one preferred share = ? = ?5
4000
for
Dividend received by Asha on 150 preferred shares = ? (5 x 150) = ? 750.
Hence, Total dividend received by Asha = ? (2100 + 750) = ? 2850.
r
L\.-\MI’LE 21 A company with 70000 shares, par value ? 300 each, shows a profit of ? 380500. The
You
s

directors announce an annual dividend of 5%


ook

(i) Find the total annual dividend paid by the company,


eB

(ii) What dividend will Ashok receive annually if he ozvns 100 shares.
SOLUTION (i) Face value of a share =? 100
Face value of 70,000 shares = ? (100 x 70000) = ? 700000
our
ad

Dividend paid by the company =5% of the face value of 70000 shares
= ? — X 7000000 = ? 350000
dY

100
Re

(ii) Since Ashok has 100 shares of face value ? 3 100 each.
Fin

Face value of 100 shares = ? (100 x 100) = ? 10000


f 5
Dividend received by Ashok = ? X 10000 =?500.
100
EXAMPLE 22
Mohan sells 5000 common shares (each of par value ? 30) of a company A which pays a
dividend of 20% at ? 30 per share. He invests the sale proceeds in ordinary shares (each of par value ? 25) of
company B that pays a dividend of 15%. If the market value of a share of company B is ? 40, find
(i) The number of shares of company B purchased by Mohan,
(ii) The change in dividend income of Mohan.
SOLUTION Number of shares of company A possessed by Mohan = 5,000
Total par value of the shares = ? (5,000)
Since company A pays dividend at the rate of 20% per annum. Then,
f 20
To dividend received by Mohan = ? X 50000 =? 10,000
100

Selling price of a share of company A = ? 30


25.9
FINANCIAL MATHEMATICS-li

Selling price of 5000 shares of company A = f (5000 x 30) = ? 150,000


(i) Market value of share of company B = ? 40
n50,ooo
= 3750
Number of shares of company B purchased by Mohan from ? 150,000 = 40

(ii) Company B pays dividend at the rate of 15% per annum.


Par value of a share of company 6 = ? 25
Par value of 3750 shares of company B = ? (3750 x 25) = ? 93,750
15
Total dividend received from company B = ? X 93,750 1 = ?14062.50
100

Change in income of Mohan = ? (14062.50 -10000) - ? 4062.50


FXAMI’l.li J ● Mohau invested ^ 12500 and purchased 1000 shares of par value ^ 10 each. How much
above par is the market value of these shares ? If the dividend per share is 30%,/mrf Mohan's income from
these shares.

w
SOLUTION Market value of 1000 shares = ? 12500
n2500
Market value of one share = ^ = n2.50.

F lo
1. 1000
Face value of one share = ? 10.

ee
Hence, the market value is ? 2.50 above par.

Fr
Face value of 1000 shares = ? (1000 x 10) = 110,000.
Since the dividend is paid at the rate of 107o. Therefore, for
10
Dividend on 1000 shares = t X 10,000 = ? 1000.
r
uoo
You
s

Hence, Mohan's income from 1000 shares is ? 1000.


ook

EXAMPi i: If the market value ofa^ 100 share is ? 125, find the amount required bij Shama to
purchase 1250 shares. What would be the gain of the original share holder from Shama if he had bought
eB

each share at 10% discount ?


SOLUTION Market value of one share = ? 125
our
ad

Market value of 1250 shares = ? (125 x 1250) = ? 156250.


Tlius, amount paid by Shama to buy 1250 shares = ? 156250.
The original share holder bought shares at 107o discount i.e. he had bought each share for ? 90.
dY
Re

Amount paid by the original share holder =? (90 x 1250) = ? 112500.


Gain to the original share holder = ? (156250 -112500) = ? 43750.
Fin

43750
Also, Gain percent = xlOO =38.88.
112500

I XAMPI .r n- A company, with a capital off 250,000, declares a semi-annual dividend of 5%. Find:
(i) the total amount of dividend declared in a year,
(ii) the amount of dividend received by Anand holding 50 shares of common stock of par value ? 20
each.

SOLUTION (i) Amount of dividend declared in first six months = 5% of f 250,000


= ? f 100 X 250,000 = f 12500

Total amount of dividend declared in a year - ? (2 x 12500)

(ii) Face value of 50 shares = ? (50 x 20) = f 1000.


25.10
APPLIED MATHEMATICS-XII

Amount of dividend received by Anand in six months =5% of ? i 000 = ? — xlOOO


100
Thus, Amount of dividend received by Anand = ? (2 x 50)
: . \ \i I’LL 2' Awan bmjs 1500 shares of a company (book value ? 10, paid up f5)att 45. If the company
declares a dividend of 35%, find zvhat percentage zuill he earn on his investment?
SOLUTION Cost of 1500 shares = ? (1500 x 45) = ? 67,500
Paid up value of 1500 shares
Since the company will pay dividend on thu paid up value. Therefore,
/ qc
Dividend on 1500 shares at the rate of 35% = ? — X 7500 = ? 2625..
100

ow
Thus, on the investment of ? 67,500, Aman gets ? 2635.
2625
Percentage gain = X 100 = 3.89%
67,500

Shikha bought 800 preferred shares of par value ? 50 each atfZ premium (Brokerage 5%).

e
After a month, the price of the shares fell and she sold 300 of the shares at ? 47 each (Brokerage 2%) and

re
rFl
sometime later she sold the remaining shares at ? 54 each (Brokerage 50 paise each share). Find her gaiin' or
:

F
loss in the transaction.

SOLUTION Cost of one preferred share of par value ? 50 each at ? 2 premium = ? (50 + 2) = ? 52.

r
Cost of 800 preferred shares = ? (800 x 52)
ou
Brokerage on 800 preferred shares = 5% of Rs 41600= ? fo
— X 41600 =?2080.
ks
100

Amount paid in buying 800 preferred shares = ? (41600 + 2080) = ? 43680.


oo

Selling price of 300 preferred shares at ? 47 each = ? (300 x 47) = ? 14,100.


Y
eB

Brokerage on 300 preferred shares = 2 % of ? 14,100 = ? — X 14,100


100

Selling price of remaining 500 shares at ? 54 each = ? (500 x 54) = r 27,000.


r
ou

Brokerage on selling 500 preferred shares at the rate of 50 paise each share
ad
Y

= ?f — x500 =?250.
100
d

Total amount received in selling 800 preferred shares = ? (14,100 - 282 + 27,000 - 250)
Re
in

= ? 40,568.
Total amount paid in buying 800 preferred shares = ?
F

43,680.
Loss = ? (43,680-40,568) = ? 3112
EXAMPLE 28 What rate percent zuill a man get from his 200 common shares of par value ^ 25 each,
bought at^5 premium, the rate of dividend being 8% ?
SOLUTION Par value of a share = ? 25.
Premium = ? 5

Market value of a share = ? (25 + 5) = ? 30.


Market value of 200 shares = ? (30 x 200) = ? 6,000.
Also, par value of 200 shares = ? (25 x 200) = ? 5000.
Dividend on 200 shares = 8% of f 5000= ? x5000
100
Dividend 400
Gain percent =
Market value
X 100 xlOO = 6 7r.
6000 3
25.11
FINANCIAL MATHEMATICS-l!

EXAMIM f How much cash must I pay for 750 shares, of par value ? 10 each, at premium of^ 3
(Brokerage 1%). If the dividend is 10% per annum, find the annual dividend on these shares. What rate
percent do I get for my investment ?
SOLUTION Par value of a share = ? 10.
Premium on a share = ? 3.
Market value of a share = ^ (10 + 3)=t 13.
Market value of 750 shares = ^ (750 x 13) = ^ 9750.
Par value of 750 shares = ? (10 x 750) = ? 7500
1
04083 X 9750 = ? 9750.
Brokerage paid = (1 % of ? 9750) = ^ 100

Total amount paid in buying 750 shares = ^ (9750 + 9750) = ? 984750

w
Dividend on 750 shares =10 % of ? 7500 = ? 750.
Gain 750

Flo
X 100 % = X 100 % = 7.61 %
Gain percent = 984750
Amount paid

ee
EXERCISE 25.1

Fr
. Find the investment in buying
(i) 400 shares of f 10 each at 2 discount;
for
ur
(ii) 500 shares of X 10 each at ? 3 below par;
(iii) 700 shares of X100 each at 8 premium;
s
(iv) 600 shares of X 10 each quoted at X 18.
k
Yo

. Find the annual dividend paid in each of the following cases :


oo

Par value of a share Number of common shares Rate of dividend


eB

(i) XIO 1500 10% per annum


(ii) ?10 1800 5% semi-annually
r

2500 5% quarterly
ou
ad

(iii) XlOO

(iv) ?10 3750 10% semi-annually


Y

(V) ?10 2500 2% per month


(Vi) ?100 5000 7.5% per annum
Re
nd

(vii) ^20 1500 2.5% quarterly


Fi

Find what a buyer would have to pay for 800 shares of X10 each quoted at X 65. What would
be the gain to the share-holder, if he had purchased the share at par?
4. Ranjana invested X 2592. When she bought shares of a company at X108 each the face value
of a share was X 100. The company paid 12.5% dividend at the end of the year. Find the
dividend earned by Ranjana.
A company declared an annual dividend of 10%. Find the annual dividend received by
Ravi owning 4000 shares of the company having a par value of X 100 each.
6. A company declared a semi-annual dividend of 7.5%. Find the annual dividend of Rohit,
owning 2250 shares of the company having a par value of X10 each.
7. Ashish bought 300 shares of a company quoted at X 450. If the brokerage be 1%, find the
amount spent by Ashish on this purchase.
8. Rekha buys 500 shares of face value ^ 10 each at a premium of X 6. If the company declares a
half-yearly dividend of 3%.
(i) How much money does she invest?
(ii) What will be her annual income from this investment ?
25.12
APPLIED MATHEMATICS-XI!

(iii) What percentage will she get on her investment ?


9. How many shares of market value ? 12.50 each can be purchased for ? 12625, if the
brokerage paid is 1%?
10. Find the income derived from 600 shares, if the face value of one share is ? 10, and the
dividend is 16%.
11
. The market price of ? 10 shares of a company is ? 18 above par. Manoj buys 450 shares of the

rw
company from the market. If the company declares a dividend of 12% at the end of the year,
find:

(i) the amount of money invested;

e
(ii) his annual income from this investment;
(iii) rate of interest received by him on this investment.

e
ulo
12. Rahman purchased 300 shares of the face value of ? 100 each from the market at ? 800 per

r
share. If the company paid a dividend of 40%, find Rahman's earning percent on this

F
investment.
13. Which is better investment:

14
oF
16%, ? 100 shares at ? 132 or 18%, ? 100 shares at ? 140?
. A company issued 125000 shares of par value ? 20 each. If the total dividend declared by the

rs
company is ? 375,000, find the rate of dividend paid by the company. Also, find the
dividend received by Monu having 1000 shares of the company.

ko
1;?. Mahesh invested ? 5500 in the shares of a company. At the end of the year, the company
of
declared 20% dividend which gave him an income of ? 500. At what premium were the
shares purchased?
o
Y
16. What income is derived by investing ? 27000 in 12%, ? 10 shares at 2 discount?
B

17. Find the annual dividend received by Amit for his 200 preferred shares and 1000 common
Y

shares, both of par value ? 100 each, if the dividend declared on a preferred share is 10% per
annum and an annual dividend of 12.5% on the common
er

shares.
18. Rohit sold 500 shares of a company with face value of a share as ? 100 and market value
u

as ? 740. What was his gain if he had purchased the shares at 100% premium?
d

19. Shikha pruchased 12000 shares of a company of par value ? 10 each, paying an aimual
o
ad

dividend of 15% at such a price that she gets 10% on her investment. Find the market value
in

of a share.

2U. Simmi buys 200 shares of par value of ? 10 each of a company, which pays an annual
Re

dividend of 8% at such a price that she gets 10% on his investment. Find the market value of
F

a share.

21. Rajneesh invested ? 18180 in the shares of face value ? 100 each of a company. At the end of
the year, the company declared dividend at 16% and Rajneesh received ? 1920 as dividend.
At what price was the share quoted, the brokerage being 1%?
22. The shares of a company are quoted at ? 80. If the nominal value of a share is ? 10 and if the
company declares 15% dividend, what does it cost Rakesh to buy 4000 shares and what
dividend does he get ?
23. A company issued 50000 shares of par value ? 100 each. If the total dividend declared by the
company is ? 125000, out of which ? 50000 have been kept in reserve fund and the
remaining is distributed as dividend, find the rate of dividend paid by the company. Also,
find dividend received by Ankush having 250 shares of the company.
24. A company has a capital of ? 10,00,000 made of ? 10 shares. A profit of ? 50,000 was declared
at the end of the year to be distributed among the share holders. Find the dividend received
by a person holding 300 shares.
25.13
FINANCIAL MATHEMATICS-11

- - The capital of a company is made up of 5000 preferred shares with a dividend of 15% per
annum, and 20000 common shares both of par value ? 100 each. The company had a profit
of ? 10 lakh out of which 6 lakhs rupees were kept for running expenses, 1.25 lakhs rupees
kept for contingent expenditure and the remaining was distributed as dividend. Find the
dividend per cent paid on a common share.
How much money will a person gain by buying 300 shares of ? 10 each at 50 paise discount
and selling them at 25 paise premium ?
■'7, A company has issued 10000 preferred shares and 50,000 common shares both of par value
? 100 each. The dividend on a preferred sharee and a common share is 12% and 17.6%
respectively. The company had a total profit of 15 lakli rupees out of which some amount

ow
was kept in reserve fund and the remaining distributed as dividend. Find the amount kept
in reserve fund.
28. A man sells 400 shares of ? 10 each at X 15 each. The company declares dividend of 21% on
these shares. With the money he gets, he buys shares of X 10 at X 8 each of some other

e
company which declares a dividend of 18%. Find the difference in his income.

re
29. Manisha sells 20,000 shares of company X, of par value X10 each, paying a dividend of 10%,

Frl
at ? 25 each. She invested the sale proceeds in ordinary shares of another company Y, of

F
par value X 25 each and paying 16% dividend. If the market value of a share of company Y is
X 40, find
ou
r
(i) the number of shares of company Y purchased by Manisha
(ii) change in dividend income of Manisha
so
kf
30. The market price of X10 shares of a company is X 16. If a man purchases 500 shares of this
company from the market (i) how much money does he invest over them (ii) what will be
oo
his annual income from this investment ? (iii) what percentage will he obtain on his
1
Y

investment, if the company declares a dividend of 7 —% per annum ? (iv) how many more
eB

shares can he buy for X 2400 ?


31. Sonia sells 5000 ordinary shares of company a, of par value X 50 each, paying 15% dividend,
ur
oY

at X 75 per share. She invested the sale proceeds in ordinary shares of a company p, with par
value X 50 each and paying a dividend of 12%. If the market value of a share of company p is
ad

X 60, find
d

(i) the number of shares of company p bought by Sonia,


in

(ii) change in the dividend income of Sonia.


Re

32. A man wants to invest X 50400 either in X 50 shares at X 6 premium and paying a dividend of
5% or in X 100 shares at 10% discount paying a dividend of 4%. Which investment will be
F

more
profitable to him ? How much more will he get from the better investment ?
ANSWERS

1. (i)? 3200 (ii) X 3500 (iii) ? 75600 (iv) ?10800

(i) X 1500 (ii) ?1800 (iii) 50,000 (iv) ?7500

(vi) ? 37500 (vii) 3000 3. ?52000, ?51200


(v) ? 6000
4. X 300 7. ? 136350 8. (i) Rs. 8000 (ii) nSO (iii) 3%
9. 1000 ;n. ?960 10. ? 15%,? 3000
(iii) X 6.66 % 12. 5%
11. (i) ?8100 (ii) ?540
' V 2nd investment is better 14. 15%, X 3000 IT 120%

16. ?4050 17. ? 14500 \h. ? 270,000 19. ?15

: XS ?150 X 320000, X 6000 1.5%, ? 375


25.14
APPLIED MATHEMATICS-XII

24. ?150 25. 10% 26. ?225 27. ? 5 lakh


?510 29. (i) ? 12500 (ii) ? 30,000
75
*^0
(i) ?8000 (ii) ?375 (iii) —% (iv) 150
16

31. (i) ?6250 (ii) No change .32. ? 50 shares at 6 premium, ? 10


HINTS TO SELECTED PROBLEMS
14. 16%, ? 100 shares at ? 132 means tliat:
Face value of a share = ? 100. Market value = ? 132 and, dividend = 16%
16 18
.'. Gain per share in 1st investment =
132' and Gain per share in 2nd investment = 140
.
18 16
Clearly . So, 2nd investment is better
' 140 132

25.6 DEBENTURES

w
Sometimes a company needs money for further expansion and diversification of its activities.

F lo
For this, the company may not issue new shares but may borrow this money from the
public/share-holders for a fixed period of time at a fixed rate of interest. In such a case, the
company issues debentures. The whole amount of debt needed by the company is divided into

ree
equal small units. These units are called debentures. For example, if a company wants to borrow
? 50 Lakhs, it may issue 25000 (say) debentures of value of ? 200 (say) each. Like share
F
certificates, a company issues debenture certificates. A debenture certificate is an
for
acknowledgment of a debt of a company. Debenture holders of a company get a fixed rate of
interest, generally payable half-yearly or yearly, irrespective of the profit or loss incurred by the
r
company. After expiry of the fixed period, the company repays the debenture money to its
You
oks

holders.
eBo

Like shares, debentures are also sold and purchased in the market. The market value of
debentures also keeps on changing. However, the interest on a debenture is always calculated
on its face value and the brokerage is calculated on its market value. If a debenture holder needs
ad
our

money before the expiry of the period for which debentures are issued, he (she) may sell
debentures in the market.

These days companies issue convertible and non-convertible debentures. Convertible


debentures or a part thereof are converted into equity shares after a specified period of time.
Re
dY

DIFFERENCE BETWEEN SHARES AND DEBENTURES


Fin

Shares Debentures

(i) Shares form a part of the capital of


(i) Debentures are a debt
the company
Share-holders receive dividend Debenture-holders receive

(ii) based on profit earned by the interest at a fixed rate,


(ii)
company. It may vary from year to irrespective of profit or loss of
year the company.
A share holder is a part owner of A debenture holder is a creditor of
(iii) (iii)
the company the company

We shall now discuss some examples to illustrate the computation of brokerage, interest etc. on
debentures.
25.15
FINANCIAL MATHEMATICS-H

ILLUSTRATIVE EXAMPLES

Cotupute the (ui)ui(il yield peveeut on 12% debentures offace vulue ^100 each nnd ovuiliwle
at ? 80 each.

SOLUTION It is given that:


Face value of a debenture = ? 100
Market value of a debenture = ? 80
Rate of Interest on debentures = 12%
Now, By investing ? 80, annual interest = ? 12
ri2
— xlOO =?15.
By investing ? 100, annual interest = t 80

Hence, annual yield = 15%

w
rxAMPLi:: Find the percent income of buyer on 67o debentures offace value ? 100 available in the
market for ? 150.

F lo
SOLUTION We have.
Market value of a debenture = ? 150

ee
Income on ? 150 is ? 6

Fr
Income on ? 100 = ? — xlOO = ?4.
150

Thus, there is 4% income on the debentures. for


ur
IXAMPU Which is better investment: 15% debentures at 8% premium or 14Vo debentures at 4/o
discount?
s
ook

Let the face value of each debenture be ? 100.


Yo

SOLUTION

Now, 15% debentures at 8% premium means :


eB

Face value of a debenture = ^ 100


Market value of a debenture = ? 108
Rate of Interest = 15%
r
ou
ad

Interest on a debenture of ? 100 = ? 15.


Thus, By investing ? 108, annual interest = ? 15
Y

15
xlOO =? 13.88.
By investing ? 100, annual interest = ? 108
Re
nd

14% debentures at 4% discount means;


Fi

Face value of a debenture = ? 100


Market value of a debenture = ? 96 [v 4%of? 100 = 4]
Rate of Interest = 14%
Interest on a debenture of ? 100 = ^ 14.
Thus, By investing ? 96, annual interest = ? 14
ri4
— XlOO =? 14.58
By investing f 100, annual interest = ? 96

Clearly, second investment yields more interest. Hence it is a better investment.


EXAMI’Li: I Mohan has 500 shares of par value ? 10 each of a company 500 debentures of par value ? 100
each. The company pays a dividend of 8% on the shares and pays an interest of 10% on its debentures.
Find the total annual income of Mohan and the rate of return on his investment.
SOLUTION Mohan's investement on shares = ? (500 x 10) = ? 5,000
Mohan's investement on debentures = ? (500 x 100) = ? 50,000
25.16
APPLIED MATHEMATICS-XII

8
Annual dividend on shares = ? 5,000 x = ?400
100

and, 10 ^
Annual intest on debentures = ? 50,000 x = ? 5,000
100 J
Mohan's total income = ? (400 + 5,000) = ? 5,400.
Total investment of Mohan = ? 5,000 + 50,000 = ? 55,000
Rate of return on Mohan's investment =
5,400 'l 9
xlOO %=9 — %
(55,000 11

^ ■ -’f .V 5 How much money wiU Shikha get b\j selling 20% debentures worth ? 6000 at the rate
of ? 120 per debenture? The face value of a debenture is ? 100 and the brokerage is 1.5%.

ow
SOLUTION Total worth of debentures = f 6000
Face value of one debenture = ? 100
6000
Number of debentures = = 60.
100
Market value of one debenture = ? 120

e
Market value of 60 debentures = ? (60 x 120) = ? 7200

re
Brokerage = 1.5%

Fl
F
f 7200
Total brokerage = ? 15 x = ?108
ur 100

r
Thus, amount received by Shikha = Selling price - Brokerage = ? (7200 -108) = ? 7092
liXAMl'l i
fo
Som Dutt invested a certain sum in 18% debentures of the face value of ? 200 each
ks
available at ? 90 and earned an annual income of^ 8100. Find the amount invested by him, if the brokerage
Yo
is 1%.
oo

SOLUTION To purchase one debenture :


eB

Amount invested = Market value of a debenture + Brokerage


90 ^
= ?90 + l%of?90 = ?l 90 + = ? 90.90
100
ur

Income obtained = Interest on ? 100 = ? 18.


ad

Thus, for an annual income of ? 18, amount invested


Yo

= ? 90.90
90.90
For an income of ? 8100, investment made = ? X 8100 = ? 40,905
d

18
Re
in

Hence, the amount invested by Som EXitt = ? 40,905.

LXAMI’LL 7 Rohit invested a part oft 5300 in 13-^ % debentures offace value oft 100 each available at
F

tlW. The rest he invested in 1100 shares available at par. At the end of the year, he received 20% dividend
on his investment on shares. If his income from shares is equal to his income from his investment in
debentures, find the amount invested in shares and debentures.
SOLUTION Suppose Rohit invested t x in debentures and the remaining t (5300 - .v) in shares.
Now,

For an investment of 1110 in a debenture, income obtained = ? —


3
40 1
For an investment of t x, income obtained = t — X X .V X
3 no 33
Dividend on his investment in shares = 20%
20
Income obtained on his investment of t (5300 - x) = t
100
x(5300-.r) _^r530Q-.\-
5
25.17
FINANCIAL MATHEMATICS-ll

It is given that the income from debentures is equal to his income from shares.
4 _ 5300 - x
33 5

20 .Y = 33 X 5300 - 33 .y => 53.Y = 33 x 5300 => .y = 3300


Hence, investment made on debentures was ?3300 and on shares ? (5300 - 3300) - ? 2000.

w
EXAMPLE 8 Ram has 200 shares, of par value UOO each, of a company paying diviiiemi at 6% per
annum. He also has 50 debentures, of par value ? 100 each, on
which the company pays an annual interest
ofW%. Find the total annual income of Ram from investment in the company,
f 6

e
SOLUTION Annual dividend on a share = ? X 100
UOO ;

e
or
Annual dividend on 200 shares = ? (200 x 6) = ^ -.;00.

r
Par value of a debenture = ? 100

F
Par value of 50 debentures = ? (50 x 100) = ? 5000

oF 10

ul
Annual interest on 50 debentures = ? xSOOO =?500.
100

rs
Thus,
Total annual income of Ram from the investment in the company = ? (1200 + 500) = ^ 1700.

ko
EXERCISE 25.2

1. Find the annual yield percent on 16% debentures of


of
face value t 100 each and available
o
Y
at ? 80 each.

of face value ? 100 each and available at


B

2. Find the annual yield percent on 15% debentures


Y

20% discount.

3. Find the annual yield percent on 20% debentures available at 20% premium.
er

4. Bhawna invested a certain sum in 12 — % debentures of the face value of ? 100 each
u
d

available at ? 85, and earned an annual income of f 4000. Find the amoimt invested by her, if
o
ad

1
the brokerage is 1 — %●
in

5. Arun wants to secure an annual income of ^ 1500 by investing in 15% debentures of face
value ? 100 each and available for ? 104 each. If the brokerage is 1%, what sum of money
Re
F

should he invest?
6. Which is better investment : 10% debentures at 10% discount or 20% debentures at 10%
premium?
7. Rakesh sold 20% debentures worth ? 7500 at 5% premium. Find the cash realised from this
sale, if the brokerage was 2%.
8. Ankur invested ? 10800 partly in 12% debentures of face value of ? 100 each available at
? 105 and partly in ? 100 shares available at par. At the end of the year, he received 16%
dividend on shares. If his income from debentures is equal to that from shares, find the
amounts invested in debentures and shares.
9. What amount of money does Rashmi get on selling 14% debentures worth ^ 16000 at 10%
discount, the face value of each being ? 100 and brokerage 1.5% ?
I n. A man invested ? 10800 partly in 12% debentures of face value ? 100 each, available at ? 105
and partly in ? 100 shares available at par. At the end of the year, he received 16% dividend
on shares. If his income from debentures is equal to that from shares, find the amounts

invested in debentures and shares.


25.18
APPLIED MATHEMATICS-XII

Anish has 1200 shares of par value? 20 each of a company and 500 debentures of par value
? 100 each of the company. Tlie company pays an annual dividend of 8% on the shares and
an interest of 12% on debentures. Find the total annual income of Anish and rate of return
on his investment.
I.?.
Komal has 1600 shares of par value ? 10 each of a company and 300 debentures of par value
? 100 each of the company. The company pays an annual dividend of 8% on shares and an
interest of 15% on debentures.

Find the total annual income of Komal and the rate of return on her investment.

ANSWERS

w
20% ?1875 :. 16-% 4. ? 27608
3
?10504 b. Second is better 7717.50 8. ? 6300,? 4500

o
?14184
10. ? 6300 and ? 4500 respectively . ? 7920,10.7%

e
? 5780,12.6%

re
25.7 STOCKS

rFl
F
Sometimes, government requires money to meet the expenses of a certain project or a big work

r
of public utility. In such cases. Government raises a loan from the public at the fixed rate of
ou
sfo
interest and issues Bonds or Promissory Notes as an acknowledgment of the debt. These bonds
are generally for ? 100 and in some cases for ? 500 and ? 1000. The government pays interest at a
k
fixed rate to the holder of these bonds. The interest is paid on the value printed on the bond
oo

certificate. This value is also called the face value. If a person buys a bond of ? 100 on which 10%
Y

interest has been fixed by the government, then the holder of such a bond is said to have 'a ? 100
eB

stock at 10%. If a person is in possession of 15 bonds of ? 100 each on which 8% interest has been
fixed by the government, then the holder of these bonds is said to have 'a ? 1500 stock at 8%'.
r
You

Usually, the government repays the loan at a date fixed at the time of issue of bonds. This date is
ad

called the maturity date. In case the holder of a stock needs money before the date of maturity,
he cannot ask the government to pay back his money. But he can sell his stock to some other
d

person (s), whereby his claim to interest is transferred to that person (s). The sale and purchase
Re
in

of stocks is done exactly in the same way as is done for shares.


F

If the selling price of ? 100 stock is ? 100 cash, the stock is said to be at par.
If the selling price of ? 100 stock is more than ? 100 cash, the stock is said to be at a premium or
above par.
If the selling price of ? 100 stock is less than ? 100 cash, the stock is said to be at a discount or
below par.
The stock is usually named by means of rate of interest it pays. Thus, the expression "9% stock at
95" refers to a stock which pays interest at the rate of 9% and ? 100 stock can be purchased for
? 95. Similarly, the expression "5% stock at 20 discount" refers to a stock which pays interest at
the rate of 5% and ? 100 stock can be purchased for ? (100 - 20) = ? 80. The expression "4.5%
stock at 102" means that ? 100 stock pays a dividend of ? 4.50 per annum and ? 100 stock can be
purchased or sold for ? 102.
The following examples illustrate the use of the above terminology.
25.19
FINANCIAL MATHEMATICS-!!

ILLUSTRATIVE EXAMPLES

Tv; ON COMPUTING THE MARKET VALUE OF A STOCK OR COMPUTAT!ON OF INVESTMENT


EX AMP! E I Find the cost of^ 5000 of 7% stock at 92.
SOLUTION Recall that the expression 7% stock at 92" means that ? 100 stock can be purchased
for ? 92 and it pays a dividend of ? 7 per annum.
Cost of ? 100 stock = ? 92
92
Cost of ? 1 stock = t
100
92
Cost of ? 5000 stock = ? X 5,000 = ? 4,600.
100

ow
EXAMlM.i': Find the cost off 7000 of 15% stock at 105.
SOLUTION Note that the expression "15% stock at 105" refers to a stock which pays interest at
the rate of 15% per annum and f 100 stock can be purchased for f 105.
Cost of f 100 stock = f 105

e
105

re
Cost of ? 1 stock = f
100

rFl
F
/'ins 1
Cost of f 7000 stock = ? ilia X 7,000 =? 7,350.
100

r
EXAMPLE Find the investment required to purchase f 125,000 of 8% stock at 92.
ou
SOLUTION It is given that: fo
ks
The market value of f 100 stock = f92
( 92
oo

Market value of f 125000 stock = f X 125,000 =? 115,000


100
Y
eB

Hence, an investment of f 115000 is required to purchase the given stock.


EXAMPLE 4 Find the cost off 15000 of 5 stock at 99, brokerage being f 1.
ur

SOLUTION Cost of f 100 stock = ^ (99 + Brokerage)


ad
Yo

Cost of f 15000 stock = f


100
X 15,000l = ^15,000.
100
d

EXAMPLE s Hoxv much money is invested in buying f 40,000 of 8% stock at 35 premium (Brokerage
Re
in
F

SOLUTION At 35 premium means that for f 100 stock, investment will be f 135. Since the
brokerage is added to the cost of stock.
Investment for ? 100 stock = f (135 + 15) = f 1365
1365
Investment for Re 1 stock = f
100

Investment for f 40000 stock = 40,000 1 = ^54,600.


100

EXAMPLE 6 Find the cash required to purchase f 165000 of 9% stock at 107, brokerage being f 1.
SOLUTION Cash required to purchase f 100 stock = f (107 + Brokerage) = f (107 + 1) = 108
/
108
X 16500 =? 17,820.
Cash required to purchase? 16500 stock = ? 100

Hence, the cash required is ? 17,820.


25.20
APPLIED MATHEMATICS-Xli

Type I/ ON FINDING THE AMOUNT OF STOCK THAT CAN BE PURCHASED FOR A GIVEN SUM OF
MONEY OR CAN BE SOLD TO RAISE A REQUIRED SUM
E\AMri.E7
How much 11% stock at 97 can be bought by investing ? 24,250 7
SOLUTION By investing ? 97, stock purchased =?100
By investing ? 1, stock purchased
By investing 124250, stock purchased = ? 25,000.
EXAMl’Li- H How much 6% stock at J 08 —
10
can
be bought by investing ? 14,300, brokerage being ] —
10
?

SOLUTION Cost of ? 100 stock = ? f 108 — + 1 —' = ?110


10 10 J

ow
Thus,
By investing f 110, stock purchased = ? 100
fm'
By investing ? 1, stock purchased = ?
.110,

e
100

re
By investing ? 14300, stock purchased = ? X 14,300 =? 13,000

rFl UlO

F
LXAMi’I.m
What amount of stock can be bought by investing^ 1133 in 4% stock at 102 brokerage

or
bein? —?
ou
* 10
ksf
SOLUTION Cost of ? 100 stock = ? f 102 — 1 ^
+ — =?103.
I, 10 10
oo

Tluis, by investing ? 103, stock purchased = X100


Y
B

By investing ? 1, stock purchased = ? —


103
re

100
By investing ? 1133, stock purchased = ? X 1133
103
oYu
ad

EXAMI’l.i; III
Hozv much 7% stock at 106 — be sold to realise ? 21,000, brokerage beins 1 —?
10 o jQ-
d

SOLUTION Here the stock is to be sold. So, the brokerage is subtracted from the selling price.
So,
in
Re

1 1 ^
To realise X 106 — 1— or ? 105, stock sold = ? 100.
F

10 10
rioo
To realise 121000, stock sold = ?
U05
X 21,000l) = ? 20,000
rxAMn.L 11
How much 4% stock at 84 ^ be sold to realise ^ 12,600, brokerage being ~?
1
SOLUTION Selling price of 1100 stock = ? 84 -
4

Brokerage
Amoimt realised in selling ? 100 stock = ? f 84 -4 1
4
= ?84.

Thus, to realise ? 84, stock sold = ? 100


100
To realise ? 12600, stock sold = ? X 12600 = ? 15,000.
84
FINANCIAL MATHEMATICS-II 25.21

1
EXAMI’LC12
A person sells his ^4800,4% stock at 6 premium. How much 3 -7o stock at 4 discount can
he buy from the sale proceeds of the former stock?
SOLUTION Selling price of ? 100 stock is ? 106.
106^
Selling price of ? 1 stock = ?
100
106
Selling price of ? 4800 stock = ? x 4800 =?5088.
100

Hence, sale proceeds of the stock = ^ 5088.


Now, the stock to be purchased is at 4 discount i.e. it is at 96.

ow
So,
By investing ? 96, stock purchased = 1100
100
By investing ? 5088, stock purchased = ? x5088 =?5300.
96

e
■y

re
EXAMPLE 13 Ravish sells t 7337, 6% stock at 96 and invests the proceeds in 8% stock at 106 - How

rFl
F
much stock does he hold nozu?

SOLUTION Selling price of ? 100 stock = ? 96

r
\
96
ou
Selling price of ? 7337 stock = ? X 7337
UOO ; fo
ks
Thus, sale proceeds of the stock = ? 704352.
2
Since the sale proceeds of the stock are to be invested in 8% stock at 106 —.
oo
Y

2
B

By investing ? 106 —, stock purchased = ? 100


3
100 X 3
re

By investing ? 1, stock purchased = ?


320
ou
Y
ad

By investing 17043.52, stock purchased = ? 100 x — X 704352 =? 6603.30


V 320

Type III ON FINDING THE CASH REALISED OR SALE PROCEEDS BY SELLING THE STOCK
d

[y
in
Re

EXAMPLE 14 Hozv miick money is obtained by selling ^ 30,000 stock at 93, brokerage being ^ ^ ^
F

SOLUTION Net selling price of ? 100 stock = ? (93 - Brokerage) = ? (93 -1.5)
91.5
Selling price of ? 1 stock = ?
100
r91.5
Selling price of ? 30,000 stock = ? X 30,000 =? 27,450.
100

EXAMPLE 15
Hoiv much money is obtained by selling ? 30,000 of 15% stock at 40 premium, brokerage
being ? 2.?
SOLUTION At 40 premium means that the cost of ? 100 stock is ? 140. Since the brokerage is
subtracted from the selling price.
Net selling price of ? 100 stock = ^ (140 - 2) = ? 138.
H38
Selling price of ? 1 stock = ?
100 J
ri38
Selling price of ? 30,000 stock = ? 100
X 30,000
25.22 APPLIED MATHEMATICS-XII

T\/pc IV ON FINDING THE GAIN OR LOSS IN BUYING AND SELLING OF STOCK


I
EXAMl’LE Ih Find the change in income by transferring"^ 6000 of} 3 -^% stock at 90 to a 10 2
% stockat

par.
SOLUTION Selling price of ? 100 stock = ? 90.
90
Selling price of ? 6000 stock = ? 100
x 6000 =?5400

1
13 '
Income from first stock = 13 — % of ? 6000 = ? 2- X 6000 X 6000 = ? 810
2 100 200

ow
10 1
Income from second stock = 10 of? 5400 =? ^ X 5400 X 5400 = ? 567
9 100 200

e
re
Loss in income = ? (810 -567) = ? 243.

rFl 1

F
CXAMI’U: 17 A man buys ?3000 stockat 90 and sells when its price rises to ? 92—.
4
Find the gain.

SOLUTION Cost of ? 100 stock = ? 90.

r
ou
S.P. of ? 100 stock = ? 92 -.
4
fo
ks
Gain on ? 100 stock = ? 92 —-90 =?—.
oo

4 4

f9
Y

1
eB

Gain on ? 3000 stock = ? — x X 3000 = ? 6750.


4 100

EXAMPLE IS blozv much would a person lose by selling ? 4250 stock at ? 87, if he brought it at 5
r

premium ?
ou
ad
Y

SOLUTION Cost of ? 100 Stock = ? 105.


S.P. of? 100 stocks? 87.
d

Loss on ? 100 stock = ? (105 - 87) = ? 18.


Re
in

18
Loss on ? 4250 stock = ? x 4250 =?765.
100
F

J
EXAMPLE 19 Which is the better investment 10 —% stockat ^ 90 or, 11% stockat par ?
2

SOLUTION For the investment in 10 - % stock at ? 90.


2

Income on ? 90 =? 10.50
10.50
Income on ? 1 = ?
90
10.50
Income on ? 100 = ? xlOO =? 11.66
90

Income onll% stock at par is ? 11 on an investment of ? 100.


Clearly, income from the first investment is more than that of second investment.
Hence, investment in 10 —% stock at ? 90 is better.
2
FINANCIAL MATHEMATICS-ll 25.23

tXAMIM.H 2i
A man sells ? 5,000 of 9% stock at ? 206 and invests the proceeds in a 13% stock at ? 139.
Find the amount of nexv stock purchased and change in income (Brokerage = ? 1).
SOLUTION Sale price of ? 100 stock = ? (106 - Brokerage) = ? (106 -1)
no5^
Sale price of ? 1 stock = ^ 100

rio5
Sale price of ? 5000 stock = ^ X 5,000 5,250
100

For the new stock, we have


Cost of ? 100 stock = ? (139 + Brokerage) = r (139 + 1) = ? 140
Thus,
For the investment of ? 140, stock purchased = ?100

w
100
For the investment of 15250 stock purchased = ? X 5,250 = f 3,750
140

Now, Income from the first stock = 9% of ? 5000 = ?

F lo — X 5,000
100
= ? 450.

ee
13
Income from the second stock = ? 13 % of ? 3750 = ? X 3,750 =?487.50.

Fr
UOO
Gain in income = ? (487.50 - 450.00) = ? 37.50. for
i:x AMI’LL 21 A person invests ? 1365 m 3% at 91. He sells out ? 1000 stock zuhen theij have risen
r
You
s

and the remainder zvhen they have fallen to 85. Hozu much does he lose or gain by this transaction ?
ook

SOLUTION By investing ? 91, stock bought = ? 100.


eB

100
By investing ? 1365, stock bought = ? X 1365 =?1500
91
our
ad

Now, C.P. of ? 100 stock = T 91


1
S.P. of X 100 stock = ? 93
2
dY
Re

Gain = S.P.-C.P. = ? 93 -91 =?2.5


2
Fin

Thus, Gain on ? 100 stock =^25


25
Gain on ? 1000 stock = ? X 1000 = ? 25
100

Remaining stock = ? (1500 -1000) = f 500.


Since the remaining stock is sold 85.
C.P. of ? 100 stock = ^ 91
S.P of e 100 stock = ? 85

Loss = C.P. - S.P- = ? (91- 85) = ? 6.


Thus, loss on ? 100 stock = ? 6

Loss on ? 500 stock = ? — x500 = ? 30


100

Hence, the net loss in the transaction = ? (30 - 25) = ? 5


25.24 APPLIED MATHEMATICS-XII

Tiipe V ON FINDING THE AMOUNT OF STOCK WHEN THE GAIN OR LOSS IN BUYING AND SELLING OF
A STOCK IS GIVEN

EXAMPLE 22
A man invested a certain sum in 5% stock at 95 ^ and sold it out when the price rose to
100 In the transaction he gained a sum oft 90. What sum did he invest ? How much stock did he buy
4

brokerage ^

SOLUTION C.P. of 1100 Stock = t 95 - + Brokerage = t 95


2 2
= t97.

( 3 1

ow
3 1
S.P. of 1100 stock = t 100 Brokerage = t 100--1- = t99-
4 4 2 4
1 1
Gain = S.P.-C.P. = ? 99--91 =t2- = t~.
4 4 4
9

e
Now, when gain is t investment = t 97

re
Fl 97

F
When gain is 190, investment = t X 90 =t 3880.
94
ur
Now, By investing 197, stock purchased = ? 100.

r
By investing 13,880, stock purchased = t
100

97
fo
X 3,880 = t 4,000
ks
Yo
EXAMPLE 23
A man invested a certain sum in 7 -^% stock at 103 and sold when the price rose to 107,
oo

gaining thereby a sum oft 36. What sum did he invest ? How much stock did he buy ?
eB

SOLUTION C.P of 1100 Stock = ?103.


S.P. of? 100 stocks? 107.
ur

Gain on ? 100 stock = S.P.-C.P. = ? (107 -103) = ?4


ad

When gain is ? 4, investment = ? 103


Yo

^ 103
-
When gain is ? 36, investment = ? — X 36 = ? 927
\ 4
d
Re
in

Thus, the sum invested = ? 927.


Now,
F

By investing ? 103, stock purchased = ? 100.


100
By investing ? 927, stock purchased = ? x927 =?900.
103
1
EXAMPLE 24 A man invests 134,000 partly in 8% stock at ? 80 and the remaining in 7 — % stock at t

90. If his annual income be 13,000, hoio much stock of each kind does he hold ?
SOLUTION Let the investment in 8% stock at ? 80 = x.

Investment in 7 ~ % stock at ? 90 = ? (34,000 - x)

First stock:

Investment of ? 80 stock yields income = ? 8


Investment of ? x stock will yield income = X X = ?^
80 10
FINANCIAL MATHEMATICS-II 25.25

Second stock:

Investment of 190 stock yields income “ ^ ^ 2


Investment of ? (34,000 -.t) stock will yield income
e f—X
2

90
X (34,000 - x) 1 = ^ 12

X 34,000
Total amiual income = ? — +
10 12

X 34,000 - X
+ = 3000 [●.● Total annual income = ? 3000]
10 12

ow
6x +170,000-5x =180,000
x= 10,000
Investment in 8% stock at ? 80 = ? 10,000

and, investment in 7 -% stock at ? 90 = ? 24,000.


2

e
EXAMPLE 25 A man invests part oft 8370 in 9% stock at 196 and the remainder in 12% stock at 1120.

re
rFl
His income from each investment is the same. Find the amount invested in each stock.

F
SOLUTION Let the investment in 9% stock at ? 96 = x
Investment in 12% stock at 1120 = ? (8370 - x).

r
First stock:
ou
Investment of 196 yields income = t9 fo
ks
( 9
Investment of ^ x will yield income - t — x x
(.96 32
oo

Second stock:
Y

Investment of 1120 yields income = ? 12


eB

Investment of t (8370 - x) will yield income = t


12
X
(8370 - x) ^ f 8370 - X

120 10
r

It is given that the income from each investment is same.


ou
ad
Y

3x _ 8370 - X => 30x = 267,840 - 32x => x = 267,840 = 4320.


32 ” 10 62

Investment in 9% stock at ? 96 =t 4320.


d

and, investment in 12% stock at 1120 = t (8370 - 4320) = t 4050.


Re
in

EXAMPLE 26 Equal sums of money are invested in 9 --% stock at ? 85 and 12 —% stock at 1117. If one
F

of these investments yields 120 more than the other, find the sum invested (Brokerage 50 paise).
SOLUTION Let ? xbe invested in each stock. Since brokerage is 50 paise, therefore, in 9 —% stock
at f 85,
l’\ 1 1
Investment of ? 85 + - i.e. 85 — yields income = ? 9 —
2) 2^ 2
f
19 X 2x

Investment of T x will yield income = ? ^ x


= ? — X
1 2 171 9
85
2J

In 12 — % stock at ? 117:
2
25.26 APPLIED MATHEMATICS'XII

1 1 1
Investment of ? 117 + - i.e. ? 117 - yields income = ? 12 -
2J 2^ 2
r25 2
Investment of ? a: will yield income = f — x X X ^5.
I 2 235 47

It is given that one of these investments yields ? 20 more than the other.
X 5x 2x 20 X 9 X 47
— =20 => = 20=> x = = 4230.
9 47 9x 47 2

Thus, the sum invested in each stock is ? 4230 and the total sum invested is ? 8460.

l)/pc VI ON FINDING THE INCOME DERIVED FROM A GIVEN QUANTITY OF STOCK

ow
EXAMIM.I-: 27 Determine the dividend obtained from ? 5300 of 8% stock.
SOLUTION Dividend from ? 100 stock = ? 8.

Dividend from ? 5300 stock = ?


f 8
x5300 =?424.
100

e
re
EXAjVIPLE 28 What dividend will be derived from ? 4000 of 5% stock at 95?
SOLUTION

rFl
Dividend from ? 100 stock = ? 5

F
Dividend from ? 4000 stock = ? — x 4000 =?2000
uoo

or
ou
NOIl- In the above example, ? 5 is dividend either on ? 100 stock or it is income on ? 95 cash investment.
So, the quotation oft 95 does not have any meaning in this problem. Ifiue want to use 195, then we will
ksf
have to convert ? 4000 stock to the cash investment.
oo

EXA.MPI.E 29 Determine the annual income derived by investing 13300 in 7% stock at 110.
Y

SOLUTION For the investment of 1110, Income derived = t 7


B

f 7
For the investment of 13300, income derived = t — x 3300 =t210.
re

110

ALITER InvesOnent of 110 buys the stock = 1100


oYu
ad

fm
Investment of 13300 buys the stock = t X 3300 = t 3000.
110
d

Now,Income on 1100 stock = t7.


in
Re

Income on 13000 stock = t — x 3000 =?210.


100 J
F

3 9
EXAMPLE 30
Find the annual income derived by investing t 3360 in 6—% stock at 88— .
4 10
1
(Brokerage 1 —).

SOLUTION C.P. of 1100 stock = ? f 88 — + 1—1 ^ =?90.


10 10
3
Investment of 190 yields income = ? 6 — =
4
/ 27 1
Investment of 13360 yields income =t — X — X 3360
I 4 90

Type VII ON FINDING THE AMOUNT OF STOCK WHEN INCOME DERIVED FROM IT IS GIVEN

EXAMIT.E 3! What amount of 5% stock will produce an annual income of 1500 ?


SOLUTION To produce annual income of 15, the amount of stock required is 1100
FINANCIAL MATHEMATICS-il 25.27

100
To produce annual income of ? 500, the amount of stock required = ? — x500 =^10,000.

EXAMPLE32

/
W}iat amount o/3^ % stock will produce an annual income of^ 77after paying an income
tax o/8- paise a rupee ?

SOLUTION Income on ? 100 stock = ? 3 -


2 2

^(25
25 1 'i
- paise = — paise =
Income tax on ? 1 = 8 — = ? —X
3 100 12

ow
Income tax on ? — ? — X - = ? —.
2 Il2 2 24

(7 7
Net income on ? 100 stock = ?
2 24) 24

Now,

e
77

re
Fl
To produce annual income of ? 24' the amount of stock required = ? 100.

F
100
ur
To produce aniiual income of ? 77, the amount of stock required = ^ x77 = ?2400.
77

r
/
fo 24
ks
EXAMPLE 33 Hozo mucli must he invested in 4 —% stock at 98 to produce the same annual income as
Yo
oo

would be obtained by investing ? 7,395 in 6 ~% stock at 105 ?


eB

3
SOLUTION By investing ^ 105, the Income = ^ ^ 4
ur

(27
, 7,3951 = ?!^
1
By investing ? 7,395, the income = ? \ —
4
x 105 28
ad
Yo

Now,
9
For the income of ? -, investment required = ? 98.
d
Re
in

13311 98 13311
F

For the income of ? investment required = ? x


28 ' 28
I 2
2 13311
= ? 98 X - X = ^ 10353.
9 28

1 9
EXAMPLE 34 How mucIi should Rahul invest in 3-% stock at 96 —
10
to get a net income off 532 after
2
1
paying an income tax of 5 paise in a rupee (brokerage 1 —)?
( 9 1
SOLUTION C.P. of f 100 stock = f 96 — + 1 —
10 \0)

Income tax on one = ? 5 paise


7 f I 7^1
Income tax on? — = ? — x —
2 20 2)
25.28 APPLIED MATHEMATICS-XII

7 7
Net income on the investment of ? 98 = ? ^ 133
2 40 40

Thus,
133
For the income of ? investment required = ? 98
40 '
40
For the income of ? 532, investment required = ? 98 X
x532 =?15,680.
V 133

EXAMPLE 35 A man has ? 3120. He invests his sum partly in 3% stock at 8% and partly in 4 -% stock
at 135. His annual income from both the stock is ? 110. Hozv much did he invest in each stock ?

w
SOLUTION Suppose he invested ? a: in first stock and the remaining ?(3120 - x) in second stock.
In first stock:
Investment of ? 81 yields income = ?3

o
3 X

e
Investment of ? x will yield income = ? — x = ?—
81 27

re
Frl
In second stock:

F
9
Investment of ? 135 yields income = ^ ~
f 9 1
^r3120-x^

or
ou
Investment of ? (3120 - .r) will yield income = ? — x — X
{3120-x) =
12 135
kfs 30
3120-X
Total income = ? — +
27 30
oo

But, the total income is given to be ? 110.


Y

X 3120 - .X
B

+ = 110 => lOx + 28080 - 9x =110 x 270 => x = 1620.


27 30
Investment in 3% stock = ? 1620.
re

Investment in 4 ^ stock = ? (3120 -1620) = ? 1500.


oYu
ad

EXAMPLE 36 Shiklw invested ? 9299 partly in 3% stock


at 93 ^ and remaining 4 ^ % stock at 102, in
d

such a manner that she gets an income of ^ 168 more on the second stock than the first. How much has she
in
Re

invested in each type of stock?


F

SOLUTION Let ? x be invested in 3% stock at 93 —.


2

Then the remaining ? (9299 - x) are invested in 4 - stock at 102.


2
1 f 3
Income on 3% stock at 93 — = ? X X
2 187/2 187

Income on 4 —% stock at 102 = ?


2 102
^x(9299-x) =? —~—x(9299-x)l=?
2x102
/ 83691-9x^
204

It is given that the investment second stock yield ? 168 more than the first stock.
83691 - 9x 6x
= 168
204 187
FINANCIAL MATHEMATICS-II 25.29

(9404083 - 9x) x 11 - 6x x 12
(83691)= = 168
2244
920601 - 99x - 72x = 168 x 2244
171x = 929601 376992=543609
543609
X = = 3179
171
1
Investment in 3% stock at 93 = ? 3179
2

And,
Investment in 4 ^ % stock at 102 = ? (9299 - 3179) = ? 6120.
EXAMPUI37 Out off 75,000, a person invested some money in 4% stock at 90 and the balance in 7%

ow
stock at 130. If the income from second investment is f 500 more than the income from the first
investment, find his investment in each.
SOLUTION Let the investment in 4% stock be ? and in 7% stock be f (75,000 - x)
In 4% stock at 90:

e
Investment of f 90 yields income = ? 4

re
Fl f 4x'
Investment of ? a: will yield income = ? —
90

F
In 7% stock at 130:
ur
Investment of f 130 yields income = f7

r
Investment of f (75,000 - x) will yield income = f — X (75,000-a:)fo
ll30
ks
Yo
Since the income from second investment is f 500 more than the income from first investment.
oo

— X (75,000 - x) - — =500
eB

130 90

52,500 7x 4x
— =500
13 130 90
ur

7x 4x 52,500
ad

+ -500
Yo

130 90 13

63,r+52.v 52,500-6,500 115y _ 46,000 => X = 36,000.


d

1170 13 1170 ~ 13
Re
in

Hence,
Investment in 4% stock = f 36,000.
F

Investment in 7% stock = f (75,000 - 36,000) = f 39,000.


LXAMn,E38 A man secures an income off 80 by investing partly in the 4% stock at 75 and partly in
3% stock at 60. If he makes 5 —%
19
on his money, what does he hold in 3% ?
SOLUTION Suppose he invested ? a: in 3% stock.
In 4% stock at 75:

For the income of ? 4, investment = ? 75


f75
For the income of f 80, investment = f —x80 = ?1500

Total investment = f (1500 + a:)

It is given that he earns 5 —%


19
of his investment.
25.30 APPLIED MATHEMATiCS-XIl

Total income =5 of ? (1500 + ,v) = ? X (1500 + y)


19 19 ^ 100
1500 + -Y
Income from 3% stock at 60 = ? -80 -(i)
19

In 3% stock at 60:

Investment of ? 60, yields income = ? 3.


f 3
Investment of ? x will yield income = ? — y ...(ii)
60 20

From (i) and (ii), we have


1500 + Y X
Y - 20 _ Y
-80 Y = 400.
19 20 19 ~20

w
Thus, he holds ? 400 of 3% stock at 60.

/i/;n- \ HI ON FINDING THE MARKET VALUE (QUOTATION) WHEN CHANGE IN INCOME IN A

F lo
TRANSACTION IS GIVEN

I XAMI’l.l A man invested ? 9000 in 9% stock at ? 90. He sold the stock zvhen price rose to ? 105 and

ee
invested the sale proceeds in 15% at some price. By doing this, his income increased by ? 675. At what

Fr
price did he purchase the second stock?
SOLUTION Suppose he purchased the second stock at price of ? y. for
First transaction:
ur
Investment of ? 90 yields income = ? 9
s
ook

Investment of ^ 9000 will yield income = ? — X 9,000 =?900


Yo

90
eB

Income from first stock = ? 900.

Now,
our

Investment of X 90, purchases stock = ?100.


ad

100
Investment of ? 9,000, purchases stock = ? X 9000 = ? 10,000
90
Y

Thus, amount of stock purchased for ? 9000 = ? 10,000.


Re
nd

The first stock is sold at 105.


Fi

rio5
Money received on selling the first stock = ? X 10,000 =? 10,500.
100

Second transaction:

It is given that the income from second transaction is ? 675 more than that of first.
Income from second stock = ^ (900 + 675) = ? 1575.
Now,
When income is ? 15, investment = X x
f X \
When income is ? 1575, investment = ? ~ x 1575 = ? 105y
15

Since the money received on selling first stock is invested in second stock .
105 Y = 10500 =:>Y= 100.

Hence, the second stock was bought at par.


FINANCIAL MATHEMATICS-il 25.31

i-:\ \\in.i 4i'


A man sells out ^ 6300 of 3% stock at 110 and re-invest the sale proceeds in 5% stock at the
current price. Bi/ doing so he increases his income b\j ? 42. What is the current price of the latter stock?
SOLUTION Let the current price of the latter stock be ? .v.
no
Selling price of first stock = t 100
X 6300 =? 6930

f 3
Income from first stock = ? X 6300 = ? 189.
100

Income from the second stock = ? (189 + 42) = ? 231


Since the second stock pays dividend at the rate of 5%.
Therefore,

ow
For the investment of ? x, income = ? 5
f5
For the investment of ? 6930, income = ? — x 6930
X

5 5 X 6930

e
- X 6930 = 231 ^ .r = = 150
231

re
X

Hence, the current price of the second stock is ? 150.

rFl
F
l)/pe IX ON FINDING THE MARKET VALUE OF THE STOCK WHICH YIELDS A GIVEN RATE OF
INTEREST

r
ou
/
r.XAMI’l,l-41
fo
What is the market value of4 -^% stock when an investment of ? 1800 produces an income
ks
o/? 72 ?
1
oo

SOLUTION Let the market value of 4 — stock be ? x. Then,


2
Y
eB

Investment of ? x yields income “ ^ ^ 2 2

( 9
^8100
r

Investment of ? 1800 will yield income = ? X 1800 =


ou

2x x
ad
Y

8100 8100
= 72 => x = = 112.50
72
d

Hence, the market value of the stock is ? 112.50.


Re
in

EXAMI’1,1.47
Anish sold ? 16,000 of 4% stock in ? 15,000. If brokerage is 1 ^,find the market price of the
F

stock.

SOLUTION The cash price of ? 16,000 stock = ? 15,000


Cash price of ? 100 stock =? x 100 ^375
^ 16,000 4

This is the price after subtracting the brokerage at 1


rS75 5\
Market price of ? 100 stock = ? + - =?95
4)

Ti/pc X ON FINDING THE RATE PERCENT OF INTEREST DERIVED FROM A GIVEN INVESTMENT

rxAMl’l r Determine the rate of interest obtained by investing in 5% stock at 120.


SOLUTION Investment of X 120 earns interest = ? 5
5 \ 1
Investment of X100 will earn interest = X — xlOO =X4-
120 6
25.32 APPLIED MATHEMATICS-XII

Thus, the rate of interest earned = 4 —%


6
7 3
EXAMPLE 44 W7irtf rate of interest is obtained by investing in 5% stock at 118 —, brokerage ^ ~ ?
/ 7 3 ^
SOLUTION CP of ? 100 Stock = ? 118 —+ 1— = ? 120.
10 10

Investment of ? 120, earns interest = ^ 5


5 ^ 1
Investment of ^ 100 will earn interest = ? — xlOO =?4-
120 6

Hence, the interest earned = 4-%


6

Type XI ON TRANSFER OF STOCK AND CHANGE IN INCOME DUE TO TRANSFER

w
EXAMPLE 45 A person holding ? 15000 in the 3% stock sells out at 95 and invests the proceeds in 4%
stock at /00 Find the change in his income allowing brokerage I

F lo
in each transaction.

SOLUTION First stock:

ee
Income from X100 stock = ? 3.

Fr
Income from ? 15,000 stock = ? — X 15000 =?450.
100
n 1
for ^187 1
Sale proceeds of ? 100 stock = ? 95-1 — = ? 93 - = Brokerage = 1 —
ur
V 2, 2 2

— xl5,000l = ? 14,025.
s

Sale proceeds of f 15,000 stock = t X


ook

2 100
Yo

Second stock:
eB

C.P. of ? 100 stock = ? I 100 -2 +1 -1


2)
= ? 102.
our
ad

By investing ? 102, stock purchased = f 100.


By investing ? 14,025, stock purchased = ? x 14,025 = ? 13,750.
dY

Income from ? 100 stock = f 4


Re
Fin

Income from ? 13,750 stock = ? — X 13,750 = ? 550.


100

Hence, change in income = Income from second stock - Income from first stock = ? (550 - 450)
EXAMPLE 46 A persofi sold ? 4000 of 3% stock at 80 and invested the proceeds thereof in 5% stock at
128. Flow much of the second stock did he buy and what was the change in his income ?
SOLUTION First stock:
Income from ? 100 stock = ? 3.

Income from ? 4000 stock = ? — x 4000 =?120


100

Sale proceeds of ? 100 stock = ? 80


80
Sale proceeds of ? 4000 stock = ? X 4,000 = ? 3200
100

Second stock:

By investing ? 128, stock purchased = ? 100.


FINANCIAL MATHEMATICS-II 25.33

rioo X 3200 = ? 2500.


By investing ? 3200, stock purchased = ? 128

Income from ? 100 stock = ? 5

Income from ? 2500 stock = f x 2500 = ? 125.


100

Change in income = Income from second stock - Income from first stock
= ? (125-120) = ? 5.
Hence, second stock bought for ?2500. Change in income = ? 5 increase.
l l/fh' I MISCELLANEOUS PROBLEMS ON STOCKS, SHARES AND DEBENTURES
EXAMPLE 47 Hozv much of 8% stock at 96 be sold so that the sale proceeds, if invested at 10% stock at
105, would increase the annual income by ? 144.

low
SOLUTION Let the stock of ? x be sold. Then,
8.V ^
Income on this stock = 8% of ? x = ?
100 25

r96.v^ ^24.
Sale proceeds of the stock = ? 100 25

ee
F
It is given that sale proceeds of this stock i.e. ?
24.Y
are invested at 10% stock at 105.

Fr
25
10 24.Y
Income on second stock = ?
105"" 25 175 for
ur
16.t 2.Y
Difference in income = ?
U75 25 175
k s
Yo

But, the annual income increases by ? 144.


oo

2.V
= 144 => x= 12,600
eB

175

Hence, ? 12,600 of 8% stock at 96 should be sold.


r

EXAMPLE48 Ravish invests ? 20,500 in 4% stock at 125 while Ravi invests ? 24,600 in another stock at
ou
ad

120. If the ratio of the incomes of Ravish to that of Ravi is 4:5, at what rate has the annual dividend been
paid to Ravi?
Y

SOLUTION Let the rate of dividend paid to Ravi be x%. Then,


Re
nd

Ravi's income = ? ^x 24,600 1 = ? 205 X


120
Fi

( 4
Ravish income = ? — X 20,500 = ? 656.
U25
It is given that:
Ravish's income : Ravi's income =4:5
656
= - => 820-Y = 3280 => x = ^ =4
205.V 5 82

Hence, the rate of dividend on Ravi's stock is also 4%.

EXAMPLE49 A person invests equal sums in 4% and 4 stocks and obtains equal income. If 4% stock
isat a discount of ^4, find the other type of stock, given that par value of each stock is ? 100.
SOLUTION Let the market value of the second stock be ? x. Also, let ? y be invested in each
stock. It is given that the 4% stock is at a discount of ? 4. This means that its market value is ? 96.
25.34
APPLIED MATHEMATiCS-XII

/ 4 ,,
Income on first stock = ? — x y =
1.96 24

1
4
Income on second stock = ? 2
-xy a
A' 2.V

It is given that the person earns equal income from the two investments. Therefore,
a=108
24 2.r

1
Hence, the other stock is 4 —% stock at ? 108.
2

I >AMri I ● l^ohanholds50share$ofparvahie^600eachofncowparii/whi chpa\/sdiviclendattliernte

low
of 5% per annum. When the price of a share rises to ? 784, the shares are sold and he invests half the sale
proceeds in 7% stock at 98 and the other half in 8% debenture at ^ 100. Find the change in his income.
SOLUTION We have. Par value of a share = ? 600
Total par value of 50 shares = ? (600 x 50) = ? 30,000

ee
5 ^
Dividend on these shares = ?

F — X 30000 =?1500.

Fr
uoo
Since each share is sold at ? 784. Tlierefore,
Total sale proceeds of 50 shares = ? (784 x 50) = ? 39,200 for
ur
It is given that half of the sale proceeds i.e. ? 19,600 are invested in 7% stock at 98 and the other
half in 8% debentures at ? 100. Therefore,
ks
Yo

f 7
oo

Income on investment in stock = ? — x 19,600 = ?1400


98
eB

8
and. Income (in the form of interest) on investment in debentures = ? X 19,600 =?1568.
100
r

Total income = ? (1400 +1568) = ? 2968.


ou
ad

Change in income = ? (2968 -1500) = ?1468.


Y

I X.AMI'I 1 Si
A man has ? 30,000 4 —% stock. Hesellshalfofitat ? 98 and the remaining at ^ 105. He
Re
nd

then reinvests the entire proceeds in 5% stock at 101 i Iftheface value ofeach of the stock is ^200, find the
Fi

amount of stock he now holds and change in his income.


SOLUTION We have.

A
Income on first stock = ? 2- X 30000 = ^ 1350
100

It is given that the man sells half of the stock i.e. ? 15,000 at ? 98 and the remaining half at ? 105.
Therefore,
98
Sale price of first half of the stock = ? X 15,000 =? 14,700
100
105
Sale price of second half of the stock = ? X 15,000 =ri5,750
100

Total sale proceeds = ? (14700 + 15750) = ? 30450


FINANCIAL MATHEMATICS-11 25.35

Now, total sale proceeds are invested in 5% stock at 101

Income on new stock - ? X 30450 = ?1500


203
2

Hence, Change in income = ? (1500 -1350) = ?150.


EXERCISE 25.3

Find the cost price of:


(i) ? 4500 of 5% stock at 95. (ii) ? 8700 of 8% stock at 5 premium
(iii) ? 2400 of 3% stock at 20 discount. (iv) ? 7600 of 8% stock at par (brokerage 1%).
(V) ? 1200 of 10% stock at 8 premium (brokerage

w
(Vi) t 20,000 of 12% stock at 20 premium (brokerage 1 ^).
(vii)

F lo
? 15000 of 5 -^% stock at X 99, brokerage being Re 1.

ee
How much 6% stock at 111 - be sold to realise X 13310, brokerage being 1

Fr
4 4
How much 10% stock at 95 be sold to realise X 8550?
4.
for
What amount of stock can be bought by investing X1560 in 8% stock at 104?
ur
D.
What amoimt of stock can be bought by investing X 1856.25 in 5% stock at 3 premium,
brokerage being —?
s
ook

8
Yo

b. How much stock must be sold to realise ? 4590 from 9 % stock at 103—, brokerage being
eB

lb
4
our

1
ad

7. How much stock must be sold to get X 2130 from a stock at X 106 —?

8. How much stock must be sold to get X 5767.50 from a stock at 96, brokerage being ^?
Y
Re
nd

How much cash is reqirired to purchase X 26400,9% stock at 104, brokerage being 1?
10. How much 10% stock at 102 can be bought by investing ? 18870?
Fi

1 [. How much money will a man get by selling X 5000 of 12% stock at 10 premium, brokerage
being —?
^ 10
12. How much money will be realised from the sale of ? 20,000 of 6% stock at 16 discount,
brokerage being —?

13. A man bought X 5000 stock at 95 and sold it when its price rose to 98 Find his gain.
3
14. What is a better investment: 7% stock at X130 or 6—% stock at X 125?
4

13. A man bought X 3750 stock at par and sold it out when it was quoted at 105. What was his
gain or loss?
25.36 APPLIED MATHEMATICS-XII

16. A man invested a certain sum in 4% stock at 102 Suddenly the price of the stock fell and
8

he sold it out at 99 — and lost ? 550 in this transaction. Find the sum invested by him and the

stock bought (brokerage 1 ^).


17. Mohan invested a sum of money in 12 -% stock at
100 ^ and when they fell to 98 he sold

them losing thereby ? 90. What did Mohan invest and what amount of stock did he buy?
(Brokerage 1 ^).
18. A person sells his ? 4800 of 4% stock at 6 premium. How much 3 stock at 4 discount can

w
buy from the sale proceeds of the former stock?

Flo
1 1
19. Rahul invested a certain sum in 10—% stock at 91 — and sold it out when the price rose to

ee
3
96-, gaining thereby ? 13.75. What sum did he invest? How much stock did he sell

Fr
(Brokerage 1 ^)?
for
ur
20. Determine the annual income from an investment of ? 2625 in 5% stock at 105.
21. Determine the dividend obtained from:
s
ok

(i) ? 2000 of 6 —% stock at 90.


Yo

(ii) ? 2580 of 12 stock at 105.


4 2
Bo

(iii) ^5300 of 8% stock.


(iv) ? 4125.50 of 9% stock at 92 ^ (Brokerage 1/5).
re

22. What amount of 4 stock will produce a half yearly income of ? 42.50?
ou
ad

23. Rameshinvested? 1728 in 2 stock at 108. Find his income after paying an income tax of
Y

1
6 - paise in a rupee.
nd
Re

24. Determine the sum of money invested in 4% stock at 105 to produce a net income of ? 146.64
Fi

after paying an income tax of 6 paise in a rupee.


25. By investing some money in 6 stock at 124, Rohit earns ? 150.80. Find the amount of

stock with him.


26. A man who holds ? 3900 of 4% stock and ? 2400 of 7% stock wants to sell out the former at 82
and the latter at 118 and re-invest the proceeds partly in 5% stock at 96 and the rest in 6%
stock at 108 so as to get exactly the same income as before. How much of each stock he buy?
27. After retiring from his service Mr. K. Mohan invests his entire amount of ? 41,600 drawn
7 1
from the P.F. in buying 15% stock at 103 -. If the brokerage is—and he has to pay income tax
8 8
@20 paise per rupee, then what would be his net income in a year?
28, A person invested ? 24000 partly in 3% stock at 75 and partiy in 4% stock at 96. Find his
investment, in each if the income derived from both the investments be ? 975.
25.37
FINANCIAL MATHEMATICS-II

29. A man invests a part of his capital of ? 163000 in 4% stock at 108 and the rest in 5 /o stock at
109 - How much must he invest in each so that he may get equal incomes from the two
2
investments?

30. A person invested ? 26000, partly in 3% stock at 81 and partly in 4% at par. If the income
derived from each is the same, find his investment in each and his total income from both.
31. A man invested the same sum in two different stocks 3 Government security at 103 -
and 4% immicipal debentures at 105. If his income from one is 93 more than the other, find
the sum invested in each stock?
32. A person invests ? 7000 partly in 3% stock at 96 and partly in 4% stock at 120. If his total

ow
income be 3 —% on his investment, how much did he invest in each?
14

33, A man has ^ 9000 invested partly in 9% stock at ? 93 ^ and partly in a 13 stock at X102, in
such a manner he gets 12% on his money. How much has he invested in each stock ?

e
re
rFl
34, Divide X 5300 in two parts such that if one part is invested in 10 -% stock at X 98 and the

F
other in 12% stock at par, the resulting income in both the cases is equal.
35. What amount will earn X 60 more, by investing in 5% stock at 115 than in 4% stock at 95?

r
A man invested X 90000 in 3% at 90. He sold the stock when price rose to 95 and invested the
ou
36.
fo
sale proceeds in 4% stock. By doing so his income increased by X 800. At what price did he
ks
buy the latter stock?
7% stock a man increases his
37, By selling out 4% stock at 96 and investing the proceeds i m
oo

mcome
by 50%. What is the price of the latter stock?
Y

A man invested ? 7275 in 5% stock at 97. Find his annual income. He sold at par and
eB

38.

invested the proceeds in 3 |% stock. Find the price of this stock, if his income was not
r

altered.
ou

of X 17,000 produces an annual


ad
Y

39. What is the market value of 7% stock when an investment


income of X 1400?
40. Find the market value of 9% stock when an investment of X 4503 yields monthly income of
d
Re
in

X 22.50 (Brokerage —).

A person investing in the 8% stock receives 10% of his money. Find the price of the stock.
F

41.

42. A stock yields 5% to an investor, a fall of ^ 5 in its prices causes it to yield 5 -%. What was
the price?
43. A man invested X 7500 in 5% debentures at 123 and sold them out at 122. With the money
realised he bought 4 ^% console at 118. Find the change in his income. (Brokerage 2 to be
charged on each transaction). 3 3
44. How much stock must be transferred from 3 —% stock at 98 —
—A to the 4% stock at 100 —4 m

order to produce an increase in income of X 33? (Brokerage 1 —).


4.5. Out of a 4 -% stock at 50, a 7% stock at 75 and a 8% stock at 82, which is least profitable?
2
25.38 APPLIED MATHEMATICS-XII

4f). Which is the better investment: 2 -% stock at 53 - (brokerage 1 -) or 3% stock at 61 —


2 4 ^ 4 10
(brokerage 1 —)? Also find the difference in the annual income of a person who has
invested ? 6930 in each.

47. How much money has a person invested in 3 stock at 90 when his income is ? 210? If

latter he sells off his stock at 91 and invests the proceeds in 4 —% at 117, what is the
difference in his income?

48. How much money has a person invested in 10 ~% stock at 90 when his income is ? 630?

w
49. Which is more profitable investment, 5 stock at par or 5% stock at 10% discount? How

much is one investing in each case when difference of income is ? 45? (Brokerage —).

Flo
10

50. A man invests ? 7371 in 4% stock at 102 - . He sells the stock when quoted at 105 and

ee
8

re-invests the money realised in 5% at 126. Find the change in income and latter stock held.

Fr
ANSWERS

for
ur
1. (i) ?4275 (ii) ?9135 (iii) ? 1920 (iv) ?7676
(v) ? 1297.20 (vi) ? 24300 (vii) 115000
ks
2. ? 12100 4. ^1500 5. ? 1800 6. ^4500
Yo

7. ^2000 8. ^6000 9. ? 27720


oo

10. ? 18500
11. ?5495 12. ? 16780 13. ? 175 14. 2nd is better
eB

15. ? 187.50 16. ^10362.50,? 10,000 17. ?1836,n800


18. ^5300 20. 1125
r

21. (i) ? 135 (ii) ? 322.50


ou

(iii) ?424 (iv) ? 401.40


ad

22. n883.89 23. ^ 37.50 2’-. ?4095 25. ?2262


Y

26. 5% stock of ? 3300,6% stock of 12650 27. ? 4800


28. Investment in 3% stock = ^ 15000, Investment in 4% stock = ? 9000
nd
Re

29. ? 90,000, ? 73000


Fi

30. Investment in 3% stock = ? 13500, Investment in 4% stock = ? 12500, Income = 100


32. ? 4000, ? 3000 33. ? 3080, ? 5920 34. ? 2800, ? 2500 35. ? 43700
36. ?100 37. ? 112 38. ? 375, ? 70 39. ? 85
40. ?150 41. ?80 42. ? 80 43. ? 30 less
44. ?10200 45. 4 -% at 50 46. 2nd, ? 15
2 47. ? 5400, No change
48. ?5400 49. 2nd, I. ^ 495.55, II. ? 500.50 50. f 12 increase, ? 6000

MULTIPLE CHOICE QUESTIONS (MCQs)


Mark the correct altermtive in each of thefolhzving:
1. Rohan invested ? 4,455 in ? 10 shares quoted at ? 8.25. If the rate of dividend be 12%, his
annua] income is
(a) ?648 (b) ?668 (c) ? 655.60 (d) ? 534.60
25.39
FINANCIAL MATHEMATICS-''

Mr. X buys ? 50 shares in a company which pays 10% dividend. If he gets 12.5% on his
investment, at what price did he buy the shares?
(a) ? 52 (b) ?48 (c) ^40 (d) ?42
Raman invests ? 5,508 in 4% stock at 102. He afterward sells out at 105 and reinvest in 5%
stock at 126. The change in his income is
(a) (b) ?9 (c) ? 10 (d) ?20
At what price should I buy a share the value of which is f 100, paying a dividend of 8% so
that my yield is 11 %?
(a) ?84 (b) ^75 (c) ? 70 (d) ? 72.72
A person has deposited ? 13,200 in a bank which pays 14% interest. He withdraws the
money and invests in ? 100 stock at f 110 which pays dividend of 15%. How much does he
gain or lose?
(a) Loses ? 48 (b) Loses ? 312 (c) Gains ? 48 (d) Gains? 132
In order to obtain an income of ? 650 from 10% stock at ? 96, one must make an investment

w
of
(a) ? 3,100 (b) ? 6,240 (c) ? 6,500 (d) ? 9,600
Mr. X iiwested ? 913 partly in 4% stock at ? 97 and partly in 5% stock at ? 107. If his income

(a) ? 485,? 428 (b) ? 475,438

F lo
from both is equal, the amount of his investment in each stock is
(c) ? 495, ? 418 (d) ? 505,? 408
Isha wants to secure an annual income of ? 1,500 by investing in 15% debentures of face

ee
H
brokerage is 1%, then sum of money he

Fr
value ? 100 each and available for ? 104 each. If the
should invest is
(a) ? 10,540 (b) ? 10,784 (c) ? 15,000
for
(d) ? 19,642
9 A man invested ? 14,400 in ? 100 shares of a company at 20% premium. If the company
r
declares 5% dividend at the end of the year, then how much does he got?
You

(d) ?720
s

(a) ?500 (b) ?600 (c) ? 650


ook

10.
By investing ? 3,450 in a 4^% stock, a man obtains an income of ? 150. The market price of
eB

the stock is
(a) ?110 (b) ?105 (c) ? 103.50 (d) ? 107.50
our

A man buys ? 25 shares in a company which pays 9% dividend. The money invested is such
ad

li.

that it gives 10% on inveshnent. The price at which he bought the shares is
(a) ? 22.50 (b) ? 22 (c) ? 45 (d) ? 20.50
A person wants to invest 140,000 in two types of bonds. The annual return is 12% on bond A
dY

12.
Re

and 16% on bond B. One of the conditions requires that the investment in bond B cannot be
more than 40% of the investment in bond A. What is the maximum return he can get per
Fin

year?
(a) ? 15,600 (b) ? 16,800 (c) ? 19,200 (d) ? 20,800
ANSWERS

1. (a) 2. (c) .3. (b) 4. (d) 5. (a) 6. (b) (a) s. (a) 0, (b)
10. (c) 11. (a) 12. (b)
CHAPTER

LINEAR PROGRAMMING

26.1 INTRODUCTION
The term 'programming' means planning and it refers to a particular plan of action from
amongst several altenaatives for maximizing profit or minimizing cost etc. Programming
problems deal with determining optimal allocation of limited resources to meet the given

w
objectives, such as least cost, maximum profit, highest margin or least time, when resources
have alternative uses.

F lo
The term 'Linear' means that all inequations or equations used and the function to be
maximized or minimized are linear. That is why linear programmmg deals with that class of
problems for which all relations among the variables involved are linear.

ee
Formally, linear programming deals with the optimization (maximization or minimization) of a

Fr
linear function of a number of variables subject to a number of conditions on the variables, in the
for
form of linear inequations or equations in variables involved,
. j- problems
In this chapter, we shall discuss mathematical formulation of linear programming .
r
that arise in trade, industry, commerce and military operations. We shall also discuss sonae
You

elementary techniques to solve linear programming problems in two variables only.


s
ook

26.2 LINEAR PROGRAMMING PROBLEMS


eB

In this section, we shall discuss the general form of a linear programming problem. To give the
general description of a linear programming problem, let us consider the following problem.
viz. chairs and tables. Processing of these
our
ad

Suppose that a furniture dealer makes two products


products is done on two machines A and 6. A chair requires 2 hours on machine A and 6 hours
on machine B. A table requires 4 hours on machine A and 2 hours on machine B. There are 16
hours of time per day available on machine A and 20 hours on machine B. Profits gained by the
dY
Re

manufacturer from a chair and a table are ? 300 and ? 500 respectively. The manufacturer is
willing to know the daily product of each of the two products to maximize his profit.
Fin

The above data can be put in the following tabular form:


Item Chair Table Mnxhmim available time

2 hrs 4hrs 16hrs


Machine A

2 hrs 20 hrs
Machine B 6 hrs

?300 ?500
Profit (in ?)
To maximize his profit, suppose that the manufacturer produces chairs and y tables per day. It
is given that a chair requires 2 hours on machine A and a table requires 4 hours on machme A.
Hence, the total time taken by machine A to produce .x chairs and y tables is 2x
+ 4y. This must be
less than or equal to the total hours available on machine A. Hence, 2x + 4y < 16. Similarly, for
machine B, we have
6x + 2y < 20.
26.2
APPLIED MATHEMATICS-XII

The total profit for x chairs and y tables is 300:r + 500y. Since the number of chairs and tables ISi
never negative. Therefore, a: > 0 and y > 0.
Thus, we have to maximize
Profit = 300a:+ 500y
Subject to the constraints
2a: + 4y < 16
6x + 2y ^ 20
a:>0, y > 0
Out of all the points {x, y) in the solution set of the above linear constraints, the manufacturer
has to choose that point, or those points for which the profit 300a: + 500y has the maximum
value.

In the above discussion if a chair costs ? 1250 and a table costs ? 3000 then the total cost of
producing x chairs and y tables is 1250a: + 3000y. Now, the manufacturer will be interested to
choose that point, or those points, in the solution set of the above linear constraints for which the
cost 1250a: + 3000y has the minimum value.
The two situations discussed above

F low
give the description of a type of linear programming
problems. In the above discussion, the profit function = 300a: + 500y or the cost function
= 1250a: + 3000y is known as the objective function. The inequations 2x + 4y < 16, 6x + 2y ^ 20
are known as the constraints and a: > 0, y > 0 are known as the non-negativity restrictions.
The general mathematical description of a linear programming problem (LPP) is
Optimize2 = cj a:j + C2 ^2 +... + a^
for Fre given below:
(objectivefunction)
Subject to
«ll x^ + flj2 X2 +a^3X3+... + ayjXtj(<,=,>)
^21 ^1 + «22 ^2 + «23 ^3 + ● ● ● + «2« ^ ^2
(constraints)
Your
eBo ks

/ ^2, ,..., Xfj >0 (non-negativity restrictions)


ad

where all Ufj's, cfy's and Cj's are constants and xj's are variables.
our

The above linear programming problem may also be written in the matrix form as foUows :
xi
Re

^2
Optimize (maximize or minimize) Z = [cj C2 cy*]
Find Y

Subject to
flu fll2 ... flj;, Xl h
fl2i fl22 fl2tt X2 h

fml ^2 ●●● flwrt ,Xti br

Xl,X2,X^, , Xfj>0
OR

Optimize (Maximize or Minimize) Z =CX


Subject to AX (<, =, >) B
X>0 ,
26.3
LINEAR PROGRAMMING

^11 ‘’iH
■^2 ^21 ^22 ■ ●● ^2n
where C=[c‘i C2-. C,,], X = .A =

.amn
^n\

26.3 SOME DEFINITIONS


In this section, we shall formally define various terms used in a linear programming problem.
As discussed in the previous section, the general form of a linear programming problem is
Optimize (Maximize or Minimize) Z = c^ a:-] + C2 ^2 + + c„ -V,

Subject to

ow
(]|1 + (?i2 ■-'^2 + + .v„ (<,=,>
i?2i + fl22 ■-^2 +

Ojfii .V-] + a,„2 A'2 + ●●●● +

e
> 0
■■^1, ^2'

re
The definitions of various terms related to a LPP are as follows:

rFl arc variables, then the linear

F
OBJECTIVE FUNCTION //Jj, Cj q, arc constants and Tj-
function Z =c-^ x-\ + C2 XJ+ ■■■ + c„ X iL’hich is to be tnaximized or minimized is called the objective
function.

r
ou
The objective function describes the primary purpose of the formulation of a linear
fo
programming problem and it is always non-negative. In business applications, the profit
ks
function which is to be maximized or the cost function which is to be minimized is called the
objective function.
oo

CONSTRAINTS The inequations or equations in the variables of a LPP which describe the conditions
Y

under which the optimisation (maximization or minimization) is to be accomplished are called


B

constraints.
re

In the constraints given in the general form of a LPP there may be any one of the three signs
ou
Y
ad

Inequations in the form of greater than (or less than) indicate that the total use of the resources
must be more than (or less than) the specified amount whereas equations in the constraints
indicate that the resources described are to be fully used.
d

NON-NEGATIVITY RESTRICATIONS These are the constraints which describe that the variables
in
Re

involved in a LPP are non-negative.


F

26.4 MATHEMA .. .. ‘ OF LINEAR PROGRAMMING PROBLEMS


In the previous section, we have introduced the general form of a linear programming problem
(LPP). In this section, we shall discuss the formulation of linear programming problems.
Problem formulation is the process of transforming the verbal description of a decision problem
into a mathematical form. There is not any set procedure to formulate linear programirung

problems. In fact, one can only learn the formulation with adequate practice. However, the
following algorithm will be helpful in the formulation of linear programming problems.
ALGORITHM

In every LPP certain decisions are to be made. These decisions are represented by decision
variables. These decision variables are those quantities whose values are to be determined.
Identify the variables and denote them by x^^ .V2, A3,...
Identify the objective function and express it as a linear function of the variables introduced in
step I.
26.4
APPLIED MATHEMATICS-XII

STliP 111
In a LPP, the objective function )nai/ be in the form of maximizing profits or mmimizing costs.
So, after expressing the objective fimction as a linear function of the decision variables, we must
find the type of optimization i.e. maximization or minimization. Identify the type of the objective
function.
STHI> IV
Identify the set of constraints, stated in terms of decision variables and express them as linear
inequations or equations as the case may be..
The following examples will illustrate the formulation of linear
programming problems in
various situations.

ILLUSTRATIVE EXAMPLES

Type I OPTIMAL PRODUCT LINE PROBLEMS


EXAMPLE
A factory produces two products Pj and P2. Each of the product Pj requires 2 hrs for
1

moulding, 3 hrs for grinding and 4 hrs for polishing, and each of the product P2 requires 4 hrs for

w
moulding, 2 hrs for grinding and 2 hrs for polishing. The factory has moulding machine availablefor
20 hrs, grinding machine for 24 hrs and polishing machine availablefor 13 hrs. The profit is^5 per unit of

F lo
P| and ? 3 per unit of P2 and the factory can sell all that it produces. For7nulate the problem as a linear
programing problem to maximize the profit.

ee
SOLUTION The given data may be put in the following tabular form:

Fr
Product
Resources
Pi Pi for Capacity
ur
Moulding 2 4 20

Grinding 3
s
2 24
ook
Yo

Polishing 4 2 13
eB

Profit in ? 5 3

Suppose x units of product P^ and y units of product P2 are produced to maximize the profit. Let
r

Z denote the total profit.


ad
ou

Since each unit of product P^ requires 2 hrs for moulding and each unit of product P2 requires
4 hrs for moulding. Hence, the total hours required for moulding for :r units of product Pj and
Y

y units of product P2 are 2jc + 4y. This must be less than or equal to the total hours available for
Re

moulding. Hence,
nd

2x + 4y< 20
Fi

This is the first constraint.

The total hours required for grinding for a: units of product Pj and y units of product P2 is
3.r + 2y. But, the maximum number of hours available for grinding is 24.
3a: -f 2y < 24
This is the second constraint.

Similarly, for polishing the constraint is 4a: + 2y < 13.


Since x and 1/ are non-negative integers, therefore .y > 0, y > 0.
The total profit for y units of product P] and y unitsofproductP2 isSY + 3y. Since we wish
to maximize the profit, therefore the objective function is
Maximize Z =5Y+3y
Hence, the linear programming problem for the given problem is as follows:
Maximize Z =5y + 3y
26.5
LINEAR PROGRAMMING

Subject to the constraints


2x + Ay< 20
3.t + 2i/ < 24
4x + 2i/<13
and, s:>0, y£0
_
EXAMPLE 2 _- toy company manufactures two types of doll; a basic version doll A and a deluxe version
doll B. Each doll of type B takes twice as long to produce as one of type A, and the company ivould have time
to make a maximum of 2,000 per day if it produces only the basic version. The supply of plastic is sufficient
fo produce 1500 dolls per day (both A and B combined). The deluxe version requires afancy dress of which
there are only 600 per day available. If the company makes proift of^ 3 and 15 per doll respectively on doll
A and doll B; how many of each should be produced per day in order to maximize profit?

ow
SOLUTION Let ;c dolls of type A and y dolls of type B be produced per day. Then,
Total profit = 3a: + 5y.
Since each doll of type B takes twice as long to produce as one of type A, therefore total time
taken to produce x dolls of type A and y dolls of type B is a: + 2y. But, the company has time to
make a maximum of 2000 doUs per day

e
re
a: + 2i/<2000

Fl
Since plastic is available to produce 1500 dolls only.

F
x + y < 1500
ur
Also fancy dress is available for 600 dolls per day only

r
y < 600 fo
ks
Since the number of dolls cannot be negative. Therefore,
Yo
x>0, i/>0
oo

Hence, the linear programming problem for the given problem is as follows:
Maximize Z = 3x + 5y
eB

Subject to the constraints


x + 2y < 2000
ur

x + y < 1500
ad
Yo

y < 600
and, a: > 0, 1/ > 0
EXAMPLE 3 A firm can produce three types of cloth, sayC-^,C2>C^. Three kinds of wool are required for
d

it, say red wool, green wool and blue wool. One unit of length C-] needs 2 metres of red wool, 3 metres of
Re
in

blue wool; one unit of cloth C2 needs 3 metres of red wool, 2 metres ofgreen wool and 2 metres of blue wool;
and one unit of c/of/iC 3 needs 5 metres of green wool and 4 metres of blue wool. Thefirm has only a stock of
F

26 metres of red wool, 20 metres of green wool and 30 metres of blue zuool. It is assumed that the income
obtained from one unit of length of cloth Cj is ^ 6, of cloth C2 is ^ 20 and of clothC^ is ? 8. Formulate the
problem as a linear programming problem to maximize the income.
SOLUTION The given information can be put in the following tabular form:
Total quality
Cloth Cl Cloth C2 Cloth C3
of wool available
2 3 0 16
Red Wool
0 2 5 20
Green Wool

Blue Wool 3 2 4 30

Income (in ?) 6 10 8
26.6
APPLIED MATHEMATICS-XII

Let Xi,X2 and.t3 the quantity produced in metres of the cloth of type CuC^ andCo
respectively.
Since 2 metres of red wool are required for one metre of cloth Cj and Xj metres of cloth Cj are
produced, therefore 2.Vj metres of red wool will be required for cloth C|. Similarly, cloth C2
requires 3x2 i^etres of red wool and clothC3 does not require red wool. Thus, the total quantity
of red wool required is 2.V| + 3a:2 + OX3.
But, the maximum available quantity of red wool is 16 metres.
2.y^ 4- 3.Y2 + OY3 <16
Similarly, the total quantities of green and blue wool required are
0Y| + 2.T2 + 5a*3 and 3.t| + 2.t2 + 4^3 respectively.
But, the total quantities of green and blue wool available are 20 metres and 30 metres
respectively.

w
O.vi + 2.V2 + 5.Y3 < 20 and 3.Vj + 2.t2 + 4.Y3 < 30
Also, we cannot produce negative quantities, therefore

Flo
.Yj > 0, A'2 > 0, .Y3 > 0
The total income is Z = 6.y^ +10^2 + 8-V3

ee
Hence, the linear programming problem for the given problem is

Fr
Maximize Z = b.Y^ + 10^2 + 8.Y3
Subject to the constraints
2.Y| +■ 3.Y2 + O-Y3 <16
for
ur
O.Y1 + 2,Y2 + 5.Y3 < 20
3.y-| + 2a'2 + 4.Y3 < 30
ks
and, x-i > 0, .Y2 > 0, .Y3 > 0
Yo
oo

FXAMI’U; 4
A furniture firm manufactures chairs and tables, each requiring the use of three machines
A, B and C. Production of one chair requires 2 hours on machine A, 1 hour on machine B, and 1 hour on
B

machine C. Each table requires 1 hour each on machines A and B and 3 hours on machine C. The profit
re

realized bif selling one chair is ? 30 while for a table the figure is ? 60. The total time available per week on
machine A is 70 hours, on machine B is 40 hours, and on machine C is 90 hours. How many chairs and
ou
ad

tables should be made per week so as to maximize profit? Develop a mathematical formulation.
Y

SOLUTION The given data may be put in the following tabular form:
Available time per
nd

Machine Chair Table


Re

week (in hours)


Fi

A 2 1 70
B 1 1 40
C 1 3 30

Profit per unit ?30 ?60

Let Y chairs and y tables be produced per week to maximize the profit. Then, the total profit for y
chairs and y tables is 30y + 60y.
It is given that a chair requires 2 hours on machine A and a table requires 1 hour on machine A.
Therefore, the total time taken by machine A to produce y chairs and y tables is (2y + y) hours.
This must be less than or equal to total hours available on machine A.
2x+y < 70
Similarly, the total time taken by machine B to produce Y chairs and y tables is (y + y) hours. But,
the total time available per week on machine B is 40 hours.
Y + y < 40
26.7
LINEAR PROGRAMMING

Finally, the total time taken by machineC to produce :c chairs and y tables is a: + 3y hours and the
total time available per week on machine C is 90 hours.
.V + 3y < 90
Since the number of chairs and tables cannot be negative.
a: > 0 and y ^ 0
Let Z denote the total profit. Then,
Z = 30.Y + 60y
Hence, the mathematical form of the given LPP is as follows:
Maximize Z = 30.y + 60y

ow
Subject to
2x + ij <70
x +y < 40
Y + 3y < 90

e
and, Y > 0, y >0

re
A manufacturer of a line of patent medicines is preparing a production plan on mcdicmes

rFl
EXAMPLE '

A and B. There are sujficient ingredients available to make 20,000 bottles of A and 40,000 bottles of B but

F
there are only 45,000 bottles into which either of the medicines can be put. Further more, it takes 3 hours to
prepare enough material to fill 1000 bottles of A, it takes one hour to prepare enough material to fill 1000

r
bottles of B and there are 66 hours available for this operation. The profit is ? 8 per bottle for A and ? 7 per
ou
fo
bottle for B. Formulate this problem as a linear programming problem.
ks
SOLUTION Suppose the manufacturer produces y bottles of medicines A and y bottles of
medicine B.
oo

Since the profit is ? 8 per bottle for A and ? 7 per bottle for B. So, total profit in producing y
Y

bottles of medicine A and y bottles of medicine Bisf (8y + 7y).


B

Let Z denote the total profit. Then,


re

Z = 8y+ 7y
ou

Since 1000 bottles of medicine A are prepared in 3 hours.


Y
ad

3y
hours.
Time required to prepare Y bottles of medicine/I = 1000
d

It is given that 1000 bottles of medicine B are prepared in 1 hour.


in
Re

y hours.
Time required to prepare y bottles of medicine B = 1000
F

Thus, total time required to prepare y bottles of medicine A and y bottles of medicine 6 is
3y y
+ hours. But, the total time available for this operation is 66 hours.
1000 1000
3y y
+ < 66
1000 1000

=> 3y + y < 66,000


Since there are only 45,000 bottles into which the medicines can be put.
Y + y < 45,000
It is given that the ingredients are available for 20,000 bottles of A and 40,000 bottles of B.
X < 20,000 and y < 40,000
Since the number of bottles can not be negative. Therefore, y > 0, y > 0.
Hence, the mathematical formulation of the given LPP is as follows;
Maximize Z = Sx+7y
26.8
APPLIED MATHEMATICS-XII

Subject to
3x + ij < 66,000
x + i/ < 45,000
X < 20,000
1/ < 40,000
and. x>0,y > 0.
EXAMPLE 6 A resourceful home decorator manufactures two types of lamps say /I and B. Both lamps
go through two technicians, first a cutter, second a finisher, lamp A requires 2 hours of the cutter's time

ow
and 1 hour of the finisher's time. Lamp B requires 1 hour of cutter's and 2 hours offinisher's time. The
cutter has 104 hours and finisher has 76 hours of time available each month. Profit on one lamp A is
^ 6.00 atid on one lamp B is f 11.00. Assuming that he can sell all that he produces, hozv many of each type
ofla7}ips should he manufacture to obtain the best return.
SOLUTION The above information can be put in the following tabular form:

e
re
Lamp Cutter's tit?ie Finisher's time Profit in ?

Flr
A 2 1 6

F
B 1 2 11

Maximum time available


ou104 76

sr
Let the decorator manufacture x lamps of type A and y lamps of type B.
Total profit = ? (6x + lly)
fo
k
Total time taken by the cutter in preparing x lamps of type A and y lamps of type B is (2x + y)
oo
hours. But, the cutter has 104 hours only for each month.
Y
2x + y < 104
reB

Similarly, the total time taken by the finisher in preparing x lamps of type A and y lamps of type
B is (x + 2y) hours. But, the cutter has 76 hours only for each month.
uY

X + 2y < 76
Since the number of lamps cannot be negative.
X > 0 and y > 0
ad
do

Let Z denote the total profit. Then, Z = 6x + lly.


Since the profit is to be maximized. So, the mathematical formulation of the eiven LPP is as
in

follows:
Re

Maximize Z = 6x + lly
F

Subject to
2x + y < 104
X + 2y <76
and. X > 0 , y > 0
EXAMPLE 7 A cotnpany makes two kinds of leather belts, A and B. Belt A is high quality belt, and B is of
lower quality. The respective profits are ? 40 and ? 30 per belt. Each belt of type A requires twice as much
time as a belt of type B, and if all belts were of type B, the company could make 1000 belts per day. The
supply of leather is sufficient for only 800 belts per day (both A and B combmed). Belt A requires a fancy
buckle, and only 400 buckles per day are available. There are only 700 buckles available for belt B. What
should be the daily production of each type of belt? Formulate the problem as a LPP.
SOLUTION Suppose the company makes per day x belts of type A and y belts of type B.
Profit = 40x + 30y.
Let Z denote the profit. Then, Z = 40x + 30y and it is to be maximized.
It is given that 1000 belts of type B can be made per day and each belt of type A requires twice as
much time as a belt of type B. So, 500 belts of type A can be made in a day.
26.9
LINEAR PROGRAMMING

I ^ 1 . But the
preparingA'belts of typev4andi/beltsoftype6is —^
So, total time taken in

company is making x belts of type A and y belts of type B in a day.


X y
+ < 1 => 2.V + y < 1000
500 1000

Since the supply of leather is sufficient for only 800 belts per day.
X + i/ < 800
It is given that only 400 fancy buckles for type A and 700 buckles for type B are available per
day.
a: < 400, y < 700

ow
Finally, the number of belts cannot be negative.
a: > 0 and y ^ 0
Thus, the mathematical formulation of the given LPP is as follows:
Maximize Z = 40a' + 30y

e
Subject to

re
Ix + y < 1000
A + y < 800

rFl
F
a: < 400
y < 700

r
.t > 0, y > 0.
ou
and.

Txmell DIET PROBLEMS


sfo , ,
EXAMPLE 8 A dietician whishes to mix tzvo types offood in such a way that the vitamin contents of the
k
. cu

mixture contain at least 8 units of Vitamin A and 10 units of vitamin C. Food T contains 2 units per kg of
oo

vitamin A and 1 unit per kg of vitamin C while food 'IF contains 1 unit per kg of vitamin A and! units
per kg of vitamin C. It costs ^ 50.00 per kg to purchase food '!' and ? 70.00 per kg to produce food U.
Y
B

Formulate the above linear programming problem to minimize the cost of such a mixture.
The gives data may be put in the following tabular form:
re

SOLUTION

Food Requirements
ou

Resources
Y
ad

I II

1 8
2
d

Vitamin A
Vitamin C 2 10
in

1
Re

Cost (in ?) 50 70
F

Let the dietician mix a; kg of food and y kg of food Tl'. Clearly, a; > 0, y > 0.
Since one kg of food 7' costs ? 50 and one kg of food 'II' costs ? 70. Therefore, total cost of .v kg of
food 'r and y kg of food 'IT is ? (50.t + 70y).
Let Z denote the total cost. Then,
Z = 5a: + 7y
Since one kc of food T contains 2 units of vitamin A. Therefore, A' kg of food'T con tarn 2Aunits of
vitamin A. One kg of food 'II' contains one unit of vitamin A. So, y kg of food 'IT contams y units
of vitamin A. Thus, x kg of food 'T and y kg of food 'IF contain 2.t + y units of vitamm A. But, the
minimum requirement of vitamin A is 8 imits.
2x + y > 8
Similarly, ottal amount of vitamin C supplied by x units of food 'I' and y units of food 'II' is
(x + 2y) units and the minimum requirement of vitamin C is 10 units.
X + 2y >10
26.10
APPLIED MATHEMATfCS-XI!

Hence, the mathematical model of the LPP is as follows:


Minimize Z = 5x + 7\j
Subject to
2x + y > 8
x+2\j > 10
and. X, 1/ > 0.

; I A diet is to conloin at least 400 units of carbohydrate, 500 units offat, and 300 units of
protein. Two foods are available: Fy which costs ? 2 per unit, and F2, which costs ? 4 per unit. A unit of
food Fi contains 10 units of carbohydrate, 20 units offat, and 15 units of protein; a unit offood F. contains
25 units of carbohydrate, 10 units offat, and 20 units of protein. Find the minimum cost for A diet that
consists of A mixture of these two foods and also meets the minimum nutrition requirements. Formulate
the problem as A linear programming problem.
SOLUTION The given data may be put in the following tabular form:
Food

w
Carbohydrate Fat Protein Cost per unit
F1 10 20

F lo
15 ?2
^2 25 10 20 ?4

Minimum requirement 400 500 300

e
Fre
Suppose the diet contains x units of food F■^ and y units of food F2.
Since one unit of food -F^ costs ^ 2 and one unit of food F2 costs ? 4. Therefore, total cost ofx units
for
of food fj and y units of food F2 is f (2x + 4y).
Let Z denote the total cost. Then, Z = 2x + 4y.
r
You

Smce each umt of food F■^ contains 10 units of carbohydrate. Therefore, x units of food -F] contain
oks

10 X units of carbohydrate. A unit of food F2 contains 25 units of carbohydrate. So, y units of


eBo

food F2 contain 25 y units of carbohydrate.


Thus, X units of food and y units of food F2 contain lOx + 25i/ units of carbohydrate. But, the
minimum requirement of carbohydrate is 400 units.
ad
our

lOx + 25y > 400


Similarly, the total amount of fat supplied afy x units of Food F^ and 1/ units of food A is
20x + lOy and the minimum requirement is of 500 units.
Re
dY

20x + lOi/ > 500


Fin

Finally, the total amount of protein supplied by x units of food and y units of food F2 is
15x + 20y. But, the minimum requirement of protein is of 300 units.
15x + 20y > 300
Clearly, x>0 and y>0.
Since we have to minimize the total cost Z = 2x + 4y.
Thus, the mathematical form of the given LPP is as follows:
Minimize Z = 2x + 4y
Subject to
lOx + 25y > 400
20x +lOy > 500
15x + 20y > 300
X, y > 0.
26.11
LINEAR PROGRAMMING

EXAMPLE 10 The objective of A diet problem is to ascertnin the quantities of certain foods that should be
eaten to meet certain nutritional requirement at minimum cost. The consideration is hunted to milk, beaf
and eggs, and to vitamins A. B, C. The number of milligrams ofeach of these vitamins contained within vA
unit of each food is given below:
Minimum daily
Vitamin Litre of milk Kg of beaf Dozen of eggs requirements

A 1 1 10 1 mg
B 100 10 10 50 mg
c 10 100 10 10 mg

ow
?1.00 ?1.10 ?0.50
Cost

What is the linear programming formulation for this problem?


SOLUTION Let the daily diet consists of x litres of milk, y kgs of beaf and z dozens of eggs. Then,
Total cost per day = ? (.v + l.lOi/ + 050z).

e
Let Z denote the total cost in Then, Z = .v + l.lOi/ + 050z

re
Total amount of vitamin A in the daily diet is (x + y + lOz) mg

rFl
F
But, the minimum requirement is 1 mg of vitamin A.
X + y + lOz >1
Similarly, total amounts of vitamins B andC in the daily diet are (lOOx + lOy + lOz) mg and
or
ou
(lOx + lOOy + lOz) mg respectively and their minimum requirements are of 50 mg and 10 mg
ksf
respectively.
’ lOOx + lOy + lOz > 50 and, lOx + lOOy + lOz > 10
Finally, the quantity of milk, kgs of beaf and dozens of eggs cannot assume negative values.
oo

X > 0, y > 0, 2 > 0


Y
B

Hence, the mathematical formulation of the given LPP is


Minimize Z = x + l.lOy + 050z
re

Subject to
oYu

X + y + lOz > 1
ad

lOOx + lOy+ 10z > 50


lOx L lOOy + lOz > 10
d

and. X > 0, y > 0, z > 0.


in
Re

Type III TRANSPORTATION PROBLEMS


Tl^e transportation problem is a special type of linear programming problem (LPP) where the
F

objective is to minimize transportation cost of a given commodity from a number of sources


or

origins (e.g. factory, manufacturing facility etc) to a number of sources or origins (e.g.
warehouse, store etc).
EXAMPLE 11 There is a factonj located at each of the tzuo places P and Q. From these locations, a certain
commodity is delivered to each of the three depots situated at A, B and C. The weekly requirements of the
depots are respectively 5,5 and 4 units of the commodity lohile the production capacity of the factories at I
and Q are 8 and 6 units respectively. The cost of transportation per unit is given below. ^ ^
To
Cost (in
From
>1 B C

16 10 15
P
12 10
10
Q
26.12
APPLIED MATHEMATICS-XII

How many units should be transported from each factory to each depot in order that the transportation
cost is minimum. Formulate the above as a linear programming problem.
SOLUTION The given information can be exhibited diagrammatically as follows:

Depot A
5 units

ow
%

?10
Factory P Depot B ?12

e
8 units
Factory Q
y units 5 units
(5-y) units I 6 units

re
Frl
F
cP
ou
S'

r
'<S>

Depot C
so
4 units
kf
oo
Y

Fig. 26.1
B

Let the factory at P transports x units of commodity to depot at A and y units to depot at B. Since
the factory at P has the capacity of 8 units of the commodity. Therefore, the left out (8 - x - y)
re

units will be transported to depot at C.


oY
u

Since the requirements are always non-negative quantities. Therefore,


ad

X>0,y>0 and 8-x-y>0 ^ x>0,y>0 and x + y<8


d

Since the weekly requirement of the depot at >1 is 5 units of the commodity and x units are
in

transported from the factory at P. Therefore, the remaining (5 -x) units are to be transported
Re

from the factory atQ. Similarly, 5 - y units of the commodity will be transported from the factory
F

atQ to the depot at B. But the factory atQ has the capacity of 6 units only, therefore the remaining
6-(5-x + 5-y)=x + y- 4 units will be transported to the depot at C. As the requirements at
the depots at A, B and C are always non-negative.
5-x>0,5-y>0 and x + y-4>0=> x<5,y<5 and x + y>4.
The transportation cost from the factory at Pto the depots at A, B and Care respectively ?16x,
lOy and 15 (8 - X - y). Similarly, the transportation cost from the factory at Q to the depots at
A,BandC are respectively ? 10(5-x), 12(5-y) and 10(x + y-4). Therefore, the total
transportation cost Z is given by
Z = 16x + 10y + 15(8-x-y)+10(5-x) + 12(5-y) + 10(x + y-4) = x-7y + 190
Hence, the above LPP can be stated mathematically as follows:
Find X and y which
Minimize Z = x-7y +190
LINEAR PROGRAMMING 26.13

Subject to
x+i/ < 8
X + If > A
X < 5

y ^5
and. X > 0,y > 0
Suppose the factory at P transports .t units of conmiodity to depot at A and y units to
depot at B. The given information can be exhibited in the following tabular form. The
transportation cost per unitof commodity from factories P and Q to depots A, B, C are shown in
the top right corner of each cell of the table. Factor P has capacity of 8 units out of which x units
are
supplied to depot at A and y units to depot at B. Remaining 8 - (x + y) units are supplied to
depot at C. Depot at A requires 5 units. It receives x units from factory P and the remaining 5 - x
units from factory Q. Similarly, the requirement of 5 units of depot at B is fulfilled by y units from

w
factory P and remaining (5 - y) units from factory Q.

F lo
Depots
A B C Productions

15

ee
16 10

Fr
P 8

tn
X y
for
8-(x+y)
●c
r
Q
(j
10 12 10
You
Q
U.
s

6-{5-.r+5-y)=x+i/-4
ook

Q 5-x 5-x or 6
4-|8-(x+y)}=x+y-4
eB

Requirements 5 5 4
our
ad

The requirements of depots at A,B and C are non-negative.


X > 0, y > 0, 8 - (x + y) > 0,5 - X > 0,5 - y > 0, and x + y - 4 > 0
x>0, y>0, x + y<8, X <5, y <5 and x + y > 4.
dY
Re

The transportation costs per unit from factories P and Q to depots at A, B and C are show in the
upper right corner of each cell of the above table. Therefore, the total transportation cost Z is
Fin

given by
Z =16x + lOy +15 {8 - (x + y) 1 + 10 (5 - x) + 12 (5 - y) +10 (X + y - 4) = x -7y +190.
Hence, the mathematical form of the above LPP is
Minimize Z = x -7y +190
Subject to
x + y <8
x + Oy <5
Ox + y <5
x +y > 4
and. x>0,y >0
A brick manufacturer has two depots, A and B, with stocks of 30,000 and 20,000 bricks
respectively. He receives orders from three builders P, Q and Rfor 15,000, 20,000 and 15,000 bricks
respectively. The cost in ^ of transporting 1000 bricks to the builders from the depots are given below:
26.14 APPLIED MATHEMATICS-XII

To p Q R
From

A 40 20 30
B 20 60 40

How should the manufacturer fulfil the orders so as to keep the cost of transportation minimum?
Formulate the above linear programming problem.
SOLUTION The given information can be exhibited diagrammatically as shown in Fig. 26.2.

Builder P
15,000

low ee
y Builder Q (20-y)
Depot A

F Depot B

Fr
30,000 ?20 20,000 ?60 20,000

for
ur
.V X

■i<y^
ks
Yo
oo

Builder R
eB

15,000
r

Fig. 26.2
ou
ad

Let the depot A transport x thousands bricks to builders P, y thousands to builder Q. Since the
depot A has stock of 30,000 bricks. Tlierefore, the remaining bricks i.e. 30 - (x + y) thousands
Y

bricks will be transported to the builder R.


Since the number of bricks is always a non-negative real nurnber. Therefore,
Re
nd

X > 0, y > 0 and 30 - (x + y) > 0 => x > 0, y > 0 and x + y < 30


Fi

Now, the requirement of the builder P is of 15000 bricks and x thousand bricks are transported
from the depot A. Therefore, the remaining (15 - x) tliousands bricks are to be transported from
the depot at B. The requirement of the builder Q is of 20,000 bricks and y thousand bricks are
transported from depot A. Therefore, the remaining (20-y) thousand bricks are to be
transported from depot B.
Now, depot B has 20 - (15 - x + 20 - y) = .\ + y -15 thousand bricks which are to be transported
to the builder R.
Also, 15 - X > 0, 20 - y > 0 and x + y -15 > 0 ^ x < 15, y < 20 and x + y >15
The transportation cost from the depot A to the builders P, Q and R are respectively ? 40x, 20y
and 30 (30 - X -y). Similarly, the transportation cost from the depot B to the builders P, Q and R
are respectively ^ 20 (15 - x), 60 (20 - y) and 40 (x + y -15) respectively. Therefore, the total
transportation cost Z is given by
Z = 40x + 20y+ 30(30-x-y) + 20(15-x) + 60(20-y) + 40(x + y-15)
Z = 30x-30y + 1800
LINEAR PROGRAMMING 26.15

Hence, the above LPP can be stated mathematically as follows:


Find X and y in thousands which
Minimize Z = 30.v - 30y + 1800
Subject to
x + y < 30
X < 15

y < 20
x + y > 15
and, X > 0, y > 0
ALITER Let the depot at A transport x thousands bricks to builder P and y thousands to builder
Q. The number of bricks transported in thousands to various builders by depots at A and B are
shown in the cells of the following table. In the upper right corner of each cell of the given table
the cost in ? of transporting 1000 bricks to builders from the depots are givei :n the following

w
table.
Builders

F lo
Available
P Q R
stocks
40 20 30

ee
30,000

Fr
/\
.V y 30-(.v+y)
Q
a.
20 60
for 40
Q
ur
20-(15-.Y+20-y)=.v+y-l ?
B or 20,000
s
ook

15-{30-(.r+y)l=.v+j/-15
Yo

15-.V 20-1/
eB

Requirements 15,000 20,000 15,000


our
ad

Since the number of bricks is always a non-negative real number. Therefore,


X > 0, y > 0, 30 - (X + y) > 0,15 - x > 0, 20 - y > 0, x + y -15 > 0
=> X + y < 30, X < 15, y < 20, X + y > 15, X > 0, y > 0
dY
Re

The total transportation cost Z is given by


Z = 40x + 20y + 30 |30 - (x + y)l + 20 (15 - x) + 60 (20 - y) + 40 (x + y -5)
Fin

Z = 30 X - 30y +1800
Hence, the above LPP can be stated mathematically as follows:
Minimize Z = 30x - 30y +1800
Subjected to
x + y < 30
x + Oy < 15
Ox+y < 20
x + y > 15
and, X > 0, y > 0
Type IV ASSIGNMENT PROBLEMS
Assignment problem is also a type of linear programming problem in which the objective is to
assign a number of jobs to an equal number of machines at a minimum cost (or maximum
profit). Each job is assigned to one and only one machine and each machine performs exactly
one job.
26.16 APPLIED MATHEMATICS-XII

EXAMm; 1:^ A company loishes to assign 3 jobs Ji, /2 and to 3 machines Mi, M2 and M^ in such
away that each job is assigned to some machine and no machine works on more than one job. Thefollowing
table depicts the cost ofasigming jobs to machines.
Machines
M1 M2 ^3
/l 8 7 6

jobs h 5 7 8

J3 6 8 7

Formulate this as a linear programming problem.


SOLUTION The following is the network formulation of the given problem:
Machine

w
F lo
ee
Fr
Let Xjj denote the assignment of job to j^ machine, such thatfor
1, if job /j is assigned to machine Mj
r
0, otherwise
You
s

That is, if job is assigned to machine Mj.Then,


ook

Xii = 1 and X12 = -Xi3 = 0 as jobs /2 and ) 3 cannot be assigned to machine M j. Also, machine
eB

cannot perform any other jobs.


^21 ='■^31=^
our
ad

Similarly, if job h is assigned to machine M3. Then,


Xi3 = 1 and | = X12 = 0. Also, X23 = .1:33 = 0.
and so on.
dY

Thus, we obtain
Re

Xii4-.ti2 +^13 =1 X11+.Y21 +X31 =1


Fin

^21 + ^22 + ^23 = 1 X12 +X22 + X32 =1


^31+-‘'^32 + -^33 =1 ■■'^13+ ^'23 +^33 =1
The cost of assignment is
2 = 8X|-{ + 7 .Y^2 ^■^21 ^^22 ® ^23 ^ '^31 ^ ^32 ^ ^33
Clearly, A;^y = 0, or 1 for / = 1, 2, 3 and ;=1,2,3
Hence, the linear programming problem is
Min Z = 8xjj + 7X12 + 6.V13 + 5x2^ + 7X22 + 8 X23 + 6 x^i + 8 X32 + 7 X33
Subject to the constraints
'■'^11+^12 + ^13 =1 ^11+^21 + ^31 =1
^21 ^22 + ^23 = ^ ^'12 + ^22 + ^32 =1
^31 ■*■■■'^32 + ^33 ^13+ ■-'^23 +^33 =1
Xjj =0 or 1 for / = 1, 2, 3 and j =1, 2, 3
LINEAR PROGRAMMING 26.17

EXERCISE 26.1

1. A small manufacturing firm produces two types of gadgets A and B, which are first
processed in the foundry, then sent to the machine shop for finishing. The number of
man -hours of labour required in each shop for the production of each unit of A and B, and

the number of man-hours the firm has available per week are as follows:

Gadget Foundry Machine-shop


A 10 5

B 6 4

1000 600
Firm's capacity per week

The profit on the sale of A is ? 30 per unit as compared with ? 20 per unit of B. The problem
is to determine the weekly production of gadgets A and B, so that the total profit is

w
maximized. Formulate this problem as a LPP.
2. A company is making two products A and B. The cost of producing one unit of products

F lo
A and B are ? 60 and ? 80 respectively. As per the agreement, the company has to supply at
least 200 units of product B to its regular customers. One unit of product A requires one
machine hour whereas product B has machine hours available abundantly within the

ee
company. Total machine hours available for product A are 400 hours. One unit of each

Fr
product A and B requires one labour hour each and total of 500 labour hours are available.
for
The company wants to minimize the cost of production by satisfying the given
requirements. Formulate the problem as a LPP.
ur
3. A firm manufactures 3 products A, B andC.Theprofitsare^'S,^ 2 and ^4 respectively. The
firm has 2 machines and below is the required processing time in minutes for each machine
oks
Yo

on each product:
o
eB

Machine Products

A B C
our
ad

4 3 5
Ml
2 2 4
M2
Y

Machines Mj and M2 have 2000 and 2500 machine minutes respectively. The firm must
Re
nd

manufacture 100 A's, 200 B's and 50 C's but not more than 150 A's. Set up a LPP to maximize
Fi

the profit.
4. A firm manufactures two types of products A and B and sells them at a profit of? 2 on
type A and ? 3 on type B. Each product is processed on two machines M-j and M2. Type A
requires one minute of processing time on Mj and two minutes of M2 type B requires one
minute on Mj and one minute on M2- The machine M^ is available for not more than 6
hours 40 minutes while machine M2 is available for 10 hours during any working day.
Formulate the problem as a LPP.
5. A rubber company is engaged in producing three types of tyres A, B andC. Each type
requires processing in two plants. Plant I and Plant II. The capacities of the two plants, in
number of tyres per day, are as follows:
Plant A B C

I 50 100 100

II 60 60 200
26.18
APPLIED MATHEMATICS~XII

The monthly demand for tyre A, B and C is 2500,3000 and 7000 respectively. If plant I costs
? 2500 per day, and plant II costs ? 3500 per day to operate, how many days should each be
nm per month to minimize cost while meeting the demand? Formulate the problem as LPP.
6. A company sells two different products A and B. The two products are produced in a
common production process and are sold in two different markets. The production process
f has a total capacity of 45000 man-hours. It takes 5 hours to produce a unit of A and 3 hours
to produce a unit of B. The market has been surveyed and company officials feel that the
maximum number of units of A that can be sold is 7000 and that of B is 10,000. If the profit is

ow
? 60 per unit for the product A and ? 40 per unit for the product B, how many units of each
product should be sold to maximize profit? Formulate the problem as LPP.
7. To maintain his health a person must fulfil certain minimum daily requirements for several
kinds of nutrients. Assuming that there are only three kinds of nutrients—calcium, protein
and calories and the person's diet consists of only two food items, I and II, whose price and

e
nutrient contents are shown in the table below:

re
Food I Food 11
Minimum daily requirement

F
Frl
(per lb) (per lb) for the nutrient
Calcium 10
ou 5 20

osr
Protein 5 4 20

Calories 2 6 13

Price (?) 60 100


kf
oo
What combination of two food items will satisfy the daily requirement and entail the least
Y

cost? Formulate this as a LPP.


B

8. A manufacturer can produce two products, A and B, during a given time period. Each of
these products requires four different manufacturing operations: grinding, turning,
re
uY

assembling and testing. The manufacturing requirements in hours per unit of products A
and B are given below.
ad

A
do

Grinding 1 2
Turning 3 1
in

Assembling 6 3
Re

Testing 5 4
F

The available capacities of these operations in hours for the given time period are: grinding
30; turning 60, assembling 200; testing 200. The contribution to profit is ? 20 for each unit of
A and ? 30 for each umt of B. The furm can sell all that it produces at the prevailing market
price. Determine the optimum amount of A and B to produce during the given time period.
Formulate this as a LPP.
9. Vitamins A and B are found in two different foods Fj and p2. One unit of food Fj contains 2
units of vitamin A and 3 umts of vitamin B. One unit of food F2 contains 4 units of vitamin A
and 2 units of vitamin B. One unit of food Fj and cost ? 50 and 25 respectively. The
minimi^ daily requirements for a person of vitamin A and B is 40 and 50 units
respectively. Assuming that any thing in excess of daily minimum requirement of vitamin
A and B is not harmful, find out the optimum mixture of food Fj and F2 at the minimum
cost which meets the daily minimum requirement of vitamin A and B. Formulate this as a
LPP.
10. An automobile manufacturer makes automobiles and trucks in a factory that is divided into
two shops. Shop A, which performs the basic assembly operation, must work 5 man-days
on each truck but only 2 man-days on each automobile. Shop B, which performs finishing
LINEAR PROGRAMMING 26.19

operations, must work 3 man-days for each automobile or truck that it produces. Because of
men and machine limitations, shop A has 180 man-days per week available while shop B
has 135 man-days per week. If the manufacturer makes a profit of ? 30000 on each truck and
? 2000 on each automobile, how many of each should he produce to maximize his profit?
Formulate this as a LPP.
11. Two tailors A and B earn ? 150 and ? 200 per day respectively. A can stitch 6 shirts and 4
pants per day while B can stitch 10 shirts and 4 pants per day. Form a linear programming
problem to minimize the labour cost to produce at least 60 shirts and 32 pants.
12. An airline agrees to charter planes for a group. The group needs at least 160 first class seats

ow
and at least 300 tourist class seats. The airline must use at least two of its model 314 planes
which have 20 first class and 30 tourist class seats. The airline will also use some of its mode!
535 planes which have 20 first class seats and 60 tourist class seats. Each flight of a
model 314 plane costs the company ? 100,000 and each flight of a model 535 plane
costs ? 150,000. How many of each type of plane should be used to minimize the flight cost?

e
Formulate this as a LPP.

re
13. Amit's mathematics teacher has given him three very long lists of problems with the
instruction to submit not more than 100 of them (correctly solved) for credit. The problem

Flr
F
in tire first set are worth 5 points each, those in the second set are worth 4 points each, and
those in the third set are worth 6 points each. Amit knows from experience that he requires
on
ou
the average 3 minutes to solve a 5 point problem, 2 minutes to solve a 4 point problem,
and 4 minutes to solve a 6 point problem. Because he has other subjects to worry about, he

sr
can not afford to devote more than 3 ^ hours altogether to his mathematics assignment.
kfo
Moreover, the first tw'o sets of problems involve numerical calculations and he knows that
oo
he cannot stand more than 2 - hours work on this type of problem. Under these
Y

circumstances, how many problems in each of these categories shall he do in order to get
reB

maximum possible credit for his efforts? Formulate this as a LPP.


14. A farmer has a 100 acre farm. He can sell the tomatoes, lettuce, or radishes he can raise. The
price he can obtain is ? 1 per kilogram for tomatoes, ? 0.75 a head for lettuce and ? 2 per
uY

kilogram for radishes. The average yield per acre is 2000 kgs for radishes, 3000 heads of
lettuce and 1000 kilograms of radishes. Fertilizer is available at f 0.50 per kg and the
ad

amount required per acre is 100 kgs each for tomatoes and lettuce and 50 kilograms for
do

radishes. Labour required for sowing, cultivating and harvesting per acre is 5 man-days for
tomatoes and radishes and 6 man-days for lettuce. A total of 400 man-days of labour are
in

available at f 20 per man-day. Formulate this problem as a LPP to maximize the farmer's
Re

total profit.
F

15. A firm has to transport at least 1200 packages daily using large vans which carry 200
packages each and small vans which can take 80 packages each. The cost of engaging each
large van is ? 400 and each small van is f 200. Not more than ? 3000 is to be spent daily on the
job and the number of large vans cannot exceed the number of small vans. Formulate this
problem as a LPP given that the objective is to minimize cost.
16. A firm manufactures two products, each of which must be processed through two
departments, 1 and 2. Tlie hourly requirements per unit for each product in each
department, the weekly capacities in each department, selling price per unit, labour cost
per unit, and raw material cost per unit are summarized as follows:
Product A Product B Weekly capacity
3 2 130
Department 1
4 6 260
Department 2
?25 ?30
Selling price per unit
?16 ?20
Labour cost per unit
?4 ?4
Raw material cost per unit
26.20
APPLIED MATHEMATICS-XII

The problem is to determine the number of units to produce each product so as to maximize
total contribution to profit. Formulate this as a LPP.
ANSWERS

1. Max. Z = 30a: + 20y .. Mini. Z = 60A: + 80y


Subject to Subject to
10a: + 6y < 1000 x + y < 500
5a: + 4y < 600 a: < 400
a:, y > 0 y > 200
a: ^ 0,y > 0
3. Max. Z = 3A: + 2y + 4z 4. Max. Z = 2x+ 3y
Subject to Subject to
4a: + 3y + 5z < 2000 x + y < 400
2a: + 2y + 4z < 2500 2x + y < 600
100 < a: <150 a: > 0,y > 0

w
y > 200
z > 50

F lo
a:>0, y > 0, z > 0.
5. Mmi. Z = 2500a: + 3500y 6. Max. Z = 60x + 40y
Subject to Subject to
50a: + 60y > 2500

e
5x+3y < 45000

Fre
100a: + 60y > 3000 X < 7000
100a: + 200y > 7000 y < 10,000
for
X, y > 0. x,y > 0
7. Mini Z = 60x + 100y S. Max. Z = 20x+ 30y
r
Subject to Subject to
You

10x + 5y > 20 x + 2y < 30


oks

5x + 4y > 20 3x + y < 60
eBo

2x + 6y > 13 6x + 3y < 200


x,y > 0. 5x + 4y < 200
X, y > 0
our
ad

9. Mini. Z = 50x + 25y 10. Max. Z = 30000X + 2000y


Subject to Subject to
2x + 4y > 40 5x + 2y < 180
dY
Re

3x + 2y > 50 3x + 3y < 135


x>0,y >0 X > 0, y > 0
Fin

11. Mini. Z=150x+200y 12. Mini. Z = 100,000x + 150,000y


Subject to Subject to
6x + lOy > 60 20x + 20y > 160
4x + 4y > 32 30x + 60y > 300
x,y>0 X > 0, y>0
13. A4flx. Z ~ Sxj + 4x2 14. Max.Z = 1850x + 2080y + 18752
Subject to Subject to
Xj + X2 + Xg < 100 x + y+z < 100
3xj + 2x2 ^^3 — 210 5x + 6y + 5z < 400
3xj + 2x2 - 1^0. X, y,z > 0
Xi,X2,X3 > 0
15. Minimize Z = 400x + 200y 1ft. Max. Z = 5x + 6y
Subject to Subject to
LINEAR PROGRAMMING 26.21

400;r + 200y < 3000 3x + 2y < 130


200x + 80t/>1200 4x + 61/ < 260
x<y X > 0,y > 0.
x>0,y>0
HINTS TO SELECTED PROBLEMS

Let X units of product A and y units of product B be manufactured. Then, the mathematical
formulation of the LPP is
Minimize Z = 60a: + 80y
Subject to
X + y <500 (Labour hours constraint)
X < 400 (Machine hours constraint)

w
y > 200 (Agreement constraint)
X > 0, y > 0
16. Suppose X units of product A and y units of product B are produced to maximize the profit.

Flo
Then,
Profit = (25-16-4)x + (30-20-4)y= 5x + 6y.

e
re
3x + 2y < 130 (Capacity constraint of Department 1)
(Capacity constraint of Department 2)

F
4x + 6y < 260
and X > 0, y > 0
ur
26.5 SOME DEFINITIONS AND RESULTS
f or
In this section, we shall discuss some definitions related to the solution of linear programming
ks
problems.
Yo

The general form of a LPP is as given below:


oo

Maximize (or minimize) Z = Cj Xj + C2 at2 + (objective function)


B

Subject to
re

flu Xi + fli2 X2 + + a^, {<, = ,>}6l


fl2i X-^ + fl22 X2 + (Constraints)
u
ad
Yo

●^1 + %}2 ^2 + ●●●● ‘hnn^n ^ m

^1, ^2/ ,x„ > 0 (Non-negativity restrictions)


d

The following are some definitions related to a LPP.


Re
in

SOLUTION A set of values of variables x^^ X2, ■■■■, x„ is called a solution of a LPP, if it satisfies the
F

constraints of the LPP.


ILLUSTRATION 1 Consider the following LPP:
Maximize Z = 4x + 5y
Subject to
x + 2y < 6
3x + y < 12
X > 0, y > 0
Clearly, x=l,y = 2;x = -2,y = 3;x = -l,y = -2;x = 2, y = -3 etc. are solutions of this LPP as
they satisfy the constraints x + 2y < 6 and 3x + y < 12. Note that x = 2, y = 4 is not a solution,
because it does not satisfy x + 2y < 6.
FEASIBLE SOLUTION A set of values of the variables x^, X2,..., x„ is called a feasible solution of a LPP,
if it satisfies the constraints and non-negativity restrictions of the problem.
In other words, a solution that also satisfies the non-negativity restrictions of a LPP, is called a
feasible solution.
26.22
APPLIED MATHEMATICS-XII

INFEASIBLE SOLUTION A Solution of a LPP is an infeasible solution, if it does not satisfy the
non-negativity restrictions.
II.LUSTRATION 2 Consider the following a LPP:
Maximize Z = 6.y + 8y
Subject to
3.t + 2}/ < 30
-Y + 2j/ < 22

w
■T, 1/ > 0
We observe that .t = 2, i/= 3; .v =5, y = 0; y = - 2, y = -l; x = 0, y =-2 etc. are soluHons of this
LPP. Out of these solutions y = 2, y = 3 and y =5, y = 0 are feasible solutions, because these
solutions also satisfy non-negativity restrictions. Remaining solutions given above are

e
infeasible solutions.

re
ro
FEASIBLE REGION The common region determined by all the constraints of a LPP is called the feasible
region and every point in this region is a feasible solution of the given LPP..

F
OPTIMAL FEASIBLE SOLUTION A feasible solution of a LPP is said to be an optimal feasible solution, if

Ful
it also optimizes (maximizes or minimizes) the objective function.
Now, we shall discuss some definitions and results related to the feasible solutions of a LPP.

sr
CONVEX SET A set is a convex set, if every point on the line segment joining any two points in it lies in it.

ko
In Figs. 26.3 to 26.4 the polygons are convex sets whereas polygon in Fig. 26.5 is not a convex set.
o
of
o
Y
erB
uY

Fig. 26.3 Fig. 26.4 Fig. 26.5

THEOREM The set of all feasible solutions of a LPP is a convex set.


o
ad
d

The proof of the above theorem is beyond the scope of the syllabus for CBSE class XII.
It follows from the above theorem that the set of all feasible solutions of a LPP is a convex
in

polygon. When we are asked to solve a linear programming problem, it always means that we
Re

have to find its optimal solution. It is known from the general mathematical theory of linear
F

programming that a LPP may or may not attain an optimal solution. However, if it attains an
optimal solution, then one of the corner points (vertices) of the convex polygon of all feasible
solutions gives the optimal solution as stated in the following theorem.
FUNDAMENTAL EXTREME POINT THEOREM An Optimal solution of a LPP, if it exists, occurs at one of
the extreme (corner) points of the convex polygon of the set of all feasible solutions.
It may happen that the two vertices of the corner polygon give the optimal value of the object) ve
function, in such a case all points on the line segment joining these two vertices give the optimal
value and the LPP is said to have infinitely many solutions. Sometimes, the convex polygon is an
empty set. In such a case, we say that the LPP has no solution.
If the feasible region for a linear programming problem is bounded i.e., it can be enclosed within
a circle, then the objective function has both a maximum and a minimum value and each of these
values occurs at a corner point of the feasible region.
If the feasible region of a linear programming problem is imbounded i.e., it extends indefinitely
in any direction, then a maximum or a minimum value of the objective function may not exist.
However, if it exists, it occurs at a corner point of the feasible region.
LINEAR PROGRAMMING 26.23

26.6 GRAPHICAL METHODS OF SOLVING LINEAR PROGRAMMING PROBLEMS


There are two graphical methods for the solution of linear programming problems. These
methods are suitable for solving linear programming problems containing two variables only. If
a LPP contains more than two variables, these graphical methods are not suitable to solve them.
Such type of problems are solved by simplex method which is beyond the scope of our
discussion. We shall, therefore, be concerned only with the graphical methods involving two
variables x and i/.
The following methods are used to solve linear programming problems graphically:
(i) Corner-Point Method (ii) Iso-profit or iso-cost method.
We shall now apply these two methods for solving linear programming problems.
26.7 CORNER-POINT METHOD

w
This method is based on the Fundamental extreme point theorem which is stated in the earlier
section.

Following algorithm can be used to solve a LPP in two variables graphically by using the

F lo
comer-point method.

ee
ALGORITHM

Fr
STFP 1 Formulate the given LPP in mathematical form if it is not so.
STt-r 11 Convert all inequations into equations and draw their graphs. To draw the graph of a linear
for
equation, put \j = 0 in it and obtain a point on x-axis. Similarly, by putting a: = 0 obtain a point
ur
on y-axis. join these two points to obtain the graph representing the equation.
STHl' 111 Determine the region represented by each inequation. To determine the region represented by
ks

an inequation replace x niid y both by zero, if the inequation reduces to a valid statement, then
Yo

the region containing the origin is the region represented by the given inequation. Otherwise,
oo

the region not containing the origin is the region represented by the given inequation.
eB

STEP IV Obtain the region in xy-plane containing all points that simultaneously satisfy all constraints
including non-negativity restrictions. The polygonal region so obtained is the feasible region
r

and is known as the convex polygon of the set of all feasible solutions of the LPP.
ou
ad

STEP V Determine the coordinates of the vertices (corner points) of the convex polygon obtained in
Step II. These vertices are known as the extreme points of the set of all feasible solutions of the
Y

LPP.
Re

Obtain the values of the objective function at each of the vertices of the convex polygon. The
nd

STEP \ [
point inhere the objective function attains its optimum (maximum or minimum) value is the
Fi

optimal solution of the given LPP.


Ki'MARk I If the feasible region of a LPP is bounded i.e., it is a convex polygon. Then, the objective
function Z = rt.v + by has both a maximum value M and a minimum value m and each of these values
occurs at a corner point of the convex polygon.
KL'.' -Pk : Sometimes the feasible region of a LPP is not a bounded convex polygon. That is, it e.xtends
indefmitely in any direction. In such cases, we say that the feasible region is unbounded. The above
algorithm is applicable when the feasible region is bounded. If the feasible region is unbounded, then we
ifnd the values of the objective function Z = ax + by at each corner point of the feasible region. Let M and m
respectively denote the largest and smallest values of Z at these points. In order to check whether Z has
maximum and minimum values as M and m respectively, we proceed asfollozos:
(i) Draw the line ax + by = M and find the open half plane ax + by>M. If the open half-plane
represented by ax + by>M has no point common ivith the unbounded feasible region, then M is
the maximum value ofZ. Otherwise Z has no maximum value.
26.24
APPLIED MATHEMATICS-XM

(ii) Draw the line ax + by =m and find the open half plane represented by ax + by <ni If the open
half-plane ax + by <m has no point common with the unbounded feasible region, then m is the
minimum value ofZ. Otherwise, Z has no minimwn value.
Following examples illustrate the above algorithm.
ILLUSTRATIVE EXAMPLES

EXAMPLE 1 Solve the following LPP graphically:


Maximize Z = 5x + 3 y
Subject to
3x + 5y < 15
5a: + 2y < 10

w
and, x,y > 0
SOLUTION
Converting the given inequations into equations, we obtain the following

F lo
equations:
3a: + 5y = 15,5x + 2y = 10, x = 0 and y = 0

ee
Region represented by 3x + 5y<15: The line 3x + 5y =15 meets the coordinate axes at
(5, 0) and (0, 3) respectively. Join these points to obtain the line 3x + 5y = 15. Clearly, (0,0)

Fr
satisfies the inequation 3x + 5y < 15. So, the region containing the origin represents the solution
set of the inequation 3x + 5y < 15. for
ur
Region Represetited by5x + 2y < 10: The line 5x + 2y = 10 meets the coordinate axes at A2 (2, 0)
and B2 (0,5) respectively. Join these points to obtain the graph of the line 5x + 2y = 10. Clearly,
s

(0, 0) satisfies the inequation 5x + 2y < 10. So, the region containing the origin represents the
ook
Yo

solution set of this inequation.


eB

Region represented by x>0 and y>0: Since every point in the first quadrant saHsfies these
inequations. So, the first quadrant is the region represented by the inequations x > 0 and y > 0.
The shaded region OA2 PB^ in Fig. 26.6 represents the common region of the above inequations.
r
ad
ou

This region is the feasible region of the given LPP.


The coordinates of the vertices {comer-points) of the shaded feasible region are O (0, 0),
Y

A2 (2, 0), P (20/19, 45/19) and Bj (0, 3).


Re
nd

y
U'
Fi

(5> 83(0,5)

Bj (0,3) ^ll9 I9J


r

m ^i(5, 0)

\
O X
-^2(2,0)

Fig. 26.6

These points have been obtained by solving the equations of the corresponding intersecting
lines, simultaneously.
LINEAR PROGRAMMING 26.25

The values of the objective function at these points are given in the following table:
Point {x, y) Value of the objective function Z = 5a: + 3y

O (0, 0) Z = 5x0+3x0 = 0

Z = 5x2+3x0 =10
^2 (2. 0)
Z = 5 X
20
h 3 X 45 ^ 235
19 19 19
U9, 19

w
(0, 3) Z=5x0+3x3=9

Clearly, Z is maximum at P (20/19, 45/19). Hence, a: = 20/19, y = 45/19 is the optimal solution
of the given LPP and the optimal value of Z is 235/19.

o
e
re
Solve the following LPP b\j graphical method:
Minimize Z = 20a + lOy

Frl
F
Subject to
A + 2y < 40
ou
r
3a + y > 30
4a + 3y > 60
so
kf
and, A,y > 0
oo
SOLUTION Converting the given inequations into equations, we obtain the following equations:
Y

A + 2y = 40, 3a + y = 30, 4a + 3y = 60, a = 0 and y = 0


B

Region represented byx+ 2y < 40: The line a + 2y = 40 meets the coordinate axes at (40, 0) and
(0, 20) respectively. Join these points to obtain the line a + 2y = 40. Clearly, (0,0) satisfies the
re
oY

inequation a + 2y < 40. So, the region in .ry-plane that contains the origin represents the solution
u
ad

set of the given inequation.


Region represented by 3a + y > 30: The line 3a + y = 30 meets a and y axes at A2 (10, 0) and
d

B2 (0, 30) respectively. Join these points to obtain this line. We find that the point O (0, 0) does
in

not satisfy the inequation 3a + y > 30. So, that region in Ay-^lane which does not contain the
Re

origin is the solution set of this inequation.


F

Region represented by Ax + 3y> 60: The line 4a + 3y = 60 meets a and y axes at A^ (15, 0) and
Bi (0, 20) respectively. Join these points to obtain the line 4a + 3y = 60. We observe that the point
O (0, 0) does not satisfy the inequation 4a + 3y > 60. So, the region not containing the origin in
Ay-plane represents the solution set of the given inequation.
Region represented by x>0,y>0: Clearly, the region represented by the non-negativity
restrictions a > 0 and y > 0 is the first quadrant in Ay-plane.
The shaded region A^ QP in Fig. 26.7 represents the common region of the regions
re
presented by the above inequations. This region expresents the feasible region of the given
LPP.
26.26
APPLIED MATHEMATICS-XK

\ 63(0, 30)
3-y + j/ = 30

4y + 3y = 60
.r + 2y = 40
Bi(0,20) Q (4 18)

P
[6,12:
Aj(40,0)
O
A,(15,0) X

ow
^2(10,0)
Fig. 26.7

The coordinates of the comer-points of the shaded feasible region are A3 (15, 0), A| (40, 0),

e
Q (4,18) and P (6,12). These points have been obtained by solving the equations of the

re
corresponding intersecting lines, simultaneously.

rFl
The values of the objective function at these points are given in the following table:

F
Point (x,}/) Value of the objective function Z = 20.y+I0i/
^3 (15, 0)

r
Z = 20 X 15 + 10 X 0 = 300
ou
A-i (40, 0) Z
fo
= 20 X 40 +10 X 0 = 800
ks
Q(4,18) Z = 20x 4 + 10x18 = 260
P (6,12) Z = 20x 6 + 10x12 = 240
oo

Outof these values of Z, the minimum value is 240 which is attained at point P (6,12). Hence,
Y

Y = 6, 1/ = 12 is the optimal solution of the given LPP and the optimal value of Z is 240.
B

r:X.<\MrLE 3 So/w the following LPP graphically:


re

Minimize and Maximize Z = 5x + 2y


Subject to
ou
Y

- 2y - 3y < - 6
ad

Y-2y < 2
3y + 2i/ < 12
d

-3x+2y < 3
Y, y > 0
in
Re

SOLUTION Converting the given inequations into equations, we get


2y + 3y = 6, y - 2y = 2, 3y + 2y =12, - 3y + 2y = 3, y = 0 and y = 0
F

Region represented by - 2x - 3y < - 6: The line - 2y - 3y = - 6 or, 2y + 3y = 6 cuts OX and OY at


Ai (3, 0) and (0, 2) respectively. Join these points to obtain the line 2y + 3y = 6.
Since O (0, 0) does not satisfy the inequation — 2y — 3y < — 6. So, the region represented by
“ 2y - 3y < - 6 is that part of XOV-plane which does not contain the origin.
Region represented by x - 2y < 2: The line y - 2y = 2 meets the coordinate axes at A2 (2, 0) and
®2 (0/ - !)● tliese points to obtain y - 2y = 2.Since (0,0) satisfies the inequation y - 2y ^ 2,so
the region containing the origin represents the solution set of this inequation.
Region represented by 3y + 2y <12: The line 3y + 2y < 12 interesects OX and OX at A3 (4, 0) and
63 (0, 6). Join these points to obtain the line 3y + 2y = 12. Clearly, (0, 0) satisfies the inequation
3y + 2y <12. So, the region containing the origin is the solution set of the given inequations.
Region represented by - 3y + 2y < 3: The line - 3y + 2y = 3 intersects OX and OY at A4 (-1, 0)
and 64 (0, 3/2). Join these points to obtain the line - 3y + 2y = 3. Clearly, (0, 0) satisfies this
inequation. So, the region containing the origin represents the solution set of the given
inequation.
LINEAR PROGRAMMING 26.27

Regioji represented by x > 0,1/ > 0: Clearly, XOY quadrant represents the solution set of these two
inequations.
The shaded region shown in Fig. 26.8 represents the common solution set of the above
inequations. This region is tlie feasible region of the given LPP.
3x + 2y= 12

ow
e
re
Frl
F
ou
or
kfs
The coordinates of the corner-points (vertices) of the shaded feasible region Pj P2 P3 P4 are
oo

P, fl? P^CL I'] p-fl 15.1 and P1I—, —|. These points have been obtained by
1 7 '7j'^U' 4J' Hi' 4 J Hi3 13
Y
eB

solving the equations of the corresponding intersecting lines, simultaneously.


The values of the objective function at these points are given in the following table:
ur

Point jx, y) Value of the objective function Z =5.y + 2^


oY

18 _ 2 94
ad

Z = 5x —+2x-= „
^ 7 7) 7 7 7
d

7 3
7 3 L+2xX = 19
Pn Z = 5x
^24 2 4
in
Re

3 15
P3 Z = 5x- + 2x — =15
^ 2 4
F

2 4
3 24 63
Z = 5x — + 2 X —■ =
13 13
^ 13 13 J 13

^ 9*4 7 3
Clearly, Z is minimum at .v = — and 1/ = — and maximum at a: = — and 1/ = —. The minimum
13 13 2 ● 4
63
and maximum values of Z are ^ and 19 respectively.
13

EX.-\MPI.E 4 Solve the following LPP graphically:


Maximize and Minimize Z = 3.v + 5y
Subject to 3a* - 4y + 12 > 0
2A--1/ + 2 > 0
2a-+ 31/-12 > 0
0<A <4

y >2
26.28
APPLIED MATHEMATICS-XII

SOLUTION The given LPP can be re-written as:


Maximize or Minimize Z = 3x + 5y
Subject to 3x - 4y > -12
2x-y > -2
2x+3y> 12
X < 4

y > 2
X > 0

Converting the inequations into equations, we obtain the following equations 3:c -4y = -12,
2x~y = ~2,2x+ 3y = 12,x = 4,y = 2andx = 0.
These lines are drawn
on suitable scale. The shaded region Pj ?2 P3 P4 Pg shown in Fig. 26.9
represents the feasible region of the given LPP.

w
F lo
e
Fre
for
r
You
oks
eBo
ad
our
Re
dY

The values of the objective function at these points are given in the following table:
Fin

Point (x, y) Value of the objective function Z = 3x + 5y


Pi (3, 2) Z = 3x3+5x2 = 19

P2(4, 2) Z = 3x4 + 2x5=22

P3 (4, 6) Z = 3x4+5x6=42

7
Z -3 X —
= 3 4 +5X— =
- 18 102
5 5 5 5 5

. 3 _ 7 79
Z = 3x —h5x —
^ 4 2 4 2 4

Clearly Z assumes its minimum value 19 at at = 3 and y = 2. The maximum value of Z is 42 at


a: = 4 and y = 6.
26.29
LINEAR PROGRAMMINC-

LXAMi'l ● Determine graphically the minimum value of the objective function Z = -50;c + 20y
Subject to constraints:
2x-y > -5
Sx + y > 3
lx-2,y < 12
a: > 0,y > 0
SOLUTION The feasible region of the system of inequations given in constraints is shown in
Fig. 26.10. We observe that the feasible region is unbounded.

w
F lo
ee
Fr
for
ur
s
ook
Yo
eB

The values of the objective function Z at the corner points are given in the following table:
r

Corner point {x, y) Value of the objective function Z = -50a: + 20y


ou
ad

(0,5) Z = -50x 0 + 20x5 = 100


Y

(0,3) Z = -50x 0 + 20x 3 = 60


Re
nd

(1,0) Z =-50x1 +20x 0 = -50

(6,0) Z = -50x 6 + 20x 0 = - 300


Fi

Clearly, - 300 is the smallest value of Z at the corner point (6, 0). Since the feasible region is
unbounded. Therefore, to check whether - 300 is the minimum value of Z, we draw the line
- 300=-50a: + 20y and check whether the open half plane -50.r + 20y <- 300 has points in
common with the feasible region or not. From Fig. 26.10, we find that the open half plane
represented by -50a: + 20y < - 300 has points in common with the feasible region. Therefore,
Z = - 50.r + 20y has no minimum value subject to the given constraints.
26.8 ISO-PROFli OR I * 30S1

Consider the following LPP


Maximize Z = 10a: + 6y
Subject to
3x + y < 12
2A: + 5y < 34
x,y > O
26.30 APPLIED MATHEMATICS-XII

The convex set of all feasible solutions of this LPP is the set of all points in the shaded region of
Fig. 26.11. Any point in this region is a feasible solution of the above LPP and only the points in
this region are feasible solutions of the above LPP. In order to solve the above LPP, we have to
find the point or points in the shaded region which give the largest value of the objective
function. For any fixed value of Z, Z = 10a* + 6i/ or, lO.v + 6y = Z is a straight line. For example,
for Z =5, lOx + 6y =5 is a straight line. Any point on the line Z =10x + 6y will give the same
value of Z. So, it is known as an iso-profit line.
y

ow
W
V
(0,12)

e
re
P(2,6)

F
Frl
^5;
ou ^2 (17,0)

osr
O A, (4,0] X

●y "^22 = lOx + 6y
kf
oo
= lO.Y + 6y
Y
Fig. 26.11
B

Also, for each different value of Z, we obtain a different line. In other words, for different values
of Z, equation Z = lOx + 6y gives a family of parallel straight lines of slope - — =
^ and any
re
uY

6
point on the line Z = lOx + 6y, for given value of Z, gives the same value of Z. These lines are
iso-profit lines. In order to maximize the objective function Z = lOx + 6y, we have to find the
ad
do

line with the largest value of Z which has at least one point in common with the shaded region.
In other words, to maximize the objective function find the line parallel to Z = lOx + 6y which is
in

farthest from the origin O and which has at least one point in common with the shaded region.
Clearly, lOx + 6y = Z^ is not farthest from the origin. However, lOx + 6y = Z2 is farthest from the
Re

origin and has a point P (2, 6) common with the shaded region.
F

Thus, we see that Z2 is the maximum value of Z, and the feasible solution which gives this value
of Z is the corner P (2, 6) of the shaded region. The values of the variables for the optimal
solution are x = 2, y = 6. Substituting these values in Z = lO.v + 6y, we get Z =56 as tlie optimal
value.

Now, consider the LPP


Minimize Z = 18x lOy
Subject to 4x + y > 20
2x + 3y > 30
x,y > 0
The convex set of all feasible solutions of this LPP is the set of all points in the shaded region of
Fig. 26.12. In order to solve this LPP, we have to find the points in the shaded region which give
the smallest value of the objective function. We observe that for any fixed value of Z, equation
18x + lOy = Z is a straight line and any point on this line gives the same value of Z. So, for some
value of Z say Zy if the line 18x + lOy = Z| has some points common with the feasible region of
26.31
LINEAR PROGRAMMING

the LPP, then all these points give the same value of Z equal to Z-j i.e. for every point in the
feasible region lying on 18.v + lOy = Zj, we obtain the same value of Z equal to Z^. The line
18-y + lOy = Zi is known as iso-cost line. Thus, I8.v + lOy = Z gives a family of parallel lines of
18 .
slope in
xy-plane. In order to find the minimum value of Z, we have to find the line nearest
10

to the origin and having at least one point common with the shaded region. Clearly,
18x -I- lOy = Z? is nearest to the origin and has a common point P (3, 8) with the shaded region.
The line 18x + lOi/ = Z3 is more closer to the origin than the line 18.r + lOy = Z2, but it does not
have any point common to the feasible region. Thus, Z2 is the minimum value of Z, and the
feasible solution which gives this value of Z is the corner P (3, 8) of the shaded region. The
values of the variables for the optimal solution are .v = 3, y = 8. Substituting these values in

w
Z =18.v -f- lOy, we get Z =128 as the optinral value of Z.

o
e
re
rFl
F
or
ou
ksf
oo
Y
B
re
oYu
ad
d
in
Re

Fig. 26,12
F

The above discussion suggests the following algorithm to solve a LPP by using iso-profit
(iso-cost) lines.
ALGORITHM
STi r Formulate the given LPP in matheinaficnl form, if it is not given so.
STF-Pll Obtain the region in xy-plane containing all points that simultaneoush/ satisfi/ all constraints
including non-negativity restrictions. The polygonal region so obtained is the convex set of all
feasible solutions of the given LPP and it is also known ns the feasible region.
sILIMll Determine the coordinates of the vertices (corner points) of the feasible region obtained in
step II.
Ml l'l\ Give some convenient value to Z and draw the line so obtained in xy-plane.
STI.r\ If the objective function is of maximization type, then draw lines parallel to the line in step IV
and obtahi a line which is farthest from the origin and has at least one point common to the
feasible region.
26.32 APPLIED MATHEMATICS-XH

If the objective function is of minimization type, then draw lines parallel to the line in step IV
and obtain a line which is nearest to the origin and has at least one point common to the feasible
region.
Find the coordinates of the common point (s) obtained in step V. The point (s) so obtained
determine the optimal solution (s) and the value (s) of the objective function at these point (s)
give the optimal solution.

ILLUSTRATIVE EXAMPLES
EXAMPl [■ 1
Solve the following linear programming problem graphically:
Maximize Z = 50x + 15y
Subject to
5x + y < 100
x + y < 60

w
-v, y > 0
SOLUTION
We first convert the inequations into equations to obtain the lines 5x + y =100,
x + y = 60,x = 0 and y = 0.

F lo
The line 5x + y = 100 meets the coordinate axes at A-^ (20, 0) and (0,100). Join these points to
obtain the line 5.r + y = 100.

ee
The line x + y = 60 meets the coordinate axes at (60, 0) and B2 (0, 60). Join these points to

Fr
obtain the line x + y = 60.
Also, a: = 0 is the y-axis and y = 0 is the x-axis.
The feasible region of the LPP is shaded in Fig. 26.13. The coordinates of the comer-points of the
for
ur
feasible region OA-^ PB2 are O (0, 0), A^ (20, 0), P (10,50) and B2 (0, 60).
y
s
ook
Yo

V
eB

Si(0,100)

Q3'
r
ou
ad
Y

62(0,60)
P (10, 50)
Re

V
nd
Fi

Qi(0, 20)

o
Pfe, o)\ -^1(20,0) ),0) X

5.V + y = 100
X +}/ - 60
Fig, 26,13

Now, we take a constant value, say 300 (i.e. 2 times the l.c.m. of 50 and 15) for Z. Then,
300 = 50t + 15y
26.33
LINEAR PROGRAMMING

This line meets the coordinate axes at P| (6, 0) and Qj (0, 20). Join these points by a dotted line.
Now, move this line parallel to itself in the increasing direction i.e. away from the origin.
P2 Q2 and P3 Q3 are such lines. Out of these lines locate a line which is farthest from the origin
and has at least one point common to the feasible region.
Clearly, P3 Q3 is such line and it passes through the vertex P (10,50) the convex polygon
OAi PB2-Hence, x= 10 and y =50will give the maximum valueofZ. The maximum valueofZ
is given by
Z = 50x10 + 15x50 = 1250.

EX.AMPi .1 ; Solve the following LPP graphically:


Maximize Z =5x + 7y
Subject to
X +y < 4
3x + Sy < 24

w
lOx + 7y < 35
x,y > 0

F lo
SOLUTION Converting the inequations into equations, we obtain the following equations:
a: + y = 4, 3x + 8y = 24, lOx + 7y = 35, x = 0 and y = 0.
These equations represent straight lines in XOY-plane.

ee
(4, 0) and (0, 4). Join these points to obtain

Fr
The line a: + y = 4 meets the coordinate axes at
the line a: + y = 4.
The line 3a: + 8y = 24 meets the coordinate axes at A2 (8, 0) and 62 (0, 3). Join these points to
for
obtain the line 3a: + 8y = 24.
ur
The line lOx + 7y = 35 cuts the coordinates axes at A^ (3.5, 0) and B3 (0,5). These points are
s
joined to obtain the line lOx + 7y = 35.
ook
Yo

Also, a: = 0 is the y-axis and y = 0 is the A:-axis.


The feasible region of the LPP is shaded in Fig. 26.14. The coordinates of the comer points of the
eB

feasible region OA3 PQB2 are O (0, 0), A3 (3.5, 0),P V “/“'Q
3 3/ v5 5
and B2 3).
r
ad
ou
Y
Re
nd
Fi

Now, we take a constant value, say 10 for Z, Putting Z =10 in Z =5a: + 7y, we obtain the line
f lO")
5a: + 7y =10. This line meets the coordinate axes at Pj (2, 0) andQi V 0, —
7 J
Join these points by
26.34 APPLIED MATHEMATICS-XII

a dotted line. Now, move this line parallel to itself in the increasing direction away from the
origin. P2 Q2 and P3 Q3 are such lines. Out of these lines locate a line farthest from the origin
and has at least one common poiirt to the feasible region OA3 PQB2-Clearly, P3 Q3 is such line
and it passes through the vertex Q (8/5,12/5) of the feasible region. Hence x = 8/5 and t/ = 12/5
gives the maximum value of Z. The maximum value of Z is given by
8 12
Z = 5x + 7x — = 24.8.
5 5
EXAMPLE.'.
Solve the following LPP graphically:
Minimize Z = 3x + 5i/
Subject to
-2x + y < 4
x+ij > 3
x-2y < 2

w
x,y >0
SOLUTION Converting the inequations into equations, we obtain the lines -2.\' + t/=4,

F lo
x + y = 3, X -2y = 2,x = 0 and y = 0.
These lines are drawn on a suitable scale and the feasible region of the LPP is shaded in

ee
Fig. 26.15.

Fr
for
ur
s
ok
Yo
o
eB
r
ou
ad
Y
Re
nd
Fi

Now, give a value, say 15 equal to (l.c.m. of 3 and 5) to Z to obtain the line 3.v + 5y = 15. This line
meets the coordinate axes at Pj (5, 0) and Qj (0, 3). Join these points by a dotted line. Move this
line parallel to itself in the decreasing direction towards the origin so that it passes through only
one point of the feasible region. Clearly, P3 Q3 is such a line passing through the vertex P of the
feasible region. The coordinates of P are obtained by solving the lines x-2y~2 and x + y = 3.
Solving these equations, we get.v = 8/3 and y= 1/3. Putting.v = 8/3 andy =l/3inZ = 3x +5y,
we get
7
Z =
.,8^1
3x —i-ox —
29
= —
3 3 3
29 8 1
Hence, the minimum value of Z is — at x = — ,y = -.
3 3 3
LINEAR PROGRAMMING 26.35

EXERCISE 26.2

Solve each of the folloiving linear programming problems In/ graphical method.
1. Maximize Z = 5x + 3y 2. Maximize Z = 9.v + 3y
Subject to Subject to
3.Y + 5i/ < 15 lx +3y <\3
5.T + 2y<10 3.Y + y ^ 5
X, y > 0 X, \j > 0
3. Minimize Z = 18.y + lOy 4. Maximize Z=50.r+30y
Subject to Subject to
4.v + y > 20 lx +y < 18
2.V + 3y > 30 3.Y + 2y < 34
y>0 .V, y > 0
6. Maximize Z =15a: + lOy

w
5. Maximize Z = 4.y + 3y
Subject to Subject to
3.V + 4y < 24 3.Y + 2y < 80

F lo
8-Y + 6y < 48 2.Y + 3y <70
y<5 .Y, 1/ > 0

ee
y <6

Fr
-Y, y > 0
7. Maximize Z = \0x + 6y 8. Maximize Z = 3.y + 4y
Subject to for
Subject to
2x + 2y < 80
3.Y + y <12
ur
2.Y + 5y < 34 2.v + 4y <120
s

-Y, y > 0
ook
Yo

9. Maximize Z=7x + lQy 10. Minimize Z = 2x + 4y


Subject to Subject to
eB

.Y + y < 30000 X + y >8


y < 12000 X + 4y > 12
our

.Y > 6000 .Y > 3, y > 2


ad

X >y
X, y > 0
Y

11. Minimize Z =5.y + 3y 12. Minimize Z = 30.y + 20y


Re

Subject to Subject to
nd

2.Y + y > 10 X + y <8


Fi

.V + 3 y >15 .V + 4y > 12
A- < 10 5.V + 8i/ = 20
i/<8 .v/y>0
A', y > 0
13. Maximize Z = 4.y + 3y Minimize Z = Y-5y + 20
Subject to Subject to
3.Y + 4y < 24 x-y>0
8a- + 6y < 48 - x + 2y>2
.y<5 x>3

y <6 y <4
.Y, y > 0 .Y, y > 0
15. Maximize Z = 3a-+5i/ Uv Minimize Z = 3a--]+5a'2
Subject to Subject to
A- + 2y < 20 .Y| + 3.Y2 ^ 3
.Y + y < 15 .Y^ + .A'2 ^ 2
26.36 APPLIED MATHEMATICS-XII

y<5 Xi, >:2 >0


x,y>Q
17. Maximize Z = 2x+3y Maximize Z = - arj + 1x2
Subject to Subject to
x + y>l -Xi + 3x2 ^10
10x + y>5 + ;C2 < 6
x + Wy>l ^1-^2 ^2
x,y>0 x^,X2>0
■ Maximize Z = x + y Maximize Z = 3xj + 4^2, if possible.
Subject to Subject to the constraints
-2x+y<l Xi-X2<~l
x<2
-Xi + X2<0
x + y<3 X-i,X2>0
. x,y>0

w
'J- Maximize Z = 3x + 3y, if possible.
Subject to the constraints
x-y<l
x + y>3

F lo
ee
>0

Fr
22. Show the solution zone of the following inequalities on a graph paper:
5x + y > 10 for
x +y > 6
;c + 4y > 12
r
X > 0,y > 0
You
s

Find X and y for which 3x + 2y is minimum subject to these inequalities. Use a graphical
ook

method.
eB

23. Find the maximum and minimum value of 2:r + y subject to the constraints:
:r + 3y > 6, ;c - 3y < 3, 3x + 4y < 24, - 3a: + 2y < 6, 5a: + y > 5, .j;, y > 0.
O I

Find the minimum value of 3a: + 5y subject to the constraints


our
ad

-2x + y <4, x + y>3, X -2y <2, X, y>0.


25. Solved the following linear programming problem graphically:
Maximize Z = 60a: + 15y
dY
Re

Subject to constraints
a: + y < 50
Fin

3a: + y < 90
x,y>0
26. Find graphically, the maximum value of z = 2a: + 5y, subject to constraints given below:
2a: + 4y < 8
3A: + y < 6
x + y <4
a: > 0, y > 0
Solve the following LPP graphically:
Maximize Z = 20a: + lOy
Subject to the following constraints
a: + 2y < 28
3A: + y <24
a:>2

x,y>o
LINEAR PROGRAMMING 26.37

28. Solve the following linear programming problem graphically:


Minimize z = 6x+ 3y
Subject to the constraints:
4x + i/>80
x + 5y > 115
3x + 2y <150
x>Q,y>Q
ANSWERS

20 45 235 2 29
X = y = — ,Z= - x = -,y = — , Z =15 or, x = - y = 0,Z =15
19' ●^19 19 7^7 3'
3. ;c = 3,v = 8,Min.Z=134 4. :c =10, y = 2, Z =560
24 27
x = — ,V=—,Z=24
r 4 „
or x=5,y= —,Z=24
7^7 -^3

w
80
6. x=^,y = 0, Z =400 or, x = 20, y =10, Z = 400 . X =1, y = 6, Z =56
3 '
9. X =18000, y =12000, Z = 246000

F lo
8. x = 20,y = 20, Z =140
x = 3,y = 4,Z =27 12. x = -,y
^ = —,Z=60
15
10. x=4, y=2, Z=16 7^ 7
24 24

ee
13. x =— ,y = —,Z=24 14. X = 4, y = 4, Z = 4 15. X = 10, y =5, Z =55
7^7

Fr
20
16. 7 17. 2 18. — 19. 3 20. Does not exist
3
21. Max value is infinity i.e. the solution is unbounded 43
for 84 15
ur
22. X = l,y = 5,Z = 13 23. Max. = — at x =
3 13'^ 3
8
s

Zr... x = 20,y = 30,Z =1650 26. x = -,Z=— 28. x=15,y=20


5'^ = 5
ook

5 ^
Yo
eB

26.9 DIFFERENT TYPES OF LINEAR PROGRAMMING PROBLEMS


In this section, we shall discuss formulation and solution of some important types of linear
programming problems viz. Diet problems. Optimal product line problems and Transportation
our

problems.
ad

26.9.1 DIET PROBLEMS

In this type of problems, we have to find the amount of different kinds of constituents/nutrients
Y

which should be included in a diet so as to minimize the cost of the desired diet such that it
Re
nd

contains a certain minimum amount of each constituent/nutrient.


Following are some examples on this type of problems.
Fi

ILLUSTRATIVE EXAMPLES

A house wife wishes to mix together two kinds offood, X and Y, in such a way that the
mixture contains at least 10 units of vitamin A, 12 units of vitamin B and 8 units of vitamin C.
The vitamin contents of one kg of food are given below:
Vitamin A Vitamin B Vitamin C
Food X: 1 2 3
Food Y: 2 2 1

One kg offood X costs ^6 and one kg offood Y costs ^ 10. Find the least cost of the mixture which will
produce the diet.
SOLUTION Let X kg of food X and y kg of food Y are mixed together to make the mixture.
Since one kg of food X contains one unit of vitamin A and one kg of food Y contains 2 units of
vitamin A. Therefore, x kg of food X and y kg of food Y will contain x + 2y units of vitamin A. But
the mixture should contain at least 10 units of vitamin A. Therefore,
X + 2y >10
26.38 APPLIED MATHEMATICS-XIt

Similarly, x kg of food X and y kg of food Y will produce 2.v + 2y units of vitamin B and 3x + y
units of vitamin C. But the minimum requirements of vitamins B and C are respectively of 12
and 8 units.

2x + 2y > 12 and 3x + y > 8


Since the quantity of food X and food Y cannot be negative.
-V > 0,1/ > 0
It is given that one kg of food X costs f 6 and one kg of food Y costs ? 10. So, .r kg of food X and y
kg of food y will cost f (6a: + lOy).
Thus, the given linear programming problem is
Minimize Z = 6A: + 10y
Subject to X + 2y >10
2x + 2y > 12
3a' + y > 8
and. A > 0,y > 0

w
To solve this LPP, we draw the lines
A + 2y = 10, 2a + 2y =12 and 3.v + y =8.

F lo
The feasible region of the LPP is shaded in Fig. 26.16.

ee
Fr
for
ur
s
ook
Yo
eB
our
ad
Y
Re
nd
Fi

The coordinates of the vertices (Comer-points) of shaded feasible region A] P2 83 are


(10, 0), Pi (2, 4), P2 (1,5) and 63 (0, 8). These points have been obtained by solving the
equations of the corresponding intersecting lines, simultaneously.
The values of the objective function at these points are given in the following table:
Point (a, y) Value of the objective function Z = 6.A + 10y
A (10, 0) Z = 6 X 10 + 10 X 0 = 60

/iz (2, 4) Z= 6x2 + 10x4 =52

2=6x1+10x5=56

63 (0, 8) 2=6x0 + 10x8 = 80

Clearly, 2 is minimum at a = 2 and y = 4. The minimum value of 2 is 52.


26.39
LINEAR PROGRAMMING

We observe that the open half-plane represented by 6.r + 10y<52 does not have points in
common with the feasible region. So, Z has minimum value equal to 52.
Hence, the least cost of the mixture is ? 52.
EXAMPLE 2 A dietician wishes to mix two types offood in such a way that the vitamin contents of the
mixture contain at least 8 units of vitamin A and 10 units of vitamin C. Food 7' contains 2 units/kg of
vitamin A and 1 unit/kg of vitamin C while food 'If contains 1 unit/kg of vitamin A and 2 units/kg of
vitamin C. It costs 15.00 per kg to purchase food 7' and ? 7.00 per kg to produce food 7/'. Determine the
minimum cost to such a mixture. Formulate the above as a LPP and solve if.
SOLUTION Let the dietician mix x kg of food 7' with y kg of food 'II'. Then, the mathematical
model of the LPP is as follows:
Minimize Z =5x + 7y
Subject to2x + y > 8
.T + 2y > 10 [See Example 8, page 26.9]

w
and. -Y,y > 0
To solve this LPP graphically, we first convert the inequations into equations to obtain the

F lo
following lines.
2x + y = 8, -Y + 2y = 10, .v = 0, y = 0

ee
The line 2.v -i- y = 8 meets the coordinate axes at A-i (4, 0) and Bj (0, 8). Join these points to obtain

Fr
the line represented by 2.v + y = 8. The region not containing the origin is represented by
2y + y > 8.
for
The line y + 2y =10 meets the coordinate axes at A2 (10, 0) and 62 (0,5). Join these points to
ur
obtain the line represented by Y + 2y=10. Clearly, 0(0,0) does not satisfy the inequation
.V -I- 2y > 10. So, the region not containing the origin is represented by this inequation.
s
ook

Clearly, .r > 0 and, y > 0 represent the first quadrant.


Yo

Thus, the shaded region in Fig. 26.17 is the feasible region of the LPP. The coordinates of the
eB

corner-points of this region are A2 (10, 0), P (2, 4) and (0, 8).
y
our
ad
Y
Re

B,{0,8)
nd
Fi

B#, 5)
P{2,4)

(f,o)
X
o A,(4, 0) /l2(10,0)
.r + 2}/ = 10
2-r + y = 8 5.Y + 7y = 38
Fig. 26.17

The point P (2, 4) is obtained by solving 2.v -t- y = 8 and y + 2y = 10 simultaneously.


26.40 APPLIED MATHEMATICS-XII

The values of the objective function 2 =5x + 7y sti the comer points of the feasible region are
given in the following table:
Point (x, y) Value of the objective function Z =5x + 7y
A2 (10, 0) 2 = 5x10+7x0 =50

P(2, 4) 2 = 5x2+7x4 = 38

fil (0, 8) 2 = 5x0 + 7x8 =56

Clearly, 2 is minimum at = 2 and y =4. The minimum value of 2 is 38.

ow
We observe that open half plane represented by 5x + 7y < 38 does not have points in common
with the feasible region. So, 2 has minimum value equal to 38 at x = 2 and y = 4.
Hence, the optimal mixing strategy for the dietician will be to mix 2 kg of food 'T and 4 kg of
food 'ir. In this case, his cost will be minimum and the minimum cost will be ? 38.00.
EXAMPLE 3 Every gram of zvheat provides 0.1 gm of proteins and 0.25 gm of carbohydrates. The

e
re
corresponding values of rice are 0.05 gm and 0.5 gm respectively. Wheat costs f 4 per kg and rice ? 6. The

rFl
minimum daily requirements of proteins and carbohydrates for an average child are 50 gms and 200 gms

F
respectively. In what quantities should wheat and rice be mixed in the daily diet to provide minimum daily
requirements of proteins and carbohydrates at minimum cost?

r
SOLUTION Suppose x gms of wheat and y grams of irce are mixed in the daily diet.
ou
fo
Since every gram of wheat provides 0.1 gm of proteins and every gram of rice gives 0.05 gm of
ks
proteins. Therefore, x gms of wheat and y grams of rice will provide O.l.r + 0.05y gms of proteins.
But the minimum daily requirement of proteins is of 50 gms.
oo

0.1x + 0.05y>50 => f^ + y > 50


Y
eB

10 20

Similarly, x gms of wheat and y gms of rice will provide 0.25a: + 0.5y gms of carbohydrates and
the minimum daily requirement of carbohydrates is of 200 gms.
r
ou

0.25a:+ 05y > 200 => -4 + ^2 > 200


ad
Y

Since the quantities of wheat and rice cannot be negative. Therefore,


d

a: > 0, y > 0
Re
in

It is given that wheat costs ? 4 per kg and rice ? 6 per kg. So, x gms of wheat and y gms of rice
4a: 6y
will cost ?
F

1000 1000

Hence, the given linear programming problem is


4x
Minimize Z - +
1000 1000
Subject to the constraints
^ +^ >50,
10 20

^ +^
^ ---
> 200,
4 2

and, a: > 0, y > 0


The solution set of the linear constraints is shaded in Fig. 26.18. The vertices of the shaded
region are A2 (800,0), P (400, 200) and (0,1000).
The values of the objective function at these points are given in the following table.
26.41
LINEAR PROGRAMMING

4x
Point{xi, A.2) Value of objective function Z = 1000
+
1000
8
A2 (800, 0) Z — X 800 + X 0 = 3.2
1000I 1000

P (400, 200) Z — X 400 + X 200 = 2.8


1000j 1000

(0,1000) z i- X 0 + X 1000 = 6
1000 1000

Clearly, Z is minimum for ;c = 400, y = 200 and the minimum value ofZis2.8.
y

ow
B,(0,1000)' .T
^ — 50
10 20"^^

e
re
\
rFl
F
62(0,400)

r
P(400,200)
ou
fo
T + |-=200
ks
0 A,(500, 0) /\2(800, 0) X
oo

4-y 6ij
Y
eB

1000 1000

Fig. 26.18
ur

4x
We observe that the open half plane represented by < 2.8 does not have points is
1000 1000
ad
Yo

common with the feasible region. So, Z has minimum value 2.8 at x = 400 and 1/ = 200.
Hence, the diet cost is minimum when .v = 400 and y = 200. The minimum diet cost is ? 2.8.
d
Re
in

EXERCISE 26.3
F

1. A diet of two foods Fj and F2 contains nutrients thiamine, phosphorous and iron. The
amount of each nutrient in each of the food (in milligrams per 25 gms) is given in the
following table:
Food
h h
Nutrients

Thiamine 0.25 0.10

0.75 1.50
Phosphorous
Iron 1.60 0.80

The minimum requirement of the nutrients in the diet are 1.00 mg of thiamine, 7.50 mg of
phosphorous and 10.00 mg of iron. The cost of Fj is 20 paise per 25 gms while the cost of F2 i:
is

15 paise per 25 gms. Find the minimum cost of diet.


26.42 APPLIED MATHEMATICS-XII

2. A diet for a sick person must contain at least 4000 units of vitamins, 50 units of minerals and
1400 of calories. Two foods A and B, are available at a cost of ? 4 and ? 3 per unit
respectively. If one unit of A contains 200 units of vitamin, 1 unit of mineral and 40 calories
and one unit of food B contains 100 units of vitamin, 2 units of minerals and 40 calories, find
what combination of foods should be used to have the least cost?
3. To maintain one's health, a person must fulfil certain minimum daily requirements for the
following three nutrients: calcium, protein and calories. The diet consists of only items I

w
and II whose prices and nutrient contents are shown below:
Food I Food II Minimum daily requirement
Calcium 10 4 20

e
Protein 5 6

ro
20

re
Calories 2 6 12

Price

F
? 0.60 per unit ^ 1.00 per unit

Fl
Find the combination of food items so that the cost may be minimum.

u
4. A hospital dietician wishes to find the cheapest combination of two foods, A and B, that

sr
contains at least 0.5 milligram of thiamin and at least 600 calories. Each unit of A contains
0.12 milligram of thiamin and 100 calories, while each unit of B contains 0.10 milligram of

ko
o
thiamin and 150 calories. If each food costs 10 paise per unit, how many units of each
of
should be combined at a minimum cost?
5. A dietician mixes together two kinds of food in such a way that the mixture contains at least
o
Y
6 units of vitamin A, 7 units of vitamin B, 11 units of vitamin C and 9 units of vitamin D. The
erB

vitamin contents of 1 kg of food X and 1 kg of food Y are given below:


uY

Vitamin Vitamin Vitamin Vitamin


A B C D

FoodX 1 1 1 2
ad
do

FoodY 2 1 3 1
in

One kg of food X costs ? 5, whereas one kg of food Y costs ? 8. Find the least cost of the
mixture which will produce the desired diet.
Re

6. A diet is to contain at least 80 units of vitamin A


F

and 100 units of minerals. Two foods Fj and


p2 are available. Food Fj costs ? 4 per unit and F2 costs ? 6 per unit one unit of food Fj
contains 3 units of vitamin A and 4 units of minerals.One unit of food F2 contains 6 units of
vitamin A and 3 units of minerals. Formulate this as a linear programming problem and
find graphically the minium cost for diet that consists of mixture of these foods and also
meets the mineral nutritional requirements.
7. Kellogg is a new cereal formed of a mixture of bran and rice that contains at least 88 grams
of protein and at least 36 milligrams of iron. Knowing that bran contains 80 grams of
protein and 40 milligrams of iron per kilogram, and that rice contains 100 grams of protein
and 30 milligrams of iron per kilogram, find the minimum cost of producing this new cereal
if bran costs ^ 5 per kg and rice costs ? 4 per kg.
8. A wholesale dealer deals in two kinds, A and B (say) of mixture of nuts. Each kg of mixture
A contains 60 grams of almonds, 30 grams of cashew nuts and 30 grams of hazel nuts. Each
kg of mixture B contains 30 grams of almonds, 60 grams of cashew nuts and 180 grams of
hazel nuts. The remainder of both mixtures is per nuts. The dealer is contemplating to use
26.43
LINEAR PROGRAMMING

mixtures A and B to make a bag which will contain at least 240 grams of almonds, 300 grams
of cashew nuts and 540 grams of hazel nuts. Mixture A costs ? 8 per kg. and mixture B costs
? 12 per kg. Assuming that mixtures A and B are uniform, use graphical method to
determine the number of kg. of each mixture which he should use to minimise the cost of
the bag.
9. One kind of cake requires 300 gm of flour and 15 gm of fat, another kind of cake requires 150
gm of flour and 30 gm of fat. Find the maximum number of cakes which can be made from
7.5 kg of flour and 600 gm of fat, assuming that there is no shortage of the other ingradients
used in making the cakes. Make it as an LPP and solve it graphically.
10. Reshma wishes to mix two types of food P and Q in such a way that the vitamin contents of
the mixture contain at least 8 units of vitamin A and 11 units of vitamin B. Food P costs
? 60 kg and Food Q costs ? 80 kg. Food P contains 3 units/kg of Vitamin A and 5 units/kg of
Vitamin B while food Q contains 4 units/kg of Vitamin A and 2 uiaits/kg of vitamin B.

w
Determine the minimum cost of the mixture.

11. One kind of cake requires 200 g of flour and 25 g of fat, and another kind of cake requires

F lo
100 g of flour and 50 g of fat. Find the maximum number of cakes which can be made from
5 kg of flour and 1 kg of fat assuming that there is no shortage of the other ingredients used
in making the cakes.

ee
12. A dietician has to develop a special diet using two foods P and Q. Each packet (containing

Fr
30 g) of food P contains 12 units of calcium, 4 units of iron, 6 imits of cholesterol and 6 units
for
of vitamin A. Each packet of the same quantity of food Q contains 3 units of calcium,
20 units of iron, 4 units of cholesterol and 3 units of vitamin A. The diet requires atleast
ur
240 units of calcium, atleast 460 units of iron and at most 300 units of cholesterol. How
s

many packets of each food should be used to minimise the am.ount of vitamin A in the diet?
ook
Yo

What is the minimum amount of vitamin A?


eB

13. A farmer mixes two brands P and Q of cattle feed. Brand P, costing ^250 per bag, contains 3
units of nutritional element A, 2.5 units of element B and 2 units of element C. Brand Q
costing ? 200 per bag contains 1.5 units of nutritional element A, 11.25 units of element B,
our
ad

and 3 units of element C. The minimum requirements of nutrients A, B and C are 18 units,
45 units and 24 units respectively. Determine the number of bags of each brand which
should be mixed in order to produce a mixture having a minimum cost per bag? What is the
dY
Re

minimum cost of the mixture per bag?


14, A dietician wishes to mix together two kinds of food X and Y in such a way that the mixture
Fin

contains at least 10 units of vitamin A, 12 units of vitamin B and 8 units of vitamin C. Tlie
vitamin contents of one kg food is given below:
Food Vitamin A Vitamin B Vitamin C

X 1 2 3

Y 2 2 1

One kg of food X costs ? 16 and one kg of food Y costs ? 20. Find the least cost of the mixture
which will produce the required diet?
15. A fruit grower can use two types of fertilizer in his garden, brand P and Q. The amounts
(in kg) of nitrogen, phosphoric acid, potash, and chlorine in a bag of each brand are given in
the table. Tests indicate that the garden needs at least 240 kg of phosphoric acid, at least
270 kg of potash and at most 310 kg of chlorine.
26.44
APPLIED r* ● ~HEMATICS-Xlj

kg per bag
Brand P Brand Q
Nitrogen 3 3.5

Phosphoric acid 1 2

Potash 3 1.5

Chlorine 1.5 2

If the grower wants to minimize the amount of nitrogen added to the garden, how many
bags of each brand should be used? What is the minimum amount of nitrogen added in the
garden?
''Ql. n :>

w
125/2 gm of food 375/4 gm of food F2; Min cost 425/4 Paise
5 units of food A and 30 units of food B

1.875 units of ^ and 2.75 units of B


2 kg of and 4 kg of B; Min cost = ? 64
F lo
Food 1: 3 units. Food II: 1 unit, Min cost ? 2.80
?41 ● ?104 ?4.6

e
20,10

Fre
? 160 at all points on the line segment joining points (8/3,0) and (2,1/2).
30 cakes of one kind and 10 cakes of second kind. for
Food P = 15 packets. Food Q = 20 packets. Minimum amount of vitamin A = 150 units.
Bags of brand P = 3, Bags of brand Q = 15, Max. amount of vitamin = 285 units.
r
; Food X = 2 kg. Food y= 4 kg. Least cost = ? 112.
You
s
ook

Brand P = 40 bags. Brand Q =50 bags. Minimum amount of Nitrogen = 470 kg.
eB

HINTS TO SELECT(=n PROBLEMS

Let 25 X gms of food Fj and 25 y gms of food F2 be used to fulfil the minimum requirements
our

of thiamine, phosphorous and iron. Then, the LLP is


ad

Minimize Z = 2Qx + \5y


Subject to 0.25x + O.lOy > 1
dY

075:c + 150y > 750


Re

1.60X + 0.80y > 10


Fin

and. x,y k 0
Let a: units of food A any y units of food B are used. Then, the LPP is
Minimize Z = 4a: + 3y
Subject to 20Qx + lOOy > 4000
x + 2y >50
40A: + 40y > 1400
and. a:, y > 0
Let X units of food I and y units of food II are used to fulfil minimum daily requirements.
Then, the LPP is
Minimize Z = 0.60a: + y
Subject to 10a: + 4y > 20
5a: + 6y > 20
2A: + 6y > 12
A:,y > 0
26.45
LINEAR PROGRAMMING

4. Let .t units of food A and y units of food B are combined. The LPP is
Minmtize Z = O.l.r + O.ly
Subject to 0.12.V + O.lOy > 05
lOO.v + 150y > 600
and. x,ij>0
5. Let A* kg of food X and y kg of food Y are mixed to produce the desired diet. The LPP is
Minimize Z = 5.v + 8y
Subject to X + 2y > 6
x + y >7
x+3y>ll
2.V + y >9
and. x,y > 0
Let the cereal contain .v kg of bran and y kg of rice. Then, the LPP is

w
7.

Minimize Z = 5.y + 4y
80 100
Subject to XX.
>
or. 20y + 25y > 22

F lo
+ yx
lOOO 1000 1000
40 30 36
XX + y X
>
or, 20.V + 15y > 18

ee
100000 100000 100000
.r, y > 0

Fr
26.9.2 OPTIMAL PRODUCT LINE PROBLEMS
for
In this type of problems, we have to determine the number of different products which should
ur
be produced and sold by a firm when each product requires a fixed manpower, machine hours,
labour hours per unit of the product, ware house space per unit of output, etc. in order to make
s
maximum profit.
ok
Yo

Following are some examples on this type of problems.


o
eB

ILLUSTRATIVE EXAMPLES

industrial machinery. It takes 1 hour of work


r

EX.^MPLE 1 A mmiufacturer produces nuts and bolts for


ad
ou

on machine A and 3 hours on machin B to produce a package of nuts xohileit takes 3 hours on macineAand
1 hour on machine B to produce a package of bolts. He earns a profit of^ 2.50 per package of nuts and Re
Y

1.00 per package of bolts. How many packages of each should he produce each day so as to ma.ximize his
profit, if he operates his machines for at most 12 hours a day? Formulate this mathematically and then
Re
nd

solve it.
Fi

SOLUTION The given information can be summarized in the following tabular form:
Time required to produce products Max. Machine hours available
Machines
Nut Bolt

1 3 12
A
3 1 12
B

2.50 1.00
Profit (in ?)

Let the manufacturer produce y packages of nuts and y packages of bolts each day.
Since machine A takes one hour to produce one package of nuts and 3 hours to produce one
package of bolts. Therefore, the total time required by machine /4 to produce y packages of nuts
and y packages of bolts is (y + 3y) hours. But machine A operates for at most 12 hours.
Y + 3y < 12
26.46
APPLIED MATHEMATICS-Xll

Similarly, the total time required by machine B to produce packages of nuts and y packages of
bolts IS (3:r + y) hours. But machine B operates for at most 12 hours.
3.r + y < 12
Since the profit on one package of nuts is ? 2.50 and on one package of bolts the profit is ? 1.
Therefore, profit on .V packages of nuts and y packages of bolts is of ?(250.v + v). Let Z denote
the total profit. Then, Z = 2.50.V + y.
Clearly, a: > 0 and y > 0
Thus, the above LPP can be stated mathematically as follows:
Maximize Z = 2.50 .v + y
Subject to X + 3y < 12
3.V -M/ < 12
and, ■T,y > 0
To solve this LPP graphically, we first convert the inequations into equations to obtain the

w
following equations.
-Y + 3y = 12, 3y + y = 12, y = 0, y = 0

Flo
The line y + 3y = 12 meets the coordinate axes at {12, 0) and (0, 4). Join these two points to
obtain the line represented by y + 3y = 12. The region represented by the inequation y + 3y < 12
IS the region containing the origin as y = 0, y = 0 satisfies the inequation y + 3y < 12.

e
re
The line 3y + y = 12 meets the coordinate axes at A2 (4, 0) and B2 (0,12). Join these points to

rF
obtain the line represented by 3y + y = 12. Since y = 0, y = 0 satisfies the inequation 3y + y < 12.
So, the region containing the origin and below the line 3Y + y=12 represents the region
represented by 3y + y < 12. ®
ur
Clearly, y > 0 and y > 0 represent all points in first quadrant.
fo
Thus, the shaded region OA2 PBj in Fig. 26.19 represents the feasible region of the given LPP.
ks
Yo
The coordinates of the corner-points of the feasible region OA2 PBi are O (0, 0), A2 (4, 0),
oo

P (3, 3) and 6^ (0, 4). These points are obtained by solving the corresponding intersecting lines
simultaneously. °
B

y
re

\
u
ad

82(0,12)
Yo
d
Re
in
F

B,(0, 4) P (3,3)

O
^2(4,0) A,(12, 0) X

3x + y = 12
Fig. 26.19

The values of the objective function at the corner-points of the feasible region are given in the
following table:
26.47
LINEAR PROGRAMMING

Point {x, y) Value of the objective function Z = 250j: + \j


Z= 250x0 + lx0 = 0
0(0, 0)
Z = 250 X 4 + 1 X 0 = 10
^2 (4, 0)
Z = 250x3 + 1x3 = 1050
P(3, 3)
2 = 250x0 + 1x4=4
(0, 4)

Clearly, Z is maximum at x = 3, y = 3 and the maximum value of Z is 10.50.


Hence, the optimal production strategy for the manufacturer wiU be to manufacture 3 packages
each of nuts and bolts daily and in this case his maximum profit will be ? 10.50.
EXAMPLE 2 oil company requires 12,000, 20,000 and 15,000 barrels
of high-grade, medium grade
and low grade oil, respectively. Refinery A produces 100, 300 and 200 barrels per day of high-grade,
medium-grade and low-grade oil, respectively, while refinery B produces 200,400 and 100 barrels per day

w
of high-grade, medium-grade and low-grade oil, respectively. If reifnery A costs ? 400 per day and refinery
B costs ? 300 per day to operate, how many days should each be run to minimize costs ivhile satisfying

Flo
requirements.
SOLUTION The given data may be put in the following tabular form:

ee
High-grade Medium-grade Low-grade Cost per day

Fr
Refinery
300 200 ?400
A 100
?300
200 400
for 100
ur
B

Minimum Requirement 12,000 20,000 15,000


ks

Suppose refineries A and 6 should run for X and y days respectively to minimize the total cost.
Yo
oo

The mathematical form of the above LPP is


eB

Y
r
ou
ad
Y
nd
Re
Fi

100.x + 200t/ = 12000

Minimize Z =400.r + 300y


26.48
APPLIED MATHEMATICS-XII

Subject to
100.1- + 2001/ > 12,000
300x4-400y > 20,000
200.1-4-1001/ > 15,000
and, A',y > 0
The feasible region of the above LPP is represented by the shaded region in Fie 26 20 The
comer points of the feasible region are (120, 0), P (60, 30) and S3 (0,150). The value of the
objechve function at these points are
given in the following table:
Point (.Y, y)
Value of the objective function Z = 400x4-300y
A2 (120, 0) Z = 400 X 120 4- 300 X 0 = 48000

w
P (60, 30) Z = 400 X 60 4- 300 X 30 = 33000
B3 (0,150)

F lo
Z = 400 X 0 4- 300 X 150 = 45000

pearly, Z is minimum when ,r = 60, y = 30. The feasible region is unbounded. So, we find the

ee
half-plane represented by 400.V -t 300y < 33000. Clearly,
the half-plane does not have points

Fr
common with the feasible region. So, Z is minimum at x =60, y = 30.
Hence, the machine A should run for 60 days and the machine B should run for 30 days to
mmimize the cost while satisfying the constraints. for
ur
EXAMPLE 3
■4 rx, f <^ompati^ produces soft drinks that has a contract ivhich requires that a minimum of 80
units of the chemical A and 60 units of the chemical B logo into each bottle of the drink. The chemicals are
ks
available in a prepared mix from two different suppliers. Supplier S has a mix of 4 units ofA and 2 units of
Yo

B that costs ? 10, the supplier T has a mix ofl unit of A and 1 unit ofB that costs ^4. How many mixes
oo

from S and T should the company purchase to honour contract requirement and yet minimize cost?
eB

SOLUTION The given data may be put in the following tabular form:
Supplier
r
ou

Minimum
ad

S T
Chemical
Requirement
Y

A 4 1 80
B 2
nd

1
Re

60
Cost per unit ?10 ?4
Fi

Suppose runits of mix are purchased from supplier Sand y units are purchased from supplier T
Total cost Z = 10x4-4y.
Units of chemical A per bottle = 4x + y. But the minimum requirement of chemical A per bottle
IS 80 umts.

4x4-y > 80
Similarly, 2x 4-y > 60. Clearly, A > 0, y > 0
Thus, the mathematical formulation of the given LPP is
Minimize Z = lOx 4- 4y
Subject to
4x4-y > 80
2x 4- y > 60
and. A > 0,y > 0
26.49
LINEAR PROGRAMMING

Now, we find the feasible region which is the set of all points whose coordinates simultaneously
satisfy all constraints including non-negativity restrictions. The shaded region in Fig. 26.21
represents the feasible region of the given LPP. The coordinates of the comer points of the
feasible region are A2 (30, 0), P (10, 40), (0, 80).
y

10a- + 41/ = 260

B,(0,80)

w
82 4x + y = 80

P (10,40)

Flo
ee
2x + y = 60

Fr
A

^2
O
for X
ur
2at + 1/ = 80
s
4.V + y = 80
k
Yo
oo

Fig. 26.21
These points are obtained by solving the equations of the corresponding intersecting lines,
eB

simultaneously.
The values of the objective function at these points are given in the following table:
r

Point {x, y) Wahic of objective function Z = 10a: + 4y


ou
ad

A2 (30, 0) Z = lOx 30 + 4x 0 = 300


Y

P (10, 40) Z = 10xl0 + 40x 4 = 260

Bi (0, 80) Z = 10 X 0 + 4 X 80 = 320


Re
nd

Clearly, Z is minimum at (10, 40). The feasible region is unbounded and the open half plane
Fi

represented by lO.r + 4y < 260 does not have points in common with the feasible region. So, Z is
minimum at a: = 10,1/ = 40. Hence, .v = 10, 1/ = 40 is the optimal solution of the given LPP.
Hence, the cost per bottle is minimum when the company purchases 10 mixes from supplier S
and 40 mixes from supplier T.
EXAMPLE 4 A dealer wishes to purchase a number offans and sewing machines. He has only ^ 5760.00
to invest and has space for at most 20 items. /I fan costs him ? 360.00 and a sezoitig machine ? 240.00. His
expectation is that he can sell a fan at a profit of^ 22.00 and a sewing machine at a profit of^ 18.00.
Assuming that he can sell all the items that he can buy, hozv should he invest his money in order to
maximize his profit ? Translate this problem mathematically and then solve it.
SOLUTION Suppose the dealer buys x fans and y sewing machines. Since the dealer has space
for at most 20 items. Therefore,
X + y < 20
A fan costs ? 360 and a sewing machine costs ^ 240. Therefore, total cost of a: fans and y sewing
machines is ? ( 360a: + 240i/). But the dealer has only ? 5760 to invest. Therefore,
360.V + 240y < 5760
26.50
APPLIED MATHEMATICS-XII

Since the dealer can sell all the items that he can buy and the profit on a fan is of ? 22 and on a

sewing machine the profit is of ? 18. Therefore, total profit on selling .v fans and \j sewing
machines is of ? (22.v + 18i/).
Let Z denote the total profit. Then, Z = 22.r + 18y.
Clearly, x,\j >0.
Thus, the mathematical formulation of the given problem i
IS

Maximize Z - 22x + 18y


Subject to
x + y < 20
360x + 240y < 5760
and, a: > 0,1/ > 0
To solve this LPP graphically, we first convert the inequations into equations and draw the

w
corresponding lines. The feasible region of the LPP is shaded in Fig. 26.22. The comer points of
the feasible region OA2 PB-^ are O (0, 0), A2 (16, 0), P (8,12) and (0, 20).

F lo
ee
24)

Fr
B,(0, 20)
for
ur
P(8,12)
s
ook
Yo

fix
. K'l
eB
r
ad
ou

o
A2i16,0) A.(20,0) X
Y

360;t + 2401/ = 5760 x + y = 20


Re

Fig. 26.22
nd

These points have been obtained by solving the corresponding intersecting lines,
Fi

simultaneously.
The values of the objective function Z at corner-points of the feasible region are given in the
following table.

Point (x, y) Value of the objective function Z = 22a: + 18y


0(0 0) Z=22x0 + 18x0 = 0
'^2 (16, 0) Z = 22xl6 + 18x 0 = 352
P(8,12) Z = 22 X 8 + 18 X 12 = 392
B] (0, 20) Z = 22 X 0 + 20 X 18 = 360

Clearly, Z is maximum at a: - 8 and y = 12. The maximum value of Z is 392.


Hence, the dealer should purchase 8 fans and 12 sewing machines to obtain the maximum profit
under given conditions.
26.51
LINEAR PROGRAMMING

EXAMPLE 5 A farm is engaged in breeding pigs. The pigs are fed on vanotis pro(^cts grozvn o^Jh^
constituents (call them X, Y nndZ), it is riecessary to
farm. In view of the need to ensure certain nutrient
buy two additional products, say, and B. One unit of product A contams ^ umfs qf Y, ^
20 units ofZ. One unit of product B contains 6 units ofX, 12 units ofY and 10 units ofZ. Theminitnum
requirement ofX, Y and Z is 108 units, 36 units and 100 units respectively. Product A costs ? 20 per
unitand product B costs f 40 per unit. Formulate the above ns a linear programming problem to minimize
the total cost, and solve the problem by using graphical method.
SOLUTION The data given in the problem can be summarized in the following tabular form:
Nutrient constituent

ow
Cost in ?
Product Y Z
X
3 20 20
A 36
12 10 40
B 6
108 36 100
Minimum Requirement

e
Let X units of product A and y units of product B are bought to fulfill the minimum requirement

re
of X, y and Z and to minimize the cost.

Flr
The mathemahcal formulahon of the above problem is as follows:

F
Minimize Z = 20x + 40y
Subject to 36x + 6y > 108
3x + 12y > 36
ou
sr
20x +lOy > 100

fo
and, x,y,z > 0
The set of all feasible solutions of the above LPP k
is represented by the feasible region shaded
darkly in Fig. 26.23. The coordinates of the corner points of the feasible region ared2 (12, U),
oo
Pi (4, 2), P2(2,6)andBi(0,18).
Y
Y
reB
uY

B
ad
do
in
Re
F

X
26.52
APPLIED MATHEMATICS-XII

Now we have to find a point or points in the feasible region which give the minimum value of
objective function For this, let us give some value to Z, say 20, and draw a dotted line
20 - 20^ + AOy. Now, draw lines parallel to this line which have at least one point common to the
feasible region and loc^e a line which is nearest to the origin and has at least one point common
to he feasible region. Clearly, such a line is = 20a- + 40i/ and it has a point (4, 2) common
ith the feasible region. Thus, Zj = 20a + 40y is the minimum value of Z, and the feasible
soluhon which gives this value of Z is the corner (4, 2) of the shaded region. The values of the
variables for the ophmal solution are a = 4, y = 2. Substituting these values in Z = 20r + 40i/ we
get Z = 160 as the optimal value of Z.
Hence, 2 units of product /I and 4 units of product B are sufficient to fulfill the minimum
requirement at a minimum cost of ? 160.
EX.AMPLE6

dollB. Each doll of txjpeB takes twice as long to produce as one doll of type Z.. The company have time to

w
make a maximum of2000 dolls of type A per day, the supply of plastic is sufficient to produce 1500 dolls
Perdayand each type requires equal amount of it. The deluxe version, i.e. type B requires a fancy dress of
which there are only 600 per day available. If the company makes a proift of?3 and ? 5 uer doll

F lo
respectively, on doll A and B; how many of each should be produced per day in order to maximize profit ?
Solve It by graphical method. ^ e j ■

ee
^obt^'^*^ ^ ^ ^ produced per day to maximize the

Fr
The mathematical form of the given LPP is as follows:
Ma.ximize Z ~ 3a + 5y
for
ur
Subject to X + 2y < 2000
a + y < 1500
(See Ex.2 on page 26.5)
y < 600
ks
and. a, y > 0
Yo
oo

The set of all feasible ^lutions of the given LPP is represented by the feasible region shaded
eB

coordinates of the corner points of the feasible region areO (0 0)


^2 (1500, 0) P (1000,500), Q (800, 600) and R (0,600). ® ' '
Now, to find a point or points in the feasible region which give the maximum value of the
r

objechve function Z = 3a + 5y, let us give some value to Z, say 1500 and draw the dotted
ou
ad

line
3a + 5y = 1500 as shown in Fig. 26.23.
Y

Y
&
nd
Re

//
is
Fi

Z, = 3j + 5y ”T

-1- ^2(0,1500)

%
B 1
Q {800, 600)
y = 600 R{0,60~^
P(1000,500)

X’ O
o’:V- a; X

^*V
r

Fig. 26.24
26.53
LINEAR PROGRAMMING

and obtain a line which is farthest from the


Now, draw lines parallel to the line 3.x + by = 1500
origin and have at least one point common to the feasible region. Clearly,
= 3.r + by is such a
line. This line has only one point P (1000,500) common to the feasible region. Thus,
Z = 3 X 1000 + 5 X 500 =5500 is the maximum value of Z and the optimal solution is .v =1000,
i/=500.
Hence, 1000 dolls of type A and 500 dolls of type B should be produced to maximize the profit
and the maximum profit is ? 5500.
EXERCISE 26.4

1. If a young man drives his scooter at a speed of 25 km/hr, he has to spend ? 2 per km on
petrol. If he drives the scooter at a speed of 40 km/hour, it produces air polution and
He has a maximum of ? 100 to spend on
increases his expenditure on petrol to ? 5 per km.
petrol and travel a maximum distance in one hour time with less polution. Express this

w
problem as an LPP and solve it graphically. What value do you find here?
2. A manufacturer has three machines installed in hisfactory. Machines I and II are capable of
being operated for at most 12 hours whereas Machine III must operate at least for 5 hours a

F lo
day. He produces only two items, each requiring the use of three machines. The number of
hours required for producing one unit each of the items on the three machines is given in

ee
the following table:

Fr
Item Number of hours required by the machine
I II
for III
ur
A 1 2 1
oks

B 2 1 5/4
Yo
o

He makes a profit of ? 6.00 on item A and ? 4.00 on item B. Assuming that he can sell all that
eB

he produces, how many of each item should he produce so as to maximize his profit?
Determine his maximum profit. Formulate this LPP mathematically and then solve it.
our
ad

3. Two tailors, A and B earn ?15 and ?20 per day respectively. A can stitch 6 shirts and 4
pants while B can stitch 10 shirts and 4 pants per day. How many days shall each work if it
is desired to produce (at least) 60 shirts and 32 pants at a minimum labour cost?
Y

4. A factory manufactures two types of screws, A and B, each type requiring the use of two
Re
nd

machines — an automatic and a hand-operated. It takes 4 minute on the automatic and


6 minutes on the hand-operated machines to manufacture a package of screws 'A', while it
Fi

takes 6 minutes on the automatic and 3 minutes on the hand-operated machine to


manufacture a package of screws 'B'. Each machine is available for at most 4 hours on any
day. The manufacturer can sell a package of screws 'A' at a profit of 70 P and screws 'B' at a
profit of ^ 1. Assuming that he can sell all the screws he can manufacture, how many
packages of each type should the factory owner produce in a day in order to maximize his
profit? Determine the maximum profit.
5. A company produces two types of leather belts, say type A and B. Belt A is a superior
quality and belt 6 is of a lower quality. Profits on each type of belt are ? 2 and? 1.50 per
belt, respectively. Each belt of type A requires twice as much time as required by a belt of
type 6. If all belts were of type B, the company could produce 1000 belts per day. But the
supply of leather is sufficient only for 800 belts per day (both A and B combined). Belt A
requires a fancy buckle and only 400 fancy buckles are available for this per day. For belt of
type 6, only 700 buckles are available per day.
26.54
APPLIED MATHEMATICS-XII

How should the company manufacture the two types of belts in order to have a maximum
overall profit?
6. A smaU manufacturer has employed 5 skilled men and 10 semi-skilled men and makes an
article in two qualities deluxe model and an ordinary model. The making of a deluxe model
requires 2 hrs. work by a skilled man and 2 hrs. work by a semi-skilled man. The ordinary
model requires 1 hr by a skilled man and 3 hrs. by a semi-skiUed man. By union rules no
man may work more than 8 hrs per day. The mamifacturers clear profit on deluxe model is
? 15 and on an ordinary model is ? 10. How many of each type should be made in order to
maximize his total daily profit.
7. A manufachurer makes two types A and 6 of tea-cups. Three machines are needed for the
manufacture and the time in minutes required for each cup on the machines is given below;

ow
Machines

I n m
A 12 18 6

e
B 6 0 9

Fl
re
Each machine is available for a maximiun of 6 hours per day. If the profit on each cup A is

F
75 paise and that on each cup B is 50 paise, show that 15 tea-cups of type A and 30 of type B
should be manufactured in a day to get the maximum profit.
ur
r
8. A factory owner purchases two types of machines, A and B, for his factory, The
requirements and limitations for the machines are as follows: fo
ks
Area occupied by the
Yo
Labourforce for each Daily output in
oo

machine machine units


Machine A 1000 sq. m 12 men 60
eB

Machine B 1200 sq. m 8 men 40


ur

He has an area of 7600 sq.m available and 72 skilled men who can operate the machines.
How many machines of each type should he buy to maximize the daily output?
ad
Yo

9. A company produces two types of goods, A and B, that require gold and silver. Each unit of
type A requires 3 gm of silver and 1 gm of gold while that of type B requires 1 gm of silver
d

and 2 gm of gold. The company can produce 9 gm of silver and 8 gm of gold. If each unit of
Re
in

type A brings a profit of ? 40 and that of type B ? 50, find the number of units of each type
that the company should produce to maximize the profit. What is the maximum profit?
F

10. A manufacturer of Furniture makes two products : chairs and tables. Processing of these
products is done on two machines A and B. A chair requires 2 hrs on machine A and 6 hrs on
machine B. A table requires 4 hrs on machine A and 2 hrs on machine B. There are 16 hrs of
time per day available on machine A and 30 hrs on machine B. Profit gained by the
manufacturer from a chair and a table is ? 3 and ? 5 respectively. Find with the help of graph
what should be the daily production of each of the two products so as to maximize his
profit.
11. A furniture manufacturing company plans to make two products: chairs and tables. From
its available resources which consists of400 square feet of teak wood and 450 man hours. It
is known that to make a chair requires 5 square feet of wood and 10 man-hours and yields a
profit of ? 45, while each table uses 20 square feet of wood and 25 man-hours and yields a
profit of ? 80. How many items of each product should be produced by the company so that
the profit is maximum?
26.55
LINEAR PROGRAMMING

12. A firm manufactures two products A and B. Each product is processed on two machines
and M2. Product A requires 4 minutes of processing time on M^ and 8 mm. onM2; product
B requires 4 minutes on Mj and 4 min. on M2 . The machine M| is available for not mwe
than 8 hrs 20 min. while machine M2 is available for 10 hrs. during any working day. The
products A and B are sold at a profit of ? 3 and ? 4 respectively.
Formulate the problem as a linear programming problem and find how many products of
each type should be produced by the Hrm each day in order to get maximum profit.
13. A firm manufacturing two types of electric items, A and B, can make a profit of ^ 20 per
unit of A and ? 30 per unit of B. Each unit of A requires 3 motors and 4 transformers and
each imit of B requires 2 motors and 4 transformers. The total supply of these per month is
restricted to 210 motors and 300 transformers. Type 6 is an export model requiring a

ow
voltage stabilizer which has a supply restricted to 65 units per month. Formulate the linear
programing problem for maximum profit and solve it graphically.
14. A factory uses three different resources for the manufacture of two different products, 20
units of the resources A, 12 units of B and 16 units of C being available. 1 unit of the first
and 1 unit of the second

e
product requires 2, 2 and 4 units of the respective resources
product requires 4,2 and 0 units of respecHve resources. It is known that the first product

re
rFl
gives a profit of 2 monetary units per unit and the second 3. Formulate the linear

F
programming problem. How many units of each product should be manufactured for
maximizing the profit? Solve it graphically.

r
A publisher sells a hard cover edition of a text book for ? 72.00 and a paperback edition of
ou
13.
fo
the same ext for ? 40.00. Costs to the publisher are ? 56.00 and ? 28.00 per book respectively
ks
in addition to weekly costs of ? 9600.00. Both types require 5 minutes of printing time,
although hardcover requires 10 minutes binding time and the paperback requires only
oo

2 minutes. Both the printing and binding operations have 4,800 minutes available each
week. How many of each type of book should be produced in order to maximize profit?
Y
eB

16. A firm manufactures headache pills in two sizes A and B. Size A contains 2 grains of aspirin,
5 grains of bicarbonate and 1 grain of codeine; size B contains 1 grain of aspirin, 8 grains of
r

bicarbonate and 66 grains of codeine. It has been found by users that it requires at least 12
ou

grains of aspirin, 7.4 grains of bicarbonate and 24 grains of codeine for providing
ad
Y

immediate effects. Determine graphically the least number of pills a patient should have to
get immediate relief. Determine also the quantity of codeine consumed by patient.
d

7. A chemical company produces two compounds, A and B. The following table gives the
Re
in

units of ingredients, C and D per kg of compounds A and B as well as minimum


requirements of C and D and costs per kg of A and B. Find the quantities of A and B which
F

would give a supply of C and Data minimum cost.


Compound Minwium requirement
A B

1 2 80
Ingredient C
3 1 75
Ingredient D
4 6
Cost (in ? ) per kg
18. A company manufactures two types of novelty Souvenirs made of plywood. Souvenirs of
type A require 5 minutes each for cutting and 10 minutes each for assembling. Souvenirs of
type B require 8 minutes each for cutting and 8 minutes each for assembling. There are 3
hours 20 minutes available for cutting and 4 hours available for assembling. The profit is 50
paise each for type A and 60 paise each for type B souvenirs. How many souvenirs of each
type should the company manufacture in order to maximize the profit?
26.56
APPLIED MATHEMATICS-XII

19. A m^ufacturermakes two products A and B. Product A sells at ? 200 each and takes
1/2 hour to make. Product B sells at ? 300 each and takes 1 hour to make. There is a
permanent order for 14ofproductAandl6ofproductB.Aworkingweek consists of 40
Hours of production and weekly turnover must not be less than? 10000. If the profit on
each of product A is ? 20 and on product B is ? 30, then how many of each should be
produced so that the profit is maximum. Also, find the maximum profit.
20. A manufacturer produces two types of steel trunks. He has two machines A and B For
completing, *e first types of the trunk requires 3 hours on machine A and 3 hours on
machme B, whereas the second type of the trunk requires 3 hours on machine A and 2 hours
on machine B. Machines /I and B can work at most for 18 hours and 15 hours per dav
r^pectrvely. He earns a profit of ? 30 and ? 25 per trunk of the first type and the second
type respectively. How many trunks of each type must he make each day to make

w
maximum profit? ^
21. A manufacturer of patent medicines is preparing a production plan on medicines, A and B.
There ^e sufhaent raw materials available to make 20000 bottles of A and 40000 bottles of
B, but there are only 45000 bottles into which either of the medicines can be put. Further it

o
takes 3 hours to prepare enough material to fiU 1000 bottles of A, it takes 1 hour to prepie

e
l^ough material to fm 1000 bottles of B and there are 66 hours available for this opiation

re
rFl
The profit IS ? 8 per bottle for A and ? 7 per bottle for B. How should the manufacturer

F
schedule his production in order to maximize his profit?
22. aeroplane can carry a maximum of 200 passengers. A profit of ?400 is made on each first
class hcket and a profit of ? 600 is made on each economy class ticket. The airline reserves at

r
ou
least 20 seats of first class. However, at least 4 times as many passengers prefer to travel by
fo
economy class to the first class. Determine how many each type of tickets must be sold in
ks
order to maximize the profit for the airline. What is the maximum profit.
23. A gardener has supply of fertilizer of type I which consists of 10% nitrogen and 6% phos
oo

phoric acid and type II fertilizer which consists of 5% nitrogen and 10% phosphoric acid,
Y
eB

ter testog the soil conditions, he finds that he needs at least 14 kg of nitrogen and 14 kg of
phosporic acid for his crop. If the type I fertilizer costs 60 paise per kg and type II fertilizer
costs 40 paise per kg, determine how many kilograms of each fertilizer should be used
ur

so
that nutrient requirements are met at a minimum cost. What is the minimum cost?
ad

24. ^ wants to invest at most ? 12000 in Saving Certificates and National Saving Bonds
Yo

^ ^000 in Saving Certificates and at least


? 4000 m National Savmg Bonds. If the rate of interest on saving certificate is 8% per annum
d

and Ae rate of interest on National Saving Bond is 10% per annum, how much money
Re
in

should he mvest ot earn maximum yearly income ? Find also his maximum yearly income,
^ea 1000 sq.m. He wants to plant fruit trees in it. He has a sum of
F

? 1400 to purchase young trees. He has the choice of two types of trees. Type A requires 10
sq.m of ground per tree and costs ? 20 per tree and type B requires 20 sq.m of ground per
tree ^d costs ? 25 per tree. When fully grown, type A produces an average of 20 kg of fruit
wluch can be sold at a profit of ? 2.00 per kg and t^e B produces an average of 40 kg of fruit
wtoch can be sold at a profit of ? 1.50 per kg. How many of each type should be planted to
achieve maximum profit when the trees are fully grown ? What is the maximum profit?
26. A cottage industry manufactures pedestal lamps and wooden shades, each requiring the
use of grmding/cutting machine and a sprayer. It takes 2 hours on the grinding/cutting
machme^and 3 hours on the sprayer to manufacture a pedestal lamp while it takes 1 hour on
e gr^dmg/cutting machine and 2 hours on the sprayer to manufacture a shade. On anv
day *e spraye^ avaUable for at most 20 hours and the grinding/cutting machine for at
most 12 hours The profit from the sale of a lamp is ? 5.00 and a shade is ? 3.00. Assuming
manufacturer can sell aU the lamps and shades that he produces, how should he
schedule his daily production in order to maximize his profit?
LINEAR PROGRAMMING 26.57

27. A producer has 30 and 17 units of labour and capital respectively which he can use to
produce two type of goods X and Y. To produce one unit of X, 2 units of labour and 3 units
of capital are required. Similarly, 3 units of labour and 1 unit of capital is required to
produce one unit of Y. If X and Y are priced at ^ 100 and ? 120 per unit respectively, how
should be producer use his resources to maximize the total revenue? Solve the problem
graphically.
28. A firm manufactures two types of products A and B and sells them at a profit of ? 5 per

w
unit of t)q5e A and ? 3 per unit of type B. Each product is processed on two machines
Mj and M2. One unit of type A requires one minute of processing time on and two
minutes of processing time on M2, whereas one unit of type B requires one minute of
processing time on Mj and one minute on M2. Machines M^ and M2 are respectively
available for at most 5 hours and 6 hours in a day. Find out how many units of each type of

e
product should the firm produce a day in order to maximize the profit. Solve the problem

re
ro
graphically.
29. A small firm manufacturers items A and B. The total number of items A and B that it can
manufacture in a day is at the most 24. Item A takes one hour to make while item B takes

F
only half an hour. The maximum time available per day is 16 hours. If the profit on one unit

Ful
of item A be ^ 300 and one unit of item B be ? 160, how many of each type of item be
produced to maximize the profit? Solve the problem graphically.
30. A company manufactures two types of toys A and B. Type A requires 5 minutes each for

sr
cutting and 10 minutes each for assembling. Type B requires 8 minutes each for cutting and

ko
o
8 minutes each for assembling. There are 3 hours available for cutting and 4 hours available
for assembling in a day. The profit is ? 50 each on type A and ? 60 each on type B. How many
of
toys of each type should the company manufacture in a day to maximize the profit?
31. A company manufactures two articles A and B. There are two departments through which
o
Y
these articles are processed: (i) assembly and (ii) finishing departments. The maximum
erB

capacity of the first department is 60 hours a week and that of other department is 48 hours
per week. The product of each unit of article A requires 4 hours in assembly and 2 hours
uY

in finishing and that of each unit of B requires 2 hours in assembly and 4 hours in finishing.
If the profit is ? 6 for each unit of A and ? 8 for each unit of B, find the number of units of A
and B to be produced per week in order to have maximum profit.
32. A firm makes items A and B and the total number of items it can make in a day is 24. It takes
o
ad
d

one hour to make an item of A and half an hour to make an item of B. Ilie maximmn
time available per day is 16 hours. The profit on an item of A is ? 300 and on one item of B is
in

? 160. How many items of each type should be produced to maximize the profit? Solve the
problem graphically.
Re

33. A company sells two different products, A and B. The two products are produced in a
F

common production process, which has a total capacity of 500 man-hours. It takes 5 hours
to produce a unit of A and 3 hours to produce a unit of B. The market has been surveyed and
company officials feel that the maximum number of units of A that can be sold is 70 and that
for B is 125. If the profit is ? 20 per imit for the product A and ^ 15 per unit for the product B,
how many units of each product should be sold to maximize profit?
34. A box manufacturer makes large and small boxes from a large piece of cardboard. The large
boxes require 4 sq. metre per box while the small boxes require 3 sq. metre per box. The
manufacturer is required to make at least three large boxes and at least twice as many small
boxes as large boxes. If 60 sq. metre of carboard is in stock, and if the profits on the large and
small boxes are f 3 and ? 2 per box, how many of each should be made in order to maximize
the total profit?
35. A manufacturer makes two products, A and B. Product A sells at ? 200 each and takes 1/2
hour to make. Product B sells at ? 300 each and takes 1 hour to make. There is a permanent
order for 14 units of product A and 16 imits of product B. A working week consists of
40 hours of production and the weekly turn over must not be less itian ? 10000. If the profit
on each of product A is ? 20 and an product B is ? 30, then how many of each should be

produced so that the profit is maximum? Also find the maximum profit.
26.58
APPLIED MATHEMATICS-XII

36. If a young man drives his vehicle at 25 km/hr, he has to spend ? 2 per km on petrol. If he
drives it at a faster speed of 40 km/hr, the petrol cost increases to ? 5/per km. He has
? 100 to spend on petrol and travel within one hour. Express this as an LPP and solve the
same.

37. An oil company has two depots, A and B, with capacities of 7000 litres and 4000 litres
respectively. The company is to supply oil to three petrol pumps, D,£,F whose
requirements are 4500,3000 and 3500 litres respectively. The distance (in km) between the
depots and petrol pumps is given in the following table:
Distance (in km)

A B

7 3

w
6 4
3 2

F lo
Assuming that the transportation cost per km is ? 1.00 per litre, how should the delivery be
scheduled in order that the transportation cost is minimum?
38. A small firm manufactures gold rings and chains. The total number of rings and chains

ee
manufactured per day is atmost 24. It takes 1 hour to make a ring and 30 minutes to make a

Fr
chain. The maximum number of hours available per day is 16. If the profit on a ring is ? 300
for
and that on a chain is ? 190, find the number of rings and chains that should be
manufactured per day, so as to earn the maximum profit. Make it as an LPP and solve it
ur
graphically.
39. A library has to accommodate two different types of books on a shelf. The books are 6 cm
s
ook
Yo

and 4 cm thick and weigh 1 kg and 1 ~ each respectively. The shelf is 96 cm long and
eB

atmost can support a weight of 21 kg. How should the shelf be filled with the books of two
types in order to include the greatest number of books? Make it as an LPP and solve it
our

graphically.
ad

40. A factory makes tennis rackets and cricket bats. A tennis racket takes 1.5 hours of machine
time and 3 hours of craftman's time in its making while a cricket bat takes 3 hours of
Y

machine time and 1 hour of craftman's time. In a day, the factory has the availability of not
Re

more than 42 hours of machine time and 24 hours of craftman's time. If the profit on a racket
nd

and on a bat is ? 20 and ? 10 respectively, find the number of tennis rackets and cricket bats
Fi

that the factory must manufacture to earn the maximum profit. Make it as an LPP and solve
it graphically.
41. A merchant plans to sell two types of personal computers a desktop model and a portable
model that will cost ? 25,000 and ? 40,000 respectively. He estimates that the total monthly
demand of computers will not exceed 250 units. Determine the number of units of each type
of computers which the merchant should stock to get maximum profit if he does not want
to invest more than ? 70 lakhs and his profit on the desktop model is ? 4500 and on the
portable model is ? 5000. Make an LPP and solve it graphically.
42. A cooperative society of farmers has 50 hectare of land to grow two crops X and Y. The
profit from crops X and Y per hectare are estimated as ^ 10,500 and ? 9,000 respectively. To
control weeds, a liquid herbicide has to be used for crops X and Y at rates of 20 litres and 10
litres per hectare. Further, no more than 800 litres of herbicide should be used in order to
protect fish and wild life using a pond which collects drainage from this land. How much
land should be allocated to each crop so at to maximise the total profit of the society?
26.59
LINEAR PROGRAMMING

43. A manufacturing company makes two models A and 6 of a product. Each piece of Model A
requires 9 labour hours for fabricating and 1 labour hour for finishing. Each piece of Model
B requires 12 labour hours for fabricating and 3 labour hours for finishing. For fabricating
and finishing, the maximum labour hours available are 180 and 30 respectively. Tlae
company makes a profit of ? 8000 on each piece of model A and ? 12000 on each piece of
Model 6. How many pieces of Model A and Model B should be manufactured per week to
realise a maximum profit? What is the maximum profit per week?
44. A factory makes tennis rackets and cricket bats. A tennis racket takes 1.5 hours of machine
time and 3 hours of craftman's time in its making while a cricket bat takes 3 hour of machine
time and 1 hour of crafhnan's time. In a day, the factory has the availability of not more than
42 hours of machine time and 24 hours of craftsman's time,
(i) What number of rackets and bats must be made if the factory is to work at full
capacity?
(ii) If the profit on a racket and on a bat is ? 20 and 110 respectively, find the maximum

w
profit of the factory when it works at full capacity.
45. A merchant plans to sell two types of personal computers a desktop model and a portable

F lo
model that will cost ? 25000 and ^ 40000 respectively. He estimates that the total monthly
demand of computers will not exceed 250 units. Determine the number of units of each
type of computers which the merchant should stock to get maximum profit if he does not

e
want to invest more than 170 lakhs and if his profit on the desktop model is 14500 and on

Fre
portable model is ? 5000.
46. A toy company manufactures two types of dolls, A and B. Market tests and available
for
resources have indicated that the combined production level should not exceed 1200 dolls

per week and the demand for dolls of type B is at most half of that for dolls of type A.
r
Further, the production level of dolls of type A can exceed three times the production of
You
oks

dolls of other type by at most 600 units. If the company makes profit of 112 and ? 16 per doll
respectively on dolls A and 6, how many of each should be produced weekly in order to
eBo

maximise the profit?


47. There are tw'o types of fertilisers Fj and F2. F| consists of 10% nitrogen and 6% phosphoric
acid and F2 consists of 5% nitrogen and 10% phosphoric acid. After testing the soil
our
ad

conditions, a farmer finds that she needs atleast 14 kg of nitrogen and 14 kg of phosphoric
acid for her crop. If F^ costs 16/kg and F2 costs 15/kg, determine how much of each type of
fertiliser should be used so that nutrient requirements are met at a minimum cost. What is
dY
Re

the minimum cost?


48. A manufacturer has three machines I, II and III installed in his factory. Machines I and II are
Fin

capable of being operated for at most 12 hours whereas machine III must be operated for
atleast 5 hours a day. She produces only two items M and N each requiring the use of all the
three machines.

The number of hours required for producing 1 unit of each of M and N on the three
machines are given in the following table:
Items Number of hours required on machines
I II III

M 1 2 1

N 2 1 1.25

She makes a profit of ? 600 and t 400 on items M and N respectively. How many of each
item should she produce so as to maximise her profit assuming that she can sell all the items
that she produced? What will be the maximum profit?
26.60
APPLIED MATHEMATICS-XII

49. There are two factories located one at place P and the other at place Q. From these locations,
a certain commodity is to be delivered to each of the three depots situated at A, B and C
The weekly requirements of the depots are respectively 5,5 and 4 units of the coinmodity
while the production capacity of the factories at P and Q are respectively 8 and 6 units. The
cost of transportation per unit is given below:
To
From Cost (in V

A B C
P 160 100 150
Q 100 120 100

How many units should be transported from each factory to each depot in order that the

ow
transportation cost is minimum. What will be the minimum transportation cost?
50. A manufacturer makes two types of toys A and R Three machines are needed for this
purpose and the time (in minutes) required for each toy on the machines is given below:
Types of Toys Machines

e
Fl
re
1 n III

F
A 12 18 6
B 6 0 9
ur
or
Each machine is available for a maximiun fo 6 hours per day. If the profit on each toy of type
sf
A is ? 7.50 and that on each toy of type B is ? 5, show that 15 toys of type A and 30 of type B
should be manufactured in a day to get maximum profit.
k
Yo
51. An aeroplane can carry a maximum of 200 passengers. A profit of ? 1000 is made on each
oo

executive class itcket and a profit of ? 600 is made on each economy class itcket. The airline
reserves at least 20 seats for executive class. However, at least 4 times as many passengers
B

prefer to travel by economy class than by the executive class. Determine how many itckets
re

of each type must be sold in order to maximize the profit for the airline. What is the
maximum profit?
u
ad

52. A manufacturer considers that men and women workers are equally efficient and so he
Yo

pays them at the same rate. He has 30 and 17 units of workers (male and female) and capital
respectively, which he uses to produce two types of goods A and B. To produce one unit of
d

A, 2 workers and 3 units of capital are required while 3 workers and 1 unit of capital is
Re
in

required to produce one unit of B. If A and B are priced at ^ 100 and ? 120 per unit
respectively, how should he use his resources to maximize the total revenue? Form the
F

above as an LPP and solve graphically. Do you agree with this view of the manufacturer
that men and women workers are equaUy efficient and so should be paid at the same rate?
53. A manufacturer produces two products A and B. Both the products are processed on two
different machines. The available capacity of first machine is 12 hours and that of second
machine is 9 hoiu*s per day. Each imit of product A requires 3 hours on both machines and
each unit of product B requires 2 hours on first machine and 1 hour on second machine.
Each unit of product A is sold at ? 7 profit and that of B at a profit of ? 4. Find the
production level per day for maximum profit graphically.
54. There are two types of fertilisers 'A' and 'B'. 'A' consists of 12% nitrogen and 5% phosphoric
acid whereas 'B' consists of 4% nitrogen and 5% phosphoric acid. After testing the soil
conditions, farmer finds that he needs at least 12 kg of nitrogen and 12 kg of phosphoric
acid for his crops. If 'A' costs ? 10 per kg and 'B' cost ? 8 per kg, then graphically determine
how much of each type of fertiliser should be used so that nutrient requirements are met at
a minimum cost.
26.61
LINEAR PROGRAMMING

55. A small firm manufactures necklaces and bracelets that it can handle per day is at most 24.
It takes one hour to make a bracelet and half on hour to make necklace. The maximum
number of hours available per day is 16. If the profit on a necklace is ? 100 and that on a
bracelet is ?300. Formulate an LPP for finding how many of each should be produced daily
to maximize the profit? It is being given that at least one of each must be produced.
ANSWERS

1. — km with the speed of 25 km/hr, — km with the speed of 40 km/hr


3 ^
2. 4 units of A, 4 units of 6, ? 40.00 3. A :5 days, B : 3 days
4. 30 packages of screw 'A' and 20 packages of screw 'B', ? 41.00
5. 200 Belts of type A and 600 belts of type B; Max. profit = ? 1300
6. 20 ordinary models, 10 deluxe models; Max profit = ? 400
8. Either 4 machines of type A and 3 machines of type
or

6 Machines of type A and no Machine of type B


9. 2 units of A, 3 units of B , Profit = ? 230

w
^ tables ; Max profit = ? 22.2
22
in. —chairs and

F lo
5
11. 24 chairs and 14 tables; Max profit = f 2200
12. 25 units of product A, 100 units of product B; Max profit = ? 475
13. 30 type A and 60 type B; Max profit = ? 2400

ree
!4. 2 units of first product, 4 units of second product; Max profit = 16 monetary units

F
15. 360 hard cover edition, 600 paper back edition. Max. profit = ? 2880
for
16. 2 pills of size A, 8 pills of size B ; Quantity of codeine = 50 grains
17. 19 kg, 13 kg; ? 254
r
18. 8 type A, 20 type 6, Max profit = ? 16
You

in. 48 units of product A, 16 units of product B, Max. profit = ? 1440


oks

20. 3, Trunks of type A, 3 Trunks of Type B, Max profit = ? 165


eBo

21. 10500 bottles of A, 34500 bottles of B, max profit = ? 325500


22. First class tickets = 40, Economy class tickets = 160, Profit = ? 64000.00
23. Type 1 Fertilizer 100 kg. Type II Fertilizer 80 kg. Cost = ? 92
24. ? 2000 in Saving Certificates, ^ 10,000 in National Saving Bonds, Income = ? 1160 per month
our
ad

25. Type A : 20 trees Type B : 40 trees. Max profit = ? 2200


26. 4 pedestal lamps, 4 wooden shades
27. 3 units of X and 8 units of V
dY

28. 60 units of type A and 240 units of type B


Re

29. 8 items of type A and 16 items of type B


Fin

30. 12 toys of type A and 15 toys of type B


31. 12 units of product A and 6 units of product B
32. 8 units of A and 16 units of B.
33. Product A ^ 25 units. Product 6 ^ 125 units. Maximum profit = ? 2375
34. Large Box = 6, Small box = 12, Maximum profit = ? 42
35. Product A : 48 units. Product B ; 16 units. Maximum profit = ? 1440
36. At 25 km/hr: 50/3 km, the 40 km/hr : 40/3 km , Maximum Distance = 30 km
37. From A : 500 litres, 3000 litres, 3500 litres to D, E, f respectively
From B : 400 litres, 0 litres, 0 litres to D, £, f respectively
38. 8 rings and 16 chains 39. 12, 6
40. LPP is maximize Z = 20:c + lOi/, subject to 15x + 3i/ < 42 , 3.r + y ^ 24 and, x, y > 0
Number of rackets = 4, Number of bats = 12
41. 200 Desktop model, 50 portable model
42. 30 hectares for crop X, 20 hectares for crop Y, Total profit = ? 4, 95,000.
43. 12 pieces of Model A, 6 pieces of Model B, Profit = ? 1,68,000
44. (i) Tennis rackets = 4; Cricket bats = 12; Max. Profit = ? 200.
26.62
APPLIED MATHEMATICS-XII

45. 200 units of desktop model and 50 units of portable model. Max. Profit = ? 1150000.
46. 800 dolls of type yl, 400 dolls of type B, Max. Profit = ? 16000.
47. Fertiliser = 100 kg. Fertiliser F2 = 80 kg. Minimum cost = ? 1000.
48. Item M = 4, Item N =4, Profit = X 4000.
49.

_ Factory at Dqyot
A B C
P 0 5 3
Q 5 0
Cost = ? 1550.
50. 400 tickets of executive class, 160 tickets of economy class, Profit = ^ 136000.

ow
^ worker and y units of capital are required to maximize the total revenue
Then, the LPP is
Maximize Z = 100.r + 120i/
Subject to: 2.Y + 3y < 30
3x + y <17

e
a: > 0, y > 0.

re
rFl
Revenue is maximum when ,r = 3, 1/ =■ 8 and Maximum
Yes, because the efficiency of a person does not depend
revenue is ^ 1260.

F
on sex (male or female)
53. Product : 2 units. Product B : 3 units
55. Necklace = 16, Bracelet = 8
54. Fertilize A : 0.3 kg. Fertilizer B : 0.21 kg

r
fo
ou
H/NTS TO SELECTED PROBLEMS
1. Maximum Z = .Y +1/
ks
Subject to 2.v + 5y <100
oo

8.Y+ 5y <200
and. a:, y > 0
Y
B

2. Suppose the manufacturer produces y units of item A and y units of item B. Then, the
mathematical form of the given LPP IS'
e

Maximize Z = 6x + 4y
ur

Subject to Y + 2y < 12
ad

2y + 1/ < 12
Yo

Y +
d

and. Y, y > 0
Re
in

3. We have to mimmize the labour cost. This means that the profit is to be maximized. For this,
suppose the tailors A and B work for y and y days respectively. Then the LPP is
F

Maximize Z = 15y + 20y


Subject to 6y + lOy > 60
4.V + 4y > 32
and. Y,y > 0
4. Suppose the manufacturer produces y packages of screws A and y packages of screws B in a
day. The LPP is r o
Maximize Z = 0.7y + 1/
Subject to 4y + 6y < 240
6y + 3y < 240
and. Y,y > O
5. Suppose the company produces y belts of type A and y belts of type B Then
Profit = 2Y + 1.5y
Since the rate of production of belts of type B is 1000 per day. Therefore, time taken to
1/
produce y belts of type B is
1000 ●. Also, since each belt of t}q?e A requires twice as much time
26.63
LINEAR PROGRAMMING

as a belt of type B, the rate of production of belts of type /I is 500 per day and consequently
—. Thus, we have
total time taken to produce .r belts of type A is 500
X
+ 1 < 1 => 2-Y + i/ < 1000
500 1000
The supply of leather is sufficient only for 800 belts per day.
.V + y < 800
Since 400 buckles are available for belt A and 700 buckles are available for belt B per day.
.t < 400, 1/ < 700
Thus, the mathematical formulation of the LPP is
Maximize Z = 2.V + 1.5]/
Subject to 2x + y ^ 1000
.T + y < 800

w
.V < 400

y < 700
and, y > 0

F lo
6. If the manufacturer makes x■^ deluxe model articles and .V2 ordinary model, then
Maximize Z = 15.VJ + IO.V2

ee
Subject to 2.V-1 + .V2 ^ 40

Fr
2xi + 3.V2 < 80
«^d, X^, X2 > 0 Tr>p-
7. Let a: tea-cups of type A and y tea-cups of type B are manufactured per
for day. Then, the LI 1 is
ur
Maximize Z =0.75.v + 0.50y
Subject to 12.V + 6y < 360
s
18.V + Oy < 360
ok
Yo

6-v + 9y < 360


o

and, -V, y ^ 0 . . . j -1 4.
eB

8. Let a: machines of type A and y machines of type B are bought to maximize the daily output.
Then, the LPP is
Maximize Z = 60a: + 40y
r
ou
ad

Subject to 1000x + 1200y < 7600


12x + 8y < 72
Y

and, -V, y ^ 0 1 1
9. Suppose the company produces .v goods of type A and y goods of type 6. The mathematical
Re
nd

IV form of the LPP is as follows;


Maximize Z = 40x + 50y
Fi

Subject to 3x + y < 9
X + 2y < 8
and, .t,y > 0
12. The LPP is as follows:
Maximum Z = + 4.t2
4.X| + 4.Y2 ^ 500
8x;[ + 4^2 5 600
Xi, X2 ^ 0
17. Let X kg of compound A and y kg of compound B are produced. Then, tlae mathematical
formulation of the LPP is as follows:
Minimize Z = 4.x + 6y
Subject to X + 2y > 80
3x + 1/ > 75
x,y > 0
26.64
APPLIED MATHEMATICS-XII

18. follows:
Let X souvenirs of type >1 and y souvenirs of type B are manufactured. Then, the LPP is as
Maximize Z = 50a: + 60y
Subject to 5x + 8y < 200
10a:+ 8y < 240
and. AT,y > 0

of the LPP IS as follows:


^‘^y^tsofproductBbeproduced.Thenthemathematicalform
Maximize Z = lQx+ 30y
Subject to 200x + 300y > 10,000
X > 14

ow
y > 16

l + y < 40
and. AT,y > 0
20. mathematical
Suppse a: tru^ of type y4 and y trunks of type B are manufactured per day. Then, the

e
form of the LPP is as follows:

Fl
re
Maximize Z = 30A: + 25y

F
Subject to 3x+3y < 18
3a: + 2y ^ 15
ur
r
and. x,y > 0
21. fo
Let Ae manufacturer produce a: bottles of medicine /I and y bottles of medicine B. Then the
ks
mathemahcal form of the LPP is as follows:
Yo
Maximize Z = 8x + 7y
oo

Subject to a: < 20000


B

y < 40000
x + y < 45000
re

3a:
< 66
1000 1000
u
ad

and, a:, y > 0


Yo

^f
me LPP IS
y economy class tickets are sold. Then the mathematical form of
d

Maximize Z = 400a: + 600y


Re
in

Subject to x + y < 2000


F

AT > 20

y > 4a:
and. x,y > 0
23. Let, kg of fertUizer I and y kg of fertilizer n are used. Then the mathematical form of the
LPP IS as follows:
Minimize Z = 60A: + 40y
Subject to > 14
100 100
6x lOx
> 14
100 100
and. A:,y > 0
I"" “offutheTLPPCerHcate
mathematical formulation and ? y in NaHonai
is as follows: Saving Bonds. Then, the
Maximize Z = —+ 1^
100 100
LINEAR PROGRAMMING 26.65

Subject to x + y < 12000


> 2000
y > 4000
and. x,y > 0
25. Let .Y trees of type A and y trees of type B are planted. Then, the mathematical formulation of
the LPP is as follows:
Maximize Z - 40.y + 60y-(20.v + 25y)
Subject to 20.Y + 25y < 1400
10.Y + 20i/ < 1000
and, .Y,y > 0
26. Let .Y lamps and y shades be manufactured by the manufacturer. Then, the mathematical
formulation of the LPP is
Maximize Z = 5y + 3y
Subject to 2y + y < 12
3x + 2y < 20
and. Y,y £ 0
27. Let x units of X and y units of Y be produced to maximize the revenue. Then, the LPP is

w
Maximize Z = 100x + 120y

31.
Subject to 2x + 3y
3x + y
X 2: 0, y

Maximize Z = 6x + 8y
Subjected to 4x + 2y < 60
< 30
<17
> 0
F lo
Let X units of A and y units of B be produced per week for maximum profit. Then, the LPP is

2x + 4y < 48
for F
ree
X, y > 0
36. Suppose he travels x km with the speed of 25 km/hr and y km with the speed of 40km/hr.
Then, the LPP is
Your
ks

Maximize Z = x + y
eBoo

Subjected to 2x + 5y<100
— <1
25 40
ad

x,y>0
our

26.9.3 TRANSPORTATION PROBLEMS

In this type of problems, we have to determine transportation schedule for a commodity from
different plants or factories situated at different locations to different markets at different
Re

locations in such a way that the total cost of transportation is minimum, subject to tlic limitations
Y

(constraints) as regards the demand of each market and supply from each plant or factory.
Find

Following are some examples on this type of problems:


ILLUSTRATIVE EXAMPLES
EXAMPLE i There is a factory located at each of the two places P aud Q. From these locations, a certain
commodity is delivered to each of these depots situated at A, B and C. The weekly requirements of the
depots are respectively 5,5 and 4 units of the commodity while the production capacity of the factories at P
and Q are respectively 8 and 6 units. The cost of transportation per unit is given below:
Cost (in ?)

A B C

P 16 10 15

Q 10 12 10

How many units should be transported from each factory to each depot in order that the transportation
cost is minimum. Formulate the above LPP mathematically and then solve it.
26.66 APPLIED MATHEMATICS-Xll

SOLUTION
For the formulation see Example 11 in section 26.4 on page 26.11.
Let the factory at P transports x units of commodity to depot at A and y units to depot at B. Then,
as discussed in Example 11 on page 26.11, the mathematical model of the LPP is as follows:
Minimize Z = x -7\j + 190
Subject to X + y < 8
x +y > 4
X < 5

]/ ^ 5
and, -T > 0, y > 0

ow
To solve this LPP graphically, we first convert the inequations into equations and draw the
corresponding lines. The feasible region of the LPP is shaded in Fig. 26.25.
y

e
Fl
re
F
ur
r
fo
ks
Yo
oo
eB
ur
ad
Yo
d
Re
in
F

The coordinates of the corner points of the feasible region A2 A3 PQ B3 B2 are A2 (4, 0),
A3 (5, 0), P (5, 3), Q (3,5), S3 (0,5) and B2 (0, 4). These points have been obtained by solving
the corresponding intersecting lines simultaneously.
The values of the objective function at these points are given in the following table:
Point (x, y) Value of the objective function Z - x-7y + 190
A2 (4, 0) 2 = 4-7x0 + 190 = 194

A3 (5, 0) Z = 5-7 X 0 + 190 = 195

P(5, 3) Z = 5-7 X 3 + 190 =174

Q(3,5) Z = 3-7x5 + 190 =158

B3 (0,5) Z = 0-7x5+190 = 155

h (0, 4) Z = 0-7x4 + 190 = 162


26.67
LINEAR PROGRAMMING

Clearly, Z is minimum at x = 0, ]/ =5. The minimum value of Z is 155.


Thus, the optimal transportation strategy will be to deliver 0,5 and 3 units from the factory at P
and 5, 0 and 1 unit from the factory at Q to the depots at A, B and C respectively. Tlie minimum
transportation cost in this case is ? 155.
EXAMPLE 2 A brick mmnifachirer has tzuo depots, A and B, with stocks of 30,000 and 20,000 bricks
respcctiveXy. He receives orders from three builders P, Q and R for 15,000, 20,000 and 15,000 bricks
respectively. The cost in ? transporting 1000 bricks to the builders from the depots are given below:
To
P Q R
From

A 40 20 30

B 20 60 40

w
How should the manufacturer fulfdl the orders so as to keep the cost of transportation minimum?
SOLUTION The formulation of this LPP is discussed in Ex. 12insection26.4o npage26.13.

F lo
Let the depot transport x thousand bricks to builder P and y thousand bricks to builder Q.
Then, the above LPP can be stated mathematically as follows:

ee
Minimize Z = 30.v - 30y + 1800

Fr
Subject to
x + y < 30
X < 15
for
ur
y < 20
x + y > 15
and, X > 0, y > 0
ks

To solve this LPP graphically, we first convert inequations into equations and then draw the
Yo
oo

corresponding lines. The feasible region of the LPP is shaded in Fig. 26.26. The coordinates of the
corner points of the feasible region A2 PQ 63 B2 are A2 (15, 0), P (15,15) ,Q (10, 20), 63 (0, 20)
eB

and B2 (0,15). These points have been obtained by solving the corresponding intersecting lines
simultaneously.
r
ou
ad
Y
Re
nd
Fi
26.68
APPLIED MATHEMATICS-Xn

The values of the objective function at the corner points of the feasible region are given in the
following table
Point (.r, y) Value of the objective function Z = 30x - 30y + 1800
: ^2(15,0) Z = 30 X 15 - 30 X 0 +1800 = 2250
P (15,15) Z - 30 X 15 - 30 X 15 + 1800 = 1800
Q (10, 20} Z = 30 X 10 - 30 X 20 +1800 = 1500
h (0, 20) Z = 30 X 0 - 30 X 20 + 1800 = 1200
' B2 (0,15) Z = 30 X 0 - 30 X 15 + 1800 = 1350

Clearly, Z is minimum at .y = 0, y = 20 and the minimum value of Z is 1200.

w
Thus, the manufacturer should supply 0,20 and 10 thousand bricks to builders P, Q and R from
depot A and 15, 0 and 5 thousand bricks to builders P, Q and R from depot B respectively. In

F lo
this case the minimum transportation cost will be ? 1200.
_ _ EXERCISE 26.5

ee
1. Two godowns, A and B, have grain storage capacity of 100 quintals and 50 quintals

Fr
respectively. They supply to 3 ration shops, D, E and F, whose requirements are 60,50 and
40 quintals respectively. The cost of transportation per quintal from the godowns to the
shops are given in the following table: for [ NCERTj
ur
Transportation cost per quintal (in ?)
From
s

A B
ook

To
Yo

D 6.00 4.00
eB

E 3.00 2.00
F 2.50 3.00
our
ad

How should the supplies be transported in order that the transportation cost is minimum ?
2. A medical company has factories at two places, A and B. From these places, supply is made
to each of its three agencies situated at P,Q and R. The monthly requirements of the
Y

agencies are respectively 40, 40 and 50 packets of the medicines, while the production
Re

capacity of the factories A and B, are 60 and 70 packets respectively. The transportation
nd

cost per packet from the factories to the agencies are given below:
Fi

Transportation cost per


packet (in ?)
From
A B
To

P 5 4

Q 4 2
R 3 5

How many packets from each factory be transported to each agency so that the cost of
transportation is minimum ? Also find the minimum cost?
ANSWERS

1. From A : 10 quintals, 50 quintals and 40 quintals toD, £, F respectively.


From B: 50 quintals, 0 quintal and 0 quintal to D, £, F respectively.
LINEAR PROGRAMMING 26.69

2. From A : 10 packets, 0 packets and 50 packets to P, Q and R respectively.


From B: 30 packets, 40 packets and 0 packets to P, Q and K respectively.
Minimum cost = ? 400.
H/NTS TO SELECTED PROBLEMS

1. Suppose godown A supplies .r quintals of grain to the ration shopD and y quintals to ration
shop £. Then, the mathematical formulation of the LPP is as follows:
Mimmize Z=6x+3ij + - (100 - a: - y) + 4 (60 - x) + 2 (50 - y) + 3 (.v + y - 60)
Subject to x+ y < 100
X < 60
y < 50
X + y > 60
and. X, y > 0
2. Let X and y packets be transported from the factory A to the agencies P and Q respectively.

w
Then, the mathematical formulation of the LPP is as follows:
Mimmize Z = 5x + 4y + 3 (60 - x - y) + 4 (40 - x) + 2 (40 - y) + 5 (x + y - 30)

F lo
Subject to X + y <60
X < 40
y < 40

ee
x + y > 30

Fr
and, X > 0, y > 0
26.10 SOME EXCEPTIONAL CASES for
Uptill now we have been discussing linear programming problems having finite unique
ur
solutions. In this section, we shall discuss some problems which either do not have solutions or
they have unbounded solutions. Consider the following linear programming problem:
s
ook

Maximize Z = 2x + 5y
Yo

Subject to the constraints


eB

X + y <4
3x+ 3y >18
X, y > 0
our

The graphical representation of constraints and non-negativity restrictions is shown in


ad

Fig. 26.27. We observe that the constraints do not have any common feasible solution. So, tlie
LPP does not have any solution.
In some linear programming problems, the common feasible region may not be bounded and
Y

the variables can take any value in the unbounded feasible region. Such type of problems are
Re
nd

said to have unbounded solutions. Consider the following linear programming problem.
Fi

3x + 3y = 18
26.70 APPLIED MATHEMATICS-XII

Maximize Z = 2x + 3y
Subject to the constrahits
x + \j>2
.r + 2y ^ 3
x,y>0
The common feasible solution set of this LPP is the set of all points in the shaded region of
Fig. 26.28. We observe that the feasible region is unbounded. So, x and y can take arbitrary large
values. Consequently, the objective fimction can be made as large as we please. Thus, we say
that the LPP has unbounded solution.

MULTIPLE CHOICE QUESTIONS (MCQs)


1.
The solution set of the inequation 2 .t + y >5 is
(a) half plane that contains the origin

w
(b) open half plane not containing the origin
(c) whole -vy-plane except the points lying on the line 2 ;r + y = 5

Flo
(d) none of these
2. Objective function of a LPP is
(a) a constraint

ee
(b) a function to be optimized
(c) a relation between the variables (d) none of these

Fr
3.
Which of the following sets are convex?
(a) {{x, y): x^ + y^ > 1} (b) {(y, X}
for
ur
(c) {{x, y): 3 + 4y^ >5} (d) {(Y,y):y>2,y<4}
4.
LetX| and X2 are optimal solutions of a LPP, then
k s
(a) X = A.X| + (1 - X) X2 , A. € X is also an optimal solution
Yo
oo

(b) X = A. X] + (1 - A.) X2,0 < A. < 1 gives an optimal solution


(c) X = A. X| + (1 + A.) X2,0 < A. < 1 give an optimal solution
eB

(d) X = A.X| + (1 + A.) X2 , A. € K gives an optimal solution


j. The maximum value of Z = 4 Y + 2 y subjected to the constraints 2 y + 3 y < 18,
r

x + y> 10; .Y, y > 0 is


ou
ad

(a) 36 (b) 40 (c) 20 (d) none of these


6.
The optimal value of the objective function is attained at the points
Y

(a) given by intersection of inequations with the axes only


Re
nd

(b) given by intersection of inequations with Y-axis only


(c) given by comer points of the feasible region
Fi

(d) none of these


7.
The maximum value of Z=4Y+3y subjected to the constraints 3y + 2y> 160,
5 Y + 2 y > 200, Y + 2 y > 80; Y, y > 0 is
(a) 320 (d) 300 (c) 230 (d) none of these
8. Consider a LPP given by
Minimum Z = 6y + lOi/
Subjected to y > 6; y > 2; 2 y + y > 10; x, y > 0
Redundant constraints in this LPP are
(a) Y > 0, y > 0 (b) Y > 6, 2 Y + y > 10
(c) 2 Y + y > 10 (d) none of these
9. The objective function Z = 4 y + 3 y can be maximised subjected to the constraints
3 Y + 4 y < 24,8 Y + 6 y < 48, Y <5, y < 6; Y, y > 0
(a) at only one point (b) at two points only
(c) at an infinite number of points (d) none of these
26.71
LINEAR PROGRAMMING

10. If the constraints in a linear programming problem are changed


(a) the problem is to be re-evaluated (b) solution is not defined
(c) the objective function has to be modified (d) the change in constraints is ignored
11. Which of the following statements is correct?
(a) Every LPP admits an optimal solution
(b) A LPP admits unique optimal solution
(c) If a LPP admits two optimal solutions it has an infinite number of optimal solutions
(d) The set of all feasible solutions of a LPP is not a converse set
12. Which of the following is not a convex set?

w
(a) \{x,y)\lx + 5y <7\ (b) |(x,y):x^ + y^ < 4
(c) [x:\x\ =5} (d)l(Ar,y);3x2 + 2y2 S 6)

Flo
13. By graphical method, the solution of linear programming problem

ee
Maximize Z = + 5^2

Fr
Subject to SaTj + 2^2 ^ 18
< 4

JC2 ^ 6 for
ur
Xj ^ 0, X2 ^ 0, is
s
(a) xj = 2, X2 = 0, Z = 6 (b) xj = 2, X2 = 6, Z = 36
k
Yo

(d) xi = 4, X2 = 6, Z = 42
oo

(c) xi = 4, X2 = 3, Z = 27
14. The region represented by the inequation system X, y > 0,i/ < 6, x + y<3is
eB

(a) unbounded in first quadrant


r

(b) unboimded in first and second quadrants


ou
ad

(c) bounded in first quadrant


Y

(d) none of these


15. The point at which the maximum value of x + y, subject to the constraints x + 2y < 70,
Re
nd

2x + y < 95, X, y > 0 is obtained, is


Fi

(a) (30, 25) (b) (20, 35) (c) (35, 20) (d) (40,15)

16. The value of objective function is maximum under linear constraints


(a) at the centre of feasible region
(b) at (0,0)
(c) at any vertex of feasible region
(d) the vertex which is maximum distance from (0,0)
17. The comer points of the feasible region determined by the following system of linear
inequalities:
0), (3, 4) and (0, 5). Let Z = px + qy, where
2x + y < 10, X + 3y < 15, X, y > 0 are (0, 0), (5,
p,q> 0. Condition on p and q so that the maximum of Z occurs at both (3,4) and (0,5) is
(a) p = q (b) p = 2g (c) p = 3q (d) q = Sp
26.72
APPLIED MATHEMATICS-XII

18. The comer points of the feasible region determined by the system of linear constraints are:

(0,10), (5,5), (15,15), (0,20). Letz =px + cjy, where p, q>0. Condition on p and q so that the
maximum of z occurs at both the points (15,15) and (0,20) is
(a) p=q (b) p=2q (c) q = 2p (d) q=3p
19. Corner points of the feasible region determined by the system of linear constraints (0, 3),

w
(1,1) and (3,0). Letz = px + qp, where p,q>0. Condition on p and q so that the minimum of z
occurs at (3,0) and (1,1) is
(a) p=2q (b) 2p=q (c) P = 3q (d) p = q

e
20. Corner points of the feasible region for an LPP are : (0, 2), (3, 0), (6,0), (6, 8) and (0, 5). Let
2 = 4a- + 6y the objective function. The minimum value of z occurs at

e
or
(a) (0, 2) only (b) (3,0) only

r
(c) the mid-point of the line segment joining the points (0,2) and (3,0) only

F
(d) any point on the line segment joining the points (0,2) and (3,0)

oF
21. Corner points of the feasible region for an LPP are : (0, 2), (3, 0), (6, 0), (6, 8) and (0, 5). Let

ul
2 = 4a + 6y be the objective function. Then, Max. z - Min z =

rs
(a) 60 (b) 48 (c) 42 (d) 18
22. The feasible region for an LPP is shown i Fig. 26.24. Let 2 = 3a-4y be the objective

ko
in

function. Maximum value of z is

y of
o
Y
YB
er

(12,6)
u
od
ad
in
Re
F

(12, 0)
o X

Fig. 26.29
(a) 0 (b) 8 (c) 12 (d) -18
23. The corner points of the feasible region determined by the system of linear constraints are

(0, 0), (0, 40), (20, 40), (60, 20), (60, 0). Tlie objective function is z = 4a + 3y.
Compar the quantity in Column A and Column B
Column A Column B
Maximum of z 325

(a) The quantity in column A is greater (b) The quantity in Column B is greater
(c) The two quantities are equal
(d) The relationship can not be determined on
the basis of information supplied.
24. The feasible region of a LPP is shown in Fig. 26.30. Letz = 3.T -4y be the objective function.
Minimum of z occurs at
LINEAR PROGRAMMING 26.73

(a) (0,0) (b) (0,8) (c) (5,0) (d) (4,10)

(4,10)

r
l B
(0,8) (6,8)

n
(6,5)

f
u
X
(0, 0) (5,0)
Fig. 26.30

i
25. In Q. No. 24, Maximum of z occurs at
(a) (5, 0) (b) (6,5) (c) (6, 8) (d) (4,10)
26. In Q. No. 24, (Maximum Value of z + Minimum Value of z) is equal to
(a) 13 (b) 1 (c) -13 (d) -17
e

AHS\NERS
F

1. (b) 2. (b) 3. (d) 4. (b) 5. (d) 6. (c) 7. (d) 8. (c) 9. (c)


10. (a) 11. (c) 12. (c) 13. (b) 14. (a) 15. (d) 16. (d) 17. (d) 18. (d)
19. (b) 20. (d) 21. (a) 22. (c) 23. (b) 24. (b) 25. (a) 26. (d)

FILL IN THE BLANK QUESTIONS (FBQs)

1. In a LPP, the objective function is always .


2. The feasible region for an LPP is always a polygon.
ad

3. If the feasible region for an LPP is , then the optimal value of the objective
function z-ax+ by may or may not exist.
4. A feasible region of a system of linear inequalities is said to be if it can be
Re

enclosed within a circle.

5. A corner point of a feasible region is a point in the region which is the. of two

boundary lines.
6. The maximum value of z=4x + 2i/ subject to the constraints
2.Y+ 3y <18, -Y + 1/ <10, Y > 0,1/ > 0, is
7. The point which provides the optimal solution of the linear programming problem
maxz = 45y + 55i/
6Y + 4y <120
3Y + 10y <180
Y > 0, y > 0
has the coordinates

8. The maximum value of z = 3y + 4y subject to the constraints y + y < 40, y + 2y < 60 y, y S 0,


IS

9. The minimum of the objective function Z=2.r + 10y for linear constraintsY-y>0,
Y -5y <-5, Y >0, y >0, is
26.74 APPLIED MATHEMATICS-XII

10. The coordinates of the point for mirdmum value of Z =7x-^y subject to the conditions
.r + 1/ <20, y >5, X > 0,1/ >0are
11. In a LPP, the linear inequalities or restrictions on the variables are called
12. In a LPP, if the objective function Z = fl.v + by has the same maximum value on two comer
points of the feasible region, then every point on the line segment joining these two points
give the same value.

13. In a LPP, the linear function which has to be maximised or minimized is called a linear
function.

14. The common region determined by all the linear constraints of a LPP is called the
region.
ANSWERS

w
1. linear 2. convex 3. unbounded -I. bounded 5. intersection

F lo
6. 36 7. (10,15) 8. 140 9. 15 10. (0, 20)
11. linear constraints 12. maximum 13. objective 14. feasible

ee
Fr
for
ur
s
ook
Yo
eB
r
ad
ou
Y
Re
nd
Fi
TABLE i

AMOUNT AT COMPOUND INTEREST


n
s = (l + 0

Periods Rate i

n
.0025
r 1
-% .004167
5
—% .005
f 1
2
-% .00583sfll2
—%1; .OOTsf'.4-%
V4 ) 12

1.0025 0000 1.0041 6667 1.0050 0000 1.0058 3333 1.0075 0000
1
2 1.0050 0625 1.0083 5069 1.0100 2500 1.0117 0069 1.0150 5625
3 1,0075 1877 1.0125 5216 1.0150 7513 1.0176 0227 1.0226 6917

ow
4 1.0100 3756 1.0167 7112 1.0201 5050 1.0235 3828 1.0303 3919

1.0125 6266 1,0210 0767 1.0252 5125 1.0295 0892 1.0380 6673
5
6 1.0150 9406 1.0252 6187 1,0303 7751 1.0355 1438 1.0458 5224
7 1,0176 3180 1.0295 3379 1.0355 2940 1,0415 5488 1.0536 9613
8 1,0201 7588 1.0338 2352 1,0407 0704 1,0476 3061 1.0615 9885
1,0695 6084

e
9 1.0227 2632 1.0381 3111 1.0459 1058 1.0537 4179

re
1.0252 8313 1.0424 5666 1.0511 4013 1.0598 8861 1.0775 8255
10
1.0660 7129 1.0856 6441

Frl
11 1,0278 4634 1.0468 0023 1.0563 9583
1.0722 9004 1.0938 0690

F
12 1.0304 1596 1.0511 6190 1.0616 7781
13 1.0329 9200 1.0555 4174 1.0669 8620 1.0785 4506 1.1020 1045
1.0355 7448 1.0599 3983 1.0723 2113 1.0848 3657 1.1102 7553
14
ou
1.1186 0259

or
15 1.0381 6341 1.0643 5625 1.0776 8274 1.0911 6478
16 1.0407 5882 1.0687 9106 1.0830 7115 1.0975 2990 1.1269 9211
1.0732 4436 1.0884 8651 1.1039 3216 1.1354 4455
17 1.0433 6072
1.0777 1621 1,0939 2894
kfs 1.1103 7176 1.1439 6039
18 1.0459 6912
1.0485 8404 1.0822 0670 1.0993 9858 1.1168 4892 1.1525 4009
19
oo

20 1.0512 0550 1,0867 1589 1.1048 9558 1,1233 6387 1.1611 8414
21 1.0538 3352 1.0912 4387 1.1104 2006 1.1299 1682 1.1698 9302
Y

1.0564 6810 1.0957 9072 1.1159 7216 1.1365 0800 1.1786 6722
B

22
23 1.0591 0927 1.1003 5652 1.1215 5202 1.1431 3763 1,1875 0723
24 1.0617 5704 1.1049 4134 1.1271 5978 1.1498 0593 1.1964 1353
re

25 1,0644 1144 1.1095 4526 1.1327 9558 1.1565 1312 1.2053 8663
1.1384 5955 1,1632 5945 1.2144 2703
oYu

26 1.0670 7247 1.1141 6836


1.1188 1073 1.1441 5185 1,1700 4512 1.2235 3523
ad

27 1.0697 4015
1.1234 7244 1.1498 7261 1,1768 7038 1.2327 1175
28 1.0724 1450
29 1.0750 9553 1.1281 5358 1.1556 2197 1,1837 3545 1.2419 5709
d

30 1.0777 8327 1.1328 5422 1.1614 0008 1.1906 4057 1.2512 7176
in

31 1.0804 7773 1.1375 7444 1.1672 0708 1.1975 8597 1.2606 5630
Re

1.0831 7892 1.1423 1434 1.1730 4312 1.2045 7189 1,2701 1122
32
33 1.0858 8687 1.1470 7398 1.1789 0833 1.2115 9855 1.2796 3706
F

34 1.0886 0159 1.1518 5346 1.1848 0288 1.2186 6621 1.2892 3434

35 1.0913 2309 1.1566 5284 1.1907 2689 1.2257 7509 1,2989 0359
36 1.0940 5140 1.1614 7223 1.1966 8052 1.2329 2544 1.3086 4537
37 1,0967 8653 1.1663 1170 1.2026 6393 1.2401 1750 1.3184 6021
38 1.0995 2850 1.1711 7133 1.2086 7725 1,2473 5152 1.3283 4866
39 1.1022 7732 1.1760 5121 1,2147 2063 1.2546 2773 1,3383 1128

40 1.1050 3301 1.1809 5142 1.2207 9424 1.2619 4639 1.3483 4861
1.1077 9559 1.1858 7206 1,2268 9821 1,2693 0774 1.3584 6123
41
42 1.1105 6508 1.1908 1319 1.2330 3270 1.2767 1203 1.3686 4969
1.1133 4149 1.1957 7491 1.2391 9786 1.2841 5951 1.3789 1456
43
44 1.1161 2485 1.2007 5731 1.2453 9385 1.2916 5044 1.3892 5642

45 1.1189 1516 1.2057 6046 1.2516 2082 1.2991 8506 1.3996 7584
1,1217 1245 1.2107 8446 1.2578 7892 1.3067 6363 1.4101 7341
46
1.2158 2940 1.2641 6832 1.3143 8642 1.4207 4971
47 1.1245 1673
1.2208 9536 1.2704 8916 1.3220 5367 1.4314 0533
48 1.1273 2802
49 1.1301 4634 1,2259 8242 1.2768 4161 1.3297 6564 1.4421 4087

1.2310 9068 1.2832 2581 1.3375 2260 1.4529 5693


50 1.1329 7171
T.2
APPLIED MATHEMATICS-XI

TABLE I
AMOUNT AT COMPOUND INTEREST
))
s = (1 + /)
Periods Rate i

1 f 5 1 r 7
.0025 -% (1
n .004167 —% .005 -% .005833 —% .0075 -%
4 ; \^2 J 12 ll2 j U J
51 1.1358 0414 1.2362 2022 1.2896 4194 1.3453 2482 1.4638 5411
52 1.1386 4365 1.2413 7114 1.2960 9015 1.3531 7254 1.4748 3301
53 1.1414 9026 1.2465 4352 1.3025 7060 1.3610 6604 1.4858 9426
54

ow
1.1443 4398 1.2517 3745 1.3090 8346 1.3690 0559 1.4970 3847
55 1.1472 0484 1.2569 5302 1.3156 2887 1.3769 9145 1.5082 6626
56 1.1500 7285 1.2621 9033 1.3222 0702 1.3850 2390 1.5195 7825
57 1.1529 4804 1.2674 4946 1.3288 1805 1.3931 0320 1.5309 7509
58 1.1558 3041 1.2727 3050 1.3354 6214 1.4012 2963 1.5424 5740
59 1.1587 1998 1.2780 3354 1.3421 3946 1.4094 0346 1.5540 2583

e
60

re
1.1616 1678 1.2833 5868 1.3488 5015 1.4176 2498 1.5656 8103
61 1.1645 2082 1.2887 0601 1.3555 9440 1.4258 9445 1.5774 2363

Frl
62 1.1674 3213 1.2940 7561 1.3623 7238 1.4342 1216 1.5892 5431

F
63 1.1703 5071 1.2994 6760 1.3691 8424 1.4425 7840 1.6011 7372
64 1.1732 7658 1.3048 8204 1.3760 3016 1.4509 9343 1.6131 8252
65 1.1762 0977 1.3103 1905 1.3829 1031 1.4594 5756
ou
1.6252 8139

or
66 1.1791 5030 1.3157 7872 1.3898 2486 1.4679 7105 1.6374 7100
67 1.1820 9817 1.3212 6113 1.3967 7399 1.4765 3421 1.6497 5203
68 1.1850 5342 1.3267 6638 1.4037 5785
kfs 1.4851 4733 1.6621 2517
69 1.1880 1605 1.3322 9458 1.4107 7664 1.4938 1068 1.6745 9111
oo
70 1.1909 8609 1.3378 4580 1.4178 3053 1.5025 2457 1.6871 5055
71 1.1939 6356 1.3434 2016 1.4249 1968 1.5112 8929 1.6998 0418
Y

72 1.1969 4847 1.3490 1774 1.4320 4428 1.5201 0514 1.7125 5271
B

73 1.1999 4084 1.3546 3865 1.4392 0450 1.5289 7242 1.7253 9685
74 1.2029 4069 1.3602 8298 1.4464 0052 1.5378 9142 1.7383 3733
re

75 1.2059 4804 1.3659 5082 1.4536 3252 1.5468 6245 1.7513 7486
76 1.2089 6291 1.3716 4229 1.4609 0069 1.5558 8580 1.7645 1017
oYu

77 1.2119 8532 1.3773 5746 1.4682 0519 1.5649 6180 1.7777 4400
ad

78 1.2150 1528 1.3830 9645 1.4755 4622 1.5740 9074 1.7910 7708
79 1.2180 5282 1.3888 5935 1.4829 2395 1.5832 7293 1.8045 1015
d

80 1.2210 9795 1.3946 4627 1.4903 3857 1.5925 0868 1.8180 4398
81 1.2241 5070 1.4004 5729 1.4977 9026 1.6017 9831
in

1.8316 7931
Re

82 1.2272 1108 1.4062 9253 1.5052 7921 1.6111 4213 1.8454 1691
83 1.2302 7910 1.4121 5209 1.5128 0561 1.6205 4045 1.8592 5753
F

84 1.2333 5480 1.4180 3605 1.5203 6964 1.6299 9360 1.8732 0196
85 1.2364 3819 1.4239 4454 1.5279 7148 1.6395 0189 1.8872 5098
86 1.2395 2928 1.4298 7764 1.5356 1134 1.6490 6565 1.9014 0536
87 1.2426 2811 1.4358 3546 1.5432 8940 1.6586 8519 1.9156 6590
88 1.2457 3468 1.4418 1811 1.5510 0585 1.6683 6085 1.9300 3339
89 1.2488 4901 1.4478 2568 1.5587 6087 1.6780 9295 1.9445 0865
90 1.2519 7114 1.4538 5829 1.5665 5468 1.6878 8182 1.9590 9246
91 1.2551 0106 1.4599 1603 1.5743 8745 1.6977 2779 1.9737 8565
92 1.2582 3882 1.4659 9902 1.5822 5939 1.7076 3120 1.9885 8905
93 1.2613 8441 1.4721 0735 1.5901 7069 1.7175 9237 2.0035 0346
94 1.2645 3787 1.4782 4113 1.5981 2154 1.7276 1166 2.0185 2974
95 1.2676 9922 1.4844 0047 1.6061 1215 1.7376 8938 2.0336 6871
96 1.2708 6847 1.4905 8547 1.6141 4271 1.7478 2590 2.0489 2123
97 1.2740 4564 1.4967 9624 1.6222 1342 1.7580 2155 2.0642 8814
98 1.2772 3075 1.5030 3289 1.6303 2449 1.7682 7667 2.0797 7030
99 1.2804 2383 1.5092 9553 1.6384 7611 1.7785 9161 2.0953 6858
100 1.2836 2489 1.5155 8426 1.6466 6849 1.7889 6672 2.1110 8384
T.3
LOGARITHMIC AND OTHER TABLES

TABLE I
AMOUNT AT COMPOUND INTEREST
s = (l + /)
fi

Periods Rate /

( 1 'l 1
n .01(1%) .01125 1-% .0125fl-% .015 1-%
2 )
.0175 1-%
V 4 ;
8 ; K 4

1.0125 0000 1.0150 0000 1.0175 0000


1 1.0100 0000 1.0112 5000
1.0226 2656 1.0251 5625 1.0302 2500 1.0353 0625
2 1.0201 0000
1.0379 7070 1.0456 7838 1.0534 2411
3 1.0303 0100 1.0341 3111
1.0457 6509 1.0509 4534 1.0613 6355 1.0718 5903

ow
4 1.0406 0401
1.0640 8215 1.0772 8400 1.0906 1656
5 1.0510 1005 1.0575 2994
1.0773 8318 1.0934 4326 1.1097 0235
6 1.0615 2015 1.0694 2716
1.0908 5047 1.1098 4491 1.1291 2215
7 1.0721 3535 1.0814 5821
1.1044 8610 1.1264 9259 1.1488 8178
8 1.0828 5671 1.0936 2462
1.1182 9218 1.1433 8998 1.1689 8721
9 1.0936 8527 1.1059 2789

e
1.1183 6958 1.1322 7083 1.1605 4083 1.1894 4449

re
10 1.1046 2213
1.1309 5124 1.1464 2422 1.1779 4894 1.2102 5977
11 1.1156 6835

Frl
1.1436 7444 1.1607 5452 1.1956 1817 1.2314 3931
12 1.1268 2503

F
1.1752 6395 1.2135 5244 1.2529 8950
13 1.1380 9328 1.1565 4078
1.1695 5186 1.1899 5475 1.2317 5573 1.2749 1682
14 1.1494 7421
1.2048 2918 1.2502 3207 1.2972 2786
15 1.1609 6896 1.1827 0932
ou
or
1.1960 1480 1.2198 8955 1.2689 8555 1.3199 2935
16 1.1725 7864
1.2351 3817 1.2880 2033 1.3430 2811
17 1.1843 0443 1.2094 6997
1.3665 3111
18 1.1961 4748 1.2230 7650 1.2505
kfs
7739 1.3073 4064
1.3904 4540
1.2368 3611 1.2662 0961 1.3269 5075
19 1.2081 0895
1.2820 3723 1.3468 5501 1.4147 7820
oo
20 1.2201 9004 1.2507 5052
1.2648 2146 1.2980 6270 1.3670 5783 1.4395 3681
21 1.2323 9194
1.4647 2871
Y

1.2447 1586 1.2790 5071 1.3142 8848 1.3875 6370


22
B

1.2934 4003 1.3307 1709 1.4083 7715 1.4903 6146


23 1.2571 6302
1.3079 9123 1.3473 5105 1.4295 0281 1.5164 4279
24 1.2697 3465
re

1.3227 0613 1.3641 9294 1.4509 4535 1.5429 8054


25 1.2824 3200
1.3812 4535 1.4727 0953 1.5699 8269
26 1.2952 5631 1.3375 8657
oYu

1.3985 1092 1.4948 0018 1.5974 5739


27 1.3082 0888 1.3526 3442
1.6254 1290
ad

1.3212 9097 1.3678 5156 1.4159 9230 1.5172 2218


28
1.3832 3989 1.4336 9221 1.5399 8051 1.6538 5762
29 1.3345 0388
d

1.3988 0134 1.4516 1336 1.5630 8022 1.6828 0013


30 1.3478 4892
1.4145 3785 1.4697 5853 1.5865 2642 1.7122 4913
31 1.3613 2740
in

1.6103 2432 1.7422 1349


Re

32 1.3749 4068 1.4304 5140 1.4881 3051


1.4465 4398 1.5067 3214 1.6344 7918 1.7727 0223
33 1.3886 9009
1.6589 9637 1.8037 2452
F

34 1.4025 7699 1.4628 1760 1.5255 6629

1.5446 3587 1.6838 8132 1.8352 8970


35 1.4166 0276 1.4792 7430
1.5639 4382 1.7091 3954 1.8674 0727
36 1.4307 6878 1.4959 1613
1.5834 9312 1.7347 7663 1.9000 8689
37 1.4450 7647 1.5127 4519
1.5297 6357 1.6032 8678 1.7607 9828 1.9333 3841
38 1.4595 2724
1.5469 7341 1.6233 2787 1.7872 1025 1.9671 7184
39 1.4741 2251

1.5643 7687 1.6436 1946 1.8140 1841 2.0015 9734


40 1.4888 6373
1.6641 6471 1.8412 2868 2.0366 2530
41 1.5037 5237 1.5819 7611
1.5997 7334 1.6849 6677 1.8688 4712 2.U722 6624
42 1.5187 8989
1.6177 7079 1.7060 2885 1.8968 7982 2.1085 3090
43 1.5339 7779
1.6359 7071 1.7273 5421 1.9253 3302 2.1454 3019
44 1.5493 1757
1.7489 4614 1.9542 1301 2.1829 7522
45 1.5648 1075 1.6543 7538
1.7708 0797 1.9835 2621 2.2211 7728
46 1.5804 5885 1.6729 8710
1.7929 4306 2.0132 7910 2.2600 4789
47 1.5962 6344 1.6918 0821
1.7108 4105 1.8153 5485 2.0434 7829 2.2995 9872
48 1.6122 2608
1.7300 8801 1.8380 4679 2.0741 3046 2.3398 4170
49 1.6283 4834

1.7495 5150 1.8610 2237 2.1052 4242 2.3807 8893


50 1.6446 3182
T.4
APPLIED MATHEMATICS-XI

TABLE I

AMOUNT AT COMPOUND INTEREST


n
s = (l + /)
Periods Rate /

1 / 1 > 1
n .01(1%) .01125 1-% .0125 1-% .015 1-% .0175 1-%
{ 8 I 4 ) K 2 J I 4 )
51 1.6610 7814 1.7692 3395 1.8842 8515 2.1368 2106 2.4224 5274
52 1.6776 8892 1.7891 3784 1.9078 3872 2.1688 7337 2.4648 4566
53 1.6944 6581 1.8092 6564 1.9316 8670 2.2014 0647 2.5079 8046
54 1.7114 1047 1.8296 1988 1.9558 3279 2.2344 2757 2.5518 7012

ow
55 1.7285 2457 1.8502 0310 1.9802 8070 2.2679 4398 2.5965 2785
56 1.7458 0982 1.8710 1788 2.0050 3420 2.3019 6314 2.6419 6708
57 1.7632 6792 1.8920 6684 2.0300 9713 2.3364 9259 2.6882 0151
58 1.7809 0060 1.9133 5259 2.0554 7335 2.3715 3998 2.7352 4503
59 1.7987 0960 1.9348 7780 2.0811 6676 2.4071 1308 2.7831 1182

e
60 1.8166 9670 1.9566 4518 2.1071 8135 2.4432 1978 2.8318 1628

re
61 1.8348 6367 1.9786 5744 2.1335 2111 2.4798 6807 2.8813 7306
62 1.8532 1230 2.0009 1733

Frl
2.1601 9013 2.5170 6609 2.9317 9709
63 1.8717 4443

F
2.0234 2765 2.1871 9250 2.5548 2208 2.9831 0354
64 1.8904 6187 2.0461 9121 2.2145 3241 2.5931 4442 3.0353 0785
65 1.9093 6649 2.0692 1087 2.2422 1407 2.6320 4158 3.0884 2574
ou
66

or
1.9284 6015 2.0924 8949 2.2702 4174 2.6715 2221 3.1424 7319
67 1.9477 4475 2.1160 2999 2.2986 1976 2.7115 9504 3.1974 6647
68 1.9672 2220 2,1398 3533
69 1.9868 9442
2.3273 5251
kfs 2.7522 6896 3.2534 2213
2.1639 0848 2.3564 4442 2.7935 5300 3.3103 5702
70 2.0067 6337 2.1882 5245 2.3858 9997 2,8354 5629 3.3682 8827
oo
71 2.0268 3100 2.2128 7029 2.4157 2372 2.8779 8814 3.4272 3331
72 2.0470 9931 2.2377 6508 2.4459 2027 2.9211 5796
Y

3.4872 0990
73 2.0675 7031 2.2629 3994 2.4764 9427
B

2.9649 7533 3.5482 3607


74 2.0882 4601 2.2883 9801 2.5074 5045 3.0094 4996 3.6103 3020
75
re

2.1091 2847 2.3141 4249 2.5387 9358 3.0545 9171 3.6735 1098
76 2.1302 1975 2,3401 7659 2.5705 2850 3.1004 1059 3.7377 9742
77 2.1515 2195
oYu

2.3665 0358 2,6026 6011 3.1469 1674 3.8032 0888


78 2.1730 3717 2.3931 2675
ad

2.6351 9336 3.1941 2050 3.8697 6503


79 2.1947 6754 2.4200 4942 2.6681 3327 3.2420 3230 3.9374 8592
80 2.2167 1522 2.4472 7498 2.7014 8494
d

3.2906 6279 4.0063 9192


81 2.2388 8237 2.4748 0682 2.7352 5350 3,3400 2273 4.0765 0378
82
in

2.2612 7119 2.5026 4840 2.7694 4417 3.3901 2307 4.1478 4260
Re

83 2.2838 8390 2.5308 0319 2.8040 6222 3.4409 7492 4.2204 2984
84 2.3067 2274 2.5592 7473 2.8391 1300 3.4925 8954 4.2942 8737
F

85 2.3297 8997 2.5880 6657 2.8746 0191 3.5449 7838 4.3694 3740
86 2.3530 8787 2.6171 8232 2.9105 3444 3.5981 5306 4.4459 0255
87 2.3766 1875 2.6466 2562 2.9469 1612 3.6521 2535 4.5237 0584
8r: 2.4003 8494 2,6764 0016 2.9837 5257 3.7069 0723 4.6028 7070
89 2.4243 8879 2.7065 0966 3.0210 4948 3.7625 1084 4.6834 2093
90 2.4486 3267 2.7369 5789 3.0588 1260 3.8189 4851 4.7653 8080
91 2.4731 1900 2.7677 4867 3.0970 4775 3.8762 3273 4.8487 7496
92 2.4978 5019 2.7988 8584 3.1357 6085 3.9343 7622 4.9336 2853
93 2.5228 2869 2.8303 7331 3.1749 5786 3.9933 9187 5.0199 6703
94 2.5480 5698 2.8622 1501 3.2146 4483 4.0532 9275 5.1078 1645
95 2.5735 3755 2.8944 1492 3.2548 2789 4.1140 9214 5.1972 0324
96 2.5992 7293 2.9269 7709 3,2955 1324 4.1758 0352 5,2881 5429
97 2.6252 6565 2.9599 0559 3,3367 0716 4.2384 4057 5.3806 9699
98 2.6515 1831 2.9932 0452 3.3784 1600 4.3020 1718 5.4748 5919
99 2.6780 3349 3.0268 7807 3,4206 4620 4.3665 4744 5.5706 6923
100 2.7048 1383 3.0609 3045 3.4634 0427 4.4320 4565 5.6681 5594
T.5
LOGARITHMIC AND OTHER TABLES

TABLE 1
AMOUNT Al’ COMPOUND INTEREST
n
s = (1 + /)
Rale /
Periods

r_3
r„i .0275 2-%
.02(2%) .0225 2-% .025 24%
V 4
.03(3%)
n ( 4 2 )

1.0250 0000 1.0275 0000 1.0300 0000


1 1.0200 0000 1.0225 0000
1.0506 2500 1.0557 5625 1.0609 0000
2 1.0404 0000 1.0455 0625
1.0768 9063 1.0847 8955 1.0927 2700
3 1.0612 0800 1.0690 3014
1.1038 1289 1.1146 2126 1.1255 0881
4 1.0824 3216 1.0930 8332

ow
1 ' 1 1452 7334 1.1592 7407
1.1040 8080 11" .
9342 1.1767 6836 1.1940 5230
6 1.1261 6242 1.1428 2544
8575 1.2091 2949 1.2298 7387
7 1.1486 856“ 1.1685 3901
0290 1.2423 8055 1.2667 7008
8 1,171" 5938 1 1948 3114
-3 67 - -.2765 4602 1.3047 7316
9 1.1950 9257 1.2217 1484

e
1.2800 8454 1.3116 5103 1.3439 1638
10 1.2189 9442 1.2492 0343

re
1.3120 8666 1.3477 2144 1.3842 3387
11 1.2433 7431 1.2773 1050

rFl 1.3448 8882 1.3847 8378 1.4257 6089


12 1.2682 4179 1.3060 4999
1.4228 6533 1.4685 3371

F
1.2936 0663 1.3354 3611 1.3785 1104
13 1.5125 8972
1.3654 8343 1.4129 7382 1.4619 9413
14 1.3194 7876
1 , ;■32 "t'-d / .021 9896 1.5579 674_
1.3458 6831 1.3962 0: n I.
1.6047 064 1

r
●1

1.4845 0562 1.5435 0944


ou
16 1.3727 8571 1.4276 2146
1.6528 4763
1 / 1.4002 4142 1.4597 4294
1.492H 871*7
1.5216 1826
1.5596 5872 fo 1.5859 5595
1.6295 6973 1.7024 3306
1.4282 4625
ks
18 1.7535 0605
1.5261 7037 1.5986 5019 1.6743 8290
19 1.4568 1117
1.6386 1644 1.7204 2843 1.8061 1123
1.5605 0920
oo

20 1.4859 4740 1.8602 9457


1.5956 2066 1.6795 8185 1.7677 4021
21 1.5156 6634 1.9161 0341
1.7215 7140 1.8163 5307
Y

22 1.5459 7967 1.6315 2212


1.9735 8651
B

1.6682 3137 1.7646 1068 1.8663 0278


23 1.5768 9926
1.9176 2610 2.0327 9411
1.6084 3725 1.7057 6658 1.8087 2595
24
re

1.9703 6082 2.0937 7793


1.6406 0599 1.7441 4632 1.8539 4410
25 2.1565 9127
1.7833 89(4.: 1.9002 9270 2.0245 4575
26 1.0734 18n 2.2212 8901
ou

i.8235 1588 1.9478 0002 2.0802 2075


Y

27 1.7068 8640
2.2879 2768
ad

1.8645 4499 1.9964 9502 2.1374 2682


28 1.7410 24'’! 2.3565 6551
1 90e' '^727 ■7.0464 0739 2.1962 0606
29 ● .7758 4469
2.2566 0173 2.4272 6247
d

1.8113 6158 1.9493 9344 2.0975 6758


30 2.5000 8035
1.9932 5479 2.1500 0677 2.3186 5828
31 1.8475 8882
in

2.5750 8276
Re

2.0381 0303 2.2037 5694 2.3824 2138


32 1.8845 4059 2.6523 3524
2.0839 6034 2.2588 5086 2.4479 3797
33 1.9222 3140 2.7319 0530
2.5152 5626
F

1.9606 7603 2.1308 4945 2.3153 2213


34
2.37;iz 051'' 2 5344 '^4 2.8138 62 !."
:.1787 93-:
jL'?2 ib-1 9752 2.8982 783:’
36 2.0398 8731 ^.2278 1642
*;70 2370 2.9852 2GO0
37 -r.'"-306 8509 ?.2779 ■>;

-2.55" 7,3035 5810 3.0747 834ii


~ 1222 -T ●● l..S 9599
38 3.1670 2698
.1547 ‘ ’ ' .. J816

2.9598 7399 3.2620 3779


2.2080 3966 2.4351 8897 2.6850 6384
40 3.3598 9893
2.4899 8072 2.7521 9043 3.0412 7052
41 2.2522 0046 3.4606 9589
2.5460 0528 2.8209 9520 3.1249 0546
42 2.2972 4447 3.5645 1677
2.6032 9040 2.8915 2008 3.2108 4036
43 2.3431 8936
2.9638 0808 3.2991 3847 3.6714 5227
44 2.3900 5314 2.6618 6444
3.0379 0328 3.3898 6478 3.7810 .
2 4378 : ●"'.721/
15 3.8950 4372
2.7829 9590 3.1138 5086 3.4830 8606
46 2.4866 1129 4.0118 9503
2.8456 1331 3.1916 9713 3.5788 7093
47 2.5363 4:?..-^ 4.1322 5188
2.9096 3961 3.2714 8956 ;7.6772 8988
48 2.5870 7039 4.2562 1944
2.9751 0650 3.3532 7680 3.7784 1535
49 2.6388 117":
3.8823 2177 4.3839 0602
2.6915 8803 3.0420 4640 3.4371 0872
50
T.6
APPLIED MATHEMATICS-XI

TABLE 1
AMOUNT AT COMPOUND INTEREST
II
s = (1 + /)
Periods
Rate /

1 1
n .02(2%) .0225 2-%
^ 4
.025 2-%
2
.0275 2-%^ .03(3%)
4 J
51 2.7454 1979 3.1104 9244 3.5230 3644 3.9890 8562
52 2.8003 2819 4,5154 2320
3.1804 7852 3.6111 1235 4,0987 8547
53 2.8563 3475 4.6508 8590
3.2520 3929 3.7013 9016 4.2115 0208
54 2.9134 6144 4.7904 1247
3,3252 1017 3.7939 2491 4.3273 1838 4.9341 2485

ow
55 2.9717 3067 3.4000 2740 3.8887 7303 4.4463 1964 5.0821 4859
56 3.0311 6529 3.4765 2802 3.9859 9236 4.5685 9343
57 3.0917 8859 5.2346 1305
3.5547 4990 4.0856 4217 4.6942 2975
58 3.1536 2436 5,3916 5144
3.6347 3177 4.1877 8322 4,8233 2107
59 5.5534 0098
3.2166 9685 3.7165 1324 4.2924 7780 4.9559 6239 5.7200 0301
60

e
3.2810 3079 3,8001 3479 4.3997 8975
61 5.0922 5136 5.8916 0310
3.3466 5140

re
3.8856 3782 4.5097 8449 5.2322 8827
62 6.0683 5120

rFl
3.4135 8443 3.9730 6467 4.6225 2910
63 5.3761 7620 6.2504 0173
3.4818 5612 4.0624 5862

F
4.7380 9233 5.5240 2105 6.4379 1379
64 3.5514 9324 4.1538 6394 4.8565 4464 5.6759 3162 6.6310 5120
65 3.6225 2311 4.2473 2588 4.9779 5826 5.8320 1974
66 6,8299 8273

r
3.6949 7357 4.3428 9071 5.1024 0721
ou
67
5.9924 0029 7.0348 8222
3.7688 7304
68 3.8442 5050
4.4406 0576
4.5405 1939
5.2299 6739
5.3607 1658 fo
6.1571 9130 7.2459 2868
ks
6.3265 1406
69 7.4633 0654
3.9211 3551 4.6426 8107 5.4947 3449 6.5004 9319 7.6872 0574
70 3.9995 5822 4.7471 4140 5.6321 0286 6.6792 5676
oo

71 4.0795 4939 7.9178 2191


4.8539 5208 5.7729 0543 6.8629 3632
72 4.1611 4038 8.1553 5657
4.9631 6600 5.9172 2806
Y

73 7.0516 6706 8.4000 1727


4.2443 6318
B

5.0748 3723 6.0651 5876 7.2455 8791


74 4.3292 5045 8.6520 1778
5.1890 2107 6.2167 8773 7.4448 4158 8.9115 7832
re

75 4.4158 3546 5.3057 7405 6.3722 0743 7.6495 7472


76 9.1789 2567
4.5041 5216 5,4251 5396 6.5315 1261
77 7.8599 3802 9.4542 9344
ou

4.5942 3521 5.5472 1993 6.6948 0043


Y

78 8.0760 8632 9.7379 2224


ad

4,6861 1991 5.6720 3237 6.8621 7044


79
8.2981 7869 10.0300 5991
4.7798 4231 5.7996 5310 7.0337 2470 8.5263 7861 10.3309 6171
80 4.8754 3916 5.9301 4530
d

7.2095 6782 8.7608 5402 10,6408 9056


81 4.9729 4794 6.0635 7357 7.3898 0701
82 9.0017 7751 10.9601 1727
in

5,0724 0690
Re

6.2000 0397 7.5745 5219 9.2493 2639


83 5.1738 5504 11.2889 2079
6.3395 0406 7.7639 1599 9.5036 8286
84 11.6275 8842
5.2773 3214 6.4821 4290
F

7.9580 1389 9.7650 3414 11.9764 1607


85 5.3828 7878 6.6279 9112 8.1569 6424
86 10.0335 7258 12.3357 0855
5.4905 3636 6.7771 2092 8.3608 8834
87
10.3094 9583 12.7057 7981
5.6003 4708 6.9296 0614 8.5699 1055
88 10.5930 0696 13.0869 5320
5,7123 5402 7.0855 2228 8,7841 5832
89 10.8843 1465 13.4795 6180
5.8266 0110 7.2449 4653 9.0037 6228 11.1836 3331 13.8839 4865
90 5.9431 3313 7.4079 5782 9.2288 5633
91 11.4911 8322 14.3004 6711
6.0619 9579 7.5746 3688 9.4595 7774
92 11.8071 9076 14.7294 8112
6.1832 3570 7.7450 6621 9.6960 6718
93 12.1318 8851 15.1713 6556
6,3069 0042 7.9193 3020 9.9384 6886
94 12.4655 1544 15.6265 0652
6.4330 3843 8.0975 1512 10.1869 3058 12.8083 1711 16.0953 0172
95 6.5616 9920 8.2797 0921 10,4416 0385 13.1605 4584
96 6.6929 3318 16.5781 6077
8.4660 0267 10.7026 4395 13.5224 6085
97 6,8267 9184 17.0755 0559
8.6564 8773 10.9702 1004 13.8943 2852
98 6,9633 2768 17.5877 7076
8.8512 5871 11,2444 6530 14.2764 2255
99 7.1025 9423 18.1154 0388
9.0504 1203 11.5255 7693 14.6690 2417 18.6588 6600
100 7,2446 4612 9.2540 4630 11.8137 1635 15.0724 2234 19.2186 3198
LOGARITHMIC AND OTHER TABLES T.7

TABLE 1
AMOUNT AT COMPOUND INTEREST
It
s = (l + /)

Periods Rate /

1 1
.035 3-% .045 4-% .05(5%) .055 5~%
n
2 .04(4%) 2 J 2

1 1.0350 0000 1.0400 0000 1.0450 0000 1.0500 0000 1.0550 0000
2 1.0712 2500 1.0816 0000 1.0920 2500 1.1025 0000 1.1130 2500
3 1.1087 1788 1.1248 6400 1.1411 6613 1.1576 2500 1.1742 4138

ow
4 1.1475 2300 1.1698 5856 1.1925 1860 1.2155 0625 1.2388 2465

5 1.1876 8631 1.2166 5290 1.2461 8194 1.2762 8156 1.3069 6001
6 1.2292 5533 1.2653 1902 1.3022 6012 1.3400 9564 1.3788 4281
7 1.2722 7926 1.3159 3178 1.3608 6183 1.4071 0042 1.4546 7916
8 1.3168 0904 1.3685 6905 1.4221 0061 1.4774 5544 1.5346 8651

e
9 1.3628 9735 1.4233 1181 1.4860 9514 1.5513 2822 1.6190 9427

re
10 1.4105 9876 1.4802 4428 1.5529 6942 1.6288 9463 1.7081 4446

Frl
11 1.4599 6972 1.5394 5406 1.6228 5305 1.7103 3936 1.8020 9240

F
12 1.5110 6866 1.6010 3222 1.6958 8143 1.7958 5633 1.9012 0749
13 1.5639 5606 1.6650 7351 1.7721 9610 1.8856 4914 2.0057 7390
14 1.6186 9452 1.7316 7645 1.8519 4492 1.9799 3160 2.1160 9146
ou
or
15 1.6753 4883 1.8009 4351 1.9352 8244 2.0789 2818 2.2324 7649
16 1.7339 8604 1.8729 8125 2.0223 7015
kfs 2.1828 7459 2.3552 6270
17 1.7946 7555 1.9479 0050 2.1133 7681 2.2920 1832 2.4848 0215
18 1.8574 8920 2.0258 1652 2.2084 7877 2.4066 1923 2.6214 6627
19 1.9225 0132 2.1068 4918 2.3078 6031 2.5269 5020 2.7656 4691
oo

20 1.9897 8886 2.1911 2314 2.4117 1402 2.6532 9771 2.9177 5749
3.0782 3415
Y

21 2.0594 3147 2.2787 6807 2.5202 4116 2.7859 6259


B

22 2.1315 1158 2.3699 1879 2.6336 5201 2.9252 6072 3.2475 3703
23 2.2061 1448 2.4647 1554 2.7521 6635 3.0715 2376 3.4261 5157
2.2833 2849 2.5633 0416 2.8760 1383 3.2250 9994 3.6145 8990
re

24

25 2.3632 4498 2.6658 3633 3.0054 3446 3.3863 5494 3.8133 9235
oYu

26 2.4459 5856 2.7724 6978 3.1406 7901 3.5556 7269 4.0231 2893
ad

27 2.5315 6711 2.8833 6858 3.2820 0956 3.7334 5632 4.2444 0102
26 2.6201 7196 2.9987 0332 3.4296 9999 3.9201 2914 4.4778 4307
29 2.7118 7798 3.1186 5145 3.5840 3649 4.1161 3560 4.7241 2444
d

30 2.8067 9370 3.2433 9751 3.7453 1813 4.3219 4238 4.9839 5129
in

5.2580 6861
Re

31 2.9050 3148 3.3731 3341 3.9138 5745 4.5380 3949


32 3.0067 0759 3.5080 5875 4.0899 8104 4.7649 4147 5.5472 6238
5.0031 8854 5.8523 6181
F

33 3.1119 4235 3.6483 8110 4.2740 3018


34 3.2208 6033 3.7943 1634 4.4663 6154 5.2533 4797 6.1742 4171

35 3.3335 9045 3.9460 8899 4.6673 4781 5.5160 1537 6.5138 2501
36 3.4502 6611 4.1039 3255 4.8773 7846 5.7918 1614 6.8720 8538
37 3.5710 2543 4.2680 8986 5.0968 6049 6.0814 0694 7.2500 5008
38 3.6960 1132 4.4388 1345 5.3262 1921 6.3854 7729 7.6488 0283
39 3.8253 7171 4.6163 6599 5.5658 9908 6.7047 5115 8.0694 8699

40 3.9592 5972 4.8010 2063 5.8163 6454 7.0399 8871 8.5133 0877
41 4.0978 3381 4.9930 6145 6.0781 0094 7.3919 8815 8.9815 4076
42 4.2412 5799 5.1927 8391 6.3516 1548 7.7615 8756 9.4755 2550
43 4.3897 0202 5.4004 9527 6.6374 3818 8.1496 6693 9.9966 7940
44 4.5433 4160 5.6165 1508 6.9361 2290 8.5571 5028 10.5464 9677

45 4.7023 5855 5.8411 7568 7.2482 4843 8.9850 0779 11.1265 5409
46 4.8669 4110 6.0748 2271 7.5744 1961 9.4342 5818 11.7385 1456
47 5.0372 8404 6.3178 1562 7.9152 6849 9.9059 7109 12.3841 3287
48 5.2135 8898 6.5705 2824 8.2714 5557 10.4012 6965 13.0652 6017
49 5.3960 6459 6.8333 4937 8.6436 7107 10.9213 3313 13.7838 4948

50 5.5849 2686 7.1066 8335 9.0326 3627 11.4673 9979 14.5419 6120
T.8
APPLIED MATHEMAT1CS-XI

TABLE 1
AMOUNT AT COMPOUND INTEREST
n
s = (l + 0
Periods Rate i

1
n .035
^
3-%
2 .04{ 4%) .045f 4-% .05(5%) .055 5-%
1
I 2 2
51 5.7803 9930 7.3909 5068 9.4391 0490 12.0407 6978 15.3417 6907
52 5.9827 1327 7.6865 8871 9.8638 6463 12.6428 0826 16.1855 6637
53 6.1921 0824 7.9940 5226 10.3077 3853 13.2749 4868 17.0757 7252
54 6.4088 3202

ow
8.3138 1435 10.7715 8677 13.9386 9611 18.0149 4001
55 6.6331 4114 8.6463 6692 i 1.2563 0817 i4.6356 3092 19.0057 6171
56 6.8653 0108 8.9922 2160 11.7628 4204 15.3r- 6 20.0510 7860
57 7.1055 8662 9.3519 1046 12.2921 6993 16.1357 3309 21.1538 8793
58 7.3542 8215 9.7259 8688 12.8453 1 758 16.9425 22.3173 5176
59 7.6116 8203 10.1150 2635 13.4233 5687 17.7R'''' OOT 23.5448 0611

e
60 7.8780 9090 10.5196 2741

re
14.0274 0793 18.6791 8589 24.8397 7045
61 8.1538 2408

rFl
10.9404 1250 14.6586 4129 19.6131 4519 26.2059 5782
62 8.4392 0793 11.3780 2900 15.3182 8014 20.5938 0245 27.6472 8550

F
63 8.7345 8020 11.8331 5016 16.0076 0275 21.6234 9257 29.1678 8620
64 9.0402 9051 12.3064 7617 16.7279 4487 22.7046 6720 30.7721 1994
65 9.3567 0068 12.7987 3522

r
17.4807 0239 23.8399 0056 32.4645 8654
ou
66 9.6841 8520 13.3106 8463 18.2673 3400
67 10.0231 3168 13.8431 1201 19.0893 6403
fo 25.0318
26.2834
9559
9037
34.2501 3880
36.1338 9643
ks
68 10.3739 4129 14.3968 3649 19.9483 8541 27.5976 6488 38.1212 6074
69 10.7370 2924 14.9727 099'=; 20.8460 6276 ^0 Q775 4813 40.2179 3008
oo

70 11.1128 2526 15.5716 1835 21.7841 3558 30.4264 2554 42.4299 1623
71 11.5017 7414 16.1944 8308 22.7644 2168 31.9477 4681 44.7635 6163
Y

72 11.9043 3624 16.8422 6241 23.7888 2066


B

33.5451 3415 47.2255 5751


73 12.3209 8801 17.5159 5290 24.8593 1759 35.2223 9086 49.8229 6318
74 12.7522 2259 18.2165 9102 25.9779 8688 36.9835 1040 52.5632 2615
re

75 13.1985 5038 18.9452 54C'.' 27.1469 9629 38.8326 8592 55.4542 0359
76 13.6604 9964
ou

19.7030 6485 28.3686 1112 40.7743 2022 58.5041 8479


Y

77 14.1386 1713
ad

20.4911 8744 29.6451 9862 42.8130 3623 61.7219 1495


78 14.6334 6873 21.3108 3494 30.9792 3256 44.9536 8804 65.1166 2027
15.1456 4013 22.1632 6834 32.3732 9802 47.2013 7244 68.6980 3439
d

80 15.6757 3754 23.0497 9907 33.8300 9643 49.5614 4107 72.4764 2628
81 16.2243 8835
in

23.9717 9103 35.3524 5077 52.0395 1312


Re

76.4626 2973
82 16.7922 4195 24.9306 6267 36.9433 1106 54.6414 8878 80.6680 7436
83 17.3799 7041 25.9278 8918 38.6057 6006 57.3735 6322 85.1048 1845
F

84 17.9882 6938 26.9650 0475 40.3430 1926 60.2422 4138 89.7855 8347

5513 63.?'; -' 5314 94.7237 .


03' . :7-j 7l 12 99.7 ‘ '
ib 1C'

90 22.1121 7595 34.1193 3334 52.5371 0530 80.7303 6505 123.8002 0591
91 22.8861 0210 35.4841 0668 54.9012 7503 84.7668 8330 130.6092 1724
92 23.6871 1568 36.9034 7094 57.3718 3241 89.0052 2747 137.7927 2419
93 24.5161 6473 38.3796 0978 59.9535 6487 93.4554 8884 145.3713 2402
94 25.3742 3049 39.9147 9417 62.6514 7529 98.1282 6328 153.3667 4684
95 26.2R2:< 2856 41 .51 7. 65.4707 9168 61.8019 !
27.1815 1006 7730 170.70"- __ ■ '
28.13>^^ 62 ●: .^.97:1? 3078 180 CRQ'; 7969
98 -9,117= ■ -.'A 1^ v55 1732 189.9945 0657
99 ."1, ■,r,r ?n..- 17^ 7nQ7 Q-fiC-
●●●■■ -442 C-V-
100 31.1914 0798 50.5049 4818 81.5885 1803 131.5012 5785 211.4686 3567
T.9
LOGARITHMIC AND OTHER TABLES

TABLE 1
AMOUNT AT COMPOUND INTEREST
II
s = (14- i)

Periods Rate i

.065 64% .07(7%) .075 74% .08(8%)


n .06(6%) ( 2 ) 2

1.0700 0000 1.0750 0000 1.0800 0000


1 1.0600 0000 1.0650 0000
1.1342 2500 1.1449 0000 1.1556 2500 1.1664 0000
2 1.1236 0000
1.2597 1200

ow
1.1910 1600 1.2079 4963 1.2250 4300 1.2422 9688
3
1.3107 9601 1.3354 6914 1.3604 8896
4 1.2624 7696 1.2864 6635
1.4025 5173 1.4356 2933 1 1693 2808
5 1.3382 2558 1.3700 8666
1.4591 42?"- 1. 303“^ 1.5433 0153 1.5868 7432
6 1.4185 1911
8148 1.6590 4914 1.7138 2427
7 1.5036 3026 1.5539 8655
1.6549 9567 1 71R1 8618 1.7834 7783 1.8509 3021
1.5938 4807

e
8
1.7625 7C"- ● ,^84 5921 1.9172 3866 1.9990 0463
9 1.6894 7896

re
1.9671 5136 2.0610 3156 2.1589 2500
1.7908 4770 1.8771 3747

Frl
10
1.9991 5140 2.1048 5195 2.2156 0893 2.3316 3900
1.8982 9856

F
11
2.1290 9624 2.2521 9159 2.3817 7960 2.5181 7012
12 2.0121 9647
2.4098 4500 2.5604 1307 2.7196 2373
13 2.1329 2826 2.2674 8750
2.5785 3415 2.7524 4405 2.9371 9362
14 2.2609 0396 2.4148 7418
ou
or
2.7590 3154 2.9588 7735 3.1721 6911
15 2.3965 5819 2.5718 1101
2.739 - 2.9521 6375 3.1807 9315 3.4259 4264
16 2.5403 5168
17 2.6927 7279 2.9170 4637 .^.1588 1521
kfs 3.4193 5264 3.7000 1805
7 1066 5438 3.3799 3228 3.6758 0409 3.9960 1950
18 2.8543 3915
3.3085 8691 3.6165 275^' 3.9514 8940 4.3157 0106
3.0255 9950
oo
19

3.5236 4506 3.8696 8446 4.2478 5110 4.6609 5714


20 3.2071 3547
Y

4.1405 6237 4.5664 3993 5.0338 3372


21 3.3995 6360 3.7526 8199
B

3,9966 0632 4.4304 0174 4.9089 2293 5.4365 4041


22 3.6035 3742
4.2563 8573 4.7405 2986 5.2770 9215 5.8714 6365
23 3.8197 4966
re

4.5330 5081 5.0723 6695 5.6728 7406 6.3411 8074


24 4.0489 3464

4.8276 9911 5.4274 3264 6.0983 3961 6.8484 7520


25 4.2918 7072
oYu

5.1414 9955 5.8073 5292 6.5557 1508 7.3963 5321


26 4.5493 8296
ad

5,4756 9702 6.2138 6763 7.0473 9371 7.9880 6147


27 4.8223 4594
5.8316 1733 6,6488 3836 7 -5759 482-1 8.6271 0639
28 5.1116 8670
d

6.2106 7245 7.1142 5705 8.1441 4436 9.3172 7490


29 5.4183 8790
7.6122 5504 8.7549 5519 10.0626 5689
in

30 5.7434 9117 6.6143 6616


Re

7.0442 9996 8.1451 1290 9,4115 7683 10.8676 6944


31 6.0881 0064
7.5021 7946 8.7152 7080 10.1174 4509 11.7370 8300
32 6.4533 8668
F

7.9898 2113 9.3253 3975 10.8762 5347 12.6760 4964


33 6.8405 8988
9.9781 1354 11.6919 7248 13.6901 3361
34 7.2510 2528 8.5091 5950
10 ●●● ' ■ 18 12..Kcaa 7042 14 7.553 4429
35 7.6860 9.06:
i13 01 11 -IP ● 19 13.5115 - ..8' 7184
36 8.1472 .
::2rv.
1 ‘ . . .-'5 2558
- .-o
7563
:,1'52 9768

12.4160 7453 14.9744 5784 18.0442 3897 21.7245 2150


40 10.2857 1794
16.0226 6989 19.3975 5689 23.4624 8322
41 10.9028 6101 13.2231 1938
17.1442 5678 20.8523 7366 25.3394 8187
42 11.5570 3267 14.0826 2214
18.3443 5475 22.4163 0168 27.3666 4042
43 12.2504 5463 14.9979 9258
19.6284 5959 24.0975 2431 29.5559 7166
44 12.9854 8191 15.9728 6209
● ' .48 3860 31.9204 4939
●15 lor^ij 106 ●.0110 ●
- oi53 34.4740 8534
46 1 ..0304 y/-.. 18.1168 1951
li- 7^15 37.2320 1217
47 15.4659 1673 19.2944 1278
.008 40.2105 7314
18 16.3938 7173 20.5485
27 J39 2
n
43.4274 1899
19 1? :.'775 0403 21.8842 057

29.4570 2506 37.1897 4603 46.9016 1251


50 18.4201 5427 23.3066 7868
T.10
APPLIED MATHEMATICS-XI

TABLE 1

AMOUNT AT COMPOUND INTEREST


s = (l + /)
n

Periods
Rate i

( 1 ^
n
.06( 6%) .065 6-% .075 7-!-%
2 ) .07(7%) { 2
.08(8%)
51 19.5253 6353 24.8216 1279 31,5190 1682 39.9789 7698 50.6537 4151
52 20.6968 8534 26.4350 1762 33.7253 4799 42.9774 0026 54.7060 4084
53 21.9386 9846 28.1532 9377

ow
36.0861 2235 46.2007 0528 59,0825 2410
54 23.2550 2037 29.9832 5786 38.6121 5092 49.6657 5817 63.8091 2603
55 24.6503 2159 31.9321 6963 41.3150 0148 53.3906 9004 68.9138 5611
56 26,1293 4089 34.0077 6065 44.2070 5159 57.3949 9179 74.4269 6460
57 27,6971 0134 36.2182 6509 47,3015 4520 61,6996 1617 80.3811 2177
58 29.3589 2742 38,5724 5233 50.6126 5336 66.3270 8739 86.8116 1151

e
59 31.1204 6307 41.0796 6173 54.1555 3910 71.3016 1894 93.7565 4043

re
60 32.9876 9085 43.7498 3974 57,9464 2683 76.6492 4036 101.2570 6367

Frl
61 34.9669 5230 46.5935 7932 62.0026 7671 82.3979 3339 109.3576 2876

F
62 37.0649 6944 49.6221 6198 66.3428 6408 88.5777 7839 118.1062 3906
63 39.2888 6761 52.8476 0251 70.9868 6457 95.2211 1177
64 127.5547 3819
41.6461 9967 56.2826 9667 75.9559 4509 102.3626 9515 137.7591 1724
ou
or
65 44.1449 7165 59.9410 7195 81.2728 6124
66 110.0398 9729 148.7798 4662
46.7936 6994 63.8372 4163 86.9619 6153 118.2928 8959 160.6822 3435
67 49.6012 9014 67.9866 6234 93.0492 9884
kfs 127,1648 5631 173.5368 1310
68 52.5773 6755 72.4057 9539 99.5627 4976
69 136.7022 2053 187.4197 5815
55.7320 0960 77.1121 7209 106.5321 4224 146.9548 8707 202.4133 3880
oo

70 59.0759 3018 82.1244 6327 113.9893 9220


71 157.9765 0360 218.6064 0590
Y

62.6204 8599 87.4625 5339 121.9686 4965 169.8247 4137 236.0949 1837
B

72 66.3777 1515 93.1476 1936 130.5064 5513


73 182.5615 9697 254.9825 1184
70.3603 7806 99.2022 1461 139,6419 0699
74 196.2537 1675 275.3811 1279
74.5820 0074
re

105.6503 5856 149.4168 4047 210.9727 4550 297.4116 0181


75 79.0569 2079 112.5176 3187 159.8760 1931 226.7957 0141 321.2045 2996
oYu

76 83,8003 3603 119.8312 7794 171.0673 4066 243.8053 7902 346.9008 9236
ad

77 88.8283 5620 127.6203 1101 183.0420 5451 262.0907 8245 374.6529 6374
78 94.1580 5757 135.9156 3122 195.8549 9832 281.7475 9113 404.6252 0084
79 99.8075 4102 144.7501 4725
d

209.5648 4820 302.8786 6046 436.9952 1691


80 105.7959 9348 154.1589 0683 224.2343 8758 325.5945 6000 471.9548 3426
in

81
Re

112.1437 5309 164.1792 3577 239.9307 9471


82 350.0141 5200 509.7112 2101
118.8723 7828 174.8508 8609 256.7259 5034
83 376.2652 1340 550.4881 1869
F

126.0047 2097 186.2161 9369 274.6967 6686


84 404.4851 0440 594.5271 6818
133.5650 0423 198.3202 4628 293.9255 4054 434.8214 8723 642.0893 4164
85 141.5789 0449 211.2110 6229 314.5003 2838
86 467.4330 9878 693.4564 8897
150,0736 3875 224.9397 8134 336.5153 5137
87 502.4905 8119 748.9330 0808
159.0780 5708 239.5608 6712 360.0714 2596
88 540.1773 7477 808.8476 4873
168.6227 4050 255.1323 2349 385.2764 2578
89 580.6906 7788 873.5554 6063
178.7401 0493 271.7159 2451 412.2457 7558
90 624.2424 7872 943.4398 9748
189.4645 1123 289.3774 5961 441.1029 7988 671.0606 6463 1018.9150 8928
91 200.8323 8190 308.1869 9448 471.9801 8847
92 212.8823 2482
721.3902 1447 1100.4282 9642
328.2191 4912 505.0188 0166
93 225.6552 6431
775.4944 8056 1188.4625 6013
349.5533 9382 540.3701 1778
94 239.1945 8017
833.6565 6660 1283.5395 6494
372.2743 6441 578.1960 2602
95 896.1808 0910 1386.2227 3014
253,5462 5498 396.4721 9810 618.6697 4784 963.3943 6978 1497.1205 4855
96 268.7590 3028 422.2428 9098 661.9766 3019 1035.6489 4751 1616.8901 9244
97 284.8845 7209 449.6886 7889 708.3149 9430
98 301,9776 4642
1113.3226 1858 1746.2414 0783
478.9184 4302 757.8970 4390
99 320.0963 0520
1196.8218 1497 1885.9407 2046
510.0481 4181 810.9498 3698 1286.5834 5109 2036.8159 7809
100 339.3020 8351 543.2012 7103 867.7163 2557 1383.0772 0993 2199.7612 5634
T.11
LOGARITHMIC AND OTHER TABLES

TABLE II
PRESENT VALUE AT COMPOUND INTEREST
-H

v„-(! + /)
Periods Rate /

1 ^ ( 7
.004167
( 5 % .005 % .005833 -^% .00751
.0025 -%
n
4 ) U2 K2 U2 J 4 )

0.9958 5062 0.9950 2486 0.9942 0050 0.9925 5583


1 0.9975 0623
0.9917 1846 0.9900 7450 0.9884 3463 0,9851 6708
2 0.9950 1869
0.9876 0345 0.9851 4876 0.9827 0221 0.9778 3333
0.9925 3734

ow
3
0.9835 0551 0.9802 4752 0.9770 0303 0.9705 5417
4 0.9900 6219
0.9794 2457 0.9753 7067 0,9713 3690 0.9633 2920
5 0,9875 9321
0.9753 6057 0.9705 1808 0.9657 0363 0.9561 5802
6 0.9851 3038
0.9713 1343 0.9656 8963 0.9601 0303 0.9490 4022
7 0.9826 7370
0.9672 8308 0.9608 8520 0.9545 3492 0.9419 7540
8 0.9802 2314
0.9349 6318

e
0.9777 7869 0.9632 6946 0.9561 0468 0.9489 9909
9

re
0.9592 7249 0.9513 4794 0.9434 9537 0.9280 0315
0.9753 4034

rFl
10
0.9552 9211 0.9466 1487 0,9380 2357 0.9210 9494
11 0.9729 0807

F
0.9513 2824 0.9419 0534 0.9325 8350 0.9142 3815
12 0.9704 8187
0.9473 8082 0.9372 1924 0.9271 7499 0.9074 3241
13 0.9680 6171
0.9434 4978 0.9325 5646 0.9217 9783 0.9006 7733
14 0.9656 4759

r
ou
0.9395 3505 0.9279 1688 0.9164 5187 0.8939 7254
15 0.9632 3949
16 0.9608 3740 0.9356 3657 0.9233 0037
fo 0.9111 3691 0.8873 1766
ks 0.9058 5277 0.8807 1231
17 0.9584 4130 0.9317 5426 0.9187 0684
0.9278 8806 0.9141 3616 0.9005 9927 0,8741 5614
18 0.9560 5117
0.9240 3790 0.9095 8822 0.8953 7625 0,8676 4878
19 0.9536 6700
oo

0.9202 0372 0.9050 6290 0.8901 8352 0.8611 8985


20 0.9512 8878
Y

0.9163 8544 0.9005 6010 0.8850 2090 0.8547 7901


21 0.9489 1649
B

0.9125 8301 0.8960 7971 0.8798 8822 0.8484 1589


22 0.9465 5011
0.9087 9636 0.8916 2160 0.8747 8531 0.8421 0014
23 0.9441 8964
re

0,8871 8567 0.8697 1199 0.8358 3140


24 0.9418 3505 0.9050 2542
0.9012 7013 0.8827 7181 0.8646 6810 0.8296 0933
0.9394 8634
ou

25
Y

0.8975 3042 0.8783 7991 0.8596 5345 0.8234 3358


0.9371 4348
ad

26
0.8938 0623 0.8740 0986 0.8546 6789 0.8173 0380
27 0.9348 0646
0.8900 9749 0.8696 6155 0.8497 1125 0.8112 1966
28 0.9324 7527
d

0.8864 0414 0.8653 3488 0.8447 8335 0.8051 8080


29 0.9301 4990
0.8827 2611 0.8610 2973 0.8398 8403 0.7991 8690
in

0.9278 3032
Re

30
0.8790 6335 0.8567 4600 0.8350 1312 0.7932 3762
31 0.9255 1653
0.8754 1578 0.8524 8358 0.8301 7046 0.7873 3262
32 0.9232 0851
F

0.8717 8335 0.8482 4237 0.8253 5589 0.7814 7158


33 0.9209 0624
0,8681 6599 0,8440 2226 0,8205 6924 0.7756 5418
34 0.9186 0972
0.8645 6365 0.8398 2314 0.8158 1035 0.7698 8008
35 0,9163 1892
0.8609 7624 0.8356 4492 0.8110 7905 0.7641 4896
36 0.9140 3384
0.8574 0373 0.8314 8748 0.8063 7520 0.7584 6051
37 0.9117 5445
0.8273 5073 0.8016 9863 0.7528 1440
38 0.9094 8075 0.8538 4604

0.8503 0311 0.8232 3455 0.7970 4918 0.7472 1032


39 0.9072 1272
0.8467 7488 0.8191 3886 0.7924 2669 0.7416 4796
40 0.9049 5034
0.8432 6129 0.8150 6354 0.7878 3101 0.7361 2701
41 0.9026 9361
0.8397 6228 0.8110 0850 0.7832 6199 0.7306 4716
42 0.9004 4250
0.8069 7363 0.7787 1946 0.7252 0809
43 0.8981 9701 0,8362 7779
0.8328 0776 0.8029 5884 0.7742 0328 0.7198 0952
44 0.8959 5712
0.8293 5212 0.7989 6402 0.7697 1329 0.7144 5114
45 0.8937 2281
0.7949 8907 0.7652 4933 0.7091 3264
46 0.8914 9407 0,8259 1083
0.7910 3390 0.7608 1127 0.7038 5374
47 0.8892 7090 0.8224 8381
0,7870 9841 0,7563 9895 0.6986 1414
48 0.8870 5326 0.8190 7102
0.7831 8250 0.7520 1221 0.6934 1353
49 0.8848 4116 0.8156 7238

0.7792 8607 0.7476 5092 0.6882 5165


50 0.8826 3457 0.8122 8785
T.12
APPLIED MATHEMATICS-XI

TABLE II

PRESENT VALUE AT COMPOUND INTEREST


-n
v„=0 + 0
Periods
Rate i

.0025
( 1% 5 1 r 7 1
n .004167! —% .005 -% .005833 —% .0075 -%
U 12 2 U2 4 ;
51 0.8804 3349 0.8089 1736 0.7754 0902 0.7433 1491 0.6831 2819
52 0.8782 3790 0.8055 6086 0.7715 5127 0.7390 0406
53
0.6780 4286
0.8760 4778 0.8022 1828

ow
0.7677 1270 0.7347 1821 0.6729 9540
54 0.8738 6312 0.7988 8957 0.7638 9324 0.7304 5721 0.6679 8551
55 0.8716 8391 0 7955 7468 0.7600 9277 0.7262 2092 0.6630 1291
56 0.8695 1013 0.7922 7354 0.7563 1122 0.7220 0920 0.6580 7733
57 0.8673 4178 0.7889 8610 0.7525 4847 0.7178 2191 0.6531 7849
58 0.8651 7883 0.7857 1230 0.7488 0445 0.7136 5890 0.6483 1612

e
59 0.8630 2128 0.7824 5208 0.7450 7906 0.7095 2004 0.643^ 8995

re
60 0.8608 6911 0.7792 0539 0.7413 7220 0.7054 0518 0.6386 9970

Frl
61 0.8587 2230 0.7759 7217 0.7376 8378 0.7013 1418 0.6339 4511

F
62 0.8565 8085 0.7727 5237 0.7340 1371 0.6972 4691
63
0.6292 2592
0.8544 4474 0.7695 4593 0.7303 6190 0.6932 0323 0.6245 4185
64 0.8523 1395 0.7663 5279 0.7267 2826 0.6891 8299 0.6198 9266
ou
or
65 0.8501 8848 0.7631 7291 0.7231 1269 0.6851 8608 0.6152 7807
66 0.8480 6831 0.7600 0621 0.7195 1512 0.6812
kfs 1234 0.6106 9784
67 0.8459 5343 0.7568 5266 0.7159 3544 0.6772 6165 0.6061 5170
66 0.8438 4382 0.7537 1219 0.7123 7357 0.6733 3387 0.6016 3940
69 0.8417 3947 0.7505 8476 0.7088 2943 0.6694 2887 0.5971 6070
oo

70 0.8396 4037 0.7474 7030 0.7053 0291 0.6655 4652 0.5927 1533
71
Y

0.8375 4650 0.7443 6876 0.7017 9394 0.6616 8668 0.5883 0306
B

72 0.8354 5786 0.7412 8009 0.6983 0243 0.6578 4923 0.5839 2363
73 0.8333 7442 0.7382 0424 0.6948 2829 0.6540 3404
74
0.5795 7681
0.8312 9618 0.7351 4115
re

0.6913 7143 0.6502 4097 0.5752 6234


75 0.8292 2312 0.7320 9078 0.6879 3177 0.6464 6989 0.5709 7999
oYu

76 0.8271 5523 0.7290 5306 0.6845 0923 0.6427 2069 0.5667 2952
ad

77 0.8250 9250 0.7260 2794 0.6811 0371 0.6389 9323 0.5625 1069
78 0.8230 3491 0.7230 1537 0.6777 1513 0.6352 8739 0.5583 2326
79 0.8209 8246 0.7200 1531 0.6743 4342
d

0.6316 0304 0.5541 6701


80 0.8189 3512 0.7170 2770 0.6709 8847 0.6279 4006
in

81 0.5500 4170
0.8168 9289
Re

0.7140 5248 0.6676 5022 0.6242 9832


82 0.5459 4710
0.8148 5575 0.7110 8960 0.6643 2858 0.6206 7771 0.5418 8297
83 0.8128 2369
F

0.7081 3902 0.6610 2346 0.6170 7809


84
0.5378 4911
0.8107 9670 0.7052 0069 0.6577 3479 0.6134 9934 0.5338 4527
85 0.8087 7476 0.7022 7454 0.6544 6248 0.6099 4135 0.5298 7123
86 0.8067 5787 0.6993 6CC- 0.6512 0644 0.6064 0400 0.5259 2678
87 0.804:’ 4600 0.6964 oaovj 0.6479 6661 0.6028 871
88 0.802’ 3915 0.69 on -
0.6447 4290 0.5993 9071 0.5181 2575
89 0.800? 3731 0.6906 9088 0.6415 3522 0.5959 1455 0.5142 6873
90 0.7987 4046 0.6878 2495 0.6383 4350 0.5924 5854 0.5104 4043
91 0.7967 4859 0.6849 7090 0.6351 6766 0.5890 2258
92 0.5066 4063
0.7947 6168 0.6821 2870 0.6320 0763 0.5856 0654 0.5028 6911
93 0.7927 7973 0.6792 9829 0.6288 6331 0.5822 1032
94 0.4991 2567
0.7908 0273 0.6764 7962 0.6257 3464 0.5788 3379 0.4954 1009
'■J 7888 3065 0.6736 ■26f' ’?.622R 2153 0.5754 7681 0.4917 2217
0.7868 6349 0.6708 7733 '.6195 2391 0.5721 3936 0.4880 6171
97 0.7849 0124 0.6680 9361 0.6164 4170 0.5688 2124 0. ;:.14 2850
98 0.7829 4388 0.6653 2143 n.6133 7483 0.5655 2236 0. -::3
>U
0.7809 9140 0.6625 G07- 0.6103 2321 0.5622 4262 0.4772
100 0.7790 4379 0.6598 1155 0.6072 8678 0.5589 8189 0.4736 9033
T.13
LOGARITHMIC AND OTHER TABLES

TABLE II
PRESENT VALUE AT COMPOUND INTEREST
-It
V
(I = (! + /)
Rate /
Periods

1 r.i r.3

n .01(1%)
.01125fl4%^
8
.0125 1-7%
V 4 )
.0115 1~%
2
.0175
V 4

0.9876 5432 0.9852 2167 0.9828 0098


1 0.9900 9901 0.9888 7515
0.9706 6175 0.9658 9777
0.9802 9605 0.9778 7407 0.9754 6106
2 0,9492 8528
0.9669 9537 0.9634 1833 0.9563 1699
3 0.9705 9015
0.9515 2428 0.9421 8423 0.9329 5851
0.9562 3770

ow
4 0.9609 8034
0.9397 7706 0.9282 6033 0.9169 1254
5 0,9514 6569 0,9455 9970
0.9281 7488 0.9145 4219 0.9011 4254
6 0.9420 4524 0.9350 8005
0,9167 1593 0.9010 2679 0.8856 4378
7 0.9327 1805 0.9246 7743
0,8877 1112 0.8704 1157
0.9234 8322 0,9143 9054 0.9053 9845
8 0,8554 4135
0.9042 1808 0,8942 2069 0.8745 922 !
9 0.9143 3982

e
0.8831 8093 0.8616 6723 0.8407 2860
0.8941 5880

re
10 0.9052 8695
0.8722 7746 0.8489 3323 0.8262 6889
11 0.8963 2372 0.8842 1142

Frl
0.8615 0860 0.8363 8742 0.8120 5788
12 0.8874 4923 0.8743 7470
0.7980 9128

F
0.8646 4742 0.8508 7269 0.8240 2702
13 0,8786 6260 0.7843 6490
0.8550 2835 0.8403 6809 0.8118 4928
14 0.8699 6297
0.8299 9318 0.7998 51-. 0.7708 7459
15 0.8613 4947 0.8455 1629
0.7576 1631
ou
or
0.8197 4635 0.7880 Si:-’
16 0.8528 2126 0.8361 1005
0.7763 0,7445 8605
0.8443 7749 0.8268 0846 0.8096 2602
17 0.7317 7990
0.7649 11""
18 0.8360 1731 0.8176 1034 0.7996 3064
kfs 0.7536 074 7 0.7191 9401
0,8277 3992 0.8085 1455 0.7897 5866
19
0.7800 0855 0.7424 7042 0.7068 2458
oo
20 0.8195 4447 0.7995 1995
0.7703 7881 0,7314 9795 0.6946 6789
21 0.8114 3017 0.7906 2542
0.7206 8763 0.6827 2028
Y

0.8033 9621 0.7818 2983 0.7608 6796


22 0.6709 7817
B

0.7731 3210 0.7514 7453 0.7100 3708


23 0.7954 4179
0.7421 9707 0.6995 4392 0.6594 3800
24 0.7875 6613 0.7645 3112
0.6480 9632
re

0.7560 2583 0.7330 3414 0.6892 0583


25 0,7797 6844 0.6369 4970
0.7476 1516 0.7239 8434 0.6790 2052
26 0.7720 4796 0.6259 9479
oYu

0.7392 9806 0.7150 4626 0.6689 8574


27 0.7644 0392 0.6152 2829
0.6590 9925
ad

0.7568 3557 0.7310 7348 0.7062 1853


28 0.6046 4697
0.7229 4040 0.6974 9978 0.6493 5887
29 0.7493 4215
0.5942 4764
d

0.7148 9780 0.6888 8867 0.6397 6243


30 0.7419 2292 0.5840 2716
0.7069 4467 0.6803 8387 0.6303 0781
0.7345 7715
in

31 0.5739 8247
0.6209 9292
Re

0.7273 0411 0.6990 8002 0.6719 8407


32 0.5641 1053
0.6913 0287 0.6636 8797 0.6118 1568
33 0.7201 0307 0.5544 0839
0.6554 9429 0.6027 7407
F

34 0.7129 7334 0.6836 1223


0.5938 6608 0.5448 7311
0.7059 142'" 0.6760 071f 0.64-' ●~7
35 O.E35C 9974 0.5355 0122
0 6684 8667 09 1'-
36 0.6989 "'-95 0.5262
1 :.-‘09
0.6920 04? 0 6610
■-i. I 72 4002
. . ; 37 0.65?
. 506.- --00

0.5512 6232 0.4996 0098


0.6716 5314 0.6392 3216 0.6084 1334
40 0.4910 0834
0.6321 2080 0.6009 0206 0.5431 1559
41 0.6650 0311 0.4825 6348
0.6250 8855 0.5934 8352 0.5350 8925
42 0.6584 1892 0.4742 6386
0.6181 3454 0.5861 5656 0.5271 8153
43 0.6518 9992 0.4661 0699
0.6112 5789 0.5789 2006 0.5193 9067
44 0.6454 4546
0.4580 9040
45 0.6390 5492 0,004- 5/ .' - \ L.J / . I

O4502 1170
0.5977
● ‘ 0.564.' 1397
46 0.6327 2764 0.1424 6850
0 5910
t n 43;.,’ 021-'
47 U.0264 6301 at-
'' 1348 5848
0.5845 0784 '<,a50b ,3-19 ■ 4893 G17“.
48 0.6202 6041 0.4273 7934
0.5440 0,4821 297;.
49 0.6141 1921 0.5780 052b

0,5373 3905 0.4750 0468 0.4200 2883


50 0.6080 3882 0.5715 7506
T.14
APPLIED MATHEMATICS-XI

TABLE II

PRESENT VALUE AT COMPOUND INTEREST


-n
v„=(l + 0
Periods
Rate '/

1 1
n
.01(1%) .01125 1-% .0125 1-% .0115 1-% .017sfl-%
^ 8 J 4 J 2 J 4 j
51 0.6020 1864 0.5652 1637 0.5307 0524 0,4679 8491
52 0.5960 5806 0.4128 0475
0.5589 2843 0.5241 5332 0.4610 6887
53 0.5901 5649 0.4057 0492
0.5527 1044 0.5176 8229 0.4542 5505
54 0.5843 1336 0.3987 2719
0.5465 6162 0.5112 9115 0.4475 4192 0.3918 6947
55

ow
0.5785 2808 0,5404 8120
56
0.5049 7892 0.4409 2800 0.3851 2970
0,5728 0008 0.5344 6843 0.4987 4461
57 0.4344 1182 0.3785 0585
0.5671 2879 0.5285 2256 0,4925 8727
58 0.4279 9194 0.3719 9592
0.5615 1365 0.5226 4282 0.4865 0594
59 0.4216 6694 0.3655 9796
0,5559 5411 0.5168 2850 0,4804 9970 0.4154 3541 0.3593 1003
60 0.5504 4962 0.5110 7887 0.4745 6760

e
61 0.4092 9597 0.3531 3025
0.5449 9962 0.5053 9319 0.4687 0874 0.4032 4726

re
62 0,5396 0358 0.3470 5676
0.4997 7077 0.4629 2222

rFl
63 0.3972 8794 0.3410 8772
0.5342 6097 0.4942 1090
64
0.4572 0713 0.3914 1669 0.3352 2135

F
0.5289 7126 0.4887 1288 0.4515 6259 0.3856 3221 0.3294 5587
65 0.5237 3392 0.4832 7602 0,4459 8775 0.3799 3321
66 0.5185 4844 0.3237 8956
0.4778 9965 0.4404 8173 0.3743 1843

r
67 0.3182 2069
ou
0.5134 1429 0.4725 8309 0.4350 4368 0.3687 8663 0.3127 4761
68
69
0.5083 3099
0.5032 9801
0.4673 2568 0.4296 7277
fo 0,3633 3658 0.3073 6866
ks
0.4621 2675 0.4243 6817 0.3579 6708 0.3020 8222
70 0.4983 1486 0.4569 8566 0.4191 2905 0.3526 7692
71 0.4933 8105 0.2968 8670
0.4519 0177
oo

0.4139 5462 0.3474 6495


72 0.4884 9609 0.2917 8054
0.4468 7443 0.4088 4407 0.3423 3000
73 0.4836 5949 0.2867 6221
Y

0.4419 0302 0.4037 9661 0.3372 7093 0,2818 3018


B

74 0.4788 7078 0.4369 8692 0.3988 1147 0.3322 8663 0.2769 8298
75 0.4741 2949 0.4321 2551 0,3938 8787
re

76 0,3273 7599 0,2722 1914


0.4694 3514 0,4273 1818
77
0.3890 2506 0.3225 3793 0.2675 3724
0,4647 8726 0.4225 6433 0.3842 2228 0.3177 7136
ou

78 0.4601 8541 0.2629 3586


Y

0,4178 6337 0.3794 7879


ad

79 0,3130 7523 0.2584 1362


0,4556 2912 0.4132 1470 0.3747 9387 0,3084 4850 0.2539 6916
80 0.4511 1794 0.4086 1775 0.3701 6679 0.3038 9015
81 0.4466 5142 0.2496 0114
d

0.4040 7194 0.3655 9683


82 0.2993 9916 0,2453 0825
0.4422 2913 0.3995 7670 0.3610 8329 0.2949 7454
in

0.2410 8919
Re

83 0.4378 5063 0.3951 3148 0.3566 2547 0.2906 1531


84 0.4335 1547 0.2369 4269
0.3907 3570 0.3522 2268 0.2863 2050 0,2328 6751
F

85 0.4292 2324 0.3863 8882 0.3478 7426 0.2820 8917


86 0.4249 7350 0,2288 6242
0.3820 9031 0,3435 7951
87 0.2779 2036 0.2249 2621
0.4207 6585 0.3778 3961 0.3393 3779
88 0.2738 1316 0.2210 5770
0.4165 9985 0.3736 3621
89
0.3351 4843 0.2697 6666 0.2172 5572
0.4124 7510 0.3694 7956 0.3310 1080 0,2657 7996 0.2135 1914
90 0.4083 9119 0.3653 6916 0.3269 2425 0.2618 5218
91 0.4043 4771 0.2098 4682
0.3613 0448 0.3228 8814 0.2579 8245
92 0.4003 4427 0.2062 3766
0.3572 8503 0.3189 0187 0.2541 6990
93 0.3963 8046 0.2026 9057
0.3533 1029 0.3149 6481 0.2504 1369
94 0.3924 5590 0.1992 0450
0.3493 7976 0.3110 7636 0.2467 1300 0.1957 7837
95 0,3885 7020 0.3454 9297 0.3072 3591 0,2430 6699
96 0.3847 2297 0.1924 1118
0.3416 4941 0.3034 4287 0.2394 7487
97 0.3809 1383 0.1891 0190
0,3378 4861 0,2996 9666 0,2359 3583
98 0.3771 4241 0.1858 4953
0.3340 9010 0.2959 9670 0,2324 4909
99 0.3734 0832 0,1826 5310
0.3303 7340 0,2923 4242 0.2290 1389 0.1795 1165
100 0.3697 1121 0.3266 9805 0.2887 3326 0.2256 2944 0.1764 2422
LOGARITHMIC AND OTHER TABLES T.15

TABLE I!
PRESENT VALUE AT COMPOUND INTEREST
-n
V„ =(1 + 0
Periods Rate i

n .02(2%)
.0225 2-% .025 2-%
1
.0275f 2-%'' .03( 3%)
V 4 ) { 2 ) i 4 J

1 0.9803 9216 0.9779 9511 0.9756 0976 0.9732 3601 0.9708 7379
2 0.9611 6878 0.9564 7444 0.9518 1440 0.9471 8833 0.9425 9591
3 0.9423 2233 0.9354 2732 0.9285 9941 0.9218 3779 0.9151 4166

ow
4 0.9238 4543 0.9148 4335 0.9059 5064 0.8971 6573 0.8884 8705

5 0.9057 3081 0.8947 1232 0.8838 5429 0.8731 5400 0.8626 0878
6 0.8879 7138 0.8750 2427 0.8622 9687 0.8497 8491 0.8374 8426
7 0.8705 6018 0.8557 6946 0.8412 6524 0.8270 4128 0.8130 9151
8 0.8534 9037 0.8369 3835 0.8207 4657 0.8049 0635 0.7894 0923
0.8367 5527 0.8185 2161 0.8007 2836 0.7833 6385 0.7664 1673

e
9

re
0.7440 9391
10
11
0.8203
0.8042
4830
6304
0.8005 1013
0.7828 9499

rFl 0.7811
0.7621
9840
4478
0.7623 9791
0.7419 9310 0.7224 2128

F
12 0.7884 9318 0.7656 6748 0.7435 5589 0.7221 3440 0.7013 7988
13 0.7730 3253 0.7488 1905 0.7254 2038 0.7028 0720 0.6809 5134
14 0.7578 7502 0.7323 4137 0.7077 2720 0.6839 9728 0.6611 1781

r
0.7430 1473 0.7162 2628 0.6904 6556 0.6656 9078 0.6418 6195
ou
15
16 0.7284 4581 0.7004 6580 0.6736
fo
2493 0.6478 7424
0.6305 3454
0.6231 6694
0.6050 1645
ks
17 0.7141 6256 0.6850 5212 0.6571 9506
18 0.7001 5937 0.6699 7763 0.6411 6591 0.6136 5892 0.5873 9461
19 0.6864 3076 0.6552 3484 0.6255 2772 0.5972 3496 0.5702 8603
oo

20 0.6729 7133 0.6408 1647 0.6102 7094 0.5812 5057 0.5536 7575
Y

21 0.6597 7582 0.6267 1538 0.5953 8629 0.5656 9398 0.5375 4928
eB

22 0.6468 3904 0.6129 2457 0.5808 6467 0.5505 5375 0.5218 9250
23 0.6341 5592 0.5994 3724 0.5666 9724 0.5358 1874 0.5066 9175
24 0.6217 2149 0.5862 4668 0.5528 7535 0.5214 7809 0.4919 3374
r

25 0.6095 3087 0.5733 4639 0.5393 9059 0.5075 2126 0.4776 0557
ou

0.5262 3472 0.4939 3796 0.4636 9473


ad
Y

26 0,5975 7928 0.5607 2997


27 0.5858 6204 0.5483' 9117 0.5133 9973 0.4807 1821 0.4501 8906
28 0.5743 7455 0.5363 2388 0.5008 7778 0.4678 5227 0.4370 7675
29 0.5631 1231 0.5245 2213 0.4886 6125 0.4553 3068 0.4243 4636
d

30 0.5520 7089 0.5129 8008 0.4767 4269 0.4431 4421 0.4119 8676
Re
in

31 0.5412 4597 0.5016 9201 0.4651 1481 0.4312 8391 0.3999 8715
32 0.5306 3330 0.4906 5233 0.4537 7055 0.4197 4103 0.3883 3703
F

33 0,5202 2873 0.4798 5558 0.4427 0298 0.4085 0708 0.3770 2625
34 0.5100 2817 0.4692 9641 0.4319 0534 0.3975 7380 0.3660 4490

35 0.5000 2761 0,4589 6960 0.4213 7107 0.3869 3314 0.3553 8340
36 0.4902 2315 0.4488 7002 0.4110 9372 0.3765 7727 0.3450 3243
37 0,4806 1093 0,4389 9268 0.4010 6705 0.3664 9856 0.3349 8294
38 0,4711 8719 0.4293 3270 0.3912 8492 0,3566 8959 0.3252 2615
39 0.4619 4822 0.4198 8528 0.3817 4139 0.3471 4316 0,3157 5355

40 0.4528 9042 0.4106 4575 0.3724 3062 0.3378 5222 0.3065 5684
41 0.4440 1021 0.4016 0954 0.3633 4695 0.3288 0995 0.2976 2800
42 0.4353 0413 0.3927 7216 0.3544 8483 0.3200 0968 0.2889 5922
43 0.4267 6875 0.3841 2925 0.3458 3886 0.3114 4495 0.2805 4294
44 0.4184 0074 0.3756 7653 0.3374 0376 0.3031 0944 0.2723 7178

45 0.4101 9680 0.3674 0981 0.3291 7440 0.2949 9702 0.2644 3862
46 0.4021 5373 0.3593 2500 0.3211 4576 0.2871 0172 0.2567 3653
47 0.3942 6836 0.3514 1809 0.3133 1294 0.2794 1773 0.2492 5876
48 0.3865 3761 0.3436 8518 0.3056 7116 0.2719 3940 0.2419 9880
0,3789 5844 0.3361 2242 0.2982 1576 0.2646 6122 0,2349 5029

0.3715 2788 0.3287 2608 0.2909 4221 0.2575 7783 0,2281 0708
T.16
APPLIED MATHEMATICS-XI

TABLE II

PRESENT VALUE AT COMPOUND INTEREST


-n
v„ =(! + /)
Periods Rate /■

1., ^
.0225 2-%
n
.02(2%) .025 2-% .0275 2-%
.03(3%)
^ 4 J { 2 J 4 J
51 0.3642 4302 0.3214 9250 0.2838 4606 0.2506 8402 0.2214 6318
52 0.3571 0100 0.3144 1810 0.2769 2298 0.2439 7471 0.2150 1280
53 0.3500 9902 0.3074 9936 0.2701 6876 0.2374 4497 0.2087 5029
54 0.3432 3433 0.3007 3287 0.2635 7928 0.2310 9000 0.2026 7019

55 0.3365 0425 0.2941 1528 0.2571 5052 0.2249 0511 0.1967 6717
56 0.3299 0613

w
0.2876 4330 0.2508 7855 0.2188 8575 0.1910 3609
57 0.3234 3738 0.2813 1374 0.2447 5956 0.2130 2749 0.1854 7193
58 0.3170 9547 0.2751 2347 0.2387 8982 0.2073 2603 0.1800 6984
59 0.3108 7791 0.2690 6940 0.2329 6568 0.2017 7716 0.1748 2508

F lo
60 0.3047 8227 0.2631 4856 0.2272 8359 0.1963 7679 0.1697 3309
61 0.2988 0614 0.2573 5801 0.2217 4009 0.1911 2097 0.1647 8941
62 0.2929 4720 0.2516 9487

ee
0.2163 3179 0.1860 0581 0.1599 8972
63 0.2872 0314 0.2461 5635 0.2110 5541 0.1810 2755 0.1553 2982

Fr
64 0.2815 7170 0.2407 3971 0.2059 0771 0.1761 8253 0.1508 0565

65 0.2760 5069 0.2354 4226 0.2008 8557 0.1714 6713 0.146J 1325
66 0.2706
0.2653
379o
3130
0.2302 6138 0.1959 8593
for
0.1668 7804 0.1421 4879
ur
0.2251 9450 0.1912 0578 0.162«i 1172 0.1380 0853
0.2601 2873 0.2202 3912 0.1865 4223 0.1580 6493 0.1339 8887
o9 0,2550 2817 0.2153 9278 0.1819 9241 0.1538 3448 ●: :328
s
70 0.2500 2761
ook

0.2106 5309 0.1775 5358 0.1497 1726 0.1262 9736


Yo

71 0.2451 2511 0.2060 1769 0.1732 2300 0.1457 1023 0.1226 1880
72 0.2403 1874 0.2014 8429 0.1689 9805 0.1418 1044 0.1190 4737
eB

73 0.2356 0661 0.1970 5065 0.1648 7615 0.1380 1503 0.1155 7998
74 0.2309 8687 0.1927 1458 0.1608 5478 0.1343 2119 0.1122 1357

0.2264 57/1 0.1884 7391 0.1569 3149 0.1307 2622 0.1089 452’
r

76 0.2220 1737 0.1843 2657 0.1531 0389 0.1272 2747 0.1057 7205
ou
ad

77 0.2176 6408 0.1802 7048 0.1493 6965 0.1238 2235 0.1026 9131
78 0.2133 9616 0.1763 0365 0.1457 2649 0.1205 0837 0.0997 0030
79
Y

0.2092 1192 0.1724 2411 0.1421 7218 0.1172 8309 0.0967 9641
80 0.2051 0973 0.1686 2993 0.1387 0457 0.1141 4412 0.0939 7710
Re
nd

81 0.2010 8797 0.1649 1925 0.1353 2153 0.1110 8917 0.0912 3990
82 0.1971 4507 0.1612 9022 0.1320 2101 0.1081 1598 0.0885 8243
Fi

83 0.1932 7948 0.1577 4105 0.1288 0098 0.1052 2237 0.0860 0236
84 0.1894 8968 0.1542 6997 0.1256 5949 0.1024 0620 0.0834 9743
85 0.1857 7420 0.1508 7528 0.1225 9463 0.U996 6540 r ■ -:47
0.1821 3157 ' ■'■'^5 5528 0.11. 0452 0.0969 -’9^ : -34
C' 0.1785 6036 0,1443 0835 0.116.: ‘"33 0.094-" ?190 -,,076'’ ”98
88 0,1750 5918 0.1411 3286 '' - --- J130 0.0918 :..0741 8639
0.1716 2665 01380 2724 0.1110 6468 0 080- 16c .'.07 256?
90 0.1682 6142 0.1349 8997 0.1083 5579 0.0870 2324 0.0699 2779
91 0.1649 6217 0.1320 1953 0.1057 1296 0.0846 9415 0.0678 9105
92 0.1617 2762 0.1291 1445 0.1031 3460 0.0824 2740 0.0659 1364
93 0.1585 5649 0.1262 7331 0.1006 1912 0.0802 2131 0.0639 9383
94 0.1554 4754 0.1234 9468 0.0981 6500 0.0780 7427 0.0621 2993
95 71 rev 9955 0.1207 7719 0.0957 7073 0.0759 8469 0.0603 :032
96 0.14:-; 1132 0.1181 1950 0.0934 3486 0.0739 5104 0.0585 6342
97 O.Ur;.-; M169 0.1155 2029 0.0911 5596' 0.0719 7181 0.0568 5769
98 0.1436 0950 0,1129 7828 0.0889 3264 0.0700 4556 0.0552 0164
0.1407 9363 0.1104 9221 0.0867 03:v;- 0.0681 7086 0.0535
100 0.1380 3297 0,1080 6084 0.0846 4737 0.0663 4634 0.052(/^
-p'
T.17
LOGARITHMIC AND OTHER TABLES

TABLE II
PRESENT VALUE AT COMPOUND INTEREST
-H
V
n = (1 + 0
Rate /
Periods

1 r_1
.045 44% .055 5-%
.035 34% .04(4%) .05(5%) ^ 2
n
2 ) 2 J

0.9615 3846 0.9569 3780 0.9523 8095 0.9478 6730


1 0.9661 8357
0.9245 5621 0.9157 2995 0.9070 2948 0.8984 5242
2 0.9335 1070
0.8889 9636 0.8762 9660 0.8638 3760 0.8516 1366
3 0.9019 4271
0.8548 0419 0.8385 6134 0.8227 0247 0.8072 1674
4 0.8714 4223
0.7835 2617 0.7651 3435

ow
5 0.8419 7317 0.8219 2711 0.8024 5105
0.7678 9574 0.7462 1540 0.7252 4583
6 0.8135 0064 0.7903 1453
0.7348 2846 0.7106 8133 0.6874 3681
7 0.7859 9096 0.7599 1781
0.7031 8513 0.6768 3936 0.6515 9887
8 0.7594 1156 0.7306 9021
0.6729 0443 0.6446 0892 0.6176 2926
9 0.7337 3097 0.7025 8674

0.6439 2768 0.6139 1325 0.5854 3058


10 0.7089 1881 0.6755 6417

e
0.6495 8093 0.6161 9874 0.5846 7929 0.5549 1050
11 0.6849 4571

re
0.5259 8152
12
13
0.6617 8330
0.6394 0415
0.6245 9705
0.6005 7409

rFl 0.5896 6386


0.5642 7164
0.5568
0.5303
3742
2135 0.4985 6068

F
0.5774 7508 0.5399 7286 0.5050 6795 0.4725 6937
14 0.6177 8179
0.5167 2044 0.4810 1710 0.4479 3305
15 0.5968 9062 0.5552 6450
0.5339 0818 0.4944 6932 0.4581 1152 0.4245 8109

r
16 0.5767 0591
0.4362 9669 0.4024 4653
ou
0.5572 0378 0.5133 7325 0.4731 7639
17
18 0.5383 6114 0.4936 2812 0.4528 0037
fo 0.4155 2065
0.3957 3396
0.3814 6590
0.3615 7906
ks
19 0.5201 5569 0.4746 4242 0.4333 0179

0.4563 8695 0.4146 4286 0.3768 8948 0.3427 2896


20 0.5025 6588
0.3248 6158
oo

0.4388 3360 0.3967 8743 0.3589 4236


21 0.4855 7090
0.3797 0089 0.3418 4987 0.3079 2567
22 0.4691 5063 0.4219 5539
Y

0.3633 5013 0.3255 7131 0.2918 7267


0.4057 2633
eB

23 0.4532 8563
0.3477 0347 0.3100 6791 0.2766 5656
24 0.4379 5713 0.3901 2147

0.3751 1680 0.3327 3060 0.2953 0277 0.2622 3370


25 0,4231 4699
0.3184 0248 0.2812 4073 0.2485 6275
r

26 0.4088 3767 0.3606 8923


0.3046 9137 0.2678 4832 0.2356 0450
ou

27 0.3950 1224 0.3468 1657


0.2550 9364 0.2233 2181
ad
Y

28 0.3816 5434 0.3334 7747 0.2915 7069


0.2790 1502 0.2429 4632 0.2116 7944
29 0.3687 4815 0.3206 5141

0.3083 1867 0.2670 0002 0.2313 7745 0.2006 4402


30 0.3562 7841
d

0.2555 0241 0.2203 5947 0.1901 8390


31 0.3442 3035 0.2964 6026
0.2098 6617 0.1802 6910
Re

0.2444 9991
in

32 0.3325 8971 0.2850 5794


0.2339 7121 0.1998 7254 0.1708 7119
33 0.3213 4271 0.2740 9417
0.2238 9589 0.1903 5480 0.1619 6321
0.3104 7605 0.2635 5209
F

34
0.2534 1547 0.2142 5444 0.1812 9029 0.1535 1963
35 0.2999 7686
0.2436 6872 0.2050 2817 0.1726 5741 0.1455 1624
36 0.2898 3272
0.2342 9685 0.1961 9921 0.1644 3563 0.1379 3008
37 0.2800 3161
0.1877 5044 0.1566 0536 0.1307 3941
38 0.2705 6194 0.2252 8543
0.2166 2061 0.1796 6549 0.1491 4797 0.1239 2362
39 0.2614 1250
0.1719 2870 0.1420 4568 0.1174 6314
40 0.2525 7247 0.2082 8904
0.2002 7793 0.1645 2507 0.1352 8160 0.1113 3947
41 0.2440 3137
0.1574 4026 0.1288 3962 0.1055 3504
42 0.2357 7910 0.1925 7493
0.1506 6054 0.1227 0440 0.1000 3322
43 0.2278 0590 0.1851 6820
0.1441 7276 0.1168 6133 0.0948 1822
44 0.2201 0231 0.1780 4635
0.1379 6437 0.1112 9651 0.0898 7509
45 0.2126 5924 0.1711 9841
0.1320 2332 0.1059 9668 0.0851 8965
46 0.2054 6787 0.1646 1386
0.1582 8256 0.1263 3810 0.1009 4921 0.0807 4849
47 0.1985 1968
0.1521 9476 0.1208 9771 0.0961 4211 0.0765 3885
48 0.1918 0645
0.1156 9158 0.0915 6391 0.0725 4867
0.1853 2024 0.1463 4112

0.1407 1262 0.1107 0965 0.0872 0373 0.0687 6652


1.1790 5337
T.18
APPLIED MATHEMATICS-XI

TABLE II

PRESENT VALUE AT COMPOUND INTEREST


=(! + /)
-II

Periods
Rate /

1 f 1 ^
n .035 3-% .045 4-% r_i
2 .04(4%) I 2 .05(5%) .055 5-%
I 2
51 0.1729 9843 0.1353 0059 0.1059 4225 0.0830 5117
52 0.0651 8153
0.1671 4824 0.1300 9672 0.1013 8014 0.0790 9635 0.0617 8344
53 0.1614 9589 0.1250 9300 0.0970 1449 0.0753 2986
54 0.0585 6250
0.1560 3467 0.1202 8173 0.0928 3683 0.0717 4272 0.0555 0948

ow
55 0.1507 5814 0.1156 5551 0.0888 3907 0.0683 2640 0.0526 1562
56 0.1456 6004 0.1112 0722 0.0850 1347 0.0650 7276 0.0498 7263
57 0.1407 3433 0.1069 3002 0.0813 5260 0.0619 7406 0.0472 7263
58 0.1359 7520 0.1028 1733 0.0778 4938 0.0590 2291 0.0448 0818
59 0.1313 7701 0.0988 6282 0.0744 9701 0.0562 1230 0.0424 7221
60 0.1269 3431

e
0.0950 6040 0.0712 8901 0.0535 3552 0.0402 5802
61 0.1226 4184 0.0914 0423

re
0.0682 1915 0.0509 8621 0.0381 5926
62 0.1184 9453

rFl
0.0878 8868 0.0652 8148 0.0485 5830 0.0361 6992
63 0.1144 8747 0.0845 0835 0.0624 7032

F
0.0462 4600 0.0342 8428
64 0.1106 1591 0.0812 5803 0.0597 8021 0.0440 4381 0.0324 9695
65 0.1068 7528 0.0781 3272 0.0572 0594 0.0419 4648 0.0308 0279
66 0.1032 6114

r
0.0751 2762 0.0547 4253 0.0399 4903
ou
67
0.0291 9696
0.0997 6922
68 0.0963 9538
0.0722
0.0694
3809
5970
0.0523 8519
0.0501 2937 fo
0.0380 4670
0.0362 3495
0.0276 7485
ks
0.0262 3208
69 0.0931 3563 0.0667 8818 0.0479 7069 0.0345 0948 0.0248 6453
70 0.0899 8612 0.0642 1940 0.0459 0497 0.0328 6617 0.0235 6828
oo

71 0.0869 4311 0.0617 4942 0.0439 2820 0.0313 0111 0.0223 3960
72 0.0840 0300 0.0593 7445 0.0420 3655 0.0298 1058
Y

73 0.0211 7498
0.0811 6232 0.0570 9081
B

0.0402 2637 0.0283 9103 0.0200 7107


74 0.0784 1770 0.0548 9501 0.0384 9413 0.0270 3908 0.0190 2471
re

75 0.0757 6590 0.0527 8367 0.0368 3649 0.0257 5150 0.0180 3290
76 0.0732 0376 0.0507 5353 0.0352 5023 0.0245 2524 0.0170 9279
77
ou

0.0707 2827 0.0488 0147 0.0337 3228 0.0233 5737


Y

0.0162 0170
ad

78 0.0683 3650 0.0469 2449 0.0322 7969 0.0222 4512 0.0153 5706
79 0.0660 2560 0.0451 1970 0.0308 8965 0.0211 8582 0.0145 5646
80 0.0637 9285 0.0433 8433 0.0295 5948 0.0201 7698
d

81
0.0137 9759
0.0616 3561 0.0417 1570 0.0282 8658 0.0192 1617 0.0130 7828
82
in

0.0595 5131 0.0401 1125


Re

0.0270 6850 0.0183 0111 0.0123 9648


83 0.0575 3750 0.0385 6851 0.0259 0287 0.0174 2963 0.0117 5022
84 0.0555 9178 0.0370 8510 0.0247 8744
F

0 0165 9965 0.0111 3765


85 0.0537 1187 0.0356 5875 0.0237 2003 0.0158 0919 0.0105 5701
86 0.0518 9553 0.0342 8726 0.0226 9860 0.0150 5637
87
0.0100 0664
0.0501 4060 0.0329 6852 0.0217 2115 0.0143 3940 0.0094 8497
88 0.0484 4503 0.0317 0050 0.0207 8579 0.0136 5657 0.0089 9049
89 0.0468 0679 0.0304 8125 0.0198 9070 0.0130 0626 0.0085 2180
90 0.0452 2395 0.0293 0890 0.0190 3417 0.0123 8691 0.0080 7753
91 0.0436 9464 0.0281 8163 0.0182 1451 0.0117 9706 0.0076 5643
92 0.0422 1704 0.0270 9772 0.0174 3016 0.0112 3530 0.0072 5728
93 0.0407 8941 0.0260 5550 0.0166 7958 0.0107 0028 0.0068 7894
94 0.0394 1006 0.0250 5337 0.0159 6132 0.0101 9074 0.0065 2032
95 0.0380 7735 0.0240 8978 0.0152 7399 0.0097 0547 0.0061 8040
96 0.0367 8971 0.0231 6325 0.0146 1626 0.0092 4331 0.0058 5820
97 0.0355 4562 0.0222 7235 0.0139 8685 0.0088 0315
98 0.0055 5279
0.0343 4359 0.0214 1572 0.0133 8454 0.0083 8395 0.0052 6331
99 0.0331 8221 0.0205 9204 0.0128 0817 0.0079 8471
100 0.0320 6011 0.0198 0004 0.0122 5663 0.0076 0449
T.19
LOGARITHMIC AND OTHER TABLES

TABLE II

PRESENT VALUE AT COMPOUND INTEREST


-II
v„ = {1 + 0
Periods Rate i

r_i
.075 7-%
n .06(6%)
.065 6-%
2 .07(7%) 2 .08(8%)

1 0.9433 9623 0.9389 6714 0.9345 7944 0.9302 3256 0.9259 2593

2 0.8899 9644 0.8816 5928 0.8734 3873 0.8653 3261 0.8573 3882
3 0.8396 1928 0.8278 4909 0.8162 9788 0.8049 6057 0.7938 3224
4 0.7920 9366 0.7773 2309 0.7628 9521 0.7488 0053 0.7350 2985

ow
0.7298 8084 0.7129 8618 0.6965 5863 0.6805 8320
5 0.7472 5817
0.7049 6054 0.6853 3412 0.6663 4222 0.6479 6152 0.6301 6963
6
0.6435 0621 0.6227 4974 0.6027 5490 0.5834 9040
7 0.6650 5711
0.6042 3119 0.5820 0910 0.5607 0223 0.5402 6888
8 0.6274 1237
9 0.5918 9846 0.5673 5323 0.5439 3374 0.5215 8347 0.5002 4897

0.5583 9478 0.5327 2604 0.5083 4929 0.4851 9393 0.4631 9349
10

e
11 0.5267 8753 0.5002 1224 0.4750 9280 0.4513 4319 0.4288 8286

re
0.3971 1376

rFl
12 0.4969 6936 0.4696 8285 0.4440 1196 0.4198 5413
0.4688 3902 0.4410 1676 0.4149 6445 0.3905 6198 0.3676 9792
13

F
0.4423 0096 0.4141 0025 0.3878 1724 0.3633 1347 0.3404 6104
14

0.4172 6506 0.3888 2652 0.3624 4602 0.3379 6602 0.3152 4170
15
0.3387 3460 0.3143 8699 0.2918 9047

r
16 0.3936 4628 0.3650 9533
0.2702 6895
ou
9
0.3165 7439 0.2924 5302
17
18
0.3713 6442
0.3503 4379
0.3428 1251
0.3218 8969 0.2958 6392
fo 0.2720 4932 0.2502 4903
ks
0.3305 1301 0.3022 4384 0.2765 0833 0.2530 6913 0.2317 1206
19

0.2837 9703 0.2584 1900 0.2354 1315 0.2145 4821


20 0.3118 0473
oo

0.2664 7608 0.2415 1309 0.2189 8897 0.1986 5575


21 0.2941 5540
0.2502 1228 0.2257 1317 0.2037 1067 0.1839 4051
22 0.2775 0510
Y

0.2349 4111 0.2109 4688 0.1894 9830 0.1703 1528


0.2617 9726
eB

23
0.2206 0198 0.1971 4662 0.1762 7749 0.1576 9934
24 0.2469 7855

0.2071 3801 0.1842 4918 0.1639 7906 0.1460 1790


25 0.2329 9863
0.1352 0176
r

26 0.2198 1003 0.1944 9579 0.1721 9549 0.1525 3866


0.1418 9643 0.1251 8682
ou

27 0.2073 6795 0.1826 2515 0.1609 3037


ad
Y

0.1714 7902 0.1504 0221 0.1319 9668 0.1159 1372


28 0.1956 3014
0.1610 1316 0.1405 6282 0.1227 8761 0.1073 2752
29 0.1845 5674

0.1741 1013 0.1511 8607 0.1313 6712 0.1142 2103 0.0993 7733
30
d

0.1419 5875 0.1227 7301 0.1062 5212 0.0920 1605


31 0.1642 5484
0.0988 3918 0.0852 0005
Re

0.1147 4113
in

32 0.1549 5740 0.1332 9460


0.1251 5925 0.1072 3470 0.0919 4343 0.0788 8893
33 0.1461 8622
0.1175 2042 0.1002 1934 0.0855 2877 0.0730 4531
0.1379 1153
F

34

0.1103 4781 0.0936 6294 0.0795 6164 0.0676 3454


35 0.1301 0522
0.0875 3546 0.0740 1083 0.0626 2458
36 0.1227 4077 0.1036 1297
0.0818 0884 0.0688 4729 0.0579 8572
37 0.1157 9318 0.0972 8917
0.0764 5686 0.0640 4399 0.0536 9048
38 0.1092 3885 0.0913 5134
0.0714 5501 0.0595 7580 0.0497 1341
39 0.1030 5552 0.0857 7590

0.0805 4075 0.0667 8038 0.0554 1935 0.0460 3093


40 0.0972 2219
0.0624 1157 0.0515 5288 0.0426 2123
41 0.0917 1905 0.0756 2512
0.0710 0950 0.0583 2857 0.0479 5617 0.0394 6411
42 0.0865 2740
0.0545 1268 0.0446 1039 0.0365 4084
43 0.0816 2962 0.0666 7559
0.0509 4643 0.0414 9804 0.0338 3411
44 0.0770 0908 0.0626 0619

0.0476 1349 0.0386 0283 0.0313 2788


45 0.0726 5007 0.0587 8515
0.0444 9859 0.0359 0961 0.0290 0730
46 0.0685 3781 0.0551 9733
0.0415 8747 0.0334 0428 0.0268 5861
0.0646 5831 0.0518 2848
47 0.0310 7375 0.0248 6908
0.0486 6524 0.0388 6679
48 0.0609 9840 0.0289 0582 0.0230 2693
0.0456 9506 0.0363 2410
49 0.0575 4566
0.0268 8913 0.0213 2123
0.0429 0616 0.0339 4776
50 0.0542 8836
T.20
APPLIED MATHEMATICS-XI

TABLE II

PRESENT VALUE AT COMPOUND ENTTEREST


-II
v„=(l + 0
Periods Rate /

1
n
.06{ 6%) .065 6-%
.07(7%) .075 l-%\ .08(8%)
^ 2 2 )
51 0.0512 1544 0.0402 8747 0.0317 2688 0.0250 1315 0.0197 4188
52 0.0483 1645 0.0378 2861 0.0296 5129 0.0232 6804 0.0182 7952
53 0.0455 8156 0.0355 1982 0.0277 1148 0.0216 4469 0.0169 2548
54 0.0430 0147 0.0333 5195 0.0258 9858 0.0201 3460 0.0156 7174
55 0.0405 6742 0,0313 1638 0.0242 0428 0.0187 2986 0.0145 1087
56 0.0382 7115

ow
0.0294 0505 0,0226 2083 0,0174 2312 0.0134 3599
57 0,0361 0486 0.0276 1038 0.0211 4096 0.0162 0756 0,0124 4073
58 0.0340 6119 0.0259 2524 0.0197 5791 0.0150 7680 0.0115 1920
59 0.0321 3320 0.0243 4295 0.0184 6533 0.0140 2493 0.0106 6592
60 0.0303 1434 0.0228 5723 0.0172 5732 0.0130 4644 0.0098 7585
61 0.0285 9843 0.0214 6218 0.0161 2834 0.0121 3623 0.0091 4431

e
62 0.0269 7965 0.0201 5229 0.0150 7321

Fl
0.0112 8951 0.0084 6695

re
63 0.0254 5250 0.0189 2233 0.0140 8711 0.0105 0187 0.0078 3977
64 0.0240 1179 0.0177 6745 0.0131 6553 0.0097 6918 0.0072 5905

F
65 0.0226 5264 0.0166 8305 0.0123 0423 0.0090 8761 0.0067 2134
66 0.0213 7041 0,0156 6484 0.0114 9928
ur 0.0084 5359 0.0062 2346

or
67 0.0201 6077 0.0147 0877 0.0107 4699 0.0078 6381 0.0057 6247
68 0,0190 1959 0.0138 1105 0.0100 4392 0.0073 1517 0.0053 3562
69 0.0179 4301 0.0129 6812 0.0093 8684
sf
0,0068 0481 0.0049 4039
70 0.0169 2737 0.0121 7664 0.0087 7275
k
0.0063 3006 0.0045 7443
Yo
71 0.0159 6921 0.0114 3346 0.0081 9883 0.0058 8842 0.0042 3558
oo

72 0.0150 6530 0.0107 3565 0.0076 6246 0.0054 7760 0.0039 2184
73 0.0142 1254 0.0100 8042 0.0071 6117 0.0050 9544 0.0036 3133
B

74 0.0134 0806 0.0094 6518 0,0066 9269 0.0047 3995 0.0033 6234
75 0,0126 4911 0,0088 8750
re

0,0062 5485 0.0044 0925 0.0031 1328


76 0.0119 3313 0.0083 4507 0.0058 4565 0.0041 0163 0.0028 8267
77 0.0112 5767 0.0078 3574 0.0054 6323 0.0038 1547 0.0026 6914
u

78 0.0106 2044 0.0073 5751


ad

0.0051 0582 0.0035 4928 0.0024 7142


79
Yo

0.0100 1928 0.0069 0846 0.0047 7179 0.0033 0165 0.0022 8835
80 0.0094 5215 0.0064 8681 0.0044 5962 0.0030 7130 0.0021 1885
81 0.0089 1713 0.0060 9090 0.0041 6787 0.0028 5703
d

0.0019 6190
82 0.0084 1238 0.0057 1916 0.0038 9520
Re

0.0026 5770 0.0018 1657


in

83 0.0079 3621 0.0053 7010 0,0036 4038 0.0024 7228 0.0016 8201
84 0.0074 8699 0.0050 4235 0.0034 0222 0.0022 9979 0.0015 5742
F

85 0.0070 6320 0.0047 3460 0.0031 7965 0.0021 3934 0.0014 4205
86 0.0066 6340 0,0044 4563 0,0029 7163 0.0019 9009 0,0013 3523
87 0,0062 8622 0.0041 7430 0.0027 7723 0.0018 5124 0.0012 3633
88 0.0059 3040 0.0039 1953 0.0025 9554 0.0017 2209 0.0011 4475
89 0,0055 9472 0.0036 8031 0.0024 2574 0.0016 0194 0.0010 5995
90 0.0052 7803 0.0034 5569 0.0022 6704 0.0014 9018 0.0009 8144
91 0.0049 7928 0.0032 4478 0.0021 1873 0.0013 8621
92 0.0009 0874
0.0046 9743 0.0030 4674 0.0019 8012 0.0012 8950 0.0008 4142
93 0.0044 3154 0.0028 6079 0.0018 5058 0.0011 9953
94 0.0007 7910
0.0041 8070 0.0026 8619 0.0017 2952 0.0011 1585 0.0007 2138
95 0.0039 4405 0.0025 2224 0,0016 1637 0.0010 3800 0.0006 6795
96 0.0037 2081 0.0023 6831 0.0015 1063 0,0009 6558 0.0006 1847
97 0.0035 1019 0.0022 2376 0.0014 1180 0.0008 9821
98 0.0033 1150 0,0005 7266
0,0020 8804 0.0013 1944 0.0008 3555
99 0.0031 2406 0.0005 3024
0.0019 6060 0.0012 3312 0.0007 7725 0.0004 9096
100 0.0029 4723 0.0018 4094 0.0011 5245 0.0007 2303 0.0004 5459
T.21
LOGARITHMIC AND OTHER TABLES

TABLE III
AMOUNT OF AN ANUIT’V

s
(1+ /)"-!
n 11
I

Rate /
Periods
f 7 1
1
.0025
f 1
-% .0041671 Ao/J .005 4% .005833 -^%
K^2
.0075 -%
4
n
, 4 12 ) K2

1.0000 0000 1.0000 0000 1.0000 0000


1 1.0000 0000 1,0000 0000
2.0050 0000 2.0058 3333 2.0075 0000
2 2.0025 0000 2.0041 6667
3.0150 2500 3.0175 3402 3.0225 5625
3 3.0075 0625 3.0125 1736
4.0452 2542

ow
4.0250 6952 4.0301 0012 4.0351 3629
4 4.0150 2502
jj02 ^ J / -r J/ W..O/55 -" -oi
5.0250 6258 5.0418 4064
i 83 6.1136 3135
6 6.0376 2523 6.0628 4831
●; oyar 7.1594 8358
,,v.381 1018 ;,1058 7939
/ 7.0527 1930 8.:-’131 7971
'■ 1414 0879 8.1'--
8 8.0703 5110 8.1176 4397
9.1821 1583 0.27' .'356
9.1514 6749

e
g 9.0905 2697
10.2666 2516 10.3443 3940

You might also like